Pediatrics Examination and Board Review [1st Edition] 0071847685, 9780071847681, 9780071847698

An Engagingly Written, Case-Based Review for the Pediatrics Board Exam McGraw-Hill Specialty Board Review: Pediatrics i

3,136 395 81MB

English Pages 846 Year 2016

Report DMCA / Copyright

DOWNLOAD FILE

Polecaj historie

Pediatrics Examination and Board Review [1st Edition]
 0071847685, 9780071847681, 9780071847698

  • Commentary
  • TRUE PDF

Table of contents :
Pediatrics Examination and Board Review......Page 1
Half Title......Page 2
Title Page......Page 4
Copyright......Page 5
Dedication......Page 6
Contents......Page 8
Contributors......Page 10
Preface......Page 14
Chapter 1 Adolescent Medicine and Gynecology......Page 16
Chapter 2 Allergic and I m munologic Disorders......Page 40
Chapter 3 Behavioral and Mental Health Issues......Page 66
Chapter 4 Blood and Neoplastic Disorders......Page 88
Chapter 5 Cardiology......Page 124
Chapter 6 Cognition, Language, and Learning Disabilities......Page 146
Chapter 7 Collagen Vascular and Other Multisystem Disorders......Page 166
Chapter 8 Critical Care......Page 180
Chapter 9 Ear, Nose, and Throat Disorders......Page 194
Chapter 10 Emergency Care......Page 218
Chapter 11 Endocrine Disorders......Page 234
Chapter 12 Ethics......Page 256
Chapter 13 Eye Disorders......Page 270
Chapter 14 Fetus and Newborn......Page 290
Chapter 15 Fluid and Electrolyte Metabolism......Page 314
Chapter 16 Pediatric Gastroenterology......Page 338
Chapter 17 Genetics and Dysmorphology......Page 376
Chapter 18 Genital System Disorders......Page 392
Chapter 19 Growth and Development......Page 418
Chapter 20 Infectious Disease......Page 436
Chapter 21 Metabolic Disorders......Page 474
Chapter 22 Musculoskeletal Disorders......Page 496
Chapter 23 Neurologic Disorders......Page 526
Chapter 24 Nutrition......Page 554
Chapter 25 Patient Safety and Quality Improvement......Page 576
Chapter 26 Pharmacology: Pain Management and Sedation......Page 588
Chapter 27 Poisoning and Environmental Exposure to Hazardous Substances......Page 610
Chapter 28 Preventative Pediatrics......Page 640
Chapter 29 Psychosocial Issues and Child Abuse......Page 658
Chapter 30 Research and Statistics......Page 676
Chapter 31 Renal and Urologic Disorders......Page 684
Chapter 32 Respiratory Disorders......Page 712
Chapter 33 Skin Disorders......Page 742
Chapter 34 Sports Medicine a nd Physica l Fitness......Page 774
Chapter 35 Substance Abuse......Page 794
Final Exam......Page 806
Answer Key......Page 822
Index......Page 824
Blank Page......Page 0

Citation preview

McGraw-Hill Education Specialty Board Review

PEDIATRICS EXAMINATION AND BOARD REVIEW

This page intentionally left blank

fir-4"...”-

McGraw-Hill Education Specialty Board Review

PEDIATRICS EXAMINATI N

Edited by ‘ Andrew R. Peterso

Clinical Associate Professor Stead Family Department of Pediatrics Carver College of Medicine University of Iowa Iowa City, Iowa

Kelly E. Wood, MD Clinical Assistant Professor Stead Family Department of Pediatrics Carver College of Medicine University of Iowa Iowa City, Iowa

New York / Chicago / San Francisco / Athens / London / Madrid / Mexico City Milan / New Delhi / Singapore / Sydney /Toronto



Copyright © 2017 by McGraw-Hill Education. All rights reserved. Except as permitted under the United States Copyright Act of 1976,

no part of this publication may be reproduced or distributed in any form or by any means, or stored in a database or retrieval system, without the prior written permission of the publisher. ISBN: 978-0-07-184769-8 MHID: 0-07-184769-3. The material in this eBook also appears in the print version of this title: ISBN: 978-0-07-184768-1, MHID: 0-07-184768-5.

eBook conversion by codeMantra

Version 1.0

All trademarks are trademarks of their respective owners. Rather than put a trademark symbol after every occurrence of a trademarked name, we use names in an editorial fashion only, and to the benefit of the trademark owner, with no intention of infringement of the trademark. Where such designations appear in this book, they have been printed with initial caps. McGraw-Hill Education eBooks are available at special quantity discounts to use as premiums and sales promotions or for use in corpo­ rate training programs. To contact a representative, please visit the Contact Us page at www.mhprofessional.com. Notice

Medicine is an ever-changing science. As new research and clinical experience broaden our knowledge, changes in treatment and drug therapy are required. The authors and the publisher of this work have checked with sources believed to be reliable in their efforts to provide information that is complete and generally in accord with the standards accepted at the time of publication. However, in view of the possibility of human error or changes in medical sciences, neither the authors nor the publisher nor any other party who has been involved in the preparation or publication of this work warrants that the information contained herein is in every respect accurate or com­ plete, and they disclaim all responsibility for any errors or omissions or for the results obtained from use of the information contained in this work. Readers are encouraged to confirm the information contained herein with other sources. For example and in particular, readers are advised to check the product information sheet included in the package of each drug they plan to administer to be certain that the information contained in this work is accurate and that changes have not been made in the recommended dose or in the contraindications for administration. This recommendation is of particular importance in connection with new or infrequently used drugs. TERMS OF USE

This is a copyrighted work and McGraw-Hill Education and its licensors reserve all rights in and to the work. Use of this work is subject to these terms. Except as permitted under the Copyright Act of 1976 and the right to store and retrieve one copy of the work, you may not decompile, disassemble, reverse engineer, reproduce, modify, create derivative works based upon, transmit, distribute, disseminate,

sell, publish or sublicense the work or any part of it without McGraw-Hill Education's prior consent. You may use the work for your own noncommercial and personal use; any other use of the work is strictly prohibited. Your right to use the work may be terminated if you fail to comply with these terms. THE WORK IS PROVIDED "AS IS." McGRAW-HILL EDUCATION AND ITS LICENSORS MAKE NO GUARANTEES OR WAR­ RANTIES AS TO THE ACCURACY, ADEQUACY OR COMPLETENESS OF OR RESULTS TO BE OBTAINED FROM USING THE WORK, INCLUDING ANY INFORMATION THAT CAN BE ACCESSED THROUGH THE WORK VIA HY PERLINK OR OTHERWISE, AND EXPRESSLY DISCLAIM ANY WARRANTY, EXPRESS OR IMPLIED, INCLUDING BUT NOT LIMITED TO IMPLIED WARRANTIES OF MERCHANTABILITY OR FITNESS FOR A PARTICULAR PURPOSE. McGraw-Hill Education and its licensors do not warrant or guarantee that the functions contained in the work will meet your requirements or that its opera­ tion will be uninterrupted or error free. Neither McGraw-Hill Education nor its licensors shall be liable to you or anyone else for any inaccuracy, error or omission, regardless of cause, in the work or for any damages resulting therefrom. McGraw-Hill Education has no responsibility for the content of any information accessed through the work. Under no circumstances shall McGraw-Hill Education and/ or its licensors be liable for any indirect, incidental, special, punitive, consequential or similar damages that result from the use of or inability to use the work, even if any of them has been advised of the possibility of such damages. This limitation of liability shall apply to any claim or cause whatsoever whether such claim or cause arises in contract, tort or otherwise.

For VAC, RAP, MEP, MKP, KJT, and W. Thanks. Andrew R. Peterson

To my family and friends. You made his happen. Kelly E. Wood

This page intentionally left blank

Contents Contributors Preface

0 0 0 0 0 0 0 0 0 0 0 0 0 0 0 0 0 0 0 0 0 0 0 0 0 0 0 0 0 0 0 0 0 0 0 0 0 0 0 0 0 0 0 0

0 0 0 0 0 0 0 0 0 0 0 0 0 0 0 0 0 0 0 0 0 0 0 0 0 0 0 0 0 0 0 0 0 0 0 0 0 0 0 0 0 0 0 0 0 0 0 0 0

Ado l escent Med ic i n e a n d Gynecology

2 A l l e rg i c a n d I m m u nologic Di sorders

0 0 0 0 0 0 0 0 0 0 0

25

3 Behaviora l and Mental Health Issues

4 B l ood a n d Neoplastic Di sorders 5 Ca rd i o l ogy

51

0 0 0 0 0 0 0 0 0 0 0 0 0 0 0 0 0

73

0 0 0 0 0 0 0 0 0 0 0 0 0 0 0 0 0 0 0 0 0 0 0 0 0 0 0 0 0 0 0 0 0 0 0 0

1 09

0 0 0 0 0 0 0 0 0 0 0 0 0 0 0 0 0 0 0 0 0 0 0 0 0 0 0 0 0 0 0 0 0 0 0 0

1 31

6 Cog nition, La n g u age, a n d Lea r n i n g Disabi l ities

ix xiii

8 Critica l Care

1 65

9 Ea r, Nose, a n d Th roat Di sorders

1 79

10 Emerg e ncy Ca re

203

11

21 9

Endocrine Di sorders

241

12 Eth i cs 0 0 0 0 0 0 0 0 0 0 0 0 0 0 0 0 0 0 0 0 0 0 0 0 0 0 0 0 0 0 0 0 0 0

255

0 0 0 0 0 0 0 0 0 0 0 0 0 0 0 0 0 0 0 0 0 0 0 0 0 0 0

275

1 7 Genetics a n d Dysmorphology

Meta b o l i c Di sorders

459

22 M u scu loske l etal Di sorders

48 1

23 N e u rologic Di sorders

51 1

24 N utrition

25 Patient Safety a n d Q u a l ity I m p rovement 26 P h a rmacology: Pa i n Ma nagement

0 0 0 0 0 0 0 0 0 0 0 0 0 0 0 0 0 0 0 0 0 0 0 0 0 0 0 0 0 0 0 0 0 0

27 Poi so n i n g a n d Envi ro n menta l 28 Preve ntative Ped iatrics

595

0 0 0 0 0 0 0 0 0 0 0

643 66 1

0 0 0 0 0 0 0 0 0 0 0 0 0 0 0 0 0 0

669

0 0 0 0 0 0 0 0 0 0 0 0 0 0 0 0 0 0 0 0 0 0 0 0 0 0

697

Ren a l a n d U rologic Disord e rs

33 Skin Di sorders

34 Sports Med ic i n e and Physical Fitness

35 S u bsta nce Abuse

573

625

30 Research a n d Statistics

32 Res pi ratory Di sorders

561

0 0 0 0 0 0 0 0 0 0 0 0 0 0 0 0 0 0 0 0 0 0 0 0

29 Psych osoc i a l Issues a n d C h i l d Abuse

31

539

727 0 0 0 0 0 0 0 0 0 0 0

759 779

0 0 0 0 0 0 0 0 0 0 0 0 0 0

299

Fi n a l Exa m

79 1

0 0 0 0 0 0 0 0 0 0 0 0 0 0 0 0 0 0 0 0

323

An swe r Key

807

15 Fluid a n d El ectro lyte Meta bol i s m 16 Ped iatric Gastroenterology

42 1

Expo s u re t o Hazard o u s S u bsta nces

1 51

Di sorders

14 Fetus a n d Newborn

21

a n d Sedation

7 Co l l a g e n Va sc u l a r a n d Other M u ltisyste m

13 Eye Di sorders

20 I nfecti o u s Di sease

0 0 0 0 0 0 0 0 0 0 0 0 0 0 0 0 0

361

1 8 Gen ital Syste m Di sorders

377

19 G rowth and Development

403

Index

809

VII

This page intentionally left blank

Contributors Dina Al-Zubeidi, MD Clinical Assistant Professor of Pediatrics Division of Gastroenterology Stead Family Department of Pediatrics, Carver College of Medicine University oflowa Iowa City, Iowa [ 1 6] William Aughenbaugh, MD Associate Professor and Program Director Department of Dermatology Vice Chair of Education in the Department of Dermatology and Director of Specialty Clinical Medical Education and Residency Preparation University of Wisconsin Madison, Wisconsin [33] LaTisha L. Bader, PhD, LP, LAC,CC-AASP Center for Dependence, Addiction and Rehabilitation (CeDAR) University of Colorado Hospital Aurora, Colorado [35] Rebecca Benson, MD, PhD Medical Director, Pediatric Pain and Palliative Care Program Medical Director for Clinical Ethics and Director, Ethics Consult Service Stead Family Department of Pediatrics University oflowa Children's Hospital, University oflowa Hospitals and Clinics Iowa City, Iowa [ 1 2] James D. Burkhalter, LISW Director of DBT Programming Social Work Specialist, Department of Psychiatry University oflowa Hospitals and Clinics Iowa City, Iowa [29] Gayathri Chelvakumar, MD, MPH Nationwide Children's Hospital Section of Adolescent Medicine Columbus, Ohio [ 1 ]

Paula Cody, MD, MPH Assistant Professor Department of Pediatrics University of Wisconsin School of Medicine and Public Health Madison, Wisconsin [ 1 ] Cassandra J . Collins, BSW, LISW Clinical Social Worker Department of Social Service University oflowa Hospitals & Clinics Iowa City, Iowa [29] Amy L. Conrad, PhD Assistant Professor The Stead Family Department of Pediatrics University oflowa Children's Hospital Iowa City, Iowa [ 6] Linda J. Cooper-Brown, PhD Clinical Associate Professor Stead Family Department of Pediatrics, Division of Pediatric Psychology The University of Iowa Iowa City, Iowa [3] Vanessa A. Curtis, MD Clinical Assistant Professor Department of Pediatrics Division of Endocrinology and Diabetes University of Iowa Carver College of Medicine Iowa City, Iowa [ 1 1 ] Anthony J. Fischer, MD, PhD Assistant Professor Department of Pediatrics Division of Allergy, Pulmonology, and Immunology University oflowa Children's Hospital Iowa City, Iowa [32]

IX

X

Contributors

Chris Hogrefe, MD, FACEP Assistant Professor Department of Medicine-Sports Medicine Department of Emergency Medicine Department of Orthopaedic Surgery-Sports Medicine Northwestern Medicine Northwestern University Feinberg School of Medicine Chicago, Illinois [27] Sandy D. Hong, MS, MD Assistant Clinical Professor Division of Rheumatology Department of Pediatrics University oflowa Children's Hospital Iowa City, Iowa [7] Erin Howe, MD University oflowa Stead Family Department of Pediatrics Iowa City, Iowa [ 1 9]

Elizabeth H. Mack, MD, MS Associate Professor of Pediatrics Division of Pediatric Critical Care Medical University of South Carolina Charleston, South Carolina [25] Jessie Marks, MD, FAAP Clinical Assistant Professor University of Iowa Carver College of Medicine Stead Family Department of Pediatrics, University oflowa Children's Hospital Iowa City, Iowa [ 14] Ross Mathiasen, MD Department of Emergency Medicine Department of Family Medicine Institute for Orthopaedics, Sports Medicine, and Rehabilitation The University of Iowa Carver College of Medicine Iowa City, Iowa [35]

Jennifer G. Jetton, MD Clinical Assistant Professor Division of Pediatric Nephrology, Dialysis and Transplantation Stead Family Department of Pediatrics University oflowa Children's Hospital Iowa City, Iowa [3 1 ]

Satsuki Matsumoto, MD Associate of Pediatrics Department of Pediatrics, Division of Neurology and Developmental and Behavioral Pediatrics Roy J. and Lucille A. Carver College of Medicine University oflowa Iowa City, Iowa [23]

Kathleen Kieran, MD, MS Associate Professor of Urology Department of Urology University of Washington/Seattle Children's Hospital Seattle, Washington [ 1 8]

Jennifer McWilliams, MD Child and Adolescent Psychiatrist Department of Behavioral Health Children's Hospital and Medical Center Omaha, Nebraska [3]

Todd Kopelman, PhD, BCBA Clinical Assistant Professor Department of Psychiatry University oflowa Hospitals and Clinics Iowa City, Iowa [ 6]

Gary Milavetz, Pharm D, FCCP, FAPhA Associate Professor and Division Head The University of Iowa College of Pharmacy Department of Pharmacy Practice and Science Division of Applied Clinical Sciences Iowa City, Iowa [26]

Kathy Lee-Son, MD, MHSc Clinical Assistant Professor Pediatric Nephrology Department Stead Family University oflowa Children's Hospital Iowa City, Iowa [3 1 ] Ashley Loomis, MD Assistant Professor, Pediatric Critical Care University of Minnesota Minneapolis, Minnesota [8] Rebecca L. Lozman-Oxman, DNP, APRN, MSN, BSN, MPH Pediatric Nurse Practitioner Pediatrics, New London Hospital/Newport Health Center New London, New Hampshire [28]

Sarah L. Miller, MD Clinical Assistant Professor Department of Emergency Medicine University oflowa Hospitals and Clinics Iowa City, Iowa [ 1 0] Ashley A. Miller, MD, FAAP Pediatrician Pediatrics Geisel School of Medicine at Dartmouth Hanover, New Hampshire [28] Lisa K. Muchard, MD Assistant Clinical Professor Department of Dermatology University of Wisconsin Madison, Wisconsin [33]

Contributors

Blaise Nemeth, MD, MS Associate Professor (CHS) Pediatric Orthopedics, American Family Children's Hospital Department of Orthopedics and Rehabilitation University of Wisconsin School of Medicine and Public Health Madison, Wisconsin [22] Benton Ng, MD Pediatric Cardiologist Pediatrics, All Children's Hospital St. Petersburg, Florida [5] Erin A. Osterholm, MD Assistant Professor of Pediatrics Department of Pediatrics, Division of Neonatology University of Minnesota Minneapolis, Minnesota [ 14] Niyati Patel, MD Assistant Professor, Pediatric Critical Care University of Minnesota Minneapolis, Minnesota [8] Andrew R. Peterson, MD, MSPH Clinical Associate Professor Stead Family Department of Pediatrics, Carver College of Medicine University of lowa Iowa City, Iowa [30, 34] Catherina Pinnaro, MD Pediatrics Resident, Department of Pediatrics University of lowa Iowa City, Iowa [ 1 2] Nathan Price, MD Clinical Assistant Professor Pediatric Infectious Diseases Stead Family Department of Pediatrics Iowa City, Iowa [20] Gregory M. Rice, MD Associate Professor of Pediatrics, Division of Genetics and Metabolism Co- Director, WSLH Biochemical Genetics Laboratory; Director, Medical Genetics Residency Program University of Wisconsin School of Medicine and Public Health Madison, Wisconsin [ 1 7] Eric T. Rush, MD, FAAP, FACMG Departments of Pediatrics and Internal Medicine University of Nebraska Medical Center and Children's Hospital and Medical Center Omaha, Nebraska [2 1 ]

Judith Regine Sabah, MD, PhD, MBA Ophthalmologist -Comprehensive and Pediatric/Adult Strabismus Operative Care, Eugene VA Healthcare System Eugene, Oregon [ 1 3 ] Melanie A. Schmitt, MD Assistant Professor of Pediatric Ophthalmology and Director of Ophthalmic Genetics Department of Ophthalmology and Visual Sciences University of Wisconsin-Madison Madison, Wisconsin [ 1 3 ] Laura Steinauer, Pharm D Candidate Student Pharmacist The University of Iowa College of Pharmacy Iowa City, Iowa [26] Natalie Stork, MD Assistant Professor University of Missouri-Kansas City School of Medicine Department of Orthopedic Surgery and Department of Pediatrics The Children's Mercy Hospital, Division of Orthopedics Section of Sports Medicine Kansas City, Missouri [22] Alex Thomas, MD Allergist/Immunologist Internal Medicine/Pediatrics Presence Sts. Mary and Elizabeth Medical Center, Advocate Children's Hospital Chicago, Illinois [2] Amy 0. Thomas, MD Allergist/Immunologist Allergy and Immunology Department Dreyer Medical Clinic- Advocate Hospital System Aurora, Illinois [2] Elizabeth C. Utterson, MD Assistant Professor of Pediatrics Division of Pediatric Gastroenterology, Hepatology and Nutrition Washington University St. Louis, Missouri [ 1 6] Jeffrey Robert Van Blarcom, MD Assistant Professor Department of Pediatrics, Division of Inpatient Medicine University of Utah Salt Lake City, Utah [ 1 5, 26] Susan S. Vos, PharmD, BCPS, FAPhA Clinical Associate Professor The University oflowa College of Pharmacy Department of Pharmacy Practice and Science Division of Applied Clinical Sciences Iowa City, Iowa [26]

XI

XII

Contri butors

Tammy L. Wilgenbusch, PhD Clinical Assistant Professor Stead Family Department of Pediatrics, Division of Psychology University oflowa Children's Hospital Iowa City, Iowa [ 6] Adam D. Wolfe, MD, PhD Assistant Professor of Pediatric Hematology-Oncology Baylor College of Medicine Children's Hospital of San Antonio San Antonio, Texas [4] Kelly E. Wood, MD Clinical Assistant Professor Stead Family Department of Pediatrics Carver College of Medicine University oflowa Iowa City, Iowa [24, 29]

Leah Zhorne, MD Clinical Assistant Professor Department of Pediatrics, Division of Neurology and Developmental and Behavioral Pediatrics Roy J. and Lucille A. Carver College of Medicine University oflowa Iowa City, Iowa [23] Derek Zhorne, MD Clinical Assistant Professor of Pediatrics Division of General Pediatrics and Adolescent Medicine Stead Family Department of Pediatrics Iowa City, Iowa [9]

Preface Welcome to the Pediatrics Examination and Board Review book. This is a comprehensive board review designed to help the reader study for the general pediatrics board examination. This text covers all the content that the American Board of Pediatrics (ABP) says you need to know for the board exam. The 35 chapters in this text correspond to the 3 5 sections of the ABP content specifications and are written by specialists in the topic areas. The majority of the content is presented as cases followed by question/answer/discussion. The discussions are in depth but written in an informal manner to avoid the feeling that you are reading a textbook. For visual learners, we have included tables, figures, and photos. The goal of this book is to make studying for the boards more engaging. Each chapter is meant to stand alone, allowing you to focus on challenging content areas or those where you may need to spend more time. A final exam is included at the end to help you test what you have learned. Each question is referenced in the book so you can go back and review what you may have missed.

Pediatricians trying to pass a national board exam are not the only ones who might benefit from this book. Anyone wanting to learn more about pediatric medicine should read this book. It provides a broad overview perfect for both early and seasoned learners. We are very proud of the final product and believe it pro­ vides the reader with an exceptional resource to cover the entire breadth of pediatric medicine. We would like to thank all of the authors who contributed to this book. Without their hard work, this book would have never come together. We would also like to thank Christie Naglieri, Andrew Moyer, Alyssa Fried, and Samantha Williams at McGraw-Hill for their help and guidance throughout. But most of all, we would like thank our friends, family, and coworkers for their love, understanding, and support during the final push to complete this project. Andrew R. Peterson, MD, MSPH Kelly E. Wood, MD

XIII

This page intentionally left blank

1

Adolescent Medicine and Gynecology Gaya t h r i C h e l va k u m a r a n d Pa u l a Cody

A 1 4-year-old boy presents to your office with concerns of delayed puberty. The patient is shorter than most of his classmates. He is active in basketball and is worried that his lack of height will affect his ability to play. His mother is 5 feet, 7 inches tall and had her first menses at age 1 4 years. His father is 6 feet, 4 inches tall and reports that he was a "late bloomer" and attained most of his adult height in college. The patient is otherwise healthy, developmentally appropriate, and not on any medications. On physical exam he is a well-appearing, well-nourished young male. He has mild acne, his testes are descended bilaterally and 3 mL in volume, and there is scant pubic hair and minimal penile development. He has grown 5 em in the last year. (See Figure 1 - 1 .) Question 1 - 1 Which of the following tests will most likely establish the diagnosis for this patient? A) CBC. B) Calculation of midparental height. C) Bone age. D) Growth hormone levels. E) Thyroid studies. Discussion 1 - 1 The correct answer is "C' The age at which puberty is considered delayed is 14 years in boys and 1 3 years in girls. Constitutional delay of puberty is the most common cause, especially in boys. This patient most likely has a constitutional delay of puberty­ short but normal growth rate and a positive family history. The best test to establish the diagnosis would be a bone age. Delayed puberty in boys is defined as lack of pubertal testicular devel­ opment (sexual maturity rating [SMR] 2) by age 14 years in boys. (See Table 1 - 1 for details.) SMR is also known as Tanner

staging, after the pediatrician who first described the sequence of secondary sexual characteristics. (See Figures 1 -2 and 1 -3 . ) Th e delay leads t o a comparative decrease i n growth velocity compared to age-matched peers, leading to short stature as the primary complaint in most patients. Prepubertal growth veloc­ ity is typically 4 to 6 cm/y in adolescent boys and increases to a peak velocity of approximately 9.5 cm/y at SMR 3 to 4. Most delayed puberty in boys is due to a constitutional delay from delayed activation of the hypothalamic-pituitary-gonadal axis. Once puberty begins patients generally have catch-up growth and attain a normal adult height. Often there is a family his­ tory of "late bloomers" or other family members with constitu­ tional delay of puberty. Delayed puberty and short stature can have significant effects on self-esteem, particularly in boys, and short courses of androgen replacement therapy may be indi­ cated. (Think back to junior high when the girls towered over the boys.) Underlying metabolic, endocrine, or systemic dis­ orders are an unlikely cause of delayed puberty in this patient given his previous normal growth velocity and development and otherwise healthy state. A bone age test will help establish the diagnosis. In constitutional delay of puberty bone age will be decreased compared with chronological age as it is more closely related to skeletal maturity and pubertal stage. A normal bone age would be seen with familial short stature and Turner syn­ drome. An advanced bone age is seen with precocious puberty for which early closure of the growth plates will result in a short adult unless treated.





Helpful Tip

Ca lculation of a m i d p a renta l height ca n help determ i n e

1 1 1 r a c h i l d's genetic height potentia l . F o r g i rls: (Mother's height i n e m cm}/2

-

For boys: (Moth er's height i n em cm}/2

+

+

Father's height i n

+

Father's height i n

6.5 em 6.5 em

2

MCGRAW-HILL EDUCATION SPECIALTY BOARD REVIEW: PEDIATRICS

2 to 20 years: Boys Stature-for-age and Weight-for-age percentiles

NAME

12 13 Mother’s Stature — Father’s St::ure — ature

14 15

AGE

RECORD # 16 17

18 19 20 cm 1

1

sT A T U

Calculate BMI: Weight (kg) + Stature (cm) + Stature (cm) x 10,000

or Weight (lb) + Stature (in) + Stature (in) x 703

in

on:

R

10 11

E

1

62

1

S T A T U R E

W E I G H T

W E l G H T

AGE 2 3 4 5 6 Published May 30, 2000 (modified 11/21/00)..

7

8

91011121314151617181920

SOURCE: Developed by the National Center for Health Statistics in collaboration with the National Center for Chronic Disease Prevention and Health Promotion (2000). http:llwww.cdc.gov/growthcharts

7,77"

I I III/ll 4? SAFER . HEALTHIER- PEOPLE”

FIGURE 1-1. Growth chart of boy in Case 1. (Reproduced with permission from the National Center for Health Statistics in collaboration with the National Center for Chronic Disease Prevention and Health Promotion [2000]. http://www.cdc.gov/growthcharts.)

CHAPTER 1



A D O L E S C E N T M E D I C I N E A N D GYN ECOLOGY

3

TA B L E 1-1 S EXUAL MATU RITY RAT I N G ( S M R ) I N MALES

SMRin Males

Pubic Hair Development

Testicular Development

Penile Development

1

No p u b i c h a i r

Prepu berta l genita l i a

Prepu berta l genita l i a

2

Spa rse, downy h a i r at base of pe n i s

E n l a rgement o f testis (vo l u m e > 4 ml), scrota l sac e n l a rg es, red der in a ppea ra nce

No c h a n g e

3

H a i r beco mes thi cker, longer, a n d c u r l i er, sti l l i n l i m ited m i d l i ne d i stribution

Conti n ued e n l a rgement of testes a n d scrot u m

Pen i s beg i n s t o g row i n length fi rst, then d i a m eter

4

Ad u lt type hair in q u a l ity but l i m ited d i stribution

Th icke n i n g a n d d a r ke n i n g o f scrota l sac with conti n u ed g rowth of testes

Cont i n ued g rowth of pe n i s, e n l a rgement of g l a n s

5

Ad u lt q u a l ity hair with spread to med i a l t h i g h s

Ad u lt a p peara n ce, a d u lt testi c u l a r vol u m e o f 1 2-27 m l

Ad u l t a p peara n ce

Data from Bord i n i B, Rosenfield R. Normal pu berta l development pa rt I I : C l i n ica l as pects of puberty. Pediatr Rev. 2 0 1 1 ;32(7):2 8 1 -292; a n d Nei n stein LS, e d . Handbook o fAdolescent Healthcare. P h i l a d e l ph ia, PA: Lippi ncott Wi l l ia m s & Wi l ki n s; 2009.

Your next patient is a 1 3 -year-old boy who is distressed because he reports that he is developing breasts. On exam you note that he has a firm, rubbery, mobile, and

tender 0.5 em mass under both nipples. His testicular exam reveals testicular volume of 6 mL and no masses. The remainder of his exam is normal, his growth and development is otherwise normal, and he is not on any medications.

GIRLS

HEIGHT SPU RT

MENARCHE

B REAST

G ROWTH RATE Height 2 in/y Weight 6 lb/y

SEXUAL MATU RITY 2 RATING

B reast and areola grow.

3

Nipple and areola form separate mound , protruding from breast.

Areola rejoins breast contour and development is complete. AGE RANGE 12.5 -18.5 y

4

5

I i r I�! r I� l r ll I l r 11 -

AGE

AGE RANGE ... 11.5-16.5 y AGE RANGE 10-16.5 y Average height 62.5 in (158.5 em) Average weight 106 lb (48 kg)

ffi ·1� � ��� [�1M bi�---B reast buds begin. AGE RANGE 8-13 y

PUBIC HAI R

Height 3 in/y Weight 17.5 lb/y

I n itial hair is straight and fine. AGE RANGE 8-14 y 11 y

-

Pubic hair Hair looks like an adult's but limited becomes coarse, darkens, in area. and spreads. 12 y 13 y



I nverted triangular pattern is established AGE RANGE 12.5-16.5 y 15 y 14 y

FIGURE 1-2. Adolescent fem a l e sexual matu ration a n d g rowth. (Reproduced with permission from Hay WW, Levin MJ, Deterd i n g RR, Abzug MJ, eds. Current Diagnosis and Treatment Pediatrics. 22nd ed. New York, NY: McGraw- H i l l Education, I nc., 20 1 4; Fig. 4-4.)

4

MCGRAW- H I L L E D U CAT I O N S P E C I A LTY BOARD REVI EW: P E D I ATRICS

APEX ST RE NGTH SPURT Height spurt 10-12 in (25-30 em) Weight 44 lb (20 kg)

BOYS

HEIGHT SPURT

G ROWTH RATE Height 2 in/y Weight 6.5 lb/y

Height 4 in/y Weight 20 lb/y

AGE RANGE 13-17.5 y

lh\1 llf\�IY\�Ii\�

PE NIS TESTES

Testes increase in size Penis grows in and skin of scrotum length. reddens. AGE RANGE 10-13.5 y 3 SEXUAL MATU RITY RATI NG

.......,,----.;;.2-,-...,

Penis grows in width.

4

Development is complete. AGE RANGE 14.5-18 y 5

PUBIC HAI R Straight hair appears at penis base. AGE RANGE 10-15 y AGE 11 y

12 y

Hair becomes curly, coarse, and dark.

13 y

Hair is full, limited in area.

14 y

15 y

Full development. AGE RANGE 14.5-18 y 16 y

17 y

FIGURE 1-3. Adolescent m a l e sexual maturation a n d g rowt h . (Reproduced with permission from Hay WW, Levin MJ, Deterd i n g RR, Abzug MJ, eds. Current Diagnosis and Treatment Pediatrics. 22nd ed. New York, NY: McGraw- H i l l Education, I nc., 20 1 4; Fig. 4-3.)

Question 2-1 The most likely diagnosis for this patient is: A) Pseudogynecomastia. B) Testicular tumor. C) Gynecomastia. D) Normal puberty. E) Phytoestrogen consumption. Discussion 2-1 The correct answer is "C:' Gynecomastia is a common con­ dition in adolescent males with a prevalence of 19.6% in 1 0 . 5 -year-old males, increasing to 64.6% by age 14 years. It is caused by an imbalance of estrogen to testosterone in pubertal males. The relative increased estrogen leads to proliferation of glandular breast tissue. It is important to differentiate gyneco­ mastia from pseudogynecomastia, which results from excess fat deposition as opposed to glandular tissue. In pseudogyne­ comastia, tissue tends to be more widely distributed and not localized to the nipple areolar complex. This patient has a nor­ mal testicular exam, making a testicular cancer unlikely. But take the opportunity to remind him to perform monthly self­ exams as testicular cancer usually presents as a painless mass. Soy products contain phytoestrogen, but eating tofu won't give you breasts. Have you ever seen an orchidometer? To avoid confusion, be sure to explain that it is standard practice to compare the patient's testicles to wooden beads on a string to determine the testicular volume.

Question 2-2 Management for this patient would include: A) Testicular ultrasound. B) Reassurance and follow-up exam in 6 months. C) Measurement of gonadotropins. D) Measurement of prolactin. E) Testosterone injections. Discussion 2-2 The correct answer is "B:' Given that this patient has a nor­ mal testicular exam and no signs of exogenous estrogen exposure or underlying disease, reassurance and follow up is appropriate.

� QUICKQUIZ You are seeing a 1 3-year-old boy who has testicular volume of 6 mL and light downy pubic hair. What is his SMR staging? A) Testicular volume SMR 2; pubic hair SMR 2. B) Testicular volume SMR 1 ; pubic hair SMR 2. C) Testicular volume SMR 2; pubic hair SMR 3 . D) Testicular volume SMR 3; pubic hair SMR 3 . E) Testicular volume SMR 2; pubic hair SMR 1 .

CHAPTER 1

Discussion The correct answer is ''A:' •

Helpful Tip

:5.� The average age of menarche in the U n ited States is

i1 1r 1 2.6 yea rs, with ra nge o f 1 1 .0 t o 1 4. 1 . Menarche occu rs

ea rlier in Africa n American a n d Mexica n American g i rls.



A D O L E S C E N T M E D I C I N E A N D GYN ECOLOGY

TA B L E 1-2 S EXUAL M ATU RITY RAT I N G ( S M R )

I N FEMALES

SMRin Females

Pubic Hair Development No pu bic h a i r

Prepu be rta l brea sts

2

Spa rse, d owny hair

Fo rmation of breast b u d , g l a n d u l a r tissue palpable u n d e r a reola; a reola i s s l i g htly widened a n d projects as a small mound

3

H a i r beco mes thicker, longer, a n d c u r l ier, sti l l i n l i m ited m i d line d i stri bution

E n l a rgement of breast with e l evation of breast conto u r a n d e n l a rgement o f a reola

4

Ad u lt type h a i r in q ua l ity but l i m ited d i stribution

Areo la fo rms a seco n d a ry m o u n d ove r conto u r of breast

5

Ad u lt q u a l ity h a i r with spread to med i a l t h i g h s

Fu l ly mature breast with conti n u o u s conto u r between a reola a n d breast

� QUICKQUIZ What is the most common breast mass in adolescent females? A) Fibroadenoma. B) Fibrocystic changes. C) Rhabdomyosarcoma. D) Hemangioma. E) Galactocele. Discussion The correct answer is "/\'. Most breast masses in adolescent girls are benign. A fibroadenoma feels like a rubbery, smooth, mobile, round mass. It is nontender and usually located in the upper outer quadrant of the breast. Other common benign masses include fibrocystic changes, cysts, abscesses, and fat necrosis from trauma. Options "D" and "E" are less common benign causes. Malignancy such as option "C" is a rare cause. Before a nipple is pierced or a hair is plucked, remember to counsel that both can cause an abscess.

You are seeing a 14-year-old girl in your office for her annual exam. When speaking with you confidentially, she mentions that she is concerned that she has not yet started her period like all of her friends. She reports breast devel­ opment starting approximately 1 year ago. On exam she has palpable breast tissue extending just beyond her areola and pubic hair that is thick and curly and primarily midline in distribution. Question 3-1 Which of the following most accurately describes her SMR staging? A) Breast SMR 1 ; pubic hair SMR 2. B) Breast SMR 3 ; pubic hair SMR 2. C) Breast SMR 1 ; pubic hair SMR 4. D) Breast SMR 3 ; pubic hair SMR 3. E) Breast SMR 5; pubic hair SMR 5. Discussion 3-1 The correct answer is "D:' The patient described in the vignette has breast and pubic hair development consistent with SMR 3 . (See Table 1 -2 for details.)

5

Breast Development

Data from Bord i n i B, Rosenfield R. Normal pu berta l development pa rt II: C l i n ical a spects of p u berty. Pediatr Rev. 2 0 1 1 ;32(7):28 1 -292; and Neinstein LS, ed. Handbook ofAdolescent Hea/thcare. Philadelphia, PA: Lippincott Wi l l ia m s & Wil kins; 2009.

Question 3-2 The first sign of puberty in females is typically: A) Breast development. B) Development of pubic hair. C) Menses. D) Body odor. E) Acne. Discussion 3-2 The correct answer is ''A:' Breast development (thelarche) is typically the first sign of puberty in girls and typically occurs between age 8 and 1 3 years (see Figure 1 -2 ) . Breast develop­ ment typically precedes pubarche (pubic hair development) though in some girls pubarche may occur first or simulta­ neously. Pubarche typically occurs 1 to 1 .5 years after breast development. Menarche occurs approximately 2.5 years after thelarche at an average age of 1 2 . 6 years in Caucasians and earlier in African Americans and Mexican Americans. Girls reach their peak height velocity of 8.25 cm/y earlier than boys at approximately SMR 3 . Peak height velocity in girls always precedes menarche. Peak height velocity occurs at approxi­ mately SMR 4 to 5 in boys.

MCGRAW- H I L L E D U CAT I O N S P E C I A LTY BOARD REVI EW: P E D I ATRICS

6





Helpful Tip



The fi rst stage of puberty for m a l es is testi c u l a r

1 1 1r enla rgement,

defi ned as a testis vol u m e of 4 m L

or g reater, or 2 . 5 e m i n d i a m eter. F o r g i rl s, i t is the



Helpful Tip

Don't forget that isolated G n RH d eficiency has been

1 1 1r associated with both Ka l l ma n syn drome a n d anosmia.

a ppea ra nce of b reast buds.

� QUICKQUIZ Which of the following is correct? A) Adrenarche results from testosterone secretion by the gonads. B) Activation of the hypothalamic-pituitary-gonadal axis (HPA) causes gonadarche and adrenarche. C) Estrogen secretion causes armpit hair development. D) All of the above. E) None of the above. Discussion The correct answer is "E:' Puberty encompasses gonadarche and adrenarche. Both are separate events, but the timing typically overlaps. Gonadarche, growth and maturation of the gonads (testes, ovaries) , is under the control of the HPA secretion of gonadotropin-releasing hormone (GnRH). Before puberty, release of GnRH from the hypothalamus is inhibited. Adrenal androgen secretion (dehydroepiandrosterone [DHEA] and androstenedione) causes pubic and axillary hair development, acne, and body odor (adrenarche) .



� I

Helpful Tip

Detection of noctu rnal l utei n izing hormone (LH) p u l ses

llr is the fi rst hormonal s i g n that puberty has sta rted . At pu berty, the hypoth a l a m u s is no longer i n h i bited a n d

releases G n R H i n a p u l satile fas h ion. G n R H sti m u lates the a nterior pitu ita ry to secrete gonadotropins-LH fi rst, then fol l icle-sti m u lating hormone (FSH). FSH and LH sti m u l ate the gonads to produce ga metes (eg g s or sperm) a n d sex hormones (estra d i o l or testosterone).

Question 3-3 What is the next step in management of this patient? A) Bone age. B) Measurement of gonadotropins. C) Reassurance and follow up in 6 months to a year if no menses. D) Thyroid studies. E) CBC. Discussion 3-3 The correct answer is "C:' This patient is progressing through the stages of puberty and will likely attain menarche in the next 6 months to 1 year. If no menses occur by age 1 6 further workup would be warranted.

A mother brings her 1 7-year-old son for his annual health maintenance exam. When you ask if she has any concerns about him, she mentions that he sleeps all the time. She also states that he was always a happy child but recently has become more withdrawn and always seems tired. She reports that his grades have been declining, and he does not seem to enjoy activities he previously enjoyed, such as playing soccer and video games with friends. Question 4- 1 The next step in diagnosing this patient is: A) Obtaining a CBC and iron studies. B) Obtaining a complete psychosocial history from the patient. C) Thyroid testing. D) Intelligence testing. E) Completion of Vanderbilt forms by parents and teachers. Discussion 4- 1 The correct answer is "B:' Adolescence is time of rapid growth and development. It can be a stressful time, manifesting as anxi­ ety, withdrawal, aggression, somatic complaints, depression, or poor coping skills such as using drugs. The most common causes of morbidity and mortality in adolescence are related to the risk-taking behavior and experimentation that is a normal part of adolescent development. Obtaining a thorough psychosocial history is important in screening for these risk-taking behaviors and identifying protective factors. The symptoms the mother has described raise concern about depression in this patient. A thorough psychosocial assessment using the HEADSSS screen­ ing tool with a follow-up depression screen will likely reveal the cause of his symptoms. How many knew that Vanderbilt scales assess for attention deficit hyperactivity disorder (ADHD) ?





1 llr

Helpful Tip

H EADSSS was developed as a psychosocial scree n i n g tooL H- Home (Who l ives with the teen? How does the teen

get a l o n g with fa m i ly?) E

-

Education (Is the teen in schoo l ? How is he or s h e

perform i n g i n s c h o o l ? S c h o o l performa nce can be a n i m porta nt i n d ication o f h o w a teen is fu nctio n i n g .) E- Eati ng (meal consi stency; body image) A - Activities D- Drugs (a lcohol, tobacco, m a rij u a na, and other drug

use, i n c l u d i n g prescription a n d over-the-cou nter)

CHAPTER 1

S

-

Sexua l ity (Sexual attraction: Are you attracted

to m a l es, fem a l es, both, neither? Sexua l behavior: Have you ever had sex, how m a ny partners, h i story of sexu a l l y tra n s m itted i nfection d i a g n osis and testi ng, condom use, contraceptive u se, last sexua l activity, h i story of forced sex?) S- Su icide/Depression S - Safety (Does the teen feel safe at home or school?

What is h i s or her exposure to violence?)

Through the HEADDSS assessment you learn that the patient is attracted to males and recently entered a relationship with a boy at school. He wrote a letter to the boy which another student found and shared with the whole class. Since then the patient reports that he is teased by many of his classmates and has been skipping classes to avoid being teased. His depres­ sion screen is positive for sadness, anhedonia (no pleasure in activities), excessive sleeping, and feelings of guilt. He denies any thoughts of self-harm or suicidality. Question 4-2 Adolescents who identify as lesbian, gay, or bisexual are at increased risk for which of the following? A) Eating disorders. B) Substance abuse. C) Depression. D) Bullying. E) All of the above. Discussion 4-2 The correct answer is "E:' Sexual development is one part of ado­ lescent development. During early adolescence pubertal develop­ ment is just beginning. At this stage of development adolescents are very focused on changes occurring in their bodies and question­ ing whether they are normal. Adolescents may begin to experience sexual fantasies and experience sexual pleasure through masturba­ tion. Sexual intercourse at this stage is uncommon, but may occur. Adolescents often experience crushes, which may be same sex or opposite sex. These patterns of attraction may or may not persist into future stages. In middle adolescence physical development is nearing completion; at this stage adolescents are forming their sex­ ual orientation and identity. Sexual experimentation is common at this stage and many adolescents may have intercourse for the first time. By late adolescence the goal is to become a sexually healthy adult with the ability to form long-lasting relationships. Sexual orientation refers to an individual's pattern of physical and emo­ tional attractions to others and involves complex components such as fantasies and feelings. Personal, family, cultural, developmen­ tal, and social factors can affect an individual's ability to identify, accept, and act on his or her attractions. Adolescents who identify as lesbian, gay, or bisexual are at an increased risk for a number of conditions, including eating disorders, substance abuse, and men­ tal health illnesses, particularly depression and anxiety. The patient has questions about how to stay safe when he does become sexually active with his partner.



A D O L E S C E N T M E D I C I N E A N D GYN ECOLOGY

7

Question 4-3 Which of the following is true about sexually transmitted infection (STI) transmission? A) HIV transmission rates are low with receptive anal intercourse. B) STis cannot be transmitted through oral sex. C) Women who have sex with women are at a low risk for STis. D) Condoms are effective at reducing STI transmission. Discussion 4-3 The correct answer is "D:' HIV transmission rates are high with receptive anal intercourse due to microtrauma during intercourse. STis can be transmitted through oral sex, and it is important to educate patients to use condoms when having oral sex to reduce the risk of contracting an STI. Studies have shown that women who have sex with women are at an increased risk of contracting human papillomavirus (HPV) , trichomoniasis, and HIV Condoms are an effective method of STI prevention and when properly used have been shown to reduce rates of transmission of HIV, gonorrhea, chlamydia, trichomonas, and hepatitis B. They can also be effective at preventing STis trans­ mitted by skin-to-skin or mucosal contact, such as herpes sim­ plex virus (HSV) , syphilis, and HPV, but only if the affected area is covered by the condom. Equally important is making sure adolescents know how to put on a condom. Pregame practice is a good idea. Question 4-4 You also counsel the patient that the most common cause of mortality in the adolescent population is: A) Cardiac disease. B) Unintentional injuries. C) Suicide. D) Homicide. E) Cancer. Discussion 4-4 The correct answer is "B:' The leading cause of death in the ado­ lescent population is unintentional injuries, with motor vehicle collisions being the most frequent cause of such injury in this population. (See Figure 1 -4.) Homicide is the second leading cause of death, followed closely by suicide. Organic disease is a less frequent cause of mortality in this age group. Screening for risk factors such as substance use, mental illness, and expo­ sure to violence is important in this population to address the leading causes of mortality. Good driving habits, including no texting while driving, should be discussed.

� QUICKQUIZ Which of the following is a risk factor for suicide? A) Bullying. B) Witnessing violence. C) Social isolation. D) Mental illness. E) All of the above.

8

MCGRAW- H I L L E D U CAT I O N S P E C I A LTY BOARD REVI EW: P E D I ATRICS

U n i ntentional i n j u ry

1 - ---------------' ]

Homicide

I

Suicide

I

Malignant neoplasms

H eart d isease

t:::J � 0

2000

4000

6000

8000

1 0000 1 2000 1 4000

FIGURE 1-4. 20 1 0 Lea d i n g causes of death in youth ages 1 5 to 24 years of age in the U n ited States. (Reproduced with permission from the Centers for Disease Control a n d Prevention, National Center for I nj u ry Prevention a n d Control, Web-based I nj u ry Statistics Query a n d Reporting System (WI SQARS). Accessed J a n u a ry 28, 20 1 5 from http://www.cdc.gov/inj u ry/wisqa rs/.)

Discussion The correct answer is "E:' Additional risk factors include family history of suicide, history of abuse, previous attempt, access to means such as firearms, alcohol and drug use, stressful events, and sexual identification other than heterosexuality. The patient's mother mentions that she recently caught him smoking pot in the garage with some friends. She is request­ ing that you drug test him without letting him know. Question 4-5 Your next step is to: A) Do as the mother requests. B) Notify the patient and perform testing regardless of his wishes. C) Notify the patient and perform the testing if he agrees. D) Refuse to perform drug testing. E) Reassure the mother that catching him guarantees he will stop using. Discussion 4-5 The correct answer is "C:' Recreational drug use is an under­ recognized cause of morbidity and mortality in adolescents. Indications for drug testing in the acute care setting include acute presentation with altered mental status, suicide attempt, unexplained seizures, syncope, arrhythmias, or the pres­ ence of toxidromal signs. In the primary care setting volun­ tary drug testing can be helpful for assessment, therapy, and monitoring. The American Academy of Pediatrics (AAP) cur­ rently cautions against involuntary drug testing of adolescents in nonemergent settings. Testing of competent adolescents without their knowledge is unethical and illegal, and without their consent is impractical. If a pediatrician suspects that a patient is abusing drugs and the patient refuses drug testing, documentation of the refusal and referral to a mental health or addiction specialist may be warranted. Given the limitations of currently available drug tests, a thorough substance abuse history often provides more useful information on drug abuse/ use than a drug test.

A 1 6-year-old girl and her mother present to your office with concerns about irregular periods. The patient had her first menses at 12 years of age and had regular monthly periods until 6 months ago when her periods stopped. She has had an accompanying 50-pound weight loss over the past 6 months. When asked further about the weight loss, she reports that she has been working on more healthful eating, has cut all desserts and junk foods out of her diet, and eats a low-fat and low-carb diet. In addition she has started running 3 miles a day in order to "get healthy?' On physical exam her vital signs are temperature 36.4°C {97.5°F) , heart rate 44 beats per min­ ute, blood pressure 96/60 mm Hg, and respirations 16 breaths per minute. She appears thin, with sallow-looking skin and dry hair. She is bradycardic on exam, with no murmurs and a regular rhythm. Her heart rate increases by 19 beats during positional changes from sitting to standing, with minimal change in her blood pressure. Her pulses are strong and sym­ metric while her fingers and toes are cool to touch. Question 5-1 Which of the following is the most likely cause of this patient's symptoms? A) Thyroid disease. B) Anorexia nervosa. C) Bulimia nervosa. D) Diabetes mellitus. E) Coarctation of the aorta. Discussion 5-1 The correct answer is "B:' Eating disorders are a common but often underdiagnosed condition in the pediatric population. The 12-month prevalence of anorexia nervosa among young females in approximately 0.4%, the prevalence of bulimia nervosa is approximately 1% to 1 .5%. Anorexia nervosa has a mortality rate of 5% to 6%, the highest of any psychiatric illness. Patients with anorexia nervosa generally present with rapid weight loss sec­ ondary to caloric restriction, which may present as elimination of "junk food" from the diet; avoidance of certain food groups, such as carbohydrates and fats; or changing to a restrictive vegan or vegetarian diet. Patients may also try and reduce weight by over-exercising or purging through self-induced vomiting or use of diuretics and laxatives. In contrast, patients with bulimia nervosa typically present with cycles ofbinging that trigger purg­ ing or inappropriate compensatory behaviors. Compensatory behaviors could include self-induced vomiting, use of diuretics, use of laxatives, fasting, or over-exercising. Patients with bulimia nervosa are typically of normal weight or overweight. Patients with anorexia nervosa often present with signs of malnutrition (eg, bradycardia); hair, skin, and nail changes, often manifesting as dry and brittle hair, nails, and skin; menstrual irregularities ( eg, amenorrhea and oligomenorrhea); orthostatic vital sign changes; cold intolerance; acrocyanosis; mood changes; and fatigue. The DSM is revised periodically and eating disorder diagnostic cri­ teria often change. Due to copyright restrictions, we are unable to print the DSM-5 diagnostic criteria for anorexia and bulimia.

CHAPTER 1



Restricted eati ng

A D O L E S C E N T M E D I C I N E A N D GYN ECOLOGY



Syncope



Hypokalemia

Severe hypochloremia ( 2 d ays a week for symptom reliefg e n e rally i n d icates i n adeq uate control and the need to step u p treatment.

FIGURE 2-4. Stepwise a p p roach for m a n a g i n g asth m a . ICS, i n h a led corticosteroid; LABA, long-acting beta 2-agonist; LTRA, l e u kotriene receptor a ntagonist; SABA, short-acting beta 2 -agonist. (Reprod uced with permission from the Natio nal Education a n d Prevention Program, Expert Pa nel 3. Guidelines for the

Diagnosis and Management ofAsthma. N I H Publ ication N u m ber 08-5846. Bethesda, MD: National I n stitute of Health, Octo ber 2007.)

42

MCG RAW-H I LL EDUCATION SPECIALTY BOARD REVI EW: PEDIATRICS



Discussion 1 3-4

Helpful Tip

� In persistent asthma, ICS are the most effective long­

=

i1 1r term controller medication and the preferred choice in

all age grou ps. To minimize adverse effects, the lowest ICS dose necessa ry to maintain control should be used.



Helpful Tip

� A LABA such

=

as sa lmeterol should not be used as a

i1 1 r quick-relief medication or monotherapy for long-term

control. As a controller medication, a LABA is combined with inhaled corticosteroids to treat moderate to severe persistent asthma.

The correct answer is "E:' The ECR-3 asthma guidelines out­ line four major components of asthma management: ( 1 ) objec­ tive measurement of lung function (spirometry, exam, history) to assess severity and monitor control, (2) patient and family education, (3) elimination of environmental factors that cause symptoms or exacerbations, and ( 4) pharmacologic therapy for both maintenance and exacerbations. Spirometry is rec­ ommended for all children 5 years of age or older for diagnos­ ing and managing asthma. To properly understand whether pharmacologic therapy is working, it is necessary to confirm good compliance and appropriate medication administration technique.



He is started on a low-dose ICS/LABA combination. Question 1 3-3 What is the appropriate time for him to be seen in follow-up to evaluate if he has achieved better control of his asthma? A) 2 to 5 days. B) 2 to 6 weeks. C) 9 to 12 weeks. D) 3 to 6 months. E) No follow up is necessary. Discussion 1 3-3

The correct answer is "B:' Two to 6 weeks is the recommended time to evaluate the level of control that has been achieved. The goal of asthma therapy is to maintain long-term control with the least amount of medication. Responsiveness to treatment is variable with each patient, and to determine if goals of therapy are being met, follow-up assessments are critical. At follow-up, decisions to increase or decrease (when possible) the dose of medication, number of medications, or frequency of medica­ tion administration is determined by the degree to which the goals of therapy are being met (level of control) . He returns for a follow-up evaluation on his low-dose ICS/ LABA medication. In the past month, he has had symptoms throughout the night, has nighttime awakenings from wheez­ ing every night of the week, and is also requiring albuterol several times per day. Question 1 3-4 In addition to prescribing medications, what else is helpful in managing his asthma as an outpatient? A) Asthma education and creating an asthma action plan. B) Pulmonary function testing (PFTs). C) Control of environmental factors. D) Assessing appropriate compliance and medication adminis­ tration technique. E) All of the above.

Helpful Tip

=-� In children, FEV,fFVC is a more sensitive measurement

i1 1r for classifying asthma severity and monitoring control than FEV, .

Question 1 3-5 How well controlled is the adolescent's asthma currently? A) Too well controlled. B) Well controlled. C) Not well controlled. D) Very poorly controlled. E) It is too early to make an appropriate assessment. Discussion 1 3-5

The correct answer is "D:' Once diagnosed, the focus of asthma management depends on whether or not the disease is controlled. The terms used to describe disease control are well, not well, or very poorly controlled. Control is based on the same components used to classify asthma severity. In addi­ tion, loss of lung function and adverse effects of treatment are considered. In general, patients with asthma that is well con­ trolled have daytime symptoms or SABA use 2 days or more per week, nighttime symptoms 2 nights per month or less, and exacerbations requiring oral corticosteroids less than 2 times per year. The main distinction between asthma that is not well controlled and very poorly controlled asthma is the presence of symptoms throughout the day, nightly, the use of albuterol several times per day, and an extreme limitation to daily activi­ ties. (See Figure 2-5.)



Helpful Tip

=-� At the time of d iagnosis and start of therapy, asthma

i1 1r is classified by severity (ie, persistent or intermittent). Afterwa rd the focus is on the deg ree of control, which wil l guide clinica l management, including medication adjustment.

CHAPTER 2



ALLERGIC AND I M M U NOLOG IC DISORDERS

43

Classificatio n of asthm a control (age � 1 2 years)

Com ponents of control Well controlled

N i g htime awake n i n g s I nterfe rence with normal activity

I m pairment

SABA use for symptom control FEV1 o r peak flow

> 2 days/week

,; 2 days/week

Sym ptom s

>

Not well controlled not daily

,; 2x/month

3-4x/month

None

Minor l i m itation

,; 2 d ays/week

> 2 days/week

80%

60-80%

pred icted/

pred icted/

personal best

personal best

Throughout the day

7xlwee k

Extre m e l y l l m ltad

Several time per day

egg > pea­ nut > tree nuts > shellfish > fish > wheat > soy. It is important to tell patients and families that similar foods may cross-react. For example, a child with a cashew allergy may be allergic to other tree nuts. Testing for allergies to similar foods may be indicated before these foods are introduced into the diet.

Discussion 1 3- 1 1

The correct answer is "B:' Metabolic disturbances, such as hyperglycemia and hypokalemia have been reported as a form ofbeta-agonist toxicity. Other side effects include tremor, tachy­ cardia, palpitations, nausea, and vomiting. Serious toxicity is rare and generally seen with systemic therapy (ie, intravenous) . Levalbuterol (pure R-isomer of albuterol) produces similar side effects as albuterol (racemic mixture of R- and S-isomers).



Helpful Tip

=-� In

oral al lergy syndrome or pol len-food al lergy,

r1 1 r symptoms of throat itching and angioedema of lips occur after eating certain raw fruits (eg, a pple, kiwi) and vegetables (eg, potato, carrot) in individuals with pollen al lergies. The food antigens cross-react with pollen antigens, causing localized symptoms. Cooked or baked forms of the food may be tolerated as the causative proteins are heat labile.

� QUICKQUIZ Which is NOT a risk factor for developing asthma? A) Parent with asthma. B) Atopic dermatitis. C) Dust mite allergy. D) Elevated serum IgE. E) All of the above. Discussion

The correct answer is "E:' Asthma has a familial component especially if the mother or father is affected. Atopic dermati­ tis and allergic rhinitis are associated with asthma. Other risk factors include recurrent wheezing and wheezing with viral respiratory tract infections, sensitization to indoor allergens

The mother has been eliminating peanuts and peanut butter from her son's diet, but she has questions about other foods that may contain peanut oil. Question 1 4-2 What would you tell the mother about exposure to peanut oil? A) Hot pressed peanut oil typically does not cause reactions in peanut-allergic children. B) Cold pressed peanut oil typically does not cause reactions in peanut-allergic children. C) Hot pressed peanut oil can potentially trigger reactions, and should be avoided in peanut-allergic children. D) All peanut oil should be avoided completely in any peanut­ allergic child.

46

MCG RAW-H I LL EDUCATION SPECIALTY BOARD REVI EW: PEDIATRICS

Discussion 1 4-2

The correct answer is ''A:' Hot pressed peanut oil typically does not cause reactions in peanut-allergic children as there is insufficient protein to bind mast cell-bound peanut-specific IgE. Cold com­ pressed oils, however, allow enough protein through the refining process to potentially trigger reactions, and thus should be avoided.



Helpful Tip

� In general, delayed introduction (after 1 1 1r

4

to 6 months of age) of highly al lergenic foods is not recommended and may increase the risk of al lergy development. Infants with atopic disease may benefit from testing or al lergist evaluation prior to the introduction of certain foods.

The infant is brought back to your office a few months later with concerns that he had an anaphylactic reaction to another food. The previous week, the family was eating at a picnic for lunch. The picnic consisted of potato salad and hotdogs. Later that night (8 hours later), he began to have vomiting and diar­ rhea that persisted for 3 days. His mother is concerned that he had an anaphylactic reaction and is allergic to potatoes. Question 1 4-3 What additional information would speak against the pos­ sibility of his symptoms being caused by anaphylaxis? A) He had eaten both of those foods previously and tolerated them well. B) Others in the family had similar symptoms. C) The timing of onset and duration of gastrointestinal symptoms. D) The absence of other systemic symptoms. E) All of the above.

Discussion 1 4-3

The correct answer is "E:' All of the above choices support a diagnosis of food poisoning rather than anaphylaxis. Anaphy­ laxis is a potentially life-threatening reaction that can affect breathing and send the body into shock. Typically, these reac­ tions simultaneously affect different parts of the body ( eg, vom­ iting and diarrhea accompanied by hives) . Most food-related anaphylaxis symptoms occur within 2 hours of ingestion; often they start within minutes. Typically symptoms do not occur in foods that were previously tolerated. Symptoms typically resolve within 24 hours of onset. Other family members having similar symptoms would support the possibility of food poisoning. The mother asks if her son will outgrow his peanut allergy as he gets older. You tell her that it is a possibility but unlikely. Question 1 4-4 Which food allergy is a child LEAST likely to outgrow? A) Almond. B) Soy.

C) Wheat. D) Milk. E) Egg. Discussion 1 4-4

The correct answer is ''A:' Children are most likely to outgrow their allergic reactions to milk, wheat, soy, and eggs, with some studies indicating between 65% and 80% remittance. Peanut, tree nut, shellfish, and fish allergies are likely to persist. Roughly 1 in 5 children will outgrow peanut allergy, with 1 in 1 0 out­ growing tree nut allergy. Over the next year, he has increasing problems with atopic dermatitis. You have prescribed him topical emollients and high-dose topical steroids without good control of symp­ toms. His typical daily diet consists of eggs, milk, wheat, and soy. He infrequently eats apples, melon, broccoli, tomato, chicken, and beef. He continues to avoid peanuts. Question 1 4-5 Of the foods below, what could be the most likely to contrib­ ute to his persistent moderate to severe atopic dermatitis? A) Peanut. B) Chicken. C) Egg. D) Broccoli. E) Tree nuts. Discussion 1 4-5

The correct answer is "C:' Food allergy can be a contributor to moderate to severe atopic dermatitis. Between 50% and 60% of children with moderate to severe atopic dermatitis have IgE­ mediated food allergy. Unlike the immediate IgE-mediated reactions of hives and anaphylaxis with foods, children with atopic dermatitis and food allergy may have a delayed reaction presenting as an acute flare or worsening of their skin disease. It is recommended that children younger than 5 years of age who have moderate to severe atopic dermatitis be evaluated for milk, egg, peanut, wheat, and soy allergies if the child con­ tinues to have atopic dermatitis even after treatment. Empiric elimination without evidence of IgE-mediated allergy is not recommended.

� QUICKQUIZ Which is NOT a method for diagnosing an IgE-mediated food allergy? A) Intradermal (intracutaneous) skin testing B) Skin prick testing. C) Serum allergen IgE testing. D) Oral food challenges. Discussion

The correct answer is ''A:' Useful tools to diagnosis an IgE-medi­ ated food allergy include skin prick testing and serum allergen

CHAPTER 2

IgE testing. Remember to diagnosis an allergy the child must have evidence of sensitization (positive test result) and clinical symptoms when the food is ingested. Intradermal testing is not recommended for food allergies. Panel testing for a large num­ ber of foods should be avoided. Remember only a small number of foods cause the majority of IgE-mediated reactions, there­ fore; testing should focus on the suspected food(s) or the most common triggering foods (eg, peanut, milk, egg) . If testing is negative but suspicion remains high or the history is uncertain, an oral food challenge should be performed. Oral food chal­ lenges are also helpful to determine whether or not an allergy has resolved.

You are treating an 8-year-old girl with recurrent acute otitis media for an ear infection. She is typically treated successfully with amoxicillin or amoxicillin/clavulanate (Augmentin) . On day 7 of her 10-day course of amoxicillin, she develops hives. You advise her mother to stop giving her this medication immediately. The hives continue for 8 days before resolving. In a follow-up call the mother reports that her daughter has no arthritis symptoms, other nonurticarial rashes, angioedema, or respiratory symptoms. Question 1 5- 1 What type o f hypersensitivity reaction i s she having? A) Type I IgE-mediated mast cell degranulation to amoxicillin. B) Type II cytotoxic antibody (IgG) formation to amoxicillin.



ALLERGIC AND I M M U NOLOG IC DISORDERS

47

C) Type III immune complex formation and deposition in response to amoxicillin. D) Type IV delayed type hypersensitivity reaction to amoxicillin. E) None of the above; this reaction is not likely due to amoxicillin. Discussion 1 5- 1

The correct answer is "E:' The fact that the medication has been previously tolerated on multiple occasions, the onset was after 7 days of use, and resolution came 8 days after stopping the medi­ cation make the possibility of this being an IgE-mediated allergic reaction less likely. A type I IgE-mediated hypersensitivity reac­ tion to a medication typically occurs within the first few expo­ sures to a new medication. The onset of symptoms is also typically during the first few doses of the medication. Symptoms typically resolve within 1 to 2 days of discontinuation of the medication. Her symptoms are also not consistent with a type II, type III, or type IV hypersensitivity reaction. Examples of such reactions would be: type II reaction-new anemia while on a medication; type III -fever, arthritis, lymphadenopathy, and hives after taking the medication for more than 1 week; and type IV-macular rash several days after starting the medication. However, rather than a reaction to the medication itself, the girl's hives are most likely related to the underlying infection. Hives in response to infections are the most common identifiable cause of hives in children. In children infections were identified in 57% of acute urticaria cases with viral upper respiratory tract or digestive infections being the most common. As a virus stimulates the immune system, mast cells can be triggered to degranulate and cause hives. This is usu­ ally self-limited process that is well controlled with higher doses of nonsedating antihistamines. (See Table 2-6.)

TA B L E 2-6 COO M B S AND G E L L CLASS I F I CATI O N O F HYP E R S E N S ITIVITY R EACTI O N S

Type I

Mechanism

Example

Description

l g E-mediated mast ce l l or baso p h i l degra n u lation

Anaphylaxis, u rticaria

Free a ntigens cross- l i n k the l g E on mast ce l l s a n d basop h i l s, which ca uses re lease of vasoactive biomolecules On set: m i n utes to h o u rs M u st have prior expo s u re to a ntigen (se n s itization)

Type II

Type Ill

Type IV

Auto i m m u ne hemo­ lytic a n e m ia, Good­ past u re syn d rome, d rug-ind uced t h ro m bocytope n i a

l g M or lgG bind to a ntigen o n a ta rget host cel l that i s per­ ceived by i m m u n e syste m as foreig n , lead i n g to cel l u la r destruction

I m m u ne co m p lex fo rmation a n d deposition lead i n g t o co m p lement activation

Serum sickness, posti nfectious g l omeruloneph ritis

Anti body (lgG) b i n d s to so l u b l e antigen, fo rm i n g c i rcu lati ng i m m u n e co m pl exes that a re deposited i n b l ood vessel wa l l s (endothel i u m), tiss ues, or both, i n itiat i n g a loca l i n fl a m matory reaction

Del ayed hypersensitivity

Contact dermatitis, posi­ tive tuberc u l i n ski n test

Cytotoxic a nti body formation

On set: m i n utes to h o u rs

On set: h o u rs H e l per T ce l l s a re activated by an a ntigen-presenting ce l l ; w h e n the a ntigen i s prese nted a g a i n i n t h e futu re, memory T cel l s activate macrophag es, ca u s i n g a n i n fl a m m atory respon se; t h i s u ltimately can lead to tissue damage Occ u rs over 1 -2 d ays

48

MCG RAW-H I LL EDUCATION SPECIALTY BOARD REVI EW: PEDIATRICS and is most severe on his popliteal fossa and antecubital fossa bilaterally.

A 5-year-old develops diffuse hives and swelling 1 5 minutes after his second dose of a sulfonamide-containing antibiotic. His symptoms resolve completely after taking diphenhydr­ amine. He has not taken any more doses of this medication. One week later, his mother brings him to your office and asks you if he is allergic to "sulfa drugs?' Question 1 6- 1 How should a drug allergy b e diagnosed i n this patient? A) Skin testing. B) Blood testing (specific IgE). C) Patch testing. D) Oral challenge. E) The diagnosis can be made by history alone. Discussion 1 6- 1

The correct answer is "E:' Allergy testing by skin or blood test to most drugs is not standardized and has very little data to sup­ port interpretations of results. Penicillin is the only drug for which a validated IgE skin test can be performed. However, his symptoms are very suggestive of an IgE-mediated type 1 hyper­ sensitivity reaction. An oral challenge may cause an anaphylac­ tic reaction and is not recommended. Based on his history and description of symptoms, he should be considered to have an allergy to sulfonamide-containing medications. Patch testing is used to identify contact allergens.

� QUICKQUIZ Which drug is NOT capable of causing direct mast cell degranulation? A) Vancomycin. B) Opioids. C) Fluoroquinolones. D) Penicillin. Discussion

Question 17-1 What is the most likely cause of his symptoms? A) Urticaria. B) Psoriasis. C) Atopic dermatitis. D) Molluscum contagiosum. E) Pityriasis rosea. Discussion 1 7- 1

The correct answer is "C:' Atopic dermatitis or eczema is an inflammatory skin condition characterized by erythema, pru­ ritus, and scaly rashes. It often appears on the arms, legs, hands, and face. The severe pruritus of atopic dermatitis can often dis­ turb sleep. Excessive scratching and excoriation of the skin can lead to superinfection. Approximately 60% of patients develop atopic dermatitis by age 1 , and another 30% experience symp­ toms by age 5. Children born into families that have a his­ tory of atopic diseases (eg, asthma or allergic rhinitis) are at an increased risk for developing eczema. Atopic dermatitis is considered to be part of the "atopic march;' which involves the development of atopic dermatitis, food allergy, allergic rhinitis, and asthma, usually in that sequential order.

� QUICKQUIZ Which of the following is/are true regarding the pathogenesis of atopic dermatitis? A) Impaired skin barrier function. B) Increased skin transepidermal water loss. C) Increased skin entry of allergens, antigens, and environ­ mental chemicals. D) All of the above. Discussion

The correct answer is "D:' In a penicillin allergy, mast cell degranulation is IgE-mediated (type 1 hypersensitivity reac­ tion). The other choices cause direct mast cell degranulation (non-IgE-mediated reaction) . Radiocontrast dye, mannitol, intravenous iron, N-acetylcysteine, corticosteroids, blood prod­ ucts, and some chemotherapy agents may also cause direct mast cell degranulation. This type of reaction was formerly called an anaphylactoid reaction and is not a true allergy.

The correct answer is "D:' An intact, healthy skin barrier is a critical first line of defense against various microbes, irritants, and allergens. The epidermis of atopic dermatitis patients is characterized by significant barrier disruption, which leads to increase transepidermal water loss. This may also be why symp­ toms are worse in the winter months when the climate is drier. Patients with atopic dermatitis also have an increased suscepti­ bility to allergic sensitization as well as microbial colonization and infections.

A 3-year-old boy is brought for evaluation of a chronic rash that has been intermittently flaring in the past year. The rash was worse in the winter months. It is dry, pruritic, and patchy

Question 1 7-2 What is the best initial treatment for the child's symptoms? A) Topical antihistamine cream. B) Oral antihistamine medications. C) Topical emollients and topical corticosteroids. D) Topical emollients and oral corticosteroids. E) Topical corticosteroid only.

CHAPTER 2



ALLERGIC AND I M M U NOLOG IC DISORDERS

49

Discussion 1 7-2

Discussion 1 7-4

The correct answer is "C:' A topical emollient (or moisturizer) helps repair the skin barrier and prevent water loss. Ointments or creams (not lotions) are preferred. The emollient should be applied immediately after bathing while the skin is still wet, to help lock moisture in the skin. The topical corticosteroid treats the underlying inflammatory process of atopic dermatitis, reducing inflammation, easing irritation, and reducing pruritus.

The correct answer is "E:' All of the listed options can be poten­ tial complications of atopic dermatitis. Lichen simplex chroni­ cus (or neurodermatitis) is a skin condition caused by chronic scratching of the skin. This condition can cause the affected skin to become discolored, thick, and leathery. Repeated scratching that breaks the skin can cause open sores and cracks. These increase the risk of infection from bacteria and viruses. Allergic contact dermatitis is common in patients with atopic dermatitis. This may be related to a compromised skin barrier. The itch­ scratch cycle can cause children to awaken repeatedly at night and decrease the quality of sleep.

You see him for a follow-up visit. While his lower extrem­ ity atopic dermatitis has improved, he continues to have per­ sistent cracked, excoriated, dry patches on his antecubital fossa bilaterally. He had food allergy testing that revealed no underlying food allergies. Question 1 7-3 What is another recommended treatment for his severe atopic dermatitis? A) Continue his current medication regimen and add the application of wet bandages on his affected skin patches. B) Continue his current medication regimen and add the use of calamine lotion. C) Discontinuing the topical corticosteroid and double the dose of his daily emollient. D) Continue his current medication regimen and start an empiric egg elimination diet. E) None of the above. Discussion 1 7-3

The correct answer is "A:' The use of wet bandages or wet wraps has been proven to be an effective treatment in severe atopic dermatitis. Wet wraps serve a few different functions. As water gradually evaporates from the bandages this cooling of the skin and helps relieve inflammation, itching, and soreness. Emol­ lients applied to the skin and then wrapped with wet bandages are deeply absorbed into the skin to provide a longer lasting moisturizing effect. In a similar manner, there is an enhanced absorption of topical steroid molecules into both the superficial and deeper layers of skin where inflammation is present. The bandages provide protection from the itching and scratching cycle so that skin gets a chance to heal properly. This has been proven to control signs and symptoms within hours to days of application. Wet wrapping can be done at home, but if the area of application is large, it is sometimes it is done in a hospital because it can be labor intensive and require nursing expertise. The mother is concerned about the long-term complications of her son's atopic dermatitis if it persists. Question 1 7-4 Which of the following are potential complications of poorly controlled atopic dermatitis? A) Sleep problems. B) Lichen simplex chronicus. C) Skin infections. D) Allergic contact dermatitis. E) All of the above.

B I B LIOGRAPHY

Bernstein IL, Li JT, Bernstein DI, et al. Allergy diagnostic test­ ing: An updated practice parameter. Ann Allergy Asthma Immunol. 2008 ; 1 00(3 suppl 3 ) : S 1 - 148. Boguniewicz M, Leung DY. Atopic dermatitis: A disease of altered skin barrier and immune dysregulation. Immunol Rev. 20 1 1 ;242 ( 1 ) :233-246. Buckley RH. Primary immunodeficiency diseases. In: Adkinson N, Busse W, Bochner B, et al, eds, Middleton's Allergy: Prin­ ciples and Practice. 7th ed. Philadelphia, PA: Mosby; 2008: 8 0 1 -829. Cox L, Nelson H, Lockey R, et al. Allergen immunotherapy: A practice parameter third update. J Allergy Clin Immunol. 20 1 1 ; 127( 1 suppl) : S 1 -55. doi: 10 . 1 0 1 6/j .jaci.20 1 0.09.034. Dezateux C, Stocks J, Dundas I, et al. Impaired airway func­ tion and wheezing in infancy: The influence of maternal smoking and genetic predisposition to asthma. Am J Respir Crit Care Med. 1 999; 1 59:403-410. Expert Panel Report 3 (EPR- 3 ) . Guidelines for the Diagnosis and Management of Asthma-Full Report 2007. NIH pub­ lication No. 08-5846. Bethesda, MD: US Department of Health and Human Services; National Institutes of Health; National Heart, Lung, and Blood Institute; National Asthma Education and Prevention Program; 2007. Frew AJ. Allergen immunotherapy. J Allergy Clin Immunol. 20 1 0; 1 25:S306-S3 1 3 . Geha R , Notarangelo L . Case Studies i n Immunology: A Clini­ cal Companion. 6th ed. New York, NY: Garland Science; 20 1 1 . Gell PGH, Coombs RRA, eds. Clinical Aspects of Immunology. Oxford, England: Blackwell; 1 963. Holt PG, Macaubas C, Stumbles PA, et al. The role of allergy in the development of asthma. Nature. 1 999;402 : B 1 2-B 1 7 . Illi S, von Mutius E, Lau S, e t al. Perennial allergen sensitiza­ tion early in life and chronic asthma in children: A birth cohort study. Lancet. 2006;368:763-770. Jackson DJ, Lemanske RF Jr. The role of respiratory infections in childhood asthma inception. Immunol Allergy Clin North Am. 20 1 0;30: 5 1 3-522. Johnston SL, Pattemore PK, Sanderson G, et al. Community study of role of viral infections in exacerbations of asthma in 9- 1 1 year old children. EMf. 1 995;3 1 0 : 1 225- 1 229. Kaplan AP. Urticaria and angioedema. In: Adkinson N, Busse W, Bochner B, et al, eds. Middleton's Allergy:

50

MCG RAW-H I LL EDUCATION SPECIALTY BOARD REVI EW: PEDIATRICS

Principles and Practice. 7th ed. Philadelphia, PA: Mosby; 2008 : 1 063 - 108 1 . Kelso JM. A second dose of epinephrine for anaphylaxis: Epi­ nephrine for anaphylaxis: How often needed and how to carry? J Allergy Clin Immunol. 2006; 1 1 7:464-465. Korenblatt P, Lundie MJ, Dankner RE, et al. A retrospective study of epinephrine administration for anaphylaxis: How many doses are needed? Allergy Asthma Proc. 1 999;20:383-386. Kuyper LM, Pare PD, Hogg JC, et al. Characterization of airway plugging in fatal asthma. Am J Med. 2003; 1 1 5( 1 ) : 6- 1 1 . Lieberman PL. Anaphylaxis. In: Adkinson N, Busse W, Bochner B, et al, eds. Middleton's Allergy: Principles and Practice. 7th ed. Philadelphia, PA: Mosby; 2008 : 1 027- 1 049. Miller EK, Edwards KM, Weinberg GA, et al. A novel group of rhinoviruses is associated with asthma hospitalizations. J Allergy Clin Immunol. 2009; 123 :98- 1 04. Morgan WJ, Stern DA, Sherrill DL, et al. Outcome of asthma and wheezing in the first 6 years of life: follow­ up through adolescence. Am J Respir Crit Care Med. 2005; 1 72: 1 2 53 - 1 258. Nelson HS. Immunotherapy for inhalant allergens. In: Adkinson N, Busse W, Bochner B, et al, eds. Middleton's Allergy: Prin­ ciples and Practice. 7th ed. Philadelphia, PA: Mosby, 2008: 1 657- 1 677. Parameswaran K, Beida J, Rowe BH. Addition of intravenous aminophylline to beta2-agonists in adults with acute asthma. Cochrane Database Syst Rev. 2000;(4) :CD002742. Pauwels RA, Lofdahl CG, Postma DS, et al. Effect of inhaled formoterol and budesonide on exacerbations of asthma. N Engl J Med. 1 997;337: 1405- 1 4 1 1 . Platts-Mills TA. The role of immunoglobulin E in allergy and asthma. Am J Respir Crit Care Med. 200 1 ; 1 64:S1 -S5. Plotnick LH, Ducharme FM. Combined inhaled anticho­ linergics and beta2-agonists for initial treatment of acute asthma in children. Cochrane Database Syst Rev. 2000; ( 4): CD000060. Polito AJ, Proud D. Epithelia cells as regulators of airway inflammation. J Allergy Clin Immunol. 1 998; 1 02(5): 7 14-7 1 8 . Review. Sampson HA Aceves S, Bock SA, et al. Food allergy: A practice parameter update-20 14. J Allergy Clin ,

Immunol. 2014; 1 34(5), 1 0 1 6 - 1 025.e 1 043. doi: 1 0 . 1 0 1 6/j. jaci.20 14.05.0 1 3 . Sampson HA Burks WA. Adverse reactions to foods. In: Adkinson N, Busse W, Bochner B, et al, eds. Middleton's Allergy: Principles and Practice. 7th ed. Philadelphia, PA: Mosby; 2008 : 1 1 39- 1 1 67. Seidman MD, Gurgel RK, Lin SY, et al. Clinical practice guide­ line: Allergic rhinitis. Otolaryngology Head Neck Surg. 20 1 5; 1 52 ( 1 suppl) : S 1 -S43. doi: 1 0. 1 1 77/0 1 945998 1456 1 600 Simons FE, Ardusso LR, Bilo MB, et al. World allergy organi­ zation guidelines for the assessment and management of anaphylaxis. World Allergy Organ f. 20 1 1 ;4(2) : 1 3-37. doi: 10 . 1 097/WOX.Ob0 1 3e3 1 8 2 1 1496c. Spahn JD, Covar R, Szefer SJ. Glucocorticoids: B. Clinical sci­ ence. In: Adkinson NJ, Bochner BS, Busse WW, et al, eds. Middleton's Allergy: Principles and Practice. 6th ed. Phila­ delphia: Mosby; 2003:887-9 1 3 . Tager IB, Ngo L , Hanrahan J P. Maternal smoking during preg­ nancy: Effects on lung function during the first 1 8 months of life. Am J Respir Crit Care Med. 1 995; 1 52:977-983. Thomas A, Lemanske RF, Jackson DJ. Approaches to stepping up and stepping down care in asthmatic patients. J Allergy Clin Immunol. 20 1 1 ; 128:91 5-924. Thomas AO, Lemanske RF Jr, Jackson DJ. Infections and their role in childhood asthma inception. Pediatr Allergy Immunol. 2014;25: 1 22- 128. TREC screening can show newborn's T cells are low. July 20 1 3 . American Academy o f Allergy Asthma and Immunology website. http:/ /www. aaaai.org/ global/latest-research­ summaries/Current-JACI-Research/TREC-screening. aspx. Accessed October 5, 20 1 5. Uekert SJ, Akan G, Evans MD, et al. Gender related dif­ ferences in immune development and the expression of atopy in early childhood. J Allergy Clin Immunol. 2006; 1 1 8 (6) : 1 375- 1 3 8 1 . von Mutius E. Paediatric origins o f adults lung disease. Thorax. 200 1 ;56: 1 53 - 1 57. Wardlaw AJ, Brightling C, Green R, et al. Eosinophils in asthma and other allergic diseases. Br Med Bull. 2000;56:985- 1 003. Wedi B, Raap U, Wieczorek D, Kapp A. Urticaria and infec­ tions. Allergy Asthma Clin Immunol. 2009;5 ( 1 ) : 1 0 . ,

Behavioral and Mental Health Issues

3

L i n d a J. Cooper- B rown a n d J e n n ifer Me Wi l l i a m s

This chapter i s meant to cover the behavior and mental health material that could b e tested o n the Pediatrics Board Exam. However, due to copyright issues, we are not able to share the diagnostic criteria tables from the Diagnostic and Statistical Manual of Mental Disorders, Fifth Edition (DSM-5). Summaries are provided, but we would encourage the reader to seek out and review the DSM-5 diagnostic criteria tables for the following disorders: •

Conduct disorder



Oppositional defiant disorder



Major depressive disorder



Depressive disorder severity



Attention deficit hyperactivity disorder

The father of a 5-year-old child is concerned that his son still sucks his thumb. This behavior is most common when they read a book together at night, while watching television, or when in the car. Question 1 - 1 What should this parent do? A) Point out to the child whenever he is seen sucking his thumb so he knows when it is occurring. B) Put a bitter substance on the thumb to make thumb sucking aversive. C) Have the child wear a glove to make it less appealing to suck his thumb. D) Identify triggers and provide alternative behaviors the child can use. E) Tell the child that his thumb might fall off if he continues to suck it. Discussion 1 - 1 The best answer is "D:' Thumb sucking is a common habit among children. Babies have natural rooting and sucking reflexes, and some can even be seen with their thumbs in their mouths in ultrasound images before birth. Babies often develop thumb (or finger) sucking to comfort or soothe themselves.

As children grow older, they often develop other coping skills to use when upset, anxious, or tired. Most children stop suck­ ing their thumbs on their own, usually between 2 and 4 years of age. Thumb sucking usually is not a concern until a child's permanent teeth come in. Because it is most likely a self­ soothing activity, children will be more successful at stopping if taught alternative ways to calm or soothe themselves rather than being punished or prevented from sucking. In this case, the child could hold the book or engage his hands when the parent is reading, or the parent could give the child a stuffed animal, Koosh ball, or other "fidget" toy while watching tele­ vision and during or car rides. The parent should involve the child in helping to identify things to hold or do that would provide comfort at the times most likely to be associated with thumb sucking.

A mother brings her 3-year-old son to the clinic for a well­ child visit. He is growing and developing within normal limits; however, the mother states that he is a very "picky" eater, preferring to eat certain foods and refusing to try new foods. She asks you if she should be concerned and what to do. 51

52

MCG RAW-H I LL EDUCATION SPECIALTY BOARD REVI EW: PEDIATRICS

Question 2-1 What percentage of toddlers are described as picky eaters by their parents? A) Less than 2%. B) Less than 10%. C) Up to 50%. D) Up to 75%. E) Close to 1 00%. Discussion 2-1 The correct answer is "C:' Variation occurs in reporting among studies, but up to 50% of toddlers are described by parents as "pickY:' meaning the child eats a limited amount or variety of food, is unwilling to try new foods, and has strong food prefer­ ences. If the child is growing appropriately, he or she is getting adequate calories. Reassurance and mealtime recommendations may help prevent the need for further intervention.

� QUICKQUIZ Which is NOT a helpful tip to prevent feeding problems from becoming major problems? A) Set a schedule for three meals and one or two snacks per day. B) Establish a routine of what meals and snacks look like. Eat at a dinner table and not on the run. C) Do not feel pressured to try the new foods being served to the child. D) Offer a new food many times as it may take 10 to 20 attempts before the child will accept it. E) Offer new foods at the start of a meal when the child may be hungriest. Discussion The correct answer is "C': Other helpful tips include the follow­ ing: ( 1 ) Establish mealtime rituals (sit together, pass dishes, practice manners and etiquette, avoid distractions, and so on. (2) Do not let the child graze on food or clinks throughout the day. The child will not be as likely to eat or try new foods if he or she is full all the time. (3) Establish a routine of what meals and snacks look like. (4) Remember that parents or caregivers serve as role models. You can't expect the child to eat something you won't eat yourself. If you want your child to eat certain foods, you have to eat them, too.

C) Ask what happens between these meals related to offering food. D) All of the above. E) None of the above. Discussion 3-1 The correct answer is "D:' These are fairly common and relatively easy-to-treat issues that can contribute to a lack of appetite. If the child is constipated, he may not be hungry. Likewise, if the child is drinking excessive amounts of fluid this can curb his appetite. I den­ tify how much fluid is needed for daily intake. Finally, eating (and drinking) throughout the day, especially close to a meal, contribute to reduced hunger. Consistent use of a treatment for constipation, scheduled meals and snacks, and discontinuing fluid intake at least 1 hour before a meal or snack can positively affect food intake.

A 2-year-old girl is brought to the clinic because her mother is worried that she is not yet toilet trained. The child is incon­ sistently dry after naps and kicks and screams every time a parent tries to have her sit on the potty seat. Grandmother told the mother that her own children were trained well before age 2 years, which has the mother especially worried. Should she be concerned? Question 4- 1 What factors contribute to toilet training readiness? The child: A) Shows interest in using a potty chair or toilet and may complain about wet or dirty diapers. B) Stays dry for 2 hours or longer during the day. C) Can follow basic directions such as "sit down'' or "follow me:' D) Can communicate through words, expressions, or posture when needing to go. E) All of the above. Discussion 4- 1 The correct answer is "E:' When it comes to potty training, parents often have unrealistic expectations. Although the mother may have been ready and trained before 2 years of age, this child is not yet ready. The American Academy of Pediatrics (AAP) recom­ mends starting potty training once the child is developmentally ready and shows signs of readiness or interest (see Table 3 1 ) . -



Parents bring their 4-year-old son to the clinic for concerns related to poor eating. They report that the child is willing to eat just about every food offered, but only eats a few bites during meals and then says he is done. Question 3-1 What would you do? A) Investigate possible constipation. B) Identify how much fluid the child is drinking during the day.

Helpful Tip

� Remember, toileti ng accidents are common when a l l lr child is stressed.

A mother brings her 4-year-old son to the clinic. She is frus­ trated because the child is not yet consistently having bowel movements in the toilet, although he urinates in the toilet.

CHAPTER 3



BEHAVIORAL AND M ENTAL H EALTH ISSUES

53

TA B L E 3 - 1 S I G N S OF REA D I N E S S THAT IT I S T I M E TO TO I L ET TRA I N

C a n wa l k t o t h e toi l et Can sit on the toi l et Can stay d ry for seve ra l h o u rs Can fol l ow one- a n d two-ste p co m ma n d s C a n com m u n icate the n eed t o use the to i l et t h ro u g h words or gestu res Wa nts to please careg ivers Wa nts i ndependence

The mother is concerned because he will be starting school soon. She wants to know if this is normal for his age. Question 5-1 Regarding bowel and bladder control, which of the follow statements is (are) correct? A) Bowel control is typically achieved before bladder control. B) Daytime bladder control should be achieved by age 4. C) Twenty percent of 5-year-olds and 10 % of 6-year-olds have primary nocturnal enuresis. D) Persistent nocturnal stooling in an older child suggests encopresis. E) All of the above. Discussion 5-1 The correct answer is ''E': The child already uses the toilet suc­ cessfully for urination and he is older than would be expected to achieve bowel control. He should be assessed for developmental delay, genitourinary abnormalities or constipation. If all is reassur­ ing, you may want to suggest specific strategies for delayed toileting:

The parents of a 1 '12-year-old girl express concern that their daughter is biting at home and in a daycare setting. Parents and staff cannot figure out a reason for her behavior, which occurs whether she is happy or upset. They tell her "no" every time it happens, but it continues to occur. Question 6- 1 What is {are) the most likely reason(s) she is biting? She is biting because: A) She lacks alternative language skills to express herself. B) She is not getting enough attention during the day. C) She is not getting enough oral stimulation. D) She gets a reaction from adults or peers. E) Both A and D. Discussion 6-1 The correct answer is "E." Biting is very common in chil­ dren. Approximately 1 0 % of children younger than 4 years of age have bitten another child at least once. S ome are wild animals needing socialization. During infancy children explore their environment using all of their senses. Any­ thing near an infant's mouth is a candidate for mouthing or biting. Biting in infancy is often an experiential occurrence. By 18 months to 2 years, children learn that their behav­ ior leads to desired outcomes and bite because it results in attention or toys. It is not usually the case that the child does not have enough of something. It is more likely that the child has learned ( over time) that biting produces the outcome more immediately. For example, a toy that another child is holding is released right away, the child who is bit­ ing gets picked up quickly, or she is offered many alterna­ tives after b eing told "no:'

Scheduled toilet-sitting times about 20 minutes after eating.



Make sure the child has his feet securely positioned on a step stool or the floor to permit adequate "pushing" or bearing down.



Provide small rewards for "trying" to have a bowel move­ ment and bigger rewards for successes.



Do not use rewards for an absence of accidents or the child may learn to withhold even more.





Helpful Tip

� If the child has a history of constipation, he or she may not I

1 1 r be motivated to use the toilet due to fear of hard stools.



Helpful Tip

� As the child learns more appropriate ways to express 1 1 1 r wants and needs, and how to wait, biting should

decrease. Parents and other adu lts can help by teaching age-appropriate language skills and rewa rding the child (with the desired attention and toys) for using words instead of biting. If biting continues to occur beyond 3V2 years of age, a referral to experts such as behavioral psychologists may be helpful.

Make sure the stool is easy to pass fortoilettraining success.



Helpful Tip

:5.� The best rewards are often highly preferred activities

i1 1 r rather than food. Advise pa rents to set these activities

aside and use them only for toileting times so they remain effective. The rewards may need to be changed weekly to keep the child motivated.

At a well-child check, the mother o f a 5-year-old mentions that her son won't listen to her when she asks a question. He continues to engage in whatever he is doing. She has to ask the question repeatedly and finally get his attention by turn­ ing his face toward her. She has tried time-outs, ultimatums, taking things away, and yelling. Nothing works.

54

MCG RAW-H I LL EDUCATION SPECIALTY BOARD REVI EW: PEDIATRICS

Question 7-1 What advice regarding limit setting should you give her? A) Increase the length of time he sits in time-out. B) Stop asking him questions if he isn't going to answer. C) Take notice when he does listen and praise him for the good behavior. D) Your efforts will be futile no matter what method is used. E) Long discussions about why his behavior is undesirable are helpful. Discussion 7-1 The correct answer is "C' Children are not miniature adults. Although they may act as though they want to be in control, they need parents to provide limits and consistent rules. These provide children with a greater sense of security and help them to learn how to get along with others and, eventually, how to discipline themselves.

TA B L E 3-2 COM M O N TRI G G E R S A N D

I NT E RV E N T I O N S F O R B E H AV I O R P R O B L E M S

Trigger

Intervention

Desire to gain access to something such as a toy, activity, or atten­ tion ("I want it now")



• •

Des i re to avoid or esca pe u n p l easant o r non preferred activities o r tasks ("I don't want to do it") Frustration, d ifficu lty com m u n icati ng



• •







P l ace c h i l d i n safe a rea to preve nt i nj u ry d u ri n g tantru m Ti me-out fro m re i nforce ment Do not g ive i n to tantru m Place c h i l d i n safe a rea to pre­ ve nt i nj u ry d u ri n g ta ntru m Ti me-out fro m re i nforce ment Do not g ive u p on orig i n a l req uest I g n o re u nt i l ca l m (may need to place in safe a rea to pre­ ve nt i nj u ry d u ri n g ta ntru m) Offe r to help after c h i l d ca l m s

Helpful Tip

:5.� Top 1 0 Tips for Discipline and Limit Setting:

i1 1 r

1.

Be consistent. "Yes" means "yes"and "no" means "no:' Make it short and sweet. Provide concrete one­ step directions: "Put the ca rs in the bin;"'put your clothes in this d rawer;' rather than "Put your toys and clothes away:' 3. Remember to "catch" your child bei ng good and praise often. Don't take good for granted. 4. Provide lots of chances to teach the correct behav­ ior. Don't assume your children know how to be good. Show them. 5. Respond quickly to good and bad. I mmediacy is important. 6. Phrase you r req uest as a statement, not a q ues­ tion. A question i m plies a choice. For exa mple, "It is time for bed," not "Are you ready for bed?" 7. Be patient. Behaviors take time to change. It can take at least a week to change problem behavior for every month it has occurred. 8. Be prepared for an increase in problems when trying to change behaviors. Behavior often gets worse before they get better as the child is learn­ ing the rules. 9. Be united as parents across time and households. 1 0. Pick and choose battles. Not making a request because you are tired is better than making a request and giving in because you cannot fol low through on the request. 2.

recently at the store he saw something h e wanted and had a major meltdown that included banging his head when told "no?' The mother has ignored his screaming, but head bang­ ing is a new behavior. She wonders why he would try to hurt himself. Question 8- 1 You tell her that head banging: A) Is a behavior that suggests deeper mental health issues. B) Is another form of tantrum behavior. C) Suggest the need for medical testing. D) Suggests he has a high pain tolerance. E) Is caused by a headache. Discussion 8-1 The correct answer is "B:' Tantrums can include behaviors that range from whining, crying, and screaming to hitting, kicking, stomping, and head banging. The causes are varied, and the behaviors can happen anywhere (see Table 3-2). Children often escalate to more severe behaviors when more minor ones do not provide the desired outcome. No matter how smart a child appears, toddlers are not very good at verbally communicat­ ing feelings, wants, or needs. As vocabulary and language skills improve, tantrums generally decrease. Identifying the triggers for tantrums leads to intervention strategies. (Would the same strategies work for adult tantrums? If so, reality television might no longer exist.) The mother is captivated. She has tried time-outs but isn't sure how to use this method of discipline successfully.

A mother brings her 3-year-old son to you because she is convinced there is something wrong with him. He just had a birthday and received many new toys, but when they were

Question 8-2 What do you tell her? A) Time-outs should always last 2 minutes. B) Time-outs should always last 5 minutes.

CHAPTER 3

C) Time-outs should last until the child has calmed down. D) The child should be allowed to read a book or other quiet activity while in time-out. E) A special time-out stool is needed to reinforce the concept to the child. Discussion 8-2 The correct answer is "C:' There are many ways to imple­ ment time- out. Some of the key components include the following: •





Time-out needs to last long enough to get the child's atten­ tion and convey that negative behaviors result in negative consequences, or put another way, negative behaviors never result in desired outcomes. It does not have to last for a set number of minutes and should last until the child has calmed. It should never end when the child is still having a tantrum. Time-out does not have to involve a special location. It should occur away from any action and fun activities. It could mean simply walking away from the child or it could mean remov­ ing the child to a carpeted, safe area. Nothing should occur while the child is in time-out. For time-out to work there has to be a "time-in" environ­ ment. Time-out is actually time out from reinforcement and time-in is all the fun and positive things, including positive attention from parents. First, throughout the day parents should acknowledge positive behavior and not take it for granted. Second, when a child has calmed after time-out, parents should let the child know what kinds of behaviors will work to get the desired outcomes.

A 3-year-old girl attends daycare all day while her parents work. When the family arrives home, is time to fix dinner and do chores. The girl often behaves well at daycare but has tantrums at home. Her parents wonder if she is not get­ ting enough attention from them and if one of them should quit work. Question 9- 1 What do you suggest? A) The mother should quit her job as daughters respond better to female caregivers. B) The father should quit his job as daughters respond better to male caregivers. C) She is not getting adequate sleep and should have an earlier bedtime. D) The parents should ignore the tantrums and hope they will cease with time. E) The parents should find time each evening to engage in a fun activity with her.



BEHAVIORAL AND M ENTAL H EALTH ISSUES

55

Discussion 9- 1 The correct answer is "E:' It is hard to comment about whether one parent should quit work, but because the daughter seems to have the most difficulties at home, it is likely that she has learned that a tantrum is the quickest way to get a parent's attention. This is likely a case of too little "time- in:' Her parents are probably busy upon arriving home and ignore her until she engages in a tantrum. Finding times to provide even intermit­ tent attention upon arrival home may reduce some of the tan­ trums. Advise parents to help the child get started with a fun activity and provide some positive attention as they prepare dinner. Spending 5 minutes with her before starting dinner, and interspersing attention may go a long way in decreasing the unwanted behaviors.

The parent of a 1 6-month-old girl describes a recent episode in which the child became upset at the park, started crying, stopped suddenly, turned slightly blue around the lips, and then stopped breathing, went limp, and fell to the ground. Less than a minute later the child recovered and returned to normal breathing and activity. The parent is convinced that the child had a seizure and is reluctant to let the child have a tantrum for fear of causing additional seizures. Question 1 0-1 What most likely occurred? A) The child held her breath to upset the parent when she could not get her way. B) The child had a breath-holding spell beyond her control. C) The child had a seizure. D) The child had an episode of supraventricular tachycardia. E) The child has amazing acting skills. Discussion 1 0-1 The correct answer is "B:' It is most likely a breath-holding spell. Breath-holding spells typically occur in children starting at 6 to 1 8 months of age and usually stop by the time the child is 7 years old. Such episodes can be upsetting to parents and others who see them. These are not seizures and do not harm the child, although a child may have a secondary anoxic sei­ zure. Furthermore, the child is not holding his or her breath on purpose to be able to pass out. The episodes can be triggered by fear, pain, anger, or frustration. Typically the child starts to cry and then no further sound is heard but the child still appears to be crying. This is different than voluntarily holding one's breath, which occurs when an individual deeply inhales and then holds his or her breath. Breath-holding spells are some­ times classified into two types of episodes: pallid and cyanotic (see Table 3 - 3 ) . Breath-holding spells may run in families and are more common in children with iron deficiency. Treatment involves reassuring the parents of the benign nature of the epi­ sodes and providing iron supplementation for those who are deficient.

56

MCG RAW-H I LL EDUCATION SPECIALTY BOARD REVI EW: PEDIATRICS

TA B L E 3-3 B R EATH - H O L D I N G S P E L LS

Pallid Episodes

Cyanotic Episodes

Trigger

Fea r, pa i n

Fru stration, a n g e r

O n set

Gasp, cry

Vigorous cryi ng

Cou rse









Becomes pale, lo s es consciousness, goes l i m p May beco m e sweaty, stiffen, jerk Lasts less than 1 m i n ute Reg a i n s co nsciousness a n d may a ppea r ti red









Rapid ly turns b l u i s h , especi a l ly a ro u n d l i ps, loses consciousness May stiffen, a rch back Lasts less t h a n 1 m i n ute Reg a i n s co nsciousness a n d returns to normal

Helpful Tip

� Although breath-holding spells are triggered when the

1 1 1r child becomes upset, they do not represent a behavior

problem and most children who have episodes of breath - holding spells are otherwise healthy. The natural tendency would be for the parent to pick up and hold the child, but this position may prolong the event. The best intervention is to leave the child lying on the ground, positioned on his or her side. No other intervention is needed as the child will resume breathing in less than 1 minute. When a child is known to have breath­ holding spel ls, a parent may avoid al lowing the child to become upset by giving in to tantrums. However, the spel ls generally decrease and stop by 7 to 8 years of age. Giving in to crying episodes and tantrums to prevent the child from becoming upset will only create behavior problems. The child will learn quite quickly to start crying to obtain desired outcomes.

� QUICKQUIZ Which strategy is NOT recommended as a way to help a child adjust to the arrival of a baby sibling? A) Allow the child to help care for the baby. B) Spend a short amount of time each day with the child. C) Allow the child to have increased screen time while he or she adjusts. D) Show the child his or her own baby pictures. E) Have patience and understand that this is a big adjustment for the child. Discussion The correct answer is "C:' Tips for helping a child prepare and for and welcome a new sibling include the following:









The child can be allowed to have a role in preparing the home. The parent can have the child help care for the baby by bring­ ing a diaper or selecting an outfit. Have the parent schedule special time with each sibling. To do this, the parent sets aside a short time each day ( eg, when the baby is napping or another adult is available to care for the baby) to do an activity of the child's choice. The parent should share special baby memories with the child as a reminder of how important he or she is to the family.

A parent comes to the clinic with 5- and 6-year-old siblings for their well-child checks. While in the clinic the children fight over the parent's cell phone, each demanding a turn. One of the children sits in the only small chair in the room, but whenever the child gets up, the other child quickly takes over the chair creating a physical battle and tattling. The par­ ent says it is like this at home, but doesn't know what to do. Question 1 1 - 1 What d o you suggest? A) Remind the parent that all siblings have some rivalry and it will go away someday. B) Have the parent encourage the children to work out their differences but assist as needed. C) Suggest the parent ground the children from video time at home and any other fun activities for today. D) Have the parent listen to each child's story and pick a side. E) Ignore the behavior even if the children get physical as long as no one gets injured. Discussion 1 1 - 1 The best answer is "B:' I t i s n o t uncommon for siblings to engage in problem behaviors with each other, but reassur­ ance alone will not help the parent. The children should be given an opportunity to find a way to work out their differ­ ences. However, if the children get physical with each other, the parent must step in and make it clear that it is never acceptable to hit, pinch, bite, punch, shove, and so on. These behaviors may require a time- out. The parent's focus should be on teaching appropriate ways to handle differences rather than doling out punishment, such as groundings or removal of activities, for conflicts. Tips for reducing sibling conflicts include the following: •



If the siblings cannot figure out a way to play together, a timer can be used to teach them how to share a desired toy or activity. This is especially useful for very young toddlers who want items immediately and have no concept of sharing. The timer signals the beginning and end of each child's turn with the toy. The parent should use an "in/out" rule for tattling: Have the child think about why he or she wants to tell on a sibling. If it is to get someone "in'' trouble, the child should not tattle

CHAPTER 3

and the parent will not listen. If it is to keep the sibling from getting hurt or "out" of trouble, the child should let the par­ ent know right away. •



Encourage the parent to show appreciation for each child's uniqueness and avoid comparisons. Rivalry often occurs when comparisons are made. Help the parent focus on each child's unique personality, ability, and temperament, and give compliments whenever possible. Remind the parent to "catch'' the children playing well together and praise them for following the rules.

For the time in your office, you may want to suggest using a timer for turns with the small chair and other items such as the cell phone. It is also acceptable to put the cell phone in time-out and focus on turns with other items in the office.

Parents bring their 2 ¥2 -year-old son to clinic and are con­ cerned that he has started awakening at night and scream­ ing. His eyes are open and he seems awake, but does not react to their attempts to calm him. They want to know what to do. Question 1 2- 1 What d o you tell them? A) He is having a seizure. B) He is acting out trying to get his parents' attention. C) He has no memory of these events. D) He is hallucinating. E) He needs to consume less sugar. Discussion 1 2- 1 The correct answer is "C:' The child is having night terrors (see Table 3 -4) .



BEHAVIORAL AND M ENTAL H EALTH ISSUES

57

The parents of a 4-year-old boy both work and need a good night's sleep. Their son often has a hard time going to sleep, taking several hours to fall asleep because he needs water, to tell them something, or has other requests. When he doesn't fall asleep he may start crying and want to come to their room. The parents ask if you can prescribe medications to help their son fall asleep and stay asleep. Question 1 3- 1 I s medication appropriate for this child and, i f so, what medication? A) Yes. B) No. Discussion 1 3- 1 Th e correct answer i s "B:' Th e difficultly this child has i n initiating sleep is behavioral. The parents need to establish good sleep hygiene habits and a consistent bedtime routine. Consistency is key. You counsel the parents that even if medication is prescribed, it is important to have good sleep hygiene. The parents look confused. Questions 1 3-2 Which strategy is a component of good sleep hygiene? A) Have a television or other electronic device in the child's bedroom that he can play with or watch until sleepy enough to fall asleep. B) Have a consistent bedtime routine but vary the actual time the child goes to bed. C) Minimize late afternoon napping so the child is tired at bedtime. D) Allow the child to do whatever he wants as long as he stays in his bedroom. E) Eat a late night sugary snack as a treat to get the child to go to bed.

TA B L E 3-4 N I G HTM A R E S V E R S U S N I G H T T E R RORS

What is the c h i l d d o i n g ?

Nig htmare

Nig ht Terror

Eyes o p e n , awa ke a n d a l e rt, frig htened, breat h i n g fa st, awa re of parent presence

Appears awa ke but confu sed, cryi ng u ncon­ trol l a bly, sha ky, breat h i n g fa st, g l azed look, does not recog n ize pa rent

When?

Seco nd h a lf of n i g ht when d rea m i n g i nte nse

Early in n i g ht when dee pest stages of sleep

I s the child awa ke?

Yes

No

Wi l l the c h i l d re m e m ber the eve nt?

Yes

No

Wi l l the c h i l d ret u rn to peacefu l s l eep?

M aybe

Yes

Best parent res ponse

I m med iate comfo rt and rea s s u ra n ce; when ca l m h e l p put child back to bed

Preve nt c h i l d fro m h u rt i n g se lf; wa it for c h i l d to ca l m ; do n ot d i scuss i n the m o r n i n g

58

MCG RAW-H I LL EDUCATION SPECIALTY BOARD REVI EW: PEDIATRICS

Discussion 1 3-2 The correct answer is "C:' Sleep hygiene refers to good habits and routines that help a child learn to fall asleep and remain sleeping. Tips for establishing good sleep hygiene include the following:

TA B L E 3-5 N O R M A L S L E E P PATTE R N S BY AG E

I nfa nts ( 1 -2 months)











Set a consistent bedtime. Determine the amount of sleep the child needs and time he or she must awaken in the morning to set the bedtime. Develop a consistent routine ( eg, taking bath, putting on pajamas, brushing teeth, reading a book) , and end with being in bed and turning off the lights.



Allow for wind down time before sleep time.



Adjust naps to prevent late sleeping during the day.



I nfa nts {3- 1 1 months)

Tod d lers ( 1 -3 yea rs) Preschoolers (3-5 yea rs) School-aged (5- 1 2 years)

The parents seem to be onboard thus far but want to know what you recommend when he keeps waking up. They have not been successful in dealing with this behavior.

Discussion 1 3-3 The correct answer is ''A:' Children typically call out or seek out a parent because they want something-usually attention. They often escalate their behavior until the parent gives in to let oth­ ers in the household sleep. Several strategies can be suggested. Advise parents to do a quick check: Parents tell the child they will come to his room to check on him as long as he is in bed. Check-in should occur for only a matter of seconds and involve looking into the room. Parents determine the timing of these check-ins and how often they occur during a night, gradually lengthening the time between check-ins. In handling problems, parents should: •





Make sure the child is safe but ignore the child's behavior, regardless of how intense or loud. Expect sleep-deprived nights. If the child leaves the room, return her without discussion or attention.

Bedtime tickets or passes can be used: •

The child is provided a set number of tickets each night to use in requesting a drink, hug, to tell the parent one last thing, and so forth, but without leaving his or her room.





Remove electronic gadgets from the bedroom. These tend to entertain and prevent sleep.

Question 1 3-3 What do you tell the parents about managing the child's fre­ quent nighttime awakening? A) Briefly check on the child if he is in his bed and provide reassurance. B) Install a lock on the outside of the door so the child cannot leave his room upon awakening. C) Allow the child to enter the parent's room as long as he sleeps on the floor and not in the bed. D) Ignore the child unless he becomes distraught. E) Alternate one parent lying down with the child until he falls asleep.





















1 0- 1 8 hou rs/day Sleep a ro u n d the clock Sleep/wa ke cycle i nteracts with need to be fed, c h a n g ed, held 9- 1 2 hou rs/d ay About 75% sleep t h ro u g h the n i g ht by 9 months of age One to fo u r naps per day lasti ng 30 m i n utes to 2 h o u rs 1 2- 1 4 hou rs/d ay By 1 8 months, most n a p once per day last i n g 1 -3 h o u rs 1 1 - 1 3 hou rs/d ay Most do n ot n a p by 5 yea rs of age 1 0- 1 1 hou rs/day May get less than opti mal sleep d u e t o sched u led activities or com peti ng attention from electronic gadgets

Every time the child calls to the parent, the parent enters the room to assist, but removes a ticket. When tickets are gone, all behavior must be ignored. Any unused tickets are collected in the morning. If any tickets remain, the parent can provide a reward in the morning.

Normal sleep patterns by age are listed in Table 3-5.

� QUICKQUIZ Which statement does NOT describe parent-infant attachment? A) It is an emotional relationship between the infant and parent. B) It is based upon the parent's response to the infant. C) It involves nonverbal actions. D) It provides the infant with a sense of security. E) It is the same as bonding. Discussion The correct answer is "E:' Attachment is an aspect of the parent­ child relationship that contributes to making the child feel safe, secure, and protected. It is not the same as bonding. A simple example can illustrate this process. A family's cat bats their infant son on the head, frightening him. The infant crawls crying to his father and is calmed after he is picked up. The father is viewed as a protector, having conveyed nonverbally that the infant is safe. Factors influencing parent- infant attachment include: •



The parent's response to the infant when he or she is hurt, frightened or upset. The amount of care (physical and emotional) provided by the parent.

CHAPTER 3

The mother of a 6-week-old boy is distraught. She tells you her baby never stops crying. It started 1 month ago and has been getting worse. It always seems to happen in the early evening. The infant will cry uncontrollably for hours. She has tried baths, car rides, stroller rides, carrying him in a sling, feeding, changing his diaper, gripe water, music, and even letting him "cry it out?' She reluctantly admits that she has found herself overly frustrated at times and doesn't want to hurt the baby. Question 1 4- 1 What d o you tell her? A) Colic begins at 2 weeks of age. B) Colic equally affects breastfed and formula fed babies. C) An infant cries 2.2 hours per day on average. Discussion 1 4- 1 Th e correct answer i s "A:' Between 10 % and 2 6 % o f infants are colicky. The etiology of colic is unclear. Colic was defined in a 1 954 article as crying more than 3 hours per day, more than 3 days per week, and more than 3 weeks in duration. It usu­ ally starts at 2 weeks, peaks at 6 weeks, and resolves by 12 to 1 6 weeks o f age. Babies with colic are more difficult t o console. Question 1 4-2 How is colic diagnosed? A) History and physical examination. B) Elevated creatine kinase level related to tensing of muscles during crying episodes. C) Elevated amylase level related to excess secretions during crying episodes. D) An abdominal radiograph showing large amounts of gas. E) All of the above. Discussion 1 4-2 The correct answer is "A:' Colic is a diagnosis of exclusion based on history and physical examination. An organic cause is found in less than 5% of infants evaluated for colic. Question 1 4-3 What do you NOT offer the mother as a management suggestion? A) Swaddle the baby in a sleep sack. B) Ask others to help watch the baby during crying episodes. C) Change to a soy formula. D) Develop a safety plan for use when she is feeling frustrated. E) Use a white noise machine or fan. Discussion 1 4-3 The correct answer is "C' Treatment includes parental sup­ port, reassurance, and education. Parents should identify other caregivers who would be willing to help. Interventions for the baby include swaddling, minimizing stimulations, white noise, and rocking. It is important to remind parents to never shake a baby! Soy and lactose-free formulas have not been shown to



BEHAVIORAL AND M ENTAL H EALTH ISSUES

59

be beneficial although many parents have already tried dietary changes. Simethicone is safe but ineffective. The limited current data do not support the use of complementary and alternative treatments, including probiotics, chiropractic treatment, and massage. (Really? The infants didn't like massage?)

A mother calls the office after hours, crying. She needs advice on dealing with her 1 1 -year-old son. This year at school he has been caught lying, stealing, and cheating on multiple occasions. He has lost school privileges and had to stay after school but the behaviors continue. The father is not con­ cerned as he feels "boys will be boys". Today the mother was called into the principal's office because her son was caught stealing candy from a school fundraiser. Question 1 5- 1 What d o you recommend? A) She should have spanked the child more as a toddler. B) She should send him to a boarding school. C) He should visit a juvenile detention center for a day to "scare him straight:' D) He should have his cell phone taken away for a short period of time. E) She should not worry as the child will outgrow the behavior on his own. Discussion 1 5- 1 Th e correct answer i s "D:' Lying and stealing fall under the cat­ egory of undesirable behaviors. There are two main strategies for dealing with such behavior: removing the positive reinforce­ ment for the behavior and providing a negative reinforcement. Both can be used to take away the positive reinforcement of the behavior. The privilege that is removed must be important to the child in order for this strategy to work. Punishment uses a negative action to reduce the behavior. Punishment may involve verbal reprimands or corporal punishment (inflicting physical pain). If used frequently verbal reprimands lose effectiveness. Spanking has been used by more than 90% of American families to stop undesirable behaviors. Spanking has negative long-term consequences and is less effective than time-out or removal of privileges; therefore, the AAP recommends that physicians help parents develop alternative methods of discipline.

A 2-year-old girl is brought by her mother for a well-child check. The toddler begins to sob and throw a tantrum when her mother steps out of the exam room to sign some paper­ work, leaving her with the nurse. When she returns, the mother apologizes profusely and states that she would like some advice on how to handle her daughter's tantrums as

60

MCG RAW-H I LL EDUCATION SPECIALTY BOARD REVI EW: PEDIATRICS

they have increased in intensity to the point that the mother is unable to leave her with anyone else. Question 1 6- 1 How would you treat the toddler's tantrums? A) Prescribe fluoxetine for separation anxiety. B) Refer the child and her mother for therapy with a psycholo­ gist who specializes in early childhood disorders. C) Reassure the mother that the child's behavior is normal for her age. D) Recommend a multimodal treatment plan including both medication and therapy. Discussion 1 6- 1 Th e correct answer i s "C:' Separation anxiety i s normal for children between the ages of 6 and 30 months. While it may be beneficial to counsel parents on how to respond to their child's anxiety, neither medication nor therapy is recommended for this age group.

An 8-year-old girl is brought to the clinic. The mother says her daughter will not leave her side out of fear that something horrible will happen to the mother. It has gotten so bad the child must sleep with the door open so she can see her par­ ents' room. She hasn't been to school for 2 weeks. Question 17-1 What do you tell the mother? A) The child's behavior is normal and often seen at this age. B) The child may have separation anxiety disorder. C) The child is likely being bullied at school. D) The behavior is a ploy by the child to avoid going to school. E) The child should be allowed to stay home from school. Discussion 17-1 The correct answer is "B:' According to the DSM-5, separation anxiety disorder (SAD) is diagnosed when the child has "devel­ opmentally inappropriate and excessive anxiety concerning separation" from home or the primary caregiver(s) . Typically onset of SAD is between 7 and 9 years of age. In SAD, the child becomes anxious at the thought of being separated from care­ givers out fear that something bad will happen to them. Every­ one experiences anxiety now and then. (Are you apprehensive about a little thing called boards? If so, congratulations you are anxious! ) Anxiety becomes a disorder or disease when it con­ sumes and interferes with life. Separation anxiety is very com­ mon and part of normal development. Think of a 2-year-old crying and clinging to a parent at the daycare drop-off. When such anxiety becomes obsessive or persists into childhood, then there is a problem. For example, a fourth grader may be tearful when first dropped off at a sleepover but quickly able to join the fun. Had the child stayed in the corner crying and obsessing, that would be SAD.



Helpful Tip

� Anxiety disorders are the leading psychiatric disorders 1 1 1r diag nosed in children.

In talking with the mother, you learn that the father was recently diagnosed with cancer. He is undergoing treatment, and doctors are optimistic that his cancer is curable. The mother tells you that the child was sometimes homesick in the past and occasionally had to leave sleepovers when she was younger, but this had improved. She emphasize that the child's behaviors were never this extreme before. Question 1 7-2 Which life event is NOT associated with the development of SAD? A) Death. B) Divorce. C) Illness. D) New baby. E) All of the above. Discussion 1 7-2 The correct answer is "D:' Signs of SAD may first appear after the child's home life is disturbed. Examples include divorce of parents, parental illness, death of a family member, financial issues affecting the home, and or a move to a new location. Question 1 7-3 Which of the following is a symptom of SAD? A) School refusal. B) Headaches. C) Fear the parent will be harmed. D) Inability to be separated from the parent for even a short time. E) All of the above are symptoms of SAD. Discussion 1 7-3 The correct answer is "E:' Like everything in pediatrics, symp­ toms vary by age. It is common for teens to worry about school performance or social interactions but these concerns resolve as the adolescent matures. Refusal to go to school out of fear of being separated from a parent is a red flag. Truancy is different. It is a deliberate avoidance of school for no good reason. You diagnose the child with SAD and have a long discussion with the mother. You provide information and support ser­ vices. You refer the child to a psychiatrist but it will take a month to get an appointment. She needs help now. Question 1 7-4 Which of the following measures is (are) appropriate for SAD? A) Selective serotonin reuptake inhibitor. B) Cognitive behavioral therapy. C) Educating parents to stop enabling the child. D) Return to school. E) All of the above.

CHAPTER 3

Discussion 1 7-4 The correct answer is "E:' SAD should not be chalked up to poor coping skills. It is often a gateway to future mental illness. A multimodal treatment plan, including parent psychoeducation, a selective serotonin reuptake inhibitor, and cognitive behav­ ioral therapy, is recommended. Parents need to be assertive, and the child needs to go to school.

A ninth grader is brought for evaluation by his mother. He has missed 20 days of school this year because of vari­ ous headaches, stomach aches, and other complaints. No physical causes have been determined despite an exten­ sive workup. When the possibility that he is trying to avoid school is brought up, his mother reports that it has been hard to make him go to school since his dad moved out last year. In addition, she knows there is a "bully problem" at the high school, and she is concerned he is being picked on. Question 1 8- 1 What i s the best next step? A) Contact the school counselor, after obtaining a release of information form from the mother, and ask about the ado­ lescent's social interactions and academic performance. B) Refer the adolescent to a psychologist who specializes in anxiety disorders. C) Suggest to the mother that she call the school's truant officer next time the adolescent refuses to go to school. D) All of the above. Discussion 1 8- 1 The correct answer i s ''A." Children and adolescents with school refusal are a heterogeneous group and the evaluation should include a variety of components. Collateral informa­ tion from school personnel is very important to help identify psychological factors such as depression and anxiety, social factors such as bullying and peer influences, and academic factors such as learning difficulties. In addition, the evalu­ ation may include a clinical interview or semistructured diagnostic interview with the patient; collateral information from parents regarding psychological, social, and academic factors; and a psychoeducational evaluation to assess for learning disorders. If, after a thorough evaluation has been completed, anxiety is felt to be the primary component of the etiology, then psychotherapy would be vital. If, on the other hand, a disruptive behavior disorder such as conduct disor­ der is determined to be the primary cause, then involvement of the legal system may be necessary. Often a multimodal treatment plan involving, parents, school officials, psycho­ therapy, and possibly medication is needed. Regardless, the cause of the school refusal must be fully evaluated first (see Table 3 - 6 ) .



BEHAVIORAL AND M ENTAL H EALTH ISSUES

61

TA B L E 3-6 S C H O O L R E F U S A L

Prevalence (%)

Mean Age of Onset (years)

Percentage of Males

Anxious

1 .6

1 0.9

47.9

Tru a nts

5.8

1 3.1

65 . 1

M ixed

0.5

Type of School Refusal

5 1 .9

At a 2-year well-child check, you ask if there are any other concerns. The father looks at the floor, then sheepishly men­ tions that his son likes to touch himself in public. The parents have tried telling him "you shouldn't touch your wee-wee;' "that's not nice;' and even "good boys don't do that:' The grandmother told the father that this is abnormal behavior and there must be something wrong. She actually wondered if the babysitter is touching him. The father asks for advice. Question 1 9- 1 What d o you tell him? A) Male infants may have erections. B) It is normal for toddlers to masturbate. C) Using nicknames for genitalia is not recommended. D) Children should not be punished for normal sexual behaviors. E) All of the above. Discussion 1 9- 1 Th e correct answer i s "E:' Normal versus abnormal sexual behaviors are differentiated based on cultural and societal beliefs. Masturbation and sexual exploration are part of normal development that begins in infancy (see Table 3-7). Since this behavior is normal, parents should be reassured and children allowed to explore without reprimand. Early in life, these behav­ iors occur in public. As the child gets older, privacy becomes important. Parents should be encouraged to use proper anatom­ ical terms for genitalia instead of nicknames, which may cause confusion.

� QUICKQUIZ Which of the following is NOT a characteristic of a resilient child? A) Becomes angry when faced with a challenge. B) Has a strong connection to the community. C) Knows he or she is loved unconditionally. D) Recognizes that his or her actions affect outcomes. E) Able to maintain perspective when dealing with problems.

62

MCG RAW-H I LL EDUCATION SPECIALTY BOARD REVI EW: PEDIATRICS

TABLE 3-7 NORMAL SEXUAL B E H AVIORS BY AG E

TA B L E 3-8 B U I L D I N G R ES I L I E N CY: T H E 7 CS

A N D DEVELO P M E NTA L STAG E

YO U N E E D

0- 1 2 months

Mastu rbation

Key Behavior

Actions for Caregivers

I nfa n cy

Explore genita l a rea d u ri n g d i a per cha nges and baths

Com petence: Knowing how to h andle a situation effectively

Enco u rage c h i l d ren to m a ke decisions

E rections 1 -4 yea rs

Mastu rbation ( p u b l ic)

Tod d l e r

To uch mother's body pa rts (eg, brea sts)

Confi dence: Believing in one's abilities

Expose genita l i a to others

Gives courage to face dif­ ficult situations

5 - 1 1 yea rs

Mastu rbation (private)

Con necti o n :

C h i l d re n

Ask or ta l k a bout sex

Building relationships with others to provide a sense of support and secu­ rity th at leads to the devel­ opment of strong values

Take clothes off

D ress up as o p posite sex Play "doctor" (explore peers' gen ita l ia) 1 2- 1 8 yea rs

Mastu rbation

Ad o lescents

Explore sexual i nti macy with others Qu esti o n i n g or awa reness of sex u a l orie ntation Sex u a l l y active

Discussion The correct answer is "A:' Resiliency allows a child to deal with the ups and downs of life. To be resilient, a child needs to know that an adult believes in and loves him or her unconditionally. Parents can provide their children with the tools they need to become resilient. In 2006, the AAP published a book that iden­ tified "7 Cs" of resilience (see Table 3-8).

A 1 6-year-old boy has recently been expelled from school for starting a fistfight with a peer for the fourth time this school year. School personnel are also concerned that he has "conned" a couple of girls into doing his homework each day by lying to them about a fictitious diagnosis of dyslexia. His mother sent him to live with his uncle 6 months ago because she was tired of "constantly arguing with him:' He had also run away from home repeatedly for 2 to 3 days at time. She suspects he might be using drugs. His uncle has noted that the adolescent has a very short fuse and gets angry easily. He was arrested and placed on probation earlier this year for shoplifting cough medicine from a local drugstore, but has not had any run-ins with authorities in the last few months. Question 20-1 What is the adolescent's diagnosis? A) Oppositional defiant disorder. B) Bipolar disorder.

C h a racter: Developing a set of values and morals Learning right from wrong beh aviors

Focu s on a c h i ld's strengths Recog n i ze a n d com p l i ­ ment when the c h i l d has done wel l Provide e m oti o n a l secu rity at home Enco u rage c h i l d ren to reach out d u ri n g to u g h times Poi n t out h ow behaviors affect others Help the c h i l d see h i m self as a caring person

Contri bution: Experiencing thejoy of helping others

Poi n t out those who a re less fo rtu nate

Co p i n g : Learning t o h andle stress

Model positive copi ng strateg ies

Contro l : Learning t o m ake wise choices

Poi n t out h ow the c h i l d 's choices contri buted to the outco m e

Model generosity

Recognizing cause and effect

C) Conduct disorder. D) Delinquency. E) Substance use disorder. Discussion 20-1 The correct answer is "C:' He has met the criteria for con­ duct disorder by displaying at least three criteria over the course of the last year, with at least one criterion in the last 6 months. He has initiated physical fights, has lied to obtain favors, has stolen items of nontrivial value, and has run away from home overnight. While one could make the argument that the adolescent meets the general description of opposi­ tional defiant disorder (ODD) given his frequent disregard for authority figures, loss of his temper, and argumentative­ ness, meeting the criteria for conduct disorder is an exclusion criteria for ODD. While many children and adolescents with bipolar disorder display disruptive behavior, the behavior is noted, by those who have regular contact with the child, as being episodic. Bipolar disorder is characterized by several

CHAPTER 3

days to weeks of manic behavior that alternates with several day to weeks of depressed behavior. A manic episode is char­ acterized by a mood changes (elevated, irritable), increased energy, decreased need for sleep, pressured excessive speech, inflated self-esteem, flight of ideas, and distractibility. Delin­ quency is considered a legal term, not a medical diagnosis. While substance abuse may certainly be playing a role in the patient's presentation, there is not enough information to make a diagnosis of a substance use disorder or identify it as the causative factor of the patient's behavior.



Helpful Tip

� People with conduct disorder are mean. They are

1 1 1r aggressive

toward people and animals, destroy property, lie, steal, and break rules. They violate the basic rig hts of others and do not respect age­ appropriate societal norms.



Helpful Tip

� People with ODD are difficu lt. They a re frequently i n

1 1 1r a n angry/i rrita ble mood and display a rg u m entative

or defiant behavior. They a re also spitefu l or vindictive. They often lose their tem per, a re easily annoyed, and wi l l argue with authority fig u res over req uests or rules. Note that this is different from conduct disorder. People with ODD fight other people. People with conduct disorder h u rt other people.

Question 20-2 What would you recommend to the adolescent's family for treatment? A) Admission to a "boot camp" residential treatment facility. B) Admission to a substance abuse treatment facility. C) A combination of risperidone and methylphenidate. D) Development of an individual treatment plan involving school, juvenile justice, and mental health professionals, as well as caregivers. Discussion 20-2 The correct answer is "D:' Conduct disorder requires a mul­ timodal treatment plan that accounts for the heterogeneous nature of its etiology. Treatment is the most effective when multiple agencies are involved so that care is coordinated and consistent through each aspect of the patient's life. Family-based programs, such as functional family therapy and multisys­ temic therapy, can be very effective. Problem-solving and skills training also can be beneficial. Currently, there is no evidence supporting medication as the primary treatment for ODD or conduct disorder. However, medication can be helpful in treat­ ing comorbid conditions such as attention deficit hyperactivity disorder (ADHD) .



BEHAVIORAL AND M ENTAL H EALTH ISSUES

63

A 10-year-old girl has been having difficulty sleeping for the last several months. Her mother reports that she is anxious all the time, but has particular difficulty at night and will lay in bed for hours worrying about things. She worries about doing well in school, her parents getting into car accidents, tornados, and Ebola as a few examples. The mother mentions that as her daughter's list of concerns has gotten longer and her sleep has worsened, she has also become more irritable and withdrawn. Question 2 1 - 1 What i s the prevalence o f her disorder? A) 0.5- 1 % . B) 3-5%. C) 1 0- 1 5%. D) > 20%. Discussion 2 1 - 1 Th e correct answer i s "B:' Anxiety disorders are chronic and reoccurring. All are characterized by uneasiness, rumina­ tion, and frequently associated with somatic complaints (see Table 3-9). Younger children may express anxiety as crying, tantrums, clinging, or outbursts of anger. Genetics play a role and frequently there is a family history of anxiety disorders. This patient has generalized anxiety disorder (GAD). Anxiety is very common and normal for children and adolescents and can be protective and beneficial. Anxiety becomes a disorder, however, when it begins to impair an individual's ability to function. Children and adolescents with GAD have chronic, excessive worry in multiple areas of their lives-school­ work, family, world events-and a minimum of one associ­ ated somatic complaint (headache, abdominal pain). People affected by GAD tend to be perfectionists. GAD is estimated to occur in 3% to 5% of children and adolescents. Specific phobias can occur in up to 1 0 % of children. Estimates of the incidence of social anxiety disorder vary widely but have been reported to be as high as 1 5% . Question 2 1 -2 What would you recommend as treatment? A) Prescribe citalopram and alprazolam. B) Refer her to a therapist who specializes in cognitive behav­ ioral therapy and prescribe citalopram. C) Refer her to a therapist who specializes in cognitive behav­ ioral therapy and prescribe alprazolam. D) Reassure her mother that the anxiety will resolve as her daughter gets older. Discussion 2 1 -2 The correct answer is "B:' Psychotherapy is recommended for children with mild to moderate anxiety disorders ( eg, GAD, specific phobia, panic disorder) . For children with moderate to severe anxiety, or children who do not respond to psychother­ apy, the addition of a selective serotonin reuptake inhibitor ( eg, citalopram) is recommended. Benzodiazepines (eg, alprazolam)

64

MCGRAW-HILL EDUCATION SPECIALTY BOARD REVIEW: PEDIATRICS

TABLE 3-9 MAJOR ANXIETY DISORDERS

Disorder

Description

Symptoms

Comorbidities

Generalized anxi-

Excessive, chronic worry

Somatic complaints

Depression

9W disorder (GAD)

Interferes with daily function

Constant worry

ADHD

Separation anxi-

Excessive, developmentally inap-

Somatic complaints

Unable to sleep alone

ADHD

home or caregivers

Nightmares about separation

Other anxiety disorders

School refusal

Social phobia

Difficulty speaking in school,

Substance abuse

new. people, attending parties

Other anxiety disorders

Avoids fearful object or situation

Separation anxiety disorder

Other anxiety disorders

ety disorder (SAD)

Social phobia

Specific phobia

Panic disorder

propriate fear over separation from

Scared or uncomfortable in social

settings or performance situations

Depression

reading aloud, talking with

Excessive, unreasonable fear of a specific thing or situation despite awareness that the fear is unreasonable

Distressed if must deal with fearful object or situat

Depression

ubstance abuse pression

g OCD

Recurrent, unexpected episodes of fear with physiologic symptoms Fear and avoidance of places or situations where escape is difficult or would draw unwanted attention (eg, agoraphobia)

Heart racing

Depression

Sweating

ADHD

Shaking

Other anxiety disorders

Na

Substance abuse

ea

Chill I

‘a

g on ofchoking ADHD, attention deficit hyperactivity disorder; OCD, obsessive c.e disorder; SSRI, selective serotonin reuptake inhibitor.

are generally not recommended for children. W W

toms of GAD may wax and wane, the cours frequently lifelong.

.-

p-

f _ A rder is

b

A 9-year-old girl is new to your practice. She has an existing diagnosis of obsessive compulsive disorder (OCD). Prior to

treatment with a combination of a selective serotonin reuptake inhibitor (SSRI) and psychotherapy, she had persistent and intrusive worries about coming in contact with germs

and becoming ill. She had developed an intricate ritual of

washing her hands a certain way several times a day and

never eating in public to keep these worries at bay.

Question 22-1 What comorbidity should you be most concerned about monitoring her for? A) Tourette’s disorder.

B) Schizophrenia. C) Generalized anxiety disorder. D) Oppositional defiant disorder.

Discussion 22-1 The correct answer is “C.” Obsessions are reoccurring or persistent thoughts or urges that drive compulsions. A compulsion is a repetitive act (ritual) or mental act done in response to an obsession or need to rigidly follow the rules. The features of OCD are unwanted and interfere with the child’s and or adolescent’s life. Examples of compulsions include hand washing, counting, and checking. A child with OCD may fear an intruder entering the house (obsession), and will check the locks over and over (compulsion) to make sure the house is safe. Over 50%

of children with OCD have a comorbid psychiatric diagnosis. Another anxiety disorder is the most common comorbidity. ADHD is also a common comorbidity. Tourette’s disorder is often associated with OCD but is less prevalent than ADHD or another anxiety disorder. Disruptive behavior disorders, such as ODD can occur as well, but less frequently than the others. Concurrent psychotic disorders are very rare. 0

.5 H

HelpfulTip

Psychiatric disorders frequently occur with other psychiatric disorders. It is important to treat the primary disorder and comorbidities.

CHAPTER 3

During a well-child visit, the father of a 7-year-old boy expresses concern about his son's fear of elevators. His son avoids getting in elevators at all costs: It can take more than 30 minutes to convince him to get into one. When forced to ride in an elevator, he becomes very distressed and often starts crying. As an aside, the father mentions that boy's mother has an intense fear of spiders. Question 23-1 What do you tell the father? A) The child has panic disorder. B) The child has a specific phobia. C) The child has panic attacks. D) The child has agoraphobia Discussion 23-1 The correct answer is "B:' The child specifically is scared of ele­ vators. A specific fear becomes a specific phobia when it causes great distress, impairment, or both. A phobia may be tied to a prior traumatic event involving the object or situation. Com­ mon phobias include animals, insects, heights, water, blood, needles, airplanes, or clowns. Risk of developing a specific pho­ bia is increased if a first-degree relative is affected. Agorapho­ bia is a generalized fear of situations or places in which escape would be difficult. Agoraphobia may be present with panic dis­ order. Panic disorder is characterized by recurrent, unexpected panic attacks. A panic attack is the sudden onset of fear and feeling of impending doom without any real danger; it may be unexpected or triggered by certain situations. Exposure to the specific phobia may trigger a panic attack such as in the patient described.

A 1 7-year-old girl is evaluated for recurrent episodes in which she relives the car crash in which she was a passenger 6 months ago. Her mother speaks for her. The adolescent's friend was killed in the crash. Going by the accident site or hearing the song that was playing on the radio just before the crash causes her to freeze and the memories flood her mind. Afterward, she has difficulty falling asleep and is irri­ table. When asked about the events, the adolescent avoids the conversation. Question 24- 1 Regarding her disorder, which of the following is true? A) Reminders of the event do not cue strong emotions or distress. B) Patients of all ages present with "flashbacks:' C) Difficultly sleeping or concentrating may persist for pro­ longed periods after the event. D) Stimuli and reminders of the event are calming.



BEHAVIORAL AND M ENTAL H EALTH ISSUES

65

Discussion 24- 1 The correct answer is "C:' Posttraumatic stress disorder (PTSD) develops after a traumatic event associated with real harm, near or actual death, or injury. The traumatic event is recurrently reexperienced. Reminders are avoided and cause significant distress. Avoidance behaviors include unwilling­ ness to talk about the trauma, feeling detached from others, decreased interest in related activities, inability to recall event details, and avoidance of stimuli associated with the trauma. Presentation varies by age. Preverbal children may be clingy, irritable, refuse to explore their environment, aggressive, dis­ rupted sleep, or hard to soothe. Verbal children may rapidly cycle through emotions or reenact the event. Adolescents may experience flashbacks or heightened arousal (insomnia, hypervigilance, angry outbursts, poor concentration) when reminded of the event. Symptoms must be present for more than 1 month for diagnosis of PTSD.



Helpful Tip

� Many psychiatric disorders are extensions of norma l 1 1 1 r behaviors and emotions except more intense, and

cause impaired functioning in one or more areas of life.

A 14-year-old boy is brought to the clinic by his family because they are concerned about his behavior. For the last 2 months he has been very irritable. He has not been sleep­ ing well and complains that it takes several hours each night before he is finally able to doze off. He has been going to school, but his grades have started to slide and he states he "just can't concentrate:' Although he has always been social and outgoing, lately he has preferred to stay home play­ ing video games online. This has been a source of multiple arguments with his father. His mother recently discovered he has been cutting his upper arm with a razor. When she confronted him about this, he became agitated and insisted that he was not suicidal. Question 25-1 What is the best diagnosis? A) Major depressive disorder. B) Bipolar disorder. C) Borderline personality disorder. D) Oppositional defiant disorder. Discussion 25-1 The correct answer is "A:' Children and adolescents with depression often present with symptoms of increased irrita­ bility rather than sadness. This adolescent's sleep difficulties, poor concentration, and social withdrawal are commonly seen in pediatric depression, as well. While patients with bipolar disorder who are experiencing a manic episode frequently

66

MCG RAW-H I LL EDUCATION SPECIALTY BOARD REVI EW: PEDIATRICS

have a decreased need for sleep, they generally do not com­ plain about their limited sleep and instead describe having a great deal of energy and feeling very motivated. Cutting can be a symptom of borderline personality disorder, but adolescents with a variety of psychiatric disorders engage in this behavior. In addition, personality disorders are not diagnosed in chil­ dren and adolescents because their personalities are not com­ pletely developed. Adolescents with depression can display oppositional and defiant behavior, but a diagnosis of ODD cannot be made if this behavior only occurs in the context of a mood disorder. The family listens intently as you discuss your concerns that their son may have major depressive disorder. The family asks, "How do you know he isn't just in a funk and isn't a nor­ mal moody teenager? " Question 25-2 What should you tell the family? A) Mood swings are common in adolescents. B) Mood swings do not interfere with daily life functions. C) Mood swings are not associated with depressive symptoms (weight changes, concentration problems) . D) Mood swings are not associated with cutting behaviors. E) All of the above. Discussion 25-2 The correct answer is "E:' Adolescents may experience mood swings and argue frequently with parents but life should not be disrupted nor should depressive symptoms of sleep disturbances, lack of energy, and so forth, be present. Depressive disorders include major depressive disorder (MDD), persistent depres­ sive disorder (formerly dysthymia), disruptive mood dysregula­ tion disorder (DMDD), and premenstrual dysphoric disorder. For diagnosis of MDD, symptoms must be present for 2 weeks, impair functioning, and must include either a depressed or irri­ table mood, or loss of interest and pleasure in activities.

Question 25-3 How should you manage the adolescent's depressive symptoms? A) Prescribe citalopram. B) Refer him for psychotherapy. C) Refer him for psychotherapy and prescribe citalopram. D) Provide reassurance and schedule frequent follow-up. Discussion 25-3 The correct answer is "C:' It is important provide education and engage the family and school. Mild to moderate symptoms may be treated with an initial trial of psychotherapy. If no or mini­ mal response, a selective serotonin reuptake inhibitor (SSRI; eg, citalopram) should be prescribed. For moderate to severe symp­ toms, psychotherapy and an SSRI should be prescribed. Frequent follow-up is important, but his symptoms require treatment; therefore, reassurance would be inappropriate. It is important to recognize and treat comorbid conditions. As many as 90% of children and adolescents with depression have an additional psy­ chiatric disorder, most frequently anxiety disorders, disruptive disorders (eg, conduct disorder), ADHD, or substance abuse.



Helpful Tip

� Major depressive disorder is characterized as mild,

1 1 1 r moderate, or severe. If you have the ba re minimum of symptoms to make the d iag nosis, it is mild depression. If you have substantial excess number of sym ptoms AND the intensity of the symptoms is seriously distressing or unmanageable, it is severe depression. Somewhere in the middle lies moderate depression.



Helpful Tip

� Suicide

risk may increase after starting a SSRI

r1 1 r medication in a depressed adolescent, therefore; close monitoring is warranted.



Helpful Tip

� Major depressive disorder is more than just being

1 1 1 r sad. In order to be diag nosed with MDD, the person

must have 2 weeks of symptoms, one of which is either depressed mood or loss of pleasure in normal ly pleasurable activities. At least five of the fol lowi ng nine symptoms must be present: Depressed mood Loss of interest or pleasure Weight loss Sleeping more or less than usual Psychomotor agitation or retardation (observa ble, not just subjective) Fatigue or low energy Feelings of worth lessness or guilt Difficulty thinking clearly Thoughts of death or suicide.



Helpful Tip

:5.� It

is important to recognize and treat comorbid

r1 1 r conditions. As many as 90% of children and adolescents

with depression have an additiona l psychiatric disorder, most frequently anxiety disorders, disruptive disorders (eg, conduct disorder), ADHD, or su bstance abuse.

� QUICKQUIZ Which of the following is NOT a risk factor for developing major depressive disorder? A) Good coping skills. B) Difficulties with academics.

CHAPTER 3

C) Bullying. D) Neglect. Discussion The correct answer is "A:' MDD is often a familial disorder as are many other psychiatric disorders including anxiety disorders and bipolar disorder. The most predictive risk factor is a fam­ ily history of depression or anxiety. Other risk factors include abuse, family dysfunction, neglect, loss of a loved one, bullying, poor coping skills, social stressors, presence of comorbid psy­ chiatric disorder, and chronic medical illness. The adolescent is prescribed an SSRI and referred to psycho­ therapy. At follow-up, he admits to thinking about suicide and feeling hopeless. He stays in his room the majority of the time avoiding others. You recognize he is at increased risk for suicide.



BEHAVIORAL AND M ENTAL H EALTH ISSUES

67

of Suicidology created a helpful mnemonic to remember the warning signs of suicide: "Is (the) path warm? " I - Ideation S P

Substance abuse

-

-

Purposelessness

A - Anxiety T - Trapped H

Hopelessness

-

W

-

Withdrawal

A - Anger R M

-

-

Recklessness Mood changes

� QUICKQUIZ

Question 25-4 What is the strongest predictor of whether he will commit suicide? A) His diagnosis of depression. B) His cutting behavior. C) The absence of guns in the home. D) The fact that he has never made an attempt before.

Which of the following is NOT a protective risk factor for suicide? A) Supportive family members. B) Good grades. C) An established relationship with a therapist. D) Passing a firearms safety course.

Discussion 25-4 The correct answer is "D:' The strongest predictor of future suicidal behavior is past suicide attempts. His depression diagnosis and cutting do put him at slightly higher risk. Nearly 90% of children and adolescents who commit suicide had a psychiatric diagnosis. Having guns in the home signifi­ cantly increases the risk of completing suicide as well. Rates of completed suicide are higher after puberty. Females are more likely to attempt suicide, but males are more likely to complete suicide.

Discussion The correct answer is "D:' Having an understanding about the risks and safety measures with firearms is not a protective fac­ tor and, in fact, easy access to firearms is a significant risk fac­ tor. Protective risk factors include having good community and family support, and access to effective care for mental health, physical health, and substance abuse concerns. Risk factors associated with suicide include a history of previous suicide attempts, a family history of suicide, untreated mental health problems or chronic illness, substance abuse, isolation, or access to lethal means. (See Table 3 - 1 0. )

Question 25-5 What is NOT a concerning sign that a child or adolescent is suicidal? A) Talking about suicide. B) Feeling hopeless. C) Feeling he or she has no purpose in life. D) Difficulty sleeping. Discussion 25-5 The correct answer is "D:' Difficultly sleeping is a symptom of depression but not necessary a red flag for suicide. Suicide is the third leading cause of death for adolescents. Talking or ask­ ing about suicide does not cause suicidal actions or ideations in children or adolescents. Primary care providers should ask about suicide as part of routine adolescent health screening. Any patient who threatens suicide, talks about hurting himself or herself, or has a plan or is looking for ways to kill himself or herself should be emergently referred to a mental health pro­ vider or emergency department. The American Association



Helpful Tip

=-� Confidential ity is important and laws vary by state. Exceptions to confidentia lity include suspected or reported child abuse, suicidal or homicidal thoug hts, and violent behaviors. Abuse must be reported to the state. The others should be reported to the legal guardians.

r1 1 r

A 1 7 -year-old is brought to the emergency department by her family. She won't sit down to be interviewed, but instead paces back and forth. Her speech is rapid and you struggle to break into her rambling monolog to ask questions. She admits she has slept only 1 to 2 hours a night for the last week, but states this isn't a problem and has allowed her to dedicate

68

MCG RAW-H I LL EDUCATION SPECIALTY BOARD REVI EW: PEDIATRICS

TABLE 3-1 0 P ROTECTIVE AND RISK FACTO RS

FOR SUICIDE

Risk Factors

Protective Factors

Previo u s suicide atte m pts

Fa m i ly s u p po rt

Fa m i ly h i story of su icide

Com m u n ity s u p port

H i story of a buse or negl ect

Access to effective me nta l health care

Mental health d i agnosis

Access to effective physical health care

Alcohol a n d s u b­ sta nce a b use

Access to effective su bsta nce a b use ca re

I m pu l sive or a g g ressive te ndencies

S ki l l s i n problem solvi ng a n d confl i ct reso l ution

Cu ltura l bel iefs that su icide i s a noble resol ution

Cu ltura l a n d be l i efs that d i sco u r­ age s u icide a n d viole nce

Isolation

Perso n a l satisfaction and a reas of ach ievement

C h ro n i c I l l ness Easy access to let h a l mea n s Barriers t o me nta l health treatment

more time to her artwork. She has been painting everything in her bedroom purple and even tried to paint the family dog when he made the mistake of entering her room. She tells you proudly that "monochrome is the next big thing" in interior decorating and she's convinced she is starting a new trend that will "spread like wildfire!" Question 26-1 How would you treat this adolescent? A) Start fluoxetine. B) Start lamotrigine. C) Start lisdexamfetamine. D) Start risperidone. Discussion 26- 1 The correct answer is "D:' She is clearly experiencing a manic epi­ sode. While lithium and anticonvulsants have been well studied as mood stabilizers for adults with mania, the atypical antipsychot­ ics (eg, risperidone) have been the most researched in children and adolescents with mania. Some patients with bipolar disor­ der require a combination of a mood stabilizer for their manic symptoms and an antidepressant such as an SSRI (eg, fluoxetine) for their depressive symptoms. However, this combination is typically not started at the same time. The mood stabilizer is usu­ ally initiated first and then the antidepressant is added cautiously

once the patient's mania is under control. Antidepressants should be used with caution in the absence of a mood stabilizer as they can exacerbate manic symptoms. Lamotrigine is used as a mood stabilizer but is not indicated to reduce mania. It is effective in treating depressive symptoms for patients with bipolar disorder. Lisdexamfetamine is a stimulant used for ADHD. Some of this adolescent's behaviors are similar to hyperactivity; however, it is unlikely that she has suddenly developed ADHD in late adoles­ cence. Stimulants should be used with caution in patients with mania as they can exacerbate manic symptoms.

A 16-year-old girl was admitted for evaluation of seizures. She was monitored by video electroencephalogram (EEG) for 2 days. Although several episodes of seizure-like body movements were recorded on the video, her EEG remained essentially normal throughout her evaluation. An exten­ sive workup was completed and no other physical cause for her movements could be found. Her mother notes that the problem began 2 weeks after one of her friends was sexually assaulted at a party. She had left the party early without her friend and has been feeling guilty about it. The mother wor­ ries that someone might have slipped something into her daughter's soda and that has caused the seizures. Question 27- 1 What i s the diagnosis? A) Hypochondriasis. B) Conversion disorder. C) Malingering. D) Factitious disorder. Discussion 27- 1 The correct answer i s "B:' The definition of conversion disor­ der is the presentation of a neurologic impairment ( eg, non­ epileptic seizures, weakness, vision changes) that cannot be explained by any medical cause. The symptoms are produced subconsciously and the patient does not believe he or she has control over them. Frequently the onset is associated with a psychological stressor. Hypochondriasis is the fear of having an illness and is not necessarily associated with the develop­ ment of symptoms. Malingering and factitious disorder are diagnosed when the symptoms are produced consciously by the patient (ie, "faked" ) . In factitious disorder, the patient is seeking primary gain; that is, he or she receives psychologi­ cal benefit from assuming the sick role. In malingering, the patient is seeking secondary gain, attempting to get out of something (eg, school) or gain something (eg, money) .

A 1 4-year-old boy is referred to your clinic with a 2-year history of abdominal pain. He cannot describe the pain well. It does not awaken him from sleep nor does it

CHAPTER 3

interfere with activities. He has had no fevers, diarrhea, vomiting, or weight loss. On examination, he is at a proper weight and height, and appears well. The exam findings, including vital signs, are normal. You review a 4-inch stack of records to find that among the hundreds of tests that have been performed all are normal except the serum vitamin D level, which is low. You suspect his symptoms are psychological. Question 28-1 Which of the following should NOT be used to manage the adolescent? A) Education. B) Referral for a second opinion with a gastroenterologist. C) Psychotherapy. D) Pharmacotherapy. Discussion 28-1 The correct answer is "B:' Psychosomatic disorders are physical manifestations of emotional stress. The clinical symptoms are not due to an underlying medical illness. Patients may present with various physical complaints, including chest pain, abdomi­ nal pain, and headache. They are often diagnoses of exclusion but extensive medical evaluations may reenforce the thought that an undiagnosed medical illness is present. A thorough his­ tory and physical exam may help identify red flags that would warrant a diagnostic evaluation. Clues include vague, inconsis­ tent, and multiple symptoms present at the same time. Patients are in good health and have benign physical exams. Education is key and should focus on coping skills. Unnecessary medical evaluation should be avoided. Comorbid psychiatric conditions should be treated. Frequent office visits may be helpful. Psycho­ therapy, antidepressants (SSRI), or a combination are useful treatments.

A healthy 8-year-old boy has been struggling in school. After completing a thorough clinical interview and obtaining rating scales from his parents and his teach­ ers, you diagnose him with ADHD and prescribe 5 mg of methylphenidate each morning. At his follow-up appoint­ ment 2 weeks later, his parents report no change in his behavior and his teacher's rating scales show that he is doing worse. Question 29-1 What is the best next step? A) Refer him for neuropsychological testing to evaluate for learning disorders. B) Switch him to an amphetamine-based stimulant C) Increase his methylphenidate dose. D) Refer him to a psychologist who specializes in social skills training.



BEHAVIORAL AND M ENTAL H EALTH ISSUES

69

Discussion 29-1 The best answer is C. According to the American Academy of Child and Adolescent Psychiatry and Texas Children's Medi­ cation Algorithm Project, an adequate trial with a methylphe­ nidate-based stimulant, an amphetamine-based stimulant, or atomoxetine is the first step in treating ADHD. He is on a low dose of methylphenidate and the dose-response relationship has been established as a linear response. Before switching to an alternative medication, the next step is to try to optimize the current medication. If he is given a trial of a higher dose and still does not respond, the next step in the algorithm is to switch to a different stimulant. If he does not respond to a second trial or it is determined that he only has difficulty in certain subjects, assessing for learning disorders can be helpful. Psychotherapy has been found to be beneficial for children with comorbid anx­ iety, mood, or disruptive behavior disorders. Common comor­ bidities include ODD, conduct disorder, substance abuse issues, learning disability, depression, and anxiety disorders. You review the patient's original presenting symptoms, which were reported 9 months before your evaluation. He was described then as "spacey:' forgetful, and careless. His mother constantly had to bring his schoolwork or lunch box to school as he would forget it. Last year, he lost his lunch card three times and his mother paid for a new one each time. He dislikes playing board games as he says they require too much focus. His mother can't understand why his room is such a mess and he can't organize it. When ask to feed the dog, he will some­ times leave the bag of food next to the empty bowl. Question 29-2 What was his initial presentation of ADHD? A) Combined presentation. B) Predominately inattentive. C) Predominately hyperactive-impulsive. D) He did not qualify for the diagnosis of ADHD. Discussion 29-2 The correct answer is "B." ADHD is characterized by hyperactivity, impulsivity, inattention, or a combination. The pathogenesis of ADHD is unknown but thought to be related to catecholamine metabolism (deficient dopamine activity) in the brain. A number of genes associated with ADHD have been identified, supporting a genetic com­ ponent. Imaging and testing suggests impaired executive function due to abnormalities in the prefrontal cerebral cortex and basal ganglia. The brain matures normally but at a delayed rate. ADHD is subdivided as predominately inattentive (meets symptom criteria for inattentive only) , predominantly hyperactive-impulsive (meets symptom cri­ teria for hyperactive- impulsive only) , or combined presen­ tation (meets symptom criteria for b oth ) . Symptoms must be present for 6 months, inappropriate for developmental level, cause impairment, onset during childhood, and occur in two or more settings.

70

MCG RAW-H I LL EDUCATION SPECIALTY BOARD REVI EW: PEDIATRICS



Helpful Tip

:5.� HEY

i1 1 r



LOOK! A SQUIRREL! Still reading? Then you probably don't have attention deficit hyperactivity disorder. Or at least not the inattentive type. However, if you are ru nning around a child's playground and blurting out the answers before you even finish reading the question, then you might have the hyperactive­ impulsive type.

Helpful Tip

� At least six of the following nine symptoms must be I

1 1 r present to diagnose inattentive-type ADHD:

Poor attention to detail Difficulty sustaining attention Does not seem to listen Does not follow through on instructions Difficulty orga nizing tasks Avoids or dislikes tasks that require sustained mental effort Loses things Easily distracted Forgetful



Helpful Tip

:5.� At least six of the fol lowing eig ht symptoms must be

i1 1 r present to diag nose hyperactivity- and

impulsivitytype ADHD: Fidgets or squirms Leaves seat inappropriately Runs a bout or climbs when not appropriate Cannot sustain quiet play "On the go" or "driven by a motor" Tal ks excessively Blurts out answers before the q uestion is completed Difficulty waiting for his or her turn

Question 29-3 What is an appropriate diagnostic evaluation for ADHD? A) ADHD checklist completed by parent and teacher. B) Brain MRI. C) EEG. D) Urine drug screen. Discussion 29-3 The correct answer is ''A:' The ADHD checklist should be com­ pleted by the parent and teacher. Medical testing such as thy­ roid function tests or lead level should be obtained as clinically indicated.

};{ QUICKQUIZ Which is a true fact about ADHD? A) Males are more frequently affected than females. B) ADHD is familial. C) Symptoms may persist into adulthood. D) All of the above. Discussion The correct answer is "D:' The male-to-female ratio is 3 : 1 . ADHD tends to run i n families. A genetic basis exists as stud­ ies have shown ADHD to be strongly inherited, with up to 76% heritance. Most children with ADHD have symptoms into ado­ lescence and some continue to have problems into adulthood that may require treatment.

A 9-year-old boy is referred to your clinic from his school due to concerns about aggressive behaviors. He is easily angered and, when angry, lashes out. He has kicked and punched objects, and broken things. He will grit his teeth, tense up his face and body, and scream. Most recently another child was using a green marker, which he wanted. When he couldn't have it he kicked the wall. Question 30-1 Regarding his aggressive behaviors, what do you not tell the school? A) His response is developmentally inappropriate. B) He has not developed proper problem-solving skills and self-control. C) He has not developed proper social skills. D) Children do not outgrow their aggression until junior high. Discussion 30-1 The correct answer is "D:' Aggression and anger in response to frustration is normal in young children. As the child learns social skills, conflict resolution, problem solving, and self-con­ trol these behaviors subside. Aggression becomes problematic when it persists, is severe, or happens frequently. For exam­ ple, a toddler whose stuffed animal is taken away by his sister may respond by hitting her. The toddler has not learned how to handle his emotions. However, a 9-year-old should be able to work out conflict without resorting to physical aggression. Have you ever heard parents tell their child, "use your words"? The parents are trying to develop coping skills in their child. Some children have a genetic or psychological predisposition to aggression. Children who witness aggression by others, includ­ ing role models and parents, are more likely to be aggressive themselves. Aggressive children may have problems processing and interpreting social cues.

CHAPTER 3

A 1 3-year-old girl comes into the clinic for a sore throat. During casual conversation, you ask about school. She won't make eye contact. When she finally answers, she tells you that the popular girls are spreading rumors about her and trying to make others not want to be her friend. You recognize she is the victim of bullying. Question 3 1 - 1 Which is NOT an example of bullying? A) The fastest child in class making fun of the slowest child. B) The science nerd and math geek getting into a fistfight. C) The high school heartthrob making fun of the class nerd. D) The leader of the popular clique of girls spreading rumors about a new girl in school.



BEHAVIORAL AND M ENTAL H EALTH ISSUES

71

A 1 6-year-old male i s brought t o the emergency depart­ ment by his mother for odd behaviors. He reports seeing unicorns. He talks about the troll god's secret plot to steal the unicorn. He has come up with a plan to save the uni­ corn. As part of his plan, he was going to paint the unicorn, TABLE 3 - 1 1

CAU S E S OF PSYC H O S I S

D r u g overdose or a b use Anti c h o l i nergics Sti m u l a nts (eg, a m pheta m i nes, M D MA) H a l l ucinogens (eg, PCP, LSD)







Drug-i n d u ced psychosis Steroids Ison iazid Anticonvu lsa nts







Discussion 3 1 - 1 The correct answer is "B." To constitute bullying, the two individuals involved cannot be on equal footing. One must be younger, smaller, or less powerful. Bullying is a form of aggression toward peers . It is a repetitive event with the intent is to harm another person physically or socially. There is an imbalance of power. The bully has either more physical or more psychological power than the victim. If two light­ weights get into a fight, it isn't bullying. But a heavyweight taking on a lightweight is bullying. Aggression displayed by boys is typically overt and physical. Aggression by girls is more likely to be covert and relational, focusing on damag­ ing someone's social status by spreading rumors, gossiping, or excluding others. Question 3 1 -2 Which is a management strategy for victims of bullying? A) Avoid hot spots where bullying occurs. B) When hot spots are unavoidable, travel with a friend who isn't bullied. C) Identify a trusted adult at school to turn to when problems arise. D) At recess, join structured activities such as a game of soccer or tag. E) All of the above. Discussion 3 1 -2 The correct answer is "E:' Options A through D are all correct.

Drug-re lated syn d romes Seroto n i n syn d ro m e Withd rawa l N e u roleptic m a l i g nant syn d ro m e







Centra l nervo u s syste m ma sses Tu mor Hyd rocepha l u s Abscess •





I ntracra n i a l i nj u ry Epid u ra l he matoma S u bd u ra l hemorrhage





Stroke Seizure M i g ra i n e M e n i n g itis or encep h al itis Syste m i c l u p u s erythe matosus Meta b o l i c or endocri ne d i sorders U rea cycle defects Thyroid sto rm Hypothyro i d i s m H a s h i moto encep h a l opathy Ad re n a l i n s ufficie ncy U re m i a Hypera m m o n e m i a



Electrolyte a bnormal ities Hypog lyce m i a Hyperg lyce m i a Hyponatre m i a







Menstrual psychosis •

Helpful Tip

:5.� Cyberbul lying

involves infl icting harm using media, including computers, social networks, and cel l phones. I n contrast to traditional bul lying, it does not involve a power imbala nce and is not always repetitive.

r1 1r electronic

Postpa rtu m psychosis Psyc h i atric d i sorders Schizo p h re n i a Depression B i po l a r d i so rder







LSD, lyserg ic acid d i ethyl a m i de; M D MA, 3,4-methylenedioxy­ metha m p heta m i ne; PCP, phe ncyc l i d i ne.

72

MCG RAW-H I LL EDUCATION SPECIALTY BOARD REVI EW: PEDIATRICS

disguising it as a horse with a horn. His mother found him rummaging through the paint collection and brought him in for evaluation. Question 32-1 Which of the following is NOT a possible cause of his psychosis? A) Substance abuse. B) Electrolyte derangement. C) Stroke. D) Conduct disorder. Discussion 32-1 The correct answer is "D:' Psychosis is defined as disrupted thinking with delusions or hallucinations. Delusions are fixed, false beliefs. Patients who have hallucinations see or hear things that aren't present. The differential diagnosis for acute psychosis is lengthy (see Table 3 - 1 1 ) . Identifying whether the symptoms are acute or chronic is helpful in narrowing the differential. Acute onset suggests a medical etiology. Chronic symptoms suggest a psychiatric cause. Before diagnosing a psychiatric disorder, other medical causes must be ruled out. Management includes ensuring the patient's safety (may require restraints) , treating reversible conditions, and completion o f a diagnostic evaluation to identify the underlying etiology. •

Helpful Tip

=-� Most children with hal luci nations do not have schizophrenia and many do not have a psychiatric disorder.

r1 1 r

B I B LIOGRAPHY

American Academy of Pediatrics. Building resilience in children. https:/ /www.healthychildren.org/English/healthy-living/ emotional-wellness/Building-Resilience/Pages/Building­ Resilience-in-Children.aspx. Accessed June 10, 2015. American Association of Suicidology. Know the warning signs of suicide. http:/ /www. suicidology.org/resources/warning­ signs. Accessed June 30, 20 1 5 . American Psychiatric Association. Diagnostic and Statistical Manual of Mental Disorders: DSM-5. Washington, DC: American Psychiatric Association; 20 1 3 . Brill SR, Patel DR, MacDonald E. Psychosomatic disorders in pediatrics. Indian J Pediatr. 200 1 ;68(7) : 597-603. Carruth BR, Ziegler PJ, Gordon A, Barr SI. Prevalence of picky eaters among infants and toddlers and their caregivers' decisions about offering a new food. J Am Diet Assoc 2004; 1 04:S57 -S64. Centers for Disease Control and Prevention. Attention-deficit/ hyperactivity disorder (ADHD ). http:/ /www. cdc.gov/ ncbddd/adhd/diagnosis.html. Accessed June 30, 20 1 5 .

Cohen GM, Albertini LW Colic. Pediatr Rev. 2 0 1 2;33(7) :332-333. doi: 1 0 . 1 542/pir.33-7 -332. Cohen JA, Bukstein 0, Walter H, et al. Practice parameter for the assessment and treatment of children and adolescents with posttraumatic stress disorder. J Am Acad Child Ado­ lese Psychiatry. 2 0 1 0;49(4):414-430. Committee on Psychosocial Aspects of Child and Family Health. Guidance for effective discipline. Pediatrics. 1 998; 1 0 1 ( 4) : 723-728. Connolly SD, Berstein GA. Practice parameter for the assess­ ment and treatment of children and adolescents with anxiety disorders. J Am Acad Child Ado lese Psychiatry. 2007;46(2) :267-283 . Dulcan M , ed. Dulcan's Textbook of Child and Adolescent Psychiatry. Washington, DC: American Psychiatric Publishing; 20 1 0 . Egger H L , Costello EJ, Angold A. School refusal and psychi­ atric disorders: A community study. J Am Acad Child Adolesc Psychiatry. 2003;42:797-807. Findling R, ed. Clinical Manual of Child and Adolescent Psy­ chopharmacology. Washington, DC: American Psychiatric Publishing; 2008. Fisher WW, Piazza CC, Roane HS, eds. Handbook ofApplied Behavior Analysis. New York, NY: Guilford Press; 20 1 3 . Ginsburg KR. Resilience i n Children and Teens: Giving Kids Roots and Wings. Elk Grove Village, IL: American Academy of Pediatrics; 2006. Michel RS. Toilet training. Pediatr Rev. 1 999;20(7):240-245. doi: 10 . 1 542/pir.20-7-240. Practice parameter for the assessment and treatment of children and adolescents with attention-deficit/hyper­ activity disorder. J Am Acad Child Ado lese Psychiatry. 2007;46(7):894-92 1 . Practice parameter for the assessment and treatment of chil­ dren and adolescents with bipolar disorder. J Am Acad Child Adolesc Psychiatry. 2007;46 ( 1 ) : 1 07- 125. Practice parameter for the assessment and treatment of chil­ dren and adolescents with depressive disorders. J Am Acad Child Adolesc Psychiatry. 2007;46 ( 1 1 ) : 1 503- 1 526. Practice parameter for the assessment and treatment of children and adolescents with obsessive-compulsive disorders. J Am Acad Child Adolesc Psychiatry. 2012;5 1 ( 1 ) :98- 1 1 3 . Reimers TM. Help! There's a toddler i n the house! Boystown, NE: Boystown Press; 20 1 1 . Roberts DM, Ostapchuk M, O'Brien JG. Infantile colic. Am Pam Physician. 2004;70(4) :735-740. Shetgiri R. Bullying and victimization among children. Adv Pediatr. 2 0 1 3;60( 1 ) :33-5 1 . doi: 1 0 . 1 0 1 6/j. yapd.20 1 3 . 04.004. Williams KE, Gibbons BG, Schreck KA. Comparing selective eaters with and without developmental disabilities. J Dev Physical Disabil. 2005; 1 7:299-309.

Blood and Neoplastic Disorders

4

Ad a m D. Wo lfe

A 12-month-old boy presents to your clinic for his health supervision visit. He is growing and developing normally. His diet consists of approximately 30 ounces of cow's milk daily, as well as mixed table foods. His heart rate is 1 22 beats per minute. Physical exam reveals gingival and conjunctival pallor and a grade 1/VI systolic ejection murmur heard over the left sternal border. Question 1 - 1 Which of the following laboratory values is likely to be elevated in this patient? A) Ferritin. B) Haptoglobin. C) Hemoglobin. D) Mean corpuscular volume. E) Platelet count. Discussion 1 - 1 The correct answer is "E:' This patient's age, history, and physi­ cal exam findings are most consistent with iron deficiency anemia (IDA) . A CBC performed in a patient with IDA would confirm decreased hemoglobin (making option "C" incorrect) and abnormal red cell indices including microcytosis (making option "D" incorrect) , and an increased red cell distribu­ tion width, reflective of insufficient iron for the construction of hemoglobin and red cells in the bone marrow. Children with IDA often have a reactive thrombocytosis, which is due to cross-reactivity of erythropoietin, elevated in the anemic state, with the megakaryocyte-stimulating thrombopoietin receptor. The blood smear should reveal red cell microcytosis, hypochromia, and anisocytosis. (See Figure 4- 1 . ) Laboratory evaluation of iron may include serum ferritin, which reflects hepatic iron stores and is therefore typically low in IDA (mak­ ing option "/\' incorrect) . Serum iron is also likely to be low, while total iron-binding capacity, reflective of hepatic transfer­ rin production in an effort to scavenge additional iron, is likely

to be elevated. Serum haptoglobin is not routinely checked in cases of suspected iron deficiency, in which it would not likely be affected (making option "B" incorrect); haptoglobin is often decreased in patients with hemolytic anemias. IDA is the most common hematologic condition in pediatrics, affecting an estimated 3% to 7% of toddlers and up to 9% of menstruat­ ing young women. Toddler-aged children are at greatest risk of IDA when consuming large amounts of cow's milk, as milk interferes with intestinal absorption of dietary iron. Data sug­ gest that children with iron deficiency, even in the absence of anemia, are at increased risk of neurocognitive deficits, lower IQ, behavior problems, and cardiovascular changes, presum­ ably due to altered function of nonhematopoietic enzymatic processes that depend on iron.

A 1 2-year-old girl with no past medical history presents to the emergency department after suffering a brief syncopal episode in her bathroom at home. Her history is remarkable for an influenza-like febrile illness last month, followed by lingering fatigue and daytime sleepiness. There is no per­ sonal or family history of symptoms like these. She has nor­ mal vital signs apart from heart rate of 1 1 2 beats per minute. Physical exam reveals an awake and alert girl lying in bed, who answers questions appropriately. She has marked facial, conjunctival, and gingival pallor, and scleral icterus. Cardio­ vascular exam reveals a grade II/VI systolic ejection murmur, capillary refill ofless than 3 seconds, and 2+ peripheral pulses in all extremities. Laboratory evaluation includes hemo­ globin 5.3 g/dL, white blood cell count 1 1 .9 x 1 03/mm\ and platelets 502 x 1 03/mm3• White cell differential includes 30% neutrophils and 60% lymphocytes. Evaluation of the blood smear reveals normocytosis and normochromia, occasional spherocytes, and no schistocytes. Serum total bilirubin and lactate dehydrogenase (LDH) are both elevated. 73

74

MCG RAW-H I LL EDUCATION SPECIALTY BOARD REVI EW: PEDIATRICS

FIGURE 4-1. Peripheral b l ood s m e a r fro m a patient w i t h i ron deficiency a n e m i a . The red b l ood cel l s exh i bit ma rked hypoch romia {ie, severe pa l lor). There is also su bsta ntia l va riation i n cel l size, or a n isocytosis. (Used with permission from Ada m D. Wolfe, M D, P h D.)

Question 2-1 Which of the following evaluations is most likely to yield this patient's diagnosis? A) Glucose-6-phosphate dehydrogenase (G6PD) enzyme activity. B) Direct antiglobulin (Coombs) test. C) Ferritin, serum iron, total iron binding capacity. D) Erythrocyte osmotic fragility test. E) Bone marrow aspiration and biopsy. Discussion 2-1 The correct answer is "B:' This patient presents with progressive symptoms of anemia, including fatigue, pallor, tachycardia, and ejection murmur, with jaundice, hyperbilirubinemia, and elevated LDH that suggest a hemolytic mechanism. Hemolytic anemias, in contrast to most nutritional anemias, typically exhibit nor­ mocytosis and normochromia on the CBC and smear. The onset of new symptoms abruptly following an acute viral illness make it probable that this patient has autoimmune hemolytic anemia (AIHA), and therefore a direct antiglobulin test is most likely to yield a diagnosis. This test will reveal the presence of antibodies directed against red cell membrane antigens. Because the mecha­ nism of red cell depletion involves clearance of antibody-coated cells by the spleen, this is a largely extravascular hemolysis that is unlikely to be accompanied by fragmented red cells as might be seen with intravascular hemolysis. The presence of occasional spherocytes is consistent with deformation of antibody-coated cells. AIHA may occur as an isolated event, or it may be associ­ ated with other immune cytopenias, such as thrombocytopenia (seen together in Evans syndrome) or neutropenia. The presence of autoimmune cytopenias in an adolescent patient should raise suspicion for the presence of other autoimmune disorders, and a thorough autoimmune family history and review of systems is appropriate. Further screening for thyroid dysfunction, systemic lupus erythematosus, and inflammatory bowel diseases may then be warranted. Management is generally aimed at quelling inflam­ mation, starting with corticosteroids. Once in remission, AIHA

may remain so following a taper of immune suppressive therapy, although recurrence is common. G6PD deficiency (option "A:') is an inherited enzymopathy that can present with severe hemo­ lytic anemia in the setting of acute illness, but it is inherited in an X-linked fashion, making it unlikely in this female patient without a family history. The timing of this patient's anemia after antecedent illness, jaundice, and normocytosis are incon­ sistent with a diagnosis of iron deficiency anemia (option "C") . Despite the presence o f occasional spherocytes, explained above, this patient's history of not having jaundice or other hematologic problems until this illness make this less likely to be hereditary spherocytosis, and osmotic fragility testing (option "D") is there­ fore not expected to be helpful. Finally, while acute leukemias are always in the differential diagnosis for an ill-appearing child with cytopenia, the history and lab findings are more consistent with a nonmalignant process. Should the patient have presented with multiple cytopenias, significant leukocytosis, or blasts on the blood smear, evaluation for leukemia with bone marrow assess­ ment (option "E") would be urgently warranted.

� •

I I

Helpful Tip

The presence of spherocytes on a periphera l blood smear may be noted in heredita ry spherocytosis or autoimmune hemolytic anemia. Order a direct antiglobulin test.

You are seeing a 6-month-old baby girl for sudden onset of jaundice. Her mother reports that the patient was healthy until 2 days ago, when she developed fever, cough, conges­ tion, and irritability. Last night, her skin appeared yellow, and this has worsened today. Mom reports that she under­ went splenectomy when she was 6 years old because of "a problem with her blood:' and the infant's maternal grand­ mother underwent splenectomy and cholecystectomy while in middle school. The exam is remarkable for a crying but nontoxic infant with temperature 38.5°C ( 1 0 1 .3°F), scleral icterus, diffuse jaundice, grade II/VI systolic ejection mur­ mur, and spleen palpable 3 em below the left costal margin. Lab work on the baby reveals white blood cell count of 14.6 x 1 03/mm3 with normal differential, platelets of 457 x 1 03/mm\ hemoglobin 7.3 g/dL, mean corpuscular volume 95 fL, reticu­ locytes 1 2%, and indirect bilirubin 6.6 mg/dL. Question 3-1 Which of the following tests is most likely to establish the diagnosis in this patient? A) Bone marrow aspirate and biopsy. B) Direct antiglobulin (Coombs) test. C) Erythrocyte osmotic fragility test. D) Glucose-6-phosphate dehydrogenase enzyme activity. E) Hemoglobin electrophoresis.

CHAPTER 4

Discussion 3-1 The correct answer is "C:' This baby exhibits an onset of acute, symptomatic anemia in the setting of a febrile illness; the presence of jaundice suggests a hemolytic process, most likely hereditary spherocytosis (HS) . This is suggested by the splenomegaly and mild macrocytosis, as well as by an auto­ somal dominant inheritance pattern indicated on family history. HS is caused by defects in structural red blood cell transmembrane proteins responsible for vertical interactions that link the cytoskeleton to the cell membrane. Mutations in genes coding for spectrin and ankyrin are most commonly associated with HS, and in some cases mutations in Protein 4.2 and Band 3 are also causative. While HS is inherited in an autosomal dominant fashion, an estimated one third of cases appear to be due to sporadic mutation. The management of HS is supportive and based on clinical severity, which can vary substantially between cases. Some neonates with more severe phenotypes exhibit symptomatic anemia brought about by the physiologic stress of birth and continue to have hemolytic episodes throughout infancy during acute illnesses. Infants with severe HS may also exhibit an exaggerated physiologic nadir within the first 2 months after birth. Infants requiring blood transfusions for HS-associated hemolytic episodes are the most likely to require splenectomy to control their disease at early ages. Note that splenectomy is discouraged in children younger than 6 years due to infection risk. At the other end of the spectrum, at least a quarter of children with HS may have only mild hemolysis, for which they adequately compensate, and develop no clinical problems. In general, individuals with HS are at increased risk of biliary sludging due to chronic red cell turnover and hyperbilirubinemia, and often undergo cho­ lecystectomy at early ages. Diagnosis of HS may be suspected based on red cell indices. Red cells of children with suspected HS are often subj ected to osmotic fragility testing, in which the cells are incubated in solutions of increasing tonicity; the weakened red cells of HS will exhibit increased hemolysis at relatively higher tonicity than control cells. Performance of this test is usually deferred until children are at least 6 months of age; therefore, this is an appropriate confirmatory test for the patient in the vignette. Because the patient is exhibiting an appropriate reticulocyte response, has no other cell lines down, and has evidence of hemolysis, a bone marrow failure syndrome or malignancy is unlikely to be causing the anemia (making option "/\_' less helpful) . The relative macrocytosis and the family history make autoimmune hemolytic anemia less likely, and while a direct antiglobulin test might be performed early in this patient's workup, it is expected to be negative (option "B" ) . Glucose-6-phosphate dehydrogenase (G6PD, option "D") deficiency could cause a clinical picture such as this patient exhibits, particularly with stress-induced hemo­ lysis, but this condition is inherited in an X-linked recessive fashion; the female patient plus family history in the vignette are therefore not suggestive of this diagnosis. Hemolysis may be seen in cases of certain hemoglobinopathies (investigated by option "E" ) , such as patients with acute exacerbation of sickle cell disease, untreated thalassemia major, or those with



BLOOD AND N EOPLASTIC DISORDERS

75

(very rare) unstable hemoglobins, but the laboratory findings in the vignette do not indicate any sickling or red cell inclu­ sions to indicate this etiology. •

Helpful Tip

:S� An infection with parvovirus B 1 9 may cause an acute

i1 1 r aplastic crisis from transient bone marrow suppression

in children with hereditary spherocytosis or other hemoglobinopathy.

� QUICKQUIZ Which is not a laboratory test finding in a child with hereditary spherocytosis? A) Spherocytes. B) Anemia. C) Low mean corpuscular hemoglobin concentration. D) Elevated mean corpuscular volume. E) Elevated lactate dehydrogenase. Discussion The correct answer is "E:' In hereditary spherocytosis, the anemia is typically normocytic to slightly macrocytic, and the mean corpuscular hemoglobin concentration (MCHC) is often elevated due to dehydration experienced by the red cells.

A 2-year-old boy is brought to your clinic for evaluation after an episode of painless gross hematuria. His mother reports that he had a red-tinged void in his diaper last night, although his void this morning was normal colored. The patient is oth­ erwise healthy, on no medications, and appears to be in no pain. His physical examination reveals a painless, left-sided abdominal mass palpated toward the flank, which does not move with respiration. Urinalysis in clinic today reveals trace heme and no red blood cells. You decide to order an abdomi­ nal ultrasound to evaluate the mass further. Question 4- 1 Which is the most likely diagnosis? A) Wilms tumor. B) Neuroblastoma. C) Splenomegaly. D) Pyelonephritis. E) Rhabdomyosarcoma. Discussion 4- 1 The correct answer is "A:' Painless gross hematuria in a toddler or preschool-aged child, even when intermittent, is highly sus­ picious for Wilms tumor (WT) . A newly-identified abdominal

76

MCG RAW-H I LL EDUCATION SPECIALTY BOARD REVI EW: PEDIATRICS C) Tumor lysis syndrome. D) Adrenal insufficiency. E) Thrombocytopenia. Discussion 4-2 The correct answer is "A:' Because WT arises within the kidney capsule, as it grows the risk of tumor rupture increases. Rupture may be caused by minimal trauma, such as falling for­ ward while running, as toddlers often do. Ruptured WT may become a life-threatening condition due to internal blood loss, although these tumors often bleed into the retroperitoneum and auto-tamponade limits the quantity of blood loss. Nevertheless, this presents a significantly greater surgical bleeding risk when the tumor is resected. Tumor rupture also seeds the abdomen, which upstages the cancer and leads to therapy with more inten­ sive chemotherapy and a wider radiation field.

FIGURE 4-2. W i l m s tumor. I n this a x i a l computed tomography image o f t h e a bdomen, there is a l a rg e encapsulated m a s s noted with i n the lower p o l e of the left kidney. The mass has no evidence of local infi ltration or calcifications. Renal parenchyma, shown by contrast u pta ke, is pushed aside by the tumor, suggesti n g an i ntrinsic ren a l p rocess. The left kidney rem n a nt appears to be g r a s p i n g or e n g u lfi n g t h e l ower-atte n u a t i o n t u m o r, a fi n d i n g sometimes descri bed a s a "c l a w s i g n :' ( U sed with permission fro m Ad a m D. Wo lfe, M D, P h D.)

mass is often palpated, and sometimes is the only finding, inci­ dental on a well-child visit, that identifies this tumor. Because the tumor is not adjacent to the diaphragm, it does not move with respirations as an enlarged spleen does (making option "C" incor­ rect) . Associated symptoms for WT are based on mass effect and may include constipation (constant or intermittent) and bladder dysfunction. Staging workup includes CT scan of the chest, abdo­ men, and pelvis; local extension and distant metastasis-often to the lungs-are both hallmarks of this disease. Therapy includes upfront resection of the affected kidney, followed by chemo­ therapy and radiation therapy dictated by stage. Cure rates for the most common forms of WT are excellent, well above 90%. Distinguishing WT from neuroblastoma (option "B"), another frequently encountered abdominopelvic tumor in the same age range, is not reliably done by physical exam or ultrasound. CT scan may be helpful, as neuroblastoma tends to exhibit calcifica­ tions, while WT arises within the kidney and pushes renal paren­ chyma toward the capsule. (See Figure 4-2.) Tissue diagnosis is required to be certain. Rhabdomyosarcoma (RMS; option "E") is a soft tissue sarcoma that may arise in nearly any tissue of the body. In boys, it is frequently found arising from the prostate or blad­ der, which would be unexpected to exhibit a flank mass if involving the midline urogenital system. RMS is also less common overall than WT. Finally, while pyelonephritis (option "D") may exhibit renal complications, the overall well appearance of the child and presence of a mass are not consistent with this diagnosis. Question 4-2 What is a potential complication of Wilms tumor? A) Tumor rupture with internal hemorrhage. B) Hypercalcemia.

A 14-month-old boy is brought to your clinic for rash. His father reports that he has a rash on his neck, elbows and legs, consisting of tiny flat red dots. He has also had increased bruising over the past week on his extremities. He is other­ wise healthy, although 2 weeks ago, he had nasal congestion, cough, and diarrhea lasting for 3 days. The patient has not had nosebleeds, bleeding gums, or blood in his urine or stool. He was circumcised at birth, without unusual bleeding. Family history is negative for easy bleeding or bruising. The patient appears very well on exam and is actively playing, running, and smiling. He has several violaceous bruises ranging in size from 2 to 10 em diameter on his extremities and trunk, and scattered petechiae in the distribution reported for the rash above. A CBC is performed, revealing a platelet count of 9 x 103/mm3; all other parameters are normal. Question 5-1 Which of the following is the best initial management for this patient's condition? A) Splenectomy. B) Anti-D immune globulin 75 meg/kg x 1 dose. C) Prednisolone 1 to 2 mg/kg/day orally for 2 weeks. D) Anticipatory guidance and observation. E) Intravenous immunoglobulin (IVIG) 0.8 g/kg x 1 dose. Discussion 5-1 The correct answer is "D:' This patient, who has isolated throm­ bocytopenia after an acute viral illness, cutaneous bruising, and no other systemic symptoms, most likely has immune thrombo­ cytopenia (ITP-note that the previous name ofimmune throm­ bocytopenic purpura has been changed to exclude "purpura'' in recent years) . ITP commonly presents in the toddler period, and due to an autoimmune response following acute viral infection, or in up to 10% of ITP cases, following vaccination for measles, mumps, and rubella (MMR) . Note that no other vaccinations have been associated with this condition. ITP is most commonly

CHAPTER 4



BLOOD AND N EOPLASTIC DISORDERS

77

T A B L E 4-1 S U M MARY O F F I RST - L I N E T H E RA P I E S F O R I M M U N E T H ROM BOCYTO P E N I A ( I T

T herapy

Dose

Time to Initial Response

Obse rvation

N/A

Wee ks

Risk of bleed i ng, a lthough severe bleed i n g i s ra re

I ntraveno us i m m u noglobu l i n (IVIG)

0.8 g/kg IV X 1 , may repeat if no response

1 -3 days

I nfu sion reaction i n c l u d i n g fever, vo miti ng, pru ritus, mya l g i as; del ayed headache; ra re hem olytic reactions

Anti-D i m m u n e g l obu l i n

50-75 m eg/kg I V x 1

1 -3 days

O n ly fo r pati e nts with Rh+ bl ood type; often cau ses 1 -2 g/d L d ro p i n hemog l ob in ; black box wa r n i n g req u i res m o n ito r i n g fo r 8 h o u rs fo r development severe hemolys i s (ra re)

Corticosteroid (eg, pred n i so n e)

Pred n i sone 1 -2 mg/kg/d ay PO x 1 -2 wee ks, taper u p to seve ra l weeks more

4- 1 4 days

No d ata su pport a specific dose, d u ration, or taper, a n d practice va ries s i g n ifica ntly; seco ndary a d re n a l i n sufficiency, hyperte n­ sion, hyperg lyce m i a , a n d g rowth concerns l i m it lon g-term use

Adverse Effects and Comments

Data from Neunert C, Lim W, Crowther M, et al; American Society of Hematology. The American Society of Hematology 201 1 evidence-based practice guideline for immune thrombocytopenia. Blood. 2 0 1 1 ; 1 1 7( 1 6):4 1 90-4207.

a self-limited condition, and current evidence-based guidelines indicate that observation alone is adequate management for uncomplicated ITP such as described in the vignette; platelet counts should be expected to correct over the ensuing weeks to months following diagnosis. The most worrisome complication of thrombocytopenia in a toddler is intracranial hemorrhage (ICH), although the incidence of spontaneous ICH associated with ITP is unknown. Treated and untreated patients appear to have similar risk of ICH, up to 0.2%. Parents should be edu­ cated to seek immediate medical attention if the child suffers a head injury, or exhibits gastrointestinal bleeding symptoms, although again these are fairly rare complications. Patients with expanding hematoma or mucosal bleeding symptoms, such as prolonged epistaxis or gingival bleeding, may warrant treatment to hasten the recovery of platelets. The current frontline thera­ pies recommended for such patients are IVIG (option "E"), anti-D immune globulin for Rh-positive individuals (option "B"), and corticosteroids (option "C"); each of these has an approximately 70% likelihood of increasing platelet counts in treated children with ITP. See Table 4- 1 for a summary of these modalities. In refractory or severe cases, splenectomy (choice A) is also a consideration, but this is discouraged as upfront therapy, especially in children younger than 6 years of age because of the increased risk of infection with encapsu­ lated organisms in asplenic individuals. Question 5-2 What is NOT a typical test ordered as part of the initial diag­ nostic workup for suspected ITP? A) Complete blood count. B) Coagulopathy panel. C) Bone marrow aspiration.

D) White cell differential. E) Peripheral blood smear. Discussion 5-2 The correct answer is "C' One cohort of 328 children with typical ITP features underwent bone marrow aspiration for evaluation of leukemia, with 0 of them having malignancy; therefore, chil­ dren with typical ITP are not recommended to undergo bone marrow evaluation to "rule out" leukemia. Atypical features include symptoms such as fevers, weight loss, fatigue, irritabil­ ity, or pain, and objective findings such as lymphadenopathy, organomegaly, or multiple abnormal cell lines on the blood count; these findings raise suspicion for non-ITP hematologic or oncologic diagnoses. Under these circumstances, evaluation by a hematologist is recommended prior to initiation of therapy.

A 16-year-old adolescent girl initially presented to your clinic with cough, generalized lymphadenopathy, and fever. Onset of symptoms was 6 months ago, with increasing pruritus, flushing, and night sweats. She occasionally felt increasing pressure in her chest while lying down, and was having diffi­ culty taking deep breaths. Physical examination was remark­ able for posterior cervical lymphadenopathy, 10 em on the right and 5 em on the left. Coarse crackles were appreciated in the lower lung fields bilaterally. Saturation of peripheral hemoglobin by oxygen (SpO ) was 86% on room air. A CBC revealed hemoglobin of 10.6 g/dL, white blood cell count (WBC) 15 x 1 03/mm3 (differential: neutrophils 15%, lympho­ cytes 73%, monocytes 8%, eosinophils 4%), and platelets

78

MCG RAW-H I LL EDUCATION SPECIALTY BOARD REVI EW: PEDIATRICS is more common in adolescents and young adults. Hodgkin lymphoma is curable with chemotherapy. In the past, patients with splenic involvement have undergone splenectomy as part of their therapy, although this is not commonly recommended for pediatric patients. Asplenic patients are at increased risk of infection with encapsulated organisms; it is recommended that splenectomy be delayed until 23-valent pneumococcal and meningococcal vaccinations can be administered. Prompt attention to fevers in splenectomized patients, with blood cul­ ture and empiric antibiotic therapy during a 48-hour sepsis rule-out, is recommended.

� •

FIGURE 4-3. This fi n d i n g was noted on excisional biopsy o f t h e patient's enlarged lymph node (Case 6). (Reproduced with permission from Kaushansky K, Lichtman MA, KBeutler E, et al: Williams Hematology, Bed. McGraw-Hill Education,

I I

Helpful Tip

A mediastinal mass is never normal and may be caused by leukemia or lymphoma. Orthopnea may be a symptom of a chest mass.

Inc., 201 0. Fig 98-34.)

382 x 1 03/mm3• An excisional lymph node biopsy reveals the finding shown in Figure 4-3. Question 6-1 Of the following, the most likely diagnosis is: A) Acute lymphoblastic leukemia. B) Hodgkin lymphoma. C) Infectious mononucleosis. D) Diffuse large B-cell lymphoma. E) Systemic lupus erythematosus. Discussion 6-1 The correct answer is "B:' Figure 4-3 exhibits a classic binucleate Reed-Sternberg cell, with the typical "owl's eyes" appearance to the nucleoli. It is pathognomonic for Hodgkin lymphoma (HL), although not all HL will present with this finding on biopsy [7] . In a patient with lymphadenopathy that is worrisome for malig­ nancy, such as the patient in the vignette, note that consulta­ tion with a pediatric surgeon is recommended to ensure that excisional biopsy is obtained; the sampling error associated with needle core biopsy increases the chance of false- negative results, need for re-biopsy, and delay in diagnosis and treatment. Apart from this piece of data, this is a teenage patient with several months of lymphadenopathy, fever, and cough, with other chest symptoms suggestive of a chest mass. The quality of symptoms may overlap with those of infectious mononucleosis (option "C"), but the duration is atypical, and the CBC did not report atypical lymphocytes that would be suspicious for Epstein-Barr virus infection. Similarly, the largely normal CBC does not raise suspicion for acute lymphoblastic leukemia (ALL, option "A''); one should expect ALL to exhibit marked cytopenias and pos­ sibly leukocytosis. The biopsy result is inconsistent with lupus (option "E" ) . The age of the patient, with constitutional illness and symptoms consistent with a chest mass, should be clues for HL. Non-Hodgkin lymphomas (NHL, option "D") might also have similar presenting findings, although among pediatric patients, NHL is more common in younger children and HL

A 3-year-old boy presents with 4 months of diarrhea, irri­ tability, anorexia, and progressive fatigue. Exam shows an ill-appearing child. Vital signs include temperature 37.2°C (98.9°F) , blood pressure 130/80 mm Hg, heart rate 140 beats per minute, respiratory rate 30 breaths per minute. On physi­ cal exam, he has periorbital ecchymoses with proptosis, mat­ ted firm nonmobile lymph nodes in his left anterior cervical chain, distended abdomen, firm mass in the abdominal right upper quadrant extending to the pelvis, and bilateral ingui­ nal adenopathy. He is pale and has bruises over his lower extremities. Labs show elevated lactate dehydrogenase of 980 units/L, hemoglobin 7.4 g/dL, platelets 650 x 1 03/mm\ and WBC 4.5 x 1 03/mm3 with a normal differential. His albu­ min is 1 .9 g/dL and creatinine is 0.7 mg/dL. Question 7-1 Which of the following next steps is most likely to yield a diagnosis for this patient? A) Blood culture. B) Coagulopathy panel. C) Manual blood smear evaluation. D) Serum alkaline phosphatase. E) Urine catecholamines. Discussion 7-1 The correct answer is "E:' This patient has findings consistent with neuroblastoma. The hypertension, tachycardia, diffuse lymphadenopathy, and abdominal mass, with low albumin and hemoglobin, are all worrisome for this diagnosis and sug­ gest widespread disease. The nearly normal white blood cell count with normal differential, in the context of a large abdominal mass, reduces suspicion for leukemia (making option "C" less helpful) . This presentation is less likely to be infection than malignancy, given the presence of a primary abdominal tumor (making option "/\' less helpful) . While some

CHAPTER 4

patients with malignancy can present with disseminated intra­ vascular coagulation (DIC), the presence of a coagulopathy would not establish the underlying diagnosis (making option "B" incorrect) . Similarly, while alkaline phosphatase may be elevated in the setting of malignancy, this is often the case with tumors involving bone and is not sufficiently specific to yield this patient's diagnosis (making option "D" incorrect) . The most specific study from the answer choices that is likely to yield a diagnosis is the urine catecholamines, which would be expected to be elevated in approximately 90% of neuro­ blastoma. Staging and risk-stratifying this tumor will require surgical biopsy; imaging with CT chest, abdomen, and pelvis; tumor-specific nuclear medicine imaging; and bone marrow aspiration and biopsy. Neuroblastic tumors can be solitary pri­ mary tumors amenable to surgical resection only, or present with widespread metastatic disease, such as described in the vignette. These patients require aggressive, multimodal ther­ apy with chemotherapy, surgical resection, radiation therapy, autologous stem cell transplantation, and immunotherapy. The patient in this vignette has "raccoon eyes;' suggestive of periorbital involvement. This is a finding often associated with neuroblastoma despite its relative rarity in clinical practice. Another clinical finding associated with metastatic neuroblas­ toma is opsoclonus-myoclonus syndrome, a paraneoplastic process associated with irregular, saccading eye movements and myoclonic movements. Both of these clinical findings are suggestive of advanced stage disease, and therefore poorer prognosis.



Helpful Tip

=t'Jl Neuroblastoma

is a malig nant tumor of the

r1 1 r sympathetic ganglia. Urinary homovanillic acid (HVA)

and vanillylmandelic acid (VMA) (catecholamines) are elevated.

A 1 6-year-old adolescent girl is being seen for a health super­ vision visit. She is a former 27-week premature infant, who suffered severe necrotizing enterocolitis in early infancy, and underwent surgical resection of necrotic terminal ileum as a result. She was followed in the surgical short gut clinic to support her nutritional needs, but you note that she has not been seen there for over 3 years. Over the past year, the patient reports becoming progressively more tired. She has trouble climbing stairways at school, and becomes light­ headed when she stands up. Within the past month, she has noticed numbness in her fingers. Her mother feels that she appears pale and tired, compared with 1 year ago. In private discussion, the patient concedes that she stopped taking her vitamin and mineral treatments, both pills and injections, between 2 and 3 years ago, because it was too difficult to remember to take them, the shots were uncomfortable, and because she has been eating well and does not feel that she



BLOOD AND N EOPLASTIC DISORDERS

79

needs the extra medication anymore. You suspect a nutri­ tional anemia to explain this patient's symptoms. Question 8- 1 Which of the following statements is most accurate regarding the clinical approach to this patient? A) Iron deficiency anemia may be excluded if the patient's red cells are normocytic. B) Folate deficiency can be distinguished from vitamin B 12 defi­ ciency on blood smear by the presence of hypersegmented neutrophils. C) The presence of paresthesia raises suspicion for vitamin B 12 deficiency. D) Folate deficiency can be distinguished from vitamin B 12 deficiency on the CBC by the presence of macrocytosis. E) A therapeutic trial of oral iron, folic acid, and B-complex vitamins will likely correct this patient's symptoms. Discussion 8-1 The correct answer is "C:' This patient, who has short gut syn­ drome and has been lost to follow up for several years, likely has multiple vitamin and mineral deficiencies. She has symptoms suggestive of progressive anemia, including fatigue, activity intolerance, and orthostatic symptoms; three likely nutritional deficiencies that could be occurring are iron, vitamin B 12 , and folate deficiency. The additional symptom of paresthesia pre­ sented in the vignette is not a symptom of anemia; however, vitamin B 12 deficiency, particularly if unaddressed and long­ standing, is associated with neurologic changes that are often permanent (making option "E" incorrect) . Vitamin B 12 is absorbed in the terminal ileum, therefore; surgical removal or inflammation can result in vitamin B 12 deficiency. Diagnosis is best made by checking levels of each of the above-listed nutri­ ents individually and providing therapeutic doses as appropri­ ate. However, initial evidence may be obtained from the CBC and manual smear evaluation. Because this patient likely has multiple anemia-associated deficiencies, it will be difficult to rule in or out a specific deficiency based on the mean corpuscu­ lar volume (MCV) ; this patient may appear normocytic in the face of a microcytosis-associated iron deficiency and concomi­ tant macrocytosis-associated folate or B 12 deficiency, or both (making option ''!\.' incorrect) . Folate and B 12 deficiencies are both associated with macrocytosis, and cannot be distinguished based on blood counts and red cell indices (option "D"). Both conditions are also associated with the presence of hyperseg­ mented neutrophils (six or more lobes per nucleus) , as shown in Figure 4-4 (option "B") . Other, nonnutritional causes of macrocytic anemia in children are fairly rare. Relative macro­ cytosis may be seen in spherocytosis syndromes, hypothyroid­ ism, myelodysplastic diseases, trisomy 2 1 , and in any condition that causes significant reticulocytosis, as reticulocytes have increased MCV with respect to mature erythrocytes. Also note that red cells with increased levels of fetal hemoglobin have an increased MCV; therefore, neonates will exhibit an "elevated"­ but normal for age-MCV at birth (ie, 1 06 ± 10 fL in term neo­ nates), which over 3 to 4 months trends down to the "normal" levels expected of children and adults (ie, 88 ± 8 fL).

80

MCG RAW-H I LL EDUCATION SPECIALTY BOARD REVI EW: PEDIATRICS

FIGURE 4-4. Hyperseg mented

neutro p h i l . This fi n d i n g

acco m p a n ies

megalobl astic a n e m i a secondary to folate o r vita m i n B1 2 deficiency. Macrocytosis of red b l ood cel l s is also noted . (Reproduced with perm ission from La posata M, ed. Laboratory Medicine: The Diagnosis of Disease in the Clinical Laboratory, 2nd ed. New York, NY: McGraw- H i l l Education, I nc; 20 1 4, Figure 1 0-1 3.)

An 1 1 -month-old boy is brought to your clinic because his parents are concerned that he has become pale. He has had regular health supervision, has no medical problems, and takes no medications. His diet is varied and appropriate for age. He is up-to-date on recommended vaccinations. All developmental milestones have been met appropriately to date. Physical exam reveals a pale, tired-appearing boy who is otherwise awake and interactive. Lab work reveals anemia with hemoglobin 7.3 g/dL and replete iron stores as measured by serum iron and ferritin. Question 9- 1 Which of the following additional exam or laboratory findings would make you concerned that this child is at increased risk of malignancy during his first three decades of life? A) Reticulocytopenia. B) Triphalangeal thumbs. C) Tall stature (height > 95th percentile) . D) Neutropenia. E) Intoeing with ambulation.

Discussion 9- 1 The correct answer is "B:' This previously healthy toddler presents with non-iron deficiency anemia, and while the differential diagnosis remains broad, two conditions that can appear similarly are transient erythroblastopenia of child­ hood (TEC) and Diamond-Blackfan anemia (DBA) . Both conditions are forms of pure red cell aplasia found in chil­ dren. While the clinical findings of TEC and DBA substan­ tially overlap, DBA is often diagnosed in infancy, while TEC is typically diagnosed around 2 years of age. The prognosis and approach to management of these conditions are very different. TEC is thought to be, in part, an immune-mediated postinfectious process; some authors have posited a relation­ ship to viral infection, while others have been unable to find an association with specific viruses. The precise mechanism ofTEC is therefore still obscure. TEC is an acquired condition that is self-limited, and the reticulocytopenia/anemia should improve with observation over the ensuing weeks to months . Transfusion is typically only recommended for children with severe, symptomatic anemia (ie, hemoglobin < 5 g/dL) and persistent reticulocytopenia. On the other hand, DBA is a congenital pure red cell aplasia linked to deficient ribosomal protein production, most commonly due to mutations in the RPS 1 9 or RPSA24 genes. Syndromic features are present in nearly 50% of patients with DBA and can include upper extremity physical anomalies involving the thumbs, cardiac and genitourinary anomalies, and facial anomalies. DBA is a cancer predisposition syndrome, and patients with DBA have a nearly 50% chance of developing a hematologic or solid malignancy by age 3 0 . Management of DBA depends on severity; supportive care includes chronic transfusion therapy, and many respond to corticosteroid therapy. Cure is affected by hematopoietic stem cell transplantation, although this may not be recommended in patients who do not have a matched related stem cell donor. Table 4-2 offers a clinical comparison of TEC and DBA.

� QUICKQUIZ What is NOT a cause of pure red cell aplasia in pediatric patients? A) Systemic lupus erythematosus. B) Parvovirus. C) Isoniazid. D) Malnutrition. E) Hypothyroidism. Discussion The correct answer is "E:' In addition to TEC and DBA, other items in the differential for pure red cell aplasia include colla­ gen vascular disease, viral bone marrow suppression ( eg, with parvovirus B 19, usually in the setting of a patient with a chronic hemolytic syndrome such as hereditary spherocytosis), severe renal disease, medications, and nutritional deficiencies.

CHAPTER 4



BLOOD AND N EOPLASTIC DISORDERS

81

TA B L E 4-2 CO M PA R I S O N O F TRA N S I ENT E RYT H R O B LASTO P E N I A O F C H I L D H O O D (TEC)

D I A M O N D - B LACKFAN A N E M I A ( D BA)

Finding

T EC

DBA

Age at diag nosis

Mea n : 26 months

Mean: 1 1 months

Med i a n : 23 months

Med i a n : 3 months

I nfection h i story

Anteced ent v i ra l i l l ness

None

Macrocytosis

Rare at d iag nosis, l i kely d u ri n g recovery

Freq uent

Reticu locyte count

Low at d i a g nosis, e l evated d u ri n g recove ry

Low

Feta l hemog l o b i n

Rarely e l evated

Freq uently e l evated

N o n h ematologic fi n d i n g s

None

Up to 25% of cases: Low bi rthwe i g ht G rowth reta rd ation M i croce p h a ly C left palate Hypoplastic, bifid, or tri p h a l a n g e a l t h u m bs Ca rdiac a b n o r m a l ities Genito u r i n a ry a b n o r m a l ities

I ncreased m a l i g n a ncy r i s k

None

Acute myeloid l e u ke m i a Myelodysplastic syn d rome Osteosa rco ma, other sarcomas Hod g k i n lym phoma He patoce l l u l a r ca rci noma

I n itial ma nagement

Obse rvation

Corticostero i d s

Data from Handin Rl, Lux SE, Stossel TP, eds. Blood: Principles and Practice o f Hematology. Phi ladelphia, PA: Lippincott, Wil l iams and Wil ki ns; 2003.

An 8-year-old girl presents to the emergency department because of lumps on the right side and back of her head. Her parents first noticed them several weeks ago and thought they were "zits" because they were soft and painless. Now, the lumps have grown and are still soft and painless. The patient has previously been healthy, although the parents report that she has been getting up five to eight times each night to void, and is drinking more than usual for the past 2 to 3 weeks. Vital signs are within normal limits. Physi­ cal exam is remarkable for soft, nontender nodules over the right temporal bone and right occipital scalp. You also notice a rash on the right pinna, which is scaly and ery­ thematous, and which the patient reports has been there for months; it is occasionally pruritic. A skeletal X-ray series is performed, and reveals a 1 .5 x 1 .5 em round, lytic lesion in the right temporal bone and another 2.5 x 3 em simi­ lar lesion in the right occipital bone, the latter of which is shown in Figure 4-5.

Question 1 0-1 Which of the following is the most likely diagnosis for this patient? A) Ewing sarcoma. B) Osteosarcoma. C) Non-Hodgkin lymphoma. D) Trauma with contusion of the skull. E) Langerhans cell histiocytosis. Discussion 1 0-1 The correct answer is "E:' Langerhans cell histiocytosis (LCH) is a proliferative disorder of dendritic cells. While it is not a clonal disease, and therefore not considered a malignancy, it can involve multiple organs and cause significant and destructive systemic disease. The average age of presentation is young, approximately 2 . 5 years, but LCH can present at any age. LCH may present with solitary or multiple bony or soft tissue tumors that may or may not be painful; in more ill-appearing children one should suspect additional organ

82

MCG RAW-H I LL EDUCATION SPECIALTY BOARD REVI EW: PEDIATRICS

You are rounding in the newborn nursery, and have been asked to perform circumcision on a 48-hour-old baby boy. He was born by spontaneous vaginal delivery at 39-4/7 weeks' gestation after an uncomplicated pregnancy and has been doing well. His mother has no medical problems and is on no medications. On screening family history, the infant's mother reports that she has never had any bleeding problems. She does note that the infant's maternal grandfa­ ther became significantly disabled due to joint problems in his 40s, and that her brother, the infant's uncle, has "some kind of bleeding problem" and has to take medication sev­ eral times per week to treat his disease. The paternal family history is negative. Question 1 1 - 1 Which o f the following factor deficiencies i s most likely to affect this baby? A) Factor V. B) Factor VII. C) Factor VIII. D) Factor XII. E) von Willebrand factor

FIGURE 4-5. S ku l l fi n d i n g i n La n g e r h a n s ce l l h i stiocytosis ( LC H ) . This p l a i n fi l m exh i b its a l a rg e lytic lesion i n the r i g ht occ i pita l b o n e . There is m i n i m a l sclerosis a n d no peri oste a l react i o n . T h i s a p pe a ra n ce i s consistent with sku l l lesions co m m o n i n LC H . (Used with permission fro m Ad a m D. Wolfe, M D, P h D.)

involvement. (See Table 4-3 for a summary of organs that may be involved.) Diagnosis is suspected based on clinical imag­ ing findings, and may be confirmed by biopsy. LCH classi­ cally will stain immunohistochemically positive for CD 1 a and S 1 00 . Involvement of specific bones of the skull (ie, mastoid, orbital, sphenoid, and temporal bones) carries additional risk of intracranial progression. This may involve the neurohy­ pophysis (posterior pituitary gland) and cause central diabetes insipidus, as was seen in the vignette, and merits MRI of the pituitary for evaluation, as well as endocrinology consultation. Patients who have LCH involvement of the liver, spleen, and bone marrow are at highest risk of treatment failure. Frontline therapy for LCH includes chemotherapeutic agents and cor­ ticosteroids, typically given over 1 year. Cure rates for even high-risk patients are greater than 80%, although significant rates of recurrence (25-40%) dictate retreatment with salvage chemotherapy for many patients before the disease resolves. None of the other option choices given would be expected to cause a lytic lesion of the skull that lacks surrounding sclerotic changes or periosteal reaction.

Discussion 1 1 - 1 The correct answer i s "C:' The family history described b y the mother is most suggestive of a bleeding disorder that follows an X-linked recessive inheritance pattern, of which factor VIII deficiency-hemophilia A-is the most common. Factor VIII deficiency affects an estimated 1 in 5000 male births, and approximately two thirds of cases are inherited; the remain­ der are due to spontaneous mutation. Factor IX deficiency­ hemophilia B, also X-linked-affects approximately 1 in 30,000 male births. Severity of hemophilias is based on the level of factor activity: severe hemophilia results from less than 1 % factor activity, moderate hemophilia occurs with 1 % to 5 % activity, and mild hemophilia i s defined as greater than 5 % factor activity. Bleeding associated with hemophilias includes j oint and muscle bleeding, unusual bruising and hematomas, mucosal bleeding, retroperitoneal bleeding, and postoperative bleeding. Patients with severe and often moderate hemophilia require prophylactic dosing of recombinant factor replace­ ment products throughout their lives to prevent j oint bleed­ ing and the development of disabling chronic hemophilic arthropathy. Patients with mild and some moderate hemo­ philias may not require scheduled prophylactic dosing, and may be treated episodically in the setting of injury, surgery, or dental procedures. In the neonatal period, the greatest bleed­ ing risks in hemophilia are associated with circumcision, heel sticks, and intracranial hemorrhage. Postvaccination intra­ muscular bleeding is also likely during the infant period. Von Willebrand disease (vWD, option "E") is among the most common bleeding disorders, and vWD and the hemo­ philias together account for at least 90% of bleeding disorder

CHAPTER 4



BLOOD AND N EOPLASTIC DISORDERS

83

TA B L E 4-3 COM M O N ORGAN I NVOLV E M E NT I N LA N G E R H A N S C E L L H I STIOCYTO S I S

Symptoms and Findings

Evaluation

Complications and Outcomes

Bone

S ku l l is most freq uent, then fe m u r, ri bs, h u m erus, ve rte brae; may cause pa i n with associ ated adjacent soft tissue extension; vertebra l lesions may cause spi­ n a l com pression

P l a i n fi l m, com puted to mogra phy sca n

Pathologic fractu re, calva r i a l wea kness

G i n g iva/ m a n d i ble

"Fl oati ng" teeth

P l a i n fi l m, com puted to mogra phy sca n

Ca n lead to tooth loss

Skin

Scaly, erythe mato u s p l a q u e; associated pruritus

None or bio psy

May resolve s pontaneously; n ew ras h may present as s i g n o f rela pse

Lym phatic tissues

Lym phadenopathy

Co m puted to mogra phy sca n, bio psy

Lung

Often asym pto matic, may have co u g h or dyspnea

Co m puted to mogra phy sca n

Previously co nsidered a "ri sk organ;' but no specific associa­ tion with treatment fa i l u re

Pitu ita ry

Polyd i psia, polyu ria; rarely other pitu ita ry hormone deficiencies

Mag netic reso na nce i m a g i n g bra i n + pitu ita ry; s e r u m e l ectro­ lytes, u ri n e I plasma osmolal ity, u r i n a lysis, wate r de privation test

L i kely permanent d i a betes i n s i p i d us, i ncreases risk for n e u rodegenerative syn d rome i n later l ife

Spleen

Abdo m i n a l d i ste nsion, easy bru i s­ i n g or bleed i ng, fati gue

Co m puted to mogra phy sca n

" Risk organ" = i n c reased cha nce of recu rrence

Bone marrow

Easy bru i s i n g or bleed i n g ; fatig ue; fevers a n d i nfections

Co m pl ete blood cou nt, bone ma rrow aspi rate a n d bio psy

"Risk organ" = i n c reased cha nce of recu rrence

Liver

Abdo m i n a l d i ste nsion, j a u n d ice, icterus, pru ritus

Co m puted to mogra phy sca n

"Risk organ" = i n c reased cha nce of recu rren ce; risk of scleros i n g c h o l a n g itis

Organ system

Data from Allen CE, Kelly KM, Bollard CM. Pediatric lymphomas and histiocytic disorders of childhood. Pediatr Clin North Am. and Demellawy DE, Young JL, Nanassy J, et al. Langerhans cell histiocytosis: A comprehensive review. Pathology.

201 5;62( 1 ) : 1 39- 1 65; 201 5;47(4):294-30 1 .

diagnoses. Most vWD is inherited in an autosomal dominant fashion. Von Willebrand factor functions as a carrier for fac­ tor VIII and is crucial for recruiting platelets to sites of injury and collagen exposure; therefore, bleeding history in patients with vWD often resembles platelet-type bleeding. Symptoms may include mucocutaneous symptoms such as easy bruis­ ing, epistaxis, gingival bleeding, and menorrhagia. It is com­ mon to encounter young women with mild vWD, who are not diagnosed until after menarche. Factor V and VII deficiencies (options "J( and "B") are inherited in an autosomal recessive pattern and are considerably rarer than the hemophilias and vWD. Factor XII deficiency (option "D") is also inherited in an autosomal recessive pattern, but this condition is not associ­ ated with clinical bleeding symptoms; it causes prolongation of the activated partial thromboplastin time but, because of its lack of clinical symptomatology, does not require any therapy.



Helpful Tip

� In hemophilia, the activated partial thromboplastin 1 1 1r time (a PTI) may be prolonged but the prothrombin

time (PT) is normal. If the a PTI is prolonged, a mixing study should be performed. Correction of the aPTI indicates a factor deficiency. Failure to correct indicates the presence of an inhibitor.

A 3-year-old girl with hemoglobin SS (ie, sickle cell anemia) presents to the emergency department with fever, wheezing, and chest pain. She has a history of mild asthma, controlled

84

MCG RAW-H I LL EDUCATION SPECIALTY BOARD REVI EW: PEDIATRICS

with as-needed use of albuterol metered dose inhaler, but the medication did not correct her symptoms today. Vital signs include temperature 39°C { 1 02.2°F}, heart rate 1 36 beats per minute, respiratory rate 38 breaths per minute. Sp0 on 2 room air is 92%. On physical exam, the patient is awake and alert, but appears anxious and distressed. She coughs fre­ quently. Chest auscultation reveals fine crackles on the right and scattered expiratory wheeze. Cardiac exam reveals a grade II/VI systolic ejection murmur. Chest X-ray identifies a right middle lobe infiltrate. CBC reveals white blood cells 19.3 x 1 03/mm3 with 80% neutrophils, hemoglobin 6.8 g/dL, and platelets 602 x 1 03/mm3• You diagnose the patient with acute chest syndrome (ACS). Question 1 2- 1 I n addition t o sending a blood culture and type and screen to the lab, which of the following correctly describes your next steps in caring for this patient? A) Administer supplemental oxygen only if Sp0 2 drops below 88%. B) Initiate total exchange transfusion with appropriately cross­ matched packed red blood cells (pRBCs). C) Avoid opioid pain medications to prevent respiratory depression. D) Begin therapy with a parenteral third-generation cephalo­ sporin and oral macrolide. E) Keep total IV fluid rate to 0.75 times maintenance to avoid circulatory overload. Discussion 1 2- 1 Th e correct answer i s "D:' ACS i s a complication o f sickle cell disease that is thought to occur in nearly 1 3 % of patients with hemoglobin SS disease, and in fewer than 1 0% of patients with other sickle cell syndromes. The syndrome is characterized by acute onset of fever, cough, and chest pain, and has clinical findings of hypoxemia and new infiltrate on chest X-ray. This syndrome's findings overlap completely with acute bacterial and viral pneumonia, and in fact this is likely one of the underly­ ing etiologies of ACS in children with sickle cell disease. Other contributing etiologies are microvascular occlusion by sickled cells in the pulmonary vascular bed, asthma exacerbation, and atelectasis. As the presence of inflammation and airway nar­ rowing can cause local vasoconstriction in pulmonary capillar­ ies, infection and asthma exacerbation may quickly cause local sickling due to relative tissue hypoxia and narrowed vascula­ ture. Therefore, ACS is thought to be multifactorial in most pre­ sentations. Untreated ACS may rapidly progress to respiratory failure and death, and for this reason patients diagnosed with ACS merit inpatient admission and management. The manage­ ment of ACS is aimed at addressing all of the above etiologies simultaneously. As with any patient who has sickle cell disease and fever, blood culture and parenteral antibiotic therapy are warranted. Treatment in children is directed toward pathogens associated with both typical and atypical pneumonias, and therefore ceftriaxone and azithromycin or comparable regimen is recommended. Patients whose hemoglobin is low (institution dependent, but typically < 8 g/dL) benefit from simple blood

transfusion to reduce sickled cells and increase oxygen-carrying capacity; most exchange transfusion programs will not perform exchange on patients with low hemoglobin (making option "B" incorrect) . Because patients with ACS-associated chest pain may exhibit tachypnea with shallow breathing, supplemen­ tal oxygen is often initiated, regardless of oximetry readings (option "A''), and adequate pain management is crucial to allow comfortable, deeper breathing (option "C") . Pain management with sickle cell disease is best accomplished with a combination nonsteroidal anti-inflammatory drug (NSAID) , such as ibupro­ fen, naproxen, or ketorolac, and an opioid. Opioid doses should be titrated for effect, and with close monitoring to avoid averse­ dation. Initial hydration to improve flow through occluded vasculature is recommended at 1 .0 to 1 . 5 times maintenance (making option "E" incorrect) . Other aspects of management of ACS include aggressive therapy for asthma in patients with this diagnosis, and incentive spirometry and early ambulation as tolerated for atelectasis.

� QUICKQUIZ Which is a complication of sickle cell disease? A) Acute vasoocclusive crisis. B) Sepsis. C) Stroke. D) Priapism. E) All of the above. Discussion The correct answer is "E:' •

Helpful Tip

=-� Fever

in a patient with sickle cell disease is a emergency. Sickle cel l disease causes functional asplenia with increased risk for overwhelming bacteria l infections, including sepsis.

f1 1 r life-threatening

A 12-month-old baby girl comes to your clinic for a health supervision visit. She has been healthy, meeting developmen­ tal milestones appropriately, and parents have no concerns today. You note on history that the baby suffered a viral­ sounding upper respiratory infection with fever 2 to 3 weeks ago, with symptoms lasting 4 days that have now resolved. The patient has continued to track at the 47th percentile for weight and 6 1 st percentile for height. Based on current guide­ lines for anemia screening, you order a CBC and discover the following: white blood cells 4.3 x 1 03/mmJ, hemoglobin 12.2 g/dL, platelets 244 x 1 03/mm3• The WBC differential, which you did not order but was nevertheless performed, reveals an absolute neutrophil count of 130/mm3 and abso­ lute lymphocyte count of 3850/mm3•

CHAPTER 4

Question 1 3- 1 Which o f the following i s the next best step i n management of this patient's neutropenia? A) Provide anticipatory guidance regarding the typically benign nature of the condition; observe with close follow up. B) Admit the patient to receive intravenous antimicrobial therapy. C) Request bone marrow aspiration and biopsy. D) Administer a course of granulocyte colony-stimulating factor (G-CSF). E) Prescribe a course of oral cephalosporin. Discussion 1 3- 1 Th e correct answer i s "A:' This patient's age, well appear­ ance, and isolated neutropenia following a viral illness are most consistent with benign neutropenia of childhood. This condition is fairly commonly discovered incidentally when a blood count is ordered for a different reason. Benign neutro­ penia of childhood is a transient condition, thought to be due to an autoimmune reaction following the inciting infection. In some patients, antineutrophil antibody testing will reveal the causative antibody; this test is not offered in many centers and lacks sensitivity, and therefore is not routinely utilized in diagnosis. Over several weeks to months following diagnosis, patients are expected to return to normal neutrophil counts. In a well-appearing child with this diagnosis, observation alone is an appropriate approach. In general, benign neutro­ penia of childhood, regardless of neutrophil count, is not asso­ ciated with severe infections. Nevertheless, prompt evaluation of fever is recommended in children with an absolute neutro­ phil count (ANC) of less than 500/mm3• Normal ANC in term newborns in their first week of life is greater than 3000/mm3, and then decreases to greater than 1 1 00/mm3 during the next 2 years. After the age of 2, normal ANC in children and ado­ lescents is greater than 1 500/mm3. Table 4-4 defines different categories of neutropenia and their associated infectious risks. In addition to benign neutropenia of childhood, other pos­ sible acquired causes of neutropenia include viral suppression (eg, with Epstein-Barr virus infection), medication induced ( eg, antiepileptics, beta -lactam antibiotics, chemotherapeu­ tic agents), and other autoimmune diseases (eg, Crohn dis­ ease, systemic lupus erythematosus). Options "B" through "E" represent approaches that might be appropriate in some children with neutropenia but would not be required as a frontline approach in a well-appearing child with incidentally diagnosed neutropenia. If a patient with benign neutropenia should become severely ill, administration of G-CSF (option "D") to hasten neutrophil recovery is appropriate.

A 17-month-old boy presents to your clinic because his par­ ents are concerned about his mobility. He is a previously healthy baby, who met all developmental milestones on



BLOOD AND N EOPLASTIC DISORDERS

85

TA B L E 4-4 D E F I N ITI O N S O F N E UTRO P E N I A A N D CO M P L I CAT I O N S

Severity

Absolute Neutrophil Count (per mm3 )

Risks

Normal

> 1 500 (> 1 1 00 if yo u n g e r than age 2 years)

None

Mild

1 000- 1 500

None; merits eva l uation fo r cause of a bnormal count ( h i story often sufficient)

Moderate

500- 1 000

M i n i ma l , risk if other i m m u n e fu ncti o n s a re i m pa i red

Seve re

200-500

I ncreased risk of i n fection fro m co m mensal orga n i s m s (eg, g ram-positive ski n fl ora, g ra m ­ negative enterics)

Profo u nd (ie, a g ra n u l ocytosis)

< 200

S i g n ificant risk of i nfection by co m me n sa l s, opport u n i stic path og ens such as Pneumocystis jiroveci

Data from Newburger PE, Dale DC. Eval uation and manage­ ment of patients with isolated neutropenia. Semin Hematol. 2 0 1 3;50(3 ) : 1 98-206.

time as of his last visit with you at 12 months of age. Mother reports today that the patient has seemed clumsier over the past 1 to 2 months and is not walking as much as he used to. She is unsure of whether he is in pain. He has vomited 4 to 5 mornings this week. Vital signs are within normal limits, and the patient is continuing to track on his previous percen­ tiles for height and weight. Physical exam reveals a boy who is awake, alert, and interactive. He has horizontal nystagmus on neurologic exam. While sitting upright on the exam table, he leans over to the right and nearly falls off the table before you catch and right his posture. He is unwilling to bear weight on his legs for gait testing today. He has no rash, bruising, pete­ chiae, lymphadenopathy, or organomegaly, and the remain­ der of your exam is unremarkable.

86

MCG RAW-H I LL EDUCATION SPECIALTY BOARD REVI EW: PEDIATRICS

Question 1 4- 1 Which o f the following diagnoses i s the most likely, given the historical and exam findings? A) Craniopharyngioma. B) Acute lymphoblastic leukemia. C) Neuroblastoma. D) Germ cell tumor. E) Medulloblastoma. Discussion 1 4- 1 Th e correct answer i s "E:' This patient presents with signs and symptoms compatible with increased intracranial pres­ sure (ICP), including ataxia, nystagmus, and vomiting, often observed in patients with a tumor that obstructs cerebrospinal fluid outflow from the third or fourth ventricle. Midline poste­ rior fossa tumors, such as medulloblastoma, can present with these findings. Medulloblastoma, a primitive neuroectodermal tumor of the posterior fossa, is the most common malignant brain tumor of childhood, often diagnosed due to symptoms of increased ICP. The next step in evaluation and management when increased ICP is suspected is often urgent CT scan of the head. If a mass is noted, (MR) imaging is recommended to fur­ ther delineate the mass. If increased ICP is identified, patients will often undergo placement of an externalized ventricular drain or a ventricular shunt to relieve the pressure. Because medulloblastoma may be metastatic to the spine at diagnosis, MR spine and diagnostic lumbar puncture are required to estab­ lish staging. A key element of prognosis for medulloblastoma­ and most brain tumors in children-is the extent of primary surgical resection. Gross total resection is associated with better outcomes than partial resection, although the extent of resec­ tion may be limited by involvement of the brainstem. Following resection, patients with medulloblastoma are treated with mul­ timodal chemotherapy and radiation therapy (XRT) to involved sites. However, in a patient as young as in the vignette, efforts are made to delay or eliminate the use of XRT due to the severe central nervous system toxicity associated with it. Alternatives to XRT in young children include intensification of chemother­ apy, and high -dose chemotherapy with autologous stem cell res­ cue. Craniopharyngioma (option "/\_') is a tumor of pharyngeal cell rests (remnants of Rathke pouch) in a primarily suprasellar location near the pituitary gland, which most commonly occurs in immediately prepubertal children and is rare in children as young as the patient in the vignette. It may cause cerebral spinal fluid ( CSF) obstruction and increased ICP, and is also associated with vision changes, behavioral abnormalities, and pubertal delay. Treatment is surgical, with or without XRT, and lifelong endocrine deficiencies (e.g. panhypopituitarism) occur subse­ quent to removal or damage of pituitary tissue. Primary germ cell tumors (option "D") may arise intracranially, most often in the pineal gland or suprasellar region. Symptoms depend on location of the tumor, but primarily include deficiencies of anterior pituitary hormones, diabetes insipidus, and visual dis­ turbances. Acute lymphoblastic leukemia (option "B"), which may involve the central nervous system, does not readily form solid tumors and is unlikely to cause the posterior fossa symp­ toms described for the patient in the vignette. Neuroblastoma

(option "C") is a small round blue cell tumor derived from neu­ roepithelial cells of the neural crests during embryonic devel­ opment. Neuroblastoma rarely penetrates the dura, making it unlikely to have caused the patient's symptoms.

� QUICKQUIZ Which of the following does not have a small round blue cell appearance by histology? A) Neuroblastoma. B) Medulloblastoma. C) Ewing sarcoma. D) Osteosarcoma. E) Neuroendocrine tumors. Discussion The correct answer is "D:' Neuroblastoma, medulloblastoma, other primitive neuroendocrine tumors, and Ewing sarcoma are so-called small round blue cell tumors because of histologic appearance. These tumors are also known as embryonal tumors, given their origin in early ontogeny; they have substantial migratory and growth potential, making them typically clini­ cally aggressive.

A previously healthy 4-year-old boy presents to your acute care clinic with bilateral leg weakness. He had episodes over the past 3 weeks consisting of pain, weakness, and numbness in his thighs, legs, and feet, which resolved after 1 to 2 hours, and therefore was not brought to medical attention. There is no history of trauma. Today, he has been experiencing another, more severe, episode over the past 4 hours: he woke up unable to stand or walk. He has not voided today. The patient is sitting in a wheelchair, in no apparent pain, alert and answering ques­ tions appropriately. Vital signs are within normal limits. On exam, the patient has normal muscle tone, bulk, and strength in all upper extremity muscle groups; the lower extremities exhibit increased tone and 2/5 strength in all muscle groups. Deep tendon reflexes are 2+ in the biceps and 4+ in the patel­ lae. Sensation is diminished in the lower extremities bilater­ ally. The patient is unwilling to attempt gait testing. Capillary refill in fingers and toes is 2 to 3 seconds, and radial and dor­ salis pedis pulses are 2+ bilaterally. You send the patient by ambulance to the emergency department. Question 1 5- 1 Which o f the following are most likely t o be part o f the imme­ diate management of this patient? A) Magnetic resonance imaging (MRI) of the spine, consults to neurosurgery and radiation oncology. B) MRI of the spine, consults to oncology and radiation oncology.

CHAPTER 4

C) MRI of the spine, consults to oncology and neurosurgery, initiation of corticosteroids. D) Computed tomography (CT) of the spine, consults to oncology and neurosurgery. E) CT of the spine, consults to oncology and radiation oncology, initiation of corticosteroids. Discussion 1 5- 1 Th e correct answer i s "C:' Spinal cord compression i s a n onco­ logic emergency, and can be seen with solid malignancies such as neuroblastic tumors and soft tissue sarcomas, hematologic malignancies such as extramedullary acute myeloid or lympho­ blastic leukemias and lymphomas, and with vascular tumors such as vertebral hemangiomas. Prompt diagnosis and therapy are required to preserve neurologic function. Based on the type of tumor, the approach to initial therapy may vary. One tumor that is known to present with cord compression, neu­ roblastoma, is amenable to chemotherapeutic, neurosurgical, and radiotherapeutic interventions, as are many spinal tumors. However, while radiation therapy may successfully treat the tumor, it is often not the initial treatment of choice in pediat­ ric patients with tumors in the spine due to long-term adverse effects on growth and function. Emergent imaging with MRI, which can illustrate the source of the tumor as well as extent of epidural and soft tissue disease, is preferred, and will assist in the preliminary differential diagnosis and in establishing whether neurosurgical or chemotherapeutic intervention will be favored. For tumors that are chemotherapy-responsive, such as neuroblastoma, chemotherapy is often preferred over lami­ nectomy and resection, again to minimize long-term effects on spinal growth. Corticosteroids are often initiated prior to che­ motherapy to reduce peritumoral edema. Neurosurgical inter­ vention may be reserved for cases of lack of response to medical therapy within the first 1 to 2 days.

You are seeing an 8-year-old boy in your clinic to estab­ lish care following a recent hospital admission. His family recently emigrated from Saudi Arabia, and you are con­ ducting today's visit through an Arabic language phone interpreter. The patient was admitted for 2 days last week during a febrile influenza-like illness, because of extreme fatigue and because his eyes became "more yellow than usual:' The parents report that the patient has otherwise been healthy. You review the hospital records, which show that the patient was admitted with fever, fatigue, dys­ pnea, icterus, jaundice, and anemia with a hemoglobin of 6 g/ dL and red blood cells with dark purple inclusions with supravital staining on blood smear. He received 1 unit of ABO-matched, packed red blood cells, tolerated this well, and his symptoms resolved within 48 hours. This had never happened to the patient before, although the par­ ents inform you that the patient's 1 0-year-old brother and his maternal uncle have had jaundice and occasional blood



BLOOD AND N EOPLASTIC DISORDERS

87

transfusions in the past, and that the patient's mother, father, and 5-year-old sister do not have these problems. On today's exam, the patient appears normal apart from subtle icterus. Question 1 6- 1 Which o f the following tests i s most likely t o reveal this patient's diagnosis? A) Bone marrow aspirate and biopsy. B) Direct antiglobulin (Coombs) test. C) Erythrocyte osmotic fragility test. D) Glucose-6-phosphate dehydrogenase (G6PD) enzyme activity. E) Hemoglobin electrophoresis. Discussion 1 6- 1 Th e correct answer i s "D:' This patient exhibits evidence of baseline hemolysis, has suffered symptomatic hemolytic ane­ mia in the setting of an acute febrile illness, and has a fam­ ily history suggestive of an X-linked disorder. Of the choices, these findings best fit with G6PD deficiency. G6PD deficiency is the most common red cell enzymopathy, and carrier sta­ tus is most prevalent in families from malaria-endemic areas of the world, including Africa, the Middle East, southern and southeastern Asia, and South America. G6PD is an enzyme critical for function of the pentose phosphate pathway, which restores important reducing substances such as NADPH and glutathione, in red blood cells. Because G6PD-deficient indi­ viduals cannot effectively utilize this pathway, their red cells are at risk of oxidant-induced damage and lysis at times of oxidative stress. Oxidative damage can cause precipitation of denatured hemoglobin, seen as Heinz bodies on staining with methyl violet, as with the patient in the vignette. Stress may come from acute infectious or inflammatory insults, such as occurred in the vignette, or due to medications and dietary causes. The most commonly discussed dietary cause is from fava beans, which are often used in Mediterranean and Middle Eastern cooking. Medications to avoid include sulfonamide antibiotics, dapsone, nitrofurantoin, and quinine-containing antimalarials, as well as naphthalene (mothballs) . As an aside, for the family in this vignette, providing information on this disease and a list of medications and foods to avoid in their native language may be challenging, but will be crucial to helping them avoid complications for the affected boys. Some individuals with G6PD deficiency will have more severe phe­ notypes than others, with some children experiencing hemo­ lytic episodes in the newborn or infant periods. Severity is suggested by G6PD enzyme activity assay: patients with less than 1 0 % activity have more severe disease, while those with 1 0% to 60% activity have more moderate disease. In general, families of east Asian and Mediterranean descent appear to exhibit more severe phenotypes. Regarding the G6PD activ­ ity assay, it is recommended that the test be performed after a patient's hemolytic episode has resolved. In most patients, maximal residual enzyme activity is retained in younger red cells (ie, reticulocytes) which comprise the bulk of available cells at the time of a hemolytic event. Therefore, testing during

88

MCG RAW-H I LL EDUCATION SPECIALTY BOARD REVI EW: PEDIATRICS

an event increases the chance of a falsely elevated enzyme activity result. Of the other answer choices, hereditary sphe­ rocytosis (assayed in option "C") could have a similar clini­ cal presentation, but this condition is usually inherited in an autosomal dominant fashion. One would expect a patient with acute leukemia or bone marrow failure (assayed in option ''A'') to have additional blood count abnormalities, and a less acute symptomatology to indicate malignancy. Autoimmune hemo­ lytic anemia (assayed in option "B") would not be expected to exhibit an X-linked inheritance pattern, and is not associated with Heinz bodies. Hemoglobinopathies such as sickle cell dis­ ease or thalassemia maj or (assayed in option "E") could cause anemia with hemolysis and j aundice in an episodic fashion, but would be expected to have additional chronic symptoms and would not be expected to exhibit X -linked inheritance.

A 5-day-old baby boy is scheduled for an establishing well­ newborn visit in your clinic. All you know about the baby is that he was born at home and has not yet seen a physician or medical provider. The mother called on day of life 2 to make the patient's appointment with you and reported that he was born without complications at 38-6/7 weeks' gestation and was doing well. The clinic receives a call from the baby's mother on the day of the appointment, in which she tells you that she is concerned that the baby is not waking from his nap this morning, is breathing shallowly, and over the past 10 minutes began shaking his right arm and leg. You advise the mother to call 9- 1 - 1 . Question 17-1 When you call ahead to the emergency department to inform them of this patient, which of the following will you recom­ mend be performed first? A) Magnetic resonance imaging (MRI) of the head with IV contrast. B) Computed tomography (CT) scan of the brain without IV contrast. C) Coagulopathy screening with platelet count, prothrombin time (PT), and activated partial thromboplastin time (aPTT). D) Intravenous administration of vitamin K. E) Transfusion with fresh frozen plasma. Discussion 17-1 The correct answer is "E:' This term neonate presents with symp­ toms suggestive of intracranial hemorrhage (ICH), and since he presumably did not receive prophylactic vitamin K at birth and appeared well initially, is likely experiencing the classic form of vitamin K deficiency bleeding (VKDB) . Very likely, all of the actions represented in the answer choices above will be taken, as imaging (options ''A'' and "B") and lab work (option "C") will be used in planning further management, but the most immedi­ ate task is to administer a therapy most likely to ameliorate the bleeding. This will most rapidly be accomplished by replenishing

the vitamin K-dependent clotting factors, which are present in physiologic quantities in fresh frozen plasma (FFP). Administer­ ing vitamin K is also recommended (option "D"), but will not yield an immediate change in the patient's bleeding diathesis. The symptoms and signs of ICH vary depending on the location of bleeding and the age of the patient. Presenting symptoms of iCH in neonates include seizures, focal weakness, feeding problems, and apneas or other respiratory events. These symptoms are simi­ lar to those of neonates who experience ischemic stroke, although arterial ischemic stroke and cerebrospinal venous thrombo­ sis tend to present in neonates less than 72 to 96 hours of age. VKDB risk can be classified into three timeframes. Very early VKDB occurs within the first day after birth, and is associated with maternal medication use that interferes with vitamin K­ dependent factor production (eg, warfarin) . Classic VKDB pres­ ents within the first week oflife, as with the patient in the vignette, and is often associated with mucocutaneous bleeding symptoms. Late VKDB can occur after 1 week oflife, up to several weeks after birth. The latter two categories ofVKDB are often associated with breastfed infants, due to the paucity of vitamin K in breast milk. Prophylactic administration of intramuscular vitamin K, which is recommended for all newborns, has been shown to significantly decrease the incidence of classic and late VKDB. Oral vitamin K is also available, although a single neonatal dose does not appear to reduce the risk of late VKDB. Depending on the normative ranges used in the laboratory, be aware that neonates, and par­ ticularly preterm neonates, will have coagulation screening tests that fall out of the "normal" ranges. This is largely due to relatively poor synthetic liver function seen in newborns. Both PT and aPTT prolongation are common in the first few days of life, due to low vitamin K-dependent factors in the former test and due to low propagation (ie, intrinsic) pathway factor levels in the latter case. Exceptions to this include factor VIII and von Willebrand factor levels, as these factors are not liver dependent and may, in fact, be elevated at birth.



Helpful Tip

� Factor VI I I and von Wi llebrand factor are synthesized in

1 1 1 r the vascular endothelium rather than the liver like all other coagulation factors.

You are following a previously healthy 1 7-year-old adoles­ cent girl for acute hepatitis. She was admitted to your service with suspected viral hepatitis last week, in the setting of acute diarrhea, nausea, pruritus, jaundice, hepatomegaly, and transaminitis. Since diagnosis, her transaminases, serum bil­ irubin, and serum chemistries have been checked daily and are stable. This morning, she is reporting increasing fatigue and shortness of breath. Apart from the jaundice and hepa­ tomegaly, you find no other abnormalities on today's exam. You obtain a CBC, which reveals white blood cell count of 2.4 x 1 03/mm3 with 1 8% neutrophils and 78% lymphocytes,

CHAPTER 4

hemoglobin 7.2 g/dL, and platelets of 64 x 1 03/mm3• Erythro­ cyte sedimentation rate is 1 00 mm/h (reference range 0-20) . Results of hepatitis A, B, and C serologies are negative. Question 1 8- 1 Which o f the following diagnoses best explains this patient's blood count findings today? A) Acquired aplastic anemia. B) Acute lymphoblastic leukemia. C) Chronic myelogenous leukemia. D) Hepatocellular carcinoma. E) Epstein-Barr virus infection. Discussion 1 8- 1 Th e correct answer i s "A:' This patient presents with acute seronegative hepatitis with subsequent pancytopenia, consis­ tent with a working diagnosis of autoimmune aplastic anemia. Antecedent autoimmune hepatitis is a frequently encountered historical finding in patients with acquired aplastic anemia. Confirmatory testing with bone marrow aspirate and biopsy is required. Acute lymphoblastic leukemia (option "B") could present with these blood count findings and is certainly in the differential diagnosis, but it would be unexpected to see this acute drop in blood counts shortly after onset of hepatitis with



BLOOD AND N EOPLASTIC DISORDERS

89

this diagnosis. The bone marrow assessment in this patient will also be used to rule out leukemia prior to therapy. Chronic myelogenous leukemia (option "C") can exhibit organomegaly, due to widespread extramedullary hematopoiesis, but is associ­ ated with marked splenomegaly, a significantly elevated white blood cell count, and many differently maturing cell forms seen on the white cell differential. Hepatocellular carcinoma (option "D") is a possible consequence of chronic hepatitis B or C infection, but should not be associated directly with acute pancytopenia. Epstein-Barr virus (EBV; option "E") can cause viral hepatitis and can be associated with bone marrow sup­ pression, and EBV serologies probably would be tested in this patient; however, the absence of lymphadenopathy and rela­ tively acute onset of pancytopenia should reduce suspicion for this diagnosis. Severe aplastic anemia (SAA) is diagnosed by hypocellularity ( < 25%) on bone marrow aspirate. Manage­ ment of SAA depends on the etiology. Acquired SAA may be caused by medications, such as certain antiepileptic drugs; in these cases discontinuation of the causative medication should be therapeutic. In a patient found to have SAA without a clear etiology, bone marrow failure syndromes are typically evaluated before an autoimmune cause is assumed. Selected bone mar­ row failure syndromes that are evaluated during a workup of newly diagnosed SAA are reviewed in Table 4-5. Management

TABLE 4-5 S E LECTED BON E MARROW FA I LU R E SYN D R O M E S CO N S I D E R E D I N T H E D I F F E

T I A L D I AG N O S I S

O F S EV E R E A P LASTIC A N E M IA

Common Clinical Findings

Confirmatory Testing

Condition

Etiology

Inheritance

Fa nconi a n e m i a

M utatio n s i n m u ltiple FA core co m p lex g e n e s

Va riable: AR, XLR (FANCB m utation)

Short statu re, rad i a l a n d t h u m b a b n o r m a l ities

C h romoso m a l brea kag e with d i e poxybuta ne o r m itomyc i n C

Dyskeratosis co ngen ita

M utatio n s i n m u ltiple te lomere m a i nte n a n ce enzyme co m pl ex genes

Va riable: AR, AD, XLR

Dystro p h i c n a i l s, ski n t h i cke n i ng, l e u ko p l a ki a

Te lomere length a n a lys i s

S hwach manDiamond syn d ro m e

M utation in S hwach m an- Bod i a n Diamond gene

AR

Pa ncreatic i n s ufficiency, skeleta l a bnormal iti es, ne utrope n i a

Pa ncreatic e nzymes, feca l elastase, SBDS gene test i n g

DiamondBlackfa n a n e m i a

Ri boso me prote i n (RPS) genes

AD

Short statu re, m i croce pha ly, thumb abnormalities, cardiac a b n o rm a l iti es, u rogenita l a b n o r m a l ities

Eryth rocyte adenos i n e dea m i nase eva l uation

Pa roxys m a l noct u r n a l hemog l o b i n u ria

M utation i n PIGA gene, codes fo r a G P I I i n kage prote i n

Acq u i red

Hemog l o b i n u ria, t h ro m bosis, hemolysis, new pancyto pe n i a

Flow cyto metry fo r C D S S a n d CD59, w h i c h a re G P I-dependent co m p l e m e n t i n h i bitors

Abbreviations: AD, autosomal domina nt; AR, autosomal recessive; GPI, glycophosphatidylinositol; XLR, X- I inked recessive. Reproduced with permission from Chirnomas SD, Kupfer GM: The inherited bone marrow failure syndromes, Pediatr Clin North Am 201 3 Dec;60(6): 1 291 - 1 3 1 0.

90

MCG RAW-H I LL EDUCATION SPECIALTY BOARD REVI EW: PEDIATRICS

of most bone marrow failure syndromes involves hematopoietic stem cell transplant. For the patient in the vignette, if she is con­ firmed to have acquired SAA, supportive care with transfusions and prompt attention to infectious symptoms will be the initial management. If an antigen-matched full sibling is available, she would be a candidate for hematopoietic stem cell transplant. For patients without a matched sibling stem cell donor, current standard of care is immune suppressive therapy with antithy­ mocyte globulin, corticosteroids, and cyclosporine A; approxi­ mately 70% of patients respond to this intervention within 2 to 3 months.

You are following a 1 3-year-old adolescent girl with iron defi­ ciency anemia (IDA), which you diagnosed 1 week ago and attributed to menorrhagia. Her bleeding has been lasting 7 days per cycle, requiring seven to eight pads per day, for 2 years. At last week's visit, she was complaining of fatigue, pallor, and exercise intolerance. Initial bloodwork revealed hemoglobin 7.9 g/dL, mean corpuscular volume (MCV) 63 fL, red cell distribution width (RDW) 1 5%, reticulocyte count 1 .3% (reference range 0.2- 1 .5%), and serum ferritin 4 ng/mL (reference range 1 1 - 300) . The patient was started on a combined oral contraceptive at that time, and began treatment with ferrous sulfate 325 mg tablets ( 65 mg elemen­ tal iron per tablet), 1 tablet by mouth twice daily. She weighs 40 kg. Today, the patient reports that she has had no men­ strual bleeding in the past week, but that her other symptoms are largely unchanged from the last visit. She reports com­ pliance with all doses of medication. Lab work today reveals hemoglobin 8 . 1 g/dL, MCV 69 fL, RDW 1 9%, reticulocyte count 5.2%, and serum ferritin 10 ng/mL. Question 1 9- 1 Which o f the following would you recommend t o the patient? A) Increase iron dose to 2 tablets twice daily. B) Continue current iron therapy and follow up in 1 month. C) Refer to hematology for intravenous iron sucrose infusion. D) Discontinue iron therapy and follow up in 1 month. E) Perform hemoglobin electrophoresis. Discussion 1 9- 1 Th e correct answer i s "B:' This patient has findings suggestive of iDA in the setting of excessive menstrual losses and is show­ ing an appropriate response to iron therapy. Patients with IDA will exhibit microcytic anemia, and often elevated RDW reflec­ tive of anisocytosis due to irregular red cell production in the iron-deficient bone marrow. Therapy is recommended with oral elemental iron (elemental iron is the bioavailable form; one must distinguish this from total iron often reported on over­ the-counter tablet preparations) , 3 to 6 mg/kg/day in one or two divided doses. Despite this wide dosing range, there are no consistent data to suggest that 6 mg/kg/day is more effective than 3 mg/kg/day (option ''A''), and the patient is responding to

the current approximately 3 mg/kg/day she is taking. The ini­ tial response to therapeutic iron will be seen as reticulocytosis. Note that this patient's initial reticulocyte count was within the reference range, but was inappropriately low for her degree of anemia. One week following initiation of therapy, this number increased appropriately. Consequently, and since reticulocytes are larger than mature erythrocytes, the MCV and RDW have increased as well. These findings are reassuring that the patient is complying with medication and experiencing a physiologic response to it. While very rare individuals are refractory to oral iron due to a defect in absorption, a patient who is tolerating and responding to oral iron therapy would not be a candidate for IV iron infusion (option "C") . A measurable rise in hemoglo­ bin may take several weeks to occur, and restoration of normal counts may take 2 to 3 months. Similarly, as the liver iron stores were presumably depleted initially, this patient's iron stores are far from restored, and the ferritin would not be expected to normalize for up to several weeks. Standard iron replenishment therapy is expected to require approximately 3 months before the patient can resume a maintenance or dietary dose (making option "D" incorrect) . Patients with thalassemia trait also pres­ ent with mild-to-moderate anemia and microcytosis and can be mistaken for having IDA. Particularly in young patients, the two conditions can coexist. It is important to follow markers of iron stores, and replenish iron as needed, before contemplating a diagnosis of thalassemia trait. Children who are iron replete, yet remain anemic and microcytic, may be clinically diagnosed with thalassemia trait; the family history may also assist in this diagnosis. Genetic testing is rarely used to confirm this diag­ nosis, although hemoglobin electrophoresis (option "E") can assist in the diagnosis of beta-thalassemia minor. In this condi­ tion, the level of hemoglobin A2 (consisting of a/ )2 globins) is often found to be elevated. However, this finding is obscured in iron-deficient patients, which is why correction of iron defi­ ciency is essential before pursuing the thalassemia trait diag­ nosis. Patients with alpha-thalassemia minor (2 gene a globin deletion) or minima ( 1 gene a globin deletion) will not exhibit changes on standard hemoglobin electrophoresis.

A 3-year-old boy is brought to urgent care clinic for fever, pain, and fatigue. He was previously healthy until 3 to 4 weeks ago, when he started seeming more tired to his par­ ents. He developed fevers 6 days ago, with oral temperatures up to 39. 1 oc ( 1 02.4°F), which have been occurring daily and are responsive to acetaminophen. His parents also feel that he has seemed paler to them, and he has bruising on his legs. Vital signs include temperature 38. 1 °C ( 1 00.6°F), heart rate 136 beats per minute, and respirations 24 breaths per minute. On exam, the patient is awake, alert, but tired-appearing. He falls asleep during the exam, awakens easily but wants to be held by his mother and is unwilling to participate in gait test­ ing. He has conjunctival pallor; bilateral cervical lymphade­ nopathy; grade II/VI systolic ejection murmur; nontender

CHAPTER 4

splenomegaly with the tip palpable 4 em below the left costal margin; bruising on the lower extremities, hips, back, and neck; and scattered petechiae on flexural surfaces. Respira­ tory effort and lung exam are normal. CBC reveals white blood cells of 36 x 1 03/mm\ hemoglobin 6. 1 g/dL, and plate­ lets 9 x 1 03/mm3• The automated white blood cell differential reported 90% lymphocytes, but the lab technician called to say that a manual pathology review is underway because the cells appear atypical. Question 20-1 Which of the following will be the most crucial to obtain as you are arranging to admit this patient to the hospital? A) Epstein-Barr virus (EBV) serologic panel. B) Computed tomography (CT) of the chest with intravenous contrast. C) Prothrombin time (PT) and activated partial thromboplas­ tin time ( aPTT). D) Serum electrolytes and uric acid. E) Child abuse consult. Discussion 20- 1 Th e correct answer i s "D:' This patient presents with findings concerning for acute leukemia, most likely acute lymphoblas­ tic leukemia (ALL) . In many cases, the blood smear will reveal presence of leukemic blasts as shown in Figure 4-6, which speeds diagnosis, although the absence of circulating blasts does not exclude ALL. It is also not possible to diagnose the subtype



BLOOD AND N EOPLASTIC DISORDERS

91

of acute leukemia based on the morphologic appearance of the blasts; flow cytometric analysis and molecular genetic evalua­ tions are required. He has symptomatic anemia and thrombo­ cytopenia, as well as fever, and will require hospital admission for diagnostic procedures and blood product transfusions, fol­ lowed by chemotherapy if the diagnosis is confirmed. Despite the elevated white blood cell count, patients with newly diag­ nosed ALL are assumed to be functionally neutropenic and the presence of fever in this patient will warrant blood culture and empiric antimicrobial therapy. One potentially threaten­ ing complication at this time is tumor lysis syndrome (TLS). TLS is brought about by a sudden lysis of a substantial quan­ tity of leukemic cells simultaneously, creating a massive serum increase in intracellular constituents that can be life-threaten­ ing. A chest X-ray is often obtained to rule out the presence of a mediastinal mass, which is most frequently encountered with T-lymphoblastic leukemia and some lymphomas; however, a CT scan is generally not required urgently (option "B"), and presence of a mediastinal mass can cause respiratory obstruc­ tion if the patient is placed supine for the scan. Some patients with subtypes of newly diagnosed acute myeloid leukemia are at elevated risk of coagulopathy and disseminated intravascu­ lar coagulation, and screening with PT and aPTT is appropriate (option "C"), but this is not the most urgent concern for the patient in the vignette. Because of the patient's systemic symp­ toms, lymphadenopathy, and organomegaly, suspicion for EBV infection is appropriate (option "A"). However, the degree of leukocytosis and refusal to bear weight on his legs makes this patient's diagnosis more likely to be malignancy. Some of the findings, including the ill appearance, bruising, and refusal to bear weight, may be consistent with nonaccidental trauma (option "E"), but this would not be expected to lead to the blood count findings, organomegaly, or lymphadenopathy.

i QUICKQUIZ Which is NOT a laboratory finding of acute tumor lysis syndrome? A) Hypokalemia. B) Hyperphosphatemia. C) Hypocalcemia. D) Hyperuricemia. E) Elevated serum creatinine.

FIGURE 4-6. Leu kemic blasts. The a b u n d a nt wh ite blood cel l s s h own a re from the blood smear of a patient with B-lym phobl astic leuke m i a . They a re abnormal i n their l a rge, i rreg u l a r, heteroch romatic nuclei, a s wel l as the presence of scant b l u e cytoplasm lacking g ra n u l es. (Used with permission from Ada m D. Wolfe, M D, Ph D.)

Discussion The correct answer is "A:' Findings of tumor lysis syndrome (TLS) can include hyperkalemia, hyperphosphatemia, hyper­ uricemia, and hypocalcemia (secondary to precipitation of cal­ cium phosphate) . For these reasons, any patient with a suspected diagnosis of a hematologic malignancy should be screened for these changes, and treated immediately for any abnormal find­ ings. Cardiac instability can be brought about by hyperkale­ mia, neurologic findings may result from hypocalcemia, and acute renal failure may be seen in patients who precipitate urate or calcium phosphate crystals in the renal tubular system.

92

MCG RAW-H I LL EDUCATION SPECIALTY BOARD REVI EW: PEDIATRICS

The greatest risk ofTLS is during the initiation of induction che­ motherapy, but TLS can also occur spontaneously before treat­ ment. Patients with TLS should be started on approximately twice-maintenance fluid infusion without potassium. This patient would receive a medication to reduce uric acid: allopu­ rinol is used in patients without hyperuricemia, while rasbu­ ricase (recombinant urate oxidase) is administered to patients with established hyperuricemia.

A term baby boy is found on day of life 1 to have petechiae in his antecubital and popliteal fossae, and around the base of the neck. A CBC reveals a platelet count of 13 x 1 03/mm\ and the platelets appear small in size on blood smear evalua­ tion. Family history reveals that the patient has two brothers, ages 6 and 2 years; the older brother has eczema, frequent nose bleeds, bruises easily, and has been admitted to the hos­ pital several times with bacterial pneumonia, while the other brother is healthy. The patient's mother has no chronic ill­ ness; her older brother died as a teenager from prolonged bleeding following a motorcycle accident. Question 2 1 - 1 Which of the following additional findings would you antici­ pate in this patient? A) Absence of radii on upper extremity X-rays. B) Marked decrease in megakaryocytes on bone marrow aspirate. C) Mutation in WAS gene on genetic testing. D) Mutation in RUNXl gene on genetic testing. E) Hyperpigmentation and microcephaly on physical exam. Discussion 2 1 - 1 The correct answer is "C:' This patient presents with congenital microthrombocytopenia and an X-linked inheritance pattern with family history of atopic disease and immunodeficiency, all consistent with Wiskott-Aldrich syndrome (WAS) . WAS is caused by a defective WAS protein, which participates in cyto­ skeletal connections that are essential for platelet and leukocyte function. The classic diagnostic triad for WAS is thrombocyto­ penia, immunodeficiency, and eczema. The platelets in WAS are small in size. Neonates with WAS are at increased risk of signifi­ cant hemorrhage and merit support with platelet transfusion. Because of the substantial risk of life-threatening bleeding or infection, long-term management includes hematopoietic stem cell transplant (HSCT) if a suitable donor is available. None of the conditions referenced in the other answer choices are inher­ ited in an X-linked fashion, but they are associated with neona­ tal thrombocytopenia. Absence of radii in a thrombocytopenic neonate (option "1\.') is associated with the aptly named throm­ bocytopenia and absent radii (TAR) syndrome. This autosomal recessive condition has an unclear molecular mechanism, but it is associated with a decrease or absence in megakaryocytes in the bone marrow (option "B"). Management of individuals with

TAR syndrome is generally supportive; patients who avoid severe hemorrhage during the infant period will typically see resolution of the thrombocytopenia during the second year. Another con­ dition associated with congenital thrombocytopenia and skel­ etal abnormalities is Fanconi anemia (FA). This is an inherited bone marrow failure and cancer predisposition syndrome with associated radial and thumb abnormalities, pigmentation abnor­ malities, genitourinary malformations, microcephaly, and short stature (option "E"). FA is most often inherited in an autosomal recessive pattern, and conveys increased risk of developing myelo­ dysplasia, leukemia, head and neck tumors, and liver tumors. Patients with FA exhibit chromosome instability diagnosed by exposure to diepoxybutane; management typically includes refer­ ral for HSCT. Mutations in RUNXl/AML gene (option "D") are associated with congenital and familial thrombocytopenias and predisposition to myelodysplasia and acute myeloid leukemias; this condition is quite rare and is curable by HSCT. In neona­ tal alloimmune thrombocytopenia, the fetus/neonate inherits a platelet antigen from the father that the mother lacks. Maternal IgG antibodies to this antigen cross the placenta causing alloim­ mune hemolysis. In autoimmune thrombocytopenia occurring in mothers with autoimmune diseases such as lupus or immune thrombocytopenia (ITP), maternal antibodies react with both the mother and the fetus/neonate's platelets. Additional causes of neonatal thrombocytopenia include drug induced, thrombo­ sis, hypersplenism, disseminated intravascular coagulation, and preeclampsia.

You are working in the emergency department when a 7-year-old boy comes in by ambulance after a bicycle versus motor vehicle crash. The patient was struck in the abdomen by the handle bars and suffered immediate left upper quad­ rant abdominal pain. His heart rate is 1 66 beats per minute, respirations 36 breaths per minute, and blood pressure is 82/44 mm Hg. On exam, he is pale, diaphoretic, dyspneic, and deeply bruised in the area of the trauma. The abdomen is diffusely tender to palpation. You suspect possible splenic laceration, and consult the surgical team. While they are en route, you obtain intravenous access and order a CBC, coag­ ulation screening labs, and blood type and cross match. You also order 0-negative blood to give immediately, which will arrive in less than 5 minutes. During this time, the patient's mother asks you whether the blood transfusion will harm her son. Question 22-1 Which of the following will you tell her is the most likely com­ plication of the red blood cell transfusion for this patient? A) Fever. B) HIV infection. C) Transfusion-related acute lung injury (TRALI). D) Gram-positive bacterial infection. E) Acute hemolytic event.

CHAPTER 4

Discussion 22-1 The correct answer is "A:' Clearly, the patient in the vignette is experiencing a severe internal hemorrhage and will require probable massive transfusion, with the benefits far outweigh­ ing the risks. However, it is not uncommon for parents to have questions and concerns relating to the safety of blood product transfusion. The overall incidence of fever associated with blood transfusion is approximately 1 per 1 00 units transfused, making it one of the most common adverse effects. Premedication or as-needed therapy with acetaminophen can prevent or treat this reaction; typically it is self-limited following transfusion. Infec­ tious complications ofblood transfusion, which used to be more frequent, are now quite rare. The estimated incidence of bacte­ rial infection, primarily due to gram-positive skin flora, with red cell transfusions (option "D") is between 0.2 and 7.4 per million units. Platelet transfusions are associated with higher rates of gram-positive infection, up to 1 per 2000 units trans­ fused if the platelets were stored at room temperature prior to administration. Viral infections are also potentially transmitted. Cytomegalovirus (CMV) may be transmitted from seropositive donors to seronegative recipients, although products that have been leukoreduced and screened are considered CMV-safe and should be appropriate for most indications. Individually tested, CMV-negative blood products are also available for use, largely with immunocompromised recipients. HIV (option "B") and hepatitis C virus transmission from blood products approaches 1 per 1 to 2 million units, making this quite a rare concern. Hep­ atitis B virus transmission is slightly more common, at between 1 in 200,000 to 500,000 units. TRALI (option "C") is a rare com­ plication of blood transfusion, associated with transference of antineutrophil antibodies or neutrophil-activating metabolites to the recipient. This leads to neutrophil activation and trap­ ping in pulmonary microvasculature and development of acute respiratory distress syndrome that can be life-threatening. The rate of TRALI in pediatrics is uncertain, but it likely occurs in fewer than 1 in 1 0,000 units. The reaction is self-limited, and with aggressive hemodynamic and respiratory supportive care, often resolves within several days. Acute hemolytic events (option "E") are also rare; they result typically from adminis­ tration of an ABO-mismatched unit. Symptoms include fever, chills, hypotension, flank pain, oliguria, and cola-colored urine. Supportive care is aimed at keeping the renal system hydrated until the reaction abates. Incidence of acute hemolytic reactions is estimated at 1 in 75,000 units. A delayed hemolytic reaction may occur in up to 1 per 1 500 transfusions, more often due to minor blood group antigen mismatch, up to 10 days after the transfusion. The hemolysis is generally less pronounced, and symptoms less dramatic, than seen in the acute reactions.

A 2-month-old girl comes to the urgent care clinic for evaluation of fever. Her mother reports that the baby has been completely healthy since birth, is growing and devel­ oping normally, but developed temperature up to 38.6°C



BLOOD AND N EOPLASTIC DISORDERS

93

( 1 0 1 .5°F) last night and has been febrile through today. The baby did not want to nurse much this morning. She has had three wet diapers over the past 8 hours and nor­ mal stools daily. Vital signs reveal temperature 38.5°C ( 1 0 1 .3°F), pulse 1 24 beats per minute, and respirations 22 breaths per minute. Physical exam reveals a baby who is alert and cries throughout the exam. The anterior fon­ tanelle is open, soft, and flat, not sunken. Mucous mem­ branes are moist. The left tympanic membrane exhibits circumferential erythema, dull light reflex with bulging, and purulent material behind the drum. Heart, lung, and neurologic exams are normal. Because of the fever, a CBC is obtained, which reveals white blood cell count 12.5 x 1 03/mm3 with a normal white cell differential, hemoglo­ bin 9.2 g/dL, and platelets 360 x 1 03/mm3• The mean cor­ puscular volume (MCV) is 88 fL. Question 23-1 Which of the following is the next best step in management of this patient? A) Intravenous ceftriaxone. B) Hemoglobin electrophoresis. C) Transfusion with packed red blood cells. D) Bone marrow aspiration and biopsy. E) Oral amoxicillin. Discussion 23-1 The correct answer is "E." This patient has been inadver­ tently identified as "anemic" during her physiologic red cell nadir of infancy, while being evaluated for left acute oti­ tis media. Although practice may vary regarding "obliga­ tory" lab work for 2 -month- old infants with fever, most practice guidelines do not suggest this when the patient has an etiology for fever identified on exam. Shortly after birth, infant erythropoiesis decreases due to an abundance of oxygen. As hemoglobin concentrations drift lower, infant erythropoietin production is stimulated and drives new erythropoiesis. The period of decreased hemoglobin concentration is referred to as the physiologic nadir and is not a pathologic process . Nadir occurs between 6 and 12 weeks of life, although the timing can vary and nadir tends to be earlier, more pronounced, and more prolonged in preterm infants . Hemoglobin levels in term infants can drop to 9 to 1 1 g/dL; as this is a normal finding for age, this patient does not require transfusion support (option "C" ) . Having only acute symptoms associated with her ear infection, this patient is a candidate for outpatient manage­ ment, and should not require p arenteral antibiotics (option "!\' ) as frontline therapy. Since she has otherwise normal blood counts, evaluation of bone marrow (option "D") is not warranted. Hemoglobin electrophoresis ( option "B") might be helpful for diagnosing beta-thalassemia trait in an iron replete child with persistent microcytic anemia, or if hemoglobinopathy is suspected, but the vignette does not provide any data to suggest these diagnoses.

MCG RAW-H I LL EDUCATION SPECIALTY BOARD REVI EW: PEDIATRICS

94



Helpful Tip

� Remember hemoglobin and MCV vary by age in infants

r1 1 r and children. Consult pediatric reference tables when

interpreting va lues. Both are highest in the neonatal period, then decrease to adult va lues in adolescence.

A 4-year-old boy with sickle cell anemia (hemoglobin SS disease) presents to the emergency department with acute onset of fever, fatigue, and pallor that began this morning. He has previously been admitted to the hospital, once for an acute vaso-occlusive event with lower extremity pain, requir­ ing intravenous opioid medications for 48 hours, but has otherwise been well at home. Medications include daily oral penicillin, folic acid, and hydroxyurea. The hydroxyurea dose was recently increased at a clinic visit with the hematologist 2 weeks ago. Vital signs currently include temperature 38.7°C ( 1 0 1 .6°F), heart rate 1 70 beats per minute, and respirations 34 breaths per minute. In general, the patient appears very tired, pale, and dyspneic. A grade III/VI systolic ejection murmur is heard over the left sternal border but is audible throughout the chest. The spleen is palpable to 7 em below the left costal margin. You are considering the diagnosis of a splenic sequestration event versus an aplastic crisis. Question 24- 1 Which of the following subsequent findings would make you more suspicious for splenic sequestration? A) Reticulocyte count of 0.2%. B) Hemoglobin 3 g/dL below the patient's baseline. C) Platelet count of 55 x 1 03/mm3• D) Absolute neutrophil count of 940/mm3• E) Left lower lobe infiltrate on chest X-ray. Discussion 24- 1 The correct answer is "C:' This patient with sickle cell disease presents with a febrile illness and symptoms suggestive of an acute drop in hemoglobin. In children with hemoglobin SS and an intact spleen (ie, those younger than 5 to 6 years of age), the most dangerous complication with this presentation is a splenic sequestration event. During sequestration, a large proportion of the patient's red blood cells become trapped in the spleen, which can subsequently expand to fill much of the abdomen. Patients with severe sequestration can drop acutely to hemo­ globin levels at a fraction of their baseline, putting them at risk for cardiac compromise or hypoxic-ischemic events. Because of entrapment of blood in the spleen, platelets also become trapped in the splenic sinusoids, leading to thrombocytopenia. Patients will also typically exhibit a reticulocytosis in an effort to replace the entrapped red cells. As a life-threatening event, prompt attention to sequestration is critical. Parents of young children are educated to palpate the patient's spleen at home in the setting of severe illness or anemia symptoms, and to seek

medical attention immediately if the spleen becomes palpable. If sequestration is suspected and severe anemia confirmed, the event may be reversed by aggressive fluid resuscitation. This often will allow release of the entrapped red cells and restoration of safer hemoglobin concentrations. Transfusion with packed red cells may also be necessary. In this patient's case, man­ agement of his fever along with obtaining blood cultures and treating with empiric parenteral antimicrobial therapy is also warranted. Another complication of sickle cell disease in chil­ dren is an aplastic crisis, associated with parvovirus B 1 9 infec­ tion and transient suppression of bone marrow erythropoiesis. The patient in the vignette might be experiencing red cell apla­ sia, although the acuity and severity of symptoms are not typi­ cal. Although parvovirus B 19 can cause transient suppression of erythropoiesis in children without sickle cell disease, children with normal red cell lifespan recover their red cell produc­ tion before becoming symptomatically anemic. Children with conditions causing reduced red cell life span, including those with sickle cell anemia, hereditary spherocytosis, or G6PD defi­ ciency, are at increased risk of exhibiting symptomatic anemia before their marrow recovers from the infection. Because it is a condition of marrow suppression, patients with aplastic crisis exhibit reticulocytopenia (option ''A''). Both aplastic crisis and splenic sequestration can exhibit a significant drop in hemo­ globin from the patient's baseline (option "B"), although the anemia from sequestration can be more severe and more acute. A patient with sickle cell disease who presents with fever and a new infiltrate on chest X-ray (option "E") might be suspected of having acute chest syndrome, but infiltrate on imaging is not a finding associated with splenic sequestration or aplastic cri­ sis. Moderate neutropenia (option "D") is also not associated with splenic sequestration. It is more probable that the patient's recent dose increase of hydroxyurea would be responsible for this finding.

A previously healthy 8-year-old girl suffered severe inhala­ tional lung injury during a house fire 10 days ago, and has been in the intensive care unit on an extracorporeal mem­ brane oxygenation (ECMO) circuit for 8 days, stably antico­ agulated per protocol with unfractionated heparin (UFH) . She has been afebrile. This morning, the patient exhibits right lower extremity soft tissue edema; Doppler ultrasound imaging reveals an occlusive deep venous thrombus in the right femoral vein. Lab work reveals white blood cell count of 8.2 x 1 03/mm3 with a normal differential, hemoglobin 1 0.4 g/dL, and platelets 34 x 1 03/mm3• Question 25-1 Which of the following is the next best step in the evaluation and management of this patient? A) Transfuse 10 mL/kg platelets. B) Replace heparin with a different anticoagulant in the ECMO circuit.

CHAPTER 4

C) Send testing for factor V Leiden (FVL) mutation. D) Send testing for heparin-induced thrombocytopenia (HIT) antibody. E) Increase the heparin dose. Discussion 25-1 The correct answer is "B." This patient likely has HIT, an immune response to heparins that occurs in approxi­ mately 0 . 3 % to 0.6% of critically ill patients receiving UFH and low-molecular-weight heparins (LMWH; eg, daltepa­ rin, enoxaparin) . HIT suspicion is based on the so- called "4 Ts;' described in Table 4-6. When the likelihood of HIT is high, the immediate next step is to eliminate all heparins. Although diagnostic testing for HIT antibody is appropri­ ate (option "D" ) , it is not recommended to obtain this result before cessation of heparin. This patient, who likely has HIT associated with UFH in the ECMO circuit, will be increas­ ingly challenging to manage. She will require anticoagulation with a nonheparin agent: for example, the direct thrombin inhibitor argatroban is considered an appropriate anticoagu­ lant in children with HIT. Once a patient has been diagnosed with HIT, she should not receive UFH or LMWH in the future. The HIT antibody is directed against platelet factor 4. While it ultimately causes rapid clearance of platelets, the antibody usually has a platelet-activating effect. Therefore, it is important to note that the most common presenting find­ ing of HIT is thrombosis, rather than bleeding, despite the coexistence of thrombocytopenia. Due to the risk of further clotting with HIT, platelet transfusion (option "A'') is not rec­ ommended unless the patient is experiencing an emergent bleeding event due to the thrombocytopenia. This patient may have an underlying inherited thrombophilic condi­ tion, such as FVL, that increases her lifetime risk of deep venous thrombosis (option "C"), but it would be less likely that a patient with FVL on an anticoagulated ECMO circuit would develop a new DVT with thrombocytopenia; HIT is



BLOOD AND N EOPLASTIC DISORDERS

95

a more likely etiology, and genetic testing would not be the next step in management for this patient. Likewise, increas­ ing the heparin ( option "E") in this clinical situation would not be appropriate given the development of new thrombus and thrombocytopenia while on an appropriately heparin­ ized circuit. •

Helpful Tip

-� Thrombosis is the most common presenting sign of hepa rin - induced thrombocytopenia (H IT) as the HIT antibody activates platelets.

i1 1 r

You are rounding on a term newborn girl in the nursery, who was born yesterday at 40-3/7 weeks' gestation by nor­ mal spontaneous vaginal delivery to a healthy, primipa­ rous mother following an uncomplicated delivery. The baby appears healthy, is feeding well, and has voided and passed meconium since birth. The patient's mother is concerned that several members of her family have had cancers at young ages and wants to get your opinion about the risks of future cancers for her new baby. You consider the possibility of an inherited cancer predisposition syndrome. Question 26-1 Regarding these syndromes, which of the following molecu­ lar changes is correctly matched to an associated malignancy? A) Trisomy 2 1 -osteosarcoma. B) Neurofibromin-1 (NFl) mutation-vestibular schwannoma. C) p53 deletion-basal cell carcinoma. D) TSC mutation-optic pathway glioma. E) Abnormal l l p l S imprinting-Wilms tumor.

TABLE 4-6 4-T SCO R I N G SYSTEM FOR S U S P I C I O N OF H E PARI N - I N D U C E D T H R O M BOCYTOpE N I A ( H IT)

Score (range 0-2 for each criterion) Clinical Finding

0

1

2

Throm bocytopenia (with respect to prior to hepa r i n i n itiation)

< 30% d ro p i n platel ets, or nad i r < 1 0 x 1 03/m m 3

30-50% d ro p i n platel ets, or nad i r 1 0- 1 9 x 1 03/m m 3

< 50% d ro p in platel ets, and nad i r ;:: 20 x 1 03/m m3

Ti m i ng of t h ro m bocytopenia o r t h ro m bosis (ass u m i ng hepa r i n ­ na·ive patient)

Wit h i n 4 d ays

U ncerta i n ti m i ng, l i kely with i n 5 - 1 0 d ays; or afte r 1 0 days

During d ays 5 - 1 0

Thrombosis

None

S u s pected t h ro m bosis or prog ress ion/recu rrence of preexisti ng t h ro m b u s

New t h ro m bosis

Other cause of t h ro m bocytope n i a ?

Defi n ite other cause identified

Poss i b l e other cause identified

No other cause identified

Scoring: 0-3, low probabil ity of HIT; 4-5, intermediate probability of HIT; 6-8, high probabil ity of HIT. Reproduced with permission from Warkentin TE: Heparin-induced throm bocytopenia in critica lly ill patients, Semin Thromb Hemost 2 0 1 5 Feb;41 (1 ) :49-60.

96

MCG RAW-H I LL EDUCATION SPECIALTY BOARD REVI EW: PEDIATRICS

Discussion 26- 1 The correct answer is "E:' A comprehensive understanding of cancer predisposition syndromes goes beyond the general pediatrics board content specifications; however, it is helpful to understand how to link abnormal physical exam findings and family history to future cancer risk. This will aid the pediatrician in identifying next diagnostic steps, and in making appropri­ ate referrals. Abnormal imprinting at 1 1 p 1 5 is associated with Beckwith-Wiedemann syndrome (BWS), which is associated with hemihypertrophy, macroglossia, omphalocele, neonatal hypoglycemia, and increased risk of Wilms tumor and hepa­ toblastoma. Screening with abdominal ultrasound is recom­ mended every 3 to 6 months until age 8 years. Hepatoblastoma may also be screened by serum alpha-fetoprotein levels obtained every 3 months until age 4 years. There is often no family his­ tory in individuals with BWS. Trisomy 2 1 , or Down syndrome (option "/\') , is most commonly associated with increased risk of hematologic malignancy. This includes increased risk of developing acute lymphoblastic leukemia, and increased risk of acute myeloid leukemia, frequently of the megakaryoblastic subtype. Children with trisomy 2 1 are exquisitely chemosensi­ tive, and have more severe adverse effects from chemotherapy than children without this difference. Therefore, leukemia treat­ ment protocols often have different therapeutic dosing and sup­ portive care recommendations for children with trisomy 2 1 . Mutation of NF1 , the causative genetic change in neurofibro­ matosis type 1 (option "B"), is associated with formation of neu­ rofibromas, optic pathway gliomas, other low-grade gliomas, and peripheral nerve sheath tumors. Vestibular schwannomas are associated with neurofibromatosis type 2, which is a rarer condition caused by mutations in the merlin gene. The Li-Fraumeni cancer predisposition syndrome is associ­ ated with mutations in tumor suppressor gene p53 (option "C"). This syndrome includes unusual tumors of many types that arise in young individuals within a pedigree. Common tumors associated with Li-Fraumeni syndrome include those that start with "B" -blood (ie, leukemias), brain, bone (eg, osteosar­ coma), and breast. Many other cancers have been associated with this condition, although skin cancers are not among them. Mutations in TSC complex genes (option "D") are associated with tuberous sclerosis, a condition with presenting symptoms that include seizures, intellectual disability, and facial angiofi­ bromas. Tumors associated with tuberous sclerosus are often benign and include cardiac rhabdomyomas, subependymal giant cell astrocytomas, and renal angiomyolipomas.

An 8-year-old boy presents to the emergency department with spreading bruising on his legs and buttocks. The symp­ toms started 1 week ago, with isolated, painful bruises on the legs, which increased and expanded until becoming nearly confluent across the entire lower extremities. Two days ago, the bruising also began to include the penis and scrotum. Today, the patient has been unwilling to walk because of pain.

On exam, the patient is in significant pain and cries out when the palpable purpura on the buttocks, genitalia, and lower extremities are touched. The tissue has alternating areas of violaceous purpura and blue-black bullous changes. Lower extremity pulses are faint and thready. Skin on the trunk, abdomen, chest, back, upper extremities, head, and neck is unaffected. When an antecubital peripheral intravenous line is placed, there is substantial and continuous oozing around the site. Lab work reveals white blood cells 1 5.8 x 1 03/mm3 with 80% neutrophils and 1 5% lymphocytes, hemoglobin 9 . 1 g/dL, platelets 33 x 1 03/mm\ prothrombin time (PT) 22 s (reference range 1 2- 1 5 s) , activated partial thromboplastin time (aPTT) 74 s (reference range 24-36 s). Question 27- 1 I n addition t o pain management, blood culture, and anti­ microbial therapy, which of the following is the next best intervention? A) Platelet transfusion. B) Fresh frozen plasma (FFP) transfusion. C) Unfractionated heparin (UFH) loading dose and mainte­ nance infusion. D) Intravenous immunoglobulin (IVIG) infusion. E) Induction chemotherapy. Discussion 27- 1 Th e correct answer i s "B:' This unfortunate patient presents with purpura fulminans and associated disseminated intravas­ cular coagulation (DIC), a life- and limb-threatening condition. Thrombi form in the microvasculature causing ischemic/necrotic damage to the skin and tissue. The most appropriate interven­ tion is to identify and treat the underlying cause, and to attempt to restore the balance of pro- and anticoagulant factors in the patient's blood as soon as possible. Of the choices given, FFP is the best therapy to initiate. Neonates with homozygous protein C deficiency present with neonatal purpura fulminans, an often fatal condition. In older children, the condition is due to sepsis or antithrombotic factor dysfunction. For example, this patient's presentation could be consistent with meningococcemia, a known inciting event of DIC with or without associated pur­ pura fulminans. Patients who are started on the oral vitamin K antagonist warfarin are at risk of warfarin-induced skin necrosis, which is a form of purpura fulminans caused by microthrombi in the skin following warfarin's depletion of proteins C and S. Some patients develop an autoantibody to protein S, causing a clinical presentation similar to that described in the vignette. For reasons that are not clear, the purpura in this clinical entity preferentially affect the lower body segment. There is no clear consensus in the critical care or hematology literature to support the use of antico­ agulation in pediatric purpura fulminans (making option "C" less desirable) . Because the patient has consumed his procoagulant factors and platelets in microvascular thromboses, the bleeding risk is elevated; heparin would further increase the risk of hemor­ rhage. Should the patient develop a severe hemorrhage, platelet transfusion (option "/\') might be considered, but as the plate­ lets are currently being consumed in thrombi that are injuring

CHAPTER 4

his skin, platelet transfusion would generally be avoided until the coagulopathy is better controlled. Another clinical entity that can exhibit lower body segment purpura is Henoch-Schonlein pur­ pura (HSP). This is a vasculitis mediated by IgA, with the clinical triad of abdominal pain, arthritis, and lower segment palpable purpura. Patients with HSP generally do not become as ill as the patient in the vignette, and since HSP is not a consumptive coag­ ulopathy, the platelet count would be expected to be normal or elevated in this condition. The patient's presentation has some overlapping features with acute myeloid leukemia (AML). The promyelocytic form of this condition may present with multiple cytopenias and DIC, due to a tissue factor-like substance released by the blasts. However, this patient's differential did not report any blasts, and the purpura ful­ rninans would be unexpected for AML. This patient might undergo bone marrow assessment during his evaluation, but chemotherapy (option "E") would not be started empirically as the next step.

A 2-day-old neonate is ready for hospital discharge with his mother, and you are conducting his discharge physical exam. The pregnancy and delivery were uncomplicated, and the mother was on no medications and had no infections. Delivery was at 37- 1/7 weeks' gestation, by normal sponta­ neous vaginal delivery, Apgar scores were 9 and 9 at 1 and 5 minutes. On your evaluation today, the baby has normal vital signs, nondysmorphic features, normal extremities, and a normal heart and lung exam. During the eye screening, you note leukocoria on the left, and a normal red reflex on the right. There is no family history of eye problems, including cataracts or tumors, or cancers of any kind. Question 28-1 Which of the following is most likely to be identified on sub­ sequent evaluation? A) Increased alpha-fetoprotein for age. B) Germline p53 mutation. C) Increased urinary catecholamines. D) Germline Rb mutation. E) Germline NFl mutation. Discussion 28-1 The correct answer is "D:' Of the options presented, this patient most likely has congenital retinoblastoma (RB). While RB is not the sole cause, or most common cause, of loss of red reflex in a newborn, it is in the differential diagnosis for this patient. RB may occur broadly in two categories: hereditary and sporadic. While the nomenclature may be misleading, hereditary RB is defined based on the presence of a germline mutation in the Rb gene, even in the absence of a family history. Patients with a single germline mutation are at high risk of acquiring a second, somatic mutation during embryonic retinal development (the so-called "second hit"), which leads to development of RB early, often diagnosed within the first year of life. Because of the presence of



BLOOD AND N EOPLASTIC DISORDERS

97

the initial germline mutation, patients with hereditary RB are at high risk of development of tumors in both eyes, even though these may not both present concurrently. RB is the most common malignant tumor affecting the eye in children and carries a sub­ stantial risk of intracranial extension if not promptly identified and treated. Treatment is based on extent of disease, and whether both eyes are affected. In unilateral RB with smaller tumors, local treatment such as laser or cryotherapy may be appropriate, while larger tumors may require enucleation. Chemotherapy may be administered via intravitreous, intra-arteriolar, or intravenous routes, depending again on tumor size and extent of invasion. Children with hereditary RB are also at higher risk of developing intracranial involvement, which most often involves the pineal gland (so-called "trilateral" RB) . Therefore, for the patient in the vignette, ophthalmologic evaluation under anesthesia and MRI of the brain will be appropriate next steps for evaluating extent of disease. Rb codes for the tumor suppressor protein RB. Patients with germline mutations are at increased risk during their life­ times of multiple malignancies associated with loss of this pro­ tein if a second, somatic mutation occurs in other tissues. The nonocular malignancy most often associated with hereditary RB is osteosarcoma. Sporadic RB is thought to be due to acquisition of two somatic mutations later during development, and for this reason often presents later, around or after 2 years of age. These patients are at low risk of bilateral and trilateral RB (bilateral eyes and brain) , although the initial evaluation and therapeutic options are the same as described above. Other tumors found within the neonatal period include hepatoblastoma, teratoma, and neuroblastoma. In the former two conditions, serum tumor markers alpha-fetoprotein (option "A'') and beta-human cho­ rionic gonadotropin may be elevated, while in neuroblastoma urine catecholamines vanillylmandelic acid (VMA) and homova­ nillic acid (HVA) may be elevated (option "C"). None of these is specifically associated with newborn leukocoria. As stated earlier, p53 mutation (option "B") is associated with the Li-Fraumeni syndrome (LFS), another hereditary cancer predisposition syn­ drome due to loss of a tumor suppressor gene. Neurofibromatosis type 1 , characterized by mutation of the NFl gene (option "E"), is associated with development of cafe-au-lait macules, neurofibro­ mas, and optic gliomas. •

Helpful Tip

� Leukocoria ("white pupil") is concerning and requires 1 1 1 r urgent

referra l to an ophthalmologist. Etiologies include cataracts and retinoblastoma. Checking the red reflex is important especially in infa nts.

A 19-month-old girl presents to the emergency department with bruising on her chest. Parents report that she was run­ ning in the backyard with her older brother, fell forward, and struck her chest on a rock approximately 2 hours ago. She has been complaining of pain at the site of injury, and

98

MCG RAW-H I LL EDUCATION SPECIALTY BOARD REVI EW: PEDIATRICS

parents note a large bruise developing at that site. They gave the patient a dose of ibuprofen for pain 2 hours ago. On your physical exam, the patient is alert, interactive, and appropri­ ate. A large, firm, tender hematoma with surrounding con­ tused tissue is noted across the sternum and midchest in an oval shape, approximately 1 5 em across and 5 em high. There are multiple old bruises, in varying states of healing, on the extremities and buttocks. The medical student who is seeing this patient with you asks whether this patient's injury might have been due to nonaccidental trauma (NAT). Question 29-1 Which of the following most accurately describes the hema­ tologic evaluation of a patient with suspected NAT? A) If the history, physical exam, and skeletal evaluation are consistent with accidental trauma, a bleeding disorder labo­ ratory screening evaluation is recommended. B) CBC, prothrombin time (PT), activated partial thrombo­ plastin time (aPTT), and platelet function analyzer (PFA1 00) evaluation are recommended as screening tests. C) The recommended bleeding disorder screening tests can vary depending on the presenting injuries that yielded the NAT suspicion. D) Consultation with pediatric hematology is recommended during evaluation of all suspected cases of NAT. E) Presence of petechiae at pressure sites, such as at edges of clothing, should increase suspicion for NAT. Discussion 29-1 The correct answer is "C:' This patient presents with hematoma and contusion suggestive of a blunt force injury to the chest, which may have been due to the mechanism described, or may have occurred in the context of NAT or physical child abuse. Multiple old bruises are present, mostly in typically innocent sites (eg, the extremities), but also at a more unusual site, the buttocks. Distinguishing innocent from suspicious bruising in a toddler can be very challenging. In the case of unusual bruising, distinguishing NAT from a bleeding disorder may also be diffi­ cult, and the two entities are not mutually exclusive. Recently, the American Academy of Pediatrics issued clear, evidence-based guidelines to assist in hematologic screening of patients with suspected NAT. The guidelines allow the pediatrician to defer bleeding disorder screening in cases of NAT that were clearly witnessed or clearly established through the history and other objective data (making option "!\' incorrect) . These guidelines suggest a different coagulopathy workup for patients with bruis­ ing versus those with intracranial hemorrhage (ICH; making option "C" correct) . The initial workup in both cases includes CBC with platelet count, PT, aPTT, and factor VIII and IX levels. For patients with bruising, screening for von Willebrand disease (vWD) with von Willebrand factor level and activity is added. For patients with ICH, in lieu of vWD screening, it is recom­ mended to evaluate for disseminated intravascular coagulation with fibrinogen level and D-dimer. PFA- 1 00 evaluation (option "B"), because it is difficult to interpret in children, subject to multiple physiologic confounders, and lacks sensitivity for mild disorders, is not recommended in this initial screening process.

Referral to hematology (option "D") is only suggested in cases in which an abnormal screen is identified, or when additional testing is desired. On physical exam, the presence of petechiae (subcutaneous bleeds smaller than 3 mm, usually smaller than 1 mm) at sites of pressure or on flexural surfaces is more sugges­ tive of a quantitative or qualitative platelet problem than NAT (making option "E" incorrect) .

A 15-year-old young man with no past medical history presents to the urgent care clinic for evaluation of right leg pain. He notes that he has had a dull, aching pain, ranging from 3 to 6 out of 10 on the pain scale, over the past 5 to 6 months. Over time, this pain seems to be increasing in intensity, and during the past week he has been waking from sleep with pain of severity 6 to 8 out of 10. The patient is an active high school football player, and while the pain persists during this activity, it does not seem to worsen. The pain improves temporarily with ibuprofen. Review of systems is otherwise negative. On physical exam, the patient is ambulatory, with an antalgic gait that favors the right leg. There is a minimally tender, palpable hard tissue mass along the lateral right tibia, which is ovoid and approximately 3 x 5 em. X-ray series of the right thigh, knee, and leg is ordered. Question 30-1 Which of the following statements is most accurate regarding possible bone tumors this patient may have? A) Tumor appearance on radiographic imaging will accurately distinguish osteosarcoma from Ewing sarcoma. B) Presence of nodules on computed tomography ( CT) of the chest favors osteosarcoma over osteoid osteoma. C) Both osteosarcoma and Ewing sarcoma metastasize to bone marrow. D) Primary tumors of osteoid osteoma tend to be larger than those of osteosarcoma. E) The prognoses of osteosarcoma, Ewing sarcoma, and oste­ oid osteoma are similar, although the therapeutic approach differs for each. Discussion 30-1 The correct answer is "B:' Osteosarcoma (OS) is a malignant tumor of bone-forming cells, arising preferentially in long bones, which most often metastasizes to the lungs. OS is fairly uncommon in younger children, with a peak incidence dur­ ing adolescence. The prognosis of multifocal or metastatic OS is considerably poorer than that of solitary OS, making accu­ rate staging with CT chest and bone scan essential. OS is a chemotherapy-sensitive tumor, but it is not sensitive to radia­ tion therapy (XRT). Therefore, frontline therapy includes multi­ modal chemotherapy, followed by resection of all sites of tumor, and additional chemotherapy. The extent of tumor necrosis (greater or less than 90%) at resection, which indicates the rela­ tive chemosensitivity of the tumor, has significant prognostic value. Osteoid osteoma, in contrast, is a benign tumor found

CHAPTER 4



BLOOD AND N EOPLASTIC DISORDERS

99

in long bones, most often seen in the femur or tibia. It does not metastasize (making option "E" incorrect) . It tends to be associ­ ated with dull, constant pain that is not associated with activity and may be more severe at night in children. This tumor tends to be small, often less than 1 . 5 em (making option "D" incor­ rect) . On plain film, there is a characteristic, lucent nidus with sclerotic rim; the surrounding inflammation is responsible for the pain, and explains the efficacy of nonsteroidal anti-inflam­ matory drugs (NSAIDs) in managing it. Osteoid osteoma may resolve over several years, particularly if treated with chronic NSAIDs, although surgical resection or radiofrequency abla­ tion is often used for definitive therapy and to alleviate pain. Ewing sarcoma is an embryonal small round blue cell tumor, which commonly arises from bone but may be associated with soft tissues. Unlike OS, Ewing sarcoma has affinity for either flat or long bones. Metastasis is most commonly to lungs and bone marrow (option "C"). Management of Ewing sarcoma includes neoadjuvant chemotherapy, surgical resection, XRT of primary tumor, or a combination of these and subsequent chemother­ apy. On plain film, shown in Figure 4-7, malignant bone tumors

FIGURE 4-8. X-rays of Ewi n g s a rcoma

may s h ow a n

o n ion-ski n

a p p e a ra n c e d u e to m u lt i p l e t h i n l ayers of woven bone d e pos ited over t h e ra p i d l y g rowi n g t u m o r s ite. ( R e p ro d u ced with p e r m i s s i o n from

Ka ntarj i a n HM, Wo lff RA, & Ko l l e r CA, eds. The MD Anderson Manual of

Medical Oncology, 2 n d ed. N ew Yo rk, NY: M c G raw- H i l l Education, I n c; 2 0 1 1 , F i g u re 40-7 Pa rt B.)

often exhibit periosteal reaction, poorly defined margins, a moth - eaten lytic pattern, adj acent soft tissue involve­ ment, and larger size when compared with benign tumors . A characteristic onion- skin appearance may be seen with rapidly growing tumors such as Ewing sarcoma, resulting from multiple thin layers of woven bone deposition over the site of tumor expansion. (See Figure 4 - 8 . ) When new bony spicules are laid down p erpendicular to the tumor growth, they may yield the "sunburst" pattern on X- rays. (See Figure 4 - 9 . ) Location of tumor can also yield impor­ tant information: for example, OS tends to involve the long bone metaphysis, while Ewing sarcoma is more often seen arising in the diaphysis. However, no radiographic criteria offer adequate specificity to distinguish these two tumors on X-rays alone (making option "/\' incorrect) .



FIGURE 4-7. Osteosarcoma. This p l a i n fi l m of the d i sta l fem u r exh i b its sclerotic changes of the d i sta l meta physis and d i a p hysis, and an adjacent soft tissue mass conta i n i n g calcifications. The l ifti n g of perioste u m by tumor expa nsion often produces a tri a n g u l a r sha pe, known as a Cad m a n triangle. (Used with permission from Ada m D. Wolfe, M D, Ph D.)

Helpful Tip

=-� With Ewing sarcoma or osteosarcoma, a sunburst pattern (both) or onion-skin appeara nce (Ewing) may be seen on X-ray, though, X-ray alone cannot differentiate between the two.

i1 1 r

1 00

MCG RAW- H I LL EDUCATION SPECIALTY BOARD REVI EW: PEDIATRICS She has had ongoing problems with diarrhea and recur­ rent oral and anogenital thrush. Vaccination history is unavailable. Blood counts on admission included a white blood cell count of 1 7 x 1 03/mm3 (differential: 95% neutro­ phils, < 1 % lymphocytes, 4% monocytes, 1 % eosinophils) , hemoglobin 1 0.5 g/dL, and platelets 585 x 1 03/mm3• Flow cytometry indicated an absence of T and B lymphocytes. Bronchoalveolar lavage performed this morning revealed the presence of Pneumocystis jiroveci. Question 3 1 - 1 After management of this acute illness, which of the follow­ ing long-term interventions is most likely to improve this patient's prognosis? A) Monthly infusion of intravenous immunoglobulin (IVIG). B) Highly active antiretroviral therapy. C) Strict adherence to recommended childhood vaccination schedule. D) Referral for hematopoietic stem cell transplant. E) Regular prophylaxis for opportunistic and fungal infections.

FIGURE 4-9. X-ray o f a n osteosa rcoma showing the c h a racteristic s u n b u rst pattern resulting from the formation of new bony spicules perpend i c u l a r to the tumor g rowth. (Reprod uced with permission from Kantarj i a n HM, Wolff

RA, & Kol l e r CA, eds. The MD Anderson Manual of Medical Oncology, 2 n d ed. New York, NY: McGraw-H i l l Education, I nc; 2 0 1 1 , Fig u re 40-7 Part A.)



Helpful Tip

� Bone awakening in the middle of the night is a red flag

1 1 1r sign for malignancy.

A 1 0-month old girl is admitted to the pediatric intensive care unit with severe respiratory distress. She presented last night with progressive cough, fever, and dyspnea and has been treated with intravenous antibiotics and aggres­ sive fluid resuscitation. She appears no better today, with persistent fever, tachypnea, cough, and fine crackles throughout the chest. There is no palpable lymphadenopa­ thy. Review of the medical history reveals that this patient has been closely followed by her pediatrician for failure to thrive, and is below the first percentile for weight.

Discussion 3 1 - 1 The correct answer is "D:' This critically ill infant exhib ­ its findings of failure to thrive, chronic diarrhea, recurrent infections with fungal and opportunistic organisms ( eg, Pneumocystis ) , lymphopenia, and absence of T and B cells, most consistent with severe combined immunodeficiency (SCID ) . The absence of reactive lymphoid tissue (lymph nodes, tonsils) , also described in the vignette, is noted with this condition. Affected patients often die from infection before their second birthday, and early stem cell transplant offers the only known cure. SCID has multiple subtypes; the T- B - presentation described in the vignette is inherited in an autosomal recessive fashion. T- B +NK- SCID is inher­ ited as an X-linked recessive condition. Within the United States and internationally, newborn screening programs are adopting successful screening programs for SCID, facili­ tating diagnosis and early transplant before severe infec­ tions occur. Lack of antibody production may also be seen in other conditions, such as common variable immunode­ ficiency. This condition most often presents after adoles­ cence with frequent sinopulmonary infections, decreased IgA, IgG, and IgM levels, but with detectable lympho­ cytes, and is treated with regular administration of IVIG ( option "A'' ) . The presentation of an infant with failure to thrive and frequent infections with opportunistic patho­ gens is also suspicious for human immunodeficiency virus (HIV) infection (treated by option "B" ) , but the absence of lymphadenopathy, normal hemoglobin and platelet counts, and absence of B lymphocytes-often elevated with HIV­ make this diagnosis less likely. B ecause this patient has no lymphocytes, she will be unresponsive to vaccinations and should not receive live viral vaccines (making option "C" ineffective) . While she will require antimicrobial prophy­ laxis ( option "E") prior to and following her transplant, this patient's overall prognosis is not likely to improve simply with this intervention.

C H A PTER 4





BLOOD AND N EOPLASTIC DISORDERS

1 01

Helpful Tip

� Eosinophilia may be present in patients with severe 1 1 1r combi ned immu nodeficiency (SCI D). The combination

of eosinophilia and lymphopenia in the context of a patient with frequent infections should make you consider SCI D.

A 7-year-old boy is seen in the acute care clinic for epistaxis. He has been having nosebleeds from both nostrils, occur­ ring spontaneously, approximately once per week, for sev­ eral years. The nosebleeds last between 5 and 1 5 minutes and eventually resolve with pressure. The patient was seen in the emergency department last week for epistaxis lasting lon­ ger than 45 minutes, requiring nasal packing to stanch the bleeding. Historically, his mother describes him as bruising easily on his extremities, back, and face. He was circumcised as a neonate and had no unusual bleeding. He has never had bleeding problems associated with vaccinations. He has never appeared to have any joint pain or difficulty walking. There is no family history of individuals with bleeding dis­ orders or bleeding symptoms. The physical exam is entirely normal, apart from several violaceous purpura noted on the lower extremities. Question 32-1 Which of the following would be appropriate as initial screen­ ing tests in evaluation of this patient? A) Bleeding time (BT), prothrombin time (PT), and activated partial thromboplastin time (aPTT) . B) Platelet count, platelet function assay, factor VIII activity, and factor IX activity. C) CBC with white cell differential and platelets, factor VIII activity, and factor IX activity. D) Platelet count, BT, von Willebrand factor (vWF) antigen, and vWF activity. E) Platelet count, PT, aPTT, vWF antigen, and vWF activity. Discussion 32-1 The correct answer is "E:' While there may be substantial varia­ tion in practice in the initial workup for a patient suspected of a bleeding disorder, it is helpful to plan screening based on the most likely etiology of bleeding derived from the history. The patient in the vignette exhibits mucocutaneous bleeding in the form of epistaxis and bruising, common in disorders of primary hemostasis most consistent with platelet-related bleed­ ing or von Willebrand disease (vWD) . Confirming an adequate platelet count is crucial in the initial workup. While the preva­ lence of qualitative platelet function disorders in pediatrics is not precisely known, it is nevertheless rare, and platelet func­ tion testing is not currently considered necessary as a first­ line screening test (making option "B" incorrect) . The most dramatic platelet function defects include Bernard-Soulier

FIGURE 4-10. Macrothrom bocytopenia. This blood smear was taken fro m a patient with Bernard-Sou l i e r synd rome, associated with throm bocyto penia and l a rg e platelet size (arrowheads). Abnormally l a rge p l atel ets may also be seen i n anti body-med iated platelet disorders, such as i m m u n e thro m bocytopenia (ITP), as wel l as i n myeloprol ife rative neoplasms. (Used with permission from Ada m D. Wolfe, M D, P h D.)

syndrome (BSS; caused by deficient platelet glycoprotein Ib/IX, the vWF receptor) and Glanzmann thrombasthenia (caused by deficient platelet glycoprotein lib-Ilia, the fibrinogen receptor) , and are often diagnosed with severe mucocutaneous bleed­ ing in early childhood. Of note, BSS is one of very few inher­ ited conditions that causes macrothrombocytopenia, or low numbers of large platelets, illustrated in Figure 4- 1 0 . The BT, while still available in many centers, has fallen out of favor as a screening test, and is particularly unhelpful in children with suspected bleeding disorders. There is considerable interlabo­ ratory variability, and the BT may be affected by factors inde­ pendent of bleeding risk, such as age, gender, skin temperature, skin thickness, and hematocrit. The test has largely been sup­ planted by other laboratory assessments (making options "!\' and "D" incorrect) . Hemophilia A and B (factor VIII and IX deficiency, respectively) also may cause mucocutaneous bleed­ ing symptoms, although the bleeding with these conditions is most often related to defective secondary hemostasis, such as intramuscular (eg, following vaccination) and intra-articular (ie, joint bleeds and hemophilic arthropathy) . The patient in the vignette had no personal or family history to raise suspi­ cion for hemophilia, although screening with PT and aPTT is typically included in initial evaluation to ensure that there are no factor deficiencies within the initiation (ie, extrinsic, mea­ sured by PT) or propagation (ie, intrinsic, measured by aPTT) pathways of coagulation (making option "C" less appropriate) . Th e most likely diagnosis for the patient i n the vignette i s vWD. Most vWD types are inherited in an autosomal recessive man­ ner, making a family history less frequent in affected individu­ als. Patients with a diagnosis of vWD, or any bleeding disorder, are discouraged from participating in contact sports (such as football, basketball and wrestling) and some limited contact

1 02

MCG RAW- H I LL EDUCATION SPECIALTY BOARD REVI EW: PEDIATRICS

sports (such as baseball, gymnastics and volleyball) , based on provider comfort. They require prompt attention to head injury or concussive symptoms, due to the risk of intracranial hem­ orrhage. Emergency management involves administration of vWF:factor VIII complex, to restore appropriate vWF levels and activity. Type 1 vWD, due to quantitative vWF deficiency, and some subtypes of type 2 vWD, due to qualitative defects in vWF, are responsive to desmopressin (DDAVP) . This hormone stimulates immediate mobilization of vWF stores and can cause transient increase in circulating vWF, but is only effective in patients who can produce vWF at baseline. The advantage of DDAVP is that it may be administered intranasally and is there­ fore easily given at home as a rescue medication. Patients with vWD often undergo DDAVP challenge in the hematology clinic to establish whether they exhibit a response to the medication.

� •

I I

Helpful Tip

Platelet dysfu nction and von Wil lebrand disease (vWD) cause mucocutaneous bleeding (epistaxis, bruising, gingiva). I n addition to mucocutaneous bleeding, factor deficiencies cause bleeding into tissues (joints, muscles).

You are seeing a 1 -month-old baby girl in clinic for a follow­ up visit. She has been growing and developing well, and her parents have no concerns today. The baby's initial newborn hemoglobinopathy screen was flagged as abnormal by the state lab, exhibiting a hemoglobin FS pattern suggestive of sickle cell disease. There is no family history of sickle cell disease, and the parents are unaware of any individuals with sickle cell trait. A repeat screen was sent at 2 weeks, with the same result. You are discussing this finding with the family today, and are in the process of referring the patient to pedi­ atric hematology. Question 33-1 Which of the following statements is most accurate regarding a newborn diagnosed with sickle cell disease? A) The patient should be protected from sickle cell complica­ tions during the first 6 months of life due to the presence of fetal hemoglobin. B) Penicillin prophylaxis will need to be initiated once the spleen has involuted, usually around 5 to 6 years of age. C) Starting hydroxyurea by age 12 months should help to protect the child from developing ischemic strokes during childhood. D) Because of development of functional asplenia, infants with sickle cell disease should receive the 23 -valent pneumococ­ cal polysaccharide vaccine instead of the standard 1 3 -valent pneumococcal conjugate vaccine (PCV- 1 3 ) . E) Parents should be taught to respond promptly t o any febrile episodes with acetaminophen, and follow up with their hematologist within 48 hours of fever.

Discussion 33-1 The correct answer is "A:' A normal newborn hemoglobinopa­ thy screen exhibits hemoglobins F (HbF, comprised of globin subunits a2y) and A (HbA, a2�J The baby in the vignette exhibits hemoglobins F (fetal) and S (HbS, a2�\) . Note that the newborn screen reports hemoglobins in descending order of quantity, and HbF will be most abundant in newborns. The patient's inability to produce HbA indicates that she lacks any normal �-globin production. This pattern fits best with either hemoglobin ss disease, or hemoglobin s - w thalassemia, a heterozygous condition with a similar phenotype. The dis­ tinction between these diagnoses should not affect the initial counseling and management of the patient. Under physi­ ologic stress, HbS polymerizes within red blood cells, caus­ ing a shape change to the classic sickle cell shape illustrated in Figure 4- 1 1 . Stressors include hypoxemia, dehydration, and infection. These abnormally shaped cells may then aggregate and occlude microvasculature, causing tissue hypoxia and endothelial inflammation and damage. The presence of HbF in excess of 30% is protective against this phenomenon, as HbF incorporates into HbS polymers and terminates their exten­ sion. Therefore, infants with sickle cell disease do not usually exhibit complications until at least the latter half of the infant period, or after HbF levels drop below 30%. Children with sickle cell disease are at risk of multiple complications. The greatest early risk comes from infection. Patients with sickle cell disease are considered functionally asplenic, regardless of the presence of a spleen; this can be confirmed by the appear­ ance of Howell-Jolly bodies visible on the blood smear, shown in Figure 4- 1 1 . These patients are at elevated risk of sepsis due to encapsulated organisms. For this reason, penicillin prophy­ laxis is typically initiated by 2 months of age and continued

FIGURE 4-11. Sickle cel l a n e m i a . This blood smear exh i bits m a ny cel l s with cha racteristic sickle sha pe. M u ltiple cel l s a l so exh i bit Howel l-J o l ly bodies, baso p h i l i c i n c l usions of red cel l s representative of rem n a nt DNA (eg, arrowhead). The presence of circulating cel l s with Howel l-Jolly bod ies, which a re norma l l y cleared by the spleen, i n d i cates fu nctional asplenia. (Used with permission from Ada m D. Wolfe, M D, P h D.)

C H A PTER 4

until splenic involution, around 5 to 6 years of age (making option "B" incorrect) . Patients with sickle cell disease should receive all recommended childhood vaccinations, including pneumococcal vaccination with PCV- 1 3 . They also should additionally receive the 2 3 -valent vaccine between ages 2 and 5 (option "E" ) . The risk of bacterial sepsis in these patients remains high, and patients with sickle cell disease who develop fever (:2: 38°C [ 1 00.4°F) must be seen immediately for blood culture and empiric parenteral antibiotic therapy (making option "D" incorrect) . Sickle cell disease also can cause vasa­ occlusive episodes, which can lead to pain, respiratory compli­ cations, renal disease, and stroke, depending on the location. Hydroxyurea stimulates increased production of HbF, and has been demonstrated in large clinical trials to provide effec­ tive prophylaxis against vasa-occlusion, painful episodes, and acute chest syndrome. Unfortunately, hydroxyurea was not shown to be effective prophylaxis against stroke in sickle cell disease (making option "C" incorrect) . Patients with sickle cell disease should undergo screening of middle cerebral artery flow velocities by trans cranial Doppler ultrasound, performed annually between ages 2 and 1 6, and those found to be at risk of stroke are referred for chronic transfusion therapy to reduce stroke risk.

A 1 2-year-old adolescent girl presents to the acute care clinic due to fatigue. She underwent menarche 4 months ago, and has had fairly heavy flow lasting 5 to 7 days every 4 weeks since then. Her mother describes her as always being pale and easily tired, and she has always been at the low end of the weight and height curves for her age. The fatigue has become more severe over these past 4 months. The patient has never had a blood test before. Her mother reports that she was diagnosed with anemia during pregnancy, and required a blood transfusion after childbirth; no other fam­ ily members have known anemia or bleeding problems. The patient overall appears well, is alert and answering ques­ tions. The patient is afebrile. Exam reveals faint scleral icterus, conjunctival and gingival pallor, tachycardia and a grade 1/VI systolic ejection murmur over the left sternal border. There is no lymphadenopathy, organomegaly, or bruising. A CBC reveals white blood cell count of 5.4 x 1 03/ mm\ hemoglobin 7.8 g/dL, and platelets 500 x 1 03/mm3• Red cell indices include mean corpuscular volume (MCV) 54 fL and red cell distribution width 1 5%. Question 34- 1 Which of the following is the most likely diagnosis for this patient? A) von Willebrand disease. B) Beta-thalassemia minor. C) Beta-thalassemia intermedia. D) Beta-thalassemia major. E) Alpha-thalassemia minor.



BLOOD AND N EOPLASTIC DISORDERS

1 03

Discussion 34- 1 The correct answer is "C:' This patient presents with a his­ tory of acute-on-chronic fatigue, and symptoms suggestive of a chronic mild anemia with hemolysis ( eg, pallor, scleral icterus, fatigue, growth delays) . Her symptoms worsened after menarche, suggesting that she was unable to compensate for menstrual blood losses. Her laboratory evaluation reveals a sig­ nificant microcytosis. The best choice from the answers given, of a microcytic anemia that is chronic but may not be diag­ nosed until late childhood or adolescence, is beta-thalassemia intermedia. Thalassemia is a collection of syndromes of vary­ ing severity, which all result from an imbalance in production of a and � globins in maturing cells of the erythrocyte lineage. The imbalance leads to formation of homotetramers of the overabundant globin subunit, and these hemoglobins are often ineffective or unstable, leading to reduced red cell lifespan and resilience. In the case of beta-thalassemia syndromes, one (minor) or both (intermedia, maj or) � globin genes is defec­ tive. Beta-thalassemia minor is not considered a disease, as adequate � globin production is sustained by the normal copy of the gene, and patients will exhibit a lifelong, mild microcytic anemia that does not cause symptoms and should not require transfusion (making option "B" incorrect) . Beta-thalassemia major results from the absence of functional � globin, leaving patients dependent on fetal hemoglobin; these patients become symptomatically anemic usually during the first 1 to 2 years of life and require lifelong blood product support (making option "D" incorrect) . Patients with beta-thalassemia intermedia typi­ cally inherit one null � globin gene, and one gene with a muta­ tion that decreases production of � globin. They therefore have more severe anemia than patients with thalassemia minor, but less than if they had thalassemia major. These patients may not be diagnosed until later childhood, or in the case of this patient, after menarche. Alpha-thalassemia syndromes are caused by inheritance of mutations in one or more of the four a globin genes. Alpha-thalassemia minima, resulting from a single gene deletion, is asymptomatic and has no reliable associated labo­ ratory abnormalities. Alpha-thalassemia minor, resulting from two gene deletions, resembles beta-thalassemia minor clini­ cally, with lifelong mild microcytic anemia that tends not to cause symptoms (making option "E" incorrect) . Hemoglobin H disease results from three deleted a globin genes and is asso­ ciated with more severe anemia, particularly during acute ill­ nesses, and the presence of hemoglobin H ( �4 globin tetramer) on electrophoresis. Alpha-thalassemia major, deletion of all a globin genes, results in inability to produce any hemoglobin F or A, and is not compatible with life unless it is diagnosed in utero and intrauterine blood transfusions are administered. A patient such as the one in the vignette, who presents with anemia after menarche, may also be suspected of having a bleeding disorder; von Willebrand disease (vWD) is the most common of these. In this case, one would expect the patient to have a normocytic anemia, consistent with acute blood loss. If the blood loss were chronic, microcytosis with iron deficiency might be seen with menorrhagia associated with vWD, but the history of lifelong fatigue, pallor, growth issues, and icterus would not be expected (making option "PI' incorrect) .

1 04

MCG RAW- H I LL EDUCATION SPECIALTY BOARD REVI EW: PEDIATRICS

A 4-year-old boy is being evaluated in the urgent care clinic for fever and cough. His mother reports that he has been having frequent febrile illnesses since he was around 6 months of age, with symptoms of ear infections, sinus infections, and at least three diagnosed pneumonias. He "always" seems ill, without any periods of being asymptom­ atic. Mom has been told on two previous occasions that the patient had abnormally low white blood cell counts. The patient was adopted at birth, and the biologic family his­ tory is unknown. On exam, the patient is tired and appears small for age. The oropharynx reveals multiple mucosal ulcerations. Tender cervical nodes are palpable in the ante­ rior and posterior groups bilaterally. Lung exam reveals fine crackles in the upper lobes bilaterally. A CBC includes white blood cells 9.7 x 1 03/mm3 with 2% neutrophils and 9 1 % lymphocytes, hemoglobin 1 2.3 g/dL with normal red cell indices, and platelet count 266 x 1 03/mm3• Chest X-ray reveals consolidation in the right and left upper lobes, con­ sistent with multifocal pneumonia. Question 35-1 Which of the following evaluations is most likely to yield an etiology for this patient's infectious history? A) Bone marrow aspirate and biopsy. B) Vitamin B 12 and folate levels. C) Antineutrophil antibody testing. D) CBC with differential three times weekly for 6 weeks. E) Epstein-Barr virus (EBV) serologic panel. Discussion 35-1 The correct answer is "A:' The patient exhibits findings of chronic neutropenia starting in infancy, with multiple febrile sinopulmo­ nary infections, previous low white cell counts, mucositis, and profound neutropenia (absolute neutrophil count = 1 0, 1 00 x 0.02 = 1 94/mm3). The differential diagnosis of infant and childhood neutropenia is broad, and includes nutritional, medication-asso­ ciated, immune-mediated, and inherited causes. For this patient, severe congenital neutropenia and cyclic neutropenia are strong considerations. In the former case, diagnosis is suggested by maturation arrest at the promyelocyte stage of neutrophil devel­ opment in the bone marrow. In cyclic neutropenia, neutrophils nadir with a periodicity of approximately 2 1 days, yielding a his­ tory of febrile illnesses and other neutropenic symptoms that coincide with the neutropenia. Other cytopenias may also occur on the same schedule. Diagnosis is made by establishing neutro­ phil periodicity by obtaining CBC with differential 3 times per week for at least two cycles (ie, 6 weeks; option "D"). The patient's history does not indicate a periodic symptomatic pattern, making this diagnosis less likely. Although the patient in the vignette may have a nutritional deficiency, as vitamin B 12 and folate deficien­ cies are associated with acquired childhood neutropenia (option "B"), the absence of abnormalities noted in the red blood cells (eg, anemia, macrocytosis) reduces suspicion for this etiology. Antineutrophil antibodies (option "C") are associated with either

congenital neutropenia due to transplacental transfer of maternal antibody, or an autoantibody produced after a childhood infec­ tion. In the former case, severe infections are likely during the first few months oflife, but the neutropenia should typically resolve by 6 months of age as maternal antibody wanes. In the latter case, neutropenia may persist for up to several years but typically is not associated with frequent or severe infections. Viral infections, such as EBV, can be associated with prolonged systemic illness and various acquired cytopenias including neutropenia (option "E") that can last for several months. However, this patient's history of frequent sinopulmonary infections with neutropenia lasting at least 3 years makes convalescent or prolonged EBV infection less likely. Patients with neutrophil counts ofless than 500/mm3 are at greatest risk of bacteremia, and should receive prompt attention to fevers, with blood cultures obtained and empiric treatment with parenteral antibiotics. The patient in the vignette would ben­ efit from intervention with granulocyte colony-stimulating factor (GCSF). This has been shown to increase neutrophil counts in patients with inherited and acquired neutropenias, and to dimin­ ish frequency, duration, and severity of infections. In some cases, patients with refractory neutropenia and recurrent life-threaten­ ing infections may be considered candidates for hematopoietic stem cell transplantation.

A previously healthy, 1 5-year-old adolescent boy presents to the emergency department with cough and dyspnea. He has been noticing progressive difficulty catching his breath for several weeks now and has been unable to sleep on his back for the past week because of the sensation of pressure in his chest. He has not had fever or acute upper respiratory symptoms. Vital signs on presentation include temperature 37.5°C (99.5°F) , heart rate 1 1 0 beats per minute, and respira­ tory rate 28 breaths per minute. The patient is awake, alert, and answering questions appropriately. You suspect that this patient has a mediastinal mass. Question 36-1 Which of the following findings on your initial evaluation is most likely to warrant treatment before a biopsy can be performed? A) White blood cell count 60 x 1 03/mm3 with 64% blasts. B) Spleen tip palpable 8 em below the left costal margin. C) Edema in the face and neck. D) Palpable cervical, supraclavicular, axillary, and inguinal lymphadenopathy. E) Serum alpha-fetoprotein (AFP) 3600 ng/dL (reference range < 10 ng/dL) . Discussion 36-1 The correct answer is "C' This patient presents with indolent progression of dyspnea and orthopnea consistent with a medi­ astinal mass. A chest X-ray will be helpful in confirming this suspicion. Causes of mediastinal masses in children include

C H A PTER 4

malignancy, infection, and venous thrombosis. Mass effect within the mediastinum can lead to the superior vena cava (SVC) syndrome. Symptoms relate to obstruction of venous drainage through the SVC and include those in the vignette; there can also be associated dysphagia and hoarseness. Physi­ cal exam findings in patients with SVC syndrome may include edema of the head, neck, and upper extremities with or with­ out cyanosis, wheezing, and stridor. In the latter case, tracheal compression should be suspected. SVC syndrome is an emer­ gency that may require intubation to secure the airway. Patients may decompensate if sedated or placed supine for a prolonged period (eg, for advanced imaging or diagnostic procedures) . Patients with progressive symptoms, which can lead t o cen­ tral neurologic findings of headache, confusion, lethargy, and visual disturbance, may require therapy before the cause of the SVC syndrome is identified. Emergency management may include radiation therapy, chemotherapy, or anti-inflammatory medication such as corticosteroids. The differential diagnosis of mediastinal mass in pediatric oncology includes leukemia, lymphoma, neuroblastoma, germ cell tumor, and sarcoma. The hematologic malignancies are suggested by findings including leukocytosis and circulating blasts (option "A"), splenomegaly (option "B"), and lymphadenopathy (option "D") . Elevated tumor markers including AFP (option "E") and beta-human chorionic gonadotropin would raise suspicion for germ cell tumor. If neuroblastoma is favored, urine catecholamines vanil­ lylmandelic acid (VMA) and homovanillic acid (HVA) may be obtained. However, none of these suspicious findings on initial evaluation constitutes an immediate life-threatening state that would merit treatment prior to obtaining a tissue diagnosis.

i QUICKQUIZ Which is NOT an oncologic emergency? A) Tumor lysis syndrome. B) Spinal cord compression. C) Hyperleukocytosis. D) Typhlitis. E) All of the above. Discussion The correct answer is "E:' Hyperleukocytosis causes hyperviscos­ ity and leukostasis affecting the lungs, gastrointestinal tract, and central nervous system. In typhlitis or neutropenic enterocolitis, the bowel wall becomes inflamed in the setting of prolonged neu­ tropenia and may perforate. Additional emergencies include SVC syndrome, increased intracranial pressure, and stroke.

A 1 6-year-old adolescent boy has recently completed 39 months of chemotherapy for T-cell acute lymphoblas­ tic leukemia (ALL) . He has been in remission since the first month of induction therapy and currently feels well.



BLOOD AND N EOPLASTIC DISORDERS

1 OS

His mother notes that the family was advised that be life­ long follow up will be required, due to increased risk of complications from chemotherapy. Question 37- 1 Which o f the following i s a risk that i s likely t o decrease over the first year following completion of chemotherapy? A) Infertility due to alkylating agents. B) Development of cardiomyopathy secondary to anthracy­ cline exposure. C) Leukemia secondary to alkylating agents. D) Immune system dysfunction secondary to myelosuppres­ sive agents. E) Leukemic relapse in the central nervous system (CNS) and testes. Discussion 37- 1 The correct answer is "D:' Treatment with chemotherapy carries numerous short-term and long-term risks. Each class of agents is associated with specific toxicities that must be acknowledged with families during informed consent discussions surround­ ing cancer therapy. It is crucial to understand that many of the risks associated with chemotherapeutic agents persist indefi­ nitely and require lifelong follow up with a specialist or primary care physician knowledgeable about cancer survivorship. Of the choices presented, the risk most likely to decrease shortly after completion of therapy is immune dysfunction. Some immune suppression is seen with chemotherapy that directly suppresses leukocyte production, which is a potential effect of nearly all chemotherapeutic drugs with the exception of vinca alkaloids (inhibitors of cell division). Other immune suppressive effects are due to the alteration of lymphocyte maturation induced by chemotherapy. Restoration of normal white cell function is usually assumed by approximately 6 months after cessation of chemotherapy. Alkylating agents such as cyclophosphamide are well known to be associated with future infertility, particularly when these agents are administered to adolescents (option "A"). The exact incidence of infertility associated with chemotherapy is unknown, but the risk for this is lifelong. Prior to initiation of chemotherapy, patients are expected to receive counseling regarding this risk, and efforts at fertility preservation should be offered. Anthracycline antibiotics ( eg, doxorubicin and dauno­ rubicin) are known to preferentially enter cardiomyocytes and cause oxidative damage. Evidence of anthracycline-associated cardiomyopathy can be identified years, or decades, after expo­ sure (option "C") . Survivorship follow up includes echocardiog­ raphy performed every 1 to 5 years, depending on cumulative drug exposure, as recommended by the Children's Oncology Group. An ironic and difficult-to-face complication of chemo­ therapy is the increased risk of secondary malignancy, most often associated with alkylating agents and topoisomerase inhibi­ tors (eg, etoposide) . The most frequently encountered malignan­ cies are hematologic, including myelodysplastic syndrome and acute myeloid leukemia, although secondary solid tumors may also be seen. Radiation therapy also increases the risk of devel­ oping hematopoietic and solid malignancies. This risk persists lifelong. The other cancer-related risk to survivors following

1 06

MCG RAW- H I LL EDUCATION SPECIALTY BOARD REVI EW: PEDIATRICS

chemotherapy is relapse (option "E"). Close follow up for disease recurrence is therefore essential for any patient who has com­ pleted treatment for childhood malignancy. Leukemic relapse, which is most fearsome if it occurs shortly after completion of therapy but may occur many years later, may occur in the bone marrow, CNS, and testes. The possibility of testicular relapse is the reason that boys with leukemia receive maintenance therapy for approximately 12 months longer than girls (ie, approximately 27 months total duration for girls, 39 months for boys).

A 2-year-old boy is admitted to the nephrology service on Tuesday for acute onset of hematuria, oliguria, and hyperten­ sion. On Monday, the patient was picked up by his mother from his father's home (they share custody of the patient) . Mom was informed that the patient had fevers of37.8 to 38.3°C ( 1 00. 1 - 1 0 1 °F) throughout the weekend, for which he received antipyretics that provided relief. By Tuesday morning, the day of admission, the mother noticed a cola-colored tint to the urine, and overall decreased wet diapers that made her con­ cerned. The working diagnosis is postinfectious glomerulo­ nephritis. Blood work on admission revealed white blood cell count 7.4 x 1 03/mm3 with a normal differential, hemoglobin 1 1 . 1 gldL, platelets 194 x 103/mm3, creatinine 2.8 mg/dL, and blood urea nitrogen (BUN) 70 mg!dL. No abnormal red cell forms were noted on manual smear evaluation. The patient is scheduled for renal biopsy on Thursday morning. A preproce­ dure platelet function assay (PFA) is performed for screening purposes on Tuesday, revealing a closure time of greater than 300 seconds (reference range 60- 1 80 s) in the initial screen. Question 38- 1 Which o f the following i s the most appropriate next step for this patient? A) Proceed with biopsy as planned, transfuse 1 unit of platelets prior to procedure. B) Repeat the PFA on Wednesday. C) Perform plasmapheresis for suspected hemolytic uremic syndrome (HUS) . D) Perform bone marrow aspiration and biopsy. E) Obtain testing for von Willebrand factor (vWF) antigen and activity. Discussion 38-1 The correct answer is "B:' This patient has renal disease that merits further evaluation and was incidentally discovered to have a prolonged platelet closure time on PFA. This screen may be performed in the setting of renal disease prior to a surgical procedure, as elevated BUN is a known cause of acquired quali­ tative platelet dysfunction. However, most patients with BUN of less than 1 00 mg/dL are unlikely to exhibit clinically significant platelet dysfunction. The most likely etiology for the patient's abnormal PFA is drug-induced platelet dysfunction, given the

history of recent antipyretic use, likely ibuprofen. Nonsteroidal anti-inflammatory drugs that inhibit cyclooxygenase- 1 enzyme interfere with platelet production of thromboxane A2 (TXA) from arachidonic acid, and the diminished TXA2 reduces the ability of platelets to activate. This effect caused by ibuprofen is reversible upon withdrawal of the drug, and the alteration of platelet function will largely resolve within less than 72 hours of the most recent dose. In the vignette case, before major changes in the patient's treatment plan are contemplated, it would appro­ priate to repeat the PFA and observe for improvement. If the patient were to have confirmed non-medication-related plate­ let dysfunction, transfusion with healthy platelets (option "!\') periprocedurally would reduce the likelihood of bleeding. The likelihood of HUS (option "C") in this case is low, as the hemo­ globin and platelet counts are normal, and no schistocytes were observed on the blood smear. Further, prior to plasmapher­ esis for HUS, therapy with eculizumab (monoclonal antibody that inhibits terminal complement pathway activation) might be attempted. As no abnormalities were noted on blood count, bone marrow assessment (option "D") is not warranted. While von Willebrand disease (option "E") often also causes prolonga­ tion of the PFA, no other data were provided in the case descrip­ tion to suggest a bleeding disorder, and an acquired platelet problem is more likely.

During rounds on a 22-hour-old term newborn boy in the nursery one morning, you notice jaundice to the lower abdo­ men that was not reported at birth. The chart indicates that the mother is G P 1 with one spontaneous abortion prior to 2 this pregnancy, took no medications during pregnancy, and received no prenatal care. She reports a history of requiring several blood transfusions following trauma in a motor vehi­ cle accident 3 years ago. Her blood type is 0, Rh negative. The baby's father has chronic anemia with mild jaundice, and required several blood transfusions as a child. The baby's vital signs are all normal. The physical exam is unremarkable apart from the jaundice. CBC on the baby reveals hemoglo­ bin 12.8 g/dL, mean corpuscular volume (MCV) 105 fL, and all other values are normal. Total bilirubin is 8.4 mg/dL. The baby is blood type A, Rh negative. Question 39-1 Which of the following etiologies best explains this baby's jaundice? A) ABO incompatibility. B) G6PD deficiency. C) Rh incompatibility. D) Beta-thalassemia major. E) Folate deficiency. Discussion 39- 1 Th e correct answer i s "A:' This patient presents within 2 4 hours of life with hyperbilirubinemia and asymptomatic, mild anemia

C H A PTER 4

for age, suggestive of a mild hemolytic condition. The mother has a history of a previous pregnancy and previous blood trans­ fusion in an emergency setting, and may have become sensi­ tized to blood group antigens from these exposures. Infants with A blood group born to mothers with 0 blood group and history of exposure to A antigen from previous pregnancy or transfusion are at highest risk of alloimmune hemolysis, called hemolytic disease of the fetus and newborn (HDFN) . It is important to note that unexposed individuals make IgM type antibodies against mismatched major blood group antigens (ie, this mother likely has circulating anti-A and anti-B IgM), but these do not cross the placenta. Prior exposure to the mis­ matched blood group, however, stimulates IgG production, and these antibodies are able to cross the placenta and lead to HDFN in subsequent pregnancies. Rh incompatibility (option "C") yields an anti-D IgG, which also causes HDFN, often with greater severity than ABO mismatch, but the mother and infant in this case are both Rh-negative. Most cases of ABO mismatch-associated HDFN do not cause problematic anemia, and hyperbilirubinemia, when present, may be treated with observation or phototherapy alone. How­ ever, some affected infants, mainly those with Rh-associated HDFN, exhibit severe symptoms with significant anemia and a greater degree of hyperbilirubinemia necessitating exchange transfusion. This condition may be diagnosed on prenatal ultra­ sound with evidence of hydrops, subcutaneous edema, and third space effusions in the abdominal, pericardia!, and pleu­ ral spaces. G6PD deficiency (assayed by option "B") may be the diagnosis for this patient's father, based on his history provided, and this can be associated with neonatal hemolysis. However, G6PD is inherited in an X-linked recessive fashion, and the patient would not have inherited it from his paternal genome. Beta-thalassemia major (option "D") is associated with chronic hemolysis and jaundice, but since newborns have relatively little � globin production, relying mainly on y globin present in fetal hemoglobin, they are not expected to exhibit substantial symp­ toms for several months. Folate deficiency (option "E") causes a nonhemolytic macrocytic anemia. Note that this patient's MCV is within the normal range for newborns, which is higher than older children due to the predominance of fetal hemoglobin. Further, newborns generally have adequate folate stores to last at least several months after birth.

B I B LIOGRAPHY

Albisetti M, Monagle P. Bleeding disorders. In: de Alarcon PE, Werner EJ, Christensen RD, eds. Neonatal Hematology: Pathogenesis, Diagnosis, and Management of Hematologic Problems. New York, NY: Cambridge University Press; 20 1 3 :286-30 1 . Allen CE, Kelly KM, Bollard CM. Pediatric lymphomas and histiocytic disorders of childhood. Pediatr Clin North Am. 20 1 5;62 ( 1 ) : 1 39- 1 65. Alter BP, D'Andrea AD. Inherited bone marrow failure syn­ dromes. In: Handin RI, Lux SE, Stossel TP, eds. Blood: Principles and Practice of Hematology. Philadelphia, PA: Lippincott, Williams and Wilkins; 2003:209-272.



BLOOD AND N EOPLASTIC DISORDERS

1 07

Anderst JD, Carpenter SL, Abshire TC; Section on Hema­ tology/Oncology and Committee on Child Abuse and Neglect of the American Academy of Pediatrics. Evaluation for bleeding disorders in suspected child abuse. Pediatrics. 20 1 3; 1 3 1 (4):e 1 3 14- 1 322. Armenian SH, Robison LL. Childhood cancer survivorship: An update on evolving paradigms for understanding pathogenesis and screening for therapy-related late effects. Curr Opin Pediatr. 2 0 1 3;25 ( 1 ) : 1 6-22. Baker RD, Greer FR; Committee on Nutrition, American Academy of Pediatrics. Diagnosis and prevention of iron deficiency and iron-deficiency anemia in infants and young children (0-3 years of age). Pediatrics. 20 1 0; 1 26(5): 1 040- 1 050. Boscainos PJ, Cousins GR, Kulshreshtha R, et al. Osteoid osteoma. Orthopedics. 2 0 1 3 ; 3 6 ( 1 0 ) : 792-800. Branchford BR, Monahan PE, Di Paola J. New developments in the treatment of pediatric hemophilia and bleeding disorders. Curr Opin Pediatr. 20 1 3;25 ( 1 ) :23-30. Cairo MS, Coiffier B, Reiter A, Younes A; TLS Expert Panel. Recommendations for the evaluation of risk and pro­ phylaxis of tumour lysis syndrome (TLS) in adults and children with malignant diseases: An expert TLS panel consensus. Br f Haematol. 20 1 0; 1 49(4) :578-586. Cappellini MD, Fiorelli G. Glucose-6-phosphate dehydroge­ nase deficiency. Lancet. 2008;371 (9606):64-74. Chalmers E, Cooper P, Forman K, et al. Purpura fulminans: recognition, diagnosis and management. Arch Dis Child. 20 1 1 ;96( 1 1 ) : 1 066- 1 07 1 . Chirnomas SD, Kupfer GM. The inherited bone mar­ row failure syndromes. Pediatr Clin North Am. 2 0 1 3;60(6) : 1 29 1 - 1 3 1 0 . Christensen RD. Reference ranges i n neonatal hematology. In: de Alarcon PE, Werner EJ, Christensen RD, eds. Neonatal Hematology: Pathogenesis, Diagnosis, and Management of Hematologic Problems. New York, NY: Cambridge University Press; 20 1 3 : 385-408. Committee on Sports Medicine and Fitness. Medical con­ ditions affecting sports participation. Pediatrics. 200 1 ; 1 07(5): 1205- 1 209. Da Costa L, Galimand J, Fenneteau 0, Mohandas N. Heredi­ tary spherocytosis, elliptocytosis, and other red cell mem­ brane disorders. Blood Rev. 2 0 1 3;27(4) : 1 67- 1 78. de Alarcon P E , Fernandez KS . Congenital thrombocytopenias and thrombocytopathies. In: de Alarcon PE, Werner EJ, Christensen RD, eds. Neonatal Hematology: Patho­ genesis, Diagnosis, and Management of Hematologic Problems. New York, NY: Cambridge University Press; 20 1 3 : 1 72-207. De Bernardi B, Balwierz W, Bej ent J, et al. Epidural compres­ sion in neuroblastoma: Diagnostic and therapeutic aspects. Cancer Lett. 2005;228 ( 1 -2):283-99. Demellawy DE, Young JL, Nanassy J, Chernetsova E, Nasr A. Langerhans cell histiocytosis: A comprehensive review. Pathology. 2 0 1 5;47(4) :294-30 1 . Dimaras H , Kimani K , Dimba EAO, et al. Retinoblastoma. Lancet. 2012;379(9824) : 1 436- 1446. Fisher MJ, Rheingold SR. Oncologic emergencies. In: Pizzo PA, Poplack DG, eds. Principles and Practice of Pediatric

1 08

MCG RAW- H I LL EDUCATION SPECIALTY BOARD REVI EW: PEDIATRICS

Oncology. Philadelphia, PA: Lippincott Williams and Wilkins; 20 1 1 : 1 125- 1 1 5 1 . Fleisher T. Primary immune deficiencies: Windows into the immune system. Pediatr Rev. 2006;27 ( 1 0 ) :363-372. Friedman AD. Wilms tumor. Pediatr Rev. 20 1 3 ;34(7) :328-330. Gajjar AJ, Robinson GW. Medulloblastoma-translating discov­ eries from the bench to the bedside. Nat Rev Clin Oneal. 20 14; 1 1 ( 1 2 ) : 7 14-722. Harrison P, Mackie I, Mumford A, Briggs C, et al; British Com­ mittee for Standards in Haematology. Guidelines for the laboratory investigation of heritable disorders of platelet function. Br J Haematol. 20 1 1 ; 1 5 5 ( 1 ) :30-44. Islam MS, Anoop P. Current concepts in the management of stroke in children with sickle cell disease. Childs Nerv Syst. 20 1 1 ;27(7) : 1 037- 1 043. Kett JC. Anemia in infancy. Pediatr Rev. 20 1 2;33(4) : 1 86- 1 87. Matthews DC. Inherited disorders of platelet function. Pediatr Clin North Am. 20 1 3;60(6): 1 475- 1 488. Miller ST. How I treat acute chest syndrome in children with sickle cell disease. Blood. 20 1 1 ; 1 1 7(20) : 5297 -5305. Mughal TI, Ejaz AA, Foringer JR, Coiffier B. An integrated clinical approach for the identification, prevention, and treatment of tumor lysis syndrome. Cancer Treat Rev. 20 1 0;36(2) : 1 64- 1 76. Neunert C, Lim W, Crowther M, et al; American Society of Hematology. The American Society of Hematology 20 1 1 evidence-based practice guideline for immune thrombo­ cytopenia. Blood. 20 1 1 ; 1 1 7 ( 1 6):41 90-4207. Newburger PE, Dale DC. Evaluation and management of patients with isolated neutropenia. Semin Hematol. 20 1 3;50(3 ): 1 98-206. Parker Rl. Transfusion in critically ill children: Indications, risks, and challenges. Crit Care Med. 2014;42(3) :675-690. Powers JM, McCavit TL, Buchanan GR. Management of iron deficiency anemia: A survey of pediatric hematology/oncology specialists. Pediatr Blood Cancer. 20 1 5;62 (5) :842-846. Price VE, Ledingham DL, Krumpel A, Chan AK. Diagnosis and management of neonatal purpura fulminans. Semin Fetal Neonatal Med. 20 1 1 ; 1 6(6) : 3 1 8-322.

Reverdiau-Moalic P, Delahousse B, Body G, et al. Evolution of blood coagulation activators and inhibitors in the healthy human fetus. Blood. 1 996;88:900-906. Rund D, Rachmilewitz E. Beta-thalassemia. N Engl J Med. 2005;353( 1 1 ) : 1 1 3 5 - 1 1 46. Scheinberg P, Young NS. How I treat acquired aplastic anemia. Blood. 20 1 2 ; 1 20(6) : 1 1 85- 1 1 96. Shields CL, Shields JA. Retinoblastoma management: Advances in enucleation, intravenous chemoreduction, and intra-arterial chemotherapy. Curr Opin Ophthalmol. 20 1 0;2 1 (3) :203-2 1 2 . Teplick A , Kowalski M, Biegel JA, Nichols KE. Educational paper: Screening in cancer predisposition syndromes: guidelines for the general pediatrician. Bur J Pediatr. 20 1 1 ; 1 70(3) :285-294. Thornburg CD, Files BA, Luo Z, et al. ; BABY HUG Investi­ gators, Impact of hydroxyurea on clinical events in the BABY HUG trial. Blood. 20 1 2 ; 1 20(22) :4304-43 10; quiz 4448. Vagace JM, Bajo R, Gervasini G. Diagnostic and therapeutic challenges of primary autoimmune haemolytic anaemia in children. Arch Dis Child. 2014;99(7):668-673. van der Spek J, Groenwold RH, van der Burg M, van Montfrans JM. TREC based newborn screening for severe combined immunodeficiency disease: A systematic review. J Clin Immunol. 2 0 1 5;35( 4):41 6-430. Vlychou M, Athanasou NA. Radiological and pathological diagnosis of paediatric bone tumours and tumour-like lesions. Pathology. 2008;40(2) : 1 96-2 16 . Warkentin T E . Heparin-induced thrombocytopenia in critically ill patients. Semin Thromb Hemost. 20 1 5;4 1 ( 1 ) :49-60. Webb J, Kwiatkowski JL. Stroke in patients with sickle cell disease. Expert Rev Hematol. 2 0 1 3;6(3):30 1 - 3 1 6 . Yu AL, Gilman AL, Ozkaynak MF, e t al; Children's Oncol­ ogy Group. Anti-GD2 antibody with GM -CSF, interleu­ kin-2, and isotretinoin for neuroblastoma. N Engl J Med. 20 1 0;363( 14): 1 324- 1 334.

5

Cardiology Benton N g

the incidence of congenital heart disease, which is commonly reported to be around 1 % . A 3-year-old boy comes for his yearly well-child check. His mother's only concern is that he is a picky eater. He has been tracking on his growth curve for both height and weight, and is developmentally appropriate. On exam, he has a grade II/ VI systolic murmur that you have not previously heard. Question 1 - 1 What is the next best step in evaluation? A) Additional physical exam maneuvers. B) Electrocardiogram (ECG) . C) Echocardiogram. D) Chest X-ray. E) Both B and D. Discussion 1 - 1 The correct answer is ''A:' Innocent murmurs are common in children and most often present in the toddler years. These mur­ murs are never louder than grade III/VI and rarely radiate. They are louder when the child is supine and softer when standing, but usually do not completely disappear with standing. These mur­ murs also come and go, and are more prominent during times of increased cardiac output, such as during a febrile illness. Question 1 -2 What is the closest estimate for the percentage of children who have an innocent murmur? A) 1 5%. B) 30%. C) 50%. D) 65%. E) 80%. Discussion 1 -2 The correct answer is "E:' The majority of children have an innocent murmur at some time. Some studies report that 85% of children have an innocent murmur. This is in contrast to

Question 1 -3 Which of the following is NOT a characteristic of an innocent murmur? A) Systolic murmur. B) Diastolic murmur. C) Continuous murmur. D) Palpable thrill. E) Both A and D. F) Both B and D. G) Options B, C, and D. Discussion 1 -3 The correct answer is "F:' An innocent murmur does not produce a palpable thrill (grade IV or higher) . Solely diastolic murmurs are never innocent. The majority of innocent mur­ murs are systolic murmurs. The venous hum is the one innocent murmur that is a continuous murmur. •

Helpful Tip

� In addition to being the only continuous murmur,

1 1 1r the venous hum is also the one in nocent murmur that is louder while sitting than supine, contrary to other innocent murmurs. It is most easily heard on the low anterior part of the neck, just lateral to the sternocleidomastoid. Turning the patient's head toward the side of the murmur can diminish or elimi nate the murmur.

A 1 6-year-old adolescent boy comes to you for his prepartici­ pation physical. He has not been seen by you in 3 years. There is a family history of coronary artery disease, including his 1 09

110

MCG RAW- H I LL EDUCATION SPECIALTY BOARD REVI EW: PEDIATRICS

grandfather, who suffered a myocardial infarction at the age of 55. His mother has both hypertension and hyperlipidemia. His weight is at the 95th percentile, his height is at the 75th percentile, his body mass index (BMI) is at the 93rd percen­ tile, heart rate is 63 beats per minute (bpm), and blood pres­ sure is 142/88 mm Hg. His physical exam is otherwise normal. Question 2-1 What is his diagnosis? A) Normal blood pressure. B) Prehypertension. C) Stage I hypertension. D) Stage II hypertension. E) None of the above. Discussion 2-1 The correct answer is "E:' Hypertensive children are more likely to become hypertensive adults. Blood pressure percentiles depend on the child's age, sex, and height. Prehypertension or high normal blood pressure is defined as a blood pressure in the 90th to less than 95th percentile. Stage I hypertension is a blood pressure in the 95th to 99th percentile plus 5 mm Hg. Stage II hypertension is any blood pressure greater than the 99th percentile plus 5 mm Hg. In this case, the patient's blood pressure falls between the 95th and 99th percentiles. But in order for the diagnosis of stage I hyperten­ sion to be made, this should be measured on at least three separate occasions. He does not need to be restricted from activities. Question 2-2 He returns for three more follow ups and his blood pressure measurements are repeatedly in the 90th to less than 95th percentile. What is the most appropriate next step? A) Therapeutic lifestyle changes. B) Urinalysis. C) Echocardiogram. D) CBC. E) Electrolytes, glucose, blood urea nitrogen (BUN), creatinine. Discussion 2-2 The correct answer is "A:' Therapeutic lifestyle changes are appropriate for patients who have prehypertension. Counseling should be done with the aim of gradual weight loss. Moderate exercise of 30 to 40 minutes per day, 3 to 5 days per week, is recommended. The goal should be to expend at least 200 kcal! day. Dietary changes are also recommended to decrease intake of salt, sugar, and fat, with 25% to 35% of total calories from fat, 50% to 60% of calories from carbohydrates, and 1 5 % of calories from protein. In general, total caloric intake should be adjusted to prevent further weight gain and help achieve a healthy weight. Over the next 2 years, his blood pressure measurements are consistently in the 95th to 99th percentile. Question 2-3 What is the most appropriate next diagnostic step? A) Serum electrolytes, glucose, BUN, and creatinine. B) Urinalysis.

C) D) E) F)

Echocardiogram. CBC. Renal ultrasound. All of the above.

Discussion 2-3 The correct answer is "E' Workup is recommended in patients with stage I hypertension for further evaluation of the etiol­ ogy of hypertension and for signs of end-organ damage. This consists of a CBC, electrolytes, renal function, urinalysis, renal ultrasound, echocardiogram, and fasting lipids. The majority of hypertension in adolescents is essential, with the majority of essential hypertension being related to obesity. In the cases of secondary hypertension, the most common etiology is renal. An echocardiogram is done as a screening test to evaluate for left ventricular hypertrophy. This can be an indication for initi­ ating antihypertensive treatment.

A 1 3-year-old adolescent girl follows up with you after being seen in the emergency department the previous afternoon for chest pain. She began having the pain while running in track practice the day before. She localizes the pain to the middle of the chest and denies radiation. She also denies any palpita­ tions or presyncopal feelings. She does note feeling short of breath. The pain lasted 5 to 10 minutes and resolved when she stopped running. Question 3-1 What further finding would be most reassuring for a noncar­ diac cause of chest pain? A) Pleuritic chest pain. B) Pain with palpation of her chest wall on exam. C) Systolic murmur on exam. D) Normal ECG. E) History of Kawasaki disease. Discussion 3-1 The correct answer is "B:' Chest pain in children is noncardiac in origin over 98% of the time. Musculoskeletal, gastrointesti­ nal, and pulmonary etiologies are common. Pain with palpation over the costochondral junction on physical exam is consistent with costochondritis. This pain is often self-limited and may be helped by nonsteroidal anti-inflammatory drugs (NSAIDs) . Classically, this i s characterized by a sharp pain, although description of chest pain can often be unreliable in the pediatric setting. Pleuritic chest pain is often reassuring as well, although this may be the presenting symptom of pericarditis. In the case of pericarditis, the pain is worse with deep breathing but is relieved by leaning forward. The pain is also usually referred to the scapular ridge. There are also classic ECG changes seen with pericarditis (diffuse ST-segment elevation for acute peri­ carditis) . A harsh systolic murmur, especially heard with a sys­ tolic click, would be consistent with aortic stenosis. With more

CHAPTER

severe stenosis, this could lead to decreased oxygen delivery in the face of increased demand during exercise, resulting in exertional chest pain. Although an abnormal ECG would help with the diagnosis of pericarditis, myocarditis, and anomalous left coronary artery from the pulmonary artery (ALCAPA), a normal ECG does not completely rule out the possibility of a cardiac etiology of the chest pain. For example, other types of anomalous coronaries, such as an anomalous left coronary from the right coronary cusp, have a normal resting ECG. A history of Kawasaki's disease increases a patient's risk for having coro­ nary artery involvement. If patients have never had coronary artery involvement as part of their Kawasaki disease, then this is less likely to be related. But patients who have coronary artery aneurysms as a result of Kawasaki disease are at increased risk for thrombosis or stenosis of their coronary arteries. Question 3-2 What is the order ofECG findings you would expect to see over the first 1 to 2 months after diagnosis of acute pericarditis?

hL (1 )

A) B) C) D) E)

(2)

(3)

1 ,2,3. 2, 1 , 3 . 2,3, 1 . 1 , 3 ,2. 3,2, 1 .

Discussion 3-2 The correct answer is "C:' ECG findings seen over time with acute pericarditis have three stages before normalization. Ini­ tially there is subepicardial myocardial damage that results in diffuse ST-segment elevation. This is most prominent in leads representing the left ventricle. Within 2 to 3 days, the ST seg­ ments return to normal. Two to 4 weeks later, the T waves become inverted. This may persist for up to 1 to 2 months before complete normalization. •

Helpful Tip

:5.� An

echocardiogram can be useful when pericarditis

i1 1r is suspected to rule out a large pericardia! effusion

or tamponade physiology, which would require pericardiocentesis. Question 3-3 All of the following are risk factors for acute pericarditis EXCEPT: A) Recent cardiac surgery. B) Recent respiratory infection. C) Renal failure.

5



CARDIOLOGY

111

D) Liver failure. E) Malignancy. F) Autoimmune disease. Discussion 3-3 The correct answer is "D:' Up to 90% of cases of acute pericar­ ditis are infectious or idiopathic in etiology. Causative viruses include coxsackie B, echovirus, adenovirus, influenza A and B, enterovirus, mumps, Epstein-Barr virus (EBV) , and many oth­ ers. Coxsackievirus is the most classic cause of acute pericar­ ditis in children. Pneumococcus and tuberculosis are the most common bacterial causes. Aspirin or NSAIDs are first-line treatments and are effective. In other cases of acute pericarditis, treatment of the underlying cause is necessary.

A 1 3-year-old girl who experienced an episode of syncope is being seen for follow up. She participates in band and had an episode during practice. She recalls feeling hot and as if her vision was starting to black out. The next thing she remem­ bers is waking up on the floor. She is unsure of how much time passed. She felt a bit dazed when she woke up, but recovered. Question 4- 1 Which of the following are most likely to be abnormal? A) ECG. B) Echocardiogram. C) Electroencephalogram (EEG) . D) Resting heart rate. E) Blood pressure. F) None of the above. Discussion 4- 1 The correct answer is "F:' The girl's symptoms are most con­ sistent with neurally mediated syncope (vasovagal syncope) . Workup of these patients is often unremarkable. In contrast, cardiac syncope classically has an abrupt onset without pre­ syncopal symptoms (tunnel vision, dizziness, lightheadedness). This patient's history is classic for neurally mediated syncope, which occurs when someone has been standing for a prolonged period of time, such as during band practice or at church. One theory is that prolonged standing results in exaggerated venous pooling and temporary decreased cerebral perfusion leading to syncope. Neurally mediated syncope also occurs after strong emotional or situational triggers. Question 4-2 Which finding is the most likely to be abnormal for a cardiac cause of syncope? A) Physical exam. B) Echocardiogram. C) Family history. D) Resting heart rate. E) Blood pressure.

112

MCG RAW- H I LL EDUCATION SPECIALTY BOARD REVI EW: PEDIATRICS

Discussion 4-2 The correct answer is "C:' Often the most suspicious piece of information for a cardiac cause of syncope when working up a patient is family history. An ECG is often part of the initial workup, but a normal ECG does not completely rule out a car­ diac cause. Cardiac causes for syncope include hypertrophic cardiomyopathy, long QT syndrome, anomalous coronary arteries, arrhythmogenic right ventricular cardiomyopathy (ARVC), Brugada syndrome, and catecholaminergic polymor­ phic ventricular tachycardia (CPVT) . Whereas the ECG is abnormal in long QT syndrome, Brugada syndrome, and many cases of hypertrophic cardiomyopathy, it can also be normal in some cases of hypertrophic cardiomyopathy, ARVC, CPVT, and with abnormal coronary arteries. The echocardiogram is abnormal with anomalous origins of the coronary arteries, if they are well visualized on the study. Hypertrophic cardiomy­ opathy should also demonstrate an abnormal echocardiogram, but it can be normal in genotype-positive patients, especially if obtained early in life before development of significant hyper­ trophy. Echocardiographic criteria exist for the diagnosis of ARVC but these are insufficient to make the diagnosis alone. The echocardiogram is normal in cases of long QT syndrome, Brugada syndrome, and CPVT. Family history of sudden death is not positive in the index case, but if present is often the most concerning part of the history that triggers further workup and referral. Question 4-3 Which of the following is the most common cause of sudden cardiac death in the young? A) Long QT syndrome. B) Hypertrophic cardiomyopathy. C) Anomalous coronary arteries. D) Mitral valve prolapse. E) Commotio cordis. Discussion 4-3 The correct answer is "B:' Hypertrophic cardiomyopathy is by far the most likely cause of sudden death in young athletes (up to 40% of cardiovascular causes) . Hypertrophic cardiomyopa­ thy is an autosomal dominantly inherited disease present in 1 in 500 people. There is a great deal of phenotypic heterogeneity, so an identified mutation does not correlate with any specific risk factors for sudden death. A murmur may or may not be pres­ ent at rest. Having the patient perform the Valsalva maneuver (bear down) will decrease the intensity of most systolic mur­ murs. In contrast, a murmur caused by obstruction secondary to hypertrophic cardiomyopathy will increase in intensity with the Valsalva.

A 2-month-old previously healthy boy presents to your clinic for a well-child visit. His parents have no major concerns. He is afebrile, his heart rate is 1 68 bpm, and his respiratory rate

is 68 breaths per minute. He has gained 7 g/day since you saw him last at 1 month. On exam, he has a grade II/VI systolic ejection murmur and his liver is palpated 3 em below the cos­ tal margin. You obtain a chest X-ray. The heart looks like a snowman. He does not appear cyanotic, but you ask the nurse to check his oxygen saturation (SpoJ Question 5-1 What is the most likely Spo2 you will obtain? A) 1 00%. B) 89%. C) 72% . D) 6 5% . E) 40%. Discussion 5-1 The correct answer is "B:' The child you see is showing clini­ cal signs of volume overload heart failure. He has had poor weight gain and is mildly tachycardic as well as tachypneic. He also has hepatomegaly on exam as well as cardiomegaly by chest X-ray. No significant pulmonary edema is noted on the chest X-ray. Two months of age is common timing for a left­ to-right shunt to appear clinically. After birth, pulmonary vas­ cular resistance drops significantly compared to that in utero, and it continues to decrease for the first 6 weeks of life before reaching normal levels. As the pulmonary vascular resistance decreases, there is an increase in the amount of left-to-right shunting, leading to symptoms of volume overload heart fail­ ure, which are very rarely present at birth. The most common congenital heart disease to present in this way is a ventricu­ lar septal defect (VSD ) . A hemodynamically significant VSD would cause many of the findings that were noted in this infant. Normal oxygen saturations would be expected with a VSD; but the murmur would be a holosystolic murmur rather than an ejection-type murmur. While there would be cardiomegaly on chest X-ray, you would not expect to see a "snowman sign:' Total anomalous pulmonary venous return (TAPVR) can also be relatively asymptomatic in the neonatal period, especially if the pulmonary venous return is unobstructed. Once the pulmonary vascular resistance has dropped, these children develop similar symptoms of heart failure. The murmur in this anatomy results from a pulmonary outflow murmur related to increased volume of blood. This would present as an ej ection­ type murmur. A "snowman sign" on chest X-ray is classic for supracardiac TAPVR with the pulmonary veins returning to the innominate vein. These children do not appear clini­ cally cyanotic, but when their oxygen saturation is checked, it is lower than normal given the right-to-left shunt across the atrial septum. They are not usually significantly hypoxic (Sp0 2 < 80%) unless there is some obstruction to pulmonary venous return. There are multiple types of TAPVR. Supracardiac TAPVR occurs when the anomalous pulmonary veins drain above the heart. (See Figure 5- l .) Commonly, all the veins come together into a confluence posterior to the left atrium. This confluence then drains superiorly through a vertical vein into the innominate vein. The oxygenated blood then enters

CHAPTER

PV PV

5



CARDIOLOGY

113

Discussion The correct answer is "B:' A PDA is not required for TAPVR and may worsen the degree of pulmonary edema and subsequent oxygen desaturations as measured by pulse oximetry. There is already significant pulmonary overcirculation with this anatomy and maintenance of the PDA with prostaglandins may worsen the degree of pulmonary overcirculation. In cases of severe hypoxia from obstruction, a PDA will only worsen the pulmo­ nary edema and would not be expected to improve oxygenation.



Helpful Tip

:S.� Muscu lar

ventricu lar septal defects (VSDs) are

r1 1r more likely to spontaneously close compared with peri membranous VSDs.

FIGURE 5-1. S u p ra c a r d i a c tota l a n o m a l o u s p u l m o n a ry ve n o u s ret u r n ( TAPVR). The a n o m a l o u s p u l m o n a ry ve i n s d ra i n a bove t h e hea rt i nto the i n n o m i nate vei n . The oxyg e n ated b l ood enters the h e a rt t h r o u g h t h e s u pe r i o r v e n a c a v a . L A , l eft atri u m ; LV, l eft ve n t r i c l e; PV, p u l m o n a ry ve i n ; RA, r i g h t atri u m ; RV, r i g h t v e n t r i c l e . ( U sed w i t h p e r m i s s i o n fro m Benton N g , M D. )

the heart through the superior vena cava ( SVC) into the right atrium. Intracardiac TAPVR occurs when the anomalous veins drain into a confluence that enters the heart through the coronary sinus. This then empties the oxygenated blood into the right atrium. Infracardiac TAPVR occurs when the veins come together into a confluence posterior to the left atrium. Rather than draining superiorly through a vertical vein, the confluence drains through a vein inferiorly, through the dia­ phragm, and empties into the hepatic vasculature. The oxy­ genated blood returns to the heart through the inferior vena cava (IVC) into the right atrium. There can also be a mixture of these three types of pulmonary venous return. The sever­ ity of presentation of TAPVR is related to any obstruction to pulmonary venous return that can occur at the individual veins, confluence, vertical veins, or atrial septum. Infracardiac TAPVR almost always has some degree of obstruction and requires prompt surgical repair. Patients with severe obstruc­ tion present with significant hypoxia and a white-out chest X-ray secondary to pulmonary edema.

� QUICKQUIZ A patent ductus arteriosus (PDA) is necessary for TAPVR. A) True. B) False.

Question 5-2 Suppose the 2-month-old has a holosystolic murmur, cardiomeg­ aly on chest X-ray, and normal Sp02• What condition is she at risk for developing if her congenital heart defect is not repaired? A) Pulmonary hypertension. B) Aortic insufficiency. C) Aortic stenosis. D) Endocarditis. E) All of the above. Discussion 5-2 The correct answer is "E:' She has a VSD. Pulmonary hyper­ tension can occur from a longstanding left-to-right shunt. Per­ manent pulmonary vascular disease is unlikely to occur in the first year of life and with smaller VSDs. With unrepaired VSDs, prolapse of an aortic valve cusp into the VSD can lead to aortic insufficiency. The insufficiency is usually not audible on exam. In the United States, this condition is most frequently seen with perimembranous VSDs. In Asians, it is more likely to occur with subpulmonary VSDs (also known as outlet, supracristal, or conotruncal hypoplasia VSDs) . The development of aortic insuf­ ficiency with subpulmonary VSDs is much more common than with perimembranous VSDs. Prolapse of the aortic valve cusp can obstruct some of the flow across the VSD, which decreases the amount of left-to-right shunt. VSDs are also associated with sub aortic obstruction, which is usually a discrete fibrous or fibro­ muscular ridge below the aortic valve. Endocarditis is rare, but is seen more often in unrepaired VSDs than in repaired defects.

� QUICKQUIZ Which of the following is least likely to lead to pulmonary hypertension? A) Ventricular septal defect (VSD). B) Atrial septal defect (ASD). C) Atrioventricular septal defect (AVSD) . D ) Patent ductus arteriosus (PDA). E) Dextro-transposition of the great arteries (DTGA) .

1 14

MCG RAW- H I LL EDUCATION SPECIALTY BOARD REVI EW: PEDIATRICS

Discussion The correct answer is "B:' As stated above, a longstanding VSD with a large left-to-right shunt can lead to pulmonary hypertension secondary to medial hypertrophy and intimal hyperplasia. It is unusual for an unrepaired ASD to lead to pulmonary hypertension even decades later. AVSD or atrio­ ventricular (AV) canal can result in a large left-to-right shunt (usually if there is a large VSD component) , which can then lead to pulmonary hypertension if unrepaired. A majority of patients with an AV canal also have trisomy 21 (Down syn­ drome) , which itself is a risk factor for pulmonary hyper­ tension. In the case of a VSD and PDA, the large pressure differential between the left ventricle (LV) and right ventricle (RV) (ie, VSD physiology) and aorta and pulmonary artery (ie, PDA physiology) can lead to large left-to-right shunts and future pulmonary hypertension. Pulmonary hypertension is also seen with increased frequency and with accelerated devel­ opment in patients with DTGA. Cardiac defects that cause pulmonary overcirculation (ie, left-to-right shunts) may cause pulmonary hypertension. FIGURE 5-2. This chest X-ray shows card iomegaly with clear l u n g fields a n d n o r m a l p u l monary vasculature. (Reproduced w i t h permission from Fuster V, Wa l s h RA, Harri ngton RA, eds. Hurst's The Heart. 1 3th ed. New York, NY: McGraw- H i l l Education; 2 0 1 1 , Fig. 1 7-20A.)

A 1 5-year-old girl is seen for an acute care visit for com­ plaints of a cough. She has had a nonproductive cough for the last 2 weeks. She denies any rhinorrhea, sneezing, or recent sick contacts. She is afebrile. Her heart rate is 1 22 bpm, her respiratory rate is 38 breaths per minute. On exam, she has a grade III/VI holosystolic murmur. Her chest X-ray shows cardiomegaly with clear lung fields. (See Figure 5-2.) Question 6- 1 Which of the following is the most likely diagnosis? A) Vascular ring. B) Ventricular septal defect (VSD). C) Dilated cardiomyopathy. D) Pneumonia. E) Foreign body aspiration. Discussion 6-1 The correct answer is "C:' Although tachycardia, tachypnea, a holosystolic murmur, and cardiomegaly can be consistent with a VSD, this presentation is unlikely to be seen in a 1 5-year-old. Primary cardiomyopathies are the most likely cause of heart failure in people with structurally normal hearts. Given that the patient is 1 5 years old, it is unlikely that she has unrecognized significant congenital heart disease, especially if she has been cared for in developed nations. Dilated cardiomyopathy is due to a genetic mutation in sarcomeric, cytoskeletal, or cell mem­ brane proteins. This leads to systolic heart failure and dilation of the ventricles. The murmur heard on exam is due to mitral regurgitation that results from the ventricular dilation and poor function.



Helpful Tip

� A holo�ystolic m ur� ur is heard with VSDs and mitral or . . regurg1tat1on. 1 1 1 r tncusp1d

A different 15-year-old girl is seen for an acute care visit for complaints of a cough and fatigue. She had a cold with fever, rhinorrhea, congestion, and cough 2 weeks ago, but most of her respiratory symptoms have resolved. She is afebrile. Her heart rate is 122 bpm, and respiratory rate is 38 breaths per minute. On exam, she has a grade III/VI holosystolic murmur. Her chest X-ray shows an enlarged heart with clear lungs fields. (See Figure 5-2.) Question 7- 1 What is the most likely cause of her antecedent respiratory infection? A) Cytomegalovirus (CMV) . B) Enterovirus. C) Parvovirus B 19 . D) Hepatitis C. E) Human immunodeficiency virus (HIV) . Discussion 7-1 The correct answer is "B:' The most common cause of myocar­ ditis classically is coxsackievirus B (a type of enterovirus) , but recent studies have identified adenovirus in a large number of

CHAPTER 5

patients. CMV, parvovirus B 1 9, hepatitis C, and HIV are also linked with myocarditis, but at lower frequencies. Biopsy is considered the gold standard for diagnosis of myocarditis, but yields clinical information in only 10 % to 20% of cases. A sig­ nificantly greater number of samples is required for accurate diagnosis than is clinically feasible. Serum biomarkers, such as troponin I and inflammatory markers, are frequently drawn but also have low predictive value. Patients with mildly decreased ventricular function typically improve within weeks to months. Of the group that present with more severe dysfunction, 50% develop chronic dysfunction, 25% progress to death or need for transplantation, and 25% have spontaneous improvement.



CARDIOLOGY

115

Right pulmon ary artery Aorta Left pulmonary artery S u perior

Patent d uctus arteriosis Main p u l monary artery

Pulmonary veins Atrial septal defect

--f/7--;oll

Right ---+­ atri u m

Pulmonary veins ---- Left atri u m

ventricle Right ----+---" ventricle

A 2-week-old infant girl is seen in the ED for poor feeding. The mother had an uncomplicated pregnancy and delivery, but the infant has not been interested in eating for the last day. She has also vomited after the few feedings she has taken. It has been over 12 hours since her last wet diaper. She is alert, but not vigorous. She is afebrile, her heart rate is 192 bpm, and respirations are 76 breaths per minute. On exam, she is tachycardic without a murmur. Her liver is 2 em below the costal margin. Capillary refill in her toes is delayed. Question 8- 1 What is the most likely cause of her symptoms? A) Systolic dysfunction. B) Diastolic dysfunction. C) Volume overload. D) Hypoxia. E) Both A and B. Discussion 8-1 The correct answer is "A:' The timing and presentation is consis­ tent with ductal-dependent systemic blood flow, such as a severe coarctation of the aorta or hypoplastic left heart syndrome (HLHS). (See Figure 5-3.) Although the patent ductus arterio­ sus (PDA) commonly closes within the first day of life, some ducts may remain partially open. The vast majority close within the first 1 0 days of life. With a severe coarctation or HLHS, once the PDA closes, the patient has insufficient systemic blood flow and presents with signs of shock and poor perfusion. This is an example of ductal-dependent systemic blood flow. While the PDA is open, there should be a predominantly right-to-left shunt from the pulmonary artery to the aorta. This results in lower oxygen saturations in the legs compared with the arms (in cases with prograde flow in the ascending aorta) . Severe obstruction causes systolic dysfunction. Diastolic dysfunction is frequently present as well, but the systolic dysfunction and inability to overcome the degree of stenosis are what lead to the problems. It is uncommon for a volume overload lesion, such as a VSD, to present with poor perfusion and shock. Severe hypoxia can eventually lead to insufficient oxygen delivery and resulting acidosis and poor perfusion, but this is not as common.

I nfe rior ----t­ vena cava

FIGURE 5-3. Hypoplastic left hea rt syndrome (HLHS). The left ventricle, asce n d i n g aorta, a n d aortic a rch a re hypoplastic. I nfa nts a re dependent on a rig ht-to-left s h u nting patent d uctus a rterious for system i c blood flow. LA, left atri um; LV, left ventricle; RA, right atri um; RV, right ventricle. (Used with permission from Benton Ng, M D.)



Helpful Tip

� Ducta l-dependent lesions are most likely to present

1 1 1r very close to birth and usually within the first 1 to 2 weeks after birth. Volume overload and left-to­ right shunt lesions are most likely to present around 2 months of age. This is related to the natural decrease in pul monary vascular resistance that occu rs after birth.



Helpful Tip

� If heart failure is suspected in a child, further evaluation

1 1 1r would consist of a pediatric cardiology consult and

likely an echocardiogram. Acute ma nagement relies on diuretics in all types of heart failure. Milrinone is also used in severe cases for its afterload reduction and chronotropic effects. Heart failure secondary to overcirculation can usually be ma naged with ora l medications in an outpatient setting.

You obtain a blood pressure reading in her right arm, which is 1 08/72 mm Hg. Question 8-2 Which medication is your first choice for treatment? A) Esmolol. B) Prostaglandin. C) Milrinone. D) Dopamine. E) Indomethacin.

116

MCG RAW- H I LL EDUCATION SPECIALTY BOARD REVI EW: PEDIATRICS

Discussion 8-2 The correct answer is "B:' The signs of poor perfusion and upper extremity hypertension, as well as age of presentation, are con­ sistent with a severe coarctation of the aorta. Using a medica­ tion to treat the hypertension, such as esmolol, risks decreasing lower body perfusion given the fixed obstruction. The first med­ ication should be prostaglandin to attempt to reopen the PDA and relieve the obstruction. Once the PDA has been completely opened and is unrestrictive, there should no longer be a gra­ dient between the upper and lower extremity blood pressures. At this point, afterload reduction with milrinone can be used if necessary to promote systemic flow. At 2 weeks of age, there is also a significant chance that the PDA will not reopen with prostaglandins. If this is the case, surgery is the only treatment option. Why choose indomethacin (a prostaglandin inhibitor) ? The ductus is already closed.



Helpful Tip

� Premature infants with clinically significant PDA require

1 1 1 r closure with medical treatment using indomethacin or

C) Need for lifetime spontaneous bacterial endocarditis (SBE) prophylaxis. D) Dilated aorta. E) Presence of bicuspid aortic valve (BAV) in a first-degree relative. Discussion 9- 1 The correct answer is "C:' A BAV is commonly asymptomatic in childhood. Over time, the valve can become calcified, stenotic, or insufficient. There is also a risk of aortic dilation unrelated to any stenosis. Recent studies show that cellular abnormalities lead to aortic dilation independent of stenosis and flow dynam­ ics. The aortic dilation increases the risk of aortic dissection. There is a 9% prevalence of BAV in first -degree relatives of patients with BAV, so screening echocardiograms are recom­ mended in first -degree relatives. In the past, the risk of endo­ carditis with BAV was estimated to be between 10% and 30%. More recent estimates place the risk between 0.3% and 2% per year. With the change to the bacterial endocarditis prophylaxis guidelines, prophylaxis is no longer recommended in patients with straightforward BAV

ibuprofen (prostaglandin inhibitors) or surgical ligation.

Question 8-3 Coarctation of the aorta is associated with an increased risk for which of the following? A) Endocarditis. B) Intracranial aneurysm. C) Gastroschisis. D) Tracheoesophageal fistula. E) Club foot. Discussion 8-3 The correct answer is "B:' Studies have shown that anywhere from 10% to 50% of patients with coarctation of the aorta have intra­ cranial aneurysms. This may be related to a diagnosis of hyper­ tension. There is currently no recommendation for screening patients with coarctation of the aorta for intracranial aneurysms.

A 14-year-old boy comes to see you for a preparticipation physical. He has normal height and weight. His heart rate is 62 bpm, respirations 18 breaths per minute, and blood pres­ sure 1 14/68 mm Hg. On exam, he has no murmur, but you notice a click. You refer the patient to the cardiology service, and he is diagnosed with a bicuspid aortic valve without ste­ nosis or insufficiency. Question 9- 1 The patient is at increased risk for all of the following EXCEPT: A) Aortic stenosis. B) Aortic insufficiency.

In addition to the stated findings of this patient's echocardio­ gram, described above, it is noted that he has a normal-sized ascending aorta and aortic root. Question 9-2 He should be allowed to participate in full activity until which of the following occurs? A) Moderate aortic stenosis or moderate insufficiency. B) Severe aortic stenosis/aortic insufficiency and dilated aortic root (> 4 em) . C) Severe aortic stenosis/aortic insufficiency or dilated aortic root (> 4 em) . D) All of the above. E) None of the above. Discussion 9-2 The correct answer is "C:' No restrictions are recommended for patients who have an aortic root/ascending aorta size of less than 4 cm.

A boy is born to a 25-year-old G,P0 (now P) mother by spon­ taneous vaginal delivery at 39 weeks' gestation. He requires minimal resuscitation. The neonate's Apgar scores are 8 and 8 at 1 and 5 minutes, respectively. He is then moved to the new­ born nursery with his mother. The next day, on evaluation, you notice down-slanting palpebral fissures, low-set ears, a webbed neck, and a grade II/VI systolic ejection murmur. You check pulse oximetry, which reads 75%. As the echocar­ diogram is started, you are told that the child appears to have severe pulmonic stenosis.

CHAPTER

Question 1 0-1 What additional finding is least likely to be present? A) Large PDA with left-to-right shunt (aorta to pulmonary artery) . B) Right ventricular hypertrophy. C) Patent foramen ovale with right-to-left shunt (right atrium to left atrium) . D) Moderate-sized PDA with right-to-left shunt (pulmonary artery to aorta) . E) Left ventricular hypertrophy. Discussion 1 0-1 The correct answer is "D:' The infant in this case has Noonan syndrome, which is associated with pulmonary stenosis. As his PDA has begun to close, he has developed significant hypoxia. An infant with right ventricular hypertrophy is likely to be pres­ ent with the severe stenosis. The shunting across the patent foramen ovale (PFO) is also likely to be right to left instead of the normal left to right, secondary to the pulmonary outflow tract obstruction. Although the PDA is likely not large given the low oxygen saturations, the direction of shunting is expected to be left to right. You would not expect to have right-to-left shunting across the PDA in the setting of pulmonic stenosis. Initial treatment in this infant would be to start prostaglandins to reopen the PDA to allow for more pulmonary blood flow. An early intervention, either balloon valvuloplasty or surgical valvuloplasty, is required. •

Helpful Tip

.s.::'ll The

pul monary valve in Noonan syndrome can be

i1 1r thick and dysplastic, which decreases the chance of successful transcatheter ba llooning. It is also associated with hypertrophic cardiomyopathy.



Helpful Tip

� Pulmonary valve stenosis not due to a dysplastic

1 1 1 r pulmonary

valve ca n often be ba l looned by transcatheter intervention successfu lly. In general, milder to moderate degrees of pulmonary stenosis rarely progress to severe stenosis and may even have a decrease in the gradient over time.

You see a 9-year-old boy in clinic for complaints of leg pain. He has been experiencing pain for the last week and his par­ ents report that they think his knees and ankles have looked swollen at times. On review of symptoms, they note that he had a sore throat last month that resolved. They deny any rhi­ norrhea, congestion, or fever. On exam today, his left knee is swollen and tender. The remainder of his musculoskeletal exam is normal.

5



CARDIOLOGY

117

Question 1 1 - 1 Which of the following additional findings would help you make a diagnosis? A) Fever. B) Elevated erythrocyte sedimentation rate (ESR) . C) Elevated white blood cell count (WBC). D) Subcutaneous nodules. E) Aspiration of synovial fluid. Discussion 1 1 - 1 The correct answer i s "D:' Th e patient has a history o f sore throat without rhinorrhea or congestion, which is suspicious for pre­ vious strep pharyngitis. On exam, he has evidence of monoar­ ticular arthritis with swelling and pain of his knee, which raises concern for rheumatic fever. The Jones criteria are used to diag­ nose acute rheumatic fever. (See Table 5- 1 . ) Diagnosis is made by fulfilling two major, or one major and two minor, criteria. The major criteria include arthritis, carditis, subcutaneous nod­ ules, erythema marginatum, and chorea. The presence of chorea alone is sufficient to make the diagnosis. Minor criteria include fever, arthralgia, elevated ESR or C-reactive protein (CRP), and prolonged PR interval on ECG. There should also be support­ ing evidence of a previous strep infection with a positive throat culture or elevated or rising strep antibody titer. Arthritis is the most common symptom and is classically described as a migra­ tory polyarthritis responsive to NSAIDs. Large joints, such as the knees, ankles, elbows, and wrists, are most frequently involved. Carditis clinically is associated with a murmur and is the next most common manifestation. The mitral and aortic valves are the dominant valves involved and of most clinical importance . Moderate to severe regurgitation can lead to heart failure. Peri­ carditis can also be present but is almost always associated with mitral or aortic valve disease. Isolated pericarditis is unlikely to be secondary to rheumatic fever. Subcutaneous nodules are uncommon and tend to occur over extensor surfaces. Erythema marginatum is also uncommon and is described as a pink mac­ ule or papule with serpiginous borders and central clearing. The TA B L E S-1 J O N E S CRITERIA F O R ACUTE

R H E U MATIC F EV E R

Major Criteria

Minor Criteria

Polya rth ritis

Fever

Ca rd itis

Arth ra l g i a s

S u bcuta neous nod u l es

Elevated i nfl a m m atory ma rkers (ESR/C RP)

Erythema m a rg i nat u m

Prolonged P R i nterva l

Chorea

CRP, (-reactive protein; ESR, erythrocyte sedimentation rate. Supportive evidence of a group A streptococcal infection is a lso required (positive throat culture or rapid strep or elevated strepto­ coccal antibody titer). Primary episode of rheumatic fever diagnosed by 2 major or 1 major and 2 minor criteria plus evidence of previous streptococcal infection.

118

MCG RAW- H I LL EDUCATION SPECIALTY BOARD REVI EW: PEDIATRICS

rash is evanescent and may change quickly. A hot bath or shower can accentuate the rash. Both erythema marginatum and nodules are associated with the presence of carditis. Sydenham chorea consists of involuntary, purposeless movements and emotional lability. The onset of chorea is usually delayed compared with arthritis and carditis. Chorea may occur 1 to 6 months after the initial infection in contrast to arthritis, which manifests between 10 days and 5 weeks postinfection. Question 1 1 -2 All of the following are recommendations for treatment of mild to moderate acute rheumatic carditis EXCEPT: A) High-dose aspirin. B) Steroids. C) Activity restrictions. D) Primary antibiotic prophylaxis. E) Secondary antibiotic prophylaxis. F) Intravenous immunoglobulin (IVIG). Discussion 1 1 -2 The correct answer is "F:' High-dose aspirin (80- 100 mg/kg/day) is recommended for mild to moderate carditis. Anti-inflammatory agents are considered standard of care as some patients improve with resolution of inflammation. There is no evidence that steroids (2 mg/kg!day for 2 weeks with a taper) are superior to aspirin, but they are recommended in moderate to severe cases of carditis. If steroids are used, aspirin is initiated with the steroid taper. Activity restrictions of some kind are recommended for 4 to 6 weeks, with some experts recommending bed rest. Primary antibiotic prophy­ laxis, most commonly with penicillin, is used for treatment of acute streptococcal infection. Secondary antibiotic prophylaxis is required in rheumatic carditis, with the duration depending on the severity. The highest risk period for recurrence of rheumatic fever is the first 1 to 2 years following an episode. Patients who have carditis in their initial episode of rheumatic fever are likely to have carditis as part of recurrent episodes. Recurrent rheumatic fever can lead to more severe valve dysfunction and chronic rheumatic heart disease. IVIG is not recommended for rheumatic carditis.

A 3-year-old girl is admitted for evaluation of persistent fever. She has had a daily fever for the last 7 days of at least 38.8°C ( 1 02°F). Her mother describes her as generally more irritable over this time. She developed a diffuse maculopapular rash yesterday. On exam, you find her quite irritable and uncoop­ erative with the exam. She is febrile to 39SC ( 1 03.2°F) . She has bilateral conjunctivitis, dry, peeling lips, swollen hands, and a diffuse maculopapular rash. Question 1 2- 1 She i s expected t o have all o f the following laboratory abnor­ malities EXCEPT: A) Anemia. B) Thrombocytopenia.

C) Hypoalbuminemia. D) Elevated CRP. E) Elevated gamma glutamyl transpeptidase. Discussion 1 2- 1 Th e correct answer i s "B:' Clinically, the young girl can be diag­ nosed with Kawasaki disease with her course of fevers, rash, conjunctival injection, extremity changes, and peeling lips. Addi­ tional clinical findings include erythema of the soles and palms, strawberry tongue, and cervical lymphadenopathy. The rash can take on various forms, including an urticarial exanthem, scar­ latiniform, erythema multiforme-like, or micropustular rash. Within 2 to 3 weeks after onset of fever, desquamation of the fin­ gers and toes begins in the periungual region. The conjunctival injection spares the limbus. There should be no exudate or pain associated with the conjunctivitis. The cervical lymphadenopa­ thy is usually unilateral in the anterior cervical triangle.

She subsequently receives IVIG and an echocardiogram to evaluate for coronary aneurysms. No aneurysms were seen on this study. She becomes afebrile after receiving IVIG with down-trending inflammatory markers and is discharged home. Question 1 2-2 What treatment and follow up are needed? A) High-dose aspirin (80- 1 00 mg/kg/day) . B) Follow-up echocardiogram. C) Warfarin. D) Steroids. E) Cardiac catheterization. Discussion 1 2-2 The correct answer is "B:' High-dose aspirin is needed only for the acute illness until the patient has been afebrile for 48 hours. Warfarin is not indicated without giant aneurysms. Steroids can be considered for failed convalescence with IVIG. A cardiac cath­ eterization is also not indicated, especially soon after acute illness. A follow-up echocardiogram is needed, even without aneurysms seen on the initial study. Coronary involvement in Kawasaki dis­ ease follows multiple stages. The first stage occurs between 0 and 9 days and consists of acute endarteritis of the coronary arteries with pericarditis, valvulitis, and myocarditis. From days 12 to 25, aneurysm and thrombus formation occur, with intimal prolifera­ tion. From days 28 to 3 1 , coronary granulation and marked inti­ mal thickening can occur. From day 40 out to 4 years, there can be scarring, stenosis, and calcification of the coronary arteries. More than 50% of aneurysms resolve in 1 to 2 years. Low-dose aspirin is recommended for at least 6 to 8 weeks, or until the inflammatory markers and coronary arteries are normal. (See Table 5-2.)

A 1 7-year-old adolescent boy with a history of tetralogy of Fallot previously palliated as an infant undergoes pul­ monary valve replacement with a bioprosthetic valve. His

CHAPTER

TA B L E 5-2 LO N G - TERM MANAG E M ENT OF KAWASAKI D I S EAS E-AM E R I C A N H EART ASS

5



CARDIOLOGY

119

I AT I O N

GUIDELINES

Risk

Drugs

Restrictions

Follow up

Invasive tests

No a n e u rys m s ever

None beyo nd 6-8 weeks

None beyond 6-8 weeks

Normal ca rd i ovascular co u n se l i ng eve ry 5 yea rs

None

Tra n s i ent ecta sia d i sa ppea r i n g in 6-8 wee ks

None beyo nd 6-8 weeks

None beyond 6-8 weeks

Normal ca rd i ovascular co u n se l i ng eve ry 5 yea rs

None

1 s m a l l to m ed i u m a n e u rysm

Low-dose a s p i r i n u n t i l reg ression of a n e u rysm

None beyond 6-8 weeks fo r patients yo u nger t h a n 1 1 years; fu rther eva l uation recommended for pati e nts older t h a n 1 1 yea rs to g u i d e restrictio n s

Annual card i o logy fo l l ow u p with echo a n d ECG; stress test/ perfusion scan every 2 yea rs

Angiogra p hy if abnorm a l perfusion sca n or stress test

� 1 l a rge or gi a n t a n e u rysm

Lo ng-term antiplatel et and warfa r i n or low-molecu l a rwei g ht h e pa r i n

Avoi d contact sports

Eve ry-6-month fo ll o w u p w i t h echo a n d ECG; stress test/ perfusion sca n yea rly

Angiogra p hy 6- 1 2 months after acute i l l ness or if other tests a re a b n o r m a l

Coro n a ry a rtery obstruction

Lo ng-term l ow-dose aspirin a n d wa rfa r i n or low-molecu l a rwei g ht h e pa r i n

Avoi d contact sports

Eve ry-6-month fo llow up with echo and ECG; stress test/ perfusion sca n yea rly

Angiogra p hy 6- 1 2 months after acute i l l ness or if other tests a re a b n o r m a l

ECG, electroca rdiogram; echo, echoca rdiogram.

initial postoperative period i s uncomplicated, but h e develops fevers on postoperative day 7 with temperatures up to 39°C ( 1 02.2°F) . Laboratory tests and blood cultures are drawn and he is started on broad-spectrum antibiotics. Both his white blood cell count and inflammatory markers are elevated. Two days later, viridans group streptococci are identified on his initial blood culture. Subsequent daily blood cultures are also preliminarily positive. Question 1 3- 1 Which o f the following would NOT fulfill the diagnosis of definite infective endocarditis? A) Increasing intensity of systolic murmur as compared with immediate postoperative period. B) Evidence of an oscillating mass on the prosthetic valve by echocardiogram. C) Glomerulonephritis. D) Conjunctival hemorrhage. E) Rheumatoid factor. Discussion 1 3- 1 Th e correct answer i s "A:' Th e Duke criteria provide a diagnostic strategy for making the diagnosis of infective endocarditis due to the variability in presentation. The criteria combine clini­ cal, microbiologic, and echocardiography findings to stratify

patients into "definite;' "possible;' and "rejected" cases. (See Table 5-3.) The patient currently fulfills one major criterion: blood culture positive for infective endocarditis. Bacteremia secondary to infective endocarditis should be continuous. Therefore it is not required that cultures be drawn only when the patient is febrile. The timing for obtaining blood culture is not strict, but three cultures drawn over the first 24 hours is felt to be sufficient. Viridans group streptococci (alpha-hemolytic streptococci) and Staphylococcus aureus are the two most com­ mon etiologies of infective endocarditis. He also fulfills two minor criteria with fever and his congenital heart disease and prosthetic valve. Although an increasing intensity of a mur­ mur may be a result of increasing obstruction secondary to a large vegetation, this is insufficient to meet criteria. Evidence of an oscillating mass by echocardiogram is a major criteria. The remainder of the findings would be minor criteria. Question 1 3-2 Which of the following is not an immunologic phenomenon of infective endocarditis? A) Osler nodes. B) Glomerulonephritis. C) Janeway lesions. D) Roth spots. E) Rheumatoid factor.

1 20

MCG RAW- H I LL EDUCATION SPECIALTY BOARD REVI EW: PEDIATRICS

TA B L E 5-3 M O D I F I E D D U KE CRITERIA FOR I N F ECTIVE E N DOCA R D I T I S

Defi n ite i nfective endocarditis

1 . Pathologic criteria •

M i croorg a n i s m demon strated by c u l t u re or h i stology i n a vegetation

Patholog ic lesions: vegetation or a bscess confi rmed by h i stolog ic exa m i natio n . 2. C l i n ical criteria •



2 major or



1 major a n d 3 m i nor o r



5 m i nor

Possible i nfective endocard itis

1 . Presence of 1 major and 1 m i nor criterion or 2. Presence of 3 m i no r criteria Rejected i nfective endocard itis

1. 2. 3. 4.

Alternative d iagnosis fo u n d Reso l ution o f i nfective endocard itis sym pto m s w i t h a nt ib iotics for :::; 4 d ays No pathologic evide nce of i nfective endoca rd itis at su rg ery/a utopsy with a nti biotic thera py for :::; 4 d ays Does not meet criteria for poss i bl e i nfective endoca rd itis

Criteria

Major criteria 1 . B lood c u l t u re positive a. Typical orga n i s m s con s i stent with i n fective endoca rd itis fro m 2 separate blood c u l t u res Vi ridans stre ptococci, Streptococcus bovis, HAC E K g ro u p, Staphylococcus aureus Com m u n ity-acq u i red enterococci i n a bsence of pri m a ry foc u s b. M icroorg a n i s m s co n s i stent w i t h i nfective endoca rd itis from persi stently positive blood c u l t u res; a t least 2 positive c u l t u res d rawn > 1 2 h o u r a pa rt c. A l l of 3 or m ajority of ;::: 4 sepa rate c u l t u res with fi rst a n d last sam ples d rawn ;::: 1 h o u r a pa rt d. S i n g l e positive c u l t u re fo r Coxiella burnetii 2. Evi dence of endocard i a l i nvo lvement •



a . Echocard iogram positive for i nfective endoca rd itis Osci l l ati ng i ntraca rd iac mass on va lve or su pporti ng structu res, i n the path of reg u rg itant jets, or o n i m pl a nted material in the a bsence of a lternative expla nation Abscess New partial d e h i sce nce of prosthetic va lve b. New va lvu l a r reg u rg itation ( i ncrease or change i n preexisti ng m u r m u r is n ot suffici ent) •





Minor criteria 1 . Pred i s position, at-risk heart co nd ition, or i ntravenous d rug use 2. Fever (te m peratu re > 38°C [ 1 00.4°F]) 3 . Va sc u l a r phenomena: m ajor a rteri al e m bo l i , septic p u l m o n a ry i nfa rcts, mycotic a n e u rys m, i ntracra n i a l hemo rrhage, conj u n ctiva l hemorrhages, a n d J a n eway lesion 4. I m m u nologic phenomena: g lo m e r u l o n e p h ritis, Osier nod es, Roth s pots, rheumatoid factor 5 . M icrobiologic evidence: positive b l ood c u l t u re, but does not m eet major criteria

HACEK, Haemophilus species, Actinobacillus actinomycetemcomitans, Cardiobacterium hominis, Eikenella corrodens, and Kingella species. Discussion 1 3-2 The correct answer is "C:' Janeway lesions are due to vascular phenomena. The other findings are immunologic phenomena. Other vascular findings include major arterial emboli, septic pulmonary infarcts, mycotic aneurysms, intracranial hemor­ rhage, and conjunctival hemorrhage.

Question 1 3-3 Which of the following is NOT considered a cardiac condi­ tion associated with the highest risk for adverse outcome from endocarditis?

CHAPTER

A) Previous episode of infective endocarditis. B) Bicuspid aortic valve with moderate stenosis and moderate insufficiency. C) Status 4 months postoperative from patch closure of a peri­ membranous VSD. D) Unrepaired tetralogy of Fallot. E) Mechanical aortic valve. Discussion 1 3-3 The correct answer is "B:' Cardiac conditions considered to be associated with the highest risk for which SBE prophylaxis with dental procedures should be considered include: •













Prosthetic valve or prosthetic material used for valve repair Previous infective endocarditis Congenital heart disease Unrepaired cyanotic heart disease, including shunts and conduits Completely repaired defects with prosthetic material during the first 6 months after procedure Repaired defects with residual defects at or near the site of prosthetic patch or device, which may inhibit endothelialization Heart transplant with valvulopathy

Question 1 3-4 Which of the following etiologies of infective endocarditis (native valve) requires the shortest course of treatment? A) Penicillin-susceptible viridans group streptococci. B) Penicillin-resistant viridans group streptococci. C) Oxacillin-susceptible staphylococci. D) Oxacillin-resistant staphylococci. E) Length of therapy is the same for all bacterial etiologies. Discussion 1 3-4 The correct answer is "A:' Endocarditis due to staphylococci (oxacillin susceptible or resistant) requires 6 weeks of intrave­ nous (IV) antibiotics. Cases caused by penicillin-susceptible viridans streptococci require 4 weeks ofiV antibiotics. If medical therapy is unable to clear the vegetation, surgery may be needed.

A 5-month-old immunized boy is brought to your clinic by his parents, who are concerned about increased sleepiness and poor feeding. They deny any symptoms of cough, rhi­ norrhea, or congestion. They report that he had previously been feeding every 2 to 3 hours but now must be woken up each time to feed. He is also taking less with each feed. On exam, he is alert and afebrile. His heart rate is 280 bpm and respiratory rate is 70 breaths per minute. His cardiac exam is significant for tachycardia. His abdomen is soft, with the

5



CARDIOLOGY

1 21

liver palpable 3 em below the costal margin. His capillary refill time is 2 to 3 seconds. Question 1 4- 1 What i s the next best treatment? A) Obtain an ECG. B) Administer intramuscular (IM) ceftriaxone. C) Place an IV and administer adenosine. D) Urinalysis and urine culture. E) Cardioversion. Discussion 1 4- 1 Th e correct answer i s "A:' This infant has signs o f compen­ sated heart failure secondary to arrhythmia. Infection should always be considered in a child with decreased activity, but this 5 -month old is afebrile and without signs of localizing infection. In addition, the physical exam finding of hepato­ megaly would be less consistent with infection but is a sign of heart failure. While he is quite tachycardic, with a heart rate of 280 bpm, he is hemodynamically stable. Therefore, there is time to obtain an ECG for diagnosis of his arrhythmia. The ECG will show a heart rate greater than 220 bpm, absence of P waves, and no R-R variability. Synchronized cardioversion is reserved for those patients who are hemodynamically unsta­ ble. The infant may require an IV and adenosine to break the arrhythmia, but other vagal maneuvers, such as ice to the face, should be attempted first.

You obtain an ECG, shown in Figure 5-4. Question 1 4-2 Which of the following is most true? A) He will require lifelong medication. B) His heart function is likely to recover. C) He is at high risk for sudden death. D) He will have a normal baseline ECG. E) He will require SBE prophylaxis. Discussion 1 4-2 The correct answer is "B." The ECG shows supraventricu­ lar tachycardia (SVT) . The most common causes of SVT in pediatrics are atrioventricular reentrant tachycardia (AVRT) , atrioventricular nodal reentrant tachycardia (AVNRT), and atrial flutter. (See Figure 5 - 5 . ) His heart failure is secondary to prolonged tachycardia, or an arrhythmia-mediated cardio­ myopathy. Once he is converted out of the abnormal rhythm, his heart function is likely to recover. Given the infant's age at presentation, he is most likely to have AVRT and there is a 50% chance that this will spontaneously resolve as the accessory pathway may become nonconductive tissue. SVT rarely causes sudden death and is well tolerated in children for prolonged periods. In the case of AVRT, there is a small increased risk for sudden death if there is antegrade conduc­ tion across the accessory pathway. This may allow for atrial fibrillation to be conducted quickly to the ventricle and result

1 22

MCG RAW- H I LL EDUCATION SPECIALTY BOARD REVI EW: PEDIATRICS I �

A

I I A Ill 1"11.v �I fP'['!�z-.. 1rr.---tf 'v-hf '-" Ill' M�· V\rr ""' �· r�lr ""rlr-r- lr'-

II

Ill

I

I



II

lll r--t II r

'

t

J - �

II -�� �II

4

ilr---v Ill'-'IV r-'rv � v I"! I --.J V

\1

\0

v-'\Jv(\/\/lN\v'\}\� Ill

F

v

aVF

.1\

� ..

Jl



"-...

['-....



/!'- ,-/rv

.,.,.. _/'

ll\

-

'\

/'

lJl\

%\.

/\

� c

, ,.,

i

c

,'\.

VI "'-..

;;

1\

I

'\

),

r

cc

.II

.n

0

"l



.n

J

v

r.

I II 'll ir-. !t v 1

I

Ill r

��

. ft

VI

�� -�

.ft

VI

t�

A

V\

ft

VL H

A , "L

�������

J

.A '

����

.A

A " '-

.LJ._..U. L-

A

l�v A

""

� ��

'V

r ""

-���

0 • •v

:v

L..cc.�

���

ILY

c 'cV '-"

-

�c...c.�-

���-�

v'

-�����

'A I V'

,. A

���-

I I

-��

FIGURE S-4. S u p raventric u l a r tachyca rd ia (SVT). ECG showing hea rt rate g reater than 220 bpm, n a rrow QRS, a n d no R-R va riabil ity. (Reprod uced with permission from Klamen DL, H i n g l e ST, eds. Resident Readiness: Internal Medicine. New York, NY: McGraw-H i l l Education; 20 1 4, Fig. 1 1 - 1 .)

in ventricular fibrillation, which is a life-threatening arrhyth­ mia. With AVRT, preexcitation (delta wave) can be seen on a baseline ECG. (See Figure 5 - 6 .) The delta wave occurs due to initial antegrade conduction through the accessory path­ way. In AVNRT, the reentrant pathway occurs essentially in the AV node. It is not possible to differentiate AVRT from AVNRT by ECG. In contrast to AVRT, AVNRT is unlikely to resolve spontaneously. AVNRT is a more common presenta­ tion in teenagers than infants. Adenosine terminates the SVT caused by both AVRT and AVNRT by blocking conduction

through the AV node, which stops the reentrant cycle. In atrial flutter, the reentrant loops are located in the atrium and do not involve the AV node. In these cases, the atrial rate may be as high as 400 to 500 bpm and is conducted to the ventricle in a 2 : 1 fashion through the AV node, resulting in a ventricular rate of 200 to 250 bpm. In this case, adenosine will block the AV node, but the reentrant loops in the atrium continue. This will result in a ventricular pause, but flutter waves will become apparent on ECG. Once the adenosine wears off, the SVT will resume.

Atrioventricu lar node Sinoatrial ---+� ii:.===��� node

SVT circuit

��of-- Left

bundle branch

Normal E lectrical Cond uction

Supraventricular Tachycard ia (SVT)

FIGURE 5-5. S u p raventricu l a r tachyca rd ia. AV, node; atriove ntric u l a r node; AVN RT, atrioventric u l a r noda l reentrant tachyca rd ia; AVRT, atrioventric u l a r ree ntrant tachyca rd ia; SA node, si noatri a l node. (Used with permission fro m Benton Ng, M D.)

CHAPTER

5



CARDIOLOGY

1 23

S pontaneous intermittent preexcitation

FIGURE S-6. Delta wave in Wolff-Parki nson-Wh ite (WPW) syndrome. This ECG s hows fi n d i n g s consistent with WPW syndrome: short PR i nterva l (< 0.2 seconds), delta waves, and widened QRS com p l exes. Preexcitation (delta wave) resu lts from a nteg rade conduction via the accessory pathway. (Reproduced with permission from Hay WW, Levin MJ, Deterd i n g RR, Abzug MJ, eds. Current Diagnosis and Treatment Pediatrics. 22nd ed. New York, NY: McGraw- H i l l Education; 20 1 4, Fig. 20-7.)

Question 1 4-3 Which of the following rhythms requires defibrillation?

A) n

p

I I



p

p



'

p

p

I

\l

-�

p

p

I

p

-�

p

I



p

p

-�

A

n

ll

p



-A _£

\j

p

p

f\

""

v

p .\

I

p/

p

p

lo

l

p

p

p

II

II

8 (Reproduced with permission from Kasper DL, Fa uci AS, Ha user S L, et a l : Harrison's Principles of Internal Medicine, 1 9th ed. McGraw- H i l l Education, I nc., 20 1 5. Fig 275-4.)

B)

v

a�F

I

!

I

,.,.. �/ J\JI v -'\II

\4

! I r1 ' !"'II./ "-11,�N, --:...-11' I"''11 �hf '-"hi �'l"�nr --0. llr-V"'-'lr-r-lin" I

II

I \/·

Iii

./fi\11.Y 'VILA'\/I /'M/'

H I ti dli-c- \J\r.H. ir-..JI, r Ul

I



I'

� -

·�



V\

,I..

'\

[£\. !A-

f1'v

lit

� r--.

f'-.J /"'-' ...1

.1-R..

.;.

I ll

h. / "-' V r.. /

"' lrJ'\!



I .IN ./

lY'

,/1\

·.rc

I

' 71.

I

.A.

, I/\

rc

I

rc

I

c),

./I

I

I

I

J .A J .A .fl

I

c.

l/1

i'

I -"' J� I

I

II

t

'�

-,ft - ,ft

,A

. ft

.A

. A - .A

�n.....c

A

., A

,..._

ft

V\..-

IA

·�

' '-" -" · �

rv

I/

I.L

,tt c II/

.n

"--�

,A "

.n

c.�

. ft

ft

'�

' _6 I

- ·��

(Reproduced with permission from Klamen DL, H i ng l e ST, eds. Resident Readiness: Internal Medicine. New York, NY: McGraw- H i l l Education; 20 1 4, Fig 1 1 - 1 .)

MCG RAW- H I LL EDUCATION SPECIALTY BOARD REVI EW: PEDIATRICS

1 24 C)

u l\

'I

-"-

A

'--�

v

lr v

hr�

�-

II

II

\_

hr r

l/ v

r--

(Reproduced with permission From Longo DL, Fa uci AS, Kasper DL, et al, eds. Harrison's Principles of Internal Medicine. 1 9th ed. New York, NY: McGraw- H i l l Education; 2 0 1 5, Fig 274-3.)

D)

''"'



VV� r "l f" /h V'-11 1' II' I I I I. 1/ 1/ I. 11 I I . IV

I f\ ill

"'

I

1/\



I I

1n 'I l l

v \

/1

Ill

��

"11 1"'1

I 1\1 1 \, ,/ I 0I.

I�

II ft 1n 1n VL. J' f\ 1 r \ I I" 1\ I I' - IJL V 'd I. U" I \ \1 \ ' lf l \. f.' I

II

.J1

Ill

II\ I

1/

I I

II

l11 I I I , I I I 1/

II

I \

�I

VI Jl

v

v

n- c11

II 1/1

I"

W3lfl II II \I I 'I I 'I

••

/'

p

Vo

\

II

_A ) rL J\

W -

17 1

1\ J

I

1-' J

II ll 1' I I\..Il l IJ l c f.j J 1 -r/ j

n I I ,.II ) \.

n �I

/I I

I

, I \

� J

If 0

dl II I' II 11 11 • .1 "- r \I f \ I I l I \ II l • 'I ' I 1/ • L IJ, I I I I I I I l cr-1

v

1/\. )'

,I

{I

W2

v

I/\

"\! ''-

'

1\. I n /\

IV

II

If

..;

11 I 1 1 �

c

IL /1

/I,_

(Reproduced with permission from Klamen DL, H i n g l e, ST (Eds). Resident Readiness: Internal Medicine. McGraw- H i l l Education, I n c., 2 0 1 4. Fig 1 3- 1 .)

E)

I

f--h n

l ei II II p 1 / ( �\/� i/c

VI If\ f\ (\ \ � c \/1\fJ I ' \ 1., � kl \1 1/ l 'l'alf. - j v ,_

v "

II ·I M r\. /'I('

lt c

v

-

I

' M- 1/ , "I '11 "

nJ

r�

n

n u

r.

' I '\ I IvI

a

A

II

l

II



u c

\.

1/

f'

rs

'

l r'l 11

IV u

II

1:1

1/

\/'

+

p n 1\ 1/ / I ( I I II II II

A 1/L

ll" I L- 71, "I \ , � � 1/ II Y"

Yl

u

c

If II 1rI I/ I ll L v

IU

j j

� ! - [\ ' .1 lfll I( U/ lo< li"'j V

I

l l

'

-,

t

tl\.

' \k' '

\

fi / IV

v

1\ / 1 \; 'v' v

1 j j

1

2

1 11'

di

1. ( In\�

I

,I

{\

II

A

I

l\

I fl

1\

I A

Jl

II �-� In! m� II

II Ill

I

I \, I II

Ill " 'let

{\

'4' WY �ct! I� �l·tl�f ��

· I I ll 'lr I ,I If '\ I 1\ I I I \lf-JJI I I• I, I I I·

'I 'I II\

I 'I I I \. II \,, I s

0.2 seconds) Evide nce of g ro u p A streptococca l i nfection Positive t h roat c u lt u re or ra pid a ntigen tests El evated a nti body titer (ASO, DNase B) CRP, (-reactive p rotein; ES R, eryth rocyte sed i mentation rate.

FIGURE 7-2 . Erythema marg i natum is one of the major criteria i n the Jones criteria for diagnosing acute rheu matic fever. It is descri bed as pink rings located on the tru n k and extrem ities. The face is spared. The lesions expand and move from one location to another. (Reproduced with perm ission from Goldsmith LA, Katz 51, Gilchrest BA, et al, eds. Fitzpatrick's Dermatology in General Medicine. 8th ed. New York, NY: McGraw- H i l l Education; 201 2, Fig. 1 60-5.)

Question 2-1 What is the diagnosis? A) Post -streptococcal arthritis. B) Lyme disease. C) Systemic lupus erythematosus. D) Henoch-Schonlein purpura. E) Immune thrombocytopenia. Discussion 2-1 The correct answer is "D:' Henoch-Schonlein purpura (HSP), or IgA vasculitis, causes a characteristic rash (nonthrombocytope­ nic palpable purpura) ; arthritis or arthralgias, or both; colicky abdominal pain; and renal disease. The history of a recent upper respiratory tract infection is classic for HSP. For the diagnosis to be HSP, the platelet count and coagulation studies must be normal. (See Table 7-4 and Figure 7-3. ) Patients with lupus usually present with an abnormal CBC, indicating leukope­ nia, thrombocytopenia, anemia, or a combination of these. The patient's Lyme titer is negative, and Lyme arthritis tends to be in the knees. This is unlikely streptococcal disease as the patient's ASO titer is negative, she has no history of pharyngitis, and it would not explain the rash. Her platelet count is normal, ruling out immune thrombocytopenia. Question 2-2 What treatment do you recommend for HSP? A) NSAIDs. B) Prednisone. C) IVIG. D) Morphine. E) Nothing.

MCG RAW- H I LL E D U C AT I O N S P E C I A LTY BOA R D REVI EW: P E D I ATRICS

1 54

TA B L E 7-4 M A N I F E STAT I O N S O F

H E N O C H - S C H O N L E I N P U R P U RA

Rash

Dark-red a n d p u rple lesions o n the l owe r extrem ities a n d buttocks (classic) Sym m etric

Edem a/Swe l l i ng

Hands a n d feet Scrot u m Aro u n d the eyes Col i cky abdom i n a l pa i n

Gastroi ntest i n a l

Vo m i t i n g I ntussu sception Hematemesis, hem atoc hezia, melena Perfo ration, ischemia Hematuria

Ren a l

Prote i n u ri a Neph ritis N e p h rotic syn d rome Hypertension End-stage kid ney d i sease N e u rologic

Headaches Seizures Ataxi a

Constituti ona I

Feve r M a l a ise Art h ra l g i a

M u sculoske l etal

Arth ritis

Discussion 2-2 The correct answer is "A:' NSAIDs are given for relief of joint and abdominal pain. Avoid NSAIDs if the patient has significant kid­ ney disease or active gastrointestinal bleeding. Save corticosteroids (prednisone) for patients with severe abdominal pain or inability to walk due to arthritis; otherwise try to avoid prescribing them. •



Helpful Tip

A l l patients w i t h HSP s h o u l d have fol low-u p u rina lysis

1 1 1r a n d

blood pressu re checks monthly for 6 months.

Ren a l i nvolvement may develop u p to 6 months after

presentation.





Helpful Tip

In atypical cases, abdominal pain or arthritis will be the

I l l r presenting

sign of HSP. The diagnosis becomes clear

when the rash develops. If in doubt, you ca n obta i n a biopsy the s k i n or kidney to look for lgA deposits.

FIGURE 7-3 . Henoch-Schon l e i n purpura (HSP). Symmetric pa lpable purpura on the lower extre m ities and buttocks of a c h i l d with H S P. (Reproduced with permission from Knoop KJ, Stack LB, Storrow AB, et al: The Atlas of Emergency Medicine, 3ed. McGraw-H i l l Education, I nc., 20 1 0. Fig 1 5-23. Photo contri butor: Ralph A. Gruppo, M D.)

� QUICKQUIZ What is the most characteristic renal finding in patients with HSP? A) Macroscopic hematuria B) Microscopic hematuria C) Proteinuria D) Hypertension E) Elevated serum creatinine Discussion The correct answer is "B:' Renal involvement occurs in 20% to 50% of children with HSP but less than 5% go on to develop end-stage renal disease. Any of the findings listed may occur, but microscopic hematuria is most common. Signs and symp­ toms of renal disease usually develop 2 to 6 weeks after presen­ tation. Nephrotic range proteinuria, elevated serum creatinine, and hypertension are signs that progressive kidney disease will develop.

CHAPTER 7



COLLAG E N VAS C U LA R A N D OTH E R M U LTI SYSTEM D I SO R D E R S

A 5-year-old boy presents with 7 days of fever to 39.4°C { 1 03°F) daily. He is irritable and has a morbilliform rash, con­ junctivitis, swelling of his feet, and large adenopathy of the neck. Laboratory tests show leukocytosis, anemia, elevated alanine aminotransferase (ALT}, and pyuria. Question 3-1 What is the diagnosis? A) Measles. B) Stevens-Johnson syndrome. C) Kawasaki disease. D) Scarlet fever. E) Adenovirus infection. Discussion 3-1 The correct answer is "C:' Kawasaki disease is a vasculitis affect­ ing children typically between 6 months and 6 years of age. Symptoms self-resolve after 2 weeks. However, without treat­ ment up to 25% of patients will develop coronary artery aneu­ rysms. Table 7-5 shows characteristic laboratory abnormalities. The differential diagnosis includes viral infections (adenovirus, measles) , Stevens-Johnson syndrome, and scarlet fever.

1 55

Discussion The correct answer is ''A:' The diagnosis of Kawasaki disease requires fever for 5 or more days plus four or more of the following five clinical characteristics: •









Conjunctivitis: bilateral, nonexudative, bulbar with limbus sparing (key) Oropharyngeal changes: cracked lips, pharyngeal erythema, strawberry tongue Cervical lymphadenopathy: unilateral, typically painless Extremity changes: swelling of the hands or feet, erythema of the palms or soles Rash: erythematous, polymorphic

Pharyngitis with exudates or ulcers, purulent conjunctivitis, and bullous or vesicular rash are not seen in Kawasaki disease. Check the perineum for the rash as it is more prominent there and may desquamate early. Weeks after the fever, peeling under the nail beds of the fingers and toes may occur. Abdominal complaints (pain, diarrhea, and vomiting) are common. (See Figures 7-4 through 7-6.}

� QUICKQUIZ Which is NOT a diagnostic criterion of Kawasaki disease? A) Exudative conjunctivitis. B) Lymphadenopathy. C) Strawberry tongue. D) Feet swelling. E) Urticarial rash. TA B L E 7-5 LABORATORY A B N ORMALITI E S I N ACUTE KAWASAKI D I SEASE

Neutro p h i l i c leu kocytosis Elevated (-reactive prote i n Elevate eryth rocyte sed i m entation rate A n e m i a for age Throm bocytosis after the fi rst wee k (ca n exceed 1 m i l l ion/mm3) Throm bocyto pen i a from d i s se m i nated i ntravasc u l a r coa g u lation (DIC)-rare Hyponatre m i a Hypoa l b u m i ne m i a Elevated seru m g a m m a g l utamyl tra n s peptidase (GGT)

FIGURE 7-4. Kawasaki d i sease. A yo u n g b o y with prolonged fever

Elevate serum l ive r tra n s a m i nases

a n d ( 1 ) red, cracked l i ps; (2) red, edematous h a nds; (3) non exudative

Sterile pyu ri a

conj u n ctivitis; and (4) morbi l l iform rash. H e has fou r of five d i a g nostic

Cerebra l s p i n a l fl u i d pl eocytos i s

from Wo lff K, Joh nson RA, Saaved ra A P, eds. Fitzpatrick's Color Atlas

Le u kocytosis i n synovial fl u i d

and Synopsis of Clinical Dermatology. 7th ed. New York, NY: McG raw-H i l l

criteria consistent with Kawasaki d i sease. (Reproduced with permission

Education; 2 0 1 3, Fig. 1 4-66.)

1 56

MCG RAW- H I LL E D U C AT I O N S P E C I A LTY BOA R D REVI EW: P E D I ATRICS



Helpful Tip

� Hydrops

:5.

of the g a l l bladder (distention not d u e to

i1 1r stones) ca n be seen at presentation i n patients with Kawasaki d i sease.

FIGURE 7-5. I n Kawasaki d i sease, the conj u n ctiva is i nj ected a n d t h e l i m bus i s spared, a n d without exudate. (Reproduced w i t h permission from Goldsmith LA, Katz 5 1 , G i l c h rest BA, et al, eds. Fitzpatrick's Dermatology in General Medicine. 8th ed. New York, NY: McGraw- H i l l Education; 2 0 1 2, Fig. 1 67-5.)

Question 3-2 What tests do you order now? A) Abdominal ultrasound. B) Echocardiogram. C) Rapid streptococcal antigen test of the oropharynx. D) Blood culture. E) Urinalysis. Discussion 3-2 The correct answer is "B:' You already established that he has Kawasaki disease so additional laboratory testing to support or look for an alternative diagnosis is not needed. A baseline echo­ cardiogram should be performed as soon as the diagnosis is sus­ pected. Treatment should not be delayed waiting for the test or its results. Aneurysms typically do not form before day 10 of the illness, so the initial echocardiogram is frequently normal. For uncomplicated cases (heart and coronary arteries normal), an echocardiogram should be performed at diagnosis, at 2 weeks and at 6 to 8 weeks after the onset of Kawasaki disease to evalu­ ate for coronary artery aneurysms.

Question 3-3 When is the ideal time to treat this patient with IVIG (intra­ venous immunoglobulin) ? A) By day 1 2 of fever. B) By day 5 of fever. C) By day 1 0 of fever. D) Never. E) After systemic steroids fail to resolve the fever. Discussion 3-3 The correct answer is "C:' Once the diagnosis is made, acute Kawasaki disease is treated with a single dose of iVIG and high­ dose aspirin. Treatment is indicated until 10 days after the onset of fever (earlier is better). The reason IVIG is effective is unknown. About 15% of patients fail to respond (defined as fever that persists or returns ::::: 36 hours after completion of the initial IVIG infusion). A second dose of iVIG is usually given. Other options include ste­ roids and infliximab but these are usually reserved for third-line therapy. Patients should be afebrile for at least 48 hours after com­ pletion of their IVIG infusion before discharge. Once the patient has been afebrile for 48 hours (some sources recommend waiting 2 weeks), aspirin can be decreased to a low dose for thrombosis prevention. Low-dose aspirin is continued for 6 to 8 weeks. It can be stopped if the echocardiogram at that time is normal. If aneu­ rysms are present, aspirin is continued indefinitely. Measles and varicella vaccines should be delayed for 1 1 months after admin­ istration of IVIG. When lose-dose aspirin is being taken, use of ibuprofen should be avoided. The child should receive a seasonal influenza vaccine because of the risk of Reye syndrome. •

Helpful Tip

� Treating a patient with Kawa sa ki d i sease before day of fever decreases the risk of coro n a ry d i sease. I 1 1r 1o

Treatment before day 5 is sometimes associated with i n c reased resista nce and need for retreatment.





I

Helpful Tip

Urethritis causes the sterile pyuria seen in Kawasaki disease.

1 1 r Urine collected by catheter or su prapubic aspirate will be normal. Order a bag or clea n-catch speci men.



Helpful Tip

�A

:5. FIGURE 7-6. Desq u a mation and erythema of the perineum i n a boy with Kawasaki d i sease. The rash freq uently sta rts i n the peri n e u m . (Reprod uced

c h i l d with fever lasti ng for 5 days or longer a n d

i1 1 r two or th ree c l i n ical criteria s h o u l d be eva l uated for incomplete Kawasaki d i sease. A diag nostic a lgorit h m

with permission from Goldsmith LA, Katz 51, G i l c h rest BA, et al, eds.

for t h i s cond ition w a s created i n 2004 b y the American

Fitzpatrick's Dermatology in General Medicine. 8th ed. New York, NY: McGraw­

Hea rt Association.

H i l l Education; 20 1 2, Fig. 1 67-3.)

CHAPTER 7







COLLAG E N VAS C U LA R A N D OTH E R M U LTI SYSTEM D I SO R D E R S

Helpful Tip

IVIG ca n cause hemolytic anemia. This has been described

1 1 1r in c h i l d ren treated for Kawasaki d i sease.

1 57

TA B L E 7-6 I NTE RNAT I O N A L LEAG U E O F

ASSOCIAT I O N S FOR R H E U M ATOLOGY ( I LAR) CLASS I F I CATI O N CRITERIA FOR J U V E N I L E I D I O PATH I C A RT H R I T I S (J IA)

Syste m i c JIA

Abrupt on set Fever, ra sh, a n d a rth ritis

A 9-year-old girl presents with fever, rash, and bilateral joint swelling of the wrists and knees. Once per day, she has a fever associated with a rash. Her temperature is 39°C { 1 02.2°F) . On exam, she appears ill and uncomfortable. She has tachycardia, tachypnea, hepatosplenomegaly, and warm, painful swelling of multiple joints. A rub is present on cardiac exam. Salmon­ colored macules are present on her back and abdomen. Question 4- 1 What is the diagnosis? A) Parvovirus B l 9 infection. B) Lyme disease. C) Systemic-onset juvenile idiopathic arthritis. D) Leukemia. E) None of the above. Discussion 4- 1 The correct answer is "C:' Juvenile idiopathic arthritis (JIA) is a group of chronic diseases causing arthritis of unknown etiology affecting children and adolescents younger than 16 years of age. Arthritis must be present in the same joint for more than 6 weeks to diagnosis JIA. Disorders described by the term JIA are grouped into seven categories. (See Table 7-6 and Figure 7-7.) In Lyme disease, the rash is annular, red, and expands to clas­ sically form a bull's eye (erythema migrans). Lyme arthritis dif­ fers from JIA in that joints have large effusions and minimal pain. Malignancy (especially leukemia) mimics JIA. Typically only one joint is involved and pain is severe. Bone pain and pain during the night are red flags for malignancy. Fever is not intermittent and arthritis is transient in parvovirus B l 9 infections. In this patient, the fever, rash, and arthritis should point you toward the diag­ nosis of systemic-onset JIA (sJIA) . Infection and malignancy are always included in the differential diagnosis for sJIA. •

Helpful Tip

:5.� Ea rly

Seros itis Orga nomegaly Lym p hadenopathy O n set: a ny age G i rl s and boys eq u a l l y affected O l i g oa rticu l a r JIA

O n set: 1 -2 yea rs of age G i rl s > boys ANA positive Persi stent: no c h a n g e i n n u m ber joi nts Exte nded: ove r t i m e i ncreases t o 4 o r more joi nts i nvo lved Polya rti c u l a r J I A (RF negative)

� 5 joi nts i nvolved

O n set: 1 -2 yea rs of age G i rl s > boys

Polya rti c u l a r J I A (RF positive)

� 5 joi nts i nvolved

O n set: adolescence G i rl s > boys

Pso riatic a rth ritis

Arth ritis a n d psori a s i s Dactylitis Nail pitt i n g or onycholys i s Psoriasis i n a fi rst-deg ree re lative

Enthesitis-re l ated a rth ritis

Arth ritis a n d enthesitis

Inflamm ation of the ten­ don, lig ament, or fascia attachment site to bone

O n set of a rth ritis in a boy aged 6 yea rs or older H LA-827 positive Sacro i l i itis or sacroi l i a c joint te nderness

i n the cou rse of J I A, sym ptoms of joint pain,

r1 1r swel l i ng, a n d stiffness a re present in the morning a n d after inactivity ( g e l phenomenon) a n d i m p rove with

Uveitis

activity. C h i l d ren may refu se to wa l k for severa l h o u rs

Fa m i ly h i story of a n kylos­ ing s pondyl itis, enthesitis­ re lated arthritis, sacro i l i itis with i n fl a m m atory bowe l d i sea se, Reiter synd rom e, or ac ute a nte rior uveitis i n a fi rst-deg ree re lative

in the morning.

Question 4-2 Which physical exam finding is NOT associated with sJIA? A) Hepatomegaly. B) Lymphadenopathy. C) Arthritis. D) Stomatitis. E) Pericardia! rub.

:::; 4 joi nts i nvolved

RF, rheu matoid factor.

1 58

MCG RAW- H I LL E D U C AT I O N S P E C I A LTY BOA R D REVI EW: P E D I ATRICS

she appears ill, with hepatomegaly and extensive bruis­ ing over her arms and legs, and is somnolent. Her labora­ tory testing reveals the following results (normal values in parentheses) : White blood cell count 3000/mm3 (5000- 1 5,500/mm3) Hemoglobin 8 mg/dL ( 1 1 .5- 1 5 g/dL) Platelet count 8 1 ,000/mm3 ( 1 50,000-400,000/mm3) Alanine aminotransferase (ALT) 75 units/L ( 1 0-25 units/L) Aspartate aminotransferase (AST) 70 units/L ( 1 0-30 units/L) Ferritin 4000 ng/mL (7- 140 ng/mL) Fibrinogen 90 mg/dL (200-400 mg/dL) Triglycerides 250 mg/dL ( < 1 50 mg/dL) INR 2.5 ESR 20 mm/h (0- 1 5 mm/h)

FIGURE

7-7. Psoriatic

a rth ritis.

Sausage-l i ke

thickening

over

the

i nterp h a l a ngeal joi nts a n d psoriasis o f t h e n a i l . (Reproduced w i t h permission from Wolff K, Johnson RA, Saaved ra AP, eds. Fitzpatrick's Color Atlas and Synopsis ofC/inical Dermatology. 7th ed. New York, NY: McGraw- H i l l Education; 2 0 1 3, Fig. 3 - 1 5.)

Discussion 4-2 The correct answer is "D:' Systemic-onset JIA (formerly Still disease) classically causes once-daily (quotidian) spiking fever accompanied by an evanescent (fades when afebrile) salmon pink-colored macular rash. Fever is typically intermittent, and children may look well between fevers. Arthritis most commonly involves the wrists, knees, and ankles. Hepatomegaly, splenomeg­ aly, and lymphadenopathy are common. Serositis, pericarditis, or pleuritis may be present with pericardia! or pleural effusions. Stomatitis is a feature of Kawasaki disease, not sJIA. Question 4-3 What is NOT a current medication used in the treatment of JIA? A) Methotrexate. B) Sulfasalazine. C) NSAIDs. D) Steroids. E) IVIG. Discussion 4-3 The correct answer is "E:' Medication treatment options include NSAIDs, disease-modifying antirheumatic drugs (DMARDs, including methotrexate) , biologic response modifiers (eg, tumor necrosis factor-alpha inhibitors, anakinra), and ste­ roids. Children with JIA should be under the management of a rheumatologist.

She presents to the emergency department with a 3-day history of fever, bruising, headache, and lethargy. On exam,

Question 4-4 How do you interpret her laboratory results? A) She has macrophage activation syndrome. B) Her results are normal. C) She has a flare of her sJIA. D) She has leukemia not sJIA. E) She has hemolytic uremic syndrome. Discussion 4-4 The correct answer is "A:' Macrophage activation syndrome (MAS) is a life-threatening complication of sJIA. It is similar to hemophagocytic lymphohistiocytosis with uncontrolled mac­ rophage and T-cell proliferation. Bone marrow aspirate shows phagocytosis of bone marrow cells by macrophages. Clinical symptoms overlap with other disorders. Distinguishing clini­ cal characteristics are bleeding and central nervous system dys­ function. In sJIA, the white blood cell count, platelet count, and ESR are elevated not decreased as in MAS. Other clues pointing to MAS include the low fibrinogen, highly elevated ferritin, and elevated triglyceride levels. Hemolytic uremic syndrome causes thrombocytopenia and anemia but not leukopenia or a coagu­ lopathy (prolonged INR) . Leukemia would not cause elevated triglycerides or, typically, a coagulopathy.



Helpful Tip

� All

=

r1 1 r joi nt

r h e u mato l o g i c swel l i n g .

d i seases

Syste m i c

can

l u pu s

present

with

eryt h e matos u s,

d e r mato myos itis, a rth ritis, a n d va scu l itis a re j u st a

few exa m ples. J o i n t swe l l i n g does n ot eq u a l j uven i l e arth ritis.



Helpful Tip

� Patients with J I A req u i re seria l eye exa ms to mon itor for r1 1 r uveitis. A positive ANA i ncreases the risk for developing

=

uveitis.

CHAPTER 7



COLLAG E N VAS C U LA R A N D OTH E R M U LTI SYSTEM D I SO R D E R S

1 59

6-year-old girl is at the clinic because of joint pain with fatigue. She denies morning stiffness. She has no diarrhea, but on review of symptoms you note abdominal pain inter­ mittently. The following laboratory tests are obtained: CBC, ESR, rheumatoid factor (negative) , and ANA (negative) . Her physical exam findings are normal. She has no synovitis sug­ gestive of inflammatory arthritis. A

Question 5-1 What is included in your differential diagnosis of arthralgia? A) Guaiac stool to look for inflammatory bowel disease. B) Celiac disease. C) Joint hypermobility. D) Hypothyroidism. Discussion 5-1 The correct answer is "B:' She has no elevation in ESR or diar­ rhea consistent with inflammatory bowel disease. Her joints are normal and not hypermobile. She has no findings of coarse hair and normal reported growth. The abdominal pain makes celiac disease as a cause of arthralgias more likely.

3-year-old girl with a new baby sister presents with recent refusal to walk. She is crawling now and refuses to stand by herself, wanting to be carried. She has a previous history of eczema. On exam, she has a rash over the knuckles, elbows, and knees; gum hyperemia; and nail beds that show periun­ gual erythema and swelling. Her laboratory testing shows the following results (normal values in parentheses) : A

ALT 200 units/L { 1 0-25 units/L) AST 350 units/L ( 1 0-30 units/L) Creatine kinase (CK) 600 units/L (20-200 units/L) Lactate dehydrogenase (LDH) 400 units/L ( 1 1 0-295 units/L) ESR 27 mm/h (0- 1 5 mm/h)

FIGURE 7-8. Dermatomyositis often i nvolves the hands as erythematous flat-topped

pa pules

over

the

knuckles

(Gottro n

sign).

Peri u n g u a l

tel a ngiectases a re a l so evident. (Reproduced w i t h permission from Kasper DL, Fa uci AS, Hauser S L et al: Harrison's Principles of internal Medicine, 1 9th ed. McGraw- H i l l Education, I nc., 20 1 5 . F i g u re 76e-64.)

milestones as the marker of weakness. Swallowing and breath­ ing may be affected. The rashes of JDM are very characteristic and may be mistaken for eczema. (See Figure 7-8.) They are ( 1 ) heliotrope rash-violet discoloration of the upper eyelids; (2) Gottron papules-pink, thickened, scaly plaques classically over the proximal interphalangeal joints and distal interphalan­ geal j oints; and (3) photosensitivity-shawl sign (a pattern of erythema that develops over the chest and neck when exposed to sunlight) . Diagnosis of JDM requires rash plus three of the following criteria: muscle weakness, elevated muscle enzymes, abnormal electromyogram (EMG), abnormal muscle biopsy, or MRI evidence of myositis. •

Helpful Tip

� The

=.

wea kness of J D M is symmetric and proxi mal,

i1 1r ca u s i n g d ifficu lty c l i m b i n g sta i rs a n d ra ising the arms

a bove the hea d . On exa m, patients have d ifficu lty l ifti n g their head, doing a sit u p, a n d sta n d i n g up from

the floor (positive Gower sign).

Urinalysis normal CBC normal Question 6- 1 What is the diagnosis? A) Malignancy. B) Discitis. C) Psoriasis. D) Juvenile dermatomyositis. E) Stress of a new sibling. Discussion 6-1 The correct answer is "D:' Juvenile dermatomyositis (JDM) is an autoimmune vasculopathy that causes proximal muscle weak­ ness and distinct rashes. Young children can present with loss of

A 9-year-old left-handed boy presents with fourth finger swelling of 2 months' duration, morning stiffness, and rash. He can no longer straighten the end of his finger. He says the skin on his arm has become thick and darker in color compared with the rest of his skin. On exam, he has a lesion with faint erythema and a blue border on his left biceps. The skin over his forearm, wrist, dorsum of hand, and fourth finger is hyperpigmented with scarring and atrophy. He has swelling of his left four metacarpophalangeal joint, and a flexion contracture of his proximal interphalangeal joint (PIP).

1 60

MCG RAW- H I LL E D U C AT I O N S P E C I A LTY BOA R D REVI EW: P E D I ATRICS

Question 7-1 What is the diagnosis? A) JIA. B) Linear scleroderma. C) Methicillin-resistant Staphylococcus aureus infection. D) Systemic lupus erythematosus. E) None of the above. Discussion 7-1 The correct answer is "B:' Juvenile scleroderma causes skin fibrosis and may be localized to the skin (morphea) or affect other organs (systemic sclerosis) . Localized scleroderma is usually self-limited and has many different subtypes. In localized scleroderma, an area of "waxy" swelling with a blue or red border develops into an indurated hypopigmented or hyperpigmented skin lesion with atrophy. In systemic sclerosis, extracutaneous involvement includes pulmonary and cardiac fibrosis, Raynaud phenomenon, renal artery hypertension, and gastrointestinal dysmotility. Heart failure from cardiopulmonary fibrosis is the most common cause of death. The patient has linear scleroderma, a type of localized scleroderma. Lesions can extend through the subcutaneous tis­ sues and muscle to the bone. Lesions crossing joints cause con­ tractures and limb-length discrepancies. Arthritis may be present in some children. (See Figure 7-9.)

};{ QUICKQUIZ What is NOT a clinical feature of sarcoidosis? A) Hilar lymphadenopathy. B) Pulmonary fibrosis. C) Uveitis. D) Erythema nodosum. E) Seizures. Discussion The correct answer is "E:' Sarcoid is rare in children and results in the formation of noncaseating (no necrosis) granulomas in multiple organs. Presentation varies by age. Older children have lung and lymph node involvement (hilar or mediastinal lymph­ adenopathy, or both, and pulmonary infiltrates). Rash, uveitis, and arthritis occur in children younger than 4 years of age. Definitive diagnosis is made by biopsy.

A 1 5-year-old male wrestler presents with a 4-month history of hip and buttock pain. He has trouble sleeping due to the pain. In the mornings, his low back is stiff but improves after he starts to move. On exam, he has pain upon palpation of the sacroiliac joint, decreased ability to bend forward, and

thickening in the left ankle with mild warmth. His hips are normal. Question 8- 1 Which of the following laboratory results is NOT expected with his diagnosis? A) Positive HLA-B27. B) Normal ESR. C) Normal CBC. D) Positive ANA. E) Negative rheumatoid factor. Discussion 8-1 The correct answer is "D:' He has juvenile ankylosing spondy­ litis (JAS) a type of spondyloarthropathy involving the spine and sacroiliac joints, causing pain and limited motion of the lumbar spine and sacroiliitis detectable on MRI. Ninety percent of patients with JAS who have sacroiliac disease are HLA-B27 positive. The juvenile spondyloarthropathies are a group of dis­ orders that cause enthesitis and arthritis. They are considered a subset of JIA (enthesitis-related arthritis; see Table 7-6) . Key clinical clues are:

FIGURE 7-9. Th is boy has l i near scleroderma, a form of local ized scleroderma (morphea) that causes skin fibrosis. Linear lesions can extend to the bone, causing contractu res of joi nts. He has a n indu rated, waxy, hypopigmented lesion extending from his thigh to his foot. (Reproduced with permission from Wolff K, Johnson RA, Saavedra AP, eds. Fitzpatrick's Color Atlas and Synopsis of Clinical Dermatology. 7th ed. New York, NY: McGraw-H i l l Education; 201 3, Fig. 1 4-S 1 .)



Boy older than 6 years



HLA-B27 positive



Arthritis of the lower extremities



Enthesitis of the Achilles tendon and plantar fascia



Back and sacroiliac joint pain and stiffness



Usually ANA and rheumatoid factor negative

CHAPTER 7





I



COLLAG E N VAS C U LA R A N D OTH E R M U LTI SYSTEM D I SO R D E R S

Helpful Tip

Watch out! Sacro i l i itis, spondyl itis, and a rthritis may be

1 1r the i n itial m a n ifestations of psoriasis a n d infl a m m atory bowel d isease.

A 1 0-year-old boy was diagnosed with erythema infec­ tiosum (fifth disease) 2 weeks ago after developing fever and the classic "slapped cheek" rash. Now he presents with a 1 -week history of joint pain when writing and playing the flute. He reports morning stiffness. On exam he has arthritis of the ankle and knee. You decide to treat him with NSAIDs and reevaluate as symptoms have been present for only 1 week. At follow up 6 weeks later, he is symptom free. Question 9- 1 What is the diagnosis? A) Postinfectious/reactive arthritis. B) Acute rheumatic fever. C) Septic arthritis. D) Oligoarticular juvenile idiopathic arthritis. E) Polyarticular juvenile idiopathic arthritis. Discussion 9- 1 The correct answer is "A:' Reactive and postinfectious arthri­ tis develop 1 to 4 weeks after a viral or bacterial infection. Most cases are preceded by gastroenteritis. Classically, reac­ tive arthritis (formerly known as Reiter syndrome) developed after an enteric or genitourinary tract infection. The classic triad of arthritis, urethritis, and conjunctivitis is uncommon in children. Many are positive for HLA-B27. Arthritis typi­ cally involves the large j oints of the legs, responds to NSAIDs, and resolves within weeks to months. Common pathogens in reactive arthritis include Salmonella, Shigella, Yersinia, Campy­ lobacter, Giardia intestinalis, and Chlamydia trachomatis. Com­ mon pathogens in postinfectious arthritis include parvovirus B l 9, rubella, varicella-zoster, herpes simplex, cytomegalovi­ rus, Ebstein-Barr virus, hepatitis B, adenovirus, enteroviruses, group A streptococcus, and mycoplasma. Differentiating between poststreptococcal reactive arthritis (PSRA) and acute rheumatic fever (ARF) is challenging. Some clinicians consider PSRA to be incomplete ARF and treat with penicillin prophy­ laxis. Unlike the arthritis of ARF, PSRA does not migrate and is less responsive to NSAIDs.

A 1 7-year-old adolescent girl presents with fatigue and widespread joint pain that is getting progressively worse. For the last year, her hands, knees, back, and ankles have ached constantly. The pain is worse with activities.

1 61

She can no longer participate in gym class and is constantly tired. On exam, she can hyperextend both elbows, lay her palms flat on the floor when standing without bend­ ing her knees, touch both thumbs to her forearms, and extend both pinky fingers to 90 degrees. Her Beighton score is 7/9. Question 1 0-1 What is NOT a recommended treatment for her condition? A) Reassurance. B) NSAIDs. C) Physical therapy. D) Narcotics. E) All of the above are recommended for treatment of her condition. Discussion 1 0-1 The correct answer is "D:' Her symptoms and a Beighton score of 6 or higher is consistent with benign j oint hyper­ mobility syndrome (BJH S ) . Physical therapy builds muscle strength to stabilize j oints. NSAIDs may be useful to treat pain. Symptoms improve with age and are not associated with long-term complications. People with BJHS are double j ointed and have musculoskeletal pain. The syndrome tends to run in families. Girls and younger children are most often affected as hypermobility decreases with age. BJHS is diag­ nosed clinically using the B eighton score. (See Table 7-7.) It is important to rule out other conditions associated with hypermobile j oints, such as Marfan syndrome and Ehlers­ Danlos syndrome.





I

Helpful Tip

I nfa nts v:'ith hypermo b i l ity may have d e layed g ross

1 1 r motor m i l estones.

TA B L E 7-7 B E I G HTON SCO R I N G SYSTE M F O R

D I AG N O S I N G B E N I G N J O I NT HYP E R M O B I L ITY SYN D RO M E

> 1 0 degrees o f hyperexte nsion of the knees

1 point for each side

> 1 0 degrees of hyperexte nsion of the el bows

1 point for each side

Pa ssive fl exio n of the t h u m b to forea rm

1 point for each side

Pa ssive exte n s i o n of the fifth fi n g e r > 90 deg rees

1 point for each side

To uch the fl oor with both pa l m s with knees stra i g ht Score � 6 i n d i cates hypermobil ity.

1 62

MCG RAW- H I LL E D U C AT I O N S P E C I A LTY BOA R D REVI EW: P E D I ATRICS

� QUICKQUIZ Which of the following statements about classic Ehlers-Danlos syndrome (EDS) is NOT true? A) It is caused by mutation in the genes ( COLSA l, COL5A2) encoding type V collagen. B) Patients can have associated cardiac defects. C) It is an autosomal recessive condition. D) Patients are prone to easy bruising. E) Diagnosis is based on clinical exam and family history. Discussion The correct answer is "C:' EDS is an autosomal dominant condition, not autosomal recessive. Classic EDS is an inher­ ited connective tissue disorder known for hyperelastic skin (stretches easily and snaps back) , poor wound healing with stretched scars, and j oint hypermobility. The skin is smooth, velvety, and fragile. Those affected bruise easily and are prone to j oint dislocation. Mitral valve prolapse and aortic root dilation are uncommon. Hernias and rectal prolapse may occur. It is one of six different types of EDS. (See Table 7-8 and Figure 7- 1 0 . )

A

� QUICKQUIZ Which is NOT a clinical feature of Marfan syndrome? A) Aortic root dilation. B) Ectopia lentis. C) Scoliosis. D) Pectus deformities (excavatum or carinatum) . E ) Hyperopia. B

TA B L E 7-8 D I AG N OSTIC CRITERIA FOR C LAS S I C

FIGURE 7-10. E h lers-Da nlos syndrome is an i n herited connective tissue d i sorder cha racterized by hyperextensible skin, poor wou n d hea l i ng, and

E H L E R S - DAN LOS SYN DROM E

joint hypermobil ity. (A) The skin stretches when p u l led then snaps back when

Major criteria

Hyperexte n s i b l e s k i n

criterion is the a b i l ity to passively flex the thumb to the forearm, a s shown

3 required

Wide, atropic scars

here. (Reproduced with perm ission from Fuster V, Wa l s h RA, Harrington RA,

J o i nt hypermobil ity (Beig hton score ;;::: 6; see Ta b l e 7-7)

Fig. 1 4-8A, B.)

M i no r criteria 7 required

released. (B) J o i nt hypermobil ity is measu red using a scori ng system . One

eds. Hurst's The Heart. 1 3th ed. New York, NY: McGraw-H i l l Education; 2 0 1 1 ,

S m ooth, velvety s k i n M o l l uscoid pse udotu mors S u bcuta neous spheroids J o i nt com p l icati ons (d islocations, spra i n s, pes p l a n us) M u scle hypoto n i a , g ross motor delay Easy bru i s i n g Herni as, recta l prola pse, cervical i n sufficie ncy Fa m i ly h i story

Discussion The correct answer is "E." Myopia, not hyperopia, is associ­ ated with Marfan syndrome. Marfan syndrome is an auto­ somal dominant connective tissue disorder caused by a mutation in the FBNl gene. It is important to recognize the condition to monitor for its cardiac complications. Patients are tall, thin, and have long extremities (arm span > height), pectus deformities, ligamentous laxity, flat feet (pes planus) , and arachnodactyly (long, thin fingers and toes ) . Th e face is long and narrow with deep- set eyes, high-arched palate,

CHAPTER 7



COLLAG E N VAS C U LA R A N D OTH E R M U LTI SYSTEM D I SO R D E R S

1 63

Question 1 1 - 1 What i s the diagnosis? A) Functional joint pain. B) Plica syndrome. C) Chondromalacia patella. D) Osgood-Schlatter disease. Discussion 1 1 - 1 Th e correct answer i s ''A:' This i s not plica syndrome a s the patient has no joint snapping on exam. It is not chondromalacia patella as there is no pain to lateral compression of the patella. This is not Osgood-Schlatter disease as he does not have tibial pain to palpation. Sometimes there is not a good anatomic or pathologic correlate to j oint pain, and we call it "functional joint pain:' No treatment is necessary, but people commonly use NSAIDs to treat the discomfort.

B I B LIOGRAPHY

F I G U R E 7-1 1. This teenage g i rl has M a rfa n syndrome. She has long l i m bs a n d fingers, scol iosis, a n d g e n u va l g u m (knock knees). (Reprod uced with permission from Va l l e D, Beaudet AL, Vogel ste i n B et al: The Online Metabolic and Molecular Bases of Inherited Disease, Bed. McGraw- H i l l Education, I nc; 2 0 1 4. Fig 206-2.)

dental crowding, and a small chin. (See Figure 7- 1 1 . ) Marfan syndrome is associated with aortic dilation, mitral or tricus­ pid valve prolapse (or both ), and an increased risk for aortic dissection. Spontaneous pneumothorax may occur as well.

A 1 3-year-old adolescent boy who plays basketball pres­ ents with complaints of knee pain with running. He does not have stiffness and his knees do not swell. His symp­ toms seem to occur after activities. Physical exam is normal, with a negative lateral patellar compression exam, no crepitus or snapping of the knees, and no tibial pain to palpation.

Berard R, Whittemore B, Scuccimarri R. Hemolytic anemia following intravenous immunoglobulin therapy in patients treated for Kawasaki disease: A report of 4 cases. Pediatr Rheumatol. 2 0 1 2; 1 0 ( 1 ) : 10 . Burke RJ, Chang C. Diagnostic criteria of acute rheumatic fever. Autoimmun Rev. 2014; 1 3 (4-5):503-507. Cassidy JT, Laxer RM, Petty RE, Lindsley CB. Textbook of Pediatric Rheumatology. 6th ed. Philadelphia, PA: Saunders; 20 1 1 . Dietz HC. Marfan syndrome. (Published April I S , 200 1 [Updated June 12, 2014] ) . In Pagon RA, Adam MP, Ardinger HH, et al, eds. GeneReviews [Internet] . Seattle, WA: University of Washington; 1 993-20 1 5 . http://www. ncbi.nlm.nih.gov/books/NBK 1 335/. Accessed February 19, 20 1 5. Gurion R, Lehman TJ, Moorthy LN. Systemic arthritis in children: A review of clinical presentation and treatment. Int f Inflamm. 2 0 1 2;20 12:271 569. Kliegman RM, Stanton BF, St. Geme JW, et al. Nelson Textbook of Pedaitrics. 1 9th ed. Philadelphia, PA: Saunders, 20 1 1 . Lanzkron S. Henoch-Schonlein purpura. [First Consult] . (Published October 3, 2 0 1 2 [Updated October 2, 2012] ) . Accessed February 2 3 , 20 1 5. Malfait F, Wenstrup R, DePaepe A. Ehlers-Danlos syn­ drome, classic type. (May 29, 2007 [Updated August 1 8, 20 1 1 ] ) . In Pagon RA, Adam MP, Ardinger HH, et al, eds. GeneReviews [Internet] . Seattle, WA: University of Washington; 1 993-20 1 5 . Available from http://www. ncbi.nlm.nih.gov/books/NBK 1 244/. Accessed Feburary 20, 20 1 5 . Newburger JW, Takahashi M , Gerber MA, et al. Diagnosis, treatment, and long-term management of Kawasaki disease: A statement for health professionals from the Committee on Rheumatic Fever, Endocarditis and Kawasaki Disease, Council on Cardiovascular Disease in the Young, American Heart Association. Circulation. 2004; 1 1 0 ( 1 7) :2747-277 1 .

1 64

MCG RAW- H I LL E D U C AT I O N S P E C I A LTY BOA R D REVI EW: P E D I ATRICS

Petty RE, Southwood TR, Manners P, et al; International League of Associations for Rheumatology. International League of Associations for Rheumatology classification of juvenile idiopathic arthritis: Second revision, Edmonton, 200 1 . J Rheumatol. 2004;3 1 (2):390-392.

Prakken B, Albani S, Martini A. Juvenile idiopathic arthritis. Lancet. 20 1 1 ;377(9783 ) : 2 1 38-2 149. Yazdany J. Systemic lupus erythematous. [First Consult] . (Pub­ lished November 1, 20 1 3 [Updated October 3 1 , 20 1 3 ] ) . Accessed February 2 3 , 20 1 5.

Critical Care

8

As h l ey Loo m i s a n d N iyati Patel

A 4-month-old infant arrives in the emergency department with a 2-day history of fever, decreasing interest in oral (PO) intake, and little to no urine output today. Mother notes that the patient is more lethargic today. Upon examination you note a well-nourished infant who arouses and moans to your exam but quickly falls back asleep in his mother's arms. Vital signs are heart rate (HR) : 1 70 beats per minute (bpm}, respi­ ratory rate (RR} : 60 breaths per minute, blood pressure (BP} : 80/60 mm Hg, temperature (T} : 38°C ( 1 00.4°F) . Multiple attempts to obtain intravenous (IV) access are unsuccessful. Question 1 - 1 What is the most appropriate next therapy? A) Administer PO acetaminophen for fever. B) Administer isotonic PO fluids. C) Obtain intraosseous (IO) access and administer a 20-mL/kg isotonic fluid bolus via IO. D) Administer PO broad-spectrum antibiotic therapy. E) Obtain an ultrasound of his abdomen. Discussion 1 - 1 The correct answer is "C:' This patient is in compensated shock. Recognize that blood pressure is often the last vital sign to decline in pediatric shock. (See Figure 8- 1 .} Shock is a syn­ drome characterized by inadequate oxygen delivery to meet metabolic demands, often resulting in acidosis, organ dysfunc­ tion, and death if not treated adequately and efficiently. Recall that when treating shock, one should obtain IV/IO access and give fluids within the first 5 minutes of the patient's arrival.

While examining your patients prior to rounds in the pedi­ atric intensive care unit (PICU), you note that a 3-year-old with otherwise normal vital signs for age has a capillary refill of 3 seconds.

Question 2-1 Which of the following is most accurate? A) Capillary refill is unaffected by the use of vasopressors. B) Environmental temperature can affect capillary refill. C) Capillary refill greater than 3 seconds is always indicative of abnormal perfusion. D) Inter-observer reliability does not affect measurement of capillary refill. E) I have never heard of capillary refill. Discussion 2-1 The correct answer is "B:' Although capillary refill is a common clinical test used in pediatrics, the inter-observer variability makes it a very insensitive tool. Studies have demonstrated a tremendous amount of inconsistency in how this test is per­ formed, thus making the interpretation of the results quite difficult. It is known that certain vasopressors-specifically those that are pure vasoconstrictors (eg, Neo- Synephrine) ­ can affect capillary refill time. Additionally, environmental temperature can affect capillary refill time in an otherwise perfectly healthy individual. Therefore, while traditionally a capillary refill of greater than 3 seconds is considered abnor­ mal, it may not always indicate abnormal perfusion. If you read the preceding explanations, option "E" is no longer applicable.

You are admitting a 23-month-old girl to the pediatric PICU secondary to new-onset gait abnormality and recent early morning headaches associated with vomiting. As you perform your initial examination, you note that she is arousable, but irritable, and her respiratory rate is 60 bpm, which is a new finding according to the emergency medical services (EMS) team that transported her to your institution. 1 65

MCG RAW- H I LL E D U C AT I O N S P E C I A LTY BOA R D REVI EW: P E D I ATRICS

1 66

1 60

us to the conclusion that this child, in compensating for the lesion-induced increased intracranial pressure, is limiting blood flow to the brain by hyperventilating and causing vasoconstric­ tion of the cerebral vasculature.

.----- Vascular resistance

e

1 20

E

0 0

0 E

80

Q) 0

(i)

11..

A 3-year-old boy is in the operating room being prepared for a tonsillectomy and adenoidectomy after a sleep study demonstrated significant sleep apnea. Shortly after the surgery has begun the anesthesiologist notes an end-tidal C02 of 90 mmHg, HR 1 75 bpm, and T 40°C ( 1 04°F) .

40

Blood pressure 25

50

75

Percent of volume deficit FIGURE 8-1. Hemodyna m ics in pediatric shock. I n compensated shock, a c h i l d may conti n u e to m a i nta i n a norma l blood pressurefo r a g e with adequate perfusion to the bra i n a n d heart provided by i ncreased cardiac output-as a resu l t of tachycardia, a n d i n c reased system i c vascu l a r resista nce. Altho u g h blood p ressu re rem a i n s n o r m a l , other o rg a n s m a y be hypoperfu sed a n d i f left u ntreated, the patient cou ld prog ress t o deco m pensated shock.

Question 3-1 What is the most likely explanation for this new-onset tachypnea? A) Impending coma from elevated intracranial pressure (ICP). B) Respiratory viral illness. C) Foreign body aspiration secondary to impaired gag reflex. D) Anxiety secondary to increased personnel and PICU setting. E) Dehydration secondary to a history of emesis. Discussion 3-1 The correct answer is "A:' This patient's constellation of symp­ toms, including gait abnormality, headaches, and vomiting, should prompt the reader to think of a mass-occupying lesion. Understanding of the Monro-Kellie doctrine (Figure 8-2) leads ICP

t

E

Q)

:::J

E

0

:::J



en :::J 0 c



t

Q)

LL.

(f) 0

Arterial volume -Brain Normal

> en :::J 0 c



Question 4- 1 All of the following are associated with increased risk for development of these clinical changes EXCEPT: A) Family history of malignant hyperthermia. B) Exposure to volatile anesthetic agents. C) Treatment with succinylcholine. D) Dantrolene therapy. E) History of muscular dystrophy. Discussion 4- 1 The correct answer is "D:' The boy in this case is demonstrating clinical signs consistent with malignant hyperthermia (MH). MH has a n autosomal dominant inheritance pattern with vari­ able penetrance; thus, a family history of MH is a known risk factor. MH typically occurs after a trigger, most commonly by volatile anesthetic agents or succinylcholine. Clinical signs consistent with MH include rapid increase in end-tidal C0 2 , tachycardia, hyperthermia, hyperkalemia, myoglobinuria, muscle rigidity, and, if not treated adequately, disseminated intravascular coagulopathy, cardiac failure, and renal failure. Patients with existing muscular dystrophy are at higher risk for development of this syndrome. Dantrolene therapy, which is used to treat MH, inhibits release of ionized calcium by the sarcoplasmic reticulum, thus interfering with muscle con­ traction. The use of dantrolene does not increase the risk of developing MH.

LL.

(f) 0

Mass Arterial volume - srarn-

_ ....

Compensated normai iCP

/ /

/ //

/

Mass / / // / /

//

Arterial volume B rain

Decompensated increased, ICP

FIGURE 8-2. The M o n ro-Ke l l i e doctrine states that the cra n i a l va u l t is a closed, rigid box with a fixed vol u m e. An i ncrease in the vol u m e of a ny of its components-bra i n, blood, or cerebrosp i n a l fl u id-wi l l result in i n c reased pressure on the va ult and displacement of one of the other components.

A 5-year-old previously healthy boy is seen in the emergency department with complaints of decreased activity tolerance and breathlessness for the past 2 days. He has not been febrile today; however, he recently recovered from what his mother thought was the flu. His vital signs are HR 140 bpm, RR 40, BP 70/50 mm Hg, and T 37SC (99SF). The emergency department physician obtains IV access, sends off laboratory studies and a blood culture, administers a total of 40 mL!kg of isotonic fluid, and administers broad-spectrum IV antibi­ otics. The patient is now noted to be more tachycardic, with increased work of breathing, and pulses that are difficult to palpate.

CHAPTER 8

Question 5-1 What is the most likely cause of this child's decompensation? A) Septic shock. B) Viral bronchiolitis. C) Bacterial pneumonia. D) Systemic inflammatory response syndrome (SIRS). E) Congestive heart failure. Question 5-2 Which of the following statements regarding this child's diagnosis is most accurate? A) A 1 2-lead ECG will confirm the diagnosis. B) An elevated C-reactive protein will confirm the diagnosis. C) A stat 2-dimensional echocardiogram will confirm the diagnosis. D) The patient is in fluid refractory distributive shock. E) The patient is in primary respiratory failure.



CRITICAL CARE

1 67

the diagnosis of cardiogenic shock is suspected, a 2-dimensional echocardiogram is needed to confirm the diagnosis. This will help determine if the patient is suffering from impaired systolic or dia­ stolic function (eg, myocarditis), or tamponade physiology (eg, pericardia! effusion). With impaired systolic function epinephrine can augment contractility. Recall that epinephrine does increase myocardial oxygen demand and has the potential to lead to myo­ cardial ischemia and infarction. If impaired diastolic function is present, milrinone can help relaxation and filling. Recall that mil­ rinone will also lead to vasodilation and must be used cautiously in the hypotensive patient. The triad of tachycardia, jugular venous distention, and narrowed pulse pressure is tamponade physiology until proven otherwise. The definitive diagnosis is clinically made, and the therapy of choice is echocardiogram-guided pericardia­ centesis if a pericardia! effusion is to blame. Aggressive diuresis in the setting of tamponade physiology could result in significantly diminished systemic venous return and cardiac arrest.

You decide to admit this patient to the PICU. Question 5-3 Which of the following is the best initial choice of therapy? A) Continue to administer another 20-mL/kg bolus followed by a vasopressor if hemodynamics do not improve. B) Initiate bronchodilator therapy and systemic steroids. C) Initiate epinephrine and milrinone infusions with conserva­ tive fluid replacement (5- 1 0 mL!kg) as needed. D) Initiate vasopressin infusion alone. E) Administer supplemental oxygen.

After transfer to the PICU, prior to initiating therapy the child develops jugular venous distention, muffled heart sounds, palpable liver edge, worsening tachycardia, and BP of 60/50 mm Hg. Question 5-4 What is the best next step in management? A) Augment preload and prepare for emergent pericardiocentesis with echocardiogram guidance. B) Aggressive diuretic therapy. C) Initiate a milrinone infusion to improve ventricular filling. D) Initiate an epinephrine infusion to help improve cardiac output. E) Perform endotracheal intubation. Discussion 5-1 th roug h 5-4 The correct answers are "E;' "C:' "C:' and ''A:' When approach­ ing a patient in shock, remember to continually reassess after each therapy. Although fluid resuscitation is the appropriate initial management for any patient in shock, the reassessment helps to determine what type of shock the patient is experiencing. After fluid resuscitation, the presence of rales, hepatomegaly, worsen­ ing tachycardia, and worsening perfusion are all signs consistent with cardiogenic shock. In patients with septic shock or systemic inflammatory response syndrome (SIRS), one would not expect a worsening clinical status following aggressive fluid resuscita­ tion. In a patient with viral bronchiolitis or bacterial pneumonia, work of breathing may continue to progress with fluid resuscita­ tion; however, worsening perfusion would not be expected. Once

A 13-year-old girl with a past medical history significant for school avoidance is brought to your office by her mother. The girl states that she is "fine;' but Mom states that she has not seemed like herself for the past 4 days. Mom reports that her daughter has been sleeping much more than usual, her appetite is much less than normal, and she has had occasional emesis. Mom also notes that the girl has intermittently been "talking nonsense:' Physical exam reveals the following: oral T 38°C ( 100.4°F), HR 85 bpm, RR 18, BP 100/65 mm Hg, and oxygen saturation (SpO) of 100% on room air. Your patient looks down at the floor and will not engage with you. Her heart and lung sounds are normal. When you palpate her abdomen, she yelps when you feel her right upper quadrant and bats your hand away. With repeat palpation of that area, you note guard­ ing without rebound and a firm liver edge 1 em below the right costal margin. Bowel sounds are present and the remainder of her abdomen is nontender. On neurologic exam, cranial nerves II through XII are intact. Reflexes are brisk throughout, and strength is 5/5 in all extremities. When you ask her to touch her nose and then your finger, she touches her right eye and then misses your finger completely. At the end of your exam, she is mumbling unintelligibly and then lies down on the exam table. Question 6-1 Your next best step is: A) Reassure her mother that this is normal early teenage behavior. B) Schedule an appointment for her next month with your child psychiatry colleague. C) Obtain a chest X-ray and clean catch urinalysis. D) Order a CBC with white blood cell count differential, elec­ trolytes, BUN, creatinine, glucose, calcium, magnesium, phosphorus, AST, ALT, GGT, alkaline phosphatase, total and direct bilirubin, PTT, INR, and fibrinogen. E) Schedule her for an abdominal ultrasound next week.

1 68

MCG RAW- H I LL E D U C AT I O N S P E C I A LTY BOA R D REVI EW: P E D I ATRICS

Discussion 6-1 The correct answer is "D:' This girl's history of somnolence, anorexia, emesis, and mental status changes indicate that an immediate diagnostic workup is required. Abdominal pain, hepatomegaly, brisk reflexes, and spatiotemporal disorienta­ tion are signs consistent with hepatic dysfunction necessitating laboratory evaluation. These subtle signs are the most common presentation of fulminant hepatic failure in children. Jaundice may be present or absent depending on the etiology of the liver disease. "Classic" findings of liver failure in adults, such as a history of drug or alcohol use, ascites, spider angiomata, palmar erythema, and asterixis, occur very late in the course of pedi­ atric patients following progression to chronic liver disease or are absent altogether. In infants, irritability, failure to thrive, a high-pitched cry, and a change in sleep rhythm may be the only symptoms noted.

The girl's laboratory results return and demonstrate normal blood count and differential, electrolytes, BUN, creatinine, calcium, magnesium, and phosphorus. Her glucose is low at 70. Her AST and ALT are significantly elevated at 3523 and 4278, respectively. GGT and alkaline phosphatase are normal. Total bilirubin is slightly elevated at 3.2, with a direct fraction of 0.2. PTT is normal, INR is prolonged at 2.6, and fibrinogen is normal. Upon further questioning when discussing these abnormalities with the girl and her mother, she admits that she ingested a bottle of pills in a suicide attempt 4 days ago. Question 6-2 Which of the following substances could account for her cur­ rent clinical condition? A) Acetaminophen. B) Aspirin. C) Diphenhydramine. D) Prenatal vitamins. E) None of the above. Discussion 6-2 The correct answer is "A:' Acetaminophen ingestion is the most common etiology of acute hepatic failure in children and adoles­ cents in the United States. If caught early and treated with inten­ sive medical therapy, survival and recovery are good. However, some patients have severe damage by the time the ingestion is discovered or progressively deteriorate and ultimately need liver transplantation. Since it has been 4 days since this girl's suicide attempt, it is too late for N-acetylcysteine treatment, and she should be transferred to a hospital with a PICU and solid organ transplant capability.

A 3-year-old boy with no significant past medical history is admitted to the PICU in respiratory distress secondary to a presumed viral bronchiolitis. Viral studies have been sent but are still pending. Upon arrival to the PICU he is placed

on continuous positive airway pressure (CPAP) ; however, despite this intervention he continues to have subcostal and supraclavicular retractions, grunting, and nasal flaring. His nurse is concerned that he seems more lethargic and recently began "head bobbing?' As the PICU team prepares to intu­ bate and mechanically ventilate the child, the attending phy­ sician asks you what size endotracheal tube you would like for this child. Question 7- 1 Assuming this child has a normal airway the best answer is: A) 2.0 uncuffed tube. B) 2.0 cuffed tube. C) 3.0 uncuffed tube. D) 3.0 cuffed tube. E) 4.5 cuffed tube. Discussion 7-1 The correct answer is "E:' The traditional formula used to determine the internal diameter of the endotracheal tube is age based: (age + 1 6)/4. For the patient in this vignette, (3 + 1 6)/4 = 4. 75. Thus, the best choice of the endotracheal tube sizes listed above is the 4.5 cuffed tube. The length-based Braslow tape recommendations are also an acceptable guide to help deter­ mine appropriate size. Cuffed tubes in a hospital setting are generally regarded as safe outside of the neonatal age group. Additionally, patients with some disease processes ( eg, poor lung compliance, elevated airway resistance, large glottic leak) may have improved mechanical ventilation with a cuffed tube.

The pediatric resident gives the patient ketamine, midazolam, and rocuronium as induction medications. He then places the endotracheal tube and attaches a carbon dioxide (CO) detector to the end of the tube. After several bagged breaths the color on the detector turns yellow. Upon auscultation, the resident notes that breath sounds are asymmetric, with right greater than left. Question 7-2 The most likely reason for this is? A) Esophageal intubation. B) Tracheal intubation. C) Right mainstem intubation. D) Left mainstem intubation. E) Resident inexperience; get the attending. Discussion 7-2 The correct answer is "C:' The persistent color change on the C0 2 detector indicates that the endotracheal tube is not in the esophagus. With appropriate tracheal intubation, one would expect to see color change on the C0 2 detector in addition to hearing bilateral air entry over both lung fields. The presence of louder breath sounds over the right lung field than the left is pathognomonic for a right mainstem intubation. The right main bronchus branches off at a less acute angle from the trachea than the left, making it the more likely bronchus to be intubated if the endotracheal tube is advanced too far.

CHAPTER 8



Helpful Tip

� End-ti d a l

C0

d etectors can be u s e d t o verify the

r1 1r position of the2 endotrachea l tu be. Remem ber, "ye l l ow

is m e l l ow, purple is poor." If it stays purple, it's l i kely

you r tube is not i n the a i rway.

The respiratory therapist adjusts the endotracheal tube to the appropriate position as confirmed by chest X-ray. Before leaving the room, the bedside nurse calls your attention to the patient's rising heart rate and declining blood pressure. His saturations remain at 98% and you continue to hear sym­ metric breath sounds. Question 7-3 What is the next best step in management? A) Naloxone bolus to treat narcotic side effects. B) 20 mL!kg isotonic fluid bolus for impaired venous return from elevated intrathoracic pressures. C) Needle decompression of a tension pneumothorax. D) Dopamine infusion for septic shock. E) Bolus stress-dose steroids for adrenal suppression. Discussion 7-3 The correct answer is "B:' Upon intubation of a patient, be aware of the changes that occur when going from negative pressure ven­ tilation to positive pressure ventilation. The overall pressure in the intrathoracic cage increases with lung expansion and can directly impair systemic venous return, especially in a patient such as this one, who has likely had a fair amount of insensible water losses with tachypnea and increased work of breathing. The patient did not receive any narcotics prior to intubation, so naloxone will do nothing to reverse his hypotension. The patient continues to have adequate Sp0 2 and symmetric breath sounds, making a tension pneumothorax very unlikely. Although the patient may have a SIRS response to his virus, the first-line treatment would not be a pressor but fluid, making option "D" incorrect. Lastly, while some drugs ( eg, etomidate, chronic ste­ roid therapy) can cause adrenal suppression, there would be no reason to assume this in a previously healthy child.

A mother brings her 5-year-old daughter to your clinic with significantly increased work of breathing for several days. When you ask the nurse to obtain a pulse oximetry reading she calls you to the bedside urgently as the child is difficult to arouse. Question 8- 1 What are the first steps you should take to manage this patient? A) Transfer to the emergency department immediately. B) Give 2.5 mg of nebulized albuterol. C) Assess airway and breathing and check for pulses.



CRITICAL CARE

1 69

D) Finish your well check; the child is just getting some much­ needed rest. E) None of the above. Discussion 8-1 The correct answer is "C:' Always remember the initial assess­ ment when approaching a difficult-to-arouse child. Your ini­ tial evaluation should include assessment of the patency of the airway, effectiveness of breathing, and adequate circulation as evidenced by the presence of a pulse. Think of your ABCs­ Airway, Breathing, and Circulation. Remember to palpate pulses centrally-brachial in infants, carotid or femoral in older children. Although this child may need to be transferred to the emergency department, it is not appropriate to move her in this state with no further intervention.

You note this child has a patent airway, continues to have some shallow, gasping breaths, and has a carotid pulse of 84 bpm. Question 8-2 What is the next best step in management? A) Call 9- 1 - 1 and activate the emergency response system. B) Deliver breaths using an appropriately sized bag-valve-mask (BMV). C) Begin chest compressions. D) Perform a blind finger sweep of the airway. E) Both A and B are correct. Discussion 8-2 The correct answer is "E:' Recall that pediatric arrests are primar­ ily respiratory in origin as opposed to their adult counterparts. Adequate breaths and oxygenation in an acutely decompensat­ ing child may be all that is needed to stabilize him or her. At this time someone should be assigned the task of activating the emergency response system, so they will be on their way while the team continues to work on the child. After providing good breaths and oxygenation be sure to reassess ABCs.

After delivering several adequate breaths by BVM, you reas­ sess the patient. She continues to be unresponsive, has ongo­ ing gasping breaths, and her carotid pulse is now 40 bpm and thready. Question 8-3 What is your next step? A) Continue BVM ventilation at a rate of 10 to 12 breaths per minute and await emergency medical services. B) Start chest compressions at a rate of 60 bpm to a depth of one quarter the anterior-posterior diameter of the chest. C) Start chest compressions at a rate of 80 bpm to a depth of more than one third the anterior-posterior diameter of the chest. D) Start chest compressions at a rate of 1 00 bpm to a depth of more than one third the anterior-posterior diameter of the chest. E) Continue BVM ventilation and find the office automated external defibrillator (AED).

MCG RAW- H I LL E D U C AT I O N S P E C I A LTY BOA R D REVI EW: P E D I ATRICS

1 70

Discussion 8-3 The correct answer is "D:' Symptomatic bradycardia (HR < 60 bpm in a child or < 100 bpm in a neonate) or pulselessness requires immediate initiation of chest compressions. Chest compressions must be hard-depth greater than one third the anterior-poste­ rior diameter of the chest with full recoil-and fast- 1 00 bpm. •



Helpful Tip

H u m the 70s tune "Stayi n' Al ive" a n d compress to the

1 1 1r beat t o g e t t h e rig ht rate. T h e ratio o f compressions

to venti lation if a n adva n ced a i rway is not present is 30:2 for single rescuer and 1 5:2 for two rescuers. Once cycl e of cardiopu l mo n a ry resuscitation (CPR) eq u a l s 1 5 or 30 com p ressions (whichever is a p p l ica b l e) a n d 2 breaths. Assessments a re d o n e every 5 cycles (about 2 m i n utes) . The u lti mate goal is to l i m it i nterru ptions of chest compressions. If a n adva n ced a i rway is present

i QUICKQUIZ Which is true regarding ventricular tachycardia? A) Nonsynchronized shocks should be delivered for pulseless ventricular tachycardia. B) Shocks should never be synchronized for ventricular tachycardia. C) AEDs can only be used in children older than 5 years of age. D) 4 J/kg should always be used per shock. E) You should have paid more attention during your pediatric advanced life support (PALS) training. Discussion The correct answers are "A and E:' In the setting of ventricular tachycardia, always check for a pulse first to guide what to do ! •

the rescuer s h o u l d aim for a goal of 8 to 1 0 breaths per

Pulse + poor perfusion perform cardioversion Synchronized shocks, start at 0.5 to 1 J/kg, and escalate to 2 J/kg. =



m i n ute. •

Pulseless perform defibrillation Nonsynchronized shocks, start at 2 J/kg, and escalate to 4 J/kg. =



While you and the nurse continue CPR your colleague brings the office AED and attaches the pad. After 5 full cycles of CPR you pause to assess the rhythm and see the rhythm below. A weak pulse is present.

AEDs can be used for defibrillation but not synchronized cardioversion. AEDs can be used for infants and children if a manual machine is not available.

Question 8-4 What is the best course of action? 0!?- . · 1 02 -1--1-74043-·-1 0G1 � I

-

�J\_

-

f/·&

+oa2 + 1 8 -

IF -: . .

u ��r-

f/-�-

H

.

-rl .

U-

-ff

u:

. -�t

A) Clear everyone from the patient and deliver a synchronized shock of 0.5 J/kg of body weight then immediately resume chest compressions. B) Clear everyone from the patient and deliver a nonsynchro­ nized shock of 2 J/kg of body weight then wait to analyze the rhythm. C) Obtain peripheral intravenous access (PIV) and deliver ade­ nosine 0. 1 mg/kg. D) Place a bag of ice to the child's face. E) Resume chest compressions; this is not a shockable rhythm. Discussion 8-4 The correct answer is "A:' The rhythm shown above is ventricu­ lar tachycardia. This can be distinguished from supraventricu­ lar tachycardia (SVT) due to a wide QRS complex. Options "C" and "D" are appropriate therapies to try if the patient had SVT. Ventricular tachycardia is a shockable rhythm, and the algo­ rithm for therapy is first a synchronized shock at 0.5 to 1 J/kg followed by immediate chest compressions, upon reassessment of the rhythm if the patient continues to have a nonperfusing ventricular tachycardia with a pulse increase to 2 J/kg synchro­ nized cardioversion.

A 17 -year-old previously healthy adolescent boy presents to the emergency department with a 1 -day history of headache, fever, myalgias, and decreased activity. He has not had urine output for the last 12 hours. Upon physical exam, tempera­ ture is 39°C ( 1 02.2°F), HR 1 25 bpm, RR 30, BP 75/45 mm Hg, and Sp02 is 94% on room air. He is unable to answer simple questions. His skin is warm to the touch, pulses are bounding throughout, and capillary refill time is instantaneous. Mul­ tiple small petechiae are noted on his chest and abdomen. Question 9- 1 Which of the following signs is/are NOT consistent with a diagnosis of sepsis? A) Fever > 38°C ( 1 00.4°F) . B) Tachycardia. C) Capillary refill time > 5 seconds. D) Capillary refill time < 2 seconds. E) All of the above are consistent with a diagnosis of sepsis. Discussion 9- 1 The correct answer is "E:' Children in septic shock may pres­ ent with signs of either "warm shock;' as manifested by the teenager with bounding pulses and flash capillary refill in the vignette, or "cold shock;' exemplified by diminished pulses and prolonged capillary refill time. International consensus defini­ tions for pediatric sepsis are shown in Table 8- 1 . The petechiae

CHAPTER 8

T A B L E 8-1 D E F I N IT I O N S O F P E D I ATRIC S E P S I S

I nfection

S u s pected or prove n i nfecti o n cau sed by a ny pathogen or a c l i n ica l syn d rome asso­ ci ated with a h i g h proba b i l ity of i nfecti o n

Syste m i c i nfl a m ­ matory response syn d rome (SI RS)

Req u i res 2/4 o f the fo l l owi ng criteria, 1 of which m u st be a bnormal te m pe rature o r a b n o r m a l leu kocyte cou nt: 1 . Core te m perature > 38SC (1 0 1 .3°F) or < 36°C (96.8°F) 2 . Tachyca rd i a (may be bradyca rd ia if < 1 yea r of age) 3 . Respi ratory rate > 2 sta n d a rd deviati o n s a bove normal fo r age or acute n eed fo r mecha n i ca l ve ntilation not re lated to n e u ro m uscu l a r d i sease or genera l a nesthesia 4. Le u kocyte co u n t el evated or depressed for age (not seco n d a ry to chemothera py) o r > 1 0% i m matu re neutro p h i l s

Sepsis

S I RS i n the presence o f or a s a res u l t o f s u s­ pected or prove n i nfecti o n

Severe seps i s

Sepsis + one o f the fo l l owi n g : 1 . Card i ovasc u l a r org a n dysfu nction 2 . Acute respi ratory d i stress syn d ro m e (ARDS) 3 . Two or more other org a n dysfu nctio n s

Septic shock

Sepsis + one o f the fol lowi ng despite recei pt of > 40 m L/kg isotonic fl u i d i n 1 hour: 1 . Persi stent hypote n s i o n 2 . N eed fo r vasoactive d rug t o m a i nta i n normal blood press u re 3. Two of the fo l l owi n g : a . U n exp l a i ned m eta b o l i c acidosis



CRITICAL CARE

1 71

Question 9-2 Within the first 5 minutes of the previous patient's arrival, which of the following actions should be taken? A) Begin high-flow oxygen. B) Establish IV access. C) Obtain a chest X-ray. D) Start dopamine at 5 meg/kg/min. E) Both A and B. Discussion 9-2 The correct answer is "E:' When managing a critically ill child, the ABCs should always be followed. This patient is maintaining his airway and breathing spontaneously but needs supplemental oxygen to improve his oxygen delivery. In order to improve his circulatory status, vascular access must be obtained immedi­ ately. The initial step in shock resuscitation, fluid bolus admin­ istration, may then begin. Question 9-3 Despite multiple attempts by nursing staff in the first 5 min­ utes, peripheral IV access cannot be obtained. The next best step is: A) Have another nurse attempt to place an IV. B) Place an intraosseous (IO) needle. C) Call the anesthesia service for a peripheral IV start. D) Call the surgical service for central line placement. E) Request transfer of the patient to the PI CU. Discussion 9-3 The correct answer is "B:' It is crucial that vascular access is obtained in the first 5 minutes of caring for a patient with shock of any type. An IO needle should be placed in a long bone (tibia, sternum, humerus) either manually or using an assistive device. It is appropriately anchored in the intraosseous mem­ brane when it feels stable to touch and flushes easily without extravasation into the tissue. It may then be used to draw blood and infuse fluids or medications, including those that require central access.

b. I ncreased l actate c. O l i g u ri a d . Capi l l a ry refi l l t i m e > 5 seco nds e. Core to peri phera l te m perature g a p > 30C

indicate thrombocytopenia from disseminated intervascular coagulation (DIC) . Finding petechiae below the nipple line is never a good sign. Ingrain this case in your head! Don't miss septic shock.





Helpful Tip

For sepsis, a n i nfection m u st be proven or suspected .

1 1 1r The i nfection does not have to be bacteria l !

Intraosseous access is successfully obtained and the nurse asks what type of fluid you would like to give through it. Question 9-4 Your response is: A) D 5NS at 1 00 mL!h. B) D 5NS 20 mL!kg over 15 min. C) NS 20 mL!kg over 5 min. D) NS 20 mL!kg over 1 h. E) D 5NS + 20 mEq/L KCl at 50 mL!h. Discussion 9-4 The correct answer is "C:' The established initial step in resus­ citation from shock of any type is fluid boluses of isotonic solu­ tion. Normal saline (NS) or lactated Ringer (LR) may be used, although NS is the safer choice in patients with poor urine output since LR contains potassium. Volume of 20 mL!kg is

1 72

MCG RAW- H I LL E D U C AT I O N S P E C I A LTY BOA R D REVI EW: P E D I ATRICS

given as fast as possible. Up to 60 mL!kg should be given in the first 15 minutes, watching for rales or hepatomegaly, which are signs of cardiac failure. Colloid solution infusion should be considered following the initial 60 mL!kg. Patients with sep­ tic shock may require up to 200 mL!kg of fluid resuscitation within the first hour to ensure adequate intravascular volume repletion. Considering his history and physical exam findings, you suspect that this patient has septic shock caused by Neisseria meningitidis. Question 9-5 The next best step in his evaluation should be: A) Obtain a computed tomography (CT) scan of the head look­ ing for cerebral edema. B) Obtain CBC, electrolytes, coagulation studies, venous blood gas, and lactate. C) Send a blood culture to the lab looking for gram-negative diplococci. D) No further diagnostic evaluation is needed before antibiotic therapy is begun. E) Perform a lumbar puncture to collect cerebrospinal fluid (CSF). Discussion 9-5 The correct answer is "D:' All of the other responses could be appropriate items in his diagnostic evaluation, but the most important next step in treating his septic shock is broad-spec­ trum antibiotic therapy. A third-generation cephalosporin such as ceftriaxone is appropriate treatment. Vancomycin may be added based on risk factors. Antibiotic administration should not be delayed to wait for diagnostic tests or studies. Protect yourself and others from infection by following droplet pre­ cautions and giving postexposure antibiotic prophylaxis when indicated.

Following oxygen administration, 10 placement, NS 60 mL/ kg, and antibiotics, the patient's heart rate has decreased to 1 1 5 bpm, Sp02 has improved to 1 00%, and respiratory rate is in the mid 20s. His BP remains low at 80/50 mm Hg, pulses are still bounding, and capillary refill time is less than 2 seconds. Question 9-6 The next best step in his management is: A) Norepinephrine 0.05 meg/kg/min via IO. B) Epinephrine 0.05 meg/kg/min via IO. C) Repeat NS 20 mL!kg bolus. D) Central line placement. E) Activated protein C. Discussion 9-6 The correct answer is ''A:' If shock persists despite 60 mL!kg of crystalloid fluid resuscitation, an inotrope should be added. Central line placement is not required as IO access may be used to infuse drugs centrally. This patient demonstrates signs

consistent with warm shock (hypotension, bounding pulses, flash capillary refill from inappropriate vasodilation), and nor­ epinephrine is the drug of choice to treat that entity due to its alpha-adrenergic vasoconstrictor effects. Dopamine could be an appropriate first-line inotropic agent to start in warm shock as well, since it may run safely through a peripheral IV line. Activated protein C was evaluated to see if its benefit to septic adults extends to children, but the trial was closed due to an increased risk of intracranial hemorrhage and poor risk-to­ benefit ratio.

You are paged to the emergency department stat to assist with a patient who is en route. He is a 4-year-old previously healthy boy who suffered a drowning in his babysitter's back­ yard swimming pool. It is estimated that he was submerged approximately 15 minutes, and the babysitter started rescue breathing and CPR after pulling him out of the water. He was intubated when EMS arrived and CPR is continuing in the ambulance. Upon his arrival to the emergency department, his rectal temperature is 30°C (86°F), pulses are palpable with ongoing compressions, and Sp02 is 90% with bagging occurring through the endotracheal tube (ETT) at a rate of 20 breaths per minute. His pupils are fixed and dilated, he does not respond to verbal stimulation, and he extends his extrem­ ities to painful stimuli. His Glasgow Coma Scale (GCS) score is 4. EMS reports that he received two code doses of epineph­ rine during the 7 -minute drive to the hospital through the 10 needle they placed at the scene. Question 1 0-1 Your management of this patient should include all of the following EXCEPT: A) Passive and active rewarming measures. B) Continued CPR, epinephrine doses through the IO needle, and defibrillation if necessary until return of spontaneous circulation or core temperature of 32-34°C (89.6-93 .2°F) . C) Ensuring appropriate ventilation and oxygenation via bag­ ging through the ETT or ventilator support. D) Discontinuation of support as resuscitation is futile. E) Immediate treatment of seizures or hypoglycemia. Discussion 1 0-1 The correct answer is "D:' Since this patient's core temperature is hypothermic, aggressive resuscitation efforts should con­ tinue regardless of whether the drowning occurred in warm or cold water. Hypothermia can have profound effects on car­ diac rhythm and contractility, which may improve rapidly once the body reaches a more normal temperature. Both pas­ sive (removal of wet clothes, warm blankets, increasing envi­ ronmental temperature) and active (warmed IV fluids, external radiant heat, ventilation with heated gas) rewarming should be

CHAPTER 8

instituted in this child. Resuscitation should continue following PALS guidelines until return of spontaneous circulation or suc­ cessful rewarming. Resuscitative efforts should cease if the child has a core temperature of 32-34°C (89.6-93.2°F) and still has not had return of spontaneous circulation. Maintaining appro­ priate ventilation and oxygenation and immediate treatment of conditions that worsen cerebral injury (seizures, hypoglycemia, hyperthermia) are paramount in the management of this criti­ cally ill child.

The boy has return of spontaneous circulation 20 minutes into his resuscitation. His core temperature has warmed to 34°C, his HR is 120 bpm, blood pressure is 80/40 mm Hg on an epinephrine continuous infusion, and oxygen satu­ rations are 94% with 60% oxygen delivered through the ventilator. He is not breathing over the set ventilator rate of 25 bpm, his neurologic exam has not changed, and he has not received any sedative or analgesia medication since he was in the ambulance. Right before he is moved to the PICU, his parents ask you about his chances for survival. Question 1 0-2 The best prognostic indicator of his future outcome is: A) His submersion time of 15 minutes. B) His age and sex. C) Spontaneous, purposeful movements within the next 24 hours. D) Absence of measured 0 2 saturation (SpO ) < 90%. E) Adequate blood pressure on � 1 vasopressor medication. Discussion 1 0-2 The correct answer is "C:' Significant improvement on serial neurologic exams in the first 24 to 48 hours following injury is the best predictor of prognosis following cerebral anoxia. Submersion times of less than 5 minutes usually result in sur­ vival and submersion times of greater than 25 minutes usually result in death, but outcomes vary widely following submer­ sion times of 5 to 25 minutes. Although boys drown more fre­ quently, there is no difference in outcome between the sexes. The incidence of drowning spikes in both the toddler and teenage years but age is not a prognostic factor for outcome. Neither level of hypoxia nor need for vasopressor support indicates future outcome.

During the boy's first 24 hours in the PICU, he remains hemodynamically stable on an epinephrine continuous infusion. His oxygenation and ventilation remain acceptable with moderate ventilator support. His core temperature has varied between 35°C and 36°C (95°F and 96.8°F) . His pupils remain fixed and dilated, he does not respond to voice, and he extends his extremities to pain. He has not received any sedation, analgesia, or muscle-relaxing drugs. Approxi­ mately 36 hours following his drowning event, he suddenly has a spontaneous rise in BP to 1 30/80 mm Hg with concur­ rent drop in HR to 65 bpm.



CRITICAL CARE

1 73

Question 1 0-3 These changes in vital signs are most likely due to: A) Worsening cerebral edema. B) Pneumothorax. C) Kink in his Foley catheter causing bladder distention. D) Improvement in his neurologic status. E) None of the above. Discussion 1 0-3 The correct answer is "A:' Brain cells respond to anoxic injury by swelling, leading to cerebral edema. This can then worsen the extent of the injury by increasing the amount of ischemic injury. This process occurs over the first 24 to 72 hours following the initial event. Hypertension, bradycardia, and irregular respira­ tions (which this patient cannot demonstrate as his respiratory pattern is set by the ventilator) form the Cushing triad, which is the classic presentation of increased intracranial pressure and impending brainstem herniation. There is no evidence that therapies to reduce intracranial pressure improve outcomes for drowning victims.

Over the next 24 hours, the boy's condition worsens. He remains hemodynamically stable and normothermic but he is no longer having any response to painful stimuli. His family relays to you that the PICU physicians have stated the need to perform brain death testing. Question 1 0-4 All of the following are criteria for brain death EXCEPT: A) Flaccid tone and unresponsiveness to deep painful stimuli. B) Pupils midposition or fully dilated with absence of light reflexes. C) Absence of corneal, cough, and gag reflexes. D) Presence of oculovestibular reflexes. E) Absence of spontaneous respiratory effort while on mechan­ ical ventilation. Discussion 1 0-4 The correct answer is "D:' The exam is consistent with brain death when oculovestibular reflexes are absent. This brainstem reflex is tested by elevating the head to 30 degrees and then irrigating each auditory canal with 10 to 50 mL of ice water, one ear at a time, waiting several minutes in between. Move­ ment of the eyes in any direction means that the reflex is pres­ ent. Further required elements of brain death testing are noted in Table 8-2. Each individual institution has its own specific policy on what exactly the process steps are to declare a patient brain dead.

The boy's initial brain death exam, including an apnea test, is consistent with brain death. His parents do not want to wait a full l2 hours to perform the second exam as they would like him to be an organ donor. They ask if there is any way the time between exams can be shortened.

1 74

MCG RAW- H I LL E D U C AT I O N S P E C I A LTY BOA R D REVI EW: P E D I ATRICS

TA B L E 8-2 R E Q U I R E M E NTS FOR ESTA B L I S H I N G

B RA I N DEATH

I rrevers i bl e and known cause of coma

Tra u m atic bra i n i nj u ry, a n oxic bra i n i nj u ry, identifi a b l e meta bo l i c d i sorder, other

Correction of factors that may i nte rfere with test i n g

Core te m peratu re > 35°C (95°F), S B P or MAP i n acce pt­ a b l e ra nge for age, no d rug effects (sedative, a n a l gesic, n e u ro m usc u l a r blocker) or meta bo l i c i ntoxication

Physical exa m

1 . Flaccid to ne a n d u n respon­ sive to deep painfu l sti m u l i 2. P u p i l s a re m i d position o r fu l ly d i l ated a n d without response to l i g ht 3. Co rneal, co u g h , a n d gag responses a re a bsent 4. Ocu l ovesti b u l a r refl exes a re a bsent 5. Sponta neous res pi ratory effo rt w h i l e on m ec h a n i c a l ve nti lation i s a bsent

Apnea test

1 . Patient re m a i n s hemody­ n a m i ca l ly sta ble a n d is not hypoxic d u ri n g test i n g 2. No res pi rato ry effo rt i s obse rved despite fi n a l PaC0 2 > 6 0 m m Hg or ;:: 20 m m Hg a bove base l i n e

Wa iting period between 2 exa m s performed by sepa rate q u a l ified atte n d i n g physicians

;:: 2 4 h o u rs i n patie nts w h o a re term newborns to 30 d ays old; ;:: 1 2 h o u rs i n patie nts 31 d ays to 1 8 yea rs old

MAP, mea n a rterial pressu re; PaC0 2, parti a l pressure o f carbon d i oxide i n a rteri a l blood; SBP, systol i c blood pressu re.

Question 1 0-5 Appropriate confirmatory testing that may shorten the inter­ val between brain death exams is: A) Electroencephalographic documentation of electrocerebral silence. B) Radionuclide cerebral blood flow determinations that docu­ ment the absence of cerebral blood flow. C) Computed tomography of the head demonstrating brain­ stem herniation. D) All of the above. E) Both A and B. Discussion 1 0-5 The correct answer is "E:' An electroencephalogram (EEG) demonstrating electrocerebral silence and cerebral blood flow

( CBF) studies documenting the absence of cerebral blood flow are both confirmatory tests for brain death declaration. Only one of them need be performed. These may be used when com­ plete brain death testing is not possible due to the underlying condition of the patient, for example hypoxia or hypotension during apnea testing. They may also function as a confirmatory test when family members request a shorter time frame between exams. In this case, the second clinical exam should still be per­ formed immediately following the EEG or CBF study and all must demonstrate no cerebral activity. Cerebral imaging is not a part of brain death declaration.

A previously healthy 1 5-year-old boy is brought to the emer­ gency department after being thrown from his snowmobile. He was wearing a helmet; opens his eyes to command; is ori­ ented to person, place, and time; localizes to painful stimuli; and is in mild respiratory distress. His vital signs are HR 1 1 5 bpm, RR 22, T 37SC ( 99SF ) , Sp02 94% on 1 00% Fi02 via nonrebreathing facemask. Question 1 1 - 1 What i s his Glasgow Coma Scale ( GCS ) score? A) 8 . B) 10 . C) 12. D) 13. E) 15. Discussion 1 1 - 1 Th e correct answer i s "D:' Remember, the GCS i s based o n three components: eye opening, verbal response, and motor response. (See Table 8-3.) The patient in this vignette loses 1 point for eye opening and 1 point for motor response, giving him a GCS score of 1 3 . Strongly consider intubation for any patient with a GCS of 8 or less, or with a rapidly declining GCS score. A modi­ fied GCS is available for preverbal infants and children.

�1 •

Helpful Tip

To remem ber the Glasgow Coma Sca l e, t h i n k:

1 r 4 eyes-4 poi nts for eye opening Jackson 5-5 poi nts for verba l V6 e n g i n e-6 poi nts for motor





Helpful Tip

Tra u matic bra i n i nj u ries ca n ca use abnormal postu ring

1 1 1r of t h e body sponta neously o r i n response t o sti m u l i . I n decorticate postu ring, the u pper body is fl exed . I n

d ecerebrate postu ring, t h e u pper body is exte nded. I n both, the l ower body is extended a n d i nterna l ly rotated. Both a re g rave signs. (See F i g u re 8-3.)

CHAPTER 8



CRITICAL CARE

1 75

TA B L E 8-3 G LASGOW CO MA SCALE

Behavior

Response

Response (preverbal children)

Score

Eye o pe n i n g

Sponta neously

Sponta neous

4

To co m m a n d/s peech

To sound

3

To pa i n

To pa i n

2

Verbal response

Motor response

N o response

No res ponse

Orie nted to person, pl ace, a n d time

Voca l izations, s m i le, i nteracts

5

Confused, but a n swers q uestions

Cri es, i rrita ble

4

I n a ppropriate wo rd s

Cries to pa i n

3

I ncom preh e n s i b l e sou n d s

Moa n s to pa i n

2

No response

No res ponse

1

Obeys co m ma n d s

Sponta neous m ovement

6

Loca l i zes t o pa i n

Withd raws fro m to uch

5

Withd raws fro m pa i n

Withd raws fro m pa i n

4

Abnormal flexion (decorti cate)

Abnormal flexio n (decorticate)

3

Abnormal exte nsion (decerebrate)

Abnormal exte n s i o n (decerebrate)

2

No response

No res ponse 15

Tota l

Upon close examination you note that the patient's right chest wall seems to move inward upon inspiration and out­ ward upon exhalation. Question 1 1 -2 What is the most appropriate course of action? A) Emergent intubation and mechanical ventilation. B) Strapping and splinting to stabilize the chest.

Decorticate or flexor posturing

C) Surgical consult and analgesia with a patient -controlled analgesic (PCA) pump. D) Initiating continuous positive airway pressure. E) Both C and D are correct. Discussion 1 1 -2 The correct answer is "D:' The patient in this vignette is suf­ fering from flail chest secondary to a blunt force from a motor vehicle accident. Flail chest typically occurs when there are several anterior and posterior fractures to multiple ribs, creat­ ing an unstable or "free" area of the chest wall that no longer contributes to lung expansion. This is clinically evidenced by the characteristic paradoxical movement of the flail area of chest-inward upon inhalation and outward upon exhalation. Although it is tempting to splint or strap the chest to stabilize it, this may actually inhibit chest wall movement and adequate inspiration and pulmonary toilet. While some cases of flail chest are severe enough to require mechanical ventilation, a patient in mild distress with a reasonable GCS should not require such invasive measures. Some cases may require surgi­ cal intervention, though the mainstay for flail chest therapy is analgesia and support with noninvasive positive pressure ventilation.

Decerebrate or extensor postu ri ng FIGURE 8-3. A b n o r m a l postu ring is s e e n w i t h severe tra u matic bra i n i nj u ries. Decorticate posturing: flexion a n d adduction o f t h e a r m s with extension a n d i nternal rotation of the legs. Decerebrate posturing: extension a n d i nternal rotation of the arms a n d legs. Decerebrate postu ring scores lower on the GCS. (Reproduced with permission from H a l l J B, Schmidt GA, Kress JP, eds. Principles and Critical Care. 4 ed. New York, NY: McGraw-H i l l Education; 20 1 5 . Fig. 86- 1 5.)

As you continue your evaluation you note the patient is tak­ ing shallow breaths, complaining of chest pain, and upon auscultation the right breath sounds are diminished as com­ pared with the left. HR is now 130 bpm, BP 125/80 mm Hg, RR 30, T 38°C ( 100.4°F), and arterial blood gases demonstrate 7.35/52/75/20 with a hemoglobin concentration of 8.4 g/dL.

1 76

MCG RAW- H I LL E D U C AT I O N S P E C I A LTY BOA R D REVI EW: P E D I ATRICS

Question 1 1 -3 What is the most appropriate next step? A) Increase the dose of the PCA to improve analgesia. B) Obtain stat an upright chest X-ray and stop your surgical colleagues before they leave the emergency department for their next consult. C) Perform stat needle decompression of the right pleural space. D) Administer broad-spectrum IV antibiotics. E) None of the above. Discussion 1 1 -3 The correct answer is "B:' This patient has clinical and labora­ tory signs consistent with a hemothorax: short shallow breaths, chest pain, tachypnea, and a low hemoglobin concentration. Hemothorax typically occurs in the setting of blunt force trauma. Diagnosis of a hemothorax can be confirmed by an upright chest X-ray. While the differential diagnoses includes pneumothorax, unless the patient is acutely demonstrating ten­ sion pneumothorax physiology with resultant hemodynamic compromise it is beneficial to obtain a chest X-ray prior to emergently decompressing the pleural space, as a hemothorax would likely not resolve with simple needle decompression. The primary mode of treatment for a hemothorax is a tube thoracos­ tomy with the potential for thoracotomy ifbleeding is significant or persistent. Transfusion of blood products may be necessary to correct coagulopathies and maintain adequate intravascular volume and oxygen delivery.

At 4:30 PM on a Friday afternoon, your nurse hears a phone message from the worried father of one of your long-term patients. He states that his child was transferred to the PICU earlier that day and the physicians there have mentioned it was due to ''ARDS?' He says that he has never heard of this before and asks you to call him to answer his questions about it. You recall that his previously healthy 2-year-old daughter was admitted to the hospital 2 nights ago by one of your part­ ners with a 3-day history of oral temperature 40°C ( 1 04°F) , cough, and increased work of breathing. The girl was hypoxic and was placed on oxygen via nasal cannula and started on antibiotics for community-acquired pneumonia. Before returning the father's call, you refresh your memory on acute respiratory distress syndrome. Question 1 2- 1 Th e review article that you read defines the consensus criteria for ARDS as all of the following EXCEPT: A) Acute onset ( < 7 days) of hypoxia from an identifiable insult. B) Frontal X-ray with diffuse infiltrates in one lung. C) Severe hypoxemia as identified by Pa0 / Fi0 2 < 200. D) Absence of left atrial hypertension. E) All of the above are consensus criteria for ARDS.

FIGURE 8-4. This rad iogra p h demonstrates typical fi nd i n g s i n acute respi ratory d i stress synd rome, i n c l u d i n g b i l atera l d iffuse a l veo l a r o pacities, with obscurement of the p u l m o n a ry vasc u l a r markings, and asymmetric consolidation.

Discussion 1 2- 1 Th e correct answer i s "B:' Th e definition o f acute respiratory dis­ tress syndrome (ARDS) requires diffuse bilateral infiltrates. (See Figure 8-4.) ARDS is a disorder marked by profound hypoxia and severely decreased lung compliance. An initial insult activates the inflammatory and coagulation cascades, which increases the usual pulmonary endothelial and epithelial per­ meability. This results in alveoli filling with protein-rich edema fluid. This leads to difficulty in oxygen transfer, inactivation of surfactant, and worsened ventilation-perfusion mismatch. Question 1 2-2 The article states that ARDS may arise from multiple etiolo­ gies. Which of the following is a possible inciting event that may lead to ARDS? A) Pneumonia (bacterial, viral, or fungal) . B) Sepsis. C) Aspiration. D) Multiple trauma. E) All of the above are possible inciting events that may lead to ARDS. Discussion 1 2-2 The correct response is "E". These are the most common causes of ARDS. Other entities that may injure the lung and result in ARDS include drowning, embolism, smoke inhalation, acute pancreatitis, blood product transfusions, drug overdose, and cardiac bypass. ARDS can be classified as primary or secondary etiologies. With primary etiologies (eg, pneumonia, lung con­ tusion), the lung injury classic to ARDS is secondary to direct

CHAPTER 8

insult to the lung parenchyma. In secondary etiologies ( eg, pan­ creatitis, sepsis), the injury is secondary to a systemic inflamma­ tory response. The distinction between primary and secondary ARDS can have implications on treatment and outcome. After your quick review, you return the father's call. He relays that his daughter was intubated upon her arrival to the PICU and is still requiring a high amount of oxygen despite several changes to her ventilator made by the pediatric intensivist. Dad is very scared and his first question to you is, "Can my daughter die from ARDS?" Question 1 2-3 You reply: A) No, ARDS is not lethal. B) Yes, ARDS can be fatal. C) Complications such as sepsis and multiorgan system failure increase the risk for death. D) Only children with chronic medical conditions die from ARDS. E) Both B and C. Discussion 1 2-3 The correct answer is "E:' Significant mortality occurs from ARDS despite advances in therapeutic options. Complications increase this risk for death. Both previously healthy children and those with chronic medical conditions are at risk to die from ARDS. Dad then asks, "What methods will the doctors use to increase her chance of survival?" Question 1 2-4 You answer: A) List her for lung transplant immediately. B) Use lung protective strategies such as low tidal volume ven­ tilation, positive end-expiratory pressure (PEEP) , and per­ missive hypercapnia. C) Start her on the medicine that treats ARDS. D) Perform radiation therapy directed at her lungs. E) None of the above. Discussion 1 2-4 The correct answer is "B:' These strategies prevent ventilator­ induced lung injury due to barotrauma, atelectrauma, and volu­ trauma. Lung transplant and radiation therapy are not used as therapeutic measures in ARDS. Unfortunately, there is no med­ icine that treats ARDS. All of the methods used to minimize damage and improve survival are supportive.



CRITICAL CARE

1 77

B I B LIOGRAPHY

Brierley J, Carcillo J, Choong K, et al. Clinical practice parameters for hemodynamic support of pediatric and neonatal septic shock: 2007 update from the Ameri­ can College of Critical Care Medicine. Crit Care Med. 2007;37(2):666-688. Chameides L, Samson R, Schexnayder S, Hazinski MF. Pediat­ ric Advanced Life Support Provider Manual. Dallas, TX: American Heart Association; 20 1 1 . Cornfield DN. Acute respiratory distress syndrome in chil­ dren: Physiology and management. Curr Opin Pediatr. 20 1 3;25(3 ) :338-343. D'Agostino D, Diaz S, Sanchez MC, Boldrini G. Management and prognosis of acute liver failure in children. Curr Gas­ toenterol Rep. 2 0 1 2 ; 1 4:262-269. Goldstein B, Giroir B, Randolph A. International pediatric sepsis consensus conference: Definitions for sepsis and organ dysfunction in pediatrics. Pediatr Crit Care Med. 2005;6( 1 ) :2-8. Holmes JF, Palchak MJ, MacFarlane T, Kuppermann N. Performance of the Pediatric Glasgow Coma Scale in children with blunt head trauma. Acad Emerg Med. 2005; 1 2(9) :8 14-8 1 9 . Kliegman RM, Stanton BF, Schor NF, e t al, eds. Nelson Text­ book of Pediatrics. 1 9th ed. Philadelphia, PA: Elsevier/ Saunders; 20 1 1 . Lobos AT, Menon K. A multidisciplinary survey on capillary refill time: Inconsistent performance and interpreta­ tion of a common clinical test. Pediatr Crit Care Med. 2008;9( 4):386-39 1 . McKiernan CA, Lieberman SA. Circulatory shock in children: An overview. Pediatr Rev. 2005;26:45 1 -460. Mews C, Sinatra F. Chronic liver disease in children. Pediatr Rev. 1 993 ; 14:436-443. Meyer RJ, Theodorou AA, Berg RA. Childhood drowning. Pediatr Rev. 2006;27: 163- 1 69. Nakagawa TA, Ashwal S, Mathur M, Mysore M. Clinical report-Guidelines for the determination of brain death in infants and children: An update of the 1 987 task force recommendations. Pediatrics. 20 1 1 ; 1 28:e720-e740. Nichols DG. Rogers' Textbook of Pediatric Intensive Care. Philadelphia, PA: Lippincott Williams and Wilkins; 2008. Schwaitzberg SD, Bergman KS, Harris BH. A pediatric trauma model of continuous hemorrhage. J Pediatr Surg. 1 988;23(7) :605-609. Yager P, Noviski N. Shock. Pediatr Rev. 20 1 0;3 1 :3 1 1 -3 1 9.

This page intentionally left blank

9

Ear, Nose, and Throat Disorders Derek Z h o r n e

A 6-year-old boy presents to your office with worsening right ear pain and itching for several days. His mother has always been concerned about a skin tag in front of his ear and is concerned that it may be infected. Physical exam reveals an isolated right preauricular skin tag with no surrounding erythema; however, he has definite pain with manipulation of his tragus and you see a thick, purulent discharge in the edematous and erythematous external auditory canal. Question 1 - 1 Which of the following prophylactic treatments is NOT recommended for his condition? A) Wearing ear plugs while swimming. B) Using acidifying ear drops before and after swimming. C) Removing cerumen from the ear on a daily basis. D) Drying the ear canal with a hair dryer. E) Avoiding trauma to the external auditory canal. Discussion 1 - 1 The correct answer is "C:' Otitis externa, also called swimmer's ear, is cellulitis of the external auditory canal. It is often exac­ erbated by heat, humidity, moisture in the ear, and localized trauma to the ear canal skin. Cerumen provides a protective bar­ rier to the underlying skin of the external ear canal and should not be removed. The old adage "Don't put anything smaller than your elbow in your ear" helps to avoid otitis externa. How does this pertain to a pediatrician's favorite pastime of ear curettage? Prophylactic measures to prevent otitis externa consist of avoid­ ing ear moisture (by wearing ear plugs or using a hair dryer after swimming), using acidifying ear drops to inhibit bacterial and fungal growth and avoiding trauma to the ear canal. Question 1 -2 All of the following therapeutic options may be indicated in the treatment of otitis externa EXCEPT: A) Oral pain medication. B) Topical antimicrobial therapy.

C) Topical glucocorticoid therapy. D) Systemic antimicrobial therapy. E) Systemic glucocorticoid therapy. Discussion 1 -2 The correct answer is "E:' There is currently no indication for the use of systemic glucocorticoids in the treatment of otitis externa. Pain is a significant complaint in otitis externa and responds best to oral pain medications. The use of topical antimicrobials targeted to the common bacterial pathogens of Staphylococcus aureus and Pseudomonas aeruginosa can be achieved with fluoroquinolone, polymyxin B and neomycin, or aminoglycoside ear drops for 7 to 10 days. Avoid the use of aminoglycoside ear drops if the tympanic membrane is perforated or cannot be visualized as they can be ototoxic. Topical glucocorticoids are often added to antimicrobial sus­ pensions and may help to decrease pain and relieve itching. If the patient is immunocompromised or there is evidence of periauricular cellulitis, then oral antimicrobial therapy is indicated.





Helpful Tip

Prea u ri c u l a r s i n u ses, pits, a n d s k i n ta g s a re c o m m o n

1 1 1r a n d

freq u e n t l y noted o n routi n e physical exa m .

Either

i n h e rited

in

an

i n c o m p l ete

a utosom a l

d o m i n a n t patte rn o r a ri s i n g s po n ta n e o u s ly, t h ey u s u a l l y c a u s e no sym pto m s . H owever, eve ryo n e with a prea u ri c u l a r s i n u s, pit, o r s k i n tag s h o u l d h a v e a fo r m a l h ea r i n g test. R e m e m b e r t h a t b o t h t h e exte r n a l e a r struct u res a n d t h e ki d n eys fo rm d u r i n g t h e s a m e period o f e m b ryog e n e s i s . I nfa nts with i s o l ated prea u ri c u l a r s i n u ses, p its, o r s k i n t a g s d o n o t routi n e l y n e e d a rena l u ltraso u n d u n l ess t h e r e a re o t h e r m a l fo r m a t i o n s o r dys m o r p h i c featu res, a fa m i l y h i story of d eafness, or a m a te r n a l h i story of g estat i o n a l d i a betes.

1 79

1 80

MCG RAW- H I LL E D U C AT I O N S P E C I A LTY BOA R D REVI EW: P E D I ATRICS

A 5-year-old girl presents to the emergency department with a chief complaint of left-sided ear pain. Her parents report that the family was at the movie theater when the girl cried "Ouch!" and then told them that her older brother had put something in her ear. The older brother is not saying any­ thing as he calmly eats his popcorn. Question 2-1 Which of the following is NOT a clinical finding associated with an acute foreign body in the external ear canal? A) Cellulitis. B) Ear pruritus. C) Bleeding. D) Tinnitus. E) Ear pain. Discussion 2-1 The correct answer is "A:' Acute foreign bodies in the ear canal are often asymptomatic and identified because of a witnessed event or based on a verbal report by the affected child. Clini­ cal findings may include ear pain, pruritus, bleeding, tinnitus, or even conductive hearing loss. If left unattended, a foreign body may lead to a secondary infection. Remember that organic matter should not be irrigated as it will absorb liquid and swell, causing increased pain and making it more difficult to remove. Batteries (especially button batteries) require imme­ diate removal as they can generate an electric current and cause a severe burn with liquefaction necrosis within 4 hours. (This leaves a person wondering if a battery really could fit in an ear. To be clear, this is not a challenge.)

� QUICKQUIZ Trauma can result in an ear hematoma (pooled collection of blood) which presents as a boggy purple swelling of the pinna. The normal folds of the ear will be difficult to visual­ ize due to swelling. This is in contrast to an ear bruise, which does not change the shape of the ear. All of the following are potential complications of an ear hematoma EXCEPT: A) Infection B) Conductive hearing loss C) Cartilage necrosis D) Development of "cauliflower ear" E) Reaccumulation of the hematoma Discussion The correct answer is "B." Auricular trauma can result in the formation of a hematoma between the perichondrium

and the cartilage of the pinna. If untreated, pressure necro­ sis of the underlying cartilage may occur with subsequent infection, cosmetic deformity, or both. The "cauliflower ear" deformity seen among wrestlers is the result of new and often asymmetric cartilage formation after a hematoma. To pre­ vent these sequelae, patients should be referred to an otolar­ yngologist for drainage and application of a carefully molded pressure dressing. Drainage should occur within 7 days of the inciting injury.

An 8-month-old girl is brought to the clinic for evalua­ tion of a fever. She has been healthy and this is her first ill­ ness. She has no known drug allergies. Her mother reports that several members of the family "had a cold" last week and the patient had several days of coughing and rhinor­ rhea that seemed to improve. Physical exam reveals an ill­ appearing infant with a temperature of 39.2°C ( 1 02.5°F) and a right tympanic membrane (TM) that is erythema­ tous and bulging with white fluid present behind the TM. You perform pneumatic otoscopy and do not appreciate any movement of the TM. (See Figure 9- 1 . ) You explain to the mother that the child has a right-sided acute otitis media (AOM ) . Question 3-1 Acute otitis media is commonly caused by any of the follow­ ing organisms EXCEPT: A) Streptococcus pneumoniae. B) Staphylococcus aureus. C) Nontypeable Haemophilus influenzae. D) Moraxella catarrhalis. E) Streptococcus pyogenes. Discussion 3-1 The correct answer is "B:' The most commonly identified bacterial organisms to cause AOM include S. pneumoniae ( 3 5 -40% ) , nontypeable H. influenzae ( 3 0 - 3 5 % ) , M. catarrh­ alis ( 1 5- 1 8 % ) , and S. pyogenes (4% ) . Remember that AOM occurs most frequently as a consequence of a viral upper respiratory tract infection (URI ) , which leads to eustachian tube inflammation and dysfunction with subsequent nega­ tive middle ear pressure and movement of secretions con­ taining the pathogenic bacteria from the nasopharynx into the middle ear. Question 3-2 Which of the following is the most appropriate initial treat­ ment for this patient? A) Clindamycin. B) Cefuroxime. C) Cefdinir.

CHAPTER 9



EAR, N O S E, A N D TH ROAT D I S O R D E RS

1 81

F I G U R E 9- 1 . Acute otitis media. (A) A normal tym panic membra n e is shown with a s h i ny l ig ht reflex. (B) The tym panic membra n e is inflamed (red), b u l g i ng, a n d has a puru lent effusion consistent w i t h acute otitis m e d i a . T h e l i g ht reflex is gone. (Used w i t h permission from Dr. Shelagh Cofer, Department o f Oto l a ryngology, Mayo C l i n ic.)

D) Amoxicillin and clavulanic acid (Augmentin) . E ) Amoxicillin. Discussion 3-2 The correct answer is "E:' According to the 20 1 3 American Academy of Pediatrics (AAP) Clinical Practice Guideline for the diagnosis and management of AOM in children, first-line therapy for AOM is high-dose amoxicillin (80-90 mg/kg of body weight/day divided twice daily) . S. pneumoniae often is resistant to lower dose amoxicillin. This resistance mech­ anism is via alteration of penicillin-binding proteins (not beta-lactamase production) and can usually be over­ come by increasing the dose of amoxicillin. If the patient ( 1 ) has been treated with amoxicillin in the past 30 days, (2) has a history of recurrent AOM unresponsive to amoxi­ cillin ( 3 ) has concurrent purulent conjunctivitis, or (4) has shown no improvement after 48 hours of amoxicillin therapy, then clinicians should prescribe an antibiotic with additional beta-lactamase coverage (option "D" ) . Concurrent conjunc­ tivitis suggests H. injluenzae infection. H. injluenzae often is resistant to penicillin via beta-lactamase production. Addi­ tion of the beta-lactamase inhibitor clavulanate usually is able to overcome this resistance mechanism. If the patient has a penicillin allergy, then alternative agents include cef­ dinir, cefuroxime, or cefpodoxime. If the patient is unable to take oral medications or compliance is a concern, then you may consider ceftriaxone 50 mg/kg given intramuscularly once daily for 3 days. Remember that AOM is painful and all patients should receive oral pain medication as well. Children younger than 2 years of age should be treated for 10 days, children older than 2 years should be treated for 7 days, and children older than 6 years should be treated for 5 days. See Table 9- 1 for recommendations on the initial management of AOM.



Helpful Tip

:S.� AOM is the most common condition for which children

r1 1r in the U n ited

States a re prescribed a ntibiotics. I t is

i m perative that clinicians fol low strict diag nostic criteria

to make an accu rate diagnosis and avoid prescri bing u n n ecessa ry a ntibiotics. See Figure 9-2 for normal TM landma rks. The diagnosis of AOM ca n be made in children who present with moderate to severe bulging of the TM or new onset of otorrhea not due to acute otitis externa and the presence of a middle ear effusion proven by pneu matic otoscopy or tym pa nometry. I n a nonverba l child, the diagnosis of AOM can be made with

mild bulging ofthe tym panic membrane and recent (< 48

hou rs) onset of ear pa in (man ifest as holding, tugging, or rubbing of the ear) or i ntense erythema of the tym pa nic mem brane and evidence of a middle ea r effusion.



Helpful Tip

:S.� Risk factors for AOM include daycare attendance, which

r1 1r increases the l i keli hood of recu rrent viral U R is, bottle­ propping in the crib, smoke exposu re, eustachian tube

dysfu nction, a n d impaired host i m m u n e defenses (especia lly consider i m m u noglobu l i n A [I gAl deficiency).

Question 3-3 Which of the following is NOT a potential complication of AOM? A) Bezold abscess. B) Tympanosclerosis. C) Tympanic membrane perforation. D) Facial nerve paralysis. E) Mastoiditis.

1 82

MCG RAW- H I LL E D U C AT I O N S P E C I A LTY BOA R D REVI EW: P E D I ATRICS

T A B L E 9-1 RECO M M E N DAT I O N S FOR THE I N ITIAL M A N AG E M E NT O F ACUTE OTIT I S MEDIA (AOM)

Age

Otorrhea with AOMa

Unilateral or Bilateral AOMa with Severe Symptomsb

6 mo-2 y

Anti biotic thera py

Anti biotic thera py

Anti biotic thera py

Anti biotic thera py or additional obse rvation

;?: 2 y

Anti biotic thera py

Anti biotic thera py

Anti biotic thera py or additional observation'

Anti biotic thera py or additional o bse rvation'

Bilateral AOMa without Otorrhea

Unilateral AOMa without Otorrhea

'Appl ies o n ly to c h i l d ren with wel l-documented AOM a n d a high certa i nty of d i a g nosis. bA toxic-a ppea ring c h i ld, persistent ota l g ia for more than 48 hou rs, tem perature g reater than 39°C ( 1 02.2°F) i n the past 48 hou rs, or if there is u ncerta i n access to fol l ow-u p after the visit. 'Th is plan of i n itial management p rovides an opportu n ity for s h a red decision making with the c h i l d 's fa m i l y for those categ ories a p p ropri­ ate for i n itial observation. If observation is offered, a mecha n i s m m u st be i n place to e n s u re fol l ow-u p and beg i n a nti bitoics if the c h i l d worsens or fa i l s t o i m p rove with i n 48-72 hours o f A O M onset. Reprod uced with permission from Fried man N R, Scholes MA, Yoon PJ. Ea r, nose, and th roat. I n : Hay WW, Levi n MJ, Deterd ing RR, Abzug MJ, eds. Current Diagnosis & Treatment: Pediatrics. 22nd ed. N ew York, NY: McG raw-H i l l ; 2 0 1 3 .

Discussion 3-3 The correct answer is ''A:' A Bezold abscess is a laterocervical abscess between the digastric and sternocleidomastoid muscles and may be seen as a complication of mastoiditis. Tympanoscle­ rosis is an acquired disorder of calcification and scarring of the TM and middle ear structures from inflammation and may result in conductive hearing loss if the ossicles are involved. Patients with AOM and subsequent TM perforation often have rapid relief of pain and subsequent otorrhea. These perforations usu­ ally heal spontaneously within several weeks, although patients should receive topical antimicrobial ear drops for 1 0 to 14 days and be referred to an otolaryngologist for a follow-up exam and hearing evaluation. The facial nerve travels through the middle ear as it courses through the temporal bone and may be exposed to inflammation during an episode of AOM with resulting facial nerve paralysis. It is a rare complication that requires both sys­ temic antibiotics and referral to an otolaryngologist for prompt myringotomy and tube placement. Mastoiditis is covered in detail

later. Other potential complications of AOM include develop­ ment of retraction pockets, cholesteatoma, chronic suppurative otitis media, intracranial infections (meningitis, epidural abscess, brain abscess), and lateral or cavernous sinus thrombosis.



Helpful Tip

=-:=11.. C h i l d ren

with

th ree

or

more d i stinct a n d

well­

r1 1 r documented episodes of AOM with in 6 months or fou r or more episodes withi n 1 2 months s h o u l d be referred to

an otola ryngolog ist for tym panostomy tube placement.

You appropriately treated your patient with 10 days ofhigh-dose amoxicillin for her right AOM and are now seeing her back for her 9 month well-child check. She is doing well and her mother has no complaints. Her physical exam at this time reveals the presence of dear fluid behind the TM without any bulging,

Right tympanic membrane Otoscopic view (Nose is to right)

Attic

-----L.-!_ Lateral view

FIGURE 9-2. Tym panic mem brane l a n d m a rks. (Reproduced with perm ission from Hay WW, Levin MJ, Deterd i n g RR, Abzug MJ, eds. Current Diagnosis and Treatment Pediatrics. 22nd ed. McGraw- H i l l Education; 20 1 4, Fig. 1 8- 1 .)

CHAPTER 9

erythema, or otorrhea. You explain to the mother that the patient now has evidence of otitis media with effusion (OME). Question 3-4 All of the following statements regarding OME are correct EXCEPT: A) Most episodes will resolve spontaneously within 3 months. B) OME is often associated with allergic rhinitis, adenoidal hypertrophy, and eustachian tube abnormalities. C) Referral for tympanostomy tube placement is indicated for hearing loss > 20 decibels (dB) . D) The initial treatment option is "watchful waiting" for 3 months. E) 5% to 10% of episodes will last 1 year or longer. Discussion 3-4 The correct answer is "C:' OME is defined as the presence of fluid in the middle ear space without associated signs or symptoms of inflammation. It may occur spontaneously due to poor eustachian tube function or as the result of an inflammatory process follow­ ing AOM. Most episodes resolve spontaneously within 3 months, although 30% to 40% of children have recurrent OME and 5% to 10% of episodes last 1 year or longer. OME is often associated with allergic rhinitis, adenoidal hypertrophy, and eustachian tube abnormalities. As most cases resolve, it is recommended to pur­ sue "watchful waiting" for 3 months if the patient is not at risk for hearing problems. Children at risk for speech or language delays are likely further affected by hearing problems from OME and should be evaluated sooner. This includes children with craniofa­ cial anomalies ( eg, Down syndrome, cleft palate, Robin sequence, or CHARGE association [coloboma, heart disease, atresia of the choanae, retarded growth and development, genitourinary anoma­ lies, and ear defects with associated deafness] ), permanent hearing loss, autism spectrum disorders, severe visual impairments, or syn­ dromes that adversely affect cognitive and linguistic development. A hearing test should be obtained in any child with OME that persists for 3 months or longer, or at any time that language delay, learning problems, or a significant hearing loss is suspected. If the patient has hearing loss of 40 dB or greater, then referral for tympanostomy tube placement is indicated because persistent hearing loss of this magnitude has been shown to impact speech, language, and aca­ demic performance. If the patient has hearing loss of 2 1 to 39 dB, then he or she should have formal language testing and may benefit from strategies to optimize the learning and listening environment. If the patient has hearing loss ofless than 20 dB, then a repeat hear­ ing test should be performed in 3 to 6 months if the OME persists. •

Helpful Tip

=� Children with OME lasting 3 months or longer and who r1 1 r have had a norma l hearing test should be reexa mined at 3-

to 6-month intervals until the effusion is no longer present or they have developed symptoms warranting referral to an otolaryngologist. Pressure equalization (PE) tubes are indicated for hearing loss of greater than 40 d B or structural abnormalities of the eardrum or middle ear. See Table 9-2

for potential complications associated with OME.



EAR, N O S E, A N D TH ROAT D I S O R D E RS

1 83

TA B L E 9-2 POTENTIAL COM P L I CAT I O N S OF OTI T I S

M E D I A WITH E F F U S I O N

Potential Physical, Behavioral, Developmental Complications M i l d i nterm itte nt ear pain, fu l l n ess, o r "popping" Seco ndary m a n ifestation of ear pa i n i n i nfa nts such as ear rubbi ng, sleep d i stu rbances Fa i l u re of i nfa nts to respond a p p ropriately to voices or envi ro n m e ntal sounds (eg, not t u r n i n g towa rd the sound sou rce) Hea r i n g loss suggested by lack of attentiveness, fa i l u re to respond to normal conversational l eve l speech, n eed fo r excessively h i g h sound l eve l s when u s i n g a u d i o eq u i p­ ment or watc h i n g TV Rec u rre nt epi sodes of AOM with persi ste nt O M E between epi sodes Problems with school performance Bala nce problems, u nexp l a i ned c l u msi ness, or del ayed g ross m otor development Del ayed s peech o r l a n g uage AOM, acute otitis media; O M E, otitis media with effusion. Data from Rosenfe l d RM, C u l pepper L, Doyle KJ, et al: C l i n ica l p rac­ tice g u id e l i n e : Otitis media with effu sion, Otolaryngol Head Neck Surg. 2004 May; 1 30(5 S u p p i ) :S95- 1 1 8.

You are now seeing your patient for her 2-year-old well-child check. She had five episodes of AOM in a 12-month-period and subsequently had tympanostomy tubes placed at 19 months of life. Mom reports that the patient has had persistent white dis­ charge from her left ear for several weeks and wonders if it is related to her tubes or a sign of another ear infection. Question 3-5 Which of the following is the most likely pathogen to cause chronic suppurative otitis media (CSOM) ? A) Pseudomonas aeruginosa. B) Klebsiella species. C) Anaerobic bacteria. D) Fungi. E) Escherichia coli. Discussion 3-5 The correct answer is ''A:' CSOM is the most common cause of hearing impairment in the developing world and one of the most common childhood infectious diseases worldwide. The World Health Organization (WHO) defines CSOM as "otorrhea through a perforated tympanic membrane present for at least 2 weeks:' It is often the sequelae of AOM. The most commonly iso­ lated organisms in decreasing order of frequency are P. aeruginosa ( 1 8-67%), Staphylococcus aureus ( 14-33%), and gram-negative

MCG RAW- H I LL E D U C AT I O N S P E C I A LTY BOA R D REVI EW: P E D I ATRICS

1 84

organisms such as Proteus species, Klebsiella species, and Esch­ erichia species (4-43%). Both anaerobic bacteria such as Bacte­ roides and Fusobacterium species as well as fungi such as Candida and Aspergillus have also been reported. There is no standard of therapy and recommended regimens vary widely. In developing countries, antiseptic ear drops ( eg, aluminum acetate, boric acid, iodine powder, or povidone-iodine) are commonly used due to low cost and ease of availability. Most clinicians in the United States use a combination of topical fluoroquinolone ear drops for 14 days with the initiation of the so-called "aural toilet;' which includes dry mopping, ear wicking, or gentle suctioning to keep the ear canal consistently dry and free of debris.

� •

I I

TA B L E 9-3 CAU S E S O F P U RU L ENT OTO R R H EA

Acute otitis med ia with pe rforated tym panic m e m bra n e I nfecti o u s co m p l icatio n s o f acute otitis m ed i a {ie, acute mastoid itis, i ntracra n i a l i nfection) Otitis externa C h ro n i c s u p p u rative otitis media Cholesteato ma Tymp an ostomy t u be d ra i nage

Helpful Tip

Otorrhea simply means d rainage from the ea r and resu lts from either problems with the external ea r ca nal or middle ea r disease with a TM perforation. See Ta ble 9-3 for causes of puru lent otorrhea. Bloody otorrhea is less common and typica lly is seen with trauma to the external canal from a foreign body or overly agg ressive cleaning of the ear canals, neoplasm, longitudinal tempora l bone fracture causing TM and external ca nal laceration, or in the setting of trau matic cerebrospinal fluid otorrhea associated with a basilar sku l l fracture.





Helpful Tip

A cholesteatoma is a squamous epith e l i u m - l i ned sac

1 1 1r that may fi l l with desq u a m ated keratin a n d behave in

by

a

loca lly destructive

prolonged

eusta c h i a n

m a n ner. tu be

U s u a l ly caused

dysfu nction

with

su bseq uent c h ronic negative m i d d l e ear pressu re that

FIGURE 9-3. Cholesteatomas result from i nvag i nation o f the pars flaccida (see Fig u re 9-2) pa rt of the tym pa nic membra ne, with accu m u lation of keratin debris a n d sloug hed squamous epithe l i a l cel ls. Cholesteatomas a re loca l ly e rosive a n d req u i re s u rgical excision. The arrowhead poi nts to

d raws i n the u pper flaccid portion of the tym pa n i c

retraction a n d the a rrow to the cholesteatoma sac beh i n d the tym p a n i c

membra ne, a cholesteatoma can erode i nto b o n e

mem brane. (Reproduced w i t h permission from Lalwa n i A K , ed. Current

a n d penetrate i nto the mastoid b o n e or d i s r u pt the ossic u l a r chain. Early cholesteatomas sta rt with s lowly

Diagnosis & Treatment in Otolaryngology: Head and Neck Surgery. 3 rd ed. McGraw- H i l l Education; 2 0 1 1 , Fig. 50-2.)

prog ressive hea r i n g loss, but patients may a l so have ea r pain, headache, or a sensation of "ea r fu l l ness." On physica l exa m, you may see a retraction pocket or a pparent TM

perforation that exudes keratin

debris o r g r a n u lation tissue. Treatment is s u rg i ca l marsupial ization o f t h e sac or its complete remova l . See F i g u re 9-3 for a n exa m p l e o f a chol esteatoma. For those scratc h i n g their heads, the top part of the TM is sucked i n making a l ittl e pouch. The pouch fi l l s with sloug hed sq uamous epit h e l i a l cells. The fi l l ed pouch acc u m u lates i n the m i d d l e ea r and mastoid, destroyi ng structures a s it g rows. It has to go, so ca l l a s u rgeon.

Question 3-6 Which of the following is an indication for tympanostomy tube insertion? A) Two episodes of AOM within 6 months. B) Three episodes of AOM within 12 months. C) Persistent TM perforations.

D) Conductive hearing loss > 40 dB associated with a middle ear effusion. E) Tympanosclerosis. Discussion 3-6 The correct answer is "D:' This was a test to see if you had been reading the helpful tips, which are helpful for this question. Indications for tympanostomy tube insertion include ( 1 ) recur­ rent episodes of AOM (defined as three or more distinct and well-documented episodes of AOM within 6 months or four or more episodes within 12 months) , (2) conductive hearing loss greater than 40 dB associated with a middle ear effusion, or (3) prevention of acquired cholesteatoma due to a retraction pocket of the TM. Options ''!\.' and "B" are both incorrect because the patient must have had three or more episodes of AOM within 6 months or four or more episodes within 12 months. Options "C" and "E" are incorrect because they are potential complications of tympanostomy tube insertions and not indications for place­ ment. Other potential complications include myringosclerosis,

CHAPTER 9

tympanosclerosis, tympanostomy tube otorrhea, and focal atro­ phy of the tympanic membrane, which increases the risk of developing retractions pockets and cholesteatomas. •



Helpful Tip

Quick Diagnosis: Benign pa roxysm a l positional vertigo

1 1 1r (BPPV) man ifests as pa roxysmal vertigo a n d nystagmus that only occu rs with certa i n head positioning. It is diagnosed using the Dix-H a l l pike maneuver, i n which

the patient is moved from the sitting position to recu mbency with the head ti lted 30 to 40 deg rees over the end of the exa m i nation ta ble a n d 30 to 45 deg rees to one side. The dysfu nction a l ear is the one that is downwa rd when sym ptoms a re el icited .

You are seeing a 20-month-old boy in your office with a chief complaint of "fever and ear swelling?' On physical exam, you see an irritable child and notice that his right auricle appears displaced inferiorly and outward. He has swelling, erythema, and extreme tenderness over his right mastoid area. His tym­ panic membrane is bulging with loss of the normal landmarks. Question 4- 1 What is the most appropriate next step in the management of this patient's condition? A) Discharge home with a prescription for high-dose oral amoxicillin. B) Admit to the hospital and start intravenous clindamycin. C) Order a plain X-ray of the skull. D) Admit to the hospital and start intravenous ampicillin. E) Order a computed tomography (CT) scan of the head and temporal bones. Discussion 4- 1 The correct answer is "E:' This patient has acute mastoiditis which is a bacterial infection of the mastoid bone and air cells. It arises from middle ear space infection that spreads by means of boney erosion or through the emissary vein of the mastoid and can occur at any age although greater than 60% of cases are in chil­ dren younger than 2 years of age. There is usually a preceding his­ tory of AOM, and patients present with fever, headache, and ear pain as well as postauricular swelling, redness, and tenderness. On physical exam, the mastoid process is swollen, erythematous, and tender while the auricle is displaced both anteriorly and inferiorly and the ipsilateral tympanic membrane shows signs of AOM. In children 2 years of age or younger, the auricle appears to be pushed "down and out:' Mastoiditis is a clinical diagnosis, although you should obtain a CT scan of the head and temporal bones to evaluate for a subperiosteal or intracranial abscess. The CT scan shows clouding of the middle ear and mastoid air cells with loss of definition of the bony septae that define the mastoid air cells. There is no apparent subperiosteal



EAR, N O S E, A N D TH ROAT D I S O R D E RS

1 85

abscess or intracranial involvement. You have consulted the otolaryngology service, who will see the patient later today. Question 4-2 What is the most appropriate next step in the management of this patient's condition? A) Discharge home after being seen by the otolaryngologist with a prescription for high-dose oral amoxicillin. B) Admit to the hospital for mastoidectomy. C) Order a plain X- ray of the skull. D) Admit to the hospital and start intravenous (IV) broad­ spectrum antibiotics. E) Consult the pediatric surgery department. Discussion 4-2 The correct answer is "D:' The most common bacterial cause of mastoiditis is Streptococcus pneumoniae, although Staphylococ­ cus aureus (including methicillin-resistant strains), Streptococ­ cus pyogenes, and gram-negative bacilli such as Pseudomonas aeruginosa and nontypeable Haemophilus influenza have all been implicated. Accordingly, the next most appropriate step is to start broad-spectrum IV antibiotics to cover likely patho­ gens, including methicillin-resistant S. aureus (MRSA). Surgical drainage with mastoidectomy is indicated for a subperiosteal or intracranial abscess or a patient who does not improve after 24 to 48 hours of IV antibiotics. A tympanocentesis or myringot­ omy with or without tympanostomy tube placement, with the middle ear fluid sent for Gram stain and culture, can help guide antimicrobial therapy. A typical course of therapy for uncompli­ cated mastoiditis is 7 to 10 days of iV antibiotics and then tran­ sitioning to oral therapy for a total 4-week course of treatment. •

Helpful Tip

:5.� There a re many possi b l e causes of ea r pa i n-or ota l g ia,

r1 1 r if you wa nt to use medical term i n o l ogy. (See Ta ble 9-4.) TA B L E 9-4 CAU S E S OF EAR PA I N

Location

Etiologies

Auricle

Tra u ma, hematoma, ecze ma, i m petigo, i n sect bites, he rpes zoster

Meatus

External otitis media, eczema, h a rd ceru men, fore i g n body, herpes zoster, trige m i n a l neu­ ra l g i a (CN V3)

M id d l e ear

AOM, mastoiditis, cholestea­ to ma, m a l i g n a ncy

Referred pa i n t h ro u g h cra n i a l ne rves V, I X , X , a n d the seco nd a n d t h i rd cervical ne rves

U ne r u pted l owe r m o l a r, denta l cari es, TMJ dysfu nction, to nsi l l i­ tis, carc i n o m a or sa rco ma of the p h a rynx, cervica l lymphaden itis, thyroid itis, trigem i n a l n e u ra l g i a

AOM, acute otitis media; TMJ, tem poroma n d i b u l a r joi nt.

MCG RAW-H I LL E D U CATION S P E C I A LTY BOA R D REVI EW: P E D I ATRICS

1 86

You are working in the emergency department and see a 3-week­ old infant girl for "trouble breathing?' The parents report that the infant has had apparent trouble breathing since birth, with frequent clear right -sided nasal discharge. Review of the birth records show that the patient was born at 40 weeks' gestation by uncomplicated vaginal delivery and had a normal newborn nursery stay. The infant is breastfeeding and the mother feels that she frequently stops feeding to "catch her breath?' Question 5-1

Given this presentation, your clinical suspicion is for: A) Meningitis. B) Viral upper respiratory infection. C) Choanal atresia. D) Chlamydia trachomatis infection. E) Heart failure. Discussion 5-1

Helpful Tip

:5.� Unilatera l choana I atresia occurs more freq uently on the

r1 1r right side and usua l ly presents later in l ife with unilateral

nasal obstruction or d ischarge. Bilateral choanal atresia presents at birth with respiratory distress and may have

"cyclical cyanosis" wherein the cya nosis improves with crying and worsens with feed ing. Suspect the diag nosis if you are una ble to pass a 6-French catheter through the nose to a depth of a bout 3 to 4 em. Confirm the diag nosis

with an axia l CT scan with intranasal contrast that will show a narrowing of the posterior nasal cavity at the level of the pterygoid plate. •

Helpful Tip

:5.� Remember,

r1 1r bilateral

approximately

50%

of

patients

All of the following are potential therapies for epistaxis (nose bleeding) EXCEPT: A) Firm pressure on the soft part of the nose below the nasal bones for 5 to 10 minutes. B) Application of topical vasoconstrictor such as oxymetazoline or phenylephrine. C) Intranasal packing. D) Vigorous nose blowing to clean out the nasal passages. E) Chemical cautery with topical silver nitrate. Discussion

The correct answer is "C:' This patient may have choanal atresia (blocked posterior nasal opening), which is the most common con­ genital anomaly of the nose and occurs in approximately 1 in 7000 live births. It is typically unilateral and is twice as common in females. Remember that neonates are obligate nasal breathers and anything that creates nasal obstruction (such as choanal atresia) may present as respiratory distress. Option ''1\.' is incorrect because the clinical picture does not fit meningitis. Option "B" is incorrect because you would not expect unilateral discharge with a viral URI. Option "D" is incorrect because you would most commonly see conjunctivitis or pneumonia as a result ofneonatal C. trachomatis infection. Although you should think of possible heart failure whenever a young child is having difficulty with feedings, the case description is clearly focus­ ing on an upper airway issue; thus option "E" is incorrect. •

� QUICKQUIZ

with

choa n a l atresia have CHARGE association. Other syn d romes that may include choanal atresia include Treacher-Collins, Kallmann, and VACTERLNATER association (Vertebral anomalies, Anal atresia, Cardiac defects, TEF [tracheoesophageal fistu la] and/or esophageal atresia, Renal anomalies, and Radial or Limb defects).

The correct answer is "D:' Epistaxis is conservatively treated by application of firm pressure to the soft part of the nose below the nasal bones for 5 to 10 minutes while sitting up and lean­ ing forward to avoid swallowing the blood. If the bleeding per­ sists, then a one-time application of a topical vasoconstrictor or chemical cautery with silver nitrate may be used. For persistent bleeding, you should obtain an otolaryngology consult as the patient may require anterior or posterior intranasal packing or more advanced surgical interventions. Option "D" is incorrect as it will likely contribute to further bleeding. The evaluation of a patient with epistaxis should include a directed history to evaluate for prior or recurrent episodes of epistaxis, history of nasal trauma, prior head and neck procedures, family history of bleeding disorders, as well as the use of nonsteroidal anti-inflam­ matory drugs (NSAIDs), warfarin, heparin, aspirin, cocaine, or alcohol. Most cases of epistaxis arise from the anterior portion of the nasal septum at Kiesselbach plexus. Less common poste­ rior bleeding may arise from branches of the anterior ethmoid or sphenopalatine artery and require an urgent otolaryngology consult for posterior packing and admission with pulse oxim­ etry monitoring. After resolution of the acute bleeding, patients with epistaxis often benefit from increased nasal moisture with a daily application of water-based ointment such as petroleum jelly (eg, Vaseline) into the nose or with twice-daily nasal saline irrigation and humidifier use.

� QUICKQUIZ In a patient with an episode of epistaxis, a hematology workup is warranted for any of the following EXCEPT: A) Frequent nose picking. B) Family history of a bleeding disorder. C) Medical history of easy bleeding such as with circumcision or dental extraction. D) Spontaneous bleeding at any site. E) Onset before age 2 years. Discussion

The correct answer is ''A:' See Table 9-5 for the differential diag­ nosis of epistaxis, and remember that digital trauma (nose pick­ ing) is a common cause of epistaxis in children. Children love to

C H A PT E R 9

TABLE 9-5 D I F F E R E N T I A L D I AG N O S I S O F E P I STAX I S

Tra u m a (nose picki ng, b l u nt tra u ma)

Med i cations (espec i a l l y nasal sprays)

Nasal polyps

N eo p l a s m s (nasopharyn­ geal carci noma, rhabdo­ myosa rco ma, lymphoma, j uve n i l e nasopharyngea l a n g iofibroma) B l eed i n g d i sorders (hemo­ p h i l ia, t h rom bocyto pe n i a, vo n Wi l l ebra n d d i sease, hereditary hemorrhagic te l a n g i ectasia) S i n u sitis

pick their noses but are not likely to admit to it. Although epi­ staxis is caused by a bleeding disorder in less than 5% of cases, a hematology workup would be indicated for all of the other situations listed above.

You are working in the emergency department and see a 1 7-year-old adolescent boy who was involved in a physical altercation at school. He was hit in the face with a baseball bat (don't ask!} and is complaining of severe nose pain and difficulty breathing through his nose. On physical exam, you notice a boggy, widened nasal septum that is very ten­ der to palpation and appreciate a whistling noise with each inspiration. You inform the patient that he has a hematoma of the nasal septum and needs immediate referral to an otolaryngologist. Question 6-1

All of the following are potential complications of an untreated nasal septal hematoma EXCEPT: A) Odontogenic infection. B) Development of a "saddle-nose deformity:' C) Nasal abscess formation. D) Cavernous sinus thrombosis. E) Meningitis. Discussion 6-1

The correct answer is "A:' Early referral of a nasal septal hema­ toma to an otolaryngologist for evaluation of the hematoma and packing of the nose with antibiotic prophylaxis is important. If unattended, the septal cartilage may undergo fibrosis, necro­ sis, and then perforate within 3 to 4 days. The subsequent loss of structural support results in the characteristic "saddle-nose deformity:' The other complications listed above as options "C:' "D;' and "E" are due to secondary infection with subsequent spread through the cavernous sinus. It is always good to know the worst -case scenario. Do not miss out on the opportunity to encourage the teenage boy to use his words when angry.







EAR, N O S E, A N D TH ROAT D I S O R D E RS

1 87

Helpful Tip

Nasal foreign bodies often present with u n i l atera l fou l ­

lllr smel l i n g

rhi norrhea, nasa l obstruction, epistaxis, or even hal itosis (bad breath). Treatment options include a trial of vigorous nose blowing or the appl ication of a nasal decongesta nt (eg, oxymetazo l i n e spray) a n d then remova l with a l l igator forceps. Refer to a n otolaryngolog i st i f the object is u n l i kely t o b e removed on the fi rst attem pt or a p pears wedged in place. Remember that disk-type batteries a re very d a ngerous nasal foreig n bodies as they can generate a n el ectrica l cu rrent a n d cause necrosis of m ucosa a n d cartilage i n

l e s s than 4 hou rs! (The rea l q u estion i s , w h y do c h i l d ren put things i n their nose a n d ea rs?)

You are seeing a 1 5-year-old adolescent boy with a past medi­ cal history of allergic rhinitis who has a chief complaint of "a persistent cold:' A review of the chart shows that he was seen 2 weeks ago with similar complaints of daytime cough and rhinorrhea. He reports feeling worse now, with new fevers up to 39.0°C ( 1 02.3°F) as well as fatigue and headache. Physical exam is notable for clear rhinorrhea and tenderness to palpa­ tion of his face. His posterior oropharynx is clear without any erythema or exudates. Question 7-1

Which of the following is the most appropriate next step in the management of this patient? A) Order a plain film X-ray of his skull. B) Order a CT scan of his head. C) Offer supportive care therapies with nasal saline rinses and pain medication. D) Prescribe a 5-day course of amoxicillin and clavulanic acid. E) Prescribe a 14-day course of amoxicillin. Discussion 7-1

The correct answer is "E:' Acute bacterial sinusitis is a common complication following viral upper respiratory tract infections and is more common in patients with allergic inflammation. Many of the presenting symptoms of sinusitis are very similar to those of a viral respiratory infection, such as daytime cough, nasal discharge, fever, and fatigue. However, it is the persistence (or worsening) of these symptoms without improvement that is suggestive of sinusitis. Adolescent patients may have associated facial pain and tenderness on exam. First-line therapy is amoxi­ cillin with or without clavulanic acid as the most commonly identified pathogens are Streptococcus pneumoniae ( �30% cases), Haemophilus influenza (30% cases), Moraxella catarrha­ lis ( � 10% cases), and respiratory anaerobes in the presence of acute maxillary sinusitis associated with odontogenic infec­ tions. Options "X' and "B" will not help distinguish between acute bacterial sinusitis and a viral respiratory infection as both produce nonspecific signs of sinus inflammation. However, a

1 88

MCG RAW-H I LL E D U CATION S P E C I A LTY BOA R D REVI EW: P E D I ATRICS

CT scan of the head or magnetic resonance imaging (MRI) of the brain and orbits should be obtained if there is clinical con­ cern for orbital or central nervous system involvement. Option "C" describes appropriate supportive therapies for patients with sinusitis, but the mainstay of treatment for acute bacterial sinusitis is antimicrobial therapy. Although length of therapy is somewhat controversial, most experts recommend treating for 10 to 28 days or until the patient has been asymptomatic for at least 7 days. Question 7-2

According to the AAP, all of the following statements are true in the diagnosis of acute bacterial sinusitis EXCEPT: A) Persistent illness lasting more than 10 days without improvement. B) Worsening course after initial improvement. C) Clear rhinorrhea that subsequently turns yellow-green. D) Severe onset of symptoms with fever and purulent nasal dis­ charge for at least 3 consecutive days. E) Symptoms may consist of nasal discharge or daytime cough. Discussion 7-2

The correct answer is "C:' Although it is a commonly held belief that clear rhinorrhea comes from viral colds and purulent (ie, yellow-green) nasal discharge comes from bacterial infections, this is incorrect. The other statements are all correct. According to the AAP, the diagnosis of acute bacterial sinusitis is made when a child with an acute URI presents in one of the following ways: •



Persistent illness (nasal discharge of any quality or day­ time cough or both) lasting more than 1 0 days without improvement or a worsening course (worsening or new onset of nasal discharge, daytime cough, or fever after ini­ tial improvement) Severe onset (concurrent fever defined as temperature of 39°C ( 1 02.2°F) or higher and purulent nasal discharge for at least 3 consecutive days

Question 7-3

All of the following are potential complications of acute bac­ terial sinusitis EXCEPT: A) Orbital cellulitis. B) Orbital subperiosteal abscess. C) Cavernous sinus thrombosis. D) Nasal abscess formation. E) Brain abscess. Discussion 7-3

The correct answer is "D:' The most common complication of acute bacterial sinusitis is orbital involvement (ie, orbital celluli­ tis [option "A" ] ) in young children with ethmoid sinusitis. Other potential complications include orbital subperiosteal abscess formation (option "B"), cavernous sinus thrombosis (option "C"), and brain abscess (option "E") resulting from intracra­ nial extension of infection. The most common complication

of frontal sinusitis is Pott puffy tumor, or osteomyelitis of the frontal bone.

� QUICKQUIZ Which of the following sinus cavities are present at birth? A) Maxillary and ethmoid. B) Maxillary and frontal. C) Maxillary and sphenoid. D) Ethmoid and frontal. E) Ethmoid and sphenoid. Discussion

The correct answer is "A:' The maxillary and ethmoid sinuses are present at the time of birth. The frontal and sphenoid sinuses begin to develop at ages 1 to 2 years but do not appear radio­ graphically until ages 5 to 8 years. Question 7-4

All of the following are known associations or risk factors for chronic sinusitis EXCEPT: A) Recurrent viral upper respiratory infections. B) Allergic rhinitis. C) Ciliary dyskinesia. D) Cystic fibrosis. E) Young children in daycare. Discussion 7-4

The correct answer is "E:' Although at risk of contracting (and spreading! ) many opportunistic infections, young children in daycare do not appear to be at risk for having chronic sinusitis, which is defined as an inflammatory disorder of the paranasal sinuses and linings of the nasal passage that lasts 12 weeks or longer. Risk factors include recurrent viral upper respiratory infections, asthma, allergic rhinitis, cigarette smoking, ciliary dyskinesia, cystic fibrosis, and immunodeficiency (especially hypogammaglobulinemia) . •

Helpful Tip

:5.� Remember

to order a CT sca n a n d consult a n

r1 1 r otolaryngolog i st if there is concern for sinus tra u m a .

You are seeing a 1 2-year-old adolescent girl with no signifi­ cant past medical history in your office for the chief com­ plaint of "sore throat?' Her mother states that the patient has had 5 days of low-grade fevers, malaise, and a sore throat. This morning she appeared more ill, with temperature of 39.4°C ( 1 03°F ) , a severe sore throat, and neck pain, and she does not want to open her mouth. Physical exam is notable for unilateral left-sided tonsillar swelling, uvular deviation to the right, and trismus.

C H A PT E R 9

Question 8-1

Based on this history and physical exam, the most likely diagnosis is: A) Viral upper respiratory infection. B) Sinusitis. C) Peritonsillar abscess. D) Strep throat. E) Mononucleosis. Discussion 8-1

The correct answer is "C:' Classic physical exam findings for a peritonsillar abscess include severe sore throat, unilateral tonsil­ lar swelling, trismus, and deviation of the uvula away from the affected side. Patients may also present with fever, drooling, ody­ nophagia, and a muffled or "hot potato' voice. Hot potato voice is a buzz word for this infection. It is usually seen in older children and adolescents after an antecedent infection such as tonsillitis (option "E"), strep throat (option "D") or a viral URI (option "/\'). The physical exam is not consistent with sinusitis (option "B"). Question 8-2

Which of the following is the most appropriate next step in the management of this patient? A) Order a CBC and blood culture. B) Order a lateral X-ray of the neck. C) Start therapy with oral corticosteroids. D) Start therapy with oral cephalexin. E) Arrange for an incision and drainage procedure. Discussion 8-2

The correct answer is "£:' A peritonsillar abscess should be drained with either needle aspiration or a formal incision and drainage pro­ cedure. The fluid should be sent for Gram stain and routine culture of aerobic and anaerobic organisms. Option "/\' is incorrect because lab tests are not needed to make the diagnosis or start therapy. Option "B" is incorrect as a lateral X-ray of the neck may be indi­ cated for evaluation of a retropharyngeal abscess but will not help make the diagnosis of a peritonsillar abscess. Option "C" is incorrect as there is little data to support the use of corticosteroids in the treat­ ment of pediatric peritonsillar abscesses. Option "D" is incorrect because, although starting antibiotics is important and cephalexin will cover two of the common organisms (Streptococcus pyogenes and Staphylococcus aureus), it will not cover respiratory anaerobes. Question 8-3

All of the following are potentially appropriate antibiotics for treatment of a peritonsillar abscess EXCEPT: A) Oral metronidazole. B) IV ampicillin-sulbactam. C) Oral clindamycin. D) IV clindamycin. E) IV ampicillin-sulbactam and vancomycin. Discussion 8-3

The correct answer is "A:' The most common pathogens in a peritonsillar abscess include group A streptococcus (Streptococ­ cus pyogenes), Staphylococcus aureus, and respiratory anaerobes.



EAR, N O S E, A N D TH ROAT D I S O R D E RS

1 89

Metronidazole (option "/\') will only provide anaerobic coverage and thus is not appropriate therapy. The other options all provide appropriate therapy. Option "E" would be the ideal combination in an area with a high prevalence of methicillin-resistantS. aureus (MRSA) because the ampicillin-sulbactam would provide cover­ age for group A streptococcus, respiratory anaerobes, and meth­ icillin-sensitive S. aureus (MSSA), while vancomycin provides optimal coverage for MRSA. Remember that the bioavailability of oral and IV clindamycin is essentially equivalent, so either is appropriate as long as the patient is not vomiting and is able to keep down the oral medication. Antibiotics should be continued for a total 14-day course of therapy, and the patient may transi­ tion to oral antibiotics once clinically improved.

You are seeing an 8-year-old boy in your office with a chief com­ plaint of "sore throat?' His mother reports that the older sib­ ling was diagnosed with strep throat earlier in the week and she ''wants to make sure that is not what is going on here?' Further history reveals the patient has had 2 days of fever higher than 39.4°C ( 103 oF) and a sore throat without any rhinorrhea or cough. Physical exam is notable for erythematous tonsils with exudates, palatal petechiae, and tender cervical lymphadenopathy. Question 9-1

Which of the following is the next most appropriate step in the care of this patient? A) Order a CBC and blood culture. B) Obtain a rapid group A strep antigen test. C) Order Epstein-Barr virus serologies. D) Order a urinalysis. E) Start therapy with oral clindamycin. Discussion 9-1

The correct answer is "B:' This patient most likely has group A streptococcus (GAS) pharyngitis or "strep throat:' The most appropriate step is to confirm the diagnosis as untreated GAS pharyngitis can lead to acute rheumatic fever, cervical adenitis, peritonsillar abscess/cellulitis, otitis media, or sepsis. The diag­ nosis of GAS can be made with a rapid antigen test as it has a specificity of 95% or greater and a sensitivity of approximately 80%. Thus, if the rapid antigen test is positive, you should treat for strep throat. However, a negative rapid antigen test requires confirmation by performing a throat culture. CBC and blood cultures (option "!\') are not helpful in the diagnosis of strep throat. Epstein-Barr virus serologies (option "C") may help diagnosis mononucleosis, which can have a similar presenta­ tion of fever, pharyngitis, and tender cervical lymphadenopa­ thy but may also include fatigue, malaise, periorbital edema, and mild hepatomegaly or splenomegaly. Checking a urinalysis (option "D") would be appropriate if you are concerned about post-streptococcal glomerulonephritis, which typically occurs 1 to 3 weeks following an episode of GAS pharyngitis and does not fit with the timing of this patient's illness. Oral clindamycin (option "E") is not the first-line therapy for GAS pharyngitis.

MCG RAW-H I LL E D U CATION S P E C I A LTY BOA R D REVI EW: P E D I ATRICS

1 90





Helpful Tip

Remember to t h i n k of the broad i nfectious d ifferentia l for a ny patient presenting with acute

1 1 1r diag nosis

pharyngitis. Vira l causes include adenovirus, coxsackie A virus, i nfl u enza, parai nfl uenza, and Epstein-Barr virus (EBV). Bacterial cau ses incl uded GAS, Arcanobacterium hemolyticum, Neisseria gonorrhea (if sexu a l ly active), or Corynebacterium diphtheriae.

A 4-year-old girl presents to the emergency department with a chief complaint of "fever?' Her mother reports that she was pre­ viously healthy but seemed to have a common cold last week. She has now had 3 days of fever, difficulty and pain with swal­ lowing, as well as apparent neck pain. Physical exam is notable for cervical lymphadenopathy, neck tenderness, and a refusal to perform cervical neck extension or lateral rotation. Her poste­ rior pharyngeal wall is erythematous and appears to be bulging.

Question 9-2

Question 1 0-1

Which of the following is an appropriate treatment regimen for GAS pharyngitis? A) Trimethoprim-sulfamethoxazole. B) Amoxicillin. C) Ceftriaxone. D) Ciprofloxacin. E) Doxycycline.

Based on this history and physical exam, the most likely diag­ nosis is: A) Viral upper respiratory infection. B) Diphtheria. C) Retropharyngeal abscess. D) Strep throat. E) Mononucleosis.

Discussion 9-2

Discussion 1 0-1

The correct answer is "B:' GAS pharyngitis is treated with either oral penicillin or amoxicillin for 1 0 days or a single intramuscular inj ection of penicillin G benzathine if there are concerns for adherence or the patient is vomiting. For penicillin-allergic patients, alternative treatments include cephalexin, azithromycin, or clindamycin. A third-generation cephalosporin such as ceftriaxone (option "C") is not indi­ cated for GAS pharyngitis, which can be treated with a more narrow-spectrum cephalosporin such as cephalexin. GAS pharyngitis should not be treated with tetracyclines (option "E" ) , trimethoprim-sulfamethoxazole (option "A''), or fluoro­ quinolones (option "D" ) .

The correct answer is "C:' This patient has a retropharyngeal abscess. Younger children often present with a preceding history ofviral URI symptoms while an older patient usually has a history of pharyngeal trauma such having had an endoscopy procedure, dental procedure, intubation, or penetrating injury such as a pencil to the back of the throat (think falling asleep at school while chewing on a pencil!). The most common symptoms include fever, neck pain, neck swell­ ing, sore throat, and odynophagia, although children may also have a muffled voice or drooling. The exam is notable for neck tender­ ness, limitation of cervical movements (especially neck extension), torticollis, and cervical lymphadenopathy. Know that in reality a toddler is not going to be overly cooperative. So be suspicious.



Question 1 0-2

Helpful Tip

� Anti microbial thera py is not i n d icated for most c h ronic 1 1 1r GAS carriers. The preva lence of GAS pharyngea l carriage in healthy c h i l d ren may be as h i g h as 20% i n

the a bsence o f a n outbrea k o f G A S . Accord ing t o t h e 201 2 Red Book, the few specific situations i n which erad ication may be i n d icated include: Loca l outbrea k of acute rheu matic fever or post­ streptococcal glomeruloneph ritis Outbrea k of GAS pharyngitis i n a closed or semi­ closed com m u n ity Fa m i ly history of acute rheu matic fever M u ltiple ("ping-pong") episodes of docu mented symptomatic GAS pharyngitis occ u rring with i n a fa m i ly for many weeks despite appropriate thera py. If

indicated,

erad ication

of

GAS

pharyngea l

All of the following are common pathogens in a retropharyn­ geal abscess EXCEPT: A) Streptococcus pyogenes (group A streptococcus). B) Staphylococcus aureus. C) Fusobacterium species. D) Prevotella species. E) Streptococcus pneumoniae. Discussion 1 0-2

The correct answer is "E:' Although a common cause of both community-acquired pneumonia and acute otitis media in chil­ dren, S. pneumoniae is not a common pathogen in retropharyn­ geal abscesses. Most of these infections tend to be polymicrobial with a combination of group A streptococcus, Staphylococcus aureus (MSSA and MRSA) and respiratory anaerobes such as Fusobacterium, Prevotella, Bacteroides, and Peptostreptococcus.

ca rriage can be d ifficult to ach ieve. The most effective thera py is 1 0 days of oral c l i n d a myci n . Other options

Question 1 0-3

include a m oxici l l i n-clavu lanic acid, azith romycin, or a combi nation of penici l l i n therapy with rifa m pi n for the last 4 days of treatment.

Which of the following is the most appropriate antimicrobial therapy for a retropharyngeal abscess? A) Ampicillin-sulbactam. B) Cephalexin.

C H A PT E R 9

C) Ceftriaxone. D) Vancomycin. E) Metronidazole.



EAR, N O S E, A N D TH ROAT D I S O R D E RS

1 91

TABLE 9-6 TONSILLECTOMY A N D ADENOIDECTOMY I n dications for Tonsil lectomy Most co m m o n i n d icati ons:



Discussion 1 0-3

( 1 ) Rec u rre nt t h roat i n fections (p h aryng itis)

The correct answer is "A:' As mentioned earlier, empiric therapy for a retropharyngeal abscess should include coverage of group A streptococcus, Staphylococcus aureus, and respiratory anaerobes, all of which can be achieved with ampicillin-sulbactam monotherapy. If there is significant concern for MRSA, then consider adding van­ comycin (option "D"). Another option would be to use clindamy­ cin monotherapy. Cephalexin (option "B"), ceftriaxone (option "C"), and vancomycin (option "D") are incorrect because they do not provide anaerobic coverage. Metronidazole (option "E") does not provide coverage for group A streptococcus or S. aureus.25





7 episodes i n past yea r and 1 or more of the fol l owi ng: fever, cervica l adenop­ athy, tonsi l l a r exudate, or positive test for g rou p A streptococcus

(2) S leep-d i sordered breat h i n g •



Concu rrent to n s i l l a r hypertrophy and Associated comorbid ities (poor g rowth, poor school performance, enuresis, behavioral problems) or

Abnormal polyso m n og ra p hy Other i n d ications i nc l u d e need to exc l u d e a t u m o r a n d treatment o f PFAPA syn d rome (period ic fever with a phthous stomatitis, p h a ryngitis and aden itis). •





Helpful Tip



Although the diag nosis of a retropharyngeal a bscess

1 1 1 r ca n be made on c l i n ical g rounds alone, imaging may be h e l pfu l to plan a s u rgical a pproach if necessa ry.

A screening latera l neck X-ray wi l l show widened prevertebra l soft tissues that exceed the a nterior­

I n dications for Adenoidectomy I nd icatio n s re late to adenoidal hypertrophy ca u s i n g nasal obstruction w i t h s u bseq uent sym pto m s a n d



i nc l ude:

posterior d i mension of the adjacent vertebral body. For a latera l neck X-ray, the c h i l d m u st sit sti l l with her head u p a n d neither extended nor flexed as either will d i stort the retropharyngeal space. An u ltraso u n d or CT



Severe nasal obstruction



Refractory c h ro n i c s i n usitis



Recu rrent acute otitis media



C h ro n i c otitis m ed i a with effu s i o n

sca n of the neck ca n identify the size of the a bscess a n d identify i m porta nt surro u n d i n g structu res. I m m ed iate s u rgical d ra i nage is i n d i cated if the patient has a i rway comprom ise or a fl uctuant a bscess. Not a l l patients req u i re s u rgical i ntervention. I ntravenous a nti biotics



should be conti n ued u ntil the patient is afebrile a n d c l i n ica l ly i m p roved; he or she c a n t h e n b e switched to ora l c l i n d a myci n or a m oxic i l l i n-clavu lanic acid for a tota l 1 4 days of thera py.

Re peat tym panostomy t u be placement Co nsideration ca n a l so be g ive n to patients with dental m a l occ l u s ion fro m pers i stent mouth breat h i ng, hypo nasal s peech and "adenoid facies" (long a n d n a rrow face, na rrow maxi l l a , steep m a n d i bl e a n d a n ove rbite) •

Compl ications of a Tonsil lectomy and Adenoidectomy Bleed i n g is most com mon com p l i cation (rate of •



Helpful Tip

� Also



known as a latera l pharyngeal a bscess, a

r1 1r pa ra p h a ryngeal a bscess is s i m i l a r to a retropharyngea l



a bscess. Neck motion is not typica l ly i m pa i red but a promi nent bulge of the latera l pharyngeal wa l l may be seen. It is treated with IV a ntibiotics with or without s u rgical d ra i nage. •



Helpful Tip

:5.� The two most common

r1 1r

ind ications for tonsil lectomy a n d adenoidectomy a re recu rrent th roat i nfections

(pharyng itis) and sl eep-d isordered breath i n g . Ta ble 9-6 s u m m a rizes i nformation a bout indications, compl ications, a n d contra i n d ications of common "T&A." Pa i n control is i m porta nt to prevent dehydration. (If it h u rts with swa l lowi ng, why wou l d a child wa nt to d ri n k? Good lu c k ration a l izing with a tod d l e r.)

0. 1 -8. 1 %) Other operative co m p l icatio n s i n c l ude tra u m a to teeth o r soft pal ate, l a ryngospasm, and a s p i ration Postoperative com p l ications i nclude nausea, vom iting, pain, dehyd ration, postobstructive p u l monary edema, nasopharyngeal stenosis, or velopharyngeal i n s uf­ ficie ncy (weak pharyngeal m u scles) with hypernasa l s peech. Mortal ity rate is repo rted to a p p roxi mate that of g en era l a nesthesia a l o n e

Contraindications t o Tonsil lectomy a n d Adenoidectomy •



Contra i n d i cations i nc l u d e c left pa l ate or s u b m ucous c left pa l ate due to risk of ve l o p h a ryngeal i n s ufficie ncy with res u ltant hypernasal s peech, bleed i n g d i s o rd e r, or acute to n s i l l itis Ta king out a n g ry i n fected ton s i l s i s genera l ly avoided but may be req u i red fo r to n s i l l itis or perito n s i l l a r a bscess u n respons ive t o medical thera py (ca l l ed a "q u i n sy" to n s i l l ecto my)

MCG RAW-H I LL E D U CATION S P E C I A LTY BOA R D REVI EW: P E D I ATRICS

1 92



Helpful Tip

:5.� Vel o p h a ryngeal

i n s uffi c i e n cy

(specifica l ly

the

r1 1r s p h i n cter) res u lts i n i m p roper c l o s i n g of t h e soft p a l ­

a t e m u s c l e i n the mouth d u ri n g s p e e c h . A i r escapes out the nose rat h e r than the mouth w h e n ta l ki n g , m a k i n g it h a rd to p ro n o u nce certa i n c o n s o n a n t sounds.

You are attending in the newborn nursery and the lactation consultant has concerns that a full-term, 2-day old male infant is not feeding well due to a short lingual frenulum. On exam, you notice that the infant is unable to protrude the tongue past the alveolar ridge and his weight is down 8% from birthweight. The mother has experience breastfeeding her other two children and feels that the infant does not latch on well during feeding. Question 1 1 -1

Which of the following is the most appropriate next step in the management of this infant? A) Order a CBC with white blood cell count differential, C-reactive protein, and blood culture. B) Start empiric IV ampicillin and gentamicin for presumed early-onset sepsis. C) Start intravenous fluids for dehydration. D) Arrange for a frenulectomy. E) Instruct the mother to increase the frequency of feeding attempts to every 2 hours. Discussion 1 1 -1

The correct answer is "D:' This infant has a tongue-tie ( ankyloglos­ sia), which results from a short lingual frenulum and subsequent difficulty with protrusion and elevation of the tongue. Tongue-tie may result in neonatal feeding difficulties or dental and speech problems later in childhood. If the infant is having difficulty with breastfeeding and the tongue cannot protrude past the alveolar ridge, a frenulectomy should be performed by a trained provider in the newborn period. A quick clip is all this infant needs.

An 8-month-old boy is brought to the emergency department for evaluation of a skin rash. He has been healthy and this is his first illness. He has no known drug allergies. His mother reports that he developed a red raised rash on his cheeks and around his mouth earlier today. The rash does not appear to bother the infant. He is afebrile and otherwise acting nor­ mally while eating a popsicle in the room. Question 1 2-1

Given this presentation, your clinical suspicion is for: A) Facial cellulitis. B) Hives.

C) Cold panniculitis. D) Eczema. E) Pressure erythema. Discussion 1 2-1

The correct answer is "C:' Cold panniculitis is sometimes called "popsicle panniculitis" as it commonly occurs in associ­ ation with eating frozen treats. The absence of systemic symp­ toms such as fever combined with a history of cold exposure (ie, eating popsicles) are very suggestive of cold panniculitis. It represents acute cold injury to the subcutaneous fat and the development of erythematous indurated nodules or plaques on the exposed skin. Lesions usually appear 24 to 72 hours after exposure to cold and gradually return to normal in 2 weeks to 3 months, although postinflammatory hyperpigmen­ tation may remain.

� QUICKQUIZ Infectious causes of pediatric parotitis (inflammation of the parotid glands) include all of the following EXCEPT: A) Mumps B) Epstein-Barr virus C) Parainfluenza virus D) Staphylococcus aureus E) Clostridium diffi cile Discussion

The correct answer is "E:' C. difficile may cause diarrhea, bloody diarrhea, pseudomembranous colitis, or fulminant colitis but does not cause parotitis. ( Having a bacteria that lives in fecal matter in your salivary gland doesn't sound appetizing. ) As a result of mumps immunization, parotitis is now uncommonly seen in children. Bacterial parotitis typi­ cally presents in infants younger than 2 months of age ( espe­ cially premature infants in the neonatal intensive care unit [NICU] ) and occasionally in children older than 10 years of age as acute-onset of fever, unilateral swelling, and ten­ derness of the parotid gland with overlying erythema. The most common pathogen is Staphylococcus aureus, although streptococci (including group B streptococcus in neonates) , gram-negative bacilli, and anaerobic bacteria can all b e involved. Management should include obtaining a culture of purulent fluid expressed from the Stenson duct and antibiotic therapy (eg, clindamycin) to cover both S. aureus and anaer­ obes. Viral parotidis is usually seen in children aged 3 to 1 0 years secondary t o mumps o r a multitude o f other viruses ( eg, Epstein-Barr virus, cytomegalovirus, enteroviruses, parainfluenza viruses, influenza viruses, human immuno­ deficiency virus [HIV] , herpes simplex viruses, coxsackievi­ rus and lymphocytic choriomeningitis virus) . Viral parotitis typically presents with several days of fever, malaise, and headache followed by parotid gland swelling that eventu­ ally becomes bilateral, with one side more affected than the

C H A PT E R 9

other. Management includes pain control, adequate hydra­ tion, sialogogues (eg, lemon drops or sour candy to facilitate ductal secretions), heat packs, and gland massage as well as good oral hygiene. •

Helpful Tip

� The

=-

pa rotid g l a n d extends from i n front of the ea r,

i1 1r to the jawl i ne, t o beh i n d the ear. O n exa m, the a n g l e o f the m a n d i b l e is not pa l pa b l e w h e n the g l a n d is e n l a rged.





Helpful Tip

A ra n u l a is a s u b l i n g u a l sa l iva ry g l a n d retention cyst

11 1 r that occu rs on the fl oor of the mouth a n d a ppears



EAR, N O S E, A N D TH ROAT D I S O R D E RS

1 93

D) Cleft lip is more common in males and cleft palate in females. E) Almost all children with cleft lip and palate require myr­ ingotomy and tympanostomy tube placement due to eusta­ chian tube dysfunction. Discussion 1 3-1

The correct answer is "C:' A bifid uvula is present in approxi­ mately 3% of children but there is an association with submu­ cous cleft palate. A submucous cleft palate is diagnosed by the classic triad of a bifid uvula, central thinning of the soft palate, and a palpable notch in the posterior border of the hard palate. It is important to identify this abnormality because affected chil­ dren have a 40% risk of developing persistent middle ear effu­ sions and are at risk for velopharyngeal incompetence resulting in hypernasal speech.

as a t h i n-wa l led, bluish cyst. Refer the patient to an



otolaryngolog ist for s u rgical management.

Helpful Tip

� Cleft l i ps a re classified as complete or incomplete a n d

=-

� QUICKQUIZ You have been asked to lecture the third-year medical stu­ dents on infectious and benign lesions of the oral cavity. In preparation for your talk, you review the important clinical findings to differentiate among these entities. Which is NOT a cause of a benign oral lesion? A) Fibroma. B) Mucocele. C) Geographic tongue. D) Aphthous ulcer. E) Fissured tongue.

i1 1r u n i latera l or bilatera l . A complete cleft l i p i m p l ies a sepa ration of the l i p that extends thro ugh the nasa l

s i l l (fl oor o f nasal open ing) a n d t h e a lveo lus i nto the pa late. An incomplete cleft lip may present as a cleft of va ria ble width with a n i ntact bridge of skin below the nasal s i l l .





1 1 1r

Helpful Tip

Although many congen ita l synd romes include clefting a s a man ifestation of a genetic a bnormal ity, these synd romes m a ke u p less than 20% of all clefts. Most cases (cleft lip or pa late, or both) a re sporadic but ca n ru n in fa m i l ies. Conditions commonly associated with a cleft palate include the fol l owing:

Discussion

Velocard i ofacial (Sh pri ntzen) synd rome-an a uto­ somal d o m i n ant d isorder ca used by deletion i n

The correct answer is "D:' See Table 9-7 for the differential diag­ nosis of oral lesions in childhood.

c h romosome 2 2 q 1 1 ; i t l i kely represents a pheno­ typic va riant of DiGeorge synd rome Va n der Woude synd rome-a n a utosomal domi­ nant d i sorder with a h i g h deg ree of penetra nce that is characterized by lower l i p sinus tracts or pits

You attend the delivery of an infant girl who was noted to have a cleft lip on prenatal ultrasound. On your initial exam, she is found to have a complete unilateral cleft lip and palate. (See Figure 9-4.) The family has lots of questions regarding this diagnosis. Question 1 3-1

All of the following are true regarding cleft lip and palates EXCEPT: A) Cleft lip occurs most often on the left side. B) Right-sided clefts are more commonly associated with syndromes. C) There is no association between a bifid uvula and a cleft palate.

Stickler syn d rome or hered ita ry a rth roophth a l mop­ athy-a n a utosomal d o m i n ant condition cha racter­ ized by a flat face, severe nearsig hted ness (myopia) and hypermobile joi nts Treacher-Col l i n s synd rome or m an d i bu lofacial dys­ ostosis-a n a utosomal d o m i n ant d isorder char­ acterized by u n derdeveloped facial bones (sma l l cheek bones, jaw a n d c h i n), downslanting eyes, cleft zygomatic bone, dental maloccl usion (over­ bite), eye (eye l i d coloboma, missing eyelashes) a bnormal ities, a n d abnormal ea rs Pierre Robin seq uence-resu lts from abnormal for­ mation of the m a n d i b l e a n d may occ u r in isolation or as a featu re of a genetic syn d rome

1 94

MCG RAW-H I LL E D U CATION S P E C I A LTY BOA R D REVI EW: P E D I ATRICS

TABLE 9-7 D I F F E R E N T I A L D I AG N O S I S O F ORAL LESIONS I N C H I L D H O O D

Diagnosis

Ca use

C l i n ica l Appeara n ce

Treatment

Hand-foot-mouth

Enterovi ru ses, g rou p A a n d B coxsackie vi ruses

Fever, oral vesicles on the bucca l m ucosa a n d to n g u e a n d s m a l l, tender cuta neous lesions on the h a n d s, feet, buttocks a n d genita l i a

S u p portive with pa i n medicati o n s and fl u i d hyd ration

Coxsackie A vi ruses

Fever a n d odynophagia d u e to pa i nfu l posterior p h a ryngea l u l cers; m a i n ly occ u r i n s u m m er a n d early fa l l

S u p portive with pa i n medicati o n s a n d fl u i d hyd ration

d isease

Herpa n g i n a

Topical thera py with 1 : 1 m ixtu re of Maalox a n d Benad ryl may be h e l pfu l•

Acute herpetic g i n g ivostomatitis

Pri m a ry H SV- 1 infection i n c h i l d hood

M u lt i p l e s m a l l 1 -3 m m u l cers o n i n ne r l i ps, bucca l m u cosa, g i ng iva, ton g u e, and a nterior to n s i l l a r p i l la rs; associated fever a n d te nder cervica l lymphaden opathy

Treatment i s sym ptomatic

Aphthous u lcer "ca n ker sore"

U n known

Pa i n fu l s ma l l, s h a l l ow, ro u n d to ova l u lcers with a g rayi s h base; occ u r o n non ke rat i n ized m u cosa l s u rfaces, so lesions on the l i ps a n d periora l lesions exc l u d e t h i s d i a g nosis; n o associ ated fever or cervica l lymphaden opathy

Topical corti costeroid (triamci nolone dental paste) may help

Ben i g n lesions of oral cavity

Oral acyclovi r thera py can decrease sym pto m d u ration if sta rted in the fi rst 3-4 days of i l l ness

Avo id sa lty o r acidic food Pa i n control with aceta m i nophen or i b u p rofen

1 . Fibroma-sm ooth, pa le p i n k protu bera nces with a ses s i l e or ped u ncu lated base that occ u r o n any m u cosa l su rface seco ndary t o co n n ective tissue hyperp lasia fro m c h ro n i c i rritation 2. Traumatic ulcer m ost com mo n ora l u l cer in c h i l d ren; ca u sed by mechan ical, c h e m i ca l or therm a l i nj u ry with the a ppea ra nce depe n d i n g o n the i nfl icted tra u ma 3. Geographic tongue (be n i g n m i g ratory g l ossitis)-chro n i c, rec u r r i n g d i sorder cha racte rized by red -pi n k, s l i g htly depressed lesions with i rreg u l a rly e l evated wh ite or ye l low borders; more co m m o n i n g i rl s 4 . Fissured tongue ( l i n g u a p l i cata)-developmenta l a n o m a l y with a pro m i nent centra l fi g u re o n t h e to n g u e fro m w h i c h s m a l l e r fi s s u res rad iate latera l ly; co m m o n fi n d i n g i n Down syn d rome 5. Hemangioma-red o r b l u i s h-red, s l i g htly ra i sed lesions that ca n occ u r i n a ny soft tissue l ocation but more co m monly o n the l i p, d o rs u m of to n g u e, g i ng iva a n d buccal m u cosa 6. Lymphangiomas-benign t u mors of the lymphatic vesse l s that may be p i n k to red d i sh-bl u e, soft a n d com press i b le; m ost co m m o n ly seen on the to n g u e, l i ps, a n d bucca l m u cosa 7. Mucocele-pa i n less swel l i ng o n the l ower l i p buccal m u cosa, u s ua l ly < 1 em d i a m eter a n d tra n s l ucent or b l u i s h i n co lor -

H SV, herpes s i m plex virus. ' Maalox = a l u m i n u m hyd roxide a n d m a g n e s i u m hyd roxide; Benadryl = d i phenhyd ra m i ne.

� QUICKQUIZ The clinical features of Pierre Robin sequence are microgna­ thia, glossoptosis (posteriorly displaced tongue), and cleft palate, with the tongue tending to prolapse backward and cause airway obstruction that can be life-threatening. All of the following are appropriate initial interventions for a patient with Pierre Robin sequence EXCEPT: A) Tracheostomy. B) Glossopexy or tongue-lip plication surgery.

C) Insertion of nasopharyngeal airway. D) Mandibular distraction surgery. E) Placing the patient in the prone position. Discussion

The correct answer is "A:' In most cases, the respiratory obstruction associated with Pierre Robin sequence is seen in the immediate neonatal period and will improve over time as the mandible grows to accommodate the tongue. Conservative measures include placing the infant in the prone positioning to allow the tongue to "fall forward" and alleviate the obstruc­ tion, placing a nasogastric feeding tube, or inserting an oral or

C H A PT E R 9



EAR, N O S E, A N D TH ROAT D I S O R D E RS

1 95

You are attending on the general pediatric service and have a 4-year-old male patient who was admitted with fever and unilateral neck swelling. Exam reveals a 3 x 4 em area of right-sided cervical swelling that has central fluctuance. It is tender to palpation and the overlying skin is erythematous. The patient was previously healthy and has no known animal exposures. He does not appear to have poor dentition. Question 1 4-1

F I G U R E 9-4. This i nfa nt has a com p l ete u n i l ateral c left l i p that extends i nto the nasal s i l l . (Reproduced with permission from La lwa ni AK, ed. Current Diagnosis & Treatment in Otolaryngology: Head and Neck Surgery. 3 rd ed. McGraw- H i l l Education; 201 1 , Fig 20-4.)

nasopharyngeal airway as a temporizing measure. However, if these measures fail then surgical intervention is warranted. The goal of surgery is to avoid a tracheostomy (option "A") . Glos­ sopexy or tongue-lip plication surgery is a procedure in which the tongue is essentially sutured in place anteriorly to the lip. More recently, mandibular distraction surgery has been used to elongate the mandibular ramus and bring the tongue forward with the mandible.



Helpful Tip

� Pri m a ry teeth

-

i1 1 r age (ra nge:

beg i n to eru pt at a ro u n d 7 months of

3-16 months). The fi rst teeth seen a re the

m a n d i b u l a r centra l i n cisors fol l owed by the maxi l l a ry

centra l i ncisors.

� •

I I

Helpful Tip

The prog nosis for viabil ity worsens ra pidly the longer a tooth is outside the mouth. Ideal ly, an avu lsed permanent tooth should be rei mpla nted i nto its socket as soon as possible after a gentle rinsing with clean water, a n d the patient should seek emergency denta l ca re. The best storage a n d tra nsport media i n order of preference is H a n k's ba la n ced salt sol ution (fo u n d i n t h e commerci a l ly ava i lable "Save-A-Tooth" kit), m i l k, sa l i ne, sal iva, or water.



� I

Helpful Tip

The loss of a primary tooth may either accelerate or delay

1 1 r eruption of the underlying permanent tooth. A general­

Acute unilateral cervical lymphadenitis is commonly caused by all of the following EXCEPT: A) Streptococcus pyogenes (GAS). B) Staphylococcus aureus. C) Atypical mycobacteria. D) Bartonella henselae. E) Human immunodeficiency virus. Discussion 1 4-1

The correct answer is "E:' This patient has acute, unilateral cervical lymphadenitis, which is most often caused by either Streptococcus pyogenes (GAS) or Staphylococcus aureus. Atypi­ cal mycobacteria are usually a more indolent presentation with a characteristic violaceous appearance overlying matted lymph nodes. Bartonella henselae causes cat-scratch disease and should be considered if there is a history of cat exposure (especially kit­ tens) . In a young infant, you should consider group B strepto­ coccus (GBS) cellulitis-adenitis, which can be a manifestation of late-onset GBS infection. Tularemia caused by Francisella tular­ ensis after contact with an infected animal such as a rabbit or pet hamster may present as the ulceroglandular syndrome, which is characterized by a papular lesion in the drainage field of the inflamed lymph node. Older children with a history of dental disease may have acute unilateral cervical lymphadenitis due to anaerobic bacteria. The clinical history is very important in a child with cervical lymphadenitis as a detailed exposure history may help elicit possible infectious associations such as brucel­ losis from ingesting unpasteurized animal milk or goat expo­ sure, atypical mycobacterium from ingesting unpasteurized milk, tularemia from a rabbit or pet hamster exposure, bubonic plague from prairie dog exposure, cat scratch disease from kit­ ten exposure, or tuberculosis from travel to an endemic area. Question 1 4-2

What is the most appropriate next step in the care of this patient? A) Order a CBC with white blood cell count differential and blood culture. B) Arrange for an incision and drainage procedure. C) Order a tuberculin skin test (PPD ) . D) Start antibiotic therapy with oral azithromycin. E) Order a rapid strep test and throat culture.

ized delay in tooth eruption may be associated with medi­

ca l conditions such as hypothyroidism, hypopituitarism, cleidocranial dysplasia, trisomy 21, or rickets.

Discussion 1 4-2

The correct answer is "B:' This patient's exam is notable for an area of central fluctuance, which indicates a drainable fluid

MCG RAW-H I LL E D U CATION S P E C I A LTY BOA R D REVI EW: P E D I ATRICS

1 96

collection. Although it is appropriate to start antibiotics effective against both Streptococcus pyogenes and Staphylococcus aureus, the most important first step is to drain the fluid collection. In a young infant with possible GBS cellulitis-adenitis, it would be important to send blood for culture as such children are often bacteremic. If you suspect either tuberculosis or atypical myco­ bacteria as a possible etiology, then administering a tuberculin skin test (PPD) would be appropriate. •

Helpful Tip

:5.� It

is

usefu l

to

r1 1r lym phadenopathy

sepa rate i nto

c h ro n i c

u n i l atera l

or

cervica l b i l atera l

categ ories, as the u n i latera l form is u s ua l ly cau sed by cat-scratch d i sease or atypica l mycobacteria,

whereas the b i l atera l form is more commonly ca u sed by vira l i nfections such as Epste i n-Barr virus ( E BV) or cytomeg a l ovirus (CMV). Remember to co nsider the poss i b i l ity of l e u kemia, lym phoma, or other m a l i g n a ncies or i m m u nodeficiencies if a patient presents with c h ro n i c cervica l lymphaden opathy.

A mother brings her 10-year-old daughter in for evaluation of a "neck mass?' She reports that the child has a history of recurrent midline neck swelling for the past several years. The patient was healthy until last week when she developed a low-grade fever, rhinorrhea, and coughing. Mom then noticed a prominent midline neck swelling that appears erythematous and tender to palpation. You observe that the swelling moves upward when the patient sticks out her tongue during your exam.

TABLE 9-8 D I F F ERENTIAL DIAGNOSIS OF PEDIATRIC N ECK MASSES

I nfectious or I nfl a m matory Neck Masses

Congenital Neck Masses

Neoplastic Neck Masses

Reactive viral lymphadenopathy (adenovi rus, rhinovi rus, enterovirus, EBV, etc)

Thyrog I ossa I d uct cysts

Lym phoma

Bacterial lymphadenitis

B ra n c h i a l c left cysts

Rhabdomyosarco ma

Dermoid cysts

N e u roblasto ma

Atypical mycobacteri u m

Te rato mas

Metastatic adenopathy

Cat-scratch d i sease (Bartonella

Thyroid cancer

henselae)

Lym p h an g i o m a (cystic hyg ro ma)

Toxoplasmosis

Thym i c cysts

H i sto plasmosis

Laryngoceles

Actinomycos i s

Lym phatic ma lformati o n s

H IV-associ ated lymphadenopathy

Hemangioma

Sarcoidosis

F i b romatosis co I I i

(Staphylococcus aureus, g ro u p

A streptococci [GAS], t u l a re m ia, bruce l losis, etc) Mycobacterium tuberculosis

Question 1 5-1

Which of the following is the most likely diagnosis? A) Lymphoma. B) Infected brachial cleft cyst. C) Cystic hygroma. D) Reactive viral lymphadenopathy. E) Infected thyroglossal duct cyst. Discussion 1 5-1

The correct answer is "E:' The differential diagnosis for a pediatric neck mass is extensive and most easily understood by grouping into broad categories of infectious or inflammatory neck masses, con­ genital neck masses and neoplastic neck masses. (See Table 9-8.) A thyroglossal duct cyst is an epithelium-lined cyst resulting from the persistence of any segment of the thyroglossal duct along its migration from the foramen cecum of the tongue to the pyramidal lobe of the thyroid. It is the most common midline neck mass in children. Remember that, during formation, the thyroid migrates down from the base of the tongue to its location in the neck. Thy­ roglossal duct cysts are typically found near the level of the hyoid bone in the midline of the neck. Clinically, they most often appear as a painless, fluctuant midline neck mass that moves upward with a

Kawasaki d i sease Ada pted with p e r m i ssi o n fro m Tinti na l l i J E, Sta pczynski J, Ma 0, et al. eds. Tin tin alli's Emergency Medicine: A comprehensive s tudy guide, 8th ed. N ew Yo rk, NY: M c G raw-H i l l ; 20 1 5 .

protruding tongue or during swallowing. It is important to evaluate thyroid function because thyroglossal duct cysts may contain ecto­ pic thyroid tissue with associated hypothyroidism. By comparison, branchial cleft cysts are round, smooth, mobile lateral neck masses found along the anterior border of the sternocleidomastoid mus­ cle. Most of these arise from incomplete obliteration of the second branchial cleft. Patients may have a history of recurrent swelling or infection in the same lateral neck area. Both thyroglossal duct cysts and branchial cleft cysts are often asymptomatic until they become infected in the setting of an upper respiratory tract infec­ tion. Acutely infected cysts require antibiotics to cover typical skin flora and anaerobes. Definitive treatment is elective surgical exci­ sion. A cystic hygroma or lymphangioma is a painless lymphatic malformation commonly located above the clavicle.

C H A PT E R 9

You are seeing a 1 5-year-old adolescent girl with a chief com­ plaint of "neck swelling:' She reports noticing a small bump in the midline of her neck for the past several weeks that seems to have gotten significantly larger in the past several days. Physical exam is notable for bilateral anterior cervical lymphadenopathy and a nontender, 2 em right-sided thyroid nodule. You notice that her voice sounds rather hoarse. Given the history and physical exam, you are concerned that she may have thyroid cancer. Question 1 6-1

All of the following are clinical features that may be associ­ ated with thyroid carcinoma EXCEPT: A) History of external radiation to the head or neck, or both. B) History of exposure to nuclear fallout. C) Bradycardia. D) Dysphagia. E) Vocal cord paralysis. Discussion 1 6-1

The correct answer is "C:' The most common presentation of a patient with thyroid cancer is the presence of a solitary thyroid nodule or mass. Although approximately 2% of children have palpable thyroid nodules, most of these are benign adenomas or cystic lesions. Patients with thyroid cancer may have a history of external radiation to the head and neck, exposure to nuclear fallout, a history of rapid growth of the thyroid nodule, a firm or fixed neck mass, hoarseness, dysphagia, or cervical lymph­ adenopathy. Thyroid cancer is divided into four main types: papillary, follicular, medullary, and anaplastic. In children, the vast majority of masses are differentiated thyroid cancer, which includes both papillary and follicular thyroid carcinomas. Med­ ullary thyroid carcinoma (MTC) is notable due to the pro­ duction of calcitonin from the parafollicular or C cells of the thyroid gland, and it may be associated with multiple endocrine neoplasia type 2A (MEN 2A) or MEN 2B. Diagnosis of thyroid carcinoma is made by fine needle aspiration biopsy.



EAR, N O S E, A N D TH ROAT D I S O R D E RS

1 97

D) A child should be referred for formal hearing testing if the caregiver has a concern regarding hearing, speech, language, or developmental delay at any age E) A failed otoacoustic emission test (OAE) indicates an abnor­ mality in the auditory nerve or brainstem. Discussion

The correct answer is "E:' An OAE does not assess the neuronal transmission of sound from the eighth cranial nerve to the brain­ stem as does an auditory brainstem response (ABR). OAE detects conductive hearing loss and ABR detects conductive and senso­ rineural hearing loss. The remaining statements are all true. Uni­ versal hearing screening is recommended because some degree of hearing loss is present in 1 to 6 per 1000 newborn infants, and congenital or acquired hearing loss in children has been shown to adversely affect speech development, language development, aca­ demic achievement, and social-emotional development. Accord­ ing to the "Year 2007 Position Statement: Principle and Guidelines for Early Hearing Detection and Intervention Programs" endorsed by the AAP, the hearing of all infants should be screened at no later than 1 month of age. Those who do not pass screening should have a comprehensive audiologic evaluation at no later than 3 months of age. Additionally, the AAP recommends hearing screening for all children at ages 4, 5, 6, 8, and 10 years. Any child with a risk factor for hearing loss, regardless of the newborn hearing screen result, should be referred for an audiologic assessment at least once by 24 to 30 months of age. (See Table 9-9.) TABLE 9-9 R I S K FACTO RS ASSOCIATED WITH H EA R I N G LO SS IN C H I L D H O O D

Caregiver concern reg a rd i ng heari ng, speech, l a n g u ag e, or developme nta l delay Fa m i ly h i story of hea r i n g i m p a i rment NICU stay of > 5 days or a ny of the fo l l owi n g : ECMO, assisted ve nti lation, expo s u re to ototoxic medications, hyperb i l i ru bi ne m i a req u i ri n g exc h a n g e tra n sfusion Confirmed neonata l i nfections associated with hearing loss such as CMV, herpes, rubella, syphilis, or toxoplasmosis Anato m i c ma lformati o n s of the head and neck

� QUICKQUIZ You are the attending in the newborn nursery and have been asked to give a lecture to the medical students and nursing staff regarding universal hearing screening. All of the following regarding hearing tests are true state­ ments EXCEPT: A) The AAP recommends that all infants have a hearing screen performed by 1 month of age. B) A behavioral audiogram is the gold standard for hearing testing. C) Mild hearing loss is present when the quietest sounds that a person can hear with his or her better ear is between 20 and 40 dB

A syn d rome that i s associ ated with hea r i n g loss such a s n e u rofi bromatosis, osteogenesis i m perfecta, U s h e r, Waa rd e n b u rg, AI po rt, Pe nd red, Jerve l l, or La nge- N i e l so n N e u rodegenerative d i sorders such as H u nter syn d rome Sensory m otor n e u ropathies such as Fried reich ataxia and C h a rcot-Marie-Tooth syn d rome Confi rmed i ncidence of i n fectious d i sease associ ated with hea r i n g loss, such as bacterial or v i ra l (espec i a l l y h e rpes virus a n d va rice l la) m e n i ngitis Chemothera py Rec u rre nt or persi ste nt otitis med ia for at least 3 months H i story of s i g n ificant head tra u m a (especi a l ly i nvo lvi ng the bas i l a r s ku l l or tem po ra l bone) CMV, cytomegalovi rus; ECMO, extracorporea l membra n e oxygen­ ation; N ICU, neonata l i ntensive ca re u n it.

MCG RAW-H I LL E D U CATION S P E C I A LTY BOA R D REVI EW: P E D I ATRICS

1 98



� I

Helpful Tip

M i l d hearing loss is present when the q u i etest sou n d s

l l r that a person ca n hear with his or her better ea r is

between 20 a n d 40 d B . These i n d ivid u a ls have some

d ifficu lty keeping u p with conversations, especia l ly i n noisy s u rro u n d i ngs. Moderate hearing l o s s is i n t h e 4 1 t o 70 d B ra nge, a n d affected people have d ifficulty keeping u p with conversations without a hea ring a i d . Severe hearing l o s s is i n the 7 1 to 9S d B ra nge, a n d affected people benefit from hea ring aids b u t often



� 1

Helpful Tips

It is i m porta nt to remem ber that OAEs or ABRs a re testing t h e i nteg rity o f the a u d itory pathway

1 1r only

and a re not true tests of hearing. Hearing can not be defi n itively considered norma l u ntil a c h i l d is mature enough to partici pate in a behaviora l a u d iogra m, which is the gold sta ndard for hearing eva l uation. Everyone remem bers the behaviora l a u d iogra m s i n school w h e n y o u were g iven earphones a n d asked to ra ise a hand when a sound was heard.

rely on l i p rea d i n g a n d may use sign l a n g uage.



Helpful Tip

1r

A va riety of objective tests have been developed for hearing scree n i n g . The choice of which to use depends

� 1 I

on the chi ld's age, deg ree of cooperation, a n d ava i l a b l e resou rces. Tympanometry measures relative changes in tym pa nic mem brane movement as air pressure cha nges in the externa l a u d itory ca n a l . It provides

the best objective m i d d l e ear assessment and is u sefu l i n the eva l uation of suspected hea ring loss. Evoked otoacoustic emissions (OAEs) are acoustic signals

generated within outer hair cel ls of the cochlea that travel in a "reverse direction" through the middle ear space and tympanic membrane out to the ear canal. These signals are generated in response to an auditory stimulus and may be detected with a very sensitive microphone-and-probe system placed in the external ear canal. The benefits of an OAE test are that you can obtain ear-specific results, it can be performed quickly at any age, it does not depend on whether the child is asleep or awake, and it provides a simple "pass/fail" report. However, an OAE test does not assess the neuro­ nal transmission of sound from the eighth cranial nerve to the brainstem and thus will not identify nerve abnor­ malities (sensorineural hearing loss). A "failed" OAE test only implies that a hearing loss of greater than 30 to 40 d B may exist or that the middle ear is abnormal. A physi­ cal obstruction ofthe ear canal (eg, vernix in a newborn) will also give a false "failed" result. An automated brainstem response (ABR) test involves placing surface electrodes on the child's head and recording neural activity generated in the cochlea, audi­ tory nerve, and brainstem in response to acoustic stimuli presented through earphones or ear inserts. The benefit of an ABR is the assessment of the eighth cranial nerve and brainstem auditory pathway in addition to the peripheral auditory system that is evaluated by an OAE. The main disadvantages of an ABR are the increased time required to perform the test (about 1 5 minutes) and the need for the child to remain quiet and not move. Consequently, infants may require sedation. A "failed" ABR test implies a hearing loss of greater than 40 d B.

� QUICKQUIZ At the end of your lecture regarding universal hearing tests, a medical student remains confused regarding the difference between conductive hearing loss and sensorineural hearing loss (SNHL) . All of the following are true statements regarding hearing loss EXCEPT: A) The most common congenital infection associated with sen­ sorineural hearing loss is toxoplasmosis. B) Conductive hearing loss results from a mechanical problem in the outer or middle ear and involves the pinna, external auditory canal, tympanic membrane, or ossicles. C) Sensorineural hearing loss results from inner ear problems involving the cochlea, eighth cranial nerve, internal audi­ tory canal, or the brain. D) The most common childhood disorder associated with con­ ductive hearing loss is otitis media. E) In general, patients need to have severe to profound bilat­ eral SNHL and little or no benefit from hearing aid use after 6 months before being considered for a cochlear implant. Discussion

The correct answer is "A:' Congenital CMV is the most common infection associated with SNHL. SNHL results from an inner ear or nerve problem and may be caused by congenital anomalies, infection (bacterial meningitis, CMV, toxoplasmosis, rubella, or syphilis) , exposure to ototoxic medications, or due to genetic inheritance. SNHL is more prevalent among premature infants with low birth weights, likely due to other factors including administration of ototoxic drugs and perinatal complications, including hyperbilirubinemia. Medications known to be asso­ ciated with permanent SNHL include aminoglycosides, mac­ rolides, vancomycin, tetracycline, aspirin, NSAIDs, high-dose loop diuretics, and chemotherapy agents such as cisplatin, 5-flu­ orouracil, and bleomycin. In comparison, conductive hearing loss results from a mechanical problem in the outer or middle ear and may be caused by congenital anomalies, infection (such as otitis externa or otitis media) , penetrating trauma, tympanic membrane perforation, otosclerosis, cholesteatoma, or malig­ nant tumors such as squamous cell carcinoma.

C H A PT E R 9



EAR, N O S E, A N D TH ROAT D I S O R D E RS

1 99

Discussion 1 7-2

You are seeing a 1 2-year-old boy whose chief complaint is that "my nose is constantly dripping:' The patient and his mother report that he has had persistent rhinorrhea and sneezing for the past 2 months. It is not associated with changes in the temperature or exposure to tobacco smoke but is definitely worse when he is asked to mow the lawn outside. On physical exam his nasal turbinates are swol­ len and erythematous bilaterally and there is a cobblestone appearance to the posterior pharyngeal wall. You diagnose him with allergic rhinitis and begin to discuss treatment options. Question 1 7-1

Which of the following is the first-line medication therapy for allergic rhinitis? A) Oral antihistamines. B) Leukotriene antagonists. C) Intranasal antihistamines. D) Intranasal corticosteroids. E) Nasal saline rinses. Discussion 1 7-1

The correct answer is "D:' The differential diagnosis of chronic rhinitis includes allergic or nonallergic rhinitis. Allergic rhini­ tis affects up to 40% of children in the United States and rep­ resents an IgE-mediated inflammatory response to allergen exposure. Symptoms include nasal congestion, rhinorrhea, sneezing, and an itchy nose, throat, or eyes. Physical exam may reveal swollen and erythematous nasal turbinates. The most important treatment modality is the use of intranasal cortico­ steroids, which has been shown to improve symptoms and help prevent progression to more severe disease. Other treatment options include oral and intranasal antihistamines as well as leukotriene antagonists. Nasal saline rinses have been shown to be helpful by washing away the allergens. Nonallergic rhinitis presents with similar symptoms, but does not seem to involve an immunologic reaction. Triggers may include environmental temperature changes or air pollution. Rhinitis medicamentosa is "rebound" nasal congestion that may occur after the discon­ tinuation of chronic nasal decongestant spray use. Look for the allergic salute-a horizontal crease across the nose from rub­ bing the tip of the nose.

On close exam, you notice a pale grapelike mass in his right nostril. He does not remember being previously told about it. He has no history of chronic cough, greasy stools, or recur­ rent nosebleeds.

The correct answer is "D:' Nasal polyps frequently present as smooth, pale, spherical mucosal masses that protrude from the middle meatus. A classic description is a "gray or pale appear­ ing grapelike mass:' They develop after recurrent episodes of mucosal edema such as may be seen in longstanding allergic rhinitis, aspirin-sensitive asthma, chronic infectious sinusitis, or cystic fibrosis. Not all polyps are benign, and they may rep­ resent malignancy such as glioma, lymphoma, neuroblastoma, juvenile nasopharyngeal angiofibroma, or rhabdomyosarcoma. Children with polyps and symptoms of allergic rhinitis should be evaluated for allergies, as they will likely benefit from using a topical nasal corticosteroid or receiving a course of oral cortico­ steroids. Any child with multiple benign nasal polyps should be evaluated for cystic fibrosis-the most common cause of polyps in children. Polyps and recurrent infections should trigger an immunologic workup. Surgical removal is an option if medi­ cal management has failed or the polyps are very large in size. However, it is not uncommon for nasal polyps to recur after sur­ gical removal. The use of intranasal corticosteroids after polyp removal may help prevent this recurrence. •

Helpful Tip

� The most common ca use of nasal

=-

i1 1r is

polyps i n c h i l d ren

cystic fi brosis. Typical res pi ratory a n d d igestive sym ptoms might be a bsent. Sweat the patient!

� QUICKQUIZ Instead let us say the 12-year-old presented for evaluation of rhinorrhea during the winter. He has cough and congestion. He still claims "my nose is constantly dripping:' With more questioning, he admits that his symptoms improve for a day or two before fully returning. He is in school and has younger sib­ lings. On exam, he has erythematous nasal mucous with clear drainage. His throat is mildly erythematous. You diagnosis him with the common cold and recommend supportive care. Which of the following statements about acute viral rhinitis is false? A) It is the most common pediatric infectious disease. B) Recurrent episodes only occur in children with an immunodeficiency. C) Healthy children may have over 1 0 illnesses per year. D) Multiple different viruses are to blame. E) A single episode typically last 7 to 10 days.

Question 1 7-2

Discussion

What is NOT a cause of his intranasal mass? A) Cystic fibrosis. B) Chronic allergic rhinitis. C) Aspirin-sensitive asthma. D) Viral rhinitis. E) Chronic bacterial sinusitis.

The correct answer is "B:' A viral upper respiratory tract infec­ tion is the most common pediatric infectious disease. Children younger than 5 years of age often have 6 to 12 illnesses per year with each episode lasting up to 10 days. No wonder parents always feel that their young children are "always sick!" Rhinovi­ ruses are the most commonly isolated agents (30-40% of cases),

200

MCG RAW-H I LL E D U CATION S P E C I A LTY BOA R D REVI EW: P E D I ATRICS

with other identified viruses including adenoviruses, coronavi­ ruses, enteroviruses, influenza, parainfluenza, and respiratory syncytial virus. The typical clinical course consists of clear or mucoid rhinorrhea, nasal congestion, and a sore throat that lasts 7 to 10 days. Fever may be present and is more commonly seen in younger children. Recurrent bacterial sinopulmonary infections would be a red flag for an immunodeficiency.

You are seeing a 4-year-old, former 26-week-premature boy in your office for his well-child check. A review of his chart shows that he was intubated in the neonatal intensive care unit (NICU} for 2 weeks but subsequently discharged home without supplemental oxygen, has a history of gastroesopha­ geal reflux for which he takes omeprazole, and was diagnosed with mild global developmental delay. His mother reports that he is doing well but that "he sounds hoarse" all the time. She reports that he frequently clears his throat and seems to have a new chronic cough that developed over the past week. He had a low-grade fever several days ago, and his younger sister has been ill recently. Question 1 8-1

All of the following are potential causes of his new symp­ toms, EXCEPT: A) Acute viral laryngitis. B) Subglottic stenosis. C) Gastroesophageal reflux. D) Vocal fold granuloma. E) Vocal fold nodule. Discussion 1 8-1

The correct answer is "E:' Vocal fold nodules are a common cause of chronic hoarseness in school-aged children and develop from repeated traumatic abuse of the vocal folds, such as from screaming or shouting. This patient is young to have developed vocal fold nodules and there is no supporting information pro­ vided to make you worry that he is "abusing his voice:' The remaining choices are all potential causes of hoarseness in this particular patient. Although hoarseness in children is most often secondary to acute viral laryngitis or benign lesions of the vocal cords, the differential diagnosis is quite broad. Acute viral laryngitis may persist for up to a week after other symptoms of an upper respi­ ratory tract infection have resolved. Gastroesophageal reflux with associated laryngopharyngeal reflux may cause chronic inflammation of the vocal folds. Subglottic stenosis and vocal fold granulomas usually occur secondary to traumatic or pro­ longed intubation. Vocal cord paralysis is a common cause of stridor in neonates or infants who have undergone a surgical intervention that may be associated with damage to the recur­ rent laryngeal nerve (such as certain congenital heart disease surgeries) . Papillomas secondary to human papillomavirus (HPV) types 6 and 1 1 typically become symptomatic in chil­ dren by causing progressive hoarseness and stridor over weeks

to months. Congenital anomalies that may cause hoarseness include laryngeal webs, laryngeal clefts, or hemangiomas. Any patient with hoarseness that has persisted beyond 2 weeks should be evaluated by an otolaryngologist and undergo laryngoscopy. B I B L I O G RA P H Y

American Academy of Pediatrics. Group A streptococcal infections. In: Pickering LK, ed. Red Book: 2012 Report of the Committee on Infectious Diseases. 29th ed. Elk Grove Village, IL: American Academy of Pediatrics; 2 01 2. American Academy of Pediatrics, Joint Committee o n Infant Hearing. Year 2007 position statement: Principles and guidelines for early hearing detection and intervention programs. Pediatrics. 2007; 120(4):898-92 1 . Baugh RF, Archer SM, Mitchell RB, et al; American Academy of Otolaryngology-Head and Neck Surgery Foundation. Clinical practice guideline: Tonsillectomy in children. Otolaryngol Head Neck Surg. 20 1 1 ; 144( 1 suppl) : S 1 -S30. Brown JC, Osincup DP. Pediatric procedures: Nasal and otic foreign bodies. In: Tintinalli JE, Stapczynski J, Ma 0, et al, eds. Tintinalli's Emergency Medicine: A Comprehensive Study Guide. 7th ed. New York, NY: McGraw-Hill; 201 1 . Brown KD, Banuchi V, Selesnick SH. Diseases of the external ear. In: Lalwani AK, ed. Current Diagnosis & Treatment in Otolaryngology: Head & Neck Surgery. 3rd ed. New York, NY: McGraw-Hill; 2 0 12 . Burrow TA, Saal HM , de Alarcon A, e t al. Characterization of congenital anomalies in individuals with choana! atresia. Arch Otolaryngol Head Neck Surg. 2009 ; 1 3 5 (6):543. Busaidy N, Habra M, Vassilopoulou-Sellin R. Endocrine malignancies. In: Kantarjian HM, Wolff RA, Koller CA. eds. The MD Anderson Manual ofMedical Oncology. 2nd ed. New York, NY: McGraw-Hill; 20 1 1 . Friedman NR, Scholes MA, Yoon PJ. Ear, nose, and throat. In: Hay WW, Levin MJ, Deterding RR, Abzug MJ, eds. Current Diagnosis & Treatment: Pediatrics. 22nd ed. New York, NY: McGraw-Hill; 20 1 3 . Goldstein NA, Hammerschlag MR. Peritonsillar, retropharyn­ geal and parapharyngeal abscesses. In: Feigin RD, Cherry JD, Demmler-Harrison GJ, Kaplan SL, eds. Textbook of Pediatric Infectious Diseases. 6th ed. Philadelphia, PA: Saunders; 2009: 1 77- 185. Harlor AD, Jr, Bower C; Committee on Practice and Ambula­ tory Medicine, Section on Otolaryngology-Head and Neck Surgery. Hearing assessment in infants and children: Recommendations beyond neonatal screening. Pediatrics. 2009; 1 24: 1252- 1263. Hoffman WY. Cleft lip and palate. In: Lalwani AK, ed. Current

Diagnosis & Treatment in Otolaryngology: Head & Neck Surgery. 3rd ed. New York, NY: McGraw-Hill; 201 2.

Joint Committee on Infant Hearing, American Academy of Audiology, American Academy of Pediatrics, American Speech-Language-Hearing Association, Directors of Speech and Hearing Programs in State Health and Wel­ fare Agencies. Year 2000 position statement: Principles and guidelines for early hearing detection and interven­ tion programs. Pediatrics. 2000; 1 06 ( 4} :798-8 1 7 .

C H A PT E R 9

Kentab OY, Qureshi N. Neck masses in children. In: Tintinalli JE, Stapczynski J, Ma 0, et al, eds. Tintinalli's Emergency Medicine: A Comprehensive Study Guide. 7th ed. New York, NY: McGraw-Hill; 20 1 1 . Klein U. Oral medicine and dentistry. In: Hay WW, Levin MJ, Deterding RR, Abzug MJ, eds. Current Diagnosis & Treat­ ment: Pediatrics. 22nd ed. New York, NY: McGraw-Hill; 20 1 3 . LeBlond RF, Brown DD, Suneja M , Szot J F. Th e head and neck. In: LeBlond RF, Brown DD, Suneja M, Szot JF, eds. DeGowin's Diagnostic Examination. l Oth ed. New York, NY: McGraw-Hill; 2014. Lichten SR. Retropharyngeal abscess. In: Gerschel J, Rauch D. eds. Caringfor the Hospitalized Child: A Handbook of Inpatient Pediatrics. Elk Grove Village, IL: American Academy of Pediatrics; 20 1 3 . Lieberthal AS, Carroll AE, Chonmaitree T, e t al. Th e diag­ nosis and management of acute otitis media. Pediatrics. 20 1 3 ; 1 3 1 :e964-e999. Lustig LR, Schindler JS. Ear, nose, and throat disorders. In: Papadakis MA, McPhee SJ, Rabow MW, eds. Current Medical Diagnosis & Treatment 201 5. New York, NY: McGraw-Hill; 2014. Lye C, Nead JA, Chase L. Parotitis. In: Gerschel J, Rauch D, eds.

Caringfor the Hospitalized Child: A Handbook of Inpatient Pediatrics. Elk Grove Village, IL: American Academy of Pediatrics; 20 1 3 . Macfadyen CA, Acuin JM, Gamble CL. Systemic antibi­ otics versus topical treatments for chronically dis­ charging ears with underlying eardrum perforations. Cochrane Database ofSyst Rev. 2006 ( l ) :CD005608. doi: 10. 1 002/ 1465 1 858.CD005608. McArdle AJ, Shroff R. Question 3 : Is ultrasonography required to rule out congenital anomalies of the kidneys and urinary tract in babies with isolated preauricular tags or sinuses? Arch Dis Child. 20 1 3 ;98:84-87. Mittiga MR, Gonzalez del Rey JA, Ruddy RM. Pediatric condi­ tions. In: Knoop KJ, Stack LB, Storrow AB, Thurman R, eds. The Atlas of Emergency Medicine. 3rd ed. New York, NY: McGraw-Hill; 2 0 10 . Rosenfeld RM, Schwartz SR, Cannon CR, e t al. Clinical prac­ tice guideline: Acute otitis externa. Otolaryngol Head Neck Surg. 2014; 1 50 ( 1 suppl) : S l -S24. Rosenfeld RM, Culpepper L, Yawn, B, Mahoney MC; AAP, AAFP, AAO-HNS Subcommittee on Otitis Media with



EAR, N O S E, A N D TH ROAT D I S O R D E RS

201

Effusion. Clinical practice guideline: Otitis media with effusion. Otolaryngol Head Neck Surg. 2004; 1 30:S95. Rubin MA, Ford LC, Gonzales R. Pharyngitis, sinusitis, otitis and other upper respiratory tract infections. In: Longo DL, Fauci AS, Kasper DL, et al, eds. Harrison's Principles of Internal Medicine. 1 8th ed. New York, NY: McGraw­ Hill; 2 0 12 . Shah RN, Cannon TY, Shores CG . Infections and disorders of the neck and upper airway. In: Tintinalli JE, Stapczynski J, Ma 0, et al, eds. Tintinalli's Emergency Medicine: A Com­ prehensive Study Guide, 7th ed. New York, NY: McGraw­ Hill; 20 1 1 . Spiegel JH, Numa W Nasal trauma. In: Lalwani AK, ed. Cur­

rent Diagnosis & Treatment in Otolaryngology: Head & Neck Surgery. 3rd ed. New York, NY: McGraw- Hill; 20 1 2 .

Summers SM, Bey T. Epistaxis, nasal fractures, and rhinosi­ nusitis. In: Tintinalli JE, Stapczynski J, Ma 0, et al, eds.

Tintinalli's Emergency Medicine: A Comprehensive Study Guide. 7th ed. New York, NY: McGraw- Hill; 20 1 1 . Suurna MV. Congenital nasal anomalies. In: Lalwani AK, ed.

Current Diagnosis & Treatment in Otolaryngology: Head & Neck Surgery. 3rd ed. New York, NY: McGraw-Hill; 20 1 2 .

Usatine RP, Smith MA, Chumley H S , Mayeaux EJ. Otitis media: Acute otitis and otitis media with effusion. In: Usatine RP, Smith MA, Chumley HS, Mayeaux EJ, eds. The Color Atlas of Family Medicine. 2nd ed. New York, NY: McGraw-Hill; 20 1 3 . Verhoeff M , Van Der Veen EL, Rovers MM, e t al. Chronic sup­ purative otitis media: A review. Int f Pediatr Otorhinolar­ yngol. 2006;70( 1 ) : 1 - 1 2. Wald ER, Applegate KE, Bordley C, et al. Clinical practice guideline: Diagnosis and management of acute bacte­ rial sinusitis in children aged 1 to 18 years. Pediatrics. 20 1 3 ; 1 32:e262-280. Wang RY, Earl DL, Ruder RO, Graham JM. Syndromic ear anomalies and renal ultrasounds. Pediatrics. 200 1 ; 108(2) :e32-e38. Weinberger PM, Terris DJ. Otolaryngology-Head and neck surgery. In: Doherty GM, ed. Current Diagnosis & Treat­ ment: Surgery. 1 3th ed. New York, NY: McGraw-Hill; 2010. Zhorne D, Nead JA, Chase L. Retropharyngeal abscess. In: Gerschel J, Rauch D, eds. Caringfor the Hospitalized Child: A Handbook of Inpatient Pediatrics. Elk Grove Village, IL: American Academy of Pediatrics; 20 1 3 .

This page intentionally left blank

10

Emergency Ca re S a ra h L. M i l l e r

You are seeing a 1 0 day-old male infant in clinic for a weight check. He was delivered at term by uncomplicated vaginal delivery. A rectal temperature obtained in triage measured 38. 1 °C ( 1 00.7°F) . The child's mother states that he has been taking less with each feeding today. His siblings have had cough, congestion, and fever over the past week. On exam the infant is sleeping but easily arouses. His fontanelle is neither bulging nor sunken. He does have some clear rhinorrhea. Mucous membranes are moist. Lungs are clear with normal work of breathing, and abdomen is benign. Extremities are warm and well-perfused. He is not jaundiced. Question 1 -1

The most appropriate management of this patient includes: A) Continue to monitor at home as he likely has a cold like his siblings. B) Administer ceftriaxone intramuscularly (IM) and discharge to home with plan to follow up in clinic tomorrow for reevaluation. C) Order a chest X-ray and obtain a catheterized urine sample for urinalysis and culture. D) Transfer to the emergency department for further evalua­ tion and hospital admission. E) Prescribe an over-the-counter cold medicine. Discussion 1 -1

The correct answer is "D:' The neonate in this scenario has a fever and decreased feeding, which should raise immediate concern for neonatal sepsis, requiring a complete infectious workup (including blood, urine, and cerebrospinal fluid [CSF] culture), hospital admission, and empiric antibiotic treatment. This scenario is frequently referred to as "the rule out:' Fever in a neonate is defined as 3 8°C ( 1 00.4°F) or higher. In an infant younger than 28 days of age, fever is a medical emergency and

requires an infectious workup, including lumbar puncture, intravenous antibiotics, and admission to the hospital. For infants 29 to 90 days old, fever denotes urgency, but treatment may be stratified by risk. Unless your office is remarkably well­ supplied, this workup would most likely be performed in a hospital setting with subsequent admission for observation. If you chose option "E;' you will frustrate your office colleagues who practice evidence-based medicine but parents will love you. (See Figure 1 0- 1 .)

� QUICKQUIZ What is the most appropriate method of measuring body temperature in pediatric patients? A) Rectal. B) Temporal. C) Axillary. D) Tympanic. E) Forehead. Discussion

The correct answer is ''A:' The rectal temperature is the most accurate noninvasive assessment of core body temperature. The American Academy of Pediatrics (AAP) recommends that rec­ tal temperature be obtained for children 0 to 3 years old and oral temperatures for children 4 years and older. However, in toddlers it may be technically difficult to measure a rectal tem­ perature because of patient discomfort and resistance. Taking a rectal temperature on a 3 -year-old may fall under cruel and unusual punishment. Tympanic temperature measurements are accurate in children 6 months of age and older if the tympanic membrane is not obstructed by wax. Axillary temperature mea­ surements may be used for screening only and are not appro­ priate for infants. Mercury thermometers should never be used due to risk of ingestion.

203

MCG RAW-H I LL E D U CATION S P E C I A LTY BOA R D REVI EW: P E D I ATRICS

204

1


" ( "L L i ,----------------__--____ -__--__________________, .

No

Blood culture U rine culture Lumbar pu nctu re Parenteral antibiotics Chest radiograph*

Yes

Option 1

Option 2

Blood cultu re Urine cu ltu re Lumbar pu ncture (CSF culture) Ceftriaxone 50 mg/kg IV or I M i ntravenously Reevaluation with i n 24 hours

Blood culture U rine culture Reeval uation within 24 hours

*Chest radiograph if signs of pneumonia: respi ratory distress, abnormal breath sounds, tachypnea, pu lse oximetry < 95%. Follow-u p of low-risk i nfants treated as outpatients with positive culture results:

Blood culture positive (pathogen ) : Admit for sepsis evaluation and parenteral antibiotic therapy pending resu lts Urine culture positive (pathogen) : Persistent fever: Ad mit for sepsis evaluation and parenteral antibiotic therapy pending resu lts Outpatient antibiotics if afebrile and well

Low-risk criteria f o r fe b rile i nfants:

Clin ical criteria: Previously healthy, term i nfant with u ncomplicated n u rsery stay > 28 days old Nontoxic clin ical appearance No focal bacterial infection on exami nation (except otitis media) Laboratory criteria: WBC count 5-1 5,000/mm 3 , < 1 , 500 bands/mm 3 , or band/neutrophil ratio < 0.2 Negative gram stain of unspun u rine (preferred), or negative urine leukocyte esterase and n itrite , or < 5 WBCs/hpf When diarrhea present: < 5 WBCs/hpf in stool CSF: < 8 WBCs/m m 3 and negative gram stain (option 1 on ly) F I G U R E 1 0- 1 . Diag nostic eva l uation of fever in an i nfa nt younger than 90 days of age.

� QUICKQUIZ How should fever be treated? A) Acetaminophen. B) Ibuprofen. C) Treatment is not required. D) Cool bath. E) Both A and B.

bundling when fever is present. Fluids should be encouraged to replace increased insensible losses with fever. Fever should be differentiated from hyperthermia, as hyperthermia can be fatal if left untreated. In hyperthermia, the body temperature increases independently without hypothalamic regulation, exceeding the body's cooling mechanisms. Hyperthermia is due to environmental, metabolic, or pharmacologic factors.

Discussion

The correct answer is "C:' Fever is a symptom of an underly­ ing infectious or inflammatory process that causes resetting of the hypothalamic thermoregulatory set point followed by the body's response to increase the temperature to match the new set point. Fever, therefore, does not require treatment in and of itself. Parents should be reassured that unless their child appears uncomfortable no medications need to be given. High fever has not been shown to cause brain damage, and the height of fever is not the trigger for febrile seizures. They should avoid excessive

Your next patient is a 10-week-old infant boy brought by his father for fever. He has had a fever of 38.8°C ( 102°F) since this morning and has not seemed interested in breastfeed­ ing throughout the day. He has had one wet diaper in the past 12 hours. Physical exam reveals a lethargic infant with tachycar­ dia and delayed capillary refill, but no other localizing signs for his fever. He has received his 2-month vaccinations. He is other­ wise healthy and was delivered at term. His father states that the patient's mother was diagnosed with influenza yesterday.

C H A PT E R

Question 2-1

What is the most appropriate management? A) Continue to monitor at home as he likely has influenza like his mother. B) Administer ceftriaxone IM and discharge to home with a plan to follow up in clinic tomorrow for reevaluation. C) Order a chest X-ray and obtain a catheterized urine sample for urinalysis and culture. D) Transfer to the emergency department for further evalua­ tion and hospital admission. E) Administer acetaminophen. Discussion 2-1

The correct answer is "D:' The infant in this scenario is an obvi­ ously ill-appearing, febrile 70-day-old. Although there is some controversy, infants in this age group who are febrile but well­ appearing and otherwise "low risk" (see Figure 1 0- 1 ) may be managed as outpatients. However, the infant in this scenario is "high risk" and obviously quite ill, with signs of shock. He is at much higher risk of serious bacterial infection (eg, sepsis, men­ ingitis, urinary tract infection) . Appropriate disposition of this patient requires rapid transfer to the emergency department for stabilization. Call 9- 1 - 1 . This infant needs an ambulance. Workup would include a complete blood count (CBC); cultures of blood, urine, and CSF; antibiotics; fluid resuscitation; and admission. •

� I

1 1r

Helpful Tip

The most common serious bacterial i nfection i n i nfa nts you nger than 9 0 days o f age is a urinary tract i nfection.

An 18-month-old girl is brought to the emergency depart­ ment with a 2-day history of fever. She has been feeling tired and wants to be held constantly. She is drinking but is not eating. She has no other symptoms or sick contacts. She is a healthy toddler with no past medical history and is up-to­ date on her vaccines. On exam, she is ill but not toxic. She is warm to the touch. There is no rash, meningismus, or focal infection on exam. Question 3-1

The most appropriate management includes: A) Perform a lumbar puncture. B) Obtain a blood culture. C) Prescribe amoxicillin. D) Obtain a urine specimen for urinalysis and culture. E) Send her home. Discussion 3-1

The correct answer is "D:' The risk of a serious bacterial infec­ tion decreases with age and vaccination. Historically, diagnos­ tic evaluation in a febrile toddler without a source included

10



E M E R G E N C Y C A RE

205

obtaining blood, urine, and possibly CSF to avoid missing an invasive infection with Haemophilus influenzae type B or Strep­ tococcus pneumoniae. Now with vaccinations, if the infant (older than 90 days) or child is well appearing the only routine test to consider would be urinalysis and urine culture for ( 1 ) all females and uncircumcised males younger than 2 years, (2) all circumcised males younger than 6 months, and (3) all children with genitourinary abnormalities.

A nurse tells you that emergency medical services (EMS) are en route with a 1 7-year-old adolescent girl who has altered mental status and a measured temperature of more than 41 oc (> 1 06°F) . Their estimated arrival time (ETA) is 5 minutes. You have a few minutes to consider your differential diagno­ sis for this patient. Question 4-1

What will you be looking for when she arrives? A) Petechiae. B) Muscular rigidity. C) Exophthalmos. D) Mydriasis. E) All of the above. Discussion 4-1

The correct answer is "E:' Temperature above 41 oc ( 1 05.8°F) may be associated with fever secondary to bacterial or viral infection or inflammation. It may also be consistent with hyper­ thermia secondary to environmental factors (heat stroke), med­ ications (neuroleptic malignant syndrome, serotonin syndrome, malignant hyperthermia) , intoxication (particularly MDMA or anticholinergics), and thyroid storm.

� QUICKQUIZ Which types of lacerations can be repaired in the office and which should prompt consideration of surgical referral? A) Simple forehead laceration. B) Scalp laceration. C) Lip laceration. D) Finger laceration with severed ligament. E) Nonlinear leg laceration. Discussion

The correct answer is "D:' Lacerations are a common presenting complaint in pediatric patients. Most are small, simple lacera­ tions on the head or scalp as a result of falls. Wounds associated with tearing or compression of the skin may create irregular or stellate lacerations, which may be associated with tissue dam­ age and poor healing. Most lacerations can be repaired in the office or emergency department with excellent cosmetic results. Wounds that should prompt further evaluation or referral include

206

MCG RAW-H I LL E D U CATION S P E C I A LTY BOA R D REVI EW: P E D I ATRICS

TABLE 1 0- 1 I N D I CATI O N S F O R AD M I N I STRAT I O N O F TETA N U S VACC I N E

Vacci nation h i story u n known or patient h a s received < 3 doses, a l l wou nds Patient has received :::: 3 doses but none i n the last 1 0 yea rs fo r s i m ple, clean wo u n d Patient has received :::: 3 doses but none i n the last 5 years fo r conta m i n ated or co m pl ex wo u n d

puncture wounds t o the neck, chest, o r abdomen; lacerations on the neck; persistent or pulsatile bleeding; tendon or nerve inju­ ries; lacerations with underlying fracture or muscle involvement; large, complicated, or heavily contaminated lacerations; retained foreign bodies; or lacerations on the face with potential for poor cosmetic outcome. Lacerations involving the vermillion border can be sutured without surgical consultation if the pediatrician is experienced and able to achieve a good cosmetic result. Sutures generally provide the best tensile strength and cos­ metic outcome, but are more time consuming and require follow up for removal. For wounds on the scalp or long linear lacera­ tions, staples are appropriate. Skin adhesives are appropriate for small, linear, low-tension wounds without continued bleeding and not located on hands, feet, or joints. Adhesive strips (Steri­ Strips) may be used in similar situations as skin adhesive; how­ ever, these are more appropriate for additional wound support after removal of sutures or staples. Puncture wounds generally should not be closed owing to the increased risk of infection. Prophylactic antibiotics should be considered for animal or human bites or suspicion for retained foreign body. Puncture wounds through sneakers are at increased risk for infection with Pseudomonas aeruginosa. All wounds should be thoroughly irrigated with sterile water, saline, or dilute iodine solution. Tap water is a usable alternative when sterile solutions are not avail­ able. Irrigation should be at relatively high pressure ( 4 to 1 5 psi) to flush bacteria, debris, and necrotic tissue out of the wound. A minimum volume of 100 mL of irrigation fluid per centime­ ter of wound is appropriate. Immunizations should be reviewed and tetanus vaccine given if indicated. (See Table 10- 1 . ) Teta­ nus immune globulin is indicated for complex or contaminated wounds when vaccination history is unknown or if patient has received less than 3 doses of tetanus vaccine. Prophylactic anti­ biotics are not needed for minor wounds.

A 4-year-old is brought to the emergency department after being bitten in the face by a dog with several resultant lacera­ tions to the nose and left cheek.

C) The wound should be gently irrigated with sterile water to prevent further tissue damage. D) All victims of dog bites should be treated with antibiotics to prevent infection. E) None of the above. Discussion 5-1

The correct answer is "E:' Dog bites in children are most com­ mon on the face and neck, while in adolescents and adults bites are more often to the extremities. The offending dog is most often one that the victim knows. Antibiotics should be considered for patients with higher risk bites, including bites to the hand or near a prosthetic joint, associated crush injury, those requiring closure, delayed presentation, or a victim with immune compromise. Question 5-2

Appropriate antibiotic prophylaxis for this child would include all of the following EXCEPT: A) Cephalexin oral. B) Amoxicillin/clavulanate oral. C) Ampicillin/sulbactam intravenous. D) Clindamycin plus trimethoprim/sulfamethoxazole oral. E) None of the above. Discussion 5-2

The correct answer is "A:' First-line oral prophylaxis for this child is amoxicillin/clavulanate (Augmentin) orally or ampicil­ lin sulbactam (Unasyn) intravenously (IV) . For penicillin-aller­ gic patients options include clindamycin or metronidazole plus doxycycline, trimethoprim/sulfamethoxazole, or moxifloxacin. Remember that doxycycline is associated with enamel hypo­ plasia and tooth discoloration and should not be used in chil­ dren younger than 8 years of age. Fluoroquinolones should be used with caution owing to risk of tendon rupture. Cephalexin has limited activity against Pasteurella multocida and therefore should not be used for bite wound prophylaxis.

The child's parents tell you that the dog that bit their child belongs to their neighbor. They are unsure if its vaccina­ tions are up to date. They are wondering if their child should receive a rabies vaccine. Question 5-3

What is your response? A) No, if the bite was provoked. B) No, if the dog may be observed. C) Yes, all dog bites require rabies vaccination. D) Yes, if vaccination status of the dog is unknown. E) Yes, both the rabies immune globulin and vaccines should be administered. Discussion 5-3

Question 5-1

Which of the following is true regarding dog bites? A) Most bites in children are to the hands or forearm. B) It is more likely that the child was bitten by a stray dog.

The correct answer is "B:' Signs of rabies in animals include anorexia, difficulty swallowing, ataxia, seizures, and abnormal behavior or vocalizations. If the dog in this scenario does not display any of these symptoms and can be observed for 10 days,

C H A PT E R 1 0

no postexposure prophylaxis (PEP) is indicated. Prophylaxis should be considered if the dog is not observable and should be given immediately if the dog is known or suspected to be rabid. Rabies PEP includes rabies immune globulin injected at or proximal to the site of injury at time of presentation and rabies vaccine given on days 0, 3, 7, and 14. For wild animal bites, the animal in question should be assumed to be rabid and PEP should be strongly considered. If the animal in question can be captured and tested, PEP can be discontinued if results are negative. Further evaluation of nonbite exposures and deci­ sion to initiate PEP can be made in conjunction with local pub­ lic health authorities. A listing of state and local resources is available from the Centers for Disease Control and Prevention (CDC; www.cdc.gov/rabies/resources/ contacts.html) .

You are seeing a 1 7-year-old adolescent boy with a hand injury. He was brought to the emergency department by his parents after they noticed his hand was becoming increasingly red and swollen. They noticed some "cuts on his knuckles" several days ago when he returned home from a party. When you examine his hand you see several horizontal linear lacera­ tions over his third and fourth dorsal metacarpophalangeal (MCP) joints, with underlying warmth, erythema, and indu­ ration. This area is exquisitely tender, and his ability to flex or extend his fingers at the MCP joints is severely limited. There is a small amount of purulent drainage from the area. He ini­ tially states he injured his hand after "punching a wan:' but when you interview him with his parents out of the room he admits that he had instead punched someone in the face. Question 6-1

The most appropriate next step in management is: A) Oral antibiotics and reevaluation in 24 hours. B) Magnetic resonance imaging (MRI) scan of the hand to evaluate for tendon injury. C) IV antibiotics and immediate consultation with hand surgeon. D) IV antibiotics and admission to the pediatric floor for observation. E) Referral to anger management counseling. Discussion 6-1

The correct answer is "C:' The wounds described are also con­ sistent with a closed fist contacting another person's teeth, or a "fight bite:' Patients may withhold this detail of their history, so any injury to the hand with these characteristics should be treated as a fight bite regardless of stated history. Polymicro­ bial infections are common after this injury, with a spectrum of complications including cellulitis, tenosynovitis, septic j oint, and osteomyelitis. Common bacterial isolates include gram­ negative bacilli, streptococcal species, Staphylococcus aureus, and Eikenella corrodens. This patient's presentation is concern­ ing for septic arthritis or tenosynovitis, given his limited range



E M E R G E N C Y C A RE

207

of motion. Management would include immediate evaluation by a hand surgeon, and admission for IV antibiotics. X-rays may be obtained to evaluate for fracture or retained foreign body ( eg, tooth fragments) . •

Helpful Tip

.s:::"'l Management

of other h u m a n bite wou nds ca n more stra ig htforwa rd. A l l wou nds should be thoroug h ly i rrigated. Pu nctu re wou nds a re genera l ly

i1 1r b e

not closed, a n d there is some controversy reg a rd i n g suture closu re o f l a rger wou nds. Delayed pri m a ry clos u re or loose a pproximation of wou n d edges a re both proposed ma nagement options. Antibiotic prophylaxis is recom mended for puncture or other deep bite wou n ds, those with u n d erlying tendon or bone, or bites to the face.

� QUICKQUIZ Most snake bites in the United States are unprovoked, often due to startling the snake in its natural habitat. A) True. B) False. Discussion

The correct answer is "B:' The likelihood of encountering a ven­ omous snake increases as one approaches the equator, there­ fore these exposures are more common in the southern United States. Poisonous snakes found in the United States include pit vipers (rattlesnakes, cottonmouths, and copperheads) and coral snakes. The majority of deaths attributed to snake bites in the United States are in men who have been intentionally handling or playing with snakes, although innocent victims do exist. Pit viper envenomation is associated with extensive local swelling, which may progress to systemic symptoms including paralysis, rhabdomyolysis, shock, and disseminated intravascular coagu­ lation (DIC) . Immediate management includes removing any tight clothing or jewelry in the area. Suctioning of the area in an attempt to remove venom is not recommended. Antivenom should be administered if available. Coral snakes inject venom via repeated bites or "chewing"; therefore, these bites are less likely to result in significant envenomation. Local edema and coagulopathy are not seen with coral snake bites, in contrast to pit viper envenomation. Systemic effects may be delayed for hours and are primarily neurotoxic and myotoxic. Patients should be monitored closely for respiratory depression and rhabdomyoly­ sis. In general, all patients with suspected snake envenomation should be monitored in a facility with intensive care capabilities.

A 12-year-old girl presents with foot pain. She states she was getting ready to head out for a hike with her family in the Arizona desert. She put her foot in her boot and felt a

208

MCG RAW-H I LL E D U CATION S P E C I A LTY BOA R D REVI EW: P E D I ATRICS

sudden sharp pain on the side of her foot. This occurred about 30 minutes ago. She is feeling well other than some tin­ gling and pain in the area. She did not look into her boot but believes she may have been bitten by a spider. On exam she has a small puncture wound to the lateral aspect of her foot without erythema or swelling. Tapping over this area causes significant increase in her pain and paresthesias. Question 7-1

Management of this patient's symptoms should include: A) Cold compress and oral pain medication followed by 4 to 5 hours of observation in the emergency department. B) Rapid administration of Loxosceles antivenom. C) Admission to the hospital for observation. D) Irrigation of the wound and discharge to home. E) Throwing away her boots. Discussion 7-1

The correct answer is "A:' The patient in this scenario has symp­ toms most consistent with bark scorpion sting, with local reac­ tion only. Scorpions are found primarily in the southwestern United States. She should be treated symptomatically and dis­ charged to home if symptoms have not progressed 4 to 5 hours after envenomation. As her symptoms are mild, antivenom is not indicated. Patients presenting for evaluation of "spider bite" are almost always not, in fact, the victims of arachnid assault. Most of these alleged "spider bites" will be diagnosed as cellulitis or abscess. However, spider bites do occur, and the common species (brown recluse and black widow spiders) are found throughout the United States. Loxosceles, or brown recluse, spiders are named for their tendency to build webs in small secluded spaces, such as the back of a closet or dark cor­ ner of a garage. They are 8 to 15 millimeters (mm) in length and light to dark brown, with a violin- or fiddle-shaped spot on their dorsal thorax. They are most common in midwestern and southern states. Brown recluse spider bites can cause local skin irritation but are not associated with paresthesias. Effects of bites are generally local skin necrosis, although major com­ plications including DIC, kidney injury, and hemolytic anemia have been reported. Most exposures can be managed with local wound care. Surgical debridement and empiric antibiotic treat­ ment are not recommended. Latrodectus, or black widow, spi­ ders are larger ( 1 2- 1 6 mm in length) and can be identified by a red or orange "hourglass" on their ventral abdomen. They are not always black, however, with colors ranging from pale brown to gray to black. A black widow spider bite is not associated with significant local reaction or paresthesias. Black widow venom causes very little local reaction, but it does contain a neurotoxin capable of inducing persistent contraction of both smooth and skeletal muscle. Severe abdominal pain, tachycardia, diapho­ resis, and muscle spasms are common presenting symptoms. Black widow envenomation is not fatal, and treatment is sup­ portive. Antivenom is available as an adjunct to pain control. More than 40 types of scorpion are found in the United States; however, the Arizona bark scorpion (Centruroides sculpturatus) is responsible for most morbidity and mortality associated with scorpion stings. Its primary habitat is in the southwestern

United States. The effects of its venom are primarily neurotoxic, with symptoms ranging from local pain and paresthesias to skel­ etal muscular hyperactivity to multisystem organ failure. Local pain and paresthesias may be worsened by tapping over the site of the sting. These cases are managed with pain control, and cool compresses. Patients should be monitored for progression of symptoms but if stable 4 to 5 hours after evenomation may be discharged to home. More severely affected patients often pres­ ent with agitation, writhing, and roving eye movements. These patients with skeletal muscular or cranial nerve involvement can be treated with benzodiazepines and analgesics followed by admission for observation. Antivenom is available for patients with more severe symptoms. (Although many people think that urine is a magical remedy for bites or stings, including those from jellyfish, it's not!)

Suppose the patient had instead been stung by an insect in the order Hymenoptera. Question 7-2

All of the following are true regarding Hymenoptera stings EXCEPT: A) The order Hymenoptera includes bees, wasps, and ants. B) Patients who have systemic reactions to bee stings should be observed for several hours, even if symptoms have com­ pletely resolved after treatment. C) Wasps and bees leave their "stinger" in the skin, which may need to be removed. D) Systemic reactions from Hymenoptera stings are not pos­ sible without prior sensitization. E) A large local reaction may develop around the sting site. Discussion 7-2

The correct answer is "D." The order Hymenoptera encom­ passes a group of stinging insects, including bees, wasps and ants. Envenomation causes immediate pain followed by devel­ opment of an erythematous wheal at the site of entry. Self­ limited local reactions can be treated with cold compresses and oral analgesics or antihistamines as needed. Bee stings may require removal of the "stinger;' a modified ovipositor, which is left in the skin. (Kudos if you know what an oviposi­ tor is.) Stings with significant local swelling (eg, in the hand) may benefit from oral corticosteroids. Multiple stings, which usually occur after nest disturbance, can cause significant sys­ temic effects and may be fatal, even in people with no history of anaphylaxis or prior exposure. Think about that the next time you are tempted to kick or knock down a nest. Anaphylactic reactions increase in incidence with increasing age. Presenta­ tion of anaphylactic or anaphylactoid reactions may include urticaria, hypotension, bronchospasm, angioedema, vomit­ ing, or diarrhea. Treatment should be rapidly initiated, includ­ ing epinephrine, steroids, and IV fluids, as well as aggressive airway management and vasoactive medications (pressors) if needed. Patients with systemic reactions should be closely observed for at least 4 to 6 hours after resolution of symptoms because of the risk of biphasic IgE-mediated reaction.

C H A PT E R

� QUICKQUIZ Treatment of exposure to North American species of jellyfish includes all of the following EXCEPT: A) Vinegar. B) Hot water. C) Sea water. D) Urine. E) Use plastic object to remove tentacles. Discussion

The correct answer is "D:' Jellyfish stings are mediated through nematocysts, which are small structures located on the tentacles and near the mouth. Nematocysts are able to attach to the skin and when triggered release venom. Contact or pressure over nematocysts triggers release; thus, it is helpful to inactivate nema­ tocysts prior to removal of tentacles from skin. Vinegar ( 4-6% acetic acid) has been shown to help prevent nematocyst discharge when applied for 30 minutes. Tentacles can be rinsed off with sea water or scraped off with a credit card. Rinsing with fresh water or urine is not recommended as hypotonic solutions may trigger nematocysts. After removal of nematocysts, soaking in hot water has been shown to deactivate some jellyfish toxins. (Hopefully, some will remember the jellyfish episode from Friends.)

A 6-year-old boy is brought for evaluation of abdominal pain after a bicycle accident. He was wearing a helmet when he struck a curb and fell forward, striking his abdomen on the handlebars. Question 8-1

He is most likely to have injured which of the following? A) Stomach. B) Pancreas. C) Duodenum. D) Liver. E) Kidney. Discussion 8-1

The correct answer is "D:' In blunt abdominal trauma, the liver and spleen are the most commonly injured organs. Hollow organs injuries are less common but may also be life threaten­ ing. Duodenal injuries, specifically duodenal hematomas, are associated with blunt trauma from bicycle handlebars but are still less common than solid organ injuries.

You are evaluating a 1 3-year-old adolescent girl who was kicked in the abdomen while she was grooming her horse. She is complaining of pain in her upper abdomen and left

10



E M E R G E N C Y C A RE

209

shoulder. When you examine her you note bruising to her left upper quadrant and significant tenderness to palpation in this area. Her left shoulder exam is completely normal. Question 9-1

Based on her history and exam, you are suspicious of what type of injury? A) Splenic injury. B) Kidney injury. C) Pancreatic injury. D) Stomach injury. E) Ovarian injury. Discussion 9-1

The correct answer is "A:' Acute splenic injury often presents as left upper quadrant abdominal pain that may be referred to left shoulder. Splenic injury should be considered in persistent hypo­ tension or shock after blunt trauma. Negative FAST (Focused Assessment with Sonography in Trauma) does not rule out splenic rupture, as bleeding may be contained within the splenic capsule. Computerized tomography (CT) scan of the abdomen with IV contrast is the preferred diagnostic study to diagnose splenic laceration or rupture. Whenever possible splenic ruptures are managed conservatively given the risk for immunocompro­ mise after splenectomy. Any child with suspected or confirmed splenic rupture requires emergent surgical consult and should be admitted for close observation. Due to the risk of massive hem­ orrhage and shock, aggressive resuscitation with IV crystalloid fluids is indicated and blood transfusion may also be necessary. Question 9-2

Which laboratory tests are typically indicated in evaluation of blunt abdominal trauma? A) CBC. B) Lipase. C) Urinalysis. D) Kidney function. E) All of the above. Discussion 9-2

The correct answer is "E:' Indicated laboratory tests include CBC (hemoglobin and platelet count), electrolytes, liver panel (eleva­ tions of AST and ALT may be seen in liver injury) , amylase and lipase, urinalysis (hematuria may be seen in injury to kidney, bladder, or urethra), lactic acid (may be elevated in tissue isch­ emia/hypoperfusion) . Coagulation studies (PT/INR, PTT) are generally not indicated in otherwise healthy children unless there is a history of coagulopathy or in cases of massive hemorrhage.

An 8-year-old girl is brought to the emergency department from the scene of an accident. Paramedics state that she was riding her bike when she was struck from behind by a car traveling 40 miles per hour (mph) . She was not wearing a hel­ met. On arrival she is agitated and crying.

210

MCG RAW-H I LL E D U CATION S P E C I A LTY BOA R D REVI EW: P E D I ATRICS

Question 1 0-1

Your initial rapid assessment of this patient should include all of the following EXCEPT: A) Measurement of pulse, blood pressure, respirations, and pulse oximetry. B) Checking peripheral pulses and capillary refill. C) Lung auscultation. D) Removal of clothing to fully expose the patient. E) Assessing level of responsiveness. F) Obtaining a detailed history of events surrounding the injury and performing a full head-to-toe exam. Discussion 1 0-1

The correct answer is ''F:' Trauma is responsible for nearly 50% of deaths in children aged 1 to 14 years. Trauma evalu­ ation therefore involves a rapid patient assessment known as the primary survey. This includes identifying and correcting potentially life-threatening abnormalities involving airway, breathing, circulation, and mental status. This is often sim­ plified to ABCDE (Airway, Breathing, Circulation, Disability [neurologic exam] , Exposure) . Airway assessment includes identifying airway obstruction or inability to protect the air­ way, and treatment includes repositioning (with cervical spine immobilization), suctioning, and securing the airway if neces­ sary. This patient's airway appears to be intact as she is awake and crying. Breathing should be assessed using respiratory rate and effort, pulse oximetry, and lung auscultation, with treatments including supplemental oxygen, needle decom­ pression or chest tube for pneumothorax, and bag-valve mask or mechanical ventilation. Circulation assessment includes examining pulse rate, blood pressure, peripheral pulses, and capillary refill. Treatments include securing intravenous access, administration of IV fluids or blood transfusion, hemorrhage

control, or vasoactive mediations (pressors) as needed. Dis­ ability includes rating of neurologic status using the Glasgow Coma Scale (see Table 1 0-2), with treatments aimed at air­ way protection and maintaining cerebral perfusion. Exposure involves removing all clothing to facilitate full exam for injury while avoiding hypothermia. Further assessment, known as the secondary survey, including detailed head-to-toe exam, obtaining AMPLE history (Allergies, Medications, Past medi­ cal history, Last meal, Events surrounding injury) should pro­ ceed after primary survey is completed.

When you examine this patient's chest you note abrasions and bruising on the left anterior and lateral chest wall. She is tachy­ pneic and pulse oximetry reads 9 1 % on room air. Her blood pressure is normal although she is tachycardic. You notice crep­ itus in the subcutaneous tissue in her neck and her trachea is deviated to the right. Her breath sounds are absent on the left. Question 1 0-2

Your next step in management is: A) Immediate chest X-ray. B) Esophagram to evaluate for esophageal rupture. C) Immediate CT of the chest with IV contrast to evaluate for pulmonary laceration. D) Needle decompression of the left chest followed by tube thoracostomy. E) Administration of supplemental oxygen via nonrebreathing face mask. Discussion 1 0-2

The correct answer is "D:' Blunt chest trauma is commonly asso­ ciated with pulmonary contusion, hemothorax/pneumothorax, and rib fractures. Less common injuries include pulmonary or

TABLE 1 0-2 G LASCOW CO MA SCALE (GCS) F O R C H I L D R E N A N D M O D I F I E D SCALE F O R I N

Behavior Eye o pe n i n g

Verbal response

Motor res ponse

Tota l

Response

NTS

Response (preve rba l c h i l d ren)

Score

S po nta neously

S po nta neous

4

To co m ma n d/s peech

To sound

3

To pa i n

To pa i n

2

No res po n se

No res ponse

Orie nted to person, place, a n d time

Voca l izations, s m i le, i nteracts

5

Confused, but a n swers q uestions

Cri es, i rrita ble

4

I n a ppropriate wo rd s

Cries to pa i n

3

I ncom preh e n s i b l e sou n d s

Moa n s t o pa i n

2

No response

No res ponse

1

Obeys co m ma n d s

Sponta neous m ovement

6

Loca l i zes to pa i n

Withd raws fro m to uch

5

Withd raws fro m pa i n

Withd raws fro m pa i n

4

Abnormal flexion (decorti cate)

Abnormal flexio n (decorticate)

3

Abnormal exte nsion (decerebrate)

Abnormal exte n s i o n (decerebrate)

2

No response

No res ponse 15

C H A PT E R 1 0

tracheobronchial laceration, myocardial contusion, aortic injury, diaphragmatic rupture, or esophageal rupture. This patient has a tension pneumothorax, as evidenced by unilateral absent breath sounds, subcutaneous emphysema, and tracheal devia­ tion away from the affected side. If untreated, further mediasti­ nal shift can lead to decreased cardiac output, hypotension, and shock, culminating in a pulseless electoral activity (PEA) arrest. Management of a tension pneumothorax requires immediate needle decompression followed by chest tube placement. Physi­ cal findings should prompt immediate treatment-do not wait for chest X-ray to confirm the diagnosis. Esophageal rupture is a relatively uncommon complication of blunt chest trauma. Signs and symptoms include chest pain, tachycardia, dyspnea, subcu­ taneous emphysema, pneumomediastinum, or pneumothorax. Diagnosis is made by esophagram or esophagoscopy. Pulmonary laceration is usually associated with nearby rib fractures; patients may present with respiratory distress, hemoptysis, hemothorax, or pneumothorax. Treatment involves airway management (par­ ticularly with hemoptysis), volume resuscitation, tube thoracos­ tomy, and emergent consultation with a pediatric surgeon. (See Figure 1 0-2.)

Your patient will open her eyes to painful stimuli. She answers questions with inappropriate words. She is moving her arms and legs spontaneously.



E M E R G E N C Y C A RE

21 1

Question 1 0-3

What is her GCS score? A) 20. B) 2. C) 1 1 . D ) 0. E) 16. Discussion 1 0-3

The correct answer is "C:' The patient's score is 2 (eyes) + 3 (verbal) + 6 (motor) 1 1 . The minimum and maximum GCS scores are 3 and 1 5, so the only option was C (see Table 1 0-2) . =

You determine this patient's GCS to be 1 1 . The alteration in consciousness leads you to focus your exam to identify signs of serious head injury. Question 1 0-4

What is NOT a sign of skull fracture in pediatric patients? A) Scalp hematoma. B) A palpable step-off. C) Post-traumatic clear otorrhea. D) Bruising behind the ear. E) Scalp laceration. Discussion 1 0-4

The correct answer is "E:' Any palpable crepitus or depression on head exam should raise suspicion for skull fracture. Signs of basilar skull fracture include CSF rhinorrhea, CSF otorrhea or bleeding from external auditory canal, hemotympanum, Battle sign (bruising posterior to the ear), and raccoon eyes (perior­ bital ecchymosis). For suspected depressed, basilar, or open skull fractures, a head CT scan should be performed to look for intracranial injuries. (See Figure 10-3.) A simple linear skull fracture in a patient without concern for nonaccidental trauma

F I G U R E 1 0-2. A tension pneumothorax is present on the right side

with no lung markings on the right a n d s h ift of the mediasti n u m to the left. I m med iate need l e deco m p ression a n d placement of a chest tube thoracotomy is i n d icated to prevent cardiopu l m o n a ry arrest. (Reproduced with permission from McKean SC, Ross JJ, Dressier DD, Brotma n OJ, G i n s berg JS, eds. Principles and Practice of Hospital Medicine. New York, NY: McGraw- H i l l Education; 201 2, F i g . 1 07-7.)

F I G U R E 1 0-3. A left occipita l l i near s ku l l fracture is visible on head CT sca n

in bone window view. (Reproduced with permission from H a l l J B, Sch m idt GA, Kress JP, eds. Principles of Critical Care. 4th ed. New York, NY: McGraw- H i l l Education; 201 5, F i g . 1 1 8-2.)

21 2

MCG RAW-H I LL E D U CATION S P E C I A LTY BOA R D REVI EW: P E D I ATRICS

and otherwise normal exam requires no additional workup. Treatment includes reassurance and analgesia. Question 1 0-5

All of the following are signs of acute increased intracranial pressure or brain herniation EXCEPT: A) Altered mental status. B) Bradycardia. C) Fixed, nonreactive pupils. D) Diarrhea. E) Impaired external ocular abduction. Discussion 1 0-5

The correct answer is "D:' Symptoms of increased intracranial pressure and brain herniation overlap as increased intracranial pressure may lead to brain herniation. You must act immedi­ ately if concerned about herniation (ie, posturing, nonreactive pupil dilation) . (See Table 10-3.) Question 1 0-6

Which is NOT an indication for head CT scan in trauma patients? A) Frontal scalp hematoma. B) Severe headache. C) Papilledema. D) Open skull fracture. E) Known diagnosis of hemophilia. Discussion 1 0-6

The correct answer is ''A:' Prolonged loss of consciousness, wors­ ening or severe headache, penetrating injury, GCS score less than 14, focal neurologic exam, signs of increased intracranial

TABLE 1 0-3 COM PA R I S O N OF F I N D I N G S I N I N CREASED I NTRACRA N I A L P R E S S U R E V E R S U S H E R N I AT I O N

Acute I ncreased I ntracra n i a l Pressu re

Hern iation

Altered me nta l statu s/ Depressed GCS

Altered m e ntal statu s/ Depressed GCS

Fu l l fonta n e l

Obtu nded

I rrita b i l ity

C u s h i n g triad:

Severe headache



Seizure



Pa p i l ledema



Hyperte nsion B radyca rd i a I r reg u l a r breat h i n g (Cheyne-Sto kes)

Neck stiffness

Fixed, d i lated p u p i l (s)­ "blown p u p i l "

Photopho bia/phonophobia

Dece rebrate postu r i n g

Vom i t i n g

Extraocu l a r m ovement pa ra l ys i s

GCS, Gla sgow C o m a Scale.

pressure or herniation, exam concerning for depressed or basilar skull fracture, delayed or prolonged seizure, bleeding disorder, clinical deterioration during the period of observation, or con­ cern for nonaccidental trauma are all indications for imaging. In the absence of the preceding findings, consider imaging versus observation for high-speed motor vehicle crash, child struck by car, falls (> 5 feet for children younger than 2 years of age or > 3 feet for infants), persistent vomiting, nonfrontal scalp hematoma (in children younger than 2 years) , or parental concern for signif­ icant behavior change. Head injuries with low-risk mechanisms, GCS score of 15, normal behavior, and exam do not require imaging. A simple frontal scalp hematoma in a well-appearing child with a normal neurologic exam does not require imaging. Kids' heads are heavy and their balance is terrible during the early phase of walking, making them prone to face plants.

You note a large boggy hematoma on this patient's right temporal/parietal scalp. You have ordered a head CT scan to evaluate for skull fracture or intracranial injury. While the CT is being performed on your patient, you are called to see another patient who was just brought to the emergency department by EMS for acute respiratory distress. Twenty minutes later you are called back to the trauma room because your first patient has become unresponsive. As you are securing the patient's airway you receive a call from the radiology department, reporting the patient's head CT results. The radiologist describes a biconvex lens-shaped area of acute hemorrhage overlying the right lateral cerebrum with midline shift. Question 1 0-7

Which of the following statements is true about this patient's diagnosis? A) This finding is almost always associated with injury to underlying brain parenchyma. B) A majority of patients with this finding report brief loss of consciousness at the time of injury. C) This injury is the result of shearing forces causing tearing of small vessels in the pia mater. D) Operative management with craniotomy is necessary in all cases. E) None of the above. Discussion 1 0-7

The correct answer is "E:' The CT findings described in this case are consistent with an epidural hematoma (EDH ) . (See Figure 1 0-4.) EHDs are generally the result of blunt head trauma, most commonly falls. They are the result of arterial or venous injury with bleeding outside the dura. Sixty to 80% are associated with overlying skull fracture, although direct injury to brain parenchyma is generally not seen. The classic scenario of head injury with brief loss of consciousness � "lucid inter­ val" � rapid deterioration is often seen in Board questions but is much less common in clinical practice. Patients may present instead with headache and persistent nausea. When the diagno­ sis of EDH is made emergent consultation with a neurosurgeon is mandated owing to the risk of expansion and herniation. The patient in this case would certainly require craniotomy with

C H A PT E R

F I G U R E 1 0-4. A biconvex lens-shaped hemorrhage overlyi ng the parietal area, consistent with an acute epidural hematoma. Epid u ra l hematomas are located between the sku l l and the d u ra and do not cross suture li nes. (Reproduced with permission from Doherty GM, ed. Current Diagnosis & Treatment: Surgery. 1 4th ed. New York, NY: McGraw- H i l l Education; 201 5, Fig. 36-8.)

evacuation of the hemorrhage. Small epidural hematomas may not require operative intervention; however, all patients should be admitted to the hospital for close observation.

10



E M E R G E N C Y C A RE

213

F I G U R E 1 0-5. A s u b d u ra l hematoma i s present over the left frontotem poropa rieta l a rea. There is mass effect with m i l d m i d l i n e sh ift. S u b d u ra l hematomas a re l ocated between the a rachnoid a n d d u ra a n d cross suture l i nes. (Reproduced with permission fro m H a l l J B, S c h m idt GA, Kress J P, eds. Principles o f Critical Care. 4th ed. New York, NY: McGraw- H i l l Education; 201 5, Fig. 1 1 8-4.)

pressure. SDH also mandates emergent neurosurgical consul­ tation; however, small or chronic SDHs often are not managed operatively.

Imagine instead that the radiologist had reported bilateral crescent-shaped areas of hemorrhage. Question 1 0-8

Which of the following is true of this diagnosis? A) This finding is likely to be associated with injury to underly­ ing brain parenchyma. B) If found in infants this injury is pathognomic for child abuse. C) This finding is usually the result of arterial bleeding. D) Compared to epidural hemorrhages, this finding has a bet­ ter long-term prognosis for complete recovery. E) None of the above. Discussion 1 0-8

The correct answer is "A:' A crescent-shaped area of hemor­ rhage on CT scan describes a subdural hematoma (SDH) . (See Figure 10-5.) This is usually the result of tearing of bridging veins that pierce the dura after falls or direct blows to the head. Associated brain injury is common; however, skull fracture is not. Because of this association with underlying brain injury, SDH is associated with 1 0% to 20% mortality and long-term neurologic sequelae are more common than in EDH. SDH should raise suspicion for nonaccidental trauma in infants but is not pathognomic. Presenting symptoms include head­ ache, seizure, vomiting, or other signs of increased intracranial

You are seeing a 14-year-old boy who is brought from a local swimming pool with neck pain after diving into the shallow end of the pool. He did not lose consciousness; however, he has complained of persistent neck pain and is refusing to turn his head due to pain. He is also complaining of weakness and tingling in his arms. Question 1 1 -1

Which of the following statements is true regarding cervical spine injury in children? A) Cervical spine fracture is most common in children younger than 8 years old. B) Hypoventilation and apnea are suspicious for high cervical spine injury with cord compromise. C) Focal neurologic deficit is concerning for brain injury, but not for cervical spine injury. D) Most cervical spine injuries occur in females. E) None of the above are true. Discussion 1 1 -1

The correct answer is "B:' Spinal cord injuries are uncommon in children occurring most frequently in teenage males. Cer­ vical spine injuries result from blunt trauma and may involve

MCG RAW-H I LL E D U CATION S P E C I A LTY BOA R D REVI EW: P E D I ATRICS

214

the bones, ligaments, blood vessels, peripheral nerves, or spinal cord. Cervical spine fracture is rare in children younger than 8 years. This age group is more likely to injure the upper cer­ vical spine. Common mechanisms are motor vehicle crashes, diving, sports-related injuries, and violence. Maintain a high index of suspicion for cervical spine injury in patients with neck pain, torticollis, altered mental status, focal neurologic deficits, or substantial torso injury. Apnea or hypoventilation also can be signs of high cervical spinal cord injury disrupting inner­ vation to the diaphragm. Management of pediatric cervical spine injury involves immobilization of the cervical spine with a cervical collar. Radiologic evaluation includes plain X-rays (anterior posterior, cross-table lateral, and odontoid views) to evaluate for fracture or dislocation/subluxation. CT scan offers increased sensitivity and specificity but also increased risk due to radiation exposure. CT should be considered if clinical sus­ picion for cervical spine injury is high and plain films are inad­ equate or if plain films are suspicious for acute injury. MRI is more sensitive for spinal cord or ligamentous injuries. MRI is generally not part of emergent evaluation but is indicated for patients with persistent neurologic abnormalities. Question 1 1 -2

Which types of injuries discovered as part of your trauma assessment should also raise suspicion for thoracic or lum­ bar spinal injury? A) Facial fracture. B) Duodenal perforation. C) Femoral fracture. D) Bladder perforation. E) None of the above. Discussion 1 1 -2

The correct answer is "B:' Injury to the thoracic or lumbar spine is associated with small bowel or pancreatic injury, pneumo­ thorax or hemothorax, lung contusion, aortic injury, and head injury. Compression fractures are the most prevalent, most often occurring in the lower thoracic or upper lumbar spine. They are associated with lower energy mechanisms (eg, falls and sports) involving axial loading and flexion. They are usually not asso­ ciated with neurologic deficit. Burst fractures (vertebral body is smashed into fragments) are the result of axial load injury, also occurring most commonly at the thoracolumbar junction. They may be associated with neurologic deficit if significant retropulsion of fragments into the spinal canal occurs. Flexion­ distraction injuries (vertebrae are pulled apart) are generally associated with more severe mechanism. They may be osseous, ligamentous, or both. •



1 1 1r

Helpful Tip

C h i l d ren with a h i story of b l u nt tra u m a a n d neurologic fi ndings suggestive of spinal cord i nj u ry but with norma l X-rays and CT fi n d i n g s a re descri bed as having a spinal cord i nj u ry without radiogra phic a bnorma l ity (SCIWORA). A spinal cord i nj u ry may sti l l be present and the c h i l d should have an M R I .

A 3-month-old girl is brought to the emergency department by ambulance for evaluation of hot water burns. The child is sleepy but arousable. There are bright read burn marks from the umbilicus down with sparing of the buttocks and no splash marks above the umbilicus. Question 1 2-1

The most accurate method of estimating the total body sur­ face area involved in a burn patient is: A) The rule of 9s. B) Palmar surface area. C) Lund and Browder chart. D) All of the above are equally accurate. E) None of the above. Discussion 1 2-1

The correct answer is "C' The total body surface area (TBSA) involved can be estimated using the Wallace Rule of 9s (see Table 1 0-4) ; however, this is less accurate in young children. The surface area of a patient's palm is approximately 1 % TBSA and can be used to approximate small burns. The Lund and Browder chart is a more accurate method of estimation. This chart includes a diagram of the body with the measured area of the burn adjusted for age. (See Figure 1 0-6.) Only partial­ thickness and full-thickness burns are included in the TBSA. Superficial burns are not included. Burns involving greater than 10% TBSA require fluid resuscitation, due to increased insen­ sible losses and systemic inflammatory response. The Parkland formula is typically used to calculate additional fluid require­ ments. (See Table 1 0-5.) Question 1 2-2

For a 20-kilogram (kg) child with a 20% TBSA burn the vol­ ume of intravenous fluid required over the next 24 hours in addition to maintenance intravenous fluids is: A) 1 000 milliliters (mL) . B) 1 600 mL. C) 800 mL. D) 1 500 mL. E) None of the above.

TABLE 1 0-4 WA LLACE R U L E OF 9s.

Body Part Head

% TBSA 9

Anterior torso

18

Posterior torso

18

Arm

9

Leg

18

Peri n e u m TBSA, tota l body su rface a rea.

C H A PT E R

10



E M E R G E N C Y CARE

215

The Lund-Browder b urn chart B u rn esti mate diagram

.

Age and area

2

1

( ) \ ) / �

l:

I n itial evaluation*

.

Signature

..

4

3

I

5

\

7

6

8

( \ \ J v "-

(

\

Date of burn

!J

Date completed

*To be completed by the admitting resident or L I P admission

i\\ JJ

:v

D N/A, please refer to O P D COM Plan or 1 st admission burn diagram

I

\ I ��

This is a working burn estimate diagram on ly. It is not as accu rate as photog raphy

CODE: Crosshatch - 2 ° Solid - 3°

B i rth-1 yr

1-4 yrs

5-9 yrs

1 0 - 1 4 yrs

1 5 yrs

Adult 7

Head

19

17

13

11

9

Neck

2

2

2

2

2

2

Anterior tru nk

13

13

13

13

13

13

Posterior trunk

13

13

13

13

13

13

Right buttock

2.5

2.5

2.5

2.5

2.5

2.5

Left buttock

2.5

2.5

2.5

2.5

2.5

2.5

Gen italia

1

1

1

1

1

1

Right upper arm

4

4

4

4

4

4

Left upper arm

4

4

4

4

4

4

Right lower arm

3

3

3

3

3

3

Left lower arm

3

3

3

3

3

3

2.5

2.5

2.5

2.5

2.5

2.5

Left hand

2.5

2.5

2.5

2.5

2.5

2.5

Right thigh

5.5

6.5

8

8.5

9

9.5

Left thigh

5.5

6.5

8

8.5

9

9.5

R i g h t lower l e g

5

5

5.5

6

6.5

7

Left lower leg

5

5

5.5

6

6.5

7

Right foot

3.5

3.5

3.5

3.5

3.5

3.5

Left foot

3.5

3.5

3.5

3.5

3.5

3.5

'-

••

Only 2 ° and 3° burns are incl uded i n the total TBSA burn percent total

\\ \j

\J v

Area

Right hand

I

I

1

I �� \

20

30

Total

-

F I G U R E 1 0-6. The Lund-Browder c h a rt provides the best estimate of the burn extent, accounting for changes in body proportions with age. (Reproduced with

permission from Goldsmith LA, Katz 51, G i l c h rest BA, et al, eds. Fitzpatrick's Dermatology in General Medicine. 8th ed. New York, NY: McGraw- H i l l Education; 201 2, Fig. E95-4. 1 .)

216

MCG RAW-H I LL E D U CATION S P E C I A LTY BOA R D REVI EW: P E D I ATRICS

TABLE 1 0-5 PA RKLA N D FORM U LA F l u i d vo l u m e ( i n m l) = We i g ht (kg) x % TBSA x 4 Half of t h i s vo l u me is g ive n ove r the fi rst 8 hou rs, the second h a lf over the next 1 6 h o u rs Ca l c u l ated vo l u m e is g ive n in addition to m a i nte n a n ce fl u i d s



� I

Helpful Tip

I n itial assessment of b u r n i nj u ries is similar to the assessment o u t l i n ed ea rlier, which fo l l ows

1 1r tra u m a

the ABCDEs of i n itia l sta b i l ization a n d pri m a ry

su rvey. Ca re s h o u l d be ta ke n d u ri n g eva l u ation a n d res u scitation t o m i n i m ize fl u i d losses a n d preve nt

Discussion 1 2-2

The correct answer is "B:' Using the Parkland formula, Fluid volume (in mL) 20 (kg) x 20 (% TBSA) x 4 1600 mL. Half (800 mL) should be given over the first 8 hours and the second half (800 mL) over the next 16 hours. This is in addition to the child's daily main­ tenance fluid needs of 1500 mL. (See Table 10-5.) =

burns involving less than 5% of TBSA can usually be managed in the outpatient setting with pain control and local wound care. Topical silver sulfadiazine or bacitracin may be used.

=

Question 1 2-3

Which of the following statements is/are correct regarding burn assessment and management? A) Partial-thickness burns are generally not painful due to disruption of nerve fibers in the burned tissue. B) Any full-thickness burn requires immediate transfer to a burn center for management. C) Burns involving the hands or feet or burns over major j oints should be evaluated at a burn center. D) Associated inhalation injury is not an indication for hospitalization. E) All of the above are true.

hypothermia. Presence of soot a ro u n d the na res, mouth, or both, or stridor/hoarse voice s h o u l d ra ise s u s picion for poss i b l e a i rway i nvolvement ( i n h a lation i nj u ry) i n burns related to fi res, as wel l as poss i b l e c a r b o n monoxide or cya n ide toxicity. D e l a y i n seeki ng treatment or i m mersion patterns s h o u l d ra ise s u s picion for child a buse.

� QUICKQUIZ What type of stress is associated with a greenstick fracture? A) Torsion. B) Compression. C) Longitudinal. D) Flexion. E) None of the above.

Discussion 1 2-3

Discussion

The correct answer is "C:' Burns involving the hands or feet or burns over maj or joints should be evaluated at a burn center. Superficial or first -degree burns involve the epidermis only. The affected skin may be red or painful but can be expected to heal fully without scarring. A sunburn is an example of a superficial burn. Superficial burns are not used in calculations of TBSA. Partial-thickness (or second-degree) burns involve the dermis. They may involve blistering but the underlying skin remains pink and moist and is still viable. Partial-thickness burns are further classified as superficial or deep. Full-thickness burns (or third degree) involve destruction of the dermis and skin is not able to regenerate. Full thickness burns result in scarring and require skin grafting. Only in full-thickness burns may pain be absent if the nerves are affected. In the period immediately fol­ lowing injury it can be difficult to distinguish between partial­ thickness and full-thickness burns, and the extent of injury may evolve over time. Management of burns depends on severity, extent of TBSA affected, and location. Patients with superficial or partial-thickness burns involving less than 5% TBSA or full­ thickness burns involving less than 2% TBSA may be managed as outpatients. Those with partial-thickness burns involving 5% to 10% TBSA or full-thickness burns involving 2% to 5% TBSA or suspected inhalational injury should be admitted to the hos­ pital for observation. Any burn encompassing greater than 10 % TBSA o r any burns t o the head, genitalia, hands, o r feet o r burns involving joints should be referred to a burn center. Superficial

The correct answer is "C:' A greenstick fracture results when longitudinal force causes a bone to bend with disruption of one side of the cortex, generally in the diaphysis. There may be an associated buckle fracture on the opposite side of the bone. Greenstick fractures are often seen in the forearm. They are unstable fractures and require immobilization to promote heal­ ing. Reduction may require completion of the fracture to cor­ rect angulation. (See Figure 1 0-7.)

A 2-year-old boy presents with arm pain. His grandmother had noticed that he was not using his left arm for the last sev­ eral hours. She states that they were crossing the street when the boy slipped. She was holding his left hand and pulled him up by his arm. On exam the patient is holding his left arm with the shoulder adducted and internally rotated, elbow flexed, and forearm pronated. He is holding his left hand with his right hand to keep it against his abdomen. There is no swelling or ecchymosis of the elbow, forearm, or wrist. You do not elicit any bony tenderness with palpation over the dis­ tal humerus, wrist, or hand. He cries and resists when you attempt to supinate his hand. His grandmother is concerned that she may have broken his arm and wants to know if he needs X-rays.

C H A PT E R

10



E M E R G E N C Y C A RE

217

Discussion 1 3-1

The correct answer is "D:' The history and exam described for this patient are typical for "nursemaid's elbow;' or subluxation of the radial head under the annular ligament. It is common in children up to 8 years old, with peak incidence around 2 years of age. With a consistent history and exam, as in this case, rou­ tine X-rays are not indicated. Always palpate the distal radius and obtain X-rays if you are worried about a fracture of the distal radius. Reduction should be attempted by supination of the forearm followed by flexion of the elbow and shoulder, or by hyperpronation of the forearm. You may feel a pop. After several minutes, and with distraction, the child should begin to use the arm normally again. Owing to apprehension from prior pain, the child may need prompting to move the affected arm. This can be accomplished by immobilizing the unaffected arm while offering an object of interest to the child. A

8

c



Helpful Tip

:5.� C h i l d ren

commonly susta i n fractu res rather than

i1 1r dislocations or spra i n s as their liga ments a re stronger than their bones.

D

A 1 7 -year-old adolescent boy presents with shoulder pain. He was tackled while playing rugby and fell, landing on his right shoulder. He had immediate pain over the superior aspect of his shoulder. On exam he has tenderness over the distal clav­ icle and acromioclavicular joint without visible deformity or crepitus. He is able to move his shoulder but complains of pain with abduction.

E

Question 1 4-1

F

G

F I G U R E 1 0-7. Fractu re patte rns in c h i l d ren: (A) Tra nsverse, (B) obliq ue, (C) spiral, (D) com m i n uted or fragmented fractu re, (E) segmental, (F) torus­ buckling/com p ression of the cortex on one side only, and (G) g reenstick­ cortex fractu res under tension on one side while the other side re m a i n s i ntact. (Reproduced w i t h permission from Ti nti na I I i J E, Stapczynski JS, Ma OJ,

et al, eds. Tin tina/lis Emergency Medicine: A Comprehensive Study Guide. 8th ed. New York, NY: McGraw- H i l l Education; 201 6, Fig. 267-2.)

Question 1 3-1

What is your response? A) X-rays are not needed as clinically he has a fracture of the distal radius. B) X-rays are needed because he won't supinate his forearm. C) X-rays are not needed because a fracture won't be evident this early. D) X-rays are not needed as he has a subluxed radial head. E) X-rays should always be obtained when a child aged 2 years or younger injures an extremity.

You suspect he sustained an injury to: A) Clavicle. B) Acromioclavicular joint. C) Scapula. D) Both A and B. E) All of the above. Discussion 1 4-1

The correct answer is "D:' The clavicle is the most commonly fractured bone in the shoulder region, with 90% of clavicular fractures occurring in the midshaft. Medial or lateral clavicu­ lar fractures are less common. The mechanism described in this case would be typical for both clavicular fracture and acromio­ clavicular (AC) separation. Given the location of tenderness on exam there is concern for lateral clavicular fracture or AC sepa­ ration. AC separation refers to strain or disruption of the AC ligament, which in severe cases is accompanied by disruption of the coracoclavicular ligament. Physical exam findings include tenderness or deformity over the AC j oint. Pain may be exac­ erbated by adducting the shoulder across the body to compress the AC joint. Plain films (X-rays) may show a widened AC joint

218

MCG RAW-H I LL E D U CATION S P E C I A LTY BOA R D REVI EW: P E D I ATRICS

space or elevation of the distal clavicle. Stress views obtained with the patient holding 5- to 1 0-pound weights in the affected arm may help to further delineate the degree of separation but are not part of routine evaluation. AC injuries are graded from I to VI according to degree of ligamentous disruption, with types I to III managed with pain control and brief immobiliza­ tion in a sling for comfort. Types IV to VI may require surgical correction; however, these injuries are associated high-energy mechanisms resulting in complete disruption of both ligaments with gross deformity of the shoulder. Clavicular fractures are managed similarly, with surgical referral indicated for open fractures, or displacement of fracture causing neurovascular compromise or tenting of the skin. Question 1 4-2

Which orthopedic injuries should raise suspicion for injury to vasculature or nerves? A) Supracondylar humeral fracture. B) Posteriorly displaced clavicular fracture. C) Pelvic fracture. D) Anterior shoulder dislocation. E) All of the above. Discussion 1 4-2

The correct answer is "E:' Medial clavicular fractures with dis­ placement posterior to the sternum may have laceration of subclavian vessels or injury to the recurrent laryngeal nerve. Anterior shoulder dislocations may result in injury to the axil­ lary nerve. Fractures of the supracondylar humerus, if dis­ placed, may also injure the median nerve and brachial artery (with medial displacement of the proximal humeral segment) and the radial nerve (with lateral displacement of the proxi­ mal fragment) . The ulnar nerve is less commonly involved but may be affected with posterior displacement of the proximal humeral fragment. Supracondylar fractures as well as fractures of the tibia or fibula, or both, are at risk for compartment syn­ drome. Elevation of the muscle compartment pressure impairs blood flow, causing nerve and muscle injury. Remember the 6 Ps: pain (earliest), paresthesia, pallor, poikilothermia, paraly­ sis, and pulselessness (late). Look for horrible pain despite anal­ gesics and pain with passive muscle stretch. Dislocation of the tibia and femur at the knee is associated with injury to the popli­ teal artery. Careful assessment of pulses, measurement of ankle­ brachial index, and duplex ultrasound are mandatory. If all are normal the patient should be admitted for serial vascular exams. If any of the preceding parameters are abnormal emergent con­ sultation with a vascular surgeon, as well as an arteriogram and

CT angiogram, should be obtained. In children, pelvic fractures may cause injury to the pelvic vessels, but this is significantly less common than in adult patients. B I B L I O G RA P H Y

Arora R, Mahajan P. Evaluation of child with fever without source: Review of literature and update. Pediatr Clin North Am. 20 1 3 ;60: 1 049- 1 062. Avarello JT, Cantor RM. Pediatric major trauma: An approach to evaluation and management. Emerg Med Clin North Am. 2007;25:803-836. Avner, JR. Acute fever. Pediatr Rev. 2009;30:5- 1 3 . Balhara KS, Stolbach A. Marine envenomations. Emerg Med Clin North Am. 2014;32:223-243. Baraff LJ. Management of fever without source in infants and children. Ann Emerg Med. 2000;36:602-6 14. Daniels AH, Sobel AD, Eberson CP. Pediatric thoracolumbar spine trauma. J Am Acad Orthop Surg. 20 1 3 ;2 1 :707- 7 1 6 . DeBoard RH, Rondeau DF, Kang C S , Sabbaj A, McManus JG. Principles of basic wound evaluation and management in the emergency department. Emerg Med Clin North Am. 2007;25:23-39. Friday B, Depenbrock P. Land envenomations. Curr Sports Med Rep. 2014; 1 3 : 1 20-125. Jamshidi R, Sato TT. Initial assessment and management of thermal burn injuries in children. Pediatr Rev. 20 1 3;34:395-404. Kennedy SA, Stoll LE, Lauder AS. Human and other mam­ malian bite injuries of the hand: Evaluation and manage­ ment. J Am Acad Orthop Surg. 201 5;23:47-57. Leonard JC: Cervical spine injury. Pediatr Clin N Am. 2 0 1 3;60: 1 1 23- 1 1 37. Leonard JC, Kupperman N, Olsen C, et al. Factors associated with cervical spine injury in children after blunt trauma. Ann Emerg Med. 20 1 0;58(2) : 145- 1 55. Schunk JE, Schutzman A. Pediatric head injury. Pediatr Rev. 2 0 1 2;33:398-4 1 1 . Skolnik AB, Ewald MB. Pediatric scorpion envenomation. Pediatr Emer Care. 20 1 3;29:98- 1 06. Spiro DM, Zonfrillo MR, Meckler GD. Wounds. Pediatr Rev. 2 0 1 0;3 1 :326-334. Steere M, Sharieff GQ, Stenklyft PH. Fever in children less than 36 months of age-questions and strategies for management in the emergency department. J Emerg Med. 2003;25: 149- 1 57. Wing R, Dor MR, McQuilkin PA. Fever in the pediatric patient Emerg Med Clin North Am. 2 0 1 3;3 1 : 1 073- 1 096.

11

Endocri ne Disorders Va nessa Cu rtis

An infant is born at 39 weeks' gestation to a healthy Gravida 1 now Para 1 Caucasian woman. Pregnancy and delivery were uncomplicated. On newborn exam, the neonate is noted to have ambiguous genitalia with a 1 . 5 em phallic structure, no palpable testicles, and pigmented labial scrotal folds. Question 1 -1

Which of the following is the most appropriate first step? A) Confirm that electrolytes are normal and then discharge the infant with outpatient follow-up by endocrinology. B) Obtain karyotyping and then discharge the infant with out­ patient follow-up by endocrinology. C) Order a pelvic ultrasound; obtain 1 7-hydroxyprogesterone, luteinizing hormone, and testosterone levels; obtain karyo­ typing; and arrange for an inpatient endocrine consult. D) Reassure the family that they can let the child choose its sex when older and advise a gender-neutral name such as Pat. E) Consult the urology service and arrange for cosmetic genital surgery as soon as possible. Discussion 1 -1

The correct answer is C: In early fetal development, both XX and XY fetuses are undifferentiated in terms of reproductive structures. In normal sexual differentiation, the bipotential gonad differentiates beginning around 7 weeks' gestational age into a testicle or ovary based on multiple genes. If the gonads become testicles, they secrete miillerian-inhibiting substance, causing the miillerian ducts to regress, and they secrete tes­ tosterone, causing the wolffian ducts to persist (which develop into the epididymis, vas deferens, and seminal vesicles). If the gonads become ovaries, the lack of miillerian-inhibiting sub­ stance allows the miillerian structures to persist (which become the oviducts, uterus, cervix, and upper vagina) , and the lack of testosterone results in wolffian duct regression. The external genitalia respond to testosterone, which is converted peripher­ ally into the more potent dihydrotestosterone, with posterior fusion of the genital folds and growth of the genital tubercle into "

a phallic structure. (See Figure 1 1 - 1 .) In this infant, physical exam alone cannot tell you whether it is an undervirilized male or an overvirilized female; however, the most immediately dan­ gerous diagnosis in the differential is that the infant is a viril­ ized female as a result of congenital adrenal hyperplasia (CAH) . In female infants with classic 2 1 -hydroxlyase deficiency CAH, excess adrenal androgen during gestation can cause ambigu­ ous genitalia with clitoral enlargement, a urogenital sinus, and fusion of the labial folds. The internal reproductive organs are normal. In male infants with 2 1 -hydroxlyase deficiency, geni­ talia are typically normal, though these infants are at risk for salt-wasting crisis. Neonates with CAH are at risk for adrenal crisis because of their inability to make cortisol and aldosterone. (See Figure 1 1 -2.) They may be identified on newborn screen or based on ambiguous genitalia. An infant suspected of CAH should be monitored closely until the diagnosis is evident.

The infant is found to have a markedly elevated ! ?-hydroxy­ progesterone ( 1 7-0H-P), 46,XX karyotype, miillerian struc­ tures on pelvic ultrasound, and normal luteinizing hormone (LH), testosterone, and estrogen levels, confirming a diagno­ sis of CAH. The team discusses results with the family and advises that the child was meant to be a female.

'

Question 1 -2

The infant will likely require all of the following medical therapies EXCEPT: A) Oral hydrocortisone. B) Oral fludrocortisone. C) Estrogen. D) Injectable hydrocortisone for illness or stress. E) Salt supplementation. Discussion 1 -2

The correct answer is "C:' Infants with classic salt-wasting 2 1 -hydroxylase CAH require glucocorticoid supplementation with hydrocortisone, with a goal of replacing the cortisol defi­ ciency and also suppressing the overactive hypothalamic-pituitary axis (HPA) and preventing further androgen excess. Infants also 219

MCG RAW-H I LL E D U CATION S P E C I A LTY BOA R D REVI EW: P E D I ATRICS

220

Epididymis

/

� M ul lerian duct - Fal lopian tube

deferens sinus

vesicle UI.V--- Prostate Female

Male

A

.AIIIEi�-i / �- Gen ital tubercle �

- Genital swelling U rethral fold and groove

--,#--

w----- Shaft of penis �

- Scrotum

--

� Penoscrotal raphe

B

Female

Male

F I G U R E 1 1 - 1 . Sexual Development. Development of the male and fe m a l e i nternal a n d external u rogen ita l tracts. (Reprodu ced with permission from Bra u nwald

E et a l : Harrison's Principles of Internal Medicine, 1 5th ed. New York: McGraw- H i l l ; 200 1 .)

require mineralocorticoid supplementation with fludrocortisone to maintain appropriate serum sodium and potassium and blood pressure. Finally, owing to the low salt content in an infant diet, newborns often require a supplemental salt solution until they are eating more table foods. For any patient who is cortisol-depen­ dent, additional therapy is needed when ill. Typically this includes tripling the hydrocortisone dose for high fevers or other signifi­ cant illness. Patients are also taught to administer intramuscular (IM) hydrocortisone if illness prevents oral intake. •

Helpful Tip

:5.� If a c h i l d with CAH (or any c h i l d who req u i res cortisol

i1 1r replacement) presents with severe i l l ness a n d i n a b i l ity to ta ke things by mouth, do not hesitate to a d m i n i ster pa rentera l hyd rocortisone at a dose of 1 00 mg/m 2 • This works out to: 25 mg hydrocortisone IV/IM for a small child or i nfa nt 50 mg hydrocortisone IV/IM for a med i u m-sized child 1 00 mg hyd rocortisone IV/IM for a l a rge child. When i n dou bt, give more.

The family is planning on having more children in the future. Question 1 -3

Which of the following is an accurate statement to help the family as they consider future pregnancies: A) There is a 1 % chance of having another child affected with CAH. B) Mothers can be treated prenatally with dexamethasone to reduce the virilization of female fetuses with CAH. C) Fathers can be treated preconception with prednisone to reduce the virilization of female fetuses with CAH. D) There is a 12.5% chance of having another child affected with CAH. E) With the rising cost of college these days, discourage another pregnancy. Discussion 1 -3

The correct answer is "B." 2 1 -Hydroxylase deficiency is an auto­ somal recessive condition (like many enzyme deficiencies­ hint, hint), and thus the chance of having another affected

C H APTER 1 1



E N DOC R I N E D I SO R D E R S

221

1 7,20-lyase

1 7 -Hyd roxylase

1 7 -Hyd roxylase

1 7 , 20-lyase

Mineralocorticoids

Glucocorticoids

Androgens (estrogens)

Zona g lomeru losa

Zona fasciculata

Zona reticularis

F I G U R E 1 1 -2. Steroid Pathway. Synthesis of endogenous steroids: m i neralocorticoids, g l u cocorticoids, and sex steroids. DH EA, Dehydroepiandrosterone. (Reproduced with permission from Hay WW, Levin MJ, Deterd i n g RR, Abzug MJ, eds. Current Diagnosis and Treatment Pediatrics. 22th ed. New York, NY: McG raw­ H i l l Education; 20 1 4, Fig. 34-8.)

child would be 1 in 4 ( 2 5 % ). Mothers can be treated prenatally with dexamethasone, which suppresses fetal pituitary adreno­ corticotropic hormone (ACTH ) , causing less overproduction of adrenal androgen, and theoretically reduces virilization in female fetuses. The treatment must be done very early in the pregnancy (prior to knowing if the fetus is affected by CAH or the fetal sex) and can have adverse effects on both the fetus and mother, and thus is not considered standard of care.

A 1 6-year-old adolescent girl presents with primary amenor­

rhea. She has had excellent growth and is several inches taller than her older sisters. She has Tanner 3 breasts and scant pubic hair. Her clitoris is prominent. Workup reveals a 46,XY karyotype, pubertal LH levels, and markedly elevated testos­ terone level for a female.

Discussion 2-1

The correct answer is "C:' Disorders of androgen action are a common form of 46,XY disorders of sexual development and occur when the internal genitalia and gonads and testosterone levels are consistent with 46,XY (male) but there is insensitiv­ ity at the level of the androgen receptor. These patients have a variable phenotype ranging from a phenotypic woman with no pubarche (complete androgen insensitivity syndrome) to an undervirilized male to a phenotypic man to isolated infertility (partial androgen insensitivity syndrome) . •



Helpful Tip

(But rea l ly a fu n fact.) Com plete a n d rogen i n sensitivity

1 1 1r syn d rome is overrepresented i n el ite fem ale ath letes and fa shion models. Th i n k a bout it.

Question 2-1

This presentation is consistent with which of the following syndromes? A) Klinefelter syndrome. B) Turner syndrome. C) Androgen insensitivity syndrome. D) Trisomy 2 1 . E) Mayer-Rokitansky-Kuster-Hauser syndrome. F) Curtis-Peterson syndrome.

A 1 0-year-old boy presents for his well-child check. His mother

raises concern that he has started youth soccer and is doing well but seems so much shorter than his peers. He has no chronic medical conditions and is not taking any medications. Review of his growth charts shows that he was born at term and was average for gestational age (AGA). He has been at the fifth per­ centile since his first birthday for both height and weight. His growth velocity has been 6 cm/y for the past year.

222

MCG RAW-H I LL E D U CATION S P E C I A LTY BOA R D REVI EW: P E D I ATRICS

to 20 years: Boys Statu re-for-age and Weig ht-for-age percentiles

2

5' 0 "

Mother's Stature Date 3/4/05

Weight 7 1b 9 oz

Age 2 weeks

12

5' 5"

Father's Stature

BMI*

Stature

NAM E

13

14

re ____:_ A_:_:_: If'-'-=-=d:.:__

_

15

16

AG E (YEARS)

/

/

-1-l-j�/,E ----9� r-./

-- ¢ '-'j- �-�

2

3

4

5

6

17

18

19

_ _ _ _ _

20

cm=- i n 76-

1 90

1------'------'-----'-------'-----t---t---'' *To Calculate B M I : Weight (kg) + Stature (em) + Stature (em) x 1 0,000 6/'l:: or Weight (lb) + Stature (in) + Stature (in) x 703 h -/

Published M ay 30, 2000 (modified 1 1 /2 1 /00) .

_ _ _ _ _ __

RECORD #

7

8

9

10

11

12

Reproduced with permission lrom the National Center for Health Statistics in collaboration with the National Center lor Chronic Disease Prevention and Health Promotion (2000). http://www.cdc.gov/growthcharts.

13

_j-L

14

15

16

-

95 � 74 90 - 1 85 7275 =-__----,_ - 1 80 70 50 1 75 168f----4 - 25 �� 1 70 ::!. 1 0,r_ �f---: __,1== -•6 ' 6'' =-F �

17

18

19

20

r;!m

SAF E R • H E A LT H I E R • P E O P L E '�

s T

A T U

R

C H APTER 1 1



E N DOC R I N E D I SO R D E R S

223

bone age and normal growth hormone secretion. To calculate a midparental height, you must first consider if the child is male or female. If the child is male, add 5 inches ( 1 3 em) to the moth­ er's height and then average with the father's height. If the child is female, subtract 5 inches ( 1 3 em) from the father's height and average with the mother's height.

Question 3-1

Which of the following is most likely to reveal his diagnosis? A) A growth hormone stimulation test. B) Karyotype. C) Parents' pubertal timing. D) Mother's and father's heights. E) Mother's height and the milkman's height. Discussion 3-1

The correct answer is "D:' In this case, if the parents are short, the child needs no further workup since he has a normal growth rate (5 cm/y) and is otherwise healthy. Parental heights are important pieces of evidence when considering a child with short stature. Short parents have short children. (See following growth curve.) Children with familial short stature are born AGA and should track along an appropriate growth curve at a normal growth rate. They typically have a normal

A 15-year-old adolescent boy presents for his well-child check. His mother raises concern that he seems to be growing poorly, and there is a widening discrepancy between his height and that of his peers. On review of his growth charts you note that, indeed, he was previously at the 15th percentile but since age 13 years his height percentile has gradually declined and is currently at the 3rd percentile. See following growth curve.

2 to 20 years: Boys

NAME

Stature-for-age and Weig ht-for-age percentiles

Date

Father's Stature Wei ht

Age

1 /2/20 1 3

2 weeks

13



BMI*

Stature

8 1l:> 1 0 oz

RECORD #

AGE (YEARS) =±

12 Mother's Stature

14

-.:.....f-:-- ....:....1

15

in

em f--= 3

4

62

s T A T

u R E

-1 55

60

1 50

58

5

6

---j=:::+� :::+ ;T

-1 45

=

- -1 40 56

54

+-

1 35

- _1 30 50

1 25

48

38 36

� 7o/ L.�v EE � .2':. -1 00 _1 05

80 E I

G H T



=!= $

-80

+

-35

::±:: -30 60 I--ER= =+-70

'

+

+-

_/

/ 1//'

.-0



$/

w7 17'1 ==7'

L

A= L -:£

-i=� LA IL.

v Vr-

1-+

�� �v v 7 X. / /

-25

�� 77v-::�

2

3

4

5

6

Published May 30, 2000 (modified 1 1 /21 /00).

7

8

9

10

11

12

13

-1 551 50

A

68

u R

60

...-:.......

90

v

75

90 - 200 85 _

so· 75

50�1= 70 _

- 1 90 1 80 -1 70 -1 - 60 -1 50 -

2 5 "I�-1 40

60 1 30

10

0"1-55 1 20 50 1 1 0 45 - 1 00 40

---+-----+-----+-----+--r- = ==I= ==

20

17

18

E

1 00 220

'

__n

16

T

95 1=95 _ 2 1 0

::±:: -+ -+ + +----1==+--=+-!-� =+=

Reproduced with permission from the National Center for Health Statistics in collaboration with the National Center for Chronic Disease Prevention and Health Promotion (2000). http://www.cdc.gov/growthcharts.

70

66

1 65

s T

_,_ -1 05- 230

/ 7v

15

72

_1 75

35

14

-

1 80

E:: � 1 70

v-

/

r-+-

v

v ./

� ·� � ===!7' f==l=; 4= =Y

74

1 85

75

T

/

I=F �

76

1 90

95 90

c= 50

---j- --1-j- -r--

y

1=1= �

in

em

64 """� = == _1 60 --+--i--+- = -= 62

1---t-I- -t

� �� � �P"' =20 40 _ --r --r -r T �� z �F'?" � 1 530 -= ""'-=I= = = � � -- 1 0 �GE (YEARS)_ lb'- kg so=

E =::::r

20

10 5

=

't-.

//r- --+

/ 17':

$ u: ry � U£ ==E==E� ==E� $ w

30

/ /

/ P,_ �

'k,

- 95

34 85 32

/v

V;:J q -"' IL_,

110

42

/ 'h

/7'/

�i/i

.L_-'--'

1,; �·7 ill� ==P= :::+=

7,

T

_ _ _ __

19

_...

+==

1//I� 7 � / /

18

-

r=

-+--::



17

i--+- %-'*1,11'7�4----j-----j- -j-

I/,, ./..

-1 1 5

44

=

IF/. Ly-�

1 20

46

40

+-

'

1 0,000

x

1 0-1 1

---j-

;-

52

w

9 8 ---j---

7

= I= -= == -= -=

- _1 60_

.

16

o/

"

*To Calculate BMI: Weight (kg) : Stature (em) : Stature (em) or Weight (I b) : Stature {in) : Stature (in) x 703

ch-'------_,E"'"n'-=o""' ---

_

19

20

=-go =-so

o-7o 30 o-- 60 25

=-so

-=-40 1 5-30 10

kg

=-ri)

m

w E I G H

T

224

MCG RAW-H I LL E D U CATION S P E C I A LTY BOA R D REVI EW: P E D I ATRICS

Discussion 4-1

Weight has been appropriate for height. He will be a fresh­ man next month and would rather go out for football than cross-country, but his parents are worried that he may be injured because of his small size.

Question 4-1

Which of the following would be a reassuring finding on history and physical exam with regard to his prognosis? A) Mother and father are both tall. B) Mother had menarche at age 1 1 years. C) Father was shaving by the time he started high school. D) He has Tanner 3 pubic hair and testicles are 10 mL. E) He has Tanner 1 pubic hair and testicles are 4 mL.

The correct answer is "E:' Constitutional delay of growth and puberty ( CDGP) is a common cause of short stature. Known col­ lectively as "late bloomers;' children with CDGP are of normal size at birth; however, a downward shift in growth rate during the first 2 years of life results in a childhood height that is lower than expected for genetic potential. The child then remains with a normal growth rate during childhood but has late entry into puberty. During the time when the child is growing at a prepubertal growth rate and friends are starting their pubertal growth spurt, the child experiences an increased height discrep­ ancy between his or her height and that of peers and downward crossing of percentiles. The child eventually does enter puberty, exhibits catch-up growth, and achieves appropriate height for their family. (See Figure 1 1 -3 . )

Stature-for-age percentile (boys,

200

2 t o 2 0 years)

1 90

78

l---

95 90 _

-

�f.--I //v //1/ vv �1 '// / //, /4�I�v v � /)"'/ �v4Ll ij1�"' ,/;v/ q/fi /'/ /v/ A�/ w v

1 80

/

- 75

v

50

!--'"

1 70

25

-

1 60

7�

1 50

E'

-2� :::J



(j)

1 40

110 1 00 90 80 70

66 63

'2 54 = � :::J

o; 51 (jj

48

��� /

45

� �Vj 0� fA�� �� �· -

69

57

h��///

1 20

72

60

#��B

1 30

10 5

75

42 39 36 33

r-

30 2

4

6

8

10

12

Age (ye a rs)

14

16

18

20

F I G U R E 1 1 -3. Th is boy has constitutional delay of g rowth and p u berty. (Reproduced with permission from Centers for Disease Control and Prevention (CDC).)

C H APTER 1 1

In evaluating the adolescent in this case, CDGP would be suspected if there is a family history oflate bloomers (eg, mother had late menarche or father was still growing after high school) and the adolescent is prepubertal on physical exam.

Based on history and physical exam, a diagnosis of CDGP is suspected. The family is still concerned and hoping for some further testing to confirm this suspicion.



E N DOC R I N E D I SO R D E R S

225

Discussion 4-1

The correct answer is "E' If the history is supportive, watchful waiting may be all that is necessary. Though nonspecific, a delayed bone age would be reassuring; a delayed bone age predicts that growth is expected to continue longer than normal allowing catch-up growth. If the history and physical exam are not sup­ portive of CDGP, you may want to consider screening for thyroid disease (TSH and free T4), growth hormone deficiency (IGF- 1 ) and for other causes o f delayed puberty (LH, FSH, testosterone).

Question 4-2

Appropriate workup may include which of the following? A) Thyroid-stimulating hormone (TSH) and free thyroxine (TJ B) Insulin-like growth factor- 1 (IGF- 1 ) . C) Luteinizing hormone (LH), follicle-stimulating hormone (FSH), and testosterone. D) Bone age X-ray. E) Watchful waiting. F) All of the above. 2 to 20 years: G i rls

A 12-year-old adolescent girl presents for her well-child check. Her family does not have any concerns. She has previously been healthy and is not taking any medications. On review of her growth charts, she is currently at the 5th percentile; how­ ever, she was previously growing along the 50th percentile until 2 to 3 years ago. See following growth curve. Fa_n---' ny '---- -----___ RECORD # _ 1 2 1 3 1 4 1 5 1 6 1 7 1 8 1 9 20

Stature-for-age and Weig ht-for-age percentiles

NAME

_ _ __

76 t==Da=t•==t==A:o•==t==�:•io=h�t=t==st=at=ure�t==BM=I·==ff==fA=GtE=��EAI4RS�)Ef�ft�=t�{ 1 9095 90 75 50 25 10 5

74 1 85 72 1 80-70 1 75 68 1 7066 1 6564 1 60 62 1 55 1 50 60

s T A T u R E

1-

s T A T u R E

1 40 1 05' '230 54 1 35 1 1 00 220 52 1 30 � '210 50 200 - 1 25 90 48 1 20 85 1 90 - -1 $ 95 46 1 1 5 80 1 80 '-'� 1 70 44 1 1 0 p?-'� � """' 90 f--- 75 1 60 42 I" -f-.--� g:;jsJ-'r 70 1 50 - 1 05 40 1 00+-��� 7� � �b � ��r-+-,_-r-+_,--r -�-+�--r-+ � =+ � r 65 1 40 /�.� 38 =95 J i. 60 50 �r-- 1 30 36 55 1 20 -- 90 25 r-- :;, 34 -85 �=· 1 0 10 32 - so 5·� 1 00 30 40 -90 80 80 -35 35 70 70 -30 ---r- ---r­ 30 60 60 -25 25 50 50 20 ::..40 40 -20 1 � ::-30 30 =15 = ======= 10 -1 0 kg =-rb 11)= kg AGE �EARS) 2 3 4 5 6 7 8 9 1 0 1 1 1 2 1 3 1 4 1 5 1 6 1 7 1 8 1 9 20 -=



w E I G H T

Published May 30, 2000 (modified 1 1 /21/00).

Reproduced with permission from the National Center for Health Statistics in collaboration with the National Center for Chronic Disease Prevention and Health Promotion {2000). http://www.cdc.gov/growthcharts.

iii!

w E I G H T

MCG RAW-H I LL E D U CATION S P E C I A LTY BOA R D REVI EW: P E D I ATRICS

226

Discussion 6-1

Her weight has continued to track along the 50th percentile. Mother is 5 feet, 3 inches tall and father is 5 feet, 1 1 inches. Question 5-1

Appropriate workup may include which of the following? A) TSH and free T4 • B) IGF- 1 . C ) Bone age X-ray. D) Growth hormone level. E) Options A, B, and C. F) No workup is necessary since she is not below the 5th percentile. Discussion 5-1

The correct answer is "E:' This child is not pathologically short (height is at the 5th percentile) ; however, her slow growth veloc­ ity and downward crossing of height percentiles is concerning and must be evaluated. Furthermore, her family is not short (midparental target height is 5 feet, 4.5 inches, which is approxi­ mately the 50th percentile) , which is another red flag for her. In a child with growth deceleration or growth failure, endocrine causes must be considered such as hypothyroidism, growth hor­ mone (GH) deficiency, and cortisol excess. Appropriate screen­ ing lab tests for hypothyroidism include TSH and free T4 • IGF- 1 with or without IGF-binding protein 3 (BP3) are appropriate screening tests to assess for GH deficiency. A random GH level would not be helpful. Finally, a bone age is nonspecific but can provide further information about height potential. •

Helpful Tip

:5.� If I G F- 1

a n d IGF-BP3 a re low, this ra ises concern for

r1 1 r g rowth hormone deficiency (G H D) but is not diag nostic. To diag nose G H D, c h i l d ren m u st u n dergo provocative g rowth hormone testing, which is typica lly done by a ped iatric endocri nologist.

The correct answer is "E:' Tall stature can simply be related to genetics (familial tall stature) . Calculate a midparental height to consider whether this is the case. Children with simple familial tall stature should have a growth rate at the upper end of normal, a normal bone age, and no dysmorphic features. If the family is not tall, then it is appropriate to consider pathologic causes of the tall stature. Hormones that promote linear growth include growth hormone, sex steroids, and thyroid hormone. Growth hormone excess is very rare in children but can result in extreme tall stature. Children with precocious puberty experience their pubertal growth spurt early, resulting in a period of tall stature. They ultimately do not end up tall for their family, however, as they will have premature epiphyseal closure (advanced bone age). Excess thyroid hormone also leads to increased linear growth with early epiphyseal closure, resulting in ultimate nor­ mal or short stature. Overnutrition or exogenous obesity also drives linear growth, resulting in modest overgrowth in child­ hood, and is a common cause of tall stature in children. As with precocious puberty and hyperthyroidism, this is associated with increased skeletal maturation resulting in a final height that is not tall. Prader-Willi syndrome, as with most endocrine causes of obesity, is associated with short stature. Look for small hands with Prader-Willi syndrome.

A mother makes an urgent visit for her 9-year-old daughter to evaluate breasts that she noticed the other day when they were trying on swimsuits for a family vacation. Mother is frantic that "my baby is developing too early!" She is hope­ ful that you can determine what is wrong and put a stop to it ASAP. While taking the history, you learn that the mother had menarche at 1 1 .5 years of age. On physical exam of the daughter, you find bilateral breast buds, some long straight pubic hairs, and a few pimples. Question 7-1

A 6-year-old girl is brought in by her parents to evaluate for tall stature. She recently had her kindergarten winter concert, and her family noted that she was a head taller than her peers and had to stand in the back row (with the naughty boys) . She has no significant past medical history and no concerns on review of systems. Her height is at the 97th percentile. Her weight is at the 99th percentile.

What is the most appropriate course of action? A) Consult surgery to remove the breast buds. B) Order lab tests to include LH, FSH, and estradiol. C) Reassure the mother that this is normal. D) Order a magnetic resonance imaging (MRI) scan of the brain to determine the cause of central puberty. E) Order lab tests to include 1 7-0H-progesterone, dehydroepi­ androsterone sulfate (DHEA-S), and androstenedione.

Question 6-1

Discussion 7-1

Differential diagnosis includes all of the following EXCEPT: A) Growth hormone excess. B) Precocious puberty. C) Familial tall stature. D) Overnutrition. E) Prader-Willi syndrome. F) Hyperthyroidism.

The correct answer is "C:' Typical female puberty is first her­ alded by breast budding and starts at 10 years of age; however, any time after 8 years of age can be normal. It is important to consider the family history, race or ethnicity, and any medi­ cal conditions when assessing an individual child. In this case, she is older than 8 years old. Her mother had menarche at 1 1 .5 years, which suggests that she was a slightly early bloomer

C H APTER 1 1

as well. Furthermore, the girl has early Tanner 2 pubic hair, which is also consistent. We can expect that she will have another 2+ years before menarche, which is often the parent's main concern. Males have a later initiation of puberty, with testicular enlargement starting around 12 years of age. Males should be evaluated for pathologic precocious puberty if they demonstrate any signs of virilization prior to 9 years of age. It is important to become comfortable with the Tanner stages and the normal timing and tempo. (See Figures 1 1-4 and 1 1- 5 . ) If the child was younger than 8 years of age, part of the workup would likely include gonadotropins (LH, FSH) and estradiol. If the child was found to have central precocious puberty ( ele­ vated LH, FSH) , a brain MRI would be indicated. If the child demonstrated signs of androgen (as opposed to estrogen) , you would consider checking the adrenal studies mentioned in option E : "



=-:=11.. r1 1 r



E N DOC R I N E D I SO R D E R S

227

� QUICKQUIZ At what age is puberty (thelarche onset) considered preco­ cious in girls? A) 7. B) 9. C) 11. D) 13. E) 15. Discussion

The correct answer is "A:' Onset of puberty before age 8 years in girls and 9 years in boys is considered precocious.

'

Helpful Tip Rule of 2s ·



Normal fema le puberty is 1 0 yea rs ± 2 years The time from thelarche to menarche is a l ittle over 2 years



Boys a re a bout 2 years beh i n d girls

An 8-year-old girl is referred by her second-grade teacher for body odor. (We are talking teenage boy hockey locker-room quality!) On history, you learn that she has had gradually pro­ gressive pubic hair for the past 2 years. She has had some mild acne, which is controlled with over-the-counter preparations. She denies breast changes or menstrual bleeding. Her mother had menarche at 13 years of age; her father just blushes and

G I R LS

H E I G HT S P U RT

M ENARCHE

B R EAST

G ROWTH RATE Height 2 i n/y Weight 6 lb/y

Height 3 in/y Weight 1 7. 5 lb/y

AG E RANG E ... 1 1 .5-1 6 . 5 y AG E RAN G E 1 0-1 6 . 5 y Average height 62 . 5 in ( 1 58.5 em) Average weight 1 06 lb (48 kg)

ffi · 1 � � �� � [ �1 M bi�---Breast buds beg i n . AG E RAN G E 8-1 3 y

SEXUAL MATUR ITY 2 RAT I N G

Breast and areola grow.

3

I n itial hair is straight and fi ne. AG E RAN G E 8-1 4 y

AG E

Areola rejoins breast contour a n d development is complete. AGE RANG E 1 2.5-1 8.5 y

4

5



I i r I� ! 'f I� l r ll I l r 11 -

P U B I C HAI R

Nipple and areola form separate mound, protruding from breast.

11 y

-

Hair looks like Pubic hair an adult's but l i mited becomes coarse, darkens, i n area. and spreads. 12 y

13 y

Inverted triangular pattern is establ ished AGE RAN G E 1 2. 5-1 6.5 y 14 y

15 y

F I G U R E 1 1 -4. The Ta nner stages of fem a l e breast a n d pubic h a i r development. (Reproduced with permission from Hay WW, Levin MJ, Deterd i n g RR, Abzug MJ, eds. Current Diagnosis and Treatment Pediatrics. 22th ed. New York, NY: McGraw- H i l l Education; 20 1 4, Fig. 4-4.)

228

MCG RAW-H I LL E D U CATION S P E C I A LTY BOA R D REVI EW: P E D I ATRICS

APEX STRE NGTH S P U RT Height spurt 1 0-1 2 in (25-30 em) Weight 44 lb (20 kg)

BOYS

H E I G HT S P U RT

G ROWTH RATE Height 2 i n/y Weight 6 . 5 lb/y

Height 4 in/y Weight 20 lb/y

AGE RAN G E 1 3-1 7 . 5 y

l h \ 1 l l l i � IY I � I i l �

PENIS TESTES

Testes increase i n size and skin of scrotum reddens. AGE RANG E 1 0-1 3.5 y

Penis grows in length .

Penis grows i n width .

....--,---;;;.2-,--.

Development is complete . AGE RANG E 1 4. 5-1 8 y 5

SEXUAL MATUR ITY RAT I N G P U B I C HAI R

Straight hair appears at penis base . AG E RAN G E 1 0-1 5 y AG E

11 y

12 y

Hair becomes cu rly, coarse, and dark.

13 y

Hair is f u l l , l i m ited i n area.

14 y

15 y

Full development. AGE RANG E 1 4. 5-1 8 y 16 y

17 y

F I G U R E 1 1 -5 . The Ta nner stages of m a l e gen ita l a n d pubic h a i r development. (Reproduced with permission from Hay WW, Levin MJ, Deterd i n g RR, Abzug MJ, eds. Current Diagnosis and Treatmen t Pediatrics. 22th ed. New York, NY: McGraw- H i l l Education; 20 1 4, Fig 4-3.)

stares at you blankly when you ask about his puberty. On exam, you find Tanner 3 pubic hair. Question 8-1

Which of the following features would be the most reassuring on your exam and workup? A) Clitoromegaly. B) Bone age of 8 years. C) Bone age of 1 1 years. D) Elevated testosterone level. E) Tanner 4 breasts.

A 1 3-year-old girl comes in for her well-child check. Her only concern is that her friends call her "Flat Stanley" based on her breast development (or lack thereof) . She is hoping to fill out her bra before high school cheerleading tryouts this sum­ mer. She was born at term with a birth weight of 5 pounds. She has had recurrent acute otitis media but otherwise is gen­ erally healthy. She has grown along the third percentile for height with appropriate weight gain. Question 9-1

Discussion 8-1

The correct answer is "B:' Adrenarche occurs when the zona reticularis of the adrenal cortex matures and results in a rise in adrenal androgens, including DHEA-S and androstenedi­ one. The adrenal androgens are responsible for female pubarche (sexual hair) , body odor, and acne. (See Figure l l -2, earlier.) Adrenarche is separate from true central puberty and typically manifests after thelarche; however, it can occur first. Premature adrenarche can be a normal variant and result in early, slowly progressive pubarche. It is associated with a normal bone age, adrenal androgen levels consistent with Tanner stage, normal testosterone levels, normal growth rate, and ultimately normal true puberty. "Benign" premature adrenarche must be distin­ guished from precocious puberty (which would include breast development and advanced bone age), late-onset CAH, or viril­ izing tumors (both of which would result in virilization/clitoro­ megaly and advanced bone age).

Appropriate workup may include which of the following? A) Bone age X-ray. B) Karyotype. C) Growth hormone level. D) Good family history of heights and pubertal timing. E) Insulin level. F) All of the above. G) Options A, B, and D. H) Options C, D, and E. Discussion 9-1

The correct answer is "G:' Delayed female puberty can be a normal variant (constitutional delay of growth and puberty) which is typically a familial trait and is associated with a delayed bone age. If there is no family history of delayed puberty or if a girl does not have any signs of breast tissue by 1 3 years, it is important to consider other causes of delayed puberty. These

C H APTER 1 1

include hypogonadotropic hypogonadism, which can be func­ tional (due to chronic disease, eating disorders, or high energy expenditure) or pathologic due to a true/permanent gonado­ tropin-releasing hormone (GnRH) deficiency (ie, Kallmann syndrome) . Differential diagnosis of delayed puberty in females also includes hypergonadotropic hypogonadism (ovarian insuf­ ficiency) , of which the most common cause is Turner syndrome (45,XO karyotype) . •

� I



E N DOC R I N E D I SO R D E R S

229

D) 16. E) 18. Discussion

The correct answer is "C:' Lack of pubertal changes (in girls, breast buds; in boys, testicular enlargement) by age 12 years in girls and 14 years in boys is considered delayed puberty.

Helpful Tip

I n add ition to delayed puberty, other featu res of Turner

1 1r syndrome include short stature (20 em shorter than mid pa renta l heig ht), widely spaced nipples, webbed

neck, cu bitus va lg us, bicuspid aortic va lve or aortic coa rctation, horseshoe kid n ey, recurrent otitis media, and h i g h risk of a utoi m m u n e d isease. (See Figure 1 1 -6.)

� QUICKQUIZ

A 14-year-old adolescent girl was seen in her local acute care clinic for an ear infection and was noted to have thyromegaly. She was referred to your clinic for further evaluation. She reports some fatigue and cold intolerance, but admits that it is winter and she has been staying up late texting with her boyfriend. On exam, she has a thyroid that is twice normal size with no discrete nodules. Thyroid function tests show ele­ vated TSH of 25 !!IU/mL and mildly low free T4 of 0.7 ng/dL. Question 1 0-1

At what age is puberty (no testicle enlargement) considered delayed in boys? A) 10. B) 12. C) 14.

The most likely cause of her hypothyroidism is: A) Iodine deficiency. B) Thyroid cancer. C) Autoimmune (Hashimoto) thyroiditis. D) Radiation exposure. E) Congenital hypothyroidism.

,------ Short stat u re Epicanthal folds Low-set ears H i g h-arched palate Micrognathia



------

.

• •



-------':--

Webbed neck

N evi Shield-shaped chest

lll"l-------:-:- Aorta c oa rctati on

� Bicuspid aortic valve

. . -------.::-= .-: o___•

Absent b reast d evelopm e nt Hypoplastic nipples Widely-spaced a reolas

---'::-'�--':---

---.::---

---

-

Renal anomalies Wide carryi ng angle

Short 4th m etacarpal Hyperconvex nails

F I G U R E 1 1 -6. Featu res of Tu rner syndrome (45,XO). (Reproduced with permission from Hoffm a n B L, Schorge JO, Schaffer Jl, et al, eds. Williams Gynecology.

2nd ed. New York, NY: McGraw- H i l l Education; 201 2, Fig. 1 8-8.)

230

MCG RAW-H I LL E D U CATION S P E C I A LTY BOA R D REVI EW: P E D I ATRICS

Discussion 1 0-1

The correct answer is "C:' Acquired hypothyroidism in the United States is most often caused by autoimmune thyroiditis, also known as Hashimoto thyroiditis. Iodine deficiency is the most common cause of hypothyroidism worldwide but is uncommon in North America because of iodine supplementation of salt. Thyroid cancer does not typically affect thyroid function. Children who are treated with radiation for tumors of the head and neck are at higher risk of hypothyroidism; this would be considered based on the history. Congenital hypothyroidism is possible but would typically present before this age. Common symptoms of hypothy­ roidism in children include fatigue, cold intolerance, constipation, dry skin, and brittle hair. Signs of hypothyroidism include growth deceleration, bradycardia, facial puffiness, and delayed relaxation phase of deep tendon reflexes. There may be a small amount of weight gain (2-4 kg), although this is not typically severe. Question 1 0-2

What is the most appropriate treatment for this patient? A) Synthetic triiodothyronine (T) . B) Synthetic T4 . C) Pork or beef desiccated thyroid. D) Nothing until her free T4 decreases further. E) Synthetic TSH.

the 80th percentile for height and weight and is approach­ ing the end of puberty. The only abnormality is thyroid gland asymmetry; he has an approximately 1 x 2 em firm nodule on the left upper lobe that you can palpate. Lab tests in clinic show a normal TSH. Question 1 1 -1

What is the most appropriate next step? A) Reassurance. B) Obtain a free T4 level. C) Order neck/thyroid ultrasound. D) Follow up in 6 months for repeat thyroid exam. E) Obtain thyroid peroxidase antibody titer. Discussion 1 1 -1

The correct answer is "C:' Thyroid nodules may be caused by benign conditions, including multinodular goiter, autoimmune (Hashimoto) thyroiditis, simple cysts, or adenomas. However, chil­ dren presenting with thyroid nodules are more likely to have cancer than adults, and carcinoma needs to be ruled out. It is helpful to start with a TSH level to assess thyroid function. Most children pre­ senting with a thyroid nodule have normal function and will need to then undergo neck/thyroid ultrasound to further characterize the lesion and determine if fine needle aspiration is necessary.

Discussion 1 0-2

The correct answer is "B:' Synthetic T4 (levothyroxine) is the treatment of choice for children with hypothyroidism and is typically started if the TSH is greater than 7 to 10 fllU /mL. With appropriate treatment, children should be expected to resume normal growth and development. Thyroid preparations that contain T3 alone (liothyronine) or in combination preparations that are made or derived from desiccated glands are not recom­ mended for routine treatment of pediatric hypothyroidism.

� QUICKQUIZ Which laboratory value is not typical of autoimmune thyroiditis (Hashimoto thyroiditis) ? A) Elevated TSH. B) Low free T4 . C) Elevated thyroid peroxidase antibodies. D) Elevated thyroglobulin antibodies. E) Elevated thyroid-stimulating immunoglobulins. Discussion

The correct answer is "E:' Thyroid-stimulating immunoglobu­ lins or TSH receptor antibodies are typically seen in Graves disease, which causes hyperthyroidism.

A 1 5-year-old adolescent boy comes in for his sports physical. He has no concerns whatsoever. He has no significant past medical history and takes no medications. On exam, he is at

You are moonlighting in the local newborn nursery to earn money for a new bike (all carbon and electronic shifting) . You are patting yourself on the back for finding this easy gig when you get a call from the state lab saying that an infant you discharged yesterday has been found to have an abnor­ mal newborn thyroid screen, with a TSH level of greater than 100 f!IUImL. Question 1 2-1

The most appropriate next step is: A) Send the primary pediatrician a note to address the abnor­ mal level at the 2-week check. B) Call the family and have them take the infant for a thyroid ultrasound. C) Nothing, since the infant looked fine at discharge. D) Arrange for the infant go for repeat labs today, and prescribe levothyroxine. E) Call the family and have the infant go for repeat labs in 2 months. Discussion 1 2-1

The correct answer is "D:' Congenital hypothyroidism occurs in approximately 1 in 3000 newborns and is one of the most common causes of preventable mental retardation. So take it seriously! There is an inverse relationship between the age at treatment initiation and IQ, making prompt recognition and treatment of utmost importance. The goal is to start supplemen­ tation by 2 weeks of age. All states in the United States include

C H APTER 1 1

congenital hypothyroidism as part of the newborn screen, although exact protocols vary. Serum TSH concentrations rise abruptly at birth due to exposure to cold and then decrease rap­ idly to about 20 fliU/mL at 24 hours of age. An early collection can cause a false positive on the newborn screen. If a screen is positive, it is important to call the family immediately, con­ firm the result with a venipuncture TSH and free T4, and start therapy. The most common cause of congenital hypothyroid­ ism is thyroid dysgenesis (85% of cases); however, patients may also have defects in thyroid hormone metabolism. Thus, thyroid imaging can be helpful to determine etiology but does not dic­ tate treatment and is not routinely done. •

Helpful Tip

� The majority of thyroid hormone is bound by thyroid­

=-

r1 1r binding g l o b u l i n (TBG). Tota l T4 assays a re affected by

TBG amou nt, although free T4 assays a re not. Hered ita ry TBG deficiency is a n X-l i n ked recessive d i sorder

that resu lts i n normal TSH with low tota l T4 and has h i storica lly ca used diagnostic d i lemmas. We now have the capa bil ity to mea s ure a free T4, which is norma l i n



E N DOC R I N E D I SO R D E R S

231

Discussion 1 3-1

The correct answer is "C:' Elevated thyroid hormone with sup­ pressed TSH is consistent with hyperthyroidism. Hyperthy­ roidism affects many organ systems, and symptoms include palpitations, jitteriness, insomnia, weight loss, polyphagia, polyuria, increased stooling frequency, and heat intolerance. Signs include tachycardia, tremor, brisk reflexes, and weight loss. Hyperthyroidism is most commonly caused by Graves disease due to stimulating autoantibodies but can also be caused by a destructive autoimmune process with passive spilling of thyroid hormone ("hashitoxicosis") . Positive TSI or thyrotropin receptor antibodies (TrAb ), markedly enlarged and smooth thyroid gland, exophthalmos (rare in kids) , and relative elevation of T 3 greater than T 4 can point to Graves disease. Positive TPO antibodies, mildly enlarged heteroge­ neous/cobblestone gland, and relative T4 greater than T3 can point to hashitoxicosis. Management of hyperthyroidism in pediatric patients includes medical therapy with methimazole or definitive therapy with radioactive iodine ablation or thy­ roidectomy. Beta-blockers can be used to treat the tachycar­ dia if necessary.

t h i s setting o f "euthyroid hypothyroxinemia."



Helpful Tip

=-� N eonata l

G raves d i sease can

occur if there is

r1 1r tra nsplacenta l passage of materna l sti m u latory TSH­

receptor a ntibodies. It occu rs i n approxi mately 2% of

A 14-year-old adolescent girl comes to the clinic with her father because of palpitations and exercise intolerance that are interfering with her basketball season. She describes a 2- to 3-month history of the concerns with associated jitteri­ ness and poor sleep. She has had a voracious appetite but has had a stable weight (her friends are jealous that she can eat a whole bag of Flamin' Hot Cheetos every night and maintain her figure) . She has not had a menstrual period for the past 2 months despite previously being regular. She gets up once or twice a night to urinate. On exam, the young woman is fidget­ ing. She is wearing a T-shirt despite the January weather. Her pulse is 1 20 beats per minute (bpm) . Her skin is warm, and she has a tremor. You suspect hyperthyroidism. Her labs show: TSH 95% ( h i g h risk)

> 75% ( h i g h i ntermed i ate risk)

< 40% (low risk)

C l i n ica l ly yel low

Fi rst 24 h o u rs

Before discharg e

None

Gestational age (weeks)

35-36

37-38

2: 41

Feedi n g

B rea stfed - d ifficu lties, wei g ht loss

B rea stfed-g o i n g we l l

For m u l a

Sibling

Received photothera py

J a u n d iced but no photothera py

Never

Hemolytic d i sease

B l ood g ro u p i ncompat i b i l ity w i t h + DAT, G6PD

Race

East Asian

H i s pa n i c

African American

Other

Ce phal ohem ato ma, bru i s i n g

Macroso m i c I DM, polycythem i a, m a l e gender

Discharged after 72 h o u rs of age

zone on nomogram

DAT, d i rect a nti body testi ng; G6PD, g l ucose-6-phophate dehyd rogenase deficiency; I DM, i nfa nt of a d i a betic mother. 'Risk is proportional to the n u m ber risk factors present.

C H A PT E R 14

concern and is not a reliable method of estimating the actual level of bilirubin in the bloodstream. It is also not acceptable to wait until neurologic symptoms arise before intervening. Measurement of the infant's total bilirubin level may be done by blood test or transcutaneous device. Transcutaneous measure­ ment is noninvasive but does not seem to be as accurate as a total serum or plasma bilirubin in certain settings, particularly following phototherapy, at high levels, or in infants with dark skin pigmentation. Your transcutaneous bilirubin measurement is 13 mg/dL, which is in the high intermediate risk zone on the nomo­ gram. The level is below the phototherapy threshold for an infant of at least 38 weeks' gestation and without neurotoxic­ ity risk factors.



FETUS A N D N EWBORN

287

The medical student following this infant notes that she has heard of both breast milk and breastfeeding jaundice, and wonders if they are the same thing. Question 5-6

What of the following best describes breast milk jaundice?

A) Breast milk jaundice typically occurs within the first few days of life before the mother's mature milk is in. B) Breast milk jaundice may last up to 3 months of age. C) Supplementation is typically recommended in cases of breast milk jaundice. D) Breast milk jaundice can be confirmed by an elevated direct bilirubin level. E) The terms are synonymous. Discussion 5-6

Question 5-5

Which is NOT a neurotoxicity risk factor when deciding if an infant needs phototherapy?

A) B) C) D) E)

Hemolytic disease. Temperature instability. Sepsis. Hypoalbuminemia. East Asian race.

Discussion 5-5

The correct answer is "E:' East Asian race is a risk factor for developing severe hyperbilirubinemia but is not a neurotoxicity risk factor. The Bhutani nomogram is used to the map total bili­ rubin levels to a risk zone based on the age of the infant at time of collection. This predicts the risk of developing severe hyper­ bilirubinemia (defined as a serum bilirubin > 95% for age) and determines the urgency for follow-up assessment, which is par­ ticularly important when planning for discharge. To determine the threshold for phototherapy, the bilirubin must be plotted on a separate nomogram. The threshold varies based on gestational age and neurotoxicity risk factors, which include hemolytic disease, asphyxia, significant lethargy, temperature instability, sepsis, acidosis, and hypoalbuminemia. A third curve is used to determine the threshold for exchange transfusion. The bilirubin nomogram and treatment threshold curves are not reproduced here. The reader should be aware of their existence, how to read them, and that higher risk babies (those with prematurity, evidence of infection or, concerns for h emo­ lysis) require earlier and more aggressive therapy. It is not use­ ful to memorize the curves or know specific cut off points for treatment. •

Helpful Tip

� Any i nfa nt with j a u n d ice noted i n the fi rst 24 hours of

i1 1 r l ife or whose bilirubin is ra pidly rising (> 0.2 mg/d l/h or 5 mg/d l/day) m ust be eva l uated for pathologic cause, often hemolytic d i sease such as ABO i ncom pati bil ity. A positive d i rect a nti body test (Coombs test) mea n s hemolysis is present.

The correct answer is "B:' Breastfeeding failure and breast milk jaundice are both causes of indirect hyperbilirubinemia in infants, resulting in increased enterohepatic circulation of bili­ rubin. Breast milk jaundice occurs between DOL 5 and 7, usu­ ally peaks within the second to third week of life, and gradually decreases over 1 to 3 months. Breast milk is thought to contain an unidentified substance that promotes intestinal absorption of bilirubin. Beta-glucuronidase, an enzyme that deconjugates bilirubin and enhances intestinal reabsorption of bilirubin has been proposed as the causal agent. Although severe hyperbili­ rub inemia necessitating intervention is unlikely in breast milk jaundice, it is important to monitor for high conjugated biliru­ bin levels or significant increases over time. If jaundice does not resolve by 12 weeks, pathologic causes should be reevaluated. A simple diagnostic test is to feed the infant formula for a few days. The bilirubin level should rapidly drop, confirming the diagnosis. Resumption of breastfeeding usually does not result in a significant increase in bilirubin. Breastfeeding failure jaun­ dice begins after 24 hours of age, peaks between DOL 3 and 5, and resolves within the first week of life. It is caused by inad­ equate milk intake, causing hypovolemia through weight and fluid loss, delayed bilirubin excretion, and increased enterohe­ patic circulation (delayed passage of bilirubin-rich meconium). •

Helpful Tip

� All newborns lose weig ht i n the first week of life, with

=.

i1 1 r most sou rces quoting 1 0% to 1 2% weig ht loss as norma l .

A neonatal resuscitation team is called to an impending delivery of a 41 -week gestational age male infant. The team is notified of thick meconium-stained fluid and persistent vari­ able heart rate decelerations with slow recovery to baseline for the past 30 minutes. The mother had spontaneous rupture of membranes approximately 20 hours ago. Vacuum-assisted vaginal delivery is being attempted as the team enters the room. The infant is delivered after two pulls on the vacuum

288

MCG RAW-H I LL E D U CATION S P E C I A LTY BOA R D REVI EW: P E D I ATRICS

device. There was one pop-off of the vacuum device between attempts. The infant is handed to the waiting neonatal inten­ sive care unit (NICU) team for assessment and treatment. He is not crying, has no spontaneous respirations, appears limp and cyanotic, and has a heart rate of 75 bpm. Question 6-1

What is the first step in the management of this infant?

A) Immediately begin tactile stimulation, including drying and gently rubbing the back to encourage respirations. B) Apply free-flowing oxygen to the infant's face as he is cyanotic. C) Suction the mouth followed by the nose with a bulb syringe. D) Intubate the trachea and suction for meconium. E) Start positive-pressure ventilation (PPV) as the infant has no spontaneous respirations.

an infant requires PPV, when central cyanosis is present, when supplemental oxygen is required, or to confirm the perception of cyanosis. Adequate respiratory effort and heart rate will deter­ mine the need for PPV, not the pulse oximetry alone. If a new­ born is not responding with increased heart rate and saturations, first reassessment of airway position and patency should be com­ pleted, followed by continuation of PPV, initiation of chest com­ pressions if the heart rate is less than 60 bpm, and administration of bolus epinephrine, preferably using the intravenous route or endotracheal tube if intravenous access unavailable. After the initiation of PPV, the newborn's heart rate increased to 1 65 bpm and the saturation levels increased to 90% on 40% fraction of inspired oxygen (FiO) . The infant is intubated in the delivery room and transported to the NICU for further care. Apgar scores of 1, 3, and 7 are obtained at 1, 5, and 10 minutes of age.

Discussion 6-1

The correct answer is "D:' This newborn remains nonvigorous at the time of delivery with meconium present. Therefore, he should be intubated immediately (prior to any stimulation) and the trachea should be suctioned. A vigorous newborn is defined as an infant who has strong respiratory efforts, good muscle tone, and a heart rate greater than 1 00 bpm. If the newborn is vigorous despite the presence of meconium, suction the mouth and nose only. If the newborn in this case remains nonvigorous after intubation and suctioning of the trachea, then additional resuscitation should be undertaken promptly.

Question 6-3

The newborn is at risk for which of the following consequences?

A) Lactic acidosis due to poor perfusion. B) Hyperglycemia due to stress counter-regulatory hormones. C) Increased risk of subgaleal hemorrhage and subsequent hypovolemic shock due to vacuum-assisted delivery. D) Increased risk of seizures due to initial hypoxia. E) Respiratory failure due to meconium aspiration. F) All of the above. Discussion 6-3

After intubation and suctioning for meconium, the infant's heart rate remains at 80 bpm and he is taking only a few gasp­ ing breaths. Question 6-2

What should the next step be in resuscitation management of this infant?

A) Apply free-flowing oxygen to the infant's face to encourage breathing. B) Continue to stimulate the infant for several minutes to increase respiratory effort. C) Provide PPV via bag-mask or T-piece resuscitator. D) Start chest compressions in response to bradycardia. E) Apply a pulse oximetry device and only start PPV if oxygen saturations remain less than 75% at 2 minutes of age. Discussion 6-2

The correct answer is "C:' The initial steps of resuscitation include appropriate positioning of the airway into the "sniffing" position and tactile stimulation for not more than 60 seconds. If a newborn has not responded to these efforts and has persistent apnea, then PPV should be started promptly. Chest compres­ sions should only be started after the infant has failed to respond to PPV and the heart rate remains less than 60 bpm. Free-flowing oxygen without PPV should only be used in an infant with ade­ quate respiratory effort. The decisions and actions during new­ born resuscitation should be based on respirations, heart rate, and color (oxygenation). Pulse oximetry should be applied when

The correct answer is "E' Newborns requiring significant resus­ citation at birth are at risk for several metabolic consequences of poor perfusion in the first few minutes of life. They must be observed closely for end-organ damage caused by hypoxemia, including lactic acidosis, liver and kidney damage, cardiovascu­ lar instability, and electrolyte abnormalities, especially hypogly­ cemia and hyperglycemia. Although suctioning of the trachea to remove meconium has been done in this case, a newborn with thick meconium at delivery still remains at risk of the consequences of meconium aspiration syndrome, including persistent pulmonary hypertension of the newborn. The con­ sequences of hypoxia to the brain are the most significant given the nonreversibility of these changes. Prevention of seizures and treatment with therapeutic hypothermia are recommended in moderate and severe cases of hypoxic ischemic encephalopathy. It is important to recognize that the use of vacuum -assist devices in vaginal deliveries is associated with increased risk of subga­ leal bleeding. Newborns should be monitored closely by means of serial physical exams and hemoglobin levels if there is any concern for subgaleal hemorrhage. Infants can rapidly decom­ pensate, developing hypovolemic shock in these situations.

A mother is admitted into labor and delivery with a 1 -day history of dysuria, fevers to 38.9°C ( 1 02°F), and now active contractions every 2 to 3 minutes for about 1 hour. Her fetus

C H A PT E R 1 4

has an estimated gestational age of 24-3/7 weeks based on last menstrual period (LMP) and a concordant 8-week ultra­ sound. The mother had regular prenatal care and an uncom­ plicated pregnancy until now. On exam, the physician notes that the mother's cervix is dilated to 8 em with a bulging bag of water. She then has spontaneous rupture of membranes with green-colored, foul-smelling fluid. She rapidly delivers a 575-g male infant prior to maternal administration ofbeta­ methasone. The NICU team is present at the delivery. Question 7-1

Which of these statements is accurate about this infant?

A) The Apgar score can be used to reliably predict asphyxia in a very preterm infant. B) The Apgar score is composed of five components: heart rate, respiratory effort, blood pressure, color, and oxygen saturation. C) The Apgar score can be affected by maternal sedation or analgesia as well as neurologic conditions such as muscle disease or cerebral malformations. D) The Apgar score is composed of five components: heart rate, respiratory effort, oxygen saturation, respiratory distress, and color. E) None of the above. Discussion 7-1



Helpful Tip

� Maternal

-

medications

ta ken

during

FETUS A N D N EWBORN

289

C) Monitor closely for low blood glucose as this infant will be at risk for hypoglycemia due to gestational age. D) Monitor closely as this infant will be at risk for apnea due to gestational age. E) All of the above. Discussion 7-2

The correct answer is "E:' Extremely low and very low birth­ weight infants are at risk for several medical morbidities beyond the basic resuscitation of airway, breathing, and circulation. The preterm neonate is at high risk for cold stress owing to rela­ tively high surface area with very immature skin. Delivery room practices to minimize hypothermia are important. The infant is also at high risk for hypoglycemia due to poor glycogen stores, relative liver immaturity, and abnormal glucose regulatory hor­ mone pathways. Very preterm infants are at risk of central apnea and may be treated with methylxanthine medications (caffeine) to minimize this risk. •

� I

The correct answer is "C' The Apgar score is composed of five components-heart rate, respiratory effort, tone, reflex irritabil­ ity, and color-each of which can be given a score of 0, 1, or 2. Hopefully you remember this from earlier. The scoring provides consistent shorthand for reporting the state of the infant and effec­ tiveness of resuscitation. It is important to recognize, however, that elements of the score, including tone, color, and reflex irritability, are partially dependent on the physiologic maturity of the infant. Thus, normal preterm infants may receive a lower score because of immaturity without evidence of asphyxia. Maternal sedation, neu­ rologic conditions, and cardiorespiratory conditions that interfere with heart rate or tone may alter the Apgar score.



Helpful Tip

I n premature i nfa nts, birthweight may b e classified as

1 1 r low, very l ow, or extremely low as fol l ows: Low birthweig ht (LBW) < 2500 g

Very low birthweight (VLBW) < 1 500 g Extremely low birthweig ht (ELBW) < 1 000 g

Question 7-3

What prognostic factors in this ELBW infant will improve survival without disability?

A) B) C) D) E)

Singleton gestation. Male sex. Active maternal chorioamnionitis. No betamethasone administration. None of the above.

preg nancy,

r1 1r del ivery, or both can affect the fetus a n d neonate.

Mag nesi u m or na rcotics g iven d u ring l a bor may cause neurologic a n d respi ratory depression i n the neonate

after del ivery. Medications taken by brea stfeed ing mothers may be tra nsm itted thro u g h the breastm i l k; an exa m ple is codeine, which has ca used neonata l death from res pi ratory depression.

Question 7-2

Which of these steps is/are important in the management of this extremely low birthweight (ELBW) preterm infant?

A) Provide thermal support immediately after delivery due to the poor thermoregulation capabilities of preterm infants. B) Use a bag or plastic covering in the delivery room to mini­ mize heat loss through evaporation or convection.

Discussion 7-3

The correct answer is "A:' This ELBW 575-g infant born at 24-3/7 weeks' gestation to a mother with active chorio­ amnionitis who was unable to receive betamethasone has several risk factors for poor prognosis. Being a single gesta­ tion (as opposed to one of multiples) improves risk status, but all of the other options worsen prognosis. Betametha­ sone is administered to mothers in two doses with 24 hours between dosing. This medication has been found to improve fetal lung maturity and decrease intracranial hemorrhage, which are both maj or causes of morbidity and mortality in this population. A fetus exposed to active maternal infection ( eg, chorioamnionitis) has a worsened probability of sur­ vival without disability owing to the high likelihood of fetal infection and multiorgan dysfunction that is associated with infection.

MCG RAW-H I LL E D U CATION S P E C I A LTY BOA R D REVI EW: P E D I ATRICS

290

� QUICKQUIZ

Question 8-1

What evaluation should be completed at this time?

Which of the following regarding intraventricular hemor­ rhage (IVH) is FALSE?

A) B) C) D) E)

IVH is graded I to IV. Hydrocephalus is a complication of iVH. Apnea and seizure are clinical manifestations of IVH. Polycythemia may result from IVH. Premature infants should have routine screening for IVH.

A) B) C) D)

Serum bilirubin. Hemoglobin to assess for polycythemia. Infant and maternal blood type. Physical exam to assess for cephalohematoma, caput, and subgaleal hemorrhage. E) Physical exam to assess hydration. F) All of the above. Discussion 8-1

Discussion

The correct answer is "D:' The severity of IVH is graded from I to IV based on the location and extent of bleeding, as follows: •







Grade I: germinal matrix Grade II: extension of grade I involving the ventricles with­ out ventricular dilation Grade III: extension grade II with ventricular dilation Grade IV: ventricular dilation with extension into the cere­ bral cortex

IVH is common in low birthweight and premature infants. The infant may be asymptomatic or have apnea, coma, leth­ argy, bradycardia, seizures, hypotension, metabolic acidosis, and anemia. Infants at risk should be screened routinely by ultrasound: •

Helpful Tip

:5.� I ntraventricu l a r

hemorrhage

(IVH)

is

a

major

i1 1r compl ication o f extreme prematu rity. T h e germ i n a l

matrix is a fra g i l e network o f blood vessels. It is prone to bleed ing with any abrupt change i n hemodynam ics, especia l ly between the time of del ivery u ntil a bout 1 week of age i n preterm i nfa nts. Bleed ing is most common i n those less than 34 weeks' gestation. I ntraventricu l a r hemorrhage a n d hypoxic i nj u ry ca n lead to loss of periventricu l a r wh ite matter

(leukoma lacia) and su bseq uent cerebra l pal sy, one of the major compl ications of extreme prematu rity.

A full-term infant is delivered at 40-2/7 weeks' gestation after a pregnancy complicated by maternal diabetes requiring insulin for blood glucose control. The delivery requires vac­ uum assistance for a relatively large macrosomic infant. The infant does well in the delivery room, requiring only stimula­ tion and drying. Blood glucose levels are slightly low in the first 6 hours but improve with frequent feeding attempts. The physician caring for the infant receives a call at 24 hours of age that the infant has a transcutaneous bilirubin reading in the high-risk zone and appears well, but somewhat sleepy and quite ruddy in color on exam.

The correct answer is "E' Jaundice in the first 24 hours of life in a neonate is pathologic, most commonly a result of hemo­ lysis. This infant has several risk factors for the development of severe hyperbilirubinemia of the newborn. The transcutaneous bilirubin level should be confirmed with serum bilirubin test­ ing. Infants of diabetic mothers are at higher risk of polycythe­ mia (central hematocrit > 65%), adding to the risk of jaundice. One of the most common causes of severe hyperbilirubinemia is ABO incompatibility, with hemolysis and resultant hyperbiliru­ binemia. Therefore, screening of mother and infant blood types is important, especially with a blood type 0 mother. Infants who have been delivered with the assistance of instrumentation such as the vacuum device are at higher risk of cephalohematoma and subgaleal bleeding that can significantly increase bilirubin levels and should be assessed regularly by physical exam. Physi­ ologic jaundice of the newborn is exacerbated by dehydration, manifested by weight loss, poor feeding with decreased stool output, and increased enterohepatic circulation, all leading to increased jaundice. (See Table 14-2.) The infant undergoes the appropriate evaluation and is found to have a serum unconjugated bilirubin of 1 3 mg/dL at 24 hours with a conjugated bilirubin of 0.4 mg/dl, a free­ flowing hematocrit of 72%, and a small cephalohematoma at the vacuum site. Both mother and infant are blood type A+. The infant has made only marginal breastfeeding attempts due to sleepiness. Question 8-2

Which is the true statement?

A) This infant does not need phototherapy for increased biliru­ bin until DOL 3. B) This infant may need a partial exchange transfusion due to polycythemia with symptoms of hyperviscosity. C) This infant does not need supplemental feedings or fluids unless the bilirubin continues to rise. D) This infant can be discharged home as long as parents agree to follow up in 24 hours for another bilirubin check. E) The conjugated bilirubin is elevated and needs additional evaluation for metabolic disease. Discussion 8-2

The correct answer is "B." This infant has a bilirubin level above the indicated threshold for treatment with photo­ therapy and therefore should start inpatient phototherapy immediately. The infant's hematocrit is elevated in a manner

C H A PT E R 1 4

consistent with polycythemia. Infants with severe hypervis­ cosity can have lethargy, tremors, respiratory distress, cyano­ sis, and even seizures or strokes. This infant needs increased fluid intake and may need a partial volume exchange trans­ fusion (involving removal of blood and replacement with saline or albumin) . Infants should not be discharged early if any risk factors for increased j aundice (hemolytic dis­ ease, intracranial bleeding, lethargy, sepsis, etc) are pres­ ent. Kernicterus (irreversible bilirubin encephalopathy) can result from severe hyperbilirubinemia. Infants with bilirubin encephalopathy are lethargic, hypotonic or hypertonic, and have a high-pitched cry. Opisthotonus, seizures, and death may occur if bilirubin is not lowered. Magnetic resonance imaging (MRI) scans shows a high T2 signal in the globus pallidus (part of the basal ganglia) . Patients surviving ker­ nicterus have severe permanent neurologic symptoms ( cho­ reoathetosis, spasticity, hearing loss, ataxia, and cognitive impairment) . Less severe injury is associated with mild neu­ rologic abnormalities, including hearing loss, which may be the only abnormality. This infant has a normal conj ugated bilirubin; however, infants who have an increased conj ugated fraction of bilirubin need additional evaluation for several serious disorders. The liver and gall bladder anatomy should be assessed for biliary atresia, choledochal cyst, or other ana­ tomic causes of biliary obstruction. Infections, especially viral hepatitis, Ebstein-Barr virus (EBV ) , cytomegalovirus (CMV) , and other neonatal viral infections, should be inves­ tigated. Severe sepsis can cause conjugated hyperbilirubine­ mia. Inborn errors of metabolism should be evaluated. Total parenteral nutrition cholestasis may be the cause in those receiving long-term intravenous nutrition.

� QUICKQUIZ What is the definition of conjugated hyperbilirubinemia?

A) Conjugated bilirubin greater than 1 mg/dL. B) Conjugated bilirubin greater than 5 mg/dL. C) Conjugated bilirubin greater than 10 mg/dL. D) Conjugated bilirubin greater than 10% total serum bilirubin. E) None of the above. Discussion

The correct answer is "E:' Conjugated hyperbilirubinemia is never normal in a neonate. Its definition is dependent on the total serum bilirubin value, with 5 being the reference value, as follows: Conjugated bilirubin > 1 mg/dL if the total serum bilirubin is < 5 mg/dL

or Conjugated bilirubin > 20% of the total serum bilirubin if the serum bilirubin is > 5 mg/dL



FETUS A N D N EWBORN

291

A 35-week gestational age infant is born by cesarean delivery to a mother with placenta previa. The infant girl is vigorous after delivery, with Apgar scores of 8 and 8. She is brought to her mother's chest. Fifteen minutes later the NICU team is called because the infant is experiencing worsening respira­ tory distress with tachypnea, retractions, and mild grunting respirations. She is placed on nasal continuous positive air­ way pressure (CPAP) and admitted to the NICU. The infant shows clinical improvement on CPAP, requiring approxi­ mately 25% Fi02• Her chest radiograph shows 1 0-rib expan­ sion with fluid in the fissure and no other focal infiltrates. You are asked to update her family. Question 9-1

You tell them: A) This is most consistent with neonatal pneumonia and sepsis. You will start antibiotics immediately and she will recover in several days to a week. B) This is most consistent with respiratory distress syn­ drome resulting from surfactant deficiency related to prematurity. She may need to be intubated for surfactant administration. C) This is most consistent with transient tachypnea of the newborn and fluid retention exacerbated by the cesar­ ean delivery without labor. She will improve in the next 24 hours . D) This is most consistent with meconium aspiration. She will improve within days to weeks. E) None of the above. Discussion 9-1

The correct answer is "C:' Transient tachypnea of the newborn is most common in late preterm and term infants who are delivered by cesarean section or precipitous delivery. Chest films demon­ strate adequate lung expansion with retained fetal lung fluid. This condition generally improves quickly over hours and affected infants very rarely need intubation and mechanical ventilation.

A 35-week gestational age infant is born by cesarean delivery to a mother with placenta previa. The infant girl is vigorous after delivery, with Apgar scores of 8 and 8. She is brought to her mother's chest. Fifteen minutes later the NICU team is called because the infant is experiencing worsening respira­ tory distress with tachypnea, retractions, and mild grunting respirations. She is placed on nasal CPAP and admitted to the NICU. The infant continues to be tachypneic, at 80 breaths per minute, has mild retractions, and requires 40% Fi02 on CPAP. Her chest radiograph shows approximately 7-rib expansion with ground glass opacities throughout all lung fields. You are asked to update her family.

292

MCG RAW-H I LL E D U CATION S P E C I A LTY BOA R D REVI EW: P E D I ATRICS

Question 1 0-1

You tell them: A) This is most consistent with neonatal pneumonia and sepsis. You will start antibiotics immediately and she will recover in several days to a week. B) This is most consistent with respiratory distress syndrome resulting from surfactant deficiency related to prematurity. She may need to be intubated for surfactant administration. C) This is most consistent with transient tachypnea of the new­ born and fluid retention exacerbated by the cesarean deliv­ ery without labor. She will improve in the next 24 hours. D) This is most consistent with meconium aspiration. She will improve within days to weeks. E) None of the above. Discussion 1 0-1

The correct answer is "B:' Although the clinical scenario is nearly identical to the one described in Case 9, this infant has persistent distress on CPAP with rising oxygen and evidence of respiratory distress syndrome on chest radiograph with decreased lung expansion and findings consistent with surfac­ tant deficiency. (See Figure 14-7.) Although respiratory distress syndrome is most common in very preterm infants, many late preterm infants of less than 37 weeks' gestation have surfactant deficiency, especially those who have not had spontaneous labor. If clinical symptoms continue to worsen, oxygen needs increase, or gas exchange is poor as evidenced by respiratory acidosis on blood gas measurements, then this infant should be intubated and surfactant administered through the endotracheal tube.

A 39-week gestational age infant is born after prolonged rup­ ture of membranes. The membranes ruptured 48 hours before admission, and the mother presented to the hospital with a temperature of 39.4°C (103°F), foul-smelling amniotic fluid, and severe abdominal pain. She is started on antibiotics shortly before delivery. The infant is born by cesarean delivery due to persistent fetal tachycardia and concerns for maternal cho­ rioamnionitis. Apgar scores are 8 and 8. The infant is initially vigorous and is placed on the mother's chest for transition. The infant is brought to the NICU for evaluation due to suspected maternal chorioamnionitis and develops respiratory distress within hours of admission. Findings include tachypnea, mild retractions, and grunting respirations. Her chest radiograph has patchy infiltrates bilaterally. You are asked to update her family. Question 1 1 -1

You tell them: A) This is most consistent with neonatal pneumonia and sepsis. You will continue antibiotics and she will likely recover in several days to a week. B) This is most consistent with respiratory distress syndrome resulting from surfactant deficiency. She may need to be intubated for surfactant administration. C) This is most consistent with transient tachypnea of the new­ born and fluid retention exacerbated by cesarean delivery without labor. She will improve in the next 24 hours. D) This is most consistent with tension pneumothorax. E) None of the above. Discussion 1 1 -1

The correct answer is "A:' This infant's respiratory distress is most consistent with an infectious cause stemming from pro­ longed rupture of membranes (PROM) and maternal infection. The most common infectious etiology in the newborn is group B streptococcal ( GBS) infection, which can lead to pneumonia, sep­ sis, or even meningitis. Escherichia coli is the next most common cause. The infant is generally infected with vaginal flora bacte­ ria, with higher infection rates associated with PROM. Empiric antibiotic coverage in the newborn period is therefore targeted at these most common organisms, and consists of ampicillin and gentamicin intravenously. (See Figure 14-8.) •



Helpful Tip

Prolonged rupture of membra nes (> 1 8 hou rs) is

1 1 1r associated with neonata l sepsis.

FIGURE

1 4-7. Res piratory

d i stress synd rome. B i l atera l g round g lass opacities, poor i nflation, a n d promi nent a i r bronchogra m s a re present o n the chest X-ray of this i nfa nt with respi ratory distress syndrome. (Reprod uced with perm ission from Tinti n a l l i J E, Stapczynski JS, Ma OJ, et a l : Tintinalli's Emergency Medicine: A Comprehensive Study Guide, 8th ed. McGraw- H i l l Education, I nc., 201 6. F i g 1 07-2.)

� QUICKQUlZ What is NOT a risk factor for neonatal sepsis? A) Rupture of membranes for 20 hours. B) Maternal chorioamnionitis. C) Prematurity.

C H A PT E R 1 4

Signs of neonatal sepsis?

Yes

Full diagnostic evaluation antibiotic therapy*

Maternal chorioamnion itis?

Yes

Limited evaluation antibiotic therapyt

GBS prophylaxis indicated for mother?

No

� No

�No

t

Routine clin ical care +

C) Respiratory failure resulting from hypoxic ischemic encephalopathy. D) Respiratory failure resulting from congestive heart failure. E) All of the above. Discussion 1 2-1

Yes



Obse rvation for � 48 h +

-

Yes



Observation for � 48 h**

Limited evaluation observation for � 48 h +

The infant has been o n the ventilator for about 6 hours when you are called to the bedside for a sudden decompensation with decreased saturations as low as 30%, bradycardia, and absent breath sounds on the right.

�No

Either < 37 wk or duration of membrane rupture � 1 8 h?

293

-

No

� 37 wk and d u ration of membrane rupture < 1 8 h?

FETUS A N D N EWBORN

The correct answer is "B:' This infant likely has hypoxemic respi­ ratory failure resulting from meconium aspiration syndrome and persistent pulmonary hypertension, but this etiology can be complicated by sepsis and hypoxic ischemic encephalopa­ thy. The chest film in meconium aspiration generally shows scattered patchy infiltrates with significant hyperinflation. The hypoxemia associated with these conditions can be extreme, at times requiring high-frequency ventilation or even extracorpo­ real membrane oxygenation (ECMO) .

tYes

Mother received intravenous penici l l i n , ampici l l i n , or cefazol i n for � 4 h before delivery?

-+J



-

Yes

'Full eval uation includes blood culture, a complete blood count (CBC) , chest radiograph (if respiratory symptoms present), and lumbar puncture (if infant is stable enough to tolerate procedu re). t umited evaluation includes blood culture and CBC (at birth and/or 6-12 hours of age). +11 signs of sepsis develop, periorm a full evaluation and start antibiotics. .. CBC at age 6-12 hours of age is recommended.

F I G U R E 1 4-8. Secondary prevention of early-onset g roup B streptococcal (GBS)

disease in newborns. (Reproduced with permission from Vera ni J R, McGee L, Schrag SJ, et al: Prevention of perinatal group B streptococcal disease-revised g u ideli nes from CDC, 201 0, MMWR Recomm Rep. 201 0 Nov 1 9;59(RR-1 0): 1 -36.)

D) Maternal colonization with GBS. E) All of the above. Discussion

The correct answer is "E:'

A postterm 42-3/7-week gestational age infant is born after induced vaginal delivery. Meconium-stained fluid was noted after rupture approximately 6 hours before delivery. The NICU team is present at the delivery. The infant's head deliv­ ers easily; however, shoulder dystocia is immediately recog­ nized and the infant's body does not deliver for approximately 4 minutes after delivery of the head. The infant is initially apneic, with very poor tone and no spontaneous movements. The NICU team immediately intubates the infant and suc­ tions the airway. The infant is admitted to the NICU with sig­ nificant hypoxemia that worsens over the first 6 hours of life. Question 1 2-1

What is etiology of the respiratory failure in this infant? A) Respiratory failure resulting from sepsis. B) Respiratory failure resulting from meconium aspiration syndrome and persistent pulmonary hypertension of the newborn.

Question 1 2-2

What is the most likely problem? A) The infant has a previously undiagnosed heart problem. B) The infant has a tension pneumothorax associated with meconium aspiration. C) The infant needs surfactant. D) The infant needs antibiotics. E) None of the above. Discussion 1 2-2

The correct answer is "B:' Air leak syndrome with pneumothorax is a common serious complication of meconium aspiration syn­ drome that results from air trapping and the high ventilator needs in this condition. Tension pneumothoraces must be recognized immediately and relieved with needle thoracentesis. Although a ductal-dependent cardiac lesion could lead to severe hypoxemia upon ductal closure, it would be uncommon for a cardiac lesion to suddenly cause absent breath sounds on the right. Although this infant may need surfactant and antibiotics during the course of illness, it would again be uncommon to have a sudden decompen­ sation with these physical findings as a result of those problems.

As the infant improves from a respiratory standpoint, the nurse notices that she does not move her right arm very often and her arm seems to be rotated in toward her body. The nurse asks what you think caused this. Question 1 2-3

The most likely cause of this problem would be: A) The peripheral IV line that was placed in the infant's arm is causing decreased movement. B) The infant has a brachial plexus injury associated with delivery complicated by shoulder dystocia. C) The infant had a perinatal stroke. D) The infant received sedation medication. E) None of the above.

MCG RAW-H I LL E D U CATION S P E C I A LTY BOA R D REVI EW: P E D I ATRICS

294

Discussion 1 2-3

The correct answer is "B:' Stretch injuries to the brachial plexus like this one, which has resulted in an Erb palsy, are associated with difficult deliveries, especially those involving shoulder dys­ tocia and macrosomia. It is important to assess for clavicular fractures as well after delivery. Some brachial plexus injuries are severe enough to require surgery; however, many improve with physical therapy and time.

};{ QUICKQUIZ What is NOT true regarding brachial plexus injuries? A) Brachial plexus injuries may be bilateral. B) Brachial plexus injuries may cause Horner syndrome if the sympathetic fibers of T l are affected. C) Clinical manifestations of a brachial plexus injury depend on the specific nerves involved. D) The most common brachial plexus injury is associated with the "waiter's tip" posture. E) Brachial plexus injuries are diagnosed by ultrasound. Discussion

The correct answer is "E:' Brachial plexus injuries are evident clinically but may require electromyography in some cases. The brachial plexus involves cervical nerves CS to CS and the first thoracic nerve T l . Brachial plexus injuries may cause arm paralysis, Horner syndrome, or hemidiaphragm paralysis from phrenic nerve involvement. Erb palsy involves injury to CS, C6, and C7. This is the most common injury. The upper arm is adducted and internally rotated; the forearm is pronated and extended; and the wrist and fingers are flexed. This is called the "waiter's tip" posture. Klumpke palsy involves injury to CS and T l , resulting in isolated hand paralysis and Horner syn­ drome. It is rare and may represent residual deficit after the upper plexus portion has recovered. •

Helpful Tip

� Macrosomia,

operative vag i n a l delivery, matern a l

r1 1r pelvic anomal ies, breech presentation, a n d shoulder dystocia a re risk factors for birth i nj u ries. I nj u ries include bruising, sca l p bleed ing or swel l i ng, brachial plexus i nj u ry, cervica l spine tran section, i ntracra n i a l hemorrhage, fractu res, a n d i ntra -a bd o m i n a l i nj u ries.

infant is breathing well and has an acceptable heart rate, but remains slightly pale with prolonged capillary refill time both centrally and peripherally. Question 1 3-1

What recommendations do you give this physician prior to arrival of the transport team? A) Place the infant in a sterile bowel bag to cover the defect and minimize insensible losses. B) Keep the defect central with minimal kinking of the bowel if possible. C) Anticipate large insensible losses; obtain intravenous (IV) access and begin IV volume resuscitation. D) Treat respiratory symptoms if needed, remain aware of apnea in this population. E) All of the above. Discussion 1 3-1

The correct answer is "E:' Although it is preferred that infants with gastroschisis be delivered in a tertiary care center with immediate access to pediatric surgeons, there is often associated polyhydramnios in these pregnancies making them high risk for preterm labor. After delivery, many infants with gastroschi­ sis lose a tremendous amount of fluid through insensible losses and require significant volume resuscitation to maintain perfu­ sion to the bowel which is at risk for compromise.

While the transport team is en route to pick up the infant with gastroschisis, labor and delivery calls for another NICU team to attend the delivery of a full-term infant with pre­ natally diagnosed omphalocele. The infant is delivered via cesarean section and remains vigorous. You note a large abdominal wall defect centrally in the area of the umbilicus that is completely covered with a gelatinous membrane. Question 1 3-2

Which of these statements about abdominal wall defects is accurate? A) Infants with gastroschisis are more likely than those with omphalocele to have associated chromosome problems. B) The definition of omphalocele is based on the presence of a membrane covering. C) The definition of omphalocele is based on the location of the defect involving the umbilical ring in contrast to gastroschi­ sis, which is lateral to the umbilicus. D) Associated cardiac defects and other anomalies are rare in omphalocele, but common in gastroschisis. E) None of the above. Discussion 1 3-2

An infant is born at 35-5/7 weeks' gestation with a prenatally diagnosed gastroschisis defect. The mother received regular prenatal care at a tertiary care facility throughout the preg­ nancy. Unfortunately, she developed rapidly progressing pre­ term labor and was admitted to a small community hospital near her home. The physician on call delivers the infant and requests transport to your center. The physician reports the

The correct answer is "C:' By definition the defect location assigns the nomenclature of gastroschisis versus omphalocele. Although omphaloceles are generally membrane covered by the umbilicus, these can rupture before or at delivery. Omphalocele is more common in infants born to older mothers and is more often associated with other anomalies, especially aneuploidy. Infants with omphalocele should receive a comprehensive

C H A PT E R 14

evaluation, including cardiac echocardiogram, karyotyping, and renal ultrasound. Gastroschisis occurs very early in preg­ nancy. The defect is generally located to the right of the umbi­ licus, and the amount of intestine outside the body may vary. Gastroschisis is not generally associated with other anomalies or chromosome problems. The incidence is higher in adolescent mothers and mothers who smoke cigarettes.





FETUS A N D N EWBORN

295

Helpful Tip

� Fetuses

=

who a re u n a b l e to swa l l ow wel l or have obstruction i n utero may present with polyhyd ra m n ios. Exa m ples i n c l u d e i nfa nts with tra­ cheoesophageal fistula, i ntesti n a l atresias, and neu­

r1 1r i ntesti n a l

ro m u scu l a r cond itions. O l i g ohydra m n ios ca n occ u r w i t h a va ri ety o f co n d itions, i n c l u d i n g feta l ren a l a n o m a l ies l e a d i n g t o decreased u r i n e output, feta l u r i n a ry output obstruction, placenta l i n sufficiency, or premature ru pture of mem bra nes. Fetu ses with

A preterm infant born at 25 weeks' gestation is now 6 weeks old (3 1 weeks corrected gestational age) . The infant has been stable for the past few weeks, breathing in room air, and receiving full enteral feedings through gavage tube that consist of fortified maternal breastmilk and some formula supplementation. Yesterday the infant had a few large gas­ tric residuals with feeding but otherwise seemed well. You are called to the bedside today because of increased abdomi­ nal distention, apneic spells, poor urine output, lethargy, and hypothermia.

severe o l i g ohyd ra m n ios may develop p u l mona ry hypoplasia owi ng to the rol e of a m n iotic fl u i d in l u n g development. They m a y a l so develop severe contrac­ tu res due to the i n a b i l ity to move with i n the uterus. The conste l l ation of I UG R, p u l m o n a ry hypo plasia, abnormal fa cies ( l ow-set ea rs, m icrog nathia, flattened nose). and l i m b a bnorma l ities i n c l u d i n g c l u b foot a re termed Potter sequence. This is a conseq uence of seve re o l i g ohyd ra m n ios in utero.

Question 1 4-1

This constellation of findings is most concerning for what condition? A) Hirschsprung disease. B) Respiratory viral illness. C) Jejunal atresia. D) Necrotizing enterocolitis. E) Tracheoesophageal fistula.

A mother presents to labor and delivery at 39 weeks' gesta­ tion for a scheduled induction due to poorly controlled gesta­ tional diabetes. She has required insulin since the 29th week and reports that most of her blood glucose readings have been greater than 200 mg/dL over the past 2 weeks. She pro­ gresses in labor after oxytocin induction and vaginally deliv­ ers a single male infant weighing 4500 g.

Discussion 1 4-1

Question 1 5-1

The correct answer is "D:' Necrotizing enterocolitis (NEC) is a disease that is most common in preterm infants who have previously been enterally fed. Early signs of NEC are often systemic signs of illness such as lethargy, apnea, and hypo­ thermia. There are also generally abdominal-specific findings, including increased abdominal distention and high gastric residuals. The disorder may lead to pneumatosis intestina­ lis with gas present below the serosal surface of the bowel or portal venous gas. Advanced NEC may lead to intestinal perforation, which is a surgical condition requiring immedi­ ate assessment and intervention. Laboratory findings includ­ ing hyponatremia, thrombocytopenia, lactic acidosis, and coagulopathy may also be present. Option ''A;' Hirschsprung disease, is not correct; this disorder is caused by absence of ganglion cells, generally in the colon, and often presents with severe constipation, abdominal distention, and possibly toxic megacolon in the newborn. Option "C;' j ejunal atresia, should generally be recognized shortly after birth or prena­ tally by evidence of significant bowel dilation proximal to the atresia, often with emesis, and poor stooling. Option "E;' tra­ cheoesophageal fistula, generally manifests early in life and is characterized by copious oral secretions, respiratory dis­ tress, and inability to easily pass an orogastric tube beyond the esophageal defect.

This infant is at increased risk for which of the following problems in the days following delivery? A) Polycythemia. B) Respiratory distress syndrome of the newborn. C) Hypoglycemia. D) Cardiac defects. E) All of the above. Discussion 1 5-1

The correct answer is "E:' Maternal diabetes mellitus causes a variety of complications that can be minimized when moth­ ers maintain good glycemic control during pregnancy. Infants remain exposed to a hyperinsulin state after delivery and can have profound hypoglycemia requiring frequent monitoring after delivery until stable. Infants of diabetic mothers (IDMs) have higher risks of polycythemia due to relative intrauterine hypoxia. Hyperinsulin states also decrease surfactant pro­ duction and therefore increase the risk of respiratory distress syndrome of the newborn. IDMs are also at increased risk for cardiac defects especially septal wall hypertrophy. These infants should undergo prenatal ultrasound screening for cardiac defects. IDM infants have also been found to have increased rates of iron deficiency and potential neurodevelopmental con­ sequences in iron dependent neuronal processes.

MCG RAW-H I LL E D U CATION S P E C I A LTY BOA R D REVI EW: P E D I ATRICS

296

� QUICKQUIZ What is NOT a cause of neonatal polycythemia? A) Delayed cord clamping. B) Prematurity. C) Twin-to-twin transfusion syndrome. D) Postterm infant. E) Placental insufficiency. Discussion

The correct answer is "B:' Additional causes include LGA, IUGR, trisomy 2 1 , hypothyroidism, Beckwith-Wiedemann syn­ drome, and chronic fetal hypoxia.

You are evaluating an infant in the newborn nursery who was born weighing 5 lb, 2 oz at 39-5/7 weeks' gestational age. The infant has small eyes with drooping upper eyelids, a flat philtrum, micrognathia, thin upper lip, and a short upturned nose. The infant fails the first newborn hearing screen. Question 1 6-1

During pregnancy, the infant was most likely exposed to what substance? A) Selective serotonin reuptake inhibitor (SSRI ). B) Oxycodone. C) Alcohol. D) Cocaine. E) Cigarette smoke. Discussion 1 6-1

The correct answer is "C:' Fetal alcohol syndrome is a spectrum of symptoms involving growth restriction, characteristic facial fea­ tures, and central nervous system problems that eventually may result in developmental delays, attention deficit hyperactivity dis­ order (ADHD), and seizures. Infants exposed to narcotics such as oxycodone can have a typical withdrawal syndrome termed neo­ natal abstinence syndrome, which is characterized by neurologic irritability, poor feeding, gastrointestinal disturbance (diarrhea), and seizures. Infants exposed to stimulants such as cocaine may also experience a withdrawal syndrome. These agents are also associated with poor intrauterine growth and an increased inci­ dence of placental abruption. Cigarette smoking is a major cause of intrauterine growth restriction and also is associated with an increased risk of sudden infant death syndrome (SIDS) postnatally. •



Helpful Tip

I ntrauteri ne g rowth restriction (IUGR) remains a com-

1 1 1 r mon

ca use of small-for-gestational age ( < 1 Oo/o for weig ht) i nfa nts. IUG R is most commonly associated with placental insufficiency. Materna l hypertension is a major ca use of placenta l insufficiency. IUG R ca n a l so be associ­

ated with congenita l infections, chromosome or other feta l anomal ies, and severe maternal malnutrition.

A male infant is born at 24 weeks' gestational age after his mother experiences preterm labor. Following a 4-month hospitalization in the NICU, he is discharged at 39-5/7 weeks' corrected gestational age. The infant is feeding well by mouth on fortified maternal breastmilk. He is discharged home on supplemental oxygen at a rate of 1 /8 L/min. Question 1 7-1

Which statement is true regarding this infant's lung disease? A) This infant has chronic lung disease of prematurity (bron­ chopulmonary dysplasia) confirmed by the need for supple­ mental oxygen beyond 36 weeks' gestational age. B) This infant's lungs were likely in the canalicular stage of development at the time of birth. C) This infant's lungs are likely between saccular and alveolar stages of development at the time of discharge. D) Vitamin A and caffeine are two medications that have been shown to decrease the severity of lung disease of this type. E) All of the above. Discussion 1 7-1

The correct answer is "E:' This infant has chronic lung disease of prematurity, also known as bronchopulmonary dysplasia. This disease is most common in infants at the extremes of via­ bility around 23 to 25 weeks' gestation at birth. It is generally defined as the persistent need for oxygen therapy at 36 weeks' corrected gestational age. The fetal lungs go through expected maturation from lung bud to pseudoglandular to canalicular to saccular to alveolar. Preterm infants lack alveolarization until several months of age, adding to difficulties with ventilation and oxygenation. Both vitamin A and caffeine produce modest improvements in chronic lung disease and are administered to many preterm infants.

An infant is born at 38-2/7 weeks' gestation to a mother with immune thrombocytopenia (ITP) . The mother's platelet count has averaged 25,000/!!L during the pregnancy. She has required platelet transfusions and intravenous immunoglob­ ulin therapy (IVIG) . Her platelet count on the day of delivery is 50,000/!!L. The infant is born by uncomplicated vaginal delivery. On physical exam he is well without bruising, pete­ chiae, or scalp swelling. Question 1 8-1

How should you manage this infant? A) The infant does not require monitoring of his platelet count as he has no signs of cutaneous bleeding on exam. B) If the infant's platelet count immediately after birth is nor­ mal, no additional monitoring is necessary.

C H A PT E R 1 4

C) The infant's platelet count should be monitored closely over the next few days until it is documented to be stable or increasing. D) The infant should receive IVIG even if his platelet count is normal. E) The majority of infants will develop severe thrombocytope­ nia with platelet counts of less than 50,000/f.LL.



FETUS A N D N EWBORN

297

Question 1 9-2

You tell them what? A) Neonatal seizures may be caused by electrolyte disturbances. B) Neonatal seizures may be caused by infection. C) Neonatal seizures may be due to a metabolic disorder. D) Neonatal seizures may be due to intracranial hemorrhage. E) All of the above.

Discussion 1 8-1

Discussion 1 9-2

The correct answer is "C:' Maternal ITP is a cause of neona­ tal thrombocytopenia. Acquired maternal antibodies cause destruction of the infant's platelets. The majority of infants will have safe or normal platelet counts after birth. The maternal platelet count is not predictive of the infant's count and maternal treatment does not decrease the risk of fetal thrombocytopenia. The infant's platelet count will decrease after delivery, reaching a nadir between DOL 2 and 5. The platelet count should be moni­ tored closely after birth until the nadir has been reached. IVIG is given to infants with severe thrombocytopenia. Thrombocy­ topenia is transient but may last for weeks to months, requiring long-term monitoring.

The correct answer is "E:' There are multiple potential etiolo­ gies for neonatal seizures. Age at presentation may be helpful in narrowing the differential diagnosis. For example, seizures from hypoxic ischemic encephalopathy typically present in the first 24 hours of life. The differential diagnosis for a lethargic or comatose infant is very similar to that for an infant with seizure.

A term newborn was born by normal vaginal delivery. The mother's pregnancy was uncomplicated. On DOL 2, the infant is noted to be making mouthing movements and appears to be bicycling his legs. He is transferred to the NICU team for management and evaluation. The movements stop during transfer and he is noted to be lethargic. Question 1 9-1

What is NOT true regarding neonatal seizures? A) Neonatal seizures may present as repetitive rhythmic con­ tractions of the arms and legs. B) Neonatal seizures may present as sustained posturing of one extremity. C) Neonatal seizures may present as sustained eye deviations or roving eye movements. D) Neonatal seizures may present as asymmetric arrhythmic body jerks. E) Neonatal seizures are not associated with changes in vital signs. Discussion 1 9-1

The correct answer is "E:' Neonatal seizures have a wide variety of clinical presentations, including apnea and fluctuations in vital signs. Seizures may cause coma, lethargy, or both. Seizures may be provoked by stimulation. Seizures may be subtle and difficult to differentiate from jitteriness. Infants typically do not have large motor movements (tonic clonic) as may be seen in an older child.

The infant is intubated and medically stabilized. An MRI brain scan and bedside electroencephalogram (EEG) are ordered. The parents arrive at the bedside.

A term male infant is born vaginally. The NICU team is called to the delivery room to assess the infant due to an abnormal­ ity on exam. His right arm is amputated below the elbow with a circumferential depression in the skin. He has normal strength, tone, and movement of the elbow and upper arm. The infant is otherwise well and without dysmorphic features. Question 20-1

What is the cause of his limb amputation? A) Amnionic band sequence. B) Vascular infarct of the distal forearm. C) Genetic syndrome. D) Uterine constraint. E) Birth trauma. Discussion 20-1

The correct answer is "A:' Amnionic band sequence is a group of disorders causing serious structural deformations from early in utero amniotic rupture with bandlike constriction or amputa­ tion. The limbs, digits, and craniofacial structures are generally affected. Abnormalities include limb amputation, digit amputa­ tion, constriction rings (circle of depressed tissue around a part of the body) , encephalocele, and facial clefts.

B I B L I O G RA P H Y

American Academy of Pediatrics Committee on Fetus and Newborn. Controversies concerning vitamin K and the newborn. Pediatrics. 2003; 1 1 2 ( 1 ) : 1 9 1 - 1 92. American Academy of Pediatrics Committee on Fetus and Newborn, American College of Obstetricians and Gyne­ cologists Practice Committee on Obstetrics. The Apgar score. Pediatrics. 2006; 1 1 7(4) : 1444- 1447. American Academy of Pediatrics Subcommitte on Hyperbili­ rubinemia, et al. Management of hyperbilirubinemia in the newborn infant 35 or more weeks of gestation. Pediat­ rics. 2004; 1 14( 1 ) :297- 3 1 6 .

298

MCG RAW-H I LL E D U CATION S P E C I A LTY BOA R D REVI EW: P E D I ATRICS

Anderson MS, Hay WW Intrauterine growth restriction and the small-for-gestational-age infant. In: Avery GB, Fletcher M, MacDonald M, eds. Averys Neonatology: Pathophysiology and Management of the Newborn. 6th ed. Philadelphia, PA: Lippincott Williams and Wilkins; 2005:490-522. Chu A, Hageman JR, Caplan MS. Necrotizing enterocolitis: Predictive markers and preventive strategies. Neoreviews. 20 1 3;4(3) :e l l 3-e120. de Ungria M Daru J. Delivery room management. In: Zaoutis LB, Chiang VW, eds. Comprehensive Pediatric Hospital Medicine. Philadelphia, PA: Mosby; 2007:235-246. Fanaroff A, Martin R. Fanaroff and Martin's Neonatal-Peri­ natal Medicine: Diseases of the Fetus and Infant. 8th ed. Philadelphia, PA: Mosby Elsevier; 2006. Gomella T, Cunningham D, Eyal, F. Neonatology: Management, Procedures, On Call Problems, Diseases and Drugs. 7th ed. New York, NY: McGraw-Hill; 20 1 3 . Hormann MD. Birth injury. In: Zaoutis LB, Chiang VW, eds. Comprehensive Pedatric Hospital Medicine. Philadelphia, PA: Mosby; 2007:255-267. Kattwinkel J, Bloom R. Textbook of Neonatal Resuscitation. 6th ed. Elk Grove Village, IL: American Academy of Pediatrics; 201 1 . Keren R . Neonatal hyperbilirubinemia. In: Zaoutis LB, Chiang VW, eds. Comprehensive Pediatric Hospital Medicine. Philadelphia, PA: Mosby; 2007:279-285. Lightdale JR, Gremse DA; Section on Gastroenterology, Hepatology, and Nutrition. Gastroesophageal reflux:

Management guidance for the pediatrician. Pediatrics. 20 1 3; 1 3 1 (5):e1 684-e1695. Madejczyk K. The well newborn. In: Zaoutis LB, Chiang VW, eds. Comprehensive Pediatric Hospital Medicine. Philadelphia, PA: Mosby; 2007:247-254. Mirkinson LJ. Hypoglycemia and infants of diabetic moth­ ers. In: Zaoutis LB, Chiang VW, eds. Comprehensive Pediatric Hospital Medicine. Philadelphia, PA: Mosby; 2007:274-278. Muller AJ, Marks JD. Hypoxic-ischemic brain injury: Poten­ tial therapeutic interventions for the future. Neoreviews. 2 0 14; 1 5(5):el 77-e 1 86. Newburger JW, Takahashi M, Gerber MA, et al. Diagnosis, treatment, and long-term management of Kawasaki disease: A statement for health professionals from the Committee on Rheumatic Fever, Endocarditis and Kawasaki Disease, Council on Cardiovascular Disease in the Young, American Heart Association. Circulation. 2004; 1 1 0 ( 1 7) :2747-277 1 . Oh W, Blackman LR, Escobedo M , et al. Use and abuse of the Apgar score. Pediatrics. 1 996;98 ( 1 ) : 14 1 - 142. Teramo K. Diabetic pregnancy and fetal consequences. Neoreviews. 2014; 1 5(3) :e83-e90. Thebaud B. Chronic lung disease in the neonate: Past, present, and future. Neoreviews. 2 0 1 3 ; 14(5) :e252-e258. Warren JB, Phillipi CA. Care of the well newborn. Pediatr Rev. 2012;33 ( 1 ) :4- 1 8 .

Fl u id a nd El ectrolyte Meta bol ism

15

J eff Va n B l a rcom

A previously healthy 1 5-month-old girl presents to the emergency department for evaluation for a 3-day history of persistent vomiting and diarrhea. Neither the stool nor the emesis has been bloody, and the emesis has not been bil­ ious. She has had little to drink or eat in the last 24 hours. She has been febrile intermittently and her temperature in triage was 38.6°C ( 1 0 1 SF ) . Physically the child appears tired and pale and she has little response to examination. Her heart rate is 1 40 beats per minute (bpm) , her respi­ ratory rate is 28 breaths per minute, and her blood pres­ sure is 95/58 mm Hg. Her peripheral pulses are somewhat difficult to find, her peripheral capillary refill is about 4 seconds, and she has a systolic ejection murmur. Her liver is not enlarged. Question 1 -1

Which answer best describes the status of this child's cardio­ vascular system? A) Septic shock. B) Cardiogenic shock. C) Compensated hypovolemic shock. D) Uncompensated hypovolemic shock. E) Distributive shock.

absence or presence of normal blood pressure. Uncompen­ sated shock is associated with hypotension. (See Table 1 5-2.) Though our patient is tachycardic and has poor capillary refill, her blood pressure is appropriate for a child of her age, so she has compensated hypovolemic shock. Her fever is likely con­ tributing to her tachycardia, but fever will not contribute to a lengthened capillary refill so it is best to attribute most of her tachycardia to hypovolemia. (See Table 1 5- 3 . ) The child's murmur is most likely caused by a hyperdynamic cardiovas­ cular state related to her fever, and she has no liver enlarge­ ment or past medical history that would suggest significant heart disease, so cardiogenic shock is not likely. Septic shock, which is possible in this child, is a form of distributive shock caused by toxins released by infectious organisms. Differenti­ ating hypovolemic shock from septic shock in an ill-appear­ ing, febrile child can be difficult. It is best to have a high index of suspicion for septic shock, but in this case the child's his­ tory is more consistent with hypovolemic shock, which is the most common type of shock seen in children. It is important to keep in mind that several forms of shock can coexist in a patient, although thankfully only on rare occasions. Rem em­ ber, cardiac output is influenced by preload (blood coming back to heart), afterload (resistance to blood being pumped out of the heart) , and contractility (pumping power of the heart) . In shock, one or more of these factors is deranged.

Discussion 1 -1

The correct answer is "C." Shock is defined as a state in which decreased tissue perfusion leads to inadequate delivery of substrate to meet metabolic demands. The tissues are not get­ ting enough oxygen ! Shock has many etiologies and can be divided into three main categories depending on the underly­ ing pathophysiologic process: cardiogenic shock, distributive shock, and hypovolemic shock. (See Table 1 5 - 1 . ) Given this child's history of gastrointestinal volume losses and decreased input, it is likely that she is hypovolemic. Whether or not shock is compensated or uncompensated depends on the



Helpful Tip

�A I

febri l e, tachyca rd ic c h i l d is most l i kely not septic, but

1 1 r assu m i n g that the c h i l d is septic until proven otherwise

is best. The i n itia l i nterventions for septic shock a re rea d i ly ava i l a b l e i n most U.S. hospita l settings: i ntravenous (IV) fl uids a n d a nti biotics.

An IV catheter is placed and blood is sent for laboratory studies.

299

300

MCG RAW-H I LL E D U CATION S P E C I A LTY BOA R D REVI EW: P E D I ATRICS

TABLE 1 5- 1 CAT E G O R I E S O F SHOCK

Hypovo l e m i c

Pathophysiology

Ca u ses

Signs a n d Sym ptom s

Decreased preload:

De hyd ration

Tachyca rd i a

1 . I nadeq u ate i nta ke 2. I ncreased losses 3 . Acute blood loss 4. I m pa i red a b i l ity to co ncen­ trate the u r i n e

Hemorrhage

Decreased u r i n e output'

Dia betes i n s i p i d u s

Prolonged ca p i l l a ry refi l l

B u rns

Wea k p u l ses

Hyperg lyce m i a

Cool extre m ities

Vo m i t i n g

Dry m uco u s m e m branes

Dia rrhea

S u n ke n fo nta n e l l e

Ad re n a l i n s ufficiency

Altered m e ntal status

Hemorrhage Card ioge n i c

Decreased contract i l ity of the heart

Myoca rd itis

Heart fa i l u re

Arrhyth m i a

Tac hyca rd ia

Ca rd i o myopathy

Decreased u ri n e output

Co ngen ital heart d i sease

Wea k p u l ses

Obstructi o n (pericard i a ! tam­ ponade, te nsion p n e u m otho­ rax, p u l m o n a ry e m b o l i s m )

Cool extre m ities

I nfa rction or i s c h e m i a

Prolonged ca p i l l a ry refi l l He patomegaly JVD P u l m o n a ry edema

Distri butive

Decreased syste m i c vasc u l a r resi sta nce

Seps i s

Tac hyca rd ia

S p i n a l co rd i nj u ry

Decreased u ri n e output

1 . Va sod i l ation 2. Ca p i l l a ry leak 3 . Th i rd spaci ng of fl u i d s i nto i nterstitial tissues

Head i nj u ry

Pe ri phera l edema

A n a p hylaxis

Brisk ca p i l l a ry refil l b

Drugs

Bou n d i n g p u l sesb

JVD, jugular venous d i stention. 'Typica l ly u ri n e output is decreased u n less cause is from kidney dysfu nction. bC haracteristics of early d i stributive shock. Question 1 -2

TABLE 1 5-2 HYPOTE N S I O N BY AG E A N D SYSTO L I C B LOOD P R E S S U R E

Acco rd i ng t o the American Heart Association Ped iatric Adva n ced Life S u p po rt criteria, hypotension i s a systol i c blood press u re l e s s than the 5 t h perce nti l e o f n o r m a l for age, na mely:

The next best intervention for this child would be to: A) Administer a 1 g dose of iV ceftriaxone. B) Administer a 10 mL/kg of body weight IV bolus of normal saline (NS). C) Administer a 20 mL/kg IV bolus of D 5 '\4 NS. D) Administer a 20 mL/kg IV bolus of NS. E) Check the child's fingerstick blood glucose.

TABLE 1 5-3 CAU S E S O F TACHYC A R D I A IN A C H I L D

< 60 m m Hg i n term neonates (0 to 28 d ays)

( I N O R D E R O F P R EVA L E N C E )

< 70 mm Hg in i nfa nts (1 month to 1 2 months)

1 . Anxiety 2. Pa i n 3 . Fever 4. Dehyd ratio n/hypovo l e m i a 5. Anemia 6. Sepsis 7. Arrhyth m i a

< 70 m m Hg + (2 x Age i n yea rs) i n c h i l d re n 1 to 1 0 yea rs of age < 90 m m Hg i n c h i l d ren 1 0 yea rs of age or older

C H A PT E R 1 5



F LU I D A N D E LECTROLYTE M ETABOLI S M

301

Discussion 1 -2

Discussion 1 -3

The correct answer is "D." Giving a 20 mL/kg NS bolus as rapidly as possible is a time-tested intervention that is the initial treatment for any child in shock. This applies to chil­ dren up to a weight of 50 kg (approximately the weight of an average 1 4-year- old ), over which the initial intervention would be 1 L of NS given as rapidly as possible. If the child were in septic shock, giving a dose of ceftriaxone as rapidly as possible would be an appropriate intervention, though the best first step would be to give IV fluid. As previously mentioned, it is more likely that this child has hypovole­ mic shock rather than septic shock. A 10 mL/kg NS bolus would not be inappropriate for this child, though there is no indication that a 20 mL/kg bolus would not be tolerated. A 1 0 mL/kg bolus is considered to be the initial intervention for a child in shock with a cardiac problem, the rationale being to avoid a worsening of congestive heart failure with a larger fluid bolus. Giving a bolus of either a hypertonic or hypotonic solution such as 'h NS is generally felt not to be a good idea, given the possibility of a rapid change in the serum sodium, leading to neurologic issues. Determina­ tion of the child's blood glucose level is rarely a bad idea but would not be the initial focus of the child's care in this case. Lactated Ringer's solution can also be given as a bolus in a resuscitation situation.

The correct answer is "B:' An acute change in weight is the best determinant of a person's current hydration status; indeed, our estimates of dehydration are reported in percentages of weight loss. However, previous weights are not often available and are not necessary for decision making in resuscitation scenarios such as in this vignette. If a child just so happens to have had a recent visit for medical care and if the weight measurements were obtained on the same scale, the level of the child's dehydration can be deter­ mined accurately. Urine output is a good indicator of hydration status and is used in virtually all inpatient settings, although it is not as reliable a measurement as an acute change in weight and at times can be misleading, such as in a dehydrated child who has diabetic ketoacidosis with an osmotic diuresis. A child's oral input is subject to the interpretation of an anxious parent or frequently is unknown, and thus is a poor indicator of a child's level of hydra­ tion. Clinical signs of dehydration may be the only available indi­ cators of a child's hydration status but can be subjective, subtle, or both. If you have time to debate the concavity of an infant's fon­ tanelle, you may not need to act with urgency. (See Table 1 5-4.) •

� 1

1 1r

Helpful Tip

The fl uid deficit of a dehyd rated child can be ca lcu lated u s i n g body weig ht as follows: Fluid deficit (in liters) = Basel ine body weight (in kg) Current dehydrated body weight

Question 1 -3

Which of the following is the best acute indicator of a child's hydration status? A) Urine output. B) Acute change in weight. C) Acute change in mental status. D) Clinical signs such as sunken eyes and dry mucous membranes. E) Input.

(in kg)



� 1

Helpful Tip

Each 1 o/o of dehyd ration is equ ivalent to a fl uid deficit

1 1r o f 1 0 m L/kg . For exa m ple, a n i nfa nt w h o is estimated

c l i n ica l ly to be 6% dehyd rated has a fl u i d deficit of

60 m L/kg that needs to be replaced .

TABLE 1 5-4 U S I N G C L I N I C A L S I G N S TO EST I M ATE T H E D E G R E E OF D E HYD RAT I O N

Mild

Moderate

Severe

Child

3% (30 m l/kg)

6% (60 m l/kg)

9% (90 m l/kg)

I nfa nt

5% (50 m l/kg)

1 Oo/o (1 00 m l/kg)

1 5% ( 1 5 0 m l/kg)

Ski n

Normal

Dry

Cool

M u co u s mem bra nes

Tacky

Dry

Parched

Eyes

Normal

Deep set

S u n ke n

Tea rs

Present

Red uced

None

% Dehyd ration ( fl u i d deficit)

Physical Findings

Fonta n e l l e

Flat

Soft

S u n ke n

Me nta l status

Ti red

I rrita ble

Letharg i c/obtu nded

Pu lses

Normal

Wea k

Fa i nt/i m pa l pa b l e

Capi l l a ry refi l l

Normal

2-3 seco n d s

> 3 seconds

U ri n e output

Norma l/decreased

Decreased

None

302

MCG RAW-H I LL E D U CATION S P E C I A LTY BOA R D REVI EW: P E D I ATRICS

Question 1 -4

Which of the following statements is FALSE? A) Dry mucous membranes are consistent with at least 5% dehydration. B) Sunken eyes are consistent with at least 5% to 10% dehydration. C) Deterioration in mental status is consistent with at least greater than 10% dehydration. D) A sunken fontanelle is consistent with at least 5% to 10% dehydration. E) Increases in blood urea nitrogen (BUN) can be used to accurately estimate the severity of dehydration. Discussion 1 -4

The correct answer is "E:' There are too many variables for a direct relationship between BUN and hydration level to be established, although it is true that elevations in BUN are con­ sistent with hypovolemia in most routine cases. Conversely, the lack of an increased BUN should not deter intervention if you believe a child is dehydrated (hypovolemic from water loss, to be more specific) . These are by no means all of the clinical signs used to determine a patient's hydration status. Heart rate, for instance, is used to gauge a child's response to rehydration, and therefore can be used as an indicator of hydration status, although tachycardia has a number of confounding causes. (See Table 1 5-3, earlier.) There obviously is some leeway in the esti­ mation of dehydration based on physical findings, but generally some history is provided when dealing with a patient. Always use all the available information to make an informed deci­ sion about what, if any, intervention needs to be undertaken. (See Table 1 5-4, earlier.)

A) B) C) D) E)

1:1. 2: 1 . 5: 1 . 10: 1 . 4:3.

Discussion 1 -6

The correct answer is "B:' Ratio aside, the usual percentages of intracellular volume and extracellular volume in the human body are facts you are just supposed to know. In a healthy, nor­ mal subject, intracellular fluid volume represents 67% of total body water; extracellular fluid volume, which includes intravas­ cular fluid and interstitial fluid, represents 3 3 % of total body water: 2: 1 . The major extracellular cation is sodium (Na+) and the major intracellular cation is potassium (K+), but there is no osmotic difference between the two spaces. You will recall that "third-spacing" represents fluid movement within the body from the two useful spaces (the intracellular and extracellular compartments) to a not-so-useful space (the transcellular com­ partment (ie, the peritoneal space in ascites, the subdural space in subdural hemorrhage) , which is as good as volume loss. Question 1 -7

What is the circulating blood volume of a typical 6-year-old child? A) 75 mL/kg. B) 50 mL/kg. C) 65% of body weight. D) 10 pounds (lb) . E) 1 00 mL/kg.

Question 1 -5

Discussion 1 -7

What is the best estimate of this child's percentage of dehy­ dration based on her clinical signs? A) 9%. B) 5%. C) 10%. D) 6%. E) 1 5%.

The correct answer is "A." The normal amount of circulat­ ing blood volume in a child varies with age and is as high as 92 mL/kg in a newborn, as low as 71 mL/kg in an 1 8 -year­ old (and even lower in adulthood) . An acute drop from "normal" to 50 mL/kg would likely create a state of uncom­ pensated shock, a point which is typically reached when a child loses 30% or so of intravascular volume. The history of the 20 mL/kg bolus is obscure, but it sounds reasonable given that it is in the neighborhood of restoring a symptomatically dehydrated child's intravascular volume back to normal, if only momentarily. Sixty-five percent of total body weight is a good estimate of a 6 -year-old's total body water, but not of his or her circulating blood volume. Remember, two thirds of the body is water. As for option D - 1 0 pounds-what kind of answer is that?

Discussion 1 -5

The correct answer is "A:' Our patient is 1 5 months old (not an infant) and estimated to be 9% dehydrated based on her capil­ lary refill of 4 seconds, weak pulses, and lethargy.

After receiving the fluid bolus, the patient's clinical status remains the same, although her capillary refill is marginally better. Her latest blood pressure measure is 90/45 mm Hg. A fingerstick blood glucose reading is reported as 78 mg/dL, and some of her laboratory studies are available. Most nota­ bly, sodium is 140 mEq/L, potassium 5.0 mEq/L, bicarbonate 16 mEq/L, creatinine 0.39 mg/dL, BUN 26 mg/dL, and lactate 4.2 mEq/L. Question 1 -6

What is the normal ratio of intracellular volume to extracel­ lular volume?

Question 1 -8

What is the next step in this patient's management? A) Discuss her case with the intensivist on call at the nearest children's hospital. B) Initiate an epinephrine drip. C) Obtain an electrocardiogram (ECG) . D) Administer a bolus of normal saline with 1 0 mEq/L of potassium bicarbonate. E) Administer another 20 mL/kg bolus of normal saline.

C H A PT E R 1 5

Discussion 1 -8

The correct answer is "E:' If you want some hand-holding and possibly a good brow-beating, go ahead and call your local intensivist, who will inform you in some fashion that the child does not, at least up to this point, meet admission criteria for the intensive care unit, nebulous though they may be. The child's blood pressure appears to have decreased some but is still ade­ quate for her age, so an epinephrine drip would not be indi­ cated. The difference in the blood pressure measurements is not likely to be clinically significant and may depend on when, and the manner in which, they were obtained. The pulse pressure (difference between systolic and diastolic pressures) has wid­ ened some, which is more common with distributive shock such as occurs with sepsis or anaphylaxis, but not enough to either be alarming or to be an indication for vasopressor medication. A good rule of thumb for pulse pressure is that it should generally be around 40 mm Hg, corresponding to a "normal" adult blood pressure of 120/80 mm Hg. The child's low bicarbonate level is consistent with her history of diarrhea, but it is not low enough to warrant the administration of bicarbonate. Whether or not the bicarbonate level is followed as a marker of the patient's response to therapy is debatable. Obtaining an ECG is not likely to be fruitful; her potassium level is elevated but this should first prompt a further examination of the lab report or a call to the lab itself to determine if there was any hemolysis on the sample. Giving another normal saline bolus is the most appropriate next step. And the next step after that, as well, assuming that her clin­ ical status remains the same. If there is no response thereafter, or if the child is deteriorating despite the administration of fluid, you may well want to discuss the case with an intensivist, this time without fear of a snide response. •



1 1 1r

Helpful Tip

The h istory of u ndertreating patients i n need of fl u i d resu scitation g oes a l l t h e way back t o the beg i n n ings of IV fl u i d therapy i n the early 1 800s, which was i n itia l ly developed for the treatment of chol era. The end points of fl u i d a d m i n istration a re c l i n ical i m p rovement, which is most l i kely, or pul monary a n d peri pheral edema, which i n a person with normal kid neys is ra rely l ife



F LU I D A N D E LECTROLYTE M ETABOLI S M

303

following is a potentially disastrous consequence of correct­ ing a patient's hyponatremia too quickly? A) Stroke. B) Osmotic demyelination syndrome. C) Cardiac dysrhythmia. D) Cerebral edema. E) Seizure. Discussion 1 -9

The correct answer is "B." Purists will point out that severe hyponatremia necessitating 3% saline is uncommon in the setting of hypovolemic dehydration from diarrhea. Nonethe­ less, osmotic demyelination syndrome (ODS) would be pos­ sible in patient with a sodium level of 1 1 5 mEq/L, so slow correction would be warranted. If that level of hyponatremia had existed in the child for less than 48 hours, rapid correc­ tion leading to an occurrence of ODS would not be likely, but generally we cannot know how long a patient's hyponatremia has existed prior to discovery. The brain osmotically adapts to chronic hyponatremia and rapid correction can lead to cellular dehydration and disruption of the blood-brain bar­ rier, which allows immune system factors to damage nerve cells (famously in the pons, as in pontine) , or so the theory goes. When hypernatremia is corrected too rapidly, osmotic flow of water into solute-heavy cells can eventually lead to cerebral edema, which is more than just a headache. Sei­ zures can occur from severe hyponatremia by itself but are not commonly seen in ODS or cerebral edema per se, which both are consequences of overly rapid sodium correction. ODS can produce irreversible neurologic symptoms (usu­ ally presenting several days after the correction of the serum sodium level) and cerebral edema can lead to death, so they are best avoided. In this child's case, a rapidly infused 20 mL! kg NS bolus given in the emergency department may well have rectified or lessened the degree of an existing hypona­ tremia and we may be none the wiser. If you answered "A;' you were on the right track with the idea that too-rapid correction of hyponatremia leads to a neurologic problem, but ultimately you were wrong. If you answered "C:' you were simply incorrect.

th reate n i n g . If a c h i l d i n compensated shock proceeds to u ncom pensated shock (ie, becomes hypotensive), you rea l ly ought to ca l l an i ntensivist, but in the meantime, g ive more crysta l loid.

The child's appearance improves and her mother states that she "has her color back:' Her heart rate has decreased to 1 00 bpm, though it rapidly increases to 140 bpm as she scrambles up her mother's torso whenever you approach. She appears more vigorous but seems to have no interest in drinking or eating. She urinates. You admit her for further treatment. Question 1 -9

Suppose that instead of 140 mEq/L, her sodium level at the time of initial evaluation was 1 1 5 mEq/L. Which of the

A 5-day-old infant boy presents to the emergency depart­ ment 30 minutes after what his mother describes as a seizure. She states that he abruptly stiffened for a few sec­ onds, then "his eyes rolled back:' This was followed by a period of sleepiness from which he seems to be recovering. He was born at term, there were no pregnancy or delivery complications, and he has done well since birth. On physical examination, he weighs 4. 1 kg and appears to be a healthy infant. He is irritable but consolable. Shortly after an IV catheter is placed and his blood is sent for laboratory stud­ ies, he begins to seize. He is given a 0.4 mg IV dose of loraz­ epam, to no avail. The laboratory technician calls with a

304

MCG RAW-H I LL E D U CATION S P E C I A LTY BOA R D REVI EW: P E D I ATRICS

critical value: his calcium level is 5.2 mg/dL. There are no other critical values. He continues to seize.

TABLE 1 5-6 C L I N I C A L M A N I F ESTAT I O N S O F

Question 2-1

Neuromuscular Man ifestations (teta ny)

What is the next most appropriate step in this child's management? A) Give another 0.4 mg IV lorazepam. B) Give a 20 mg/kg IV loading dose of fosphenytoin. C) Give a dose of IV calcium gluconate. D) Repeat a blood draw for an ionized calcium measurement. E) Give a dose of iV calcium chloride.

Wea kness

Discussion 2-1

The correct answer is "C' Although giving another dose of lorazepam or a loading dose of fosphenytoin are appropri­ ate therapies for a seizing child, this infant's seizure is most likely caused by hypocalcemia. The treatment of choice for a newborn with symptomatic hypocalcemia (seizing, irritable, or both) is 1 00 mg/kg of IV calcium gluconate given over a 1 0 - to 20-minute period. Seizures resulting from hypocal­ cemia are unlikely to stop until the serum calcium level has been normalized. IV calcium chloride would be appropriate if it was more readily available than calcium gluconate, but calcium chloride is less desirable given a greater propensity for venous and local tissue destruction. The ionized calcium level is a more accurate measure of the physiologically active form of calcium in the serum, but given a markedly low cal­ cium level in an actively seizing newborn, the next best step in management would be to administer calcium intrave­ nously in an attempt to stop the seizure. Confirming that the infant's calcium level is indeed low is necessary, but given the presented information it would be reasonable to assume that his measured calcium level was accurate and to proceed accordingly. If the infant was not symptomatic, an ionized calcium measurement would be the next step. Do not forget to investigate why the infant is hypocalcemic after you stop the seizure. (See Table 1 5- 5 . ) Acute-onset hypocalcemia can cause seizures, as in this infant, along with a range of clinical manifestations. (See Table 1 5-6.)

ACUTE-O N S ET HYPOCALC E M I A

Pa resthesias, parti c u l a rly periora l and i n the extre m ities Seizures I nvo l u nta ry m u scle contractions (twitc h i ng, carpa l spasm, l a ryngospasm) I nd uced fi n d i ngs: Trou sseau s i g n : Carpa l spasm fo l l owi ng 3 m i n utes of arm ischemia i n d uced by i nflation of a bl ood pressu re cuff C hvoste k s i g n : Facial m u scle contraction i n d uced by m ec h a n i c a l sti m u lation of the fac i a l ne rve Cardiac Man ifestations

Prolonged QT i nte rva l Arrhyt h m i a Hypotension Heart fa i l u re





Helpful Tip Cal c i u m infusion pea rls:

The c h i l d 's card i ovascu l a r

1 1 1r status must b e closely mon itored d u ri ng I V ca lcium

infusion given the possibil ity of cardiac a rrhyth mias

related to the changing seru m ca lcium level. The infusion should be given slowly, over 10 to 20 minutes. Extravasation of ca lcium-containing fl uids is both menta l ly and physica l ly traumatic, so close attention must be paid to the IV site. Treatment of such extravasations usually involves su bcuta neous or intradermal administration of hya l u ronidase to limit the destruction and consultation with plastic surgery.

Question 2-2

TABLE 1 5-5 CAU S E S O F HYPOCALC E M I A A N D HYPERCALC E M I A

Hypoca lcemia

Hyperca lcemia

Vita m i n D deficiency

Hype rvita m i nosis D

Hypopa rathyro i d i s m

Hyperpa rathyro i d i s m

Maternal hyperpa rathyro i d i s m

M a l i g n a ncy

Hyperphosp hatem i a

Thiazide d i u retics

Hypomag nese m i a

Su bcuta neous fat necrosis

Pa ncreatitis

Wi l l i a m s synd ro m e

Drugs

G ra n u lo mato u s d i sease

Ethylene g lyco l i ngestion

Hyperthyro i d i s m

Which of the following is most likely to cause a patient's serum calcium measurement to appear falsely low? A) Hypoalbuminemia. B) Hyperalbuminemia. C) Acidosis. D) Hypermagnesemia. E) Alkalosis. Discussion 2-2

The correct answer is "A:' About 40% of serum calcium is bound to protein, mostly to albumin. A low level of albumin in the serum will cause the routinely measured serum calcium level to be low but does not affect the ionized calcium level, which is the hormonally regulated form of calcium. A high level of serum albumin, which is usually only seen in children in the setting of severe dehydration, would have the opposite effect. Serum pH

C H A PT E R 1 5

has an effect on the binding of calcium to albumin (with alka­ losis promoting binding and acidosis decreasing it) , so changes in pH affect the ionized calcium level but do not affect the mea­ sured serum calcium level. A low level of serum magnesium can be associated with hypocalcemia but would not cause the serum calcium measurement to be falsely low. Whenever there is a question about a patient's calcium level, an ionized calcium level should be obtained.

A mother brings her 2¥2 -week-old daughter for a routine well-child check. Looking at the infant's chart, you see that her heart rate and respiratory rate seem to be normal for her age, and her weight is 3. 7 kg. The mother states that the infant is generally doing well, but when you ask about the feeding schedule, she says that the infant has been intermittently vomiting for about a week. The mother has not been par­ ticularly concerned about this vomiting because the infant's older brother had reflux when he was an infant, which was messy but did not require any intervention. She notes that her daughter has been breastfeeding readily and most often vomits within a few minutes after finishing a feed, although on occasion the vomiting occurs an hour or so after feeding. The vomit is not bloody or bilious, but occasionally is force­ ful. The infant has been stooling with less frequency of late. Physically the infant appears well. The mother reports that birth weight was 8 pounds, 1 1 ounces. The infant's physical examination is unrevealing.



F LU I D A N D E LECTROLYTE M ETABOLI S M

305

but should not drive an evaluation or the lack of an evaluation. Ordering a chemistry panel might put you on the right track, but why not go directly to a more definitive test if you think you know what the problem is? A normal chemistry pattern would not rule out pyloric stenosis, but a normal ultrasound would. Similarly, an abdominal radiograph would not be helpful, unless you were concerned that a child had a lower obstruction such as duodenal atresia or malrotation with a volvulus, but this infant has no history of bilious vomiting. Pyloric stenosis is much more common as well. A pH probe is usually a hard sell for a gastroenterologist and would not likely be useful in this case, even if the infant did indeed have gastroesophageal reflux, which in many cases need not be obj ectively substantiated. She could have gastroesophageal reflux, but this history warrants further investigation. You may not be asked to convert a weight from pounds to kilo­ grams on the pediatric board certification test, but as a pedia­ trician, you will need to do it on a regular basis as long as the English system of measurement persists.

An abdominal ultrasound is obtained and findings are consistent with pyloric stenosis. Question 3-2

If a chemistry panel were obtained in this dehydrated infant with pyloric stenosis, what pattern would be expected? A) Hypochloremic metabolic alkalosis. B) Mixed respiratory and metabolic acidosis. C) Hyperchloremic metabolic acidosis. D) Hyperkalemia. E) Hypernatremia.

Question 3-1

Discussion 3-2

As the child's pediatrician, you have determined your level of concern with this history. What should you do next? A) Order a chemistry panel. B) Order an abdominal radiograph. C) Reassure the mother that the infant does indeed have reflux and that no intervention is necessary. D) Order a pH probe. E) Order an abdominal ultrasound.

The correct answer is ''A:' Although this is a situation nota­ bly associated with pyloric stenosis, persistent vomiting of just about any cause can lead to similar laboratory findings. The main electrolytes lost with vomiting are hydrogen and chloride, as in hydrochloric acid, a product of the stomach essential in the initiation of digestion. Loss of these two sub­ stances leads to a hypochloremic metabolic alkalosis. Because of the dehydration, acidosis can also be seen, particularly if the vomiting is rapidly progressive. A hyperchloremic metabolic acidosis can be seen in the setting of diarrhea, given a loss of bicarbonate and other anions in the stool, the hyperchloremia caused by a reaction of the kidneys to the loss of serum anions by retaining chloride. A hyperchloremic metabolic acidosis without a history consistent with bicarbonate loss (most com­ monly diarrhea) or without a history of exogenous acid input should prompt an investigation for a renal tubular acidosis. By the way, chloride is a player but rarely if ever has the cen­ tral role in electrolyte disturbances. Hyperkalemia is not often encountered in a child with diarrhea or vomiting unless the blood draw was difficult, in which case the child's potassium level is unlikely to be truly elevated. Hypernatremia certainly can be seen in a dehydrated child, but is not the most fre­ quently encountered metabolic derangement pattern seen in a child with pyloric stenosis.

Discussion 3-1

The best answer is "E:' The infant does not fit the traditional demographic, but this vignette should raise a concern for pyloric stenosis. She is vomiting and remains below her birth­ weight more than 2 weeks later, which together are both­ ersome. A lack of physical examination findings does not exclude pyloric stenosis; it would be hard to be absolutely cer­ tain that the infant's examination was normal in that regard. Pyloric stenosis is indeed more frequently seen in firstborn males, is frequently associated with proj ectile vomiting, and presents more frequently at about 1 month of age, but this does not exclude that possibility in this infant. "Proj ectile" vomiting tends not to be a particularly useful piece of history in many situations given a parental fondness for using the word; its presence or absence should be taken into account

306

MCG RAW-H I LL E D U CATION S P E C I A LTY BOA R D REVI EW: P E D I ATRICS

Back to our patient: Blood for lab studies is drawn, confirming that she does indeed have a hypochloremic metabolic alkalo­ sis. Incidentally, her sodium level is 145 mEqiL, potassium is 4.5 mEqiL, and blood glucose is 62 mg!dL. Her potassium level is normal, but hypokalemia is common in pyloric stenosis. She needs a pyloromyotomy, but first she needs to be hydrated. Question 3-3

What fluid is most appropriate for this infant's preoperative hydration needs? A) D 5 1lz NS with 20 mEq KCl per liter. B) D 1 0 lJt NS with 40 mEq KCl per liter. C) 1lz NS with 20 mEq K acetate per liter. D) D 5 1lz NS with 40 mEq KCl per liter. E) Lactated Ringer's. Discussion 3-3

The correct answer is "D:' The choice may be somewhat debat­ able, but the principles are as follows: ( 1 ) She has not been eating well and should be NPO (nil per os, nothing by mouth) prior to her operation; therefore, she needs some glucose in the fluid to keep her brain happy; the choice between D 5 and D 1 0 is not straightforward, but her blood glucose level is normal so she probably does not need D! O ' (2) Although her potassium is normal, children with persistent vomiting from pyloric stenosis usually have a total body potassium deficit, so giving more potas­ sium than the typical maintenance amount (20 mEq KCl!L given at a maintenance rate) would be appropriate. (3) The choice of the NaCl content in the fluid is also not a simple determination, but 1lz NS is typical in this situation; lJt NS has been associated with problematic iatrogenic hyponatremia in hospitalized children and probably should not be used. On the other hand, hyperna­ tremia is more likely when a child with ongoing free water loss is not given an adequate amount of fluid than when an adequate amount of fluid with a higher NaCl content is given, so giving 1lz NS and even normal saline is safe. In any case, a high index of suspicion for an electrolyte problem in a hospitalized child who is NPO will likely keep you out of trouble. If the infant appears dehydrated, a 20 mL/kg NS bolus would be appropriate, followed by D5 1lz NS with 40 mEq KCl per liter. The rationale for not giving a base such as acetate or bicarbonate to a child with an alkalosis should be evident. Question 3-4

Speaking of IV fluid, what are the components of lactated Ringer's solution? A) Na+ 1 54 mEqiL, K+ 4 mEqiL, Cl- 1 58 mEqiL, lactate 3 giL. B) Na+ 1 30, K+ 4, Ca2 + 2.7 mEqiL, Cl- 109, lactate 28 mEqiL. C) Na+ 0, K+ 0, dextrose 50 giL. D) Na+ 1 54, K+ 0, Cl- 1 54, dextrose 50. E) Na+ 1 30, K+ 4, Cl- 109, lactate 28. Discussion 3-4

The correct answer is "B:' Lactated Ringer's does indeed have lactate, calcium, and potassium. Option "C" is D 5W (W = water) , option "D" represents the components o f D5NS, option ''!\.' will

not routinely be found in a hospital pharmacy, and option "E" is not possible without involving another anion, in keeping with the law of electroneutrality.

A 6-month-old infant girl is brought to clinic for evalua­ tion after 3 days of progressively worsening vomiting and diarrhea. The vomiting has been increasing to the point that the infant has had very little input today, according to her mother. Her input has consisted of small amounts of formula intermittently. Her mother is not sure of her urine output because of the large amount of stool that the infant has been having. Neither the emesis nor the stool has been bloody, and the emesis has not been bilious. She last vomited about 30 minutes before your exam, and her last stool was shortly thereafter. There are no other bothersome aspects of the infant's present or past medical history. She appears about as tired as her mother but responds to examination as expected for her age; her eyes do not appear to be sunken and her oral mucosa is at worst a little tacky. Her capillary refill is less than 2 seconds. She is holding a bottle with formula in it. She is afebrile, her heart rate is 1 00 bpm, respiratory rate is about 30 breaths per minute, and blood pressure is 95165 mm Hg. Question 4-1

What should be done to help this infant? A) Begin a trial of a clear liquid such as Pedialyte. B) Place an IV catheter, order lab studies, and give a normal saline bolus. C) Begin a trial of formula. D) Place an intraosseous line and give a normal saline bolus. E) Give a dose of loperamide. Discussion 4-1

The correct answer is "A:' This is a straightforward case of mild dehydration in a generally healthy infant, of the type that a gen­ eral pediatrician will see a lot. There certainly are different ways to approach the situation, but in this case a trial of a clear liquid would be the most appropriate. The things that the stomach does not like when it is ailing are mainly ( 1) anything in large volume, ( 2 ) solids, and ( 3 ) anything fatty, including milk or formula; so we should avoid those. She is not more than 5% dehydrated accord­ ing to our previously discussed clinical findings for dehydra­ tion, so drastic rehydration measures such as intraosseous fluid and even IV fluid are probably not necessary at this point. What exactly constitutes a large volume is debatable; perhaps starting with a dropperful or an ounce of Pedialyte, depending on the size of the infant, and seeing what happens would be the most appro­ priate first step. Antidiarrheal drugs such as loperamide are not recommended in routine cases such as this because they are not curative, they can be sedating (not good for a child who needs to drink), and they can, at least in theory, delay the expulsion of the diarrhea-causing agent. You can temporize the infant's diar­ rhea momentarily, but what's going to come out is going to come

C H A PT E R 1 5

out. Placing a nasogastric (NG) tube and slowly administering a liquid such as Pedialyte may also be appropriate, despite a lack of acceptance for this modality in the United States. Question 4-2

The following are components of oral rehydration ther­ apy (ORT), as specified by the World Health Organization (WHO), EXCEPT: A) Potassium. B) Sodium. C) Zinc. D) Chloride. E) Citrate. Discussion 4-2

The correct answer is "C:' While it may not be important to mem­ orize all of the properties of ORT, it is probably useful to have some basic understanding of what those components are and why Pedialyte, the unflavored variety in particular, does not taste very good. Zinc, although commonly lacking in the diets and bodies of children in developing countries, is not a WHO-recommended component of ORT. Although it is similar, Pedialyte is not ORT per se. The sodium and chloride concentrations are lower than in ORT, which makes Pedialyte more palatable. The flavored versions contain the sweetener sucralose, also to increase the palatability. The concentrations of sodium and glucose need to be the same to take advantage of their 1 : 1 linked transport in the small intestine, which is the speediest method of fluid absorption; the faster the better in an ill, vomiting child. The glucose concentration of ORT needs to be somewhat low so as not to promote an osmotic effect and contribute to the diarrhea. (The child already has enough of that.) Sports drinks and sodas contain much more carbohydrate and much less sodium; it is up to the individual to decide what is more palatable. Dehydrated children will drink Pedialyte or ORT because they are thirsty and not very picky. But any fluid is better than no fluid, so if all a child will drink is a sweetened carbonated soda, let him or her have it. As for our patient, starting with small amounts of Pedialyte and slowly increasing the frequency or the amount is often enough to return the infant to a better hydration status, feeling better, and (somewhat paradoxically) not vomiting. And the hassle and pain of placing an IV line is avoided. Suppose the child appeared to be more ill than our current patient, was tachycardic, and had a prolonged capillary refill (> 3 seconds) . Suppose she was known t o have cystic fibrosis. Suppose you obtained some laboratory studies. •



1 1 1r

Helpful Tip

The WHO-recommended properties ofORT are as follows: Tota l osmolal ity between 200 a n d 3 1 0 mOs m/L ·

Eq u i molar concentrations of g l ucose and sod i u m G l ucose concentration < 20 g / L ( 1 1 1 m moi/L) Sod i u m concentration between 60 a n d 90 m Eq/L Potassi u m concentration between 1 5 and 25 m Eq/L Citrate concentration between 8 a n d 1 2 m moi/L Ch loride concentration between 50 and 80 m Eq/L



F LU I D A N D E LECTROLYTE M ETABOLI S M

307

Question 4-3

What electrolyte abnormality is more common in dehydrated patients with cystic fibrosis than in otherwise healthy dehy­ drated patients? A) Hyponatremia. B) Hypochloremia. C) Hypokalemia. D) Hyperbicarbonatemia. E) All of the above. Discussion 4-3

The correct answer is "E:' The point is that the evaluation of a dehydrated patient with cystic fibrosis (CF) likely will require more vigilance than that of an average dehydrated patient, but do not ever call your patients "average;' at least not to their parents. You will recall that the screening test for CF is the sweat chloride test; high amounts of chloride in the sample are consistent with a diagnosis of CF. Therefore it stands to reason that CF patients lose more chloride (and more sodium) in their sweat, making them more prone to hypochloremia and hyponatremia, especially when they have been exercis­ ing in a warm environment. They may even be chronically hyponatremic, which leads to increased aldosterone secretion and further disturbances such as hypokalemia and metabolic alkalosis, otherwise known as hyperbicarbonatemia. Dehydra­ tion acutely worsens the chronic stresses on their electrolyte metabolism. Question 4-4

Which of the following would be an appropriate maintenance IV fluid order for a 23-kg child? A) We need the child's body surface area to properly determine a fluid rate. B) D 5 'l2 NS with 20 mEq/L KCl at 63 mL!h. C) D 5 NS with 20 mEq/L KCl at 63 mL!h. D) D 5 'l2 NS with 20 mEq/L at 1 000 mL!kg/day. E) D 5W at 80 mL!kg/day. Discussion 4-4

The correct answer is "B:' Exactly what constitutes a child's "maintenance" IV fluid need is debatable; probably the only true maintenance fluid is total parenteral nutrition (TPN) . To maintain hydration, children need to replace the water they lose through urine and stool and the water that is expelled through their skin and lungs, otherwise known as insensible loss. The most prevalent method for determining mainte­ nance fluid needs in children has been with us since at least 1 957, when Doctors Holliday and Segar published their seminal paper in Pediatrics. They furthered the notion that water needs are best tied to energy expenditure and gave us what has become known as the "4:2 : 1 rule:' Using that rule, a 2 3 -kg child would require 4 mL!kg/h of fluid for his or her "first" 10 kg, 2 mL!kg/h of fluid for the "second" 10 kg, and 1 mL!kg/h for every kilogram beyond those two sets of 1 0 , totaling 63 mL!h for the child i n question. Th e electrolytes added to the water are those that are most important to us, at amounts that were decided upon by Holliday and Segar

308

MCG RAW-H I LL E D U CATION S P E C I A LTY BOA R D REVI EW: P E D I ATRICS

relative to their concentrations in human and cow milk. Our bodies do require magnesium, but not enough that it needs to be included in the replacement fluid. As noted previously, \4 NS does not include enough salt. Using normal saline for a maintenance fluid is acceptable, but Jt2 NS is arguably the best. Five percent dextrose provides just enough calories to keep us from catabolizing ourselves if we are not eating. Option "E" is a good answer for a newborn, but 23 kg is a good birthweight for a dolphin, and who knows what it needs? Other methods of estimating human maintenance IV fluid needs have been developed, such as using body surface area, but none have had the staying power of the 4:2 : 1 rule. Being precise and using the number 63 is unnecessary, although it is not too hard to program 63 mL/h into an infusion pump. Try it some­ time. (See Table 1 5-7.) Question 4-5

For our 23-kg patient, which of the following is correct regarding his maintenance needs? A) His daily sodium needs are 46.8 mEq. B) His daily sodium needs are 1 54 mEq. C) His daily potassium needs are 46.8 mEq. D) Jt2 NS provides his exact sodium needs. E) He needs 1 L of fluid daily. Discussion 4-5

The correct answer is ''A:' To determine a "maintenance" IV fluid requirement, you need to calculate the daily water, sodium, and potassium needs. Daily, 3 mEq of Na+ and 2 mEq of K+ are needed for every 1 00 mL of fluid. For our 23-kg patient: Daily fluid needs: 1 500 mL + (20 mL/kg/day x 3 kg) = 1 560 mL (see Table 1 5-7) Daily Na+ needs: (3 mEq/ 100 mL) Daily K+ needs: (2 mEq/ 100 mL)

x

x

1 560 mL = 46.8 mEq

1 560 mL = 3 1 .2 mEq

To calculate what fraction of NS is needed, divide the patient's Na+ need by the Na+ concentration of NS (46.8/ 1 54 = 0.3); V3 NS would be perfect. Now add 3 1 .2 mEq KCl!L. A perfect fluid would be D 5 V3 NS + 30 mEq KCl!L.

TABLE 1 5-7 DA I LY A N D H O U RLY F LU I D R EQ U I R E M E NTS U S I N G T H E H O L L I DAY-SEGAR F O RM U LA

Wei g ht ( kg)

24-Hour Ca lcu lation

Hou rly Ca lcu lation

Fi rst 1 0 kg

1 00 m l/kg/day

4 m l/kg/h

Seco nd 1 0 kg

1 000 ml + 50 m l/ kg/day

40 m l + 2 m l/ kg/h

1 5 00 m l + 20 m l/ kg/day

60 m l + 1 m l/ kg/h

Each additional kg > 20 kg

A 12-year-old girl is hospitalized for management of pneu­ monia with a parapneumonic effusion. Despite being encouraged to eat, she has been eating poorly since admis­ sion, although she has been drinking "a lot;' despite being on IV fluid. Her mother asks when she is going to get bet­ ter, especially since her energy level seems to be deterio­ rating. She largely appears well but is somewhat pale. She appears well-hydrated and has no edema. Her urine output has been steadily dropping since admission. Her vitals have been stable within normal limits for the last several days. The decision is made to start her on parenteral nutrition. Prior to starting TPN, the hospital pharmacist asks for some "baseline" lab tests. Shortly thereafter, you answer a page to the child's nursing unit, and her troublesome lab results are reported to you: Sodium 120 mEq/L, potassium 7.0 mEq/L, albumin 1 .9 g/dL, hemoglobin 8 g/dL, and white blood cell count (WBC) 25,000/mmJ. There is no record of hemolysis. Her BUN is 16 mg/dL, creatinine 0.65 mg/dL, and glucose 95 mg/dL. The remainder of her chemistry panel is normal. Question 5-1

What should be rectified first? A) Sodium. B) Appetite. C) Potassium. D) Albumin. E) Hemoglobin. Discussion 5-1

The correct answer is "C:' The potassium level is the most both­ ersome, so deal with that first. A potassium level of 7 mEq/L or higher needs your immediate, undivided attention. Hypona­ tremia (serum sodium < 1 3 5 mEq/L) has variable and nonspe­ cific effects and can certainly make a person tired, but that level of hyponatremia is less urgent than a truly elevated potassium level, which could lead to a fatal arrhythmia, including asys­ tole-fatal, as in death and increased malpractice premiums. Poor appetite, low albumin, low sodium, and low hemoglobin are all commonly seen in this setting, each of which should be addressed in due time, though none of them are emergent issues. These lab results promptly become your first priority, so you drop your donut and proceed to the patient's unit to deal with the situation. When you arrive, she looks no worse than she did in the morning and responds to questions appropriately. The nurse states that she thinks the T waves have been some­ what peaked on the patient's cardiorespiratory monitor tracing. A look at her current tracing shows what appears to you to be a normal sinus rhythm. Question 5-2

Of the following, which is NOT an appropriate intervention for true hyperkalemia? A) IV infusion of calcium gluconate. B) IV infusion of insulin- and dextrose-containing fluid. C) Sodium polystyrene sulfonate (Kayexalate) .

C H A PT E R 1 5

D) An intranasal dose of desmopressin. E) IV infusion of normal saline and a dose of furosemide. Discussion 5-2

The correct answer is "D:' Calcium gluconate does not lower serum potassium levels per se, but it does lessen the pro­ dysrhythmia effect of hyperkalemia and is used in conjunc­ tion with interventions that do lower the serum potassium, such as the administration of dextrose and insulin. These together precipitate a shift of potassium from the extracel­ lular compartment to the intracellular compartment, where most of our potassium ( �95%) belongs. Fluid administration followed by a dose of furosemide, a non-potassium-sparing diuretic, is also helpful. Sodium polystyrene sulfonate lowers serum potassium by exchanging its sodium ions for a patient's potassium ions while in the intestines. Desmopressin (syn­ thetic antidiuretic hormone) is not a recommended treatment for hyperkalemia.



F LU I D A N D E LECTROLYTE M ETABOLI S M

309

lower the serum potassium is to give a dose of beta-agonist, which promotes potassium uptake into cells. The most readily available beta-agonist in a children's hospital is albuterol, which is a safe intervention even if the patient's potassium is not truly elevated. (See Table 1 5-8.) Hypokalemia (serum potassium < 3 mEq/L), on the other hand, causes muscle weakness up to the point of paralysis; all sorts of cardiac arrhythmias, includ­ ing ventricular tachycardia; and renal dysfunction. Hypokale­ mia in children is most often seen with gastroenteritis or with diuretic use. Levels below 3 mEq/L typically cause symptoms and should be rectified. This is preferably done using oral potas­ sium chloride or with IV doses of 0.25 to 0.5 mEq/kg of K+ (a "potassium rider") for those who cannot take oral medica­ tion. Prunes and chard are high in potassium, if only you could get a child to eat them.

TABLE 1 5-8 CAU S E S OF HYPOKA L E M I A A N D HYP E R KA L E M I A

Question 5-3

What should be the next step in this patient's management? A) Give an IV dose of calcium gluconate. B) Repeat the chemistry panel and obtain an ECG. C) Review her cardiorespiratory tracing for the last 2 hours. D) Give an oral dose of Kayexalate. E) Start an insulin drip at 0. 1 units/kg/h. Discussion 5-3

The best answer is "B:' Again, a truly elevated potassium level requires urgent attention and steps should be taken to rectify it. The proper thing to do is to assess the totality of the situation first. As far as we have been told, our patient does not have an obvious reason to have hyperkalemia, such as tumor lysis syn­ drome or a crush injury, but she has been on IV fluid, which can be mixed incorrectly. Laboratory reports are not always per­ fectly accurate and the sample may well have been hemolyzed, but doing nothing is probably not appropriate in this situation. Repeating the lab draw promptly and obtaining an ECG are the correct first steps. Obtaining an ECG is appropriate not only for confirmation of any suspected cardiorespiratory monitor find­ ings, which are not as sensitive as a 1 2-lead ECG, but also serves as the best way to later confirm a response to your interven­ tions if there are rhythm abnormalities. A leisurely perusal of the last 2 hours of monitor tracings is unlikely to be fruitful but is likely to earn you the ire of the child's nurse, who is a person of action. Sodium polystyrene sulfonate takes time to lower the serum potassium ( 1 -2 hours) and therefore should not be the first intervention for true hyperkalemia. Insulin can be given, although it would be problematic if administered by itself. The hyperkalemia treatment involving insulin should include a glu­ cose infusion, usually 0.5 mg/kg over 30 minutes with a 0. 1 unit/ kg one-time dose of insulin. The best way to immediately pre­ vent the occurrence of a life-threatening arrhythmia is to give a 0.5 to 1 mL!kg IV dose of 10% calcium gluconate, which stabi­ lizes the myocardium, but usually you have time to sort things out before intervening pharmacologically. The quickest way to

Hypoka l e m i a

(K+ < 3 m Eq/L) I ncreased re n a l excretion

Hyperka l e m i a

( K+ > 5.5 m Eq/L) Decreased re n a l excretion



Hypera ld ostero n i s m



Kid ney d i sease



Renal t u b u l a r acidosis



Hypoa ld ostero n i s m



Ba rtte r syn d ro m e



Add i son d i sease



Gite l m a n syn d rome





D i u retics Drugs (am photericin)







I ncreased g a stroi ntesti n a l losses

Co ngen ital a d re n a l hyperplasia Potass i u m-spa r i n g d i u retics Drugs Aldosterone i nsensitivity (pseudohypoa ld oste­ ro n i s m )

Cel l u l a r destruction Tu mor lysis syn d rome •



Vom i t i n g



Rhabdomyolysis



Gastric sucti o n i n g



Hemolys i s



Dia rrhea





Laxative a buse

Cel l u l a r s h ift i n g

Cel l u l a r s h ift i n g



Meta b o l i c a l kalosis



Beta-a g o n i sts (a l b uterol)





Insulin





Hyperthyro i d i s m





Meta b o l i c acidosis Beta-blockers Hyperka l e m i c period ic pa ra lysis

Fa m i l i a l hypoka l e m i c period ic paralys i s

Iatrog e n i c •

Tra n sfusion o f stored red blood cel l s

Ad m i n i stration o f IV fl u id s or TPN without K+

Iatrog e n i c •

Ad m i nistration of I V fluids or TPN with excess K+

MCG RAW-H I LL E D U CATION S P E C I A LTY BOA R D REVI EW: P E D I ATRICS

310





Helpful Tip

Pseudohyperka lemia is common. Most often it resu lts

1 1 1r from a hemolyzed speci men obta i ned by squeezi ng

the heel too hard d u ri n g speci men col l ection. The lysed cel l s release potassi u m . Pseudohyperkalemia resulting from severe leu kocytosis or throm bocytosis occu rs when the cel ls clot i n the col lected specimen.

The lab is called and the tech apologizes for forgetting to note the hemolysis. To be thorough, a repeat potassium level is drawn and an ECG is obtained, both of which are normal.

causes of her hyponatremia. Her lung pathology puts her at risk for SIADH, so it is reasonable to be leaning toward that diagnosis. Her urine output alone would not provide all the required information, but a low urine output would be con­ sistent with SIADH. Psychogenic polydipsia is not common in otherwise healthy children but is not out of the question. Establishing a diagnosis of SIADH, and then acting appro­ priately, first requires a comparison between urine osmolality and serum osmolality and a look at the urine sodium, so the answer to the above question is option "C." The serum osmo­ lality can be ordered separately and measured directly, but it also can be estimated by the following equation, conveniently using laboratory tests that in most cases have already been obtained: (2

Question 5-4

All of the following ECG findings can be seen in a patient with hyperkalemia EXCEPT: A) Peaked T waves. B) Prolonged PR interval. C) Prominent U waves. D) Absent P waves. E) Asystole. Discussion 5-4

The correct answer is "C:' U waves can be a normal finding but also can be indicative of hypokalemia, not hyperkalemia. As previously discussed, hyperkalemia can cause asystole, and asystole is pretty close to death, so the answer is not option "E:' Hyperkalemia is known to produce all of the other findings, which are listed in order of their occurrence as the potassium level increases.

Now it is time to deal with the patient's hyponatremia. She reportedly has been drinking a lot, iatrogenic hyponatremia is common in the inpatient setting, and hyponatremia and syndrome of inappropriate antidiuretic hormone secretion (SIADH) are solidly linked in your impressionable mind, despite the rare occurrence of SIADH in children. Question 5-5

How can you determine the cause ofthe patient's hyponatremia? A) Check her urine output for the last several days. B) Measure her antidiuretic hormone (ADH) level. C) Measure her serum osmolality, urine osmolality, and urine sodium. D) Give a fluid bolus and check her serum sodium level thereafter. E) Check her cortisol level. Discussion 5-5

The correct answer is "C:' The first step is to assess her hydra­ tion status, which is essential in determining the cause of the hyponatremia (Na+ < 1 3 5 mEq/L) . She is reportedly well hydrated and not overly hydrated (hypervolemic) , as is sug­ gested by the lack of edema, which together limit the possible

x

Na+) + (BUN/2.8) + (Glucose/ 1 8 )

Th e 2.8 and the 1 8 i n this equation are mg/dL-to-mmol!L con­ version factors to make terms agree, and the multiplication of the Na+ by 2 accounts for the anions (other osmoles) associated with the sodium cations. In SIADH, the urine osmolality will be inappropriately high ( > 100 mOsm/kg) in comparison to the serum osmolality. If the urine sodium is in the range of 20 to 30 mEq/L or higher in a euvolemic, hyponatremic patient with an inappropriately high urine osmolality, the main considerations are SIADH, hypothyroidism, and glucocorticoid deficiency. It will, of course, be 25 mEq/L when you order it. Measurement of ADH levels, though intuitive, has not been shown to be clinically useful. (See Figure 1 5 - 1 . ) Tracking down the cause of hyponatremia in a patient with a complex set of findings is not always straightforward ( eg, in a patient with meningitis who has heart failure and is taking diuretics). If we learn the prin­ ciples we should be able to figure things out. Sometimes a fluid bolus followed by repeated lab values is the most appropriate intervention. Our patient's calculated serum osmolality is 2 5 1 mOsm/kg, her measured urine osmolality i s 2 2 0 mOsm/kg, and her urine sodium is 34 mEq/L. She is not simply drinking too much, which would give her a low urine osmolality. In the proper setting ( eg, this previously healthy child with a parap­ neumonic effusion), these results, at least initially, are most con­ sistent with a diagnosis of SIADH. The urine is concentrated with elevated sodium while the serum is dilute.



Helpful Tip

:a� The

sod i u m

level

r1 1r (pseudohyponatremia)

may in

be the

fa lsely presence

low of

hyperg lycem ia, hyperl i pidem ia, a n d hyperproteinemia.



Helpful Tip

� When t h i n king a bout the causes of hyponatremia, fi rst

=

r1 1 r determ i n e the patient's fl uid status (ie, hypovolem ia, euvolem ia, or hypervolemia). Use this i nformation

to identify the relative causes. It beats memorizing a l a u n d ry l ist of etiologies.

C H A PT E R 1 5



F LU I D A N D E LECTROLYTE M ETABOLI S M

31 1

Normal serum sodium 1 35-145 meqfl Normal serum osmolality

Measured: -275-295 mOsm/kg Calculated value should be within 10-15 of measured, through in mmoi/L

- Psychogenic polydipsia - Potomania (far too much beer)

Normal urine osmolality -50-1 500 mOsm/kg Normal urine sodium (random)

Reference range not established; generally should be over 20 meq/L The numbers may not add up well and the cutoff levels are not exact, but the patient will have a history that should be helpful

() Kidneys holding on to electrolytes Extrarenal solute loss - Vomiting diarrhea - Third spacing - Skin losses (i.e. sweat)

Urine sodium should be

Kid neys losing electrolytes

>26

- Renal insufficiency - Mineralocorticoid deficiency (K+ usually high) - Cerebral salt wasting

- SIADH - Medications: some antiepileptics and chemotherapy agents (mimickers of ADH) - Glucocorticoid deficiency - Diuretics

Mostly not the kidneys' fault

The kidneys' fault Chronic renal failure

- Congestive heart failure - Cirrhosis Nephrotic syndrome

F I G U R E 1 5- 1 . Causes of hyponatremia.

� QUICKQUIZ Why are urine and serum osmolality reported in mOsm/ kg while the calculated estimate of serum osmolality is in mmol/1? A) I don't know. B) Do I need to know this? C) I will feel like an expert after this question. D) Is this relevant? E) I don't like chemistry. Discussion

The best answer is "C:' or at least we hope it will be after read­ ing the explanation that follows. Millimoles per liter of solu­ tion and milliosmoles per kilogram of solvent are measures of osmolarity and osmolality, respectively. Osmolality and osmolarity can differ, depending on the solvent. One liter of water, by definition, equals 1 kg, but 1 L of serum does not. Confusing as this may be, the difference is small enough not to be clinically relevant. In solving the earlier equation, we are technically calculating a serum osmolarity, but let's not get hung up on the words. The most common laboratory testing methodology for osmolality utilizes the difference in freez­ ing temperatures between two solutions of different osmolal­ ity; it does not distinguish between osmoles such as sodium,

glucose, and so on. The laboratory testing for sodium, glucose, and BUN relies on different methods; therefore, it is expressed in different units. The difference between the measured serum osmolality and our calculated serum osmolarity is referred to as the "osmolal gap;' which if elevated in a patient without an acidosis points to an alcohol ingestion, usually isopropyl alcohol or ethanol.



Helpful Tip

:5.� Th i n k a lcohol

ingestion if the plasma osmolal gap is

i1 1 r elevated! If ethanol

is ru led out, consider isopropyl

a lcohol, metha nol, or ethylene g lycol . Remem ber, drinking rubbing a lcohol, antifreeze, or deicing solutions is not recommended.

Question 5-6

The most appropriate intervention for this patient with SIADH should be: A) Administer a dose of tolvaptan (vasopressin receptor antagonist) . B) Postpone starting TPN. C) Start a 3% sodium infusion. D) Give an IV NS bolus. E) Restrict fluid intake.

31 2

MCG RAW-H I LL E D U CATION S P E C I A LTY BOA R D REVI EW: P E D I ATRICS

Discussion 5-6

The best answer is "E:' A serum sodium level of 120 mEq/L, though attention grabbing, is not universally dangerous, but it should be monitored closely and corrected. Treating the under­ lying condition may be all that is needed, but if the patient is drinking excessively she could continue to have fluid balance problems, so her oral intake should be restricted. Starting TPN would be appropriate for nutritional support if needed; although her oral intake and the TPN together should be kept under her maintenance fluid need (60% of maintenance is suggested) . Vasopressin receptor antagonists have a role in the treatment of hyponatremia, particularly in adults with heart failure, cirrhosis, and SlADH, but their use in children has been limited and would not be warranted in this case. A 3% NaCl solution would also not be warranted. This patient has too much free water not too little salt. Since we think we know what the problem is, giving a fluid bolus also would not be the most appropriate intervention. Question 5-7

Of the following, all are predisposing factors for the develop­ ment of SIADH EXCEPT: A) Head trauma. B) Pneumonia. C) Lithium use. D) Lung cancer. E) Meningitis. Discussion 5-7

The correct answer is "C:' Certain medications, most notably in pediatrics the antiseizure medications carbamazepine and oxcarbazepine and selective serotonin reuptake inhibitors such as fluoxetine and sertraline, are known to alter ADH metabo­ lism and cause SIADH, but lithium is not among them. Lithium has in fact been associated with the development of nephro­ genic diabetes insipidus and was at one time used as a treatment for SIADH . Pneumonias, malignancies (small cell lung cancer, in particular) , and central nervous system disturbances such as head trauma, meningitis, and neurosurgical procedures are all known to cause SlADH. Persistence of hyponatremia is, at best, unpleasant and should be rectified. How this is rectified depends on the situation, but if a child has seizures or altered mental status attributable to hyponatremia, the best treatment is to give 3 to 5 mL/kg of 3% NaCl solution over a short interval ( �30 minutes) , check the sodium thereafter, and repeat the infu­ sion if the symptoms do not abate. For further, or less urgent, sodium replacement, an initial 3% NaCl solution infusion rate can be estimated by multiplying the patient's body weight in kilograms by the desired rate of increase in serum sodium in milliequivalents per liter per hour. For example, a 10 kg child whose serum sodium you would like to increase at a rate of 0.5 mEq/L/h requires an infusion of 3% NaCl at 5 mL/h. A con­ servative rate of change of no more than 8 mEq/L in a 24-hour period is recommended, although disagreement about the rate exists. In any case, changes in serum sodium can be unpre­ dictable, so frequent lab draws are warranted, perhaps as fre­ quently as every 2 hours. An appropriate therapeutic end point is a sodium level of 120 to 125 mEq/L, which assumes that the

underlying mechanism of the hyponatremia is being addressed. Remember, the clinical symptoms of hyponatremia depend on duration (acute or chronic) and severity (how low the value is) . With a sodium value o f 1 2 0 mEq/L o r less, seizures are likely to occur. Acute problematic symptoms such as seizing are less likely when the sodium value is 125 mEq/L or greater. In the set­ ting of seizures, the sodium should be rapidly raised by 5 mEq/L or above the level of 125 mEq/L. •

Helpful Tip

� Avoid

=-

correcting the sod i u m too q u ickly i n patients

r1 1r with hyponatremia or hypernatremia to avoid osmotic

demye l i nation or cerebra l edema, respectively. This is especia l ly true if the condition is c h ronic. A general ru le is to correct no faster than O.S m Eq/Uh.





1 1 1r

Helpful Tip

Administration of 1 m Ukg of 3% sod i u m ch loride w i l l increase the serum sod i u m by 1 m Eq/L.

It is a blustery November Sunday evening in Maine, and you are passing through the emergency department (ED) to the physician's parking lot after rounding in the nursery when the ED physician waves you down for some help. A 2¥2-year­ old boy has just been brought in by his father because "He's acting funny, kind of like he's drunk:' As far as dad knows, the child has not been sick lately and he apparently was fine when he woke up this morning before the mother left for work. The father figured there was something wrong when he found the boy on the floor of the living room at about 6 PM drooling, with a "glassy look in his eyes:' In addition to bab­ bling slowly and insensibly on the way to the hospital, the boy vomited once. He is mildly tachycardic but otherwise appears well, aside from being a little unsteady on his feet and mostly oblivious to the world about him in a happy sort of way. The father seems to have all his wits about him. Question 6-1

The child was brought to the ED, so he is almost certainly going to undergo some lab studies, but which ones would you advocate? What labs should be obtained? A) Chemistry panel. B) Serum alcohol level. C) Blood gas. D) Serum drug screen. E) All of the above. Discussion 6-1

The correct answer is "E:' The patient is not of childbearing age, so he probably does not need a serum beta-hCG test, but he needs all of those listed above, plus some more. The history

C H A PT E R 1 5

strongly suggests an ingestion of some sort. Short of being tied to their children during all waking hours, parents cannot know what they are up to all the time. Sometimes parents need to attend to other needs, and toddlers are curious and always ready to put just about anything into their mouths, particu­ larly liquids in colorful cups. The father's thought that his son is acting as if he is drunk points us in certain directions, as well it should. Question 6-2

Which test is least likely to be helpful in the initial evalua­ tion of a child with a suspected methanol or ethylene glycol ingestion? A) Urinalysis. B) Serum methanol or ethylene glycol concentration. C) Blood gas. D) Chemistry panel. E) Serum osmolality. Discussion 6-2

The correct answer is "A:' All of these are useful tests in estab­ lishing, or ruling out, a methanol or ethylene glycol exposure in a child, but a urinalysis is least helpful given that oxalate crystals are not specific to ethylene glycol ingestions and are a late find­ ing. Serum methanol and ethylene glycol tests are not readily available in many hospital labs, which limits their usefulness, so we often need to rely on some old-school, readily available testing to point us in the right direction so we can limit the damage from such ingestions. If you are thinking anion gap, you are on the right path. Methanol and ethylene glycol are not acids, but their metabolites in the body are, which can and do cause an increased anion gap metabolic acidosis. The elevated osmolal gap was discussed above. Let's hope you were paying attention.



Helpful Tip

:5.� Why

is the C0



F LU I D A N D E LECTROLYTE M ETABOLI S M

Question 6-3

The anion gap represents: A) The number of measured serum anions minus the unmea­ sured serum anions. B) The discrepancy between the number of serum cations and serum anions. C) 1 2 . D) The number of unmeasured serum anions minus the unmeasured serum cations. E) A number that increases with a decrease in serum osmolality. Discussion 6-3

The correct answer is "D:' Not many things are as exciting as a big anion gap, unless it is your child who has it. The num­ ber of serum anions must always equal the number of serum cations, so option "B" is incorrect. The "unmeasured" cations are calcium, magnesium, and potassium. These frequently are measured, but the anion gap traditionally is calculated from the lab values reported on a standard basic metabolic pro­ file. The K+ can be used in the calculation, but the number is small and does not vary to a large degree, so it is usually left out. The normal, "unmeasured" serum anions are phos­ phate, sulfate, and various proteins and organic acids. The number 12 is a fine number that coincidentally is within the realm of a normal anion gap, but that is only a number and we need some lab values in an actual patient to calculate an anion gap. If the K+ is ignored, the value of a normal anion gap can range from 8 to 1 6 , but in the question we are being asked what the value actually represents, which technically is the number of unmeasured anions minus the number of unmeasured cations. An increased anion gap without an acidosis is not a com­ mon pediatric scenario; therefore, we can focus on situa­ tions in which an increased anion gap is associated with a metabolic acidosis. An increased anion gap clues us in to the presence of abnormal anions, which are a subset of the unmeasured serum anions. Where did they come from?

(carbon d i oxide) on a meta bolic

i1 1 r profi le i nterpreted2 as HC03 (bicarbonate)? The s i m p l e

a n swer i s , t h a t i s the w a y it is reported . T h e more c o m p l i cated a n swer stem s from the lack of a rea d i ly

ava i l a ble system fo r the d i rect measurement of seru m HC0 3 • The C0 2 reported on a basic meta bolic profi le is the tota l a m ount of C0 2 prod uced from the acid ification of a seru m sa m p l e, which converts all of the HC0 3 , C0 2, a n d H 2 C0 3 present (95% of w h i c h is HC0 3 ) i nto C0 2, which can be measured a n d i s what is reported . The HC0 3 reported on a seru m b l ood gas a n a lysis is a l so not a d i rect measure of seru m HC0 3 ; the va l u e is ca l c u l ated from the seru m pH a n d

Total anions

Total cations

U nmeasured cations 1-------t - - - - - - - - - -

I

- - - - - - - - - -

Sod i u m

Unmeasured anions

Anion gap t------1

Ch loride

pC0 2 u s i n g the H e n derson-Hasselbach equation, which you may reca l l fro m u n d e rg ra d biochemistry ( a n d which is where it wi l l stay fo r o u r pu rposes) . Both of these methods i ntrod uce a n error i nto the measurement of HC0 3 •

31 3

Bicarbonate

314

MCG RAW-H I LL E D U CATION S P E C I A LTY BOA R D REVI EW: P E D I ATRICS

For those who like lists, the mnemonic MUDPILES captures most of the possibilities: M - Methanol U - Uremia (isocyanic acid and uric acid) D - Diabetic ketoacidosis (acetoacetic acid and beta­ hydroxybutyric acid) P - Paraldehyde, paracetamol I - Inborn errors of metabolism, isoniazid L - Lactic acidosis E - Ethylene glycol

S - Salicylates Paracetamol is what acetaminophen is called in most coun­ tries outside of the United States and paraldehyde is a seda­ tive, mainly used to stop seizures, that is not available in the United States. For those who like processes, consider this: An increased anion gap with an acidosis, not surprisingly, repre­ sents a gain of acid in the blood. The H+ is buffered in large part by HC0 3 , causing the laboratory-reported HC0 3 to drop, and the associated abnormal anions are left to wreak havoc. Where did the rogue acid come from? Perhaps from the body itself: lactate, ketoacids (diabetic ketoacidosis, DKA), or the acids that build up when the kidneys fail. Perhaps not from the body: methanol, ethylene glycol, paraldehyde, toluene (the breakdown products of which are toxic acids) , and medica­ tions such as isoniazid, aminoglycosides, and salicylates, including aspirin, bismuth subsalicylate (the active ingredient in Pepto-Bismol), and methyl salicylate (the active ingredient in Bengay and oil of wintergreen, which is a source of highly concentrated salicylate) . Other than a gain in acid, the other main category of metabolic acidosis (non-anion gap) results from either a loss of, or a failure to generate, HC0 3 • This can occur through net base loss, as in gastrointestinal losses from diarrhea or with renal losses from diuretic use or proxi­ mal renal tubular acidosis, or from a failure of the kidney to excrete normally produced acids, as in renal insufficiency and distal renal tubular acidosis. Question 6-4

Of the following, which is the most common cause of a non-anion gap metabolic acidosis in a child? A) Renal tubular acidosis. B) Cholestyramine. C) Acute kidney insufficiency. D) Respiratory insufficiency. E) Diarrhea. Discussion 6-4

The correct answer is "E:' The acidosis is caused by the loss of bicarbonate in the stool. The body prefers a pH close to 7.4, so the lungs compensate rapidly for this loss of bicarbonate by stepping up the excretion of C0 via an increased respiratory 2 rate. A respiratory acidosis is not a metabolic acidosis. All of the other listed items can cause an acidosis, but none are as com­ mon as diarrhea.

Question 6-5

Speaking of diarrhea, what is a normal stool output for a human? A) 15 g/kg/day. B) 1 5 mL/kg/h. C) 30 g/kg/day. D) Up to 200 g/day in an adult. E) Two stools a day. Discussion 6-5

The correct answer is "D:' Anything looser than a child's nor­ mal output is interpreted by most parents as diarrhea, but we need to know what normal is so we can either be reassuring or not reassuring, as the situation requires. There is quite a bit of variability in the consistency, frequency, and volume of what can be considered to be normal, but infants typically have 5 to 1 0 g/kg/day of stool output and an adult will have up to 200 g/day. Two stools a day may be "normal;' but what about the volume and consistency? Some practitioners consider diarrhea to be a stool output of greater than 15 g/kg/day, so 30 g/kg/day is enough to be considered abnormal. Urine out­ put is generally measured in milliliters per kilogram per hour, and a normal urine output in a fully hydrated child is 1 to 2 mL/kg/h, although it can be considerably more in a normal, overly hydrated child.

Remember our patient? Let's say his anion gap is indeed elevated at 24 (> 20 is usually worthy of attention ) . His blood gas shows a metabolic acidosis: pH 7.28, PC02 38, HC0 3 18. You start running down the list of possibilities with his father. He admits to putting some antifreeze in the car yes­ terday, though he is not willing to say that he left any in a location that was accessible to his child. There is no aspirin or alcohol in the home. Some of the lab reports are still pending. Question 6-6

What is the most appropriate intervention for this child? A) Intravenous fomepizole. B) Ethanol infusion. C) Peritoneal dialysis. D) Ritualistic bloodletting. E) Intravenous fluid at 1 . 5 times the maintenance rate. Discussion 6-6

The correct answer is "A:' Ethanol can be used, but fomepizole is the preferred treatment for suspected ingestion of antifreeze. Both are competitive inhibitors of alcohol dehydrogenase, and therefore keep ethylene glycol from being metabolized into toxic compounds. The parent compounds cause central nervous system depression but are otherwise relatively nontoxic. It is the acids produced from metabolism that are toxic. Sodium bicarbonate and dialysis are also used. Peritoneal dialysis would not, however, be the preferred method. Bloodletting still exists in "Western'' medicine; we just call it a partial exchange transfusion, which is a treatment for polycythemia in a new­ born. Whether or not it is performed as a ritual is a matter of preference and interpretation.

C H A PT E R

15



F LU I D A N D E LECTROLYTE M ETABOLI S M

31 5

I ncreased

A 29-day-old previously healthy infant has had nasal conges­ tion and a cough for about a week. He is diagnosed with respiratory syncytial virus (RSV) bronchiolitis and admitted because he is hypoxic. Several hours after arrival, his nurse tells you that he now has retractions, nasal flaring, and an increasing supplemental oxygen need. A blood gas measure­ ment is obtained, revealing the following: pH 7.29, PC02 66, HC0 3 30.4.

I

Metabolic

I Respiratory I

I

Metabolic

I Respi ratory I

Normal pH 7 . 0 , Pco2 40, HC0 24 3 F I G U R E 1 5-2. Acid-base status.

Question 7-1

Which of the following statements is FALSE?

A) For each acute 10 mm Hg uncompensated increase in PC02, there is a 0.08 point decrease in pH. B) For each acute 10 mm Hg increase in PC02, there is a 1 mmol!L increase in HC03• C) For each chronic 10 mm Hg increase in PC02, there can be up to a 4 mmol!L compensatory increase in HC03• D) The kidneys compensate for a respiratory acidosis by excret­ ing H+. E) The lungs can rapidly and completely compensate for any metabolic alkalosis. Discussion 7-1

The correct answer is "E:' Functional though they are, the lungs cannot always "fully" compensate for a metabolic alkalosis, given that we can only lower our respiratory rate so much and continue to live. Perhaps this keeps the body aware that what­ ever fishy is going on needs to be rectified.

normal, average pH is 7.40; a normal, average HC03 is 24; and a normal, average PC02 is 40. In the current scenario, the pH is lower than the normal 7.40, so the child has an acidosis. Easy enough. His PC02 is higher than normal, which is characteristic of a respiratory acidosis, and his history is consistent with the development of a respiratory acidosis. His HC03 is higher than normal at 30.4. If the child had a pure, uncompensated respira­ tory acidosis, his pH should have decreased by approximately 0.2 1 (0.008 x [66 - 40] ), resulting in a pH of 7. 1 9 (7.40 - 0.2 1 ) and his HC03 should have increased spontaneously by 2.6 ( [ 66 40] x 0. 1 ) to 26.6. But his pH is 7.29 and his HC03 is 30.4, so his kidneys must have compensated, which they do by excreting H+ (functionally increasing the serum HCOJ He may have lost H+ or gained HC03 somewhere else, but we can keep it simple. (See Table 1 5-9.) This child has a partially compensated respira­ tory acidosis. The chronic change in HC03 quoted in the answer choices for Question 7- 1 (0.4 mmol!L per 1 mm Hg increase in

Question 7-2

Our patient's blood gas results are consistent with what meta­ bolic circumstance?

A) B) C) D) E)

An uncompensated respiratory acidosis. An uncompensated respiratory alkalosis. A compensated metabolic alkalosis. A partially compensated respiratory acidosis. A mixed metabolic and respiratory acidosis.

Discussion 7-2

The correct answer is "D:' Metabolic disturbances are found in patients who have histories of illness, and we have been pre­ sented with a child with respiratory difficulty. As we are aware, respiratory difficulties can lead to hypoxia, hypercarbia, or both, so most likely we are looking at a respiratory acidosis in this scenario, which will get you off on the right foot. (See Figure 1 5-2.) Extrapolating from the commonly used relation­ ships in the options listed above, an acute, uncompensated increase of 1 mm Hg of PC02 causes a decrease of 0.008 points on the pH scale and a 0. 1 mmol!L increase in HC03• The HC03 and PC02 increase together based on their natural balance in the blood. If you note more than the spontaneous increase in HC03 you know that the kidneys have been compensating. To begin to answer questions such as the one above, you must know what normal is for all of these values. Keeping it simple, a

TABLE 1 5-9 S O M E AC I D- BASE R U L E S Respi ratory Acidosis

Acute respi ratory acidosis, no re n a l co m pe n sati o n : F o r e a c h 1 m m Hg i ncrease i n PC02 , there wi l l b e a 0 . 1 m moi/L i ncrease i n HC03 • C h ro n i c respi ratory acidosis, maxi m u m re n a l com pensa­ tion: For each 1 mm Hg i n c rease in PC02 , there w i l l be a 0.4 m mo i/L i ncrease i n HC0 3 • Respi ratory Alkalosis

Acute respi ratory a l ka l osis, no re n a l co m pensati o n : F o r e a c h 1 m m Hg decrease i n PC0 2 , there wi l l b e a 0.2 m m oi/L decrease i n HC0 3 • C h ro n i c respi ratory a l ka l osis, maxi m u m re n a l compensa­ tion: For each 1 mm H g decrease in PC02 , there wi l l be a 0.5 m mo i/L decrease i n HC0 3 • Meta bolic Acidosis

For each 1 m m o i/L decrease i n HC0 3 ' there w i l l be a 1 .2 m m Hg decrease i n PC0 2 • Meta bolic Alkalosis

For each 1 m m o i/L i n c rease i n HC0 3 ' there wi l l be a 0.7 m m Hg i n c rease i n PCOz -

MCG RAW-H I LL E D U CATION S P E C I A LTY BOA R D REVI EW: P E D I ATRICS

316

PCO) represents the kidneys' maximal compensating power, which could change his HC03 by 1 0 ( [66 - 40] x 0.4) up to a total of 34. He's at 30.4, which tells you he has compensated some but not as much as is theoretically possible, as might be guessed by his rather unfavorable pH of 7.29. Once you figure all of this out, the next step is to figure out what, if anything, can be done to get his values back to normal. This particu­ lar child probably needs some respiratory support, assuming that no one has invented "anti-RSV:' The kidneys get to work soon after a disturbance in the force is detected but exercise their compensatory power judiciously over a period of 3 to 5 days. The lungs are able to compensate for a metabolic aci­ dosis much faster, on the order of several minutes, if allowed to do their j ob without hindrance. The lungs work fast enough that we rarely, if ever, need to worry about whether they have compensated appropriately for a metabolic disturbance. We are all compensating for something, no? Perhaps you did not get accepted to medical school on the first try, or your dad is famous, or some such. •



1 1 1r

Helpful Tip

I n acid-base d i sturbances, t h e C0 2 a n d HC0 3 fol low each other. They go u p or down together.

Question 7-3

Which of the following is FALSE?

A) Capillary blood gas measurement gives an accurate measure of P02• B) Capillary blood gas PC02 and pH measurements are com­ parable to arterial blood gas measurements. C) The HC03 measure on a blood gas measurement, regardless of source, is a calculated value. D) Venous blood gas PC02 is regularly 5 mm Hg higher than arterial blood gas PC02• E) Capillary and venous blood gas measurements can be used to accurately delineate an acid-base disorder. Discussion 7-3

The correct answer is "A:' All of the other responses are true.

venous blood gas measurement are completed, the results of which are as follows: pH 7.48, PC02 30, HC03 22. Question 8-1

What is the nature of this patient's acid-base disturbance?

A) B) C) D)

A compensated respiratory alkalosis. An uncompensated respiratory alkalosis. A metabolic alkalosis. A compensated metabolic alkalosis. E) A mixed metabolic and respiratory alkalosis. Discussion 8-1

The correct answer is "B:' Again, the first step is to determine whether the pH is high or low. This child's pH is high, so she has an alkalosis. Her HC03 and PC02 are not normal, but her HC03 is pretty close. As usual, knowing something of the child's his­ tory is helpful. The concern here is the child's respiratory status, given her past and present histories. Most likely we were looking to see if a respiratory acidosis accompanied the child's respira­ tory distress. This is obviously not the case, so just what is going on here? Her bicarbonate level is lower than 24, which is not out of what could be considered the normal range. To say that she had a metabolic alkalosis would be an overstatement. Putting that together with her low PC02 of 30, she appears to have a respiratory alkalosis, which could make sense given the stated difficulty in obtaining the sample and some resultant screaming and yelling, loosely translated into medicalese as hyperventila­ tion. But is that the only disturbance? Probably, but to be sure you can utilize some grade school arithmetic. The change in her PC02 from an assumed baseline of 40 is - 1 0, which would be accompanied by an anticipated change in her HC03 of 0.2 mul­ tiplied by the change in the PC02 which is -2. Her HC03 is at , the anticipated value of 22 (24 - 2). Therefore you can be confi­ dent that she has an acute, uncompensated respiratory alkalosis, which would not be unanticipated with the history. Some blood gas patterns, of course, can represent multiple derangements, which will require some thought to decipher. Examples include acute-on-chronic respiratory acidosis, acute respiratory acido­ sis with a metabolic acidosis, metabolic acidosis with an acute respiratory acidosis, and so on. To arrive at a correct blood gas interpretation, a guess may need to be made regarding the likely derangement followed by some math to determine if that inter­ pretation is correct. Question 8-2

An anxious 3-year-old girl with asthma is brought in for evaluation of a cough and nasal congestion that has been worsening for 3 days. Her mother also states that the child has not been drinking well nor urinating as much as usual over the past several days. She appears somewhat dehy­ drated and is mildly tachycardic at about 1 20 bpm. She is mildly febrile at 38.4 oc ( 1 0 1 . 1 oF) . Her work of breathing appears to be increased, although her level of anxiety and squirminess make it hard to determine just how significant it is. Her respiratory rate is in the upper 20s. After some trouble obtaining the sample, a basic chemistry panel and

What common household substance, if ingested, can pro­ duce a metabolic acidosis with an accompanying respiratory alkalosis?

A) B) C) D)

Ice melter. Baking soda. Baking powder. Aspirin. E) Drain cleaner. Discussion 8-2

The correct answer is "D:' Aspirin is acetylsalicylic acid, one of the most commonly encountered salicylates, which we considered

C H A PT E R

in an earlier question. Although acetylsalicylic acid is a weak acid, it is able to cause a lactic acidosis by disruption of cellu­ lar metabolism, as are all medicinal salicylates. Salicylates also tend to ( 1 ) stimulate the respiratory center to produce a respi­ ratory alkalosis, (2) produce some interesting blood gas results, and (3) participate in the development of Reye syndrome. Here is a characteristic blood gas pattern, which you can play around with in your head: pH 7.48, PC02 22, HC03 14. Baking soda is sodium bicarbonate and is most commonly stored in yellow boxes. Baking powder is sodium bicarbonate, monocalcium phosphate, and sodium aluminum sulfate and is most com­ monly stored in red cans. Each could cause an alkalosis, but not the pattern in question. Drain cleaners usually contain strong bases, which cause local tissue damage but are rarely ingested in amounts able to cause significant systemic acid-base distur­ bances; apparently they are mostly buffered by the esophagus. Ice melter, depending on the brand, contains sodium chloride, potassium chloride, magnesium chloride, urea, calcium magne­ sium acetate, or a combination of these chemicals, none of which would cause an acidosis. Whether or not you knew the content of some of these substances may be immaterial, but you should know the effects of aspirin in an overdose situation, because you may well encounter such circumstances on the board exam or in practice, and you want what is best for your patients, even the hypothetical ones.

An 8-year-old boy is brought to your office for evaluation of a 6-day history of gradually worsening cough, loss of energy, and loss of appetite, which put a damper on his family's visit to Disneyland. He has had a fever for the past several days and has been drinking some, but his mother is not sure how much. His brother had a similar flu-like illness of late. You see in the chart that the patient was diagnosed by your part­ ner with type 1 diabetes 6 months ago, in December, after an emergency department visit for vomiting and diarrhea, which turned out to be diabetic ketoacidosis ( DKA ) . You also see that his weight at that visit was 41 kg. It is 37 kg today. At first glance, he does indeed appear tired. He is tachycardic, but his blood pressure is within normal limits. A blood glu­ cose level is obtained and is too high to be measured. He is promptly sent to the emergency department for further evaluation. Once there, a fluid bolus is given and blood is obtained for laboratory studies. The fluid bolus is repeated. His blood glucose is confirmed to be quite high at 642 mg/dL. Interestingly, the C02 on his metabolic panel is 21 mEq/L. His blood gas measurement shows a comparable HC0 3 of 20 mEq/L. His pH is 7.37 with a PC02 of 4 1 . He does not have DKA, but you wonder if he is suffering from hyperosmolar hyperglycemic state ( HHS ) . Question 9-1

Which of the following is NOT a common laboratory finding in a patient with HHS?

A) B) C) D) E)

15



F LU I D A N D E LECTROLYTE M ETABOLI S M

31 7

Blood glucose greater than 600 mg/dL. Increased serum ketones. Elevated serum osmolality. Pseudohyponatremia. Normal pH.

Discussion 9-1

The correct answer is "B:' This is one of the distinguishing factors between HHS and DKA. HHS is not associated with an acidosis, which in DKA is caused by serum ketoacids, more commonly referred to as serum ketones. HHS is a rare entity that most commonly occurs in type 1 diabetic patients in the setting of another illness, usually something severe such as sepsis or a significant bacterial pneumonia-the type that you can see without the help of a radiologist. The exact pathophysi­ ology has not been worked out, but it can occur when a diabetic patient has not been receiving insulin and has been drinking a lot of sugar-containing drinks, not unlike our little friend in the vignette. Pseudohyponatremia is a result of hyperglycemia, which occurs in both HHS and DKA. Since the serum glucose contributes to serum osmolality, the blood glucose should be lowered slowly so that cerebral edema does not ensue. Even if the serum potassium is elevated, patients with HHS very likely will have a total body potassium deficit, and insulin will drive K+ into the intracellular space, so hypokalemia should be antic­ ipated. Aiming to rectify the serum glucose and the fluid deficit over 48 hours is appropriate, but probably should not occur without the presence and input of an endocrinologist and an intensivist.

It is August in Arizona, and a 9-day-old girl is brought to an urgent care facility for evaluation because she has not been feeding well for several days. She has solely been breastfed. There is no other bothersome history: No fever, no rash, no exposure to ill persons, no concerning information. The urgent care physician is worried when she sees the neonate and sends her to the emergency department ( ED ) for treat­ ment. In the ED she appears tired but not truly lethargic in the medical sense; an IV line is placed, blood is obtained for laboratory studies, and she is given a 20 mL/kg normal saline bolus. After obtaining the history, the ED physician decides that the neonate would be best served by admission, and you, as the pediatrician on call, agree to take over her care. She improves following fluid administration in the ED, but when you arrive shortly thereafter, you are told that her serum sodium is 1 65 mEq/L. She appears well and her vital signs are within normal limits; the only other exam finding of note is mild jaundice. Her mother tells you that her birth­ weight was 3.5 kg, and you notice that her weight in triage was 3.0 kg. The child's mother also asks if it is normal for the child to sweat while feeding, as she has done occasionally since birth.

318

MCG RAW-H I LL E D U CATION S P E C I A LTY BOA R D REVI EW: P E D I ATRICS

water, although it is not an uncommon practice. Given a history of inadequate oral intake, as in this case, it is most likely that the neonate simply has hypernatremic dehydration caused by poor intake and possibly an abnormal breast milk sodium con­ centration. It is not possible to eliminate diabetes insipidus as a possibility in this patient without adequate testing, starting with urine and serum osmolalities, but diabetes insipidus is rare and this history represents somewhat of a classic scenario for simple hypernatremic dehydration. (See Figure 1 5-3.)

Question 1 0-1

What most likely is the principal cause of this child's hyper­ natremia (serum sodium > 145 mEq/L) ?

A) B) C) D) E)

Lack of supplemental water. Diabetes insipidus. Poor feeding leading to dehydration. Neglect or abuse. Excessive sweating and insensible fluid losses.

Discussion 1 0-1

The correct answer is "C:' Hypernatremia (serum sodium > 145 mEq/L) is not common in children, but a general pedia­ trician may encounter this scenario. Presenting signs include irritability, lethargy, fever, vomiting, and seizures. Addition­ ally, these infants can have a high-pitched cry and tachypnea. We know from this neonate's weights that she is not optimally hydrated; a healthy 9-day-old should be somewhat close to birthweight. Her weight plus the mother's concern about feed­ ing should alert you to the presence of a problem. The amount of breast milk that the child is ingesting, or not ingesting in this case, is the most likely issue. If the mother appears reasonably healthy, the breast milk will also be reasonably healthy, but in some cases of breastfeeding failure the sodium content of breast milk has been known to be high. Some component of increased insensible losses would be expected for an infant in a warm envi­ ronment, but it is unlikely that this neonate's inadequate weight could be accounted for by insensible losses alone. Sweating dur­ ing feeding can be a tip-off to the presence of heart disease in infants but is not a specific finding; all infants have the capacity to sweat. Normal infants also have an intact thirst mechanism but are dependent on caregivers for their fluid intake. Neglect or abuse is, and should be, on the differential for just about any medical issue in a child, but is not the most likely issue with our patient, given a plausible history that is consistent with the findings. There is no American Academy of Pediatrics (AAP) recommendation for a breastfed infant to receive supplemental

Question 1 0-2

What findings would be expected if the child had hypernatre­ mia due to diabetes Insipidus (DI)?

A) B) C) D) E)

High serum osmolality, low urine osmolality. Low serum osmolality, low urine osmolality. High serum osmolality, high urine osmolality. Low serum osmolality, high urine osmolality. Her serum sodium will always be abnormal.

Discussion 1 0-2

The correct answer is "A:' Serum osmolality is mainly determined by the serum sodium level, as reflected in the equation by which a serum osmolality is calculated, and would be expected to be high in any hypernatremic individual. The problem in DI is a lack of antidiuretic hormone or the lack of a proper response to antidi­ uretic hormone from the kidneys, so a person with DI will uri­ nate an excessive volume that will give them a tendency toward hypernatremic dehydration. For reference, urine osmolality most frequently is between 300 and 800 mOsm/kg but can range from 50 to 650 mOsm/kg in a neonate (infant younger than 1 month of age), the upper limit of which increases to 1 500 mOsm/kg in older children. In a dehydrated child, the kidneys should be hold­ ing onto whatever water they can, producing a concentrated urine with a high osmolality, but a lack of ADH or a lack of a response to ADH makes that impossible. Children with DI who can drink enough to overcome a lack of ADH can have a normal serum

1

Hypernatremia (Na+ > 1 45 mEq/L)

I

1

iWeight excess Na+

1

I

I

I atrogenic Exogenous Na+ Hyperaldosteronism

Labs

F I G U R E 1 5-3. Causes of hypernatremia.

1

I J.Weight excess H20 loss I

1

Kidney losses - Diabetes insipidus - Postobstructive diuresis - Di u retics

1

iUrine Na+ i U rine volume J.Specific gravity

I nadequate water intake - l mparised th rist - No access - Neglect

I nsensible losses - G1 - S kin - Respi ratory



J.Urine Na+ J. U rine volume !Specific gravity

C H A PT E R 1 5



F LU I D A N D E LECTROLYTE M ETABOLI S M

31 9

sodium level. And, of course, children who are given a properly prescribed ADH analog such as desmopressin (DDAVP) will most often also have a normal serum sodium.

D) Normal saline. E) D 5 \4 NS.

Question 1 0-3

The correct answer is "C:' LR has an osmolality of 275 mOsm! kg, D 5W has an osmolality of 277, normal saline has an osmo­ lality of 308 and D5 \4 NS has an osmolality of 354, all of which can be considered isotonic because they have an osmolality that is reasonably close to normal serum osmolality, which is 275 to 295 in a healthy, normal child. D 5 \t2 NS has an osmolality of 43 1 , which is sufficiently hypertonic that we can call it hypertonic. The upper limit of fluid osmolality acceptable for peripheral IV infusion is generally considered to be 900 mOsm/kg. For refer­ ence, standard TPN has an osmolality of over 1 800, which is why TPN should not be given through a peripheral IV catheter. Now, back to our hypernatremia quagmire. Isotonic also means "no solute free water:' which is a good notion to remember, given that \t2 NS contains 50% solute free water, and \4 NS contains 75% solute free water. This makes our calculations somewhat less taxing. A child with a 500 mL solute free water deficit would need 1 L of \t2 NS or 750 mL of \4 NS to replace the solute free water deficit. The initial rate should give the child half the deficit in 24 hours plus maintenance needs. Losses in addition to the usual insensitive losses, such as with a drainage tube (eg, extra­ ventricular drain or chest tube), will need to be replaced as well. The type of fluid to be used is a matter of opinion and depends on the situation. A sensible approach is the following: D5 \t2 NS for hypernatremia associated with Na+ and water loss such as with gastroenteritis, and D5 \4 NS for hypernatremia associated with free water loss alone such as with breastfeeding failure or DI. The sodium level probably should not be changed by more than 0.5 mEq/L/h. Experiments to determine the most rapid safe rate of correction are not likely to get past a well-intentioned institu­ tional review board (IRB), so the optimal rate will probably never be known. Using a child's dehydrated weight underestimates his or her water deficit, but our calculations are estimates based on estimates, rife with opportunity for error, so calculate a starting rate then set forth and measure the child's sodium frequently. It is late and we are tired, so let's not work through an example.

Discussion 1 0-5

What should be the next step in this patient's medical management?

A) B) C) D) E)

Give another NS bolus. Calculate the child's free water deficit. Give her a \t2 NS bolus. Check the sodium content of the mother's milk. Start her on IV fluid at 1 . 5 times the maintenance amount.

Discussion 1 0-3

The best answer is "B:' It is hard to deny the diagnostic and ther­ apeutic utility of an NS bolus, but in this case, with its limited differential, we first need to figure out how much water the child needs to attain euvolemia and eunatremia, so the best option is "B:' An elevated milk sodium level has been associated with insufficient lactation, but this is not routinely evaluated. There is no rationale for option "E:' Question 1 0-4

Roughly what percentage of a person's body weight is accounted for by water?

A) B) C) D) E)

75% in a newborn, 65% in an adult. 75% in an 8-year-old, 45% in an adult. 90% in a newborn, 65% in an adult. 55% in a newborn, 70% in a teenager. It is too variable to accurately say, and we do not really need to know.

Discussion 1 0-4

The correct answer is "A:' However, option "E" certainly is a tempting answer. It may not come up very often, but the num­ ber is used to calculate the free water deficit in a hypernatremic dehydrated patient in need of iV fluid, so it is a good number to keep in mind. The estimates vary by age, situation, fitness level (ie, body fat), and by your informational source, but we need to start somewhere. Let's say 60%. Here is the salient equation: Free water deficit Current total body water x ( [Current plasma Na/ 140] - 1 ) There i s n o way to avoid some mental gymnastics in the deter­ mination of the fluid needs of a hypernatremic patient. You need to figure out how much electrolyte free water is needed to dilute the patient's sodium to the point that the serum sodium is normal, or 140 mEq/L, as in the equation. Then you give them the water they need, but in the form of an available IV fluid, without forgetting to provide their ongoing maintenance needs and to replace their ongoing losses. But first, answer the next question. =





Helpful Tip

Replacing the fl uid deficit i n a dehyd rated child ta kes

I 1 1 r longer that you might think if you fol l ow the book. For

isonatremic and hyponatremic dehydration: replace half the deficit over the first 8 hou rs, then the remaining half

over the next 1 6 hours. I n hypernatremic dehyd ration: replace half the deficit over the fi rst 24 hou rs, then the remaining half over the next 24 hours.

Question 1 0-5

Which of the following IV fluid concoctions is not isotonic?

A) Lactated Ringer solution (LR) . B) D SW. C) D 5 \t2 NS.

Being the unfortunate recipient of the night shift, you accept a patient from the emergency department (ED) with the modicum of enthusiasm that you can garner at 02: 1 5 . Just adopted from a mother in a neighboring state, the patient

320

MCG RAW-H I LL E D U CATION S P E C I A LTY BOA R D REVI EW: P E D I ATRICS

is a male infant who is 30 days old, had a fever of 38.3°C { 1 0 1 oF) earlier that day at home, and was dutifully brought to the ED for evaluation. According to the ED physician, the infant appears well and has no other notable history, other than not feeding as well as the day before and having fewer wet diapers. In the ED, blood, urine, and cerebrospi­ nal fluid ( CSF) were obtained for laboratory analysis. You discuss the lab results with the ED physician, who details the CBC result: WBC 1 1 ,200/mm\ hemoglobin 1 5.4 g/dL, and platelet count 240,000/mm 3 • The differential shows 56% neutrophils and 37% lymphocytes and a smattering of other uninteresting white cells. The ED physician reports that the metabolic profile was unremarkable with the excep­ tion of mildly elevated liver enzymes. The CSF shows a WBC count of 1 5,000 and a red blood cell (RBC) count of 1 432, and the ED physician comments that the spinal tap was not his smoothest effort. The urinalysis was unremarkable. An IV line was placed and the infant was given a dose of IV cefotaxime and a normal saline bolus, followed by IV fluid at a maintenance rate. You, apparently more aware of your hospital's febrile infant protocol, are somewhat alarmed by the elevation of the liver enzymes (AST 1 45 units/L, ALT 192 units/L) , and you inquire about the possibility of her­ pes simplex virus (HSV) exposure. The biologic mother's history is not known to the adoptive parents in detail, but they were told that the mother was healthy and the deliv­ ery was an uncomplicated caesarian section. The newborn required supplemental oxygen for "several hours" but was able to leave the nursery within 2 days. No cold sores are known to have been in proximity to the infant, but you rec­ ommend adding an HSV polymerase chain reaction (PCR) test to the patient's CSF testing as well as starting the infant on a course of acyclovir. These items are accomplished and the infant later arrives on the floor in good shape. The vitals are normal, the infant is vigorous, and you are unable to locate any skin or mucosal lesions consistent with an HSV infection. After discussing your thoughts with the parents, the infant is set on medical autopilot and you head off to the call room. Fast forward 36 hours. The infant has done well with the exception of not feeding very well. He was given acetamino­ phen for fever early in his stay, but he has not had a fever for about 1 2 hours. The blood, urine, and CSF cultures are showing no growth to date. There was a slight snafu with the spinal fluid HSV PCR, which was not set up until the day after his admission, so the result is still pending. The infant is still being given acyclovir and cefotaxime. He appears well and has essentially returned to his usual self, with the exception of suboptimal feeding. His urine output over the last 24 hours is 2.4 mL/kg/h and his weight has increased from 3.24 to 3.27 kg. Given his continued need for IV fluid and your curiosity about the value of his liver enzymes, you order a metabolic profile. Lo and behold, everything, including his liver enzymes, is normal with the exception of a BUN of 42 mg/dL and creatinine of 1 .2 mg/dL. Yikes! What have you done?

Question 1 1 -1

What is the most likely cause of the child's elevated BUN and creatinine?

A) B) C) D) E)

HSV interstitial nephritis. Prerenal acute kidney injury. Nephrotoxic acute kidney injury. Hypoxic/ischemic acute kidney injury. Obstructive uropathy.

Discussion 1 1 -1

The best answer is "C:' This is a case of acute kidney injury (AKI), which is an acute loss of kidney function, in this case manifested by an acute rise in BUN and creatinine, although AKI covers the spectrum from relatively simple issues such as the one presented here all the way to the necessity for dialysis. All of the options would be good answers for the differential diagnosis list during morning report, but when a patient has received a medication known to be nephrotoxic and has no other obvious historical sug­ gestion of an offending process, it is reasonable to point a finger at the medication. Acyclovir is just such a medication. Therefore the most reasonable answer is nephrotoxic AKI. Just for record, however, prerenal processes are the most common form of acute renal insufficiency in children, in which the kidneys are intrinsi­ cally functional. We have been told that the infant's urine output is a robust 2.4 mL/kg/h, which further helps us in our quest for medical truth. Nephrotoxic AKI usually is a nonoliguric process; normouric, we could say. Obstructive uropathy in a child most commonly manifests with a resultant urinary tract infection (UTI), which is not the case for this infant, although without suf­ ficient testing (ie, imaging), it could not absolutely be ruled out. Rapid elevation of the creatinine and BUN, as in this case, argues strongly against an obstructive uropathy, which would more likely have a more insidious course or be found after a UTI. This, and a more obvious offending agent, should be enough to avoid further testing at this point. As for the other potential answers, HSV interstitial nephritis is uncommon and is simply not the best of the given options, and the infant's history, though not known in detail, does not heartily support a diagnosis of hypoxic/ischemic AKI, which is much more commonly seen in premature or criti­ cally ill infants and children. Question 1 1 -2

What should the next step in the infant's management be? A) Fluid restriction to two th irds of th e maintenance level.

B) C) D) E)

Pester the lab for the result of the HSV PCR. Lower the dose of the acyclovir. Give a dose of furosemide. Genetic testing to further determine susceptibility to wors­ ening renal injury.

Discussion 1 1 -2

The correct answer is "B:' What we are getting at here is a com­ mon theme for medicine in general: removal of the offend­ ing agent. The child's AKI, as discussed, is most likely due to the acyclovir, so the question becomes: "Do we really need the medication?" Consequently, the next step in this case is to

C H A PT E R

determine the necessity of the medication. Lowering the dose of the offending medication, particularly with acyclovir, is an appropriate measure if the medication is deemed essential to the care of the child and the kidneys are not catastrophically failing, but in our case the acyclovir is probably no longer necessary. Whether it was truly necessary from the beginning is a convo­ luted story, which is covered in another chapter of this book. In a case such as this, stopping the medication and increasing the infant's fluid intake would be appropriate. Nonsteroidal anti­ inflammatory drugs are also common AKI offenders, but this little guy is younger than 6 months old so its acetaminophen for him should his fever return. Let us say that the lab has com­ pleted the study by the time you call and that it is negative, so you are able to discontinue the acyclovir. As for option "!\' and fluid restriction, you are running the risk of adding a prerenal confounder to the infant's problem, so that is not a good answer. For prerenal AKI, increasing the infant's fluid intake would be appropriate. Diuretics such as furosemide may play a role in severe, oliguric renal failure but are not likely to be beneficial in this case. Genetics probably does play a role in a child's suscep­ tibility to the development of AKI, but genetic testing to further define that susceptibility is unlikely to be of benefit here. Say, for the sake of argument, that despite an appropriate input of fluid the infant's urine output for the previous 24 hours had been 0.3 mL/kg/h, and that was the reason that you wanted to check chemistries, as well you should. Say the BUN and creatinine results were the same as previously mentioned and the infant's weight had increased to 3.4 kg (a gain of 730 g) . Now it would be imprudent to increase his fluid intake, unless you would like to add pulmonary edema, hyponatremia, hypertension, or any mixture of these to the problem list. Detailed and frequent assessments of the infant's weight, blood pressure, input, out­ put, and electrolytes, and frequent physical examinations would be in order. The most important of these is the infant's weight, which should perhaps be taken more than once a day, much to the nursing staff's chagrin. And, oh yes-you probably should get a nephrologist involved if you have gotten this far.

� QUICKQUIZ Common physiologic derangements seen in acute renal fail­ ure include all of the following EXCEPT:

A) B) C) D) E)

Hyponatremia. Hyperkalemia. Anemia. Metabolic acidosis. Hypocalcemia.

Discussion

The answer we seek is "C:' Anemia is the result of a decrease of renally excreted erythropoietin, which is a feature of chronic renal failure-as is osteodystrophy, while we are on the subject.

15



F LU I D A N D E LECTROLYTE M ETABOLI S M

321

All of the other options are seen in both acute and chronic renal failure, as a result of the kidneys not doing what they normally do. If the kidneys do not or cannot excrete acid, metabolic acidosis results; if they cannot excrete water, fluid overload and hyponatremia result; and if they cannot excrete potassium? Well you get the idea. Ultimately, as we should all know, the go-to intervention for severe renal failure is dialysis. Question 1 1 -3

Speaking of dialysis, which of the following is NOT an indi­ cation for dialysis?

A) Serum bicarbonate persistently below 10 mEq/L. B) Persistent azotemia. C) Clinically significant electrolyte disturbance refractory to other management. D) Serum creatinine in excess of 10 mg/dL. E) Intractable fluid overload leading to, in particular, cardiac compromise. Discussion 1 1 -3

The correct answer is "B:' Dialysis should occur when the kid­ neys have sufficiently demonstrated an inability to perform their usual functions to a degree that is potentially life-threatening. In keeping with that idea, persistent azotemia, that being a high BUN without symptoms, is not specifically an indication for dialysis. Though it is not difficult to state absolute indications for dialysis, in reality the decision is a bit more nuanced and frequently based on a combination of factors. Constitutional factors such as fatigue and anorexia also play a role in the initia­ tion of dialysis, although in less of an acute sense. B I B L I O G RA P H Y

Awad S, Allison SP, Lobo DN. The history of 0.9% normal saline. Clin Nutr. 2008;27(2) : 1 79- 188. Brenkert TE, Estrada CM, McMorrow SP, Abramo TJ. Intra­ venous hypertonic saline use in the pediatric emergency department. Pediatr Emerg Care. 201 3;29: 7 1 . Moritz LM, Ayus JC. Disorders o f water metabolism in children: Hyponatremia and hypernatremia. Pediatr Rev. 2002;23( 1 1 ) :371 -380. Moritz ML, Ayus JC. Preventing neurological complica­ tions from dysnatremias in children. Pediat Nephrol. 2005;20 : 1 687- 1 700. Ril ey AA, Arakawa Y, Worley S, Duncan BW, Fukamach i K. Circulating blood volumes: A review of measurement techniques and a meta-analysis in children. ASAIO ]. 201 0;56( 3) :260-264. Sarnaik AP, Meert K, Hackbarth R, Fleischmann L. Manage­ ment of hyponatremic seizures in children with hyper­ tonic saline: A safe and effective strategy. Crit Care Med. 199 1 ; 19( 6):758-762 Verbalis JG, Goldsmith SR, Greenberg A, et al. Diagnosis, evaluation, and treatment of hyponatremia: Expert panel recommendations. A m J Med. 20 13; 126( 10 suppl 1 ) : S 1 -4.

This page intentionally left blank

Ped iatric Gastroenterol ogy

16

D i n a A I -Zu bei d i a n d E l iza bet h U tterson

Question 1 -2

You are evaluating a child younger than 2 years of age with acute abdominal pain, fever, and peritoneal signs. Question 1 -1

Which of the following is high on your differential diagnosis list?

A) B) C) D) E)

Intussusception. Hirsch sprung disease. Henoch -Sch Onlein purpura (HSP) . Incarcerate d h ernia. Perforate d appen dix.

Discussion 1 -1

The correct answer is "E:' All of th e liste d options can cause severe acute ab d ominal pain, but a perforate d appen d ix causes peritoneal signs. Creating an age-appropriate d ifferen­ tial diagnosis is th e most important step in making th e right diagnosis and planning th e d iagnostic evaluation. This avoi d s th e "sh otgun" approach . The ch ief complaint of ab d ominal pain may seem overwh elming but a few key d etails can h elp in narrowing th e differential: ( 1 ) acute or ch ronic pain, (2) age of th e patient, (3) presence or absence of fever, (4) degree of impairment, and (5) presence of peritoneal signs. Peritoneal signs of a perforate d appen d ix inclu d e guarding an d reboun d tenderness. The patient often lays still and winces with th e slightest movement. A good test is to inconspicuously bump the bed to see if it causes pain. Peritoneal signs are never nor­ mal or good. You must rule out an acute surgical abdomen! (See Table 1 6- 1 . ) An ultrasound is performed, which confirms the diagnosis of a ruptured appendix with abscess formation. Appendici­ tis usually presents with periumbilical abdominal pain that moves to the right lower quadrant in less than 24 hours.

This typical presentation may be absent in all of the following EXCEPT:

A) B) C) D)

Children younger than age 2 years. Retrocecal appendix. Perforated appendix. Fecalith-impacted appendix.

Discussion 1 -2

The correct answer is "D:' Typically, an appen d icitis results from obstruction of th e appendiceal lumen by a fecalith . The appendix becomes inflamed, irritating the peritoneum. As the peritoneum becomes inflamed, the pain localizes to the right lower quadrant (McBurney point) with rebounding, guarding, and a positive Rovsing sign (palpating the left lower quadrant hurts the right lower quadrant) . The child may have vomiting and fever. He or she will prefer to lie still. The parents may tell you that going over bumps on the car ride caused pain. Ask the child to hop; he or she will not, as it is painful. Recognizing a classic presentation is easy; it is the atypical presentations that will burn you. Less than 2% of cases of appendicitis occur in children younger than age 2; however, more than 70% of cases present with perforation. Toddlers and infants present with diffuse abdominal pain accompanied by guarding, vomiting, and diarrhea that is frequently misdiagnosed as gastroenteri­ tis. A child with a retrocecal appendix presents with general­ ized abdominal pain. A positive iliopsoas sign (pain with right hip extension) is associated with retrocecal appendicitis. The rectal exam will be painful as you push on the inflamed appen­ dix. With perforation, the pressure in the appendix is released and the abdominal pain suddenly improves for a short time period. It is a clinical diagnosis, but in equivocal or typical presentations imaging may be needed. In children, computed tomography ( CT) scan has a sensitivity of greater than 90% and a specificity of 85% to 90%. Ultrasound has a sensitiv­ ity of 88% and specificity of 94%. For an ultrasound scan to be useful, the appendix must be seen, which is very operator dependent. 323

MCG RAW-H I LL E D U CATION S P E C I A LTY BOA R D REVI EW: P E D I ATRICS

324

TABLE 1 6- 1 D I F F E R E N T I A L D I AG N O S I S O F ACUTE A B D OM I NA L PA I N BY AG E

Organ System

I nfa nt

Child

S u rg i ca l

Appendicitis

Appendicitis

Appendicitis

I ntussusception

I ntussusception

Tra u m a

M a l rotation with m i d g ut volvu l u s

I ncarcerated i n g u i n a l hernia

I ncarcerated i n g u i n a l hernia

Tra u m a with perforation, hemato ma

Adolescent

Omental i nfa rction or torsion Gastroi ntest i n a l

Acute chol ecystitis G a l l sto nes Pa ncreatitis Hepatitis I nfl a m m atory bowe l d i sease Hemolytic u remic syn d rome

Renal Gen itou r i n a ry Gynecologic

Testicu l a r to rsion

Pye l o n e p h ritis

Kid ney sto ne

Kidney sto ne

U ri n a ry tract i nfecti o n

Testic u l a r torsion

U ri n a ry tract i nfecti o n

U ri n a ry tract i nfection

Ova r i a n torsion

Ova r i a n torsion Ecto pic preg na ncy Pe lvic i nfl a m m atory d i sease R u ptu red ova r i a n cyst Pneumonia

P u l m o n a ry Endocrine I nfecti o u s

Gastroe nteritis

Pneumonia

Dia betic ketoacidosis

Dia betic ketoacidosis

Gastroe nteritis

Gastroe nteritis

Co l itis

Co l itis

Streptococca l p h a ryngitis

Psoa s a bscess

Mesente ric aden itis R h e u m atolog ic He matologic/On cologic

H e noch-Scho n l e i n p u r p u ra M a l i g n a ncy

Psyc h i atric

Va sa-occ l u sive crisis

Va sa-occl u sive crisis

Porphyria

Porphyria

M a l i g n a ncy

M a l i g n a n cy

Somatic or fu n cti o n a l

Somatic or fu ncti o n a l Conversion d i sorder



Helpful Tip

:5.� The

r1 1r

m n emonic PANT-Pa in, Anorexia, Nausea, Tem peratu re-describes the sym ptoms of a n acute a ppendicitis.

She has no fever or diarrhea. On exam, she is afebrile, obese, and uncomfortable. She has tenderness in the right upper quadrant. Her direct bilirubin, GGT, and transaminase levels are elevated.

Question 2-1

Of the list below, the most likely diagnosis is: A 1 3-year-old African American girl presents with acute right upper quadrant pain radiating to the shoulder. She had one episode of nonbloody, nonbilious emesis and is nauseated.

A) B) C) D)

Gastroenteritis. Choledocholithiasis. Splenic rupture. Pancreatitis.

C H A PT E R 1 6

Discussion 2-1

The correct answer is "B:' Choledocholithiasis refers to gall­ stones in the common bile duct. Pain, or "biliary colic;' arises when a stone temporarily obstructs the biliary tree. Children and adolescents present with right upper quadrant pain, vom­ iting, and sometimes jaundice. The stone should be removed surgically. Stone removal by endoscopic retrograde cholangio­ pancreatography (ERCP) is becoming more common. A right upper quadrant ultrasound is performed, which shows a thickened gallbladder wall. The nurse calls you to report that your patient has a new fever of 38.5°C ( 1 0 1 .3°F) . The technician notes that when she pushed the transducer into the right upper quadrant, the patient caught her breath.



Disease

Sym ptoms

Cholelithiasis

G a l l stones with i n g a l l bladder May be asymptomatic B i l i a ry col i c < 6 h o u rs

Choled ocholithiasis

Abnormal l a borato ry tests U rgent re mova l of stone Cholecystitis

I nfl a m mation a n d i nfection of g a l l bladder B i l i a ry col i c > 6 h o u rs Fever, peritoneal s i g n s N o t j a u n d iced U rgent chol ecystectomy Bacterial i nfection of g a l l blad­ der and b i l i a ry tree

C h o l a n g itis

Fever, peritoneal s i g n s

Discussion 2-2

The correct answer is "B:' The patient has acute cholecystitis, infection, and inflammation of the gallbladder, with a posi­ tive Murphy sign. Acute cholecystitis is usually associated with gallstones. Acalculous cholecystitis (no gallstones) occurs in ill patients with serious infections such as streptococcal sepsis. Both present with prolonged right upper quadrant pain that radiates to the back or right shoulder, fever, and leukocytosis. Although options "C;' "D;' and "E" are important, this patient needs an urgent cholecystectomy. "Biliary colic" can be seen with biliary stones and acute cholecystitis. With stones the pain typically resolves within 6 hours, but it is more persistent with acute cholecystitis. Fever, abnormal laboratory tests, peritoneal signs, or a combination of these findings, are more typical of cholecystitis.

J a u n d iced Anti biotics and u rgent d ucta l deco m pression 'Right u pper quadrant pa i n that may rad iate to the back or right shoulder is common to a l l .

ducts, or both) from obstruction of the bile ducts (stones, stenosis, choledochal cyst) . Signs include right upper quadrant pain, fever, and jaundice. Blood cultures are frequently positive. Treatment includes IV antibiotics, IV fluids, and relief of the obstruction. (See Table 1 6-2.) •





Helpful Tip

B i l i a ry colic from cholecystitis lasts longer a n d is

1 1 1r accompan ied b y fever a n d peritoneal sig ns, w h i c h a re

G a l l stone i n co m mo n b i l e d u ct J a u n d ice

Question 2-2

ERCP. Cholecystectomy. Intravenous (IV) antibiotics. IV opioids. IV fluids.

Helpful Tip

=tJl Consider

the g a l l bladder lazy in hyd rops of the

i1 1r g a l l bladder. I t is d i stended a n d contracts poorly but is not i nfected, i nfla med, or fi l l ed with stones. The

cond ition is self-l i m ited, caused by a systemic i l l ness, and does not req u i re remova l of the g a l l bladder. A

not characteristic of b i l i a ry colic from cholelithiasis.

classic exa m p l e is Kawasa ki d isease. Question 2-3

What is NOT a complication of cholelithiasis/choledocho­ lithiasis?

A) B) C) D)

Cholangitis. Pancreatitis. Sepsis. Gastritis.

Discussion 2-3

The correct answer is "D:' Cholangitis is a bacterial infection of the gallbladder and biliary tree (intrahepatic or extrahepatic

325

TABLE 1 6-2 B I L I A RY TRACT D I S EA S E"

What is the next step in her management?

A) B) C) D) E)

P E D I ATRIC GASTRO ENTEROLOGY

� QUICKQUIZ Which is NOT a risk for gallstones in pediatric patients?

A) B) C) D) E)

Obesity. Family history of gallstones. Inflammatory bowel disease. Sickle cell disease. Parenteral nutrition.

MCG RAW-H I LL E D U CATION S P E C I A LTY BOA R D REVI EW: P E D I ATRICS

326

Discussion

The correct answer is "C:' In a young child who is not obese or with strong family history of choledocholithiasis, consider the underlying hematologic process (hemolytic anemias) . Pig­ mented stones are associated with hemolysis. Cholesterol stones are common in children who are school-aged and older. The laboratory results for your patient with acute cholecystitis keep rolling in as the afternoon ticks away. Her lipase is elevated at 300 units/L. You recognize that she also has pancreatitis.

TABLE 1 6-3 CAU S E S O F ACUTE PA N C REATITIS Anato m i c

A n n u l a r pan creas Pa ncreatic d ivis u m

Metabolic

Hyperca lcem i a Hyperl i pi d e m i a Hypertrig lyceride m i a I n born e rrors o f meta b o l i s m

B i l i a ry tract

Cholelithiasis

Question 2-4

Choledocholithiasis

All the following are highly suggestive of acute pancreatitis in children EXCEPT:

B i l e d uct ste nosis

A) Acute-onset epigastric pain with radiation to the back or left scapula. B) Elevation of amylase and lipase more than three times the upper limit of normal. C) Abdominal ultrasound findings of choledocholithiasis and dilated pancreatic duct with edema. D) History of episodic suprapubic abdominal pain. Discussion 2-4

The correct answer is "D:' Diagnosis of acute pancreatitis requires at least two of the following three criteria: ( 1 ) Abdomi­ nal pain consistent with pancreatitis, (2) elevation of amylase and lipase at least three times the upper limit of normal, and (3) radiographic evidence of pancreatitis. Too bad you did not order an abdominal ultrasound at the same time as the right upper quadrant one.

Drug/toxin

Antiseiz u re med icatio n s A n t i psyc hotics Ketog e n i c d i et Alcohol

I nfecti o u s

Vi ra l (Epstei n-Ba rr, cytomega lovi rus, enterovirus, i nfl uenza, m u m ps)

Auto i m m u ne/ rheumato logic

Autoi m m u ne d i sease Kawa saki d i sease L u p u s erythematosus H e noch-Scho n l e i n p u r p u ra I nfl a m m atory bowe l d i sease

Heredita ry

PRSS 7, SPINK I, and CFTR gene

m utatio n s Cystic fi bros i s

Other

Hemo lytic u re m i c syn d rome

You do a quick mental check to make sure you are not missing another problem. Could gallstones be the common denomi­ nator of everything? You start to formulate yet another differential diagnosis list.

Tra u m a B u rn s I d i o path ic

Question 2-5

Which of the following is the most common cause of acute pancreatitis?

A) B) C) D) E)

Viral infection. Hyperlipidemia. Cholelithiasis. Medications. Burns.

Discussion 2-5

The correct answer is ''A:' Viral etiologies include influenza, entero­ virus, Epstein-Barr, cytomegalovirus, mumps, and varicella-zoster viruses. Remember, patients with acute pancreatitis can get really sick and develop sepsis, shock, third spacing/capillary leak, mul­ tisystem organ failure, and pleural effusions. Options "B" through "E" are also causes of acute pancreatitis. (See Table 16-3.) •



1 1 1r

Helpful Tip

Did you know that scorpion bites can ca use acute pa ncreatitis? N o w you do.

� QUICKQUIZ True or False: Chronic pancreatis is reversible.

A) True. B) False. Discussion

The correct answer is "B:' Acute pancreatitis is reversible. Chronic pancreatitis causes progressive, irreversible, inflamma­ tory destruction of the pancreas, eventually leading to pancre­ atic insufficiency (endocrine and exocrine) . Call a GI specialist. Reoccurrence is not a good sign. (See Table 1 6-4.) •



1 1 1r

Helpful Tip

C h ronic pancreatitis may be the presenting sign of a lcohol a buse in adolescents.

C H A PT E R 1 6

TABLE 1 6-4 CAU S E S OF C H R O N I C PA N C R EATITIS Anato m i c

A n n u l a r pa n creas Pa ncreati c d ivis u m

Meta b o l i c

Hyperca lcem i a Hyperl i pi d e m i a I n born e rrors o f meta b o l i s m



P E D I ATRIC GASTRO ENTEROLOGY

327

These are tissue bands that cross and obstruct the duodenum). Any infant who presents with bilious emesis is an emergency case. There is also an atypical presentation. (Are you surprised?) Intermittent twisting with pain and vomiting may occur in older children. It is frequently misdiagnosed as cyclic vomiting. The infant arrives. She appears ill, with a distended abdo­ men that is tender to the touch. She is tachycardic with cool extremities. An IV is placed and she receives fluid resuscita­ tion. You call the surgeons.

B i l i a ry tract

Choledocholithiasis

Drug/tox i n

Alcohol

Question 3-2

Smoking

The best modality for diagnosis of malrotation with midgut volvulus is:

B i l e d uct ste nosis

Auto i m m u ne/ r h e u m ato logic

Autoi m m u ne Sj ogren syn d rome L u p u s erythematosus I nfl a m matory bowe l d i sease

Heredita ry

PRSS 7 , SPINK 7 , and CFTR gene

m utatio n s Other

C h ro n i c ren a l fa i l u re Cystic fi bros i s I d i o path ic

A) B) C) D)

Barium enema. Upper gastrointestinal (UGI) series. Abdominal ultrasound. Upper endoscopy (esophagogastroduodenoscopy [EGD] ) .

Discussion 3-2

The correct answer is "B:' A UGI series diagnoses malrota­ tion without volvulus. Anytime you think volvulus, your heart rate should increase. Time is bowel. Call the surgeons. Do not wait. Look for a beak or corkscrew sign with volvulus. For mal­ rotation, check the location of the duodenal jejunal flexure (ligament of Treitz). It will be on the right with malrotation. (See Figures 16-1 and 1 6-2.)

The mother of a 3-week-old infant calls the office to report the following: Thirty minutes ago the infant started vomit­ ing; it looks "grass green''; she cries if you touch her belly and is starting to get sleepy. You tell her to drive immediately to the emergency department: this is an emergency. You are worried about malrotation with volvulus. Question 3-1

Most patients with malrotation present at what age?

A) B) C) D)

After 1 year of age. In the first month of life. In adolescence. Immediately after birth.

Discussion 3-1

The correct answer is "B:' This infant has malrotation with vol­ vulus-a surgical emergency. Rotational anomalies may become symptomatic at any age; however, more than 80% present in the first month oflife, often in the first week. Risk of acute volvulus is highest in the neonatal period. In malrotation, the intestine fails to rotate in utero and fixate in the correct position. It is adhered by a narrow pedicle of mesentery, which allows the intestine to twist around itself. The cecum ends up in the right upper quad­ rant. The risk is volvulus, most often at midgut. This occurs when the intestine strangulates by twisting, cutting off the blood supply and blocking the flow of fecal contents and gas through the tract. Bad outcomes (perforation, infarction) ensue if it is not surgi­ cally corrected. (You may have heard the term Ladd bands used.

F I G U R E 1 6- 1 . M a l rotation. I n utero the i ntestine fa i l s to rotate a n d fixate i n the correct positions, resulting i n the cecu m i n the right upper q u a d ra nt a n d the t h i rd part of the duodenum a n d jej u n u m to the right of the m i d l i ne. Bands of tissue (Ladd bands) adhere to the cecum, crossing the duodenum, a n d may cause obstruction. (Reproduced with perm ission from Ti nti n a l l i J E, Stapczynski JS, Ma OJ, et a l : Tintinalli's Emergency Medicine: A Comprehensive Study Guide, 7th e d i t i o n . M c G raw H i l l Ed u cation, I n c; 2 0 1 1 . Fig u re 1 24-4, Pg 843.)

MCG RAW-H I LL E D U CATION S P E C I A LTY BOA R D REVI EW: P E D I ATRICS

328

Discussion 4-1

The correct answer is "A:' Intussusception causes acute intes­ tinal obstruction in children younger than age 2 years. Tele­ scoped bowel causes obstruction, blockage of blood flow, and eventual necrosis, perforation, and peritonitis. Most cases occur in infants (ie, before 12 months of age) . There may a his­ tory of preceding viral respiratory illness or gastroenteritis. The classic presentation is colicky abdominal pain, vomiting, "sausage-shaped" abdominal mass, and hematochezia ("cur­ rant jelly" poop), although rarely does this occur. The vomiting progresses from nonbilious to bilious. Infants may present with lethargy concerning for sepsis. Sometimes intussusception is found incidentally on imaging. Question 4-2

Most cases of intussusception involve what region?

A) B) C) D) E) F I G U R E 1 6-2. Volvu l u s. T h i s 1 0-day-old i nfa nt presented with bilious emesis. Her abdominal X-ray shows a d i l ated proxi m a l bowel with m i n i m a l gas past t h a t poi nt, consistent with volvu lus. (Reproduced with permission from Brunica rd i FC, Andersen DK, B i l l i a r TR, et al, eds. Schwartz's Principles of Surgery. 1 Oth ed. New York, NY: McGraw- H i l l Education; 201 5, Fig. 39-1 6.)





1 1 1r

Helpful Tip

Ileocecal (ileocolic) . Colocolic. Jejunojejunal. Ileoileal. Ileo-ileocolic.

Discussion 4-2

The correct answer is "A:' The ileocecal or ileocolic region is most commonly involved. Small bowel intussusception is less common. Pretend for a moment that the patient is 7 years old. In a child of that age, you should consider a lead point causing intussusception.

B i l ious emesis i n a n i nfa nt, especia lly a newborn, is a n

Question 4-3

emergency. I f d u e to volvu l us, emergency s u rgery is needed. If d u e to i ntussusception, emergent red uction

Which is NOT a lead point?

with air contrast enema is needed .

A) B) C) D) E)

Mekel diverticulum. Polyp. Bowel wall hematoma. Duplication cyst. Meconium plug.

Discussion 4-3

A 12-month-old boy is brought to the clinic with a chief complaint of abdominal pain. He appears to be a healthy infant. His mother reports that he had a slight cough last week. Today he has had repeated bouts during which he cries inconsolably and pulls his legs up. He has vomited twice. Between episodes he is well, but the episodes seem to be increasing in frequency. There is no history of fever, diar­ rhea, or rash. Question 4-1

What is NOT a presenting sign associated with this condition?

A) B) C) D) E)

Rash. Vomiting. Lethargy. Abdominal pain. Abdominal mass.

The correct answer is "E:' Intussusception is most commonly idiopathic. Less than 10% of cases involve a pathologic lead point. Children and infants outside the typical age range (3 months to 3 years) are more likely to have a lead point. The lead point acts to drag the bowel inside a distal segment of bowel. Tumors, vascular malformations, and lymph nodes are other lead points. •

Helpful Tip

:5.� Feca l

r1 1r cases

occult blood testing is positive i n 7S% of

of i ntussusception. The cu rra nt j e l ly stool in i ntussusception is a late-a nd bad-fi n d i n g . It

i n d i cates bowel wa l l ischemia and necrosis.

You have your diagnosis and call the interventional radiologist.

C H A PT E R 1 6

Question 4-4

In the management of intussusception all of the following statements are true EXCEPT:

A) Air (pneumatic) enema is safe and fast and uses less radia­ tion, with a success rate of 75% to 95%. B) Hydrostatic reduction with liquid contrast or saline has a high success rate. C) Enema reduction is indicated in children with shock and signs of peritonitis. D) Open surgical reduction is indicated when air/hydrostatic reduction is unsuccessful. Discussion 4-4

The correct answer is "C:' If you strongly suspect an intussuscep­ tion, go for the air enema. Air reduction is safe and successful in the majority of cases. It diagnoses and treats the condition-a two-for-one deal! If unsure, consider an ultrasound, looking for the target sign or bull's eye. If bad findings are present (shock, perforation, peritonitis), do not squirt anything into the rectum. This includes air. Call the surgeon ASAP! Question 4-5

Abdominal mass was mentioned as a sign of intussusception. It was even compared to food, as is everything in pediatrics. If the mass is not a "sausage-shaped" intussusception, what else could it be?

A) B) C) D) E)

Distended bladder. Testicular torsion. Neuroblastoma. Ovarian cyst. Choledochal cyst.



P E D I ATRIC GASTRO ENTEROLOGY

329

D) Celiac disease. E) All of the above. Discussion

The correct answer is "E:'

� QUICKQUIZ In the management of acute intestinal obstruction all the following steps are indicated in your initial management EXCEPT:

A) B) C) D) E)

Bowel decompression using a large-bore nasogastric (NG) tube. Fluid resuscitation. Early surgical intervention in cases of malrotation. Correction of electrolyte abnormalities. All of the above.

Discussion

The correct answer is "E:' The management depends on the eti­ ology. If the cause is not a surgical emergency, a trial of con­ servative management is reasonable in partial obstruction, including options "A;' "B;' and "D:' In patients with malrota­ tion, early surgical intervention (Ladd procedure) is important to prevent volvulus. Patients with volvulus need emergent sur­ gery, fluid resuscitation, and IV antibiotics. Those with intus­ susception require enema reduction. You need to know what is causing the obstruction.

Discussion 4-5

The correct answer is "B:' Surprise, surprise! The differential diagnosis of abdominal masses depends on the patient's age. The big one not to miss is malignancy, which may present as a child of any age with an abdominal mass. Here is a partial list of pediatric abdominal masses: Wilms tumor, constipation, intussusception, gastric distention, hydronephrosis, cystic kid­ ney disease, splenomegaly, hepatomegaly, distended bladder, Ewing sarcoma, rhabdomyosarcoma, neuroblastoma, ovarian cyst, ovarian torsion, enlarged uterus (pregnancy, hydrome­ trocolpos) , choledochal cyst, and lymphoma. Just think of all the structures in the abdomen, then start running through a list of possibilities based on each structure. For example, kidney masses include Wilms tumor, multicystic dysplastic kidney, hydronephrosis, ureteropelvic junction obstruction, and so on.

A 1 2-year-old girl is referred to the gastroenterology clinic for evaluation of chronic abdominal pain. She cannot describe the pain and vaguely rubs the center of her abdo­ men when asked where it hurts. You ask her to point with one finger and she says she can't. She reports the pain as continuous and 10 out of 10. She has missed school. She denies vomiting, fever, diarrhea, or weight loss. The pain is not associated with eating and does not wake her from sleep. On exam, she appears well but claims to be in pain. She is not tachycardic. She winces in pain when you lightly palpate her abdomen but the pain is distractible. She has no oral ulcers or arthritis, and the rectal exam is normal, with no perianal skin tags or fissures. You suspect functional abdominal pain. Question 5-1

� QUICKQUIZ Which condition is associated with intussusception?

A) Cystic fibrosis. B) Henoch-Schonlein purpura. C) Hemolytic uremic syndrome.

Functional abdominal pain is typically associated with which of the following findings?

A) B) C) D) E)

Involuntary weight loss. Significant vomiting. Nocturnal symptoms. Periumbilical location. Fever.

MCG RAW-H I LL E D U CATION S P E C I A LTY BOA R D REVI EW: P E D I ATRICS

330

Discussion 5-1

The correct answer is "D:' The symptoms of functional gastro­ intestinal (GI) disorders (irritable bowel syndrome, function abdominal pain, abdominal migraine) cannot be explained by an organic cause. Diagnosis of functional abdominal pain can often be made without any specific laboratory testing based on a good history and a thorough physical exam in the absence of red flags, which include abnormal exam, involuntary weight loss, significant vomiting, nocturnal symptoms, growth retar­ dation, delayed puberty, GI blood loss, unexplained fevers, family history of inflammatory bowel disease, and consistent right upper or lower quadrant pain. Between episodes of pain the child is well. The pain is poorly defined and periumbili­ cal or poorly localized. Ask about a family history of irritable bowel syndrome, fibromyalgia, chronic fatigue syndrome, and constipation. Patients have often undergone extensive test­ ing. Testing only reenforces the belief that there is something wrong. Parents may have a hard time accepting that the pain is not due to a medical disorder but rather is psychologic. Remember, the pain in functional abdominal pain is real and symptoms are not created intentionally, as in malingering or factitious disorder. Avoid medicalizing patients with these complaints. •

� Diag nostic

CRITERIA I N C H I LDREN

Rec u rre nt a bdom i n a l pai n/d i sco mfo rt: •

3 days per month for the past 3 months, and

Two or more of the fol l owi n g : •





I m proved w i t h poo p i n g Sta rted after a c h a n g e i n poo p i n g freq ue ncy (d i a rrhea, constipation) Sta rted after a change in the poo p's a p peara nce

� QUICKQUIZ Currently the best clinical definition of irritable bowel syndrome (IBS) includes all the following EXCEPT:

A) Recurrent abdominal pain at least 3 days per month in the last 3 months. B) Daily abdominal pain occurring at night. C) Pain associated with improvement after defecation. D) Onset of pain associated with changes in stools. Discussion

Helpful Tip

=-

TABLE 1 6-5 I R RITA B L E BOWEL SYN D R O M E

criteria for fu nctional a bd o m i n a l pa i n

i1 1r include ( 1 ) c h ronic pa in, ( 2 ) no red flags, (3) norma l exa m, a n d (3) negative feca l occult blood test.

The correct answer is "B:' IBS is a clinical diagnosis. The symp­ toms cannot be explained by an alternative medical diagnosis such as malignancy or an inflammatory process. Definitions vary but the Rome III criteria are most often cited. (See Table 16-5.)

The fecal occult blood test is negative. You discuss the diagnosis and treatment. Question 5-2

What is NOT an effective treatment for functional abdominal pain?

A) B) C) D)

Acid-suppressive medications. Education and reassurance. Hypnotherapy. Cognitive behavioral therapy.

A 1 5-year-old boy presents with a history of abdominal pain occurring twice a week for the past 6 months. The pain is more of a dull ache and it is not related to eating. The unpleasant feeling in his gut improves after he poops. He denies vomit­ ing, fever, rash, mouth ulcers, joint symptoms, or weight loss. He is on polyethylene glycol 3350 (Miralax) for hard "poop balls:' His exam and vital signs are normal. Question 6-1

Discussion 5-2

The correct answer is "A:' Children need to get back to liv­ ing. This does not mean they will be pain free. Step one is creating a strong relationship with the child. The current lit­ erature does not support dietary restriction of fructose or lac­ tose, fiber supplementation, or probiotics to treat functional abdominal pain. Data support the use of cognitive behavioral therapy and hypnotherapy. There is limited evidence to sup­ port the use of medications, including selective antispas­ modics, cyproheptadine, acid-blocking medications, gastric motility agents, loperamide, herbal supplements, and antibi­ otics, to treat functional GI disorders. Low-dose antidepres­ sants may be useful in children with underlying psychiatric illness, such as anxiety.

All of the following may be helpful in your management of this patient EXCEPT:

A) Serologic testing for celiac disease. B) Hydrogen breath test (lactose, fructose, or lactulose intolerance). C) Dietary recall/diary. D) Colonoscopy. Discussion 6-1

The correct answer is "D:' This adolescent meets the Rome III cri­ teria for IBS. The American College of Gastroenterology (ACG) IBS Task Force recommends that routine diagnostic testing not be performed in patients with typical symptoms of IBS. How­ ever, inflammatory bowel disease (IBD) may coexist with celiac

C H A PT E R

disease, with a fourfold increase in IBS symptoms in patients with celiac disease. Lactose or fructose intolerance (or both) in IBS patients may cause IBS-like symptoms. Colonoscopy is not recommended unless alarming symptoms and signs are present. Question 6-2

Peptic ulcer disease, often caused by Helicobacter pylori infection, usually presents with:

A) Abdominal pain located in the epigastric, right upper quadrant, or left upper quadrant area. B) History of bloating and early satiety. C) Pain worse after ingestion of a fatty meal. D) Chronic arthralgias and back pain. E) Mouth ulcers. Discussion 6-2

The correct answer is "A:' The two most common causes of pep­ tic ulcer disease (PUD) are H. pylori infection and nonsteroi­ dal anti-inflammatory drugs (NSAIDs). PUD is uncommon in children. Note that H. pylori gastritis is different from H. pylori PUD. The role of H. pylori in abdominal pain without ulcer dis­ ease is unclear. The ESPGHAN (European Society for Paediatric Gastroenterology, Hepatology and Nutrition) and the NASP­ GHAN (North American Society for Pediatric Gastroenterol­ ogy, Hepatology and Nutrition) guidelines for H. pylori infection recommend that initial diagnosis be made by gastric biopsy histopathology plus a positive rapid urease test or positive cul­ ture. Serology is not reliable. Treatment includes a proton pump inhibitor and antibiotics. After treatment, the stool antigen test or urea breath test should be checked to ensure eradication. Each of the options listed above is consistent with a diagnosis other than H. pylori. Option "C" is seen with gallstones. Option "E" is seen with celiac disease and IBD. Option "D" is seen with Crohn disease. Option "B" may be seen with lactose intolerance.

16



P E D I ATRIC GASTRO ENTEROLOGY

a lactose breath hydrogen test may not be needed if the patient improves on a lactose-free diet. Clinical symptoms of lactose intolerance include diarrhea, abdominal pain, and flatulence after the ingestion of milk or milk-containing products. These symp­ toms have been attributed to low intestinal lactase levels (hypo­ lactasia), which may be due to mucosal injury or, much more commonly, reduced genetic expression of the enzyme lactase­ phlorizin hydrolase (ie, lactase). African Americans and Asians are most frequently affected. In some Asian countries more than 95% of population is lactase deficient after 5 years of age. Empiric placement on a gluten-free diet is not indicated unless celiac serology is obtained and is abnormal, and results of small intesti­ nal biopsies are consistent with celiac disease.

� QUICKQUIZ What is the most common cause of chronic abdominal pain in children and adolescents?

A) B) C) D) E)

Functional Abdominal Pain Pancreatitis Celiac Disease Gallstones Renal Colic

Discussion

The correct answer is ''A:' Functional abdominal pain is the number one cause of chronic abdominal pain in children and adolescents. (See Table 1 6-6.) TABLE 1 6-6 CAU SES OF C H R O N I C ABDOM I NAL PAI N Gastroi ntest i n a l

Constipation I nfl a m m atory bowe l d i sease Gastritis Esophagitis Peptic u l cer d i sease

A previously healthy 14-year-old girl is complaining of daily bloating, gassiness, loose stools, and occasional nonbloody, nonbilious vomiting. There has been no recent travel or dietary changes. She has a family history of Crohn disease. Her growth, findings on physical exam, and lab results are normal.

Ce l i a c d i sease Food a l lergy Lactose i nto lera n ce Ga stropa res i s G a l l stones

Question 7-1

All the following steps are appropriate steps in management EXCEPT:

A) B) C) D)

Dietary lactose restriction. Endoscopy and colonoscopy. Stool ova and parasite culture. Gluten-free diet.

Pa ncreatitis Ren a l

Ren a l co l i c

Gen itou r i n a ry

Obstructive u ropathy

Gynecologic

Endometriosis Dysmenorrhea M ittelsch merz Hematocol pos

Discussion 7-1

The correct answer is "D:' The differential diagnosis for her symp­ toms include a parasitic GI infection, such as giardiasis, and celiac disease. Dietary lactose restriction if suspicion is high for lactose intolerance can be a good initial step, and further testing with

331

Hematolog ic Psyc h i atry

M a l i g n a ncy Somatic/fu n cti o n a l d i sorder Conversion d i sorder

MCG RAW-H I LL E D U CATION S P E C I A LTY BOA R D REVI EW: P E D I ATRICS

332



Helpful Tip

� Visceral abdominal pa in may be sensed as coming from a

=

r1 1 r different location; this is ca l led referred pain. For exa mple,

subsides, with normal health in between these severe episodes. Her parents recall having to bring their daughter to the emer­ gency department for IV rehydration when she was younger.

pa in from gall bladder disease may be felt in the right

Question 9-1

shoulder and pa in from pancreatic disease i n the back.

The diagnostic approach may include:

An 8-year-old girl presents to the clinic with pain in the middle of her abdomen that began last night and has gotten progressively worse. She asked not to go to the movies today with her family because it was so bad. She has had two prior episodes in the past, each lasting a few hours. Between epi­ sodes she is well. The mother denies fever, weight loss, diar­ rhea, nocturnal symptoms, vision changes, seizure, trauma or rash. On exam, the girl is uncomfortable. When you look in her eyes with the ophthalmoscopic, she winces and grabs her head. The disc margins are crisp. Her exam, includ­ ing neurologic exam, is normal. Her laboratory tests are normal, including a complete blood count, liver function tests, electrolytes, creatinine, urinalysis, and inflammatory markers. A rapid streptococcal antigen test is negative.

A) B) C) D) E)

Discussion 9-1

The correct answer is "E:' The most likely diagnosis is cycli­ cal vomiting syndrome. Cyclic vomiting syndrome is just that: abrupt recurrent episodes of repeated barfing that are short lived. Diagnosis requires three or more episodes in the prior year with vomiting-free intervals in between. A family history of migraines may be present. No single test is diagnostic. Many experts recommend a minimal workup and, if symptoms fit cri­ teria, therapy may be initiated. National guidelines for children and adolescents state that all of the diagnostic criteria below must be met for a diagnosis of cyclic vomiting syndrome:

Question 8-1

What is the most likely diagnosis?

A) B) C) D) E)

Cyclic vomiting syndrome. Functional dyspepsia. Abdominal migraine. Functional abdominal pain. Constipation.

Discussion 8-1

UGI series. Complete metabolic panel. Dietary recall. Upper endoscopy. All of the above.











At least five attacks in any time interval, or a minimum of three attacks during a 6-month period Episodic attacks of intense nausea and vomiting lasting 1 hour to 10 days and occurring at least 1 week apart Stereotypical pattern and symptoms in the individual patient Vomiting during attacks occurs at least four times per hour for at least 1 hour Return to baseline health between episodes Not attributed to another disorder

The correct answer is "C:' All the listed options are possible causes of recurrent, acute abdominal pain, but the most likely diagnosis is best teased out by a good history and physical exam. Options "PI' through "D" fall under the umbrella of functional GI disor­ ders. In this patient, there are no red flags present. A family his­ tory of migraines would be helpful. The diagnosis of abdominal migraines using the Rome III criteria requires ( 1 ) intermittent episodes of severe, acute periumbilical pain; (2) episodes lasting 1 hour or more; (3) weeks to months between separate episodes during which the child is well; ( 4) pain that interferes with activi­ ties; (5) associated anorexia, nausea, vomiting, headache, photo­ phobia, or pallor (need two or more); (6) inability to explain the findings by another diagnosis; and (7) two or more episodes in the preceding year. Functional dyspepsia must cause persistent or recurrent pain in the upper abdomen, show none of the bowel changes seen in IBS, and not be explained by a different diagnosis.

A 6-week-old breastfed male infant presents with nonbilious, nonbloody projectile emesis with progressive worsening and recent weight loss. He has been afebrile without diarrhea. He eats voraciously then vomits, only to want to eat again.

An 8-year-old girl has a history of stereotypical episodes of nausea and vomiting that have occurred at least six times in the past year. Vomiting lasts for 2 hours and then completely

The correct answer is "B:' IHPS occurs in approximately 2 to 3.5 newborns per 1000 live births. It is more common in males than females (4: 1 to 6: 1) and in premature infants. It may run in



Question 1 0-1

The most likely diagnosis is:

A) B) C) D) E)

Gastroesophageal reflux disease (GERD). Infantile hypertrophic pyloric stenosis (IHPS). Malrotation with volvulus. Vascular ring. Formula intolerance.

Discussion 1 0-1

C H A PT E R 1 6

families. Approximately 30% of cases occur in first-born children (approximately 1 .8- fold increased risk). The etiology is unknown. Symptoms usually begin between 3 and 5 weeks of age, and very rarely occur after 12 weeks of age. Vomiting in IHPS is typically forceful and nonbilious, and tends to occur immediately after feeding. The force and timing can help to distinguish IHPS from physiologic gastroesophageal reflux, in which most episodes of vomiting are not forceful and may occur 1 0 minutes or more after the meal. A history of bilious vomiting does not exclude IHPS but should raise concern about more distal intestinal obstruc­ tion, such as malrotation with volvulus or Hirschsprung disease. Vascular rings typically have associated respiratory symptoms such as stridor. Option "E" is incorrect as the case clearly says the newborn is breastfed. Here's a tip if you missed that: slow down, speedy reader! IHPS is diagnosed by ultrasound and treated surgi­ cally after rehydration and correction of electrolyte abnormalities.

� QUICKQUIZ Speaking of electrolyte abnormalities, what is NOT an elec­ trolyte derangement seen with pyloric stenosis?

A) Hypokalemia. B) Hypochloremia. C) Metabolic alkalosis. E) Hyperkalemia. Discussion

The correct answer is "D:' In early cases, electrolytes may be normal.



P E D I ATRIC GASTRO ENTEROLOGY

333

disease. In diagnosing the latter, look for irritability, poor growth, apnea, or a combination of these findings. Question 1 1 -2

How should you evaluate this patient?

A) B) C) D) E)

Obtain a UGI study. Change formulas. Order a pH probe study. Start an H2-receptor antagonist. None of the above.

Discussion 1 1 -2

The correct answer is "E:' In the majority of infants, a focused history and physical examination will confirm that the reflux is uncomplicated, and little further evaluation or intervention is required. The first step in the evaluation is to determine whether the infant has any warning signs that would suggest an underlying disorder other than GER. The second step is to determine if the infant has secondary complications of the reflux, such as esophagi­ tis, respiratory symptoms, or failure to thrive. This step is guided by whether the infant has associated problems with weight gain, feed­ ing refusal, irritability, apnea, or gross or occult blood in the stool.

A 2-month-old male infant presents to the emergency depart­ ment with acute onset of bilious emesis and a history of poor feeding and abdominal distention. His last bowel movement was over 3 days ago. The most likely diagnosis is malrotation with volvulus. Question 1 2-1

The best diagnostic test in this case is: A 2-month-old male infant has effortless nonbilious, non­ bloody emesis (which can on occasion be forceful), occur­ ring 10 to 30 minutes after each meal. He has been tracking at the 1 5th percentile for weight and height since birth. He has been started on anti-acid therapy without any change in symptoms. He is mostly content and continues to feed well despite these episodes. Question 1 1 -1

The most likely diagnosis is:

A) B) C) D)

Gastroesophageal reflux. Malrotation with volvulus. Pyloric stenosis. Vascular ring.

Discussion 1 1 -1

The correct answer is ''A:' This is a classic "happy spitter:' Key clues include good growth, no cyanotic or apneic spells, no irri­ tability, nonforceful emesis, nonbilious emesis, and a reassur­ ing exam. Management is focused on reassurance and parental education about the natural history of gastroesophageal reflux (GER) and the difference between GER and GERD. D equals

A) B) C) D)

Abdominal ultrasound. UGI series. Upper endoscopy. Barium enema.

Discussion 1 2-1

The correct answer is "B:' A limited upper gastrointestinal (UGI) contrast series is the best examination to visualize the duode­ num and assess for malrotation and midgut volvulus. It should be performed, whenever possible, under fluoroscopy and by an experienced pediatric radiologist. Sensitivity of a limited UGI series in infants with signs of malrotation is approximately 96%. Do you feel a sense of deja vu?

� QUICKQUIZ Which is NOT associated with duodenal atresia?

A) B) C) D) E)

Bilious emesis. Polyhydramnios. "Double bubble" sign. Trisomy 2 1 . All of the above.

MCG RAW-H I LL E D U CATION S P E C I A LTY BOA R D REVI EW: P E D I ATRICS

334

Question 1 3-1

Recommendations include all the following EXCEPT:

A) B) C) D)

Start an oral rehydration solution. Give ondansetron by mouth as needed. Consider and IV fluid bolus if moderately dehydrated. Instruct that he is to be NPO (no eating or drinking) .

Discussion 1 3-1

The correct answer is "B:' In patients with clinically significant vomiting, the use of antiemetics, particularly serotonin (5-HT) receptor antagonists such as ondansetron, has facilitated the administration of oral rehydration therapy by reducing vom­ iting. In children with vomiting due to gastroenteritis, only a single oral dose of ondansetron is needed to reduce vomiting, facilitate the administration of oral rehydration therapy, and reduce the need for intravenous fluids. It is a useful adjunct to oral rehydration therapy and may be used to reduce vomiting in children with gastroenteritis. Maybe slip a little ondansetron in Pedialyte? Administration of 5-HT3 receptor antagonists also prevents nausea and vomiting such as occurs with chemotherapy. F I G U R E 1 6-3. D o u b l e B u b b l e . Abd o m i n a l X-ray showi ng the "double bubble" sign i n a newborn with duodenal atresia. Bubble 1 is the d i l ated prox i m a l duodenum. Bubble 2 is the d i l ated stomach. Note the absence of dista l bowel gas. (Reproduced with permission from Brunicardi FC, Andersen DK, B i l l i a r TR, et al, eds. Schwartz's Principles of Surgery. 1 Oth ed. New York, NY: McGraw- H i l l Education; 201 5, Fig. 39-1 3.)

� QUICKQUIZ True or False: The differential diagnosis for vomiting is short.

A) True. B) False. Discussion Discussion

The correct answer is "E:' On abdominal X-ray, the stomach and proximal duodenum are dilated ("double bubble" sign) and there is no gas distally. Approximately, 25% of infants with duodenal atresia have trisomy 2 1 . Other anomalies including congenital heart disease may also be present. (See Figure 16-3.) •



1 1 1r

Helpful Tip

Signs of i ntesti n a l obstruction include a bdom i n a l d istention, bil ious emesis, a n d lack of stoo l i n g or fa i l u re to pass meco n i u m i n newborns. If you suspect obstruction, get an a bdom i n a l X-ray.

The correct answer is "B:' The American Board of Pediatrics expects you to formulate an age-appropriate differential diag­ nosis of vomiting. If we made a chart, it would take up at least 2 pages. Work through each case systematically. Start by deter­ mining the following (not all-inclusive): ( 1 ) bilious or nonbil­ ious, (2) bloody or nonbloody, (3) age of patient, (4) diarrhea or no diarrhea, (5) febrile or afebrile, (6) peritoneal or nonperi­ toneal signs, and (7) recurrent, chronic, or acute. The presence of peritoneal signs suggests a surgical process. Bilious emesis suggests obstruction distal to the stomach. You get the rest. You need a differential to guide your workup and begin the jour­ ney toward finding the pot of gold at the end of the rainbow (a diagnosis)-or concluding that it is reflux and moving on. Remember to think about every organ system, not just the GI tract. Vomiting is a common player for all.

A 2-year-old boy with a 2-day history of nonbilious vom­

A 1 3-year-old female patient presents with daily regurgita­

iting and nonbloody diarrhea is brought to the clinic by his mother. She reports that he has thrown up 12 times in the past 24 hours and cannot keep anything down. He last peed 12 hours ago. The mother received a notice this week that another child at the daycare he attends was sick with a "stomach bug:' On exam, the boy appears ill but not toxic. He is slightly tachycardic, has tears, and has normal capil­ lary refill.

tion witnessed by parents. During or after eating, swallowed food it brought back up, chewed, and reswallowed without vomiting or pain. Question 1 4-1

The most likely diagnosis is:

A) GERD. B) Rumination syndrome.

C H A PT E R 1 6



P E D I ATRIC GASTRO ENTEROLOGY

335

C) Malrotation. D) Celiac disease. E) Gastritis.

D) Upper GI series. E) All of the above.

Discussion 1 4-1

The correct answer is "C:' In older children and adolescents, the patient history and physical exam are adequate to diagnose GERD and rule out other diagnoses. An upper GI series does not rule GERD out or in. If you were looking for a structural issue, then an upper GI would be useful. A pH probe study does not detect nonacidic reflux. It is useful to correlate reflux with symptoms. If the diagnosis of GERD is in question, the patient refractory to medical treatment, red flags are present, or you are looking for a complication then endoscopy with biopsy sampling is indicated.

Discussion 1 5-2

The correct answer is "B:' Rumination disorder is the recurrent effortless regurgitation of food that is rechewed and either swal­ lowed or spit out. It is a habit that serves as self-stimulation, similar to biting nails. In patients of normal intelligence, it is voluntary but not intentional. Rumination can be habitually reversed using diaphragmatic breathing to counter the urge to regurgitate. Alongside reassurance, explanation, and habit reversal, patients are shown how to breathe using their dia­ phragms prior to and during the normal rumination period. A similar breathing pattern can be used to prevent normal vomit­ ing. Breathing in this method works by physically preventing the abdominal contractions required to expel stomach contents. Regurgitation is bringing up swallowed food or secretions into the mouth not out of habit or pleasure. It is typically involun­ tary, except in the case of eating disorders.

A 12-year-old boy has a history of intermittent epigas­ tric pain and heartburn with worsening nocturnal cough. Symptoms improved but have not resolved on histamine H2 blocker therapy. Question 1 5-1

All the following are appropriate steps in management except:

A) B) C) D) E)

Start a proton pump inhibitor (PPI). Avoid spicy meals. Avoid late meals before going to bed. Start a prokinetic agent. Obtain a GI specialist consult and upper endoscopy.

Discussion 1 5-1

The correct answer is "A:' The symptoms described are clas­ sic for GERD-epigastric pain after eating, substernal burning, and nocturnal symptoms (lying flat) . H2 blocker failure is an indication for a PPI trial (step-up therapy) and referral to a gas­ troenterologist for further evaluation and follow up. PPis are superior to H2 blockers for the relief of GERD symptoms. Pro­ kinetic therapy is not indicated in the management of simple GERD. Lifestyle modifications, including no smoking, avoid­ ance of late-night eating, and dietary changes are reasonable to consider in addition to medications. However, some might choose a little heartburn over a diet devoid of coffee, caffeine, or chocolate.





1 1 1r

Helpful Tip

Use of acid-blocking med ication should be j u d icious as the r i s k o f ga stroenteritis a n d p n e u m o n i a is increased

i n i nfa nts and c h i l d ren treated with h i sta m i n e H2receptor a ntagonists, proton p u m p i n h i bitors, or both.

You discuss the patient's symptoms and talk about GERD with him. You explain the treatment. Astutely, the 12-year-old asks, "What does this mean for me when I am old like you?" Question 1 5-3

Which is NOT a complication of GERD?

A) B) C) D)

Esophagitis. Tonsil hypertrophy. Esophageal stricture. Dental erosions.

Discussion 1 5-3

The correct answer is "B:' Patients who are neurologically impaired, obese, or have anatomic abnormalities of the upper GI tract are at risk for severe, chronic GERD. Complications include esophagitis, esophageal strictures, and Barrett esopha­ gus (metaplastic changes in the mucosa of the esophagus).

� QUICKQUlZ Which is NOT a symptom of GERD?

A) B) C) D) E)

Wheezing. Dental erosions. Posturing. Cough. All of the above.

Discussion Question 1 5-2

What evaluation is necessary to diagnosis GERD?

A) Endoscopy. B) pH probe study. C) Patient history.

The correct answer is "E:' GERD may present in ways other than regurgitation and heartburn. Laryngospasm, aspiration, apnea, failure to thrive, oral aversion, dysphagia, and noctur­ nal cough are few examples. GERD can worsen wheezing and respiratory symptoms in asthmatic patients with moderate to

336

MCG RAW-H I LL E D U CATION S P E C I A LTY BOA R D REVI EW: P E D I ATRICS

severe disease. Sandifer syndrome (opisthotonic posturing and arching) is a sign of reflux and may be confused with a seizure.

A 4-year-old boy returns to clinic for follow-up of chronic nonbilious vomiting and failure to thrive. He has been treated with a PPI for the past 6 months with ongoing weight loss. He has a significant past medical and family history of mild asthma, seasonal allergies, and moderate to severe eczema. Question 1 6-1

The most likely diagnosis is:

A) B) C) D)

Eosinophilic esophagitis. GERD. Celiac disease. Malrotation with volvulus.

Discussion 1 6-1

The correct answer is "A:' A personal or family history of atopy, nonresponse to adequate PPI therapy, and failure to thrive are concerning for eosinophilic esophagitis. However, diagnosis is still clinicopathologic, so endoscopy with esophageal biopsy sampling is needed to confirm diagnosis before further therapy. Eosinophils are not present in the esophagus unless there is pathology. Food impaction, dysphagia, vomiting, and abdomi­ nal pain are typical signs and symptoms. Treatment includes dietary modification (removal of allergic foods), acid suppres­ sion, and topical steroids. Steroid inhalers such as fluticasone are sprayed into the mouth without a spacer and swallowed. Esophageal dilation is needed if strictures develop.

acute bloody diarrhea, and patients appear ill. C. difficile colitis is possible, but without presence of blood and prior history of anti­ biotics exposure it is less likely than Giardia intestinalis. The wil­ derness adventure is a giveaway, as would be travel to a developing country. Giardiasis is a parasitic infection of the small intestine known to cause daycare outbreaks and infect international travel­ ers. It is spread through water, food, and fecal-oral routes. This is why campers are advised to boil, filter, or treat fresh water when in the wilderness. Asymptomatic infection is common. Symptom development is delayed by 1 to 2 weeks. Acute infection causes diarrhea with smelly, fatty stools and abdominal cramps. Fever is uncommon. Diarrhea may last up to 1 month. Chronic infection may cause malabsorption, diarrhea and weight loss. His ova and parasite stool microscopy is positive for Giardia. Question 1 7-2

What is the treatment of choice?

A) B) C) D) E)

Antidiarrheal medication. Probiotics. Amoxicillin. Metronidazole. Albendazole.

Discussion 1 7-2

The correct answer is "D:' Metronidazole, tinidazole, or nitazoxanide are first-line treatment. Tinidazole is approved for children over the age of 3 and is given as a single dose. Albenda­ zole and mebendazole are alternative options for children and have fewer side effects. Asymptomatic infection and carriers should not be treated.

r1J •

A 5-year-old previously healthy male patient has daily watery diarrhea of 3 weeks' duration associated with intermittent abdominal pain. There is no history of vomiting, fever, or bloody stools and he has not taken antibiotics recently. His parents note that the family went on a backpacking trip, but that was 5 weeks ago. The boy is otherwise healthy, has been growing well, and has a normal appetite. His vital signs and exam are normal, including a rectal exam. Question 1 7-1

Stool testing is likely to be consistent with:

A) B) C) D) E)

Giardiasis. Clostridium diffi cile colitis. Rotavirus infection. Enterotoxigenic Escherichia coli (ETEC). Normal.

Discussion 1 7-1

The correct answer is "A:' Presence of daily diarrhea with abdom­ inal pain suggests an infectious etiology or inflammatory bowel disease. Rotavirus is not a cause of chronic diarrhea. ETEC causes

Helpful Tip

Acq u i red lactose i ntolera nce is common after giardiasis.

The boy's father asks about using loperamide, which he read about on the Internet. He tells you, "International travelers use it all the time:' Question 1 7-3

What do you recommend?

A) B) C) D)

Loperamide Diphenoxylate. Morphine. None of the above.

Discussion 1 7-3

The correct answer is "D:' Antimotility agents, options ''!\.' and "B;' are not recommended for patients with acute diarrheal ill­ nesses accompanied by fever or bloody diarrhea as the clear­ ance of the pathogen is delayed and may cause more severe disease. These medications do not treat the infection. In infants and children, use of an antimotility agent to treat gastroenteritis increases the risk of serious adverse effects.

C H A PT E R 1 6



P E D I ATRIC GASTRO ENTEROLOGY

337

Discussion 1 9-1

A 22-month-old girl who was recently hospitalized for recur­

rent febrile urinary tract infection presents with a 5-day his­ tory of bloody stools. Her mother made sure she completed her 14-day course of antibiotics. Her stool testing is consis­ tent with C. diffi cile colitis. Question 1 8-1

What is the best initial treatment option?

A) B) C) D) E)

Oral vancomycin. Intravenous vancomycin. Oral metronidazole. Intravenous metronidazole. Oral fidaxomicin.

Discussion 1 8-1

The correct answer is "C:' Isn't it funny that an antibiotic­ associated infection is treated with antibiotics? Oral metronida­ zole remains first -line treatment. Metronidazole may be repeated for the initial reoccurrence. Oral vancomycin is used for a second recurrence, and often a pulsed taper dosing regimen is utilized. Fidaxomicin is approved for patients older than 18 years. This is a reminder to prescribe antibiotics wisely, and to choose the nar­ rowest spectrum of activity and the shortest course possible.

The correct answer is ''A." The patient has Guillain-Barre syn­ drome, acute immune-mediation polyneuropathy causing ascending paralysis. Campylobacter jejuni infection has been established as a trigger of Guillain-Barre syndrome. Diarrhea is self-limited and lasts for a mean of 7 days. Abdominal pain may persist after resolution of diarrhea, and weight loss of 5 kg or more may be observed. It has been estimated that in 30% to 40% of cases Guillain-Barre syndrome is attributable to Campylobacter infection, which typically occurs between 1 and 2 weeks before the onset of neurologic symptoms. The infection is most often acquired from preparing and eating undercooked or raw chicken. (Wash that cutting board!) Other sources include unchlorinated fresh water and raw milk-a reminder of why pasteurization of milk is a good thing. Campylobacter infections also may cause reactive arthritis (postinfectious) . •

Helpful Tip

:5.� Campylobacter

a n d Yersinia i nfections can present

i1 1r w i t h sym ptoms s i m i l a r t o a p pend icitis (pseudoappen­ dicitis), with a bdominal pa i n that occu rs before the

dia rrhea .

Question 1 8-2

Which of the following is true with regard to Clostridium difficile?

A) B) C) D)

It primarily causes bloody diarrhea. If present, it always indicates active infection. Prevalence decreases with age. Retesting should be done 1 week after treatment.

Discussion 1 8-2

The correct answer is "A:' Generally retesting is recommended at least 6 weeks or more after treatment, if needed, and generally is not required. Asymptomatic colonization is present in 30% to 40% of some populations (eg, infants), and the prevalence of true infection increases with age.

A 2-year-old boy presents with a 3-month history o f diar­

rhea described as daily blow outs with pieces of raw veg­ etables visible in the bowel movement. He has as many of four bowel movements per day, with stools that become looser throughout the day. Despite these ongoing episodes of diarrhea, he is growing and thriving. His physical exam is normal, but as you watch him he guzzles 3 cups of juice in 15 minutes. Question 20-1

The best initial step in your management is:

A) B) C) D) E)

Limit excessive fluid intake. Order upper endoscopy with biopsy sampling. Order screening lab tests for malabsorption. Perform stool studies. Order sweat chloride testing.

A 1 7-year-old boy comes to the clinic with ascending weak­

ness in his lower extremities as well as facial muscle weak­ ness. He had presented 2 weeks earlier with moderately severe abdominal pain and diarrhea. Question 1 9-1

Which of the following was the most likely cause of his GI symptoms?

A) B) C) D)

Campylobacter gastroenteritis. Clostridium diffi cile colitis. Ulcerative colitis. Autoimmune enteropathy.

Discussion 20-1

The correct answer is ''A:' The key here is a toddler with diar­ rhea. Functional diarrhea is defined as the painless passage of three or more large, unformed stools during waking hours for 4 weeks or more, with onset in infancy or the preschool years, and without failure to thrive or a specific definable cause. This common, benign disorder has also been termed chronic non­ specific diarrhea of childhood or toddler's diarrhea. Children with functional diarrhea usually pass stools only during wak­ ing hours. Early-morning stools typically are large and semi­ formed, and then stools become progressively looser as the

338

MCG RAW-H I LL E D U CATION S P E C I A LTY BOA R D REVI EW: P E D I ATRICS

TABLE 1 6-7 D I AG N OSTIC C R I T E R I A F O R F U N C T I O N A L D I A R R H EA

Th ree or more l a rg e, loose, non bl oody stools per day Pa i n less to pass Stool s occ u r d u ri n g the day o r awa ke t i m e Present fo r > 1 month O n set between the ages of 6 to 36 months No fa i l u re to th rive or red flag sym pto m s

A 3-week-old full-term healthy breastfed infant girl presents with a 3-day history of blood in the stool. She has been eat­ ing well, acting normal, and has gained weight appropriately since birth. There has been no history of fever or sick contacts. On exam, she is a well-nourished newborn with a soft abdo­ men. No anal fissures are seen on the external anus. There is no bruising or petechiae. You review her newborn record to verify that she received intramuscular vitamin K after birth. Question 22-1

The initial step in management should be:

day progresses. (See Table 1 6-7.) In some cases, the diarrhea is associated with excessive intake of fruit juice or other osmoti­ cally active carbohydrates and improves when intake of these foods is moderated. Other than this precaution, restrictions to the diet or other interventions are neither necessary nor help­ ful. In particular, restriction of dietary fat may be counterpro­ ductive. Make sure no red flags are present before making this diagnosis (ie, bloody diarrhea, fever, weight loss, poor growth, etc-you get the picture) . If the evaluation suggests functional diarrhea, the following dietary changes should be trialed: ( 1 ) Reduce o r eliminate fruit juice o r other osmotically active car­ bohydrates; (2) liberalize the fat content of the diet to 35% to 50% of total calories.

A 2-month-old exclusively breastfed male infant has a his­ tory of intermittent bloody stools for 3 days. He is afebrile, gaining weight, and thriving without any other concerns. On exam, he is happy with a soft belly with good bowel sounds.

A) B) C) D)

Stool analysis for Clostridium diffi cile infection. Abdominal X-ray, with two views. Complete blood count and coagulation profile. Maternal dietary exclusion of cow's milk protein.

Discussion 22-1

The correct answer is "D:' This is a classic scenario for a patient with allergic proctocolitis resulting from allergy or intoler­ ance to cow's milk protein. It classically presents around 4 weeks of age with bright red blood from the rectum. The key clue is that the newborn is healthy, gaining good weight, and looks well on exam. Specialized testing-including flex­ ible sigmoidoscopy with biopsy sampling, often done at the bedside without sedation-will confirm the diagnosis but is not necessary if the history is classic and exam is reassur­ ing. Infants are frequently colonized with C. diffi c ile, and this infant has no risk factors for infection. An abdominal X-ray would look for necrotizing enterocolitis, which can occur in term infants, but again her exam is reassuring. A CBC and coagulation studies would screen for a coagulopathy, but this infant has no other signs of bleeding or bruising and received postpartum vitamin K.

Question 21 -1

You are most likely to recommend:

A) B) C) D)

Soy-based formula. Amino acid-based formula. Maternal dietary dairy restriction. Stool testing for Clostridium diffi cile.

Discussion 21 -1

The correct answer is "C:' Clinical findings of cow's milk allergy frequently appear during the first few months of life, often within days or weeks after the introduction of a cow's milk­ based formula into the diet, although symptoms may also occur with exclusive breastfeeding if the mother ingests cow's milk. Signs and symptoms may include vomiting, fussiness, failure to thrive, diarrhea with or without blood, and positive fecal occult blood test. Patients with cow's milk allergy present with a wide range of immunoglobulin E (IgE)- and non-IgE-mediated clin­ ical syndromes. Dietary elimination of suspected culprit (milk protein) is the first step in management. Formula-fed babies should be switched to a protein hydrolysate formula.

A 5-year-old boy presents with a 6-day history of diarrhea, which has become bloody in the past 48 hours. He is admitted to the hospital, and his laboratory results reveal leukocytosis with reduced hemoglobin and platelet counts. His creatinine and blood urea nitrogen (BUN) are elevated at 0.8 mg/dL and 35 mg/dL, respectively. His urine output is decreased, and he has had severe abdominal cramps. On exam, he is tachy­ cardic, hypertensive, and mildly pale. He is uncomfortable and has mild tenderness to palpation. Bowel sounds are pres­ ent. No bruising, edema, or petechiae are noted. Question 23-1

The most likely organism is:

A) B) C) D)

Yersinia. Clostridium diffi cile. Salmonella. Escherichia coli.

C H A PT E R 1 6

Discussion 23-1

The correct answer is "D:' Acute bloody diarrhea with abdomi­ nal cramps and fever is suggestive of bacterial gastroenteritis or colitis. Pathogens include Salmonella, Shigella, Yersinia, Cam­ pylobacter, and Clostridium diffi cile. Bloody stools and evidence of early renal dysfunction are consistent with hemolytic uremic syndrome (HUS) associated with Shiga-toxin-producing E. coli (STEC) or Shigella infection. Most cases of HUS are caused by E. coli 0 1 57:H7 (STEC strain).

� QUICKQUIZ



P E D I ATRIC GASTRO ENTEROLOGY

339

(malabsorptive). Patients present with life-threatening diar­ rhea in the newborn period. Osmotic diarrheas (malab­ sorptive) stop when fasting or the problem carbohydrate is eliminated from the diet. There are several types, including glucose-galactose malabsorption, congenital lactase deficiency, congenital sucrase-isomaltase deficiency, and enteric anendo­ crinosis. Both sucrase-isomaltase deficiency and fructose mal­ absorption manifest in infants after 3 to 6 months of life when fruit is introduced in the diet. Concentrated formula (ie, forti­ fied) may cause an osmotic diarrhea, but it is not life-threaten­ ing and is easy to identify from the history.

What is the most common cause of diarrhea in children worldwide?

A) B) C) D)

Salmonella. Norwalk virus. Rotavirus. Enterotoxigenic E. coli.

Discussion

The correct answer is "A:' Rotavirus is the leading cause of severe acute gastroenteritis (vomiting and severe diarrhea) among children worldwide. Two different rotavirus vaccines are currently licensed for infants in the United States. The vaccines are RotaTeq (RVS) and Rotarix (RV 1 ) . Before being licensed, both vaccines were tested in clinical trials and shown to be safe and effective. In these studies, during approximately the first year of an infant's life, rotavirus vaccine was found to prevent 85% to 98% of rotavirus illness episodes that were severe, and to prevent 74% to 87% of all rotavirus illness episodes.

A 1 0-day-old female infant presents with protracted severe diarrhea for the past week. She is down 15% from birth­ weight. The mother is unsure when she last peed. She contin­ ues to have diarrhea even if not eating. Her albumin is 2 g/dL. Her chloride and sodium levels are normal. She has a meta­ bolic acidosis and elevated BUN and creatinine. On exam, she is dehydrated and listless. Question 25-1

The most likely diagnosis is:

A) B) C) D) E)

Microvillus inclusion disease. Congenital chloride diarrhea. Autoimmune enteropathy. Tufting enteropathy. Enterocyte heparan sulfate deficiency.

Discussion 25-1

A 5-day-old formula-fed neonate presents with severe, life­ threatening diarrhea. Her diarrhea is watery and continu­ ous. With fasting the diarrhea stops. Her stool is positive for reducing substances. On exam, the neonate is dehydrated and listless. Question 24-1

What is the most likely cause?

A) B) C) D)

Congenital lactase deficiency. Sucrase-isomaltase deficiency. Fructose malabsorption. Improperly mixed formula.

Discussion 24-1

The correct answer is "A:' Congenital lactase deficiency is extremely rare and manifests in newborns. Sepsis is always the number one diagnosis to rule out in an ill newborn. Let's stay on the topic of diarrhea rather than being sidetracked by the toxic newborn differential. Congenital diarrheas are rare causes of chronic diarrhea and are either secretory or osmotic

The correct answer is "E:' All of the options are congenital secretory diarrheas. Option "E;' enterocyte heparan sulfate deficiency is a type of congenital protein-losing enteropathy (PLE). The other choices do not typically involve severe enteric protein loss. PLE is a type of secretory diarrhea that leads to massive stool protein loss with subsequent hypoalbuminemia. It may be congenital or acquired. Stool alpha - 1 antitrypsin is positive. Acquired causes include severe cow's milk allergy, celiac disease, inflammatory bowel disease, post-Fontan proce­ dure (congenital heart disease surgery) with right-sided heart failure, and giardiasis. Acquired forms result from mucosal injury or increased lymphatic pressure in the gut. The molecu­ lar basis of congenital PLE is unknown. However, it has been shown that sulphated glycosaminoglycans may be important in regulating vascular and renal albumin loss. Congenital secre­ tory diarrheas begin at birth with profuse watery diarrhea even when fasting. It may be impossible to distinguish the watery diarrhea from urine. There are several types, including congen­ ital chloride diarrhea, congenital sodium diarrhea, microvillus inclusion disease, and tufting enteropathy. As a nongastro­ enterologist, knowing the details of each type of congenital diarrhea seems impractical. It is probably enough to ( 1) rec­ ognize that diarrhea in the newborn period may be congenital,

340

MCG RAW-H I LL E D U CATION S P E C I A LTY BOA R D REVI EW: P E D I ATRICS

(2) distinguish if diarrhea is secretory or osmotic through a fasting challenge, and (3) support the newborn's hydration and nutrition status while calling your GI colleagues. In infants with profuse insensible fluid losses from diarrhea, it is impor­ tant to monitor urine output. If urine cannot be distinguished from poop, you should place an indwelling urinary catheter.

A 4-day-old male infant presents with constipation since birth and abdominal distention in the past 24 hours. He has had no vomiting or fever. He was born at term after an uncom­ plicated pregnancy. There is no history of polyhydramnios. Question 26-1

� QUICKQUIZ

You are mostly concerned about:

True or False: The differential diagnosis of diarrhea is long.

A) True. B) False.

A) B) C) D)

Functional constipation. Ectopic anus. Spinal dysraphism. Hirschsprung disease.

Discussion 26-1 Discussion

The correct answer is "A:' Like vomiting, diarrhea raises a laun­ dry list of possibilities. Diarrhea can be classified as secretory or malabsorptive, acute or chronic, or bloody or nonbloody. Ask about nocturnal symptoms. A simple first test is to have the child fast. Secretory diarrhea will continue despite the child being NPO and occurs at night. For malabsorptive diarrhea to continue the child needs to eat. The next simple test is a fecal occult blood test. What is the time frame-acute or chronic? Finally, is the patient a neonate, infant, toddler, child, or ado­ lescent? Now you can start thinking about the exhaustive list of causes of diarrhea, but at least the differential has been nar­ rowed some. (See Table 1 6-8.)

TABLE 1 6-8 CAU S E S O F D I A R R H EA

Category

Exa m p l e

I nfecti o u s

Vi ra l Bacterial Pa ra sitic

Co ngen ital

Co ngen ital lactase d eficie ncy

Mal a bsorptive

Ce l i a c d i sease

Autoi m m u ne enteropathy Cystic fi bros i s I nfl a m matory bowe l d i sease A l l e rg ic Endocrine

Pa ncreati c i n s ufficie ncy Cow 's m i l k prote i n a l l e rgy or

A 2-month-old male infant presents with fever, bloody diarrhea, and feeding intolerance. His history is notable for short-segment Hirschsprung disease status post-anorectal resection and pull-through on day of life 5. Question 27-1

The best step in management is:

A) Administer IV antibiotics and perform gastric decompres­ sion by means of a nasogastric tube. B) Administer IV antibiotics and perform rectal irrigation. C) Send stool for Clostridium diffi cile testing and start an empiric course of oral metronidazole. D) Perform flexible sigmoidoscopy with biopsy sampling.

i nto lera n ce

Discussion 27-1

Hyperthyro i d i s m

The correct answer is "B:' Acute enterocolitis is a life-threatening emergency associated with Hirschsprung disease. Enterocolitis can be the initial presentation for a patient with Hirschsprung disease, but even after surgical resection of aganglionic bowel, patients are at risk for enterocolitis for months. Before you dismiss vomiting, fever, explosive diarrhea, abdominal pain, and distention in a patient with a history of Hirschsprung dis­ ease make sure you are not missing enterocolitis. Abdominal X-rays will show intestinal obstruction. The pathophysiology of

Hormo ne-secret i n g t u m o rs M i sce l l a neous

The correct answer is "D:' Hirschsprung disease is caused by failure of the ganglion cells to migrate to the colon. The "agangli­ onic" segment cannot relax to allow contents to pass normally. Consider it a functional obstruction. More than 95% of infants with Hirschsprung disease fail to pass meconium in the first 24 hours. The diagnosis is best made by rectal suction biopsy; examination shows absence of ganglion cells in the submucosa. Barium enema may still be normal but should be obtained. On rectal exam, the rectum is not dilated and empty; gas and poop may squirt out. Be aware that newborns with Hirschsprung dis­ ease may develop an acute life-threatening intestinal obstruc­ tion resulting in bilious vomiting and abdominal distention with rapid clinical deterioration.

Functional d i a rrhea (tod d l e r 's d i a rrhea) Anti biotics Laxative a buse Constipation with encopres i s

C H A PT E R 1 6

Hirschsprung-associated enterocolitis is not clear. Rectal irriga­ tion with saline cleans out (decompresses) the colon, potentially making the infection less severe.



P E D I ATRIC GASTRO ENTEROLOGY

341

disease or other etiologies such as small left colon are the cause. A history of delayed passage of meconium would be expected in such disorders but is absent in this case.

J% QUICKQUIZ Which of the following is true about Hirschsprung disease?

A) It is more commonly diagnosed in females. B) In most patients it is associated with other congenital anomalies. C) Surgery is performed between the ages of 3 and 4 years. D) Enterocolitis can occur before or after surgical correction. Discussion

The correct answer is "D:' Short segment Hirschsprung disease (the common form) is diagnosed more often in males. Total colonic/intestinal forms occur in females and are often familial. Surgery is rarely performed after first few months of life, unless the diagnosis was initially missed. •

Helpful Tip

� Most newborns should pass meco n i u m wit h i n the fi rst

:5.

r1 1r 4 8 hours of life. I f not, consider H i rschsprung d isease and obta i n a recta l suction biopsy.

A 5-year-old girl has a history of large-caliber infrequent bowel movements for 9 months, and more recently developed fecal incontinence. She has frequent streaks of poop in her underwear. Her stools are hard and have clogged the toilet several times. She has intermittent periumbilical abdominal pain. Her neurologic exam is normal, including reflexes in her lower extremities. She has no sacral dimple or hair tuft. Her abdomen is soft but slightly full. Her rectal exam is nor­ mal except for palpation of a "poop ball?' Question 29-1

The best initial management is:

A) Prescribe an osmotic daily stool softener and scheduled toilet sitting. B) Recommend increased daily fiber and fluid intake. C) Perform stool testing for pH and fecal fat. D) Order an abdominal X-ray with both supine and upright views. Discussion 29-1

A 3-month-old male infant who was born with a single umbilical artery, vertebral anomalies, radial agenesis, and a single kidney presents with a history of intermittent consti­ pation for the past 2 months. Question 28-1

The best next step is:

A) B) C) D)

Daily stool softener and follow-up in 6 months. Barium enema study. Screening laboratory studies for hypothyroidism. Surgical referral for possible anorectal malformation.

The correct answer is "A:' The diagnosis in this case is functional constipation with overflow fecal incontinence (encopresis) . This is the most common cause of constipation and results from voluntary stool withholding with retention. The best manage­ ment is a combination of stool softener (osmotic stool softener is one of the best choices) and timed toilet sitting. Testing is not usually indicated as long as the history and exam do not raise any red flag signs (ie, delayed passage of meconium, abnormal reflexes, abnormal rectal tone, sacral dimple, vomiting, fever, poor weight gain) . Increasing fiber and fluid intake is a good idea; however, once chronic constipation and incontinence is established, a clean-out dose of the stool softener with sched­ uled timed toilet sitting is needed, followed by maintenance therapy. (See Table 1 6-9.)

Discussion 28-1

The correct answer is "D:' The presence of vertebral and limb anomalies is concerning for VACTERL association (Vertebral, Anorectal, Cardiac, TracheoEsophageal fistula, Renal, Limb anomalies) , also known as VATERR association. It is not a syn­ drome but rather a group of malformations that occur together. Anorectal abnormalities include imperforate anus and anal atresia. A thorough physical exam and possible surgical evalu­ ation should be top priority. Hypothyroidism can cause con­ stipation. Although the newborn screen has likely ruled out (> 95% likelihood) congenital hypothyroidism, checking levels of thyroid -stimulating hormone (TSH) and free thyroxine (T 4 ) is reasonable. Barium enema can be done at this age to assess whether the rectosigmoid index is reversed in Hirschsprung



Helpful Tip

� Excessive

:5.

i nta ke of cow's m i l k (> 32 ou nces per

r1 1 r day) is a common ca use of constipation in todd lers.

Approximately 24 ounces per day is adequate to meet the body's needs.

A 6-year-old boy with a 2-year history of constipation is cur­ rently on polyethylene glycol 3350 (Miralax) therapy daily but continues to have daily fecal incontinence.

342

MCG RAW-H I LL E D U CATION S P E C I A LTY BOA R D REVI EW: P E D I ATRICS

TABLE 1 6-9 M E D I CATI O N S TO TREAT CO N ST I PATI O N

Medication Class

Exa m p l e

M o d e o f Action

Route

Laxatives

Senna

I r ritates the s m ooth m u scle; sti m u lates the i ntesti ne to contract

Ora l

Softe n s stool con s i stency by p u l l i ng fl u i d i nto stoo l; i n c reases stool freq uency; nonsti m u l a nt

Ora l

M a kes stoo l s l i c k s o it is easier t o pass

Ora l

B i sacodyl

Recta l

G lycer i n Osmotic stool softe ners

Polyethyl ene g lyco l 3 3 5 0 (M i ra l ax) Lact u l ose M agnesi u m hyd roxide

Lu brica nts

M i nera l o i l

Question 30-1

Question 31 -1

The most likely cause is:

The primary source of bilirubin production is:

A) B) C) D)

A) B) C) D) E)

Lack of scheduled daily toilet sitting. Presacral mass. Ectopic anus. Nonretentive fecal soiling.

Hepatocytes. Gallbladder. Hemoglobin. Platelets. Skin.

Discussion 30-1

The correct answer is ''A:' Stool softener use without scheduled toilet sitting (every morning and after every meal for 3 to 5 minutes, with properly supported feet) often does not correct constipation problem. Detailed history on daily toilet sitting (or lack thereof) usually gives you the diagnosis. Make sure the neurologic exam is normal, with an anal wink, and there is no sacral dimple to suggest underlying spinal cord pathology such as a tethered cord.

� QUICKQUIZ Which is NOT a cause of constipation?

A) B) C) D)

Hypercalcemia. Tethered cord. Insufficient water intake. Hyperthyroidism.

Discussion

The correct answer is "D:' Hypothyroidism causes constipa­ tion. Constipation may be ( 1 ) functional (withhold stool)-for example, forced potty training or sexual abuse; (2) anatomic­ for example, imperforate anus or anteriorly displaced anus; or (3) physiologic-for example, cystic fibrosis or spinal cord pathology.

A newborn girl is noted to be jaundiced on the first of life. Her total bilirubin level is 1 5 mg/dL, with a direct bilirubin of 1 mg/dL, and the direct antiglobulin testing is positive. She is started on phototherapy.

Discussion 31 -1

The correct answer is "C:' Neonatal jaundice is a common occurrence and is most often nontoxic and transient. How­ ever, complications can occur if the total bilirubin level is too high or there is an elevated conjugated bilirubin level. Bili­ rubin is formed from the degradation of heme compounds. Heme is converted to biliverdin by heme oxygenase, and bili­ verdin is converted to bilirubin by biliverdin reductase in the reticuloendothelial cells of the spleen and liver. The water­ insoluble unconjugated bilirubin is transported to the liver primarily bound to albumin. It is taken up by hepatocytes and conjugated by uridine diphosphoglucuronate (UDP)-gluc­ uronosyltransferases (UGT), enzymes in the glucuronidation pathway. Conjugated bilirubin is secreted in bile by a specific transporter. In the small intestine, some of the conjugated bilirubin is hydrolyzed to unconjugated bilirubin. Most of the unconjugated bilirubin is then excreted in the stool, but some is reabsorbed into the bloodstream and returned to the liver in a process known as enterohepatic circulation. Therefore, if an infant has hemolytic anemia, the hyperbilirubinemia is more severe. Question 31 -2

In differentiating indirect from direct hyperbilirubinemia, a fractionated bilirubin analysis is necessary. When is the level of direct bilirubin abnormal?

A) When the direct bilirubin level is greater than 0.5 mg/dL or the percentage of direct bilirubin is less than 10% of the total bilirubin, or both. B) When the direct bilirubin level is greater than 2 mg/dL or the percentage of direct bilirubin is greater than 20% of the total bilirubin, or both.

C H A PT E R 16

C) When the direct bilirubin level is greater than 1 mg/dL or the percentage of direct bilirubin is 5% of the total bilirubin, or both. D) When the direct bilirubin level is greater than 1 mg/dL or the percentage of direct bilirubin is 15% of the total biliru­ bin, or both.



HYP E RB I L I RU B I N EM I A

I nc reased prod uction



1 1 1r

For a tota l bilirubin of 5 mg/d l or less, the d i rect component should be less than 2 mg/d l. For a tota l bilirubin g reater than 5 mg/d l, the d i rect com ponent should be less than 20% of the tota l .

A 14-year-old boy presents t o your office with complaints of fatigue, upper respiratory symptoms, fever, and "yellow eyes:' A multiplex nasal swab confirms influenza A infection. Laboratory analysis includes the following pertinent results: total bilirubin 3.5 mg/dL; direct bilirubin 0.4 mg/dL; and normal AST, ALT, albumin, PT, and glucose levels. Direct antiglobulin testing for hemolysis is negative. Question 32-1

What is the most likely etiology of the hyperbilirubinemia?

A) B) C) D)

Gallstone obstruction. Liver failure in the setting of systemic viral infection. Hemolytic anemia. Gilbert syndrome.

Discussion 32-1

The correct answer is "D:' Gilbert syndrome is an autosomal dominant condition characterized by mild indirect hyperbili­ rubinemia resulting from impaired bilirubin conjugation with otherwise normal liver function tests and absence of hemolysis. It most often presents in pubertal boys. During stress or illness, mild scleral icterus or jaundice is often observed. Children with Gilbert syndrome have a mutation in UGTlAl, which encodes for bilirubin-UGT. Bilirubin-UGT conjugates bilirubin into a water-soluble form excreted in bile. Gilbert syndrome is benign and is not associated with negative implications for long-term health. (See Table 1 6- 1 0.) •



1 1 1r

Helpful Tip

When investigating i n d i rect hyperb i l i rubi nem ia, order a d i rect a ntiglobu l i n test (OAT) to ru le out hemolysis.

Hemolys i s Po lycythem ia Extravasati o n of b l ood i nto tissues (cephalohe matoma, bru i s i ng)

Discussion 31 -2

Helpful Tip

343

TABLE 1 6- 1 0 CAU S E S OF I N D I R ECT

Concea led hemorrhage (ad re n a l g l a nd)

The correct answer is "B:' High total bilirubin levels often are associated with a slightly elevated direct bilirubin level-even when the problem is indirect hyperbilirubinemia. These param­ eters help to distinguish pathologic from nonpathologic direct bilirubin levels. •

P E D I ATRIC GASTRO ENTEROLOGY

Enterohepatic c i rcu lation B reast m i l k j a u n d ice Physiologic j a u n d ice of the newborn Decreased hepatic u pta ke

Medications

Decreased conj u g ation

G i l bert syn d ro m e Crig l e r- N ajja r syn d rome Hypothyro i d i s m

Suppose the 14-year-old had a total bilirubin of 3.5 mg/dL, with a direct component of 2.5 mg/dL, consistent with a direct hyperbilirubinemia. On exam he is febrile, toxic, and has right upper quadrant abdominal pain. Question 32-2

Which diagnostic test should be ordered?

A) B) C) D) E)

Right upper quadrant ultrasound. Sweat chloride testing. Cytomegalovirus ( CMV) serology. Urinalysis. Prothrombin and partial thromboplastin times (PT, PTT).

Discussion 32-2

The correct answer is "A:' Liver function tests, including a gamma glutamyl transferase (GGT) level and a fractionated bilirubin analysis, are the first tests ordered when evaluating a child who is jaundiced. The direct bilirubin level is the fork in the road. Indirect hyperbilirubinemia points to red blood cell hemolysis. Direct hyperbilirubinemia points toward cholestatic liver disease. Elevated transaminases (ALT, AST) indicate hepa­ tocellular damage. Elevated alkaline phosphatase (AP) and GGT indicate cholestasis. The second round of testing depends on the individual scenario and the most likely diagnosis. In this case a right upper quadrant ultrasound will look for suspected gall­ bladder pathology (cholecystis, cholangitis). (See Table 1 6- 1 1 . )

A 16-year-old previously healthy girl presents to your office with pharyngitis, fever, cervical lymphadenopathy, and fatigue. Her throat swab is negative for group A streptococcal

344

MCG RAW-H I LL E D U CATION S P E C I A LTY BOA R D REVI EW: P E D I ATRICS

TABLE 1 6- 1 1 CAU S E S O F D I RECT HYP E R B I L I RU B I N E M I A I nfectious

Cholang itis Vi ra l hepatitis

Heredita ry d i sorders of b i l i ru b i n tra n s port

Roto r syn d rome

Cytomega l ovirus Epstei n-Barr virus

Drugs

Herpes s i m plex virus

H i sto plasmosis

Parentera l n utrition Meta b o l i c d i sorders

N i e m a n n-Pick d i sease

Sepsis

G lycogen sto rage d i sease

U ri n a ry tract i nfecti o n Bacterial i n fections

M itoch on d rial d i sorders

Auto i m m u ne hepatitis

Wi lson d i sease

Pri m a ry scleros i n g chola n g itis

Cystic fi bros i s

Pri m a ry b i l i a ry ci rrhos i s Extra hepatic b i l i a ry obstruction

G raft-versus-host d i sease

A l p h a - 1 -antitrypsi n deficiency

B i l i a ry atresia

Hemochromatosis

Choledoch a l cyst

Endocrine

G a l l stones I n spi ssated bile syn d rome I ntra h e patic b i l i a ry d i sease

Galactose m i a Tyrosi n e m i a

Liver a bscess

I m m u ne medi ated

Medications (aceta m i nophen, rifa m pi n, oxaci l l i n) Alcohol

Va rice l la-zoster virus Co ngen ital i nfections

Dubin-J o h n so n syn d ro m e

Hypothyro i d i s m Hypopitu ita rism

Other

Va sc u l a r a n o m a l ies

M a l i g n a ncy

Budd-C h i a ri synd ro m e

B i l i a ry atresia Alag i l le syn d rome (intra hepatic d uct paucity)

S i n u so i d a l obstruction syn d ro m e (vena-occ l u s ive d i sease)

Ca ro l i d i sease (congen ita l hepatitic fi bros i s)

Ca rd i a c i n s ufficie ncy a n d hypo perfusion

M a l i g n a ncy

Trisomies 2 1 , 1 8

infection. Initial biochemical evaluation is remarkable for a white blood cell count of 3000/mm3 with some atypical lym­ phocytes, normal hemoglobin and platelet count, and mild transaminase elevation (AST 88 units/L, ALT 92 units/L) . Monospot is negative. Question 33-1

infectious mononucleosis caused by EBV or CMV A teenager, pharyngitis, and reactive lymphocytosis-who cares about a negative Monospot, which can be negative early in infection (and in young children). Liver transaminases are transiently elevated in 80% of cases. You could consider sending serology testing for EBV and CMV or prescribe watchful waiting. She is healthy, not jaundiced, and with only mild elevations of her transaminases.

What is the most likely cause of her hepatitis?

A) B) C) D) E)

Epstein-Barr virus (EBV) infection. Hepatitis B infection. Cytomegalovirus (CMV) infection. Autoimmune hepatitis. Alcohol toxicity.

Discussion 33-1

The correct answer is "D:' This adolescent girl has viral hepatitis. Elevated transaminases indicate hepatocellular injury or inflam­ mation. Her clinical presentation is most consistent with acute

� QUICKQUIZ What is NOT a cause of viral hepatitis?

A) B) C) D) E)

Epstein-Barr virus (EBV). Herpes simplex virus (HSV). Respiratory syncytial virus (RSV). Varicella-zoster virus. Cytomegalovirus.

C H A PT E R 1 6



P E D I ATRIC GASTRO ENTEROLOGY

345

Discussion

Question 3 5-1

The correct answer is "C:' Viral causes include EBV; CMV; hepatitis A, B, C, D and E; HSV; human herpesviruses; measles; rubella; varicella; adenovirus; and human immunodeficiency virus (HIV) .

In addition to tests to assess liver synthetic function, you would recommend:

A 1 9-year-old male adolescent presents to the student health center for evaluation of acne. On exam, you note he has hep­ atomegaly. You order blood work; his ALT is 95 units/L and AST 70 units/L. He is not taking any medications. He denies weight loss, vomiting, fever, odd-colored stools, or jaun­ dice. He drinks on the weekends but no more than two to three drinks per weekend. He tests positive for hepatitis C. With additional questions, he admits to IV drug use.

The correct answer is "E:' Wilson disease is an autosomal reces­ sive disease with impaired copper transport. The excess copper accumulates in the liver and brain. It often manifests in the sec­ ond decade of life with a primary hepatic presentation (hepa­ titis, jaundice, cirrhosis, liver failure). Psychiatric features can also result from the impaired biliary copper excretion, which leads to copper accumulation in the liver and other organs (brain). Ophthalmologists can confirm the presence of Kayser­ Fleischer rings with slit lamp. Lifelong treatment is necessary and aimed at chelating the stored copper and thus removing it from affected organs.

A) B) C) D) E)

Alpha- 1 -antitrypsin phenotype. Ophthalmology exam with slit lamp. Hepatobiliary iminodiacetic acid (HIDA) scan. Ceruloplasmin and 24-hour urine copper level. Both B and D.

Discussion 3 5-1

Question 34-1

Which is not a cause of chronic hepatitis?

A) B) C) D) E)

Alcohol abuse. Hepatitis B infection. Nonalcoholic fatty liver disease. Infectious canine hepatitis. Autoimmune hepatitis.

Discussion 34-1

The correct answer is "D:' The question should have specified in humans. Consider this a reminder to read questions carefully. Chronic hepatitis is most often infectious. Initial diagnostic evaluation should focus on viral testing; if negative, workup for autoimmune and metabolic processes should occur. The his­ tory should include risk factors for hepatitis B and C and HIV infection, including sexual activity, history of transfusion, and illicit drug use. Symptoms of chronic hepatitis may be mild or absent. Chronic hepatitis of any etiology may progress, leading to the development of liver failure, cirrhosis, and possible need for liver transplant.

Your young colleague looks at you in confusion and asks, "What liver synthetic function tests? " Question 3 5-2

Which of the following tests do you tell him to order?

A) B) C) D) E)

PT and PTT. Sodium. Hemoglobin. 1 ,25-dihydroxyvitamin D. Glucose.

Discussion 35-2

The correct answer is "A:' (See Table 16-12.)

TABLE 1 6- 1 2 LIVER F U N CT I O N TESTS Synthetic fu nction

Proth ro m b i n t i m e (PT), pa rti a l t h ro m bop lasti n t i m e (PTT); i nternatio n a l normal ized ratio (I N R) Albumin

An 1 8-year-old female adolescent returns from her first semester of college and presents to your office with fatigue and yellow skin and eyes. Her mother accompanies her to this visit and informs you that her daughter has been act­ ing "paranoid" and "different than usual" since she returned home for winter break. Last night, she had epistaxis that persisted for several hours. You are concerned about syn­ thetic liver dysfunction in this setting. On exam, her liver is enlarged and she has mild jaundice with scleral icterus. Her mental status appears compromised. You begin the process of hospital admission. You need to give report to the inpa­ tient GI team.

A m m o n i a ( N H 3) He patoce l l u l a r i nj u ry

A l a n i n e a m i n otra n sferase (ALT) and aspa rtate a m i n otran sfe rase (AST) Lactate de hyd rogenase (LDH)

Chol esta sis

B i l i ru bi n-d i rect component Gamma g l utamyl tra n sferase (GGT) A l ka l i ne phosp hatase (AP) U robi l i nogen

346

MCG RAW-H I LL E D U CATION S P E C I A LTY BOA R D REVI EW: P E D I ATRICS

A 2-month-old male infant presents with jaundice. Initial exam is remarkable for jaundice, scleral icterus, and hepato­ megaly without a palpable spleen. Initial laboratory results were remarkable for AST 1 50 units/L, ALT 300 units/L, GGT 82 units/L, and normal PT, INR, albumin, and glucose. His total bilirubin was 6 mg/ dL, with a direct bilirubin of 3.2 mg/ dL. The workup was negative for infectious etiologies, and an ultrasound showed mild hepatomegaly with a homogenous liver without signs of portal hypertension or sonographic evidence of biliary obstruction. Subsequently, a HIDA scan showed good excretion of bile into the intestine. Question 36-1

What are the next steps in the care of this patient?

A) B) C) D)

Alpha- 1 -antitrypsin level and phenotype. Liver biopsy. Bronchoscopy. Both A and B.



I.

,.



� #' .;_

,

• a

•'

� C> �

;:



4



F I G U R E 1 6-4. Alpha-1 -Antitrypsin Liver Biopsy. In a l pha-1 -antirypsin deficiency, m uta nt proteins (pink g lobu les) can not exit the hepatocyte due to protein mutation. (Reproduced with permission From, Kasper DL, Fa uci

AS, Ha user SL, Longo DL Ja meson J L, Losca lso J (Eds) Harrison's Principles of Internal Medicine, 1 9ed. McGraw- H i l l Education, I n c., 201 5 . Fig 366E- 1 9.)

Discussion 36-1

The correct answer is "D:' Infants with alpha- 1 -antitrypsin (AT) deficiency may present with liver involvement (direct hyper­ bilirubinemia) between 1 and 2 months of age. Older children may present with portal hypertension after a period of "unex­ plained" neonatal jaundice. The classic alpha- 1 -AT phenotype is homozygous "PiZZ"; however, we often recommend obtaining an alpha- 1 -AT level in addition to the phenotype to guide the diagnostic evaluation since the phenotype result may take 10 to 14 days. Although the alpha- 1 -AT level is an acute phase reac­ tant, deficient levels at baseline often do not increase substan­ tially in the setting of illness. According to the current literature, heterozygous individuals (alpha- 1 -AT phenotype = PiMZ) may or may not develop liver disease. Lung disease (emphysema) in alpha- 1 -AT deficiency typically occurs in adults. In alpha- 1 -AT­ deficient individuals, cigarette smoking reduces mean survival by more than 20 years. The pathogenesis of alpha- 1 -AT defi­ ciency involves a mutant protein (single amino acid substitu­ tion) that is trapped within cells and, thus, deficient in the blood and body fluids. The histopathology of alpha - 1 -AT deficiency in hepatocytes reflects the "buildup" of the mutant protein within hepatocytes. (See Figure 1 6-4.) Of course, certain etiologies of cholestasis in infants are time sensitive and if surgery is needed, this should be done as quickly as possible. The normal HIDA scan ruled out biliary atresia for this infant.

Question 3 7-1

Which of the following blood tests would you ask the labora­ tory to add first in this setting?

A) B) C) D) E)

EBV serologies. Alpha- 1 -AT level. Fractionated bilirubin. PT and INR. Albumin.

Discussion 3 7-1

The correct answer is "C:' Breastmilk jaundice may cause pro­ longed neonatal jaundice but after 4 weeks you need to know if the hyperbilirubinemia is indirect or direct. Direct hyperbiliru­ binemia in infancy is a sign of cholestasis due to extrahepatic biliary obstruction, defects in bile synthesis, other metabolic etiologies, Alagille syndrome, and infections. (See Table 1 6- 1 1 , earlier.) The direct bilirubin is elevated at 3 mg/dL. You are worried about biliary obstruction, especially biliary atresia. You rec­ ognize the need for an urgent evaluation as the clock is tick­ ing if surgical intervention is needed. Question 3 7-2

A 1 -month-old infant girl who is breastfed presents for rou­ tine well-infant evaluation and is noted to have jaundice. Her exam is normal otherwise and her growth is good. Ini­ tial laboratory evaluation is remarkable for the following: comprehensive metabolic panel with AST of 66 units/L, ALT 78 units/L, total bilirubin 7 mg/dL, and a CBC with normal white blood cell, hemoglobin, and platelet counts.

In the setting of extrahepatic biliary atresia, which might you see?

A) B) C) D) E)

Small, atrophic, or absent gallbladder on liver ultrasound. Butterfly vertebrae on X-ray. Kayser-Fleischer rings on slit lamp examination. Pulmonary atresia identified on echocardiogram. Gallstones on liver ultrasound.

C H A PT E R 1 6



P E D I ATRIC GASTRO ENTEROLOGY

347

Discussion 3 7-2

Question 3 7-4

The correct answer is "A:' Extrahepatic biliary atresia is a progressive, idiopathic disorder. Patients develop biliary obstruction in the neonatal period, presenting with j aundice and acholic stools (pale, chalk, or clay colored) . Parents may not notice the funny-colored poop. Fortunately stool color cards and free smartphone applications are available that can help determine whether the poop is normal colored. Another nuclear medicine study that is used to assess biliary function in this setting is a HIDA scan (to see if bile is excreted into the small intestine) . Often, a liver biopsy is necessary to assess for bile duct proliferation histopathologically.

Which is NOT a potential clinical presentation of a choledochal cyst?

The ultrasound is consistent with extrahepatic biliary atresia. You sit down to tell the mother that her infant will need urgent surgery. She is tearful and asks when it will be done as her baby is so tiny.

A) B) C) D) E)

Fetal abdominal cystic mass. Indirect hyperbilirubinemia. Cholangitis. Direct hyperbilirubinemia. Pancreatitis.

Discussion 3 7-4

The correct answer is "B:' Choledochal cyst is congenital anomaly with cystic dilation of the biliary tree. The classic triad is abdomi­ nal pain, jaundice, and right upper quadrant mass, but presenta­ tion varies by age. The presentation in infants is similar to biliary atresia, with obstructive jaundice, acholic stools, and hepato­ megaly. Children and adolescents present with abdominal pain, cholangitis, jaundice, or pancreatitis. The cyst causes obstruction of bile flow causing a direct, not indirect, hyperbilirubinemia.

Question 3 7-3

In the setting of extrahepatic biliary atresia, what is the optimal age for a Kasai (hepatoportoenterostomy) procedure? A) At 6 months of age after the infant has grown and developed

(to guard against surgical complications). B) At 4 months of age, because the infant can begin to transi­ tion to solid foods after recovering from surgery. C) At 3 months of age, because the bile ducts are better visualized. D) Before 2 months of age to have the greatest success in obtain­ ing bile flow.

A 14-year-old girl presents with elevated transaminases. Over the last month, she has had fever and fatigue. She has gotten progressively weaker and can no longer lift her arms above her head. On exam, she appears to have faint purple eyeshadow on and scaling over her knuckles. She is unable to keep her leg lifted off the bed for more than 10 seconds. Her AST is 500 units/L and is ALT 250 units/L. Bilirubin, gamma glutamyl transferase (GGT), lactate dehydrogenase, albumin, alkaline phosphatase, and coagulation studies are normal.

Discussion 3 7-3

What is the cause of her elevated transaminases?

The correct answer is "D:' With respect to the Kasai procedure, the timing of surgery correlates with outcome. In the Kasai procedure, a loop of small intestine is directly anastomosed to the liver. Infants younger than 60 days have a greater than 80% chance of reestablishing bile flow. If the Kasai procedure is per­ formed after age 90 days, the percentage of patients successfully reestablishing bile flow drops to less than 20%. Therefore, it is very important to see infants in your office at 1 month of age for checkup. If the infant is yellow, do not ignore it.

A) B) C) D) E)

Question 38-1





Helpful Tip

A n i nfa nt presenti n g with d i rect hyperbi l i ru b i n e m i a

1 1 1r m u st

have extra h e pa t ic b i l i a ry obstruction­ specifi ca l l y, b i l i a ry atres i a - r u l e d o u t a s t i m e l y s u rg i c a l i n terve n t i o n i s req u i red to save t h e l iver.

Let's say, instead, that the same 1 -month-old who pre­ sented to her well-infant check with a direct hyperbilirubi­ nemia and elevated transaminases had different findings on ultrasound. This time, cystic dilation of the common bile duct is seen.

Myositis/rhabdomyolysis. Muscular dystrophy. Hemolysis. Alcohol. All of the above.

Discussion 38-1

The correct answer is ''A:' Elevated liver transaminases may originate from the liver, muscle, and red blood cells. Typically in liver pathology outside of alcohol toxicity, the ALT is greater than the AST. Extrahepatic sources should be considered when the pattern is reversed. In this case our patient has dermato­ myositis. Her creatine kinase was elevated at 4000 units/L. Of note, alkaline phosphatase is also found in bone. Elevated GGT supports a liver source.

A 1 3-year-old previously healthy boy faints at a basketball game and is taken to your office for evaluation. The history is remarkable for recent passage of black stools, and the exam is remarkable for pale skin and conjunctiva, and tachycardia. You suspect upper GI bleeding.

348

MCG RAW-H I LL E D U CATION S P E C I A LTY BOA R D REVI EW: P E D I ATRICS

Question 39-1

Question 39-2

Another historical element that could go with this picture is:

What would you expect the hemoglobin value to be based on his clinical presentation?

A) B) C) D) E)

Recent pseudoephedrine (Sudafed) administration. Recent ibuprofen administration. Recent bismuth subsalicylate (Pepto Bismol) administration. Recent diphenhydramine (Benadryl) administration. Both B and C.

A) B) C) D)

12 mg/dL. 1 1 .5 mg/dL. 10 mg/dL. 6 mg/dL.

Discussion 39-1

Discussion 39-2

The correct answer is "E:' Nonsteroidal anti-inflammatory drugs (NSAIDs) are known to cause gastric ulcers. Pepto Bis­ mol in the original (not children's) formula contains bismuth subsalicylate, which is an aspirin derivative that can also cause gastric ulcers. Bleeding originating from the upper GI tract (before the ligament of Treitz) may present as hematemesis (including coffee ground) or melena (black stools) . Lower GI tract bleeding typically presents as hematochezia or bright red blood from the rectum. Warning: A brisk upper GI bleed may present as hematochezia if the blood travels quickly through the intestinal track. Either may cause a positive fecal occult blood test. The etiology of a GI bleed is dependent on the child's age and history, as well as the location (upper or lower) . (See Table 1 6- 1 3.)

The correct answer is "D:' Most often, there is a precipitous drop in hemoglobin when the body has not been able to compensate (ie, fainting occurs) . This occurs with acute blood loss. With chronic GI blood loss, the body is able to compensate to main­ tain a normal blood pressure. Patients are more likely to present with fatigue and pallor rather than syncope.

You push the patient i n a wheelchair t o the lab for blood­ work. You want the results fast and don't want him sitting in the waiting room forever. His coagulation studies and plate­ let count were normal. The lab is working on his blood type and cross match; hemoglobin was not normal.

TABLE 1 6- 1 3 CAU S E S OF U P P E R A N D LOWER GASTRO I NTEST I N A L B L E E D I N G

U pper G l Bleed i n g

Lower G l Bleed i n g

Swa l l owed maternal bl ood

A n a l fi s s u re

Swa l l owed nasopharyn­ geal blood (epistaxis)

M a l rotation with vo lvu l u s

Esophagitis

Necrotizi ng enteroco l itis

Gastritis

A l l e rg i c proctoco l i t i s

Peptic u l cer d i sease

C ro h n d i sease

Esophageal va rices

U l cerative co l itis

Cro h n d i sease

Meckel d ivertic u l u m

Coag u lo pathy

I ntussu sception

M a l lory-Weiss tea r

I nfectious co l itis I ntesti n a l polyps Henoch-Sch o n l e i n p u r p u ra Coag u lo pathy Hemorrhoids Lym phoid hyperp lasia M a l i g n a ncy

r1J •

Helpful Tip

I ron deficiency anemia is a sign of occu lt Gl bleed i n g .

Question 39-3

You nearly faint when you read his hemoglobin value. The most appropriate next step is:

A) Emergent upper endoscopy (EGD) for likely upper GI bleed. B) Surgery consult. C) Hemodynamic stabilization with a large-bore IV catheter placement and fluid bolus. D) Repeat of the lab test; it has to be a lab error. E) Transfer to the emergency department. Discussion 39-3

The correct answer is "C:' The key to this question is the first step to take. Remember your ABCs-"C" is for circulation. With acute blood loss, you should resuscitate with IV fluids. Once stabilized, the patient should be transferred to the emer­ gency department by ambulance to be transfused with packed red blood cells and have an EGD to find and fix the source of bleeding. In a patient with anemia due to chronic blood loss, go gingerly with fluid resuscitation to avoid putting the patient into heart failure. With acute hemorrhage, go for it. When a critical lab value returns, it is always good to stop and ask, is it plau­ sible? In this case, the answer is yes. The patient is at the emergency department receiving a blood transfusion when he is wheeled to the endoscopy suite for EGD. Question 39-4

The most likely finding on an EGD in this setting is:

A) B) C) D) E)

Nothing; the bleeding source is more distal. Esophagitis. Gastritis. Larger gastric ulcer with visible vessel. Esophageal varices.

C H A PT E R 1 6



P E D I ATRIC GASTRO ENTEROLOGY

349

Discussion 39-4

Discussion

The correct answer is "D:' Options "B" through "D" can all cause hematemesis. This patient does not have signs of portal hyper­ tension, so varices are unlikely. Esophagitis and gastritis are more indolent conditions that do not present with acute hem­ orrhage. A large gastric ulcer would fit with this scenario. The visible vessel predicts a significant increased risk of rebleeding if no endoscopic intervention is performed.

The correct answer is "A:' Vomited blood may be bright red or have a coffee ground appearance. Stomach acid breaks down hemoglobin, giving it a coffee-ground color. Bright red vomit indicates a brisk or acute bleeding source (eg, esophageal vari­ ces, large ulcer) . Coffee ground emesis is typical of gastritis, esophagitis, or swallowed blood.

Question 39-5

Which is a cause of peptic ulcer disease in adolescents?

A) B) C) D) E)

Alcohol. Helicobacter pylori infection. Medications. Critical illness. All of the above.

A 2-month-old healthy term breastfed infant presents with streaks of bright red blood in an otherwise normal-appearing stool. She is happy, and her abdominal exam is benign. She has been growing well. Question 40-1

Discussion 39-5

The most likely etiology is:

The correct answer is "E:' Consider binge drinking in adoles­ cents with gastritis. Common medications include NSAIDs and steroids. H. pylori infection should be considered in the absence of another risk factor such as NSAID use. The infection is treated with antibiotics.

A) B) C) D) E)





Helpful Tip

Teenag ers who swa l low p i l l s without water risk having

1 1 1r the

p i l l become lodged i n the lower esophagus.

Prolonged contact with the p i l l cau ses i rritation of the

m u cosa, referred to as pill esophagitis. This has been reported with tetracyc l i nes (doxycycl i ne, m i n ocyc l i ne), which a re often prescribed for acne.

Question 39-6

After the results of the EGD are reviewed with the team, the most appropriate treatment is:

A) B) C) D)

Discharge home with an oral proton pump inhibitor (PPI). Admit the patient overnight for additional IV fluids. Discharge home with sucralfate. Admit for IV PPI therapy, serial monitoring of lab values, and observation for rebleeding.

Discussion 39-6

The correct answer is "D:' There is always a risk of rebleeding, even after an endoscopic intervention, so the patient must be monitored closely.

� QUICKQUIZ What is not a cause of coffee ground emesis?

A) B) C) D) E)

Mallory-Weis tear. Esophagitis. Dental trauma. Epistaxis. All of the above.

Lymphoid hyperplasia. Cow's milk protein intolerance. Meckel diverticulum. Intestinal polyp. Intussusception.

Discussion 40-1

The correct answer is ''A:' Lymphoid hyperplasia is very com­ mon in infants and should resolve spontaneously without nec­ essary interventions. Intussusception causes colicky ab dominal pain, emesis, and dark maroon-colored poop (late finding) . Options "C" and "D" are discussed later, in Cases 42 and 43. Question 40-2

The best management advice for this problem is:

A) Have the breastfeeding mother remove all cow and soy protein from her diet. B) Stop breastfeeding. C) Reassurance without changes. D) Prescribe PPI medication. Discussion 40-2

The correct answer is "C:' Removal of cow's milk proteins from the maternal diet has not been shown to change the course of benign lymphoid hyperplasia and may decrease maternal milk supply if her calorie intake is not sufficient in the setting of dietary elimination.

A 5-year-old white male child presents with painless rectal bleeding. His vital signs and exam are normal. His hemoglo­ bin is 12 mg/dL. The child drinks his bowel preparation like a champ, and a colonoscopy is performed. Question 41 -1

What is most likely to be found?

A) Juvenile polyp. B) Lymphoid hyperplasia.

MCG RAW-H I LL E D U CATION S P E C I A LTY BOA R D REVI EW: P E D I ATRICS

350

C) Severe colitis. D) Internal anal fissure. E) Malignancy.

� QUICKQUIZ What is NOT a cause of a positive fecal occult blood test?

Discussion 41 -1

The correct answer is "A:' The most common cause of pain­ less rectal bleeding in this age group is a juvenile polyp. An intestinal polyp is a benign hamartoma that causes painless GI bleeding. The polyp may fall off with increased bleeding. Multiple intestinal polyps (> 10) with a family history of pol­ yps is consistent with juvenile polyposis syndrome. Due to the increased risk of malignancy, coloscopies should be performed every few years in patients with this syndrome. Peutz-Jeghers syndrome is an inherited condition characterized by multiple pigmented spots on the lips and inside of the cheeks and mul­ tiple GI hamartomatous polyps. When investigating a lower GI bleed, key questions relate to whether the bleeding is ( 1 ) pain­ less or painful, (2) bright red or tarry, and (3) stool consistency. For example, a patient with an infection may present with loose stools mixed with bright red blood and mucus associated with cramping.

A 1 3-year-old boy presents with rapid red blood per rectum that is painless. His hemoglobin is 5 mg/dL. An IV line is placed and he is given fluids and transfused with packed red blood cells. Question 42-1

The next best step in investigating the source of bleeding is:

A) Colonoscopy to diagnosis and treat the bleeding source. B) Upper endoscopy (EGD) since the rapid bleeding must be gastric in origin. C) Meckel scan. D) Liver biopsy. Discussion 42-1

The correct answer is "C:' A Meckel diverticulum often contains gastric mucosa, which secretes acid and can cause an ulceration of the adjacent ileal mucosa, resulting in significant bleeding. The bleeding is often brisk (therefore, it may be bright red in color despite the source being proximal to the colon) and pain­ less. Remember the Rule of 2s: affects 2% of the population, males 2 times more likely than females, 2 inches long, 2 feet from the ileocecal valve, and contains 2 types of tissue (gastric and intestinal) . •



Helpful Tip

A) B) C) D) E)

Crohn disease. Gastritis. Horseradish. Swallowed blood. Iron supplements.

Discussion

The correct answer is "E:' The question did not differenti­ ate between false and true positive results. False-positive fecal occult blood tests may be caused by eating meat, tomatoes, fresh cherries, turnips, and horseradish. Iron supplements do not cause false-positive tests. True positives detect hidden bleed­ ing somewhere in the digestive tract that is not visible to the human eye. The bleeding may be anywhere along the GI tract. It is commonly positive with smoldering processes such as Crohn disease, peptic ulcer disease, malignancy, and cow's milk pro­ tein intolerance. Current iron deficiency anemia from chronic GI blood loss is common.

A 2-year-old boy passed a bowel movement in his diaper with some bright red blood mixed within the soft stool sub­ stance. He did not cry or display discomfort during or after defecation. When wiping him, you see a small mass protrud­ ing from the left side of the anal orifice. This spontaneously reduces into the rectum when you try to examine it further. The family history is negative for polyposis syndromes. Question 43-1

The most likely cause of this finding is:

A) B) C) D)

Vascular malformation. Juvenile polyp. Rectal prolapse. Both B and C.

Discussion 43-1

The correct answer is "B:' Painless rectal bleeding in a 2-year­ old is most commonly due to a solitary rectal juvenile polyp. A rectal prolapse is characterized by the protrusion of concentric rings of rectal mucosa and is not typically described as unilateral in position. A rectal prolapse can be easily reduced and is most common in infants. Finding the underlying cause of prolapse is important. Rectal prolapse may occur in patients with cystic fibrosis, constipation, pertussis (increased abdominal pressure with coughing), and infectious enteritis. Often, a picture of the finding helps to distinguish between the two entities.

I ntussusception a n d Meckel d iverticu l u m may present

1 1 1 r with dark ma roon-colored stool descri bed as "currant jel ly."

After confirmation of your clinical suspicion (both endo­ scopically and histologically) and removal of the polyp, this child will need to return to the GI clinic:

C H A PT E R

16



P E D I ATRIC GASTRO ENTEROLOGY

351

Question 43-2

A) B) C) D)

Only if rectal bleeding recurs and persists. Annually. In 6 months. In 1 month, when you will recheck his hemoglobin.

Discussion 43-2

The correct answer is ''A:' Solitary juvenile polyps are benign and do not warrant further follow-up.

A 3-year-old boy presents for a well-child check-up with a distended liver edge palpable 4 em below the right costal margin. The history is remarkable for recent increases in soft abdominal girth without other problems. He is growing and developing normally. His parents have not witnessed any excessive bleeding or bruising. Question 45-1

The next step in evaluation should include:

A 1 3-year-old girl presents to your clinic with new-onset hematochezia for 1 week. She is currently passing two soft, formed stools per day, each with blood mixed throughout the stool substance. Her history is otherwise unremarkable. Her family history is remarkable for fatal colon cancer in her pater­ nal grandfather at 38 years, colon cancer in her father at age 26 years (treated with chemotherapy and colectomy), and colon cancer in her 1 9-year-old brother, who is currently undergoing chemotherapy after colectomy. Her exam is nor­ mal, without evidence of tachycardia or other signs of signifi­ cant anemia. Question 44-1

You recommend colonoscopy to better define the source of bleeding and find the following endoscopically:

A) B) C) D)

A single mass in the rectum that has polypoid features. Gardens of small, sessile polyps throughout the colon. Nothing in the colon; the bleeding must be more proximal. Endoscopic features of chronic colitis.

Discussion 44-1

The correct answer is "B:' The most likely etiology is familial adenomatous polyposis (FAP) due to the significant colon can­ cer affecting family members at very young ages. FAP is an auto­ somal dominant condition that causes hundreds to thousands of colon polyps and eventual colon cancer if untreated. It is due to a mutation in a tumor suppressor gene (APC, adenomatous polyposis coli).

A) Laboratory testing, including complete liver function panel, serum glucose, and CBC. B) Abdominal ultrasound. C) Abdominal ultrasound and the following laboratory tests: AST, ALT and albumin. D) Abdominal ultrasound and the following laboratory tests: complete liver hepatic function panel, serum glucose, PT, INR, and CBC. Discussion 45-1

The correct answer is "D:' In this setting, sonographic evidence is necessary as is confirmation of normal liver synthetic func­ tion. PT, INR, albumin, and ammonia are markers of synthetic function. Altered synthetic function indicates liver failure. Hepatomegaly is never normal and results from congestion, hepatitis, or a tumor. It may be part of a viral illness or represent malignancy. It is important to take a careful history and per­ form a thorough exam looking for signs of portal hypertension and splenomegaly. Portal hypertension results from obstruction of portal blood flow and is most often caused by cirrhosis. It most commonly presents as esophageal variceal bleeding. Sple­ nomegaly may be caused by a liver process (cirrhosis, portal, or hepatic vein thrombosis) or a manifestation of the same sys­ temic process (viral infection, malignancy) . The history and physical exam will guide your workup. For example, hepato­ splenomegaly in a girl with classic infectious mononucleosis does not require additional evaluation. Incidental hepatomeg­ aly, as described in this patient, is a different story. The abdominal ultrasound shows a mass in the liver. Which of the following etiologies is less likely in this setting? Question 45-2

Question 44-2

After the results of the colonoscopy are reviewed (including histopathology of the biopsy samples obtained during the procedure) , you recommend the following:

A. B. C. D.

Watchful monitoring with serial hemoglobin measurements. Surgical consultation for colectomy. Chemotherapy. Radiation therapy.

Discussion 44-2

The correct answer is "B:' The treatment for FAP is colectomy in an attempt to avoid colon cancer.

A) Hepatoblastoma. B) Fibroadenoma. C) Hemangioma. Discussion 45-2

The correct answer is "B:' Fibroadenoma occurs in females in their 20s and 30s, especially in those patients taking oral contraceptive pills. Liver hemangiomas are associated with multiple skin hem­ angiomas and may be asymptomatic. Infants with a large liver hemangioma may develop high output heart failure. Hepatoblas­ toma is malignant tumor of the liver primarily affecting children younger than 2 years of age. Children with Beckwith-Wiedemann

352

MCG RAW-H I LL E D U CATION S P E C I A LTY BOA R D REVI EW: P E D I ATRICS

syndrome, trisomy 21, and familial adenomatous polyposis are at increased risk for developing hepatoblastoma. The liver mass takes up more than 75% of the liver itself. A biopsy of the mass confirms a neoplastic process. Question 45-3

Which is the best treatment option in this setting?

A) B) C) D)

Liver transplant and subsequent chemotherapy. Chemotherapy. Radiation. Watchful monitoring.

Discussion 45-3

The correct answer is "A:' The child has a hepatoblastoma. He is the right age, and most often children present with abdomi­ nal distention, as he did. Complete resection is the mainstay of treatment. In this setting, a liver transplant will be necessary due to the large size of the mass.

A 2-year-old girl presents to your clinic with diarrhea of 6 weeks' duration, a distended abdomen, and weight loss. When fasting, the diarrhea resolves. You are concerned about malabsorption in this setting.

A

B

F I G U R E 1 6-5. Celiac Disease (Anterior and Profi le). This young boy has celiac

Question 46-1

Which is NOT an initial diagnostic stool test to obtain?

A) B) C) D) E)

Stool cultures. Microscopy for ova and parasites. Fecal occult blood test. Rotavirus stool antigen. Stool alpha- 1 -antitrypsin.

Discussion 46-1

The correct answer is "D:' The pooping stops with fasting, so think malabsorption. Malabsorption results from either ( 1 ) failure to absorb nutrients (ie, problem with the intestinal mucosa) or (2) failure to digest the nutrients for absorption (ie, problem with pancreatic function). Malabsorption can be gen­ eral (multiple nutrients) or selective (isolated fat, protein, car­ bohydrates, vitamins). Symptoms include diarrhea, abdominal distention, and poor weight gain. Signs include muscle wasting (toothpick extremities, flat buttocks) and loose skin for loss of underlying fat. In older children, stunting may occur. Edema suggests protein-losing enteropathy (PLE) . Bloating suggests carbohydrate malabsorption. Greasy, fatty stools suggest fat malabsorption. (See Figure 1 6-5.) Bacterial etiologies are less likely in the setting of chronic diarrhea without other features (bloody diarrhea, cramping) . Infectious diarrhea is typically secretory, with the exception of chronic diarrhea from Giardia. Chronic giardiasis can cause severe malabsorptive diarrhea with significant weight loss, so parasite testing (O&P) is a must. Stool cultures and parasite testing should make the first round of testing. Tests of nutrient malabsorption are also in the first

disease with sig nificant protein energy m a l n utrition. He shows the classic a p pearance of abdom i n a l d i stention, hanging ski nfol ds, loss of su bcuta neous fat, and m uscle wasting (th i n a rms and legs, flat buttocks). (Used with permission from Eyad M. H a n na, M D, U n iversity of Iowa.)

round of testing and include fecal alpha- 1 -antitrypsin to test for protein malabsorption, stool-reducing substances for car­ bohydrate malabsorption, fecal fat for fat malabsorption, and fecal elastase to assess pancreatic exocrine function. A low level of fecal elastase means the pancreas is out of gas. Malabsorp­ tion in cystic fibrosis is due to pancreatic insufficiency, so fecal elastase would be low in this patient population. •

� I

Helpful Tip

Any time you a re asked a bout a diag nostic worku p,

1 1r always sta rt with a

thoro u g h history a n d physica l exa m . When done well, these w i l l point you towa rd the rig ht d iag nosis which g u ides the diag nostic worku p.

Question 46-2

The appropriate blood tests to order for this patient should include the following:

A) B) C) D) E)

CBC. Peripheral smear. Tissue transglutaminase antibody and total IgA. Peripheral blood smear. All of the above.

C H A PT E R

Discussion 46-2

The correct answer is "E:' A CBC with differential will detect anemia, neutropenia (Shwachman-Diamond syndrome), and lymphopenia (lymphatic issues, severe combined immunodefi­ ciency). Option "C" screens for celiac disease (gluten-sensitive enteropathy) . To recap, the initial diagnostic testing for malab­ sorption should include: •

























Bacterial stool cultures Microscopy for ova and parasites Fecal occult blood test Stool leukocytes: marker of inflammatory process Stool alpha - ! -antitrypsin: protein malabsorption Fecal fat: fat malabsorption (duh!) Stool-reducing substances and pH: carbohydrate malabsorption Stool elastase: pancreatic exocrine function CBC with differential Peripheral blood smear Tissue transglutarninase (TTG) -IgA antibody level and total IgA: screen for celiac disease Comprehensive metabolic panel Vitamin D (25-hydroxyvitamin D)

This list is not set in stone but rather a framework to work from. Once you get the results, you can determine your next steps. The child's TTG-IgA level is greater than 100 units, which is strongly positive. Question 46-3

The next step in management should include:

A) B) C) D) E)

Colonoscopy. Bone marrow biopsy. Abdominal ultrasound. Upper endoscopy with duodenal biopsy. Trial of a gluten-free diet.

Discussion 46-3

The correct answer is "D:' This child's TTG-IgA antibody level needs to be confirmed by biopsy. Celiac disease is an immune-mediated sensitivity to gluten. The mucosa of the small bowel villi is damaged, leading to malabsorption. TTP­ IgA antibody level is one of th e best screening tools for celiac disease. However, its use is complicated by the fact that it is an IgA antibody, and there is a significant percentage of the pop­ ulation for whom IgA deficiency is a reality. Therefore, assess­ ment of the total IgA level is often obtained in conjunction with the TTG-IgA level. Celiac disease is unlikely if TTG-IgA antibody testing is negative with a normal total IgA level but it is not ruled out completely as testing can be falsely nega­ tive. The TTG-IgG antibody test is also utilized, especially in the setting of a low total IgA level; however, it is less specific and sensitive overall. The gold standard for the diagnosis of celiac disease continues to be histopathologic assessment of the duodenal mucosa.



� I

16



P E D I ATRIC GASTRO ENTEROLOGY

353

Helpful Tip

lgA a nti-TIG a nti body is the preferred screening test

1 1 r for celiac d i sease in c h i l d ren over the age of 2 yea rs.

Endoscopy is performed, and multiple biopsies of the duode­ num are obtained for the pathologist to review. Question 46-4

The most likely histopathologic features found by the pathol­ ogist include:

A) B) C) D) E)

Goblet cells. Mastocytes and atypical white blood cells. Blunting of the villi with intraepithelial lymphocytes. Melanocytes. Granulomas.

Discussion 46-4

The correct answer is "C:' This is the typical histopathologic picture of celiac disease. Duodenal villous atrophy can be seen with small bowel bacterial overgrowth, Crohn disease, eosinophilic enteritis, giardiasis, and malnutrition. This is why diagnosis requires more than one element-signs and symptoms, serology, and biopsy. Question 46-5

When the child returns for follow-up and review of the test results to date, you recommend the following treatments:

A) B) C) D) E)

Chemotherapy. Gluten-free diet. Soy-free diet. Stopping all laxatives. Metronidazole.

Discussion 46-5

The correct answer is "B:' Dietary elimination of gluten (ie, a pro­ tein found in wheat, barley, and rye) is the treatment for celiac disease. The treatment is lifelong and needs to be followed strictly in an attempt to avoid complications of celiac disease, which are most often caused by chronic malabsorption of vital nutrients. The mother forgot to mention that she has celiac disease. She remarks, "Oops, was that an important detail? " You politely ask how she did not remember to bring this up earlier. Question 46-6

Which of the following conditions places patients at an increased risk for celiac disease?

A) B) C) D) E)

Trisomy 2 1 . Type 1 diabetes mellitus. Autoimmune thyroiditis. IgA deficiency. All of the above.

Discussion 46-6

The correct answer is "E:' Children and adolescents with an affected first -degree relative, Williams syndrome, Turner

MCG RAW-H I LL E D U CATION S P E C I A LTY BOA R D REVI EW: P E D I ATRICS

354

syndrome, alopecia areata, type 1 diabetes mellitus, autoim­ mune hepatitis, autoimmune thyroiditis, and Addison disease are at increased risk for celiac disease. Do you see an autoim­ mune theme? •



Helpful Tip

Celiac l i kes friends a n d is freq uently fou n d i n patients other a utoi m m u ne d iseases such as type 1 dia betes mell itus or a utoi m m u ne thyroid itis.

1 1 1r with

� QUICKQUIZ What is NOT a symptom of celiac disease?

A) B) C) D) E)

Diarrhea. Weight gain. Vomiting. Abdominal distention. Mouth ulcers.

Discussion

The correct answer is "B:' Okay, that was too easy. Celiac dis­ ease can manifest with the typical features of malabsorption but it has a few tricks as well. Constipation, intussusception, rectal prolapse, mouth ulcers, discolored teeth or enamel, dermatitis herpetiformis (weird rash) , peripheral neuropathy, and short stature are also manifestations of celiac disease. •



1 1 1r

Helpful Tip

I ron deficiency a nemia u n responsive to i ron thera py is a m a n ifestation of celiac d i sease.

test may be used to diagnosis SIBO although they are not per­ fect. Jejunal aspirate cultures are more sensitive and specific. Lactulose is a sugar that is not absorbed. The excess bacteria in the small intestine should produce a "double peak'' phenomena on a breath test, showing an initial peak due to baseline fermen­ tation by the bacteria in the small intestine and a second peak when the lactulose reaches the bacteria who happily reside in the large intestine, releasing more methane as they feast on the unabsorbed sugar. The hydrogen or methane gas is absorbed, excreted in the lungs, and measured in the breath. This fermen­ tation will translate into increased gut motility with associated diarrhea and increased malodorous intestinal gas. The [ 14 C] ­ d-xylose test is another way to check for SIBO that uses xylose, a sugar metabolized by gram-negative anaerobic bacteria, and urine instead of breath. Question 47-2

Treatment of SIBO in this case would most often be accom­ plished with which of the following?

A) B) C) D)

Oral antibiotics. Laxatives. Change in diet. IV antibiotics.

Discussion 47-2

The correct answer is "A:' Flagyl, neomycin, and rifaximin are popular antibiotics to give orally for bacterial overgrowth. Question 47-3

Carbohydrate malabsorption is assessed by which of the fol­ lowing methods?

A) B) C) D)

d-Xylose test. Lactose breath hydrogen test. Fructose breath hydrogen test. All of the above.

Discussion 47-3

A 1 3-year-old boy who has a history of short bowel syndrome resulting from gastroschisis presents for follow-up with a 2-week history of increased stool frequency including passage of loose stools and abundant malodorous gas. Question 47-1

The best breath test to choose for assessment of the current problem is:

A) B) C) D)

Sucrose. Fructose. Lactulose. Urea.

Discussion 47-1

The correct answer is "C:' Small intestinal bacterial overgrowth (SIBO) causes inflammation, fermentation (makes a lot of gas), and polynutrient malabsorption and maldigestion. It may be associated with short gut. The lactulose breath test and d-xylose

The correct answer is "D:' All of these tests assess for carbohy­ drate malabsorption and thus intolerance. The history is often helpful in selecting which sugar to test. The subject fasts over­ night then is fed the assumed "problem" carbohydrate (lactose, sucrose, fructose, glucose). In malabsorption, the sugar is not absorbed and passes to the large intestine, where the bacteria feast upon it producing hydrogen gas. Hydrogen gas is absorbed into the bloodstream and then sent out of the body through the lungs. Increased hydrogen gas in a breath suggests carbohydrate malabsorption. If testing for SIBO, the d-xylose test or lactulose breath test is used, which you just read about in Question 43- 1 . •



Helpful Tip

If you can neither digest nor a bsorb carbohyd rates,

1 1 1 r you r colonic flora wi l l

prod uce gas when you eat carbohyd rate-conta i n i n g foods causing bloating a n d flatu lence. Neither is beneficia l to a romantic relatio n s h i p.

C H A PT E R

A 3-month-old infant girl who was born at full term has failed to gain weight appropriately despite formula fortifica­ tion of breast milk. She is taking 6 ounces of fortified breast milk every 3 hours and does not have any feeding difficul­ ties, tachypnea while eating, or perioral cyanosis. Her length and fronto-occipital circumference are also challenged but are following curves under the standard growth curve. She is admitted to the hospital for observation and additional evaluation.

16



P E D I ATRIC GASTRO ENTEROLOGY

355

syndrome, specifically. It is a genetic condition with characteris­ tics facies, bone dysplasia, neutropenia/cytopenias, and pancre­ atic exocrine insufficiency.

A 2-month-old infant who was evaluated for direct hyper­ bilirubinemia was found to have extrahepatic biliary atresia (EHBA) and underwent a Kasai procedure at 62 days of life. Question 49-1

Question 48-1

Tests done during her hospital stay should include:

A) B) C) D) E)

Colonoscopy. Bone marrow biopsy. Sweat test. Bone mineral density analysis. Echocardiogram.

Discussion 48-1

The correct answer is "C:' Even if the newborn screen was nor­ mal, a sweat test should be done if there is clinical suspicion for cystic fibrosis. The other tests are either not applicable or should not be done as "first-line" analysis. Cystic fibrosis may cause pancreatic exocrine insufficiency resulting in fat malab­ sorption. Pancreatic enzymes are needed to metabolize and absorb fat. Signs and symptoms of cystic fibrosis include failure to thrive, poor weight gain, and steatorrhea-big, greasy, fatty stools. Treatment is pancreatic enzyme replacement therapy. (Don't forget about your fat-soluble vitamins!)

� QUICKQUIZ Which is NOT a cause of fat malabsorption?

A) B) C) D)

Shwachman-Diamond syndrome. Lymphangiectasis. Lipase deficiency. Amylase deficiency.

Discussion

The correct answer is "D:' Fat malabsorption should scream pancreatic exocrine insufficiency or biliary tract disease. Pan­ creatic lipase digests fats with the help of bile salts, followed by absorption through a normal intestine. In pancreatic insuffi­ ciency, fecal elastase is low and fecal fat elevated (quantitative measurement). If you want to get fancy, suck some duodenal juice and analyze it. Treatment is replacement enzymes and fat -soluble vitamin supplementation. It is important to identify the underlying reason for the malabsorption. Additional condi­ tions on the list of fat malabsorption culprits include chronic pancreatitis, protein-energy malnutrition, bile acid deficiency, terminal ileum disease, Johanson-Blizzard syndrome, Pearson syndrome, cystic fibrosis, and lymphangiectasis. The American Board of Pediatrics wants you to know Shwachman-Diamond

Considering that a substantial percentage of patients under­ going Kasai for EHBA will experience progressive synthetic liver function and eventually need liver transplantation, which is the most appropriate formula to continue during infancy?

A) B) C) D)

Enfamil!Similac. Nestle Good Start. Alimentum. Pregestimil.

Discussion 49-1

The correct answer is "D:' Pregestimil contains medium-chain triglycerides (MCTs). These fats are water soluble and absorbed directly through the intestinal villi without need for bile acids or pancreatic lipase. The consumption of MCTs helps to decrease the amount of fat malabsorption in the setting of liver or pan­ creas compromise and thus promotes the nutritional status of the infant.

A 5-year-old girl presents with bloody diarrhea of 2 months' duration. Stool studies performed at her local pediatrician's office were negative for bacteria (including Clostridium difficile) and parasitic pathogens. Her most recent hemoglo­ bin, checked late last week, was 9 mg/ dL. She is passing six to eight bloody stools per day and awakening twice nightly to defecate. She complains of lower abdominal cramping prior to defecation but is otherwise comfortable. She denies nau­ sea, vomiting, or anorexia. Her weight is down 5 pounds. Her vital signs are remarkable for a heart rate of 142 bpm, normal blood pressure, and normal temperature. Her abdominal exam is benign. She has no perianal skin tags, fissures, or fistulas. Her rectal exam is remarkable for gross red blood, which is confirmed guaiac positive. Question 50-1

Your next step(s) in management should be to:

A) B) C) D) E)

Repeat the hemoglobin measurement today. Schedule a colonoscopy in 2 weeks. Perform an abdominal CT scan today. Admit to the pediatric floor for IV fluids. Both A and D.

356

MCG RAW-H I LL E D U CATION S P E C I A LTY BOA R D REVI EW: P E D I ATRICS

Discussion 50-1

The correct answer is "E:' The most important first step is to sta­ bilize her hemodynamic status. She may need a blood transfu­ sion and undoubtedly needs some IV fluids while waiting for the repeat hemoglobin measurement. Waiting 2 weeks for a colo­ noscopy is not appropriate as her clinical colitis is moderately active and may become fulminant without near future interven­ tion. Moderate to severe disease (fever, tachycardia, orthosta­ sis, abdominal tenderness) requires hospitalization. Admission will allow for stabilization, timely completion of a colonoscopy, and likely initiation of treatment. The needed diagnostic test is a colonoscopy with biopsies, not an abdominal CT scan (which would expose her to unnecessary radiation) . CT scan would be indicated if you were worried about an acute surgical process. The American Academy of Pediatrics' "Choosing Wisely" cam­ paign would not support your decision to irradiate her without strong conviction. Her clinical picture is consistent with colitis. The differential diagnosis includes bacterial infection, C. diffi cile infection, Henoch-Schonlein purpura, and systemic vasculitis. Stool cultures and colonoscopy are the most important diagnos­ tic tests for this child. Question 50-2

The patient undergoes a colonoscopy, and the most likely finding is:

A) Pancolitis with histologic evidence of cryptitis, crypt abscesses, and signs of chronic inflammation. B) C. diffi cile pseud omembranous colitis. C) A clean colon; the bleeding must be more proximal. D) Vascular malformations throughout the colon. Discussion 50-2

The correct answer is "A:' The clinical picture best fits a new presentation of ulcerative colitis, which most often presents as pancolitis (involvement of the entire colon) in young children. Symptoms of ulcerative colitis include rectal bleeding, diarrhea, fever, and defecation-associated abdominal pain. Ulcerative colitis and Crohn disease are idiopathic inflammatory diseases of the GI tract. They are collectively referred to as inflamma­ tory bowel disease (IBD). The cause is unknown. They are life­ long disorders and are categorized based on symptoms. Crohn disease can involve any part of the GI tract from the mouth to the anus. Ulcerative colitis affects only the colorectal area. Peri­ anal disease is common with Crohn disease but not ulcerative colitis. GI tract inflammation seen with Crohn disease is not continuous. Areas of inflammation are separated by normal areas of mucosa, so-called skip lesions. On biopsy evaluation, inflammation is transmural (affecting the entire intestinal wall) with noncaseating granulomas. In ulcerative colitis, inflamma­ tion is continuous, starting at the rectum. On biopsy evaluation, inflammation is limited to the mucosa. Complications of Crohn disease include fistulas, perforations, abscesses, and strictures. Question 50-3

C) Methylprednisolone. D) Tacrolimus. Discussion 50-3

The correct answer is "C:' IV corticosteroids are helpful "rescue" therapies in the setting of IBD. Corticosteroids should be con­ sidered temporary treatments, and evaluation should be ongo­ ing to determine the most appropriate maintenance therapy for this problem. Patients who need corticosteroid rescue often need immunomodulator medications to maintain control of their disease.

A 1 3-year-old boy presents to his pediatrician for evalua­ tion of anorexia and weight loss. He denies abdominal pain, nausea, and vomiting but does acknowledge early satiety. His mother also mentioned that he is more tired than usual, often napping daily in addition to sleeping for 9 hours each night. The family history is negative for chronic diseases. His vital signs are normal. His weight is at the fifth percentile for age (downward trending from the 25th percentile at his last well­ check) . His abdomen is soft, with tenderness and a palpable inflammatory mass in the right lower quadrant. Question 51 -1

The most likely etiology to explain this constellation of symp­ toms and exam findings is:

A) B) C) D)

Ruptured appendicitis. Eating disorder. Crohn disease involving the terminal ileum. Parasitic infection (eg, giardiasis) .

Discussion 51 -1

The correct answer is "C:' Crohn disease can cause inflamma­ tion anywhere in the GI tract. As such, manifestations depend on the area involved. Crohn disease of the small intestine is often insidious in presentation, leading to weight loss and fatigue without many overt GI symptoms. However, severe abdominal pain with nausea and vomiting occurs in the setting of a com­ plication of Crohn disease; namely, small bowel obstruction. Growth ( eg, short stature) and development ( eg, going through puberty normally) can all be severely compromised by Crohn disease. (See Table 16- 14.) TABLE 1 6- 1 4 S I G N S AND SYMPTO M S OF CRO H N D I S EA S E

Gastroi ntest i n a l m a n ifestations Dia rrhea Abdo m i n a l pa i n Wei g h t loss

The initial treatment that you recommend based on the colo­ noscopy findings includes:

Vo m i t i n g

A) Oral vancomycin. B) IV metronidazole.

Mouth u l cers

Peri a n a l d i sease (s k i n tags, fi s s u res, fi st u l a s, a bscesses)

C H A PT E R

Question 51 -2

You order blood tests to be drawn and find which abnormalities?

A) Positive blood culture for Salmonella. B) Hemoglobin 10.5 mg/dL with mean corpuscular volume (MCV) 69 fL, albumin 3 g/dL, erythrocyte sedimentation rate (ESR) 26 mm/h, and C-reactive protein (CRP) 40 mg/dL. C) Hemoglobin 16 mg/dL with MCV 105 fL, albumin 5 g/dL, and normal ESR and CRP. D) Hemoglobin 1 1 mg/dL with MCV 90 fL, albumin 3 . 7 g/dL, normal ESR, and CRP 12 mg/dL. Discussion 51 -2

The correct answer is "B." Patients with new-onset Crohn disease most often have microcytic anemia due to iron defi­ ciency. Because Crohn disease is a type of IBD, inflammatory indices are often elevated in the setting of active disease. Due to active intestinal inflammation, there is often a compo­ nent of malabsorption which is confirmed by hypoalbumin­ emia (protein malabsorption due to damaged mucosa with impaired absorption) .

16



P E D I ATRIC GASTRO ENTEROLOGY

357

TABLE 1 6- 1 5 EXTRA I NTEST I N A L M A N I F ESTAT I O N S O F I N F LA M M ATO RY B O W E L D I S EA S E

Ora l u l cers or stomatitis Fever Arth ritis Short statu re Del ayed pu be rty I ron d eficiency a n e m i a Arth ritis or a rt h ra l g i a s Uveitis E rythema nodos u m Va sc u l itis Th ro m bosis Pri m a ry scleros i s chola n g itis H e patitis Osteoporosis N e p h ro l i t h i a s i s

Question 51 -3

What test(s) should be ordered next to help you make your suspected diagnosis?

A) Capsule endoscopy. B) Upper and lower endoscopy with biopsies to confirm histologic evidence of chronic intestinal inflammation. C) Abdominal/pelvic magnetic resonance enterography (MRE). D) Hydrogen breath test. E) Both B and C.

Pa ncreatitis

Discussion 51 -4

The correct answer is "A." Option "D" is pathognomonic for celiac disease- remember t h e earlier discussion? Vil­ lous blunting can be seen in the setting of Crohn disease; however, one does not typically see intraepithelial lympho­ cytosis, too.

Discussion 51 -3

The correct answer is "E:' The gold standard for diagnosing Crohn disease continues to be upper and lower endoscopy with biopsy sampling to confirm characteristic histologic fea­ tures. However, the area affected in small bowel Crohn disease may not be reached with a typical endoscope or colonos­ copy; in such cases, imaging of the remaining small intestine is needed. As noted earlier, in children CT scan is reserved for surgical signs with or without fever in an attempt to spare radiation exposure. A capsule or PillCam is a useful tool for examining the small intestinal mucosa; however, this should not be performed without recent imaging (eg, MRE, CT, UGI small bowel follow-through) to ensure safe passage of the cap­ sule, especially th rough areas of active intestinal inflammation (which narrows the intestinal lumen). The main risk of capsule endoscopy is capsule retention, which could necessitate emer­ gency surgery.

}% QUICKQUIZ Which is NOT an extraintestinal manifestation of iBD?

A) B) C) D)

Purpura. Ankylosing spondylitis. Thrombosis. Cholelithiasis.

Discussion

The correct answer is "A:' (See Table 16-15.) Purpura are seen with Henoch -Schi:inlein pupura.

Question 51 -4

What histopathology is pathognomonic for Crohn disease?

A) Mucosal granulomas with features of cryptitis, crypt abscess formation, and chronic inflammation. B) Low-grade dysplasia in the colon. C) Intestinal metaplasia. D) Villous blunting with intraepithelial lymphocytosis.

An 8-year-old girl presents with her mother, who reports that her daughter has had discharge on her underwear for the past week. Prior to this finding, the girl found it difficult to sit at her desk at school due to pain near her anal orifice. Once the discharge started, her pain resolved. Her family history is remarkable for ulcerative colitis (father) .

358

MCG RAW-H I LL E D U CATION S P E C I A LTY BOA R D REVI EW: P E D I ATRICS

Question 52-1

Question 52-4

On exam, you find the following abnormality:

You complete your testing and are ready to begin treatment. The therapy with the most efficacy in the scientific literature for this problem is:

A) Vaginal candidiasis. B) Folliculitis in the perineal region. C) Apparent fistula tract to the left of the anal orifice, which releases white, purulent material from the orifice when the area is manipulated. D) Cellulitis of the perianal skin.

A) B) C) D)

Corticosteroids (eg, prednisone). Biologic or anti-TNF agents ( eg, infliximab). lmmunomodulators (eg, azathioprine) . Antibiotics (eg, metronidazole) .

Discussion 52-1

Discussion 52-4

The correct answer is "C:' A perianal fistula involving her vagina is the most likely etiology in this setting. This is often the presentation of perianal Crohn disease. Perianal findings are common with Crohn disease and may include skin tags, anal fissures, abscesses, or fistulas.

The correct answer is "B:' Biologic medications have been shown to be efficacious in treating perianal Crohn disease.

Next, blood tests and upper endoscopy and colonoscopy are arranged.

A severely malnourished 16-year-old girl presents to her pediatrician for evaluation. Her mother has not been able to get her to eat more than 500 kcal!day for several weeks. Despite losing 15 kg and having a BMI of 16, the girl says she does not want to eat because she is already fat. Her heart rate is 56 bpm. You admit her to the hospital. The adolescent medicine service is consulted.

Question 52-2

You should also include which of the following when making the colonoscopy reservation?

A) An exam (of the perianal skin) under anesthesia with a pediatric surgeon. B) A gynecologic exam with a pediatric gynecologist. C) A bone marrow biopsy with a pediatric oncologist. D) A perineal ultrasound with a pediatric radiologist. Discussion 52-2

The correct answer is "A:' Pediatric gastroenterologists and surgeons should work in conjunction for these problems. If a fistulous tract is found, a pediatric surgeon may prefer place­ ment of a Seton drain to allow correct healing of the fistula tract over time. A perineal ultrasound will not provide enough information about a possible fluid collection in the pelvis. A perianal fistula is found, and you need to decide the best treatment for this problem. Question 52-3

You should complete the following tests before starting treat­ ment in this setting:

A) B) C) D)

Bone marrow biopsy. PPD (a test for tuberculosis). Pelvic imaging in the form of MRI or CT scan. Both B and C.

Question 53-1

When starting nasogastric feedings, what is the most signifi­ cant concern with respect to the patient's electrolytes?

A) B) C) D)

Hyperkalemia. Thiamine deficiency. Hypophosphatemia. Vitamin D deficiency.

Discussion 53-1

The correct answer is "C:' The concern is for refeeding syn­ drome. The pathogenesis includes depleted phosphate stores during starvation. When the patient is fed carbohydrates, phos­ phate moves into the cells under the influence of insulin and the serum phosphate level is depleted, causing tissue hypoxia and associated complications. The risk of potassium alteration in patients with anorexia nervosa and other malnourished states is typically low potassium, which can lead to weakness. Question 53-2

The risk of electrolyte abnormalities persists for how long in this setting?

Discussion 52-3

A) B) C) D)

2 days. 2 weeks. 6 weeks. 8 weeks.

The correct answer is "D:' You need to ensure the absence of a pelvic fluid collection (ie, abscess) before starting potent anti­ inflammatory treatments aimed at perianal fistulae. Often, bio­ logic therapies ( eg, infliximab) are the most helpful treatments for perianal fistulae. Testing for tuberculosis exposure is neces­ sary before starting these treatments.

The correct answer is "B:' The concern regarding electrolyte abnormalities in the setting of refeeding syndrome persists for 2 weeks after feedings are started; therefore, these patients must continue to be monitored closely during this period. The key

Discussion 53-2

C H A PT E R

is starting slow, monitoring electrolytes frequently, and supple­ menting (phosphorus) as needed. Some patients may require telemetry monitoring as hypophosphatemia may result in bad cardiac outcomes-arrhythmias or heart failure. The heart can atrophy, too. B I B L I O G RA P H Y

Baumgart DC, Sandborn WJ. Crohn's disease. Lancet. 2012;380(9853): 1 590- 1 605. doi: 1 0 . 1 0 1 6/ so 140-6736( 12 )60026-9. Bishop WP. Pediatric Practice Gastroenterology. McGraw-Hill Education, Inc., New York, NY: 2010. Catassi C, Fasano A. Coeliac disease. The debate on coeliac disease screening-are we there yet? Nat Rev Castro­ enteral Hepatol. 2014; 1 1 (8) :457-458. doi: 10. 1038/ nrgastro.20 14. 1 19. Centers for Disease Control and Prevention. Provider informa­ tion: Rotavirus VIS. http://www. cdc.gov/vaccines/hcp/ vis/vis-statements/rotavirus-hcp-info.html. Accessed March 9, 20 15. Chiou E, Nurko S. Management of functional abdominal pain and irritable bowel syndrome in children and adolescents. Exp Rev Gastroenterol Hepatol. 201 0;4(3):293-304. doi: doi: 10. 1 586/egh. 1 0.28. Chitkara DK, van Tilburg M, Whitehead WE, Talley N. Teach­ ing diaphragmatic breathing for rumination syndrome. Am J Gastroenterol. 2006; 1 0 1 ( 1 1 ) :2449-2452. Cohen GM, Albertini LW Colic. Pediatr Rev. 2012;33(7): 332-333. doi: 1 0. 1 542/pir.33-7-332. Committee on Infectious Diseases; American Academy of Pediatrics; Kimberlin DW, Brady MT, Jackson MA, Long SS. Red Book. 201 5 Report of the Committee of Infectious Diseases. 30th ed. http:/ /www.redbook.solutions.aap.org/ redbook.aspx. Accessed June 14, 2015. Diibritz J, Muhlbauer M, Domagk D, et al. Significance of hydrogen breath tests in children with suspected carbo­ hydrate malabsorption. BMC Pediatr. 2014;14:59. doi: 1 0. 1 1 86/ 147 1 -243 1 - 14-59. Daneman A. Malrotation: The balance of evidence. Pediatr Radio/. 2009;39(suppl 2):S164-S 1 66. Egan LJ, Sandborn W Taking a closer look at IBD. Gut. 20 14;63(2) :el. doi: 10. 1 136/gutjnl-201 3-305424. Gorsche JR, Vick L, Boulanger SC, Islam S. Midgut abnormali­ ties. Surg Clin North Am. 2006;86(2) :286-299. Harb, R, Thomas, DW Conjugated hyperbilirubinemia: Screening and treatment in older infants and children. Pediatr Rev. 2007;28(3) :83-9 1 . Koletzko S , Jones NL, Goodman KJ, et al. Evidence-based guidelines from ESPGHAN and NASPGHAN for Helicobacter pylori infection in children. J Pediatr Gas­ troenterol Nutr. 20 1 1 ;53 (2) :230-243. doi: 10. 1 097/ MPG.Ob013e3 1 82227e90.

16



P E D I ATRIC GASTRO ENTEROLOGY

359

Munck A, Gargouri L, Alberti C, et al. Evaluation of guide­ lines for management of familial adenomatous pol­ yposis in a multicenter pediatric cohort. J Pediatr Gastroenterol Nutr. 20 1 1 ;53(3) :296-302. doi: 1 0 . 1 097/ MPG.Ob0 1 3e3 1 82 1 98f4d. Murch SH, Winyard PJ, Koletzko S, et al. Congenital entero­ cyte heparan sulphate deficiency with massive albumin loss, secretory diarrhoea, and malnutrition. Lancet. 1 996;347(90 1 1 ) : 1299- 1 3 0 1 . Nachamkin I, Alios BM, H o T. Campylobacter species and Guillain-Barre syndrome. Clin Microbial Rev. 1 998; 1 1 (3):555. Ohhama Y, Shinkai M, Fujita S, Nishi T, Yamamoto H. Early prediction of long-term survival and the timing of liver transplantation after the Kasai operation. J Pediatr Surg. 2000;35(7) : 1 03 1 - 1034. Ordas I, Eckmann L, Talamini M, Baumgart DC, Sandborn WJ. Ulcerative colitis. Lancet. 2012;380(9853 ) : 1 606- 1 6 1 9. doi: 10. 1 0 1 6/S0 140-6736( 12)60 1 50-0. Park T, Wassef W Nonvariceal upper gastrointestinal bleed­ ing. Curr Opin Gastroenterol. 2014;30(6) :603-608. doi: 10. 1097/MOG.0000000000000 123. Rome Foundation. Rome III disorders and criteria. http:/ I www.romecriteria.org/criteria. Accessed June 13, 20 15. Rubio-Tapia A, Hill ID, Kelly CP, Calderwood AH, Murray JA. ACG clinical guidelines: Diagnosis and management of celiac disease. Am J Gastroenterol. 20 13; 108(5):656-676; quiz 677. doi: 1 0. 1 038/ajg.20 1 3.79. Sandborn WJ. Crohn's disease evaluation and treatment: Clini­ cal decision tool. Gastroenterology. 2014;147(3):702-705. doi: 10. 1 053/j.gastro.2014.07.022. Sandborn WJ, Hanauer S, Van Assche G, Panes J, Wilson S, Petersson J, Panaccione R: Treating beyond symptoms with a view to improving patient outcomes in inflamma­ tory bowel diseases. J Crohns Colitis. 20 14 Sep 1 ;8(9): 927-35. doi: 10. 1 0 1 6/j.crohns.20 14.02.02 1 . Epub 2014 Apr 6. Sondheimer JM and Hurtado CW, (Eds) . The NASPGHAN Fellows Concise Review of Pediatric Gastroenterology, Hepatology and Nutrition. 20 1 1 Srinath, Arvind I., & Lowe, Mark E. (20 13). Pediatric Pancre­ atitis. Pediatrics in Review, 34(2), 79-90. doi: 1 0. 1 542/ pir.34-2-79 Suchy, Sokol and Balistreri. Liver Disease in Children Third Edition. 2007 Thakkar K, Fishman DS, Gilger MA. Colorectal polyps in childhood. Curr Opin Pediatr. 2012 Oct;24(5):632-7. doi: 10. 1 097/MOP.Ob0 1 3e3283574 1 9f. Theander G, Triigardh B. Lymphoid hyperplasia of the colon in childhood. Acta Radio/ Diagn (Stockh) . 1 976 Sep; 1 7(5A):63 1 -40. Walker, A. et al. Pediatric Gastrointestinal Disease. Fourth Edition, 2004.

This page intentionally left blank

Genetics a nd Dysmorphol ogy

17

G reg R i ce

Question 2-1

What is the inheritance pattern shown in this pedigree?



Affected

A) B) C) D) E) F)

Autosomal dominant disorder with complete penetrance. Autosomal dominant disorder with incomplete penetrance. Autosomal recessive disorder. X-linked recessive disorder. X-linked dominant disorder. Mitochondrial disorders .

Discussion 2-1 Question 1 -1

What is the inheritance pattern shown in this pedigree?

A) B) C) D) E) F)

Autosomal dominant disorder with complete penetrance. Autosomal dominant disorder with incomplete penetrance. Autosomal recessive disorder. X-linked recessive disorder. X-linked dominant disorder. Mitochondrial disorders.

Discussion 1 -1

The correct answer is ''A:' With autosomal dominant disor­ ders, heterozygotes are affected. Males and females are equally affected. With complete penetrance, all individuals with one dominant mutant allele will be affected. An affected individual will have an affected parent, and 50% of offspring of an affected parent will be affected. Male-to-male transmission distinguishes this from X-linked dominant disorders.

The correct answer is "B:' In autosomal dominant disorders with incomplete penetrance, not all heterozygotes are affected. An individual can have the dominant mutant allele but not express it. Yet, he or she can still pass it to offspring who are then affected. Variable expression of autosomal dominant disorders is different. All individuals with the dominant allele are affected, but symptoms vary from person to person. �

Question 3-1

What is the inheritance pattern shown in this pedigree?

A) B) C) D) E) F)

Autosomal dominant disorder with complete penetrance. Autosomal dominant disorder with incomplete penetrance. Autosomal recessive disorder. X-linked recessive disorder. X-linked dominant disorder. Mitochondrial disorders. 361

362

MCG RAW-H I LL E D U CATION S P E C I A LTY BOA R D REVI EW: P E D I ATRICS

Discussion 3-1

Discussion 5-1

The correct answer is "C:' With autosomal recessive disorders, homozygotes are affected. Heterozygotes are carriers for the trait. Males and females are equal affected. All individuals with two recessive mutant alleles will be affected. An affected individ­ ual will not have an affected parent who is a carrier. Twenty-five percent of offspring of parents who are both carriers will be affected whereas 50% of offspring will be carriers.

The correct answer is "E:' With X-linked dominant inheritance, males and females may be affected. Male-to-female transmis­ sion (ie, father to daughter) may occur. Male-to-male trans­ mission (ie, father to son) of X-linked genes does not occur. One hundred percent of daughters born to affected fathers will be affected. All sons born to affected fathers are normal. Males are typically more affected than female, and the trait may be lethal in males.

Question 4-1

What is the inheritance pattern shown in this pedigree?

A) B) C) D) E) F)

Autosomal dominant disorder with complete penetrance. Autosomal dominant disorder with incomplete penetrance. Autosomal recessive disorder. X-linked recessive disorder. X-linked dominant disorder. Mitochondrial disorders.

Discussion 4-1

The correct answer is "D:' With X-linked recessive inheritance, only males are affected and females are carriers. Transmission is female to male (ie, mother to son), and 50% of sons born to mothers who are carriers will be affected. X-linked genes are never passed male to male (ie, father to son) .

Question 6-1

What is the inheritance pattern shown in this pedigree?

A) B) C) D) E) F)

Autosomal dominant disorder with complete penetrance. Autosomal dominant disorder with incomplete penetrance. Autosomal recessive disorder. X-linked recessive disorder. X-linked dominant disorder. Mitochondrial disorders.

Discussion 6-1

The correct answer is "E' Mitochondrial disorders are inher­ ited from the mother as nearly all mitochondria come from the mother. One hundred percent of offspring of an affected mother are affected. Affected fathers produce no affected offspring. •

� I

Question 5-1

What is the inheritance pattern shown in this pedigree?

A) B) C) D) E) F)

Autosomal dominant disorder with complete penetrance. Autosomal dominant disorder with incomplete penetrance. Autosomal recessive disorder. X-linked recessive disorder. X-linked dominant disorder. Mitochondrial disorders.

Helpful Tip

M u l tifactorial d i sorders (cleft lip or palate, spina bifida)

1 1r a n d traits (heig ht, i ntel ligence) a re caused by the action of mu ltiple genes or gene-environmental effects.

You are covering pediatrics at a large community hospital and receive a call on a Saturday night from the labor and delivery nurse who is concerned that a recently born term infant might have trisomy 2 1 . The nurse reports that newborn is stable, feeding well, and has normal pulse oximetry. He was born fol­ lowing an uncomplicated pregnancy to a healthy 28-year-old woman. There is no family history of pregnancy loss or aneu­ ploidy. In the morning your evaluation of the newborn reveals a flatted midface, upslanting palpebral fissures, small low­ set ears, nuchal redundancy, and borderline microcephaly.

C H A PT E R 1 7



G E N ETICS A N D DYS M O R P H O LOGY

363

it is therefore important for them to be aware of this informa­ tion for future family planning. Additionally, a small percentage of children with trisomy 2 1 are mosaic, meaning they have both a normal cell line and a trisomy 21 cell line. The developmental outcomes in these children are more variable and in some cases can be better. Infants with trisomy 21 are at high risk of having congenital heart disease. Approximately 40% of affected infants have an abnormal echocardiogram, endocardial cushion defects (also known as atrioventricular [ AV] canal defects) are classic, but isolated atrial and ventricular septal defects also occur frequently. Individuals with trisomy 2 1 are at increased risk for hypothy­ roidism and hearing loss. Some infants with trisomy 2 1 have a leukemoid reaction in which the white blood cell count becomes transiently elevated with the presences ofblasts. These infants are at increased risk of developing leukemia later in childhood. Question 7-2 F I G U R E 1 7- 1 . This 8-month-old has trisomy 21 with the typical facies, i n c l u d i n g upslanting pa l pebral fissu res, epicanthal folds, l a rge tong ue, flat face, low-set ears a n d brachycephaly. (Reproduced with permission from Va l l e D, Beaudet AL, Vogel stein B et a l : The Online Metabolic and Molecular Bases of Inherited Disease, Sed. McGraw- H i l l Education, I nc; 201 4. Fig. 63- 1 .)

(See Figure 1 7-1.) The cardiac exam is normal. The newborn's hands are small and there are single palmar creases. Based on the typical facial features you make a clinical diagnosis of trisomy 21 and discuss your findings with the family. The parents were not expecting this news and become upset. They wonder how this could have happened because the mother had a normal ultrasound and is only 28 years old. Question 7-1

Which of the following tests does NOT need to be performed for this newborn before discharge?

A) B) C) D) E) F)

Karyotype of the newborn. Echocardiogram. Thyroid-stimulating hormone {TSH) level. Newborn hearing screen. CBC. All of the above tests should be performed within the new­ born period in a child with suspected trisomy 2 1 .

Which of the following conditions is/are more common in children with trisomy 2 1 ?

A) B) C) D) E) F) G)

Refractive errors o f the eye and amblyopia. Leukemia. Cognitive disability. Duodenal atresia. Atlantoaxial (cervical spine) instability. Celiac disease. All of the above.

Discussion 7-2

The correct answer is "G:' All of these conditions are more common in children with trisomy 2 1 . This list helps to guide trisomy 2 1 -specific anticipatory guidance for the pediatrician. Infants with trisomy 21 are more likely to have one or more of the following medical conditions: intestinal obstructions such as duodenal atresia or stenosis, imperforate anus, tracheo­ esophageal fistula, and Hirschsprung disease. (See Figure 1 7-2.)

Discussion 7-1

The correct answer is "E' The newborn has typical physical manifestations of trisomy 2 1 or Down syndrome. Other physical features include a sandal gap between the first and second toes, hypotonia, and large tongue. Down syndrome is usually due to an extra copy of chromosome 21 (trisomy 2 1 ) , which is usually the result of maternal nondisjunction. All infants with suspected trisomy 2 1 should have a karyotype performed, even if the phy­ sician is confident of the diagnosis. This is because 4% of cases of trisomy 21 are due to an unbalanced robertsonian transloca­ tion (2 chromosomes join) between chromosome 21 and another acrocentric (centromere near one end rather than the center) chromosome (eg, 14 or 15). The recurrence risk for parents who are carriers of the balanced form of the translocation is higher;

F I G U R E 1 7-2. T h e "double bubble" sign is suggestive o f d u o d e n a l atresia, which is associated with trisomy 2 1 . (Reproduced with permission from Brunica rd i FC, Andersen DK, B i l l i a r TR, et al, eds. Schwartz's Principles ofSurgery.

1 Oth ed. New York, NY: McGraw- H i l l Education; 20 1 5, Fig. 39- 1 3, p. 1 6 1 5.)

MCG RAW-H I LL E D U CATION S P E C I A LTY BOA R D REVI EW: P E D I ATRICS

364

It is very important that children with trisomy 21 be engaged in early intervention therapies as soon as possible. All chil­ dren with trisomy 2 1 have cognitive disability; however, the spectrum is broad and can range from borderline to severe cognitive disability. In recent years some high-functioning young adults with trisomy 2 1 have attended college. It is important to inform parents about the complete range of outcomes while at same time not giving false hope for "normal'' development. The parents of this newborn were surprised that they could have a child with trisomy 21 because the mother was young and the ultrasound normal. Question 7-3

Which of the following statements is true regarding the risk of having a child with trisomy 2 1 ?

A ) Prenatal ultrasound i s highly sensitive in detecting a fetus with trisomy 2 1 . B ) Most children with trisomy 2 1 are born to mothers of advanced maternal age. C) If a mother carriers a balanced robertsonian translocation between chromosomes 14 and 2 1 , her risk of having a child with trisomy 2 1 is 50%. D) Most children with trisomy 21 are born to mothers who are not of advanced maternal age. Discussion 7-3

The correct answer is "D:' Although advanced maternal age (AMA) increases the risk of having a child with trisomy 2 1 , most women who have children with this genetic disorder are not AMA. In fact, although the absolute risk of having a child with Down syndrome increases with age, more children with trisomy 2 1 are still born to younger women. The risk of hav­ ing a child with trisomy 2 1 at age 40 years is around 1 %; the risk at 28 is much lower. Parents are often surprised to find out at delivery that their newborn has trisomy 2 1 ; therefore, empathic and accurate counseling is critical at this time. Some parents find the diagnosis difficult to accept. In these cases the confirmatory karyotype can be helpful in the process of accep­ tance. When used alone, a 20-week prenatal ultrasound will detect trisomy 2 1 in only 50% of the cases. Prenatal screening tests such as the quad screen can increase that sensitivity, but this is still only a screening test. High-risk results would need to be confirmed by examination of the fetal karyotype follow­ ing amniocentesis or chorionic villus sampling (CVS). The risk of having a child with trisomy 2 1 for a mother (of any age) who carries a balanced 14:2 1 robertsonian translocation is around 14%, which is much higher than her age-related risk but still nowhere near 50%. •



1 1 1r

You are called to evaluate a term newborn in the delivery room. The mother received little prenatal care and measured small for dates. At delivery the newborn is found to be small for gestational age and dysmorphic, with microcephaly, a unilateral cleft lip and palate, clenched hands, and rocker bottom feet. The infant has a loud systolic murmur, weak cry, and recurrent episodes of apnea. Question 8-1

What is the most likely diagnosis in this newborn?

A) B) C) D)

Trisomy 18. Trisomy 1 3 . Cornelia d e Lange syndrome. CHARGE syndrome.

Discussion 8-1

The correct answer is "A:' This newborn has the classic features of trisomy 1 8 (Edwards syndrome). Affected children are small and thin and often have cleft lip and palate. Congenital heart and renal anomalies are common. The most recognizable features are clenched hands with overlapping fingers (see Figure 1 7-3) and rocker bottom feet. Prognosis is grim; most infants die in the first several months oflife without aggressive support. Apnea, seizures, severe feeding problems, and aspiration are common. Survivors have profound developmental delays. Infants with a mosaic form of trisomy 1 8 may be less severely affected. It is important to distinguish this condition from trisomy 13 (Patau syndrome). Infants with trisomy 1 3 often have midline cleft lip and palate, which is often associated with midline brain anoma­ lies (holoprosencephaly), and hypotelorism (eyes close together) .

Helpful Tip

The facies of trisomy 21 may be s u btle i n a newborn a n d may be more pronou nced with cryi ng or as the i nfa nt g rows.

F I G U R E 1 7-3. Disti n g u i s h i n g cha racteristics of trisomy 1 8 i ncl u de rocker

bottom feet and clenched hands with overl a p p i n g fi n g ers. (Reproduced with permission from Fuster V, Wa lsh RA, Ha rrington RA, eds. Hurst's The Heart. 1 3th ed. New York, NY: McGraw- H i l l Education; 201 1 , Fig. 1 4- 1 3.)

C H A PT E R 1 7

F I G U R E 1 7-4. An i ris coloboma is typical of CHARGE syndrome. (Reproduced with permission from Hay WW, Levi n MJ, Deterd ing RR, Abzug MJ, eds. Current Diagnosis and Treatmen t Pediatrics. 22nd ed. New York, NY: McGraw- H i l l Ed ucation; 201 4, F i g . 1 6-20, p . 469.)

Scalp defects (cutis aplasia), polydactyly, and congenital heart disease are also common features. The prognosis is similar to that for trisomy 18. Cornelia de Lange syndrome (CDLS) also results in multiple congenital anomalies and small size, but the findings in this case are not suggestive of CDLS. CDLS leads to typical dysmorphic characteristics that are present in the newborn, including synophrys (high-arched and connected eyebrows), long eyelashes, and a long philtrum. Most affected individuals are short statured, with microcephaly and small hands. Reduction defects of the hands and forearms can occur. Renal and cardiac anomalies also occur. CHARGE syndrome results in multiple congenital anomalies but in a predictable pattern: Coloboma (see Figure 17 -4), Heart defects, Atresia choanae, Retardation of growth, Genital anomalies, and Ear anomalies.

The correct answer is "E." The child in the case has 22q 1 1 deletion syndrome (also known as DiGeorge syndrome) . All the issues except the one listed as option "E" are commonly

the mnemonic CATCH-22 to remember the

r1 1r features of 22q 1 1 .2 deletion or DiGeorge syn d rome: Cardiac defects

C-

Abnormal facies hypoplasia C - Cl eft palate

A-

T - Thym us H

- Hypoca lcem ia/hypopa rathyroidism 22q 1 1 .2 deletion

22

The newborn in this case is at risk for all of the following health issues EXECPT:

Discussion 9-1

Helpful Tip

=-::'11.. Use

Question 9-1

Pierre Robin syndrome. Hypocalcemia secondary to hypoparathyroidism. Cognitive delays. Immunodeficiency. Direct hyperbilirubinemia secondary to a paucity of bile ducts.

365

associated with 22q 1 1 .2 deletion syndrome. Paucity of bile ducts is typically associated with Alagille syndrome. Deletion of 22q 1 1 .2 is the most common chromosomal microdeletion syndrome and is associated with conotruncal cardiac malfor­ mations such as tetralogy of Fallot. Hypoplasia of the thymus can result in immunodeficiency owing to abnormal lympho­ cyte maturation and function. The parathyroid glands may also be hypoplastic, which leads to hypoparathyroidism and subsequent hypocalcemia, tetany, and seizures. Around 40% of individuals have cognitive delays. Pierre Robin syndrome can occur secondary to the micrognathia (small j aw) . (See Figure 1 7-5.)

gram shows tetralogy of Fallot. The newborn is small for dates and also has mildly dysmorphic facial characteristics, including micrognathia, bulbous nasal tip, and round face.

A) B) C) D) E)

G E N ETICS A N D DYS M O R P H O LOGY

F I G U R E 1 7-5. This newborn has Pierre Robin syndrome with m icrog nathia a n d a hypoplastic m a n d i ble. (Reproduced with permission from Fuster V, Wa lsh RA, Harrington RA, eds. Hurst's The Heart. 1 3th ed. New York, NY: McGraw- H i l l Education; 201 1 , Fig. 1 4-7.)



A newborn is noted to have a cyanosis and respiratory dis­ tress. A chest X-ray shows absence of the thymic silhouette and several vertebral anomalies. A follow-up echocardio­



-

� QUICKQUIZ Which is a feature of Pierre Robin sequence?

A) B) C) D) E)

Cleft lip. Prognathia. Glossoptosis. Proptosis. Maxillary hypoplasia.

366

MCG RAW-H I LL E D U CATION S P E C I A LTY BOA R D REVI EW: P E D I ATRICS

Discussion

The correct answer is "C:' Pierre Robin syndrome results from mandibular hypoplasia. It is characterized by micro­ gnathia, glossoptosis (posteriorly displaced tongue) , and cleft soft palate. As the mandible is small, the tongue is displaced posteriorly causing upper airway obstruction. Treatment of the airway obstruction may include prone positioning, nasal pharyngeal airway (trumpet) , labioglossopexy (tongue-lip adhesion), mandibular distraction, and tracheostomy. Pierre Robin syndrome may occur in isolation or as part of another genetic syndrome.

You evaluate a 4-year-old girl for developmental delay and hyperactivity. She is very social and has good expressive lan­ guage skills despite her other cognitive deficits. She has short stature and mildly dysmorphic facial characteristics, includ­ ing a long but well-formed philtrum and puffy periorbital region. She had a history of hypercalcemia in infancy. On physical exam you notice a 2/6 systolic murmur. A follow-up echocardiogram reveals supravalvular aortic stenosis. Question 1 0-1

The child in the case has which of the following condition?

A) B) C) D) E)

Fetal alcohol syndrome. Prader-Willi syndrome. Angelman syndrome. Williams syndrome. Rett syndrome.

Discussion 1 0-1

The correct answer is "D:' Williams syndrome is due to a chromosomal microdeletion at 7ql l and is associated with cognitive delays and relatively preserved expressive language. Affected children are described as having elfin facies. Some individuals have an outgoing "cocktail party" personality and may have proclivity toward music. The deletion results in the loss of the elastin gene, which is responsible for the pathog­ nomonic cardiac lesion, supravalvular aortic stenosis. Renal artery stenosis, short stature, and infantile hypercalcemia also occur. Fetal alcohol syndrome (FAS), results in mildly dysmorphic facial characteristic, neurodevelopmental delays, and poor growth. The typical facial features of FAS include a smooth philtrum (area between the nose and upper lip) and small palpebral fissures (eye openings) . (See Figure 1 7-6.) Children often struggle in school, particularly with atten­ tion. Prader-Willi syndrome (PWS) results from imprinting abnormalities on chromosome 1 5, usually due to deletion of the paternal copy of 1 5q 1 1 - 1 3 or maternal uniparental disomy (UPD, which occurs when both chromosome copies come from one parent) for chromosome 1 5 . The maternal copy is imprinted and thus not expressed, so either a paternal deletion or maternal UPD will result in no functional copy of the criti­ cal genes. Children with PWS are hypotonic at birth and this

F I G U RE 1 7-6. C h i l d ren with fetal a l cohol synd rome have a smooth philtrum,

short pa l pebra l fissures, a n d a thin u pper l i p. Notice the long, smooth philtrum i n this child. (Reprod uced with permission from Fuste r V, Wa lsh RA, Ha rrington RA, eds. Hurst's The Heart. 1 3th ed. New York, NY: McGraw- H i l l Education; 201 1 , Fig. 1 4- 1 5.)

often leads to poor feeding. Many affected infants develop fail­ ure to thrive and require gastrostomy feedings. Boys may have a small phallus. At age 2 or 3 years the children begin to gain excessive weight due to absent satiety controls, which leads to overeating. Food-seeking behaviors are common, and families may have to lock cupboards. Without aggressive intervention most children with PWS will become obese. Growth hormone administration can be helpful in building lean muscle mass. Most children have mild to moderate cognitive disability and short stature. Angelman syndrome (AS) usually results from deletion of the maternal copy of chromosome 1 5ql l - 1 3 or paternal UPD. The paternal copy is imprinted and thus not expressed, leaving no functional copy of the critical region sec­ ondary to either of these mechanisms. Children with AS have global developmental delays, cognitive disability, wide-based gait (ataxia), happy demeanor, inappropriate laughter, micro­ cephaly, and seizures. Children with AS have been described as "happy puppets:' Most affected individuals develop little expressive language. Rett syndrome is an X-linked dominant condition that classically affects girls. Girls with Rett syn­ drome often develop normally for the first 4 to 6 months, then have regression of motor and language skills along with pro­ gressive microcephaly and seizures. Most classically affected girls are cognitive disabled and have little expressive language. Midline hand-wringing behaviors are common.

C H A PT E R 1 7



G E N ETICS A N D DYS M O R P H O LOGY

367

You see a 7 -year-old girl who is new to your practice for a well-child exam. You notice that she has borderline short stature and mildly unusual facial characteristics, includ­ ing downslanting eyes, low-set ears, and a broad neck. Her father has similar facial features and short stature. The father remarks that he struggled in school. The child's cognitive development has been normal. The physical exam also reveals a pectus carinatum of the chest and a cardiac murmur. A follow-up echogram shows mild pulmonic stenosis. Question 1 1 -1

What syndrome is the girl most likely affected with?

A) B) C) D)

Turner syndrome. Achondroplasia. Noonan syndrome. Russell-Silver syndrome.

Discussion 1 1 -1

The correct answer is "C:' The child and the father both have Noonan syndrome. In a girl with short stature it is important to distinguish Noonan syndrome (see Figure 1 7-7) from Turner syndrome (see Figure 1 7-8). Both conditions have the physical hallmarks of lymphedema, including low-set ears and nuchal webbing. Noonan syndrome is inherited in an autosomal domi­ nant fashion and therefore can affect both males and females.

F I G U R E 1 7-7. This c h i l d has Noonan synd rome. Notice the low-set ea rs

and broad neck. (Reproduced with permission from Fuster V, Wa lsh RA, Ha rrington RA, eds. Hurst's The Heart. 1 3th ed. New York, NY: McGraw- H i l l Education; 201 1 , F i g . 1 4- 1 7.)

F I G U R E 1 7-8. This girl has Tu rner synd rome. Aga i n , notice the low-set ears and broad neck. (Reproduced with permission from Fuster V, Wa lsh RA, H a rrington RA, eds. Hurst's The Heart. 1 3th ed. New York, NY: McGraw- H i l l Education; 201 1 , F i g . 1 4- 1 4.)

Turner syndrome results from monosomy X ( 45,X), is not inher­ ited, and only affects females. Girls with Turner syndrome have primary ovarian insufficiency (hypogonadism), resulting in lack of development of the secondary sexual characteristics and infer­ tility. Other features include short stature, shield-shaped chest, widely spaced nipples, and increased risk of coarctation of the aorta. Some girls with Turner syndrome have a mosaic karyo­ type. Girls who have a mosaic karyotype, with the presence of a Y chromosome (45,X +46,XY), are at risk for developing gonad­ oblastoma and should have their primitive gonads removed. Girls with Turner syndrome have normal cognitive develop­ ment. Individuals with Noonan syndrome often have short stat­ ure, dysmorphic facial features (as described above), pulmonary valve stenosis, and chest wall abnormalities (pectus excavatum or carinatum). Other features include cognitive delays in 40% of affected individuals, bleeding diathesis, and cardiomyopathy. Noonan syndrome is secondary to heterozygous mutations in several genes that use the RAS pathway, including PTPNl l . Muta­ tions in the genes that cause Noonan syndrome can be inherited from an affected parent in a dominant fashion (as in this case) or can occur de novo. Achondroplasia is the most common skeletal dysplasia. It is secondary to mutations in the gene FGFR3, which can be inherited from an affected parent in a dominant fashion or can occur de novo. This condition results in disproportion­ ate (short-limbed) dwarfism, with rhizomelic (upper segment) foreshortening of the limbs. Affected individuals have short stature and macrocephaly, with typical facial features including midface hypoplasia and frontal bossing. The fingers are short and splayed. Intelligence in normal. Russell-Silver syndrome (RSS) also results in short stature; however, these individuals have pro­ portionate short stature (no rhizomelia) with a normal head size.

368

MCG RAW-H I LL E D U CATION S P E C I A LTY BOA R D REVI EW: P E D I ATRICS

The body mass index is often very low. Infants with RSS are born very small and grow poorly. Their bodies are very tiny while their head size is normal, with a triangular-shaped face (broad fore­ head, narrow chin). Fifth finger clinodactyly is present. RSS also increases the risk of hypoglycemia. Some individuals with RSS have development delays.

u:

You are evaluating a 1 6-year-old boy for a sports physical. He has tall stature, flexible joints, and a pectus excavatum. On physical exam he is Tanner stage 5. His arms and legs seem disproportionately long. He has been getting straight J(s in school and is the captain of his basketball team. His mother had a similar body habitus and passed away suddenly at age 35 from an aortic dissection. Question 1 2-1

Which of the following syndromes is this adolescent boy most likely affected with?

A) B) C) D) E)

Klinefelter syndrome. Classical homocystinuria. Marfan syndrome. Ehlers-Danlos syndrome. Fragile X syndrome.

Discussion 1 2-1

The correct answer is "C:' Marfan syndrome is a connective tis­ sue disorder that results in a disproportionate body habitus with long arms and legs (arachnodactyly), leading to tall stature. (See Figure 1 7-9.) Pectus excavatum is common. Progressive aortic root enlargement can result in aortic dissection and death if not recognized and treated early. Ocular manifestations include ectopia lentis (dislocation of the lens). Cognitive development is

F I G U R E 1 7-9. Typica l m a n ifestations of Ma rfa n syndrome include ectopia lentis, ta l l statu re, a n d a rachnodactyly (shown). The thumb sign is positive (th u m b extends past the fifth fi nger when bent i nto the hand). (Reproduced with permission from LeBlond RF, Brown DD, Su neja M, Szot J F, eds. DeGowin's Diagnostic Examination. 1 Oth ed. New York, NY: McGraw- H i l l Education; 201 5, Plate 29.)

F I G U R E 1 7- 1 0. Klinefelter syndrome (47,XXY) causes primary hypogonadism.

Affected males a re ta l l, with small testes and gynecomastia. This male a l so has sparse body hair with a female pubic hair pattern. (Reproduced with permission from Gardner DG, Shoback D, eds. Greenspan's Basic & Clinical Endocrinology. 9th ed. New York, NY: McGraw- H i l l Education; 201 1 , Fig. 1 2-7.)

normal. Mutations in the fibrillin- 1 gene cause Marfan syndrome, and it can be inherited from an affected parent (as in this case) in a dominant fashion or can occur de novo. It is important to dis­ tinguish Marfan syndrome from classical homocystinuria, which is a metabolic disorder resulting in markedly elevated plasma and urine homocysteine. These individuals can have tall stature and ectopia lentis but are not at increased risk for aortic dila­ tion and often have development delays. Classical homocystin­ uria also increases the risk of coronary artery disease and stroke due to the atherosclerotic effects of homocysteine. Klinefelter syndrome results from the presence of an extra copy of the X chromosome in a male (47,XXY). Males with Klinefelter syn­ drome are relatively tall and have small testes. (See Figure 1 7 - 1 0.) Testosterone deficiency can result in delayed puberty. Infertility is common. Most boys have normal intelligence quotient (IQ) but specific learning disabilities can be seen. Ehlers-Danlos syndrome (EDS) is a connective tissue disorder that results in joint hypermobility; soft, stretchy "doughy" skin; and abnormal wound healing. (See Figures 1 7- 1 1 and 17-12.) Joint disloca­ tions and subluxations are common in EDS. There are several forms of the condition, identified as classic, hypermobile, vascu­ lar, arthrochalasia, dermatosparaxis, and kyphoscoliosis types. Individuals with classic-type EDS (formerly EDS 1 and EDS 2) have the typical skin and joint features. Those with vascular-type

C H A PT E R 1 7



G E N ETICS A N D DYS M O R P H O LOGY

369

children (boys and girls) with cognitive disability. Other features include macrocephaly, large ears, and male macro-orchidism in adulthood. Anticipation may occur with fragile X syndrome and other trinucleotide repeat disorders. In genetic anticipation, the signs and symptoms of the disorder present earlier and become more severe with each successive generation. •

Helpful Tip

=-� Frag i l e

X syn d rome is the most common i n herited

r1 1r ca use of cog n itive disa b i l ity that may affect boys a n d girls despite bei ng i n herited i n a n X-l i n ked fashion.

F I G U R E 1 7- 1 1 . In Ehl ers-Da nlos synd rome, the skin is elastic, soft, a n d hea l s poorly. (Reproduced w i t h permission from Goldsmith LA, Katz 51, G i l c h rest BA, Pa l l e r AS, Leffe l l DJ, Wolff K, eds. Fitzpatrick's Dermatology in General Medicine. 8th ed. New York, NY: McGraw- H i l l Education; 201 2, Fig. 1 3 7-1 .)

EDS (formerly EDS 4) are at increased risk for arterial rupture. Fragile X syndrome is the most common inherited cause of cog­ nitive disability. It is an X-linked disorder, but both boys and girls can be affected. The condition is due to the expansion of trinucle­ otide (CGG) repeats in the FMRl gene when passed from a carrier mother to offspring. Boys with a large CGG repeat expan­ sion (> 200 repeats) will have cognitive disability. Around 50% of girls with a large CGG repeat expansion will have cognitive disability due to skewing of X chromosome inactivation. There­ fore, it is important to perform fragile X molecular testing on all

You are evaluating a male infant for his 9-month health supervision visit. You find that he is near the 95th percen­ tile for height, weight, and head circumference. His mother tells you that he was large for gestational age and had several episodes of neonatal hypoglycemia. On physical exam you notice that he has a large tongue and one of his legs is larger than the other in both girth and length. The abdominal exam is unremarkable. Question 1 3-1

Which of the following screening tests should be performed? A) Lead level. B) Abdominal ultrasounds. C) Bone age. D) Serum alpha-fetoprotein (AFP) . E) Both B and D. Discussion 1 3-1

The correct answer is "E:' This infant likely has Beckwith­ Wiedemann syndrome (BWS). Children with BWS are often large, with macroglossia and have distinctive earlobe creases. Hemihypertrophy of a limb, omphalocele, and diastasis recti are common. Neonatal hypoglycemia can occur but is usually transient. Children with BWS or isolated hemihypertrophy are at increased for developing embryonal tumors such as Wilms tumor and hepatoblastoma. The outcomes for children with the tumors are improved through early detection. Therefore, all children with BWS should have abdominal ultrasounds per­ formed every 3 to 6 months until age 8 years, and serum AFP levels every 3 months until age 4 years. The same screening also applies to children with isolated hemihypertrophy.

F I G U R E 1 7- 1 2. Joint laxity a n d hypermobil ity is associated with E h l e rs­ Danlos syndrome. (Reproduced with permission from Fu ster V, Wa lsh RA, Ha rrington RA, eds. Hurst's The Heart. 1 3th ed. New York, NY: McGraw- H i l l Education; 201 1 , F i g . 1 4-8 B.)

A 3-year-old girl with a history of autism and cognitive delays presents to the emergency department with a generalized seizure. On physical exam you notice several white patches on her skin. A computed tomography (CT) scan of the head reveals several calcified subependymal nodules in the brain.

370

MCG RAW-H I LL E D U CATION S P E C I A LTY BOA R D REVI EW: P E D I ATRICS

A

B

F I G U R E 1 7- 1 3. Hypomela notic macule, also known as an ash-leaf s pot, is common in tuberous sclerosis complex. These spots a re more obvious u nder Wood l a m p exa mi nation. (Reproduced with permission fro m Wolff K, Johnson RA, Saaved ra AP, eds. Fitzpatrick's Color Atlas and Synopsis of Clinical Dermatology. 7th ed.

New York, NY: McGraw- H i l l Education; 20 1 3, Fig. 1 6-2 A, B.)

Question 1 4-1

This girl likely has which of the following conditions? A) Neurofibromatosis type 1 . B ) Congenital toxoplasmosis. C) Tuberous sclerosis complex. D) Klippel-Feil syndrome. E) Neurofibromatosis type 2.

posterior hairline and fusion of the cervical vertebrae, resulting in a short neck and torticollis. Malformations of the scapula (Spren­ gel deformity) can also occur. Neurofibromatosis type 2 (NF2) is largely associated with vestibular schwannoma occurring in adulthood and results in hearing loss and vertigo.

Discussion 1 4-1

The correct answer is "C:' This child has classic features of tuber­ ous sclerosis complex (TSC) . This condition results in benign central nervous system (CNS) tumors, including subependymal nodules and cortical tubers. However, malignant transformation can lead to subependymal astrocytoma. Seizures are common and some children have autism and developmental delay. None­ theless there is a large degree of variability within this condition, and some individuals are very mildly affected. Characteristic skin lesions called hypomelanotic macules (also known as ash­ leaf spots) are lightly pigmented and ovoid. (See Figure 17-13.) Tumors of the retina and kidney can also occur. Screening con­ sists of monitoring for malignant transformation with cranial and renal imaging and developmental programing. Neurofi­ bromatosis type 1 (NF 1 ) is associated with skin lesion and CNS tumors; however, the skin lesions, known as cafe-au-lait spots, are hyperpigmented rather than hypopigmented. (See Figure 1 7- 14.) Cutaneous neurofibromas and hamartomas (Lisch nodules) of the iris are common. Optic glioma can occur in childhood, and malignant nerve sheath tumors and pheochromocytoma are more common in adulthood. Around 40% to 50% of affected individuals have developmental or cognitive delays. Both NF 1 and TSC may be inherited in an autosomal dominant fashion from an affected parent or may occur de novo. Owing to the extreme variability of both conditions, it is possible that a parent could be mildly affected and previously undiagnosed; this may lead to the false assumption that the disorder was secondary to a de novo event. Klippel-Feil syndrome is associated with a low

A newborn infant boy is noticed at delivery to have absence of the left thumb and hypoplasia of the right thumb, with bowing of forearms. A loud systolic murmur is heard on exam. X-rays of the arms reveal bilateral radial hypoplasia. A chest X-ray is normal.

F I G U R E 1 7- 1 4. Cafe-au-lait spots a re associated with neurofi bromatosis type

1 . The raised l u m ps in the photo a re cuta neous neurofi bromas. (Reprod uced with permission from Fuster V, Wa lsh RA, Ha rrington RA (Eds). Hurst's The Heart, 1 3ed. McGraw- H i l l Education, I nc., 201 1 . Fig 1 4-7.)

C H A PT E R 1 7

C) D) E) F)



G E N ETICS A N D DYS M O R P H O LOGY

371

Apert syndrome. Pfeiffer syndrome. Fetal hydantoin syndrome. All of the above.

Discussion

F I G U R E 1 7- 1 5 . This c h i l d has severe b i l atera l rad i a l ray hypoplasia. (Reproduced with permission from Lichtma n MA, Shafer MS, Fel g a r RE, Wa ng N, eds. Lichtman's Atlas of Hematology. New York, NY: McGraw- H i l l Education; 2007, Fig. XI .A.96.)

Question 1 5-1

This infant most likely has which of the following disorders? A) Fanconi anemia. B) Thrombocytopenia absent radius syndrome. C) Diamond-Blackfan anemia. D) Holt-Oram syndrome. E) VACTERL association. Discussion 1 5-1

The correct answer is "D:' All of the disorders listed in options "!\.' through "E" can result in radial ray abnormalities, which are con­ genital deficiencies of radial bone in the forearm and the thumb. (See Figure 1 7- 1 5.) It is important to be able to distinguish one from the other. Holt-Oram syndrome results in radial hypoplasia with thumb aplasia or hypoplasia along with an atrial or ventricu­ lar septal defect. Fanconi anemia often results in radial ray defects with hypoplastic thumbs and aplastic anemia later in childhood. Other findings may include short stature, cafe-au-lait spots, and developmental delays. Affected individuals have an increased risk of developing cancer, particularly if exposed to ionizing radia­ tion. Thrombocytopenia absent radius (TAR) syndrome results in neonatal thrombocytopenia with absent or hypoplastic radii; however, the thumb is always present. Diamond-Blackfan anemia results in congenital anemia with thumb and radial hypo­ plasia. VACTERL association (Vertebral anomalies, Anal atresia, Cardiac defects, TracheoEsophageal fistula, Renal anomalies, and Limb deformities) often produces radial hypoplasia and must be included in the differential diagnosis for this infant. However, it is less likely than Holt -Oram to be the cause of this infant's anoma­ lies given the normal chest X-ray.

The correct answer is "B:' Craniosynostosis results from the pre­ mature fusion of one or more cranial sutures. Crouzon, Apert, and Pfeiffer syndromes are all autosomal dominant conditions characterized by craniosynostosis, shallow orbits, proptosis, hypertelorism, and high foreheads. Crouzon syndrome pro­ duces a beaked nose, frontal bossing, and midface hypoplasia. Apert syndrome results in a short anteroposterior diameter of the head, flat occiput, syndactyly (mitten hands) , and devel­ opmental delay. Pfeiffer syndrome causes broad thumbs and great toes. Fetal hydantoin syndrome (also called fetal Dilantin syndrome) is characterized by poor growth, hypoplastic nails and distal phalanges, a broad nasal bridge, wide fontanelle, and metopic ridging. Mothers who take phenytoin (Dilantin) during pregnancy have a 10% of having a fetus with fetal hydan­ toin syndrome. •

Helpful Tip

� Cra n i osynostosis

i nvolving the sag itta l, coronal, and

r1 1r l a m bdoid sutu res resu lts i n a cloverleaf sku l l deform ity.

� QUICKQUIZ Which karyotype listed below is associated with tall stature, behavioral problems, and poor school performance? A) 47,XXX. B) 47,XYY. C) 47,XXY. D) 46,XY. E) 46,XX. Discussion

The correct answer is "B:' Males with an extra Y chromosome (47,XYY) tend to be tall, have a long face and large ears, and suffer from behavioral problems, including hyperactivity, dis­ tractibility, and temper tantrums. Females who are 47,XXX are typically tall, commonly have learning disabilities, and are uncoordinated. Fertility is normal for both 47,XYY and 47,XXX individuals. Option "C" is Klinefelter syndrome, option "D" is a normal male, and option "E" is a normal female.

� QUICKQUIZ Which syndrome is NOT associated with craniosynostosis? A) Crouzon syndrome. B) Trisomy 2 1 .

A 2-year-old boy is being evaluated in the genetic clinic. He has aniridia and hypospadias. He is developmentally delayed and is receiving early intervention services.

372

MCG RAW-H I LL E D U CATION S P E C I A LTY BOA R D REVI EW: P E D I ATRICS

F I G U R E 1 7- 1 6. This c h i l d h a s bilatera l a n i ridia. S m a l l i r i s remnants a re present tem pora lly in each eye. Aniridia is associated with Wi lms tumor due to a deletion

in contiguous genes. (Reproduced with permission from Hay WW, Levin MJ, Deterd ing RR, Abzug MJ, eds. Current Diagnosis and Treatment Pediatrics. 22nd ed. New York, NY: McGraw-H i l l Education; 201 4, Fig. 1 6-2 1 , p. 469.)

Question 1 6-1

You recommend screening for Wilms tumor. Why? A) Children with aniridia are at increased risk for Wilms tumor. B) Aniridia and Wilms tumor may occur together as part of a contiguous gene syndrome. C) The association results from a deletion affecting a number of neighboring genes, including PAX6, the aniridia gene, and WTl , the Wilms tumor gene. D) None of the above. E) All of the above. Discussion 1 6-1

The correct answer is "E:' Contiguous gene syndromes result from gene abnormalities (usually deletions) that span more than one neighboring gene. WAGR syndrome (Wilms tumor, Aniridia, Genitourinary anomalies, and mental Retardation) results from a large deletion of many individual genes that are located contiguously on chromosome 1 1 . (See Figure 1 7- 1 6.)

A 6-year-old girl is referred to your clinic after failing the school hearing test. The mother has to repeat things to the child but attributes this to her "daydreaming" rather than hearing loss. The girl's face is narrow and flat, with downslant­ ing palpebral fissures. There are clefts over her zygomatic bones. Her auricles are poorly formed, with a small external ear canal opening. Question 1 7-1

This child most likely has which of the following disorders? A) Treacher Collins syndrome. B) Pierre Robin sequence. C) Stickler syndrome. D) Marshal syndrome. E) Van der Woude syndrome. Discussion 1 7-1

The correct answer is "A:' Treacher Collins syndrome (man­ dibulofacial dysostosis) involves abnormal formation of the facial bones. It is characterized by malar and mandibular

F I G U R E 1 7- 1 7. This g i rl has Treacher Col l i n s syn d rome. Notice the m a l a r

hypoplasia, downslanting pal pebral fissures, zygomatic b o n e clefts, m icrognathia, a n d a bsent lower eyelas hes. (Reproduced with permission from Brunica rd i FC, Andersen DK, B i l l i a r TR, et al, eds. Schwartz's Principles of Surgery. 1 Oth ed. New York, NY: McGraw- H i l l Education; 201 S, Fig 45- 1 9 A, p. 1 847.)

hypoplasia, zygomatic bone clefts, downslanting palpebral fissures, malformed external ears, and absent lower eyelashes. (See Figure 1 7- 1 7.) Conduction hearing loss is common. The craniofacial anomalies may result in a narrow airway and respi­ ratory distress requiring prone positioning or tracheostomy. Stickler syndrome involves midface and mandibular hypoplasia, high-degree myopia, flattened face with anteverted nares, and arthritis. Marshal syndrome is similar to Stickler syndrome with a flattened face, short depressed nose, anteverted nares, myopia, and eyes that appear to be large. Lower lip pits and cleft lip or palate, or both, are seen in van der Woude syndrome.

A young mother is referred to you for prenatal genetic counseling. She is 20 weeks' pregnant and had a prenatal ultrasound last week that was normal. The fetus is moving well, and her pregnancy has been uncomplicated thus far. Her brother and maternal uncle have Duchenne muscular dystrophy (DMD). Last week at her ultrasound, she found out she is having a boy. Question 1 8-1

What is the risk of her son being affected by DMD if the mother is a carrier? A) 1 00%. B) 50%. C) 25%. D) 10%. E) 3 % .

C H A PT E R 1 7

Discussion 1 8-1

The correct answer is "B:' DMD is an X-linked recessive disor­ der. Only males are affected. X-linked traits are never passed from father to son. If a mother is a carrier, then 50% of her sons will be affected and 50% of her daughters will be carriers. This is illustrated by the Punnett square for X-linked recessive inheri­ tance, shown below: X< (carrier) X

y

X XX XY

The mother asks if there is any testing you can perform now to see if her son will be affected by DMD. Question 1 8-2

Of the following, which is NOT an accurate reply to her question? A) Prenatal ultrasound can identify anomalies and or malformations. B) Chorionic villi sampling and amniocentesis may result in a spontaneous abortion. C) The quad screen looks at alterations in serum markers to screen for specific genetic disorders such as trisomy 2 1 . D ) Amniocentesis removes a small amount of amnionic fluid containing fetal cells that cultured for testing. E) All prenatal screening tests are invasive. Discussion 1 8-2

The correct answer is "E:' Some prenatal screening tests (ultrasound, quad screen) identify fetuses at increased risk for a genetic condition or birth defect by means of noninva­ sive testing. Newer testing using cell-free fetal DNA from the mother's blood can screen for fetal aneuploidy, but fetal DNA must be detected. To determine if the disorder is present (yes/ no), fetal tissue must be tested. This is called prenatal diagnosis and is often done using invasive procedures such as chorionic villi sampling, amniocentesis, or both. If these tests are incon­ clusive, umbilical cord blood sampling may be performed. This is a high-risk procedure and should be used in limited situations. The quad screen tests the levels of alpha-fetoprotein (AFP) , estriol, human chorionic gonadotropin (beta-hCG) , and inhibin A in the maternal blood. It is a screening test to iden­ tify fetuses at increased risk of having trisomies 2 1 , 1 8 , and 1 3 . A n elevated AFP may also indicate a n open neural tube defect. Ultrasound may detect an increase in nuchal translucency (marker of chromosomal disorders) and congenital anomalies or malformation such as gastroschisis or clubfoot. In patients who undergo in vitro fertilization, embryos can be tested prior to uterine implantation to identify those free of a specific genetic condition. This is called preimplantation genetic diagnosis. No screening test is perfect. False positive test results may occur. To determine with certainty whether a genetic disorder is present, confirmatory fetal testing is required, but this requires invasive means that are not without risk, including fetal loss.



G E N ETICS A N D DYS M O R P H O LOGY

373

i QUICKQUIZ Which neuromuscular disorder is NOT correctly paired with its mode of inheritance? A) Juvenile myotonic muscular dystrophy; autosomal dominant. B) Spinal muscular atrophy; autosomal recessive. C) Becker muscular dystrophy; X-linked dominant. D) Congenital myotonic dystrophy; maternal transmission. E) Mitochondrial myopathy; maternal transmission. Discussion

The correct answer is "C:' Becker's muscular dystrophy and Duchenne muscular dystrophy share a common genetic defect but Becker is a milder phenotype; therefore, both are X-linked recessive conditions. Congenital myotonic dystrophy is almost exclusively transmitted from the mother. Often the mother is unaware she has the disorder until after the newborn is diag­ nosed. Mitochondrial myopathies are maternally transmitted except those involving nuclear DNA, which follows autosomal dominant or recessive inheritance patterns.

You see a 2-year-old boy in clinic because his mother has been concerned about his poor language development. At 27 months he uses three words that all pertain to his favor­ ite talking train show and does not form sentences. He gets his mother attention by pulling her hand and does not point or use other gestures. He is unable to follow single-step com­ mands and rarely turns in response to his name. During your evaluation you notice that he has poor eye contact, is self­ directed, and is fascinated with turning the handle of the door. His motor skills are within the normal range and he is normally grown. On physical exam he is a handsome tod­ dler with no dysmorphic features. There is no family history of pregnancy loss, autism, or developmental delay. An earlier audiogram was normal. You suspect that the child has autism. Question 1 9-1

Following a formal diagnosis of autism, which genetic test would be most indicated? A) Fluorescent in situ hybridization (FISH) for 7ql l deletion. B) Microarray comparative genomic hybridization (CGH). C) Molecular testing for fragile X syndrome. D) Karyotype. E) DNA sequencing of the PTPNl l gene. F) Next-generation sequencing panel for autism. G) Whole exome sequencing. H) Both B and C. I) Both D and A. J) None of the above. Discussion 1 9-1

The correct answer is "H:' This autistic child is normally grown and has no dysmorphic features; therefore, he has nonsyndromic

374

MCG RAW-H I LL E D U CATION S P E C I A LTY BOA R D REVI EW: P E D I ATRICS

autism. The standard care of care for the genetic diagnosis of nonsyndromic autism or intellectual disability, or both, is the combination of microarray CGH and fragile X molecular test­ ing (polymerase chain reaction [PCR] and Southern blot) . This combination of testing is used when a specific condition is not strongly suspected by history or physical exam. Both males and females should undergo fragile X testing as 50% of females with a full expansion will have intellectual disability. Microarray CGH is a new technology that surveys the entire genome for copy number variation (chromosomal deletions and duplications). This technology has completely replaced karyotype as the diag­ nostic method of choice for the following indications: autism, intellectual disability, syndromic short stature, dysmorphic

features, and congenital anomalies. Microarray CGH will only detect unbalanced genomic abnormalities; therefore, karyotype still has a role in the prenatal and preconception settings look­ ing for balanced rearrangements that would predispose a carrier parent to having a child with an unbalanced karyotype. These balanced rearrangements include translocations and inversions. Karyotype is also used to confirm the common trisomy disor­ ders (2 1 , 18, 1 3 ) when suspected clinically. These aneuploidies can be detected by microarray CGH but karyotype is more cost effective for this indication. FISH for 7 q 1 1 would look for Wil­ liams syndrome; however, nothing in this case suggests that specific syndrome. FISH has a very limited role in the genetic diagnosis of children; this technology should only be used to

TABLE 1 7- 1 CO N S I D ERAT I O N S I N G E N ET I C TESTI N G

Test

Pros

Con s

Ka ryotype

Whole g e n o m i c a p p roach

Ve ry l ow reso l ution, s o o n ly l a rge struct u ra l rea rra n g e m e nts a re seen

Ca n detect bala nced a b n o r m a l ities that may pred i s pose that carrier to preg na ncy loss o r an u n ba l a nced offspring U sed i n the prenata l sett i n g for d etection of a n e u ploidy M i c roa rray co m­ pa rative g e n o m i c hybri d ization (CG H)

Whole genomic a p p roach

Wi l l not detect bala nced rea rra n g e m e nts

H i g h resol ution; ca n detect s ma l l copy n u m ber va ria nts, i nc l u d i n g all known c h romoso m a l m icrod eletion a n d d u pl ication syn d romes

S m a l l va ria nts may have u n known s i g n ifica nce

New arrays can detect exon-leve l deletio n s/ d u pl ications F l u o rescent i n situ hybrid i zation (FISH) for specific locus

Looks i n high reso l ution at specific d i sease­ associated reg ions (eg, Wi l l i a m s syn d rome)

DNA seq uencing

Looks fo r s pecific po i nt m utati o n s i n a s u s pected gene

S h o u l d not g ive va ria nts of u n known s i g n ifi ca n ce

U sed when a specific gene i s s u s pected c l i n ica lly

May detect normal va riation O n ly d etects copy n u m ber va riation; wi l l n ot detect D N A point m utatio n s O n ly assays a s i n g l e c h romoso m a l reg ion O n ly d etects copy n u m ber va riation; wi l l n ot detect D N A point m utatio n s May m i ss sma l l non-seq ue nce-ba sed D N A cha nges such a s s m a l l deletions or tri n u c l eo­ tide repeats O n ly looks at a s i n g l e gene at a t i m e Wi l l n o t detect copy n u m ber va riati o n s

Next-gen eration DNA seq uencing pa n e l s

Can look fo r DNA seq uence changes i n m a ny genes at the same t i m e based on phenotype (eg, a utism)

M a y m i ss sma l l non-seq uence-based D N A c h a n g e s such as sma l l deletions or trinucleotide repeats Wi l l not detect copy n u m ber va riati o n s Expe n s ive

Whole exo me seq u e n c i n g

Can assay all known genes i n one test Exce l lent test for d ifficult cases in wh ich other tests have been negative

Relatively l ow sens itivity to s pecific DNA cha nges Difficult bioi nformatics a n d i nterpretation May m i ss small non-seq uence-based DNA cha nges such as s ma l l deletions or tri n ucleotide repeats Wi l l not detect copy n u m ber va riati o n s Expe n s ive

C H A PT E R 1 7

confirm a chromosomal microdeletion or duplication syndrome when a specific syndrome such as Williams or 22q 1 1 deletion is strongly suspected. Microarray CGH will also diagnose these conditions as well as many others and therefore is a better first­ line test in most cases. DNA sequencing of PTPNl l would test for Noonan syndrome; again, nothing in this vignette is sug­ gestive of that condition. DNA sequencing of a specific gene is performed when a condition that is suspected is caused by a DNA point mutation. Recently, next-generation DNA sequenc­ ing panels have become available. These panels allow for the sequencing of many genes (in some cases hundreds) at the same time. This approach is useful when the condition has many sep­ arate genetic causes that are indistinguishable clinically, such as autism, hearing loss, or epilepsy, or when many genes all cause the same syndrome, as in Noonan syndrome. An autism panel might be a good second step but the initial testing should be microarray CGH and fragile X syndrome molecular testing as these tests have higher yield. Finally, whole exome sequencing allows for sequencing of the entire protein coding areas ( exons) of the genome. This technology is used when a specific condi­ tion or syndrome is not suspected clinically and other tests such as microarray CGH are negative. Table 1 7 - 1 describes com­ monly performed genetic tests. B I B L I O G RA P H Y

Allanson JE, Roberts AE. Noonan syndrome. (200 1 Nov 1 5 [Updated 201 1 Aug 4] .) In: Pagon RA, Adam MP, Ardinger HH, et al, eds. GeneReviews [Internet] . Seattle, WA: University of Washington, Seattle; 1 993-20 1 5 . http:// www. ncbi.nlm.nih.gov/books/NBK 1 1 24/ Bondy CA. Care of girls and women with Turner syndrome: A guideline of the Turner Syndrome Study Group. J Clin Endocrinol Metab. 2007;92 ( 1 ) : 1 0-25.



G E N ETICS AND DYS M O R P H O LOGY

375

Bull MJ. Health supervision for children with Down syndrome. Pediatrics. 20 1 1 ; 1 28(2):393-406. Dean JCS. Marfan syndrome: Clinical diagnosis and manage­ ment. Eur J Hum Genet. 2007; 1 5 (7):724-733 . Friedman JM. Neurofibromatosis 1 . ( 1 998 Oct 2 [Updated 20 14 Sep 4] .) In: Pagon RA, Adam MP, Ardinger HH, et al, eds. GeneReviews [Internet] . Seattle, WA: University of Washington, Seattle; 1 993-20 1 5 . http://www. ncbi.nlm. nih.gov/books/NBK1 1 09/ Hoyme HE, May PA, Kalberg WO, et al. A practical clinical approach to diagnosis of fetal alcohol spectrum disorders: Clarification of the 1 996 Institute of Medicine criteria. Pediatrics. 2005; 1 1 5 ( 1 ) : 39-47. Jones KL. Smith's Recognizable Patterns of Human Malformation. 6th ed. Philadelphia, PA: Elsevier; 2006. McDonald-McGinn DM, Emanuel BS, Zackai EH. 22q1 1 .2 dele­ tion syndrome. ( 1 999 Sep 23 [Updated 20 1 3 Feb 28] .) In: Pagon RA, Adam MP, Ardinger HH, et al, eds. GeneReviews [Internet] . Seattle, WA: University of Washington, Seattle; 1993-20 15. http://www.ncbi.nlm.nih.gov/books/NBK1 523/ Miles JH. Autism spectrum disorders-a genetics review. Gen. Med. 20 1 1 ; 1 3 (4):278-294. Saul RA, Tarleton JC. FMR1 -related disorders. ( 1 998 Jun 1 6 [Updated 2 0 12 Apr 26] .) In: Pagon RA, Adam MP, Ardinger HH, et al, eds. GeneReviews [Internet] . Seattle, WA: University of Washington, Seattle; 1 993-20 1 5 . http:// www. ncbi.nlm.nih.gov/books/NBK1 384/ Solomon BD, Muenke M. When to suspect a genetic syndrome. Am Pam Physician. 201 2;86(9) :826. O n l i n e Resou rces

GeneReviews: http:/ /www.ncbi.nlm.nih.gov/books/NBK1 1 1 6 Online Mendelian Inheritance in Man (OMIM): http://www. ncbi.nlm.nih.gov/omim

This page intentionally left blank

18

Gen ita l System Disorders Kath leen K i e ra n

An 8-month-old girl presents with a 2-day history of increas­ ing fussiness, fevers (the highest is 39.8°C [ 1 03.6°F] ), and decreasing oral intake. Her parents note that she has been feeding less frequently and in smaller volumes, and she has had fewer wet diapers than usual. They also report that she has had several febrile episodes in the past and has been treated for ear infections. Her urine has also been quite foul-smelling over the last day. On examination, she is cry­ ing loudly and warm to the touch. Her blood pressure is normal, and perfusion is fine. She does not have tears. She is very uncooperative with the examination, but appears to have more discomfort when the right flank is palpated. You suspect a urinary tract infection. Question 1 -1

Which of the following is the most appropriate method to use in collecting urine for a sample? A) Voided urine into a bag adhered to the perineum, because the child is not toilet trained. B) Midstream urine collection, because the female urethra has commensal microorganisms. C) Catheterized urine sample, because a voided sample may be contaminated. D) Suprapubic aspirate, because catheterization may be trau­ matic for the child. E) No urine collection is needed prior to initiation of therapy, because the child is oliguric and ill.

would be challenging in a non-toilet-trained child. The parents would win an award if able to catch a pee sample from an infant. Catheterization, when performed correctly, is associated with a low risk of contamination and is not traumatic for the child (particularly before the age of genital awareness) . Suprapubic aspiration (SPA) is the method least likely to be associated with contamination, but this procedure is not commonly performed and some practitioners may not be facile or comfortable with it. Urine cultures should be collected by SPA or catheterization. Unless the child has clear signs of hemodynamic instability (not present in this scenario) , urine should be collected prior to the initiation of antimicrobial therapy to confirm the suspected diag­ nosis of urinary tract infection (UTI) and to guide antimicrobial therapy. Although antibiotic susceptibility panels may not return until 72 hours after the urine is cultured, community prevalence of certain bacterial species and antibiograms showing local anti­ biotic resistance patterns are usually available within institutions. •



Helpful Tip

In, 201 1 the America n Academy of Pediatrics (AAP)

1 1 1r publ ished cli nical practice guideli nes for diag nosis and

ma nagement of the first UTI in febrile infa nts and children aged 2 to 24 months old. lf a UTI is suspected, a clinician has

two choices: ( 1 ) obtain urine for culture and urinalysis via SPA or catheterization, or (2) obtain a bag urine specimen for urinalysis. If a bnormal (positive for leu kocyte esterase, microscopic pyu ria, or positive for nitrites), a catheterized or SPA culture should be collected. A bag urine specimen should never be sent for culture.

Discussion 1 -1

The correct answer is "C:' Bag urine specimens, while con­ venient, may be associated with false positive urinalyses if the perineum is not well cleansed prior to specimen acquisition. Cultures from bag specimens are only helpful if negative. Mid­ stream urine collections, while feasible in older children, would require knowledge of when the child would void, an ability to clean the urethra, and an ability to catch midstream urine, which

Question 1 -2

Which is NOT a risk factor for a UTI? A) African American race. B) Fever � 39°C { 1 02.2°F). C) Uncircumcised. D) Fever without an alternative source of infection. E) Female gender. 377

378

MCG RAW-H I LL E D U CATION S P E C I A LTY BOA R D REVI EW: P E D I ATRICS

Discussion 1 -2

The correct answer is "A:' Five percent of febrile infants with no identified fever source will have a UTI. Nonblack or white race is a risk factor. Girls are at higher risk than boys. Uncircum­ cised boys are at higher risk than those who are circumcised. Age younger than 12 months is also a risk factor.

refrigerated) and processed. To diagnosis a UTI, both pyuria or bacteruria on urinalysis and a positive urine culture are required. This avoids unnecessary treatment of asymptomatic bacteruria. One would imagine that most practitioners are not routinely trained in urine sniffing. Question 1 -5

A catheterized urine sample is collected. Question 1 -3

Assuming this infant has a UTI, which of the following uri­ nalyses is most likely to be seen when her urine is evaluated? A) Specific gravity 1 .0 1 0, 2+ leukocyte esterase, negative nitrites, negative ketones; microscopy shows 2-3 white blood cells (WBCs)/high-power field (hpf) and 5- 1 0 RBCs/hpf. B) Specific gravity 1 . 0 1 5, negative leukocyte esterase, negative nitrites, negative ketones; microscopy shows 2-3 WBCs/hpf and 1 -2 RBCs/hpf. C) Specific gravity 1 .020, 2+ leukocyte esterase, negative nitrites, 3+ ketones; microscopy shows 5- 1 0 WBCs/hpf and 5- 1 0 RBCs/hpf. D) Specific gravity 1 .025, 2+ leukocyte esterase, positive nitrites, negative ketones; microscopy shows 3-5 WBCs/hpf and 0-3 RBCs/hpf. E) Specific gravity 1 .025, 3+ leukocyte esterase, positive nitrites, 3+ ketones; microscopy shows 30-50 WBCs/hpf and 1 5-20 RBCs/hpf. Discussion 1 -3

The correct answer is "E:' The vignette describes a child who is dehydrated, with poor oral intake and decreased urine out­ put; this would be consistent with a higher urine specific grav­ ity and the presence of ketones in the urine. Leukocyte esterase is an enzyme produced by WBCs; thus, its presence would be associated with WBCs on microscopy. Pyuria (defined as 5 or more WBCs/hpf) , rather than hematuria, is the most sensitive and specific marker for UTI. Gram-negative bacteria, which are the most common etiologic agents for UTis, convert nitrates to nitrites. Although a urinalysis for a child with a UTI may not show nitrites, typically this occurs when the causal organism is not a gram-negative bacterium or when the urine is collected quickly after the suspicion of a UTI has been established (con­ version of nitrates to nitrates can take up to 4 hours) . Question 1 -4

What is NOT a diagnostic criterion for a UTI in a febrile infant or child aged 2 to 24 months? A) Diagnosis of a UTI requires an abnormal urinalysis. B) A positive urine culture is � 50,000 colony-forming units of a single, urinary pathogen. C) Urine culture must be obtained by adequate means. D) Urine does not need to be fresh to perform a urinalysis. E) Smelly urine is all that is needed. Discussion 1 -4

The correct answer is "D:' Urine specimens need to be freshly collected ( < 1 hour at room temperature or < 4 hours if

The organism that is associated with most UTis in otherwise healthy children is: A) Escherichia coli. B) Pseudomonas aeruginosa. C) Enterococcus faecalis. D) Streptococcus pneumoniae. E) Streptococcus viridans. Discussion 1 -5

The correct answer is "A:' E. coli remains the most common organism causing UTis.

The girl undergoes a voiding cystourethrogram (VCUG ) after her UTI has been appropriately treated. The images show grade V vesicoureteral reflux. Question 1 -6

What is the next step in management? A) Observation. B) Trimethoprim-sulfamethoxazole 2 mg/kg once daily. C) Amoxicillin 20 mg/kg three times daily. D) DMSA renal scan. E) Surgical correction. Discussion 1 -6

The correct answer is "B:' Reflux is graded on a scale from I to V, with I being reflux into the ureter alone, II being reflux into the calyces with preservation of the calyceal architecture, III being reflux into the calyces with blunting of the calyces, IV being mild ureteral tortuosity, and V being severe ureteral tortuosity. More recent publications have suggested that other radiographic findings, such as distal ureteral diameter, and timing of reflux during the micturition cycle, may refine the predictive value of reflux grade for spontaneous resolution. A meta-analysis cited in the AAP recommendations noted that antibiotic prophylaxis was associated with a decreased risk of UTI recurrence in chil­ dren with grade V reflux. Among children with a lower grade of reflux, no significant differences in UTI recurrence rates were observed in children prescribed antibiotic prophylaxis com­ pared with those who were not, but factors such as compliance may account for these findings. Amoxicillin is typically used for antibiotic prophylaxis in children younger than 2 months of age, since the immature neonatal liver may be unable to conju­ gate trimethoprim-sulfamethoxazole (Bactrim) for elimination, resulting in jaundice. The volume of distribution for amoxicil­ lin is the entire body, whereas trimethoprim-sulfamethoxazole concentrates in the urine, allowing a lower dose to be given in order to achieve a locally active antibiotic concentration. DMSA (dimercaptosuccinic acid) renal scans are not endorsed as part of the routine evaluation of a child with vesicoureteral reflux but

C H A PT E R 1 8



G E N ITA L SYSTEM D I SO R D E R S

379

Discussion 1 -7

F I G U R E 1 8- 1 . Vesicouretera l reflux, urine movi ng from bladder i nto the upper u r i n a ry tract, is eva l u ated with a voiding cysto u reth rog ra m (VCUG). It is g raded by l evel of u r i n a ry refl ux a n d d i l ation as fol l ows: G rade 1-refl ux i nto u reter on ly; g rade 1 1-refl ux i nto u reter and ren a l pelvis; g rade I l l-refl ux with d i lation of the u reter and ren a l pelvis; g rade IV-refl ux with d i l ation of the u reter a n d ren a l pelvis with b l u nting of the calyces; a n d g rade IV-massive refl ux with tortuous d i lation of the u reter. Left-sided g rade IV vesicouretera l refl ux is shown in this i mage. (Reproduced with permission from Doherty GM, ed. Current Diagnosis & Treatment: Surgery. 1 4th ed. New Yo rk, NY:

McGraw- H i l l Education; 201 5, Fig. 38-7.)

may be helpful to identify children with renal scars or decreased renal function. Surgical correction is generally reserved for children who have failed conservative management (ie, those who develop breakthrough UTis on antibiotic prophylaxis or are unable to comply with antibiotic prophylaxis) , or who show signs of renal damage; it is not first-line therapy for the manage­ ment of vesicoureteral reflux. (See Figure 1 8 - 1 . ) Make note that according to the AAP, a VCUG is not routinely needed. After the first UTI, a renal ultrasound should be performed. If abnor­ mal, then a VCUG should be obtained. Question 1 -7

Parents of children with vesicoureteral reflux should be counseled that: A) VCUGs should be performed in all siblings as there is a 75% prevalence of reflux in first -degree relatives of children with reflux. B) Surgical correction of the reflux will prevent the child from getting another UTI until at least adulthood. C) Prophylactic antibiotics will not change the risk of renal scarring. D) Optimized elimination habits are associated with fewer UTis and a higher rate of reflux resolution. E) Resolution rates in secondary vesicoureteral reflux reflect the reflux severity and laterality as well as the age and gender of the child.

The correct answer is "D:' When discussing the management of vesicoureteral reflux with parents, it is important to differenti­ ate between UTis and vesicoureteral reflux. In many cases, par­ ticularly in toilet-trained children, UTI risk can be decreased markedly by managing constipation, increasing fluid intake, and promoting frequent urination. Although poor elimina­ tion habits (holding it) have been associated with persistence of vesicoureteral reflux, only very high-grade reflux has been shown to be independently associated with an increased risk of UTI. If these habits persist, the child will continue to be at risk for lower tract UTis even after correction of the reflux. Pro­ phylactic antibiotics are associated with the same or decreased risk of UTI (depending on the reflux grade), and since renal scarring has been shown to occur only in the presence of infected urine, may protect kidneys from the development of new scars. Between 25% and 3 3 % of siblings of children with reflux have been found to have reflux as well; the variation in prevalence reflects children who were screened without symp­ toms compared with those who underwent imaging based on infection or other symptoms. Presently, a renal ultrasound is recommended for siblings of children with reflux, with a VCUG to follow if renal anatomic abnormalities are identified. Siblings with no renal structural abnormalities who develop UTis should be managed according to current AAP guide­ lines. Children with reflux should be carefully evaluated for secondary reflux, in which lower urinary tract abnormalities ( eg, neurogenic bladder related to myelomeningocele, or myo­ genic bladder secondary to posterior urethral valves), rather than an anatomic abnormality at the ureterovesical junction, is the underlying causal factor. Most cases of low-grade primary vesicoureteral reflux resolve spontaneously.



Helpful Tip

:5.� Void i n g

dysfu nction has been associated with a n

i1 1 r increased risk of UTis a n d delayed (and decreased)

resol ution rates for vesicou retera l refl ux. Be s u re to ask toi l et-trai ned c h i l d ren a bout their e l i m i nation ha bits.





Helpful Tip

Recurrent UTis a re associated with ren a l sca rri n g . Early

1 1 1r a nti biotic treatment i s better than

late. Ca regivers need to seek i m m ed iate ca re for future febrile i l l nesses

to eva l uate for a repeat UTI .

A 28-year-old G I PO (now 1 ) mother has just given birth to a 6 pound, 7 ounce infant at 39-4/7 weeks' gestation. The pregnancy was uncomplicated and the labor progressed appropriately. The mother underwent several prenatal ultrasounds, all of which showed normal levels of amniotic fluid and no fetal anomalies, but chose not to be informed

380

MCG RAW-H I LL E D U CATION S P E C I A LTY BOA R D REVI EW: P E D I ATRICS

low cortisol, resulting in adrenal gland enlargement. Inadequate mineralocorticoid production results in salt-wasting crisis with hyponatremia, hyperkalemia, hypovolemia, and cardiovascular collapse if untreated. Classic CAH (no enzyme activity) presents at birth while nonclassic milder forms (partial enzyme activity) present later in childhood. Because of the frequency of CAH and its potentially severe consequences, CAH testing is included on newborn panels for early identification. (See Figures 1 8-2 and 18-3.) •

Helpful Tip

� Newborn boys with classic CAH have normal genita l i a . 1 1 1r Without newborn screeni ng, they a re u n d iag nosed

until presenting with a salt wasting crisis at 1 to 2 weeks of age with s i g n ifica nt weight loss, dehyd ration, hyponatremia, and hyperkalemia.

F I G U R E 1 8-2. Ambiguous genita l i a i n congen ita l a d renal hyperplasia.

Question 2-2

The g i rl i n this picture has congen ita l adrenal hyperplasia with a m biguous external genita lia resu lting from excess a d renal a n d rogens. The scrotol a b i al folds a re fused, the cl itoris is e n l a rged, a n d a pen i l e u rethra has formed. No testes will be pa lpable, suggesting a d i sorder of sexual differentiation. Males with congen ita l a d renal hyperplasia have normal external genita l i a . (Reproduced w i t h permission from Brunica rd i F C , Andersen DK, B i l l i a r TR, et al, eds. Schwartz's Principles of Surgery. 1 Oth ed. New York, NY: McGraw- H i l l Education; 201 5, F i g . 39-36.)

What is the inheritance pattern of the most common form of this condition? A) Autosomal dominant. B) Autosomal recessive. C) X-linked dominant. D) X-linked recessive. E) Sporadic.

of her baby's gender. On examination, the infant has a good cry and Apgar scores of 8 at 1 minute and 9 at 5 minutes. The abdomen is soft. The genitalia are not clearly male or female. (See Figure 1 8-2.)

The correct answer is "B:' The gene encoding 2 1 -hydroxylase is on chromosome 6p, adjacent to the human leukocyte antigen (HLA) genes, and 2 1 -hydroxylase deficiency is inherited as an autosomal recessive trait. Some rare cases are transmitted as a new mutation or as mutations on chromosomes other than chromosome 6.

Question 2-1

Question 2-3

What is the primary concern when evaluating this infant? A) Blood pressure. B) Electrolyte levels. C) Urine output. D) Oral intake. E) Respiratory distress.

How is CAH diagnosed? A) Elevated blood 1 7-hydroxyprogesterone. B) ACTH stimulation test. C) Newborn screening. D) Elevated amnionic 1 7-hydroxyprogesterone. E) All of the above.

Discussion 2-2

Discussion 2-1

Discussion 2-3

The correct answer is "B:' In any infant with external genitalia that cannot be definitively categorized as male or female, con­ genital adrenal hyperplasia ( CAH) must be excluded. CAH is the most common cause of disorders of sexual differentiation in newborns. There are several different enzyme deficiencies that may be associated with CAH; in the most common, 2 1 -hydrox­ ylase deficiency, a lack of the 2 1 -hydroxylase enzyme results in a relative excess of 1 7 -hydroxyprogesterone ( 1 7 -OHP) and shunt­ ing of the intermediate products into the sex steroid pathway, resulting in masculinization. Cortisol and aldosterone produc­ tion are decreased while androstenedione and dehydroepi­ androsterone (DHEA) production are increased. Secretion of adrenocorticotropic hormone (ACTH) increases in response to

The correct answer is "E:' A very high level of 17 -OHP in the blood is diagnostic of CAH. If 1 7 -OHP is only mildly elevated, an ACTH stimulation test should be performed. The level of 1 7-0HP will increase and cortisol will remain low. In children with salt-wasting, serum renin will be elevated and aldosterone low. Question 2-4

1 1 -beta-hydroxylase deficiency is associated with: Hypertension. Hypokalemia. Hyperglycemia. Hypotonia. Elevated transaminases.

A) B) C) D) E)

C H A PT E R

18



G E N ITA L SYSTEM D I SO R D E R S

381

D H EAI

tAndrostenedione

Estrone

1

L___t_ li_ e s_t_ o s_t_ e_ ro_ n_ e __jf-. Estradiol

11 B-Hydroxylase

Cortisoll A

Estrone

1

Estradiol

11 B-Hydroxylase

Cortisoll 8 F I G U R E 1 8-3. Depicted is the pathway of a d renal synthesis of m i n e ra locorticoids, g l u cocorticoids, a n d a d renal a n d rogens a n d changes seen in congenita l adrenal hyperplasia (CAH). (A) Classic CAH is d u e to 2 1 -hyd roxylase deficiency (red X) with excess a d renal a n d rogens a n d deficiency of cortisol a n d a l dosterone. (B) Less common ly, CAH is due to 1 1 -beta-hyd roxylase deficiency (green X) with excess a d renal a n d rogens, cortisol deficiency but preserved m i neralocorticoid

fu nction fro m high l evel s of deoxycorticosterone. DH EA, dehyd roepiandrosterone.

Discussion 2-4

The correct answer is "A:' 1 1 -Beta-hydroxylase deficiency is the second most common cause of congenital adrenal hyperplasia, resulting in cortisol deficiency, excess adre­ nal androgens, and preserved mineralocorticoid function. In 1 1 -beta-hydroxylase deficiency, corticosterone cannot be synthesized from 1 1 -deoxycorticosterone owing to the missing enzyme. Deoxycorticosterone is a weak mineralo­ corticoid, but when present in large amounts, contributes to salt and water retention as well as high blood pressure. In 1 1 -beta-hydroxylase deficiency, 1 7-0HP levels are normal and 1 1 -deoxycortisol and deoxycorticosterone are elevated. Children with 1 1 -beta-hydroxylase deficiency are also at risk for hypoglycemia since glucocorticoid synthesis cannot be completed. (See Figure 1 8- 3 , earlier. )

A twin newborn male is born after a dichorionic, diamni­ otic pregnancy that was otherwise uncomplicated. He and his brother were born at 36 weeks' gestation following an uncomplicated normal spontaneous vaginal delivery. He weighs 5 pounds, 3 ounces, and his brother weighs 7 pound, 5 ounces. His Apgar scores were 8 at 1 minute and 9 at 5 minutes. On examination, he is found to have a dorsal hooded fore­ skin and the urethral meatus is noted at the ventral midshaft. The testes are bilaterally descended and palpably normal and there are no hernias or hydroceles. His brother has normal Tanner 1 male external genitalia with a circumferen­ tially intact foreskin. The parents request that both boys be circumcised.

382

MCG RAW-H I LL E D U CATION S P E C I A LTY BOA R D REVI EW: P E D I ATRICS

Question 3-1

Which of the following should be included in parental counseling? A) Prior to circumcision, he should undergo a renal ultrasound and VCUG to exclude any coexisting abnormalities of the genitourinary tract. B) Circumcision should be deferred in boys with hypospadias. C) The AAP recommends neonatal circumcision for all boys (unless there is a religious or cultural contraindication) , because this significantly reduces the risk o f penile cancer and transmission of viruses such as human immunodefi­ ciency virus (HIV) and human papillomavirus (HPV) . D) Neonatal circumcision can be safely performed without anesthetic as newborns do not feel pain and the benefits of anesthesia are outweighed by the risks. E) He should undergo hormonal testing prior to circumcision to rule out a disorder of sexual differentiation. Discussion 3-1

The correct answer is "B:' Hypospadias is seen in approximately 0.33% of newborn boys (although this proportion may be vari­ able since mild forms may not be captured, and home births may not be included) . It is a congenital anomaly in which the urethral meatus is located on the ventral penile shaft rather than the tip of the glans. Don't forget that in anatomic position the penis is erect. In most cases, hypospadias occurs as a sporadic disorder. There are several theories regarding its development, including failure of the glanular urethra to recanalize and failure of the ventral glans and prepuce to close. Hereditary factors and endocrine disruptors may be associated with an increased risk of hypospadias. The phallus forms at 10 weeks and the remain­ der of the genitourinary tract at 6 weeks; because the external genitalia and the remainder of the genitourinary system form at different times, evaluation for coexisting abnormalities is not necessary in the absence of findings such as prenatal hydrone­ phrosis. Similarly, evaluation for disorders of sexual differentia­ tion is undertaken when there is an abnormality of the gonad (eg, undescended testis), severe hypospadias, or evidence of other hormonal abnormalities. In particular, children with bilat­ eral undescended testes with a phallic structure should be eval­ uated for possible congenital adrenal hyperplasia, regardless of whether or not hypospadias is also present. (See Figure 1 8-4.) The exact method used to repair hypospadias is dependent on many factors, including the tissue quality, meatal location, and coexisting abnormalities (eg, chordee), but circumcision is generally deferred in order to preserve the prepuce for possible repair should this tissue be needed. (Prepuce is a fancy way of saying foreskin.) As neonatal circumcision is associated with a decreased risk of HPV and HIV transmission later in life and the risk of penile cancer is substantially lower in circumcised males, the AAP feels the benefits outweigh the potential risks and that families should have access to the procedure, includ­ ing insurance coverage. However, the AAP has stopped short of routine endorsement of circumcision for all infants. When neonatal circumcision is performed, local anesthetic with a penile block should be employed; sugar water alone may be an adjunct to but is not a substitute for anesthesia. Circumcision

) A. Ph imosis

B . Paraph i mosis

C. Hypospadias

D. Epispadias

F I G U R E 1 8-4. Abnorm a l ities of the Foreskin. (A) P h i mosis-the foreskin is

tight a n d cannot be retracted over the glans penis. (B) Pa ra p h i mosis-the foreskin has been retracted over the glans a n d can not be red uced, causing swe l l i ng. (C) Hypospadias-the u rethral meatus opens on the ventral side (underside) of the penis. (D) Epispadias-the u rethral meatus opens on the dorsal side (top) of the penis. (Reproduced with permission from Le Blond RF, Brown DD, Suneja M, Szot J F, eds. DeGowin's Diagnostic Examination. 1 Oth ed. New York, NY: McGraw-H i l l Education; 201 5, Fig. 1 2-7 A-D.)

should be deferred in children with penile abnormalities that may necessitate reconstructive surgery, such as chordee, bur­ ied penis, micropenis (a penis that, when stretched, had a length more than two standard deviations below the mean penile length for age), and penile torsion. Boys with micro­ penis should also be assessed for endocrine abnormalities, particularly hypopituitarism, and the position of the testes should be assessed. •



Helpful Tip

Not a l l hypospadias repa i rs req u i re the foreski n for

1 1 1r reconstruction.

However, genera l ly the prepuce is left i n place so that it is ava i la ble for u reth ra l reconstruction or s k i n coverage a t t h e t i m e o f repa i r. Rarely, the foreski n may be c i rcu mferentia l ly i ntact a n d

t h e meatus l ocated abnorma lly ventra l ly; t h i s is ca l led a mega meatus with i ntact prepuce.

A 4-year-old girl has a history of daytime urinary inconti­ nence for the last 6 months. The incontinence has waxed and waned, and some days she is able to stay completely dry but has never been dry for more than a day or two at a time. She also has nocturnal enuresis every night and has never been dry at night. Her parents state that she was toi­ let trained at 2.5 years old. They report that she often is seen squatting with her heel in her perineum. Her daytime wetting is associated with urinary urgency, and she often runs to the bathroom when she has to void, only to leak urine as she is pulling down her pants. Her parents are quite frustrated because some of the other children have started to make fun of her at school when the wetting acci­ dents occur.

C H A PT E R

18

Discussion 4-1

The correct answer is "C:' Imaging studies are indicated in chil­ dren who have suggestion of anatomic abnormalities or a failure to improve after a reasonable interval on appropriate treat­ ment. Intermittent incontinence in children is most commonly associated with abnormal voiding habits and, in many cases, constipation or otherwise abnormal stool habits. A voiding diary is helpful in obtaining accurate and more objective data than his­ torical reporting alone. A careful history and physical examina­ tion should be performed to assess for possible neurologic causes. Although both anticholinergic therapy and pelvic floor physical therapy have been successfully employed to treat children with urinary incontinence, treatments are most effective when they are symptom-directed, again making the voiding diary helpful. Initiation of therapy in the absence of an understanding of the underlying etiology may result in ineffective therapies being tried and the family becoming increasingly frustrated.

On further questioning, you learn that the girl voids infre­ quently, using the school restroom only once daily. She has a large, hard stool every third or fourth day, and has fecal leakage once or twice weekly.

G E N ITA L SYSTEM D I SO R D E R S

383

B ristol Stool C hart

Question 4-1

Which of the following is the most appropriate next step in evaluation and treatment? A) Renal ultrasound. B) Voiding cystourethrogram. C) Voiding diary. D) Initiation of anticholinergic therapy. E) Initiation of pelvic floor physical therapy.



Type 1

• ••

:

Separate hard lumps, like nuts (hard to pass)

Type 2

Sausage-shaped but lu mpy

Type 3

Like a sausage but with cracks on the su rface

Type 4

Like a sausage or snake, smooth and soft

Type 5

Soft blobs with clear-cut edges

Type 6

Fl uffy pieces with ragged edges, a mushy stool

Type 7

Watery, no solid pieces. Entirely Liquid

F I G U R E 1 8-5. Bristol stool chart. (Reproduced with perm1ss1on from

Henderson MC, Tierney LM J r, Smeta na GW, eds. The Patient History: An Evidence-Based Approach to Differential Diagnosis. 2nd ed. New York, NY:

McGraw-H i l l Education; 201 2, Fig. 32-1 .)

Question 4-2

severity of constipation. The Bristol stool scale provides a stan­ dardized measure of stool appearance and consistency that can be used by both providers and patients. (It also reinforces pedia­ tricians' love of food references.) Management of constipation is multimodal, with scheduled toilet sitting, increased fluid intake, and intake of agents to soften the stool (eg, polyethylene glycol). Enemas may be required to assist in regular stooling but are gener­ ally not employed as first-line therapy. (See Figure 1 8-5.)

Which of the following is most likely to be true regarding her stool habits? A) She likely has an inadequate intake of fruits and vegetables. B) She should undergo evaluation for a neurologic disorder given the coexisting urinary incontinence. C) The severity of constipation should be evaluated periodi­ cally with abdominal X-rays. D) The Bristol stool scale provides an objective means of assess­ ing stool habits and the success of therapies. E) Enemas are first-line treatment for this condition as oral therapies result in unpredictable results.

Continuous urinary incontinence in girls should raise concern for an ectopic ureter.

Discussion 4-2

Question 4-3

The correct answer is "D:' Although many factors have been impli­ cated in the etiology of childhood constipation, many children with constipation have healthy, well-balanced diets that include fruits and vegetables. Limited diets have, however, been associated with an increased risk of developing constipation. Constipation has been associated with a decreased quality oflife. Most constipa­ tion in children is functional constipation, meaning that it is not associated with an underlying anatomic or physiologic disorder; most commonly, it is due to holding behaviors that allow increased reabsorption of water from stool in the colon. Although stool can be seen on abdominal films, X-rays have not been found to con­ sistently independently correlate with the objective or subjective

Parental counseling regarding ureteral ectopia should include which of the following statements? A) Boys and girls are at equivalent risk of urinary incontinence with ureteral ectopia. B) Ureteral ectopia is typically associated with dysplastic changes in the most caudal portion of the ipsilateral kidney. C) Excision of the renal parenchyma drained by the ectopic ureter is necessary to reduce the risk of pyelonephritis and future development of hypertension. D) Ureteral ectopia may be associated with an increased risk of epididymitis. E) Duplicated ureters occur in 25% of the population.

MCG RAW-H I LL E D U CATION S P E C I A LTY BOA R D REVI EW: P E D I ATRICS

384

Discussion 4-3

Discussion 5-1

The correct answer is "D:' An ectopic ureter does not enter the trigonal area of the bladder. When duplication is present, the upper pole ureter is ectopic. Ureteral duplication occurs in about 1 % of the population, and arises when the ureteric bud (a deriva­ tive of the wolffian system) divides. The more caudal and medial orifice drains the upper pole of the kidney, while the lower pole of the kidney is drained by the more cranial and lateral orifice. The upper pole (cephalad portion) may be dysplastic. In some cases, the upper pole ureter can move so caudally that it remains attached to other derivatives of the wolffian system draining into the vas deferens or epididymis. Boys do not present with incontinence like girls but rather manifest pain and infection of the affected organs (ie, epididymitis) In girls, the ectopic ureter may drain from the perineum into the vagina, uterus, or rec­ tum. In boys, ectopic ureters are not associated with continuous incontinence. Surgical management is required when ureteral ectopia is associated with infections, pain, or hypertension, with the first two reasons being much more common than the third. Management options include excision of the upper pole moiety or altered drainage of the upper pole via ureteral reimplantation or ureteroureterostomy.

The correct answer is "A:' Nocturnal enuresis, or bedwetting, is diagnosed when a child older than 5 years of age is incontinent of urine at night. Although the true prevalence of nocturnal enuresis may be underreported owing to factors such as embar­ rassment, between 5% and 10% of children have bedwetting at age 7. About 10% to 1 5 % of children experience resolution of their bedwetting every year without additional therapy. To keep it simple, think 8% of 8-year-olds.



Helpful Tip

:S.� When

treating d ayti me e n u resis, physical therapy

r1 1r to assist i n opti m izing pelvic floor fu nction may be helpfu l as a n adj u n ct to i m p rovement of voi d i n g a n d defecation ha bits. Biofeedback a n d physica l

therapy using visual cues to assist in isolating targeted muscle g ro u ps have been associated with higher i m p rovement rates.

You are seeing an 8-year-old-girl who has never been dry at night for more than 2 nights at a time. She goes to bed at 8 PM and wakes at 6 AM. She does not awaken during the night. She voids three to four times during the day and has a bowel movement every other day. The bedwetting is beginning to become bothersome as she does not want to take part in sleepovers because she is embarrassed. Question 5-1

Which of the following is true when counseling her parents? A) Approximately 10% of children have nocturnal enuresis at the age of 8. B) Approximately 20% of children have nocturnal enuresis at the age of 8. C) Approximately 30% of children have nocturnal enuresis at the age of 8. D) Approximately 40% of children have nocturnal enuresis at the age of 8. E) Approximately 50% of children have nocturnal enuresis at the age of 8.

On examination, the girl is found to have an unremarkable cardiopulmonary examination, a soft and nontender abdo­ men, and no weakness in the bilateral lower extremities. Her back is straight, with no tufts of hair, discoloration, or dimples. She has Tanner 1 female genitalia and her labia minora are connected by a thin vertical line of tissue blocking part of the vaginal opening. Question 5-2

Which of the following is true regarding the treatment of this condition? A) Hormone levels should be assessed to evaluate for a disorder of sexual differentiation. B) Topical estrogen cream (enough to cover the parent's index fingertip) should be applied twice daily. C) Formal labioplasty is needed. D) Topical anesthetic and incision may be performed in the clinic but is often unsuccessful. E) Treatment may be deferred if the patient is asymptomatic. Discussion 5-2

The correct answer is "E:' Labial adhesions occur in up to 2% to 3% of girls and should be treated if the child is symptomatic. Common associated symptoms include dysuria and daytime incontinence (often from urine pooling behind the adhesions) . If treatment is desired, topical estrogen cream is typically the first-line therapy, although combination creams containing estrogen and betamethasone have also been shown to be suc­ cessful. Steroid cream of any type should be applied with an applicator to avoid potential systemic administration to the administrator. In cases where conservative treatment has failed, lysis of adhesions in the office with topical anesthetic or under general anesthetic may be required; the decision to perform the procedure in a particular location is generally made based on the demeanor of the child, not the technical difficulty of the procedure. Although many girls experience resolution of the labial adhesions with increased estrogen levels at puberty, estro­ gen levels do not seem to differ between girls with and without labial adhesions. Though the precise etiology of labial adhesions remains unknown, local irritation is a likely factor, and avoiding perineal irritants and practicing good hygiene is recommended. (See Figure 1 8-6.)

Despite compliance with timed elimination, fluid restriction, and use of a bed alarm, the girl continues to have wetting every night. She and her parents have instituted a program

C H A PT E R

18



G E N ITA L SYSTEM D I SO R D E R S

385

Question 5-4

During counseling about DDAVP (desmopressin), they should be told: A) DDAVP should be given before bed, and the girl may con­ tinue to drink water until bedtime. B) The intranasal administration method is considered safer than the oral route. C) The girl should have an electrocardiogram (ECG) prior to initiation of therapy. D) Up to three pills (0.2 meg each) may be used to achieve dry­ ness, and instructions given to titrate the dose up every 2 to 3 nights until dryness is achieved. E) The girl may be more difficult to awaken in the morning as she will likely sleep more soundly. Discussion 5-4

F I G U R E 1 8-6. This you n g g i r l has labial adhesions. A t h i n vertical l i ne is visi ble i n the center where the la bia have fused together. (Reproduced with

permission from DeCherney AH, Nathan L, Laufer N, Roman AS, eds. Current

Diagnosis & Treatment: Obstetrics & Gynecology. 1 1 th ed. New York, NY:

McGraw- H i l l Education; 201 3, Fig. 37-1 6.)

where she is able to change her sheets herself at night, and her parents are not angry with her for the bedwetting. The girl is doing well in school and is growing appropriately. The bedwetting, however, is limiting her participation in activi­ ties such as sleepovers and scout camp. Her mother has asked if there is any other option to manage the nocturnal enuresis for "special occasions:' Question 5-3

Pharmaceutical therapy for nocturnal enuresis: A) Is useful for children with high nocturnal urine production secondary to diabetes or renal concentrating defects. B) Generally results in resolution of the enuresis more quickly than if pharmaceutical agents were not used. C) Will allow the child to sleep through the night. D) Is most useful in conjunction with increased fluid intake before bedtime. E) May be associated with significant side effects. Discussion 5-3

The correct answer is "E:' Further discussion follows after Question 5-4.

The girl's parents are trying to decide if pharmaceutical therapy is right for them.

The correct answer is "D:' Pharmaceutical agents used for the treat­ ment of enuresis include DDAVP ( desmopressin) and imipramine (a tricyclic antidepressant) . DDAVP works by decreasing water secretion in the kidney, while the mechanism of imipramine is unknown. In general, pharmacologic therapy of enuresis masks, but does not resolve, the enuresis. Children may sleep through the night if they do not need to awaken to change their clothes and sheets, but there is no evidence that either medication directly impacts sleep duration or quality. Administration of DDAVP should be oral (the intranasal formulation, which is no longer available in the United States, is associated with an increased risk of hyponatremia), and fluid intake should be limited to avoid hyponatremia and fluid overload. Up to three pills at a time may be used; titration is generally encouraged every 2 to 3 days to allow for nightly variations in fluid intake that may affect dryness. •

Helpful Tip

:5.� The DDAVP nasal

spray has a black box wa rning for

i1 1r hyponatremia. Pa rents m u st be ca refu l ly counseled on

the potential side effects of DDAVP a n d i m i pra m i ne,

a n d c h i l d ren ta ki ng these medications m u st be fol l owed cl osely. G iven the risk of QT prolongation with i m i p ra m i ne, a n ECG is recom mended before sta rting thera py.

A 26-year-old G 1 PO (now 1) mother gave birth to a boy (Apgar scores of 9 and 9 at 1 and 5 minutes, respectively) at 39-3/7 weeks' gestation. At the 20-week ultrasound, the fetus was noted to have left hydronephrosis with slight thinning of the parenchyma. The right kidney had no hydronephro­ sis and normal sonographic parenchymal echotexture. The bladder was visualized. The amnionic fluid level was normal. The pregnancy had otherwise been progressing well, without complications. During the pregnancy, the mother had no problems with blood pressure, and no significant swelling of the face or hands. The newborn has been feeding well and has had multiple wet diapers since delivery 16 hours earlier.

386

MCG RAW-H I LL E D U CATION S P E C I A LTY BOA R D REVI EW: P E D I ATRICS

Blunted calyces

Ureteropelvic junction obstruction

Hydroureter

(especially distally)

A peristaltic segment

F I G U R E 1 8-7. Congenital u reteral obstruction. Causes of prenatal hyd ronephrosis i n c l u d e u reteropelvic j u nction ( U PJ ) obstruction, u reterovesical j u nction (UVJ ) obstruction, vesicou reteral reflux, or posterior u rethral valves. The image at left shows rig ht-sided U PJ obstruction with hyd ronephrosis. The image at right shows left UVJ obstruction with hyd rou reteronephrosis. (Reproduced with permission from McAninch JW, Lue TF, eds. Smith & Tanagho's General Urology. 1 8th ed. New York, NY: McGraw- H i l l Education; 20 1 3, Fig. 37-9.)

Question 6-1 The first step in postnatal evaluation of the newborn is: A) Placement of a urethral catheter. B) Measurement of serum creatinine. C) Renal ultrasound. D) Voiding cystourethrogram (VCUG) . E ) MAG-3 renal scan. Discussion 6-1 The correct answer is "C:' A renal ultrasound will provide a non­ invasive assessment of the renal anatomy. As newborns may be relatively dehydrated in the early postnatal period, it is important to recognize that an ultrasound early in life (particularly on day of life 1 ) will often underestimate the degree of hydronephrosis. If there is sonographic suspicion for upper tract abnormalities (eg, persistent postnatal hydronephrosis, abnormal parenchymal echogenicity), then consideration of further radiographic studies is reasonable. A VCUG is performed by inserting a catheter into the bladder, filling the bladder with contrast, and then having the infant void. The VCUG will assess for abnormalities such as vesico­ ureteral reflux, bladder diverticula, incomplete bladder emptying, and outlet obstruction (eg, posterior urethral valves) . A MAG-3 scan will quantify the relative function of each renal unit and will also demonstrate any points of obstruction ( eg, ureteropelvic junc­ tion [UPJ] obstruction, ureterovesical junction [UVJ] obstruction) . Renal scans are generally deferred until 1 month of age, unless there is an extenuating circumstance such as suspicion for bilateral UPJ obstructions, because the neonatal kidney has relatively poor

concentrating ability. If the newborn is voiding spontaneously and there is no clinical suspicion for bladder outlet obstruction, then catheter placement is not required. Serum creatinine in the early postnatal period is more reflective of maternal rather than neonatal renal function, although in cases of severe renal dysfunction in an infant, it may be elevated. (See Figure 1 8-7.)

An 1 1 -year-old boy presents to the emergency department following a fall from an all-terrain vehicle (ATV) . Vital signs are temperature of 37°C (98.7°F), heart rate 92 beats per min­ ute (bpm) , respirations 12 breaths per minute, and blood pressure 1 26/75 mm Hg. He has bruising around the left flank and is very tender to palpation in this area. He is alert and oriented, has some pain on deep inspiration, and is mov­ ing all extremities. He is able to void and has grossly bloody urine. Computed tomography ( CT) scan of the abdomen and pelvis shows a left UPJ disruption. Question 7-1 The grade of the renal injury is: A) 1 . B) 2. C) 3 . D) 4 . E) 5.

CHAPTER 1 8

Discussion 7-1 The correct answer is "D:' Grade 1 renal injuries are subcapsular hematomas. Grade 2 renal injuries are lacerations into the super­ ficial renal parenchyma. Grade 3 injuries are lacerations into the deep renal parenchyma. Grade 4 renal injuries include damage to segmental vasculature as well as entry into the collecting sys­ tem. In this case, the UPJ is disrupted. Grade 5 renal injuries include main renal artery thrombosis and shattered kidney. Question 7-2 Management of this injury includes all of the following EXCEPT: A) Bedrest. B) Serial hematocrit measurement. C) Drainage of the collecting system. D) Repeat imaging of kidneys. E) Exploratory laparotomy. Discussion 7-2 The correct answer is "E:' Most renal injuries can be managed with bedrest and serial hematocrit measurement. Collecting system entry should be managed with internal (ureteral stent) or external (percutaneous nephrostomy tube) drainage. Although repeat imaging is not always performed for low-grade injuries, it is routine for high-grade injuries in order to ensure viability of the renal parenchyma and delineate the anatomy following an injury. Emergent surgical repair is performed only rarely, when bleeding from the kidney cannot be controlled or when the patient is undergoing surgical intervention for other injuries. In the latter case, the risks and benefits of surgical repair ver­ sus conservative management should be carefully considered, because bleeding can often be controlled by tamponade within the retroperitoneum, and release of a retroperitoneal hematoma (from the kidney or from an injury to the liver or spleen) may be difficult to control. Question 7-3 When counseling parents and patients, which of the follow­ ing is true regarding renal trauma? A) Children with a history of solitary kidney should not be allowed to take part in sports given the increased risk of renal injury in this population. B) Renal units with underlying structural abnormalities are at increased risk of damage with renal trauma. C) Hypertension may be seen for the first several years after renal injury and typically reflects damage to the large renal vessels. D) Children are at decreased risk of UPJ disruption and renal pedicle avulsion compared with adults. E) In children with a history of trauma, microscopic hematuria is an indication for cross-sectional imaging to exclude renal trauma as the cause. Discussion 7-3 The correct answer is "B:' Children with a solitary kidney or abnormal renal anatomy may be at increased risk of injury compared with the general population when participating in



G E N ITA L SYSTEM D I SO R D E R S

387

contact sports, but much of this risk can be controlled with use of patient education and appropriate padding. Patients with a history of renal trauma are at an increased risk for development of late hypertension, which is most commonly caused by local scarring causing compression of the renal unit with resulting increase in renin and angiotensin levels (Page kidney) . Children are actually at increased risk for avulsion injuries caused by sud­ den deceleration when compared with adults, perhaps owing to the relative decrease in adipose tissue surrounding the kidney in the pediatric population. Finally, although microscopic hema­ turia (> 5 RBCs/hpf) alone is an indication for cross-sectional imaging in adults, in children microscopic hematuria plus a plausible mechanism of injury is needed before cross-sectional imaging is undertaken.





Helpful Tip

Renal i nj u ries should a l ways be suspected i n the setting

1 1 1r of i nj u ries t o nea rby org a n s (spleen, l iver, spi ne), o r when the mecha n i s m of i nj u ry cou l d potentia l ly cause renal i nj u ry (eg, sudden deceleration). I n c h i l d ren, there is less peri neph ric fat to cushion the kid n ey, so they a re at increased risk for ren a l i nj u ries compared with a d u lts.

A 3-day-old boy was born at term after an uncomplicated pregnancy. His mother has brought him to the clinic because at his postnatal examination he was noted to have a descended left testicle and a nonpalpable right testicle. The mother does not recall being told anything about the genitalia being abnormal at birth, and also does not recall seeing the right testis in the scrotum during warm baths. On examination, the boy has Tanner 1 male external geni­ talia with a circumcised phallus and orthotopic meatus, and a palpably normal descended left testicle. The right testicle is nonpalpable. No hernia or hydrocele is palpable on either side. Question 8-1 Which of the following is the most appropriate course of action at this point? A) Observation. B) Ultrasound of the inguinal canal and scrotum. C) Intramuscular human chorionic gonadotropin (hCG) therapy. D) Referral to a pediatric urologist for exploration and possible orchidopexy. E) Intramuscular testosterone therapy. Discussion 8-1 The correct answer is ''A:' Approximately 2% of male infants are born with an undescended testicle, the vast majority of which descend by 3 months of age. Early in gestation the testes descend from the abdomen to the inguinal ring. Later, at 25 to

MCG RAW-H I LL E D U CATION S P E C I A LTY BOA R D REVI EW: P E D I ATRICS

388

30 weeks' gestation, the testes move through the inguinal canal to the scrotum. Although the exact factors governing descent are not known, abdominal pressure, gubernacular location, and hormone responsiveness appear to play a role (although to different degrees in different people). There is a postnatal testosterone surge at 2 to 3 months in term infants (occurring later in premature infants); therefore, observation to deter­ mine whether the testicle remains undescended after this time period is warranted. Hormone therapy (hCG and testosterone) has not been shown to be effective in promoting testicular descent. Inguinal and scrotal ultrasounds have not been shown to be cost-effective in identifying the presence and location of a nonpalpable gonad. An undescended testis is a risk factor for torsion. In contrast, retractile testes are those that can be brought down to the scrotum and stay in place following crem­ asteric fatigue, but which may have a strong cremasteric reflex and thus may reside higher in the scrotum, or on occasion, in the inguinal canal much of the time. Approximately 30% of retractile testes may ascend over time; thus, annual examina­ tions should be performed by a skilled provider. Care should be taken to relax the child and to provide adequate time for the cremasteric reflex to fatigue.

undescended) , with the risk increasing with more proximal testicular location and later orchidopexy. The most recent studies have recommended orchidopexy earlier than 1 year of age to achieve maximal fertility benefit. It is recommended that boys with a solitary functioning testicle wear a cup for contact sports. Testicular atrophy has been estimated to occur in 20% to 30% of orchidopexies for intra-abdominal testes, and an inguinal approach for staged procedures has been asso­ ciated with a lower risk of atrophy.





Helpful Tip

Routi ne sonography to locate a n u n d escended testicle

1 11r does not change s u rg ica l management in most cases, a n d is not cost effective.



Helpful Tip

:S.� Newborns

r1 1r but

with

male-looki ng

externa l

gen ita lia

bi latera l nonpalpable testes should u n dergo

eva l uation for d i sorders of sexual d ifferentiation, as a

fem a l e i nfa nt with severe CAH ca n be morpholog ica lly identical t o a m a l e i nfa nt wi t h bilatera l n o npalpable





Helpful Tip

gonads.

C rypto rc h i d i s m i s a s sociated w i t h prematu rity a n d

1 11 r low b i rthweig ht. The testes m ove i nto t h e scrot u m

t h ro u g h t h e i n g u i n a l ca n a l at 2 5 to 3 0 weeks' gestation.

Question 8-2 When counseling parents regarding the need to consider sur­ gical intervention for an undescended testicle, which of the following is true? A) Surgery should be performed no sooner than 6 months cor­ rected age given anesthetic considerations. B) Fertility parameters may be abnormal with undescended testes, but typically normalize provided that surgery is per­ formed before the age of 2 years. C) Undescended testes are at an increased risk for development of neoplasia, but this risk normalizes when orchidopexy is performed before the age of 1 year. D) Boys who have undergone orchidopexy should wear a cup when playing contact sports. E) Testicular atrophy occurs in up to one third of boys under­ going open inguinal orchidopexy. Discussion 8-2 The correct answer is ''A:' Many testes that are undescended at birth descend by 2 to 3 months of age, in conjunction with the postnatal testosterone surge. In boys with testes that remain undescended, surgery is planned after the age of 6 months given anesthetic considerations (although more recent litera­ ture suggests that elective surgery as early as 4 months of age is safe) . There is an increase in the risk of neoplasia in both testes in the setting of cryptorchidism (even if only one is

A 7-year-old boy presents with a 3-hour history of right testic­ ular pain arising acutely without associated history of trauma. He has associated nausea and vomiting. On examination, the right hemiscrotum is swollen, tender, and erythematous. The testis is very tender to palpation and is higher than the left. You suspect testicular torsion. Question 9- 1 The next appropriate step is: A) Emergent scrotal exploration for testicular detorsion. B) Cord block and manual detorsion in the emergency department. C) Scrotal ultrasound with Doppler. D) Measurement of serum tumor markers (AFP, beta-hCG, LDH). E) Administration of ketorolac for pain relief. Discussion 9-1 The correct answer is ''A:' Testicular torsion often presents as abrupt onset of testicular or scrotal pain, but patients may also have referred abdominal pain and systemic symptoms such as nausea and vomiting (the embryologic origin of the testes is in close proximity to the kidneys, along the gonadal ridge) . The exact symptoms are dependent on the severity of the torsion, the duration of the torsion, and the patient's pain threshold, and so providers must have a high index of suspicion. Testicular torsion has a bimodal distribution, with infants and teenagers most commonly affected, although

CHAPTER 1 8

torsion has been reported in all age groups, including adults. Teenagers are most commonly affected. The increasing weight of the testis during puberty is thought to play a role. The testis twists around the spermatic cord, interrupting blood flow. If testicular torsion is suspected, maneuvers to institute detor­ sion should be promptly instituted. Surgical exploration is the standard of care for testicular torsion, as it allows visual confirmation of both the viability of the testis and the success of detorsion maneuvers. When surgical exploration cannot be performed promptly, manual detorsion may be attempted; the testicle is gently rotated outward as if one were opening a book. In many cases, the testicle has twisted more than one full rotation, and so manual detorsion should be considered an adjunct to, rather than a substitute for, operative intervention. Pain medication, including a cord block or narcotic medica­ tions, may be helpful in relaxing the cord and limiting pain so that this can be accomplished. Ketorolac and other anti­ inflammatories that inhibit platelet function are not recom­ mended in the preoperative patient, given the increased risk of bleeding. Scrotal ultrasound with Doppler should be used to confirm the presence of blood flow in patients in whom the examination is equivocal, and in no case should delay surgi­ cal intervention. Testicular torsion is generally not associated with testicular masses, and so measurement of tumor markers before surgery is unnecessary unless other signs of a testicu­ lar mass are present. Torsion of the appendix testis (a rem­ nant of the miillerian system) or the appendix epididymis (a remnant of the wolffian system) typically manifests with testicular and scrotal pain and swelling, and a blue dot (the infarcted appendage) may be seen through the scrotal skin. Systemic symptoms are much more rare with appendix tor­ sion. Appendix torsion can be managed conservatively, with anti-inflammatory medications, ice, and scrotal support. (See Figures 1 8-8 and 1 8-9.)

F I G U R E 1 8-8. Testicular Tors ion. The boy h a s torsion o f h i s left testicle. The hemiscrot u m is swol len, red, and tender. O n exa m, the testis wou ld be e l evated i n the scrot u m a n d te n d e r, a n d the c re m a ster refl ex wo u l d n o t be present. ( R e p rod u ced w i t h p e r m i s s i o n fro m Knoop KJ, Sta c k LB, Sto rrow AB, Th urman RJ (Eds). The Atlas of Emergency Medicine, 3ed. McGraw­ H i l l Education, I nc., 201 0. Photo contri butor: Patrick McKenna, M D.)



G E N ITA L SYSTEM D I SO R D E R S

389

FIGURE 1 8-9. Blue Dot Sign. I n torsion of a testicular appendage, the infa rcted a ppendage may be seen under the scrota l skin, a ppea ring as a b l u e d o t . (Reproduced w i t h permission fro m K n o o p K J , Stack L B , Storrow AB, Th urman RJ (Eds). The Atlas of Emergency Medicine, 3ed. McGraw- H i l l Education, I nc., 201 0. Photo contributor: Javier A. Gonza l ez d e l Rey, M D.)



Helpful Tip

� The cremaster refl ex (el evation of the testis when 111r the

i n ner t h i g h is touched) ca n h e l p d ifferentiate

between torsion of the testis and torsion of a testicu l a r

a ppendage. It is a bsent i n the former but present i n t h e latter condition.

Question 9-2 Which of the following is true of neonatal testicular torsion? A) Risk factors include large birthweight infants and those who have been delivered after their due dates. B) Most torsions occur after birth rather than in utero. C) Operative salvage rates are approximately 50%. D) The testicle is typically twisted externally and intravaginally. E) Most cases of neonatal torsion are associated with testicular tumors. Discussion 9-2 The correct answer is "A:' Neonatal torsion affects approximately 2.5% of newborns. Risk factors include infants who are large for gestational age, those delivered after their estimated due date or after a prolonged labor, and those born to mothers with a history of diabetes. Tumors are not typically seen in association with neonatal torsion. Although the exact etiology is unknown, torsion is thought to arise when the testicle twists as a result of comparatively poor fixation of neonatal tissues. In compari­ son to testicular torsion in older children, neonatal torsion is typically extravaginal (outside the tunica vaginalis), rather than intravaginal. It is suspected that most "neonatal" torsions actu­ ally occur antenatally. Considerable debate remains concerning the best treatment for neonatal torsion; owing to the low opera­ tive salvage rate and the risks of anesthesia in the neonate, some urologists advocate conservative management. However, in the first 30 days of life, an increased risk of contralateral torsion has

MCG RAW-H I LL E D U CATION S P E C I A LTY BOA R D REVI EW: P E D I ATRICS

390

been observed; thus others have endorsed operative interven­ tion for fixation of the contralateral testis.



Helpful Tip

=.� I ntrava g i n a l

(inside the tunica vag i n a l is) testicular

i11r torsion is a ssociated with the "be l l cla pper" deformity,

which occu rs when the testis is n ot attached norm a l l y i n the scrot u m . I n stead, the testis hangs i n the scrotum i n a horizonta l position a n d is more mobile. The d eform ity is typica l ly bi latera l; therefore, both testes should be corrected d u ri n g su rg ery for torsion.

The patient's 9-year-old cousin had presented to his pediatri­ cian recently with a 1 -day history of scrotal pain and swelling, as well as low-grade fever. On examination, the scrotum was swollen and erythematous, particularly over the left side, the testis was nontender, and a tender mass was felt over the upper pole of the testis. Question 9-3 The structure most likely responsible for this presentation is derived from the same embryologic origin as the: A) Pyramidalis. B) Lower third of the vagina. C) Uterus. D) Ureter. E) Parasympathetic nerves. Discussion 9-3 The correct answer is "C:' In a child of this age, the most likely cause of this presentation is torsion of the appendix testis, which is a remnant of the miillerian system. The miillerian system also gives rise to the uterus and upper third of the vagina. The lower third of the vagina develops from the genitourinary sinus. The ureters are derivatives of the wolffian system. Nervous tissues are not involved in this presentation. (See Figure 18-10.) Question 9-4 What is the most appropriate management? A) Emergent surgical exploration. B) Anti-inflammatory medication. C) Ciprofloxacin. D) Trimethoprim -sulfamethoxazole. E) Scrotal ultrasound with Doppler. Discussion 9-4 The correct answer is "B:' Torsion of the appendix testis is man­ aged conservatively, with ice, scrotal support, and oral anti­ inflammatory agents. Antibiotics are not indicated (and in any event, ciprofloxacin is not recommended for children given the risk of tendon rupture) . Although scrotal ultrasound with Doppler may be used to demonstrate blood flow to the testis, clinical examination is generally adequate for the diagnosis of both testicular torsion and torsion of the appendix testis. In both conditions, the scrotum may be erythematous, swollen,

F I G U R E 1 8- 1 0. Formation of the Male and Female I nternal Genita l i a . A fetu s is b o r n w i t h the e m b ryologic precu rsors for both genders' i nternal genita l i a . I n males, the wolffian d u cts form the epid idym i d i s, v a s deferens, sem i n a l vesicles, and ejaculatory d u cts. T h e testis secretes mu l l erian-i n h i biting su bsta nce causing the mu l lerian d u cts to regress. I n females, the mullerian ducts form the female i nternal genita l i a : fa l lopian tu bes, uterus, cervix, and upper part of the vag i n a . Without testosterone, the wolffian ducts do not d iffe rentiate. (Reproduced with permission from M cA n i n c h JW, Lue TF, eds. Smith & Tanagho's General Urology. 1 8th ed. New Yo rk, NY: McGraw- H i l l Education; 201 3, F i g . 43-2.)

and tender. In testicular torsion, the cremaster reflex is absent and the testis is tender, elevated, and may be horizontal in the scrotum. In appendix testis or epididymis torsion, the cremaster reflex is intact, a tender mass on the upper pole of the testis is palpable, and a blue dot may be seen on the scrotum.

The clinic seems to be besieged by males with scrotal pain. Today a I S-year-old presents with a 4-day history of aching in his scrotum, fever, and burning with peeing. He has a new sexual partner and does not use condoms. On exam, his testis exam is normal but he has tenderness posteriorly. His urinal­ ysis shows pyuria. Question 1 0-1 Which of the following is the most likely cause of his symptoms? A) Chlamydia trachomatis. B) Adenovirus.

CHAPTER 1 8

C) Escherichia coli. D) Neisseria gonorrhoea. E) Mumps. Discussion 1 0-1 The correct answer is "A:' The epididymis serves as a sort of sperm storage shed located on the posterior side of the testis. This adolescent boy has bacterial epididymitis. In a sexually active male, the most common cause is chlamydia. In addi­ tion to the organisms listed, other possible causes include Mycoplasma pneumoniae, adenovirus, and enterovirus. Pain may be acute or subacute, with possible urinary symptoms and or fever. On exam, tenderness is localized to the epididy­ mis, the cremaster reflex is present, and elevation of the testis may relieve the pain (Prehn sign). The Prehn sign is absent in torsion but it is not sufficiently reliable to differentiate between epididymitis and torsion. If in doubt, order a Doppler ultra­ sound. Management includes obtaining cultures, treating the infection, and reminding the boy to use condoms. Reflux of sterile urine or ectopic insertion of a ureter resulting in non­ infectious inflammation is the most common cause of epi­ didymitis in younger children. Mumps is the most common cause of orchitis. Orchitis is most commonly caused by a viral infection but may be bacterial. If epididymitis is present, the etiology is more likely bacterial.



torsion,

epid idym itis,

testicu l a r

appendage

Henoch-Schon l e i n

purpura,

torsion, orchitis,

tra u ma, a n d i n ca rcerated hernia.



I

391

Question 1 1 -1 What is the most likely cause of her pain? A) Ovarian torsion. B) Tuboovarian abscess. C) Ectopic pregnancy. D) Ruptured ovarian cyst. E) Hematocolpos. Discussion 1 1 -1 The correct answer is "A:' In torsion, the adnexa twist, cut­ ting off the blood supply to the ovary and fallopian tube. An ovarian cyst may cause torsion. Early recognition and surgery is important to save the ovary. Tuboovarian abscess, ectopic pregnancy, and ruptured ovarian cyst should be ruled out. A pelvic ultrasound with Doppler is the imaging study of choice. Physiologic ovarian cysts are common in fetuses and neonates as a result of maternal hormones and in adolescent pubertal females as a result of failure of the follicle to ovulate. Rup­ ture causes acute onset of pain and bleeding. An imperforate hymen blocks the drainage of menstrual products, resulting in the accumulation of blood and fluid in the uterus called hematocolpos. A bulging bluish hymen is present on exam. Girls present with amenorrhea and cyclical abdominal or pel­ vic pain. The contents can be secondarily infected (pyometra) . Our patient's presentation i s acute, and she told you her period was 1 week ago.

d ifferential diag nosis of scrota l pa i n i n cl udes

r1 1r testicular



G E N ITA L SYSTEM D I SO R D E R S

Helpful Tip

� The

=-



Helpful Tip

A 4-year-old girl presents with a 1 -week history of vaginal itching and redness. The child is potty trained and wears cotton underwear. She loves bubble baths and dance. She has had no fever, abdominal pain, urinary symptoms, or bleeding. On exam, she has normal external genitalia with erythema and irritation of the labial majora.

M a n y c h i l d re n w i t h testicu l a r t o r s i o n retrospectively

1 1r reca l l

prior episodes of s i m i l a r pa i n , typica l l y of

s h o rter

d u ration,

that

reso lved

sponta n eously

("interm ittent torsion"); t h u s, c h i l d ren with repeated e p i sodes of severe but self-l i m ited pain without other a p parent etiolog ies may ben efit fro m eva l uation by a u ro l o g i st.

A 1 4-year-old girl presents with acute onset of severe abdominal pain localized to the left lower quadrant. She feels sick to her stomach and has vomited several times. She denies trauma, fever, or vaginal discharge. Her last menstrual period was 1 week ago. On exam, she is very uncomfortable and cannot lie still. Her external genitalia are normal, no cervical motion tenderness is present, her abdomen is diffusely tender, and a mass is palpable in her left adnexa.

Question 1 2-1 It is important to tell parents or caregivers: A) Her dance leotards should be tight fitting. B) Bubble baths are fine, especially with scented soap. C) It doesn't matter which way she wipes. D) She should wear cotton underwear. E) Keeping her vaginal area moist will help. Discussion 1 2-1 The correct answer is "D:' Vulvovaginitis in the prepuber­ tal child is due to local irritation and associated with poor hygiene, bubble baths, scented soaps, obesity, tight-fitting clothes, and foreign bodies. Look for retained toilet paper. Patients should wear cotton underwear, avoid tight-fitting clothes, keep the area dry, wipe front to back, minimize mois­ ture, and avoid bubble baths or scented bath products, Warm soaks in clean bath water may help but it is important to dry the genital area afterward before putting on underwear. If symptoms do not improve, consider a foreign body or infec­ tious cause.

392

MCG RAW-H I LL E D U CATION S P E C I A LTY BOA R D REVI EW: P E D I ATRICS

Question 1 2-2

Which is NOT an infectious cause of vulvovaginitis? A) Streptococcal pyogenes. B) Candida vagina/is. C) Pinworms. D) Yersinia. E) All of the above. Discussion 1 2-2 The correct answer is "E:' If symptoms are persistent or discharge is purulent, cultures should be obtained. Candida is associated with antibiotics, immunosuppression, and wet diapers. Abuse should be suspected if vulvovaginitis is caused by a sexually transmitted pathogen in young girls.

A male fetus was noted at the 20-week ultrasound to have bilateral hydronephrosis and a thickened bladder wall. He was delivered at 32 weeks' gestational age weighing 2690 g. The delivery was uncomplicated, but shortly after birth the newborn developed respiratory distress and was intubated. Examination shows a palpable bladder and normal Tanner 1 male external genitalia with bilateral descended testes. Chest X-ray shows a right-sided pneumothorax. After stabilization a voiding cystourethrogram is performed and shows a dilated posterior urethra and a thickened bladder. Question 1 3-1

Initial management of this condition is which of the following? A) Urethral catheter placement. B) Cystoscopy. C) Postnatal renal ultrasound. D) Measurement of serum electrolytes and creatinine. E) Vesicostomy. Discussion 1 3-1 The correct answer is "A:' Posterior urethral valves affect male infants, causing bladder outlet obstruction. (See Figure 18-11.) Typically newborns present with delayed voiding, but some void normally. Although serum creatinine and electrolytes should be assessed, and a renal ultrasound should be obtained, the most urgent matter is obtaining adequate drainage of the lower urinary tract, which is blocked by the posterior urethral valves. Subse­ quently pressure increases in the bladder then the upper urinary tract, resulting in development of hydroureteronephrosis. The renal parenchyma may be irreversibly damaged from the high pressure in the upper tract. If obstruction is severe enough to cause oligohydramnios, respiratory distress and pneumothora­ ces may develop from lung hypoplasia. Remember, voided urine enters the amnionic fluid. Normal amnionic fluid is required for lung development. Prior to undergoing surgery to ablate the valves, pulmonary function should be optimized since ven­ tilators utilize positive-pressure ventilation (which can worsen pneumothoraces), and serum electrolytes should be normalized.



Helpful Tip

:5.� When

eva l uating a newborn for hyd ronephrosis or

i1 1 r other u ri n a ry tract pathology, ren a l u ltrasou nd may

be fa lsely negative if obta i ned on the fi rst few days of l ife before u ri n e prod uction has increased. It is okay

to order but keep i n m i n d you s h o u l d repeat it at 1 to 2 weeks of age.

Question 1 3-2

In this case, optimal initial surgical management of the posterior urethral valves is best achieved by: A) Prolonged catheter drainage. B) Transurethral ablation of the valves. C) Vesicostomy. D) Bilateral ureterostomy. E) Bilateral pyelostomy. Discussion 1 3-2 The correct answer is "B:' While initial catheter drainage should be performed, prolonged catheter drainage may increase the risk of urethral pressure necrosis and UTis. Transurethral resec­ tion of the valves is an option in children in whom the urethra is large enough to accommodate the scope. In most very small or premature infants, the initial procedure is a vesicostomy since transurethral resection cannot be safely performed. In infants and children with tortuous ureters in whom an adequate decline in serum creatinine cannot be achieved with vesicostomy alone, ureterostomy or pyelostomy (bringing the ureter and renal pelvis to the skin, respectively) , or both, may be needed to opti­ mize drainage.



Helpful Tip

� Posterior u reth ra l va lves may ma n ifest i n older m a l e 1 1 1r c h i l d ren a s problems with daytime continence o r with bedwetting, although this presentation is less common

with the increased use of prenata l i m a g i n g .

A 2-year-old girl is brought into clinic today because her mother felt an abdominal mass while she was giving the child a bath. The girl is otherwise healthy and was deliv­ ered at term after an uncomplicated pregnancy. Her family history is significant for two cousins with Wilms tumor. A renal ultrasound shows a large echogenic renal mass on the right. Question 1 4-1 All of the following should be included in the evaluation of this mass EXCEPT: A) Mesoblastic nephroma. B) Renal cell carcinoma. C) Wilms tumor.

CHAPTER 1 8



G E N ITA L SYSTEM D I SO R D E R S

393

F I G U R E 1 8- 1 1 . The poste rior u reth ra is e n l a rged on voi d i n g cystou reterogram i n this boy with posterior u rethral va lves. (Reproduced with permission fro m McAn i n c h JW, Lue TF, eds. Smith & Tanagho's General Urology. 1 8th ed. New York, NY: McGraw- H i l l Education; 201 3, Fig. 4 1 - 1 .)

D) Neuroblastoma. E) Hypertrophied column of Bertin. Discussion 1 4-1 The correct answer is "E:' A hypertrophied column of Bertin is a column of renal cortical tissue that extends to the renal sinus; since it is normal renal parenchyma, it has normal uptake and excretion of contrast compared with the remainder of the kid­ ney. Mesoblastic nephroma, renal cell carcinoma, Wilms tumor, and neuroblastoma are all abnormal findings. Neuroblastoma may develop from any neural tissue, including the adrenal gland, whereas the other masses are seen in the kidney. Mesoblas­ tic nephroma is most commonly seen in newborns and young infants, Wilms tumors in toddlers, and renal cell carcinoma (typically translocation-type) in older children. However, these age ranges are not strict, and so physicians should be aware of the differential diagnosis for all renal masses. (See Figure 18-12.)

F I G U R E 1 8- 1 2. This 3-yea r-ol d has a Wi l m s tumor of the right kid ney (white arrow), a m a l i g nant tumor that presents as an asym ptomatic abdo m i n a l

m a s s i n todd lers. (Reproduced w i t h permission from Brunica rd i F C , Andersen DK, B i l l i a r TR, et al, eds. Schwartz's Principles of Surgery. 1 Oth ed. New York, NY: McGraw- H i l l Education; 201 5, Fig. 39-37.)

394

MCG RAW-H I LL E D U CATION S P E C I A LTY BOA R D REVI EW: P E D I ATRICS

This toddler has a Wilms tumor, a malignant tumor of the kidney. Wilms tumors are the most common cause of a renal mass. It typically presents as an asymptomatic abdominal mass. Hematuria, hypertension, or both may be present. Question 1 4-2 The chromosome on which the gene responsible for Wilms tumor resides is: A) lp. B) 3p. C) l lp. D) 1 7p. E) 22q. Discussion 1 4-2 The correct answer is "C:' The WTl gene is located on chromo­ some l lp l 3 - 1 5. Question 1 4-3

Syndromes associated with Wilms tumor include all of the following physical examination findings EXCEPT: A) Cafe-au-lait spots. B) Macroglossia. C) Hypospadias and undescended testes. D) Aniridia. E) Hemihypertrophy. Discussion 1 4-3 The correct answer is ''A:' Cafe-au-lait spots are associated with neurofibromatosis. The most common syndromes asso­ ciated with Wilms tumor are Beckwith-Wiedemann (macro­ glossia), WAGR (Wilms tumors, Aniridia, Genital anomalies, mental Retardation), and Denys-Drash. Associated isolated anomalies include aniridia, hypospadias, hemihypertrophy, and cryptorchidism.

The tumor is excised and the pathology returns as 475 g, favorable histology, negative margins. There was no intraop­ erative tumor spill. The girl does well and is discharged from the hospital. Question 1 4-4

The most appropriate further treatment is: A) Surveillance with periodic cross-sectional imaging. B) Surveillance with renal ultrasound. C) Chemotherapy with vincristine, doxorubicin, and cyclo­ phosphamide. D) Flank radiotherapy. E) Total abdominal radiotherapy. Discussion 1 4-4 The correct answer is ''A:' In the United States, Wilms tumors are generally treated according to the Children's Oncology Group protocols. For children 2 years of age and younger, with favorable histology tumors, negative margins without spill, and

tumors weighing less than 550 g, additional therapy is not rec­ ommended. These children are followed on the surveillance protocol. Children with favorable histology tumors receive chemotherapy with vincristine and dactinomycin or with vin­ cristine, doxorubicin, and dactinomycin. Cyclophosphamide is not used as an initial chemotherapeutic agent in Wilms tumor. Radiation therapy is reserved for children with anaplastic his­ tology, positive margins, extensive intraperitoneal disease, or intraoperative tumor spill.

An 1 1 -year-old girl presents to the emergency department complaining of a 3-day history of increasing left flank pain radiating to the left labia majora. She had one episode of emesis and one episode of pink urine. She has had no fevers, and her urine does not have a foul odor. She is oth­ erwise healthy. She has a family history of kidney stones in multiple relatives. On examination, she is afebrile, alert and interactive, in obvious pain, and has left flank and abdominal tenderness to palpation. She has no rebound tenderness or guarding on abdominal exam. Urinaly­ sis shows 3 to 5 WBCs/hpf, 1 1 to 20 RBCs/hpf, negative nitrites, and no bacteria. Complete blood count shows a WBC count of 10.2 K/mm3, hematocrit 38.4%, and plate­ let count 375 K/mm3• The basic metabolic panel shows sodium of 1 37 mEq/L, potassium 4.6 mEq/L, chloride 1 1 2 mEq/L, bicarbonate 26 mEq/L, BUN 13 mg/dL, and creatinine 0.5 mg/dL. Question 1 5-1 The initial imaging test should be: A) Renal ultrasound. B) Intravenous pyelogram (IVP). C) Computed tomography (CT) scan of the abdomen and pelvis with oral and intravenous contrast. D) Magnetic resonance imaging (MRI) of the abdomen and pelvis with gadolinium. E) KUB (kidney, ureter, bladder) X-ray. Discussion 1 5-1 The correct answer is ''A:' Renal ultrasound has been shown to accurately diagnose renal stones but not ureteral stones in at least 90% of children when employed as the initial imaging modality. This reduces radiation exposure, which is especially important for children who may develop multiple stones. The AAP's "Image Gently" campaign urges clinicians to consider whether a test with ionizing radiation is necessary, or whether the same clinical information can be gleaned from a test with­ out ionizing radiation. The most sensitive imaging study is a noncontrast helical CT, which can show ureter, radiolucent, and small stones that may be missed with other imaging modalities. KUB is useful to demonstrate radiopaque stones, but these may be obscured by overlying bowel gas. IVP and CT scan with contrast can demonstrate delayed drainage from

CHAPTER 1 8



G E N ITA L SYSTEM D I SO R D E R S

395

Discussion 1 5-3 The correct answer is "E:' Management includes pain con­ trol, hydration, and antibiotics if a concurrent urinary tract infection is present. Tamsulosin (Flomax) is a selective alpha- 1 blocker used to help with stone passage; these recep­ tors are located in the distal ureter and the bladder trigone as well as in the prostate in males. In adults, medical expul­ sive therapy has been associated with decreased cost and increased rate of spontaneous stone passage. There is limited experience with tamsulosin in children; early reports sug­ gest that it may be successfully employed to encourage stone passage in the pediatric population as well, but the benefit has not been fully established. Indications for a ureteral stent include suggestion of infection behind an obstructing stone, acute kidney inj ury, obstruction of all functional renal units, immunocompromise ( eg, chemotherapy, diabetes mellitus, immunosuppressive medications), or failure to achieve ade­ quate pain control.



F I G U R E 1 8- 1 3. U reter Stone. A 6-m m stone (red arrow) is present in the

prox i m a l left u reter on noncontrast helical CT. (Reproduced with permission from Tinti n a l l i J E, Stapczynski JS, M a OJ, Yealy DM, Cline DM, Meckler G D, eds. Tintinalli's Emergency Medicine: A Comprehensive Study Guide. 8th ed. New York, NY: McGraw-H i l l Education; 201 6, Fig. 94- 1 .)

Helpful Tip

� Kid n ey

=.

stones s h o u l d be col l ected a n d a n a lyzed to

r1 1r help identify a n u nd erlyi ng ca use a n d prevent future stones.

On hospital day 2, she passes the stone. She is pain free and has no complaints. a stone, but the contrast may obscure smaller stones. MRI is not a useful modality to demonstrate renal and ureteral stones. (See Figure 1 8 - 1 3 . ) Question 1 5-2 Which is NOT a presenting sign of nephrolithiasis in children? A) Microscopic hematuria. B) Asymptomatic. C) Irritability. D) Abdominal pain. E) All of the above. Discussion 1 5-2 The correct answer is "E:' Microscopic hematuria is present in nearly all cases of nephrolithiasis. Gross hematuria, while important to ask about, is not always present. Some patients may have a concurrent urinary tract infection.

Ultrasound reveals the girl has a left-sided renal stone. Question 1 5-3

Further management in the emergency department should include: A) Tamsulosin. B) Trimethoprim -sulfamethoxazole. C) Nephrology consultation for acute kidney injury. D) Urology consultation for ureteral stent. E) Admission for pain control.

Question 1 5-4 The stone composition is most likely: A) Calcium oxalate dihydrate. B) Calcium phosphate. C) Uric acid. D) Ammonium acid urate. E) Struvite. Discussion 1 5-4 The correct answer is "A:' Calcium oxalate (monohydrate or dihydrate) stones are most common; calcium phosphate stones are seen less commonly. Uric acid stones typically arise in patients with conditions associated with an increase in purine byproducts (eg, Lesch-Nyhan syndrome, gout) . Ammonium acid urate stones are associated with diarrhea and laxative use. Struvite stones arise in the presence of infected urine and are often seen in patients with chronic UTis from urea-splitting bacteria (eg, Proteus, Klebsiella) . Large struvite stones that fill the renal pelvis and calyces are called staghorn calculi. A diag­ nostic workup is important to identify underlying metabolic conditions (eg, hypercalciuria) predisposing to stone reoccur­ rence. Testing should include stone analysis, 24-hour urine collection, urinalysis with culture, basic metabolic panel, and phosphate, magnesium, and uric acid levels. Check parathyroid hormone and vitamin D levels if the calcium or phosphate levels are elevated.

She is asked to complete a 24-hour urine collection.

MCG RAW-H I LL E D U CATION S P E C I A LTY BOA R D REVI EW: P E D I ATRICS

396

Question 1 5-5

The abnormality most likely to be seen on this test is: A) Hypercalciuria. B) Hypomagnesemia. C) Low total urine volume. D) Hypercitraturia. E) Elevated urine pH. Discussion 1 5-5 The correct answer is "C:' The most common abnormality seen on 24-hour urine collection performed for urolithiasis is decreased total urine volume. Low urine volume, high urine solute levels (eg, calcium, oxalate, uric acid), and decreased urine inhibitors are risk factors for stone formation. Although abnormalities of calcium metabolism (eg, hyperparathyroid­ ism) may be associated with stone formation, hypercalciuria is uncommon in the absence of these conditions. Both mag­ nesium and citrate are considered inhibitors of lithogenesis; thus, decreased urinary concentrations of these ions may be observed. Hypomagnesemia would not be detectable on a urine test. An alkaline urine pH is associated with the development of calcium phosphate stones or struvite stones. Question 1 5-6

Dietary recommendations to prevent stone recurrence in this patient should include all of the following EXCEPT: A) Limited sodium intake. B) Limited calcium intake. C) Low oxalate diet. D) Increased fluid intake. E) Lemonade.

A 1 7-year-old boy presents for evaluation of a "lump" in the right testicle that he noticed in the shower 2 days ago. It has not changed in size since then and is not painful. He has had no abdominal pain, no cough, no recent illnesses, and states that he is not sexually active. On examination, the left testicle is descended and palpably normal, but on the right side, there is a firm area at the superior pole of the testis. A scrotal ultrasound is obtained showing a testicular mass. (See Figure 18-14.) Question 1 6-1 Which of the following is the LEAST appropriate next step in management? A) Repeat scrotal ultrasound in 2 weeks. B) Check serum levels of alpha-fetoprotein, beta-human cho­ rionic gonadotropin (beta-hCG), and lactate dehydrogenase (LDH). C) Discuss fertility concerns, including sperm banking. D) Obtain a chest X-ray. E) Referral to a urologist. Discussion 1 6-1 The correct answer is "A:' A solid testicular mass is an indication for exploration and surgical resection. Testicular cancer is the

Discussion 1 5-6 The correct answer is "B:' Inhibitors of stone formation include magnesium and citrate, while increased urinary oxalate and sodium have been shown to predispose to stone formation. In many calcium stones, increased urinary sodium serves as the nidus for crystallization; decreased sodium intake has been associated with a lower rate of stone recurrence, while decreasing calcium levels has not. A low­ oxalate diet, while challenging to follow, can be effective in decreasing the urinary oxalate, although the maj ority of urinary oxalate is endogenously produced. Increased fluid intake increases urine output and not only provides more solvent in which solute might dissolve, but also reduces urinary stasis. All children with stones should increase their daily fluid intake. Lemonade is an excellent source of citrate in the form of citric acid.



Helpful Tip

=� U ro l ithiasis

is beco m i n g i ncrea s i n g ly common i n

i1 1 r c h i l d ren, although the exact etiologic factors have not been wel l descri bed . C h i l d ren may pass l a rger stones

more ea sily than a d u lts beca use their u reters are more d i stensible.

F I G U R E 1 8- 1 4. Testicu l a r Ca ncer. This patient has a l a rge mass in h i s left hem iscrotum. Despite often being slow-g rowi ng tumors, testicu l a r cancer often presents q u ite late. (Reproduced with permission from Brunicardi FC, Andersen DK, B i l l i a r TR, et al, eds. Schwartz's Principles of Surgery. 1 Oth ed.

New York, NY: McGraw-H i l l Education; 201 5, Fig. 40- 1 .)

CHAPTER 18

most common solid neoplasm in older adolescent boys and young men. Workup includes measurement of tumor markers (alpha­ fetoprotein, LDH, and beta-hCG) . Once a neoplasm is confirmed, further therapy is dictated by staging of the tumor, including sur­ gical pathology and the results of a metastatic workup.





I

l lr

Helpful Tip



Not a l l solid masse a re testi c u l a � neoplasms. Testicu l a r hamartomas, testicu l a r m 1. c rohths, a n d epidermoid cysts have a l so been described. Pa rtial orchiectomy may be appropriate for ben i g n processes. In younger c h i l d ren, yol k-sac tumors a re the most common testicular neoplasm, but teratomas may a l so occ u r. Leyd ig cel l tumors, which a re hormona l l y active, may present i n patients with precocious pu berty, i n c l u d i n g increased m u s c l e m a s s a n d a d eepened voice.

A 10-month-old infant boy is brought for evaluation by his mother after she noticed a bulge in his groin. She tells you that he was crying at the time, and when he stopped crying, the bulge was no longer there. He has not been irritable nor has he had a fever. She did not notice any redness in the groin area. On exam, a reducible mass is noted in the inguinal canal consistent with an indirect hernia. Question 1 7-1 What is NOT a cause of inguinal swelling? A) Indirect hernia. B) Spermatic hydrocele. C) Lymphadenopathy. D) Retractile testis. E) None of the above. Discussion 1 7-1 The correct answer is "E:' Other causes include lymphadenitis, malignancy, and even an appendicitis if the appendix is inside the hernia sac. In the abdomen, the process vaginalis is cre­ ated from peritoneum. The testes descend through the inguinal canal, pushing the process vaginalis into the scrotum. Once the testes are home safe and sound, the process vaginalis obliterates. If the process vaginalis remains wide open, an indirect inguinal hernia forms, allowing organs to move into the inguinal canal. If it remains open but narrows, a communicating hydrocele filled with peritoneal fluid forms. If it remains open in the middle but closes on both ends, a hydrocele of the spermatic cord develops.





Helpful Tip

� � T h 1 s 1s lr Il

A er ia that ca n n ot be reduced is ca l l ed incarcerated. an

emergency a n d

req u i res

i m m ediate

treatment to avoid stra n g u lation at the point where blood fl ow to the hernia contents is compromised.



G E N ITA L SYSTEM D I SO R D E R S

397

A 16-year-old boy was found on routine physical examination to have a left testicular mass. He notes that the mass is associated with an occasional "aching" feeling in the left testicle, but no nausea. The mass enlarges when he lifts heavy objects or strains, and decreases in size when he lies down. He has no other com­ plaints. On examination, both testicles are approximately 5 x 3 x 2 em and of normal consistency, and he has a palpable plexus of veins in the left scrotum that reduces in the supine position. Question 1 8-1 Management of this condition includes: A) Endovascular injection of sclerosing agents. B) Observation. C) Ligation of the spermatic cord. D) Microscopic denervation of the spermatic cord. E) Orchiectomy. Discussion 1 8-1 The correct answer is "B:' Varicoceles are dilated veins ofthe pampi­ niform plexus that surrounds the spermatic cord in the scrotum and may feel like a "bag of worms:' The management of varicocele in adolescents remains a matter of debate, but most experts agree that surgery should be performed in children with severe pain, sig­ nificant (> 10-1 5%) and persistent discrepancy in testicular size, and those engaged in activities likely to promote progression of the varicocele (eg, varsity sports, entry into the armed forces). When varicocelectomy is undertaken, first -line options for surgical man­ agement include laparoscopic clipping of the gonadal vessels, open inguinal approach, or subinguinal microscopic varicocelec­ tomy. Each approach has benefits and drawbacks: the laparoscopic approach is easy, but ligation oflymphatics and the testicular artery may be associated with testicular atrophy and hydrocele formation, respectively. The inguinal approach is familiar to most surgeons and there are often fewer veins and an increased ability to spare the testicular artery at this location, but this approach requires signifi­ cant activity limitations postoperatively given the risk for hernia, and may be more painful. Subinguinal microscopic varicocelec­ tomy is associated with the best artery- and lymphatic-sparing results as well as an earlier return to activities. Endovascular man­ agement is generally reserved for surgical failures, and denervation of the spermatic cord is appropriate for refractory orchalgia. There is no indication for orchiectomy if the testicle is otherwise healthy. Question 1 8-2

If no palpable varicocele were found on physical examina­ tion, but retrograde flow was seen in the veins of the left pampiniform plexus on the scrotal ultrasound, the boy and his family should be counseled that: A) Endovascular repair is less likely to be successful. B) Prolonged observation is not recommended as the varico­ cele may progress to become symptomatic. C) Monthly testicular examinations with a clinician are neces­ sary to ensure adequate testicular growth. D) He likely has decreased sperm motility on semen analysis. E) Surgical repair is not undertaken for subclinical varicoceles.

398

MCG RAW-H I LL E D U CATION S P E C I A LTY BOA R D REVI EW: P E D I ATRICS

Discussion 1 8-2 The correct answer is "E:' Given the indications for consid­ eration of surgical repair of a varicocele, subclinical varico­ celes should not be repaired. Annual examination (unless there is a change in symptoms or anatomic findings) is gen­ erally adequate to assess for progression, although testicular self-examinations are recommended in all pubertal boys and postpubertal men to assess for testicular cancer. There are no data to suggest that subclinical varicoceles adversely affect sperm motility.

Question 1 8-3

If this mass were located on the right side rather than the left side, additional evaluation would include: A) Retroperitoneal ultrasound. B) CT scan of the abdomen and pelvis. C) Measurement of serum follicle-stimulating hormone and luteinizing hormone. D) Scrotal ultrasound with Doppler. E) Measurement of serum testosterone. Discussion 1 8-3 The correct answer is "A:' Varicoceles are more common on the left than on the right, owing to the insertion of the gonadal vein into the left renal vein at a 90-degree angle. On the right side venous flow is more continuous as the gonadal vein drains directly into the inferior vena cava (IVC ) . Iso­ lated left varicoceles and bilateral varicoceles generally do not require additional evaluation. However, in the case of isolated right- sided varicoceles, retroperitoneal imaging should be undertaken to rule out a mass or other pathology causing increased local venous pressure (IVC or right renal vein thrombosis ) . Initial evaluation can be performed with a retroperitoneal ultrasound, with cross-sectional imag­ ing reserved for patients in whom the ultrasound does not provide good- quality images and those in whom additional characterization of the mass is necessary. Scrotal ultrasound may be used to determine the absolute size of both testes, although it has not been shown to be more accurate than physical examination by an experienced clinician. Serum hormone levels are not indicated in the absence of clinical evidence of an endocrinopathy.

A 4-year-old boy who was not circumcised at birth owing to parental preference has difficulty completely retract­ ing the foreskin, and he has "whitish" material that comes from under the foreskin on occasion. He has not had uri­ nary tract infections (UTis) and has no ballooning of the foreskin with voiding. His parents would prefer to keep him uncircumcised.

Question 1 9-1 When counseling the boy and his family, which of the follow­ ing is true? A) He should undergo circumcision because the AAP now encourages circumcision for all boys unless there is a religious or cultural objection. B) He should undergo circumcision because phimosis at this age is abnormal. C) He should not undergo circumcision because this will increase his risk of UTis. D) He should not undergo circumcision until he is treated with antibiotics and the white discharge stops. E) None of the above. Discussion 1 9-1 The correct answer is "E:' Although the AAP has issued a 20 1 2 statement saying that circumcision may b e associated with a decreased risk of UTis, penile cancer, and sexually transmit­ ted infection, the procedure was not recommended for all male infants. Phimosis, the failure of the foreskin to retract, may be present even in older children and is not problematic in the absence of infections, irritation, or pain. The phimosis is normal or physiologic. As the child grows the foreskin separates from the glans becoming retractable. Some children will have smegma (dead skin cells) expressed from under the foreskin, but this alone is not an indication for circumcision. Topical steroid cream (beta­ methasone) may be used to release adhesions in children whose parents wish to preserve the foreskin. Paraphimosis is a condition that occurs when the retracted foreskin becomes edematous and cannot be reduced, resulting in a subsequent decrease in blood flow to the glans. It is a urologic emergency and should be treated by prompt reduction of the foreskin, or incision of the foreskin to relieve the pressure. (See Figure 18-4B, earlier.) B I B LIOGRAPHY

Acker A, Jamieson MA. Use of intranasal midazolam for man­ ual separation of labial adhesions in the office. f Pediatr Ado lese Gynecol. 20 1 3;26: 1 96- 198. American Academy of Pediatrics, Task Force on Circum­ cision. Circumcision policy statement. Pediatrics. 20 1 2 ; 1 30:585-586. Amerstorfer EE, Haberlik A, Riccabona M. Imaging assess­ ment of renal injuries in children and adolescents: CT or ultrasound? J Pediatr Surg. 2 0 1 5 ;50:448-455. Ammenti A, Cataldi L, Chimenz R, et al. Febrile urinary tract infections in young children: Recommendations for the diagnosis, treatment and follow-up. Acta Paediatr. 20 1 2; 1 0 1 :45 1 -457. Best SL, Sivalingam S, Penniston KL, Nakada SY. Radiographic and laboratory data ("the Megaprofile") can accurately guide medical management in the absence of stone analysis. J Endaural. 2 0 1 5;29:357-36 1 . Bisogni S , Olivini N , Festini F. Undertreated and untreated pain should be considered an adverse event of neonatal circumcision. JAMA Pediatr. 2 0 1 4; 1 68: 1 076- 1 077.

CHAPTER 1 8

Bleeker MC, Heideman DA, Snij ders PJ, et al. Penile cancer: Epidemiology, pathogenesis and prevention. World J Ural. 2009;27: 1 4 1 - 1 50. Boettcher M, Bergholz R, Krebs TF, et al. Differentiation of epididymitis and appendix testis torsion by clinical and ultrasound signs in children. Urology. 20 1 3;82:899-904. Brioude F, Lacoste A, Netchine I, et al. Beckwith-Wiedemann syndrome: Growth pattern and tumor risk according to molecular mechanism, and guidelines for tumor surveil­ lance. Harm Res Paediatr. 20 1 3 ;80:457-465. Bulum B, OzT mutation in the IVD gene. Fortunately, the infant is doing well with no sequelae of the condition at the present time. The parents are incredulous, denying that the condition is real, and breaking into tears at several times during the encounter. They are very concerned that their son is going to die suddenly and are afraid to leave him by himself to sleep. Question 5-1

Aside from restriction of leucine and education on dietary needs in this condition, which intervention could be most helpful for this patient and family? A) Give the infant supplementation with ubiquinol, which is proven to decrease the number of metabolic episodes. B) Follow with the family in 1 month to further investigate their adaptation to the diagnosis. C) Repeat molecular testing, as it is often unreliable and this may represent a false-positive result. D) Place the infant on an apnea and bradycardia monitor to decrease parental anxiety about sudden death. E) Have the parents meet with a genetic counselor to discuss their reaction to this condition and discuss the natural his­ tory in greater detail. Discussion 5-1 The correct answer is "E:' All hail the genetic counselor. Unlike pediatricians or geneticists, who receive little to no formal train­ ing in counseling, genetic counselors have extensive training in helping patients and their families work through difficult diag­ noses from a psychosocial standpoint. Following up with the family in a month is probably a good idea on general principle but does not address the parents' immediate concerns about their child. Repeating genetic molecular testing would not be indicated, as the testing is highly reliable and the result is very unlikely to be incorrect. Placing the infant on an apnea and bra­ dycardia monitor may appear reasonable but does not address root concerns and may end up reinforcing the parents' anxiety

errors of meta bolism, giving the patient g l u cose a n d i ntravenous (IV) fl u i d s wi l l b e h e l pfu l a n d potentia l l y l ifesavi ng, so this should be done a s q u ickly as possible.

A 7 -day-old infant girl is brought into the emergency depart­ ment unconscious. Her parents accompany her and report a history of an uneventful pregnancy and delivery at 39 weeks' gestational age, with no unusual postnatal events, and dis­ charge on day of life 2. The parents report that the infant's behavior was normal during the first 3 days at home, but for the past 2 days she has not wanted to feed as much. They were not terribly concerned until about an hour before admission when she became difficult and then impossible to awaken. Emergency department personnel report that they already have plans to evaluate the infant for a cardiac lesion and infectious disease. Question 6- 1 What are the most important immediate steps in evaluation and treatment of this critically ill infant? A) Obtain an electrocardiogram (ECG) to look for occult arrhythmia. B) Measure glucose, lactic acid, and ammonia, and start IV flu­ ids with at least 1 0 % dextrose concentration. C) Start the infant on ampicillin and gentamicin for presumed sepsis. D) Await the results of an echocardiogram before making fur­ ther management decisions. E) Obtain plasma amino acids and urine organic acids to look for metabolic disease. Discussion 6-1 The correct answer is "B:' When a child presents with a criti­ cal illness, it is vitally important to determine acid-base status, ammonia level, and blood glucose, as these will have immedi­ ate management implications and results will be quickly avail­ able. The usefulness of dextrose-containing IV fluids cannot be overstated, as most inborn errors of metabolism feature some

464

MCG RAW-H I LL E D U CATION S P E C I A LTY BOA R D REVI EW: P E D I ATRICS

amount of volume depletion and even if hypoglycemia is not present, dextrose provides a ready energy supply and suppresses catabolism. In some metabolic disorders, a further cardiac eval­ uation is appropriate, but it would be difficult to argue that it should be the most important step. Likewise, although order­ ing an echocardiogram and placing the patient on antibiotics are both reasonable in the management of a critically ill infant, evaluation for an inborn error of metabolism should not wait. Plasma amino acids and urine organic acids are also indicated for this patient, but in most locations they will need to be sent out to a biochemical laboratory and will not be back for sev­ eral days. Even those fortunate enough to have an in-house bio­ chemistry lab that performs these tests will have to wait for the results, and it would not be acceptable to wait for results before providing treatment for the patient.

Question 7-1 What class of disorder is most likely the cause of this patient's hypoglycemia? A) Congenital hyperinsulinism. B) Glycogen storage disease. C) Ketotic hypoglycemia. D) Organic aciduria. E) Congenital disorder of glycosylation type la (CD G l a) . Discussion 7-1 The correct answer is ''A:' The fact that the infant is large for gestational age and has hypoketotic hypoglycemia in the absence of organomegaly is suggestive of hyperinsulinism. (See Figure 2 1 - 1 .) Glycogen storage diseases generally present with organomegaly. Ketotic hypoglycemia is excluded by the text of the case study. (Were you reading carefully?) An organic aciduria is unlikely given the infant's prominent hypoglycemia without prominent acidosis. Congenital disorders of glycosyl­ ation can cause hypoglycemia, although it is not prominently featured in CDG l a. Fatty acid oxidation disorders present after the neonatal period with hypoketotic hypoglycemia after fast­ ing or during an illness.

You are asked to see a formerly premature male infant in the neonatal intensive care unit (NICU) for hypoglycemia. The infant was born at 34 weeks' gestational age, and did not require ventilation, although he was on oxygen for a short period of time. The infant, now 38 weeks' corrected gesta­ tional age, has continued to struggle somewhat with feed­ ing. He is rather large for his gestational age and continues to require a rather high rate of glucose infusion. You note that he does not have hepatomegaly. His most recent serum glucose level was 76 mg/dL, and he continues to receive a 12.5% dextrose concentration infusion. If the glucose infu­ sion is stopped, serum glucose levels drift down rather quickly. The NICU providers have started an evaluation for hypoglycemia, and neither ketosis nor lactic acidosis has been seen.

t

I I

I

I

Hypoglycem i a

!

G l ucagon test

!



I

Pan hypopitu itarism Adrenal i n sufficiency O r g a n i c acidemia M itochond rial defects

I

!



Ketones absent

!

I

No response

I t

I

U ri nary ketones +

Blood g l u cose t

A female neonate is evaluate d at 5 days of age for poor feed­ ing and alteration in the level of consciousness. The moth­ er's pregnancy was normal, and the infant was delivered at 39 weeks' gestational age with Apgar scores of 9 and 9 at 1 and 5 minutes, respectively. There were no problems in the immediate postnatal period. The neonate has two older sib­ lings who are healthy. Examination reveals a normally sized and nondysmorphic neonate who is listless with poor tone

I

Fasting/starvation Galactosem i a Hereditary fructose intolerance

Hyperi n s u l i nemic states Nesidoioblastosis Persistent hyperi n s u l i n e m i c hypoglycemia of infancy I nfant of a d iabetic mother Exogen o u s i n s u l i n Fatty acid oxidation d isorder M itochondrial d isorder

F I G U R E 2 1 - 1 . S i m p l ified diagnostic approach to hypoglycemia. (Reproduced with permission from Tinti n a l l i J E, Stapczynski JS, Ma OJ, Yealy DM, C l i n e DM, Meckler G D, eds. Tintinalli's Emergency Medicine: A Comprehensive Study Guide. 8th ed. New York, NY: McGraw- H i l l Education; 201 6, Fig. 1 44- 1 .)

C H A P T E R 21

and prolonged capillary refill. There is no organomegaly, and the cardiac examination is normal. Question 8- 1 What is the most important next step in this neonate's evalu­ ation and management? A) Evaluation should start with glucose, blood gases, lactic acid level, ammonia level, and serum ketones. B) Plasma amino acids, urine organic acids, and acylcarnitine profile should be obtained. C) IV fluids containing 1 0 % dextrose should be started at 1 . 5 times the maintenance rate. D) Both A and C. E) None of the above. Discussion 8-1 The correct answer is "D:' Patients who are critically ill and are suspected of having an inborn error of metabolism should have their acid-base status, glucose level, and ammonia level measured immediately upon presentation. However, one should not wait until those results are available to begin treat­ ment with dextrose-containing IV fluids, which can be lifesav­ ing. Measurement of the plasma amino acids, urine organic acids, and acylcarnitine profile is very important for making a definitive biochemical diagnosis, but those results will be unavailable for some time and it would be inappropriate to wait for treatment.

A 4-month-old girl is brought to the emergency depart­ ment after an initial seizure. Her serum glucose is low at 37 mg/dL, and lactic acid is moderately elevated. She is noted on examination to have massive hepatomegaly, and her face is described as "doll-like" with rather chubby cheeks. While in the hospital, the infant's glucose level starts to drop after only 2 hours, and administration of glucagon does not improve serum glucose, but does transiently worsen lactic acidosis. Question 9-1 What is the most likely reason for the infant's clinical findings? A) A defect in beta-oxidation of medium-chain fatty acids. B) A defect in phosphatase, which converts glucose-6-phosphate into glucose. C) A defect in the enzyme that converts phenylalanine into tyrosine. D) A defect in the myophosphorylase activity in skeletal muscle. E) A defect in metabolism of sphingolipids. Discussion 9-1 The correct answer is "B:' Defects in glucose-6-phosphatase cause glycogen storage disease type I ( GSD I, von Gierke disease), which results classically in hepatomegaly and also



M ETA B O L I C D I SO R D E R S

465

frequently causes hypoglycemia, which can happen after a rather short fast. Glucagon does not raise the blood glucose level as glycogen cannot be utilized. Defects of fatty acid oxida­ tion can certainly cause hypoglycemia with fasting, although typically this occurs after prolonged fasts. Additionally, these defects generally do not cause hepatomegaly to the degree seen in this case. Conversely, disorders of sphingolipids may cause hepatomegaly but do not frequently cause hypoglyce­ mia. Defects in metabolism of phenylalanine to tyrosine cause phenylketonuria and would cause neither hypoglycemia nor hepatomegaly. Lastly, myophosphorylase deficiency causes gly­ cogen storage disease type V (GSD V, McArdle disease), which frequently causes muscle cramping and breakdown, but with­ out hepatomegaly.

An infant with severe hypotonia is referred to the clinic for evaluation. The infant also has failure to thrive, recur­ rent respiratory infections, and cardiac hypertrophy. Acid alpha-glucosidase level is measured on a dried blood spot and is found to be less than 1 % of normal. Molecular analy­ sis of the GAA gene shows that the infant is homozygous for the common p.Arg854Ter mutation, confirming diagnosis of GSD II (Pompe disease) . Enzyme replacement therapy (ERT) is the mainstay of treatment in patients with Pompe disease. Question 1 0-1 What can be stated about the use or efficacy of ERT in this disease? A) Timing of therapy has little bearing on the outcomes for patients who are treated. B) ERT is helpful in reducing cardiac disease burden but does not affect the course of disease for skeletal muscle. C) Infusion reactions are common and usually require the patient to discontinue therapy. D) IgG-antibodies to ERT seem to have little bearing on the efficacy of the treatment. E) Patients who commence treatment in the first 6 months of life have better ventilator-independent survival, cardiac out­ comes, and developmental milestones. Discussion 1 0-1 The correct answer is "E:' Pompe disease in its classic infantile form causes hypertrophic cardiomyopathy, severe generalized hypotonia followed by death from cardiopulmonary failure by 1 year of age. Patients who commence treatment early gener­ ally have better responses to therapy, although not all patients respond equally. Therefore, timing is important in treatment. Patients may have improvements in both cardiac and muscle disease. Infusion reactions occur in about half of patients who are being treated, although it is usually possible to continue treatment. High-titer IgG antibodies may reduce the efficacy of enzyme replacement therapy.

MCG RAW-H I LL E D U CATION S P E C I A LTY BOA R D REVI EW: P E D I ATRICS

466

An 1 6-year-old boy who plays left tackle for his high school football team complains of cramping in his muscles after practice. You, as the team physician, dig a bit further and find that he has experienced a number of such episodes, usually after fairly strenuous practices. Laboratory evalu­ ation shows normal glucose at 97 mg/dL, normal blood urea nitrogen (BUN) of 12 mg/dL, and normal creatinine at 1 .0 mg/dL. His creatine kinase (CK) was markedly ele­ vated at 22 1 6 IU/L. The patient denies use of performance­ enhancing substances and a urine drug screen is normal. You wonder about a glycogen storage disease as a cause of his problems. Question 1 1 -1 Which of the following is the most likely cause of this patient's findings? A) GSD type II. B) GSD type III. C) GSD type V. D) GSD type VI. E) GSD type VII. Discussion 1 1 -1 The correct answer is "C:' Patients with GSD V (McArdle dis­ ease) present with muscle cramping, poor exercise tolerance, and rhabdomyolysis. It would not be uncommon for a patient to present in adolescence during strenuous athletic activities. GSD II (Pompe disease) is a lysosomal storage disease that manifests with skeletal and cardiac findings. Although adult­ onset forms of this condition exist, this patient's presentation would be atypical for that condition. GSD III is a liver glycogen storage disease that causes hepatosplenomegaly and myopathy. It would be an unlikely presentation in an otherwise healthy adolescent. GSD VI (Hers disease) and VII (Tarui disease) are rarer GSDs.



Helpful Tip

� G lycog e n

stora g e d i s e a s e s ca n b e c h a l l e n g i n g t o

r1 1r d i a g n o s e a n d

have very d i ffe rent presenta t i o n s .

Pa t i e n t s w i t h G S D I (vo n G i e rke d i s e a s e) pres­

e n t w i t h low b l oo d g l u cose, fa sti n g i n tol era n ce, g rowt h fa i l u re, a n d h e patos p l e n o m e g a ly. Patients w i t h GSD II (Pompe d i sease) present w i t h m u s c l e

A 6-day-old girl presents t o the emergency department with her parents, who relate a history of decreased level of con­ sciousness that has worsened over the past 48 hours. The parents state that they received a call from the state labora­ tory about an abnormality the infant's newborn screen but did not understand what the concern was. On examination, the infant has tachypnea and decreased level of alertness with poor capillary refill. She is otherwise nondysmorphic. Labo­ ratory evaluation shows metabolic acidosis, and interestingly the urine shows the presence of reducing sugars. Blood cul­ tures are drawn and broad-spectrum antibiotic therapy is commenced. Within 12 hours, both sets of blood cultures show growth of a pathologic organism. Question 1 2-1 Given the presentation of the patient, which organism is most likely to have been found? A) Streptococcus pyogenes. B) Escherichia coli. C) Staphylococcus aureus. D) Streptococcus pneumoniae. E) Klebsiella pneumoniae. Discussion 1 2-1 The correct answer is "B:' The case history for this patient includes clues to the diagnosis of galactosemia, including evi­ dence of sepsis at presentation, an abnormal newborn screen, and presence of reducing sugars on urinalysis (indicating errors in sugar metabolism) . Galactose and glucose form the disaccha­ ride lactose. One important aspect of galactosemia is its associa­ tion with sepsis in patients who do not have prompt treatment, E. coli being the most common organism isolated in such cases. Therefore, for the primary care provider, it is crucial that these patients be placed on galactose-free feeding (no lactose) as soon as a diagnosis is made. Other findings include liver dysfunction and cataracts. Remember, soy formula is the drink of choice in galactosemia.

After stabilization of the patient, you as the diligent pedi­ atrician call the state laboratory to find out exactly what was abnormal on the newborn screen. However, since you studied metabolic diseases extensively in preparing for your examination you already have a pretty good idea of what was found.

wea k n e s s a n d h ea rt fa i l u re, a n d a l s o h e pato­ s p l e n o m eg a ly. E n zyme re p l a c e m e n t i s ava i l a b l e fo r t h i s c o n d i t i o n . Pa t i e n t s w i t h G S D I l l present w i t h h e patos p l e n o m e g a ly a n d myo pat hy. Patients w i t h G S D V ( M cArd l e d i s e a s e ) present w i t h exerc i s e ­ i n d u ced c ra m ps a n d r h a b d o myolys i s . O t h e r fo r m s of G S D m a n ifest va r i a b l y w i t h h e patos p l e n o m eg ­ a ly, m u s c l e wea k n e s s o r c ra m p i n g , hypog lyce m i a , a n d g rowth c o n c e r n s .

Question 1 2-2 What laboratory abnormality should you suspect in this patient? A) Elevation of CS (octanoylcarnitine) . B) Elevation of phenylalanine and decreased tyrosine. C) Presence of succinylacetone. D) Reduced activity of galactose- I - phosphate uridyltransferase (GALT). E) Reduced activity of biotinidase.

C HAPTER 2 1

Discussion 1 2-2 The correct answer is "D:' Classic galactosemia is caused by mutations in the gene that encodes the galactose- ! -phosphate uridyltransferase enzyme (GALT). GALT is part of the enzy­ matic pathway that metabolizes galactose to glucose for energy utilization. Elevation of octanylcarnitine is typical of a diag­ nosis of medium-chain acyl-CoA dehydrogenase deficiency (MCADD). Elevation of phenylalanine with decreased tyro­ sine is seen in patients with phenylketonuria (PKU). Presence of succinylacetone is seen in patients with type I tyrosinemia. Reduced activity of biotinidase would, of course, be seen in bio­ tinidase deficiency.

Your patient has recovered well from her acute health prob­ lem and is thriving with the benefit of appropriate dietary intervention. Thus far her growth and development have been normal, to the delight of her parents. Question 1 2-3

In addition to providing them advice about the recurrence risk of this condition, what should you tell the parents about long-term sequelae that may not be prevented by diet? A) All long-term sequelae are preventable by dietary interventions. B) Learning disability and premature ovarian failure are more common. C) Liver failure is unavoidable and not prevented by diet. D) Cataract development occurs in treated and untreated patients at the same frequency.



M ETA B O L I C D I SO R D E R S

467

Question 1 3-1 What is the most likely diagnosis in this patient? A) Krabbe disease. B) Fabry disease. C) Hurler syndrome (MPS I). D) Hunter syndrome (MPS II). E) Morquio disease (MPS IV) . Discussion 1 3-1 The correct answer is "C:' Patients with Hurler syndrome tend to present very early in life with typical features of mucopolysac­ charidosis (MPS) such as hepatosplenomegaly, developmental delay, coarse facial features, and limitations in joint movement. (See Figure 2 1 -2 ) . MPS is a lysosomal storage disorder in which glycosaminoglycans accumulate in the lysosomes. Corneal clouding is present in MPS I. Other features include short stat­ ure, skeletal dysplasia, hearing loss, hydrocephalus, and persis­ tent nasal drainage. MPS II shares a number of features with MPS I, but it has an X-linked inheritance pattern and does not feature corneal clouding, both of which would be inconsistent with the findings described for this child. Patients with MPS IV may present with short stature but not cognitive delays. The description given is also inconsistent with diagnoses of Krabbe disease and Fabry disease.

Discussion 1 2-3 The correct answer is "B:' Patients with classic galactosemia have an increased risk of learning disabilities, speech deficits, and premature ovarian failure, which does not appear to be pre­ vented, even with scrupulous adherence to diet. Liver failure is a complication that is typically seen in patients when treatment is delayed or not commenced. Cataract development appears to be more common in patients who are untreated, but it does not appear that risk of cataract is eliminated in patients who are treated with diet.



� I

1 1r

Helpful Tip

Galactosemia is associated with sepsis, pa rticula rly E. coli.

An 1 8-month-old girl presents to the clinic with find­ ings of poor linear growth and developmental delay. Her facial features are becoming coarse, and she has devel­ oped an inability to fully extend the joints. On further investigation, you note that she has corneal clouding and hepatosplenomegaly.

F I G U R E 2 1 -2. This c h i l d has a m u copolysaccha ridosis type I ( H u rler synd rome) characterized by short statu re, coarse facies, e n l a rged tongue, joint stiffness, developmental delay, and hepatomega ly. An enzymatic defect resu lts i n the accu m u lation of g l ycosa m i nog lyca ns i n the lysosomes. (Reproduced with permission from Va l l e D, Bea udet AL, Vogelstei n B et a l : The Online Metabolic and Molecular Bases of Inherited Disease, Sed. McGraw- H i l l Education, I nc; 201 4. F i g 1 36-6.)

MCG RAW-H I LL E D U CATION S P E C I A LTY BOA R D REVI EW: P E D I ATRICS

468



Helpful Tip

=� I n

comparing MPS I a n d I I , rem e m ber that a h u nter

r1 1 r needs clea r eyesig ht, so H u nter syn d rome (MPS I I ) does

not have cornea l cloudi ng, whereas H u rler syn d rome

(MPS I) does.

Question 1 3-2 Which of the following laboratory investigations would be the most likely to provide a definitive diagnosis in this patient? A) Carbohydrate-deficient transferrin. B) Very long-chain fatty acids. C) Acylcarnitine profile. D) Lactic acid level. E) Urine glycosaminoglycans. Discussion 1 3-2 The correct answer is "E:' Patients with mucopolysaccharidoses have characteristic patterns of abnormalities on urine glycos­ aminoglycan (GAG) profile, and this is likely to make the diag­ nosis. Carbohydrate-deficient transferrin is used to diagnosis congenital disorders of glycosylation, which are not suspected in this patient. Very long-chain fatty acids are used to diagnose peroxisomal disorder, which we also do not expect here. Lactic acid level and acylcarnitine profile are common tests to screen for metabolic disorders but would not diagnose a patient with MPS I. Question 1 3-3 Which is NOT a radiographic finding of MPS disorders? A) Delayed cranial suture closing. B) Thick ribs. C) Hypoplastic flared iliac bones. D) Metacarpals with narrowing proximally. E) Coarse trabeculated long bones. Discussion 1 3-3 The correct answer is "A:' MPS disorders produce character­ istic skeletal abnormalities known as dysostosis multiplex. Therefore, diagnostic evaluation should include a skeletal sur­ vey. Radiographic signs include short, thick long bones with irregular metaphyses and epiphyses; thick ribs; enlarged skull with thickened calvarium and craniosynostosis; ovoid vertebral bodies; and gibbus deformity (thoracolumbar spine kyphosis). Closure of the sutures is premature, not delayed.

Congenital hyperinsulinism can be a cause of particularly severe hypoglycemia, which requires continuous administra­ tion of significant amounts of glucose in the neonatal period. Question 1 3-4 Which of the following long-term interventions would be appropriate for this patient? A) Therapy with diazoxide. B) Therapy with octreotide.

C) Partial resection of the pancreas. D) High-carbohydrate diet. E) All of the above. Discussion 1 3-4 The correct answer is "E:' Patients with hyperinsulinism may require any of all of these interventions depending on the sever­ ity of disease. Pharmacologic therapy and dietary therapy by themselves may not be sufficient to decrease insulin secretion, and in some patients a partial pancreatectomy may be required to reduce the bulk of beta islet cells and insulin secretion.

A 6-year-old child presents with onset of acute abdominal pain, and pancreatitis is diagnosed. This is the second presen­ tation of pancreatitis for this child. His parents report that he is otherwise healthy and takes no medication. On physical exam he also has hepatosplenomegaly and xanthomas on his elbows and Achilles tendons. Question 1 4-1 What laboratory investigation is most likely to lead to the correct diagnosis for this patient? A) Lactic acid. B) Lipid panel. C) Fingerstick glucose. D) Urine organic acids. E) Carbohydrate-deficient transferrin. Discussion 1 4-1 The correct answer is "B:' This patient has a disorder of lipo­ protein metabolism, in this case familial lipoprotein lipase defi­ ciency. Patients with this disorder present in childhood with severe hypertriglyceridemia, pancreatitis, cutaneous xantho­ mata, and hepatosplenomegaly. They may also have accumula­ tion of chylomicrons in the blood, giving a milky appearance that may be appreciated with phlebotomy. Although lactic acid and urine organic acids are frequently measured in patients with metabolic disorders, they are unlikely to be helpful in this instance. Patients with pancreatitis may indeed have deviations in their glucose level, but such discovery is unlikely to lead toward a diagnosis. Transferrin isoelectric focusing may show a carbohydrate-deficient pattern in patients with congenital disorders of glycosylation but is unlikely to shed any light on a lipoprotein disorder.

A family presents to the clinic with their two children, aged 2 and 5 years, who are known to be healthy. The father, who is 34 years old, relates a personal history of extremely premature

C HAPTER 2 1

cardiovascular disease with coronary artery bypass grafting at age 26. He is presently taking three different lipid-lowering agents to control his extremely high low-density lipoprotein (LDL) cholesterol. Question 1 5-1 Assuming that the father is homozygous for familial hyper­ cholesterolemia, what should you tell him about the proper next step in management for his children? A) Begin weekly LDL apheresis. B) Reassure the father that his children are unlikely to be affected. C) Commence high-dose HMG-CoA reductase therapy in both children. D) Commence bile acid sequestrant therapy in both children. E) Order a lipid panel and institute appropriate lifestyle changes. Discussion 1 5-1 The correct answer is "E:' If the father in this scenario is a homozygote for familial hypercholesterolemia, his children would be obligate heterozygotes and in this condition hetero­ zygotes also have elevated serum cholesterol and increased risk of disease, with 50% of men and 25% of women affected hav­ ing coronary artery disease by age 50. In very young children who have heterozygous familial hypercholesterolemia (HeFH), it would be appropriate to start measurement of a lipid panel at age 2 years. However, it would not be appropriate to start defin­ itive therapy such as bile acid sequestrants or statins (HMG­ CoA reductase inhibitors) at such a young age. LDL apheresis is used in some cases for patients with homozygous familial hypercholesterolemia (HoFH), but would not be appropriate in such young children. Telling the parent that the children are unlikely to be affected is inappropriate. The proper course of action in this case is for both children to be screened with a lipid panel. Should any lifestyle factors be contributing to risk, those also should be addressed. The present guidelines sug­ gest treatment of children with HeFH using statin medications starting at age 8 years.

� QUICK QUIZ Type I Gaucher disease i s a relatively common disorder of sphingolipid metabolism that leads to health concerns involving multiple organ systems. Which of the following findings would NOT be consistent with a diagnosis of Gaucher disease type I? A) Hepatosplenomegaly. B) Low bone mineral density. C) Intellectual disability. D) Thrombocytopenia . E) Interstitial lung disease.



M ETA B O L I C D I SO R D E R S

469

Discussion The correct answer is "C:' Gaucher disease is a lysosomal stor­ age disease-specifically involving sphingolipidoses-that affects the cell's ability to recycle cellular glycolipids. Lyso­ somes are the digestive system of the cell. In Gaucher disease, glycolipids build up in the lysosomes of macrophages. The lipid-laden macrophages then accumulate in affected organs {liver, spleen, lungs, bone marrow) . Type I Gaucher disease is characterized by a number of conditions, including hepa­ tosplenomegaly, low bone density, and painful bone crises. Patients also may have leukopenia, thrombocytopenia, and anemia in any combination or together. They may suffer from a variety of lung diseases, including interstitial lung disease and pulmonary hypertension. There is some evidence to suggest that monoclonal gammopathies, including multiple myeloma, may be more common in patients with Gaucher disease than in the general population. Intellectual disability would be incon­ sistent with a diagnosis of Gaucher disease. Gaucher disease is treated by replacement of the missing lysosomal enzyme, glucocerebrosidase.

An adolescent boy presents with the lesions shown in Figure 2 1 -3. He has been suffering from episodic pain in his hands and feet that has not been adequately explained. His mother reports that she also has had tingling in her hands for a number of years, but the pain has never been severe. Question 1 6-1 Which of the following choices represents a major risk for early death in this adolescent male patient? A) Hematologic malignancy. B) Cirrhosis. C) Immune deficiency. D) Cerebrovascular disease. E) Alzheimer dementia. Discussion 1 6-1 The correct answer is "D:' This patient presents with a history consistent with Fabry disease, which is an X-linked lysosomal storage (sphingolipidosis) disorder arising from deficient activ­ ity of the alpha-galactosidase enzyme. Patients with this disor­ der frequently suffer from painful acroparesthesia (tingling of the extremities), and a large percentage of them present with angiokeratoma. (See Figure 2 1 -3.) As patients age, their risk for cardiovascular and cerebrovascular disease increases. Patients also frequently have renal involvement and progress to end­ stage renal disease. Although the classic findings of this con­ dition have been historically described in males {X-linked) , it is understood that females may also have manifestations of the disease, which may be relatively mild but can be as severe as those in affected males.

MCG RAW-H I LL E D U CATION S P E C I A LTY BOA R D REVI EW: P E D I ATRICS

470

C) Vomiting. D) Hyperammonemia. E) Lactic acidosis. Discussion 1 7-1 The correct answer is "D:' Decreased level of consciousness and vomiting are common signs of a variety of metabolic conditions and are not terribly specific. Likewise, a number of conditions can cause lactic acidosis, particularly mild lactic acidosis. The patient's blood glucose level is also not particularly low, which is often the case in a urea cycle disorder. However, prominent hyperammonemia in a situation such as this is suggestive of a urea cycle disorder as the primary metabolic defect. Early rec­ ognition of this significant metabolic disorder is very important for its successful treatment. The longer hyperammonemia is present, the worse the neurologic outcome will be.

F I G U R E 2 1 -3. Patients w i t h Fa bry disease develop angiokeratomas s u c h as the one l ocated on the p e n i s i n this fi g u re. (Reproduced w i t h permission from Fuster V, Wa lsh RA, H a rri ngton RA, eds. Hurst's The Heart. 1 3th ed. New York, NY: McGraw- H i l l Education; 201 1 , Fig. 1 4- 1 1 .)



Helpful Tip

� Enzyme replacement therapy (ERT) has revol ution ized 1 1 1r the treatment of severa l genetic d i sorders, a n d

increasing n u m bers o f prod ucts a re ava i la b l e on t h e ma rket. T h e fol lowi ng prod ucts a re presently ava i l a b l e

i n the U n ited States: For Gaucher d i sease ( lysosomal storage d i sorder [LSD])-i m i g l u cerase, vel a g l u cerase, ta l i g l ucerase For Fa bry d i sease (LSD)-aga lsidase beta For Pompe disease (glycogen storage d isease [GSD] I I)­ agalsidase alfa For H u rler d i sease (mucopolysaccha ridosis [M PS] I)­ I a ronidase For H u nter d isease (MPS 1 1)-idu rsulfase For Ma rotea ux-La my d i sease (MPS Vl)-galsu lfase For Morq u i o A d i sease (MPS IV)-elosu lfase alfa

Patients with urea cycle disorders may experience symptoms of acute hyperammonemia, but also may experience chronic symptoms if control of the disorder is inadequate. For most patients, this requires daily management of their condition. Question 1 7-2 Which of the following treatments is/are indicated for the daily care of a patient with a urea cycle disorder? A) Carnitine supplementation. B) Protein restriction. C) Ammonia scavengers such as sodium phenylbutyrate. D) Both B and C. E) Options A, B, and C. Discussion 1 7-2 The correct answer is "D:' Patients with urea cycle disorders benefit from protein restriction, although the level of protein restriction required varies by patient. Sodium phenylbutyrate is an ammonia scavenger that allows patients with urea cycle disorders to have an alternate pathway for ammonia excretion. Treating patients who have a urea cycle disorder with L-carnitine will probably not harm them, but it is unlikely to be productive.



� A patient presents in the neonatal period with decreased level of consciousness and inability to feed. The patient has also experienced spells of vomiting and is lethargic on exami­ nation. Initial laboratory studies show glucose of 61 mg/dL, a lactic acid level of 3.3 mmol/L, and an ammonia level of 320 mg/dL. The metabolist on call evaluates the patient and tells you that she is concerned for a urea cycle disorder.

Helpful Tip

The u rea cycle is responsible for detoxifying a m monia

1 1 1r into

u rea, which ca n then b e easily excreted. (See

F i g u res 2 1 -4 and 2 1 -S.) Ammonia is produced as a byprod uct of protein metabolism. The treatment of u rea

cycle d isorders has a few centra l themes. First, if a person consu mes less protein, he or she wil l also prod uce less a mmonia so protein restriction is recommended. Second, many patients with u rea cycle disorders also benefit from amino acid supplements, genera l ly either citru lline or argini ne. Third, if dieta ry thera py and amino acid therapy

Question 1 7-1

Which of the patient's findings is most specific for a urea cycle disorder? A) Decreased level of consciousness. B) Hypoglycemia.

a re not sufficient to control the a m monia level (a nd they freq uently a re not), patients wi l l be treated with an a mmonia scavenger. Th e most common ora l scavenger is sod i u m phenyl butyrate, but others exist.

C H A P T E R 21

� � �H2N

M ETA B O L I C D I SO R D E R S



471

Arg i n i n osuccinate

F marate

Aspartate

H2N I

',

',

/c

yto

I

C = NH2+

C=O I

HN I

(CH2 b

I

Citru l l i n e + N O

Arg i n i n e

HN I

(CH2 b

I � )I II y C I

I

HC- NH3+

HC - NH3+

Ornithine

co o -

coo-

H3W

Pi

(CH2 b

Carbamoyl phosp hate

U rea

HC - N H 3+

NH2

1

1

coo-

=0

NH2

NH/ � NH3

F I G U R E 2 1 -4. U rea Cyc le. T h e u rea cyc l e converts a m m o n i a , t h e byp rod uct o f p rote i n meta b o l i s m , i nto u rea fo r excret i o n from t h e body. D i s r u pt i o n of t h i s process res u lts in va r i o u s d e g rees of hypera m m o n e m i a . Cyto, cyto p l a s m ; M ito, m itoc h o n d r i a . (Repro d u ced w i t h p e r m i s s i o n from B a r rett KE, Boitano 5, B a r m a n 5M, B rooks H L, eds. Ganong's Review of Medical Physiology. 24th ed. New Yo rk, NY: M c G raw-H i l l Education; 2 0 1 2, Fig. 1 -20.)

A female neonate is brought by her parents to the pediatrics office after 1 day of poor feeding and worsening vomiting over the past few hours. She has a decreased level of con­ sciousness and poor muscle tone. A basic metabolic panel shows a metabolic acidosis with increased anion gap, CBC shows thrombocytopenia and neutropenia, and ammonia is mildly elevated. The neonate was born in a different state and the parents report "there was something wrong with the new­ born screening;' but since their daughter was doing well they did not follow up on it. Question 1 8-1 What is the most likely diagnosis for this neonate? A) Phenylketonuria. B) Propionic acidemia. C) NARP syndrome (neuropathy, ataxia, and retinitis pigmentosa) . D) Medium-chain acyl-CoA dehydrogenase deficiency. E) Nonketotic hyperglycinemia. Discussion 1 8-1 The correct answer is "B:' The neonate has findings typical of an organic acidemia, in which patients have a metabolic acidosis

Net reaction

2NH3

Hepatocyte

0

+

C02

=

U rea +

H20

NH 3+ I

II

H2N - C - N H - (CH2b-CH-CooCitru l l i n e

Aspartate

2 AMP

cooI



NH 3+

N H 2+ II

I

-ooC -CH2-CH - N H - C - N H - (CH2b-CH- CooArg i n i n e succinate Ornit h i n e

@

N H 3+

I + H3N - (CH2b-CH-Coo-

Fumarate

NH 2+ II

NH 3+ II

H2N -C- N H - (CH2b-CH-CooArg i n i n e

0 II

Cytosol

H2N - C - N H2 U rea To c i rculation

G)

Carbamoyl synthetase

®

Arg i nosuccinate synthetase

@

Arg i n i n e succinate lyase

@

Argi nase

F I G U R E 2 1 -5. U rea Cycle. The u rea cycle controls the processi n g of N H 3 to u rea for excretion. (Reproduced with permission from Ba rrett KE, Barman 5M, Boita no 5, & Brooks H L (Eds). Ganong's Review of Medical Physiology, 25th ed. McGraw- H i l l Education, I nc., 201 6. Fig 1 -20.)

472

MCG RAW-H I LL E D U CATION S P E C I A LTY BOA R D REVI EW: P E D I ATRICS

and frequently present with poor feeding and vomiting. Patients with organic acidemias also commonly have thrombocytopenia or neutropenia from bone marrow suppression. The history in this case is inconstant with the diagnosis of phenylketonuria, as a neonate with untreated PKU is not likely to have developed symptoms yet. Patients with NARP also likely would not present in the neonatal period, and would not present in this fashion. Patients with MCADD may present early in life with critical ill­ ness that is usually caused by hypoglycemia, without prominent acidosis (minimal to no ketones) . Nonketotic hyperglycinemia manifests as a neurologic disorder, including encephalopathy and seizures. Acidosis is not a prominent feature.

A 5-year-old boy is brought to the emergency department after 2 days of vomiting and decreased level of consciousness. Laboratory evaluation shows significant metabolic acidosis. He had previously been healthy, but several days before vom­ iting started had an apparently mild viral illness. The boy has had generally normal growth, but has significant macroceph­ aly that has not been well-explained. Since admission, he has been medically stable but has developed unusual choreoath­ etoid movements. Question 1 9-1 What disorder best fits the patient's clinical findings? A) 3-Methylcrotonyl-CoA carboxylase deficiency. B) Medium-chain acyl-CoA dehydrogenase deficiency. C) Glutaric aciduria type I. D) Succinate semialdehyde dehydrogenase deficiency. E) Postinfectious encephalitis. Discussion 1 9-1 The correct answer is "C' Patients with type I glutaric aciduria frequently are noted to have macrocephaly prior to their inciting metabolic crisis, which may resemble other organic acidemias with vomiting, poor feeding, and metabolic acidosis. However, the disorder is somewhat unique in that metabolic crises fre­ quently cause damage to the basal ganglia, resulting in choreo­ athetosis. Unlike other organic acidemias glutaric aciduria type I rarely manifests in the newborn period. Affected children may have subdural hemorrhages. A child with MCADD or succinate semialdehyde dehydrogenase deficiency would not resemble the boy in this description, presenting with hypoglycemia or neurologic findings, respectively. Postinfectious autoimmune encephalitis is possible in this context given a viral prodrome, but would not explain the patient's acidosis or his macrocephaly.

Patients with methylmalonic academia require intensive long-term management of their disease. In the short term, the focus becomes dietary in nature with implementation of a low-protein diet, and in some patients routine cobalamin

injections. These measures are used to keep the patient out of acidotic crisis. However, as the disease progresses, many long-term sequelae can develop that are problematic for the patient. Question 20-1 Which of the following is NOT a typical long-term conse­ quence of infantile-onset methylmalonic acidemia? A) Tubulointerstitial nephritis. B) Metabolic stroke. C) Intellectual disability. D) Cardiomyopathy. E) Pancreatitis. Discussion 20-1 The correct answer is "D:' Cardiomyopathy is not a typical long­ term effect of methylmalonic acidemia, an organic acidemia. However, tubulointerstitial nephritis, metabolic stroke, intel­ lectual disability, and pancreatitis can also be seen in patients with this condition. These can occur even in patients with good dietary control who do not have recurrent metabolic crises.





Helpful Tip

Patients with o rg a n i c a c i d e m i a s ca n present early in

1 1 1r t h e neonatal period with letha rgy, vo m i t i n g , hypo­

to n i a , i r rita b i l ity, and a c i d o s i s . These a re n o n s pecific

fi n d i ng s and m i m i c s e p s i s, but s h o u l d a lways be c o n s i d e red in such a patient. C h ro n i c seq u e l a e of o rg a n i c a c i d e m i a ca n i n c l u d e meta b o l i c stro ke, pan­ c reatitis, a n d neutrope n i a .

A 2-month-old male neonate presents to the emergency department with poor feeding and tachypnea. Examination reveals very poor muscle tone and hepatomegaly. The neo­ nate had intermittent hypoglycemia soon after birth, which appeared to resolve. An echocardiogram shows severe biven­ tricular dysfunction consistent with dilated cardiomyopa­ thy. Pericardial effusion is present. An ECG shows frequent premature ventricular complexes with intermittent atrio­ ventricular (AV) block. Liver transaminases are moderately elevated. Question 21 -1 Which of the many fatty acid oxidation disorders does this patient most likely have? A) Very long- chain acyl-CoA dehydrogenase deficiency (VLCADD ) . B) Medium- chain acyl-CoA dehydrogenase deficiency (MCADD) . C ) Carnitine palmitoyltransferase 1 (CPT - 1 ) deficiency. D) Short-chain acyl-CoA dehydrogenase deficiency (SCADD). E) Long-chain acyl-CoA dehydrogenase deficiency (LCADD) .

C H A P T E R 21

Discussion 21 -1 The correct answer is "A:' Patients with VLCADD are at high risk for cardiac manifestations, such as dilated cardiomyopathy with heart failure and arrhythmia, and also may have hypoglycemia and hepatic dysfunction. Patients with long-chain 3-hydroxyacyl­ CoA dehydrogenase (LCHAD) deficiency may also have cardiac manifestations. Patients with MCADD, SCADD, or CPT- 1 often have other manifestations of disease, but a predominantly cardiac presentation would be unusual. "Long-chain acyl-CoA dehydro­ genase deficiency" (LCADD) sounds like a very worrisome disor­ der, but as far as we know it does not actually exist. Don't confuse LCHAD (real) with LCADD (not real) .

� QUICK QUIZ Which i s the most likely diagnosis i n a patient presenting in early childhood with hypoglycemia who has excessive amount of total carnitine in an acylcarnitine analysis? A) Very long-chain acyl-CoA dehydrogenase deficiency (VLCADD) . B) Medium-chain acyl-CoA dehydrogenase deficiency (MCADD) . C ) Carnitine palmitoyltransferase 1 (CPT- 1 ) deficiency. D) Short-chain acyl-CoA dehydrogenase deficiency (SCADD). E) Long-chain acyl-CoA dehydrogenase deficiency (LCADD) . Discussion The correct answer is "C:' CPT -1 is the rate-limiting step for transport of fatty acids into the mitochondria, and is unique in that it shows elevations in CO, or unesterified carnitine. Patients frequently present early in life with hypoketotic hypoglycemia after fasting. They may also develop liver dysfunction and renal tubular disorders. Patients with VLCADD tend to have eleva­ tions in C 14, C 1 4: 1 , and C 1 4:2. Patients with MCADD have elevations in CS. Patients with SCADD have elevations in C4. LCADD does not exist. (Were you paying attention when you read the preceding question?)







M ETA B O L I C D I SO R D E R S

473

and then closes it again. You note that the infant startles in an obvious manner, with extension of his arms and legs. On examination you are not able to get the boy to track visu­ ally, and a macular cherry red spot is visible on funduscopic examination. On palpation, there is no organomegaly. Question 22-1 What is the most likely diagnosis in this patient? A) Krabbe disease. B) Tay-Sachs disease. C) Niemann Pick disease type A. D) Gaucher disease. E) Metachromatic leukodystrophy sphingolipidoses. Discussion 22-1 The correct answer is "B:' Tay-Sachs disease, a lysosomal storage disorder (sphingolipidosis), is characterized by the early onset of hypotonia and muscular weakness. Often, visual tracking is lost very early in the disease process. Patients have a "cherry red" spot on the macula that is typical of this condition and some other related conditions. (See Figure 2 1 -6.) An exaggerated startle reflex is characteristic as well. Children with Tay-Sachs disease have a relentless clinical course characterized by neurologic dete­ rioration, seizures, and blindness. Average life expectancy for a child with acute infantile Tay-Sachs disease is less than 4 years. Although patients with Gaucher disease or Niemann-Pick type A (which are other sphingolipidoses) can have the cherry red spot, both of those disorders feature prominent hepatospleno­ megaly, and patients with neuronopathic forms of Gaucher dis­ ease may have neurodegeneration. Patients with Krabbe disease present in infancy with irritability, spasticity, and developmental delay, with regression and evidence of leukodystrophy on neu­ roimaging. Metachromatic leukodystrophy generally manifests

Helpful Tip

It is i m po rta nt to u n dersta nd what tests to order to

1 1 1r confi rm the d iag nosis of a meta bolic d isorder.

Order plasma a m i n o acids for u rea cycle d efects a n d

a m i n o a c i d d isorders Order urine org a n i c acids for orga n i c acidemias Order a n acylcarnitine profi le for fatty acid oxidation d i sorders

Worried parents bring their 4-month-old son to the clinic because of motor weakness and low muscle tone, which they feel is getting worse. While you are interviewing the par­ ents, one of your nurses opens the examination room door

F I G U R E 2 1 -6. Tay-Sachs Disease. A cherry red s pot is present on the macula i n patients with Tay-Sachs, Gaucher, and N i e m a n n-Pick type A d iseases, all of which a re s p h i n g o l i pidoses, a type of lysosom a l storage disorder. (Reproduced with perm ission from the National I n stitute of Health ( N I H ) and the National Eye I n stitute.)

474

MCG RAW-H I LL E D U CATION S P E C I A LTY BOA R D REVI EW: P E D I ATRICS

after 1 year of age with weakness and hypotonia. Children who have gained speech eventually lose that ability, and vision and hearing are lost. If your patient has an exaggerated startle reflex and a cherry red spot, think Tay-Sachs disease.

The parents have brought their now 1 2-month-old child with hexosaminidase A deficiency for follow-up care. He has continued to suffer from the effects of the disease, and his mother asks you what interventions are appropriate for his supportive care. Question 22-2 Which of the following interventions would NOT be appropriate? A) Ensuring adequate nutrition. B) Pharmacologic treatment of seizures. C) Management of airway and respiratory compromise. D) Treatment of chronic constipation. E) Enzyme replacement therapy. Discussion 22-2 The correct answer is "E:' Tay-Sachs disease is caused by a defi­ ciency of the enzyme hexosaminidase A. At present, no enzyme replacement therapy exists. Several strategies to treat this disease have been proposed but none is approved at the present time. Supportive care is indicated for children with this condition and includes nutritional support and management of respiratory con­ cerns. Chronic constipation and seizures are also typical prob­ lems seen as the disease progresses, and their management is also appropriate. Honest discussion among family members and medical providers regarding the goals of therapy is appropriate to ensure that medical interventions are implemented with the goal of relieving suffering and not merely because the option exists.

A patient presents in the neonatal period with lethargy and hypotonia. The neonate subsequently develops myoclonus and spells of central apnea, and requires mechanical ventila­ tion for 3 weeks before starting to recover spontaneous res­ piration. An electroencephalogram (EEG) is obtained and shows a burst-suppression pattern. Laboratory studies show a notable absence of ketosis or acidosis. Question 23-1 What amino acid is likely to be elevated in both the plasma and the cerebrospinal fluid of this neonate? A) Glycine. B) Threonine. C) Phenylalanine. D) Methionine. E) Proline. Discussion 23-1 The correct answer is "A:' This neonate has nonketotic hyper­ glycinemia (also known as glycine encephalopathy), presenting

with the typical neurologic findings of this condition, including a classic burst-suppression EEG pattern. The patient does not have ketosis or acidosis, which distinguishes this disorder from other organic acidemias that have been variably described as ketotic hypoglycemias (a somewhat archaic description). Elevations in threonine are exceedingly rare, although at least one case report from 1 978 describes this finding in a male infant. Elevations in phenylalanine are typically seen in patients with phenylketon­ uria (PKU), but these patients are normal at birth with neuro­ logic findings that develop over time. Elevations in methionine can be seen in patients as a reflection of an immature liver or with hepatocellular damage. They can also be seen in patients with cystathione beta-synthetase deficiency (homocystinuria) or methionine adenosyltransferase deficiency. Elevations in proline are seen in patients with hyperprolinemia, which is often asymp­ tomatic but can cause seizures and intellectual disability.

Maple syrup urine disease is a disorder of branched-chain amino acids, and the specific cause is a decrease in the activ­ ity in the branched-chain alpha-ketoacid dehydrogenase complex. If untreated, the disease leads to ketonuria, poor feeding, and encephalopathy. Patients with this condition have a maple syrup or burnt sugar smell to their body fluids, particularly the cerumen in the ear canals. Question 24-1 Which of the following amino acids is not considered a branched-chain amino acid that would be elevated in a patient with maple syrup urine disease? A) Leucine. B) Isoleucine. C) Alloleucine. D) Histadine. E) Valine. Discussion 24-1 The correct answer is "D:' Histadine is not a branched-chain amino acid. This is a question you might be able to reason out even if you do not recall any amino acid chemistry. One might assume (correctly) that leucine, isoleucine, and alloleu­ cine share a common chemical structure. This being the case, none of them could be the odd amino acid out. You would only then have to remember either that valine is a branched-chain amino acid or that histadine has an imidazole functional group to arrive at the correct conclusion. If nothing else, you would have a 50-50 shot at guessing right.

Historically phenylketonuria (PKU), an amino acid disorder, has constituted an important share of children with devel­ opmental disabilities. The results of untreated PKU can be

C H A P T E R 21

truly devastating for these individuals. Although mandatory screening for the disorders means that you are unlikely to manage patients with untreated PKU in your practice, you are very likely to care for a patient who has received treat­ ment for the disorder. Thus it is important to know what the potential complications of untreated PKU could be. Question 25-1 All of the following are potential complications in an untreated patient with PKU EXCEPT: A) Seizure disorder. B) Cardiomyopathy. C) Poor pigmentation of the hair and skin. D) Microcephaly. E) Severe eczema. Discussion 25-1 The correct answer is "B:' Patients with PKU are not believed to be at risk for cardiomyopathy. Phenylketonuria is an inborn error of metabolism that results from a deficiency of the enzyme phenylalanine hydroxylase, which prevents phenylalanine from being metabolized to tyrosine. The resulting deficiency of tyro­ sine and buildup of phenylalanine in blood and tissues have widespread effects in the body. Patients with untreated PKU are at risk for severe behavioral problems and intellectual impair­ ments, and can have microcephaly and seizure disorders, with variable structural brain changes visible on MRI. The deficiency in tyrosine results in decreased production of melanin and is also detrimental to the production of dopamine, which con­ tributes to neurocognitive deficits in the brains of patients with untreated PKU. The skin condition and the musty body odor often attributed to the disorder result from excretion of phenyl­ alanine byproducts through alternate means.

Treatment of patients with PKU involves the restriction of phenylalanine in the diet and the supplementation of other amino acids to ensure protein sufficiency. However, in many cases additional non-diet-based therapy is appropriate for optimal management. Question 25-2 Which of the following therapies is approved by the U.S. Food and Drug Administration (FDA) for treatment of PKU? A) Adeno-associated virus 1 (AAV l ) -based gene therapy. B) Miglustat. C) IV phenylalanine hydroxylase. D) Sapropterin hydrochloride. E) Carglumic acid. Discussion 25-2 The correct answer is "D:' Sapropterin hydrochloride (Kuvan) is FDA-approved for the treatment of PKU. It is a pharmaco­ logic form of the tetrahydrobiopterin cofactor for phenylala­ nine hydroxylase. Not all patients respond to sapropterin, but those who do can have significant decreases in their blood phenylalanine level. Gene therapy does not yet exist, and IV



M ETA B O L I C D I SO R D E R S

475

phenylalanine hydroxylase is not used, although investigation of enzyme replacement through other pathways is ongoing. Miglustat is substrate reduction therapy for disorders such as Gaucher disease. Carglumic acid is used in a rare urea cycle dis­ order called NAGS (N-acetylglutamate synthase) deficiency.

An 1 8-year-old patient presents to your practice after a long absence. He is known to have PKU and, after consulting your records, it is clear that he had reasonable adherence to diet at least until age 12, when social problems within his family resulted in poor dietary adherence. Since that time, he has been essentially lost to follow-up. He presents with a case worker as he has recently been released from a brief incar­ ceration for some petty crimes. He now carries diagnoses of bipolar disorder and attention deficient hyperactivity disor­ der and is being treated with methylphenidate, valproic acid, and sertraline. He is not following a low-phenylalanine diet. Question 26-1 What is the best next step in his medical management? A) Measure the patient's intelligence quotient (IQ) to assess level of cognitive functioning. B) Increase the dosage of sertraline and add a heterocyclic antidepressant. C) Check the patient's phenylalanine level and restart him on a low-phenylalanine diet. D) Obtain an MRI of the brain with and without contrast. E) Refer the patient to psychiatry for evaluation. Discussion 26-1 The correct answer is "C:' Surprisingly, this is not an unheard of scenario. Patients with PKU who are treated throughout the early childhood critical period escape the effects of intel­ lectual impairment that are a well-known consequence of the disorder. However, when the restrictive diet is stopped, patients frequently complain of poor focus, impulsiveness, mood swings, and psychiatric distress. When phenylalanine control is improved, patients report improvement in these symptoms and generally better functioning. Measuring this patient's IQ is unlikely to be helpful in the quest for improving his functioning and well-being as it is likely to be normal, albeit with decreases in attention and perhaps processing speed. Increasing the dos­ age of sertraline and adding another antidepressant or referral to psychiatry may be reasonable management strategies for a patient with treatment-resistant depression, but would not be the best first step in this case. MRI of the brain in a patient such as this may show white matter changes, but their discovery is unlikely to change management from the prescription for a low­ phenylalanine diet. Such white matter changes also commonly resolve when the patient's phenylalanine levels return to better control. The artificial sweetener aspartame contains phenylala­ nine and must be avoided by patients with PKU, which means no diet soda!

MCG RAW-H I LL E D U CATION S P E C I A LTY BOA R D REVI EW: P E D I ATRICS

476



Helpful Tip

=� PKU

is one of the prototypica l i n born errors of

r1 1r meta bolism, a n d is one of o u r fi rst treatment success

stories. It is clea r that d ieta ry i nterventions a re effective in mainta i n i ng the health of patients with PKU a n d also

clear that the d iet should idea l ly be m a i nta i ned for l ife. Patients who a re treated but su bseq uently go off

and can be inherited from either parent. Options "]>;' and "D" are neurodevelopmental disorders that would be present from infancy; hence, the patient would not have been previously normal. Option "B" describes the typical mutation for type II myotonic dystrophy. Option "E" does describe a mitochon­ drial disorder, but one that would present differently and most importantly is not consistent with the family history.

of d i et may experience d ifficu lty with focu s, increased i m p u l s iveness, and worsen i n g anxiety. The take-home message for the pri m a ry care physicia n is "diet for l ife."

An 8-year-old boy with previously normal development pres­ ents for care after an initial tonic-clonic seizure. He has had a poor appetite and frequent headaches, which have been resis­ tant to conventional therapies. Prior to these episodes, the child had been healthy. The examination is only significant for proximal muscle weakness. Metabolic acidosis is noted, with a lactic acid level of 3.9 mmol/L. Evaluation of a muscle biopsy sample shows ragged red fibers. The family history is significant for mother with type 2 diabetes mellitus and hear­ ing impairment. Question 27-1 What is the most likely molecular cause of the patient's findings? A) Deletion located at 1 p36. 1 . B ) Trinucleotide expansion at the DM2 gene. C) Missense mutation in the MT-TLl gene in the patient's mitochondrial (mt) DNA. D) Uniparental disomy of chromosome 14. E) Biallelic loss in the POLG gene in the patient's nuclear (n) DNA. Discussion 27-1 The correct answer is "C:' This patient has typical presentation for MELAS (Mitochondrial Encephalopathy, Lactic Acidosis and Stroke) syndrome, a mitochondrial disorder. The mitochon­ dria perform oxidative phosphorylation via their respiratory chain to produce adenosine triphosphate (ATP)-energy. In mitochondrial disorders, the respiratory chain is dysfunctional. This question appears on its face to be impossibly difficult, but there are several important clues in the case description. First, the patient presents with acute onset of neurologic complaints. Second, he has lactic acidosis and ragged red fibers, which are typical of mitochondrial disorders. Third, there is mention that his mother has unusual health concerns, including deaf­ ness. Fourth, he has proximal muscle weakness consistent with a myopathy. All of these point toward a mitochondrial disor­ der, and specifically one that is encoded in the mitochondrial genome as opposed to the nuclear genome. Remember, mtDNA is inherited exclusively from the mother so some disorders are maternally inherited. Others result from nDNA mutations

A 1 7-year-old boy presents to the clinic complaining of bilateral blurring of his central vision. He noted blurring initially in the right eye but did not alert his parents. Over the past week he has noticed onset of blurring in his left eye. He describes no pain with his vision loss. Examination by an ophthalmologist showed disk swelling but was otherwise normal. His maternal grandmother had onset of vision loss in her early 20s and is now legally blind, but no other family members have vision loss. Question 28-1 What is the most likely natural course for this patient's vision? A) The patient is unlikely to experience further vision loss. B) The patient is likely to experience further vision loss acutely, but full recovery is the rule. C) The patient is likely to experience slow vision loss over the next 10 to 20 years. D) The patient is likely to experience onset of sensorineural hearing loss along with vision loss. E) The patient will continue to experience further vision loss and will stabilize only with near-complete vision loss within a few months. Discussion 28-1 The correct answer is "E:' This patient described is experienc­ ing symptoms of Leber hereditary optic neuropathy (LHON), a disorder characterized by painless loss of vision that proceeds to near-complete loss of vision within a few months. This mater­ nally inherited mitochondrial disorder (mtDNA mutation) results in painless bilateral vision loss and typically presents in the teenage years. Most patients experience only vision loss, and males are approximately five times more likely than females to experience vision loss from this condition. It is unlikely that a patient would experience complete or even significant resolu­ tion of symptoms, and it is unlikely that hearing loss would be associated with LHON.

A 2-year-old boy who is developmentally delayed presents with new onset of self-injurious behavior, biting his lips and fingers, sometimes quite severely. He has been hypotonic since shortly after birth and has failed to gain significant motor milestones, with frequent dystonic movements. He was

C HAPTER 2 1

recently diagnosed with choreoathetoid cerebral palsy due to this constellation of findings. His mother notes that her older brother passed away at a young age without diagnosis. Question 29-1 Overproduction of which molecule could adequately explain this patient's clinical findings? A) Uric acid. B) Arginosuccinic acid. C) Ammonia. D) Lactic acid. E) Glycine. Discussion 29-1 The correct answer is "A:' This patient has findings that are typical of Lesch-Nyhan syndrome, including cardinal features of develop­ mental delay, movement disorder such as choreoathetosis or dysto­ nia, and self-injurious behavior, particularly biting behaviors. The latter often start in the second year of life. An X -linked disorder caused by mutations in the HPRT (hypoxanthine-guanine phos­ phoribosyltransferase) gene, Lesch-Nyhan syndrome leads to over­ production of uric acid from impaired purine metabolism. Patients with arginosuccinic aciduria, hyperammonemia, lactic acidemia, or hyperglycinemia generally will not present with movement dis­ orders or self-injurious behavior. Most disorders that would cause these findings would also be autosomal recessive in nature, and the affected maternal uncle is suggestive of an X-linked disorder. Do not forget that biting and self-injurious behaviors are big clues for Lesch-Nyhan syndrome.



M ETA B O L I C D I SO R D E R S

477

to organs including the brain, where copper-requiring enzyme function is impaired. In some patients, parenteral administration of copper has improved functional outcomes although this does not appear to be a universal phenomenon.





Helpful Tip

Both insufficient and excess metal del ivery to end organs

1 1 1r can be ha rmfu l to humans, and present with d ifferent clinica l featu res. Copper is an essentia l component of a

n u mber of importa nt metalloenzymes. Menkes d isease involves insufficient copper del ivery to tissues such as the brain, where the deficiency leads to neurodegeneration. Wilson d isease resu lts from inability to adequately excrete copper into the bile; this causes copper to be stored within tissues such as the liver, brain, and eye. The excess copper can produce movement disorders, psychiatric disease, hepatic fai l u re, and rena l tubular dysfu nction. Zinc is a cofactor in dozens of im portant enzymes and has important reg u latory fu nctions. Hered itary deficiency of zinc ca n result from mutations in genes responsible for absorption of zinc from the intestines. One

consequence

is acrodermatitis enteropathica,

which consists of diarrhea, infections, poor g rowth, and dermatitis occurring a round body orifices, pa rticula rly perianal. An excess of zinc, as seen in the condition hyperzi ncu ria with hypercalprotectinemia, can present with infections, hepatosplenomegaly, a rth ritis, anemia, and infla m matory reactions. Magnesium is essential for several chemical reactions within the body, and inborn errors of metabolism that cause hypomag nesemia can result in irrita bility and seizu res. Hypoca lcemia ca n also be va riably present and ca n result in teta ny and nephrocalci nosis.

An infant with Menkes disease may appear to be normal at birth, with manifestations of neurodegeneration such as seizures and hypotonia only becoming evident by about 3 months of age. However, even at birth if scalp hair (brittle, depigmented) is present it may exhibit the microscopic pili torti that gives Menkes disease its alternate moniker, kinky hair syndrome. Question 30-1 Disruption of the metabolism of which metal is responsible for this condition? A) Copper. B) Cobalt. C) Zinc. D) Nickel. E) Magnesium. Discussion 30-1 The correct answer is "A:' Although a large number of metals have important activities in the human body, Menkes disease is a dis­ order of copper metabolism. Mutations in the ATP7A gene result in failure of intracellular trafficking of copper, which prevents extracellular excretion. This results in decreased copper delivery

A 6-year-old boy with previously normal growth and devel­ opment presents with decreasing school performance and onset of erratic behavior over the past 4 months. A screening hearing examination conducted by the school nurse showed hearing loss that was not present on similar screening done 18 months earlier. His parents note that their son has seemed listless with poor weight gain. They also note that he contin­ ues to be very tan despite a long winter in Vermont. There is no family history of similar concerns in the family. A MRI scan of the brain is obtained. (See Figure 2 1 -7.) Question 31 -1 What is the most likely diagnosis for this patient? A) Glutaric aciduria type I. B) Metachromatic leukodystrophy. C) Infection with Lyme borreliosis. D) X-linked adrenoleukodystrophy. E) Miinchausen syndrome by proxy.

478

MCG RAW-H I LL E D U CATION S P E C I A LTY BOA R D REVI EW: P E D I ATRICS

A

8

c

F I G U R E 2 1 -7. ALD. Wh ite matte r changes a re seen on this bra i n M R I of a male patient with adrenoleukodystrophy. There is hyperintensity of the posterior periventric u l a r wh ite matter. The leukodystrophies a re a d iverse g ro u p of neurodegenerative d i sorders cha racterized by demye l i nation of the wh ite matter of the bra i n . (Reproduced with permission from Kasper DL, Fa uci AS, Hauser S L et a l : Harrison's Principles of Internal Medicine, 1 9th McGraw- H i l l Ed u cation, I n c., 201 S. Fig 44 1 E-56.)

Discussion 31 -1 The correct answer is "D:' This patient has presented with neuro­ logic deterioration caused by white matter changes to the brain. Even if nothing else is known about the condition, it could be narrowed to being a leukodystrophy. The leukodystrophies are a diverse group of disorders characterized by degeneration of the white matter of the brain. Disorders include vanishing white matter disease, Krabbe disease, Pelizaeus-Merzbacher disease, and Canavan disease. Patients with X-linked adreno­ leukodystrophy (X-ALD) also have adrenal insufficiency, with features that include poor weight gain and bronze skin from increased ACTH secretion. Lastly, the MRI obtained shows posterior periventricular white matter changes, which are typi­ cal of patients with X-ALD. Glutaric aciduria type I can present with neurologic changes, but they are more frequently related to movement. Metachromatic leukodystrophy (MLD) can cer­ tainly present with cognitive dysfunction, but ataxia is also a common presenting feature. Patients with MLD also do not generally have adrenal insufficiency as a presenting feature. Lyme disease would typically not present in this fashion, and Miinchausen syndrome by proxy would be a diagnosis of exclu­ sion in a patient with concerning health problems.

B I B LIOGRAPHY

Adam BW, Flores SR, Hou Y, Allen TW, De Jesus VR. Galactose- I- phosphate uridyltransferase dried blood spot quality control materials for newborn screening tests. Clin Biochem. 2 0 1 5;48(6) :437-442. Antenor-Dorsey JA, Hershey T, Rutlin J, et al. White matter integrity and executive abilities in individuals with phe­ nylketonuria. Mol Genet Metab. 20 1 3 ; 1 09(2) : 125- 1 3 1 . Aronica E , van Kempen AA, van der Heide M , et al. Congeni­ tal disorder of glycosylation type Ia: A clinicopathological report of a newborn infant with cerebellar pathology. Acta Neuropathol (Berl). 2005 ; 1 09:433-442.

Baris HN, Cohen IJ, Mistry PK. Gaucher disease: The meta­ bolic defect, pathophysiology, phenotypes and natural history. Pediatr Endocrinol Rev. 2 0 1 4; 1 2 (suppl l ) :72-8 1 . Bensend TA, Veach PM, Niendorf KB. What's the harm? Genetic counselor perceptions of adverse effects of genet­ ics service provision by non-genetics professionals. J Genet Couns. 2 0 1 4;23 ( 1 ) :48-63. Bley AE, Giannikopoulos OA, Hayden D, Kubilus K, Tifft CJ, Eichler FS. Natural history of infantile G(M2) gangliosi­ dosis. Pediatrics. 20 1 1 ; 128(5):el233-e l 24 1 . Bonnet D , Martin D , D e Lonlay P, e t al. Arrhythmias and conduction defects as presenting symptoms of fatty acid oxidation disorders in children. Circulation. 1 999; 1 00:2248-2253. Brunzell JD, Deeb SS. Familial lipoprotein lipase deficiency, apo CII deficiency and hepatic lipase deficiency. In: Scriver CR, Beaudet AL, Sly WS, Valle D, eds. The Meta­ bolic and Molecular Bases of Inherited Disease. 8th ed. New York, NY: McGraw-Hill; 200 1 :2789-2 8 1 6 . Channon S, Goodman G, Zlotowitz S, Mockler C , Lee PJ. Effects of dietary management of phenylketon­ uria on long-term cognitive outcome. Arch Dis Child. 2007;92:2 1 3-2 1 8 . Clow CL, Laberge C , Scriver CR. Neonatal hypertyrosinemia and evidence for deficiency of ascorbic acid in Arctic and subarctic people. Can Med Assoc f. 1 975; 1 1 3 (7):624-626. Croffie JM, Gupta SK, Chong SK, Fitzgerald JF. Tyrosinemia type 1 should be suspected in infants with severe coagu­ lopathy even in the absence of other signs of liver failure. Pediatrics. 1 999; 1 03:675-678. de Sain-van der Velden MG, Diekman EF, Jans JJ, et al. Dif­ ferences between acylcarnitine profiles in plasma and bloodspots. Mol Genet Metab. 20 1 3; 1 1 0 ( 1 -2) : 1 1 6- 1 2 1 . Dionisi-Vici C , Deodato F, Roschinger W, Rhead W, Wilcken B. 'Classical' organic acidurias, propionic aciduria, meth­ ylmalonic aciduria and isovaleric aciduria: Long-term outcome and effects of expanded newborn screening

C HAPTER 2 1

using tandem mass spectrometry. J Inherit Metab Dis. 2006;29:383-389. Fernandes J, Saudubray J-M, van den Berghe G, Walter JH. Inborn Metabolic Disease. 4th ed. Heidelberg: Springer; 2006. Ficicioglu C, Hussa C, Gallagher PR, Thomas N, Yager C. Monitoring of biochemical status in children with Duarte galactosemia: Utility of galactose, galactitol, galactonate, and galactose 1 -phosphate. Clin Chern. 20 1 0;56(7) : 1 1 77- 1 1 82. Hennermann JB, Berger JM, Grieben U, Scharer G, Van Hove JL. Prediction of long-term outcome in glycine encephalopathy: A clinical survey. J Inherit Metab Dis. 20 12;35:253-26 1 . Hussain K , Aynsley-Green A , Stanley CA. Medications used in the treatment of hypoglycemia due to congenital hyperinsulinism of infancy (HI) . Pediatr Endocrinol Rev. 2004;2(suppl 1 ) : 1 63 - 1 67. Kaback MM, Desnick RJ. Hexosaminidase A deficiency. ( 1 999 Mar 1 1 [updated 20 1 1 Aug 1 1 ] .) In: Pagon RA, Adam MP, Ardinger HH, et al, eds. GeneReviews [Internet] . Seattle, WA: University of Washington, Seattle; 1 993-20 14. http:// www.ncbi.nlm.nih.gov/books/NBK1 2 1 8/. Kaler SG, Liew CJ, Donsante A, Hicks JD, Sato S, Green­ field JC. Molecular correlates of epilepsy in early diag­ nosed and treated Menkes disease. f Inher Metab Dis. 20 1 0;33:583-589. Kasapkara c;:s, Cinasal Demir G, Hasanoglu A, Turner L. Continuous glucose monitoring in children with glycogen storage disease type I. Bur J Clin Nutr. 20 14;68( 1 ) : 1 0 1 - 105. Lucia A, Ruiz JR, Santalla A, et al. Genotypic and phenotypic features of McArdle disease: Insights from the Spanish national registry. J Neural Neurosurg Psychiatry. 20 12;83(3) :322-328. Muenzer J, Wraith JE, Clarke LA. Mucopolysaccharidosis I: Management and treatment guidelines. Pediatrics. 2009; 1 2 3 : 1 9-29. Nesbitt V, Pitceathly RD, Turnbull DM, et al. The UK MRC mitochondrial disease patient cohort study: Clinical phenotypes associated with the m.3243A>G mutation­ Implications for diagnosis and management. J Neural Neurosurg Psychiatry. 2 0 1 3;84:936-938. Nordestgaard BG, Chapman MJ, Humphries SE, et al; Euro­ pean Atherosclerosis Society Consensus Panel. Familial hypercholesterolaemia is underdiagnosed and under­ treated in the general population: Guidance for clini­ cians to prevent coronary heart disease. Bur Heart f. 20 1 3;34:34 78-3490. Orteu CH, Jansen T, Lidove 0, et al. Fabry disease and the skin: Data from FOS, the Fabry outcome survey. Br J Dermatol. 2007; 1 57:33 1 -337. Ortiz RG, Newman NJ, Shoffner JM, Kaufman AE, Koontz DA, Wallace DC. Variable retinal and neurologic manifesta­ tions in patients harboring the mitochondrial DNA 8993 mutation. Arch Ophthalmol. 1 993; 1 1 1 : 1 525- 1 530. Parini R, Corbetta C. Metabolic screening for the newborn. J Matern Fetal Neonatal Med. 20 1 1 ;24(suppl 2) :6-8.



M ETA B O L I C D I SO R D E R S

479

Parviz M, Vogel K, Gibson KM, Pearl PL. Disorders of GABA metabolism: SSADH and GABA-transaminase deficien­ cies. J Pediatr Epilepsy. 20 1 4;3 (4) : 2 1 7-227. Petraitiene I, Barauskas G, Gulbinas A, et al. Congenital hyperinsulinism. Medicina (Kaunas). 2 0 1 4;50(3): 1 90- 1 9 5 . Picca S, Bartuli A , Dionisi-Vici C. Medical management and dialysis therapy for the infant with an inborn error of metabolism. Semin Nephrol. 2008;28(5) :477-480. Pierro A, Nah SA. Surgical management of congenital hyperinsulinism of infancy. Semin Pediatr Surg. 20 1 1 ;20( 1 ) :50-53. Prater SN, Banugaria SG, Dearmey SM, et al. The emerging phenotype of long-term survivors with infantile Pompe disease. Genet Med. 20 1 2 ; 1 4:800- 8 1 0 . Robey KL, Reck JF, Giacomini KD, Barabas G, Eddey GE. Modes and patterns of self-mutilation in persons with Lesch-Nyhan disease. Dev Med Child Neural. 2003;45: 1 67- 1 7 1 . Robinson JG. Management of familial hypercholesterolemia: A review of the recommendations from the National Lipid Association Expert Panel on Familial Hypercholes­ terolemia. J Manag Care Pharm. 20 1 3 ; 1 9(2) : 1 39- 149. Santosh Rai PV, Suresh BV, Bhat IG, Sekhar M, Chakraborti S. Childhood adrenoleukodystrophy-Classic and variant-Review of clinical manifestations and magnetic resonance imaging. J Pediatr Neurosci. 2 0 1 3;8(3) : 1 92- 1 97. Saudubray JM, Martin D, de Lonlay P, et al. Recognition and management of fatty acid oxidation defects: A series of 107 patients. f Inherit Metab Dis. 1 999;22(4):488-502. Scarpa M, Almassy Z, Beck M, et al; Hunter Syndrome European Expert Council. Mucopolysaccharidosis type II: European recommendations for the diagnosis and multidisciplinary management of a rare disease. Orphanet J Rare Dis. 20 1 1 ;6:72. Scriver CR, Kaufman S. Hyperphenylalaninemia: Phenylala­ nine hydroxylase deficiency. In: Scriver CR, Beaudet AL, Sly SW, Valle D, eds. The Metabolic and Molecular Bases of Inherited Disease. 8th ed. New York, NY: McGraw-Hill; 200 1 : 1 667-724. Seashore MR. The organic acidemias: An overview. (200 1 Jun 27 [updated 2009 Dec 22] .) In: Pagon RA, Adam MP, Ardinger HH, et al, eds. GeneReviews [Internet] . Seattle, WA: University of Washington, Seattle; 1 993-20 14. http:// www.ncbi.nlm.nih.gov/books/NBK 1 1 34/. Strauss KA, Morton DH. Branched-chain ketoacyl dehydroge­ nase deficiency: Maple syrup disease. Curr Treat Options Neural. 2003;5( 4):329-34 1 . Strauss KA, Morton DH. Type I glutaric aciduria, part 2 : A model of acute striatal necrosis. A m J Med Genet. 2003; 1 2 1 C:53-70. Summar M, Tuchman M. Proceedings of a consensus con­ ference for the management of patients with urea cycle disorders. J Pediatr. 200 1 ; 1 38:S6- 1 0 . Tomatsu S, Shimada T, Mason RW, e t al. Establishment of glycosaminoglycan assays for mucopolysaccharidoses. Metabolites. 20 1 4;4( 3 ):655-679.

480

MCG RAW-H I LL E D U CATION S P E C I A LTY BOA R D REVI EW: P E D I ATRICS

Tuchman M, Lee B, Lichter-Konecki U, et al; Urea Cycle Dis­ orders Consortium of the Rare Diseases Clinical Research Network. Cross-sectional multicenter study of patients with urea cycle disorders in the United States. Mol Genet Metab. 2008;94( 4):397 -402. Vockley J, Andersson HC, Antshel KM, et al; American College of Medical Genetics and Genomics Therapeu­ tics Committee. Phenylalanine hydroxylase deficiency: Diagnosis and management guideline. Genet Med. 20 14; 1 6(2): 1 88-200.

Waggoner DD, Buist NR, Donnell GN. Long-term prognosis in galactosaemia: Results of a survey of 350 cases. J Inherit Metab Dis. 1 990; 1 3:802-8 1 8 . Waisbren SE, Potter NL, Gordon CM, e t al. Th e adult galactos­ emic phenotype. J Inherit Metab Dis. 2 0 1 2;35:279-286. Wolf B. The neurology of biotinidase deficiency. Mol Genet Metab. 20 1 1 ; 1 04:27-34. Yu-Wai-Man P, Griffiths PG, Hudson G, Chinnery PF. Inher­ ited mitochondrial optic neuropathies. J Med Genet. 2009;46: 1 45- 1 58.

22

M uscu l oske l eta l D isorders Nata l i e Sto rk a n d B l a ise N e m eth

A 14-month-old boy presents with refusal to bear weight on his right leg after falling off the bed. X-rays demonstrate a midshaft femur fracture. Medical records demonstrate prior visits for a wrist fracture and a tibial fracture over the past 3 months. Physical examination reveals multiple bruises. Skel­ etal survey demonstrates healing right distal radius and right spiral tibial fractures, but no other healing or new fractures. Head magnetic resonance imaging (MRI) does not reveal any intracranial injury and there are no retinal hemorrhages. Question 1 - 1 Which of the following findings is least consistent with a pos­ sible diagnosis of osteogenesis imperfecta? A) Blue sclera. B) Dental caries. C) Negative family history. D) Low vitamin D level. E) Short stature. Discussion 1 -1 The correct answer is "D:' Concerns regarding osteogenesis imperfecta are frequently entertained in cases of unusual or fre­ quent fractures and may be mistaken for nonaccidental trauma. Numerous genetic causes have been identified, most com­ monly in the formation of bone involving collagen (COLlA l , COL1A2, and others) . Vitamin D levels are normal. Inheritance patterns of the genetic abnormalities include autosomal domi­ nant, recessive, and X-linked forms, but current classification remains based on phenotype. Type I, or nondeforming, is the mildest, and patients typically achieve normal height. Type II is lethal in the perinatal period. Type III often presents at birth with frequent fractures and results in progressive deformity, and type IV is the moderate form. Type V involves ossification of the interosseous membrane, hypertrophic callus formation, or both. Blue sclera are common in types I and III, but indi­ viduals with type IV often have white sclera. Patients often have

triangular facies and macrocephaly, and may have hearing loss. Short stature, easy bruising, and joint hypermobility may also occur. X-rays often demonstrate osteopenia or wormian bones of the skull.



Helpful Tip

::5.� Although

osteog enesis i m perfecta is pa rt of the

i1 1r d ifferential diag nosis i n t h e etiology of fractu res i n

i nfa nts a n d c h i l d ren, it is i m porta nt t o remem ber that

c h i l d ren with osteog enesis i m perfecta may sti l l be victims of c h i l d a b u se.

A 6-month-old girl presents as a new patient for a well-child check. Her length is below the third percentile and her weight is at the 1 Oth percentile. Head circumference is greater than the 90th percentile. Her mother is 5 feet, 4 inches tall (50th percentile), and her father is 5 feet, 8 inches (25th percentile) . She has normal eyes, a prominent forehead, and a prominent curve to her spine. She also demonstrates mild hypotonia. Question 2-1 The most likely diagnosis is: A) Achondroplasia. B) Osteogenesis imperfecta. C) Klippel-Feil syndrome. D) Trisomy 2 1 . E ) Familial short stature. Discussion 2-1 The correct answer is "A:' Achondroplasia is a skeletal dyspla­ sia arising from a defect in fibroblast growth factor receptor 3 (FGFR3) . Patients are of short stature with normal height through the trunk but shortening of the extremities. Although fingers may be short, the most pronounced shortening is of the upper 48 1

MCG RAW-H I LL E D U CATION S P E C I A LTY BOA R D REVI EW: P E D I ATRICS

482

arms and thighs (rhizomelic shortening) . Macrocephaly with frontal bossing is often present, and midface hypoplasia pre­ disposes to otitis media and obstructive apnea. Abnormalities at the base of the skull and cervical spine predispose to hydro­ cephalus, cervical instability, hypotonia, and central apnea. Orthopedic manifestations include excessive thoracic kyphosis, hyperlordosis, spinal stenosis, and bowed legs, often requir­ ing surgical correction. Patients often have ulnar deviation of the fifth digits creating the appearance of a "trident hand:' Osteogenesis imperfecta is a genetic syndrome associated with frequent fractures, and while short stature may be present in more severe forms, the other findings are atypical. Klippel-Feil syndrome involves congenital fusion of the cervical spine and a short, webbed neck. Patients with trisomy 2 1 have short stat­ ure, short fingers, and hypotonia but demonstrate other charac­ teristic findings (see Chapter 1 7) . This patient's length is below the fifth percentile, much lower than predicted by midparental height, making familial short stature less likely, especially when taking the other clinical findings into consideration.



in certain genetic syndromes, such as diastrophic dysplasia or Larsen syndrome. In addition to the multiple joint contractures, which can include hand and foot deformities as well as dislocate hips, amyoplasia is characterized by cylindrical-shaped limbs with lack of flexion creases, dimpling at joints, and normal cognitive function. Difficulty swallowing and breathing may occur at birth. Abnormal cognitive development would suggest an underlying neurologic abnormality as the cause of multiple joint contractures.

� •

1 I

Helpful Tip

Arth rogryposis is the c l i n ical fi n d i n g of joint contractu res in m u ltiple jo i nts. I d entifying a n u n derlyi ng cause is i m porta nt i n d eterm i n i ng associated m edi­ cal issues, prog nosis a n d g u i d i n g treatment for the patient a n d fa m i ly.

Helpful Tip

=til Other

bone dysplasias, such as m u ltiple epi physea l

r1 1r dyspla sia, spondyloepiphysea l dysplasia, a n d cam pto­ melic dyspla sia, may m a n ifest with short statu re, l i m b

bowing, o r s p i n e deform ities. Genetics eva l uation i s helpfu l i n making a d efi n itive diag nosis o f t h e specific dysplasia.

A newborn male is found to have hyperextended knees and bilateral clubfeet. Both hips are flexed and have limited abduction; the spine appears straight and without any cuta­ neous lesions. The pregnancy was unremarkable, other than decreased fetal movement on prenatal ultrasounds. Question 3-1 All of the following would be consistent with a diagnosis of amyoplasia EXCEPT: A) Cognitive delay. B) Lack of flexion creases at joints. C) Bilateral hip dislocations. D) Difficulty swallowing. Discussion 3-1 The correct answer is "A:' Arthrogryposis is the presence of congenital joint contractures, usually associated with decreased fetal movement due to either a restricted intrauterine environ­ ment or intrinsic muscular or neurologic disorders, and occurs in approximately 1 in 1 0,000 live births. The most common cause is amyoplasia, an abnormality of muscle development often referred to as arthrogryposis multiplex congenita. Arthro­ gryposis may also occur in patients with abnormalities of the central nervous system, in the presence of oligohydramnios, or

A 4-month-old boy with torticollis returns to clinic for a well­ child check. His head is preferentially held in a tilt to the left and rotation to the right. He also has flattening of the right occiput. Shaking keys to the right and left side reveals that he can rotate 90 degrees to the right and 60 degrees to the left. He has a palpable mass within the left sternocleidomastoid muscle. His parents have tried adjustments to his home envi­ ronment, consisting of laying him in his crib with his right side toward the wall and a music box to his left, to stimulate him to turn more to his left side. Question 4-1 What is the most appropriate next step in management? A) Continued home management. B) Referral for physical therapy. C) MRI evaluation of the mass. D) Referral to pediatric orthopedics. E) Referral to orthotics. Discussion 4-1 The correct answer is "B:' Patients with congenital muscu­ lar torticollis have tightness within the sternocleidomastoid muscle thought to be secondary to in utero positioning. This muscle tightness results in a tilt of the head toward the side of tightening and rotation to the contralateral side. Presentation is typically between 2 weeks and 2 months of life. Initially, treat­ ment with modification of the home environment is appropri­ ate. If there is no improvement over 2 to 4 months, then referral to physical therapy is most appropriate. Occasionally infants will have a palpable mass within the sternocleidomastoid mus­ cle; the etiology of this "tumor" is unknown, but since it appears fibrotic on microscopic analysis, theories include arterial occlu­ sion or venous congestion from in utero positioning causing avascularity of the muscle or potentially even primary fibrosis of the muscle. Ultrasound, not MRI, is the preferred initial imaging

C H A PT E R 22

modality as the mass is in proximity to the skin surface and this modality does not require sedation. Ultrasound should reveal a heterogeneous mass within the sternocleidomastoid muscle; MRI might be warranted if there was an atypical appearance on ultrasound. Presence of a "tumor" may suggest decreased response to physical therapy, but since surgical release of the sternocleidomastoid typically does not occur until after 1 year of age, referral to orthopedics is usually not necessary unless the infant fails to improve with physical therapy over 2 to 4 months. Plagiocephaly often occurs in the presence of torticollis, and cranial remolding orthoses are a consideration, but it can often be prevented with proper physical therapy and resolution of the torticollis.

A mother presents with her 6-month-old son whose head is tilted to the right and rotated to the right. She first noticed this abnormal positioning at 2 months of age, and it has not changed. He has developed some flattening of the right occiput. No esotropia, exotropia, or nystagmus is noted. His neck appears shortened and his hairline low. His right scapula is smaller than the left and slightly elevated. Cervical spine X-rays are as shown. (See Figure 22- 1 . ) Question 5- 1

What is the most likely diagnosis? A) Congenital muscular torticollis. B) Klippel-Feil syndrome. C) Ocular torticollis.



M U S C U LOSKELETAL D I S O R D E RS

483

D) Benign paroxysmal torticollis. E) Sandifer syndrome. Discussion 5-1 The correct answer is "B:' Klippel-Feil syndrome encompasses the combined findings of shortened neck with low hairline in the presence of cervical vertebral fusions. (See Figure 22- 1 . ) Approx­ imately 20% of patients have Sprengel deformity, with a small, elevated scapula due to incomplete somite migration during fetal development. The typical presentation of congenital muscular torticollis is an infant with a tilted head that is rotated to the con­ tralateral side due to tightness within the sternocleidomastoid muscle on the side of the tilt. In ocular torticollis, children with strabismus or amblyopia tilt their head to reduce the contribution of the eye with abnormal vision. Benign paroxysmal torticollis is a rare cause of torticollis, thought to be a migraine-related disor­ der based on family history, that occurs every few days to months and resolves completely between episodes. Sandifer syndrome is posturing of the torso and turning of the head related to gastro­ esophageal reflux disease in the presence of a hiatal hernia. Other causes of torticollis include trauma, malignancy, and inflamma­ tion in the soft tissue structures adjacent to the vertebra, such as retropharyngitis or lymphadenitis (Grisel syndrome).





Helpful Tip

If a patient with torticol l i s does not i m p rove after 2

1 1 1r to 4 months, obta i n anteroposterior (AP) a n d latera l

cervica l X-rays to eva l u ate for vertebra l body a n o m a l ies of Kli ppei-Fei l . If Cl is not wel l visua l ized due to the

ti lted head from the tortico l l is, a n X-ray of the latera l sku l l may be h e l pfu l .





1 1 1r

Helpful Tip

Cervica l spine abnormal ities, whether d u e to fusion or congenital i nsta b i l ity, may result i n exclusion from certa i n sports or activities, depen d i n g on the abnormal ity a n d level of activity.

A 12-month-old girl presents with her parents who are con­ cerned that she is standing "crooked:' She has just started walking and they have no concerns regarding her gait. Exam­ ination reveals that her spine is curved. X-rays reveal mul­ tiple vertebral body anomalies, including hemivertebrae and butterfly vertebrae.

F I G U R E 22- 1 . Latera l cervical spine X-ray. Note fusions at C3-4 posteriorly (block arrow). (Used with permission fro m Blaise Nemeth, M D, MS.)

Question 6-1 All of the following should be performed EXCEPT: A) Renal ultrasound. B) Echocardiogram. C) Referral to pediatric orthopedics. D) Referral to pediatric neurology.

484

MCG RAW-H I LL E D U CATION S P E C I A LTY BOA R D REVI EW: P E D I ATRICS

She reports the pregnancy was uncomplicated and the delivery went well. Apgar scores were 8 and 9 at 1 and 5 minutes, respectively. The newborn's mother and father are concerned because his feet are turned in. They do not remember their first child's feet looking this way after delivery. Mom states that the night nurse mentioned it was likely caused by the way he was positioned in utero. You begin your newborn exam; as you approach the feet, you notice the toes on both feet point inward. Question 7-1 Which of the following clinical findings define(s) congenital clubfoot? A) Metatarsus adductus. B) Hindfoot varus. C) Rigid equinus. D) Cavus. E) All of the above.

F I G U R E 22-2. Posteroanterior (PA) scoliosis X-ray i n a n i nfa nt demonstrating m u ltiple seg mentation and fusion abnormal ities of the u pper thoracic spine i n a patient with congenital scoliosis secondary to vertebral ma lformations. (Used with permission from Blaise Nemeth, M D, MS.)

Discussion 6-1 The correct answer is "D:' Congenital scoliosis encompasses curves of the spine associated with vertebral abnormalities due to failures of formation or segmentation of the vertebral bodies. (See Figure 22-2.) Since the renal and cardiac systems develop at the same time as the spine (eg, VACTERL association) , patients with congenital scoliosis should undergo evaluation with renal ultrasound and echocardiogram. Spinal dysraphism, such as syrinx or tethered cord, may also occur in the presence of vertebral body anomalies. Referral to neurology is not neces­ sary, but referral to neurosurgery would be appropriate if there are clinical concerns or findings on MRI. MRI may be indicated if there are abnormal neurologic findings; otherwise, necessity may be left to the discretion of the consulting pediatric orthope­ dist. The curve should be followed for progression necessitating surgery, which may occur at any point during growth.

Discussion 7-1 The correct answer is "E:' Congenital clubfoot (talipes equin­ ovarus) is a complex deformity affecting the bones and soft tissues of the lower extremity. It is clinically defined by four characteristic findings: cavus, metatarsus adductus, hindfoot varus, and rigid equinus. (See Figure 22-3.) The etiology of clubfoot is not well understood but it is thought to be multifac­ torial, with both genetic and environmental influences. Clubfoot can be idiopathic or associated with other neuromuscular diag­ noses, with an incidence around 1 in 1 000 live births. Prenatal ultrasounds have detected the deformity as early as 12 weeks'

>
5 em) . Amputation or early prosthetic fitting is considered for severe discrepancies with a proj ected difference greater than 20 em.





Helpful Tip

The d ifferentia l diag nosis for leg-length d i screpa ncy is

1 1 1r q u ite wide; thus, a thorough h i story a n d physical m u st be cond ucted to identify potential organic ca uses. Leg­

length d i screpa ncies up to 1 em a re fa i rly commo n a n d often id iopathic. Discrepa ncies l ess than 2 em a re fa i rly wel l tolerated with a s i m p l e shoe l ift correcting half the d ifference (up to a 1 e m shoe l ift ca n be comforta bly

F I G U R E 22-5. T2-weig hted M R I scan demo nstrating edema with i n the meta physis and epiphysi s of the fem u r (bright wh ite). (Used with permission from Blaise Nemeth, M D, M S.)

tolerated inside a shoe).

A 2-year-old boy presents with a limp. His parents first noticed it about 5 days ago, and it has progressively worsened since. He has become increasingly fussy with weight bearing. Today he developed a temperature of 37.9°C ( 1 00.2°F) . On exam, he has limited flexion of his right knee and is tender over the distal femur. The knee is not swollen or warm to touch. X-rays are unremarkable. CBC reveals a white blood cell (WBC) count of 10,1 00/mm3 with 70% neutrophils, C-reactive protein (CRP) of 7, and erythrocyte sedimenta­ tion rate (ESR) of 29. Question 1 6-1 What is the most appropriate next step in management? A) Observation. B) MRI scan. C) Ultrasound. D) CT scan. Discussion 1 6-1 The correct answer is "B:' These findings are concerning for possible infection. Children may experience infections of the musculoskeletal system involving bones (osteomyelitis) , septic arthritis, or muscle (pyomyositis) . With lack of joint swelling,

osteomyelitis would be most likely, although early septic arthri­ tis may not present with clinically detectable swelling. Early in the course of osteomyelitis, X-rays may be negative. MRI is the most helpful next imaging tool, if an area of concern can be localized, to identify early osteomyelitis, demonstrating edema and early destruction within the bone. (See Figure 22-5). A bone scan may be helpful if an area of involvement cannot be identified. Ultrasound is useful for imaging soft tissue struc­ tures, including evaluation for joint effusions, especially if there is concern for septic arthritis, and to guide aspirations, but is less helpful in evaluating bone. CT scan works well to high­ light bone abnormalities but is more helpful in trauma than when infection is suspected. Treatment of osteomyelitis usually requires only antibiotics, initially intravenous (IV), followed by oral therapy, and surgery is required only when there is an area of focal abscess (see Figure 22-6) or associated septic arthritis. Osteomyelitis often involves the metaphysis of the long bones, occurring as the result of hematogenous spread, although eruption into the joint may occur, most commonly in the proximal femur or hip, or in infants. Infectious organisms may be identified by direct culture of the infected area or on blood culture. Resolution of changes on MRI can take weeks to months, so repeat imaging with MRI is not helpful to evaluate for resolution of infection; following declines in WBC and ESR is more helpful, reserving repeat MRI for patients who are clini­ cally worsening, which may raise concern for potential subperi­ osteal abscess requiring surgical irrigation and debridement.

MCG RAW-H I LL E D U CATION S P E C I A LTY BOA R D REVI EW: P E D I ATRICS

492

F I G U R E 22-6. T2-weighted a x i a l image (left) and Tl -weig hted coronal i mage (rig ht) o f periostea l a bscess related t o dista l fi b u l a r osteomyel itis. (Used with permission from Matthew Halanski, M D.)





Helpful Tip

X-rays a re often negative in patients presenting with

1 1 1 r osteomyel itis. MRI

may be necessa ry for diag nosis if

X-rays a re negative. M R I abnormal ities ta ke months to resolve, so seri a l M R i s a re not i n d i cated in patients with osteomyel itis who a re i m p rovi ng based on c l i n ical a n d laboratory para meters.

A 1 3-year-old girl presents with a swollen right knee. She first noticed it 2 days ago. She will walk, but she limps and appears uncomfortable. Her temperature is 38°C ( 1 00.4°F ) . Her right knee is swollen and slightly warm to touch. She cries with attempts to flex the knee. X-rays are negative. CBC demonstrates WBC 1 5,000/mm3 with 82% neutrophils, CRP of 1 0, and ESR of 45. She is taken to the operating room for suspected septic arthritis. Question 1 7-1 What is the most likely causative organism? A) Staphylococcus aureus. B) Streptococcus pyogenes. C) Neisseria gonorrhoeae. D) Kingella kingae. E) Salmonella. Discussion 1 7-1 The correct answer is "A:' Pyogenic arthritis arises from bacte­ rial infection of the j oint. This can result from direct hema­ togenous seeding of the j oint or when osteomyelitis occurs in

an intra-articular metaphysis and ruptures through the bone into the j oint. Diagnosis is an urgency as j oint infection results in j oint destruction within 3 to 5 days and permanent long­ term sequelae. (See Figure 22-7.) The greatest yield in recover­ ing an organism includes performing blood culture and j oint aspirate. Ultrasound may be helpful in identifying j oint effu­ sions, especially in the hip, and to guide aspiration. Treatment includes emergent irrigation and debridement of the j oint in the operating room and initiation of antibiotic therapy. The most common causative organism in both osteomyelitis and septic arthritis is staphylococcus aureus. Methicillin-resistant strains are increasing in prevalence, so the preferred antibi­ otic treatment of choice typically varies based on regional prevalence. In areas where methicillin-sensitive strains pre­ vail, initiation of treatment may begin with nafcillin, but in methicillin-resistant areas, vancomycin is preferred, although clindamycin may be reasonable if strains are D-test negative. S. pyogenes (group A beta-hemolytic streptococcus) is another common cause of septic arthritis, but less frequent than S. aureus. N. gonorrhoeae is a potential causative organism in sexually active individuals that is treated with penicillin; patients may experience polyarthralgia before onset of arthri­ tis. K. kingae is an emerging cause of septic arthritis, primarily in children under 3 years of age, that may make up many of the reported "culture-negative" cases of septic arthritis as it is a fastidious organism that may be missed on routine cultures; recovery is improved by use of culture bottles. Salmonella is a common cause of musculoskeletal infection in individuals with sickle cell disease. Infections from historically significant organisms, such as Streptococcus pneumoniae and Haemophi­ lus influenza type b have been reduced by vaccinations but represent important considerations in unimmunized children or infants too young to receive immunizations.

C H A PT E R 22



M U S C U LOSKELETAL D I S O R D E RS

493

any trauma, and she has not had any fevers. They have not noticed any swelling. Physical examination reveals limited motion at her right hip. There is no focal tenderness through­ out her thigh, lower leg, or foot. There is no swelling at her knee. X-rays of her femur, tibia, and foot are normal. Question 1 8-1 What is the most appropriate next step in evaluation and management? A) Hip ultrasound with aspiration. B) Continued observation. C) Consultation with pediatric orthopedics. D) CBC with differential and ESR. E) Lyme titer

F I G U R E 22-7. Ful l-length orthoroentgenogram of patient with a history of right d i sta l femoral osteomye l itis a n d su bseq uent septic a rth ritis resulting i n osteonecrosis of the medial fem u r. As a resu lt, this patient has developed

g rowth arrest and va rus deformity. Fortu nately, the patient presented u pon development of the septic arthritis and was treated i m mediately, preventing osteonecrosis of the rest of the joint (lateral fem u r and prox i m a l tibia). (Used with permission from Blaise Nemeth, M D, M S.)



Helpful Tip

� While

5 . aureus

is the most common ca use of

r1 1r musculos keleta l infections in c h i l d ren, consider other o rg a n i s m s when performing testing a n d i n itiati ng

a ntibiotics to i m p rove d etection a n d coverage in patients with a history of travel, other medical issues, o r incomplete i m m u n ization status.

A 2-year-old girl presents with a limp and decreased weight bearing on her right leg. This was first noticed by her par­ ents this morning after she awoke. Her parents do not recall

Discussion 1 8-1 The correct answer is "D:' Children with a limp present a difficult diagnostic challenge owing to the broad differential diagnosis, which ranges from the benign to the gravely concerning. A sys­ tematic approach is helpful in evaluating these patients, starting with a detailed examination to localize the area of involvement. X-rays are helpful in assessing for fractures, tumors, or develop­ mental orthopedic issues, such as Legg-Calve-Perthes disease or late-presenting developmental dysplasia of the hip, which may manifest with limited hip motion. When X-rays are negative, the two most common causes of hip pain in a child include septic arthritis and transient (ie, toxic) synovitis. The physical exam in both cases is remarkably similar, but differentiation is important as untreated septic arthritis can cause significant destruction of the joint within 4 to 5 days; therefore, timely diagnosis is para­ mount. Kocher described four findings, originally determined retrospectively but validated prospectively, that help differentiate septic arthritis from transient synovitis: fever (temperature > 38°C [ 1 00.4°F] ) , refusal to walk, WBC count greater than 12,000/mm3 and ESR greater than 40. Patients with none of these criteria were unlikely to have septic arthritis, whereas those with three or four were very likely to have septic arthritis. It is important to remem­ ber that the Kocher criteria are only part of the decision-making process, and subsequent prospective studies have shown that patients with none of the criteria may have as much as a 1 6.9% chance of having septic arthritis. Therefore, the criteria should be used in the context of the remainder of the history, exami­ nation, and clinical concern. As a result, CBC would be helpful in guiding decision making in this patient. If septic arthritis is a concern, regardless of Kocher criteria, then ultrasound of the hip with aspiration of any detectable fluid for definitive diagno­ sis may be indicated. Pyogenic arthritis typically contains greater than 50,000 WBCs/mm3 and requires treatment with irrigation and debridement of the joint, as well as antibiotics. An orthope­ dist should be consulted emergently for treatment of the infected joint or to assist in diagnosis in the emergency department. Transient synovitis occurs primarily in the hip, and the etiology is unknown. In contrast to reactive arthritis or inflammatory arthritis from juvenile idiopathic arthritis, transient synovitis will resolve spontaneously without any treatment other than support­ ive measures, so observation may be appropriate once clinical

MCG RAW-H I LL E D U CATION S P E C I A LTY BOA R D REVI EW: P E D I ATRICS

494

concern for septic arthritis has resolved. Occasionally, initial pre­ sentation of Legg-Calve-Perthes disease may be misdiagnosed as transient synovitis, as X-rays may be negative, but recurrence of limp and subsequent development of radiographic changes of the femoral head ultimately result in diagnosis. Lyme disease, caused by Borrelia burgdorferi, transmitted by the Ixodes tick, is a consid­ eration in endemic areas; arthritis is a late manifestation occur­ ring months after inoculation and is typically nonpainful.



� I

Helpful Tip

Kocher criteria (inabil ity to a m bu l ate, T > 3 8°C [ 1 00.4°F],

1 1 r WBC

>

1 2,000, ESR

>

40) assist in g u i d i n g eva l uation

but should not d i ctate decision making i n patients

with history a n d exa m fi n d i n g s that a re concern i n g for septic a rth ritis versus tra nsient synovitis. Defi n itive diag nosis is based on u ltrasou nd identification of effusion, joint aspirate, a n d resu lts from microscopy and culture of syn ovial fl u i d i n concern i n g cases.

syndrome can result in irreversible damage to the associated nerves and muscles. This damage can occur in 4 to 6 hours and thus is a medical emergency. Fractures are the most common injury associated with acute compartment syndrome; other risk factors include crush injury, restrictive splints or casts follow­ ing a fracture or manipulation of a fracture, and vascular injury. Specifically, supracondylar fractures and tibial shaft fractures are the most common fractures associated with compartment syndrome. Pain out of proportion to the injury or pain requir­ ing increasing analgesia is classically an early symptom and the most sensitive symptom of acute compartment syndrome. Other symptoms include a tense swollen extremity, paraesthe­ sias, pallor, paralysis, and pulselessness. However, many of these are late findings and indicate vascular compromise. Diagnosis requires a high suspicion for acute compartment syndrome and can often be made clinically in the older, alert, reliable patient. However, in younger or obtunded patients, intracompartmental pressures may be required to confirm the diagnosis. Treatment requires emergent fasciotomy of the affected compartment(s).



Helpful Tip

=t'Jl One should You are moonlighting in the emergency department when a 16-year-old boy presents with a left leg injury. He was play­ ing in a football game earlier in the day and ended up at the bottom of the pile. He states his leg was planted while he was twisted underneath a large pile. He heard and felt a large crack and developed immediate onset of pain over the left shin and lower extremity. There was visible deformity and he was unable to bear weight on his left lower extremity. He was transferred by ambulance to the emergency department, where X-rays demonstrated a displaced midshaft tibia and fibula fractures. He is noted to have a fair amount of swelling. Exam demon­ strates normal sensation over the dorsal and plantar aspects of his foot. Normal and symmetric pulses are palpated in both lower extremities. He is able to move all of his toes without dif­ ficulty. While you await the orthopedist, you begin educating the resident about compartment syndrome. Question 1 9-1 Which statement is false regarding acute compartment syndrome? A) Definitive diagnosis is made by attaining intracompartmental pressure measurements. B) Pain requiring increasing analgesia is a late finding. C) Emergent fasciotomy is treatment of choice. D) It can be a complication associated with fracture or crush injury. Discussion 1 9-1 The correct answer is "B:' Acute compartment syndrome results from increased interstitial pressure within a finite compart­ ment, leading to vascular compromise of the associated nerves and muscles within that compartment and ischemic injury. Although relatively uncommon in children, acute compartment

have a h i g h suspicion for com partment

r1 1r syn d rome in a c h i l d who susta i ned a s u p racondyl a r

fracture o f the d i sta l h u m e rus, or a t i b i a l shaft or e m i nence fractu re, a n d is req u i ri ng i ncreasing a n a lgesia d u e to prog ressive pa i n . Check the compa rtments!

Your next patient while moonlighting in the emergency depart­ ment is a 4-year-old boy who presents after falling on his left arm, which was fully extended, while playing on the monkey bars at school. He developed immediate pain in his left arm, which was initially rather diffuse in nature. However, once the patient was taken to the nurse's office, the pain appeared to concentrate around the elbow. No obvious deformity was noted; however, the elbow began to swell shortly after the fall. Due to the pain and swelling, the family was called and advised to take the patient to the doctor. In the emergency department X-rays were obtained and demonstrated a type II supracon­ dylar fracture of the left distal humerus. While awaiting the orthopedic consultation, you begin to discuss the evaluation and management of pediatric elbow injuries with the resident. Question 20-1 Which statement is false regarding the patient's injury? A) The presence of a posterior elbow fat pad can be normal, and comparison views should be obtained. B) Ulnar nerve function can be checked by asking the patient to cross the index and middle fingers. C) A fall on an outstretched hand is the most common mecha­ nism of injury. D) The brachial artery is the most common vascular injury associated with this injury.

C H A PT E R 22

Discussion 20-1 The correct answer is ''A:' Supracondylar fractures are the most common type of elbow fracture in children. This fracture often occurs after a fall on an outstretched hand (FOOSH), with subsequent hyperextension of the elbow. The fracture is commonly classified into three types. Type I is a nondisplaced or minimally displaced fracture. Type I fractures can present as an occult fracture, without a fracture line visible on X-ray. In the absence of a fracture line on X-ray, the presence of a fat pad sign can indicate an occult fracture. Specifically, the posterior fat pad visible on X-ray indicates a j oint effusion, raising concern for an occult fracture in the setting of an acute injury in the skeletally immature patient. (See Figure 22-8.) Type I supracondylar fractures generally respond well to non­ operative treatment in a long arm cast for about 3 weeks. Type II supracondylar fractures are characterized by obvious disrup­ tion of one cortex (often the anterior) with the opposite cortex intact (often posterior). Type II supracondylar fractures also commonly have some degree of associated angulation or exten­ sion. Due to the angulation, type II supracondylar fractures often require reduction and surgical stabilization. Type III supracondylar fractures involve disruption of both cortices with complete displacement, requiring reduction and further surgical stabilization. Supracondylar type II and III fractures should be referred urgently to orthopedics. Neurovascular injuries can occur in the setting of supracondylar fractures.



M U S C U LOSKELETAL D I S O R D E RS

495

Thus, a thorough neurovascular exam should be conducted on the initial encounter and each subsequent encounter. The median nerve is the most commonly injured nerve. Sensation and function of the median nerve can be checked by assessing sensation along the palmar aspect of the index and long fin­ ger and active flexion at the interphalangeal j oint (asking the patient to form an "0" with the thumb and index finger) . The radial nerve is the second most common nerve injured. Sensa­ tion of the radial nerve is assessed along the dorsal first web space. Motor function can be assessed by having the patient extend his or her thumb ("thumbs up" ) . Distal sensation of the ulnar nerve is assessed along the palmer aspect of the small finger. Motor function can be checked by have the patient actively abduct the fingers ("crossed finger") .



Helpful Tip

=-� Remem ber the ma ntra "exa mine (a nd image) the joint

r1 1 r above and below." El bow i nju ries ca n occur in the context of other u pper extremity inj uries, so complete eva l uation

of FOOSH inju ries involves examining the wrist, el bow, and shoulder, and imaging may include not only wrist, forearm, and shoulder X-rays, but also el bow X-rays (and vice versa).

In clinic the following day, your first patient is a 20-month-old boy who presents with refusal to bear weight through his right lower extremity. His mother states he was playing in his room and came running to show her something. She heard him fall and went to see if he was okay. He was lying next to a toy on the floor crying. She is unsure what happened, but thinks he may have slipped on the toy. She did not appreciate any swelling or bruising of his leg. However, the patient refused to bear any weight through his right lower extremity and thus she called right away to make an appointment. She reports he has other­ wise been well, no fevers, normal appetite, no other joint swell­ ing or erythema. No prior upper respiratory or gastrointestinal symptoms. On exam the patient is well appearing, nontoxic, and afebrile. There is no effusion appreciable throughout his right lower extremity (knee or ankle). He has normal range of motion at the ankle, knee, and hip. He is fairly anxious dur­ ing the exam making assessment of focal tenderness difficult. X-rays of his femur, tibia, and bilateral hips are negative for any obvious fracture lucency at this time, and there are no signs of effusion on X-rays.

F I G U R E 22-8. Su pracondylar fractu re. Note that the fracture is not visible, but the a nterior and posterior fat pad signs a re. (Reproduced with permission from Tinti n a l l i J E, Stapczynski JS, Ma OJ, Cline DM, Cyd u l ka RK, & Meckler G O (Eds). Tintinalli's Emergency Medicine: A Comprehensive Study Guide, 7 t h ed. New York, NY: McGraw H i l l Education, I nc. Fig u re 1 33 - 1 2, Pg 898.)

Question 21 -1 The most likely diagnosis is: A) Juvenile rheumatoid arthritis. B) Occult fracture of the distal tibia. C) Legg-Calve-Perthes disease. D) Transient synovitis. E) Developmental dysplasia of the hip.

MCG RAW-H I LL E D U CATION S P E C I A LTY BOA R D REVI EW: P E D I ATRICS

496

Discussion 21 -1 The correct answer is "B:' The toddler's fracture was initially described as an occult or obscure fracture of the distal tibia. This type of fracture takes its name from its most common sufferer: a young ambulator (commonly 9 months to 3 years old) who loses balance and twists while falling, causing a low-energy and rather minor torsional injury. Not too uncommonly, the injury is unwitnessed. Often, the presentation includes a new-onset limp or refusal to bear weight. As with any injury, especially fracture, in a child, an appropriate history should be obtained, making sure the fracture pattern fits the mechanism. If not, or if other con­ cerns are present, nonaccidental trauma should be considered; any fracture in a nonambulatory patient should raise concern for nonaccidental trauma. Care should be taken to evaluate signs and symptoms consistent with other etiologies of new-onset limp or refusal to bear weight ( eg, transient synovitis, septic arthritis, etc). AP and lateral X-rays of the affected lower extremity should be obtained. Initial films may be negative for fracture lucency. An internal oblique X-ray of the tibia and fibula may be benefi­ cial in these cases, providing another view. However, these too may be negative in some cases. If X-rays are negative, but his­ tory and exam are consistent with concern for an occult fracture, treatment should be instituted. Treatment often consists of cast immobilization in a long leg cast. In cases where initial X-rays are negative for fracture lucency, follow-up X-rays 10 to 14 days after injury may help to confirm the diagnosis. Treatment consists of cast immobilization for about 3 to 4 weeks.



Helpful Tip

.s.::'Jl Fractu res

i n c h i l d ren may not a l ways be rea d i l y

i1 1r a p pa rent on i n itial X-rays; thus, sus picion for a n occult fracture s h o u ld be ra ised i n the setting of a n acute i nj u ry. C h i l d ren with type I supracondyl a r fractu res may present with refu sal to use the affected arm fol l owing a FOOSH i nj u ry. X-rays may be negative for a fractu re l i ne; however, ca re s h o u l d be ta ken to assess for the fat pad signs i n d icati ng an effusion a n d su bseq uently a n occult fractu re. Todd ler's fractu res a re a n other common i nj u ry in you n g c h i l d ren that may present with l i m ited c l i n ica l and radiogra phic signs.

These fractu res d o n ot occu r i n the nona m b u l atory patient a n d thus should ra ise concern if seen in this popu lation. Additional ly, fractu res a re concern i n g for c h i l d a buse when the i nj u ry mecha nism d o e s not fit the type of fractu re o r other h istorica l or physica l fi n d i n g s a re present.

tournament, evaluated the patient shortly after the injury. Her mother is unsure what the diagnosis from the trainer was but knows that rest, ice, and wrapping the ankle with an elastic bandage were advised. She was also advised to follow up with the doctor this week. On exam the patient has mild swelling over the lateral aspect of the ankle. Sensation is intact and she has appropriate pulses. Motor function is intact, with ability to wiggle her toes. Movement at the ankle is limited and pain­ ful. With palpation, the point of maximal tenderness is over the lateral malleolus at the level of the joint line. No significant tenderness extends anterior to the lateral malleolus. There is no significant translation when cupping the heel and attempt­ ing translation of the talus on the tibia. X-rays demonstrate a skeletally immature patient, with no obvious fracture or dislo­ cation, but with mild soft tissue swelling over the lateral ankle. Question 22-1 What is the most likely diagnosis? A) Septic arthritis. B) Lateral ankle sprain. C) Tendinopathy. D) Salter-Harris I fracture of the distal fibula. Discussion 22-1 The correct answer is "D:' The physis is made up of different zones of cartilaginous cells, which provide longitudinal growth of long bones. This structure is inherently weaker relative to the associ­ ated ligaments. Thus, skeletally immature individuals are more likely to sustain an injury to the growth plate (fracture) than a sprain (ligamentous injury). Clinically, patients present with his­ tory of trauma, overuse, or both. Pain is produced with active and passive range of motion of the associated joint, in addition to palpation over the affected physis. Depending on the sever­ ity of the injury and fracture, various degrees of swelling, bruis­ ing, and pain may be present. X-rays vary depending on the type of injury, as described below. However, it is important to note that X-rays often are negative for obvious fracture lucency in the setting of a nondisplaced Salter-Harris I fracture. Thus, clinical exam is important in diagnosis of these injuries. Physeal inju­ ries are commonly described using the Salter-Harris classifica­ tion. This classification provides a standard method to describe the fracture pattern. In addition, the Salter-Harris classification provides guidance for treatment and some information regarding expected prognosis. (See Figure 22-9.) Treatment varies depend­ ing on the fracture and age of the patient. Displaced physeal frac­ tures should be urgently referred to an orthopedist. Nondisplaced fractures can be splinted and referred to a pediatric orthopedic or sports medicine specialist within 7 to 10 days.



A 9-year-old girl presents for evaluation of a right ankle injury she sustained over the weekend in a basketball game. She went up for a rebound and "landed funny" on her right foot and ankle. She had difficulty ambulating afterward and had to be helped off the court. An athletic trainer, volunteering at the

Helpful Tip

.s.::'Jl The health ca re provider s h o u l d have a low threshold

i1 1 r for fractu re in a s keleta l ly i m mature patient or athlete.

C h i l d ren with open g rowth plates a re more l i kely

to susta i n a physea l i nj u ry (fractu re) than a spra i n (liga ment i nj u ry).

C H A PT E R 2 2

II

Ill

IV



M U S C U LOSKELETAL D I S O R D E RS

497

v

F I G U R E 22-9. Salter-Ha rris classification of physea l fractu res: 1-fractu re extends through the physis; 1 1 -fractu re extends through the physis and i nto the meta physis; I l l -fractu re extends t h rough the physis and i nto the epiphys is; IV-fractu re extends through the physis and i nto the meta physis and epiphysis; V-crush i nj u ry. (Used with perm ission from Blaise Nemeth, M D, MS.)

An 8-year-old right-hand dominant male patient presents for evaluation of right shoulder pain. While playing in a little league game over the weekend he was hit in the shoul­ der by a line drive. He had immediate onset of pain over the proximal aspect of his shoulder. There was mild bruising, which his parents thought was the cause of his pain. How­ ever, due to persistent pain and refusal to use the arm they decided to seek medical care. The patient denies pain in the elbow and denies any pain in his other extremities, including the left shoulder. On exam, you note tenderness with palpa­ tion over the right proximal humerus. You decide to obtain X-rays of his right humerus to evaluate further. On review of the X-rays, you notice a cystic lesion of the right proximal humerus. (See Figure 22- 10.) In talking with radiology, this appears most consistent with a unicameral bone cyst. Question 23-1 What do you discuss with family regarding the natural his­ tory of this bone lesion? A) This lesion requires an emergent referral to oncology and orthopedic surgery for prompt treatment. B) The lesion remains dormant in childhood and typically expands after skeletal maturity. C) The lesion starts near the physis, typically migrates away from the physis with growth, and can spontaneously resolve. D) The lesion is very aggressive, causing significant disruption of the cortex, pain, periosteal reaction, and associated soft tissue swelling. Discussion 23-1 The correct answer is "C:' This child has a unicameral bone cyst or a simple bone cyst. Unicameral bone cysts are typically found in the metaphysis of long bones. The proximal humerus and proximal femur are two common locations where this cyst can occur; the calcaneus is another location. The diagnosis is often made when patients present for care with a pathologic fracture through the cyst, as the isolated cyst generally does not produce symptoms. The diagnosis can be made given the characteristic

F I G U R E 22- 1 0. Cystic lesion consistent with u n icameral bone cyst. (Reproduced with permission from Doherty GM (Ed). Cu rrent Diagnosis & Treatment: S u rgery, 1 4ed. McGraw-H i l l Education, I nc., 201 5 . Fig 40-40.)

MCG RAW-H I LL E D U CATION S P E C I A LTY BOA R D REVI EW: P E D I ATRICS

498

appearance on X-ray. Radiographically, the lesion appears as an isolated central, lucent lesion, located within the medullary canal of the metaphysis of the affected long bone. In an active lesion, it often lies immediately adjacent to the physis. Cortical thinning may be present; however, the cortex generally remains intact, unless a pathologic fracture has occurred. The differen­ tial diagnosis includes nonossifying fibroma, chondroblastoma, and focal fibrous dysplasia. Observation is a treatment option for asymptomatic cysts. If a fracture is present management includes appropriate treatment of the fracture. Surgical treat­ ment is indicated for large cysts in weight-bearing or dominant limbs or cysts at high risk of fracture. Intralesional corticoste­ roid injection have been used with some success in treatment of unicameral bone cysts; however, multiple injections may be required. Surgical curettage and bone grafting or intramedul­ lary pinning is occasionally necessary.



Helpful Tip

=t'll U n icameral bone cysts a re a common bone lesion i n

r1 1r c h i l d ren a n d may present i ncidenta l ly, or with i ndolent onset of pa i n a n d stress fractu re or acute fractu re

throug h the cyst a n d t h i n cortex.

affect any bone but are most commonly found in the femur, tibia, and spine. Although the natural history of these lesions is often unpredictable, they can demonstrate rapid expansion and growth causing localized pain and swelling. Aggressive lesions found within the spine grow to produce compressive nerve symptoms. X-rays will demonstrate a radiolucent lesion, which is expansile in nature, with the metaphysis often appear­ ing to "balloon'' wider than the physis with associated cortical thinning. The lesion can cause cortical disruption and perios­ teal reaction. The differential diagnosis includes osteosarcoma, chondroblastoma, giant cell tumor, and fibrous dysplasia. Given the aggressive nature and appearance of the lesion, incisional biopsy is recommended to confirm the diagnosis. Treatment often requires curettage and bone grafting or excision. Between 20% and 30% of patients have recurrence, typically within 12 to 24 months.



Helpful Tip

=t'll U n icameral

bone cysts a re often fou n d i m med iately

r1 1r adjacent to the physis. They a re ben i g n cysts, typica l ly asym ptomatic u n less a pathologic fract u re is present.

Aneurys mal bone cysts a re ben i g n but potentia l ly agg ressive, expa n s i l e bone cysts that often m i m i c a ma l i g nant process. An incisional biopsy is req u i red to confi rm d i a g n osis, and s u rg ica l cu rettage versus excision for treatment.

A 1 3-year-old girl presents for evaluation of right leg pain. She denies any injury prior to onset of pain. She states the pain has progressively increased over the past few weeks. It is located over her right shin and is fairly constant. At times it is worse with ambulation. She is accompanied by her mother, and both deny any history of skin changes or swelling. On exam the patient's gait appears symmetric without a limp. The skin over her shin is intact without bruising or erythema. With palpation, she has pain located over the proximal tibia on the right relative to the left. AP and lateral X-rays of her right tibia and fibula demonstrate a rather large, expansive, lucent lesion located in the proximal tibia at the metadiaphy­ seal junction. There is associated cortical thinning, disrup­ tion, and associated periosteal reaction. Question 24-1 What is the most appropriate next step? A) Referral to oncology to start chemotherapy. B) Referral for an incisional biopsy to confirm diagnosis. C) Reassure family and plan to follow up in 9 to 12 months with repeat X-rays. D) Intralesional corticosteroid injection. Discussion 24-1 The correct answer is "B:' The patient has any aneurysmal bone cyst-a type of cyst that is typically benign but can be a rather aggressive bone lesion, often mimicking a malignant process. Aneurysmal bone cysts generally occur in adolescents, with the peak incidence in the second decade of life. They can

A 12-year-old girl presents for her well-child check. On standing her right shoulder is slightly higher than the left, and the Adams forward bending test demonstrates elevation of the ribs to the right of her spine. Her neurologic exami­ nation is normal, including sensation, strength, and reflexes in upper and lower extremities. Skin is clear, including over the lumbar spine. Her height is 1 30 em; her right leg mea­ sures 68 em and her left leg measures 68.4 em. Standing, full-length spine X-rays demonstrate a 25-degree thoracic curve to the right and a 25-degree lumbar curve to the left. (See Figure 22- 1 1 .) No vertebral anomalies are noted. Question 25-1 The most likely cause her curve is: A) Congenital scoliosis. B) Leg-length discrepancy. C) Tethered spinal cord. D) Marfan syndrome. E) Unknown. Discussion 25-1 The correct answer is "E." Scoliosis, defined as a curve of greater than 10 degrees as measured by the Cobb method (see Figure 22- 1 1 ) , occurs in 3 % to 5% of adolescents, equally in boys and girls. Girls more commonly present for evaluation

C H A PT E R 22



M U S C U LOSKELETAL D I S O R D E RS

499

term adolescent idiopathic scoliosis is used. Numerous theories exist, considering the association of scoliosis with neurologic, bone, and connective disorders and the prevalence during puberty, but the exact cause is unknown in most cases.

The patient's mother reveals that there is a cousin with sco­ liosis who required surgery, and she is concerned that her daughter might need surgery. Question 25-2 What is the most reliable predictor for curve progression in a female patient with adolescent idiopathic scoliosis? A) Patient age. B) Curve magnitude. C) Sexual maturity rating. D) Family history. E) Type of curve.

F I G U R E 22-1 1 . PA X-ray of the spi ne, as noted in the u pper left. As a res u lt, the patient's right side is to the rig ht, and the cardiac s i l houette and gastric air b u b b l e a re to the l eft. ( U sed with permission from B l a ise N e m eth, M D, M S.)

due to more significant curves. Spinal curves can occur due to abnormalities of vertebral body development (in which case the term congenital scoliosis is used) , tethered cord (often sig­ naled by cutaneous lesions over the lumbar spine or an abnor­ mal neurologic exam with hyperreflexia, spasticity, or foot deformities on one or both sides) , and neuromuscular diseases or genetic syndromes, such as Marfan syndrome, neurofibro­ matosis, or achondroplasia, in which cases other findings of the syndromes should be present. Children younger than 1 0 years o f age are more likely t o have scoliosis secondary t o one of these causes. Physical exam utilizing the Adams forward­ bending test is the primary means of evaluation and detection. Curves related to leg-length discrepancies occur in the lumbar spine and resolve when the leg-length difference is corrected. Typically a leg-length difference of 0.5 to 1 em is necessary to create a clinically identifiable curve. If no other cause of the curve is identified and the patient is at least 10 years old, the

Discussion 25-2 The correct answer is "C:' While age is important in defining adolescent idiopathic scoliosis (ie, older than 10 years) , risk of progression is related to both curve magnitude and proximity to skeletal maturity, which is closely related to sexual maturity rating. Larger curves have an increased risk of progression, but skeletal maturity status determines the extent to which pro­ gression is likely to occur. Patients who have not yet passed peak height velocity have the greatest risk of progression. For females, menarche occurs shortly after peak height velocity, and skeletal maturity occurs about 2 years later. Boys take longer to reach skeletal maturity. On scoliosis X-rays, the Risser sign is used to estimate remaining growth potential after peak height velocity. The Risser sign utilizes the ossification of the iliac crest apophysis, which starts after peak height velocity. The iliac crest is divided into four quadrants (see Figure 22- 1 2 ) , and as ossi­ fication proceeds, the iliac crest apophysis grows posteriorly. Chronologie age and sexual maturity rating may vary greatly at any given age.

The patient is premenarchal. Question 25-3 What is the most appropriate next step in management? A) Return in 6 months for repeat evaluation. B) Referral to physical therapy. C) Referral to orthopedics for bracing. D) Referral to orthopedics for surgery. Discussion 25-3 The correct answer is "C:' Adolescent idiopathic scoliosis typi­ cally progresses during times of growth, but currently it is not possible predict how quickly and to what degree. There is greater risk of progression in younger individuals and in the presence of higher degree curves. Bracing has been shown to decrease the rate of progression to surgery and is usually indicated for growing individuals with curves exceeding 20 to 25 degrees, so referral to pediatric orthopedics is appropriate. Surgery is usu­ ally not indicated unless curves exceed 40 to 50 degrees. Beyond

500

MCG RAW-H I LL E D U CATION S P E C I A LTY BOA R D REVI EW: P E D I ATRICS

F I G U R E 22- 1 2. Ossification o f the i l iac crest apophysis, as seen on a PA scol iosis X-ray, is used to approxi m ate time fol l owing peak height velocity u ntil skel eta l matu rity. (I mage has been cropped to exclude spine and magn ify fi n d i n g s of the left i l iac crest.) No ossification is g raded as Risser 0; lo/o to 25% is Risser 1; 26% to 50% is Risser 2; S l o/o to 75% is Risser 3; a n d g reater than 75% is Risser 4. Closure of the a pophysis is Risser 5 and represents skeletal matu rity. This patient wou l d be assig ned Risser 2 status si nce the i l iac crest a pophysis ossification has advanced posteriorly i nto the second q u a d rant. (Used with permission from Blaise Nemeth, M D, MS.)

this point curves may progress after skeletal maturity, and patients may develop cardiorespiratory compromise. Physical therapy has not been shown to be effective in preventing significant curve pro­ gression. Continued monitoring would not be appropriate in this case given the curve parameters and skeletally immature status of the patient suggesting high risk for curve progression, although for a patient older than 10 years of age with minimal asymmetry on Adams forward-bending test (or radiographic curve < 20 degrees), follow-up evaluation in 6 months is recommended with referral and X-ray if progression is noted.

A 14-year-old boy presents with his mother, who is con­ cerned about his posture. She tells him to sit up straight, but he continues to slouch. She first had concerns 2 years ago and thinks his posture is worsening. He denies pain or neurologic symptoms. On forward bending, there is no asymmetry to his spine or ribs, but he has an abrupt forward angulation to his spine. X-rays are obtained (see Figure 22- 13) demonstrat­ ing 55 degrees of kyphosis and wedging of multiple vertebral bodies with irregularity of the endplates. Question 26-1 The most appropriate next step in management is: A) Referral to physical therapy. B) Referral to pediatric orthopedics. C) Referral for chiropractic manipulation. D) Continued observation.

F I G U R E 22- 1 3. Latera l scol iosis X-ray demonstrating wed g i n g of vertebral bodies (block arrows) and endplate irreg u l a rities (arrowheads). (Used with permission from Blaise Nemeth, M D, MS.)

Discussion 26-1 The correct answer is "B:' Normal kyphosis of the thoracic spine is 25 to 45 degrees. Scheuermann kyphosis is defined as increased thoracic kyphosis with anterior wedging of at least 5 degrees involving three or more consecutive vertebral bod­ ies. (See Figure 22- 1 3 . ) The etiology is unknown. Scheuermann kyphosis progresses during adolescence and bracing may slow progression in individuals who are still growing, although refer­ ral to pediatric orthopedics for further discussion and shared decision making with the patient and family is most appro­ priate. Surgery typically is not considered until curves exceed 75 to 85 degrees. Physical therapy and chiropractic manipula­ tion have not been shown to decrease the risk of progression of Scheuermann kyphosis, although physical therapy may be useful for patients with associated muscular back pain. Patients with postural kyphosis have normal X-rays and resolution of their kyphosis with laying down; observation or physical therapy may be appropriate, depending on patient and family preference.

C H A PT E R 22

A 4-year-old boy presents with persistent right leg pain. His mother states that for the past 2 weeks he has been "walking funny?' He has continued to run and play, but he will stop playing earlier than everyone else secondary to pain. She has noticed he generally points to his right thigh and knee when complaining of pain. She denies any history of injury prior to onset of pain. The patient has been otherwise well and at his baseline health. There is no history of fevers, no other joint swelling or erythema, and no abnormal rashes. His gait exam in the hallway demon­ strates asymmetry. You notice his upper body tends to lurch or lean laterally toward the right side through stance phase on his right leg. On examination of his hips, you also note asymmetry in abduction, in the frog-leg position, with about 60 degrees of abduction on the right relative to about 75 degrees of abduction on the left. An AP/frog-leg X-ray of the pelvis demonstrates the findings seen in Figure 22- 14. Question 27-1 All of the following statements regarding the patient's diag­ nosis are true EXCEPT: A) Residual deformity and hip joint congruity at skeletal matu­ rity are two of the most important prognostic factors. B) Age of onset before 8 years of age is associated with a worse prognosis. C) Long-term studies report most patients do well until the fifth decade. D) Girls have a worse prognosis than boys. Discussion 27-1 The correct answer is "B:' Legg-Calve-Perthes (LCP) disease is defined as idiopathic osteonecrosis of the femoral head. The true etiology is unknown, though many theories have been proposed. LCP is more common in males than females, with a peak age of onset between 4 and 8 years. It can affect both hips in about 10%



M U S C U LOSKELETAL D I S O R D E RS

501

to 12% of cases. Patients often present with an insidious onset of limp and pain. Pain can be nonspecific, with patients describing hip, thigh, or knee pain, or a combination of these. Pain is gener­ ally worse with activity. On exam, patients with an acute presenta­ tion demonstrate an antalgic gait. They also have limited range of motion of the affected hip, most commonly involving hip abduc­ tion and internal rotation. Four different radiographic stages are used to describe LCP. The first, appropriately named the initial stage, is characterized by synovitis and early necrosis. On radio­ graphs, there may be subtle widening of the joint space, a subchon­ dral fracture of the femoral epiphysis, or both. During the second stage, fragmentation, the femoral epiphysis begins to collapse. Radiographs demonstrate fragmentation of the femoral epiphy­ sis with areas of increased radiolucency apparent. The degree of deformity depends on the amount of collapse that occurs during this phase. The third stage, reossification, is characterized by new bone formation. The final stage is the healed stage. In this stage the residual deformity of the femoral head and proximal femur is apparent. The true natural history of LCP is unknown; however, various long-term follow-up studies have provided us with what is currently known. Most patients do well without significant pain or limitations until about the fifth decade. Thereafter hip function begins to deteriorate. Most patients develop degenerative joint disease by the sixth and seventh decades of life. Residual femoral head deformity and hip joint congruity at skeletal maturity, age at onset, and sex are all prognostic factors. Age of onset before 6 to 8 years carries a better prognosis due to the remodeling potential. Female patients also appear to have a poorer prognosis relative to their male counterparts. The treatment goals include preserving a round femoral head and joint congruency in addition to main­ taining hip range of motion. Treatment is somewhat controversial and varies depending on the age, symptoms, severity of femoral head involvement, and joint congruency. Nonsurgical treatment options include activity limitations, physical therapy, and abduc­ tion bracing or casting. Surgical treatment includes both femoral or pelvic surgical options.





Helpful Tip

Consider h i p pathol ogy i n a l i m p i n g child who

1 1 1r com p l a i n s of knee pa i n .

FIGURE 22-1 4. Legg-Calve-Perthes disease o f t h e left hip. X-ray demonstrates fragmentation and flattening of the femoral head. (Reproduced with permission from Skinner H B, McMahon PJ: CURRENT Diagnosis & Treatment in Orthopedics, 5th ed. New York: McGraw-Hill; 201 4.)

A 9-year-old female soccer player presents with bilateral heel pain. The pain started about 3 to 4 weeks ago. There was no known injury prior to onset of pain. The pain is located over the bilateral heels and is worse with activity, specifically, worse after soccer activities. Rest and ibuprofen alleviate the pain. The family denies any swelling or redness of the heels. She did have to sit out of part of the last game secondary to pain. On exam, her gait is normal without evidence of antalgia. Bench exam demonstrates mild heel cord contractures with passive dorsiflexion just a few degrees past neutral (legs extended) . She has a positive calcaneal squeeze test bilaterally.

502

MCG RAW-H I LL E D U CATION S P E C I A LTY BOA R D REVI EW: P E D I ATRICS

Question 28-1

Which of the following is characteristic of this patient's diagnosis? A) It is a rare cause of heel pain in the preteen age group. B) Treatment often involves repeated corticosteroid injections or surgery. C) Maximal pain is elicited with medial and lateral compres­ sion along the posterior calcaneus. D) This condition is more common in females than males. E) Patients often present with morning stiffness and swelling. Discussion 28-1 The correct answer is "C:' This patient has Severs disease, also known as calcaneal apophysitis, a common cause of heel pain in children between 8 and 1 3 years of age. It is more common in boys than girls, is bilateral about 60% of the time, and is often seen in children participating in soccer, basketball, running, and gym­ nastics. Patients present with heel pain that is worse with activity. Clinically, the calcaneal squeeze test (medial and lateral compres­ sion of the posterior third of the calcaneus) reproduces the pain. In addition, patients may have associated heel cord tightness and midfoot pronation. Severs disease is a clinical diagnosis. X-rays (AP calcaneal view and lateral view) are generally not necessary but should be considered to rule out other causes of heel pain if history and exam are not classic. Calcaneal apophysitis does not present after skeletal maturity. Treatment includes a combination of activity modifications, anti-inflammatories, and addressing any biomechanical issues with the use of orthotics, gel heel cups, or both. Occasionally, a short leg walking cast or walking boot may be used in patients with more significant pain or activity limitations. Symptoms resolve, on average, after about 2 months with a range between 1 and 6 months.

Discussion 29-1 The correct answer is "B:' This patient has Osgood-Schlatter disease (OSD) , a common cause of anterior knee pain in patients between 10 and 1 5 years of age. OSD is a traction apophysitis that occurs at the insertion of the patellar tendon at the tibial tubercle. Sinding- Larsen-Johannsen (SLJ) dis­ ease, another apophysitis responsible for anterior knee pain, occurs at the inferior pole of the patella. Both OSD and SLJ are thought to occur secondary to overuse of the extensor mechanism and thus are common in athletes participating in jumping sports. OSD is more common in males and is typi­ cally diagnosed with history and clinical exam. Patients pres­ ent with anterior knee pain that localizes to the tibial tubercle. Pain is worse with activity, and improved with rest and anti­ inflammatories. Some patients may develop mild swelling or a prominence over the tibial tubercle. X-rays (AP and lat­ eral knee) should be considered in atypical clinical cases to evaluate for other pathology ( eg, tumor, fracture, infection ) . They commonly demonstrate fragmentation of the tibial tubercle, but this finding may also be seen in asymptomatic patients so is not diagnostic. (See Figure 22- 1 5 . ) As with other apophysitis conditions, O SD, is an age-limited pro­ cess and generally improves with skeletal maturity; however, some patients continue to report pain with kneeling after skeletal maturity. Treatment includes activity modifications, ice, and anti-inflammatories. Physical therapy can be used as an adjunct to focus on quadriceps and hamstring stretching and strengthening exercises. More recently, an inj ection of hyperosmolar dextrose has been used with some success in recalcitrant cases of OSD.

A 1 3-year-old previously healthy boy presents for evaluation of bilateral knee pain, left greater than right. He states pain was gradual in onset and denies any significant knee injury prior to onset of pain. The pain is located over the anterior, inferior aspect of his knee and is worse with running and jumping. It has increased since he started basketball season a month ago. On exam, he has symmetric range of motion of the knees bilaterally, and no appreciable swelling. His exam is negative for a ligamentous injury. The point of maximal tenderness is along the tibial tubercle bilaterally. X-rays dem­ onstrate fragmentation of the tibial tuberosity. Question 29-1 Of the following, the most likely cause of the patient's symp­ toms is: A) Physeal fracture of the proximal femur. B) Traction apophysitis at the insertion of the patellar tendon on the tibia. C) Avascular necrosis of the femoral head. D) Acute fracture of the tibial tubercle.

F I G U R E 22- 1 5. Osgood-Sch l atter d i sease. It is common, but not necessa ry, to see fragmentation of the tibial tubercle apophysis, s i m i l a r to the diagram. (Reproduced with permission from Skinner H B, McMahon PJ, eds. Current Diagnosis & Treatment in Orthopedics. 5th ed. New York, NY: McGraw- H i l l Education; 20 1 4, F i g . 1 0-29.)

C H A PT E R 22

Your first patient of the afternoon is a 1 2-year-old boy who recently transferred to your clinic for care. He presents for evaluation of right knee pain that began about 2 months ago when he twisted his knee as he was pushed into the pool. He states his whole leg hurt at that time and he had difficulty bearing weight. He was seen in a local emer­ gency department and X-rays of his knee were negative (the family did not bring a copy today) . He was diagnosed with a knee sprain and discharged home with crutches. He was seen by a physician a few weeks after the injury and instructed to wean off the crutches as tolerated. He contin­ ued to use the crutches for another month and discontin­ ued use about 1 week ago because school started, despite continued pain. On exam, you notice asymmetry in his gait. He demonstrates antalgia and appears to lean to the right with stance phase. On bench exam he has vague knee pain when flexing the right hip and knee relative to the left. He has no pain with palpation around the knee, and the exam is negative for any ligamentous injury. In addi­ tion, you note about 40 to 50 degrees of abduction on the right and about 70 to 75 degrees of abduction on the left. He also has limited internal rotation at about 10 degrees on the right and about 30 degrees of internal rotation on the left. Question 30-1 What is the most appropriate next step? A) AP and lateral X-rays of femur. B) AP, lateral, Merchant, and tunnel X-rays of knee. C) Referral for MRI of right knee. D) AP and frog-leg X-rays of pelvis. E) Referral to orthopedics. Discussion 30-1 The correct answer is "D:' Slipped capital femoral epiphysis (SCFE) is a physeal fracture involving the proximal femoral physis that results in anterior displacement of the proximal femoral neck and shaft. It is an orthopedic emergency. Obesity is the most common risk factor. SCFE occurs more commonly in males, and up to 60% of cases may have bilateral involve­ ment {either at presentation or later) . Endocrinopathies (thyroid disease, hypopituitarism, and renal osteodystrophy) are another common risk factor and should be considered in the workup of patients with an atypical presentation. Clini­ cally, patients present with a variety of symptoms, including pain located over the anterior groin, thigh, knee, or a com­ bination of these. When a child presents with knee pain, it is important to also examine the gait and hips to assess for pos­ sible hip pathology referring pain distally to the knee. Pain can be acute or insidious in onset. Other associated findings include limp (antalgic or Trendelenburg gait, out-toed gait) and decreased range of motion (most commonly involving flexion, abduction, and internal rotation) . Diagnosis is based on history, clinical exam, and radiographic studies. Many



M U S C U LOSKELETAL D I S O R D E RS

503

F I G U R E 22- 1 6. S l i pped capital femoral epi physis. Note the step-off between the meta physis and epi physis on this frog leg-latera l view of the right h i p. (Used with permission fro m Nata l i e Stork, M D.)

cases of SCFE are diagnosed by AP and lateral films of the pelvis. Up to 50% of patients can have bilateral involvement on presentation. Thus, it is important to obtain radiographs of the bilateral hips and pelvis at the time of presentation. The AP view may show irregularity along the physis and or widening of the physis. The lateral hip film is important, as it is more sensitive in detecting mild slips. On the lateral film the posterior translation of the head relative to the femoral neck and shaft becomes more apparent. (See Figure 22- 1 6. ) MRI i s a n important tool when there i s a high clinical suspi­ cion for SCFE but X-rays are normal. This presentation has been referred to as a pre-slip SCFE. Increased signal on both sides of the physis seen on MRI is consistent with a pre- slip SCFE. Various classification systems are used to describe the fracture. One approach focuses on the time frame, clas­ sifying fractures as acute (symptom onset < 3 weeks) versus chronic {prodrome of symptoms > 3 weeks ) , and acute on chronic. The classification of SCFE as stable (the patient can bear weight either with or without crutches) versus unsta­ ble (patient unable to bear weight with or without crutches) can be helpful in making treatment decisions. Classification systems also are available for assessing severity based on the degree of slip measured radiographically. Patients diagnosed with SCFE should be made non-weight bearing immedi­ ately upon diagnosis despite ambulatory status prior to diag­ nosis, duration of symptoms, or degree of slip. In addition, upon diagnosis, the patient should be urgently or emer­ gently (in the case of unstable slips) referred to an ortho­ pedist. Treatment generally requires surgical fixation of the fracture. Complications associated with missed diagnosis or untreated SCFE include further progression and subsequent osteonecrosis of the femoral head leading to various other complications.

MCG RAW-H I LL E D U CATION S P E C I A LTY BOA R D REVI EW: P E D I ATRICS

504



Helpful Tip

� An obese adolescent with a pa i nfu l

=

limp a n d a nterior

r1 1r thigh pain, g ro i n pain, or u n s pecified knee pa i n should

ra ise concern for SCFE. Patients demonstrate a n

a nta l g i c a n d Trendelenburg g a i t with l i m ited ra nge of motion (flexion, a bd u ction, a n d i nterna l rotation).

A 4-year-old boy presents with a complaint of back pain for the last 3 days. Today he awoke and refused to get out of bed. He is laying down when you enter the exam room and has dif­ ficulty rising to a sitting position, pointing to his lower back as the area of discomfort. His mother denies vomiting, diar­ rhea, urinary frequency, or complaints of nausea or dysuria. His temperature is 38°C ( 1 00.4°F). His abdomen is soft and he has negative obturator but positive psoas signs bilater­ ally. He is tender over the spine in the lumbar region, and his paraspinous muscles are tense and also tender. Neurologic examination of the lower extremities is normal, other than decreased strength with hip flexion due to back discomfort. Question 31 -1

The most likely diagnosis is: A) Histiocytosis. B) Urinary tract infection. C) Discitis. D) Spondylolysis. E) Appendicitis. Discussion 31 -1 The correct answer is "C:' Back pain in children should always raise concern for a pathologic etiology. In younger children, spine-associated abnormalities may manifest with back pain, abdominal pain, or refusal to walk, stand, or even sit. Discitis is the most likely cause of back pain in a young child, espe­ cially if fever is present, although fever is not always present. Often patients present with ill-defined symptoms and normal lab values, leading to delayed diagnosis. The psoas sign may be positive due to irritation from the psoas at its origin at the level of the spine. Discitis can be differentiated from pyelonephritis with associated costovertebral angle tenderness by the absence of urinary symptoms, although urinalysis may be helpful to rule out urinary tract infection. X-rays may reveal narrowing of the disc space at the level of involvement. MRI demonstrates edema within the disc and the adjacent vertebral bodies and is the diagnostic modality of choice. (See Figure 22- 1 7.) Antibiotics typically yield resolution of symptoms, and bracing may provide comfort; Staphylococcus aureus is the most common infectious organism. Histiocytosis (previously termed eosinophilic granu­ loma) may occur in the spine and is the most common spinal tumor in children. The clinical presentation may be very similar to discitis, but X-rays will demonstrate vertebra plana (flatten­ ing of the vertebral body) . (See Figure 22- 1 8.) Patients may have other lesions, and biopsy evaluation confirms the diagnosis.

F I G U R E 22- 1 7. Sag itta l MRI scan of the l u m ba r spine shows i n c reased signal i n the disc s pace and soft tissue posterior to the vertebral bodies suggestive of d iscitis. This patient had blood cultures positive for Staphylococcus aureus, confi r m i n g the diag nosis. (Reproduced with permission from McKean SC, Ross JJ, Dressier DD, Brotman OJ, G i n s berg JS, eds. Principles and Practice of Hospital Medicine. New York, NY: McGraw- H i l l Education; 201 2, Fig. 200- 1 .)

Treatment usually proceeds with chemotherapy. Most intraspi­ nal soft tissue tumors do not cause back pain but instead pres­ ent with abnormalities in neurologic exam or radicular pain, or both. Spondylolysis, a stress fracture of the vertebra, is a rare cause of back pain in younger children unless there is an epi­ sode of acute trauma with immediate onset of back pain.

A 13-year-old girl presents with complaints of low back pain. Her pain started 1 month ago, about 2 weeks after starting gym­ nastics. She does not recall any traumatic episode that occurred at the time of pain onset. Initially the pain occurred only during activity, but it has progressively worsened in severity and is now present all the time. She points to the right side of her low back as the area of maximum discomfort. She denies any radiation of pain down her leg, and has no encopresis or enuresis. Heat and ibuprofen help her pain. Examination demonstrates pain with back extension, but minimal limitation or discomfort with for­ ward flexion. Neurologic examination is normal. Question 32-1 What is the most likely diagnosis? A) Spondylolysis. B) Herniated disc.

C H A PT E R 22



M U S C U LOSKELETAL D I S O R D E RS

505

not be necessary if there is clinical suspicion and advanced imag­ ing will be performed regardless. Bone scan is the most sensitive modality for identification (see Figure 22-20) but does not help in determining the acuity of the fracture, or differentiating other potential causes ( eg, osteoid osteoma), and involves relatively high radiation exposure. CT scan is the most specific modality, providing good bony detail, but utilizes high radiation doses and does not detect stress reactions ("pre-fractures"). MRI avoids ion­ izing radiation and is useful if there is concern regarding other abnormalities (eg, discitis in a younger patient); false negatives, when compared with bone and CT scans, may occur due to lack of edema in the dense pars interarticularis and use of disc-spe­ cific, rather than pars-specific, imaging protocols. Patients with spondylolysis should be referred to a sports medicine or ortho­ pedics specialist for management. Treatment protocols vary from relative rest to bracing, followed by physical therapy to correct biomechanical issues contributing to stress at the pars interar­ ticularis (such as tight hamstrings) and gradual return to play. Herniated discs are rare in children, although disc bulging and desiccation is commonly found incidentally on MRI; disc hernia­ tion presents in a manner similar to that in adults: sudden onset of low back pain that is worse with flexion and often produces radicular symptoms. Treatment involves acute pain management, typically for 1 to 2 weeks, and physical therapy for long-term "back health:' Mechanical low back pain also occurs commonly in adolescents but should be a diagnosis of exclusion, considering the other etiologies in the differential diagnosis. Onset is usually insidious, and the pain may limit activity but is often worse with sitting or standing and better with movement. Radicular symp­ toms are absent, and advanced imaging may not be necessary if the pain is of long duration without change in symptoms. Physi­ cal therapy is the mainstay of treatment, addressing biomechani­ cal issues contributing to altered spinal mechanics. F I G U R E 22- 1 8. Lateral l u m ba r X-ray demonstrating vertebra plana (flattening) of L 1 (arrow) secondary to infi ltration from histiocytosis. This patient presented with refusal to wa l k secondary to back pa i n . Note that the disc s paces a bove a n d below a re preserved, u n l i ke i n discitis (Fig u re 22-20). (Used with permission fro m Blaise Nemeth, M D, MS.)

C) Mechanical low back pain. D) Endometriosis. Discussion 32-1 The correct answer is "A:' The most common pathologic cause of acute-onset, atraumatic back pain in adolescents is isthmic spondylolysis, a stress fracture within the pars interarticularis, a narrow part of the posterior ring of the vertebra, as a result of repetitive overuse. (See Figure 22- 19.) The most common loca­ tion is at LS, although other lumbar vertebrae may be involved. Spondylolysis typically develops in school-aged children, and athletes in sports that involve spinal extension, such as gym­ nastics and football linemen, are at greatest risk for develop­ ment. Physical examination typically demonstrates pain with back extension. AP and lateral X-rays of the lumbar spine may demonstrate the fracture, especially if both sides are involved. Oblique X-rays may help improve diagnostic accuracy but may

You are meeting a family who is establishing care prior to the birth of their infant in a few months' time. The mother-to-be reports she recently had a good friend with a daughter who was diagnosed with hip dysplasia. Both parents would like to know more about hip dysplasia and the risk factors associ­ ated with this condition. Question 33-1 All of the following combinations of history and exam would raise your suspicion for developmental dysplasia of the hip EXCEPT: A) A male infant born to a G P mother with an intermittent 2 2 high-pitched hip click at the termination of abduction. B) A female infant born to a G l 1 mother with asymmetry noted in the height of her knees when flexed at 90 degrees. C) A male infant born by cesarean section due to breech pre­ sentation with limited abduction. D) A female infant with torticollis on exam, born to a G l 1 mother with a maternal history positive for hip dysplasia.

MCG RAW-H I LL E D U CATION S P E C I A LTY BOA R D REVI EW: P E D I ATRICS

506

B

A

c

F I G U R E 22- 1 9. (A) Latera l l u m ba r X-ray demo nstrating l u cency with i n the pars i nterarticularis of LS (arrow) in an adolescent soccer player with low back pa i n for 2 months. ( B ) T h e l u cency is more easily seen on t h i s sag itta l C T s c a n t h rough t h e left L S pars i nterarticularis (arrow) o f t h e s a m e patient. (C) An axial image from the same CT scan a l so demonstrates the fracture through the left pars i nterarticularis (arrow), as wel l a s hypertrophic hea l i ng of a n early spondylo lytic lesion on the right side (block arrow). (Used with permission fro m Blaise Nemeth, M D, MS.)

Discussion 33-1 The correct answer is "A:' Developmental dysplasia of the hip (DDH) is a general diagnosis of abnormal hip development that encompasses a variety of pathologic findings relating to the rela­ tionship between the femoral head and acetabulum. The true etiology ofDDH is unknown, although it is thought to be multi­ factorial, including both genetic and environmental influences. Risk factors include a positive family history, female sex, breech presentation, and a restricted intrauterine environment (first born, multiple gestation, oligohydramnios, large for gestational

age) . Often diagnosis is made on exam, shortly after birth; how­ ever, DDH may be detected at any age.

A 10-week-old female infant presents for her well-child check. She was recently placed in foster care, and her foster mother states they are still working on gathering her medical records.

C H A PT E R 22

F I G U R E 22-20. Bone scan in a d ifferent patient demonstrating increased u pta ke of tech netium-99m i n the right pars interartic u l a ris of LS in this pediatric patient with acute onset of low back pa i n . The fi n d i n g of

spondylolysis was confirmed by CT sca n. (Used with permission from Blaise Nemeth, M D, MS.)

The foster mother knows the infant was born at term by cesar­ ean section secondary to breech presentation. Her biological mother is 15 years old, and the foster mother believes this was her first child. On exam, the infant is well appearing, her head is round, and she demonstrates full range of motion of her neck. Knees are symmetric in height when flexed at 90 degrees. With abduction you notice mild asymmetry, with 75 degrees of abduction on the right and 65 to 70 degrees on the left. No overt dunk is noted when performing Ortolani and Barlow tests during your exam today. Given her risk factors-female, breech presentation, first born-and exam findings you are concerned for hip dysplasia (DDH). Question 34-1 What is the most appropriate next step? A) Make an urgent referral to orthopedics. B) Obtain an AP and frogleg film of her pelvis. C) Reassure the foster mother and follow up at her 4-month checkup. D) Obtain an ultrasound of her hips bilaterally. Discussion 34-1 The correct answer is "D:' Clinically, patients with DDH can present with a variety of different signs and symptoms, depend­ ing on age and the pathology involved. The exam may be nor­ mal in patients with mild dysplasia. When present, certain exam



M U S C U LOSKELETAL D I S O R D E RS

507

findings will raise the concern and may also provide diagno­ sis of DDH. In neonates and young infants the Ortolani and Barlow tests have been used to assess for stability (ie, evidence of a dislocated or dislocatable hip). The Ortolani test assesses for reduction of a potentially dislocated hip. With this test the hip is flexed and gently abducted. A positive test occurs if there is a palpable clunk when the hip is brought into this position, indi­ cating reduction of the femoral head into the acetabulum. The Barlow test assesses whether a reduced hip can be subluxed or dislocated. This test is performed by gently bringing the hip into flexion and adduction while palpating for an associated clunk indicating movement of the femoral head out of the acetabulum. The Ortolani and Barlow tests are more useful in the neonatal period. After 2 to 3 months of age these tests become less reli­ able. In older infants or children the physical exam may demon­ strate limited abduction or a painless limp, or both. Asymmetric thigh folds or a leg-length discrepancy may be present in the context of a dislocated hip. Proper physical exam remains the most important aspect in the diagnosis of DDH. Imaging can provide further information to help support the diagnosis. The choice of imaging modality depends on the age of the patient. In infants younger than 4 to 6 months, the femoral head is not yet ossified and cannot be visualized on plain X-rays. This limits the use of plain X-rays in the young infant population. In these young infants, ultrasound is helpful for assessing anatomy and allows for a static and dynamic assessment of the hip anatomy. In infants younger than 6 weeks of age, screening ultrasounds may be overly sensitive and may result in overtreatment. Uni­ versal ultrasound screening in infants is generally not practiced in the United States for a number of reasons, including but not limited to low disease prevalence, cost, lack of trained personnel to assure quality imaging and interpretation, subjectivity, and a high false-positive rate.

B I B LIOGRAPHY

Arkader A, Gebhardt MC, Dormans JP. Bone and soft tissue tumors. In: Weinstein SL, Flynn JM, eds. Lovell and Winters Pediatric Orthopaedics. Vol l . 7th ed. Philadelphia, PA: Wolters Kluwer Health/Lippincott Williams and Wilkins; 2014. Arndt CAS. Neoplasms of the bone. In: Kleigman RM, Stanton BF, St Geme JW, Schor NF, Behrman RE, eds. Nelson Textbook of Pediatrics. 1 9th ed. Philadelphia, PA: Elsevier/ Saunders; 20 1 1 . Baxter WR, Kocher M, Ganley T. Sports medicine in the growing child. In: Weinstein SL, Flynn JM, eds. Lovell and Winters Pediatric Orthopaedics. Vol 2. 7th ed. Philadel­ phia, PA: Wolters Kluwer Health/Lippincott Williams and Wilkins; 2014. Brooks W, Gross R. Genu varum in children: Diagnosis and treatment. J Am Acad Orthop Surg. 1 995;3 (6):326-335. Caird MS, Flynn JM, Leung YL, Millman JE, D'Italia JG, Dormans JP. Factors distinguishing septic arthritis from transient synovitis of the hip in children. A prospective study. J Bone Joint Surg Am. 2006;88(6) : 1 2 5 1 - 1257. doi: 1 0.2 1 06/JBJS.E.002 16.

508

MCG RAW-H I LL E D U CATION S P E C I A LTY BOA R D REVI EW: P E D I ATRICS

Dubnov-Raz G, Ephros M, Garty B-Z, et al. Invasive pedi­ atric Kingella kingae infections: A nationwide collab­ orative study. Pediatr Infect Dis ]. 2 0 1 0;29(7) :639-643. doi: 1 0 . 1 097 /INF.Ob0 1 3e3 1 8 1 d57a6c. Dunbar JS, Owen HF, Nogrady MB, Mcleese R. Obscure tibial fracture of infants-the toddler's fracture. J Can Assoc Radial. 1 964; 1 5 : 1 36- 1 44. Engstrom P, Tedroff K. The prevalence and course of idio­ pathic toe-walking in 5-year-old children. Pediatrics. 20 12; 1 30(2) :279-284. doi: 1 0. 1 542/peds.20 1 2-0225. Farsetti P, Weinstein SL, Ponseti IV. The long-term functional and radiographic outcomes of untreated and non­ operatively treated metatarsus adductus. J Bone Joint Surg Am. 1 994;76(2):257-265. Flaherty EG, Perez-Rossello JM, Levine MA, et al. Evaluating children with fractures for child physical abuse. Pediatrics. 20 14; 1 3 3 (2) :e477 -e489. doi: 10. 1 542/peds.20 1 3 -3793. Flynn JM, Skaggs DL. Supracondylar fractures of the distal humerus. In: Flynn JM, Skaggs DL, Waters PM, eds. Rockwood and Wilkins' Fractures in Children. 8th ed. Philadelphia, PA: Lippincott Williams and Wilkins; 20 1 5. Friedman JE, Richard S. Davidson. Leg-length discrepancy. In: Kleigman RM, Stanton BF, St Geme JW, Schor NF, Behrman RE, eds. Nelson Textbook of Pediatrics. 1 9th ed. Philadelphia, PA: Elsevier/Saunders; 20 1 1 . Halanski MA, Noonan KJ. Limb-length discrepancy. In: Weinstein SL, Flynn JM, eds. Lovell and Winters Pediatric Orthopaedics. Vol 2. 7th ed. Philadelphia, PA: Wolters Kluwer Health/Lippincott Williams and Wilkins; 2014. Heath CH, Staheli LT. Normal limits of knee angle in white children-genu varum and genu valgum. J Pediatr Orthop. 1 993 ; 1 3 (2):259-262. Hecht AC, Gebhardt MC. Diagnosis and treatment of unicam­ eral and aneurysmal bone cysts in children. Curr Opin Pediatr. 1 998; 1 0 ( 1 ) :87-94. Herman MJ, James J. McCarthy. The principles of pediatric fracture and trauma care. In: Weinstein SL, Flynn JM, eds. Lovell and Winters Pediatric Orthopaedics. Vol 2. 7th ed. Philadelphia, PA: Wolters Kluwer Health/Lippincott Williams and Wilkins; 2014. Hosalkar HS, Spiegel DA, Davidson RS. The foot and toes. In: Kleigman RM, Stanton BF, St Geme JW, Schor NF, Behrman RE, eds. Nelson Textbook of Pediatrics. 1 9th ed. Philadelphia, PA: Elsevier/Saunders; 20 1 1 . Kay RM, Kim YJ. Slipped capital femoral epiphysis. In: Weinstein SL, Flynn JM, eds. Lovell and Winters Pediatric Orthopaedics. Vol 2 . 7th ed. Philadelphia, PA: Wolters Kluwer Health/Lippincott Williams and Wilkins; 2 0 1 4 . Kocher MS, Mandiga R , Zurakowski D , Barnewolt C, Kasser JR. Validation of a clinical prediction rule for the dif­ ferentiation between septic arthritis and transient synovitis of the hip in children. J Bone Joint Surg Am. 2004;86-A(8) : 1 629- 1 635. Krause BL, Williams JP, Catterall A. Natural history of Osgood-Schlatter disease. J Pediatr Orthop. 1 990; 1 0 ( 1 ) :65-68. Lincoln TL, Suen PW. Common rotational variations in children. J Am Acad Orthop Surg. 2003; 1 1 (5):3 1 2-320.

Micheli LJ, Ireland ML. Prevention and management of cal­ caneal apophysitis in children: An overuse syndrome. J Pediatr Orthop. 1 987;7( 1 ) :34-38. Mosca VS. The foot. In: Weinstein SL, Flynn JM, eds. Lovell and Winters Pediatric Orthopaedics. Vol 2. 7th ed. Philadelphia, PA: Wolters Kluwer Health/Lippincott Williams and Wilkins; 2014. Mubarak SJ, Owen CA, Hargens AR, Garetto LP, Akeson WH. Acute compartment syndromes: Diagnosis and treatment with the aid of the wick catheter. J Bone Joint Surg Am. 1 978;60(8) : 1 0 9 1 - 1 095. Oetgen ME, Peden S. Idiopathic toe walking. J Am Acad Orthop Surg. 20 1 2;20(5) :292-300. doi: 1 0 . 5435/ JAAOS-20-05-292. Rathj en KE, Kim HKW. Physeal injuries and growth dis­ turbances. In: Flynn JM, Skaggs DL, Waters PM, eds. Rockwood and Wilkins' Fractures in Children. 8th ed. Philadelphia, PA: Lippincott Williams and Wilkins; 2014. Rosman NP, Douglass LM, Sharif UM, Paolini J. The neurol­ ogy of benign paroxysmal torticollis of infancy: Report of 10 new cases and review of the literature. J Child Neural. 2009;24(2) : 1 55 - 1 60. doi: 1 0. 1 1 77 /0883073808322338. Sankar WB, Horn BD, Wells L, Dormans JP. The Hip. In: Kleigman RM, Stanton BF, St Geme JW, Schor NF, Behrman RE, eds. Nelson Textbook of Pediatrics. 1 9th ed. Philadelphia, PA: Elsevier/Saunders; 20 1 1 . Schoenecker PL, Rich MR. The lower extremity. In: Wein­ stein SL, Flynn JM, eds. Lovell and Winters Pediatric Orthopaedics. Vol 2. 7th ed. Philadelphia, PA: Wolters Kluwer Health/Lippincott Williams and Wilkins; 2014. Schwend RM, Shaw BA, Segal LS. Evaluation and treatment of developmental hip dysplasia in the newborn and infant. Pediatr Clin. 2 0 1 4;6 1 (6) : 1 095- 1 1 07. doi: 10. 1 0 1 6/ j .pcl.20 1 4.08.008. Sharrard WJ. Knock knees and bow legs. Br Med f. 1 976; 1 ( 60 1 3) :826-827. Skaggs DL, Flynn JM, Grill, F, Moseley CF. Leg length inequal­ ity. In: Skaggs DL, Flynn JM, eds. Staying out of Trouble in Pediatric Orthopaedics. Philadelphia, PA: Lippincott Williams and Wilkins; 2006. Staheli LT. Rotational problems in children. Instr Course Lect. 1 994;43: 1 99-209. Tiwari A, Haq AI, Myint F, Hamilton G. Acute compartment syndromes. Br J Surg. 2002;89(4) :397-4 1 2 . doi: 1 0 . 1 046/j .0007 - 1 323 .2002.02063 .x. Topol GA, Podesta LA, Reeves KD, Raya MF, Fullerton BD, Yeh H. Hyperosmolar dextrose injection for recalcitrant Osgood-Schlatter disease. Pediatrics. 20 1 1 ; 1 28(5):e1 1 2 1 -el l28. doi: 10. 1 542/peds.20 1 0 - 1 9 3 1 . Warman ML, Cormier-Daire V, Hall C , et al. Nosology and classification of genetic skeletal disorders: 20 1 0 revision. Am J Med Genet A. 20 1 1 ; 1 55A(5) :943-968. doi: 1 0 . 1 002/ ajmg.a.33909. Weinstein SL. Developmental hip dysplasia and dislocation. In: Weinstein SL, Flynn JM, eds. Lovell and Winters Pediatric Orthopaedics. Vol 2. 7th ed. Philadelphia, PA: Wolters Kluwer Health/Lippincott Williams and Wilkins; 2014. Weinstein SL. Legg-Calve-Perthes syndrome. In: Weinstein SL, Flynn JM, eds. Lovell and Winters Pediatric Orthopaedics.

C H A PT E R 2 2

Vol 2. 7th ed. Philadelphia, PA: Wolters Kluwer Health/ Lippincott Williams and Wilkins; 2014. Weinstein SL, Dolan LA, Wright JG, Dobbs MB. Effects of bracing in adolescents with idiopathic scoliosis. N Engl ! Med. 2 0 1 3;369( 16): 1 5 1 2- 1 52 1 . doi: 1 0 . 1 056/ NEJMoal 307337. Wells L, Sehgal K. The knee. In: Kleigman RM, Stanton BF, St Geme JW, Schor NF, Behrman RE, eds. Nelson Textbook of Pediatrics. 1 9th ed. Philadelphia, PA: Elsevier/Saunders; 20 1 1 .



M U S C U LOSKELETAL D I S O R D E RS

509

Wells L, Sehgal K. Torsional and angular deformities. In: Kleigman RM, Stanton BF, St Geme JW, Schor NF, Behrman RE, eds. Nelson Textbook of Pediatrics. 1 9th ed. Philadelphia, PA: Elsevier/Saunders; 20 1 1 . White GR, Mencio GA. Genu valgum in children: Diagnostic and therapeutic alternatives. J Am Acad Orthop Surg. 1 995;3(5) :275-283.

This page intentionally left blank

23

N e u ro l og i c D isorders Sats u ki Mats u m oto a n d Lea h Zhorne

A I S-year-old obese adolescent girl presents to the clinic with a 1 -month history of near-daily, throbbing, early morning headaches and nausea. She is being bullied at school because of her weight and acne. Her medications include a combined oral contraceptive pill (OCP) and retinoic acid cream. Her mother thinks she is faking symptoms to stay home from school. Her vital signs and general, and neurologic exams are normal except for blurred optic disc margins. Question 1 -1 What is the most likely diagnosis? A) Tension headache due to social stressors. B) Chronic migraine. C) Idiopathic intracranial hypertension. D) Acute hydrocephalus. E) None of the above. Discussion 1 - 1

The correct answer is "C:' The early morning headache with nausea and papilledema on exam should clue you in to increased intracranial pressure as the underlying cause of the patient's symptoms. Although option "D;' acute hydrocephalus, could also cause these symptoms, the patient would likely present in a more emergent manner with vomiting, altered level of con­ sciousness, and possibly focal neurologic findings. Idiopathic intracranial hypertension (IIH), also known as pseudotumor cerebri syndrome, can be truly "idiopathic"; however, it is also associated with obesity, pregnancy, endocrinopathies, medica­ tions (tetracyclines, retinoic acid), vitamin A toxicity, and ane­ mia. Although the neurologic exam in affected children should otherwise be normal, cranial nerve ( CN) VI palsy is allowed as it is a "false localizing sign'' of increased intracranial pressure.

Question 1 -2 What is the next appropriate step in her evaluation? A) Lumbar puncture with opening pressure. B) Magnetic resonance imaging (MRI) of the brain with magnetic resonance venography (MRV) of the head. C) Treatment with a carbonic anhydrase inhibitor. D) Intravenous opiates. E) Nutrition consultation. Discussion 1 -2

The correct answer is "B:' You first need to assess for secondary causes of increased intracranial pressure with an MRI scan of the brain (looking for tumor and hydrocephalus) and an MRV (looking for venous sinus thrombosis) prior to performing a lumbar puncture with opening pressure (option "/\_') in the left lateral decubitus position. If her MRI and MRV are normal (venous sinus narrowing is allowed) and the opening pressure is elevated (> 25 em H 0 in most cases) with normal cerebro­ 2 spinal fluid ( CSF) cell count, she may be treated with a carbonic anhydrase inhibitor (option "C") to lower her CSF pressure. IIH is typically co-managed by ophthalmology and neurol­ ogy specialists. The risk in untreated IIH is permanent vision loss. Patients who are refractory to medications may require optic nerve sheath fenestration or ventriculoperitoneal shunt placement.





Helpful Tip

Sym ptoms of increased i ntracra n i a l pressu re include

1 1 1 r headaches that wa ke a pati ent up in the m i d d l e of the

n i g ht or occ u r fi rst thing i n the morning, especi a l ly

those with acco m panyi ng vom iti n g . Other wa rning signs i n c l u d e Va lsalva- or exertion-i n d uced headaches. Physica l exa m fi n d i n g s i n c l u d e pa pil ledema a n d foca l neurologic signs (CN VI palsy) .

51 1

MCG RAW-H I LL E D U CATION S P E C I A LTY BOA R D REVI EW: P E D I ATRICS

51 2



Helpful Tip

=� Neuroimaging

is indicated in children with headaches

r1 1 r who have signs of increased intracranial pressu re, seizures,

an abnormal neurologic exa m, or focal neurologic

sym ptoms (weakness, n u m bness). It should also be considered in children with new-onset severe headaches

(< 1 month d u ration) or worsening headaches, and in children you nger than 6 years of age.

You are seeing, a 1 0-year-old boy with asthma and attention deficit hyperactivity disorder (ADHD), for his well-child check. On review of systems he complains of frequent head­ aches during which he stops playing, is irritable with others, and demands that the television {TV) and lights be turned off. He is also uncharacteristically still and prefers to lie on the couch. He describes the headache as being "like a ham­ mer hitting his head:' He sees spots and then feels numbness and tingling in his left arm for 15 minutes before the head­ ache begins, but these go away once the headache starts. He is obese, but his examination is otherwise normal. Question 2-1 What is the most likely diagnosis? A) Tension-type headache. B) Migraine with aura. C) Hemiplegic migraine. D) Basilar-type migraine. E) Cluster headache. Discussion 2-1

The correct answer is "B:' A migraine is a long ( 4- to 72-hour) headache characterized by two of more of the following char­ acteristics: unilateral, pulsating quality, moderate-to-severe intensity, and worsening that causes the sufferer to avoid physical activity; and associated with one or more of the fol­ lowing: nausea, vomiting, or photophobia and phonopho­ bia. Some of these symptoms can be inferred by this child's behavior (light and noise avoidance) . A typical aura includes sensory or visual phenomena (weakness is excluded) that are fully reversible, last 5 to 60 minutes, and are followed shortly by a migraine. Hemiplegic migraine (option "C") includes an aura of fully reversible weakness (must first rule out intracranial pathology) , and cluster headache (option "D") includes an aura of several symptoms referable to the brainstem (ataxia, aphasia, vertigo, tinnitus, etc). Tension­ type headache (option "A") is essentially the opposite of a migraine and defined as a headache with two or more of the following characteristics: bilateral, mild-to-moderate sever­ ity, nonthrobbing (pressing, tight), not aggravated by physi­ cal activity, and without nausea or vomiting. The patient may have photophobia or phonophobia, but not both. Clus­ ter headaches (option "E") and other trigeminal autonomic

cephalgias are uncommon in children and by definition must include autonomic features such as unilateral conj unctival injection, tearing, rhinorrhea, or eyelid edema. It can be a bear to remember all of the headache subtypes, so put the Interna­ tional Headache Society's website (http:/ /ihs-classification.org/ en/) on your favorites list. It contains a handy index of diag­ nostic criteria. (Bet you have a headache from reading all of this ! ) Question 2-2 What treatment will you recommend? A) Lifestyle management with adequate sleep, regular meals, improved hydration, avoidance of caffeine, stress manage­ ment, and regular physical activity. B) Acetaminophen or ibuprofen as needed, no more than 1 0 times per month. C) Headache diary. D) Prophylactic medication if headaches are disabling or occur more than once a week. E) All of the above. Discussion 2-2

Of course the correct answer is "E"! Most pediatric headaches can be managed through simple lifestyle changes and the judi­ cious use of over-the-counter analgesics. If your patient uses any as-needed (PRN) meds more than 1 0 times per month, he may develop a medication overuse headache. Although select trip­ tans have been approved for use in children by the Food and Drug Administration (FDA) , nonsteroidal anti-inflammatory drugs (NSAIDs) typically work just as well if utilized at a proper dose at the onset of the headache.

� QUICK QUIZ Which should NOT b e used for management o f headaches in the clinic? A) NSAIDs. B) Acetaminophen. C) Opiates. D) Meditation (stress reduction) . E ) Sleep. Discussion

The correct answer is "C:' Opiates are not an appropriate choice for outpatient headache management.

When you see the patient in follow up 2 months later, his head­ aches are worse. Despite your expert counseling on proper lifestyle management, his mother cannot seem to get him to stop drinking soda, and he will not go to bed before 1 0:00 PM because he watches TV for hours after dinner. His mother is frustrated because he is now having daily headaches and she does not know what to do. They have adhered to your order of using ibuprofen less than 10 times per month.

C H A PT E R 23

Question 2-3

Which prophylactic medication would be the best choice for this child? A) Cyproheptadine. B) Propranolol. C) Topiramate. D) Valproic acid. E) None of the above. Discussion 2-3

The correct answer is "C:' When planning a prophylactic medication, you must think about the patient's comorbidities. Topiramate may cause weight loss and thus would be the pre­ ferred choice for an obese patient. At higher doses, such as when it is used for seizure medication, it can also cause word­ finding difficulties and brain fog. Another potential side effect is kidney stones. Option "B" is incorrect because beta-blockers should be avoided in patients with asthma. Options "!\' and "D" are not the best choices for an obese patient as their side effects include weight gain.

A 2-year-old girl is brought to the emergency depart­ ment by her family because she has been acting strangely ever since she was awakened from her afternoon nap. Her mother says the girl was at her baseline when she and her 4-year-old brother were put down for their naps 3 hours ago. When the mother went to wake her up, the girl was difficult to arouse and seemed confused. On examination, the patient's vital signs are heart rate 70 beats per minute, respiratory rate 12 breaths per minute, and blood pressure 75/50 mm Hg; she has a Glasgow Coma Scale score of 1 0 : she opens her eyes briefly only t o voice o r tactile stimula­ tion; and she cries and pulls away when you pinch her arm. She does not speak. Her general examination is unremark­ able except for miotic pupils. Question 3-1 Define her mental status. A) Alert. B) Lethargic. C) Obtunded. D) Stuporous. E) Comatose.



N E U ROLOG I C D I SO R D E R S

51 3

abnormal state, so please do not document lethargy when you really mean tired); option "C:' obtunded = opens eyes only to tactile or verbal stimulation; option "D;' stupor = opens eyes only to pain; and option "E;' coma = a state of unresponsive­ ness or unconsciousness without eye opening that is due to brain dysfunction. The differential diagnosis for pediatric altered mental status or encephalopathy is enormous but can be broken down into three main categories: toxic/metabolic, infectious, and structural. It is important to remember that maj or organ system dysfunction (liver, kidneys, heart, lungs) can lead to encephalopathy as well.



Helpful Tip

� M eta bo l i c

=-

eti o l og ies

r1 1r yo u r d iffe re n t i a l

should

be very

high

on

d i a g n os i s i n a n e n c e p h a l o p a t h i c

neon ate, a n d m eta b o l i c tests ( e g , u r i n e o rg a n i c a c i d s

a n d a m mo n i a ) s h o u l d be i n c l u d ed i n yo u r first l i n e o f test i n g .

Your initial lab work comes back normal, including full elec­ trolytes (sodium, potassium, chloride, bicarbonate, glucose) , renal function (creatinine, blood urea nitrogen), liver func­ tion tests (alanine aminotransferase, aspartate aminotrans­ ferase), ammonia, and blood gases. A noncontrast computed tomography (CT) scan of the head is also normal. When you go back to the family to gather more information for the his­ tory, the little brother proudly volunteers that he found some "candy" and gave it to his sister while they were supposed to be sleeping. You ask mother what medications are accessible in the house. She notes that her husband recently had back surgery and was prescribed hydrocodone for postoperative pain. You quickly administer naloxone to the child, and she returns to her baseline mental status. Question 3-2 Which of the following acute ingestions would NOT be expected to cause a depressed mental status? A) Amitriptyline. B) Aspirin. C) Cyanide. D) Lead. E) All of the above would cause a depressed mental state. Discussion 3-2

Discussion 3 - 1

The correct answer is "C:' It is exceedingly important to cor­ rectly document the patient's mental status in this situation. If you cannot recall what the words listed in options "!\' through "E" mean, then you should document what you see on exam (the patient opens eyes only to voice/pain, etc) . From most alert to least, the options choices for this question are defined as follows: option ''A;' Alert = fully awake (duh); option "B;' lethargic = difficulty staying awake (but it connotes a clearly

The correct answer is "D." Although lead can cause perma­ nent neurologic sequelae, it would be unusual for a child to be exposed to enough lead to cause an acute poisoning with coma. Lead is more likely to build up over time with repeated exposures. The substances classically known to induce coma include central nervous system (CNS) depressants such as sedatives, narcotics, alcohol, and barbiturates, as well as tricyclic antidepressants, cholinergics, cyanide, and carbon monoxide.

514

MCG RAW-H I LL E D U CATION S P E C I A LTY BOA R D REVI EW: P E D I ATRICS

� QUICK QUIZ What i s a potential cause o f altered mental status i n a pedi­ atric patient? A) Intracranial hemorrhage. B) Acute disseminated encephalomyelitis. C) Hyperinsulinemia. D) Acute respiratory failure. E) All of the above. Discussion The correct answer is "E:' Option "B" actually has the base term encephalitis as part of its disease name! Hyperinsulinemia may cause hypoglycemia, leading to altered mental status. Hyper­ capnia or hypoxia, or both, may also cause altered mental sta­ tus. Remember the mnemonic AEIOU-TIPS when evaluating a patient for altered mental status:

A - Ammonia

E - Electrolytes (sodium, glucose, calcium) I - Infection 0-

Overdose

U - Uremia T - Trauma I - Intussusception P - Psychiatric

Discussion 4- 1

The correct answer is "A:' The evaluation of a floppy infant is based on the clues found in the history and physical exam that help you determine if the hypotonia is central or periph­ eral in origin. (See Table 23- 1 . ) This infant most likely has a peripheral cause of his hypotonia; therefore, you should check the CK level to help narrow your differential. In muscular dys­ trophies the CK level is significantly elevated, but this is not the case in congenital myopathies and spinal muscular atro­ phy (SMA) . This infant's presentation (with profound hypo­ tonia, absent reflexes, and tongue fasciculations) is strongly suggestive of SMA type 1 , which involves progressive loss of the anterior horn cells. You confirm the diagnosis by send­ ing a sample for molecular testing of the SMNl gene. Treat­ ment is supportive. An MRI of the brain (option "B") would be more appropriate if you were worried about a central cause of hypotonia. EMG/NCV (option "C") can be useful in differ­ entiating whether peripheral hypotonia or weakness is related to nerve (anterior horn cell or motor nerve/axon disorders) , neuromuscular junction (acquired o r congenital myasthenic disorders, or botulism), or muscle disorders but would not be an appropriate first step. If you have a high suspicion for SMA from the start, you should send that specific gene test early in the workup. "D" isn't appropriate as this infant has signifi­ cant gross motor developmental delay. Referral for services is important in neuromuscular disorders but the first step in this case is a diagnostic work-up. Spinal muscular atrophy (SMA) is classified 1 through 4 depending on the severity of features

S - Stroke or Seizure TA B L E 2 3 - 1 C H A RACTE R I S T I C S OF C E NTRAL

VERSUS P E R I P H E RA L HYPOTO N I A

Ce ntra l

You are seeing a 4-month-old boy for his well-child check. He was a former 38-week average gestational age (AGA) infant born to a 23-year-old G.P0 (now 1 ) mother by normal spon­ taneous vaginal delivery after an uncomplicated neonatal course. His Apgar scores at birth were normal. His parents are concerned because he still cannot hold his head up, but note that he has no problems eating. On exam he is alert and reactive with normal general exam findings. His neurologic exam is notable for proximal greater than distal weakness, hypotonia with significant head lag, tongue fasciculations, absent Moro reflex, and absent biceps and patellar reflexes (you even used a real reflex hammer!) . There is no family his­ tory of neuromuscular disorder. You are worried about the cause of his hypotonia, areflexia, and weakness. Question 4-1 What is the next appropriate diagnostic step? A) Obtain a creatine kinase (CK) level. B) Order an MRI of the brain. C) Order an electromyogram (EMG) and a nerve conduction velocity (NCV) test. D) Provide reassurance and close follow-up. E) Refer for early access and intervention.

Pe r i p h e ra l

Menta l statu s

I m pa i red, let h a rg i c

I ntact, a l ert

Reflexes

N o r m a l t o brisk

D i m i n i s hed or a bsent

Strength

Movement t h ro u g h post u ra l refl exes, axial hypoton i a, m i l d to no wea kness

More profo u n d wea kness, no m ovement t h ro u g h pos­ t u ra l reflexes

Other org a n syste m dysfu nction

May have m u lt i p l e co ngen ita l ma lfor­ matio n s, dys m o r­ p h i c featu res, bra i n ma lformations, seizu res, a b n o r m a l breat h i n g

Poor feed i n g , respi ratory co m prom i se

Exa m c l ues

Scissoring on verti­ cal s u s pension, fi sted h a n d s with cortical th u m bs

M u scle fa scicu­ lations, to n g u e fa sciculations, m u scle atrophy, wea k/soft cry, wea k cou g h

C H A PT E R 23

and age of onset. SMA type 1 (Werdnig-Hoffmann disease) is the most common and severe with onset in early infancy. Decreased fetal movement may be noted by the mother in late pregnancy. SMA type 2 and type 3 are milder and present in late infancy or early in toddlerhood. SMA type 4 onsets in adulthood.

A 1 0-day-old male infant is admitted to the pediatric intensive care unit (PICU) for fever, seizures, and som­ nolence. His anterior fontanelle is full. A CT scan of the head did not show acute abnormalities. CSF showed ele­ vated white blood cells with mostly neutrophils, elevated protein, and low glucose. You suspect meningitis and start antibiotics. Question 5- 1 What are the most likely organisms in a patient of this age? A) Streptococcus pneumoniae, Neisseria meningitidis, Haemophilus influenzae type b. B) N. meningitidis, S. pneumoniae. C) Group B Streptococcus, Escherichia coli, Listeria monocytogenes. D) S. pneumoniae, N. meningitidis, L. monocytogenes. Discussion 5-1 The correct answer is "C." It is important to know the com­ mon organisms causing acute bacterial meningitis in each age group. (See Table 23-2.) The most common causes of men­ ingitis in a newborn are group B Streptococcus, E. coli, and Listeria. The typical empiric antibiotic regimen for neonatal meningitis is ampicillin and cefotaxime or gentamycin. Liste­ ria is resistant to cephalosporins, which is why you need to include ampicillin. Group B Streptococcus is sensitive to peni­ cillins. E. coli is a gram-negative rod, and is covered by cefo­ taxime and gentamycin.

TA B L E 23-2 CAU S E S OF ACUTE BACT E R I A L

M E N I N G I T I S B Y AG E

Age Gr o u p

Co m m o n Orga n i s m s

Newborn t o i nfa nt (3 months)

G ro u p B Streptococcus, Escherichia coli, Listeria monocytogenes

I nfa nt (> 3 months) to c h i l d

Streptococcus pneumoniae, Neisseria meningitidis, Haemophi/us influenzae type b

Ado l escent t o you n g a d u lt

N. meningitidis, S. pneumoniae

Older a d u lt

S. pneumoniae, N. meningitidis, Listeria monocytogenes



N E U ROLOG I C D I SO R D E R S

51 5

An 1 8-year-old college freshman is brought to the emergency department (ED) for altered mental status. His roommate reports that the patient was well this morning but when the roommate returned tonight, he found the patient on the floor, barely responsive. The roommate does not suspect drug or alcohol exposure. In the ED, the patient looks toxic. He is febrile, hypotensive, and obtunded, with diffuse petechiae. The head CT scan was unremarkable, and a drug screen was negative. You suspect meningitis. Question 6-1 What is the most likely organism causing his meningitis? A) Streptococcus pneumoniae. B) Escherichia coli. C) Listeria monocytogenes. D) Neisseria meningitidis. E) Enterovirus. Discussion 6-1 The correct answer is "D:' N. meningitidis is one of the common organisms causing meningitis among adolescents and young adults (college students, military personnel) . The doubling time of this organism is very short, leading to the rapid clinical progression. N. meningitidis can cause disseminated intravascular coagulation (DIC), with resulting petechiae, purpura, and adrenal insufficiency (Friderichsen-Waterhouse syndrome). A third-gen­ eration cephalosporin, such as ceftriaxone is the therapeutic choice.





Helpful Tip

Petechia and purpura i n meningitis should prompt you

1 1 1 r to think meningococcus; however, these fi ndings are

also seen in Streptococcus pneumoniae and Haemophi/us influenzae meningitis.

You are the senior resident in the emergency department in July when a 3-year-old boy who was sick with fever, cough, and congestion for 4 days presents with irritability, headaches, and a persistent fever. His exam is notable for photophobia and pain with neck flexion. There are no focal neurologic findings, altered mental status, or papilledema. You suspect meningitis and plan to perform a lumbar puncture (LP). Question 7-1 With what symptoms should you first do neuroimaging prior to LP? A) Imaging is required before any LP when you suspect meningitis. B) Any focal neurologic symptoms or altered mental status. C) Papilledema. D) Seizures. E) Options B, C, and D.

516

MCG RAW-H I LL E D U CATION S P E C I A LTY BOA R D REVI EW: P E D I ATRICS

Discussion 7-1

The correct answer is "E:' Routine neuroimaging is not needed unless the patient has seizures, you are suspicious for a mass lesion (eg, brain abscess) , or there are abnormalities on neuro­ logic examination. You should also consider imaging for neo­ nates with meningitis.

Initial CSF results show no organisms on Gram stain, but a high white blood cell count with lymphocytic predominance, mildly elevated protein, and normal CSF glucose.

Discussion 7-3

Question 7-2

What is the most likely etiology? A) Enterovirus. B) Streptococcus pneumoniae. C) Neisseria meningitides. D) Partially treated bacterial meningitis (mother forgot to mention the antibiotics he was on last week) . E) None of the above. Discussion 7-2

The correct answer is "A:' Enteroviral meningitis is the most likely pathogen given the time of year (summer) and the patient's CSF results, which are suggestive of a viral aseptic meningitis. While pretreated bacterial meningitis is possible, the initial CSF studies would be expected to show a neutrophilic predominance and a low glucose level. Other etiologies of aseptic meningitis include other viruses and bacterial pathogens such as tuberculous, spiro­ chete, or rickettsial infections. Noninfectious etiologies are also possible and include disorders such as lupus, malignancy, and Kawasaki disease, as well as medications ( eg, NSAIDs, intrave­ nous immunoglobulin [IVIG] ) . (See Table 23-3.)

Over the course of the next day, your patient becomes pro­ gressively more lethargic and has a focal seizure.

TA B L E 23-3 C E R E B ROS P I N A L F LU I D COM P O S I T I O N

I N M E N I N G IT I S

Eti o l ogy

Question 7-3 What acute complications of meningitis could account for these new symptoms? A) Cerebral edema. B) Syndrome of inappropriate antidiuretic hormone secretion (SIADH) . C) Subdural effusion. D) Dural venous sinus thrombosis. E) All of the above.

WBC Co u nt

Protein

ii

i

Bacterial

Glucose

Neutro ph i l predomina nce

ja

Vi ra l

Lym p hocyte predomina nce

Normal to m i l d ly I

ja

i

Mycobacterial

Normal

The correct answer is "E:' All of the listed options are poten­ tial complications of meningitis. Careful fluid management is key in meningitis treatment. Acute management of cerebral edema can include steroids, mannitol, diuretics, brief hyper­ ventilation if intubated, and hypertonic saline solution. Sub­ dural effusion may require surgical intervention if there is suspicion for empyema or if it causes increased intracranial pressure. Intracranial thrombosis may require anticoagulation therapy. Long-term complications of meningitis can include permanent neurologic deficits such as intellectual impair­ ment, hydrocephalus, spasticity, and seizures. Hearing loss is also a noted complication of bacterial meningitis and warrants testing in this population.

� QUICK QUIZ Which i s NOT a characteristic o f the CSF i n acute bacterial meningitis? A) Elevated protein. B) Elevated white blood cell count. C) Elevated glucose. D) Positive Gram stain. E) Neutrophilic predominance. Discussion

The correct answer is "C:' In acute bacterial meningitis, the white blood cell count is typically elevated (> 1 000 cells/mm3) and comprises primarily neutrophils. The protein content is elevated, and the Gram stain may be positive for bacteria. Hypoglycorrhachia (low CSF glucose) strongly suggests a bac­ terial infection. Be sure to obtain concurrent serum glucose to calculate the CSF-to-serum glucose ratio (normally �60%). The normal number of white blood cells in the CSF varies slightly by age, with 0 to 12 for neonates, 0 to 6 for infants 1 to 2 months old, and 0 to 7 for infants and children older than 2 months of age. (See Table 23-3, earlier.)

Monocyte predomina nce

ja

Fungal

Lym p hocyte predomina nce

i



Helpful Tip

:5.� Aseptic

meningitis

r1 1r i nfla m mation

is

a

catch-a l l

term

for

CSF

(leukocytosis) with negative bacteri a l

cultu res. T h e most c o m m o n ca use is vira l me ningitis.

a

M i l d ly el evated to h i g h .

C H A PT E R 23

A 7 -year-old girl presents to the emergency department with a 4-day history of fever to 38.9°C ( 1 02°F}, headache, photo­ phobia, and irritability. Her parents brought her in because she "has not been acting right" for the past 24 hours. They note that she is sleepier, is slow to respond, and seems "out of it:' The parents also mention that she developed a "cold sore" on her lip last week. You are testing her for nuchal rigidity (absent) when she has a focal seizure. An electroencephalo­ gram (EEG) shows a temporal lobe seizure focus. A CT scan does not show any contraindication to lumbar puncture (LP}. CSF studies show a mononuclear pleocytosis, elevated pro­ tein, and normal glucose. Question 8- 1

What diagnosis and organism do you suspect? A) Meningitis; enterovirus. B) Encephalitis; enterovirus. C) Encephalitis; herpes simplex virus type 1 . D ) Encephalitis; West Nile virus. E) Meningitis; chikungunya virus. Discussion 8-1

The correct answer is "C:' Encephalitis caused by herpes simplex virus (HSV) type 1 infection should be high on your differential diagnosis given the clinical picture consistent with encephalitis (brain inflammation manifesting as altered level of conscious­ ness, fever, headache, and possibly focal neurologic features such as seizure) in a child with a history of a recent herpes lesion (cold sore) . An overlap syndrome of meningoencephalitis may also develop, in which both the meninges (nuchal rigidity) and brain are affected. HSV is a common cause of encephalitis, second only to enterovirus in children. Other infectious causes include viral agents such as arboviruses and influenza, as well as a myriad bac­ terial, amoebic, and parasitic agents. If you are suspicious of HSV encephalitis and the first CSF HSV PCR is negative, repeat the test because false negatives have been known to occur in early infection. Always start your patient on empiric acyclovir while awaiting the test results, as morbidity and mortality are high with­ out therapy. Treatment of herpes encephalitis includes 2 1 days of intravenous (IV) acyclovir followed by an end-of-therapy LP for repeat CSF HSV PCR testing. The repeat testing ensures infection resolution prior to stopping IV acyclovir. 1 3



N E U ROLOG I C D I SO R D E R S

51 7

Discussion 8-2 The correct answer is "A:' Intoxication causes CNS depression (encephalopathy) not inflammation (encephalitis) . Viruses are the most common cause of encephalitis. Anti-NMDA (N-methyl-D-aspartate) receptor encephalitis is an autoim­ mune process that manifests with behavior and personality changes, then progresses to autonomic instability, seizures, coma, and even death. It is commonly a paraneoplastic process in adults (associated with ovarian teratoma) . It is one of several antibody-mediated encephalitis conditions that are all rare in children. (See Table 23-4.}



Helpful Tip

.s.::'ll Encephal itis a n d encephalopathy both ca use cerebra l

i1 1r dysfu nction . T h e d ifference is that enceph a l itis resu lts from i nfla m mation of the bra i n ; encephal opathy is a n o n i nfla m matory ma n ifestation of a systemic i l l n ess.

TA B L E 23-4 CAU S E S O F E N C E P H A L I T I S

Ca use

Exa m p l e

I nfectious

Ente rov iru s Pa rechovi rus H SV 1 a n d 2 Adenovi rus Epstei n-Barr virus H IV West N i l e virus St. Lo u i s v iru s Ca l ifornia encepha l itis virus LaCrosse encep h a l itis virus Ea ste rn eq u i n e encepha l i t i s v i rus Weste rn eq u i n e encepha l itis virus Rickettsia rickettsii (Rocky Mounta i n s potted fever) Borrelia burgdorferi (Lyme d i sease) Bartonella henselae (cat-scratch d i sease) Mycoplasma pneumoniae

You start empiric treatment for HSV and monitor the patient closely. You start mentally reviewing the differential diagno­ sis of encephalitis in a child. Question 8-2 Which is NOT a cause of encephalitis? A) Intoxication. B) Arbovirus infection. C) Cat-scratch disease. D) Malignancy. E) Autoimmune disorders.

Pa rasites Fu n g u s Autoi m m u ne

Anti- N M DA rece ptor encepha l i t i s

Posti nfectious

Acute d i ssemi nated encephalomyel itis

Va sc u l itis Meta b o l i c M a l i g n a ncy H IV, h u m a n i m m u n odeficiency virus; HSV, herpes s i m plex vi rus; N M DA, N-methyi-D-aspa rtate.

518

MCG RAW-H I LL E D U CATION S P E C I A LTY BOA R D REVI EW: P E D I ATRICS

Apparently it is seizure day in the emergency department. The third chart you open is a 9-year-old boy who has been transferred from outside the hospital. He has a history of chronic otitis media, was diagnosed with another ear infection last week, and was placed on amoxicillin. He presented to the emergency department with persistent fevers and new-onset seizure characterized by unrespon­ sive staring, lip smacking, and then generalized jerking. The outside CT scan was "abnormal" so he was sent to you for further evaluation. On examination, the child is back to baseline mental status, and the parents deny any alteration of consciousness before or after the seizure. You see a hypodense lesion in the left temporal lobe. (See Figures 23- 1 and 23-2.) Question 9- 1 What is the lesion, and what will be your next diagnostic step? A) Acute stroke; brain MRI without contrast. B) Acute stroke; brain MRI and MRV with and without contrast. C) Brain tumor; brain MRI with and without contrast. D) Brain abscess; brain MRI with and without contrast. E) Brain arteriovenous malformation (AVM) ; MRV with and without contrast.

F I G U R E 23-2. Temporal lobe bra i n a bscess on M R I . (Used with permission from Nathan Price, M D.)

Discussion 9-1

The correct answer is "D:' Although stroke, tumor, and AVM are included in the differential diagnosis for new-onset sei­ zure in the setting of a brain lesion, this is more likely to be an abscess given the infectious history. Infection of the middle ear or sinuses can rarely lead to more invasive infection of nearby areas such as the meninges and brain. Brain abscess secondary to otitis media tends to affect the temporal lobe, whereas sinus­ itis can lead more often to frontal lobe abscess. This complica­ tion should be suspected if the patient has altered mental status, meningeal signs, seizures, or focal neurologic abnormalities. An MRI with and without contrast will better define the lesion and assist with medical and surgical management. Abscesses are characteristically ring-enhancing with contrast studies.



Helpful Tip

� Ea r

=-

a n d s i n u s i nfections ca n extend i ntracra n i a l ly,

r1 1 r ca using

m e n i n g itis, encephal itis, epid u ra l a bscess,

s u bd u ra l e m pyema, bra i n a bscess, d u ra l venous

throm bosis, cavernous s i n u s throm bosis, stroke, and osteomyel itis of the s ku l l .

� QUICK QUIZ F I G U R E 23-1 . Tem poral lobe bra i n a bscess on CT sca n . (Used with permission from Nathan Price, M D.)

Which i s NOT a common risk factor for a brain abscess? A) Congenital heart disease. B) Chronic otitis media.

C H A PT E R 23

C) Endocarditis. D) Viral meningitis. E) Craniotomy. Discussion The correct answer is "D:' Pathogenesis ofbrain abscess involves hematogenous seeding (endocarditis) , direct spread from con­ tiguous structures such as sinuses and middle ear (untreated otitis media), and direct inoculation in head trauma (open skull fracture, oral trauma) and neurosurgical cases. Congenital heart disease, especially with right- to-left shunting, may cause a brain abscess in the setting of bacteremia. The causative organism reflects the mechanism of infection. For example, brain abscess that formed due to complication of sinusitis reflects the common sinusitis organisms (Streptococcus pneumoniae, Haemophilus influenzae, anaerobes) . Organisms from the lungs (Aspergillus, Cryptococcus, Nocardia) can seed the brain through the blood­ stream; this most often occurs in immunocompromised hosts. Hematogenous spread from endocarditis is often caused by Staphylococcus aureus and Streptococcus viridans. A positive blood culture for S. viridans is not always a skin contaminant!



N E U ROLOG I C D I SO R D E R S

519

not urinated all day today despite drinking plenty of fluids. He has no preexisting health problems and no prior episodes of weakness, but did have a mild upper respiratory infection I 0 days ago. On exam, his mental status, cranial nerves, and upper extremity function are intact. He has decreased sensa­ tion below the level of the umbilicus, with diminished reflexes and flaccid asymmetric leg weakness (affecting the right more than the left) . He is catheterized and a large amount of urine is drained from the bladder. MRI of the spine reveals a contrast-enhancing, demyelinating lesion in the lower tho­ racic spine. CSF is notable for elevated protein with a normal WBC count. Question 1 0-1 What is the most likely diagnosis? A) Guillain-Barre syndrome. B) Multiple sclerosis. C) Transverse myelitis. D) Spinal cord tumor. E) None of the above. Discussion 1 0-1

MRI with and without contrast is performed in your 9-year­ old patient, revealing a solitary 2.5 mm ring-enhancing lesion consistent with abscess, and no signs of cerebral edema or mass effect. You perform a lumbar puncture and initial studies are pending. Question 9-2 What factors will lead you to consider antibiotic therapy alone over medical and surgical management (aspiration, drainage, excision) ? A) Isolation of a causative organism in the CSF with concomitant meningitis. B) Abscess location that is very deep or in a high-risk area. C) An unstable, poor surgical candidate. D) All of the above. Discussion 9-2

The correct answer is "D:' Multiple factors will weigh into this treatment decision, including the need to isolate an organism for proper treatment. Given that your patient was pretreated with antibiotics, it may not be possible to identify the organism without surgical intervention. Likewise, he is an excellent surgi­ cal candidate given that he is otherwise healthy and has a small, solitary abscess without significant mass effect. Antibiotic regi­ mens to treat a parenchymal abscess are prolonged and typically range from 3 to 6 weeks, with repeat imaging often used to assist in determining length of therapy.

A I S-year-old boy presents to the emergency department with a 5-day history of progressive leg weakness and numb­ ness. He has not been able to walk for the past 2 days and has

The correct answer is "C:' Transverse myelitis (TM) is an acute to subacute, often postinfectious inflammation of the spinal cord that can lead to significant flaccid weakness and depressed or absent reflexes below the level of the lesion. There is often a sensory level and, depending on the site of the lesion, bowel and bladder dysfunction. It is treated with high-dose IV ste­ roids or plasma exchange, or both. TM can be distinguished from Guillain-Barre syndrome (option "A''), another common postinfectious neurologic syndrome, by the presence of the spi­ nal level and the focal lesion in the spinal cord. Guillain-Barre syndrome is classified as a polyradiculoneuropathy (typically demyelinating) and affects the peripheral nervous system (PNS; nerves and nerve roots) , whereas TM affects the CNS (spinal cord) . Although TM may be the initial sign of a future demye­ linating disease, you cannot diagnose multiple sclerosis (option "B") at this time without evidence of previous clinical attacks of weakness or old lesions on MRI. Option "D;' a spinal cord tumor, is most definitely on your differential diagnosis and is the reason you obtained emergent neuroimaging in the emer­ gency department.

A IS-month-old girl is brought to the clinic for language regression. You do a brief chart review before going in to see her. She was born at term, has no significant past medical history, and has met all developmental milestones on time. Notably, her head circumference has dropped from the 25th to the 2nd percentile in her last few well-child checks. Her mother reports that she has been fussier, is not sleeping well, has lost interest in her surroundings, and is no longer feed­ ing herself. In addition, she has stopped saying several of the words she previously knew. On examination, you note a small

MCG RAW-H I LL E D U CATION S P E C I A LTY BOA R D REVI EW: P E D I ATRICS

520

child with no dysmorphic features. There is no hepatospleno­ megaly and her funduscopic exam is normal. She has poor eye contact. She appears mildly anxious and has an irregu­ lar breathing pattern with periods of breath-holding. She is wringing her hands together in the midline in a nonpurpose­ ful fashion. Her gait is clumsier than expected for her age. During the exam, she has a brief complex partial seizure. There is no family history of neurologic conditions.

a first-time generalized seizure that lasted less than 2 min­ utes and self-resolved 20 minutes ago. She is afebrile and has no history of recent head trauma or illness. She is sleepy on exam, but follows commands and has a completely nor­ mal neurologic exam. Results of a full electrolyte panel with renal and liver function tests and a urine drug screen are all normal. Question 1 2-1

Question 1 1 -1

What is the most likely diagnosis? A) Tay-Sachs disease. B) Krabbe disease. C) Rett syndrome. D) Autism. E) None of the above.

What is the next appropriate management step? A) Obtain a head CT scan stat. B) Obtain an EEG before discharge from the ED. C) Counsel the family about epilepsy and provide a prescrip­ tion for levetiracetam. D) Order an outpatient EEG. E) Admit the patient overnight for hospital observation.

Discussion 1 1 - 1

Discussion 1 2- 1

Th e correct answer i s "C' Neurodegenerative disorders present with developmental regression or arrest and include numerous hereditary and metabolic causes. Rett syndrome is a neurode­ generative disorder characterized by an initial period of normal development followed by a period of rapid developmental regres­ sion that typically occurs around 12 months of age (range, 5 to 18 months) . This is accompanied by postnatal deceleration of head growth, complete or partial loss of purposeful hand use and language, stereotypical hand movements (midline wringing, clapping, tapping), and gait ataxia. Patients then have a period of relative stability following the regression, unlike other neurode­ generative diseases that continually worsen. However, they even­ tually have motor regression. Girls are almost exclusively affected. While awake many children with Rett syndrome cycle between hyperventilation and hypoventilation/apnea. Management is supportive, and survival is usually into adulthood. Children with Krabbe disease (option "B"), a lysosomal storage disorder, may initially present with irritability; however, the disorder usually manifests earlier in infancy, around 4 months of age. It is a rap­ idly progressive demyelinating disorder characterized by progres­ sively increased muscle tone, psychomotor regression, blindness, and seizures. Tay-Sachs disease (option "A") likewise manifests with psychomotor regression around 6 months of age and is accompanied by a cherry red spot on funduscopic exam and hepatosplenomegaly. Autism (option "D") should not be accom­ panied by marked developmental regression or microcephaly.

The correct answer is "D:' Outpatient EEG is recommended based on the American Academy of Neurology practice param­ eter for a first-time afebrile (unprovoked) seizure of a child aged 6 months or older. The only reason to get an urgent EEG in the ED (option "B") is if there are concerns for ongoing subclini­ cal status epilepticus. This patient is sleepy because she is still postictal, but she readily follows commands and has a normal neurologic exam, making subclinical status unlikely. Laboratory studies are recommended if clinically appropriate to evaluate for toxic or metabolic etiologies (hyponatremia, hypoglycemia, ingestions) . Neuroimaging recommendations are also clinically based. An emergent head CT (option "A'') is indicated only to rule out an acute intracranial process that may require immedi­ ate intervention. There is no need to obtain a CT scan for every child with a seizure who walks through the door! MRI is always the preferred imaging choice for seizures, but this can be done on an outpatient basis. In most cases, these children should all receive an outpatient EEG, MRI, and neurologic consultation. Option "C" is incorrect, as there is no benefit in treating a child with a first-time seizure rather than waiting to see if he or she develops epilepsy. Patients are more likely to develop epilepsy if they have an abnormality on EEG, prior febrile seizures, a prior insult known to cause seizures (encephalitis) , nocturnal seizures, or a postictal Todd paralysis.



Helpful Tip

:a� Any

c h i l d with loss of developmenta l m i l estones

r1 1 r (reg ression) wa rra nts a neurolog ic consu ltation a n d M R I o f the bra i n . These s h o u l d be completed u rg ently.

You are rotating in the emergency department (ED) and pick up the chart of a previously healthy 5-year-old girl who was brought in by emergency medical services (EMS) with

� QUICK QUIZ I n which o f the following situations would patients require emergent neuroimaging? A) Seizure after high-speed collision with a tree in a skiing accident. B) Seizure accompanied by early morning headache, vomiting, and papilledema. C) Seizure followed by prolonged postictal period (more than several hours). D) Seizure with postictal weakness of the right arm and leg. E) All of the above.

C H A PT E R 23

Discussion

The correct answer is "E:' Emergent neuroimaging is required to rule out acute intracranial processes in all of these scenar­ ios. Option "X' is worrisome for an epidural hematoma; option "B" is suggestive of acute hydrocephalus; option "C" suggests an underlying toxic, metabolic, or infectious etiology; and although option "D" may simply be a postictal Todd paralysis, you must rule out an acute stroke or other intracranial pathol­ ogy in anyone with a focal neurologic deficit.



Helpful Tip

::'Jl Epilepsy

.s.

r-1 1r seizu res

is defi ned as two or more u n provoked occurring at l east 2 4 hours a pa rt. Once

the diag nosis is made, c h i l d ren a re placed o n an a ntiepileptic med ication based on their seizu re type.

The majority of c h i l d ren ca n be completely contro l l ed with medications (60-70%). Attem pts to wea n off of medications a re typica l ly made if the patient rema i n s seizu re-free for 2 yea rs on med ications.

Question 1 2-2

Would your initial evaluation and differential diagnosis change if the child was a 5-day-old infant with a first afebrile seizure? A) No, a seizure is a seizure, and workup is independent of age. B) Yes, I would be more worried about serious underlying pathology. Discussion 1 2-2

The correct answer is "B:' Neonatal seizures are a whole different beast. First of all, their appearance is different than typical child­ hood seizures because neonatal brains are immature and not well myelinated. As a result, neonates cannot have a classic general­ ized tonic-clonic seizure. Typical neonatal seizure manifestations include apnea and stiffening, rhythmic jerking of one limb or side of the body, jerking of multiple limbs but at different times, or even bicycling movements of the legs. Neonatal seizures require emergent workup looking for underlying metabolic or toxic eti­ ology, structural brain defects, and serious bacterial infection. Some examples include hypocalcemia, drug withdrawal, menin­ gitis, sepsis, hypoxic ischemic encephalopathy, urea cycle disor­ ders, intracranial hemorrhage, and glycine encephalopathy.



N E U ROLOG I C D I SO R D E R S

521

not respond to touch or voice. About 1 5 seconds later she resumes blowing on the tissue. When you ask her about it she has no idea that she stopped blowing. Her parents note that this was exactly what happened at dinner. Question 1 3-1 What is the most likely diagnosis and recommended treatment? A) Complex partial epilepsy; oxcarbazepine or carbamazepine. B) Childhood absence epilepsy; ethosuximide. C) ADHD; stimulant medication. D) Behavioral staring spells; no treatment necessary. Discussion 1 3- 1

Th e correct answer i s "B:' Childhood absence epilepsy i s an epilepsy syndrome characterized by brief seizures (5- 1 5 sec­ onds) consisting of behavioral arrest, staring, and occasional repetitive, nonpurposeful facial movements (blinking, chew­ ing) called automatisms. The seizures can occur upwards of 1 00 times per day and can significantly impact learning abilities. The characteristic EEG finding is a 3 -Hz generalized spike-and­ wave pattern. The seizures can usually be provoked in the office by hyperventilation (tissue blowing) . Interestingly, children with absence epilepsy also have coexisting learning issues and ADHD, so careful screening will still be necessary once seizures are controlled. Ethosuximide is the first-line treatment. The vast majority of children outgrow the seizures. A minority also have generalized tonic-clonic seizures. Although it can be difficult to distinguish a complex partial seizure (option "X') from an absence seizure, typically complex partial seizures last longer (> 1 minute) and are preceded by an aura and followed by a postictal period of fatigue. They are not induced by hyperven­ tilation. Behavioral staring or daydreaming (options "C" and "D") can be distinguished from seizures by the fact that they are interruptible by voice or touch. (Children may ignore being called, but have a hard time ignoring a tickle or pinch! )



Helpful Tip

::'Jl Seizu re m i m ics i n c l u d e breath-h o l d i n g spel ls, Sand ifer

.s.

d rome (refl ux-related opisthotonic posturing), tics, r-1 1r syn syncope, and self-sti m u lation or g ratification behavior. These m i mics ca n be d ifferentiated from seizu re by a

ca refu l h i story of event timi ng, trigg ers, a n d the a b i l ity to d i stract a c h i l d out of an episode.

A 6-year-old girl presents to the clinic with her mother for evaluation of a possible learning disability or attention def­ icit disorder (ADD). She is having a hard time learning in kindergarten and is often caught "daydreaming:' Last night at dinner she suddenly stopped eating, stared for 15 seconds, and then resumed as if nothing was wrong. This event scared the parents so they brought her in today. She is a normal, healthy child with an intact neurologic examination. When you have her blow repeatedly on a tissue, she suddenly stops blowing, stares straight ahead with eyelid fluttering, and does

� QUICK QUIZ Which o f the following i s true regarding seizures? A) Partial seizures may or may not impair consciousness. B) Generalized seizures always involve changes in motor activity or tone. C) Absence seizures are focal seizures. D) Simple partial seizures may be preceded by an aura. E) Complex partial seizures always involve lip smacking.

522

MCG RAW-H I LL E D U CATION S P E C I A LTY BOA R D REVI EW: P E D I ATRICS

TA B L E 23-5 S E I Z U R E C H A RACTE R I ST I C S

Simple Pa rti a l

Co m p l ex

Seizu re Type

( a u ra)

Pa rti a l

Abse n ce

G e n e ra l ized

Spasms

Leve l of co nscious ness

Prese rved

I m pa i red

I m pa i red

I m pa i red

I m pa i red

Appeara n ce

Foca l jerking, stiffness, or sensory sym ptom

Foca l stiffness or jerki n g

Behavior arrest, facial a utomati s m s (b l i n ki ng, chewi ng)

Genera l ized stiffness, jerking, or both

Brief flexi o n or exte nsion of a r m s/a bdomen, looks l i ke a "cru n c h"

Typical d u ration

< 2 min

< 2 min

5-20 sec

< 2 min

C l u sters u po n awa ke n i n g

y

I nfa nti l e

Posti cta l period

N

N

y

N

AED

Oxca rbazepi ne, l eveti raceta m, l a m otri g i n e

Ethos uxi m i d e, va l p roic acid, l a m otri g i n e

Leveti raceta m, top i ra m ate, va l p roic acid

ACTH or vigabatrin (if c h i l d has tu bero u s sclerosis)

EEG

Foca l d i sc h a rg es/a b n o r m a l ities

3-Hz genera l ized s p i ke-a nd-wave patte rn

Genera l ized d i sc h a rges

Hypsa rrhyt h m i a

ACTH, a d renocorticotropic hormone; AED, a ntiepileptic d rug; EEG, el ectroencephalogram.

Discussion The correct answer is "A:' Seizures are classified as partial or generalized. Consciousness is not impaired with simple partial seizures. Complex partial seizures may be preceded by an aura, cause impaired consciousness, and may or may not be associated with automatisms (lip smacking, drooling) . Partial seizures can evolve into generalized seizures. Generalized seizures include absence, myoclonic, tonic, clonic, tonic-clonic, and atonic. (See Table 23-5.)

A 13-year-old postmenarchal girl comes to the clinic with con­ cerns of early morning clumsiness after several episodes of breaking dishes and glasses at breakfast. She describes an uncon­ trollable brief jerk of her shoulders and arms that causes them to fling out, resulting in the broken dishes. On further question­ ing, she admits to having had a few episodes where she woke up having wet the bed at night and with blood in her mouth from biting his tongue. Her exam is normal. You suspect seizures and order an MRI, which is normal, and an EEG, which shows a 4- to 6-Hz generalized polyspike-and-wave pattern. Question 1 4-1 What is the most likely diagnosis? A) Juvenile myoclonic epilepsy. B) Benign rolandic epilepsy. C) Complex partial epilepsy. D) Childhood absence epilepsy. E) None of the above.

Discussion 1 4- 1

The correct answer is ''A:' Myoclonic seizures are brief, invol­ untary (usually single) , asymmetric j erks typically involving the arms. They characteristically occur in the morning hours, lead­ ing to dropped toothbrushes, curling irons (and burns! ) , and broken dishes. Children are often accused of being clumsy until they have their first generalized tonic-clonic seizure. A minority of children also have absence seizures. Benign rolandic epilepsy (option "B") is a form of autosomal dominant epilepsy that is typically "benign" because the majority of children only have a handful of seizures over their lifetime. It typically manifests around 8 years of age and remits by age 14. The typical seizure occurs at night and consists of one-sided facial tingling followed by twitching, drooling, and difficulty speaking. The child is fully aware of what is going on. The seizure can spread to involve the arm or even go on to a generalized tonic-clonic seizure. Treat­ ment is not needed unless seizures are frequent or disruptive, in which case oxcarbazepine or carbamazepine is preferred. The seizures described in this patient are all generalized (myoclonic and presumed generalized tonic-clonic) and would not be part of complex partial or absence epilepsy. Question 1 4-2 What is the proper treatment? (Bonus question: What are the most common side effects of the following drugs?) A) Oxcarbazepine. B) Valproic acid. C) Levetiracetam. D) Ethosuximide. E) Any of the above.

C H A PT E R 23

Discussion 1 4-2

The correct answer is "C:' Levetiracetam is a well-tolerated medication. Typical side effects include behavioral issues (irri­ tability, moodiness) . Although valproic acid (option "B") is also used, it is preferentially avoided in women of childbearing age due to the potential for birth defects (neural tube defects). It can also cause pancreatitis, liver failure, and thrombocytopenia and requires careful blood monitoring. Ethosuximide (option "D") is used in childhood absence epilepsy (but note that it does not treat generalized tonic-clonic seizures). Side effects include gastrointestinal upset and risk of Stevens-Johnson syndrome (which also can occur with lamotrigine) . Oxcarbazepine (option "A'') is an excellent choice for partial seizures but may actually worsen generalized seizures. Side effects include hyponatremia and ataxia. Carbamazepine has a similar side effect profile to oxcarbazepine, with the added concern for bone marrow sup­ pression and DRESS (drug reaction with eosinophilia and sys­ temic symptoms). All seizure medications have the potential to cause cognitive issues and sleepiness at higher doses. For intrac­ table seizures, the child may be placed on a ketogenic diet.

During another busy day in clinic, a 5-month-old boy is referred to you for evaluation of atypical movements. The infant was born at term and had been healthy with normal development. A few weeks ago, he started having attacks dur­ ing which his body crunches up repeatedly then stops. The dusters frequently occur right after he wakes up. The mother shows you a video on her phone. During the episode, the infant's head thrusts forward while his entire body repeat­ edly flexes together and then extends. Question 1 5-1 What is the most likely cause of these events? A) Infantile spasms. B) Lennox-Gastaut syndrome. C) Dravet syndrome. D) Sleep myoclonus. E) Tremors. Discussion 1 5-1 The correct answer is "A:' Infantile spasms (also known as West syndrome) begin in the first year oflife with peak onset between 4 and 6 months of age. Seizures occur in clusters and consist of head bobbing with flexion and extension of the trunk and extremities (like opening and closing a jackknife) . The infant may cry out before seizing. The EEG shows hypsarrhythmia. (If a test question mentions hypsarrhythmia, stop reading and look for infantile spasms in the answer choices.) Approximately 70% of children have an underlying CNS disorder or malformation. Lennox-Gastaut syndrome is characterized by multiple seizure types, difficult to control seizures, cognitive impairment, and a slow spike-and-wave pattern on EEG. Severe myoclonic epi­ lepsy of infancy (Dravet syndrome) is associated with intrac­ table seizures beginning in the first year of life.





N E U ROLOG I C D I SO R D E R S

523

Helpful Tip

:5.� When

eva l uating a c h i l d who may be seizi ng, put

i1 1r you r hand on the s h a ki n g body pa rt to see if you ca n

exti n g u i s h the movement. If you ca n, the c h i l d is not having a seizu re.

There is a full moon and the emergency department is full of seizing children. EMS brings in another seizing patient; this time it is a 9-month-old girl who has been having rhythmic twitching of her left side for 25 minutes. The paramedics gave a dose of IV lorazepam 5 minutes ago. She is febrile to 38.8°C ( 1 02°F) and her airway and breathing are stable. She has no past medical history, is up-to-date with immunizations, and has never had a seizure before. Question 1 6-1 What is the next treatment step for status epilepticus? A) Fosphenytoin 20 mg/kg IV B) CT of the head stat. C) Repeat dose of IV lorazepam. D) Phenobarbital 20 mg/kg IV E) Neuromuscular blocking medication. Discussion 1 6- 1

As with any critically ill patient, first complete a primary survey, the ABCs, and stabilize the patient's cardiorespiratory status. Also think about calling a code blue. Regarding the seizing, the cor­ rect answer is option "C": repeat the dose of IV lorazepam if the seizure does not stop 5 minutes after the first dose. If the infant continues to seize 5 minutes after the second dose, your next step in the treatment of status epilepticus will be an IV bolus of an antiepileptic drug such as fosphenytoin or phenobarbital. It is helpful to obtain a drug level 2 to 4 hours after the antiepileptic load in case further medications are needed. Iatrogenic paralysis (option "E") will stop the infant's body from jerking, but it will not stop the seizure. For practical purposes, status epilepticus is defined as a seizure lasting more than 5 minutes, because this is typically when you will intervene. It is more traditionally defined as a seizure lasting more than 30 minutes, or two or more seizures without return to baseline in between. (See Table 23-6.)

You administer the second dose of lorazepam and the seizure stops. You obtain a full electrolyte panel, including calcium, glucose, sodium, and magnesium; urinalysis; urine drug screen; and CBC. All labs come back normal. The patient returns to baseline and looks well, and you go to counsel the parents. She has no signs of meningitis and has not received any antibiotics recently. Question 1 6-2 What is her diagnosis? A) Simple febrile seizure. B) Complex febrile seizure.

524

MCG RAW-H I LL E D U CATION S P E C I A LTY BOA R D REVI EW: P E D I ATRICS

TABLE 23-6 M A N AG EM E NT O F STAT U S

E P I L E PTI C U S

Ti m e i n m i n utes

Action

0-5

Sta b i l ize the patie nt: ABCs: assess a i rway, breat h i ng, c i rc u lation Obta i n access (IV or i ntraosseo u s) Ad m i n i ster oxyg en

EEG, and MRI are not indicated. LP should be considered in ( 1 ) a child with clinical signs o f meningitis, (2) a 6 - to 12-month­ old who is not fully immunized, or in whom the immunization status is unknown, or (3) a child who has been pretreated with antibiotics.



Helpful Tip

=-::'E... Beware

of ca l l i ng everything a febrile seizu re. Pay

i1 1 r ca refu l attention to the age cutoffs in the defi n ition of febrile seizu res. A 5-month-old with fever a n d

seizu re has a serious i nfection u n t i l proven otherwise.

Obta i n stat b l ood g l u cose a n d correct if l ow

Li kewise, a 7-yea r-old with a fever and seizu re deserves fu rther worku p a n d neurologic eva l uation.

Obta i n bl ood wo rk Ide ntify a n d treat reversi ble causes 5-1 0

G ive med i cati ons: Lorazepam, 0.05-0 . 1 mg/kg IV, o r Diazepam, 0.2-0.5 mg/kg I V

1 0- 1 5

If seizu res persists: Repeat loraze pa m or d i azepam

1 5-25

If seizu re persi sts, load with one of the fo l l owi n g :





Helpful Tip

Psychogenic nonepil eptic events (pseudoseizu res)

1 1 1r a re

a conversion d isorder best managed through

psychological ca re. Clues to psychogenic events

include

eye

closu re,

i ntact

awareness

d u ri n g

general ized s h a king, pelvic th rusting, prolonged or waxing a n d wa n i n g events, a n d lack of a posticta l period. The u nd erlyi ng stressors a re not always

Fosphenytoi n, 1 5-20 mg P E/kg IV or I M

a p pa rent (or even pa rticula rly severe), but patients

Phenyto i n , 1 5-20 mg/kg I V

should always be screened for a buse.

Phenoba rbita l, 1 5-20 m g/kg I V 25-40

If seizu re persi sts, g ive one o f the fol l owi n g : Leveti raceta m, 20-30 mg/kg I V Va l p roate, 20 mg/kg IV Phenoba rbita l (if fosphe nyto i n or phe nyto i n g iven) Add itional phenyto i n or fosphenyto i n

40-60

If seizu re persists: Pentobarbital, lorazepam, o r genera l a nesthes ia-ind u ced coma Avoi d para lytics

I M, i ntra m uscu l a r; IV, i ntravenous; PE, phenyto i n equ iva l ents.

Discussion 1 6-2

The correct answer is "B:' A simple febrile seizure occurs between 6 and 60 months of age, is associated with a fever of 38°C ( 1 00.4°F) or higher, is generalized, lasts 15 minutes or less, and only occurs once in 24 hours. Febrile seizures are consid­ ered complex if they are focal (as in our patient), last longer than 15 minutes (as in our patient) , or reoccur in a 24-hour period. Children with simple febrile seizures have only a slightly higher incidence of epilepsy (2% versus 1 %) than the general popula­ tion and typically do very well. Children with complex febrile seizures are slightly more likely to develop epilepsy. Febrile seizures occur in families. Evaluation of simple febrile seizures includes evaluation for the source of fever. Routine lab testing,

A 2-year-old previously healthy girl presents to the emergency department for unsteady gait. She was in her normal state of health until last night. This morning, she has not been able to walk since she woke up. Otherwise, she is in good spirits and does not appear to be in pain. She is currently afebrile but had a viral illness 2 weeks ago. On exam, she is alert and talks nor­ mally. She has no focal neurologic signs, but has truncal ataxia and dysmetria when she reaches for her toy. She is ataxic and unable to walk unassisted. Reflexes are normal throughout. Question 1 7-1 What is the most likely diagnosis? A) Drug ingestion. B) Posterior fossa tumor. C) Acute cerebellar ataxia. D) Acute demyelinating encephalomyelitis (ADEM). E) Behavioral. Discussion 1 7-1 The correct answer is "C:' Acute cerebella ataxia is a postinfec­ tious autoimmune process, and is thought to be due to cross­ reactivity of antiviral antibodies to cerebellar epitopes. It is most commonly associated with varicella zoster virus and Epstein­ Barr virus, but can occur with other infectious processes. A pre­ viously healthy child develops ataxia mostly affecting his or her gait over hours to a few days (rapid progression) . Mental status

C H A PT E R 23

is not affected. MRI may be normal or may show enhancement of cerebellum (cerebellitis) . CSF usually shows mild pleocyto­ sis. Acute cerebellar ataxia is usually a self-limiting disease, and affected children improve over weeks to a few months. Progno­ sis for complete recovery is excellent. Only a minority of patients experience persisting ataxia. ADEM is also a postinfectious pro­ cess, but patients have encephalopathy (mental status change), multifocal deficits on exam, and multifocal lesions on MRI. A child with a posterior fossa tumor does not present with acute ataxia unless there is acute associated hemorrhage or it leads to acute obstructive hydrocephalus. Unlike the child described in the case, such patients do not appear well.



Helpful Tip

•::'Jl Toxic exposu re is the

r1 1r ataxia

most common ca use of acute

i n c h i l d ren. I n tod d l e rs, accidenta l i n g estion

is most common, whereas recreational d rug use is

often suspected i n adolescents. In add ition to ataxia, patients also have s l u rred s peech and menta l status change. Alcohol, benzodiazepines, antiepileptics, a n d d r u g s o f a b u s e a l l m a y c a u s e these sym ptoms.

i QUICK QUIZ Which o f the following antiepileptic medications i s NOT associated with drug-induced tremors? A) Phenytoin. B) Carbamazepine. C) Sodium valproate. D) Levetiracetam. Discussion The correct answer is "D:' Phenytoin, carbamazepine, and sodium valproate can all cause tremors.

A 7-year-old boy presents to your office for evaluation of worsening gait instability and incoordination. His symptoms started shortly after he started walking as an infant and have been progressive. He is experiencing increasing difficulty with walking, and he sometimes trips and falls. His hands are shaky. He has had frequent sinus infections and pneumonias. Physical exam is remarkable for dysarthria, dysmetria on fin­ ger to nose, truncal ataxia, and ataxic gait. Dilated blood ves­ sels are noted on the conjunctiva, auricles, and nasal bridge. (See Figure 23-3.) Question 1 8-1

The most likely diagnosis in this patient is: A) Friedreich ataxia. B) Ataxia telangiectasia. C) Neuroblastoma.

F I G U R E 2 3 - 3 . Conj u nctival



N E U ROLOG I C D I SO R D E R S

telangiectasias

in

a

patient

with

525

ataxia­

telangiectasia. (Reproduced with permission from Goldsmith LA, Katz 51, Gilchrest BA, Pal ler AS, Leffell DJ, Wolff K, eds. Fitzpatrick's Dermatology in General Medicine. 8th ed. New York, NY: McGraw-Hill Education; 201 2, Fig. 1 43-8 .)

D) Guillain-Barre syndrome. E) None of the above. Discussion 1 8-1 The correct answer is "B:' Ataxia telangiectasia is characterized by progressive childhood onset ataxia, oculocutaneous telangi­ ectasias, sinopulmonary infections, hematologic malignancies (leukemia and lymphoma), and radiosensitivity. Telangiectasias may not be present initially. Patients are immunodeficient and have recurrent sinopulmonary infections requiring aggressive treatment. It is an autosomal recessive disorder, and the caus­ ative gene is the ATM (ataxia telangiectasia mutated) gene, which is involved in DNA damage response and cell cycle regu­ lation. Alpha-fetoprotein is elevated in the majority of patients. Friedreich ataxia is also an autosomal recessive disorder present­ ing with gait ataxia. Symptoms typically begin around puberty (later than ataxia telangiectasia) . Loss of lower extremity deep tendon reflexes is typical. Other clinical features include diabe­ tes and cardiomyopathy. Opsoclonus-myoclonus syndrome is a rare immune-mediated condition, characterized by irritability, ataxia, opsoclonus, myoclonus, and developmental regression. Opsoclonus consists of chaotic, often conjugate, rapid irregular eye movement in all directions. Myoclonus involves uncoordi­ nated jerking of the limbs and trunk. Neuroblastoma is discov­ ered in about half of cases of opsoclonus-myoclonus syndrome. The course of symptoms for ataxia telangiectasia evolves over days (not years) in Guillain-Barre syndrome.



Helpful Tip

•::'Jl C h i l d ren

with ataxia telang iectasia a re extremely

r1 1r sen sitive t o rad iation. I ma g i n g with ionizing rad iation

(X-rays, CT sca ns) s h o u l d not be ordered casual ly, g iven their a l ready increased risk of m a l i g n a ncy.

� QUICK QUIZ Dopamine antagonists such a s antipsychotics and some anti­ emetics can cause acute dystonic reactions (gaze deviation, torticollis, twisting of the trunk) .

526

MCG RAW-H I LL E D U CATION S P E C I A LTY BOA R D REVI EW: P E D I ATRICS

Choose a drug that can reverse acute dystonia caused by anti­ dopaminergic agents. A) Dantrolene. B) Bromocriptine. C) Diphenhydramine. D) Metoclopramide. E) All of the above. Discussion The correct answer is "C:' Diphenhydramine is effective in treat­ ing acute drug-induced dystonia. Benztropine is also effective. Dantrolene and bromocriptine are used for treatment of neuro­ leptic malignant syndrome, which is also caused by dopamine antagonists. Metoclopramide (Reglan) is an antiemetic and a dopamine antagonist; therefore, it can cause dystonia and neu­ roleptic malignant syndrome.

A 1 0-year-old girl is brought to the clinic for evaluation of behavioral changes and abnormal movements. A few weeks ago, she became anxious and emotionally labile, crying for no apparent reason. She has become fidgety. She recently started to have random writhing movements of her arms and legs, and she has not been able to sit still during class. Her hand­ writing is affected. A few months ago, she had an acute febrile illness with headache and sore throat. The symptoms went away without treatment. On exam, she has good strength and no focal neurologic signs, but she has difficulty maintaining hand grip {repeatedly squeezing and releasing the examiner's fingers) . She has facial tics and unpredictable, purposeless flowing movements of her arms and legs.

the risk of recurrent rheumatic fever. Although most patients with Sydenham chorea do not require symptomatic treatment with medications, when the impairment is considerable, anti­ epileptic medications such as valproate or carbamazepine can be utilized. In such cases, you will need to confirm the diagnosis before discussing symptomatic treatment.

� QUICK QUIZ I n what percentage o f children with Sydenham chorea is serology negative for GABHS? A) 1 %. B) 5%. C) 25%. D) 60%. Discussion The correct answer is "C:' Most children with Sydenham cho­ rea have positive serology (ASO and anti-DNase B antibodies), but more than 25% have negative serology. Negative serology does not rule out Sydenham chorea (acute rheumatic fever) . The anti-DNase B titer remains elevated longer compared with the ASO titer, and may be more helpful as Sydenham chorea devel­ ops several weeks to months after a GABHS infection.

A 7-year-old boy is being evaluated for facial movements. While you are speaking with the father, you hear the boy repeatedly sniffing.

What would you do next? A) Evaluate her for ADHD. B) Refer her to a psychiatrist. C) Obtain diagnostic studies, including antistreptolysin 0 (ASO) , and anti-DNase B antibodies, as well as throat culture. D) Start her on valproate.

Question 20-1 Which of the following is the most likely cause of the boy's symptoms? A) Tourette syndrome B) Habit cough C) Tardive dyskinesia D) Chorea

Discussion 1 9-1 The correct answer is "C:' Sydenham chorea is the most com­ mon cause of chorea in childhood. It occurs several weeks to several months after an untreated group A beta-hemolytic streptococcus (GABHS or Streptococcus pyogenes) or "strep throat" infection. Sydenham chorea is one of the major mani­ festations of acute rheumatic fever, along with migratory arthri­ tis, carditis, erythema marginatum, and subcutaneous nodules. Difficulty with hand grip ("milkmaid's grip") is due to motor impersistence (inability to maintain a posture). Most children with Sydenham chorea have positive serology, although this is not necessary as Sydenham chorea alone is enough to diagnosis acute rheumatic fever. Children should be evaluated for con­ current carditis. Long-term prophylactic antibiotic treatment for the prevention of GABHS infections is required to reduce

Discussion 20-1 The Best answer is ''!\.' Tourette syndrome is characterized by involuntary motor and vocal tics that have their onset in child­ hood. The cause is unknown. It is frequently associated with obsessive compulsive disorder (OCD), ADHD, or learning problems. Children can often suppress their tics but then "tic out" in private. Children feel an urge before and sense of relief after a tic. Examples of motor ticks include blinking, grimacing, shrugging, kicking, jumping, and obscene gestures. Examples of vocal tics include swearing/obscenities (coprolalia) , repeat­ ing words (echolalia), sniffing, barking, and throat clearing. The school should be educated that the tics are not intentional or bad behavior. Medications such as risperidone (antidopaminer­ gic), clonidine, or guanfacine may be prescribed if the tics are severe and interfering with school, work, or social interactions.

Question 1 9-1

C H A PT E R 23

An 8-month-old girl with myelomeningocele is seen in the emergency department for changes in behavior. She has a ventriculoperitoneal (VP) shunt, and her neurogenic blad­ der is managed by intermittent clean catheterization. She has been fussy for the past few days, has not been eating well, and has vomited a few times. She has been afebrile. She awakes during the exam, but drifts back to sleep. Her anterior fon­ tanelle is bulging. She has limited upward gaze. Her blood pressure is moderately elevated compared with her previous normal measurements. Question 21 -1

What would likely yield the correct diagnosis? A) Urinalysis and urine culture. B) Head CT scan. C) Urine toxicology screen. D) Tap the VP shunt reservoir and send CSF for analysis, including Gram stain and culture. Discussion 21 -1 The correct answer is "B:' Between 60% and 90% of patients with myelomeningocele have hydrocephalus, and it occurs more frequently with higher lesions. VP shunts are often used for treatment of hydrocephalus. A VP shunt can be complicated by shunt failure and infection. In cases of shunt failure, patients present with signs and symptoms of increased intracranial pres­ sure (ICP), such as headache, irritability, lethargy, vomiting, and eye movement abnormality. They may have hypertension with reflex irregular bradycardia. In infants, increasing head circum­ ference, bulging anterior fontanelle, split sutures, and "setting sun" sign (limited upward gaze) may be noted. Some patients with shunt failure present with more subtle symptoms, such as "not acting normal" or moodiness. It is important to consider the possibility of a shunt malfunction when evaluating a patient with a VP shunt presenting with nonspecific complaints. The diagnostic study of choice is a noncontrast head CT scan, which often shows increased ventricle size. Single-shot T2-weighted MRI is becoming the initial imaging study of choice at some institutions. It provides the same information as a head CT scan and avoids radiation. Sedation is usually not required, as the test can be performed quickly. Be aware that programmable shunts may require resetting as the valve-pressure may be changed after exposure to an MRI magnetic field. Neurosurgical consul­ tation is required whenever a shunt malfunction or infection is suspected. Be prepared to hear "It's not the shunt;' to which you will reply, "It's the shunt:' Urinary tract infections are com­ mon in children with bladder dysfunction. However, the patient described has clear signs and symptoms of increased I CP; there­ fore, urinary tract infection is not at the top of the differential. A patients with toxic ingestion can present with acute mental status change, and it is a reasonable differential diagnosis. How­ ever, given signs of increased ICP in a patient with VP shunt, shunt malfunction is more likely. Although shunt infection is possible, imaging is done first before collecting a CSF sample.



N E U ROLOG I C D I SO R D E R S

527

Shunt infections most commonly occur within the first few months of placement. The most common organisms are coag­ ulase-negative staphylococci followed by Staphylococcus aureus. Patients with shunt infections may be asymptomatic or present with signs of increased ICP, peritonitis (VP shunts) , septicemia (ventriculoatrial [VA] shunt) , or cellulitis (distal external por­ tion). Treatment includes IV antibiotics, removal of the shunt, and placement of an external ventricular drain.

� QUICK QUIZ What i s NOT a risk factor for hydrocephalus? A) Intraventricular hemorrhage. B) Bacterial meningitis. C) Posterior fossa tumor. D) Chiari malformation. E) All of the above. Discussion The correct answer is "E:' Obstructive hydrocephalus develops whenever the normal flow of CSF is blocked and fluid accumu­ lates within the ventricles or subarachnoid space. It presents with signs and symptoms of increased ICP. CSF is primarily produced by the choroid plexus in the ventricles. CSF flows through the ventricular system (lateral � third � fourth ven­ tricle) into the subarachnoid space, where it is absorbed by the arachnoid granulations into the venous circulation. Remember, CSF flows from the lateral ventricles through the foramen of Monroe into the third ventricle; through the cerebral aqueduct to the fourth ventricle; and through the foramina of Luschka and Magendie into the subarachnoid space. Risk factors for developing hydrocephalus include CNS malformations, CNS infection, intraventricular hemorrhage, trauma, and tumors.





1 1 1r

Helpful Tip

Folic acid supplementation d u ri n g preg na ncy can prevent many cases of neura l tube defects. It is recommended that all women of c h i l d bearing age ta ke 400 to 800 m i crog ra m s (meg) of fo l i c acid per day. If the wo m a n h a s a neural tu be defect o r has g iven b i rth to a c h i l d with neural tu be defect, the reco m m ended dose of fo l i c acid is 4 g (4000 meg) per d ay.

� QUICK QUIZ Patients with myelomeningocele are at risk for which o f the following? A) Constipation. B) Urinary tract infection. C) Vesicoureteral reflex. D) Renal failure. E) All of the above.

MCG RAW-H I LL E D U CATION S P E C I A LTY BOA R D REVI EW: P E D I ATRICS

528

Discussion The correct answer is "E:' Spinal cord pathology can lead to bowel and bladder dysfunction. Clean intermittent catheteriza­ tion is done to reduce the risk of urinary tract infection and bladder overdistention, which can lead to vesicoureteral reflex, hydronephrosis, and chronic kidney disease. In some cases, a vesicostomy is placed. Ultrasonography and urodynamics are often utilized to assess the morphology and function. Constipa­ tion and fecal incontinence are often managed medically (timed toileting, fiber, stool softener, enemas) .



Helpful Tip

�A

=-

r1 1r a n

C h i a r i type I I A r n o l d-C h i a ri

d i s placement

of

m a lformation (a l s o known a s m a lformation) the

l ower

is

d ownwa rd

cerebel l u m

and

cerebe l l a r to n s i l s t h ro u g h t h e fo ra m e n m a g n u m, with a concu rrent mye l o m e n i n g ocele. Sym pto m s of C h i a r i m a lformation res u l t fro m com p res s i o n of t h e b ra i n stem a n d c ra n i a l n e rves a n d i n c l u d e dys p h a g ia , a s p i ration, b reath i n g p ro b l e m (hoa rse voice, strider, a p n ea), a n d a r m wea kness. A C h i a ri I m a lformation i s d ownwa rd d i s p l a c e m e n t of t h e cerebe l l a r to n s i l s o n l y, without a concu rrent mye l o m e n i ngocele. A

A pregnant woman's alpha-fetoprotein, checked as part of screening for neural tube defects, is elevated at 20 weeks' gestation. A prenatal ultrasound reveals signs of a neural tube defect. She and the father-to-be want more informa­ tion, and they are referred to the neurology clinic for a prenatal consultation visit. You discuss the various types of neural tube defects; anticipated care needs for the newborn, including surgery in the postoperative period; long-term complications; and need for long-term multi­ specialty care. Question 22-1

When discussing neural tube defects, what do you tell them? A) Neural tube defects result from failure of the neural tube to close between the third and fourth week of gestation. B) Spina bifida occulta may have cutaneous markers. C) In a meningocele the meninges herniate through a midline defect in the lumbosacral spine. D) A sacral dimple or cutaneous marking may indicate an underlying tethered cord. E) All of the above.

syri nx (fl u i d-fi l l ed cavity with i n t h e s p i n a l cord) ca n be a ssociated with a C h i a ri I ma lfo r m a t i o n . (See Fig u re 2 3 -4.)

Discussion 22-1 The correct answer is "E" Spinal dysraphism is a collective term for malformations of the spinal cord. Neural tube defects are included under the term spinal dysraphism. Myelomeningocele, also known as spina bifida, is the most common neural tube defect. The proximal or distal portion of the neural tube may be affected. The various types of neural tube defects are sum­ marized in Table 23-7.

TA B L E 23-7 TYP E S O F N E U RA L TUBE D E F ECTS

Open s p i n a l defects: •



M e n i ngocele: hern iation of the m e n i nges t h ro u g h a l u m bosacra l defect Myelomen i ng ocele: hern iation of the m e n i nges a n d s p i n a l cord t h ro u g h a l u m bosacra l defect

Closed s p i n a l defects (spina bifida occu lta ) : •





Tethered cord: attachment of the cord to d i sta l structu res Dermal sinus tract: com m u n ication between the skin a n d the s p i n a l co rd C a u d a l reg ression: i n co m p l ete fo rmation of the sacrum

Cra n i a l defects: •

F I G U R E 23-4. Sag itta l M R I showing downward d i s p l acement of the cerebe l l a r to nsils and a n associated syrinx i n this patient with a Chiari ma lformation. (Reproduced with permission from Doherty GM, ed. Current Diagnosis & Treatment: Surgery. 1 4th ed. New York, NY: McGraw- H i l l Education; 201 5, Fig. 36-1 8.)



Encephaloce le: herniation of the m e n i nges a n d cerebra l co rtex, cerebe l l u m, or part o f the bra i n stem t h ro u g h a m i d l i ne s ku l l defect Anencepha ly: fa i l u re of the part of the bra i n , bra i n ­ stem, a n d s ku l l t o develop

C H A PT E R 23



Helpful Tip

:5.� Encepha loceles

may i nvolve the a nterior portion

r1 1r of the skull, man ifesting with nasal bridge swe l l i ng, hypertelorism, proptosis, nasal mass, or CSF rhi norrhea.

Add it to you r differential d iag nosis of nasal obstruction.

J% Q U I C K Q U I Z Which i s NOT an overlying cutaneous marker o f a closed spi­ nal neural tube defect such as a tethered cord? A) Sacral dimple. B) Hair tuft. C) Lipoma. D) Hemangioma. E) None of the above. Discussion The correct answer is "E:' Trick question! Options "!\' through "D" are all correct. Cutaneous markers over the sacrum or coc­ cyx may indicate underlying issues. Order an ultrasound (new­ born) or an MRI of the spine.

J% Q U I C K Q U I Z



N E U ROLOG I C D I SO R D E R S

529

upgoing bilaterally. Increased tone is noted at the ankles. He has mild bilateral toe walking. A brain MRI shows increased hyperintensity on T2 images at the frontal and occipital horns of lateral ventricle. Question 23-1

Which condition would best explain his presentation? A) Neonatal hyperbilirubinemia. B) Cerebral palsy. C) Perinatal stroke. D) Congenital cytomegalovirus infection. E) None of the above. Discussion 23-1 The correct answer is "B:' The physical exam finding of lower extremity hyperreflexia, spasticity, ankle clonus, and extensor toe signs (upper motor neuron signs), along with the history of prematurity and the described brain MRI finding (periven­ tricular leukomalacia or white matter injury of prematurity) all point toward the diagnosis of spastic diplegia, one type of cere­ bral palsy. In addition to spastic diplegia, cognitive deficits and behavioral!attentional deficits are some of the clinical correlates of periventricular leukomalacia. (See Figure 23-5.) Kernicterus is caused by brain damage due to high level of bilirubin in new­ born period. Patients develop hearing loss, dystonic cerebral palsy, and abnormal eye movements (limited upward gaze) . Perinatal stroke is most common in the left middle cerebral

True o r false: Most patients with an epidural hematoma have a typical history of initial loss of consciousness, followed by a lucid interval, then clinical deterioration. A) True. B) False. Discussion The correct answer is "B:' In contrast to classic teaching, only about 20% of patients with epidural hematoma experience loss of consciousness. They typically present with mental status changes or focal neurologic findings, or may have more subtle symptoms such as headache and emesis.

A 4-year-old boy is brought in for evaluation of behavior issues. He was born at 27 weeks' gestational age. He was mildly delayed in achieving early developmental mile­ stones even corrected for his prematurity, but he is mostly "caught up" per the mother. However, his preschool teacher has been expressing concerns about his behaviors. He can­ not sit still during circle time, and he has difficulty follow­ ing directions. The child is hyperactive and is exploring the exam room while you talk to his mother. He needs to be redirected multiple times during the exam. You note hyper­ reflexia at the knees and Achilles in bilateral lower extremi­ ties. He has several beats of ankle clonus bilaterally. Toes are

F I G U R E 23-5. Axial bra i n M R I showing periventric u l a r leukomalacia. (Reproduced with permission from Maitin I B, Cruz B, eds. Current Diagnosis and Treatment: Physical Medicine and Rehabilitation. New York, NY: McGraw­ H i l l Education; 20 1 5, Fig. 20-3.)

530

MCG RAW-H I LL E D U CATION S P E C I A LTY BOA R D REVI EW: P E D I ATRICS

A

B

F I G U R E 23-6. U ltraso u n d i mages of an i nfa nt with periventric u l a r calcifications due to congenital cytomegalovirus i nfection. (Reproduced with permission from C u n n i n g h a m FG, Levene KJ, Bloom S L, et al, eds. Williams Obstetrics. 24th ed. New York, NY: McGraw- H i l l Education; 201 4, Fig. 64-3A&B.)

artery distribution, and usually causes focal deficits or seizures. Congenital cytomegalovirus infection is the most common congenital infection in the United States, with prevalence of approximately 1 % . Between 10% and 1 5 % of affected patients develop symptoms (intrauterine growth restriction, microceph­ aly, hepatosplenomegaly, j aundice, thrombocytopenia, and reti­ nitis) . Brain imaging shows periventricular calcifications. (See Figure 23-6.)

� QUICK QUIZ Which i s NOT a cause o f cerebral palsy? A) Stroke. B) Prematurity. C) Traumatic brain injury. D) Hypoglycemia. E) Intracranial hemorrhage. Discussion The correct answer is "D:' Cerebral palsy is a disorder of movement, muscle tone, or posture that is caused by a non­ progressive disturbance that occurred in the fetal or infant brain. Etiology includes perinatal brain injury (hypoxic isch­ emic encephalopathy [HIE] , stroke), brain injury related to prematurity (periventricular leukomalacia, intraventricular hemorrhage) , developmental abnormality (brain malfor­ mation, genetic or metabolic abnormality) , postnatal brain injury (kernicterus, CNS infection, traumatic brain injury) , and prenatal risk factors (congenital TORCH infections, toxin exposure) . Obstetric complications are commonly blamed as a cause of cerebral palsy, but in fact perinatal asphyxia only accounts for 6% to 28% of such cases. Cerebral palsy is classi­ fied by the predominant motor abnormality and affected body area. Spastic hemiplegia (classically due to perinatal stroke), spastic quadriplegia (classically due to HIE) , spastic diplegia (prematurity) , and dystonic cerebral palsy (kernicterus, HIE) are some of the examples.

An 8-year-old previously healthy girl presents to the emergency department after a focal seizure with secondary generalization. She is afebrile. She returns to her baseline after brief observa­ tion. Her skin exam is normal. A head CT scan shows a contrast­ enhancing mass without surrounding edema or mass effect. Question 24-1 What is the most likely cause of the lesion seen on head imaging? A) Glioblastoma multiforme. B) Arteriovenous malformation. C) Brain abscess. D) Sturge Weber syndrome. Discussion 24-1 The correct answer is "B:' Arteriovenous malformations (AVMs) are collections of abnormal blood vessels that connect arteries to veins without the intervening capillaries. The imaging shows a typical contrast -enhancing tangled clump of vessels. AVMs can hemorrhage, with clinical presentation of sudden headache, altered mental status, and possible focal neurologic deficits. (See Figure 23-7.) Glioblastoma multiforme is an aggressive tumor, and has a heterogeneous appearance, with surrounding edema and mass effect. A brain abscess has a different appearance at different stages of its evolution, but typically has ring enhance­ ment. (See Figures 23- 1 and 23-2, earlier.) A tram-track sign is a reflection of cortical calcifications due to leptomeningeal vascular malformations in Sturge-Weber syndrome. It often is seen in the posterior lobe and has characteristic gyriform parallel lines.

A 5-year-old African American girl with known sickle cell disease presents to the emergency department with drooling and left-sided weakness since yesterday. Yesterday was July

C H A PT E R 23



N E U ROLOG I C D I SO R D E R S

531

TA B L E 2 3 - 8 CAU S E S O F P E D I AT R I C STROKE

Ischemic

Sickle ce l l d i sease Co ngen ital heart defects (ca rd iogenic t h ro m bi, rig ht-to­ left s h u nt) Clotti ng d i sord e rs (anti phos p h o l i pid anti bodi es, factor V Leiden d eficiency, prote i n C o r S deficiency, prot h rom b i n m utation) Hype rhomocyst i n e m i a Arte riopathy (a rterial d i ssection, Moya m oya d i sease, I nfecti o n s) Hemorrhagic

Arte riove n o u s ma lformation

23-7. Axial brain MRI demonstrating a large arteriovenous malformation. (Reproduced with permission from Waxman SG, ed. Clinical Neuroanatomy. 27th ed. New York, NY: McGraw-Hill Ed ucation; 20 1 3, Fig. 1 2-23.) FIGURE

4th, and the patient was having fun blowing and spinning a pinwheel. Her parents noticed that she was not moving the left side of the body well last night. She was drooling as well, which is unusual for her. They decided to bring her in today as her condition is not improving. She has not had fever, cough, diarrhea, or headache. The patient does not have a history of seizures, and her parents have not noticed any seizure-like episodes. There is no family history of seizures, migraines, or early stroke or myocardial infarction. On exam, the patient has left-sided lower facial weakness, as well as decreased strength of the left arm and leg. The sensory exam is diffi­ cult to interpret given her age. Deep tendon reflexes are exaggerated on the left arm and leg. A magnetic resonance angiogram (MRA) of the brain shows bilateral stenosis of the intracranial internal carotid arteries with collateral vessels. Question 25-1 What is the most likely cause of her symptoms? A) Hemiplegic migraine. B) Todd paralysis. C) Brain hemorrhage. D) Ischemic stroke. E) None of the above. Discussion 25-1 The correct answer is "D:' The patient likely suffered an isch­ emic stroke with underlying vascular pathology in the setting of hyperventilation (blowing on a pinwheel), leading to cerebral vasoconstriction. Moyamoya disease is characterized by progres­ sive stenosis of the intracranial internal carotid arteries with for­ mation of collateral vessels. Moyamoya means "puff of smoke" in Japanese, describing the appearance of the fine networks of

the collateral blood vessels. When the condition occurs with a well-known associated disorder, such as sickle cell disease, neu­ rofibromatosis l, and trisomy 2 1 , it is categorized as Moyamoya syndrome. Typically, both internal arteries are affected, but some cases have unilateral involvement only. The child described in this case had ischemic lesions at the right middle cerebral artery territory on brain MRI. Hemiplegic migraine presents with usu­ ally reversible weakness often associated with headache. Todd paralysis is one of the differential diagnoses for new-onset hemi­ plegia, but the lack of convulsion in this case makes Todd paraly­ sis unlikely. Brain hemorrhage can lead to hemorrhagic stroke, but patients typically also have headache. Furthermore, the vast majority of vascular complications seen in Moyamoya disease in children are ischemic. (See Table 23-8.)



Helpful Tip

� Sickle cel l d i sease is the most common cause of stroke r1 1r in c h i l d re n . Other etiologies i n c l u d e congenita l heart d i sease, CN S

i nfection, tra u ma, vasc u l a r lesions,

thromboph i l ia, a n d g enetic conditions.

� QUICK QUIZ Which i s NOT a presenting sign o f stroke i n pediatric patients? A) Seizure. B) Hemiparesis. C) Altered mental status. D) Tremor. E) Aphasia. Discussion The correct answer is "D:' Presentation varies by age. Infants typically have seizures and altered mental status while older children present with hemiparesis, headache, and focal deficits (aphasia, vision changes) .

MCG RAW-H I LL E D U CATION S P E C I A LTY BOA R D REVI EW: P E D I ATRICS

532

TA B L E 23-9 D I F F E R E N T I A L D I AG N O S I S OF

G U I LLA I N - B A R R E SYN D RO M E

A 1 0-year-old girl is admitted to the general pediatrics floor for progressive weakness. She started to have weakness in her legs several days ago, and the weakness has progressed to involve her arms and face. She has not been able to walk since yesterday. She denies back pain and bladder or bowel dysfunction. Until her leg weakness started, she had been healthy except for a diarrheal illness 3 weeks ago. Deep ten­ don reflexes are not present in the legs and are diminished in the arms. CSF and electrophysiologic study results are con­ sistent with Guillain-Barre syndrome.

Presentations a n d D i sti n g u i s h i n g Featu res Tic k paralys i s

Presents with acute ataxia, asce n d i n g flaccid paralys i s, or both. Remova l of tick i s the treatment.

Bot u l i s m

Desce n d i n g (not asce n d i n g ) wea kness, p u p i l l a ry a b n o r­ m a l ities, a n d b u l ba r sym ptoms. I nfa nt botu l i s m i s caused by colon ization of the i m mature bowe l by botu l i n u m spores a n d occu rs i n patients yo u n g e r t h a n 1 yea r o f a g e . I n older patie nts, botu l i s m is cau sed by botu l i n u m tox i n i n food or wo u n d s.

Myasthenia g ravis

Fatigable wea kness. Ptosis, d i plopia.

Vi ra l myositis

M u scle pain, tenderness, a n d wea kness after a v i r a l i n fection. El evated creat i n e ki nase.

Spinal cord pathology (tra ns­ verse mye l itis, s p i n a l co rd co m p ression)

Back pain, bowe l or bladder dysfu nction, sen sory level

Question 26-1

Regarding the management of this patient, which statement is NOT correct? A) Her forced vital capacity (FVC) should be monitored. B) Her blood pressure should be monitored closely. C) IVIG or plasma exchange should be considered. D) Oral steroid should be given. Discussion 26-1 The correct answer is "D:' The pathophysiology of Guillain-Barre syndrome is thought to be an autoimmune phenomenon in the setting of recent infection or immunization. Various pathogens have been implicated, including cytomegalovirus, Epstein-Barr virus, varicella zoster virus, Campylobacter jejuni (this child's probable diarrheal illness), Mycoplasma pneumoniae, and Hae­ mophilus injluenzae. Patients present with ascending weakness, usually occurring 1 to 4 weeks after the infection or immuniza­ tion. Weakness can involve respiratory muscles leading to respi­ ratory compromise, and this can occur rapidly. Therefore, FVC needs to be monitored intermittently to identify patients at risk. Patients whose FVC is 20 mL/kg or less, as well as those with rapidly progressing weakness, bulbar palsy, quadriparesis, and cardiovascular autonomic dysfunction need to be monitored in the PICU. More than 10% of patients require intubation and mechanical ventilation during the course of the illness. Auto­ nomic dysfunction such as cardiac dysrhythmias, hypotension, hypertension, paralytic ileus, and bladder dysfunction are com­ mon, occurring in about 50% of patients at the peak of the illness. Cranial nerve involvement can occur in up to 50% of patients, with involvement of the facial nerve being the most common. IVIG or plasma exchange hastens recovery. Steroids, either intra­ venous or oral, are not indicated. Oral corticosteroids may actu­ ally hinder improvement based on adult studies.



A 1 5-year-old boy presents to your office with a 1-day history of unilateral facial weakness. On exam, he is unable to fully close his right eye. He also cannot raise his right eyebrow. The nasolabial fold on the right side is less prominent compared with the left side. His extraocular movement is full bilater­ ally, and he has normal pupillary responses. Strength in the arms and legs is full bilaterally. Question 27-1 Where do you localize the lesion? A) Motor cortex. B) Facial nerve. C) Neuromuscular junction. D) Facial muscles.

Helpful Tip

:5.� CSF

ana lysis i n G u i l l a i n-Ba rre syn d rome typica l l y

r1 1 r shows a n elevated prote i n l evel without pleocytosis

(a l b u m i nocytologic d issociation). CSF resu lts may not

match the expected pattern, especia l ly when the CSF is obta i ned early i n the cou rse of i l l ness. CSF protein beg i n s to rise towa rd the end of the fi rst week of i l l ness, and pea ks i n 4 to 6 weeks. Ta ble 23-9 s u m m a rizes the d ifferential diag nosis i n G u i l l a i n-Ba rre syn d rome.

Discussion 27-1 The correct answer is "B:' Due to hi-hemispheric innervation of facial nerve nuclei, a lesion above the facial nerve nucleus (as in stroke) produces facial weakness below the forehead. In the case of facial nerve palsy, the entire face is weak on the affected side, including the forehead. It is therefore very important to assess the presence or absence of forehead weakness when evaluating a patient with facial weakness. (See Figures 23-8 and 23-9.)

C H A PT E R 23



N E U ROLOG I C D I SO R D E R S

533

years ago as she was no longer able to keep up with her peers. When she walks, her feet drop, forcing her to lift her legs higher to avoid stumbling. Her father has a similar condition and was diagnosed with hereditary motor and sensory neu­ ropathy (Charcot-Marie- Tooth disease) . Question 28-1 Which finding do you NOT expect on her physical exam? A) High-arched feet. B) Decreased position and vibration sense in feet. C) Calf hypertrophy. D) Difficulty with heel walking. E) Absent or depressed deep tendon reflex at Achilles. Discussion 28-1 The correct answer is "C' Charcot-Marie-Tooth ( CMT) disease is a demyelinating disorder of the peripheral nerves. Patients pres­ ent with foot weakness and often have foot drop, leading to step­ page gait (raising legs higher to compensate for the foot drop). Affected children are unable to walk on their heels, owing to weakness of foot dorsiflexion. High-arched feet (pes cavus) are common. Hammer toe deformity develops later. A characteristic inverted "champagne bottle" or "stork leg" appearance of legs is caused by atrophy of the muscles. There is length -dependent posi­ tion and vibration sense loss beginning in the feet. Hand muscles may become weaker later in the course of the disease, leading to difficulty with fine motor skills. Option "C:' calf hypertrophy, is associated with Duchenne muscular dystrophy.

F I G U R E 23-8. Left fac i a l nerve i nj u ry in a 2-day-old i nfa nt. (Reproduced with

permission from C u n n i n g h a m FG, Levene KJ, Bloom SL, et al, eds. Williams Obstetrics. 24th ed. New Yo rk, NY: McGraw- H i l l Education; 201 4, Fig. 33-3.)

When you diagnose facial nerve palsy or Bell palsy in Lyme­ endemic regions, testing for Lyme disease is indicated if season is right, as Lyme disease can cause facial nerve palsy (unilateral or bilateral) . Idiopathic facial nerve palsy or Bell palsy generally has a good prognosis with minimal residual deficits. However, eye care needs to be discussed to prevent corneal injury. Eye ointment, eye drops, and occasionally an eye patch are recom­ mended. Oral steroids are often prescribed.

You are evaluating a term newborn boy just after delivery. The pregnancy was complicated by gestational diabetes and excessive maternal weight gain. The newborn was delivered

A 1 3-year-old girl presents with an abnormal gait. She has always been a bit clumsy, often twisting her ankles, and she never learned how to ride a scooter. She quit basketball a few Left

Right Motor cortex



Sweet •

Salt

V (SA) Facial

(V I I )

V I I (VA) Sour

nerve

Bitte r

IX (SA)

IX (VA) Epiglottis

A

8

F I G U R E 23-9. Facial nerve (CN VII). (A) Central and peripheral motor i n nervation of the face. The forehead receives motor projections from both hemispheres and the l ower face (eyes and bel ow) from the contra latera l hemis phere on ly. (B) Somatic afferent (SA, touch) and viscera l afferent (VA, taste) i n nervation of the tong ue. (Reproduced with permission from Ami noff MJ, Green berg DA, & Simon RP (Eds). Clinical Neurology, 9th ed. McGraw-H i l l Education, I nc., 201 5. Figure 1 - 1 7)

534

MCG RAW-H I LL E D U CATION S P E C I A LTY BOA R D REVI EW: P E D I ATRICS

vaginally with vacuum assistance. His birth weight is 3900 g. On exam, he has decreased movement of his right arm. He is holding the right arm internally rotated with elbow extension and forearm pronation. Tone in the right arm is decreased. Tone in the rest of the extremities is normal. He is moving his left arm and both legs well. You also note that the right eyelid is droopy and the left pupil is larger than the right. The differ­ ence in pupil size is more apparent in the dark. Question 29-1 Are the eye findings related to the arm weakness? A) Yes, the eye findings are related to the arm weakness. B) No, the eye findings are not related to the arm weakness. C) Unsure. (You haven't a clue but you have a 50-50 shot of guessing correctly.) Discussion 29-1 The correct answer is "A:' The patient has Erb palsy, which is a type of brachial plexus injury. The brachial plexus is formed by nerve roots arising from C5 through T l . Superior trunk or C5 and C6 nerve roots are involved in Erb palsy, which is the most common type of obstetric brachial plexus injury. When the oculosympathetic pathway, which travels through the spi­ nal cord, is also damaged, it causes ipsilateral Horner syndrome (ptosis, miosis, and anhidrosis). (See Figure 23- 1 0.) A physi­ cal therapist, neurologist, orthopedic surgeon, or neurosurgeon may be involved in the care. Most patients with Erb palsy have a good prognosis. However, if no or minimal improvement is noted during the first 3 months or so, surgical intervention may be considered.

An 1 1 -year-old girl has experienced ptosis and diplopia for the past 3 weeks. Her symptoms worsen during the course of the day, and ptosis worsens with sustained upward gaze. The acetylcholine receptor antibody is positive, and you make a diagnosis of myasthenia gravis. Question 30-1 What is the pathophysiologic mechanism of myasthenia gravis? A) Acetylcholine release into the neuromuscular junction is blocked. B) Autoantibodies bind to acetylcholine receptors, leading to lysis of the acetylcholine receptors. C) Mutations occur in acetylcholine receptor subunits. D) Acetylcholinesterase activity is decreased. E) None of the above. Discussion 30-1 The correct answer is "B:' Myasthenia gravis is the most common disorder of neuromuscular transmission. It is an autoimmune process in which antibodies binding to acetylcholine receptors lead to lysis of the acetylcholine receptors. Most patients pres­ ent with ocular symptoms such as ptosis and diplopia (ocular

myasthenia gravis) . Other muscles can also be involved, and patients can progress to generalized myasthenia gravis. Bulbar weakness (dysarthria, dysphasia), masticatory weakness (jaw fatigue, jaw closure weakness), respiratory weakness, and axial! limb weakness need to be monitored. The edrophonium (Ten­ silon) test, electrophysiologic studies, and a positive acetylcho­ line receptor antibody test aid the diagnosis. Edrophonium is a short-acting acetylcholinesterase inhibitor and improves the symptoms transiently. The classic electrophysiologic finding is decremental response of the compound muscle action potential with repetitive stimulation of a motor nerve. Treatment involves pyridostigmine (Mestinon) , which is an anticholinesterase. Immunosuppressive agents such as prednisone, cyclosporine, azathioprine, and mycophenolate mofetil are also used. Plas­ mapheresis and IVIG are usually reserved for myasthenic crisis or acute worsening. The mechanism listed in option "!\' occurs in botulism, as the toxin blocks the calcium-dependent release of acetylcholine. Children with infantile botulism present with constipation, hypotonia, poor feeding, and weak cry. The paral­ ysis in botulism is descending, whereas that in Guillain-Barre syndrome is ascending. Congenital myasthenic syndromes are a group of disorders caused by genetic defects of the neuromuscu­ lar junction (presynaptic defects, synaptic defects, postsynaptic defects). Acetylcholine receptor subunit mutation (option "C'') is one of the examples of congenital myasthenia syndrome due to postsynaptic defect. Decreased acetylcholinesterase activity (option "D") is the goal of treatment, not the underlying patho­ physiology, of myasthenia.

A 4-year-old boy is brought for evaluation because of gait concerns. Gestational and birth histories were unremarkable, and he started walking at 18 months. However, recently he began having difficulty going up the stairs, and now he has to pull himself up holding onto the hand-rails. He is slightly behind his peers in terms of learning. He has not had any recent illnesses. There is no pertinent family history. On exam, you note hypotonia and enlarged calf muscles. When he runs, a waddling gait is apparent. When he stands up from the floor, he places his hands on his knees and then his thighs to stand up. Question 31 -1 What would you do next? A) Start physical therapy and see how he improves. Follow up in 6 months. B) Check serum creatine kinase (CK) . C) Order a muscle biopsy. D) Obtain a brain MRI. E) Order genetic testing. Discussion 31 -1 The correct answer is "B:' Muscular dystrophies are a group of progressive myopathies characterized by muscle weakness. The

C H A PT E R 23



N E U ROLOG I C D I SO R D E R S

535

A Hypothalamus Ophthalmic division of trigeminal nerve Long c i l iary nerve

To sweat glands of forehead

Fi rst n e u ro n

8 F I G U R E 23-1 0. Oculosym pathetic pathway affected i n Horner synd rome. I nj u ry to these pathways causes ptosis, miosis, and a n hyd rosis to the ipsilateral face. (Reproduced with permission from Ami noff MJ, G ree n berg DA, Simon RP, eds. Clinical Neurology. 9th ed. New York, NY: McGraw- H i l l Education; 201 5, Fig. 7-1 0.)

boy's presentation is typical of Duchenne muscular dystrophy (DMD). Patients may have mild developmental delays in the first few years of life. Delayed walking is often seen. Proximal leg weakness usually becomes apparent by 3 years of age. Typi­ cally the child has difficulty rising from the floor and negotiat­ ing stairs. Additional features such as waddling gait, Gower sign (walking the hands up the legs when standing up from

the floor) , calf pseudohypertrophy, and lumber lordosis also appear. Patients often become wheelchair bound in early teen­ age years. Kyphoscoliosis and cardiomyopathy are later prob­ lems. DMD is an X-linked recessive disorder, affecting about 1 in 3500 live male births. It is caused by a mutation in the dys­ trophin gene on chromosome Xp2 1 . Dystrophin is a critical protein in muscle. Serum CK is very elevated in patients with

MCG RAW-H I LL E D U CATION S P E C I A LTY BOA R D REVI EW: P E D I ATRICS

536

DMD due to muscle fiber degeneration. When a mother is a carrier, there is 50% chance of transmitting the dystrophin mutation in each pregnancy. Sons who inherit the abnor­ mal gene will be affected, and daughters who inherit the abnormal gene will be carriers. Therefore, it is important for the parents to be counseled about future pregnancies. Becker muscular dystrophy (BMD) is caused by the same mutation as DMD and is also inherited as an X-linked recessive disorder. Compared to DMD, BMD has a later age of onset and milder symptoms including being able to ambulate for longer. Car­ diac involvement is present, as in DMD.





Helpful Tip

C reatine kinase (CK) is a relatively i nexpensive test,

1 1 1r a n d you should consider checking C K when eva l uating

a boy with developmenta l delay, weakness, or gait abnormal ities to screen for DMD.



Helpful Tip

=.� Muscle

pathology causes proxi m a l m u scle weakness

r1 1r (strug g l e going down sta i rs). Nerve pathology cau ses d ista l muscle weakness (strug g l e going up sta i rs).

A 1 3-year-old boy is urgently referred to your clinic. He is always sleepy and has a hard time getting up in the morn­ ing. If his parents allowed, he would easily sleep all day. He still will take naps after school and on weekends. He some­ times yells out just as he is falling asleep. When his mother checks on him, he is fast asleep. Recently at school, his friend made a sarcastic comment about a teacher. The adolescent started laughing then abruptly fell to the floor. He felt that his legs "gave out:' He did not lose consciousness. Shortly after, he was able to stand up and was fine. This has happened two more times, prompting a referral. His exam is entirely normal. Question 32-1 Which is NOT a symptom of his condition? A) Daytime sleepiness. B) Insomnia. C) Hallucinations. D) Drop attacks. E) Sleep paralysis. Discussion 32-1 The correct answer is "B:' Narcolepsy is a chronic disorder. Excessive daytime sleepiness (sleeping for prolonged periods, difficulty waking up, daytime naps in older children) must be present and is typically the first sign. Hallucinations occur as the child is starting to wake (hypnopompic) or fall asleep (hyp­ nagogic) . Cataplexy (drop attacks) is an abrupt loss of bilateral

muscle tone, often triggered by strong emotion, which may cause the child to fall to the floor. The other cardinal feature of narcolepsy is sleep paralysis. Treatment includes good sleep hygiene, avoiding CNS depressants, taking power naps, and medication (modafinil) to treat daytime sleepiness. B I B LIOGRAPHY

Barkovich AJ, Raybaud C. Pediatric Neuroimaging. Philadelphia, PA: Lippincott Williams and Willkins; 2 0 1 2 . Duffner PK, Berman P H , Baumann RJ, e t al. Febrile seizures: Guideline for the neurodiagnostic evaluation of the child with a simple febrile seizure. Subcommittee on Febrile Seizures. Pediatrics. 20 1 1 ; 1 27(2) :389-394. doi: 1 0 . 1 542/ peds.20 10-33 1 8 . Fenichel GM. Th e hypotonic infant. In: Clinical Pediatric Neu­ rology: A Signs and Symptoms Approach. 6th ed. Philadel­ phia PA: Elsevier Saunders; 2009: 1 53- 1 76. Fenichel GM. Paroxysmal disorders. In: Clinical Pediatric Neu­ rology: A Signs and Symptoms Approach. 6th ed. Philadel­ phia PA: Elsevier Saunders; 2009: 1 -48. Fenichel GM. Psychomotor retardation and regression. In:

Clinical Pediatric Neurology: A Signs and Symptoms Approach. 6th ed. Philadelphia PA: Elsevier Saunders; 2009: 1 1 9- 1 52. Glaser C, Long SS. Encephalitis. In: Long, SS, Pickering LK, Prober CG, eds. Principles and Practice of Pedi­ atric Infectious Diseases. 4th ed. Philadelphia, PA: Elsevier; 2 0 1 2:297-3 14. https:/ /www - clinicalkey­ com.proxy.lib.uiowa.edu/#! /content/book/3-s2.0B978 1 43772702900044 1 . Accessed April 28, 20 1 5 . Glaser C, Strober JB. Para- and postinfectious neurologic syndromes. In: Long SS, Pickering LK, Prober CG, eds.

Principles and Practice of Pediatric Infectious Diseases. 4th ed. Philadelphia, PA: Elsevier; 2 0 1 2 : 3 14-3 1 9 . https:// - clinicalkey-com.proxy.lib.uiowa.edu/#!/content/ book/3 -s2.0-B978 1 43 7727029000453 . Accessed April 28, 20 1 5. Harrison CJ. Focal suppurative infections of the nervous system. In: Long SS, Pickering LK, Prober CG, eds. Prin­ ciples and Practice of Pediatric Infectious Diseases. 4th ed. Philadelphia, PA: Elsevier; 2 0 1 2 : 3 1 9-330. https:// www - clinicalkey-com.proxy.lib.uiowa.edu/#!/content/ book/3 -s2.0-B978 1 437727029000465. Accessed April 28, 20 1 5. Hershey AD. Headaches. In: Kliegman RM, Stanton BF, St Geme JW, Schor NF, Behrman RE, eds. Nelson Textbook of Pediatrics. 1 9th ed. Philadelphia, PA: Saunders; 20 1 1 :2039-2046. https:/ /www. clinicalkey.com/ # !/content/ book/3 -s2.0-B978 1 43770755700588 1 . Accessed January 24, 20 1 5. Hirtz D, Ashwal S, Berg A, et al Practice parameter: Evalu­ ating a first nonfebrile seizure in children: Report of the quality standards subcommittee of the American Academy of Neurology, the Child Neurology Society, and the American Epilepsy Society. Neurology. 2000;5 5 : 6 1 6-62 3 . I H S Classification: International Classification o f Headache Disorders (ICHD)-2. International Headache Society www

C H A PT E R 23

website. http:/ /ihs-classification.org/en/. Accessed January 24, 20 1 5. Lewis, DW. Headaches in infants and children. In: Swaiman K, Ashwal S, Ferriero DM, Schor NF, eds. Swaiman's Pediatric Neurology: Principles and Practice. 5th ed. Edinburgh, Scotland: Elsevier Saunders; 2 0 1 2 : 880-899. https:/ /www. clinicalkey.com/ #!I content/book! 3 -s2. O­ B978 1 437704358000639. Accessed January 24, 20 1 5. Liptak GS, Dosa NP: Myelomeningocele. Pediatr Rev. 20 1 0;3 1 ( 1 1 ) :443-450. Mann K, Jackson MA: Meningitis. Pediatr Rev. 2008;29 ( 1 2):41 7-430. Marcdante KJ, Kliegman RM. Altered mental status. In: Marcdante KJ, Kliegman RM, eds. Nelson Essentials of Pediatrics. 7th ed. Philadelphia, PA: Elsevier Saunders; 20 1 5:634-642. https:/ /www. clinicalkey.com/#! /content/ book/3-s2.0-B978 1 45575980400 1 843. Accessed May 30, 20 1 5 . Mink J W, Zinner SH: Movement disorders I I : Cho­ rea, dystonia, myoclonus, and tremor. Pediatr Rev. 20 1 0;3 1 (7) :287 -295. Mix AC, Romero JR. Central nervous system infections. In: Zaoutis LB, Chiang VW, eds. Comprehensive Pedi­ atric Hospital Medicine. Philadelphia, PA: Elsevier; 2007:340-35 1 . O'Donnell KA, Ewald, MB. Poisoning. In: Kliegman RM, Stanton BF, St Geme JW, Schor NF, Berman RE, eds. Nelson Textbook of Pediatrics. 1 9th ed. Phila­ delphia, PA: Elsevier; 20 1 1 :250-270. Accessed May 30, 20 1 5. https:/ /www. clinicalkey.com/#!/content/ book/3-s2.0-B978 1 43 7707557000580 Ogle JW, Anderson MS. Infections: Bacterial & spirochetal. In: Hay WW Jr, Levin MJ, Deterding RR, Abzug MJ, eds. Current Diagnosis & Treatment: Pediatrics. 22nd ed. New York, NY: McGraw-Hill; 20 1 3 . http://accessmedicine.



N E U ROLOG I C D I SO R D E R S

537

mhmedical.com/ content.aspx?bookid= 1 0 1 6&Sectio nid=6 1 607535. Accessed May 30, 20 1 5 . Peredo DE, Hannibal MC. Th e floppy infant: Evaluation of hypotonia. Pediatr Rev. 2009;30;e66. doi: 10. 1 542/ pir.30-9-e66. Plosa EJ, Esbenshade JC, Fuller MP, Weitkamp JH: Cytomega­ lovirus infection. Pediatr Rev. 20 1 2;33(4) : 1 56- 163. Roos KL, Brosch JR. Meningitis and encephalitis. In: McKean SC, Ross JJ, Dressler DD, Brotman DJ, Ginsberg JS, eds. Principles and Practice of Hospital Medicine. New York, NY: McGraw-Hill; 2 0 1 2 . http://accessmedicine.mhmedi­ cal.com/ content.aspx?bookid=496&Sectionid=4 1 304 1 86. Accessed May 30, 20 1 5. Rosen BA: Guillain-Barre syndrome. Pediatr Rev. 2 0 1 2;33(4) : 1 64- 1 7 1 . Schunk JE, Schutzman SA: Pediatric head injury. Pediatr Rev. 2 0 1 2;33(9):398-4 1 1 . Shinnar S , Berg AT, Moshe SL, et al. The risk of seizure recur­ rence after a first unprovoked afebrile seizure in child­ hood: An extended follow-up. Pediatrics. 1 996;98(2 pt 1 ) : 2 1 6-225. Singer HS, Mink JW, Gilbert DL, Jankovic J. Movement Disor­ ders in Childhood. Philadelphia, PA: Saunders Elsevier; 2010. Stephan M, Carter C, Ashfaq S. Pediatric emergencies. In: Stone C, Humphries RL, eds. Current Diagnosis & Treatment Emergency Medicine. 7th ed. New York, NY: McGraw-Hill; 20 1 1 . http:/ I accessmedicine.mhmedical. com/content.aspx?bookid=385&Sectionid=40357266. Accessed April l6, 20 1 5. Swaiman KF, Ashwal S, Ferriero DM, Schor NF, eds. Swaiman's Pediatric Neurology. Philadelphia, PA: Elsevier Saunders; 2012. Tsao CY: Muscle disease. Pediatr Rev. 2 0 1 4;35(2) :49-6 1 .

This page intentionally left blank

N utritio n

24

Ke l l y E. Wood

While eating a snack you start reading the nutritional label. You note the total calories listed and start thinking about daily nutritional needs. Question 1 - 1 Which factor is NOT used to calculate daily energy needs? A) Health conditions. B) Age. C) Activity level. D) Height. E) Gender. Discussion 1 -1 The correct answer is "A:' The estimated energy requirement is the estimated daily calorie needs for both baseline metabo­ lism and growth. The complicated equation is based on age, gender, height, weight, and physical activity of healthy chil­ dren. It will not account for the increased needs seen with acute and chronic illnesses. Calorie and nutrition needs rela­ tive to body size are greatest in infancy when rapid growth and brain development are occurring. Birth weight triples during the first year of life. After infancy, the stages of child­ hood followed by puberty have the next highest energy needs per body size.

You look at the fat content. You wonder what this means for different age groups. Question 1 -2 Which of the following statements is true? A) Dietary fat requirements are greatest in adolescence. B) Total fat intake should not exceed 25% of daily calories. C) Fat is not needed for absorption of vitamin D. D) Fat is the main energy source for infants. E) Trans fats decrease the risk of heart disease.

Discussion 1 -2 The correct answer is "D:' Fat (lipid) is broken down by lipase (pancreatic enzyme), combined with bile and fat-soluble vita­ mins, absorbed in the jejunum, and enters the lymphatic sys­ tem for transport to the blood. Any disruption of this process will cause malabsorption with steatorrhea. Newborns have decreased ability to absorb fat due to low pancreatic lipase and bile acid activity. Fat is the most calorie dense nutrient. The majority of dietary fat is triglycerides. The high calorie require­ ments of infants to support growth and brain development are met by having a diet high in fat; 50% of infant calories come from fat. Vitamin D is a fat -soluble vitamin. Saturated and trans fatty acids increase low-density lipoprotein (LDL) cholesterol and the risk of coronary heart disease.

The label lists the total grams of carbohydrates and protein. The front wrapper boldly proclaims "no added sugars:' Question 1 -3 Which of the following is false? A) Added sugars should be less than 25% of daily calories. B) Glucose is the primary energy source for the brain. C) Fiber is a digestible protein. D) Protein should make up 10% to 35% of daily calories. E) Protein intake is limited in some inborn errors of metabolism. Discussion 1 -3 The correct answer is "C:' Carbohydrates supply the body with glucose especially the central nervous system, which is the larg­ est utilizer of glucose for energy. Digestive enzymes (amylase and disaccharidases) break down carbohydrates into mono­ saccharides for absorption in the small intestine. Amylase is produced in the saliva and pancreas. Disaccharidases, such as lactase, are brush border enzymes found in the small intestine villi epithelium. Newborns are born with decreased amylase and lactase, which can cause asymptomatic lactose malabsorp­ tion. Carbohydrates are the main energy source of children

539

540

MCG RAW-H I LL E D U CATION S P E C I A LTY BOA R D REVI EW: P E D I ATRICS

and adolescents. Carbohydrates should constitute 45% to 65% of daily caloric intake. Sugars, such as fructose in high-fructose corn syrup, are frequently added to beverages to improve the taste. Intake of added sugars is linked to obesity, diabetes, and cavities. Fiber is a nondigestible carbohydrate important to pre­ vent constipation and overeating. Protein is needed for every cell in the body, and intake must meet amino acid needs. If intake is inadequate, muscle is broken down to meet the body's needs. In certain hypermetabolic states, such as burns, protein intake should be increased but in general increased intake is not needed. Intake of protein or specific amino acids may need to be limited in gout, renal disease, and certain inborn errors of metabolism.

You are caring for a premature infant in the neonatal inten­ sive care unit. She is "feeding and growing?' With her cur­ rent feedings, her weight gain and growth velocity are on target. The mother asks why you are adding fortifier to her breast milk. Question 2-1 Which of the following is true? A) Premature infants have increased nutritional needs. B) Iron stores are built up during the second trimester. C) Unfortified breast milk provides adequate protein. D) Fortification increases the risk of necrotizing enterocolitis. E) Fortifier is added to thicken the milk. Discussion 2-1 The correct answer is "A:' Nutrition is important for brain devel­ opment in premature infants. The third trimester is important for building fat, glycogen, protein, vitamin, and mineral stores. To build up missing stores and grow, premature infants have high nutrient needs especially protein, iron, calcium, and phos­ phorus. Breast milk is fortified to meet the preterm infant's nutritional needs especially protein. Adding human milk forti­ fier increases the calories, protein, vitamin, iron, calcium, and phosphorus content. Fortification meets the infant's nutritional needs while providing the benefits of human milk (decrease necrotizing enterocolitis, late-onset sepsis).

C) Dehydration. D) Zinc deficiency. E) All of the above. Discussion 3-1 The correct answer is "E:' This toddler has kwashiorkor. Maras­ mus and kwashiorkor are two types of severe protein energy malnutrition. Features of both may be present in the same child. Marasmus is associated with total calorie deficiency with loss of fat stores and muscle wasting. Affected infants and children appear emaciated, tired, and apathetic. Their skin and hair is thin. Bradycardia, hypotension, and hypothermia may be pres­ ent. The buttocks and extremities are shrunken with minimal muscle mass. The skin hangs due to loss of subcutaneous fat. (See Figures 24- 1 through 24-3.) Kwashiorkor is associated with energy and protein deficiency but the full pathophysiology is not understood. Edema (anasarca) is the defining character­ istic. In contrast to marasmus, fat stores are relatively preserved and height and weight are normal or near normal for age. A weeping, scaly, reddish-brown rash; brittle hypopigmented hair; and irritability may be present. Children with severe malnutri­ tion are at increased risk for infection including sepsis, pneumo­ nia, and diarrhea. Death is frequently due to an overwhelming infection. Dehydration may result from acute or chronic diar­ rhea. Nutrient deficiencies including zinc are very common.

Hair easily pl uckab l e ,

Apathy

Fatty l ive r Abdomen p rotu berant (may h ave ascites) Skin frag i l e , s l o w to heal --r----- Body fat d i m i n ished

t Cell-med iated i m m u n ity

While visiting a developing country, you meet a young tod­ dler. His face is full and he has a pot belly. His hair is brittle with depigmented areas. A reddish weeping rash is present on his legs and groin. Pitting edema of his feet, legs, and peri­ orbital areas is present. Question 3-1

He is at risk of which of the following? A) Death. B) Sepsis.

Lab tests :

t Albumin t Transferrin and total

i ro n - b i n d i n g capacity

F I G U R E 24- 1 . Kwashio rkor. Manifestations of kwashiorkor. (Reproduced with permission from M u rray RK, Bender DA, Botham KM, Ken n e l l y PJ, Rodwe l l VW, Wei I PA, eds. Harper's Illustrated Biochemistry. 29th ed. New York, NY: McGraw-H i l l Education; 201 2, Fig. 57-1 7.)

C H APTER 24



N UTRITION

541

D) Hair loss. E) None of the above. Discussion The correct answer is "B:'

While in the newborn nursery, you talk with a new mother who is breastfeeding. She is exhausted and considering changing to formula feeding for her newborn. She asks if formula is the same as human milk.

F I G U R E 24-2. Kwashio rkor Dermatitis. A "flaky pai nt" or "crazy pavement" dermatitis is seen i n c h i l d ren with kwashiorkor. (Reproduced with permission from Goldsmith LA, Katz 51, G i l c h rest BA, Pa l l e r AS, Leffe l l DJ, Wolff K, eds. Fitzpatrick's Dermatology in General Medicine. 8th ed. New York, NY: McGraw­ H i l l Education; 201 2, Fig. 1 30-3.)

� QUICK QUIZ Th e defining characteristic o f kwashiorkor is: A) Hanging skin. B) Edema. C) Diarrhea.

Question 4- 1 Which of the following statements is true? A) Human milk contains more casein than whey protein. B) Cow's milk formula is lactose free. C) Both provide 1 00% of daily vitamin D requirements. D) Gastric emptying is faster for newborns fed human milk. E) Carbohydrates make up the majority of calories. Discussion 4-1 The correct answer is "D:' The American Academy of Pediat­ rics (AAP) recommends exclusive breastfeeding for the first 6 months of life with continuation for 1 year or longer. Formula has 50% more protein than human milk. The primary protein in human milk is whey (70%) whereas casein (80%) predominates in cow's milk formulas. Whey protein is easier to digest and pro­ motes faster gastric emptying. The majority of calories in both come from fat. Human milk contains lipase, which increases fat absorption. Lactose is the primary carbohydrate found in human and cow's milk formula. Human milk is deficient in vita­ min D, and sun exposure is limited in infants; therefore, breast­ fed infants should be supplemented with 400 IU of vitamin D daily to prevent rickets. The vitamin K content of human milk is low. All newborns, especially those who are breastfed, should receive a single intramuscular injection of vitamin K after birth. The iron content of human milk is low but better absorbed and should be adequate for the first 6 months of life. Question 4-2 Which is an absolute contraindication to breastfeeding? A) Galactosemia. B) Active maternal tuberculosis infection. C) Herpes simplex virus breast lesions. D) Maternal substance abuse. E) All of the above.

F I G U R E 24-3. M a rasmus. A c h i l d with extreme marasmus with atroph ied arms, loose skin, and no su bcuta neous fat. (Reproduced with permission from Goldsmith LA, Katz 51, G i l c h rest BA, Pa l l e r AS, Leffe l l DJ, Wolff K, eds. Fitzpatrick's Dermatology in General Medicine. 8th ed. New York, NY: McGraw­ H i l l Education; 201 2, Fig. 1 30-1 .)

Discussion 4-2 The correct answer is "E:' Galactosemia is a congenital disor­ der of impaired metabolism of galactose. Galactose and glucose make up the disaccharide lactose. Human and bovine milk naturally contain lactose. Infants with galactosemia require soy formula and lifelong avoidance of lactose. Tuberculosis is transmitted by respiratory droplets to those in close contact.

MCG RAW-H I LL E D U CATION S P E C I A LTY BOA R D REVI EW: P E D I ATRICS

542

Breastfeeding should be avoided to minimize close contact between the mother and infant. Herpes simplex virus infec­ tions can be deadly in young infants, especially newborns. Direct contact with the lesions may transmit the infection from the mother to the infant. Mothers infected with human immunodeficiency virus (HIV) or human T-cell lymphotropic virus should not breastfeed. Both viruses are transmitted through breast milk. Some medications, drugs, and alcohol are transmitted in breast milk. Before starting any medication, it is important to ensure it is safe to take while breastfeeding. Mothers who abuse drugs should not breastfeed. Amphet­ amines, cocaine, tetrahydrocannabinol (THC) , and phencycli­ dine (PCP) are passed through breast milk and can adversely affect the infant.





Helpful Tip

Breast m i l k conta i n s i m m u nologic proteins, i n c l u d i n g

1 1 1r secretory l g A a n d

lactoferri n . l g A is i m porta nt for

m u cosal i m m u n ity, helping to protect a g a i n st d i a rrhea l

You are seeing an infant girl for her 4-month well-child check. She was recently switched from breast milk to formula. Her father is worried she is constipated. Her stools are soft, non­ bloody, and brown but she does not poop every day. She does not strain or act in pain when passing a bowel movement. As a newborn, she passed meconium on the first day of life. She is having 8 wet diapers per day. She is growing well. On exam, she is happy and healthy. He is very worried. Question 5- 1 What do you tell him? A) She is constipated. Use glycerin suppositories. B) She has cow's milk protein intolerance. Switch to a protein hydrolyzed formula. C) She is dehydrated. Give her two ounces of water every day. D) She may have Hirschsprung disease. You will refer her to a gastroenterologist. E) She is normal. Now let's talk about vaccines.

a n d respi ratory i l l n esses. (See Ta ble 24- 1 .)

TA B L E 24- 1

B R EASTF E E D I N G P ROTECTS AGA I N ST

T H E S E CO N D I T I O N S I n fections

Otitis m ed i a Respi rato ry i nfections Dia rrhea a n d g a stroi ntest i n a l i nfections C h r o n i c Conditions

I nfl a m matory bowe l d i sease Dia betes m e l l itus type 1

Discussion 5-1 The correct answer is "E:' Constipation is more common in for­ mula-fed infants, but she is not constipated. Her stools are soft and effortless to pass. Formula-fed infants stool less frequently than breastfed infants. Formula-fed infants have formed, brown stools in contrast to the semiliquid, seedy, yellow stools of breastfed infants. Glycerin suppositories are safe to use in this age group but she does not need medical treatment. Cow's milk protein intolerance may present as failure to thrive, bloody stools, diarrhea, or constipation. Your patient is fat, healthy, and growing. Infants do not need water or juice. Formula or human milk will meet their fluid needs. Infants have 8 to 10 wet diapers per day. Fewer than this may suggest dehydration. Other signs of dehydration would be weight loss, tachycardia, sunken fon­ tanelle, dry lips, and poor capillary refill. Hirschsprung disease is unlikely without a history of delayed passage of meconium.

Ast h m a Eczem a M a l i g nancy

Leu ke m i a Lym p h o m a Prematurity

Necrotizi ng enteroco l itis



Helpful Tip

=-� Breastfed infants eat 8 to 1 2 times per day. Formula-fed

r1 1r infants eat less frequently, usually every 3 t o 4 hours for 6 to 8 feedings per day. I ncreased feeding frequency is the result of faster gastric emptyi ng of breast m i l k than formula and not due to insufficient maternal milk su pply.

Sepsis N utrition a n d Gastroi ntestinal Conditions

Obesity Other

Sudden i nfa nt death synd rome (SI DS) Data from Klei n m a n RE, Com m ittee o n N utrition, eds. Pediat­ ric Nu trition Handbook. 6th ed. E l k G rove Vi l l a g e, I L: A m e rica n Aca d e my of Ped iatri cs; 2009; a n d Sect i o n on B rea stfeedi n g . B rea stfee d i n g a n d t h e use of h u m a n m i l k. Pediatrics. 2 0 1 2; 1 29(3) :e82 7-84 1 .

A 6-week-old male infant is brought to the emergency depart­ ment for evaluation of bright red blood in his stool. He has no history of fever, vomiting, diarrhea, rash, bruising, or leth­ argy. He received an intramuscular injection of vitamin K after birth. He is formula fed and has been growing well. The family noticed the bright red blood tonight in his stool when changing his diaper. On exam, he is alert, well-nourished and

C H APTER 24

healthy. His abdomen is soft with good bowel sounds. His anus has no lesions or fissures. An abdominal X-ray is normal. Question 6-1 What is best first action to take? A) Check coagulation studies. B) Obtain a thorough history. C) Obtain stool cultures. D) Obtain an abdominal ultrasound. E) Consult surgery. Discussion 6-1 The correct answer is "B:' The infant in this case likely has aller­ gic proctocolitis due to cow's milk protein intolerance, which can occur in formula and exclusively breastfed infants. Cow's milk is the most common food allergy of infants and toddlers, with the highest incidence in infancy. Gastrointestinal symptoms are most common but other organ systems may be involved. (See Table 24-2.) Adverse reactions may be IgE mediated (allergy) , non-IgE mediated (intolerance), or mixed. Differentiation TA B L E 24-2 COW'S M I L K PROT E I N A L L E RGY

AND I NTO L E R A N C E SYM PTO M S IN I N FA NTS

AND CHI LDREN

Gastroi ntest i n a l

and diagnosis is difficult. Improvement on an elimination diet with subsequent return of symptoms when cow's milk protein is reintroduced confirms the diagnosis. Negative cow's milk protein IgE and skin prick testing does not rule out the diag­ nosis. Bloody stools can occur with a coagulopathy. The infant received vitamin K prophylaxis after birth and is formula fed. Bacterial infections can cause bloody diarrhea and would be detected by stool cultures. Infection is unlikely as the infant is well and is not having diarrhea. Intussusception can present with bloody stools but typically in an older infant or toddler. Classically infants have colicky abdominal pain, vomiting, and bloody stools described as "currant jellY:'





Helpful Tip

An ora l c h a l lenge of cow's m i l k to confi rm the diag nosis

1 11r o f cow's m i l k protein i ntolerance may b e ski pped i n

situations that wou l d b e too risky, s u c h as a n a phylaxis, h ives, wheezi ng, or stridor. These patients should be managed by a specia l ist.

You suspect cow's milk protein intolerance in this 6-week-old infant.

Gastroeso phageal refl ux

How should the infant be managed? A) Instruct the parents to switch to an extensively hydrolyzed formula. B) Instruct the parents to switch to a soy formula. C) Instruct the parents to switch to a partially hydrolyzed formula. D) Prescribe diphenhydramine. E) No management or treatment is necessary.

Constipation Protei n-los i n g entero pathy Abd o m i n a l pa i n Food i m pacti on I ro n d eficiency a n e m i a O r a l aversion or refu s a l t o eat Vom i t i n g Anorexia R h i norrhea Wheezi ng o r stridor C h ro n i c co u g h U rticaria Ato pic de rmatitis Ang ioedema Syste m i c

543

Question 6-2

Dia rrhea

Ski n

N UTRITION

Fa i l u re t o th rive B l oody stools (gross and occ u l t)

Res pi rato ry



Ana phylaxis Shock- l i ke sym pto m s with seve re vo m iti ng, d i a rrhea, a n d m eta b o l i c acidosis (food protei n-ind u ced enteroco l itis, [FPI ES])

Ada pted with permission from Kol etzko S, N iggema n n B, A rato A, et al: Diagnostic a pproach a n d management of cow's- m i l k protein a l lergy i n i nfa nts a n d c h i l d ren: ESPG HAN G l Comm ittee practica l g u i d e l i nes, J Pediatr Gastroentero/ Nutr. 2 0 1 2 Aug;55(2):22 1 -229.

Discussion 6-2 The correct answer is "A:' He should be started on an extensively hydrolyzed formula, which contains only whey or casein pep­ tides. For severe disease or failure to improve, an amino acid­ based formula is indicated. Beware-both types of formula taste bad and are costly. A soy formula is not preferable as 10% to 15% of infants with cow's milk intolerance have soy sensitivity as well. Breastfeeding mothers will need to adopt a diary-free diet. After 1 year of age, most infants can have cow's milk reintroduced into their diet. Differentiating from lactose intolerance may be diffi­ cult as most hydrolyzed and amino acid formulas are lactose free. Lactose intolerance, other than congenital lactase deficiency, typ­ ically presents at an older age and does not produce respiratory, cutaneous, or severe gastrointestinal symptoms.





Helpful Tip

Congenital lactase deficiency (pri mary lactase defi-

1 11 r ciency) is ra re. Sym ptoms beg i n at birth with watery diar­

rhea after the fi rst feed ing. I nfa nts should be changed to

soy form u l a (lactose free).

544

MCG RAW-H I LL E D U CATION S P E C I A LTY BOA R D REVI EW: P E D I ATRICS

The infant is lost to follow-up. He presents 2 months later with diarrhea. He has gained weight. On exam, he has gen­ eralized pitting edema, a full abdomen, and respiratory dis­ tress. On chest X-ray he has bilateral pleural effusions. His serum albumin level is 2 g/dL, with normal urinalysis, and coagulation studies.

TA B L E 24-3 CAU S E S OF PROTE I N - LOS I N G

E N T E R O PATHY ( P L E )

M ucosa l I nj u ry

I nfl a m mation a n d u l cerative d i seases

C ro h n d i sease U lcerative co l itis I nfecti o n s

Question 6-3 Which is NOT a cause of his condition? A) Nephrotic syndrome. B) Celiac disease. C) Intestinal lymphangiectasia. D) Fontan procedure. E) Cirrhosis with portal vein thrombosis. Discussion 6-3 The correct answer is ''A:' Protein-losing enteropathy (PLE) results in gastrointestinal protein loss, hypoalbuminemia, diar­ rhea, and anasarca. PLE is caused by intestinal mucosal damage or lymphatic system abnormalities. Severe cow's milk protein intolerance is one of many different causes. (See Table 24-3.) Other causes of hypoalbuminemia must be excluded. In PLE, stool is positive for alpha - 1 antitrypsin. Hypogammaglobu­ linemia may be present. Lymphopenia may occur with lym­ phatic pathology. Functional imaging can localize the site of protein loss. Nephrotic syndrome causes hypoalbuminemia and edema due to loss of protein in the urine. Proteinuria will be present on urinalysis.

M a l i g n a ncies G raft-ve rsus-host d i sease Necrotizi ng enteroco l itis N o n u l cerative d i seases

Food i n d u ced ente ropathy Eosi noph i l i c gastroente ritis Va sc u l itis (S LE, H S P) Lym phatic Abnormal ities

Co n g e n ital d i sease



Obstruction

� QUICK QUIZ Which i s an indication for soy formula use? A) Families wanting a vegetarian diet. B) Acute gastroenteritis. C) Prevention of atopic disease. D) Colic. E) Premature infant. Discussion The correct answer is ''A:' Soy formula is indicated in galactose­ mia, congenital lactase deficiency, and when a vegetarian diet is

Sarcoidosis C ro h n d i sease Lym p h o m a



Elevated lym phatic pressu re from elevoted venous pressure

� QUICK QUIZ

Discussion The correct answer is "E:'

Pri m a ry i ntesti n a l lym p h a n g i ectasia

Acq u i red d i seases

Synd romes

Which i s NOT a cause of hypogammaglobulinemia? A) Protein-losing enteropathy. B) Malnutrition. C) Liver failure. D) Nephrotic syndrome. E) Angioedema.

Ce l i a c d i sease

Congestive heart fa i l u re Con strictive pericard itis Noonan Tu rner Kl i ppei-Tre na u nay

Other

Post-Fo nta n proced u re for co n g e n ital heart d i sease

HSP, Henoch-Schi:inlein purpura; S LE, systemic l u pus erythematosus. Ada pted with permission from Bra a ms ka m p MJ, Dolman KM, Ta bbers M M : C l i n ical practice. Protein-losing enteropathy in c h i l d ren, Eur J Pediatr. 2 0 1 0 Oct; 1 69( 1 0) : 1 1 79- 1 1 85 .

preferred. Premature infants have increased osteopenia when fed soy formula. There is no proven value in the use of soy formula for the management and/or prevention of colic or atopic disease.

A medical student is shadowing in your clinic. She asks when solid foods may be introduced into the diet of infants. Question 7-1 Which of the following should you tell her? A) Pureed meats are good first foods to introduce. B) Cow's milk can be introduced before 1 year of age. C) Exclusive breastfeeding should continue until 9 months of age.

C H APTER 24

D) Egg should not be introduced until 2 years of age to prevent development of a food allergy. E) None of the above. Discussion 7-1 The correct answer is ''A:' Complimentary feeding includes the introduction of foods or liquids other than breast milk. Timing of introduction of solid foods is a common question from care­ givers. The AAP recommends exclusive breastfeeding for the first 6 months with continuation until l 2 months of age. Solid foods can be introduced at 4 to 6 months of age. The infant must be developmentally ready, including good head control, and loss of the tongue-thrust reflex. One single-ingredient food should be introduced at a time, waiting 3 to 5 days to introduce another new food. Should the infant have an allergic reaction it will be easier to identify the responsible food. The AAP recommends iron-fortified infant cereals and pureed meats as good first foods as they contain ample protein, iron, and zinc. Pureed fruits or vegetables can be added next. It may take repeated exposure before a new food is accepted. Introduction of cow's milk before 1 year of age increases the risk of developing iron deficiency anemia as cow's milk is low in iron and may cause gastrointesti­ nal blood loss in infants. Some authorities recommend delaying the introduction of certain foods (eg, egg, peanuts, soy) to pre­ vent the development of food allergies. Data are currently lack­ ing or conflicting to delay the introduction of such potentially allergenic foods.



Helpful Tip

� Cow's

=-

m i l k does not meet the g rowi n g i nfa nt's

r1 1r nutritional needs a n d s h o u l d not b e i ntrod u ced i n the

fi rst yea r of l ife. I ron, zinc, vita m i n E, a n d essential fatty acid d eficiencies as wel l as excessive ren a l sol ute load

from p rotein a n d sod i u m a re a ssociated with early i ntroduction . B reastfeed i n g or formula feed i n g s h o u l d conti nue for the fi rst yea r o f l ife.



� I

Helpful Tip



N UTRITION

545

Discussion The correct answer is "C:' Vitamin B 1 (thiamine) is a water­ soluble vitamin. Water-soluble vitamins are easily absorbed but not stored in the body, requiring ongoing intake to prevent deficiencies. With the excess excreted, the risk of toxicity is low. (See Table 24-4 and Figure 24-4.) Fat-soluble vitamins are stored in the body and need pancreatic enzymes and bile acids for absorption. (See Table 24-5.)

A 12-year-old girl presents with diarrhea that is worse with eat­ ing. Stool fecal fat is elevated. Her growth is poor. She has club­ bing and a chronic cough. She complains that her eyes burn and are dry. On exam, she has foamy, triangle-shaped infil­ trates on her conjunctiva. Her funduscopic exam is normal. Question 8-1 You suspect a vitamin deficiency, but which one? A) Vitamin D. B) Vitamin K. C) Vitamin A. D) Vitamin E. E) Vitamin C. Discussion 8-1 The correct answer is "C:' The girl described has cystic fibrosis with fat malabsorption due to pancreatic insufficiency. Condi­ tions associated with fat malabsorption, such as celiac disease, cystic fibrosis, biliary tract disease, and pancreatic insufficiency, result in fat-soluble vitamin (A, D, E, K) deficiencies. She has xerophthalmia (dry eyes) and Bitot spots (keratin accumulation in the conjunctiva) on exam, which is consistent with vitamin A deficiency. Vitamin A is important for proper vision and eye health. Deficiency may increase morbidity and mortality from measles. Supplementation for those deficient or living in vitamin A-deficient areas during active measles infection has been shown to decrease morbidity and mortality. Routine prophylaxis is not indicated for healthy infants and children who are well fed.

I nfa nts should not d r i n k fru it j u ices i n the fi rst 6 months

1 1r o f l ife. For c h i l d ren u p t o age 6 years, 1 OOo/o j u ices s h o u l d

be offered a n d l i m ited t o 4 t o 6 ounces o f j u ice per day.

C h i l d ren 7 to 1 8 yea rs o l d may have 8 to 1 2 ounces of j u ice per d ay. Fruit d r i n ks, sport d r i n ks, soda, or other sugar-sweetened beverages should be d i scou raged.

� QUICK QUIZ Which i s NOT a fat-soluble vitamin? A) Vitamin E. B) Vitamin K. C) Vitamin B 1 . D) Vitamin A. E) Vitamin D.

A 3-year-old African American girl is brought to the clinic. She does not like milk and gets minimal sun exposure as the family lives on a busy street requiring adult supervision to play outside. You are worried about vitamin D deficiency. Question 9-1 Which of the following is NOT a risk factor for vitamin D deficiency? A) Prematurity. B) Sunscreen (sun protection factor 30 or greater) . C) Dark pigmented skin. D) Vegan diet. E) Unlimited sun exposure.

546

MCG RAW-H I LL E D U CATION S P E C I A LTY BOA R D REVI EW: P E D I ATRICS

TA B L E 24-4 WAT E R - SOLU B L E VITA M I N S

Vita m i n B , (Th i a m i ne)

Vita m i n B 2 (Ri boflavi n)

Sou rce

Deficiency

Toxicity

Leg u m e s

None known

Yea st

Wet beriberi: ca rd i o myo pathy, cong estive heart fa i l u re, edema

B rown rice

Dry beriberi: peri pheral neu ro pathy

Whole g ra i n ce rea l

I nfa nti l e beri beri: hoa rse c ry, a phon ia, vo m iting, shock

Po rk

Wer n icke encephalopathy: ataxia, confusion, nystag­ m u s, ophtha l m oplegia

Meat

A n g u l a r stomatitis (see Fig u re 24-4)

Da i ry

G l ossitis

Eggs

Seborrheic dermatitis: gen ital a rea, nose

None known

G reen vegeta bles Fortified cerea l s Vita m i n B 3 (Niacin)

Vita m i n B 5 (Pantothenate)

Meat

Pel l a g ra: dermatitis of s u n -exposed skin (d ry, cracked, t h i ckened skin), d i a rrhea, and dementia

Flushing

Da i ry Eggs

Cheilosis

H ives

Pruritus

Bea n s

Vom i t i n g

Fortified cerea l s

Elevated l iver enzymes

Egg yo l k

B u r n i n g feet syndrome: d i stal pa resthesia

None known

Cheilosis

Neu ropathy Photose nsitivity

Liver Milk B rocco l i

Vita m i n B 6 (Pyridoxi n e)

Meats G ra i n s

A n g u l a r stomatitis

Vegeta bles

G l ossitis

N uts

Seizures i n i nfa nts I rrita b i l ity Anemia

Vita m i n B 7 (Biotin)

Egg yo l ks

Hypoto n i a

Milk

Dermatitis

Meat

Alopecia

None known

Vegeta bles Vita m i n B 9 (fo late)

G reen vegeta bles

Megaloblastic a n e m i a (macrocytic)

Liver

Hype rseg me nted neutro p h i l s

Yea st

N e u ra l t u b e defects

Masks vita m i n B , 2 deficiency i n perni­ cious anemia

Fortified cerea l/bread Vita m i n B , 2 (Co ba l a m i n )

Meat

Megaloblastic a n e m i a (macrocytic)

None known

Fish

Cheilosis

Da i ry

Peri phera l n e u ro pathy

Eggs

N e u ro psych iatric sym pto m s

Elevated l eve l s may be associated with m a l i g n a ncy, a uto­ i m m u n e co n d i ­ tions, re n a l fa i l u re, a n d l ive r d i sease

Hype rseg me nted neutro p h i l s

(Continued )

C H APTER 24

TA B L E 24-4 WAT E R - SOLU B L E VITA M I N S

Vita m i n C (Ascorbic acid)



N UTRITION

( CONTINUED)

Sou rce

Deficiency

Toxicity

Citrus fru its Strawberries

Scu rvy: ecchymoses, bl eed i n g g u ms, perifo l l i c u l a r hemorrhage

Fa lse-negative stool g u a i a c

Tomatoes

Coi l ed "corkscrew" h a i rs

Dia rrhea

Potatoes

I m pa i red wo u n d h ea l i ng

Brussel s p routs

Hysteria

Abd o m i n a l bloati n g

Spinach

547

Pa i n fu l extre m ities Dia rrhea

Data from Kl e i n m a n RE, Comm ittee on N utrition, eds. Pediatric Nutrition Handbook. 6th ed. Elk G rove Vi l l age, I L: American Aca demy of Ped iatrics; 2009.

Discussion 9- 1 The correct answer is "E:' Sunlight exposure is the maj or source of vitamin D for children. Fortified foods such as milk or breakfast cereals are good sources of dietary vitamin D • 2 Vitamin D 3 is synthesized in the skin by exposure to sun­ light. Both are converted to 25-hydroxyvitamin D (25-0H-D, calcidiol) in the liver then transported to the kidney to form 1 ,25-dihydroxyvitamin D ( 1 ,25-0H -D, calcitriol) . Calcitriol, 2 the active form of vitamin D, increases intestinal absorption of calcium and phosphorus and renal reabsorption of calcium. Risk factors for vitamin D deficiency include prematurity, dark skin pigmentation (interferes with sunlight absorption) , inadequate sun exposure, certain medications, fat malabsorp­ tion, obesity, liver and kidney disease, and inadequate dietary intake. Medications associated with vitamin D deficiency include certain anticonvulsants and antiretrovirals, gluco­ corticoids, and antifungals (ketoconazole) . Using a sunscreen with a sun protection factor of 30 or greater decreases vitamin D production in the skin. All infants and children younger

than 18 years of age should receive supplemental vitamin D per current recommendations. Supplementation should begin in the first few days of life for breastfed infants. Formula-fed term infants who consume at least 33 ounces per day do not need additional supplementation. (See Table 24-5.)



Helpful Tip

:5.� Vita m i n D d eficiency causes decreased bone m i neral­

r1 1 r ization, hypoca lcemia, a n d hypophosphatemia, which

may result i n seizu res, teta ny, m uscle weakness, rick­ ets, or osteoma lacia a n d increased susceptibil ity to

i nfections.

When examining the toddler, you note bilateral genu varum, widening of her wrists, and frontal bossing. The mother states she noticed her child's bowlegs when the child started walking and it has been getting worse with time. Question 9-2 What additional possible clinical features would you expect in this patient? A) Delayed anterior fontanelle closure. B) Rachitic rosary. C) Craniotabes. D) Harrison grove. E) All of the above.

F I G U R E 24-4. A n g u l a r Stomatitis. Erosions that macerate and bleed at the corners of the mouth a re seen i n B-com plex vita m i n (ri boflavi n and pyridoxine) deficiencies, protei n energy m a l n utrition, and zinc deficiency. (Reproduced with perm ission from Goldsmith LA, Katz 51, G i l c h rest BA, Pa l l e r A S , Leffe l l D J , Wolff K , e d s . Fitzpatrick's Dermatology i n General Medicine. 8th ed. New York, NY: McGraw- H i l l Education; 201 2, Fig. 1 30-7.)

Discussion 9-2 The correct answer is "E:' This toddler has nutritional rick­ ets from inadequate vitamin D intake. Rickets results from inadequate mineralization of growing bone at the growth plate (physis). This results in pathologic enlargement of the physis and metaphysis which leads to bowing of the affected bone. Impaired mineralization in skeletally mature individu­ als (growth plates closed) affects the bone matrix, result­ ing in osteomalacia. Rickets is classified as calcipenic or

548

MCG RAW-H I LL E D U CATION S P E C I A LTY BOA R D REVI EW: P E D I ATRICS

TA B L E 2 4 - 5 FAT - SOLU B L E VITA M I N S

Dieta ry

Vita m i n A (Reti nol, reti n a l , retinoic acid, reti nyl esters)

Vita m i n D (C holeca l ciferol [D), e rgoca lciferol [D))

Refe re n ce

F u n ction/

Sou rce

I nta ke

Ta rget

Deficiency

Toxicity

Animal sources: d a i ry, fish, l i ver, eg g s

Va ries b y age: 1 000-3000 1 U

Eye

N i g ht b l i n d ness

Anorexia

Epithe l i a l ce l l s

Xe rophtha l m i a (d ryness) Bitot s pots

I ncreased i ntracra n i a l pressu re

Keratoma lacia

Liver damage

Cornea d a mage

Pee l i n g ski n ra s h

Ret i n o pathy

Pa i n fu l bone lesions

G reen/ye l l ow veg eta bles (ca rotene): carrots, brocco l i , sweet potatoes

S u n l ig ht

Seve re measles i n fection

Fatty fi s h

400 1 U/day (i nfa nts)

Vita m i n Dfo rtified foods, i nc l u d i n g m i l k

600 1 U/day ( 1 - 1 8 yea rs of age)

Bone Ca lci u m reg u l ation

Osteo ma lacia (skeleta lly mature): pathologic fractu res, hypoca lcemia with or without teta ny, hypo phosphate m i a

Goal 25-0H-D leve l s > 2 0 ng/m l

Vita m i n E (Toco phero l s)

Oils

R i c kets (skel eta l ly i m m at u re): cra n iota bes, rach itic rosa ry, g e n u va rus (bow leg s), splayi n g of wrists

Antioxidant CNS

M otor a n d sensory n e u ro pathy

Meat

Eye

Ataxia

Vegeta bles

S keleta l m u scle

Hemo lytic anemia in premature i nfa nts

G ra i n s

Va ries b y age: 4- 1 5 m g

Reti n a l degeneration

Vita m i n K (Phyl loq u i no n e [K, ] , menaq u i none [K2], menadione [K))

G reen veg eta bles

A I : 2-2.5 m eg/ day (infa nts)

Oil

Va ries b y age: 30-75 m eg/day ( 1 - 1 8 yea rs of age)

Fru its Seeds Cow's m i l k

Hypoprot h ro m b i n e m i a

Coag u l ation: factors I I , VI I , IX, X, C, a n d S

H e m o rrhage: G l , G U , l u n g s, joi nts, C N S

Bone m i n era l ization

Vita m i n K-deficient bl eed i n g of newborn

I ntesti n a l bacte ria

Te ratogen Hyperca l ce m i a N e p h ro l i t h i a s i s N e p h roca l c i nosis Ectopic ca l cification Constipation Depression

I m pa i red response to i ro n i n anemia I m pa i red neutro p h i l fu nction Sepsis in premature i nfa nts Hemolys i s Hyperbi l i ru binemia i n i nfa nts receivi ng l a rg e pa re ntera l dose of synthetic vita m i n No toxi city from oral i nta ke

C h e m ical synthesis AI, adequate i nta ke; C N S, centra l nervou s system; Gl, gastrointesti nal; GU, gen itou rinary; I U i nternational u n its. Data from Klei n m a n RE, Comm ittee on N utrition, eds. Pediatric Nutrition Handbook. 6th ed. E l k G rove Vi llage, I L: American Academy of Pediatrics; 2009.

C H APTER 24



N UTRITION

549

TA B L E 24-6 C L I N I C A L M A N I F E STAT I O N S

O F R I C KETS

S ke l eta l

N o n s ke l eta l

Del ayed a nterior fo nta n e l l e closure

Bone pa i n

Fronta l a n d parietal bos s i n g

Hypoca l c e m i c sei­ zu res or teta ny

Cra n i otabes (soft sku l l bones) Wid e n i n g of the wrist or a n kles G e n u va r u m (bowlegs) : tibia bowing G e n u va l g u s (knock knees): fe m u r bowi ng Cox ve ra: d i stal rad i u s/u l n a bowing Rachitic rosa ry: e n l a rgement of costocho n d ra l j u nctio n s Ha rrison s u l c u s g roove: fl a r i n g of ribs at d i a p h ra g m level Pigeon breast deform ity

Poor g rowth

Denta l a bnormal ities M u scle wea kness Motor delays A m b u lation d ifficu lties Leth a rgy I rrita b i l ity Viscero ptosis: s i n ki n g or sagg i n g o f a bdo m i n a l viscera (pot bel ly)

Kyphosco l i o s i s Data from N ield LS, Ma haja n P, Joshi A, Ka mat D. Rickets: Not a d isease of the past. Am Fam Physician. 2006;74(4):6 1 9-626.

F I G U R E 24-5. Gen u Varum. X-rays showing bowing of the long bones (genu

va rum) and fl a red, irreg u l a r physes i n a child with rickets. (Reproduced with permission from Skinner H B, McMahon PJ, eds. Current Diagnosis & Treatment in Orthopedics. 5th ed. New York, NY: McGraw- H i l l Education; 20 1 4, Fig. 1 0-3.)

phosphopenic. Phosphopenic rickets usually results from renal phosphorus wasting. Calcipenic rickets is most com­ monly due to insufficient intake, absorption, or metab o ­ lism of vitamin D. Initial skeletal manifestations occur at sites of rapid bone growth, including the distal forearm, knee, and costochondral j unctions and are influenced by the child's age and weight-bearing p atterns of limbs. Fore­ arm changes are commonly seen in infants . Walking tod­ dlers classically have exaggerated bowing of the legs (genu varum) . (See Table 24-6 and Figure 24- 5 . ) An anteroposte­ rior X- ray of the knee or wrist is can be helpful to diagnosis rickets . Changes seen include widening of the distal physis, metaphyseal fraying, flaring or cupping, decreased b one density, angular deformities of the arm and leg bones, and pathologic fractures. (See Figure 24- 6 . )

Discussion The correct answer is "C:' In vitamin D-deficient rickets, serum vitamin D (calcidiol and calcitriol) , calcium, phosphorus, and urine calcium are low. Alkaline phosphatase, parathyroid hor­ mone, and urine phosphorus levels are increased. Alkaline phosphatase levels are a useful marker of disease activity with levels frequently exceeding 1 500 IU/L.



Helpful Tip

� Vitamin

=-

r1 1r dietary

K exists in two natura l forms: K, comes from K, i s synthesized by g ram-negative

sou rces;

bacteria in our i ntestines. Broad-spectrum antibiotics may cause vitamin K deficiency from decreased synthesis

of vita mi n

K,.

� QUICK QUIZ Which laboratory value i s elevated i n vitamin D-deficient rickets? A) Vitamin D. B) Phosphorus. C) Alkaline phosphatase D) Calcium. E) Urine calcium.





Helpful Tip

I nt ra m u scu l a r vita m i n K p reve nts l i fe-t h reate n i n g

1 1 1r h e m o r r h a g e i n newborns. Prior reports h ave associ­

ated i n t ra m u s c u l a r vita m i n K with a n i n c reased risk of l e u ke m i a . Th i s association h a s not been p roven, and vita m i n K prophylaxis i s reco m m e n d ed fo r a l l newborns.

MCG RAW-H I LL E D U CATION S P E C I A LTY BOA R D REVI EW: P E D I ATRICS

550

A

B

F I G U R E 24-6. Metaphyseal Changes in Rickets. Wideni ng, fraying, and c u p p in g of the d i stal meta physis of the rad i u s and u l na in a c h i l d with rickets. (Reproduced with permission fro m Fel d m a n D, Pike JW, Ada m s JS: Vitamin D. 3 rd edition. London: Academic Press/Elsevier; 201 1 )



Helpful Tip

=-� Remember

"bones, stones, a bdom i n a l g roans, a n d

r1 1 r psych iatric moa ns" for the sym ptoms of hyperca lcemia from vita m i n D toxicity. Bones: pa in, fractu res Stones: kid ney stones, polyuria G roa ns:

constipation,

anorexia,

peptic

u l cers,

pancreatitis Moa ns: confusion, d epression, dementia

� QUICK QUIZ Which water-soluble vitamin i s found only i n animal prod­ ucts (meat, dairy, eggs) ? A) Vitamin B3 (niacin). B) Vitamin B12 (cobalamin). C) Vitamin B9 (folate) . D) Vitamin B 1 (thiamine) . E) Vitamin B7 (biotin).

Discussion The correct answer is "B:' Water-soluble vitamins (B complex and C) are obtained from dietary sources mostly fruits and veg­ etables except vitamin B12 (cobalamin) , which is found exclu­ sively in animal products.

A 1 6-year-old girl presents with burning and tingling in her legs for the past few months that is getting progressively worse. She is having trouble remembering things at school. At home, she is fighting more with her parents. On exam, she is ataxic, her tongue is smooth, and she has impaired vibra­ tory sensation. Her blood work is notable for hemoglobin of 10 g/dL, mean corpuscular volume (MCV) of 1 10 fL, and hypersegmented neutrophils. Question 1 0-1 Which of the following is NOT a potential cause of her vitamin deficiency? A) Vegan diet. B) Ileum resection. C) Ulcerative colitis. D) Treatment with a proton pump inhibitor medication. E) Gastritis. Discussion 1 0-1 The correct answer is "C' She has vitamin B12 deficiency. Vita­ min B12, cobalamin, is a water-soluble vitamin that is important for red blood cells and the central nervous system. Animal prod­ ucts such as meat and eggs are the only dietary source. Dietary deficiency is rare but may occur in those following a strict vegan diet, including breastfed infants of vegan mothers. Vitamin B12 requires intrinsic factor (produced in the stomach) for absorp­ tion in the ileum. Gastric resection or bypass, ileum resection, Crohn disease, HIV infection, or acid-suppressing medications may impair absorption leading to deficiency. Pernicious anemia,

C H APTER 24

an autoimmune condition, causes vitamin B 1 deficiency due to 2 a lack of intrinsic factor. Vitamin B 1 and folate deficiency may 2 result in a macrocytic (MCV > 1 00 fl) megaloblastic anemia with hypersegmented neutrophils, but only vitamin B 1 defi­ 2 ciency produces neurologic symptoms. Neurologic manifes­ tations may be irreversible even with treatment. Classically a symmetric peripheral neuropathy affecting the legs with ataxia, altered sensation, and weakness develops. Neuropsychiatric manifestations include memory loss, irritability and personality changes. Treatment consists of oral, intramuscular, or intranasal cobalamin. Ulcerative colitis is not associated with vitamin B 1 2 as neither the ileum nor stomach is involved. Question 1 0-2 Which of the following may be associated with folate deficiency? A) Trimethoprim antibiotic. B) Chronic hemolytic anemia. C) Goat's milk. D) Alcohol. E) All of the above. Discussion 1 0-2 The correct answer is "E:' Folate and vitamin B 1 2 are needed for red blood cell DNA synthesis. Deficiency of either leads to impaired erythropoiesis and megaloblastic anemia. Folate is found in animal products and leafy vegetables. Deficiency is most commonly nutritional from inadequate intake. In the United States, fortification of flour with folic acid has decreased the prevalence of folate deficiency. Goat's milk is deficient in folate. Infants should not be fed goat's milk without other dietary sources of folate. Certain conditions, such as chronic hemolytic anemia and pregnancy, have increased folate needs which if not met result in deficiency. Alcohol inhibits folate absorption and deficiency is seen in those who abuse alcohol, especially when concurrent malnutrition is present. Trimethoprim, methotrex­ ate, and phenytoin all interfere with folate metabolism and have been associated with folate deficiency. Folate deficiency is treated with oral folic acid.







N UTRITION

551

He has swollen bleeding gingiva, multiple large ecchymosis, and a petechial rash around the hair follicles of his legs. The anterior costochondral junctions are prominent. You notice a cut on his leg that is not healing. He has no lymphadenopa­ thy, hepatosplenomegaly, arthritis, or jaundice. Question 1 1 -1 Which is the most likely cause of his symptoms? A) Malignancy. B) Disseminated intravascular coagulation (DIC) . C) Liver failure. D) Henoch-Schonlein purpura (HSP) . E) None of the above. Discussion 1 1 -1 The correct answer is "E:' The boy in this vignette has scurvy from vitamin C (ascorbic acid) deficiency. On dietary history, he is picky eater. Humans do not make vitamin C but rather obtain it by eating fruits (citrus) and vegetables. In vitamin C deficiency, synthesis of the collagen walls of blood vessels is disrupted, resulting in hemorrhagic signs and symptoms (ecchymosis, purpura, gingival hemorrhage, petechiae) . Bone pain from subperiosteal hemorrhage may present as pseudo­ paralysis. His physical exam is consistent with scurvy with perifollicular hemorrhages, signs of poor wound healing, and costochondral beading known as scorbutic rosary. (See Table 24-4 and Figures 24-7 and 24-8.) Vitamin C deficiency is treated with oral ascorbic acid. Patients with malignancy may present with thrombocytopenic bruising and bleeding as well as bone pain. The lack of fever, weight loss, lymphade­ nopathy, and hepatosplenomegaly is reassuring. Coagulation studies would be normal, ruling out DIC and liver failure, which may also manifest with j aundice. HSP is an IgA vasculi­ tis that produces palpable purpura, but this boy lacks arthritis and abdominal pain. Scurvy is often mistaken for vasculitis if cutaneous findings are misinterpreted as purpura. However,

Helpful Tip

Folate d eficiency d u ri n g preg na ncy is a ssociated

1 1 1r with neura l tu be d efects. A l l p reg nant women should receive s u p plementa l fol i c acid.

A 1 0-year-old boy presents with bruising that has been getting worse over the past few weeks. His bruises occur with mini­ mal trauma. More recently his gums have been bleeding. His legs ache and he does not want to walk. He has not had fever, weight loss, or abdominal pain. On exam, he appears tired.

F I G U R E 24-7. Scu rvy. Perifo l l i c u l a r hemorrhage and fol l i c u l a r hyperkerato­

sis, which if pro m i nent may be m i staken as pa l pa b l e purpura associated with vascu l itis. (Reprod uced with permission from Ka ushansky K, Lichtman MA, Prchal JT, eta a l : Williams Hematology, 9ed. New York: McGraw- H i l l Education, I nc; 20 1 6. Fig u re 1 22-25.)

552

MCG RAW-H I LL E D U CATION S P E C I A LTY BOA R D REVI EW: P E D I ATRICS

Question 1 2-1 He has a deficiency of what trace mineral? A) Fluoride. B) Iodine. C) Zinc. D) Chromium. E) Copper.

F I G U R E 24-8. Corkscrew H a i rs. P l ugged h a i r fol l icles c a u s e h a i r s hafts t o c o i l and a re referred t o as "corkscrew hai rs:' (Reproduced w i t h permission from McKean SC, Ross JJ, Dress ier DD, Brotman DJ, G i n s berg JS, eds. Principles and Practice of Hospital Medicine. New York, NY: McGraw- H i l l Education; 2 0 1 2, Fig 1 47-1 6A.)

this patient's rash, gum bleeding, and costochondral changes are not characteristic of the conditions listed.

An 8-week-old male infant is brought to the clinic to evaluate a diaper rash. His growth has been poor and he has had prolonged diarrhea. His mother has tried zinc oxide cream and a topic antifungal medication without improvement. On exam, he is thin and has erythematous, well-defined, scaly plaques with bullae and erosions. The rash is on his cheeks, around his mouth, and in his diaper area.

A

Discussion 1 2-1 The correct answer is "C:' This infant has acrodermati­ tis enteropathica from zinc deficiency. Trace elements or minerals are present in low concentrations in the body but necessary for many enzymatic pathways. Zinc is needed to grow, taste and smell, heal wounds, and fight infections. Zinc is given in developing countries to prevent and treat infections such as diarrhea and pneumonia. Acrodermatitis enteropathica, a hereditary defect in zinc absorption, causes diarrhea, dermatitis, and alopecia. The eczema-like rash has red, scaling plaques that develop bullae, crusting, and ero­ sions in a U- shaped distribution on the face, perioral, acral, and diaper areas. (See Figure 24-9.) Fluoride prevents den­ tal caries but in excess can stain the tooth enamel (fluoro­ sis ) . Iodine deficiency results in goiter and hypothyroidism. Concurrent selenium, iron, or vitamin A deficiency worsens the effects of iodine deficiency. To prevent deficiency, table salt has added iodine. Chromium is a cofactor for insulin action; deficiency causes impaired glucose tolerance. Skin and hair depigmentation, microcytic hypochromic anemia (unresponsive to iron), neutropenia, and osteoporosis occur with copper deficiency. Menkes (kinky hair) syndrome and Wilson disease are hereditary defects in copper metabolism. Deficiencies of other trace minerals can also cause significant systemic effects. Selenium deficiency causes cardiomyopathy and myositis. Tremor, seizures, arrhythmias including tors­ ades de pointes, and hypocalcemia are seen with magnesium deficiency.

B

F I G U R E 24-9. Acrodermatitis Enteropathica. Erythematous, wel l-demarcated lesions with crusting, sca l e, and erosions i nvolving the a nogenital a n d periora l

a reas. (Reproduced with permission fro m Goldsmith LA, Katz 51, G i l c h rest BA, Pa l l e r AS, Leffel I DJ, Wo lff K , eds. Fitzpatrick's Dermatology i n General Medicine. 8th ed. New York, NY: McGraw- H i l l Education; 201 2, Fig. 1 30-1 OA, B.)

C H APTER 24



N UTRITION

553

Question 1 3-3

A IS-month-old boy is referred for evaluation of anemia (hemoglobin IO mg/dL) . You learn the infant has been receiv­ ing cow's milk since 9 months of age. Question 1 3-1 What is the best next course of action in the care of this I S-month-old? A) Switch him to formula. B) Start empiric treatment with iron. C) Refer for developmental screening. D) Dietary modification alone. E) Repeat his blood work in 3 months.

Discussion 1 3-1 The correct answer is "B:' Early introduction of cow's milk before 1 year of age is associated with iron deficiency. Cow's milk is deficient in iron and may cause gastrointestinal blood loss in infants. In the presence of risk factors, a pre­ sumptive diagnosis of iron deficiency anemia can be made and empiric treatment with iron without additional testing may be trialed. If the hemoglobin level has not improved within 1 month, iron studies should be obtained. Alterna­ tively the AAP recommends obtaining confirmatory iron studies as only 40% of 1 -year-olds with anemia have iron deficiency and follow-up testing is poor. Dietary modifica­ tions, including limiting cow's milk intake to 24 ounces per day and eating iron-rich foods, should be made in addition to iron supplementation. Question 1 3-2

Which patient group is NOT associated with an increased risk of iron deficiency and iron deficiency anemia? A) Premature infants. B) Menstruating girls. C) Breastfed infants. D) Toddlers. E) Postpubertal boys.

Discussion 1 3-2 The correct answer is "E:' Iron deficiency is the most com­ mon nutritional deficiency in the United States . Premature infants are born with low iron stores as are infants with intra­ uterine growth restriction and those born to mothers with diabetes or anemia. Adolescent girls are at increased risk from menstrual blood loss. After 4 months of age, breast­ fed infants need iron supplementation to meet their growing needs. Infants who are formula fed should receive only iron­ fortified formula. Toddlers who drink excessive cow's milk or are picky eaters will have insufficient iron intake. Boys are at risk during their pubertal growth spurt as demands may exceed stores.

What has NOT been associated with iron deficiency? A) Improved test scores. B) Anemia. C) Restless leg syndrome. D) Lead poisoning. E) Developmental delays. Discussion 1 3-3 The correct answer is "A:' Consequences of iron deficiency include anemia and possible adverse neurodevelopmental out­ comes, including decreased cognition, lower test scores, and motor delays. The neurologic effects may occur before the child becomes anemic. Intestinal absorption of lead is increased, which may result in lead poisoning. Restless leg syndrome, breath-holding spells, febrile seizures, fatigue, and pica have been associated with iron deficiency. Question 1 3-4 Which laboratory result is seen with iron deficiency anemia but NOT iron deficiency? A) Decreased serum ferritin. B) Elevated serum transferrin receptor 1 concentration. C) Decreased reticulocyte hemoglobin concentration. D) Decreased mean corpuscular volume. E) Elevated red blood cell distribution width. Discussion 1 3-4 The correct answer is "D:' Universal hemoglobin screening is recommended at 12 month of age and for older children with iron deficiency risk factors. Hemoglobin screening detects ane­ mia but not iron deficiency. Capillary samples should be con­ firmed by venous measurements to avoid false positive results. Recommended confirmatory iron testing includes serum ferri­ tin and C-reactive protein (CRP) levels or reticulocyte hemo­ globin concentration (CHr) . A low serum ferritin is a sensitive marker for iron deficiency but it is an acute phase reactant with false elevation in the setting of inflammation as measured by ele­ vation in the CRP. Reticulocyte hemoglobin (CHr) and serum transferrin receptor 1 (TfR 1 ) concentrations are not affected by inflammation. A low CHr has been shown to be the strongest predictor of iron deficiency in children. Currently, TfR1 stan­ dards have not been developed for children. An elevated red cell distribution width (RDW) is an early marker of deficiency. (See Table 24-7.)

An 8-month-old boy is hospitalized with vomiting and diar­ rhea. He is rotavirus-positive. He has been rehydrated with intravenous fluids, and his vomiting has resolved. However, he continues to have diarrhea. He is drinking well and acting hungry. On rounds you are told that he is on a clear liquid diet due to ongoing diarrhea.

554

MCG RAW-H I LL E D U CATION S P E C I A LTY BOA R D REVI EW: P E D I ATRICS

TA B L E 2 4 - 7 LABO RATORY TEST R E S U LTS I N I RO N D E F I C I E N CY V E R S U S I RO N D E F I C I E N CY

EM IA

I ro n Deficiency

I ron Defi c i e n cy A n e m i a

Hemog l o b i n

Normal

Decreased

Mean corpusc u l a r vo l u m e (MCV)

Normal

Decreased

Mean corpusc u l a r hemog l o b i n co ncentration (MCHC)

Normal

Decreased

Red ce l l d i stribution width (RDW)

I nc reased

I nc reased

Serum i ro n

Decreased

Decreased

Tota l i ron-b i n d i n g ca pacity (TIBC)

I nc reased

I nc reased

Ferrit i n

Decreased

Decreased

Tra n sferri n sat u ration

Decreased

Decreased

Tra n sferri n rece ptor 1 conce ntration (TfR l )

I nc reased

I nc reased

Reti c u locyte hemog l o b i n concentration (CH r)

Decreased

Decreased

Data fro m Z i m m e r m a n n M B, H u rre l l RF: N ut r i t i o n a i ro n d efi c i e n cy, Lancet 2007;3 7 0 ( 9 5 8 6 ) : 5 1 1 - 5 2 0 .

Question 1 4-1

What do you say in response? A) Oral intake should be delayed for 72 hours in a child with diarrhea. B) Oral intake should be limited to clear liquids for the first 72 hours of the acute illness. C) Oral intake should be advanced to a full liquid diet but solid foods should be delayed until the diarrhea resolves. D) Dairy products should be avoided during an acute diarrheal illness. E) Oral intake should be started as soon as possible with any illness. Discussion 1 4-1 The correct answer is "E:' Oral intake should be started as soon as the child is able. Solids should be restarted as tolerated by the child not based on an arbitrary time period. There is no medical reason to withhold food simply because this patient is still hav­ ing diarrhea. The BRAT (bread, rice, applesauce, toast) diet is no longer recommended as a gateway to reintroducing solid foods after an acute gastrointestinal illness. Absorption of nutrients is decreased with acute diarrhea but not nonexistent. Prolonged periods without enteral nutrition may cause small bowel villi atrophy with subsequent diarrhea. Feeding the gut when able is always a good thing.

The boy is discharged home. Two weeks later he is brought to the clinic for evaluation as he continues to have diarrhea. He is happy, afebrile, and eating well. He is not vomiting. His diarrhea is watery without blood. He takes standard infant formula. On exam, he is a fat, afebrile, healthy infant with plenty of drool. He has regained the weight he lost from his acute illness. Question 1 4-2 What is the cause of his prolonged diarrhea? A) Secondary lactose intolerance. B) Cow's milk protein intolerance.

C) Osmotic diarrhea from excessive juice intake. D) Clostridium diffi cile infection. E) Fat malabsorption. Discussion 1 4-2 The correct answer is ''A:' B efore the vaccine, rotavirus was the "winter buddy" of respiratory syncytial virus (RSV) . It kept hospital beds full of infants with protracted diarrhea. This child is doing well, eating, and gaining weight. He is hydrated. Protracted diarrhea after an acute gastrointestinal illness may be due to transient lactase deficiency from injury to small bowel villi. Lactase, the enzyme that breaks down lactose, is located in the distal tips of the intestinal microvilli. This period of transient deficiency will resolve with time. Feeding the infant through it or imposing a brief period of a nonlactose formula (soy) feeding may be reasonable though unnecessary. Question 1 4-3 What is NOT a cause of secondary lactose intolerance? A) Bacterial overgrowth. B) Celiac disease. C) Crohn disease. D) Giardiasis. E) Ethnic lactose malabsorption. Discussion 1 4-3 The correct answer is "E:' Secondary means it is an acquired condition. You cannot acquire a new ethnicity. Lactose malab­ sorption is common in Asian and African Americans. They are born with decreased lactase activity. Options "B;' "C:' and "D" are conditions that damage the small intestinal villi in a manner similar to rotavirus. Treatment is aimed at the underlying cause while awaiting repair of the villi. Symptomatic treatments may include decreasing diary intake or substituting nondairy prod­ ucts (soy, almond milk) .

C H APTER 24

A 9-year-old girl with chronic kidney disease is admitted to the hospital. She is on the renal transplant list and has been admitted for insertion of a dialysis catheter. The child is thin and pale. You enter a general diet order for her. The next morning, your attending is not happy with your choice. Question 1 5-1 Which is true regarding dietary restrictions and needs with renal disease? A) Total fluid intake is never restricted. B) Potassium requirements are increased. C) Phosphorus requirements are increased. D) Sodium intake may be restricted. E) Nutrition is a minor detail. Discussion 1 5-1 The correct answer is "D:' Dietary restrictions, needs, and deficien­ cies depend on the stage of kidney disease. A child with acute renal failure is very different from one with dialysis-dependent disease. Total fluid intake may be restricted depending on glomerular func­ tion. If a patient is taking a diuretic agent, unlimited fluid intake is typically counterintuitive. Caloric and protein needs are greater in a child with kidney disease. Protein intake should not be restricted, especially for those on dialysis (associated with protein loss). Those with chronic disease are at risk for malnutrition, especially if the child has associated nausea, vomiting, or lack of appetite. Renal disease may cause hyperkalemia and hyperphosphatemia so giving extra potassium would not be a good idea. Sodium goes hand in hand with fluid retention and hypertension.

You begin thinking about nutritional supplements. Vitamins and minerals are good things, but which need to be supple­ mented in this patient? Unfortunately, a dietician is not on call at 3 AM in your hospital. You decide to try and find the answer by yourself. Question 1 5-2 Which is NOT true regarding nutritional deficiencies associ­ ated with chronic renal disease? A) Metabolism of the fat-soluble vitamins A, E, and K is impaired. B) Bone disease (renal osteodystrophy) is common with pro­ gressive chronic kidney disease. C) Hypovitaminosis D includes deficiencies of both 25-hydroxyvitamin D and 1 ,25-dihydroxyvitamin D. D) Potassium intake may need to be restricted. E) Phosphorus intake must be restricted. Discussion 1 5-2 The correct answer is "A:' Bone metabolism and mineralization is disrupted in chronic kidney disease. Secondary hyperparathy­ roidism develops as a result of hyperphosphatemia from renal phosphorus retention. Management includes dietary restriction or phosphate binders. Vitamin D is supplemented, with dosing dependent on the degree of deficiency. Calcium supplementa­ tion is not always needed to maintain a normal calcium level.



N UTRITION

555

The kidney is the potassium-wasting machine of the body. Depending on the degree of failure, potassium may need to be restricted. Nutritional supplementation (oral, feeding tube) may be needed if a child cannot meet his or her needs. Vitamins (except vitamin D) and minerals should be supplemented if the child is not meeting 1 00% of the daily recommended intake (DRI). Hypervitaminosis A may develop in those with advanced kidney disease from decreased renal excretion of metabolites.

An adolescent boy with cystic fibrosis is admitted because of an acute decline in pulmonary function. His weight is down 2 kg from 3 months ago. He has not filled any prescriptions for 2 months. Question 1 6-1 Which nutritional problem is he unlikely to have related to his cystic fibrosis? A) Vitamin D deficiency. B) Hyponatremia. C) Vitamin C deficiency. D) Protein calorie deficiency. E) Hyperglycemia. Discussion 1 6-1 The correct answer is "C:' Children with cystic fibrosis have increased calorie and protein needs. The exact amount depends on the severity of their lung disease. Those with pancreatic insuf­ ficiency need replacement enzyme therapy in order to absorb fat and fat-soluble vitamins. Fat-soluble vitamins (A, D, E, K) are supplemented in addition to pancreatic enzymes. Excess sodium is lost in the sweat of children with cystic fibrosis. This patient is at risk for hyponatremia if he is sweating significantly. If he has cystic fibrosis-related diabetes, he may develop hyper­ glycemia. Surgical feeding tubes may be needed for children unable to gain weight or who have persistent malnutrition.

A 2-year-old girl is admitted for routine chemotherapy. She has neuroblastoma. Following procedures, you order a nutri­ tion consult. Question 1 7-1 What should you list as the reason for your consult? A) The child does not like the patient cafeteria menu. She needs food vouchers for the visitor cafeteria. B) All children on chemotherapy require parenteral nutrition, and you need help ordering it. C) The child needs iron supplementation, but you are not sure if it should be given via enteral or parenteral route. D) The child needs to have her growth measurements plotted. E) The child is receiving high-dose chemotherapy and is at high risk for developing malnutrition. You would like to know her DRI of calories and protein.

MCG RAW-H I LL E D U CATION S P E C I A LTY BOA R D REVI EW: P E D I ATRICS

556

Discussion 1 7-1 The correct answer is "E:' The most common characteristic of cancer-associated malnutrition is anorexia. Energy needs are increased due to the metabolic demand of the cancer. Treatment may cause symptoms of anorexia, nausea, vomiting, constipation, diarrhea, altered taste, and mouth sores. Digestion and absorp­ tion may be impaired. Children may be unable to eat due to scheduled procedures and surgeries. All contribute to malnutri­ tion in cancer patients. Children receiving high -dose or comb ina­ tion chemotherapy are at high risk for malnutrition and may need nutritional support. If oral intake is inadequate, enteral tube feed­ ing or parenteral nutrition should be started. Tube feedings are the first choice. As the patient's intake improves, it is important to avoid excessive weight gain. If she is receiving frequent blood transfusions, she does not need additional iron.

A I S-year-old boy was involved in a rollover car crash. The car ignited and he was badly burned. He is being admitted to the burn unit in critical condition. Question 1 8-1 What nutritional derangements do you anticipate? A) Lipid catabolism. B) Protein catabolism. C) Insensible fluid losses. D) Electrolyte disturbances. E) All of the above. Discussion 1 8-1 The correct answer is "E:' Nutritional support is important to prevent malnutrition, help heal wounds, and handle the increased energy expenditure due to the hypermetabolic state. Burns cause a hypermetabolic response characterized by increased protein catabolism, lipid catabolism, and oxygen consumption with loss of muscle mass. Burn victims may lose huge amounts of fluid from evaporative losses. Enteral support is best, preferred, and maintains the gut integrity. Enteral tube feedings may be needed. Vitamin A and C supplementation help with wound healing.





Helpful Tip

The most i m porta nt n utritional i ntervention i n a n i l l

1 1 1r c h i l d i s adequate protein i nta ke. Protein needs may be 50% g reater than base l i n e d u ri n g acute i l l ness or

stress.

A 5-year-old child with congenital heart disease and chronic heart failure presents with acute worsening of his heart fail­ ure. He is on a fluid-restricted diet and takes diuretics daily.

His mother struggles to keep him out of the refrigerator. He is constantly trying to sneak drinks. He went to a birthday party earlier in the day. When no one was watching, he drank 10 juice boxes. He is admitted and placed back on strict fluid restriction. Question 1 9-1 What are common nutritional issues encountered in children with congenital heart disease? A) Total daily caloric needs are increased in the setting of heart failure. B) Formula may be fortified to meet calorie needs in an infant on a fluid-restricted diet. C) Potassium supplementation may be required. D) Fluid restriction can be challenging for children. E) All of the above. Discussion 1 9-1 The correct answer is "E:' Congenital heart disease is frequently associated with chronic heart failure and increased calorie needs. In young infants, fluid intake is typically not restricted and diuretics are used to maintain a euvolemic state. Fortifica­ tion of formula is frequently still needed to meet caloric needs. Children receiving diuretics that cause renal potassium wasting may need oral potassium supplementation. As evident by our young friend, not being able to drink when you want is chal­ lenging and may be difficult for a child to understand.

A 1 7-year-old girl comes to the adolescent clinic to discuss birth control. To be thorough, you ask about her diet and whether she has any weight concerns. Six months ago she watched a documentary on slaughterhouses. From that point on she has been following a strict vegan diet. Question 20-1 Which nutritional deficiency are you least worried about in this girl? A) Iron deficiency. B) Vitamin B 1 deficiency. 2 C) Protein deficiency. D) Vitamin D deficiency. E) Calcium deficiency. Discussion 20-1 The correct answer is "C:' Vegetarians come in every flavor of the rainbow. It is important to ask specifically what the per­ son will not eat: red meat, fish, fowl, eggs, dairy products, or any combination of these; what he or she is eating (alternative protein sources); and what supplements (vitamin and miner­ als) he or she is taking. Vegetarians should find alternative pro­ tein sources such as legumes, tree nuts, or tofu. Calcium- and vitamin D-fortified foods can ensure adequate intake in some­ one who "doesn't do dairY:' Vegans run the risk of becoming

C H APTER 24

deficient in vitamin B 1 2 . Regular consumption of vitamin B 1 2 fortified foods or allowing dairy products in the diet should be encouraged for both vegetarians and vegans. The RDI of iron is greater for herbivores than for carnivores because the iron in a vegetarian diet has lower bioavailability.



� I

Helpful Tip

I ro n d eficiency is the most common micron utrient

l lr d eficiency in c h i l d ren a n d i nfa nts who eat a vegeta ria n d iet.

� QUICK QUIZ Which o f the following statements regarding nutritional needs in neurologically impaired children is true? A) Children with neurologic disorders always have increased caloric needs. B) All children with neurologic disorders require enteral tube feedings. C) Vitamin D deficiency is not an adverse effect of some anti­ seizure medications. D) Obesity does not occur in children with neurologic impairment. E) Oral-motor dysfunction contributes to malnutrition in chil­ dren with neurologic impairments. Discussion The correct answer is "E:' Children with neurologic impair­ ment have special nutritional needs. If swallowing or sucking is impaired or the child cannot meet nutritional goals, enteral tube feedings typically through a surgical feeding tube are necessary. Calorie needs vary greatly. Children with spasticity (cerebral palsy) have increased energy requirements and have decreased feeding efficiency owing to impaired oral-motor skills (eg, it takes longer to swallow and chew) . Those with neuromuscu­ lar disorders such as spinal muscular atrophy have lower daily caloric needs. Children with a low resting energy expenditure rate may become obese despite eating very little. Vitamin D defi­ ciency is common with certain antiseizure medications. A keto­ genic diet may be prescribed for those with intractable seizures. Adverse effects of the diet include hypoglycemia, dehydration, vomiting, loss of appetite, constipation, hypertriglyceridemia, kidney stones, and pancreatitis. Osteoporosis is very common in this population and pathologic fractures may occur. Factors contributing to poor bone health include lack of sun exposure, antiseizure medications, lack of weight-bearing activity, and inadequate intake of calcium, phosphorus, and vitamin D.

A 5-month-old male infant is brought into clinic because his father has questions about starting solid foods. The infant is exclusively breastfed. His sibling has a peanut allergy, the



N UTRITION

557

father has hay fever, and the mother has asthma. To mini­ mize allergen exposure from her breastmilk, the mother has eliminated dairy, eggs, wheat, soy, peanuts, shellfish, and nuts from her diet. The mother does not want the child to start solid foods until he is at least a year old, at which time he will follow a soy-, dairy-, egg-, peanut-, and nut-free diet until age 5. The father does not agree. He wants your opinion as a medical professional. Question 21 -1 What do you tell him? A) The infant's risk of food allergy is increased as his sibling has a peanut allergy. B) Breastmilk is more likely than formula to trigger an allergic reaction. C) A restrictive maternal diet during pregnancy and while breast­ feeding protects against the development of food allergies D) The safest time to introduce solid foods is at 12 months of age. E) Introducing more than one food at a time is recommended. Discussion 21 -1 The correct answer is "A:' Cow's milk, soy, egg, fish, shellfish, peanuts, and tree nuts are common causes of food allergies. An infant is at risk if his or her mother, father, or sibling has asthma, eczema, allergic rhinitis, or food allergy. Though it sounds like a good idea, restricting a mother's diet has not been shown to be protective. (Hooray for mom; she can eat again ! ) Breast milk is the best option in this case as it builds the immune system and is least allergenic. Exclusive breastfeeding for 4 to 6 months is recommended. When the infant is 4 to 6 months old, single ingredient foods should be introduced one at a time. This approach helps to identify whether or not a food caused a reaction. If more than one food is introduced concurrently and the infant has a reaction, it will be difficult to know which food caused the reaction. Feeding the infant solids should not be delayed beyond 6 months. Families do not need to wait to introduce certain foods. Delayed introduction of potential allergenic foods has not been clearly shown to be beneficial, even in at-risk infants.

A 5-year-old girl has chronic liver cholestasis with diarrhea and poor weight gain. She is receiving medium-chain triglyc­ eride (MCT) oil supplementation. You order laboratory tests to assess her nutritional status. Question 22-1 Which is NOT associated with cholestatic liver disease? A) Coagulopathy. B) Hypercalcemia. C) Vitamin A deficiency. D) Hyperparathyroidism. E) Vitamin D deficiency.

MCG RAW-H I LL E D U CATION S P E C I A LTY BOA R D REVI EW: P E D I ATRICS

558

Discussion 22-1 The correct answer is "B:' Children with impaired bile acid secre­ tion have impaired absorption of fat and fat -soluble vitamins. Therefore, vitamins A, D, E, and K should be supplemented. Intravenous or intramuscular vitamin K may be needed. A low­ fat diet supplemented with MCT oil can decrease steatorrhea as MCT does not require bile salts for intestinal absorption and boosts calorie intake. Children with chronic liver disease may develop hepatic osteodystrophy (a form of rickets) . Vitamin D deficiency and hypocalcemia cause secondary hyperparathy­ roidism, resulting in bone reabsorption. Vitamin D deficiency is only partially to blame as osteopenia may persist even after vitamin D stores have been restored.

A 1 6-year-old boy is the starting center on his football team. He comes to the office for follow-up care of his acne. You ask about this year's team and how the season is going. During the conversation, he mentions that his coach is encouraging him and his teammates to take protein supplements.

Question 23-2

How do you counsel him about energy and sports drinks? A) They may contain caffeine. B) They increase the risk of dehydration. C) They may cause tremors. D) They will help you focus but only for a short time. E) All of the above. Discussion 23-2 The correct answer is "E:' Energy drinks, including those with caffeine, ginseng, and other stimulants, will boost an athlete's performance but only for a short time. An athlete who drinks energy drinks is at increased risk for dehydration and heat­ related illness. Muscle tremors and incoordination may occur, so our young athlete will need to hang on to the ball.





Helpful Tip

A fa m i ly's culture i nfl uences feed i n g practices, foods

I 1 1 r a n d beverages consumed, a n d how the food is prepa red or served. A provider m u st consider cultura l beliefs a n d practices w h e n m a k i n g n utrition recommendations.

Question 23-1

How do you respond? A) Adding nonfat powdered milk to foods is an alternative to commercial protein powders. B) Amino acid supplementation in a healthy athlete is not necessary. C) "Weight gainers" may result in excessive fat gain. D) Dietary supplements may vary in quality. E) All of the above. Discussion 23-1 The correct answer is "E:' A well-balanced diet is the key to ath­ letic success. Diet supplements are not regulated like prescription drugs. Product quality and purity vary even from reliable sources. A healthy athlete who is eating a well-balanced diet does not need protein or amino acid supplements. These supplements have not been shown to significantly improve performance in younger athletes and may decrease performance in endurance sports. "Weight gainers" are high-calorie protein supplements with up to 2000 kilocalories (kcal) per serving. If used as recommended, they add bulk but it will be fat not muscle. If an athlete cannot meet his or her protein needs, adding powdered milk to foods is a reason­ able and cheap alternative to over-the-counter supplements.





Helpful Tip

For athletes looking to i ncrease muscle mass, resista nce

1 1 1r tra i n i n g i s key a n d should be done o n nonconsecutive days (per i ndivi d u a l muscle g ro u p) .

You ask about other dietary supplements he is taking. He found a new energy drink called "Sleep No More?' After drinking it, he is really able to focus.

A 1 0-week-old male infant is admitted to the hospital with failure to thrive. He is only 30 g above his birthweight. He has profuse, watery, yellow-colored diarrhea. The family cannot differentiate between stool and water. He is on standard cow's milk formula and eats well, taking in 1 1 0 kcal/kg body weight per day. His local physician has tried fortifying his formula to 27 kcal per ounce, but the infant has not gained weight. He does not sweat with feedings, vomit, or spit-up. His suck and swallow is normal, and it takes 10 minutes to eat a 4-ounce bottle. He is admitted for nutritional support and diagnos­ tic evaluation. You are concerned he has cow's milk protein intolerance. Question 24-1 What type of formula should you order for your patient? A) Intact cow's milk protein formula. B) Soy formula. C) Extensively hydrolyzed protein formula. D) Amino acid-based formula. E) Both C and D are options. Discussion 24-1 The correct answer is "E:' This child has malabsorptive diarrhea due to severe cow's milk protein-induced enteritis. Extensively hydrolyzed protein formula contains peptides and amino acids. It is first-line treatment for infants with cow's milk or soy protein intolerance and may be used for other causes of malabsorption (prolonged diarrhea, short gut syndrome) . If the infant does not improve on a hydrolyzed formula or has severe protein hyper­ sensitivity, an amino acid formula is indicated. Both hydrolyzed

C H APTER 24

and amino acid formulas taste bad and are expensive. Infants are more likely to drink these formulas if introduced early in life before their sense of taste is well developed. The indications for soy formula include galactosemia, parental choice for a vegetar­ ian diet, and lactose intolerance. Infants with severe failure to thrive often require formula fortification to increase the number ofkilocalories per ounce. Watch out-concentrated feedings may cause osmotic diarrhea.

You decide to start the infant on an amino acid formula owing to the severity of his symptoms. He does not like the taste and is not meeting his nutritional goals. On a brighter note, his diarrhea has stopped after changing formulas. Question 24-2 What should you do next to nourish this infant? A) Add cherry syrup to the formula to improve its palatability. B) Fortify his formula to 27 kcal per ounce so his total intake goal is less. C) Switch to intact cow's milk protein formula as you know he will take it. D) Place an enteral feeding tube and provide supplemental enteral feedings. E) Place an indwelling venous catheter and start parenteral nutrition. Discussion 24-2 The correct answer is "D:' Infants and children who are unable to meet their caloric intake needs require supplemental nutri­ tion. Enteral feedings through a nasogastric or transpyloric feeding tube are preferred over parenteral nutrition. Enteral feedings stimulate the gut and prevent gut atrophy. Parenteral nutrition is associated with iatrogenic electrolyte imbalances, hypo- and hyperglycemia, complications from an indwelling venous catheter, cholestasis, and liver damage. Situations in which a feeding tube may be needed include prematurity, acute respiratory illness, burns, aspiration, and chronic medical con­ ditions associated with anorexia or increased metabolic needs. If enteral feedings are not tolerated (severe gastroesophageal reflux, delayed gastric emptying, pancreatitis, malabsorption, prolonged diarrhea) or contraindicated (necrotizing enterocoli­ tis, intestinal obstruction or perforation, anatomic abnormality, extreme prematurity) , parenteral nutrition is indicated. Paren­ teral nutrition delivered through a peripheral venous catheter has a dextrose concentration limit of 12.5% or less and it cannot contain calcium. This avoids sclerosis and inflammation of the vein. Electrolytes should be monitored closely when a patient is receiving parenteral nutrition.

You have decided on a formula and enteral supplemental feedings. Question 24-3 Now which feeding tube should you choose for this infant? A) Nasojejunal feeding tube. B) Nasogastric feeding tube. C) Orogastric feeding tube.



N UTRITION

559

D) Gastrotomy tube. E) Jejunostomy tube. Discussion 24-3 The correct answer is "B:' An enteral feeding tube inserted through the mouth or nose is not indicated for long-term use. If long-term use is needed, a surgical percutaneous enteros­ tomy tube should be placed. Nasogastric tubes are easy to insert and are the correct choice for most patients. A trans­ pyloric tube (nasoj ejunal or j ejunostomy) may be indicated in cases of severe gastroesophageal reflux or delayed gastric emptying, aspiration, intolerance of gastric feeding, or supe­ rior mesenteric artery syndrome, and in patients requiring artificial ventilatory support. Question 24-4 What is NOT a complication of enteral tube feeding? A) Tube obstruction. B) Tube dislodgement. C) Discomfort with placement. D) Nasal septum necrosis. E) All of the above. Discussion 24-4 The correct answer is "E:' Additional complications of nasa­ gastric or transpyloric tubes include misplacement, chronic irritation of the gastrointestinal wall from suctioning, and per­ foration (trachea, lung, gastrointestinal tract) . Surgical tubes are complicated by bleeding, infection, intestinal perforation, tube misplacement, tube problem (clogged, broken, migration) , and leakage around the tube.

You decide to place a nasogastric tube. The infant will be allowed to feed orally with the remaining formula given via the feeding tube. The nurse asks whether you would like the feedings given as bolus or continuously. Question 24-5 Which of the following statements is/are true? A) Continuous feedings are more likely to cause reflux than bolus feedings. B) Transpyloric feedings may be given as boluses. C) Bolus feedings are more physiologic. D) There is no indication for continuous feedings. E) All of the above. Discussion 24-5 The correct answer is "C:' Bolus feedings simulate oral feed­ ings-intermittent and given for a similar time period. Mini­ mal supplies are needed (eg, no feeding pump) . Bolus feedings are typically used during the day but not at night as the latter are more likely to cause gastroesophageal reflux. Continuous feedings should be used for overnight feedings or intolerance to bolus feedings. Bolus feedings cannot be given through trans­ pyloric feeding tube.

MCG RAW-H I LL E D U CATION S P E C I A LTY BOA R D REVI EW: P E D I ATRICS

560

� QUICK QUIZ What electrolyte abnormality i s associated with refeeding syndrome? A) Hypomagnesemia. B) Hypophosphatemia. C) Hypokalemia. D) All of the above. E) None of the above. Discussion The correct answer is "D:' Hypophosphatemia is most common.





Helpful Tip

A c h i l d or i nfa nt with m a l n utrition s h o u l d be moni­

l l l r tored for refeed i n g syn d rome a n d have feedings rein­ troduced slowly.

B I B LIOGRAPHY

American Academy of Allergy Asthma and Immunology. Prevention of allergies and asthma in children: Tips to remember. http:/ /www. aaaai.org/ conditions-and­ treatments/libraryI at -a-glance/prevention -of-allergies­ and-asthma-in-children.aspx. Accessed June 2, 20 1 5 . American Academy o f Pediatrics. Nutrition and supplement use. http:/ /www.healthychildren.org/English/healthy­ living/nutrition/Pages/Nutrition -and -Supplement-Use. aspx. Accessed June 3, 20 1 5. American Academy of Pediatrics. Performance-enhancing substances. http:/ /www.healthychildren.org/English/ healthy-living/ sports/Pages/Performance-Enhancing­ Substances.aspx. Accessed June 3, 20 1 5. Baker RD, Greer FR, Committee on Nutrition. Diagnosis and prevention of iron deficiency and iron-deficiency anemia in infants and young children (0-3 years of age) . Pediatrics. 20 1 0 ; 1 26(5) : 1 040- 1 050. doi: 1 0 . 1 542/ peds.20 1 0-2576. Bhatia J, Greer F, Committee on Nutrition. Use of soy protein-based formulas in infant feeding. Pediatrics. 2008; 1 2 1 (5): 1 062- 1 068. doi: 1 0 . 1 542/peds.2008-0564. Braamskamp MJ, Dolman KM, Tabbers MM. Clinical practice. Protein-losing enteropathy in children. Eur J Pediatr. 20 1 0 ; 1 69( 1 0) : 1 1 79- 1 1 85. doi: 1 0 . 1 007/ s0043 1 -0 1 0- 1 235-2. Golding J, Paterson M, Kinlen LJ. Factors associated with childhood cancer in a national cohort study. Br J Cancer. 1 990;62(2):304-308. Holick MF, Binkley NC, Bischoff-Ferrari HA, et al. Evaluation, treatment, and prevention of vitamin D deficiency: An Endocrine Society clinical practice guideline. J Clin Endocrinol Metab. 20 1 1 ;96(7) : 1 9 1 1 - 1 930. doi: 10. 1 2 1 0/ jc.20 1 1 -0385.

King D, King A. Question 2: Should children who have a febrile seizure be screened for iron deficiency? Arch of Dis Child. 2 0 1 4;99 ( 1 0) : 960-964. doi: 1 0. 1 1 36/ archdischild-2014-306689. Kleinman RE, Committee on Nutrition, eds. Pediatric Nutrition Handbook. 6th ed. Elk Grove Village, IL: American Academy of Pediatrics; 2009. Kliegman RM, Stanton BF, St Geme JW, Schor NF, Behrman FR, eds. Nelson Textbook of Pediatrics. 1 9th ed. Philadelphia, PA: Saunders, Elsevier; 20 1 1 . Koletzko S , Niggemann B , Arato A, et al. Diagnostic approach and management of cow's-milk protein allergy in infants and children: ESPGHAN GI Committee practical guide­ lines. J Pediatr Gastroenterol Nutr. 20 1 2;55(2) :22 1 -229. doi: 10. 1 097/MPG.Ob0 1 3 e3 1 825c9482. Mahant S, Cohen E, Cunningham S, Fine BR. Feeding tubes and enteral nutrition. In: Rausch DA, Gershel JC, eds.

Caringfor the Hospitalized Child: A Handbook of Inpatient Pediatrics. Elk Grove Village, IL: American Academy of Pediatrics; 20 1 3 :493-499. Misra M, Pacaud D, Petryk A, Collett -Solberg PF, Kappy M, Drug and Therapeutics Committee of the Lawson Wilkins Pediatric Endocrine Society. Vitamin D defi­ ciency in children and its management: Review of current knowledge and recommendations. Pediatrics. 2008; 1 22(2) :398-4 1 7. doi: 1 0 . 1 542/peds.2007 - 1 894. Nield LS, Mahaj an P, Joshi A, Kamat D. Rickets: Not a disease of the past. Am Pam Physician. 2006;74( 4) : 6 1 9-626. Oski FA. Iron deficiency in infancy and childhood. N Engl J Med. 1 993;329(3 ) : 1 90- 1 93 . doi: l 0 . 1 056/ NEJM 1 993071 53290308. Prince A, Groh-Wargo S. Nutrition management for the promotion of growth in very low birth weight prema­ ture infants. Nutr Clin Pract. 20 1 3 ;28(6) :659-668. doi: 1 0. 1 1 77/08845336 1 3 506752. Sahay M, Sahay R. Renal rickets-practical approach. Indian J Endocrinol Metab. 20 1 3 ; 1 7(suppl l ) : S35-44. doi: 10.41 03/2230-82 1 0. 1 1 9503. Schanler RJ, Shulman RJ, Lau C. Feeding strategies for premature infants: Beneficial outcomes of feeding fortified human milk versus preterm formula. Pediatrics. 1 999; 1 03(6 Pt 1 ) : 1 1 50- 1 1 57. Section on Breastfeeding. Breastfeeding and the use of human milk. Pediatrics. 20 1 2 ; 1 29(3):e827-84 1 . doi: 1 0 . 1 542/ peds.20 1 1 -3552. Yan AC, Jen MV. Skin signs of pediatric nutritional disorders. Curr Probl Pediatr Adolesc Health Care. 20 1 2;42(8):2 12-2 1 7. doi: 1 0 . 1 0 1 6/j.cppeds.20 1 2 .02.003. Zehetner AA, Orr N, Buckmaster A, Williams K, Wheeler DM. Iron supplementation for breath-holding attacks in children. Cochrane Database Syst Rev. 20 1 0;(5) :CD008 1 32. doi: 1 0 . 1 002/ 1 465 1 858.CD008 1 32. pub2. Zimmermann MB, Hurrell RF. Nutritional iron deficiency. Lancet. 2007;370(9586) : 5 1 1 -520.

Patie nt Safety a n d Qua l ity I m prove m e nt

25

E l iza beth H . M a c k

A child is undergoing sedation for a lumbar puncture. Dur­ ing the preprocedure time out, it is determined that she has diabetes. The child's mother indicates that her daughter did not have her glucose checked this morning despite taking long-acting insulin last night and not eating or drinking this morning. The mother's concern prompts you to check the child's blood glucose level, which is low. She is given dextrose­ containing fluids prior to sedation and no harm is done. Question 1 -1 This event can be described as a(n) : A) Near-miss event. B) Adverse reaction. C) Sentinel event. D) Adverse event. E) Medication error. Discussion 1 -1 The correct answer is "A:' The harm did not reach the patient, and therefore this type of error is identified as a near miss. Inves­ tigating the root causes of such events, and the safeguards that prevented them from reaching the patient, are just as important as investigating those errors that reach the patient. Question 1 -2

According to the Institute of Medicine's landmark report To Err Is Human, preventable medical errors were estimated to cause how many deaths in hospitalized patients annually in the United States? A) 400-980. B) 4000-9800. C) 44,000-98,000. D) 444,000-980,000. E) 4,400,000-9,800,000.

Discussion 1 -2 The correct answer is "C:' To Err Is Human estimated that 44,000 to 98,000 inpatients died annually due to preventable medical error. This statistic is quoted regularly throughout the medi­ cal and lay literature. Although the validity of these estimates has been questioned, this report served as a "call to action" for health care providers nationwide. Question 1 -3

To Err Is Human estimated the cost associated with prevent­ able adverse events in U.S. inpatients annually to be: A) $17,000. B) $17 million. C) $17 billion. D) $17 trillion. E) Unmeasurable. Discussion 1 -3 The correct answer is "C:' To Err Is Human estimated that pre­ ventable medical errors in the United States cost $17 billion annually. Although the validity of this figure has been ques­ tioned, this report served as a "call to action" for health care providers nationwide.

A 3-year-old boy is prescribed ampicillin for his pneumonia despite his severe penicillin allergy. Question 2-1 This event is classified as a(n) : A) Near-miss event. B) Preventable adverse event. C) Nonpreventable adverse event. D) Lapse. E) Latent error.

561

562

MCG RAW-H I LL E D U CATION S P E C I A LTY BOA R D REVI EW: P E D I ATRICS

Discussion 2-1 The correct answer is "B:' Whether or not this allergy was documented, the issue reached the patient and was prevent­ able. A near miss represents an error that does not reach the patient. A latent error, often created by employees as a work­ around, is typically a system-based problem relating to poor design or workflow that occurred days, weeks, or months before the event. Alternatively, a latent error may not yet be recognized, and thus is often more difficult to identify. A lapse represents memory failure; for example, a nurse forgets to fol­ low a step in the medication administration policy, resulting in an intravenous medication not being undamped and there­ fore delayed. Question 2-2 Suppose the 3-year-old boy had no known drug allergies but developed hives after receiving ampicillin. This event would then be classified as a(n) : A) Nonpreventable adverse event. B) Preventable adverse event. C) Blunt error. D) Lapse. E) Latent safety threat. Discussion 2-2 The correct answer is "A:' Though the adverse drug event did cause harm, presumably this reaction could not have been anticipated as the child had no prior history of drug allergy or predisposing condition. Blunt error is another term for latent error. Question 2-3

Children are at particularly high risk for adverse drug events because of: A) Overbearing parents, especially during medication administration. B) Weight-based dosing and lack of published dosing recommendations. C) Renal and hepatic metabolism that is not as mature as in the elderly. D) The wide variety of child-friendly formulations developed by drug companies. E) Lack of clinical pharmacists who are trained in pediatrics. Discussion 2-3 The correct answer is "B:' Children are at higher risk for adverse drug events because ( 1 ) dosing is often weight based, and prac­ titioners deal with a large range of weights in the pediatric pop­ ulation; (2) doses may vary by indication; (3) there is a lack of available dosage formulations; ( 4) there is a lack of concentra­ tions appropriate for pediatric patients, requiring compounding and other manipulation; and (5) young or delayed children are often unable to communicate.

A 1 3-year-old girl is admitted to the pediatric intensive care unit (PICU) . At the same time, a 9-month-old intubated infant develops a pneumothorax. A critical laboratory value for the 1 3-year-old was not noted until 3 hours after admission. Question 3-1

PICUs are known to have higher error rates than other loca­ tions in children's hospitals because: A) They are typically located farthest from emergency resources such as code carts and code teams. B) Fatigue, distraction, and stress are major issues in the context of a huge number of tasks to be performed daily. C) Violent threats are frequent due to stressed family members. D) PICUs are more likely to be staffed by residents and fellows. E) PICUs are more likely to be staffed by inexperienced nurses and respiratory therapists. Discussion 3-1 The correct answer is "B:' Human factors such as distraction, fatigue, and stress are dangerous in the context of the number of tasks that must be accomplished for a critically ill patient.

A new intravenous (IV) pump is introduced on the unit in your children's hospital. A nurse who was out on maternity leave during the in-service education for the new pump is midway through hanging a new IV fluid bag when her other patient's father asks if she can stop his daughter's feeding pump from beeping. When she returns to finish hanging the IV fluids, she discovers the entire 1 liter bag of D 5 0.45% sodium chloride has infused in 15 minutes. Question 4-1 Through the root cause analysis, the following should be determined: A) The nurse has a high degree of culpability in this case. B) The charge nurse is responsible for the error due to inad­ equate staffing. C) The nurse's lack of education is responsible for this error. D) The other patient's father is responsible for distracting her and causing this error. E) This error reached the patient due to many system-level issues, not one individual. Discussion 4- 1 The correct answer is "E:' This error represents the "Swiss cheese model;' as described by James Reason. As with most errors, no one person is to blame. This error resulted from a distraction in addition to a system failure to educate all staff who would be working with the new pumps. Likely many other contributing factors would be identified through the root cause analysis. See Figure 25-6 later in this chapter.

C H A PT E R 25

Question 4-2 A hospital-wide initiative is under way to improve quality of care and communication among staff members and reduce the number of sentinel events. Evidence has demonstrated that one way to facilitate good communication in the inpatient setting among care teams is to: A) Require detailed computerized progress notes by all mem­ bers of the care team. B) Implement rounds led by the patient, family, or both. C) Ensure that the paper chart with copies of the progress notes accompanies the patient on all transports. D) Implement multidisciplinary family-centered rounds, including a discussion of daily goals. E) Enter duplicate paper and computerized orders to ensure the plan is followed. Discussion 4-2 The correct answer is "D:' Multidisciplinary rounds that include a discussion of daily goals have been shown to be an effective way of communicating and achieving improved patient out­ comes. Communicating in the chart by means of progress notes is clearly not ideal.



Helpful Tip

:5.� Get out from beh i n d the computer. Patient outcomes

r1 1r a re

i m p roved b y fa m i ly-centered m u ltidisci p l i n a ry

rou n d s, n ot deta i led computerized progress notes.

Question 4-3 The 2006 Joint Commission National Patient Safety Goals addressed the fact that this factor contributed to the majority of sentinel events in the hospital: A) Incomplete documentation. B) Poor communication. C) Lack of proper education for staff. D) Patient and family dissatisfaction. E) Inadequate staffing.



PATI ENT S A FETY A N D QUALITY I M P ROVE M E N T

563

Discussion 4-4 The correct answer is "C:' Event reporting is a key aspect of patient safety culture as measured by the Agency for Healthcare Research and Quality's culture of safety survey. In the context of a strong safety culture, reporting is easy to do, free of retaliation, and results in meaningful change when opportunities are identified.



Helpful Tip

:5.� Medical errors a n d sentinel events most often are the

r1 1 r result of poor com m u n ication. Ta l k with other staff i n person rather than t h ro u g h progress notes.

Question 4-5 The model for improvement is a powerful tool for driving quality improvement utilizing cycles involving: A) Perform, Declare, Synchronize, Assign. B) Prioritize, Decide, Sequence, Abandon. C) Prepare, Delve, Sample, Actualize. D) Publish, Demand, Structure, Allocate. E) Plan, Do, Study, Act. Discussion 4-5 The correct answer is "E:' The model for improvement describes a methodology for driving quality improvement. The goal is identi­ fied, a measurable aim is stated, the key drivers are developed, and then the Plan, Do, Study, Act cycle is repeated until positive change is sustained (often indefinitely) . (See Figure 25- 1 .)

A 1 0-year-old boy with osteosarcoma of the left lower leg undergoes amputation of the right lower leg. After disclosing the error to the family, the boy still must undergo amputa­ tion of the left lower leg.

Discussion 4-3 The correct answer is "B". Poor communication has been shown to be the most common contributor to medical errors and sen­ tinel events.

Question 5-1 This situation is best classified as a: A) Nonpreventable adverse event. B) Slip. C) Near-miss event. D) Lapse. E) Sentinel event.

Question 4-4 A safety culture where voluntary reporting of events effec­ tively occurs will involve: A) Employees scheduling appointments with the chief quality officer to report errors. B) Assigning culpability and punishing accountable employees. C) Changes resulting from reported events where opportuni­ ties are identified. D) Requiring individuals to give their full name when reporting. E) Hospital attorneys meeting with "whistleblowers:'

Discussion 5-1 The correct answer is "E:' A sentinel event is an unexpected occur­ rence involving actual or potential death or serious physical or psychological injury. A slip or lapse is an error due to an uncon­ scious lapse in automatic task performance or memory due to dis­ traction; lapses are failures to execute. A near miss is an error that does not result in an adverse event or a "close call:' A preventable adverse event occurs when harm caused by medical error reaches the patient; harm that reaches the patient in the absence of medi­ cal error is classified as a nonpreventable adverse event.

564





MCG RAW-H I LL E D U CATION S P E C I A LTY BOA R D REVI EW: P E D I ATRICS

What changes



Set objective

for next cycle?



Ask q uestions/make predictions

Can the •

change be implemented?

Plan to answer the questions

Act

Plan

(who/where/when) •

Collect data to answer q uestions



Complete the analysis of the data





Compare data

Study

Do

to predictions



Carry out the plan

Sum marize



Collect the data

what was



learned

D) Dosing will be automatically adjusted for organ dysfunction. E) Doses can be automatically held if certain laboratory param­ eters are met.

Begin analysis of the data

F I G U R E 2 5- 1 . The PDSA (Plan, Do, Study, Act) cycle is repeated u ntil change is successfu l i n q u a l ity i m p rovement projects. (Reproduced with permission from Wac hter RM, ed. Understanding Patient Safety. 2nd ed. New York, NY: McGraw- H i l l Education; 201 2, Fig. 3-3.)

A 7 year old male is admitted with hemolytic uremic syn­ drome (HUS) . He has not urinated in over 10 hours and his serum creatinine is 3.5 mg/dL. Ibuprofen is ordered to treat pain as needed despite his acute renal failure. Question 6-1

Which of the following has been shown to reduce inpatient pediatric medication errors? A) Hard stops for doses more than 5% outside the recom­ mended range. B) Involvement of clinical pharmacists trained in pediatric pharmacotherapy. C) Performance of medication reconciliation by nurses via ver­ bal order. D) Avoiding conversion of pediatric medication orders to com­ puterized provider order entry. E) Use of trigger tools to prompt the use of antidotes. Discussion 6-1 The correct answer is "B:' The involvement of clinical pharma­ cists in inpatient teams has been shown to reduce medication errors. Verbal orders are highly discouraged. Trigger tools are a mechanism for detecting potential errors; for example, order­ ing naloxone may indicate that a patient received too much narcotic. Hard stops are sparingly used, and pharmacists com­ monly round medication doses with a wide therapeutic window to reduce cost/waste and increase ease of administration. Question 6-2 Dose-range checking is extremely useful in preventing pedi­ atric medication errors primarily because: A) Most pediatric medication errors occur in the ordering phase. B) Transcription and legibility errors are no longer an issue. C) Indication or clinical condition will be forced to match the appropriate dose.

Discussion 6-2 The correct answer is "A:' Seventy-four percent of medication errors and 79% of potential adverse drug events stem from the ordering phase. For a given drug, there is often a wide dosing range due to a wide range of age, weight, and height among our pediatric population. Thus, clinical decision support termed "dose-range checking" recognizes the appropriate dose range for a particular patient's age and size and will prevent a variety of errors in the ordering phase. Question 6-3 Verbal ordering still occurs in the computerized order-entry era. Errors related to verbal orders can be mitigated by: A) Limiting use of verbal ordering to computer "downtime:' B) Transcribing these orders onto paper to avoid the pharmacy profiling process. C) Limiting use of verbal ordering to emergent situations with emphasis on repeat-back technique. D) Limiting use of verbal ordering by anesthesiologists in the operating room. E) Requiring nurses to double-check doses with a calculator. Discussion 6-3 The correct answer is "C:' Unfortunately computerized physi­ cian order entry (CPOE) does not prevent verbal ordering. Ver­ bal orders should be limited to situations in which the ordering provider cannot possibly enter the order, and the use of repeat­ back technique should be employed. During computer down­ time, there should be a paper order backup system in place that still involves pharmacy profiling orders.





Helpful Tip

Verbal o rd e rs a re a sou rce of potentia l medica l error. If

1 1 1r a verba l order is necessa ry, t h e person taking t h e order s h o u l d repeat back the order to verify it is correct.

Question 6-4 Medication reconciliation is defined as the process whereby: A) The computerized list of home medications in the electronic health record is continued on admission, transfer, and discharge. B) The reported home medication list is added to inpatient medications on admission to the hospital. C) Doses of home medications are checked using dosing calcu­ lators on admission to the hospital. D) An up-to-date medication list is created and compared to the patient's medication orders at times of transition such as admission, transfer, and discharge. E) A printed list of discharge medications is handed to the patient so he or she can resolve any discrepancies with the pharmacy.

C H A PT E R 25



PATI ENT S A FETY A N D QUALITY I M P ROVE M E N T

565

Discussion 6-4 The correct answer is "D." Medication reconciliation involves resolving discrepancies in the medication list at all points of transition as the patient moves through the health care system.

process with required co-signatures. The double-check requires verifying the "rights" against the order (medication, dose, patient, route, formulation, time, documentation) before administration and when the dose or medication/administra­ tion has changed.

Question 6-5

Question 6-7 Trigger tool methodology is used to: A) Automatically order reversal agents for potentially hazard­ ous medications (eg, narcotic-naloxone). B) Add corollary orders for medications with common side effects (eg, narcotic-stool softener) . C) Prompt providers to consider laboratory values when ordering relevant medications ( eg, creatinine-acyclovir) . D) Screen for potential drug-drug interactions (eg, warfarin-itraconazole) . E ) Detect potential adverse drug events by screening for anti­ dote administration (eg, Kayexalate-potassium) .

An effective strategy to prevent inpatient medication errors related to dispensing involves: A) Storing sound-alike, look-alike drugs in the same area and labeling them with tall-man lettering. B) Stocking a variety of concentrations, when available, for each medication. C) Placing the automated dispensing device in a central loca­ tion in the unit hallway. D) Having the pharmacy prepare and dispense as many medi­ cations as possible. E) Having nurses compound medications on the inpatient unit. Discussion 6-5 The correct answer is "D:' Having the pharmacy prepare and dispense as many medications as possible reduces the risk of medication errors. Exceptions include emergency medications available in the automated dispensing device. The dispensing area should be free of distraction. Stocking only one concentra­ tion of a medication is helpful in avoiding dispensing errors. In addition, sound-alike, look-alike drugs should be stored in separate areas and labeled with tall-man lettering.





Helpful Tip

Ta l l-man lettering uses lowercase a n d u ppercase letters

1 1 1r to h i g h l ig h t d ifferences a m ong d rugs with look-a l i ke names; for exa m p l e, DOPa m i n e a n d DOBUTa m i ne.

The Joint Commission requires each institution to create a list of high-alert medications that have a high risk of harm or death if administered improperly. Question 6-6

Most institutions require the following for high-alert medications: A) Administration via smart pump. B) Restricting administration to certain areas. C) Tall-man lettering. D) Administration by the attending physician. E) Independent double-check process. Discussion 6-6 The correct answer is "E:' Most institutions include medica­ tions such as chemotherapeutics, anticoagulants, insulin, and neuromuscular blocking agents on the high-alert medica­ tion list. Generally, the mechanism in place to handle these high-alert medications involves an independent double-check

Discussion 6-7 The correct answer is "E:' Trigger tool methodology is a quality improvement tool used to detect potential adverse drug events. For example, Kayexalate administration might indicate that an inappropriate amount of potassium was administered (or not discontinued) for the patient's clinical condition. Likewise, administration of naloxone might indicate that too much nar­ cotic was given or that the patient was not monitored appropri­ ately. Administration of these antidotes may also represent the patient's state on admission or an unpreventable adverse event, but it is worth doing a concise chart review to see if there is a lesson that can be learned. Question 6-8 The gold standard for IV medication delivery involves: A) Administration by the attending physician after double­ checking the "rights" with the nurse. B) Verification of the "rights;' barcode administration, and use of smart pump guardrails. C) Pharmacist and nurse double-check followed by IV push administration. D) Use of a basic infusion mode, rather than smart pump guardrails, for children. E) Avoidance of programming of the smart pump drug library for pediatric patients. Discussion 6-8 The correct answer is "B:' The rights should be verified when administering any medication (medication, dose, patient, route, formulation, time, documentation) . Barcode admin­ istration involves scanning the patient's identification (ID) bracelet and then verifying the order. Some institutions uti­ lize technology wherein the order will program the pump based on the child's weight and order. However, if a medi­ cation or patient is labeled improperly, these technologies are not foolproof. It would be safe to wager that most physi­ cians' working knowledge of an IV pump ends with the alarm silence button.

566

MCG RAW-H I LL E D U CATION S P E C I A LTY BOA R D REVI EW: P E D I ATRICS

Drivers to beco m i n g board certified Secondary d river

You are appointed to your hospital's quality and safety over­ sight team. The first project is aimed at improving the direct admission process. Question 7- 1 Which of the following describes the best approach for iden­ tifying ways in which a process or product can fail, and esti­ mating the associated risk of this failure? A) Statistical process control charts. B) Pareto charts. C) Key driver diagrams. D) Run charts. E) Failure mode and effects analysis. Discussion 7-1 The correct answer is "E:' Failure mode and effects analysis involves determining the impact of failure on the patient, the likelihood of failure, and the ability to detect failure. A Pareto chart combines a column and a line graph, relying on the princi­ ple that the vast majority of events are caused by a few things. A key driver diagram is structured logic chart with multiple levels depicted as connected boxes. It is a way to look at a problem by breaking it down into parts. A run chart is a line graph of data plotted over time that is used to look at trends-think of patient satisfaction data! (See Figures 25-2 through 25-4.) Question 7-2

Shewhart, or statistical process control, charts can indicate: A) Special cause variation by 6 steadily increasing points above a centerline. B) Common cause variation by 6 steadily decreasing points above a centerline. C) Random variation by 8 points above a centerline.

F I G U R E 25-3. Key Driver Diagra m . This is a structu red logic chart that

breaks a problem down i nto its pa rts.

D) Statistical control by 8 points below a centerline. E) Single point on the upper or lower control limit. Discussion 7-2 The correct answer is ''A:' Common cause, or random, varia­ tion is inherent to a system or process. All systems and pro­ cesses have common cause variation, and systems that only have common cause variation are in statistical control. Special cause variation indicates there is a new factor signal that is not always present in the process. Statistical process control, or Shewhart, charts can help an analyst distinguish between common and special cause variation. Upper and lower con­ trol limits generally represent three standard deviations above and below the centerline. Special cause variation is typically represented by a single point outside the upper or lower con­ trol limit, 6 steadily increasing points in a row, 2 of 3 points in the outer one- third approaching the upper or lower control limit, or 8 points in a row above or below the centerline. (See Figure 2 5 - 5 . )

Reason why you haven't not studied more 1 00% Happi ness & residency

90%

��------+ 80 o/o

Picu. rotation ends

70% 60%

40% 30% 20% 1 0%

L_���

_l__L O o/o

_L____L__i____�__L____L__�__

__

P rocrastination Lack of

N o time

Too smart

Plan to



1 00

50%

0 t5 � (/) (/) Q) c

·c..

50

a. ctl I

t

First day

wing it 0

G raduation

PL1

t

t

Winter push

Last i n patient rotation

P L2 --------+ PL3 Time

F I G U R E 25-2. A Pareto chart combi nes a col u m n and l i ne g ra p h . It is based

on the working theory that 80% of events a re attributa b l e to 20% of causes.

F I G U R E 25-4. Run Chart. This is a line g raph of data plotted over time that is used to look at trends.

C H A PT E R 25

1-----r---�-- U pper control l i m it Center l i n e Lower control l i m it



PATI ENT S A FETY A N D QUALITY I M P ROVE M E N T

567

record. Risk management personnel can often help rehearse or guide disclosure.





Helpful Tip

Disclos u re of med ica l errors does not result in more

1 1 1r lawsu its a n d may h e l p the fa m i ly fi n d closu re and reg a i n tru st.

F I G U R E 25-5. Statistical process control chart. This type of chart also known

as a Shewhart chart helps d i sti n g u i s h between common and special cause va riation.

During a code, a nurse inadvertently administers an improper dose of epinephrine drawn up by another staff member. Unfortunately the infant dies. The nurse administering the dose calls in sick to work for the next 3 days. Question 8- 1 The proper course of action by management includes: A) Approving a temporary leave of absence until the nurse has taken a medication safety course. B) Termination. C) Providing the nurse with counseling resources and institu­ tional support. D) Requiring the nurse to apologize to the family. E) Transferring the nurse to another unit within the institution. Discussion 8-1 The correct answer is "C:' "Second victim" syndrome has seri­ ous potential consequences, and employees should be sup­ ported when human errors occur. This error was clearly a result of multiple systems issues. Additionally, management of the error should not focus on the patient's outcome only, but rather the errors and decisions leading up to the final error. Clearly what happened to the patient and his or her family is awful, but it is important to make sure that such errors do not happen again. Question 8-2

Disclosure of the details leading up to and involving a medi­ cal error should: A) Consider the risk oflawsuit prior to meeting with the family. B) Only occur with the hospital attorney present. C) Not be acknowledged in the medical record. D) Only be made by the patient's primary care provider. E) Happen in a timely, thorough fashion with the assistance of risk management personnel. Discussion 8-2 The correct answer is "E:' Failure to communicate is often the trigger for lawsuits. Disclosure of an error does not increase the risk of lawsuits, and in fact often helps the family regain trust and come closer to closure. Disclosure should occur as soon as possible after an event and be documented in the medical

Question 8-3

Patients and their families who experience a medical error should be offered: A) Compensation for all expenses related to the diagnosis and encounter. B) Compensation for pain and suffering, but not for related medical bills. C) Information about how the institution plans to prevent sim­ ilar future events. D) Names and credentials of all parties involved in the error. E) An in-person apology by each of the people involved in the error. Discussion 8-3 The correct answer is "C:' Patients should be offered compen­ sation related to any consequences of the error, but not nec­ essarily the entire encounter or admission, including events preceding the error. Many patients and their families just want to know the results of an investigation or root cause analysis, and how the institution plans to avoid harming patients in a similar situation in the future.

The nurses on a pediatric ward note a significant amount of variation among the admitting providers caring for infants with bronchiolitis. Question 9- 1 The strategy most likely to improve outcomes and reduce variation would involve: A) Individual providers writing their own order sets based on their clinical experience. B) Respiratory therapists writing verbal orders for the treat­ ments that seem to improve each infant. C) Assigning certain nurses to certain providers' patients so that they learn their ordering patterns. D) A multidisciplinary group writing evidence-based order sets driving care of this population. E) Pharmacists making evidence-based recommendations in the chart for each patient. Discussion 9-1 The correct answer is "D:' Reduction in variation is a huge source of cost savings and improved care delivery. Creation of evidence-based order sets or pathways is an efficient way to drive reduction in variation in care. Bronchiolitis guidelines

568

MCG RAW-H I LL E D U CATION S P E C I A LTY BOA R D REVI EW: P E D I ATRICS

exist, but translating guidelines into practice often takes many years beyond publication of the guideline. Often different quality improvement methods are used together to bring about improvement. Question 9-2

One method you might use: A) Lean methods to reduce variation. B) The model for improvement to determine common cause variation. C) Six sigma to reduce waste. D) Statistical process control, or Shewhart, charts to determine an aim. E) Key driver diagrams to map out factors likely to affect the desired outcome. Discussion 9-2 The correct answer is "E:' Lean methods, based on the Japanese manufacturing industry and Toyota Production System, refers to a methodology that maximizes value and eliminates waste. The Langley, or Institute for Healthcare Improvement (IHI), model for improvement involves creating an aim statement, then creating, testing, and studying interventions focused on reaching the aim. Six sigma focuses on reducing variation and defects. Statistical process control, or Shewhart, charts are help­ ful in determining common versus special cause variation. Key driver diagrams facilitate communication of information within a team around factors affecting the desired improvement. Question 9-3

Quality improvements most likely to succeed include: A) Education provided to all staff within one discipline. B) Loosely defined aims. C) Ownership and buy-in from front-line providers rather than leadership. D) Limited and defined scope. E) Feedback and presentation of data at the conclusion of the project. Discussion 9-3 The correct answer is "D:' Common causes of quality improve­ ment project failure include a scope that is too large, lack of involvement of key stakeholders, lack of data feedback, loosely defined aims, and lack of leadership support. Question 9-4 When special cause variation is demonstrated within a statis­ tical process control chart: A) Common cause variation is no longer possible. B) The upper or lower control limit is proven inaccurate. C) The system is said to be in statistical control. D) The reason for the special cause can be easily identified. E) The centerline or mean is recalculated. Discussion 9-4 The correct answer is "E:' Statistical process control, or Shewhart, charts can help an analyst distinguish between common and

special cause variation. When special cause is demonstrated, the centerline shifts. The team can speculate as to the reason for the special cause, but within complex systems it may be dif­ ficult to isolate one single variable. Systems with only common cause variation are in control: most points are near average, a few points are near the control limits, and no points are beyond the control limits. Question 9-5 The model for improvement asks the following question: A) What is the reliability of this intervention? (aim) B) Is this intervention generalizable? (measure) C) How will we know if a change is an improvement? (interventions) D) How many patients should we affect? (scale) E) How often should we measure? (frequency) Discussion 9-5 The correct answer is "C:' The model for improvement asks three questions: ( 1 ) What are we trying to accomplish (aim statement) ? (2) How will we know if a change is an improve­ ment (measure)? and, (3) What changes can we make that will result in improvement (interventions)? Question 9-6

Since human fallibility cannot be avoided, in order to opti­ mize results we must: Human factors refers to the interaction between humans and other humans, products, equipment, procedures, and their environment. A) Redesign systems and other exogenous factors to prevent errors from occurring. B) Use disciplinary action to discourage human error. C) Focus on the human behavior component as equipment and technology are also fallible. D) Recognize that endogenous factors such as stress, fatigue, and sleep deprivation are unavoidable. E) Utilize education repeatedly to ensure human error is minimized. Discussion 9-6 The correct answer is "A:' Human error is unavoidable, but the aver­ age error rate is affected by endogenous factors that degrade human capability, such as sleep, stress, and fatigue as well as exogenous factors, such as the systems within which we operate. Redesigning systems to prevent the "Swiss cheese model" from materializing is the optimal approach. (See Figure 25-6.) Imagine a stack of slices of Swiss cheese. If the slices are stacked so that the holes line up an error can easily pass through a hole causing a mistake or accident. If the holes are staggered, an error may pass through one slice but will be caught by the next slice (layer of defense).

A new hospital is being planned to accommodate the needs of a growing community.

C H A PT E R 25



PATI ENT S A FETY A N D QUALITY I M P ROVE M E N T

569

medications or breastmilk. (Hospitals without all private rooms could use this to their advantage in a marketing campaign that touts: "Private hospital rooms compromise safe patient care. We care about our patients, so we double-bunk whenever possible:')

Hazard/e rror

Accident/ patient harm

F I G U R E 25-6. The "Swiss Cheese Model" of Organ izational Accidents. Most accidents occur when a n error i s not detected repeated ly beca use fa i l u res i n a system l i n e u p. To detect errors a n d prevent accidents, t h e s l i ces o f Swiss cheese m u st be stacked so that the holes don't overlap so that a n error that passes t h rough one defense strategy will be detected by the next.

Question 1 0-1 When building a new children's hospital, increased vertical­ ity of the building must be considered as it may have unin­ tended consequences such as: A) Faster code response. B) Slower pharmacy turnaround times. C) Closer-knit relationships between units and floors. D) Faster transit times from the emergency department. E) Faster transit times from the operating room.

Question 1 0-3 The Consumer Assessment of Healthcare Providers (CARPS) Child Hospital survey assesses which of the following in rela­ tion to other hospitals? A) Patient and family satisfaction and engagement. B) Quality metrics such as hospital-acquired infections. C) Compliance with standard process measures such as venous thromboembolism prophylaxis. D) Cost-effectiveness of care delivered. E) Expert opinion of quality of care delivered. Discussion 1 0-3 The correct answer is "A:' The CAHPS Child Hospital survey is the pediatric version of HCAHPS (Hospital Consumer Assess­ ment of Healthcare Providers and Systems), which assesses patient satisfaction in comparison to other similar institutions.

Recent concerns over unsafe sleep practices on the general pediatric inpatient unit have been brought to the medical director's attention. To improve patient safety, a unit wide educational campaign regarding safe sleep is being planned. Question 1 1 -1

Discussion 1 0-1 The correct answer is "B:' Increasing verticality can delay phar­ macy turnaround times, code response times, and delivery times for specimens or medications that cannot be delivered by tube sys­ tems. Verticality may also increase transit times to and from the operating rooms or emergency department. Careful consideration must be given to location of critical services such as pharmacy, and satellite locations may be considered within or near critical units.

One might hear the following statement from a staff member in a unit with the most mature learning culture: A) "Let's do it the way we've always done it; change is too risky:' B) "After that last event happened, we should fix the system so that it doesn't happen again:' C) "If we follow all the rules, we'll avoid causing any harm:' D) "Let's fix this as I think it could cause harm in the future:' E) "Here, nothing is more important than patient and staff safety:'

Question 1 0-2

In the process of building a new neonatal intensive care unit, providers and architects decide to transition to all private rooms. This decision is most likely to have a negative impact on compliance with: A) Noise regulations. B) Barcode medication administration. C) Hand hygiene. D) Ventilator associated pneumonia bundles. E) Double-checking breastmilk and high-alert medications. Discussion 1 0-2 The correct answer is "E:' Moving from a pod arrangement to private rooms will decrease the likelihood that another nurse will be standing close by, and has been observed to decrease com­ pliance with double-checking processes such as for high-alert

Discussion 1 1 -1 The correct answer is "E:' The components of learning and cul­ ture are on a spectrum, from least to most mature: unmindful (A), reactive (B), systematic (C), proactive (D), and generative (E). Question 1 1 -2 The structured communication system SBAR-or situation, background, assessment, recommendation-is a mechanism that: A) Exposes the transmitter to significant vulnerability. B) Conveys a message to a provider after verbal orders have been entered. C) Promotes critical thinking by the transmitter and suggests a solution to the receiver. D) Is meant only for messages from nurses to physicians. E) Avoids shared mental models as this can narrow one's thinking.

570

MCG RAW-H I LL E D U CATION S P E C I A LTY BOA R D REVI EW: P E D I ATRICS

Discussion 1 1 -2 The correct answer is "C:' SBAR is a structured communica­ tion system meant to promote analysis of the situation by the messenger and suggest a potential solution to the recipient. This type of tool avoids "read my mind" type of thinking and creates a shared mental model.

A neonatal intensive care nurse may hear thousands of alarms per shift. The staff worry that they may ignore an important alarm, but they cannot possibly treat each alarm with the same degree of concern. Question 1 2-1 An effective approach to improving the sensitivity and speci­ ficity of the alarms heard in this unit would be to: A) Silence alarms as soon as they are heard, then evaluate the situation afterward. B) Lower the low heart rate, oxygen saturation, and blood pres­ sure alarm limits. C) Turn off the low heart rate, oxygen saturation, and blood pressure alarm limits. D) Systematically study the trigger, tone, and legitimacy of the alarms heard in this unit. E) Encourage parents to look for color change, then silence alarms if they are reassured. Discussion 1 2-1 The correct answer is "D:' Alert fatigue is a very real and dan­ gerous phenomenon. The various alarms should be system­ atically studied to determine what proportion are legitimate, and whether the issue pertains to limit settings, equipment, or another factor. (Have you ever noticed that an inpatient unit and its alarms can sound like an arcade? Now if only skee-ball was standard equipment on all hospital units, the noise might not be so bad.)

You walk into an operating room and note a scrub nurse try­ ing to perform a "time out:' but she is brushed off repeatedly by the surgeon. Question 1 3-1

In an attempt to build a culture of safety, in addition to sup­ porting the scrub nurse who tried to be compliant, your actions should include: A) Disciplining the surgeon. B) Determining the scope of the problem and implementing widespread safety culture interventions. C) Disciplining the surgeon's supervisor.

D) Assuming this to be an isolated incident since the scrub nurse was so insistent. E) Terminating all those involved. Discussion 1 3-1 The correct answer is "B:' Likely this instance is a symptom of a larger safety culture problem, and the scope of the issue should be investigated. If this is determined to be a widespread issue, there will need to be large-scale safety culture interven­ tions. Leaders will need to be sure there are supportive policies, adequate and appropriately placed champions, user-friendly reporting systems, and lack of retaliation in response to event reporting.

Your staff have come to you as they have repeatedly observed a pediatric gastroenterologist failing to follow proper hand hygiene protocol when he enters patient rooms. Question 1 4-1 An effective response to this situation is likely to involve: A) Taking the situation straight to human resources and rec­ ommending termination as compliance with hand hygiene is a hospital policy. B) Having a "cup of coffee" conversation with the physician, encouraging the staff to speak up, and holding all employees accountable. C) Explaining to the physician that the standards are higher for gastroenterologists due to the nature of their business. D) Asking the staff to notify you if they note any further noncompliance. Discussion 1 4-1 The correct answer is "B:' "Coffee cup" conversations are gen­ erally the first step in examining such issues and can help determine the validity of the allegation as well as reasons for noncompliance. However, staff should be encouraged to address situations such as this with the individual. Leaders will be responsible for ensuring that staff can confront such situations without fear of retaliation. B I B LIOGRAPHY

Argent AC, Montgomery VL. Quality improvement, patient safety, and medical error. In: Nichols DG, ed. Rogers' Textbook of Pediatric Intensive Care. 4th ed. Philadelphia, PA: Lippincott; 2008. Fernandez-Llamazares CM, Calleja-Hernandez MA, Man­ rique-Rodriquez S, Perez-Sanz C, Duran-Garda E, Sanjurjo-Saez M. Impact of pharmacist interventions in reducing paediatric prescribing errors. Arch Dis Child. 20 1 2;97:564-568. Frush KS, Krug SE. Pediatric Patient Safety and Quality Improvement. New York, NY: McGraw Hill; 20 1 5.

C H A PT E R 25

Hughes RG, Clancy CM. Working conditions that support patient safety. Nurs Care Qual. 2005;20:289-292. Kohn LT, Corrigan JM, Donaldson MS, eds. To Err Is Human: Building a Safer Healthcare System. Washington, DC: National Academy Press; 2000. Langley GJ, Moen RD, Nolan KM, Nolan TW, Norman CL, Provost LP. The Improvement Guide: A Practical Approach



PATI ENT S A FETY A N D QUALITY I M P ROVE M E N T

to Enhancing Organizational Performance. 2nd ed. San Francisco, CA: Jossey-Bass; 2009. Pronovost P, Berenholtz S, Dorman T, et al. Improving communication in ICU using daily goals. J Crit Care. 2003; 1 8 : 7 1 -75. Reason J. Human Error. Cambridge: Cambridge University Press; 1 990.

571

This page intentionally left blank

Pha rma col ogy: Pa i n Ma n a g e m e nt a n d Sedatio n

26

Pa rt 1 Pa i n M a n a gement S u s a n S. Vos, G a ry M i l avetz, La u ra Ste i n a u e r, a n d J eff Va n B l a rcom

A mother calls your office to ask if her child should take a medication with food or on an empty stomach. She does not know the name of the medication. She tells you the pills are small, pink, and diamond shaped and asks, "Isn't that enough information for you to know what medication he is taking?"

The father of your 4-year-old patient calls the clinic. His son will not swallow a medication because of the taste. The par­ ents tried holding him down and squirting the medicine in his mouth, but he bit his mother. The father asks if there is anything that can be done to make it taste better.

Question 1 - 1 Which of the following are the most important consider­ ations regarding taking medications with food? A) Decreased gut irritation. B) Acid degradation. C) Increased absorption. D) Decreased absorption. E) All of the above.

Question 2-1 Which of the following is the best option? A) Change to a better-tasting medication. B) Mask the taste of the medication with flavoring additive. C) Mix the medication with the child's favorite soft food. D) All of the above.

Discussion 1 -1 The correct answer is "E:' All of the listed options are impor­ tant considerations when thinking about food and medications. Sometimes medications are taken with food to decrease the likelihood of an upset stomach (eg, ibuprofen) . Some drugs can undergo degradation due to changes in gut pH ( eg, penicillin V) . Food may improve the absorption of some medications ( eg, amoxicillin) . Food may also bind with a drug, causing decreased absorption ( eg, tetracycline) . Additionally, food may change the absorption rate and extent of medications. Lastly, some foods can change the metabolism of a medication ( eg, grapefruit or grapefruit juice, which interferes with numerous medications).

Discussion 2-1 The correct answer is "D:' One could consider all of these options when the palatability of a medication is decreasing the patient's adherence to the regimen. Knowing the taste of liquid medications is an important consideration when prescribing medications to children. Often, steroids and certain antibiot­ ics are among the worst offenders when it comes to palatability. Changing to a more tolerable-tasting liquid is an option, if one is available. There are some flavoring additives that can be added, but this is not always an option. Some pharmacists are able to extemporaneously compound a medication with a flavoring agent. The parent could mix a medication with a small amount of the child's favorite food ( eg, chocolate pudding or applesauce) . However, you should be sure to recommend a small amount of 573

MCG RAW-H I LL E D U CATION S P E C I A LTY BOA R D REVI EW: P E D I ATRICS

574

food and have the patient consume it right away since there are very little data on the stability of medications when mixed with food. Other options to consider are "chasing" the medication with a strong-flavored drink (eg, chocolate milk, cola, juice) .

� QUICK QUIZ A normal, healthy patient begins taking "Medication X' once daily. "Medication X' has a half-life of 12 hours. If you start a normal dose of the medication on Monday at 8:00 AM and repeat the dose daily, when will the drug level reach steady state concentration? A) Tuesday at 8:00 AM. B) Wednesday at 8:00 AM. C) Thursday at 8:00 AM. D) Friday at 8:00 AM. Discussion The correct answer is "C:' Half-life refers to the amount of time it takes for the body to reduce the serum concentration by half. When a medication is taken on a regular basis, an ongo­ ing process occurs in which a continuous amount of the drug is absorbed, metabolized, and eliminated. Eventually, the drug reaches a relatively consistent serum concentration inside the body, called steady state. It typically takes between 5 and 6 half­ lives for a drug to reach steady state. In this case, it would take about 60 to 72 hours to reach a steady-state concentration. Drug levels should be measured once the drug reaches steady state. •



Helpful Tip

The h a lf-l ife of a d rug is the a mount of time it ta kes the

1 1 1r body t o red uce t h e serum d rug concentration b y ha lf. It ta kes 5 to 6 half-l ives to reach a steady state.

C) Self-induction of metabolism. D) Drug interaction. Discussion 3-1 The correct answer is "C:' Carbamazepine exhibits self-induction of metabolism. The drug is metabolized more rapidly. Regard­ ing option ''A;' typically, clinical laboratories do not report a level back if there is any suggestion of error occurring when they include controls or blinded samples. Although option "B" is always possible, it is unlikely in this case because of the his­ tory of no missed doses prior to the level being obtained. Since no other medications are being reported in this patient, option "D" is not likely to have occurred. But, it is always a good idea to ask about any new medications, over-the-counter medications, herbal products, and so on. •

Helpful Tip

:S.� With

carba mazepine, i n itia l ly check a serum drug

r1 1 r concentration after severa l days of steady-state dosing

with no m issed or extra doses. The self-i n d u ction of meta bol ism occu rs over severa l weeks to months with ma i ntenance use. Therefore, serum concentrations should be mon itored every 3 to 4 months.

A 5-year-old girl is carried into the emergency department by her father. She began seizing approximately 40 minutes ago and the seizure has just stopped following a dose of intramus­ cular (IM) lorazepam. The father reports that his daughter is currently taking phenytoin (50 mg Infatabs given as 1 '12 tablets twice daily), and she has not missed any doses in the last week. She weighed 40 pounds at her kindergarten checkup last month. Question 4- 1 What is the patient's daily total dosage of phenytoin in mil­ ligrams per kilogram of body weight? A) 1 8 mg/kg/day. B) 8 mg/kg/day. C) 5.5 mg/kg/day. D) 4 mg/kg/day.

You are seeing an 8-year-old Caucasian girl with a history of generalized tonic-donie seizures. Owing to poor control on phenytoin, she was switched 3 months ago to carbamazepine, 200 milligrams (mg) by mouth twice daily. Her initial serum level S days after starting therapy was 7. 1 micrograms/millili­ ter (mcg/mL) . The level was drawn 4 hours after her morning dose, and she reports no missed or extra doses. The goal peak levels of 4 to 12 mcg/mL occur 4 to 5 hours after a dose. On routine recheck of her serum drug concentration today, her level is 4.5 mcg/mL (the level was measured 4 hours after her morning dose, with no missed or extra doses) .

Discussion 4-1 The correct answer is "B:' The patient's weight is 40 pounds, which is equivalent to 1 8 . 1 kg. Remember, 2.2 pounds equals 1 kg. The total daily dosage is 1 50 mg (75 mg twice daily) . There­ fore, the weight-based dosage per day is 8.2 mg/kg/day. The typical maintenance dosage range for pediatric patients taking phenytoin is 7 to 1 0 mg/kg/day.

Question 3-1 Which of the following factors is most likely influencing the most recent serum level? A) Laboratory error. B) Poor adherence to her regimen.

Question 4-2 What should you do next regarding the girl's phenytoin regimen? A) Give a rapid infusion of phenytoin. B) Give a partial loading dose of phenytoin.

C H A PT E R 26

C) Draw a phenytoin level. D) Discontinue phenytoin and start a different medication. E) All of the above. Discussion 4-2 The correct answer is "C:' To be clear, you would first sta­ bilize the patient and administer intravenous lorazepam for status epilepticus. When a patient has a seizure while tak­ ing phenytoin, you need to know the drug level at which the patient has seizure activity. Phenytoin is used widely for controlling seizures. Phenytoin has a narrow therapeutic index and dosing and management is often poorly executed. You may decide to give a partial loading dose; however, you must know the patient's current phenytoin level before you can calculate the partial loading dose. Rapid infusion of phenytoin is not recommended due to severe hypoten­ sion and cardiac arrhythmias. This warning is considered a "boxed warning" or "black box warning" that is given by the U. S. Food and Drug Administration (FDA) . This is the FDA's strongest warning and can require the pharmaceutical company to print the warning on the box of the medication or in literature describing this severe and sometimes life­ threatening effect. For phenytoin, the rate of administration should not exceed 50 mg per minute (mg/min) in adults and 1 to 3 mg/kg/min in pediatric patients. •





P H A RMACOLOGY: PA I N M A N AG E M ENT A N D S E DATION

575

Your 5-year-old patient is stabilized and ready to be dis­ charged from the facility. Her maintenance dose of phenytoin has been increased to 50 mg Infatabs, given as 2 tablets twice daily ( 1 1 mg/kg/day) . Question 4-4 When should you check another phenytoin level? A) Today. B) Tomorrow. C) In 1 week. D) In 1 month. Discussion 4-4 The correct answer is "C:' The serum half-life of phenytoin is variable b ecause it follows Michaelis-Men ten pharmaco­ kinetics . This type of nonlinear pharmacokinetics occurs when the drug molecules overwhelm the body's ability to metabolize the drug. This causes the serum drug concentra­ tions to increase in a disproportionate manner. The half-life for phenytoin in adults is about 24 hours; therefore steady­ state concentrations are typically reached in 5 to 6 days . Trough concentrations should be measured to ensure that the patient does not experience seizure activity toward the end of the dosing interval as that serum drug concentration decreases.

Helpful Tip

If a b l a c k box wa r n i n g i s i s s u ed by t h e U.S. Food a n d

1 1 1r Drug

Ad m i n i stration, p h a rmaceutica l co m pa n ies

may be req u i red to p r i n t t h e wa r n i n g descri b i n g the l i fe-th reate n i n g effect o n t h e box o r o n t h e paper

i n sert.

Upon further questioning of the girl's father, you discover she has recently started enteral nutrition supplementation and has been taking the phenytoin at the same time of day as she is drinking her PediaSure. Question 4-3 What impact will this have on the bioavailability of phenytoin? A) Increased phenytoin bioavailability. B) Decreased phenytoin bioavailability. C) No effect on phenytoin bioavailability. Discussion 4-3 The correct answer is "B:' Bioavailability is the fraction of an administered dose of unchanged drug that reaches the systemic circulation. In this case, food and enteral feedings will decrease the bioavailability of phenytoin and patients may experience sub­ therapeutic levels of phenytoin. Patients should be told to take phenytoin on an empty stomach and to take the medication in a consistent manner each day. Phenytoin is a substrate of the enzymes CYP2C 1 9, CYP2C9, and CYP3A4 and therefore has numerous potential drug interactions. It is important to consider drug and nutrient interactions for patients taking phenytoin.

}% Q U I C K Q U I Z Which o f the following statements about linear pharmacoki­ netics is false? A) Changes in drug dosing and steady-state drug concentra­ tion are proportional. B) Changes in drug dosing and steady-state drug concentra­ tion are disproportional. C) Clearance is independent of the dose of the drug. D) The drug's half-life is independent of the drug's plasma concentration. E) Clearance is independent of dosing schedule. Discussion The correct answer is "B:' In linear pharmacokinetics, when the dosage of a drug is changed the steady-state plasma drug con­ centration changes proportionally. For example, if a drug dos­ age is tripled then the steady-state concentration will also triple. When the drug concentration changes more or less than what would be predicted based on the change in dosage, the pharma­ cokinetics are said to be nonlinear.



Helpful Tip

:5.� Pha rmacoki netics

i1 1 r a

stu d ies what the body does to

d rug a n d i n c l u d es drug a bsorption, d i stribution,

meta bolism,

and

e l i m i nation.

Pha rmacodyna m ics

stu dies what the drug does to the body.

576

MCG RAW-H I LL E D U CATION S P E C I A LTY BOA R D REVI EW: P E D I ATRICS

� QUICK QUIZ Which i s NOT a n associated laboratory finding with antisei­ zure medications? A) Metabolic acidosis. B) Thrombocytopenia. C) Elevated liver enzymes. D) Pancytopenia. E) All of the above. Discussion The correct answer is "E:' Valproic acid may cause dose-related thrombocytopenia. Topiramate may cause a metabolic acido­ sis. Carbamazepine may cause neutropenia and pancytopenia. Oxcarbazepine may cause elevated liver enzymes. This is not an exhaustive list of possible laboratory abnormalities or drugs associated with laboratory derangements.

A IS-month-old girl presents to the emergency department with fever. Her mother reports that "she has slept all day and won't open her eyes?' Her weight is 9.6 kg. After lumbar puncture is performed and cerebrospinal and blood cultures are obtained, she is started on the following intravenous (IV) antibiotic therapy: Vancomycin, 144 mg IV every 6 hours {60 mg/kg/day) Cefotaxime, 540 mg IV every 6 hours (225 mg/kg/day)



� I

Helpful Tip

Desi red troug h serum concentrations of va ncomycin to

1 1 r treat a suscepti ble i nfection ra nge from s to 20 mcg/m L

depending on the organism a n d severity and location of i nfection. H i g h vancomycin troug h level s a re associated

with renal toxicity. If using va ncomycin for an extended period, it is i m porta nt to monitor troug h d rug levels!

� QUICK QUIZ Which o f the following statements i s false? A) Dosing of renally excreted medications needs to be adjusted in patients with chronic kidney disease. B) Pharmacokinetics vary in neonates. C) Peak drug levels are used to monitor toxicity. D) It is important to monitor for drug toxicity by checking drug levels. E) All of the above are true. Discussion The correct answer is "C:' In general, peak drug levels are fol­ lowed for therapeutic reasons, and trough levels to monitor for toxicity. Staying with vancomycin as our example, this drug is renally excreted. Therefore, dosing needs to be adjusted based on the patient's glomerular filtration rate to avoid toxicity. In neonates (especially those born prematurely) and infants, phar­ macokinetics vary widely as the kidneys are immature. Renal processes gradually mature over the first year of life.

Acyclovir, 190 mg IV every 8 hours ( 60 mg/kg/ day high dose) •

On the second day of therapy a trough level reveals the vanco­ mycin serum concentration to be 35 mcg/mL. Question 5- 1 Which of the following actions should be taken to treat this child? A) Change the interval to every 1 2 hours. B) Reduce the dose to 1 00 mg every 6 hours. C) Change the interval to every 24 hours. D) Leave the dose unchanged and recheck the level. Discussion 5-1 The correct answer is "A:' When checking levels, it is important to know the goal trough and peak levels. In meningitis, a high peak level is needed to increase central nervous system penetra­ tion. A trough level of 35 mcg/mL is toxic. Save the kidneys! Option "X' is the best answer because it should attain a simi­ lar peak but allow the drug to be more fully excreted over the subsequent dosing interval. Option "B" will reduce the peak but maintain a higher serum trough level. Option "C" will likely allow the trough to go sufficiently low as to be ineffective. Option "D" risks keeping the trough to high and permit toxicity to occur.



Helpful Tip

Dosag e o f pediatric medications is wei g ht based.

1 1 1r I t i s i m porta nt t o have a n accurate weight t o avoid s u bthera peutic or excessive dosi ng of medications.

A 1 2-year-old boy who weighs 40 kg is taking pimozide, 10 mg/day at bedtime, for the treatment of Tourette syn­ drome that is unresponsive to other measures. Another care­ giver provides him with a prescription for erythromycin to treat acne. Within a few days of starting the erythromycin, he notices an irregular heartbeat. He is subsequently diagnosed with torsades de pointes with a prolonged QTc interval. Question 6-1 What is the mechanism of the drug interaction between pimozide and erythromycin? A) Erythromycin stimulates the metabolism of pimozide. B) Pimozide stimulates the metabolism of erythromycin. C) Pimozide inhibits the enzyme that metabolizes erythromycin. D) Erythromycin inhibits the enzyme that metabolizes pimozide.

C H A PT E R 26

Discussion 6-1 The correct answer is "D:' Medications can be substrates, inhibi­ tors, or inducers of the metabolism of other compounds. Cyto­ chrome P450 enzyme 3A4 (CYP3A4) is responsible for the metabolism of both pimozide and erythromycin and is inhibited by erythromycin, allowing the accumulation of excess pimozide to occur. This is an example of a pharmacokinetic interaction, whereby one drug affects the other's absorption, distribution, metabolism, or excretion. Supratherapeutic serum drug levels of pimozide are associated with torsades de pointes. Cytochrome P450 is a family of enzymes responsible for drug metabolism; these enzymes are primarily found in the liver. At least ten phase 1 cytochrome P450 enzymes are involved in drug metabolism. Some of the metabolites of these phase 1 enzymes may retain pharmacologic activity. Phase 2 enzymes facilitate elimination from the body. Additionally, drug metabolism enzymes mature at different rates from birth to young adulthood. •

Helpful Tip

:5.� Eryt h romycin

is commonly a ssociated with d rug

r1 1r i nteractions throug h i n h i bition of CYP3A4 (an enzyme that contri butes to the d rug's metabolism). CYP3A4 is one of the main d rug-meta bolizing enzymes fou n d i n the l iver. T h e metabolism o f d ru g s t h a t a re hepatica l l y cleared u s i n g this enzyme m a y be affected .





P H A RMACOLOGY: PA I N M A N AG E M ENT A N D S E DATION

577

Discussion 7-1 The correct answer is "D:' Scopolamine is an anticholinergic agent. Common central nervous system side effects include drowsiness, dizziness, confusion, hallucinations, and memory disturbances. Other common side effects associated with anti­ cholinergic agents include dry mouth, dry eyes, constipation, tachycardia, and urinary retention. Recall the cholinergic tox­ icity mnemonic, SLUD (Salivation, Lacrimation, Urination, Defecation) and think the opposite. Question 7-2 What type of adverse drug event (ADE) is this patient experiencing? A) Pharmacokinetic. B) Pharmacodynamic. C) Pharmacogenomic. Discussion 7-2 The correct answer is "B:' In a pharmacodynamic-related side effect, the medication dosing and mode of delivery are right but the drug causes an adverse event related to its pharmacologic effect on the body. Had the dosing or mode of delivery been wrong, a pharmacokinetic adverse event would have occurred. In this scenario, the dosing and mode of delivery are correct for a patient 1 2 years of age or older. An adverse event would have been pharmacogenomic if this drug had been metabolized dif­ ferently because of his racial or ethnic background.

Helpful Tip

� When prescri bing med ications, it is i m porta nt to have

:5.

r1 1r a n u p-to-date l ist o f medications t h e patient is ta ki ng

to check for possible d rug i nteractions. Concom ita nt a d m i n istration of certa i n d rugs can a lter the seru m concentrations of other d rugs.

A 13-year-old Caucasian boy who weighs 45 kg has a history of motion sickness. He is going on a Caribbean cruise with his par­ ents and siblings. In an attempt to avoid motion sickness while onboard the ship, the parents and child visit a travel clinic for advice and help. A physical exam and review of systems reveals a healthy adolescent who is not on any chronic medications. The provider prescribes a scopolamine patch ( 1 .5 mg) to be placed behind the ear about 4 hours before boarding the cruise ship. This should be left on for 3 days and then discarded. It may be repeated every 3 days for the duration of the cruise. On day 2 of the cruise, the parents have difficulty arousing the boy, and he is confused and cannot remember where he is or why he is there. Question 7- 1 Which of the following pairs are adverse effects of scopolamine? A) Dry mouth and enuresis. B) Bradycardia and sweating. C) Diarrhea and constipation. D) Drowsiness and confusion.



Helpful Tip

� The U.S. FDA has a n o n l i n e p rog ra m ca l led MedWatch

:5.

r1 1 r that is i ntended to be a mecha n i s m to report A DEs a n d mon itor postma rketing s u rvei l l a nce o f med ication

safety.

}% Q U I C K Q U I Z A n idiosyncratic drug reaction: A) May result from overdosing of a drug. B) Is a rare and unpredictable event. C) Is an extension of the drug's pharmacologic effect. D) None of the above. Discussion The correct answer is "B:' Idiosyncratic drug reactions are believed to be immune mediated, rare, and unpredictable. The reaction is not dependent on the dose of the drug nor does it involve the known pharmacologic effects of the drug. An adverse drug reaction is an extension of how the drug is sup­ posed to work and may result from giving too much of the drug. For example, diphenhydramine has known anticholinergic effects. Overdosage results in anticholinergic toxicity (excessive symptoms). If syncope occurred this would be an idiosyncratic reaction, as syncope is not a known pharmacologic effect of diphenhydramine.

578

MCG RAW-H I LL E D U CATION S P E C I A LTY BOA R D REVI EW: P E D I ATRICS

A 7-week-old girl born at 30 weeks' gestational age is cur­ rently receiving caffeine citrate, 5 mg/kg/dose daily, for apnea of prematurity. Following 3 weeks of caffeine therapy in the neonatal intensive care unit, she develops feeding intolerance, which includes gastroesophageal reflux and irritability reported by the parents and nurse during and after feeding. Question 8- 1 Which of the following is the most likely factor causing her feeding intolerance? A) Caffeine. B) Gestational age. C) Postmenstrual age. D) Feeding technique. Discussion 8-1 The correct answer is ''A:' Methylxanthines such as caffeine and theophylline have been the mainstay of treatment for apnea of prematurity for many years. They stimulate the central respiratory drive and decrease the number of apnea episodes. Although caffeine typically is used for apnea due to its ease of administration, wide therapeutic index, and tolerability, some­ times theophylline is used. Methylxanthines are typically safe; however, they can cause adverse drug reactions that include feeding intolerance, diuresis, diaphoresis, and urinary calcium excretion. It is always important to consider adverse drug reac­ tions when new symptoms occur in a patient. In the absence of caffeine administration, the other options could explain the feeding intolerance.

A 1 2-year-old boy comes to the clinic for an annual physical. Upon examination, you notice he has gray dental staining on his permanent teeth.

contraindicated in children owing to the risk of permanent lesions of the cartilage of weight-bearing joints, they may be prescribed if there is no alternative. Ceftriaxone and other drugs in the sulfonamide class are used with caution in jaundiced neo­ nates and infants because of the risk of kernicterus. Kernicterus may be caused by certain drugs that displace bilirubin from pro­ tein binding sites, causing hyperbilirubinemia. Chlorampheni­ col is contraindicated in neonates and infants because of the risk of "gray baby syndrome;' which results from their inability to metabolize the drug. The syndrome produces a constellation of symptoms, including vomiting, poor feeding, respiratory dis­ tress, and cyanosis, and may lead to death. The takeaway point is that medications safe in adults may pose risks in children given their immature development.





Helpful Tip

Tetracyc l i n e drugs a re contra i n d icated i n preg nant

1 1 1r women, n u rs i n g mothers, a n d c h i l d ren you nger than

8 yea rs (dentition may be permanently sta i ned) u n l ess

benefits outweigh the risks.

A 7-month-old infant girl and her mother visit your clinic for the second time this week. The infant's weight today is 16 pounds, unchanged from the visit 4 days ago. Today, the mother reports her daughter has "the worst diaper rash she has ever seen:' Four days ago, the infant was diagnosed with acute otitis media and was given a prescription for amoxicillin with clavulanate (Augmentin), 2.5 mL by mouth twice daily for 10 days. The mother reports that they have not missed any doses. The infant's bowel movements have increased in fre­ quency, but they are not described as bloody or watery. Upon physical exam, you notice the skin in the diaper area appears bright red with wet-looking patches. There are no satellite lesions, and the rash is confined to the diaper area.

Which drug is the most likely cause of dental staining of per­ manent teeth? A) Tetracycline. B) Levofloxacin. C) Ceftriaxone. D) Chloramphenicol.

Question 1 0-1 Which of the following is the most likely cause of this infant's diaper rash? A) Her age. B) Allergic reaction to the antibiotic. C) Diarrhea associated with antibiotic use. D) Allergic reaction to the type of wipe used. E) Allergic reaction to the type of diaper used.

Discussion 9-1 The correct answer is ''A:' He may have received a tetracycline medication or his mother may have received this medication while she was pregnant or breastfeeding. Dental staining and darkening of permanent teeth have been reported in patients who received tetracyclines during the time of tooth crown formation. However, recent data have shown that this may not be an issue with doxycycline when used at appropriate doses. Although levofloxacin and other quinolones are often

Discussion 1 0-1 The correct answer is "C:' One known and common side effect of amoxicillin with clavulanate is diarrhea, which has an inci­ dence ranging from 3% to 34%. Diarrhea has been shown to be a significant risk factor for diaper dermatitis. Pseudomem­ branous colitis resulting from Clostridium diffi cile infection can be associated with antibiotic use and is described as watery, bloody stools occurring at a frequency of about 1 5 stools per day. Treatment for antibiotic-associated pseudomembranous

Question 9-1

C H A PT E R 26

colitis typically includes an oral regimen of metronidazole or vancomycin. It seems ironic that the treatment of an adverse effect of one antibiotic is another antibiotic.





Helpful Tip

Beta-lacta m a ntibiotics including amoxicil l i n a re bacteri-

1 1 1 r cidal d rugs that i n hibit bacteria l cel l wa l l synthesis. Bac­ tericidal a ntibiotics kil l bacteria. Bacteriostatic a ntibiotics i n h i bit the g rowth or reproduction of bacteria.

Question 1 0-2 What is the best treatment option for this patient's diaper dermatitis? A) Air out the skin. B) Use barrier cream. C) Keep the skin clean. D) All of the above. Discussion 1 0-2 The correct answer is "D:' The most effective way to treat irritant diaper rash is to reduce skin contact with urine and feces. This can be done by airing out the skin, using a barrier cream, and keeping the skin clean. The goals of diaper dermatitis treatment are typically prevention first, and then cure. Application of a barrier cream is recommended at every diaper change. This can be recommended to prevent and also treat existing diaper der­ matitis. Nonprescription topical barrier creams include ingredi­ ents such as zinc oxide, petrolatum, or both.







P H A RMACOLOGY: PA I N M A N AG E M ENT A N D S E DATION

579

Discussion 1 0-3 The correct answer is "A:' Because of the likelihood of fungal infection in this patient secondary to antibiotic use, a topical antifungal (eg, nystatin, clotrimazole) could be recommended for up to 14 days. Talcum powders are not recommended for diaper dermatitis due to the risk of inhalation in the infant. Cornstarch is also not recommended as it can worsen fungal­ related diaper dermatitis. Topical steroids could aggravate symptoms and cause adverse systemic effects.

A 12-year-old African-American male has an 8-year history of severe persistent asthma. He uses 2 puffs of an albuterol metered­ dose inhaler (MDI) for treatment of acute respiratory symptoms and 2 puffs twice daily of fluticasone, 44 meg/puff MDI, for maintenance control. Over the past 2 months, he has needed four refills of albuterol while obtaining a monthly refill of his flutica­ sone. He states he is using his albuterol inhaler five or six times a day with minimal relief of symptoms. He wakes up at night a couple of times a week with coughing. His asthma is poorly con­ trolled and needs additional therapy; therefore, you decide to increase his fluticasone to 220 meg/puff, in 2 puffs twice a day. Question 1 1 -1 Which of the following adverse effects is/are associated with long-term inhaled corticosteroid use? A) Increased heart rate. B) Longitudinal growth suppression. C) Skin thinning. D) Serum electrolyte abnormalities.

Helpful Tip

Treatment of d i a per ra sh i n c l u d es a combi nation

1 1 1 r of

measu res, which a re most effective when used

together. The letters ABC DE a re a usefu l way to remem­ ber all of these mea s u res.

A - Air out the skin by allowing the child to go diaper-free B Barrier; apply a generous amount of a diaper paste, cream, or ointment to protect the skin -

C Clean; keep the skin clean using frequent bathing with water and mild cleanser -

D Disposable diapers; consider using disposable rather than cloth diapers during an episode of diaper rash -

E - Educate; teach parents and caregivers how to prevent a recurrence of diaper rash Question 1 0-3

Since this patient has severe diaper dermatitis and has con­ tinued to have symptoms despite treatment with barrier creams, which of the following could be recommended? A) Topical antifungal cream. B) Topical steroid cream. C) Talcum powder. D) Cornstarch.

Discussion 1 1 -1 The correct answer is "B:' Option "/\' may be associated with short-acting beta-agonist use (eg, albuterol) , option "C" may be associated with topical corticosteroids, and option "D" is unre­ lated. Adverse effects of long-term inhaled corticosteroids are dose dependent and include hoarseness, oropharyngeal candi­ diasis, adrenal suppression, cataracts, glaucoma, immunosup­ pression, short stature, and decreased bone mineralization. Inhaled corticosteroids are used to minimize systemic adverse effects of long-term systemic corticosteroids. Although inhaled corticosteroids have high topical potency in the airways, they have limited-but still some-systemic effects, especially in high doses. Management includes using the lowest dose pos­ sible or using alternative (nonsteroidal) medications.



Helpful Tip

:5.� Patients

who use inha led corticosteroids should be

i1 1 r mon itored for g rowth, fat red istri bution, blood pressu re, mood, behavior, and sleep pattern changes as wel l

as cata ract formation. When patients exhi bit these adverse effects, the dose should be red uced to the lowest effective amount that controls the asthma or an a lternative medication s h o u l d be used.

MCG RAW-H I LL E D U CATION S P E C I A LTY BOA R D REVI EW: P E D I ATRICS

580

A 16-year-old girl weighing 50 kg is being treated for urticaria of unknown origin. She was started on cimetidine, 300 mg at night, after having no response to H1 antihistamine agents. She is a straight-A student in high school but after a week of cimetidine treatment, she develops confusion, headaches, and right upper quadrant pain. Her provider stops the cimetidine and the adverse events cease after 1 week off therapy. Question 1 2- 1 Which adverse events are associated with cimetidine? A) Confusion. B) Headache. C) Abdominal discomfort. D) All of the above. Discussion 1 2- 1 Th e correct answer i s "D:' Cimetidine i s a competitive blocker of histamine at the H receptor of the gastric parietal cells. It is 2 important to remember that H receptors are found outside the 2 stomach, including in the brain, lungs, endocrine and exocrine glands, gastrointestinal muscle, genitourinary system, and skin. This answer makes sense if the medication is used to treat a skin condition and toxicity affects multiple organ systems. When prescribing a medication, it is important to think of possible organs that may be affected in addition to the targeted organ.





Helpful Tip

H 2 receptors a re p ri m a rily fou nd in the pa rieta l cel l s

1 1 1r of t h e stomach

a n d i n h i bit gastric acid secretion.

C i meti d i n e is meta bol ized by the CYP450 system i n the l iver a n d is a ssociated with m a ny drug i nteractions.

seen by her pediatrician but was not prescribed an antibi­ otic to her mother's disgust. The mother has been giving her daughter acetaminophen, ibuprofen, and occasional aspirin because she had never seen her daughter this uncomfortable with "the flu:' Question 1 3-1 Which medication is the most likely cause of the patient's current situation? A) Acetaminophen. B) Ibuprofen. C) Aspirin. Discussion 1 3-1 The correct answer is "C:' There is strong evidence that aspi­ rin therapy following a viral illness (eg, influenza A or B, vari­ cella, adenovirus) may cause Reye syndrome. Reye syndrome is an acute noninflammatory encephalopathy that occurs along with fatty degeneration in the liver. It may cause liver failure, increased intracranial pressure, and death. Signs and symptoms of Reye syndrome include vomiting following a viral illness and confusion that may progress to coma and seizures. Laboratory testing reveals sky-high values for liver function tests, as well as coagulopathy, hyperammonemia, and hypoglycemia. Following the issuances of the Surgeon General's warning as well as pack­ age warnings on all products containing aspirin in the 1 980s, the incidence of Reye syndrome has decreased.



Helpful Tip

� Aspirin

=.

should a l ways be avoided d u ri ng febrile

r1 1r i l l nesses i n c h i l d ren d u e t o the risk of Reye syn d rome,

a n d providers should conti n u e to rem i n d ca reg ivers of

this wa rning. One exception is Kawasaki d i sease. There is a l ways an exception to the rule!



Helpful Tip

=-� P roton

p u m p i n h i bitors a n d H 2 blockers should be

r1 1r j u d iciously prescribed a s u s e i n ped iatric patients i s a ssociated with increased risk of pneumonia, acute d ia rrhea l i l l ness, and, i n prematu re i nfa nts, necrotizi ng enterocol itis.





Helpful Tip

Long-term use of H 2 blockers is associated with vita m i n

1 1 1r 8 , 2 deficiency, which causes anemia a n d i rreversible peri pheral neu ropathy.

A 1 0-year-old girl is brought to the emergency department by her mother. She is responding inappropriately to ques­ tions and displaying combative behavior, vomiting, tachy­ cardia, and hypotension. Ten days earlier, the girl had been

� QUICK QUIZ Which i s NOT an adverse effect associated with nonsteroidal antiinflammatory drugs (NSAIDs) ? A) Gastritis. B) Acute kidney injury. C) Tinnitus. D) Asthma exacerbation. E) All of the above. Discussion The correct answer is "E:' NSAIDs are used for their antiinflam­ matory, antipyretic, and analgesic properties. Gastrointestinal (GI) symptoms are the most common adverse effects and are generally mild, but serious adverse events, including ulceration, bleeding, and intestinal perforation, may occur. Ibuprofen should be avoided in patients with kidney problems and certain GI conditions.

C H A PT E R 26



P H A RMACOLOGY: PA I N M A N AG E M ENT A N D S E DATION



An 8-year-old Latino boy has a 5-year history of moderate, persistent asthma. He uses 2 puffs of an albuterol metered­ dose inhaler (MDI) for treatment of acute respiratory symp­ toms and 2 puffs twice daily of a beclomethasone MDI for maintenance control. Over the past 2 months, he has needed six refills of albuterol while obtaining a monthly refill of his beclomethasone. He states he uses his albuterol inhaler six or seven times a day with minimal relief of symptoms. He wakes up at night a couple of times a week with coughing. The assessment is that his asthma is poorly controlled and needs additional therapy. The asthma clinic prescribes salmeterol by dry powder inhaler, 1 puff twice a day.

Helpful Tip

� Beta 2-agonists

=-

581

such as a l buterol cause hypoka lemia

r1 1r beca use of i ntrace l l u l a r shifting of potassium a n d may be used i n the treatment of acute hyperka lemia.

A 4-week-old girl born at 28 weeks' gestation is being treated for disseminated Candida albicans, with known kidney involvement. Conventional amphotericin B has been started. As you are reviewing the morning lab results, you notice her serum creatinine is 2 mg/dL and are concerned about nephrotoxicity.

Question 1 4-1

How do long-acting beta2 -selective sympathomimetic ago­ nists differ pharmacologically from short-acting agents? A) Longer duration of effect on airways. B) Exert an additional effect on inflammation. C) Can be used as oral therapy. D) Fewer cardiac side effects. Discussion 1 4-1 The correct answer is "A." All beta-agonists have a minimal effect on reducing inflammation, so option "B" is not the correct answer to the question. Salmeterol is a long- acting beta-agonist, and albuterol (or levalbuterol) is a short- acting beta-agonist. Since there are no oral formulations of long­ acting beta-agonists available, option "C" is incorrect. Owing to B 1 -receptor stimulation, beta-agonists cause dose-depen­ dent cardiac toxicity, including tachycardia, palpations, and hypertension. A patient must understand which inhaler should be used for relief of symptoms and which inhaler or inhalers should be used daily to prevent symptoms from occurring. The patient must understand that different inha­ lation techniques are used with the two types of inhalers. Long- acting beta-agonists are not used in the treatment of acute respiratory symptoms because of their slow onset of effect. A long-acting beta-agonist should not be used alone, but only in combination with an inhaled corticosteroid. This is a black box warning by the FDA because of the risk of death from asthma.



Helpful Tip

� A long-acti ng

-

beta-agon ist s h o u l d o n ly be added to

r1 1r i n ha l ed corticosteroid therapy when maxi m u m d oses of i n haled corticosteroids have fa i led to control asthma

sym ptoms. Short-acti ng beta-agon ist i n h a lers a re always necessa ry for the treatment of acute sym ptoms

of asth ma beca use of their ra pid onset of action. When using both a long-acti ng beta-agonist a n d a n i n h a l ed corticosteroid, providers may consider a combi nation prod uct to i m p rove patient adherence.

Question 1 5-1 Which of the following would be the best choice to treat her infection now that she has developed nephrotoxicity? A) Discontinue amphotericin B; start fluconazole. B) Switch to a lipid-based formulation of amphotericin B. C) Prolong the dosing interval to every 48 hours. D) Discontinue amphotericin B; start echinocandin ( Caspofungin). E) Both A and D. Discussion 1 5-1 The correct answer is "E:' Option ''!\_' would be a possible choice because fluconazole has good concentration in the urine and activity against C. albicans. However, if this was empiric treat­ ment you would not want to use fluconazole because some non-albicans species are resistant to fluconazole. Option "D" is a possible choice because echinocandins have been shown to cover C. albicans and are well tolerated in neonates. Conven­ tional amphotericin (amphotericin B) is readily distributed in the kidney; therefore, switching to the lipid formulation may decrease the drug penetration to the site of infection and you may still have kidney damage. A lipid formulation could be used if the infection was outside the kidney, in the central nervous system, for example. Option "C" is not optimal because of the increased serum creatinine. (See Table 26- 1 .}

A 12-year-old boy with attention deficit hyperactivity dis­ order (ADHD) is taking methylphenidate, 20 mg at 8:00 AM and 1 2:00 noon, followed by 10 mg at 3:00 PM. His symp­ toms had been under control on methylphenidate in the morning, but he recently added the afternoon dose because of afterschool activities requiring him to be able to focus. He was diagnosed with ADHD at the age of 7 and has no other comorbid disorders. His other medications include acetaminophen as needed for headaches and loratadine as needed for seasonal allergies. Recently, his teacher noticed that he is in a "zombie-like" state and has expressed some concern to the parents.

Ill 00 N

TA B L E 26-1 COM M O N TOX I C I T I E S O F A N T I F U N G A L D R U G S

I nfu sion

Bone Ma rrow

G la

Ca rdiac

Reactions

S u p p ression

Yes

ltraconazo l e, posaconazole

A l i b; itraconazo le'

No

No

No

Yes

No

No

Yes

Yesct

No

No

Yes

No

No

Yes

No

No

No

Yes

Yes

No

No

Yes

Toxicity

H e patic

Re n a l

CNS

P h otopsia

Rash

Azoles

Yes

IV voriconazo l e

Vo riconazo le

Voriconazole

Amphotericin B

Yes

Yes

No

Echi noca n d i n s

Yes

No

5-Fiucytosine

Yes

No

C N S, centra l nervous system; Gl, gastroi ntesti n a l . •Al l medications a d m i n istered ora l ly have potential t o c a u s e G l d i scomfort. bQTc prolongation. 'Myopathy. dAnemia associated with decreased erythropoi etin prod uction.

C H A PT E R 26

Question 1 6-1 Which of the following would you recommend at this time? A) Increase the dose of methylphenidate. B) Add clonidine. C) Decrease the dose of methylphenidate. D) Switch to atomoxetine (Strattera) . Discussion 1 6-1 The correct answer is "C:' Lowering the dose of methylphe­ nidate is the recommended management. Increasing doses of stimulants may worsen symptoms. Adding clonidine (an alpha -adrenergic agonist) would cause more sedation. Chang­ 2 ing to atomoxetine at this point would not be recommended because the patient has not failed to respond to the stimulant; he is likely experiencing a dose-related adverse drug reaction.





Helpful Tip

Methyl phen idate (Rita l i n, Concerta)

ca n

. 1 1 1r positive urine d rug screen for a m pheta m mes.

cause a

A 1 2-year-old girl was recently started on dextroamphet­ amine for treatment of ADHD. Her symptoms have been well controlled since starting treatment and she has shown great improvement in school and social activities. However, she has recently experienced mild weight loss and decreased appetite. Question 1 7-1 What should you recommend to address the recent weight loss? A) Discontinue dextroamphetamine and start atomoxetine. B) Eat a high-caloric meal at breakfast and bedtime. C) Discontinue dextroamphetamine and start methylphenidate. D) Decrease the dose of dextroamphetamine. Discussion 1 7-1 The correct answer is "B:' Adverse effects are quite common with stimulant therapy, and appetite suppression and weight loss are known adverse effects of stimulants. To manage this adverse reaction, you can recommend a high-caloric meal prior to giv­ ing the dose of stimulant, at bedtime, or both. Other adverse effects, such as hypertension, hallucinations, blurred vision, or dysphoria, would be reasons to discontinue the stimulant ther­ apy. Additional side effects of stimulant medications include insomnia, moodiness, and abuse or misuse.



Helpful Tip

� Sti m u lant medications u sed to treat A D H D have h i g h

=

r1 1 r potential for abuse a n d misuse. If a patient req u i res

freq uent refi l l s or refuses to ta ke med ication holidays, suspect

a b u se.

Remem ber,

prescri bing "speed."

you

a re

essenti a l l y



P H A RMACOLOGY: PA I N M A N AG E M ENT A N D S E DATION

583

A 1 6-year-old girl was started on fluoxetine, 20 mg daily for depression, 3 months ago. Her mood has improved, but her pants are fitting tighter and she is eating more. She wants to know if this weight gain is related to her medication. You tell her it is complicated. Question 1 8-1 What is NOT an adverse effect of selective serotonin reuptake inhibitor (SSRI) use? A) Decreased libido. B) Drowsiness. C) Insomnia. D) Weight loss. Discussion 1 8-1 The correct answer is "D:' SSRis block the uptake of serotonin in the central nervous system, causing increased serotonin activity. SSRis should be started at the lowest dose possible to avoid adverse reactions, which many are dose dependent. The patient's regimen of 20 mg daily is an appropriate starting dos­ age for fluoxetine. Dosing is titrated to effect and serum drug levels are not followed. Weight gain is a reported adverse effect of SSRis but the association is not clear-cut. Once a patient is no longer depressed, his or her appetite may return causing the patient to gain back any weight that was lost. Thus, the weight gain may not be a direct effect of the medication per se. Of the SSRis, paroxetine has the highest incidence of weight gain. Weight gain with fluoxetine is less likely. Sexual dysfunction includes decreased libido, inability to have an orgasm (anorgas­ mia) , and difficulty achieving an erection. It is a no-win situa­ tion. If you are depressed, your libido is likely to be low. If you are treated, your libido may stay low. However, decreasing the dose or dividing the dose during the day may decrease adverse effects.



Helpful Tip

� Antidepressa nts

i n c l u d i n g SSRis carry the black box

r1 1r wa r n i n g that t h e risk o f suicide a n d suicidal t h i n ki n g

m a y i n crease i n c h i l d ren a n d adolescents after sta rting medication. Every patient sta rted on a n a ntidepressa nt should be monitored closely for suicidal ity.

A few months later, the adolescent is admitted to the hos­ pital with confusion, fever, and diarrhea. Her muscles are rigid and stiff. She is tachycardic and hypertensive, and has a fever of 40°C ( 1 04°F). She cannot remember her name and is clearly confused. Her reflexes are brisk with clonus, and she has a fine resting tremor. From her records, you see she has been refilling the fluoxetine prescription monthly. She was recently diagnosed with methicillin-resistant Staphylococcus aureus (MRSA) cellulitis and started on an oral antibiotic­ but she cannot remember its name.

MCG RAW-H I LL E D U CATION S P E C I A LTY BOA R D REVI EW: P E D I ATRICS

584

Question 1 8-2

Which medication may interact with an SSRI to cause sero­ tonin syndrome? A) Linezolid. B) Lithium. C) Tramadol. D) St John's wart. E) All of the above. Discussion 1 8-2 The correct answer is "E:' The patient was prescribed linezolid­ an oral antibiotic used to treat MRSA infections. Serotonin syndrome may occur with serotonergic medications (SSRis, sero­ tonin norepinephrine reuptake inhibitors [SNRis] ), especially when used with other drugs that increase serotonin activity or impair the metabolism of serotonin. Linezolid is a monoamine oxidase inhibitor (MOAI) . MOAis decrease the metabolism of serotonin and may precipitate serotonin syndrome when taken with an SSRI. Other drugs that may cause serotonin syndrome when taken with an SSRI include fentanyl, carbamazepine, val­ proate, tricyclic antidepressants, triptans, and dextrometho­ rphan. When a patient is taking an SSRI, it is important to check for drug-drug interactions when prescribing a new medication. •

Helpful Tip

� Signs a n d sym ptoms of seroton i n synd rome include r1 1r a ltered mental status, a utonomic insta bil ity, neuromus­ cular changes, G l sym ptoms, and seizu res.

A 40-day-old infant was treated successfully with 21 days of IV acyclovir for disseminated neonatal herpes simplex virus (HSV) infection. She is receiving suppressive therapy with oral acyclovir. Question 1 9-1 Which is NOT a potential adverse effect of taking acyclovir? A) Neutrophilia. B) Neutropenia. C) Elevated transaminases. D) Acute kidney injury. E) Uremia. Discussion 1 9-1 The correct answer is ''A:' Antiviral medications are used to treat serious viral infections. Acyclovir may cause neutropenia, acute kidney injury, or acute liver injury. Oseltamivir (Tamiflu) may cause acute psychosis in pediatric patients. It is impossible to know every side effect of every medication. Just remember to consider a drug reaction when formulating your differential diag­ nosis, regardless of whether the medication in question is new. •



Helpful Tip

Acute kid n ey o r l iver i nj u ry and bone ma rrow suppres­

l l lr sion

a re common adverse effects of many d ifferent

medications.

An ECG is obtained. Her QTc is prolonged at 0.48 seconds. Question 1 8-3

True or false: SSRis can cause QT prolongation. A) True. B) False. Discussion 1 8-3 The correct answer is ''A:' SSRis can cause QT prolongation and ventricular arrhythmias (torsades de pointes). Careful consider­ ation should be given to patients with congenital long QT syn­ drome, a history of prolonged QT, or a family history of long QT syndrome. •

Helpful Tip

:5.� Abru pt d iscontin uation

of SSRis without a taper ca n

r1 1r cause d i scontin uation synd rome, with development of fl u l i ke symptoms-d izzi ness, nausea, c h i l l s, a n d muscle

A 5-year-old Caucasian girl weighing 20 kg has a history of intermittent asthma. Her asthma is controlled with an alb­ uterol MDI with a valved-holding chamber. She uses her inhaler and assist device about once a month, which pro­ duces complete resolution of her asthma symptoms. During a school physical exam, her pediatrician diagnoses paroxysmal supraventricular tachycardia and places her on propranolol, 5 mg twice daily. Shortly after she starts the propranolol, her parents notice she is coughing several times a day. Question 20-1 What is the likely mechanism of her coughing? A) Adverse drug reaction from propranolol. B) Drug interaction. C) Worsening of lung disease. D) Viral upper respiratory infection.

aches.





Helpful Tip

It is u nclear if SSRis a re safe in preg nancy. F l u oxetine

1 1 1r is t h e best choice. Pa roxetine increases t h e risk o f con­ gen ita l hea rt d efects.

Discussion 20-1 The correct answer is "B:' Propranolol is a non-cardioselective beta-receptor blocking agent (ie, it blocks beta 1 and beta recep­ 2 tors). Beta -receptor blockade causes bronchospasm. Some 2 beta-receptor blocking agents (eg, atenolol and metoprolol) at lower doses are less likely to aggravate lung symptoms because they selectively block beta receptors in the heart and not the

C H A PT E R 26

lungs. Although cough (option ''A'') is reported in the literature as a side effect of propranolol, given that the patient has a history of asthma, medication-induced bronchospasm is the most likely reason. Option "C" is possible but usually patients with asthma demonstrate more symptoms than just coughing if their asthma is not being controlled. Similarly, option "D" is unlikely without the common additional symptoms associated with a viral upper respiratory infection such as fever, sore throat, or rhinitis.

� •

1 I

Helpful Tip

Patients with asthma have a i rways that a re hyperresponsive to triggers that usua l ly do not cause prob­ lems to nonasth matic patients.

i QUICK QUIZ Which i s an adverse effect o f beta-blocker drugs? A) Tachycardia. B) Hyperglycemia. C) Fatigue. D) Hypertension. E) Hypokalemia. Discussion The correct answer is "C:' Adverse effects of beta-blockers include bradycardia, hypoglycemia, hypotension, and hyperkalemia.

An 8-year-old girl (weight, 40 kg; height, 1 32 em) presents to your clinic with her mother. She is complaining of increas­ ing frequency and intensity of headaches, fatigue, and blurred vision. She was seen by her primary care provider 6 months ago. The blood pressures recorded at that time were 1 1 7/78 mm Hg and 1 19/79 mm Hg (95th percentile) . At that time, she was encouraged to make therapeutic lifestyle changes such as decreasing her sodium intake and increas­ ing her physical activity. She currently has normal renal and hepatic function and is not taking any other medications. Today, she has blood pressure readings of 1 3 1 /89 mm Hg and 133/90 mm Hg (99th percentile) . Question 21 -1 Which of the following is the best initial therapy for her? A) Losartan, 1 00 mg once daily. B) Captopril, 20 mg three times a day. C) Guanfacine, 2 mg at bedtime. D) Amlodipine, 10 mg once daily. Discussion 21 -1 The correct answer is "B:' Captopril has a short half-life, mak­ ing it easy to titrate to the desired effect. Its short half-life decreases the chance of angiotensin-converting enzyme (ACE)



P H A RMACOLOGY: PA I N M A N AG E M ENT A N D S E DATION

585

inhibitor-induced hypotension. Enalapril and lisinopril are other ACE inhibitors that can be used. They have longer half­ lives and only require once- or twice-daily dosing, which may increase adherence and improve quality of life. Losartan, an angiotensin receptor blocker (ARB), and amlodipine, a calcium channel blocker, would both be acceptable as a first-line agent but the dose is too high. Guanfacine, a central alpha-agonist, is not typically used as an initial agent due to the incidence of adverse effects and is usually only used for adjunctive agents in refractory hypertension or in hypertensive emergencies. ACE inhibitors and ARBs interfere with the renin-angiotensin-aldo­ sterone system (RAAS), causing vasodilation and decreased renal sodium and water retention. A brief review of pathophysi­ ology helps clarify the mechanism of action of these drugs. Angiotensinogen is converted to angiotensin I by renin (pro­ duced in the kidneys) . Angiotensin I is converted to the active form angiotensin II by ACE in the lungs. Angiotensin II causes vasoconstriction and release of aldosterone from the adrenal glands. Aldosterone increases renal sodium and water absorp­ tion and potassium excretion. Calcium channel blockers lower blood pressure by peripheral artery vasodilation. Guanfacine blocks sympathetic nervous system activity, resulting in vasodi­ lation and a decreased heart rate. Question 21 -2 Which of the following medications does NOT have direct effect on humoral regulation or RAAS? A) Lisinopril. B) Valsartan. C) Atenolol. D) Aliskiren. Discussion 21 -2 The correct answer is "C:' Atenolol, a cardioselective (at lower doses) beta-blocker, helps regulate blood pressure by acting on parts of the central nervous system. Cardioselective means it antagonizes only beta 1 receptors, which results in reduction of cardiac output through negative inotropic and chronotropic effects. Other beta-blockers, such as propranolol and nadolol, block beta 1 and beta receptors. This additional beta blockade 2 2 can result in pulmonary cross reactivity, but only a minimal num­ ber of pediatric patients are affected. Lisinopril (ACE inhibitor), valsartan (ARB), and aliskiren (direct renin inhibitor) all act on different parts of the RAAS, effecting humoral regulation.

At a follow-up visit 1 month later, the girl's blood pressure continues to be uncontrolled and you decide to add a loop diuretic, furosemide. Question 2 1 -3 Which of the following adverse effects occurs only with loop diuretics and not other classes of diuretics? A) Dyslipidemia. B) Hypochloremic metabolic alkalosis. C) Hyponatremia. D) Ototoxicity.

586

MCG RAW- H I LL E D U C AT I O N S P E C I A LTY BOA R D REVI EW: P E D I AT R I C S

Discussion 21-3 The correct answer is "D:' Ototoxicity is specific to furose­ mide. All diuretics may cause a change in blood cholesterol and electrolyte levels. Electrolyte levels are usually monitored in patients taking diuretics. Although the risk of ototoxicity is rare, it is a significant adverse reaction that should be avoided. The exact mechanism is unknown; however, it is likely dose

related. Using the lowest effective dose of furosemide and giv­ ing divide doses may decrease this risk. Loop diuretics are associated with hypokalemia, hypocalcemia, increased urinary calcium secretion, and nephrocalcinosis (neonates) . Thiazide diuretics are potassium sparing and used to treat hypercalci­ uria (associated with kidney stones) as they decrease urinary calcium excretion.

Part2

Sedation JeffVan Blarcom

An otherwise healthy 9-year-old boy is hospitalized for treatment of encopresis after outpatient treatment has been unsuccessful. Despite having been introduced to the idea of a nasogastric tube prior to admission, he refuses to allow placement of one once in the hospital. In lieu of a struggle, the decision is made to sedate him for the procedure.

may not help, just as in other situations, but is not revered for its use as a sedative per se. (See Table 26-2.)

Question 22-1 Of the following, what would be the most appropriate medi­ cation to sedate a child for a short, likely pain-free procedure? A) Morphine. B) Midazolam. C) Chloral hydrate. D) Diphenhydramine. E) Ketamine.

Question 22-2 Which of the following is properly matched with its definition? A) Minimal sedation: a drug-induced depression of conscious­ ness during which patients cannot be easily aroused but respond purposefully. B) Moderate sedation: a drug-induced state during which patients respond normally to verbal commands. C) Deep sedation: a drug-induced depression of conscious­ ness during which patients respond purposefully to verbal commands. D) General anesthesia: a drug-induced loss of consciousness during which patients are not arousable, even by painful stimulation. E) None of the above.

Discussion 22-1 The correct answer is "B:' The necessity for sedating children in certain situations can and will continue to be debatable, but sometimes it should be considered to make everyone's life easier, most importantly the child's. Classified as a "short-acting" ben­ zodiazepine, midazolam would be the best of the listed options for the task at hand. Nasogastric tube placement is noxious but in most situations is not painful, making an analgesic such as morphine unnecessary. Anxiolysis, or minimal sedation, is what we seek here. A longer acting agent such as chloral hydrate could also be used, particularly if you are planning on taking several hours to get the tube in. Previously chloral hydrate was commonly used in child younger than 3 years of age, but it is falling out of favor as it can take forever to kick in and once it does it has a prolonged effect. Ketamine, which is an analgesic (pain-killer) and a sedative, could also be used, but this proce­ dure should not be painful and ketamine has no reversal agent. Midazolam is also a good amnestic agent. (If the child cannot remember it, did it really happen?) Diphenhydramine may or

Discussion 22-2 The correct answer is "D:' To clarify and simultaneously get all our adverbs straight, minimal sedation implies that a child can respond to commands normally, moderate sedation implies that a child can respond purposefully but not normally, deep sedation implies that a child can respond purposefully but cannot be aroused easily, and general anesthesia implies that a child is not arousable even to painful stimuli. All of these definitions refer to patients who have been drugged, of course. Useful though they are, these definitions imply intended end points, not strictly controlled steady-state conditions. In some children, particularly those who are not communicative, the levels of sedation can be hard to distinguish. It should be noted that use of the term conscious sedation, at least according to the American Academy of Pediatrics (AAP), is no longer considered acceptable, given its rather contradictory literal meaning. The different levels of sedation are correlated with differing levels of airway protection and cardiovascular effects: Minimally sedated children are able to maintain their airway

CHAPTER 26



P H A RMACOLOGY: PA I N M A N AG E M E N T A N D S E DAT I O N

587

TABLE 26-2 M I SC E L LA N E O U S P E D I AT R I C S E DAT I O N D R U G S

Drug

Description

Uses

Keta m i ne

Di ssociative agent that has sedative, a n a l g e ­ sic, a n d a m n estic properties

Com monly used in veterinary med icine ("horse tran­ q u i l izer") but not often used i n adult h u mans due to risk of hypertension, dysphoria, and ag itation

Prese rves a i rway refl exes and has l ittl e effect on res pi ratory d rive

Pro pofo l

Dexmedetom id i ne

C h lora l hyd rate

In pediatric patie nts, rarely causes l a ryngospasm

When g ive n IV, peak o n set of action occ u rs with i n 1 m i n ute a n d d u ration of action i s a bout 1 0- 1 5 m i n utes

Associ ated with a n emergence reacti o n cha racter­ ized by h a l l uci nation, confusion, a n d a d rea m l i ke state

When g ive n IM, peak o n set of action is a bout 5 - 1 0 m i n utes a n d d u ration of action i s 1 5-30 m i n utes

Ca u ses i n c reased i ntra-cra n i a l and i ntra-oc u l a r pressu re a n d s h o u l d b e avoided w h e n either i s i ncreased

Sedative agent without a ny a n lagesic or a m n estic properties

Most freq uent use i s as a n i n d uction agent fo r genera l a nesthesia

Fu ncti o n s s i m i la rly to barbitu rates in the h u ma n body

Ad m i n i stered as a conti n u o u s IV i nfu sion

Rapid o n set of action (with i n 40 seco n d s) a n d short d u ration of action ( 1 -3 m i n utes)

S h o u l d be avoided i n patients who have a n e g g a l l e rgy

A l p h a -ad renerg i c rece pto r (l i ke c l o n i d i ne) 2 that provides sedation without respi ratory depression

Ad m i n i stered as a conti n u o u s IV i nfu sion

Oral sedative, use of which has persi sted despite advent of other sedatives si nce its fi rst med i c i n a l use in the late 1 800s

Conti n u es t o be u s e d beca use o f i t s favo ra ble safety profi l e a n d fa m i l i a rity, a l t h o u g h use i s decrea s i n g

On set of action with i n a bout 20 m i n utes of ora l a d m i n istration

Excessive dosing can cause apnea and hypotension

Reaches its pea k effect at 30-60 m i n utes

Looks l i ke milk a n d conta i n s l i pi d s

Main ped iatric use is for prolonged sedation i n a n i nte nsive ca re sett i n g

(If yo u "s l i p ped someone a M i ckey;' yo u gave that person a d r i n k laced with ch lora l hyd rate)

I M, i ntra m uscu l a r; IV, i ntravenous.

without assistance, and their blood pressure and heart rate are not affected by the medication; moderately sedated children are the same; deeply sedated children may require airway or ventilatory support, and their blood pressure and heart rate may be affected; and anesthetized children will likely have a dismal outcome if not properly supported. The child in our scenario is given a dose of nasal midazolam and, once he is dozing, the tube is successfully placed with a minimum of fuss. Later he asks you where the tube came from. Success! Question 22-3 Which of the following is true? A) Sedation of children should only occur in a hospital setting. B) Experience in placing a larygeal mask airway is necessary for a sedation practitioner. C) A "crash cart" should be available whenever a child is sedated. D) Nitrous oxide is no longer considered to be an acceptable medication for sedating children. E) Propofol should only be administered in the operating room under the auspices of an anesthesiologist.

Discussion 22-3 The correct answer is "C:' Sedation of children need not occur in a hospital setting, but all of the necessary equipment should be read­ ily available. Advanced airway management skills, such as mastery of placement of a laryngeal mask airway, would be helpful but are not a requirement for a sedation practitioner. Skill in tracheostomy would also be helpful, but those who are adept at that do not fre­ quent sedation areas outside of the operating room, nor should they. An ability to effectively ventilate an overly sedated child is a must, however. Pediatric advanced life support (PALS) training is not an absolute requirement for a practitioner providing minimal sedation (anxiolysis), but it is a requirement, per AAP sedation guidelines, for any person providing more than minimal sedation to a child. Anxiolytic medications are frequently given at similar dosages for reasons other than sedation for procedures and gener­ ally do not require elaborate monitoring or observation protocols, but any planned sedation of a greater degree mandates the presence of the proper personnel, equipment, and preparation. Although nitrous oxide (Np), not to be confused with nitric oxide, has been used for medical purposes for a long time, it is still a perfectly acceptable sedative and analgesic for procedural sedation. And it has also proved useful in drag racing. It cannot be used alone for

588

MCG RAW- H I LL E D U C AT I O N S P E C I A LTY BOA R D REVI EW: P E D I AT R I C S

general anesthesia unless a hazardous degree of hypoxia is accept­ able, but it is easy to administer to a child willing to accept a face mask, it is relatively inexpensive, and its effects come and go rap­ idly. It is, however, rather notorious for causing postanesthesia nau­ sea. Propofol is another handy sedative and does not necessarily need to be administered only by an anesthesiologist. It is powerful enough that painful procedures can be performed on an oblivious patient given an adequate dose, but it is not an analgesic per se. The effects of both nitrous oxide and propofol wear off shortly after the inhalation (nitrous oxide) or the IV infusion (propofol) is stopped. (See Table 26-2.) If whatever you did to the patient when he or she was out was painful, it may well hurt after the sedative wears off, so an analgesic agent (painkiller) should be given as well.

� •

Helpful Tip

Sedation may cause res piratory and ca rd iovascu l a r

1 11r d e p ression. Be prepa red to su pport a c h i l d whenever the word sedation is uttered. Grab a m a n i ki n of choice a n d practice you r bag-a n d-mask s ki l ls.

A 9-month-old is discovered to have what sounds like a patho­ logic murmur during a well-child check. He appears healthy, exhibits good growth and development, and has no signs con­ sistent with heart failure. He is sent to get an echocardiogram the following week. For his sedated study, the proper proce­ dures are followed: emergency airway equipment is available, the appropriate personnel are present and properly trained, the child is monitored with continuous pulse oximetry and cardiorespiratory monitoring, the child has been NPO (nil per os) for the recommended amount of time, and the echocardio­ gram technician has showered and is ready to go. Question 23-1 Of the following, what would be the most appropriate medication(s) to sedate a child for an echocardiogram? A) Chloral hydrate, midazolam, and diphenhydramine. B) Pentobarbital. C) Propofol. D) Midazolam. E) Morphine.

Discussion 23-1 The correct answer is "B:' Although echocardiogram techni­ cians might prefer that every child be deeply sedated with a propofol drip so they could get to lunch on time, many would argue that the level of sedation provided by propofol, and the necessity of placing an IV line, would be excessive or even coun­ terproductive. A 9-month-old is unlikely to cooperate enough to ensure an adequate study without sedation, and distraction techniques such as toys, television, pacifiers, and clowns are useful but unlikely to provide the needed tranquility. All of the listed medications can be used to sedate a child for an echo­ cardiogram, but pentobarbital offers the best sedation profile for such an endeavor. Echocardiograms usually take between 1 5 and 90 minutes, and pentobarbital is an intermediate-acting barbiturate that will cover this time frame adequately. Barbitu­ rates are sedative-hypnotic (ie, anxiolytic and soporific) but not analgesic medications, which when dosed appropriately afford the opportunity for the successful completion of an echocardio­ gram. Echocardiograms are not painful procedures and only require sedation, so morphine, which is a sedative and an anal­ gesic, would not be necessary although it could also get the job done. Use the right tool for the right job, some say. Midazolam, a benzodiazepine, would also provide appropriate sedation but might not last long enough for a complete study: Grumpy child, grumpy tech. There is of course no prohibition to giv­ ing a child repeated doses of an agent for a sedated procedure or study, but administering multiple doses is disruptive, as is whatever behavior necessitated another dose. Using a multidrug regimen offers no significant advantage over a single, appropri­ ately dosed agent and can increase the likelihood of variable or unwanted effects. (See Table 26-3.) The echocardiogram proceeds without difficulty and is com­ pleted in 15 minutes. The child's mother has other things to get done that afternoon and is anxious to get away from all things medical. Question 23-2 When is it appropriate to discharge the child to home or errands with a caregiver? A) For narcotics or benzodiazepines, after the reversal agent has been given. B) 1 hour after completion of the echocardiogram. C) Once the child is no longer nauseated.

TABLE 26-3 B E N ZO D I AZ E P I N E S CO M M O N LY U S E D I N P E D I AT R I C S

Common

Time of Peak

Duration of

Commonly Used

Benzodiazepine

Trade Name

Effect

Action

Dosage Forms

M idazo l a m

Versed

0.5-1 h o u r

3 h o u rs

PO, IV, nasal

Lorazepam

Ativa n

2-4 h o u rs

1 0-20 h o u rs

PO, IV

Diazepam

Va l i u m

1 -2 h o u rs

36-200 h o u rs

PO, IV, P R

Al prazo l a m

Xanax

1 -2 h o u rs

6- 1 2 h o u rs

PO

IV, i ntravenous; PO, by mouth; PR, per rect u m .

CHAPTER 26

D) Once the child has returned to baseline neurologic status and vital signs are normal. E) After two half-lives of the sedative agent have passed. Discussion 23-2 The correct answer is "D:' If a child has normal vital signs and has returned to baseline neurologic status after sedation, he or she is likely ready to go. A good test of recovery is the child's demonstration of the ability to drink without difficulty. Recov­ ery from sedation commonly is a nauseating experience and having a child drink without subsequent vomiting is a good sign that he or she is unlikely to arrive at the emergency department later that day or the next dehydrated and unhappy. Vomiting by itself is unpleasant and aspiration events, though uncom­ mon, are best not provoked. Arbitrarily applying a time limit to a child's recovery, as in options "B" and "E;' is not a sound practice given the variable effects that a medication can have on a child. On occasion it may take longer than anticipated for a child to return to baseline neurologic status. Most of the effects of a medication, fortunate and unfortunate, can be antici­ pated and should be known by the practitioner before giving the medication to a patient. Administration of a reversal agent can be an unpleasant experience, which may well be contrary to the sedation goal of minimizing pain and discomfort but prob­ ably would be in order if the choice is reversal or intubation. The child would nonetheless need to be monitored thereafter until he or she has returned to baseline neurologic status. Some awareness of what constitutes normal vital signs for children of a particular age is also a necessity. Parental statements such as "She always has a high heart rate" should be accepted diplomati­ cally but should not be unquestioned, in a sedation recovery or otherwise. Question 23-3 The most common adverse events encountered in the seda­ tion of children by nonanesthesiologists are of what nature? A) Cardiac (eg, arrhythmia, arrest). B) Respiratory (eg, hypoxia, hypoventilation) . C) Inadequate sedation. D) Oversedation. E) Aspiration. Discussion 23-3 The correct answer is "C:' Inadequate sedation with subsequent failure to complete a study may not be the worst adverse out­ come, but it is nonetheless an adverse outcome. In a study by Cote and colleagues from 2000, 1 3 % of sedated children were inadequately sedated. The next most common category of adverse events involved the respiratory system. The events in this category included respiratory depression, most commonly oxygen desaturation (54 of 1 140, or 4.7%), and a rare neces­ sity for positive-pressure ventilation (2 of 1 140) . Thankfully, adverse events that result in long-term morbidity or mortality are uncommon; most of these can be attributed to medication errors, lack of appropriate monitoring or personnel, or sending a child home before appropriate recovery. Vigilance is always in order.



P H A RMACOLOGY: PA I N M A N AG E M E N T A N D S E DAT I O N

589

i QUICKQUIZ Which medication does NOT provide analgesia? A) Propofol. B) Ketamine. C) Morphine. D) Nitrous oxide. E) All of the above provide analgesia. Discussion The correct answer is ''A:'



Helpful Tip

:5.� Sedation

is not the same as a n a lgesia. Know the

i1 1r i n d ications a n d

effects of the med ications that a re

used. Chem ica l d i straction d oes not treat pa i n .

A 4-year-old boy i s brought to the emergency department in the early afternoon after falling while trying to learn to ride his new bike. The bike was not significantly damaged but the same cannot be said of the boy's left forearm, which is obviously deformed. The child appears to be in a fair amount of pain. There is no vascular or neural compromise distal to the injury, so you have some time to get some history; notably: He is an otherwise healthy child who has not had anything to eat since a healthy, well-balanced breakfast early that morning. His most recent ingestion was some ginger ale on the way to the hos­ pital. He has never been sedated or been operated upon. The remainder of his physical examination is normal. An IV line is successfully placed and he is given a dose of morphine to good effect. An X-ray confirms a moderately displaced radial frac­ ture, which is not a sustainable position for the bone or the child in general, so preparations are made to reset the fracture. Question 24-1 For a nonemergent procedure requiring sedation, what is the minimum recommended time between the ingestion of solid food and the administration of a sedative? A) 1 hour. B) 2 hours. C) 6 hours. D) 8 hours. E) 12 hours. Discussion 24-1 The correct answer is "C:' NPO status is an element of the child's history that should be taken into account prior to sedation. Most sedating medications can also cause nausea and we do not want any partially digested breakfast items in our patient's right mainstem bronchus, especially if the procedure for which the child is to be sedated is not emergent. Other historical elements

590

MCG RAW- H I LL E D U C AT I O N S P E C I A LTY BOA R D REVI EW: P E D I AT R I C S

TABLE 26-4 ASA PHYSICAL STAT U S CLASS I F I CATION

Class

Description A normally hea lthy pati ent

II

A patient with m i ld syste m i c d i sease (eg, contro l led reactive a i rway d i sease)

Ill

A patient with severe syste m i c d i sease (eg, a c h i l d who i s actively wheezing)

IV

A patient with severe syste m i c d i sease that i s a con sta nt th reat to l ife (eg, a c h i l d with statu s asth maticus)

v

A mori b u n d patient who is not expected to s u rvive without the operation (eg, a patient with severe card i o myopathy)

ASA - American Society of Ansthesiolog i sts

that should be evoked are gathered in a standard history and physical exam, with the addition of any past experiences with sedation or anesthesia and any family history of problems with anesthesia. Although this child is described as a normal, healthy child, an assessment of his neck and his airway should be under­ taken such that the sedating provider will be aware of how dif­ ficult it will be to ventilate or intubate the child should that be necessary, or if the child is even safe to be sedated without first securing his airway and ventilating him with the help of an anes­ thesiologist and an anesthesia machine. The overall assessment of the child's health can be used to stratify the risk of an adverse event using the American Society of Anesthesiologists' (ASA) classification system. It is recommended that anesthesiology be consulted for sedations of children in classes III and higher. (See Table 26-4.) A child's Mallampati classification, which is determined by looking into the oropharynx while the child, in a sitting position, sticks out the tongue, is a proxy for determining the difficulty of endotracheal intubation. If you can see the base of the uvula and the tonsillar pillars, and the child is class I, it should be an easy intubation. If all you can see is the palate, the child is class IV (hard intubation) and you may not be able to get a tube in the right spot if need be. If you can successfully bag­ mask-valve ventilate and place a nasogastric tube, you still may be on solid ground, however. Ginger ale can be considered a clear liquid, which requires a 2-hour waiting period; solid items such as pancakes and sausage require at least a 6-hour waiting period. Breast milk requires a shorter waiting period than infant formula. Yet another plug for breast feeding. (See Table 26-5.)

TABLE 26-5 ASA 2 0 1 1 P R E O P E RATIVE FA ST I N G

R E CO M M E N DAT I O N S

Clear l i q u i d s

2 h o u rs

B reast m i l k

4 h o u rs

I nfa nt for m u l a

6 h o u rs

S o l i d food

6 h o u rs

ASA, American Society of Ansthesiolog ists.

During the time in which all of the appropriate personnel and equipment are gathered for sedating the child and reset­ ting his fracture, his father continues to appear very anxious. He eventually says that his son is in pain again. Question 24-2 How would the child's pain be best treated now? A) Give a dose of acetaminophen. B) Tell his father to wait in another room, given that he seems to be heightening the anxiety level. C) Give another dose of opioid. D) Put the boy's favorite movie on. E) Give a dose of diazepam. Discussion 24-2 The correct answer is "C:' The overriding principle in the admin­ istration of opioid medications for the relief of pain is: Titrate to effect. There is no need to let him suffer. Weight-based dosing should be viewed only as a starting point. Dosing of sedatives in general is a guess based on medical experience; the same dose of a particular drug could have different effects on different chil­ dren. We know the effects of opioids, wanted and unwanted, but those should not deter us from using opioids when they will be helpful. Acetaminophen may not be sufficiently potent and likely will take longer than desired to have an effect, but would not be contraindicated. Giving a dose of another opioid would also be acceptable, assuming it has a reasonably fast onset and short duration of action and assuming that the child's fracture is going to be reset in a timely fashion. Perhaps fentanyl may be used, but not methadone or oxycontin (a sustained-release form of oxycodone). Morphine, in this setting, would be use­ ful because it has a rapid onset of action if given IV and the effect will last for quite some time. The child's arm is likely to cause some pain for a while. A benzodiazepine would be useful in alleviating some of his anxiety, but benzodiazepines do not have analgesic properties and pain is the bulk of his problem at this point. Diazepam has a long duration of action and may well complicate both the child's sedation and recovery from seda­ tion, so would not be a desirable medication here. The euphoric effect of an opioid likely will alleviate most of the child's anxiety, but it may not be unwarranted to add a anxiolytic. Quite pos­ sibly option "B" is the only inappropriate answer here. We cer­ tainly see a fair amount of secondary gain-influenced behavior in pediatrics, but this is a situation in which a child has an obvi­ ous need for pain relief and removing an anxious parent will only exacerbate the child's discomfort. No doubt part of being a pediatrician is the ability to calm an anxious parent. If you coin­ cidentally see that the child has lice, now may not be the best time to mention it. In a medical setting where children can be anticipated to be anxious or in pain, caregivers, stickers, dolls, toys, music, movies, goofy employees, child life, and other such nonpharmocologic modalities should always be available and liberally utilized. Suppose a child with osteogenesis imperfecta comes to medical attention with a new fracture. He takes opi­ oids for pain on a regular basis. Do not start with, or default to, standard weight-based dosing. Talk to his caregivers to find out

CHAPTER 26

what works for him; he almost certainly will need more than an opioid-nai"ve patient. Question 24-3 Concerning opioids, which of the following is NOT true? A) Heroin was initially synthesized as an alternative to morphine. B) Opioids comprise opiates, synthetic compounds, and endogenous compounds such as endorphins. C) Narcotics and opiates are one and the same. D) Morphine can be considered to be the prototypical opiate agonist. E) Opiates are derived from opium alkaloids from the poppy plant. Discussion 24-3 The correct answer is "C:' If we are to discuss sedatives and anal­ gesics, we need to know what we are talking about in specifics. All opiates are narcotics but not all narcotics are opiates. A narcotic, as defined in medicine, is a substance that induces a state of stupor, which includes all legal and illegal "mind-altering" substances­ opiates, marijuana, and cocaine, to name a few. Opiates are opium alkaloids derived directly from the poppy plant. Opioids include the substances listed above in option "B:' Fentanyl, as an example of a synthetic opioid, is produced through a complicated process that does not include anything from the poppy. (See Table 26-6.)

� •

I



P H A RMACOLOGY: PA I N M A N AG E M E N T A N D S E DAT I O N

particularly in older children and adults, but are not known to occur with opioid administration. Chest wall rigidity is a fright­ ening occurrence that is a known side effect of fentanyl, a much more potent and faster acting opioid than morphine. As we have discussed, fentanyl is useful for short, painful procedures such as the resetting of a displaced fracture, though other newer agents such as propofol have gained acceptance for use in emer­ gency departments. Chest wall rigidity can make ventilating a sedated patient very difficult; thankfully we have naloxone to reverse the effects of any opioid. Naloxone should be nearby any time an opioid is given, especially in a controlled environment like an inpatient unit or emergency department. All of these effects should make the necessity of proper monitoring appar­ ent. Constipation, nausea, vomiting, and pruritis are also well­ known effects of opioids. Allergic reactions, from urticaria up to anaphylaxis, are possible. Remember the history and physical exam? A discussion of allergies is part of that, but that does not eliminate the possibility of an allergic issue. Morphine causes histamine release and thereby causes itching, but this reaction is not an allergy per se. Synthetic opioids are useful for opiate­ allergic patients (notably fentanyl, meperidine, and methadone) . •

Helpful Tip

:5.� N a l oxone is freq u ntly g iven to reverse opioid-i nd uced

r1 1 r res p i ratory depress ion. F u l l reversa l wi l l ta ke away a l l

a n a lgesia leaving you with a patient writ h i n g i n pa i n .

Helpful Tip

Abrupt reversa l i n a opioid-dependent patient may

Opioid medications g iven s h o u l d be titrated to effect

1 1r not

591

lead to seizu res and withd rawa l . Use the sma l l est dose

solely based on recommended weight-based

poss i b l e to reverse the u nwa nted effect while kee p i n g

dosi n g . No two patients a re a l i ke. A patient dependent

the desired effect.

on opiods wi l l need higher doses so do not sta rt at the same dose a s a n opioid-na "lve patient.

Question 24-4 Which of the following is NOT a known complication of opi­ oid administration? A) Chest wall rigidity. B) Respiratory depression. C) Emergence hallucinations. D) Hypotension. E) Anaphylaxis. Discussion 24-4 The correct answer is "C:' Emergence phenomena such as delir­ ium or hallucinations are known adverse effects of ketamine,

The boy in question is properly sedated with propofol and his fracture is promptly and properly reset. Perhaps we should dis­ cuss what kind of monitoring was used during his procedure. Question 24-5 Which of the following is NOT considered a necessity when monitoring a moderately or deeply sedated patient? A) Continuous pulse oximetry. B) A licensed practitioner whose sole responsibilty is monitoring the patient. C) Continuous cardiorespiratory monitoring. D) Blood pressure measurements every 5 minutes. E) End-tidal carbon dioxide monitoring.

TABLE 26-6 O P I O I D S CO M M O N LY U S E D I N P E D I AT R I C S

Opioid

Common Trade Name

Duration of Action

Commonly Used Dosage Forms

Morph i n e

(Ma ny)

4-5 h o u rs

PO, IV, I M

Hyd romorphone

Dilaudid

4-5 h o u rs

PO, IV

Oxycodone

OxyCo nti n

3-4 h o u rs

PO, IV

Fenta nyl

D u ragesic

1 - 1 .5 h o u rs

IV

Meth adone

Dolop h i n e

4-6 h o u rs

PO, IV

I M, i ntra m uscu l a r; IV, i ntravenous; PO, ora l .

592

MCG RAW- H I LL E D U C AT I O N S P E C I A LTY BOA R D REVI EW: P E D I AT R I C S

Discussion 24-5 The correct answer is "E:' End-tidal C0 monitoring is useful 2 for monitoring sedated patients, particularly if the view of their body is obstructed by something like a magnetic resonance imaging (MRI) scanner, but currently is not seen as a necessity for most sedation situations. The other listed safety measures are seen as mandatory. The most important of these is the seda­ tion provider, who, by paying close attention to the patient, can avert most untoward occurences. All sedation of whatever level is indeed sedation, so the practitioner responsible for the child should be prepared for things to go awry. Our accepted termi­ nology regarding the levels of sedation, namely minimal seda­ tion, moderate sedation, deep sedation, and general anesthesia, represents a spectrum of altered mental status and children can certainly go from one state to another without announcing their intentions. Adverse events and outcomes are rare, but there is no excuse not to be ready. A working knowledge of the medica­ tions to be used and airway management skills are a necessity. This may sound intimidating, but a calm demeanor and knowl­ edge of the workings of a bag-valve-mask contraption and its intended effects on the human body should keep everyone clean and out of trouble. All obligatory sedation equipment can be summarized by the mnemonic SOAP ME: S - Suction (a suction apparatus and an appropriate suction catheter) 0- Oxygen (a supply of oxygen and a delivery apparatus; eg, nasal cannula)

A- Airway (airway management equipment; eg, nasopha­ ryngeal and oropharyngeal airways, laryngoscope and blades, endotracheal tubes, stylets, face mask, bag-valve-mask or equivalent device) P - Pharmacy (drugs needed to support life during an emergency, including antagonists such as naloxone and flumazenil) M - Monitors (pulse oximeter with appropriate probes, sphygmomanometer, end-tidal carbon dioxide monitor, ECG/cardiorespiratory monitor, stethoscope [old-fashioned but useful] ) E- Equipment (special equipment for particular cases; eg, defibrillator)

An 1 1 -year-old boy is hospitalized for treatment of menin­ gitis. The infection appears to be viral based on his labora­ tory studies, but bacterial infection has not been ruled out; therefore he is admitted for IV antibiotics at least for a day or two while the blood and spinal fluid cultures mature. Beyond that, his head pain is probably significant enough to get him admitted. He was given a IV dose of morphine in the emergency department prior to admission and now he is upstairs, obviously uncomfortable. He likes the room dark because "the light hurts his head:' he does not want to move

his head, and he says that his back hurts at the site of his lum­ bar puncture. He is stable both from a respiratory standpoint and from a cardiovascular standpoint, although he is some­ what tachycardic with a heart rate of 1 10 beats per minute, which seems most likely to be due to his pain and the anxiety of being in a unfamiliar environment. Question 25-1 Of the following, which option would be a good first step in controlling his pain? A) Ibuprofen on demand. B) A PCA (patient-controlled analgesia) machine. C) Acetaminophen on demand. D) Fentanyl on demand. E) Regularly scheduled acetaminophen. Discussion 25-1 The correct answer is "E:' It could be reasonably assumed that his pain is going to last for a while, perhaps a day or two, and we should have it within our power to alleviate most of his pain. Regularly scheduled analgesia is likely to be more beneficial than on-demand dosing, so the on-demand options probably are not a good starting point. Appropriately and regularly dosed acet­ aminophen will be metabolized in due course by a healthy liver, and as such will be safe quite possibly to give for many days, should the need arise or should you be questioned by a parent. Pain from meningitis most often does not rise to the point of necessitating a PCA machine. PCA would not be the first step in controlling the pain of the child in question, although if the child is requiring opioids at a regular interval, it may be appro­ priate to take that step. A patient in certain situations can be anticipated to benefit from PCA and an acute pain service con­ sult, if the service is available. Pancreatitis or a major operation such as an open reduction and internal fixation of a fractured femur, for instance. Do not, under any circumstances, ask the acute pain service consultant about chronic pain control, yours or anyone else's, without the expectation of redirection. Question 25-2 What is the pain-killing mechanism acetaminophen? A) Unknown. B) Anti-inflammatory property. C) Inhibition of cyclooxygenase. D) Endorphin effect. E) Placebo-like effect.

of

action

of

Discussion 25-2 The correct answer is ''A:' The mechanism of action of acetamin­ ophen is indeed unknown, but it is effective in reducing fever and controlling pain. NSAIDs (nonsteroidal anti-inflammatory drugs)-notably aspirin, ibuprofen (Motrin, Advil) , ketoralac (Toradol) , and naproxen (Aleve, Naprosyn) -block the action of cyclooxygenase 1 and 2 in the inflammatory process, thus diminishing pain related to inflammatory conditions, such as injury and infection.

CHAPTER 26

By the way, increasing the amount of ibuprofen or acetamin­ ophen is unlikely to be fruitful unless they were underdosed from the start. There appears to a ceiling for the effectiveness of both of these drugs, unlike the effect of opioids.

� •

I

1 1r

Helpful Tip

Patient-contro l l ed a n a l g esia (PCA) is com plex a n d s h o u l d not be i m plemented by u ntra i ned person nel, although a worki ng knowledge of the s u bj ect is usefu l for a ny c l i n icia n worki ng i n a n i n patient setti n g . The P C A device u s ua l ly req u i res a n awa ke patient of somewhat s o u n d m i nd, a b l e to comprehend what it is i ntended to d o. C h i l d ren you nger than perhaps age 5 years a re not l i kely to u nd e rsta n d its u se, even if d i l igently i n structed. The PCA d evice a l lows the patient to receive a prog ra m m ed a m o u nt of opioid at a progra m med m i n i m u m i nterva l, a n d if need be, a d m i n ister a basal (conti n u o u s) rate of opioid. That may be co m m o n knowledge, but the fol lowi n g a re the pa ra meters that m u st be set for the device to fu nction, for which there is no s n a p py m n e m o n ic: Loa d i n g bolus C l i n ic i a n bolus N u m ber of c l i n i c i a n bol uses per h o u r Lockout (ti m e i nterva l) Basal medication i nfusion rate •

Tota l drug over time



Maxi m u m n u m ber of patient demand doses per hour

Obviously this is not for the u n i n itiated. Additional ly, terms can va ry depen d i n g on the m a n ufactu rer. More i nformation a bout PCA devices can be obta i n ed d u ri n g a n a n esthesia residency.

The child's mother expresses concern, not about acetamino­ phen, but about the possibility of addiction if he gets "a lot of narcotics:' Question 25-3 Which of the following is true? A) Physical dependency can be anticipated after 2 weeks of regular opioid use. B) Physical dependency can be anticipated after 2 days of con­ tinuous use (ie, a drip). C) Dependency is not common when appropriate dosing of opioids is used for an appropriate indication. D) Psychological dependency can be reasonably anticipated after three doses of opioid. E) Withdrawal symptoms are unavoidable after the adminis­ tration of more than 1 . 5 mg/kg of fentanyl or an equipotent dose of another opiate. Discussion 25-3 The correct answer is "C:' You reassure his mother that in the anticipated amount of time during which his pain



P H A RMACOLOGY: PA I N M A N AG E M E N T A N D S E DAT I O N

593

appropriately necessitates medication, dependency is highly unlikely to occur. Additionally, and more importantly, he is unlikely to experience any symptoms of withdrawal once he no longer needs any pain medication. He may well become constipated, which is a problem that you address with his mother. Although the likelihood of dependency in a particular situation cannot be accurately calculated, it can be predicted, and in some situations has been quantified: After a cumulative dose of greater than 60 mg/kg of midazolam, there is a 1 7% to 30% incidence of withdrawal symptoms; after a total fen­ tanyl dose of greater than 1.5 mg/kg there is a greater than 50% chance of withdrawal symptoms (not "unavoidable") ; and after a continuous infusion of fentanyl for more than 9 days, 1 00% of infants developed withdrawal symptoms. Now, having embed­ ded those numbers into your fund of knowledge, you should also keep in mind that any time a patient has been given a significant amount of benzodiazepine or opioid, a high index of suspicion for withdrawal is warranted. What constitutes a "significant amount" can be left to the eye of the beholder, but either withdrawal or addiction would be extremely unlikely in the scenario presented here. BIBLIOGRAPHY American Academy of Pediatrics Committee on Infectious Diseases. The use of systemic fluoroquinolones, policy statement. Pediatrics. 2006; 1 1 8 : 1 287- 1 292. American Academy of Pediatrics, American Academy of Pedi­ atric Dentistry, Cote CJ and the Work Group on Seda­ tion, et al. Guidelines for monitoring and management of pediatric patients during and after sedation for diag­ nostic and therapeutic procedures: An update. Pediatrics. 2006; 1 1 8:2587-2602. Bauer LA, eds. Applied Clinical Pharmacokinetics. 2nd ed. New York, NY: McGraw-Hill; 2008. Belay ED, Bresee JS, Holman RC, et al. Reye's syndrome in the United States from 1 9 8 1 through 1 997. N Engl l Med. 1 999;340: 1 377- 1 382. Benavides S, Nahata M. Pediatric Pharmacotherapy. Lenexa, KS: American College of Clinical Pharmacy; 20 1 3 : 1 55- 1 78. Bhutta AT, Savell VH, Schexnayder SM. Reye's syndrome: Down but not out. South Med f. 2003;96 ( 1 ) :43-45. Cote CJ, Karl HW, Notterman DA, Weinberg, JA, McCloskey C. Adverse sedation events in pediatrics: Analysis of medications used for sedation. Pediatrics. 2000; 1 06( 4) :633. Grossman ER, Walchek A, Freedman H. Tetracyclines and per­ manent teeth: The relation between dose and tooth color. Pediatrics. 1 9 7 1 ;47:567-570. Guerrini R. Epilepsy in children. Lancet. 2006;367:499-524. Gulian JM, Gonard V, Dalmasso C, et al. Bilirubin displace­ ment by ceftriaxone in neonates: Evaluation by determi­ nation of free bilirubin and erythrocyte-bound bilirubin. J Antimicrob Chemother. 1 987; 1 9 :823-829. Hart LS, Berns SD, Houck CS, et al. The value of end-tidal C02 monitoring when comparing three methods of conscious sedation for children undergoing painful procedures

594

MCG RAW- H I LL E D U C AT I O N S P E C I A LTY BOA R D REVI EW: P E D I AT R I C S

in the emergency department. Pediatr Em erg Care. 1 997; 1 3 : 1 89 - 1 9 3 . H o CW, Loke KY, Lim YY, Lee Y S . Exogenous Cushing syn­ drome: A lesson of diaper rash cream. Harm Res Paediatr. 20 14;82( 6):4 1 5-4 1 8. Hoeger PH, Stark S, Jost G. Efficacy and safety of two different antifungal pastes in infants with diaper dermatitis: A ran­ domized, controlled study. J Eur Acad Dermatal Venereal. 20 1 0;24(9) : 1 094- 1 098. Lexi-Drugs. Lexicomp. Hudson, OH: Wolters Kluwer Health, Inc. Available at: http:/ /online.lexi.com. Accessed January 1 6, 20 1 5 . Malviya S, Voepel-Lewis T, Tait AR. Adverse events and risk factors associated with the sedation of children by non­ anesthesiologists. Anesth Analg. 1 997;85: 1 207- 1 2 1 3 . National Heart Lung, and Blood Institute. Guidelines on asthma. [April 20 1 2 . ] http://ww.nhlbi.nih.gov/health­ pro/ guidelines/ current/ asthma-guidelines/full-report. Accessed on June 1 , 20 1 5. Practice guidelines for preoperative fasting and the use of pharmacologic agents to reduce the risk of pulmonary aspiration: Application to healthy patients undergo­ ing elective procedures: An updated report by the American Society of Anesthesiologists Committee on Standards and Practice Parameters. Anesthesiology. 2 0 1 1 ; 1 14(3) :495-5 1 1 . Rybak LP. Pathophysiology of furosemide ototoxicity. J Otolar­ yngol. 1 982; 1 1 (2) : 1 27- 1 3 3 . Saez-llorens X , Macias M, Maiya P, e t al. Pharmacokinetics and safety of caspofungin in neonates and infants less than 3 months of age. Antimicrob Agents Chern other. 2009;53 (3):869-875. Sanborn PA, Michna E, Zurakowski D, et al. Adverse car­ diovascular and respiratory events during sedation of

pediatric patients for imaging examinations. Radiology. 2005;237:288-294. Stamatas GN, Tierney NK. Diaper dermatitis: Etiology, manifestations, prevention, and management. Pediatr Dermatol. 2 0 1 4;3 1 : 1 -7. Subcommittee on Attention-Deficit/Hyperactivity Disorder, Steering Committee on Quality Improvement and Man­ agement. ADHD: Clinical practice guidelines for the diagnosis, evaluation, and treatment of attention-deficit/ hyperactivity disorder in children and adolescents. Pediat­ rics. 20 1 1 ; 128: 1 007 and suppl. Thyagarajan B, Deshpande SS. Cotrimoxazole and neonatal kernicterus: A review. Drug Chern Toxicol. 2 0 1 4;37: 1 2 1 - 1 29. Tobias JD. Tolerance, withdrawal, and physical dependency after long-term sedation and analgesia of children in the pediatric intensive care unit. Crit Care Med. 2000;28( 6):2 1 22-2 1 32. Todd SR, Dahlgren S, Traeger MS, et al. No visible den­ tal staining in children treated with doxycycline for suspected Rocky Mountain spotted fever. J Pediatr. 20 1 5 ; 1 66: 1 246- 1 2 5 1 . U.S. Food and Drug Administration. MedWatch: Th e FDA safety information and adverse event reporting system. [June 3, 2 0 1 5 . ] http:/ /www.fda.gov/safety/medwatch/ default.htm. Accessed on June 5, 20 1 5. Warden CN, Bernard PK, Kimball TR. The efficacy and safety of oral pentobarbital sedation in pediatric echocardiogra­ phy. J Am Soc Echocardiogr. 2 0 1 0;23 ( 1 ) :33-37. Yldzda D, Yapcoglu H, Ylmaz HL. The value of capnography during sedation or sedation/analgesia in pediatric minor procedures. Pediatr Emerg Care. 2004;20 : 1 62- 1 65.

Poisoning and Environmental Exposure to

27

Hazardous Substances Christopher Hogrefe

A 2-year-old boy is brought into the emergency department with altered mental status, appearing lethargic. The boy was found in his parents' bathroom surrounded by various bottles, including household cleaners, over-the-counter medications, and a couple of unspecified prescription medications. He has otherwise been healthy, and his recent 2-year-old well-child visit was unremarkable. The boy has received all of his vacci­ nations and has no known sick contacts. Question 1-1 What information should be gathered next to assist in treat­ ing this ill-appearing patient? A) The specific household cleaners found at the scene. B) The names of the over-the-counter medications in the bathroom. C) The types and doses of medications prescribed to the parents. D) Any interventions that may have been administered prior to arrival. E) All of the above. Discussion 1-1 The correct answer is "E:' The approach to a patient with an unknown ingestion should start with gathering as much infor­ mation about the scene as possible. The patient's vital signs and physical examination can help immensely in identifying poten­ tial etiologies, but care should be given to understanding the likely causative agents based on exposure. Collecting insight into the facts suggested above will help direct conversations with the Poison Control Center and guide potential interventions. While evaluating the patient you find that he is suffering from epistaxis. Inspection of his diaper reveals a red tinge to the front of his diaper.

Question 1-2 What additional inquiry may further your diagnosis and potential treatment? A) Has the child been ill recently? B) Are there any alternative/complimentary medications in the home? C) Has the child had any traumatic injuries lately? D) All of the above. E) None of the above. Discussion 1-2 The correct answer is "B:' Option "!\' has previously been addressed, and option "C'' is unlikely given the distribution of the patient's symptoms. However, option "B" references an important aspect of the evaluation of patients with suspected ingestions that should not be ignored. Approximately 38% of adults in the United States, and an additional 12% of children, use alternative/complimentary treatments. Such medications include glucosamine, garlic supplements, ginseng, herbal pills, and scores more. In this case, fish oil toxicity can lead to hem­ orrhagic symptoms, including but not limited to epistaxis and hematuria. There have even been case reports of hemorrhagic stroke. Be sure to inquire about alternative/complimentary medications in poisoning cases. The Poison Control Center can be an extremely valuable resource when treating patients with suspected poisonings. Question 1-3 What information is not available from the Poison Control Center? A) Guidance directly to families. B) Disposition recommendations. C) Treatment options and dosages. D) Diagnostic considerations. E) Additional details regarding the patient's past medical history.

595

596

MCG RAW- H I LL E D U C AT I O N S P E C I A LTY BOA R D REVI EW: P E D I AT R I C S

Discussion 1-3 The correct answer is "E:' Contacting the Poison Control Center (by dialing 1 -800-222- 1 222 from anywhere in the United States) is an important step in the management of suspected poison­ ings. In fact, it is a tremendous resource for families as well, given that an estimated 90% of pediatric poisonings can be managed from home when the Poison Control Center is con­ tacted before medical intervention is sought. With that said, the Poison Control Center can provide useful information in the management of poisonings, as listed above. However, they are not a central repository for personalized medical information. That information should be obtained elsewhere (ie, from the patient, family, the hospital's medical record system, etc).



Helpful Tip

:5.� I n

patients with sus pected but u n known i n g estions,

r1 1r a l ways consider the possi b i l ity of m u ltiple i n g estions,

pa rti c u l a rly in cases of potential su icide. Utilizing the

Poison Control Center ( 1 -800-222- 1 222) can be a very va l ua b l e resou rce in diagnosing a n d treating these patients.

NISDJI� He uP.

1-800-222-1222

Left, Reproduced with permission from the C h i l d ren's Hospital

effective in producing emesis; however, it has not been shown to be effective at improving outcomes. The American Academy of Pediatrics (AAP) recommends that ipecac not be stored in the home. In fact, the United States Food and Drug Admin­ istration (FDA) is considering classifying it as a prescription­ only medication. The other answers are all important points to highlight when counseling families on preventing poisonings, including advising children of the risks of exposure to com­ mon household products and the medications found in their homes.

� QUICKQUIZ Which of the following regarding ingestions and poisonings is false? A) Always consider complimentary/alternative medications in your differential for toxic ingestions. B) Remember to utilize the Poison Control Center as a valu­ able resource in diagnosing, treating, and dispositioning patients. C) Counseling parents and children can help in preventing future ingestions. D) Remember to consider the possibility that multiple sub­ stances were ingested. E) Encourage families to keep syrup of ipecac in the home but to call the Poison Control Center before administering to a child after ingestion.

of Pitts b u rg h of U PMC a n d the Pitts b u r g h Po ison Center; R i g ht, Reprod u ced with permission from the U.S. Depa rtment of Health a n d Human Services.

The 2-year-old makes a full recovery. Before discharge and during future well-child visits, it is important to counsel families regarding the prevention and treatment of acciden­ tal ingestions or poisonings. Question 1-4 Which of the following is NOT worth including during such a discussion? A) Using cabinet and drawer locks. B) Elevating substances out of reach. C) Keeping ipecac accessible in the house. D) Using childproof bottles and containers. E) Counseling children directly on the general dangers of com­ mon household substances and medications.

Discussion 1-4 The correct answer is "C:' Syrup of ipecac was formally rec­ ommended to help induce vomiting following a potential toxic ingestion. Subsequent studies have shown that it is very

Discussion The correct answer is "E:'

A 1 5-year-old girl on the high school track and field team has been suffering from a particularly painful case of bilat­ eral medial tibial stress syndrome (ie, shin splints) over the past week. In order to continue training for the upcoming district track meet she has been taking an over-the-counter oral medication three times a day along with applying an over-the-counter topical cream on her bilateral shins. One morning before school her parents find her lying on her bed complaining of abdominal pain, nausea, mild dizziness, and a ringing in the ears. The patient presents to your clinic sec­ ondary to these symptoms. Question 2-1 To what clinical toxidrome do you attribute her presentation? A) Acetaminophen. B) Salicylates. C) Tricyclic antidepressants. D) Topical anesthetics (eg, lidocaine) . E) Opioids.

CHAPTER 27



P O I S O N I N G A N D ENVI RO N M E NTA L EXPOS U R E TO HAZA R D O U S S U BSTA N C E S

Discussion 2-1 The correct answer is "B:' Abdominal pain and nausea are common symptoms that can be associated with numerous medications, including acetaminophen, non-steroidal anti­ inflammatory drugs (NSAIDs), and opioids. Even dizziness can result from some of these medications. However, ringing in the ears should raise one's suspicion for salicylate toxicity or NSAIDs, as opposed to the other options. Aspirin (a salicylate) is readily accessible as an over-the-counter medication, and it is also found in numerous over-the-counter analgesic creams. Patients can exhibit systemic symptoms from topical anesthet­ ics such as lidocaine (eg, LMX, Bactine) , including dizziness. However, systemic toxicity requires very large amounts and typically results in cardiovascular (bradycardia, arrhythmias, cardiac arrest) or neurologic (altered taste, paresthesias, agi­ tation, seizures) manifestations, or both. Lastly, keep in mind that several other commonly used household products include salicylates, including oil of wintergreen, antiseptic mouth­ washes, and bismuth subsalicylate (ie, Pepto-Bismol) . Ingesting as little as 5 mL of wintergreen oil can be fatal in a child.



Helpful Tip

.s.::'Jl The use of aspirin i n c h i l d ren has been associated with

i1 1r Reye synd rom e, a potentia l ly l eth a l cond ition that ca n

result in n u merous sym ptoms, i n c l u d i n g fatty l iver, hyperrefl exia, cerebra l edema, a n d m u ltiorg a n fa i l u re.

The AAP advises a g a i nst the use of aspirin i n febri l e i l l nesses for patients you nger than 1 9 years o l d . The u n i ntentional i n gestion of aspirin by c h i l d re n has also d ropped a s a resu lt of d ecrea s i n g the d ose of chewa ble, fl avored as pirin ta blets to 8 1 mg, l i m iting the n u m ber of ta b l ets per bottle to 36, a n d e m phasizing c h i l d ­ resistant bottles.

You obtain a salicylate level on the patient. It is 35 mg/dL, within the range of mild to moderate toxicity. Question 2-2 What should you utilize to guide your management now and moving forward? A) Clinical symptoms. B) Arterial blood gases (ABGs) . C) Actual salicylate level and its trend after treatment. D) Done nomogram. E) Rumack-Matthew nomogram. Discussion 2-2 The correct answer is "A:' The patient in this case is suffering from chronic salicylate toxicity rather than an acute expo­ sure. This alters the utility of the Done nomogram, which is only useful in acute salicylate ingestions. Additionally, many have argued that this nomogram is of minimal clinical signifi­ cance at best given that it does not predict the need for therapy. The actual salicylate level is of importance; however, it is less

597

important in chronic salicylate exposure and should not trump a patient's clinical symptoms. ABGs are beneficial and may be helpful in guiding some aspects of treatment, but this test is not more significant than the clinical presentation. Lastly, the Rumack-Matthew nomogram is germane to a different toxi­ drome (discussed later) . Th e results o f her laboratory tests begin to return. She is found to have an anion gap metabolic acidosis and a pH of 7. 1 on her ABGs. Question 2-3 Given the patient's presentation, which treatment should you initiate for her salicylate toxicity? A) Activated charcoal. B) Intravenous sodium bicarbonate. C) Acetazolamide. D) Both A and B. E) Both B and C. Discussion 2-3 The correct answer is "D:' This question is difficult given the chronic nature of the ingestion. However, it is still recom­ mended that patients who have consumed salicylates be admin­ istered activated charcoal. Some authorities have advocated for multidose activated charcoal, but this remains controversial. Indications for urinary alkalization include a rising salicylate level, metabolic acidosis, and serum concentration greater than 30 mg/dL in acute ingestions. Sodium bicarbonate should be used to help enhance the elimination of the salicylate, in the process also alkalinizing the urine. Acetazolamide is contrain­ dicated in salicylate ingestions as it causes the renal elimination of bicarbonate and a subsequent systemic metabolic acidosis. In general, forced diuresis in these patients should be avoided.

� •

Helpful Tip

Sa l i cylate poiso n i n g causes a m ixed m eta bol ic acidosis

1 1 1r a n d res p i ratory a l ka losis.

Activated charcoal and repeated doses of sodium bicarbon­ ate fail to improve the patient's clinical symptoms. In fact, she appears to be more lethargic upon reevaluation. Repeat laboratory analysis shows a worsening metabolic acidosis, an indication for hemodialysis. Question 2-4 Which of the following is NOT an indication for hemodialy­ sis in salicylate toxicity? A) Central nervous system depression. B) Acute heart failure. C) Salicylate level greater than 40 mg/dL. D) Coagulopathy. E) Seizures.

598

MCG RAW- H I LL E D U C AT I O N S P E C I A LTY BOA R D REVI EW: P E D I AT R I C S

Discussion 2-4 The correct answer is "C:' Absolute salicylate levels do not determine the need for hemodialysis in isolation, unless the level is greater than 80 to 1 00 mg/dL in acute ingestions, greater than 50 to 60 mg/dL in chronic ingestions, or the level contin­ ues to rise despite treatment. The patient in this case is the vic­ tim of a chronic ingestion, as previously stated. Indications to pursue hemodialysis in salicylate toxicity include acute renal failure (acute kidney injury) , congestive heart failure, pulmo­ nary edema, altered mental status, coma, seizures, cerebral edema, and worsening metabolic acidosis or rising salicylate level despite the aforementioned interventions ( eg, activated charcoal, sodium bicarbonate) .



Helpful Tip

::'Jl l n

.s.

sa l icylate toxicity, activated c h a rcoa l a n d sod i u m

i1 1 r bicarbonate a re the treatments of choice; hemodia lysis

ca n be consid ered i n patients who do not res pond to

treatment and for s pecific i n d ications (eg, seizu res, coa g u l o pathy).

� •

1 1 1r

Helpful Tip

Sa l icylate toxicity can be acute or c h ron ic; consider common over-the-cou nter medications as potentia l c u l p rits

(eg,

aspirin,

mouthwashes,

topica l

pain­

rel ievi ng crea ms, etc) . The Done nomogram is of l i m ited c l i n ica l util ity.

A 2-year-old girl is brought into the emergency depart­ ment by her mother after the patient was found playing with an open bottle of naproxen. For the preceding 45 minutes, the child had been playing with her siblings without direct supervision. The bottle had only recently been opened, and while it is not clear how many pills had been taken previously there are more than twelve 200 mg pills missing. You find pill residue in the child's mouth, but she is otherwise asymptom­ atic at this time. Question 3-1 What initial intervention should be implemented? A) Copious oral hydration. B) Aggressive intravenous hydration. C) Activated charcoal. D) Gastric lavage. E) No intervention is indicated at this time. Discussion 3-1 The correct answer is "C:' Enough evidence exists in this case to warrant prompt intervention. And while the time

course and amount of naproxen (an NSAID) consumed is not precisely known, one should assume that it has been up to 45 minutes since the child ingested up to 2400 mg of the medication. Accordingly, activated charcoal is the initial treat­ ment of choice. This intervention is indicated in children who consume more than 1 00 mg/kg or five adult doses of an NSAID. Presentation should generally be within 1 hour if administering activated charcoal. Gastric lavage has fallen out of favor and is no longer recommended given the associated risks (eg, aspiration) and relative dearth of evidence to support improved outcomes. Hydration (oral or otherwise) may aid in addressing sequelae of NSAID toxicity (ie, renal toxicity) , but this is not the primary initial treatment needed. Question 3-2 How much and how often should the activated charcoal be administered? A) 1 to 2 g/kg up to 90 g orally, once. B) 2 to 4 g/kg up to 200 g orally, once. C) 1 to 2 g/kg up to 90 g orally, twice. D) 2 to 4 g/kg up to 200 g orally, twice. E) 3 g/kg up to 1 50 g orally, once. Discussion 3-2 The correct answer is "A." The ideal dose of activated char­ coal should not exceed 90 g. Given orally, the weight-based dose is 1 to 2 g/kg and it is typically administered as a slurry ( sometimes mixed with sorbitol; premixed slurry formula­ tions are commercially available ) . Multiple-dose activated charcoal (MDAC) may further decrease absorption rates. However, its efficacy has not been extensively studied, and it is not thought to result in significantly improved outcomes. (See Table 2 7- 1 . ) You call the Poison Control Center to review the patient's case, including her signs and symptoms of toxicity, labora­ tory evaluation, and acute management. You report that you have given activated charcoal. The Poison Control Center asks if the child is symptomatic.

TABLE 27-1 I N D I CATIONS A N D CONTRA I N D I CATIONS

FOR ADM I N I STE R I N G ACTIVATED CHARCOA L

Indications

Contra indications

Prese ntation with i n 1 h o u r o f i ng estion

Altered me nta l status (a s p i ration risk)

I ng esta nt a bsorbed by c h a rcoa l

I ng esta nt not a bsorbed by c h a rcoa l (eg, i ron, a lcohol, or l ith i u m)

I ng esta nt is potenti a l ly toxic or may cause i nj u ry

Bowe l obstruction or pe rforation I n a b i l ity to protect a i rway (aspi ration risk)

CHAPTER 27



P O I S O N I N G A N D ENVI RO N M E NTA L EXPOS U R E TO HAZA R D O U S S U BSTA N C E S

Question 3-3 Acute NSAID toxicity does NOT result in which of the follow­ ing sequelae? A) Aseptic meningitis. B) Gastrointestinal hemorrhage. C) Seizures. D) Hypertension. E) Hepatocellular injury. Discussion 3-3 The correct answer is "D:' NSAID toxicity typically results in hypotension rather than hypertension. Gastrointestinal symp­ toms are most common, including abdominal pain, vomiting, and diarrhea. Aseptic meningitis has been reported in cases secondary to hypersensitivity (along with asthma exacerba­ tions) . Gastrointestinal hemorrhage can result and is treated by standard means. Seizures have been associated with specific NSAID use, in particular piroxicam, naproxen, ketoprofen, and mefenamic acid. The treatment for such seizures involves the use of benzodiazepines. Lastly, liver function can be impaired as a result of excessive NSAID ingestion in addition to being associ­ ated with cholestasis. Acute kidney injury may occur.



glucose), and a urinalysis at minimum-and no further symp­ toms should be observed for 4 to 6 hours to ensure no further development of symptoms or sequelae. Patients with abnormal laboratory tests should be admitted for further observation and supportive measures (eg, intravenous hydration) as appropri­ ate. A nephrologist should be consulted in the setting of acute kidney injury.

� QUICKQUlZ Regarding NSAID toxicity, which of the following is true? A) Generally, NSAIDs ingestions and overdoses are benign. B) Patients who are otherwise asymptomatic may be safely dis­ charged after 4 to 6 hours of observation. C) Patients should be assessed for acute kidney injury, and if present a nephrologist should be consulted. D) Gastrointestinal symptoms are the most common with NSAID toxicity. E) All of the above. Discussion The correct answer is "E:'

Helpful Tip

� Know you r

=-

599

NSAI Ds! There can be d ifferences i n the

i1 1r presentation among patients who present with specific NSAID ingestions (eg, naproxen can cause seizures.) . lt is

not n ecessa rily i m porta nt to memorize these s u btleties; however, it is i m porta nt to identify which NSAID was consu med to a ntici pate potential c l i n ica l symptoms and prepa re for possi b l e su bseq uent treatment.

She has thrown up once but is otherwise doing well. You have ordered laboratory testing, and are waiting for the results. The mother asks if the child will have to spend the night in the hospital.

A 17 -year-old girl is brought into the emergency department by ambulance. She was found sleeping sonorously in her bedroom lying next to an empty bottle of acetaminophen. Her best friend contacted the patient's parents after seeing a social media post detailing the patient's breakup with her boyfriend. Upon arrival at the emergency department her vital signs are stable and she is arousable. The patient notes nausea and generalized malaise. She admits to consuming a large quantity of acetaminophen in an attempt to harm her­ self roughly 4 hours ago, but she is unsure of the number of pills ingested.

Question 3-4 In the setting of an asymptomatic patient with a normal labo­ ratory evaluation, how long should the patient be observed? A) No observation is needed. B) 4 to 6 hours. C) 8 to 12 hours. D) 12 to 18 hours. E) At least 24 hours.

Question 4-1 Given the patient's presentation, what other symptoms might you expect to find with further probing or upon physical examination? A) Right upper quadrant abdominal pain. B) Emesis. C) Spontaneous bleeding (eg, from the gums, ecchymosis). D) Seizures. E) Hypotension.

Discussion 3-4 The correct answer is "B:' Generally speaking, NSAID toxic­ ity is relatively benign and symptoms typically develop within 4 hours of ingestion. Ingestions of 1 00 mg/kg or less are usu­ ally asymptomatic. Patients with normal laboratory tests­ complete blood count (CBC) with differential, basic metabolic panel (including blood urea nitrogen [BUN] , creatinine, and

Discussion 4-1 The correct answer is "B:' The patient is still in the early stages of acetaminophen toxicity, which typically entails nausea, vom­ iting, and malaise. These nonspecific symptoms mimic many other processes, which can lead to underappreciating the poten­ tial severity of illness if the proper historical points are not elic­ ited (or provided for you, as in this case). Right upper quadrant

600

MCG RAW- H I LL E D U C AT I O N S P E C I A LTY BOA R D REVI EW: P E D I AT R I C S

pain and spontaneous bleeding can occur, although these find­ ings develop further along in the course of acetaminophen ingestions. Seizures and hypotension are not typically associ­ ated with acetaminophen toxicity.



Helpful Tip

.s:::"'l Remember that co-i n g estion with other med ications,

i1 1r i n c l u d i n g opioids a n d cold med ications, is common i n

aceta m i nophen toxicity. I n fact, there a re more than 1 00 cold med ications that conta i n aceta m i nophen. Be s u re to consider, a n d treat when a pp ropriate, these co-i n g estions.

You remember that timing is important with acetaminophen ingestions and that symptoms depend on time elapsed since ingestion. The Poison Control Center mentioned that the progression of acetaminophen toxicity is generally described in phases. Question 4-2 Which of the following pairings of phase timing with symp­ toms and laboratory abnormalities is INCORRECT? A) Phase 1 (30 minutes to 24 hours postingestion) -7 nausea, vomiting, malaise. B) Phase 2 (24 hours to 72 hours postingestion) -7 right upper quadrant pain, oliguria, elevated liver transaminases, pro­ thrombin time (PT), and international normalized ratio of PT (INR) . C) Phase 3 (72 hours to 96 hours postingestion) -7 peak liver function testing abnormalities, acute liver and renal failure, jaundice. D) Phase 4 (96 hours to 10 days postingestion) -7 hepatic encephalopathy. E) All of the above are correct. Discussion 4-2 The correct answer is "D:' The phases and associated time courses are all paired correctly. However, hepatic encephalopathy is indic­ ative of phase 3. At this stage if a patient's PT/INR continues to rise or renal insufficiency develops there is a high likelihood that hepatic transplantation may be necessary. Phase 4 brings with it either resolution of hepatic abnormalities or continued progres­ sion to complete hepatic failure and death. Here is an abbreviated version that may help keep the phases of acetaminophen toxicity straight. Phase 1 : mild symptoms (:::; 24 hours)

Question 4-3 How should you proceed, given this information? A) Discharge the patient as her level is not hepatotoxic. B) Observe the patient for an additional 4 hours. C) Admit the patient for 24-hour observation. D) Initiate medical management. E) Prepare the patient for hemodialysis . Discussion 4-3 The correct answer is "D:' Ingestions of greater than 200 mg/ kg or 10 g (whichever is less) are considered potentially toxic. The patient's initial acetaminophen level is in the "possibly hepato­ toxic" range according to the Rumack-Matthew nomogram. (See Figure 27- 1 .) It is important to note that the acetaminophen level in cases of toxicity should be drawn at 4 hours postingestion or at presentation if the suspected ingestion was more than 4 hours prior. Acetaminophen levels before that point in time may warrant treat­ ment if markedly elevated (an ominous sign), but a normal level prior to the 4-hour mark does not rule out acetaminophen over­ dose or the need for treatment. It is not appropriate to discharge this patient, and medical treatment should be initiated given the relatively high level so acutely. Hemodialysis is not indicated for the treatment of acetaminophen toxicity. (See Table 27-2.) You astutely decide to initiate medical treatment for this patient. Well-trained, you recall that N-acetylcysteine (NAC) is the treatment of choice. Question 4-4 Which route and loading dose should be utilized to adminis­ ter this medication initially? A) Oral; 140 mg/kg. B) Oral; 70 mg/kg. C) Intravenous; 140 mg/kg. D) Nasogastric tube; 140 mg/kg. E) Nasogastric tube; 70 mg/kg Discussion 4-4 The correct answer is ''A." The preferred initial route of admin­ istration ofNAC is orally at a loading dose of 140 mg/kg. Often this medication is diluted in juices or soft drinks to facilitate consumption. The utilization of antiemetics ( eg, ondansetron)

TABLE 27-2 I N D I C AT I O N S FOR TREATM E N T O F

A C U T E ACETA M I N O P H E N TOX I C ITY

Phase 2: hepatoxicity and nephrotoxicity develop ( 1 -3 days)

Aceta m i nophen l eve l a bove the treatment l i ne on Ru mack-M atthew nomogra m

Phase 3: acute liver failure and acute kidney injury develop (3-4 days)

I ng estion > 200 mg/kg or 1 0 g a n d aceta m i nophen level resu lts n ot ra pid ly ava i l a ble

Phase 4: recovery or death from liver failure occurs ( 4- 10 days)

U n known t i m e of i n g estion with detecta ble aceta m i no­ phen l eve l

Shortly after the patient's arrival you astutely ordered an acet­ aminophen level. Her 4-hour level returned at 1 50 mmol/L.

U n known t i m e of i n g estion with elevated l ive r tra n s a m i nases

CHAPTER 27

200 r--

1 300

1 50

1 000 900 800 700



"E

Cl) u

5 ...J

u

r-r-r-r--

E

500 -

300 -

40 r--

250 ...J

� .1::1

.E

.r:. E c.. 0 c:

Qj





.\ " . . "' � 1'\ . r\ . '\ ..

200 -

10 9 8 7

r-r-r-r-r--

5 r-4 r-r--

Possible Toxicity



.. \ .\ .

Q)



ra

� . . i\

"'

� (])E 0 en ..!!! 20 r-- E e C.. o (.) ·e

.

400 -

50 r--

30 r--

601



-

600

60 r--

c: 0

P O I S O N I N G A N D ENVI RO N M E NTA L EXPOS U R E TO HAZA R D O U S S U BSTA N C E S

2000 -

300 r--

1 00 90 80 70





1 00 90 80 70 60

-

. .\ . '

Toxicity U n l i kely

.\ . \ •

50

.

40 30 -



. "' . .

Recommend treatment if level is above broken l i n e

20

2 r-Take level 10 at I east 4 h o u rs pos!-ingestion

4

8

12

16

20

24

26

H o u rs post-ingestion

F I G U R E 27- 1 . The Rumack-M atthew nomogram is used for assessi n g aceta m i nophen toxicity. Treatment i s reco m mended for aceta m i nophen l evels a bove the lower nomogram l i n e (broken line). Serum leve l s obta i ned prior to 4 h o u rs after ingestion a re u n i nterpreta ble. (Reprod uced with permission from Tinti n a l l i J E, Sta pczynski J S , Ma OJ, Yealy D M , C l i n e D M , Meckler G D, e d s . Tin tinalli's Emergency Medicine: A Comprehensive Study Guide. 8 t h e d . N e w Yo rk, NY: McGraw- H i l l Ed ucation; 201 6, F i g . 1 90-2.)

should be entertained to facilitate the administration of this medication. Since this patient is not actively vomiting, oral administration is the best answer. If the patient does happen to vomit up the NAC, an attempt to repeat the dose orally after giving an antiemetic should be pursued. Persistent vom­ iting is an indication to attempt administration by nasogastric tube. For patients who cannot tolerate either enteral routes, intravenous NAC can be given. The dosing of the intrave­ nous medication is dictated by the package insert and can be guided by the Poison Control Center. Of note, once ini­ tiated NAC is then given orally every 4 hours at 70 mg/kg for 1 7 additional doses or until the patient's acetaminophen

level is nontoxic. Overall, when initiated within 8 hours of ingestion, NAC is 1 00% hepatoprotective. Activated charcoal should also be administered if the patient presents 2 hours or less after ingestion.

� •

Helpful Tip

For acute aceta m i nophen i ngestions, a serum level

1 1 1 r m u st be obta i ned 4 h o u rs after i ngestion. Norma l

levels before this time point do not r u l e out l iver i nj u ry nor do they obviate the need for treatment.

602

MCG RAW- H I LL E D U C AT I O N S P E C I A LTY BOA R D REVI EW: P E D I AT R I C S

� •

Helpful Tip

Patients

with

acute

acetami nophen

toxicity

often

1 1 1r present with minimal sym ptoms (eg, nausea, vomiting,

or malaise). Be sure to have a high index of suspicion. Util ize the Rumack-Matthew nomogram to help guide

treatment decisions. N-acetylcysteine is the treatment of choice in acetami nophen toxicity.

A 6-year-old boy presents in the arms of his father to the emergency department. He feels cool to the touch and is min­ imally responsive. His heart rate is appropriate at 65 beats per minute but his blood pressure is 70/30 mm Hg. The patient appears apneic. The boy's father found him in the bathroom. They have no prescription medications in the house. There are no signs of external trauma on the child, who has no past medical history and has otherwise been healthy of late. Question 5-1 Which of these medications could be responsible for the patient's presentation? A) Aspirin. B) Ibuprofen. C) Lomotil. D) Acetaminophen. E) Prenatal vitamins with iron. Discussion 5-1 The correct answer is "C:' Lomotil is a combination of diphenox­ ylate and atropine, which acts as an opioid. This patient has many signs consistent with opioid toxicity, including cool skin (suggestive of hypothermia) , hypotension, and apnea (respira­ tory depression). One would expect a bradycardic heart rate as well; however, the atropine (anticholinergic) in the Lomotil may result in a normal or low-normal heart rate. Other typical signs of opiate overdose include pinpoint pupils (although this does not occur in this particular ingestion secondary to the effects of atropine), central nervous system depression, and palpita­ tions. Aspirin overdoses should precipitate tachypnea. NSAID toxicity may appear similarly (very rarely) , although signs of hypothermia are not present. These patients typically have a benign course and are much less likely to be this critically ill. Acetaminophen and iron ingestions primarily present with gas­ trointestinal manifestations.

� •

Helpful Tip

Opioid ingestions ca n a lso result from na rcotic prescri p­

l l lr tions

such as hyd rocodone and oxycodone. Another

consideration that should be considered is heroin. The use of heroin has been on the rise since 2007, in pa rt beca use it has become more readily ava i lable a n d is a cheaper a lternative to prescription na rcotics.

Question 5-2 Aside from activated charcoal, what medical intervention should be considered next? A) Whole bowel irrigation. B) Sodium bicarbonate. C) Intravenous normal saline. D) Physostigmine. E) Naloxone. Discussion 5-2 The correct answer is "E:' While a case could be made for several of the options listed, initially the most appropriate answer is naloxone, which is an opioid-antagonist. Effective in minutes, naloxone can be administered intravenously, intra­ muscularly, or subcutaneously. Whole bowel irrigation for patients who present less than 1 hour after an opioid inges­ tion can be considered, but it is not the treatment of choice. Sodium bicarbonate is not initially indicated, and while intra­ venous normal saline might be appropriate in the setting of hypotension, as is present in this case, it should not precede the administration of naloxone. Lastly, physostigmine would antagonize the effects of the atropine in Lomotil. Yet, this may result in significant bradycardia and would do nothing for the opioid toxicity itself. Question 5-3 Approximately how long does naloxone have efficacy? A) 1 5 to 30 minutes. B) 30 to 60 minutes. C) 1 to 2 hours. D) 2 to 4 hours. E) Up to 8 hours. Discussion 5-3 The correct answer is ''A:' Naloxone remains effective for roughly 1 5 to 3 0 minutes, which can be problematic for most opioid ingestions. Typical opioid action can last for hours, which means that the usual dose for naloxone treatment (0. 1 mg/kg/dose in infants and young children and 0.0 1 mg/kg/dose in neonates) may need to be repeated. With ingestions of long-acting opioids such as methadone, patients may require increased doses of naloxone, repeated doses, or even a continuous infusion. Absorption of the diphenoxylate is delayed owing to atropine-induced delayed gastric motility, which may cause prolonged or delayed symptoms (24 hours after ingestion) . Two hours into his evaluation the young boy stabilizes after the administration of several doses of naloxone and a bolus of normal saline. His vital signs return to normal, and his physical examination is unremarkable. Question 5-4 What is this patient's disposition? A) Pediatric consultation in the emergency department. B) Observation for 2 additional hours. C) Observation for 4 additional hours.

CHAPTER 27



P O I S O N I N G A N D ENVI RO N M E NTA L EXPOS U R E TO HAZA R D O U S S U BSTA N C E S

D) Admission for 24 hours. E) Discharge to home promptly. Discussion 5-4 The correct answer is "D:' Owing to the prolonged half-life of diphenoxylate, particularly compared with the short half-life of naloxone, and prolonged symptoms from atropine-induced delayed gastric motility, 24-hour admission is recommended for all patients who require the administration of naloxone. Discharge before this period of observation leaves the patient vulnerable to a return of the initial symptoms secondary to opioid toxicity ( eg, hypotension, central nervous system depression, apnea, etc) . For other opioid ingestions, patients can be discharged if asymptomatic 6 hours after ingestion or if asymptomatic 4 hours after the administration of naloxone.

� QUICKQUIZ Which statement regarding opioid ingestion is false? A) Naloxone should not be administered to neonates. B) Symptoms of opioid toxicity commonly include hypotension, bradycardia, respiratory depression, miosis, hypothermia, and seizures. C) Naloxone is the treatment of choice for opioid toxicity, but remember that the half-life of this medication is often shorter than that of the opioid ingested. D) It is important to consider concomitant ingestions resulting from combination products ( eg, hydrocodone­ acetaminophen, etc). E) Full opioid reversal in patients with opioid dependence may precipitate withdrawal. Discussion The correct answer is ''A:' Naloxone may be given to neonates. Are you NRP (neonatal resuscitation program) certified? The dose is 10 times less than that given to infants and young children.



Helpful Tip

� Sym ptoms of opioid toxicity commonly i n c l u d e hypo­

r1 1r tension, bradyca rdia, res p i ratory d epression, m iosis,

603

C) Sustained-release drugs. D) Packages of cocaine. E) Ibuprofen. Discussion The correct answer is "E:' Others that should be considered include ingestions of enteric-coated pills, illicit drug pack­ ages, and other drugs not bound by charcoal, such as iron and lithium.

An 8-year-old girl is on a camping trip with her mother. She returns from the woods, and her mother notices that she appears to be confused and tachypneic. The mother checks her pulse and it seems accelerated. You happen to be accompanying the group on this outing. After quickly evaluating the patient you suspect anticholinergic effects. Emergency services are called and an ambulance is dis­ patched to the camp to facilitate transportation for further medical care. Question 6-1 Which symptom is NOT a part of the classic toxidrome associated with this child's presentation? A) Dry as a bone. B) Red as a beet. C) Blind as a bat. D) Cool as a cucumber. E) Mad as a hatter. Discussion 6-1 The correct answer is "D:' The classic presentation of a patient with anticholinergic toxicity is mad as a hatter (altered mental status, hallucinations), hot as a hare (hyperthermia) , red as a beet (flushed skin) , dry as a bone (dry mucous membranes, skin, or both) , blind as a bat (mydriasis-induced blurry vision) , and full as a flask (urinary retention ) . Cool as a cucumber is the opposite physiologic response to an anticho­ linergic exposure.

hypothermia, and seizu res. N a l oxone is the treatment

of choice i n such patients, but rem e m ber that the half-l ife of t h i s medication is often shorter than that of the opioid i n g ested . Be s u re to consider concom itant i n g estions (eg, hyd rocodone-aceta m i nophen, etc).

� QUICKQUIZ What is NOT a potential ingestant for which whole bowel irrigation should be considered? A) Iron pills. B) Lithium pills.



Helpful Tip

:5.� Hyperthyro i d i s m

(eg, thyroid storm) may present

r1 1 r c l i n ica l ly in a m a n ner s i m i l a r to a ntich o l i n e rg i c toxicity.

Question 6-2 Which plant is NOT potentially responsible for her presentation? A) Jimson weed. B) Nightshade. C) Mushrooms. D) Belladonna. E) Foxglove.

604

MCG RAW- H I LL E D U C AT I O N S P E C I A LTY BOA R D REVI EW: P E D I AT R I C S

Discussion 6-2 The correct answer is "E:' Foxglove is also known as Digitalis purpurea, and exposure to this plant can cause cardiac dys­ rhythmias, shock, cardiovascular collapse, agitation, lethargy, and gastrointestinal symptoms. However, foxglove does not precipitate a typical anticholinergic toxidrome. Each of the other options can precipitate anticholinergic toxicity. Regard­ ing mushrooms, those with anticholinergic properties include Amanita muscaria and Amanita pantherina. Also, keep in mind that medications can result in an anticholinergic toxidrome as well. These medications include atropine, scopolamine, antihis­ tamines (eg, diphenhydramine), cyclobenzaprine, and antispas­ modics (eg, hyoscyamine) . Once the ambulance arrives the patient is placed on the car­ diac monitor. Her vital signs show a normal temperature and blood pressure. Peering from the periphery, you observe the monitor closely. You find that she appears to have the most common cardiac monitor/ECG abnormality associated with anticholinergic toxicity. Question 6-3 What is that finding? A) Sinus tachycardia. B) QRS prolongation. C) Atrioventricular (AV) block. D) Right bundle branch block. E) Ventricular bigeminy. Discussion 6-3 The correct answer is "A:' ECG findings associated with anti­ cholinergic exposure are generally nonspecific, in contrast to some other toxicities. Although each of the options listed can result, sinus tachycardia is the most common ECG finding. Question 6-4 What is the treatment of choice for this patient? A) Benzodiazepines. B) Phenobarbital. C) Aggressive intravenous hydration. D) Physostigmine. E) Sodium bicarbonate. Discussion 6-4 The correct answer is "D:' Depending on a patient's presenting symptoms, each of the aforementioned options may be appro­ priate. However, given this case the patient would benefit most from physostigmine. This medication reverses both the central and peripheral anticholinergic effects. It should be used with caution as it can cause dysrhythmias and cholinergic crises. Its use should be avoided in those with asthma, cardiovascu­ lar disease, or concurrent or suspected tricyclic antidepres­ sant overdose. The pediatric dose is 0.02 mg/kg intravenously over 5 minutes. In the case of this patient, she has not had any

seizures, negating the use of benzodiazepines or phenobarbital. Nor is she hypotensive, and while intravenous hydration may be beneficial it will not directly address any active symptoms. Sodium bicarbonate is effective in some cardiac dysrhythmias, but one has not been identified in this case.

� QUICKQUIZ Which is NOT a symptom of a cholinergic crisis? A) Diarrhea. B) Drooling. C) Urinary retention. D) Sweating. E) Pinpoint pupils. Discussion The correct answer is "C:' The symptoms of cholinergic excess can be remembered using the mnemonic SLUD (Salivation, Lacrima­ tion, Urination, Diarrhea) or DUMBELS (Diarrhea, Urination, Miosis, Bronchospasm, Emesis, Lacrimation, Sweating) .

A group of rebellious teenagers conduct an unsupervised house party one evening. It is billed as a "pharm party" or "pill party" in which the guests bring prescription medications that are then combined into a large bowl. Adventurous party-goers can choose to sample random, unknown pills. Three of the patrons who engage in the festivities become ill. After the local authori­ ties break up the soiree, the three patients needing medical attention are brought to your emergency department by ambu­ lance for further evaluation and treatment. The EMS crew was able to ascertain that a preponderance of the medications col­ lected at the party were antihypertensive medications. The first patient presents with a heart rate of 33 beats per minute and mild hypotension, and his bedside glucose level is 55 mg/dL. You astutely suspect that the patient is suffering from ill-effects related to a beta-blocker overdose. Question 7-1 Which medication is NOT used in the management of such an ingestion? A) Atropine. B) Sodium bicarbonate. C) Glucagon. D) Insulin. E) Inamrinone. Discussion 7-1 The correct answer is "B:' The initial treatment of choice in beta-blocker toxicity, particularly in the setting of bradycar­ dia, hypotension, or both, is atropine (0.02 mg/kg/dose IV in

CHAPTER 27



P O I S O N I N G A N D ENVI RO N M E NTA L EXPOS U R E TO HAZA R D O U S S U BSTA N C E S

the pediatric population) and intravenous fluids. Although its efficacy is not outstanding, it is recommended as a primary treatment modality. Glucagon (0.03-0. 1 mg/kg as a bolus followed by 0.07 mg/kg/h as an infusion) should then be attempted if atropine is unsuccessful. Other medications such as insulin (which promotes improved myocardial metabolism) and inamrinone (a phosphodiesterase inhibitor with posi­ tive inotropic effect) can be attempted as well. Sodium bicar­ bonate may be effective in treating an arrhythmia secondary to beta-blocker toxicity, but it is not a primary treatment. Of note, another treatment option includes calcium gluconate or calcium chloride. These medications improve conduction, inotropy, and blood pressure. The next party-goer to require your attention has a very simi­ lar presentation to the first patient. Although her heart rate is 33 beats per minutes and she is hypotensive, her blood glucose level is 1 78 mg/dL. The Poison Control Center is contacted, and it is suggested that the patient may have con­ sumed a significant amount of a calcium channel blocker. Question 7-2 Which strategy should be used to treat this patient? A) Calcium gluconate bolus. B) Calcium gluconate continuous infusion. C) Calcium chloride bolus. D) Calcium chloride continuous infusion. E) Norepinephrine. Discussion 7-2 The correct answer is "A:' Calcium salts may be given for the treatment of calcium channel blocker overdose, but not all patients respond. Calcium chloride may cause tissue necrosis from extravasation and should be given through central venous access; therefore, calcium gluconate may be preferred. The ideal dose for the administration of calcium gluconate in the pedi­ atric population is 60 mg/kg/dose, which may be repeated at the same dosage in 10 minutes, if necessary. Keep in mind that calcium channel blockers are notorious for refractory brady­ cardia and hypotension. If repeated doses of calcium gluconate are administered be sure to follow the patient's serum calcium levels or consider a continuous infusion, or both. Vasopressor infusions ( eg, norepinephrine) may be beneficial in hypotensive patients. As with beta-blocker ingestion, glucagon, insulin with glucose, and atropine may be used.

� •

Helpful Tip

Differentiating between beta-blocker overdoses and

I 1 1 r ca lcium

channel overdoses can be very d ifficult as

both ca use bradyca rdia a n d hypotension. The patient's g l u cose level may provide additional insight to help delineate

between

the

two.

Beta-blockers

cause

hypog lycemia and ca lcium channel blockers ca use hyperg lycemia.

605

With treatment for the first two patients underway, you turn your attention toward patient #3. She too is bradycardic, but she is found to be hypertensive. Additionally, her breathing is quite slow, and her pupils are pinpoint. With stimulation her heart rate and breathing improve. Question 7-3 What is the most likely causative antihypertensive agent for this presentation? A) Diltiazem. B) Hydrochlorothiazide. C) Lisinopril. D) Clonidine. E) Metoprolol. Discussion 7-3 The correct answer is "D:' The patient presentation is similar to an opioid overdose with respiratory depression and miosis standing out among her symptoms. Clonidine is a centrally act­ ing alpha2 -agonist, which mimics opioid effects. Generally, this medication first causes hypertension before its antihypertensive effects surface, sometimes up to 1 0 hours later. Children tend to be more affected by this medication than adults. Diltiazem is a calcium channel blocker that results in hypotension, whereas metoprolol is a beta-blocker that likewise leads to hypotension shortly after excessive ingestion. Hydrochlorothiazide is a loop diuretic that causes neither apnea nor miosis. As for lisinopril, this medication does have some activity on opioid receptors, but like loop diuretics it does not result in respiratory depression or miosis. Naloxone should be given if central nervous system or respiratory depression is present, but not all patients will respond.



Helpful Tip

� Atro p i n e

=.

is a g o o d first treatment for sym ptomatic

i1 1r bradyca rdia, a d m i n istered a t 0.02 mg/kg/dose IV. It may buy you time to decide you r n ext step.

A I S-year-old male patient who had been diagnosed with postconcussive syndrome had previously been started on amitriptyline for symptoms associated with his con­ cussion. Unable to participate in football, he developed worsening depression. He presents to the emergency department after admitting to consuming a "handful" of his 10 mg amitriptyline tablets. His vital signs are within normal limits, and he does not appear to be in any distress. You obtain a stat ECG upon the patient's arrival, which is shown below.

606

MCG RAW- H I LL E D U C AT I O N S P E C I A LTY BOA R D REVI EW: P E D I AT R I C S

k¥ .... v

r- ..ll /

\

v- Jl

II

L

/� '\

__{_ ::,_\._ '\ v 'V

N

'\

Jl

.\lL

r---r-r--11 -f'\ - /' J\ /"'. \."f

��-

IV

1--'\

il

1./

r ,n

\

.....,

�fJp�- v

v

v

-v·

lr-..

I'

"-./

-._I

I

v

'\

i'-'

!/"\

f

---'

/'--, k. .n



I,

1\ /' I[\ IR h I I)i/ 't

v

V I

V'

/

- .d

llf

!A._

v

....,,

I

1\ t

iL\}

II

A-

/\

-1- M

����-�

J.

r

I \

,A

I 1/

"

'7

i\.

�-d'-

y_

3

.I

I

� In

'J$

)llI\

y_

f\

V-

J1

1\ II

)._

,/

I( I

��

/"> \)

I \

n

I

/"- . n

"

V' "' / ./\ /...... \ IV



\r

/'

- _L \

v

II

v

\

'\

/'

I

jV

0-.



r

v

A.

V\

v

I\

l/'

/

v-,

llnr

1-r---

\

1/

v

r .(\

1 r---r-r-1 -0.

1-r---

I \

"

v

/

"'

1-r---

1-r---

(Reprod u ced with perm ission from Life in the Fastla ne.com. http://lifei nthefastla ne.com/ecg- l i bra ry/basics/tca-overdose/)

Question 8-1 Which of the following would you expect to find on the ECG of a patient with a tricyclic antidepressant overdose? A) Sinus tachycardia. B) QTc prolongation. C) QRS complex widening. D) Right bundle branch block. E) All of the above. Discussion 8-1 The correct answer is "E:' QRS widening (> 1 00 msec) is a common initial finding associated with tricyclic antidepres­ sant (TCA) overdoses. Due to blockade of sodium channels in the His-Purkinje system and myocardium, this ECG finding usually precedes bundle branch blocks. Tachycardia is almost always present at some point in a patient's course, but QRS wid­ ening is more concerning and dictates management moving forward. Prolongation of the PR interval, ST/T-wave changes, heart block, ventricular fibrillation or tachycardia, torsades de pointes, and asystole may occur as well.

Bolus doses of 1 to 2 mEq/kg should be administered intrave­ nously and repeated as needed. Epinephrine is generally not utilized in this context, with dopamine or norepinephrine pre­ ferred for cases of significant hypotension. Calcium chloride or calcium gluconate have no role in the management of TCA overdoses. Procainamide (increases myocardial toxicity) and flumazenil (increases risk of seizure) are contraindicated in such cases.





Helpful Tip



TCAs block sod i u m a n d potassi u m c h a n n e l s, l ea d i n g t o i n c reased card i ovascu l a r toxicity. O t h e r d epres­ sa nts (pa rtic u l a rly new a ntidepressants such

as

cita lopra m ) block a m i n e reu pta ke and ca use less l ife-th reate n i n g toxicity. Look for QRS widen i n g on ECG a n d treat with sod i u m bica rbonate. Patients with sym ptoms after 6 h o u rs of observation ( i n c l u d i n g tachyca rdia), co-i ngestions, dysrhyt h m ias,

Question 8-2 What is the initial treatment that should be initiated for this adolescent in the setting of a TCA overdose with QRS widening? A) Epinephrine. B) Procainamide. C) Calcium chloride/calcium gluconate. D) Sodium bicarbonate E) Flumazenil. Discussion 8-2 The correct answer is "D:' Sodium bicarbonate is the most effec­ tive and initial treatment of choice for abolishing arrhythmias, shortening the QRS duration, and increasing blood pressure.

or a ltered menta l status wa rra nt a d m ission. Psych iat­ ric eva l uation may be necessa ry if a su icide atte m pt is sus pected.

Once the sodium bicarbonate is ordered you reevaluate the patient. Clinically he appears to be deteriorating. Question 8-3 Which symptom would you NOT expect relative to his TCA toxicity? A) Hypotension. B) Seizures. C) Agitation.

CHAPTER 27



P O I S O N I N G A N D ENVI RO N M E NTA L EXPOS U R E TO HAZA R D O U S S U BSTA N C E S

D) Lethargy. E) Bradycardia. Discussion 8-3 The correct answer is "E:' Bradycardia develops later in the course of TCA toxicity once a patient's catecholamines are depleted. Initially, patients exhibit tachycardia resulting from a blockade of norepinephrine and concomitant anticholiner­ gic effects. The other options are all potential symptoms that can develop as a result of this poisoning. Hypotension may be refractory to initial treatment and require vasopressor support (norepinephrine) . In fact, seizures can be particularly detri­ mental, as the associated acidemia that develops can worsen subsequent cardiovascular toxicity.

607

Discussion The correct answer is ''A:' Tricyclics cause hypotension. The cardiovascular, central nervous system, and peripheral nervous system (anticholinergic effects) are affected by TCA ingestion. Do you remember mad as a hatter, hot as a hare, and blind as a bat? If not, you did not learn the signs and symptoms of anticholinergic toxicity discussed earlier. Remember, physostig­ mine should not be used to treat anticholinergic toxicity in the setting of a TCA ingestion.



Helpful Tip

� Antidepressa nts

=-

other than tricycl ics have d ifferent

r1 1r cardiovascu l a r toxicity. For i n sta nce, selective seroto n i n

reu pta ke i n h i bitors (SSRis) have l ess ca rd iovascu l a r

toxicity i n genera l a n d a w i d e r m a rg i n for safety. •

Helpful Tip

� Do not l et wel l-appea ri n g

=-

Cita l opram is the most cardiotoxic SSRI a n d may have patients with a s u spected

r1 1r TCA i n gestion fool you . They can d eteriorate ra pid ly, developing

delayed ca rd iovascu l a r toxicity, genera l ly more than 1 2 h o u rs after i n g estion.

s i g n ifica nt ca rd iovascu l a r compro m ise

that necessitates emergent i ntervention.

The patient's clinical course continues to worsen, and he develops a tachydysrhythmia. Question 8-4 With sodium bicarbonate already administered, what is the treatment of choice for this cardiac abnormality in the setting of a TCA overdose? A) Lidocaine. B) Metoprolol. C) Procainamide. D) Physostigmine. E) Flecainide. Discussion 8-4 The correct answer is 'A: In general, antiarrhythmic drugs should be avoided. Lidocaine is the preferred second-line agent in the treatment of tachydysrhythmias in the setting of TCA toxicity. Metoprolol, a beta-blocker, will not affect sodium channels. Pro­ cainamide (a class lA medication) and flecainide (a class lC agent) are actually contraindicated, as they can result in a "quinidine-like" effect on the myocardium, increasing the toxicity of the TCA. Lastly, physostigmine is also contraindicated because of the risk of cardiac toxicity potentially deteriorating into asystole. '

'

� QUICKQUIZ What is NOT a sign or symptom of TCA ingestion? A) Hypertension. B) Coma. C) Blurred vision. D) Seizure. E) Heart block.

A 1 6-year-old boy and his best friend are brought into the emergency department by his mother very late at night. The mother states that the teens had attend an unsupervised party in the woods that night. You interview the son first, who stumbles as he walks from the door to the bed. He reports consuming beer for the first time. His vital signs reveal tachy­ cardia ( 1 1 0 beats per minute) but are otherwise unremark­ able. Upon examination his speech is a bit slurred, but he is able to follow commands appropriately. Question 9-1 Which of the following is the least relevant question when evaluating this patient? A) How much alcohol did you consume? B) When did you eat last? C) Have you fallen or sustained any additional trauma? D) Did you take any other drugs or medications? E) Do you have any other medical conditions? Discussion 9-1 The correct answer is "B:' Evaluating patients intoxicated with ethanol may appear mundane, particularly in patients who otherwise appear well. However, there are important questions that should be asked to rule out more significant pathology. For instance, one should take care to investigate whether the amount of alcohol consumed roughly correlates with the clini­ cal presentation. If this patient had only consumed one can of beer but was slurring his speech and was tachycardic, an inves­ tigation for other causes of his symptoms should be entertained. A traumatic fall can be the cause of such a presentation or the sequela from alcohol intoxication. Concomitant medical condi­ tions (eg, diabetes mellitus) can also affect the presentation of these patients. Additionally, it is critically important to inquire

608

MCG RAW- H I LL E D U C AT I O N S P E C I A LTY BOA R D REVI EW: P E D I AT R I C S

about the ingestion of other medications or illicit substances. Alcohol intoxication can mask the toxicity of other medications, particularly in severely intoxicated patients. Although it may affect the rate of clearance, the last meal consumed is of far less clinical significance. The patient admits to consuming four cans of beer in less than an hour. His physical examination reveals no other sig­ nificant abnormalities, including signs of trauma. You elect to obtain a plasma ethanol level, which registers 1 50 mg/dL. Question 9-2 What treatment should be initiated? A) An intravenous bolus of normal saline. B) Thiamine, folate, and a multivitamin. C) Glucose. D) Activated charcoal. E) None; observe the patient. Discussion 9-2 The correct answer is "E:' Patients with ethanol intoxication and no co-ingestions warrant observation and supportive mea­ sures if needed. They may benefit from antiemetics and hydra­ tion, but this is not necessary in all, or even most, patients. If the patient can tolerate oral hydration and it is safe to do so, this intervention should be attempted first. The administration of thiamine, folate, and a multivitamin are typically reserved for malnourished patients who consume the bulk of their daily caloric intake from alcohol. Glucose may be warranted in patients who develop hypoglycemia from severe alcohol intox­ ication, although this is not evident in this case. Lastly, patients may benefit from gastric decontamination (activated charcoal) with co-ingestions but not as a rule for isolated ethanol inges­ tions. Remember alcohols (ethanol, methanol, ethylene glycol) are rapidly absorbed and do not bind to charcoal. Knowing the patient's alcohol level can help predict how long it will take for metabolism of the alcohol (serum level of 0). The ethanol level typically decreases by 1 5 mg/ dL/h in a nonalcoholic patient. The level for the adolescent described in this case should be 0 in 10 hours. While you were treating the first patient your colleague was attending to his friend. He asks for your thoughts on the case. Apparently, this patient did not consume beer; he was handed a bottle of "moonshine" at the party. The patient has no idea how much "moonshine" he consumed, but after his maiden voyage into the world of alcohol con­ sumption he is rethinking his decision to drink it. The patient reports significant nausea with four associated bouts of nonbilious, nonbloody emesis. His abdomen is diffusely painful, and his vision is blurry. Question 9-3 What do you suspect to be the cause of his presentation? A) Ethanol. B) Ethylene glycol.

C) Methanol. D) Isopropyl alcohol. E) Glycerol. Discussion 9-3 The correct answer is "C:' The patient's presentation, particu­ larly the blurry vision, should significantly raise your suspicion for methanol toxicity, which can be found in "moonshine:' Often there is an 8- to 30-hour latent period that can exist before the onset of symptoms, but this is not always the case. A normal eye examination may be present initially, but as formic acid (a methanol metabolite) accumulates, it can have direct toxic effects on the retina and optic nerve. A funduscopic examination may reveal hyperemia of the disc, papilledema, or optic neuropathy. Question 9-4 Given the apparent methanol toxicity, which laboratory test­ ing (or associated value) will not be beneficial in managing this patient's presentation? A) Urinalysis. B) Serum anion gap. C) Serum osmolar gap. D) Ethanol level. E) Methanol level. Discussion 9-4 The correct answer is "A:' A urinalysis is not likely to add any­ thing to the treatment of this patient. Calculating the anion gap, which is (Na+) - (Cl- + HC0 3 -), can help confirm an anion gap metabolic acidosis, which is present in methanol poisonings. The osmolar gap equals the measured osmolarity minus the calcu­ lated osmolarity. A normal value is -7 to 10 mOsm. A widened serum osmolar gap, or (2 * Na+) + (BUN/2.8) + (Glucose/ 1 8 ) + (EtOH/4.6), also helps confirm methanol toxicity. However, a low or negative osmolar gap does not completely rule out a methanol ingestion (or another toxic alcohol ingestion) . The ethanol level can be useful not only in calculating the osmolar gap but also in guiding treatment. Lastly, methanol concentrations do confirm the presence of methanol. But, if a patient's presentation is late, the methanol concentration may be normal due to the formation of the associated toxic metabolites (ie, formaldehyde and formic acid) . Formaldehyde is metabolized to formic acid, which is pri­ marily responsible for toxicity. In these cases a high anion gap metabolic acidosis should be present.



Helpful Tip

� The

=

m n e m o n i c M E DIE A is hel pfu l i n reca l l i ng the

r1 1r su bsta nces that ca n ca use a n i ncreased osmolar gap: M - Metha nol

E - Eth a n o l

D

- D i u retics/Di l u ents

( e g , m a n n itol, sorbitol) 1 - Isopropyl a lcohol

E - Ethylene g lycol

A - Acetone, a m monia

CHAPTER 27



P O I S O N I N G A N D ENVI RO N M E NTA L EXPOS U R E TO HAZA R D O U S S U BSTA N C E S

The patient has both an anion and osmolar gap; a methanol level is pending. You decide to initiate treatment for this patient. Question 9-5 What medication(s) is/are appropriate for methanol ingestions? A) Fomepizole (4-methylpyrazole [4-MP] ) . B) Ethanol. C) Activated charcoal. D) Both A and B. E) Both A and C. Discussion 9-5 The correct answer is "D:' Methanol toxicity results from its metabolites (formic acid) rather than the parent alcohol. Methanol is primarily metabolized by alcohol dehydrogenase. Fomepizole and ethanol are appropriate treatments for metha­ nol overdoses. Fomepizole works by inhibiting alcohol dehy­ drogenase and is FDA-approved for methanol ingestions. Its indications include an accidental ingestion of methanol (even a sip), intentional methanol ingestion, and altered mental sta­ tus or visual symptoms with an otherwise unexplained osmolar gap or anion gap metabolic acidosis. This medication should be administered in these contexts even before a methanol level has returned. Ethanol has a higher affinity for alcohol dehydroge­ nase than methanol, thereby slowing the production of form­ aldehyde and formic acid. While not FDA-approved, it is still an appropriate treatment either orally or intravenously. Serum ethanol levels must be followed when administering ethanol, with a target range of 1 00 to 1 50 mg/dL. Activated charcoal does not absorb methanol, and it has no role in methanol ingestions. Additionally, since methanol is rapidly absorbed, gastric lavage is not likely to be beneficial. Dialysis rapidly clears methanol and its metabolites and may be needed in cases of severe toxicity.



Helpful Tip

:5.� E n h a nced e l i m i nation via d i a lysis may be used to treat

r1 1r metha nol, ethylene g lycol, l i t h i u m , theophyl l i ne, a n d sa l i cylate toxicity.

� •

l l lr

Helpful Tip •

Eth a n o l toxicity is g e n e ra l ly treated with s u p­

609

A 3-year-old boy is brought into the emergency department after his father found him with a bottle of antifreeze. The liq­ uid had been spilled on the ground surrounding the child. The child's clothes are stained and his breath smells sweet. He has vomited twice and is sleepy. On exam, he is somnolent but arousable. His gait is ataxic.

Question 1 0-1 Which is expected to be seen on laboratory testing? A) Anion gap metabolic acidosis. B) Elevated serum osmolar gap. C) Hypocalcemia. D) Elevated creatinine. E) All of the above.

Discussion 1 0-1 The correct answer is "E:' Ethylene glycol is found in anti­ freeze. Ethylene glycol and methanol both may be found in de-icing solutions, windshield wiper fluid, and industrial solvents. Often the toxic alcohol is present in high con­ centrations. Toddlers may ingest ethylene glycol due to its sweet taste. Ethylene glycol is metabolized to glycoaldehyde, glycolic acid, glyoxylic acid (glycolate) , and oxalic acid. The latter two are responsible for toxicity. Mild toxicity presents similar to intoxication from ethanol, with central nervous system depression, ataxia, nausea, and vomiting. Severe tox­ icity may cause acute kidney injury, seizures, coma, and cere­ bral edema. Similar to methanol, ethylene glycol causes an anion gap metabolic acidosis and elevated osmolar gap, but hypocalcemia and elevated creatinine are unique to ethylene glycol ingestion.

Question 1 0-2 What organ is targeted by the toxic metabolites of ethylene glycol? A) Retina. B) Liver. C) Heart. D) Kidney. E) Muscle.

po rtive m ea s u res, but be ce rta i n to eva l u ate fo r co-i n g estio n s, s i g n s of tra u ma, or oth e r comorbid medical conditions. M etha n o l is meta bol ized to form ald ehyde a n d formic acid, which d i rectly affect the retina a n d optic nerve. Be s u re to perform a fu n d uscopic exa m i nation. •

The anion a n d osmolar g a p ca l c u l ations i n possi ble m eth a n o l i n g estions are q u ite beneficial i n confirm­ i ng the d i a g n osis and g u i d i n g treatment.



The use of fom epizole or ethanol, or both, is i n d i­ cated for m ethanol i n g estions.

Discussion 10-2 The correct answer is "D:' Oxalic acid combines with calcium, causing hypocalcemia and the formation of calcium oxalate crystals. Glycolate-induced renal tubule damage and precipi­ tation of calcium oxalate crystals cause acute kidney injury. Treatment of ethylene glycol ingestion is the same as that for methanol ingestion: fomepizole, ethanol, dialysis, or a combina­ tion of these measures. Dialysis removes the toxic alcohol and its metabolites as well as fomepizole and ethanol; therefore, dos­ ing should be increased for both.

61 0

MCG RAW- H I LL E D U C AT I O N S P E C I A LTY BOA R D REVI EW: P E D I AT R I C S

i QUICKQUIZ

D) How did you come into contact with the substance-ingestion, inhalation, or another way? E) What was the intent behind using the substance?

Which of the following is true regarding alcohol ingestion? A) Methanol is metabolized to glycolate. B) Fomepizole is used in the treatment of methanol and ethylene glycol ingestions. C) Ethylene glycol metabolites are toxic to the retina and optic nerve, so a funduscopic exam should be performed in cases of ingestion. D) Methanol and ethylene glycol ingestion causes a nonanion gap metabolic acidosis. E) Ethylene glycol ingestion always requires dialysis. Discussion The correct answer is "B:' Ethylene glycol is metabolized to glycolate and is toxic to the kidneys. Methanol is metabo­ lized to formic acid and is toxic to the retina and optic nerve. Fomepizole or ethanol are used as treatment, but dialysis may be required for some ingestions of methanol or ethylene glycol. Ingestion causes an anion gap metabolic acidosis and increased osmolar gap.

A 14-year-old boy is brought into the emergency depart­ ment after his father caught him hovering over a substance that the family had been using to thin paint as they remodel their home. The patient is clearly euphoric, singing and skip­ ping about the room. A strong aromatic presence is found on the patient (including his breath) . His vital signs reveal mild tachycardia. The patient's only physical examination finding is diffuse wheezing. Question 11-1 Which question is least relevant in evaluating this patient? A) What was the specific product around which he was found? B) Have you ever done this in the past? C) How much of the substance was used or how many times did you use the substance?

Discussion 11-1 The correct answer is "B:' The patient's past exposure to this substance does not alter his management moving forward, as it does not suggest an increased risk of an adverse outcome. However, the other questions do further the evaluation of this patient. Obtaining as many details as possible about the product in question helps guide your potential intervention and can aid the Poison Control Center in making recommendations. The amount of the exposure to the substance can also suggest the extent of toxicity. Additionally, being aware of the manner in which the substance was consumed can provide insight into the organ systems most likely to be affected. Lastly, being cognizant of whether the patient intended harm to himself has a signifi­ cant impact on the disposition of the patient. After further inquiry, you ascertain that the patient had been huffing (inhaling a substance from a soaked rag) xylene, a hydrocarbon substance commonly used for thinning paint. Additionally, he states that he inadvertently swallowed some of the substance during his hijinks. Your further evaluation, particularly in the setting of his wheezing, includes obtaining a chest X-ray. Question 11-2 All of the following findings can be seen in hydrocarbon inhalation EXCEPT: A) Bibasilar or perihilar infiltrates, or both. B) Lobar consolidation. C) Pneumomediastinum. D) Pleural effusion. E) Tension pneumothorax. Discussion 11-2 The correct answer is "E:' Hydrocarbons are commonly found around the home and encompass a range of products ( eg, paint thinners, lamp oil, kerosene) that can be inhaled, ingested, or both. (See Table 27-3.) The most common chest X-ray finding

TABLE 27-3 CO M M O N H O U S E H O L D P R O D U CTS T H AT CONTA I N HYDROCA R B O N S

Aliphatic

Aromatic

Alicyclic

Halogenated

Essential Oils

Meth a n e

Benzene

P i n e o i l (Pine Sol)

C h loroform

Camphor

Buta ne

To l u e n e

Tu rpenti n e

M ethyl ene ch loride

C i n na m o n

Gaso l i n e

Xylene

P i n e o i l (Pine Sol)

C l ove o i l

Kerose ne

Euca lyptus

M i nera l sea l oil (fu r n i t u re pol i s h )

Pen nyroya l

Paraffi n wax Propa n e Ta r

Pe pperm i nt

CHAPTER 27



P O I S O N I N G A N D ENVI RO N M E NTA L EXPOS U R E TO HAZA R D O U S S U BSTA N C E S

61 1

avoided, particularly in aromatic hydrocarbon exposure. This medication sensitizes the myocardium and can lead to poten­ tially lethal dysrhythmias. This 14-year-old boy will require admission secondary to his pulmonary symptoms and abnormal chest X-ray, among other indications. Not all patients with hydrocarbon expo­ sure require admission. Question 11-4 Which is NOT an appropriate criterion for discharge? A) Symptomatic patient who quickly becomes asymptomatic. B) Asymptomatic patient with an abnormal chest X-ray. C) Patient observed for 2 hours without symptoms. D) Patient without evidence of intent to harm self. E) Asymptomatic patient with a normal chest X-ray and normal pulse oximetry. F I G U R E 27-2. Hyd roca rbon aspiration pneumonitis. A pediatric chest X-ray that reveals peri h i l a r and bibas i l a r infi ltrates, as can be seen in hyd rocarbon aspiration pneumon itis. (Reproduced with permission from Tinti n a l l i J E, Stapczynski JS, Ma OJ, Yealy DM, C l i n e DM, Meckler G D, eds. Tintinalli's Emergency Medicine: A Comprehensive Study Guide. 8th ed. New Yo rk, NY:

McGraw- H i l l Education; 2 0 1 6, Fig. 1 99-1 B.)

associated with hydrocarbon aspiration is increased bronchovas­ cular markings and bibasilar or perihilar infiltrates, or both. (See Figure 27-2.) These abnormalities can actually arise as quickly as 20 minutes postexposure or be delayed by hours. Other chest X-ray abnormalities associated with hydrocarbon inhalation include lobar consolidation (which is uncommon) along with pneumo­ mediastinum and pleural effusions (which are both rare). Tension pneumothorax is not associated with hydrocarbon exposure. Question 11-3 In an attempt to treat this patient's xylene exposure, which of the following measures should be initiated? A) Nasogastric aspiration. B) Activated charcoal. C) Corticosteroids. D) Sodium bicarbonate. E) Epinephrine. Discussion 11-3 The correct answer is "A:' Aromatic hydrocarbons are among those substances that respond to small-bore nasogastric tube aspiration. The CHAMP mnemonic can aid in remembering when nasogastric aspiration is indicated: Camphor, Halogenated hydrocarbons, Aromatic hydrocarbons, Metals ( eg, mercury and lead), and Pesticides. Before implementing this treatment, be sure to consider the risk of aspiration. Activated charcoal is not likely to be effective in the current setting. Corticosteroids might be considered because of the patient's wheezing, but they do not have an appreciable effect on the hydrocarbon itself. Sodium bicarbonate does not have a role in the management of hydrocarbon toxicity. Lastly, epinephrine (which could other­ wise be considered in the setting of bronchospasm) should be

Discussion 11-4 The correct answer is "C:' Patients should be observed for at least 6 hours following hydrocarbon exposure. If asymptom­ atic at that time patients can be discharged to home. This even applies in the setting of an asymptomatic patient with an abnor­ mal chest X-ray provided that his or her vital signs are normal. It is imperative, however, that this type of patient have reliable follow-up. Barring other abnormalities during the evaluation, symptomatic patients who quickly become asymptomatic (with­ out intervention) are considered appropriate for discharge, as are patients who have remained asymptomatic with a normal chest X-ray and vital signs. Finally, prior to discharge it is impor­ tant to ensure that the patient is not a threat to himself or herself. The patient recovers while hospitalized but no one on the health care team has addressed his "huffing" habit. You talk to the patient about the dangers of hydrocarbon inhalation. Question 11-5 Which of the following will you tell him? A) Use may result in irreversible brain damage. B) He is at risk for sudden death. C) He may develop an erythematous rash on his face. D) Both A and B. E) All of the above. Discussion 11-5 The correct answer is "E:' Hydrocarbons are intentionally inhaled as a means to "get high" and may be a gateway drug to other illicit substances. They are cheap, legal, and readily avail­ able. The practice may be called huffing, bagging, or sniffing. Onset of effect is rapid (seconds) but the duration is short ( 1 5 to 4 5 minutes) . An eczema-like rash ("glue-sniffers rash") may develop around the mouth and on the face due to the drying effect of hydrocarbons. Sudden cardiovascular collapse and death (sudden sniffing death) are rare but may occur. Long­ term use may result in permanent brain damage, including cog­ nitive impairment, neuropathy, and cerebellar dysfunction.

61 2

MCG RAW- H I LL E D U C AT I O N S P E C I A LTY BOA R D REVI EW: P E D I AT R I C S



Helpful Tip

�.

Hyd roca rbons a re commonly fou n d a ro u n d the home a n d encom pass a ra n g e of prod ucts that ca n be i n h a l ed, i n gested, or both. Obta i n a ch est X-ray a s a component of the eva lua­ tion for hyd roca rbon exposu re, looking for b i ba s i l a r a n d peri h i l a r infiltrates a s the most c o m m o n fi n d i n g s. S u p portive ca re is genera l ly the treatment of choice for hyd roca rbon toxicity, a lthough nasogastric aspi­ ration should be considered for certa i n su bclasses (eg, halogenated and a romatic hyd roca rbons).

A 9-year-old girl is brought in after picking a handful of pur­ ple bell-shaped flowers. She has thrown up twice and states that her vision is blurry, and she feels weak. Her heart rate is 32 beats per minute. Question 12-1 What is the cause of her presentation? A) Jimson weed. B) Poinsettia. C) Foxglove. D) Lilac. E) Petunia. Discussion 12-1 The correct answer is "C:' Foxglove (Digitalis purpurea), along with lily-of-the-valley and oleander, contains cardiac glycosides and can produce a digitalis effect. The patient classically pres­ ents with weakness, syncope, visual changes, nausea, vomiting, anorexia, confusion, and bradycardia. Toxicity results in mul­ tiple different arrhythmias. Hyperkalemia can occur as well. Treatment entails activated charcoal. Digibind (digoxin-specific antibody Fab fragments) is the specific antidote, which is indi­ cated in patients with cardiovascular collapse, a potassium level greater than 5.5 mEq/L, or life-threatening arrhythmias. Toxicity does not always correlate with the serum digitalis level. Jimson weed produces anticholinergic effects, including tachycardia. Poinsettia toxicity is less severe than once thought, with mild gastrointestinal discomfort as the primary symptom. Lilac and petunia plants are nontoxic. It has been speculated that Vincent van Gogh suffered from digitalis toxicity, as his paintings appear to contain haloes and primarily yellow hues similar to those occurring from toxicity-induced glaucoma and xanthopsia (objects appear to have a yellow hue).

appeared sweaty. She states that he has been teething, but this amount of drooling seemed excessive. Additionally, the child had a couple of bouts of profusely runny diarrhea. Although a couple of other kids at the daycare recently had a viral gastro­ enteritis, the acute onset of this symptom appeared abnormal to her. Thus, she brought him into your clinic to have the boy evaluated. Further inquiry reveals there are no unusual plants or vegetables in the garden. However, you learn that it was recently treated with an unspecified insecticide. You immedi­ ately suspect organophosphate exposure. Question 13-1 Other classic symptoms that you might expect in this patient include all of the following EXCEPT: A) Mydriasis. B) Bradycardia. C) Emesis. D) Bronchospasm. E) Increased urination. Discussion 13-1 The correct answer is "A:' This is the classic presentation of cholinergic toxicity, which can be seen in organophosphate exposure from insecticides, pesticides, and chemical nerve agents (sarin, VX gas), and is perhaps best remembered with the DUMBELS mnemonic: D

-

Diarrhea, diaphoresis

U - Urination M -Miosis, muscle fasciculations B - Bradycardia, bronchorrhea, bronchospasm E - Emesis L - Lacrimation S - Salivation In this case, one might expect the patient to have pinpoint pupils (miosis) as opposed to dilated pupils (mydriasis). It is also important to remember that seizures can occur in up to 25% of cases of pediatric organophosphate toxicity (compared with 3% of adult exposures). Ingestion of wild mushrooms con­ taining muscarine (toxin) can also result in cholinergic toxicity. Organophosphates inhibit acetylcholinesterase, resulting in excess acetylcholine and increased postsynaptic receptor stimu­ lation in the parasympathetic nervous system (muscarinic) and at the neuromuscular junction (nicotinic). Muscarine binds directly to acetylcholine postsynaptic receptors of the parasym­ pathetic nervous system.



Helpful Tip

:5.� Sym ptoms of the choli nergic toxid ro m e i n todd l ers can

i1 1 r be very difficult to d ifferentiate. For i n sta nce, i ncreased

uri nation ca n be difficult to g a u g e at times, a s ca n

An adventurous 2-year-old boy was brought in by a home day­ care provider. She reports that the child had been playing in the garden when he developed watery eyes, began drooling, and

i ncreased sal ivation. Have a low i n d ex of suspicion when the c l i n ica l context sugg ests poss i b l e expo s u re or even access to org a n o phosphates.

CHAPTER 27



P O I S O N I N G A N D ENVI RO N M E NTA L EXPOS U R E TO HAZA R D O U S S U BSTA N C E S

Question 13-2 When evaluating this patient, which of the following labora­ tory testing is essential in guiding the treatment course? A) Plasma acetylcholinesterase level. B) ABGs. C) CBC with differential. D) Basic metabolic and electrolyte panel. E) None of the above. Discussion 13-2 The correct answer is "E:' Although the plasma acetylcholin­ esterase level will help confirm the diagnosis, one should not wait for this result nor is it necessary before initiating treatment. Institute treatment based on the clinical presentation and index of suspicion. The other laboratory tests may assist in evaluating for concomitant end-organ damage and subsequent response to treatment, but they do not provide additional insight into when treatment should be initiated either.

Discussion 13-4 The correct answer is "E:' One of the most important aspects of managing a patient with an organophosphate exposure is to remove the clothing from the patient and consider a shower if it is possible. Failure to do so can lead to persistent symp­ toms (cutaneous absorption) even in the face of appropriate therapy. Intravenous hydration should be considered as symp­ toms necessitate, and naloxone may be warranted in situa­ tions when a patient is altered with a question as to the cause. (See Table 27-4.) Frequent suctioning can be very beneficial in significant exposures, particularly until the atropine aids in drying up the secretions. Sodium bicarbonate does not have a significant role in the management of these cases.



Discussion 13-3 The correct answer is "D:' Appropriate treatments for the cholinergic toxidrome include both atropine and pralidoxime. Atropine, at a dose of 0.05 to 0.2 mg/kg, blocks acetylcholine at the muscarinic receptor sites and works within 1 to 4 minutes. Pralidoxime (2-PAM) , regenerates cholinesterase by reversing the phosphorylation of the enzyme reversing neuromuscular blockade (nicotinic receptors) . It has a synergistic effect with atropine, and symptoms generally start to resolve within 1 0 to 40 minutes. The dose of 2-PAM is 25 to 50 mg/kg. These medi­ cations can be given until muscarinic findings subside, but be wary of using pupillary response as the only indicator of treat­ ment efficacy. Epinephrine does not have a role in such cases. Benzodiazepines may be used to treat seizures. Question 13-4 In addition to initiating the appropriate antidote therapy, all of the following treatment interventions should be strongly considered in organophosphate toxicity EXCEPT: A) Decontamination and removal of clothing. B) Frequent suctioning. C) Intravenous hydration. D) Naloxone. E) Sodium bicarbonate.

Helpful Tip

::'Jl l n

.s.

organophosphate poiso n i ng, treatment i n c l u d i n g

i1 1r atropine a n d p ra l idoxi me (2-PAM) s h o u l d be i n itiated

based on the c l i n ical presentation a s opposed to

A plasma acetylcholinesterase level is ordered from the laboratory, but it will be quite some time before it returns. You astutely elect to initiate treatment for this young boy. Question 13-3 What medication is the treatment of choice for organophos­ phate toxicity? A) Atropine. B) Pralidoxime (2-PAM) . C ) Epinephrine. D) Either A or B. E) Either A or C.

61 3

wa iting for a plasma acetylc h o l i n estera se leve l .

TABLE 27-4 A N T I DOTES A N D I N G ESTA N TS

Ingestant/Toxicant

Antidote

Sa l i cyl ate

None

Nonsteroid a nti­ i nfl a m m atory d ru g s ( N SAI Ds)

None

Aceta m i nophen

N-acetyl cyste i n e (NAC)

Tricyc l i c antidepressa nts

None

I ro n

Deferoxa m i n e

Lith i u m

None

Sel ective se roto n i n re u pta ke i n h i bitors (SSRis)

None

D i p h e n hyd ra m i ne

P hysostig m i n e

Opioids I na rcotics

N a l oxo ne

Insulin

G l ucagon

S u lfonyl u reas

Octreotide

Be nzod iazepi nes

F l u maze n i l

Org a n o p hosphates

Atro p i n e

H e pa r i n

Prota m i n e

Dextrose

Fresh frozen plasma Wa rfa r i n

Vita m i n K Fresh frozen plasma

Beta-blocker

G l ucagon

Carbon monoxide

Hyperba ric oxyg en

Ethylene g lyco l/Met h a n o l

Fomepizo l e and Eth a n o l

614

MCG RAW- H I LL E D U C AT I O N S P E C I A LTY BOA R D REVI EW: P E D I AT R I C S

On a very cold winter night a 5-year-old boy is brought into the emergency department by his parents. They report that he woke them up complaining of feeling sick to his stomach and having a significant headache. When they tried to walk him back to his room, he stumbled into a wall. These symptoms concerned the parents enough to seek medical intervention. The boy had oth­ erwise been healthy recently. On arrival, his vital signs reveal tachycardia to 1 15 beats per minute, tachypnea at 20 respira­ tions per minute, and a normal pulse oximetry reading of 98%. Question 14-1 Pertinent additional questions related to this patient's pre­ sentation include all of the following EXCEPT: A) Does anyone in the home have similar symptoms or otherwise feel ill? B) Have you noticed any issues with your furnace recently? C) Has the child or family recently taken a long trip? D) Does the child have a space heater or other source of supple­ mental heat in the room? E) Do you have any pets? Discussion 14-1 The correct answer is "C:' The presentation of this patient is concerning for carbon monoxide poisoning. Carbon monoxide is an odorless, tasteless gas. Poisoning may result from smoke inhalation (eg, house fire), malfunctioning furnaces, poor ven­ tilation when using a fuel-burning device (eg, heaters, char­ coal grills, generators), or running vehicle engines in enclosed spaces. The symptoms of this toxicity can be subtle and nonspe­ cific, including headache, dizziness, ataxia, confusion, nausea, vomiting, and dyspnea. Thus, gathering additional information about possible sources or other affected individuals and pets can be exceptionally valuable. Be sure to ask whether anyone else in the home has symptoms, including pets. Asking about the status of the furnace ( eg, "Has your furnace been serviced recently? "; "How old is your home or furnace?") can provide valuable insight. Sources of supplemental heat can result in car­ bon monoxide toxicity and should be considered. A patient's recent travel history is not germane to the evaluation of possible carbon monoxide poisoning. Cherry red lips and nail beds are useful when present but frequently are absent.

� •

Helpful Tip

Fu rnaces, space h eaters, fires, a n d automobile exha u st

1 1 1r a re a l l commonly considered as potentia l ca uses of carbon monoxide poiso n i ng,

but rem e m ber that

su bsta nces conta i n i n g methylene ch loride (eg, pa int removers, fu rniture stri ppers) can a l so cause carbon monoxide toxicity.

Upon further questioning you learn that everyone in the home feels well. However, the patient has been using a space heater in his room because of a recent cold snap.

Question 14-2 What laboratory testing should be obtained to help confirm the diagnosis while you initiate treatment? A) CBC with differential. B) Basic metabolic panel. C) ABGs. D) Carboxyhemoglobin level (COHb). E) No testing is required; pulse oximetry is all that is needed to guide management of this patient. Discussion 14-2 The correct answer is "D:' The COHb, measured by co-oxime­ try of a blood sample, is vital in the management of this patient. Although the value of this test may be deceptive at times ( eg, not markedly elevated despite the patient being critically ill, or unexpectedly low if a significant amount of time has passed) and may not correlate with the severity of poisoning, it is the first test that should be obtained when carbon monoxide exposure is expected. Normal values tend to be elevated to approximately 3%, although patients who smoke can have a COHb value up to 10%. ABGs would be beneficial in the management of the patient to assess for hypoxemia (and even metabolic acidosis, in severe cases); however, this test would not confirm the diagnosis. A CBC with differential and basic metabolic panel can provide some useful information as well ( eg, anion gap), but again they will not confirm the diagnosis. Remember, too, that pulse oximetry readings would likely be normal in carbon monoxide poisoning because this device cannot differentiate between oxyhemoglo­ bin, carboxyhemoglobin, and other abnormal hemoglobins (ie, methemoglobin) . Once the patient's blood work i s obtained and sent t o the laboratory for analysis you immediately institute treatment. Question 14-3 What is the first intervention that should be provided to this patient? A) Intravenous dextrose. B) Supplemental oxygen. C) Rapid sequence intubation. D) Methylene blue. E) Thiamine. Discussion 14-3 The correct answer is "B:' The most important initial treatment for carbon monoxide poisoning is to provide 1 00% oxygen by face mask. If the patient is not protecting his or her airway, intu­ bation may be necessary. However, that is not the case in this scenario. High-flow oxygen decreases the half-life of COHb from 300 minutes to approximately 90 minutes. Intravenous dextrose may be necessary as a supportive measure but is not a first-line treatment. Thiamine is reserved for patients with altered mental status of unknown etiology, which is most common in adults. Lastly, methylene blue is the antidote for methemoglobinemia. He is receiving supplemental oxygen and remains unsteady on his feet. The lab calls. The young boy's COHb level is 22%.

CHAPTER 27



P O I S O N I N G A N D ENVI RO N M E NTA L EXPOS U R E TO HAZA R D O U S S U BSTA N C E S

Question 14-4 What intervention should now be considered, given his presentation? A) Hyperbaric oxygen. B) Continued high-flow supplemental oxygen. C) Observation without supplemental oxygen. D) Rapid sequence intubation. E) None of the above. Discussion 14-4 The correct answer is "A:' Indications for hyperbaric oxygen therapy in acute carbon monoxide poisoning are unclear and controversial. The goal of hyperbaric therapy is to prevent long­ term neurocognitive damage. Possible indications include loss of consciousness, syncope, persistent neurologic symptoms, cardiac ischemia on ECG, elevated COHb level in pregnant patients, and a high COHb level. For many providers this may necessitate a transfer to a facility with the appropriate resources. Treatment with hyperbaric oxygen will further reduce the half-life of COHb to 20 minutes. Additional supplemental oxygen therapy may be beneficial for the patient, but more aggressive intervention is indicated. And, although the patient is ill, he is otherwise stable; thus rapid sequence intubation is not necessary at this time. In this case, the patient has persis­ tent neurologic findings (eg, ataxia) and an elevated COHb level despite high-flow supplemental oxygen, therefore; hyperbaric oxygen therapy is indicated.

� QUICKQUIZ What is NOT measured by co-oximetry testing of a blood sample? A) Oxyhemoglobin. B) Carboxyhemoglobin. C) Methemoglobin. D) Reduced hemoglobin. E) All of the above. Discussion The correct answer is "E:' A standard pulse oximeter is not useful to screen for carbon monoxide poisoning or methemo­ globinemia. It cannot differentiate COHb or other abnormal hemoglobins from oxyhemoglobin, thus giving a falsely elevated normal reading. Do not be falsely reassured by a normal pulse oximetry reading.

A 1 6-year-old female patient inadvertently swallowed her imitation tongue piercing, which consisted of two metallic pieces. Plain X-rays of the abdomen (anteroposterior and lateral) suggest that the metallic objects are in the fundus of the stomach.

61 5

Question 15-1 What should be your next step in the management of this patient? A) No interventions are necessary; the patient can be dis­ charged to home. B) Observe the patient for at least 4 hours to ensure passage of the magnets. C) Administer a promotility agent to facilitate passage of the magnets. D) Consult a pediatric surgeon for an exploratory laparotomy. E) Consult a pediatric gastroenterologist to discuss removing the magnets. Discussion 15-1 The correct answer is "E:' Magnet ingestion by pediatric patients is a growing concern, particularly as magnets are being used increasingly in products. The use of magnets in commercial products includes jewelry as in the aforementioned case. This patient has swallowed two magnets that formed her imitation tongue ring. The number of magnets and their location are the most important concerns in managing these patients. Initially, a provider should obtain anteroposterior and lateral X-rays of the abdomen to assess their location within the abdomen. Single magnet ingestions found in the esophagus or stomach should be considered for endoscopic removal, although some advocate for serial imaging to ensure movement of the magnet in otherwise asymptomatic patients. Multiple magnets or magnetic/metallic objects may attract one another between two loops ofbowel caus­ ing pressure necrosis, perforation, obstruction, or volvulus. All patients with multiple magnet ingestions should have all magnets that are accessible by endoscopy removed. If the magnets are fur­ ther along the gastrointestinal tract in an asymptomatic patient, serial abdominal X-rays can be obtained every 4 to 6 hours. How­ ever, removal by enteroscopy or colonoscopy is still advised if the magnets can be accessed. Patients who are symptomatic after ingesting multiple magnets located beyond the reach of endos­ copy should be evaluated by a pediatric surgeon for consideration of a surgical intervention. Promotility agents are not used to facil­ itate passage of magnets through the gastrointestinal tract.

A 7-year-old boy is playing outside his home, an old farm­ house, unsupervised. His father finds him lying on the ground seizing. He is brought directly into the emergency department where you evaluate him. Although his seizures have stopped, physical examination reveals dark linear lines on the gingiva. Question 16-1 What is the likely cause of the patient's presentation? A) Iron toxicity. B) Organophosphate toxicity. C) Lead toxicity. D) Anticholinergic toxicity. E) Ethylene glycol toxicity.

616

MCG RAW- H I LL E D U C AT I O N S P E C I A LTY BOA R D REVI EW: P E D I AT R I C S

Discussion 16-1 The correct answer is "C:' The patient history suggests an older home, possibly one painted with lead-based paint. The occurrence of seizures may not be specific, but it is associ­ ated with severe lead toxicity and may be both prolonged and refractory. Burton lines are blueish-black lines on the gin­ giva seen in lead toxicity, though uncommonly. This constel­ lation of contextual clues and symptoms is not associated with the other options listed. Lead is a neurotoxin, and even at minimally elevated levels is associated with long-term effects on cognition and learning. The primary sources are lead paint, and dust and soil contaminated by flakes of lead paint. Certain folk remedies may contain lead. Symptoms of lead intoxication include irritability, headaches, abdomi­ nal pain, constipation, and clumsiness. Severe toxicity may cause seizures, coma, vomiting, and encephalopathy. Inter­ vention is based on the blood lead level of a venous sample. An abnormal capillary value must be confirmed by a venous lead level. A lead level less than 10 mcg/dL is acceptable. Patients with values higher than 10 mcg/dL require repeat testing, monitoring, education, and possible environmen­ tal investigation for lead sources or treatment, depend­ ing on the exact value. (See Table 2 7 - 5 . ) A diet containing adequate vitamin C, iron, and calcium is recommended. Parents should be educated about lead toxicity, especially as it relates to infants and toddlers, who like to put obj ects in their mouths.

Suspecting chronic lead toxicity, you order a venous lead level.

Question 16-2 In chronic cases, at which level do most patients become symptomatic? A) Greater than 10 mcg/dL. B) Greater than 20 mcg/dL. C) Greater than 40 mcg/dL. D) Greater than 60 mcg/dL. E) Greater than 1 00 mcg/dL. Discussion 16-2 The correct answer is "D:' A normal lead level is less than 10 mcg/dL, whereas levels greater than 1 00 mcg/dL can result in severe encephalopathy. Generally, it is thought that acute lead exposures resulting in lead levels greater than 40 meg/ dL can result in symptoms. However, while manifesting acutely, this patient's presentation is more consistent with chronic exposure (suggested by the farmhouse exposure and presence of gingival changes) . Clinical examination findings may not become appar­ ent until the lead level surpasses 60 mcg/dL.



Helpful Tip

:S.� There is controversy a s to t h e lead l evel at w h i c h even

r1 1r asym ptomatic c h i l d re n s h o u l d be treated. Reg a rd l ess, any level g reater t h a n 1 0 mcg/d L s h o u l d pro m pt a n

i nvestigation i nto poss i b l e sou rces o f l ead exposu re.

The patient's venous lead level returns at 80 mcg/dL. It has been 3 hours since his initial presentation.

TABLE 27-5 T R EATM E N T OF L E A D TOX I C ITY

Route of

Blood Lead Concentration

Classification

Interventions

Chelation Agent

Administration

1 0- 1 9 mcg/d l

Mild i ntoxication

Diet, Ed ucation•

None

NA

Diet, Ed ucation•

None

NA

Di me rca ptos ucci n ic acid (succ i m e r)

PO

20-44 m cg/d l

Envi ro n mental eva l uation ± Bowel deconta m i nation 45-69 m cg/d l

Moderate i n toxication

Diet, Ed ucation• Envi ro n mental i nvestigation

;:: 70 m cg/d l Sym ptomatic

Severe i ntoxicati on

or

Ca Na E DTA 2

IV

Hospita l ize

BAL (d i merca prol)

IM

± Bowe l deconta m i nation

and

± Bowel deconta m i nation

I ntolerant of succi mer

Ca Na E DTA 2

IV

BAL, British a nti-Lewisite; Ca N a EDTA, ca l c i u m disod i u m ethyl ened i a m i n e-tetraacetate; C N S, centra l nervous system; I M, i ntra m uscu l a r; 2 IV, i ntravenous; NA, not a p p l icable; PO, ora l . 'Education i n c l udes d i etary a n d enviro n menta l c h a n g es.

CHAPTER 27



P O I S O N I N G A N D ENVI RO N M E NTA L EXPOS U R E TO HAZA R D O U S S U BSTA N C E S

Question 16-3 What treatment should you implement? A) Activated charcoal. B) Dimercaprol (British antilewisite [BAL] ) . C ) Calcium disodium ethylenediamine-tetraacetate (CaNa2 EDTA) . D) Deferoxamine. E) Both B and C. Discussion 16-3 The correct answer is "E:' The initial treatment in severe lead toxicity is dimercaprol (BAL) and CaNa2 EDTA. Both medica­ tions are chelating agents. CaNa2 EDTA should be started after the second dose of BAL is administered. Chelation therapy is continued for 5 days. Lead levels should be rechecked after stop­ ping therapy, as repeat courses may be needed. Activated char­ coal is not effective in lead toxicity. Deferoxamine is a treatment agent for iron ingestions. The magical number to start chelation therapy is a lead level of 45 meg/ dL or higher. A value of 70 meg/ dL or higher, symptoms of lead poisoning, or inability to toler­ ate oral succimer requires hospital admission and treatment with intravenous chelation therapy. If lead flecks are present on an abdominal X-ray, whole bowel irrigation should be performed. Consult the Poison Control Center or an experienced toxicolo­ gist when managing lead toxicity. After 5 days of treatment with BAL and CaNa2 EDTA, the patient appears well and at his baseline. He has had no addi­ tional seizures. His lead level is rechecked and found to be 40 mcg/dL. Interestingly, his lead level the day before was 1 5 mcg/dL. Question 16-4 How can you explain this result? A) The child has been exposed to additional lead. B) There has been a treatment failure. C) A laboratory error has likely transpired. D) The child has hematochromatosis. E) Lead redistributes into the serum after chelation. Discussion 16-4 The correct answer is "E:' Chelation therapy (with either BAL or CaNa2 EDTA) results in lead redistribution into the serum. Thus, while a patient may be clinically asymptomatic, one might note a rise in the serum lead level. This is not uncom­ mon and does not signify a treatment failure. It is unlikely that a patient would be exposed to additional lead while hospitalized. Although a laboratory error should be considered, it is far less likely to occur. Lastly, hematochromatosis is an inherited condi­ tion involving iron accumulation.



Helpful Tip

:5.� A lead l evel of 45 mcg/d l o r h i g h e r req u i res chelation

r1 1r thera py. M a ke s u re the c h i l d wi l l ta ke the ora l succi mer, as it has a "rotten egg" odor.

61 7

A 14-year-old boy was practicing a new magic trick with a nickel. In his attempt to make the coin disappear, and while technically successful, he inadvertently "swallowed" the coin. He presents to the emergency department after this episode. He appears to be in no acute distress. The patient's vital signs are normal. A chest X-ray shows a circular radiopaque object just above carina. Question 17-1 What is the next step in your management of this patient? A) Consult pediatric gastroenterology for endoscopy. B) Consult pediatric pulmonology for bronchoscopy. C) Secure additional testing. D) Observe the patient in the emergency department. E) Discharge the patient; no further evaluation or intervention is necessary. Discussion 17-1 The correct answer is "C:' The information provided by the chest X-ray is not sufficient to determine if the patient has aspi­ rated or swallowed the coin. As such, selecting a disposition for the patient is difficult at this juncture. Obtaining a lateral chest X-ray might provide additional insight. Aspirated coins often become tilted when caught on tracheal rings. Thus, one might see an angulated coin. Another benefit of the lateral view is that one might be able to appreciate the air-filled trachea distinctly from the location of the coin, suggesting that it has been swal­ lowed. (See Figure 27-3.) Additional imaging determines the coin is in the patient's distal esophagus. He is having no trouble breathing or swallowing. Question 17-2 What should be done now? A) Consult pediatric gastroenterology for endoscopy. B) Consult pediatric pulmonology for bronchoscopy. C) Secure additional testing. D) Observe the patient. E) Discharge the patient; no further evaluation or intervention is necessary. Discussion 17-2 The correct answer is "D:' Asymptomatic patients with esoph­ ageal coins may be observed for 24 hours to see if the coin will pass into the stomach. Coins in the proximal esophagus are less likely to pass. An esophageal coin should be removed if the patient is symptomatic (cannot swallow secretions), the time of ingestion is unknown, there is uncertainty about whether obj ect is a disc (button) battery or coin, or the obj ect fails to pass into the stomach after 24 hours of observation. (See Table 27-6.)

618

MCG RAW- H I LL E D U C AT I O N S P E C I A LTY BOA R D REVI EW: P E D I AT R I C S

i QUICKQUIZ Which is NOT an indication for emergent endoscopy with removal of the foreign body? A) A toddler with a swallowed safety pin located in the esophagus. B) A 1 0-year-old who is unable to swallow his secretions and has a swallowed quarter in the proximal esophagus. C) A 1 7 -year-old with a disc (button) battery in his distal esophagus. D) A 5-year-old with multiple magnets in the stomach. E) All of the above are indications for emergent endoscopy.

A

B

Discussion The correct answer is "D:' Multiple magnets in the stomach require urgent endoscopic removal but not emergent.

A 3-year-old girl found her way into her mother's medicine cabinet roughly 7 hours prior to presentation at your clinic. The only medication in the cabinet was an iron supplement for iron-deficiency anemia. This bottle was opened when the mother found the young girl playing at the cabinet, but it was not initially clear if the girl had consumed any tablets. Shortly after being found the girl reported some vague "tummy pain;' which resolved. After a period of observation at home, the mother thought it best to have her daughter evaluated. Upon arrival the child appears well and has no complaints. Her vital signs are within normal limits.

F I G U R E 27-3. Esophageal forei g n body. Posteroa nterior (A) a n d latera l (B) chest X-rays s how a rou n d radiopaque fo rei g n body in the esophagus consistent with a coi n . A lateral chest X-ray may h e l p identify the a n ato m i c location o f the c o i n , as the a i r-fi l led trachea can often be d e l i neated fro m the esophagus. (Reprod uced with permission from Stone CK, H u m p h ries RL, eds. Current Diagnosis and Treatment: Emergency Medicine. 7th ed. New York, NY:

McGraw-H i l l Education; 2 0 1 1 , Fig. 50-9A, B.)

TABLE 27-6 SWA L LOWED F O R E I G N B O D I ES T H AT

R E Q U I R E E N DOSCO P I C R E M OVA L

S h a r p o bject H i g h- powered mag net M u ltiple m a g n ets Lo ng o bject (> 6 em) Lead-conta i n i ng objects Coi n s in the esop h a g u s > 24 h o u rs Button/d isc battery i n esop h a g u s Button/d isc battery o r cyl i n d rical battery i n stomach > 48 h o u rs Ai rway obstruction (tracheal com pression) Esophageal obstruction (ca n not swa l l ow secretions) I ntesti n a l obstruction

Question 18-1 What test will provide you with the most efficient means for determining whether the patient consumed any iron tablets? A) CBC with differential. B) Liver function tests. C) Serum iron level. D) Computerized tomography (CT) scan of the abdomen and pelvis. E) Abdominal X-ray. Discussion 18-1 The correct answer is "E:' Iron tablets are radio-opaque, and thus they should be apparent on X-ray in the gastrointestinal tract of a patient who has consumed them. However, there are caveats to consider with regard to the utility of abdominal X-rays in iron ingestions. The absence of radio-opaque pills or tablets does not rule out iron ingestion, including a potentially lethal ingestion. It could also be the case that the iron has dissolved. Additionally, liquid preparations and children's chewable tablets and multivitamins are not often visible on X-rays. A CBC with differential may suggest signs of anemia due to gastrointestinal hemorrhage from iron toxicity, while liver function tests may sug­ gest end-organ damage later in a patient's course. However, neither test is likely to help determine if any iron has been consumed.

CHAPTER 27



P O I S O N I N G A N D ENVI RO N M E NTA L EXPOS U R E TO HAZA R D O U S S U BSTA N C E S

Serum iron levels should certainly be evaluated, as peak absorp­ tion of the iron occurs between 2 and 6 hours after ingestion. In a patient with acute iron toxicity, an anion gap metabolic acido­ sis, leukocytosis, hyperglycemia, or a combination of these find­ ings may be present. With that said, one can gain more prompt insight into a possible ingestion with X-rays, utilizing the serum iron level as further evidence of an iron ingestion. A CT scan of the abdomen and pelvis would likely reveal iron tablets if they have been consumed. Yet, the significant radiation, finan­ cial cost, and delay in securing this imaging negates its utility in the initial assessment of potential iron ingestions. Iron toxic­ ity depends on the quantity of elemental iron ingested. Differ­ ent formulations (ferrous fumarate > ferrous sulfate > ferrous gluconate) contain different amounts of elemental iron. Inges­ tions of less than 20 mg/kg of elemental iron typically do not result in symptoms. Toxic levels generally occur with ingestions of greater than 40 mg/kg elemental iron and lethal levels with ingestions of greater than 60 mg/kg elemental iron.

� •

1 1 1r

Helpful Tip

I ro n i n g estions h a v e h i storica l ly resu lted i n the h i g h est i n g estion-related morta l ity i n c h i l d re n . With that said,

61 9

manifests 2 to 4 weeks later. It should be noted that patients can skip any of these stages. But if a patient does not experience stage 1 within 6 hours of an ingestion the amount of iron consumed is likely not significant. This "6-hour rule'' does not apply if enteric­ coated tablets were consumed.

� •

1 1 1r

Helpful Tip

I ro n is d i rectly corrosive to the gastroi ntesti n a l m ucosa a n d may ca use l ife-th reaten i n g hemorrhage, necrosis, or both.

Her iron level returns from the laboratory at 550 mcg/dL. Question 18-3 What is the first-line therapy that should be initiated for this patient? A) Activated charcoal. B) Gastric lavage. C) Oral deferoxamine. D) Intravenous deferoxamine. E) Sodium bicarbonate.

c h i l d ren's chewa ble i ron prod ucts have been shown to be safe for use a s d i rected, although they freq uently wi l l not show u p on plain fi l m i m a g i n g .

X-rays of the child's abdomen confirm the presence of radio­ opaque tablets in the stomach. Question 18-2 Given the apparent iron ingestion, in which stage of iron toxicity is this patient? A) Stage 1 . B ) Stage 2 . C ) Stage 3 . D ) Stage 4. E) Stage 5. Discussion 18-2 The correct answer is "B:' The stages of iron toxicity are impor­ tant, particularly the second stage. During stage 1 (30 minutes to 6 hours after ingestion), patients typically experience nonspe­ cific gastrointestinal symptoms such as abdominal pain, nausea, emesis, and diarrhea or gastrointestinal hemorrhage (hemateme­ sis, melena) may occur. Stage 2 can be clinically problematic, as it is referred to as the latent stage. Typically commencing 6 to 12 hours after ingestion, patients are often asymptomatic, with their previous gastrointestinal symptoms having resolved. It is not uncommon for patients to present at this stage. Between 12 hours and 48 hours after ingestion patients can enter stage 3, denoted by hypoperfusion, shock, metabolic acidosis, coma and acute liver failure with hypoglycemia and coagulopathy. This can progress to stage 4 (2 to 6 weeks after ingestion), highlighted by liver fail­ ure from hepatic cirrhosis and emesis from gastric outlet or small bowel obstructions from intestinal scarring. This stage usually

Discussion 18-3 The correct answer is "D:' A peak serum iron level should be obtained 4 to 6 hours after ingestion. Deferoxamine is a highly spe­ cific iron chelator. It binds with ferric iron, forming a water-soluble compound excreted by the kidneys. Because it results in more constant levels in the bloodstream, intravenous deferoxamine is preferred over other formulations. Indications for the use of defer­ oxamine include metabolic acidosis, severe symptoms, and serum level greater than 500 mcg/dL or greater than 350 mcg/dL when pill fragments are present on X-ray. The preferred dosing of this medication is an initial continuous infusion rate of 1 5 mg/kg/h, increasing the dose as needed. Maximum doses up to 45 mg/kg/h have been reported. The infusion rate should be decreased if hypo­ tension occurs. Activated charcoal will not absorb iron, and stud­ ies have not shown gastric lavage to improve outcomes in such ingestions. Sodium bicarbonate, without an associated metabolic acidosis, is not a therapy for iron toxicity. Of note, endoscopic removal of the tablets or whole bowel irrigation ( eg, polyethylene glycol at 1 0- 1 5 mL/kg/h) would be appropriate treatment options as well, given the presence of pills in the patient's stomach. The pharmacist calls you regarding your order for intrave­ nous deferoxamine. There is some question as to whether this medication is actually indicated. Question 18-4 Which of the following is NOT an indication for implementing deferoxamine treatment in the setting of an iron overdose? A) Altered mental status. B) Persistent gastrointestinal symptoms. C) A serum iron level greater than 500 mcg/dL. D) Hypotension. E) Metabolic alkalosis.

620

MCG RAW- H I LL E D U C AT I O N S P E C I A LTY BOA R D REVI EW: P E D I AT R I C S

Discussion 18-4 The correct answer is "E:' The presence of a metabolic acidosis is an indication to initiate deferoxamine therapy, but metabolic alkalosis is not. The other options are all appropriate reasons to commence with treatment. In the case described, the patient had a serum iron level greater than 500 mcg/dL and the pres­ ence of pill fragments on plain films. Also consider that patients with late presentations (> 8 hours) may have artificially lower serum iron levels owing to redistribution intracellularly yet still warrant deferoxamine therapy.

� QUICKQUIZ Regarding iron toxicity, which of the following is false? A) Iron is often radiolucent, and plain films of the abdomen can be useful in detecting an ingestion. B) Iron ingestions can involve many different potential sources, including iron tablets, toys, weights, and imported items such as pottery or cooking vessels. C) The stages of iron toxicity are important, particularly stage 2, when patients may appear asymptomatic. One must have a high index of suspicion at this stage. D) The treatment of choice for iron toxicity is intravenous deferoxamine, which has several indications, including an iron level greater than 500 meg/ dL or an iron level greater than 350 mcg/dL when pill fragments are seen on plain films. E) Patients who have accidentally ingested less than 40 mg/kg of elemental iron with only mild gastrointestinal symptoms may be watched at home. Discussion The correct answer is ''A:' Iron is radio-opaque not radiolucent, but iron tablets may not always be seen on X-ray.

D) Liquefaction necrosis of the esophagus due to a caustic alkali ingestion. E) Both coagulation and liquefaction necrosis of the esophagus due to a caustic acid ingestion. Discussion 19-1 The correct answer is "D:' Dishwasher and laundry pods are concentrated forms of cleaning detergents, which have become an increasing hazard for young children. Not only is their con­ centrated formulation a concern for a more severe esophageal or airway injury if ingested or aspirated, their appearance is often bright and shiny, leading young children to perceive them as candy. Common household agents such as oven and drain cleaners along with traditional laundry detergents can also lead to significant esophageal injury if ingested. Acidic preparations of such substances lead to coagulation necrosis. These types of injuries tend to be less severe because they are self-limiting as the coagulation process prevents deeper pen­ etration of the caustic substance. Alkali ingestions, including those that might result from the ingestion of a dishwasher detergent pod or classically lye, can lead to liquefaction necro­ sis. This type of injury causes the disintegration of the mucosa and potentially perforation of the esophagus. In treating these ingestions, neutralizing substances, diluting agents (eg, milk) , and activated charcoal are not recommended. Patients with symptoms (dysphagia, hematemesis, retrosternal chest pain) or oral burns following an ingestion or those known to have ingested a substance associated with significant esophageal injury (eg, drain cleaner) should undergo endoscopy. Other­ wise, asymptomatic patients should be observed for several hours to ensure no further complications develop (so long as the substance ingested is of low causticity) . Long-term compli­ cations include the formation of esophageal strictures, which may cause obstruction.

� •

An IS-month-old toddler was cruising through the kitchen one morning when he discovered a bright and shiny dish­ washer detergent pod on the floor. The young lad promptly placed this directly into his mouth. Minutes later his mother noticed the presence of detergent granules around his mouth. The patient was immediately brought into your emergency department for further evaluation and treatment. Question 19-1 What is the immediate concern regarding this ingestion? A) Coagulation necrosis of the esopagus due to a caustic acid ingestion. B) Liquefaction necrosis of the esophagus due to a caustic acid ingestion. C) Coagulation necrosis of the esophagus due to a caustic alkali ingestion.

Helpful Tip

After caustic i n g estion of an a l ka l i or acidic agent,

I 1 1r patients s h o u l d not be forced to vom it. This may cause repeat esophagea l burn as the caustic su bsta nce moves back i nto the esopha g u s .

A 6-year-old boy presents to your clinic with his mother, who is concerned that he may have inadvertently swallowed a small, cylindrical battery from his Billy the Bot toy. The child denies any associated symptoms, and he appears well. His vital signs are within normal limits, and his pulmonary and abdominal examinations are unremarkable. You obtain plain films of the chest and abdomen to evaluate for the presence of a battery. A small radio-opaque object seemingly consistent with a battery is noted in the stomach.

CHAPTER 27



P O I S O N I N G A N D ENVI RO N M E NTA L EXPOS U R E TO HAZA R D O U S S U BSTA N C E S

621

Question 20-1 What is your next course of action? A) Consult a pediatric gastroenterologist to discuss endoscopic removal of the battery. B) Admit the patient for serial abdominal examinations and repeat imaging. C) Observe the patient in the clinic with repeat plain films in 6 hours. D) Discharge the patient to home with follow-up as needed. E) Discharge the patient to home with follow-up scheduled for 2 days. Discussion 20-1 The correct answer is "E:' The management of battery inges­ tions is important as they may result in caustic alkaline injuries to the esophagus. Fortunately, there are several guidelines that can help with the management of patients with battery inges­ tions. When the battery (disc or cylindrical) is found to be in the stomach, any child younger than 5 years old (symptom­ atic or otherwise) should be seen by a pediatric gastroenter­ ologist at the initial presentation. For asymptomatic children 5 years of age and older, a pediatric gastroenterologist should be consulted if the battery remains in the stomach on follow­ up plain films 48 hours after the ingestion. A disc (button) bat­ tery located in the esophagus is a different story. This requires emergent removal to prevent necrosis. Thus, in this case the asymptomatic 6-year-old can be discharged to home with a regular diet and normal activity level with instructions to return in 48 hours for repeat imaging. If the battery remains in the stomach at that time it should be removed endoscopi­ cally. Follow-up imaging is also indicated if the patient devel­ ops symptoms, to prove passage of the battery if it has not been visualized in the stool l to 2 weeks after ingestion, and to mon­ itor the weekly progression of the battery if it has not passed and has been previously seen in the intestines on plain films. Batteries that find their way out of the stomach and into the intestines generally pass without complication. Lastly, keep in mind that the larger the battery ( > 1 5-20 mm) the more likely it is that there will be some difficulty with passage.

� •

Helpful Tip

A d isc (b utton) battery located in the esoph ag u s is a n

1 1 1r emergency whereas a coin m a y n ot b e . On X-ray, look for a d o u b l e-ring o r halo a ppea ra n ce, which refl ects the battery's biconvex sha pe. (See F i g u re 2 7-4.)

As a civic-minded physician, you agree to speak at the local school to parents, students, and teachers regarding health and the environment for Earth Day. Although the majority of the presentation is left to your discretion, the organiz­ ers have asked you to touch on a couple of specific topics aimed at highlighting the interplay between the health of the

F I G U R E 27-4. A d i s c battery is l ocated in t h e prox i m a l e s o p h a g u s . O n X-ray, a d i s c battery h a s a d o u b l e - r i n g o r a s u rrou n d i n g h a l o, w h i c h h e l ps d i ffe rentiate it fro m a c o i n . ( Re p ro d uced w i t h p e r m i s s i o n fro m K n o o p KJ, Sta c k L B, Sto r row AB, Th u r m a n RJ ( E d s ) . The A tlas of Emergency Medicine, 3 e d . M c G raw- H i l l E d u ca t i o n , I n c., 2 0 1 0. P h oto contri butor:

Scott M a n n i n g, M D.)

community and the environment. It is suggested that you spend a portion of your presentation discussing the possible dangerous substances that could find their way into the com­ munity's food and water supply if individuals and industries are not vigilant. Question 21-1 Which of the following is NOT an example you might cite as such a contaminant? A) Mercury. B) Fluoride. C) Arsenic. D) Escherichia coli (E. coli) . E) Cryptosporidium. Discussion 21-1 The correct answer is "B:' Toxic substances can contaminate food and water supplies, impacting the health of an entire com­ munity. Fluorinated water has been shown to reduce cavities in children by up to 40% in children. Fluoride is generally not thought to be a significant hazard to a community's food or water supply. The other remaining options present notewor­ thy potential harms to public health, particularly to developing children. Heavy metals such as mercury and arsenic or bacteria (eg, E. coli and Cryptosporidium) can result in both acute illness and chronic sequelae. One ofyour talking points will center on the chemical hazards that can exist in the community.

622

MCG RAW- H I LL E D U C AT I O N S P E C I A LTY BOA R D REVI EW: P E D I AT R I C S

Question 21-2 Which of the following is NOT a common source of contami­ nation that may affect the health of the community? A) Pesticide runoff. B) Nuclear power. C) Electric or magnetic fields ( eg, power lines, household appliances). D) Automobile exhaust. E) Industrial waste. Discussion 21-2 The correct answer is "C:' It is worthwhile to be aware of the potential chemical hazards that exist around us. The list of these entities is lengthy and includes all the options listed above except electric and magnetic fields. Recent studies have sug­ gested that these fields (which are generated by electrical power lines and household appliances) likely do not result in leukemia, brain tumors, or other types of cancer as previously speculated. To date, the literature on the topic suggests that only extremely high levels of exposure to magnetic fields (affecting < 1 % of the child population) may result in an increased risk. However, even this conclusion is not clear.

F I G U R E 27-5. Anth rax spores from the bacteria Bacillus anthracis have been u sed in biological wa rfare. Cuta neous anthrax can man ifest with s m a l l pru ritic p a p u l e s t h a t q u ickly evolve i nto a ves icle t h a t necrosis i nto a n u l cer with a black eschar as seen i n this photogra p h . (Reprod uced with permission

The final area to be emphasized during your noble endeavor relates to possible chemical exposures in the home. Question 21-3 In contemplating this issue, which question will be least suc­ cessful in highlighting the various types of chemical hazards in their homes? A) Do you have any exotic pets? B) Are you currently remodeling your home? C) Do you work with chemicals on the job site? D) Does your job entail working in or around those that are ill? E) Do you smoke tobacco products? Discussion 21-3 The correct answer is "A:' Substances found within the home and those brought home can all impact the health of the children in a home. While exotic pets may present some health risks they are not likely to be chemical in nature. On the other hand, during a home remodeling there are often numerous cleaning products introduced into the home. These products can emit fumes that can present a health risk, represent a source for a possible cutane­ ous exposure or burn, and serve as a source for an unintentional ingestion. Similarly, exposure to chemicals on the job site can be transferred to one's family. Sprayed chemicals such as insecticides or pesticides, industrial solvents, and other substances can pres­ ent a home health hazard. Similarly, illnesses can be transmitted from someone working in or around the health care profession to family members in the home. And lastly, it is well known that exposure to tobacco smoke can have a significant impact on the health of everyone in the home, not just the primary user. For each of these entities, one should also consider the duration of exposure and the age(s) at which the exposure transpired. The physiologic effects of such exposures can be cumulative and result

from Tinti n a l l i J E, Sta pczynski JS, Ma OJ, Yealy DM, Cline DM, Meckler G D, eds. Tintinalli's Emergency Medicine: A Comprehensive Study Guide. 8th ed. New

York, NY: McGraw- H i l l Education; 2 0 1 6, Fig. 9-2.)

in more pronounced symptoms. Additionally, younger patients may be more susceptible to certain hazardous materials or more likely to suffer long-term sequelae. Remember, when evaluating patients for potential toxic ingestions, be sure to consider the home setting and parents' activities as well.



Helpful Tip

� The

sig ns and sym ptoms of a toxic exposu re a re

r1 1r i nfl u enced by both t h e d u ration o f a n d a g e a t which the exposu re occ u rs.

BIBLIOGRAPHY American Academy of Pediatrics Committee on Drugs. Naloxone dosage and route of administration for infants and children: Addendum to emergency drug doses for infants and children. Pediatrics. 1 990;86:484. Brent J, McMartin K, Phillips S, et al. Fomepizole for the treat­ ment of methanol poisoning. N Engl J Med. 200 1 ; 344:424. Brok J, Buckley N, Gluud C. Interventions for paracetamol (acetaminophen) overdose. Cochrane Database Syst Rev. 2006;(2) :CD003328. Chamberlain JM, Klein BL. A comprehensive review of naloxone for the emergency physician. Am J Emerg Med. 1 994; 1 2:650. Committee on Environmental Health. Lead exposure in chil­ dren: Prevention, detection, and management. Pediatrics. 2005; 1 1 6(4) : 1 036- 1 046. doi: 1 0 . 1 542/peds.2005- 1 947.

CHAPTER 27



P O I S O N I N G A N D ENVI RO N M E NTA L EXPOS U R E TO HAZA R D O U S S U BSTA N C E S

Greene SL, Dargan PI, Jones AL. Acute poisoning: Under­ standing 90% of cases in a nutshell. Postgrad Med ]. 2005;8 1 (954) :204-2 1 6 . Hampson NB, Piantadosi CA, Thorn SR, Weaver LK. Practice recommendations in the diagnosis, management, and prevention of carbon monoxide poisoning. Am J Resp Crit Care Med. 20 1 2 ; 1 86 ( 1 1 ) : 1 095- 1 1 0 1 . doi: 1 0. 1 1 64/ rccm.20 1207 - 1284CI. Hann G, Duncan D, Sudhir G, West P, Sohi D. Antifreeze on a freezing morning: Ethylene glycol poisoning in a 2-year­ old. BMJ Case Rep. 20 1 2 Mar 27; 2 0 1 2 . doi: 1 0. 1 1 36/ bcr.07.20 1 1 .4509. Ikenberry SO, Jue TL, Anderson MA, et al. Management of ingested foreign bodies and food impactions. Gastrointest Endosc. 20 1 1 ;73(6) : 1 085- 1 09 1 . Kao LW, Nafiagas KA . Carbon monoxide poisoning. Emerg Med Clin North Am. 2004;22:985. Kerr GW, McGuffie AC, Wilkie S. Tricyclic antidepressant overdose: A review. Emerg Med f. 200 1 ; 1 8 ( 4):236-24 1 . Kramer RE, Lerner DG, Lin T, et al. Management of ingested foreign bodies in children: A clinical report of the NASP­ GHAN Endoscopy Committee. ! Pediatr Gastroenterol Nutr. 2 0 1 5;60:562. Krenzelok EP, Proudfoot AT. Salicylate toxicity. In: Haddad LM, Winchester J, eds. Clinical Management of Poison­ ing and Drug Overdose. Philadelphia, PA: WB Saunders; 1 998:675.

623

Lanthony P. [Van Gogh's xanthopsia] . Bull Soc Ophtalmol Fr. 1 989;89( 1 0) : 1 1 3 3 - 1 1 34. Mills KC, Curry SC. Acute iron poisoning. Emerg Med Clin North Am. 1 994; 1 2:397. National Capital Poison Center. NBIH button battery ingestion triage and treatment guideline. http://www. poison.org/ battery/guideline.asp. Accessed June 28, 20 1 5 . Rumack B H , Matthew H. Acetaminophen poisoning and toxic­ ity. Pediatrics. 1 975;55:87 1 . Sidell FR. Clinical effects of organophosphorus cholinesterase inhibitors. J Appl Toxicol. 1 994; 14: 1 1 1 . Smolinske SC, Hall AH, Vandenberg SA, et al. Toxic effects of nonsteroidal anti-inflammatory drugs in overdose. An overview of recent evidence on clinical effects and dose­ response relationships. Drug Saf 1 990;5:252. Thomas TJ, Pauze D, Love JN. Are one or two dangerous? Diphenoxylate-atropine exposure in toddlers. J Emerg Med. 2008;34( 1 ) : 7 1 -75. Tormoehlen LM, Tekulve KJ, Nafiagas KA. Hydrocarbon toxic­ ity: A review. Clin Toxicol (Phila). 2 0 1 4;52:479. UW Health. Common plants: What's poisonous and what's not? American Family Children's Hospital and Wisconsin Poi­ son Center. https://www. uwhealth.org/files/uwhealth/ docs/ pdf/poisonous_ plants.pdf. Accessed on June 12, 20 1 5. Warniment C, Tsang K, Galazka SS. Lead poisoning in chil­ dren. Am Pam Physician. 20 1 0;8 1 (6) :75 1 -757.

This page intentionally left blank

28

Preventative Pediatrics Ashley M iller an d Rebecca Lozman-Oxman

A 2-year-old girl presents in mid-October for her yearly rou­ tine vaccinations. She is up to date with her primary vaccina­ tions. She has never had the flu shot before, but her parents would like her to have it this year because she had the flu last year. The patient has a history of wheezing at 2 months of age associated with respiratory syncytial virus (RSV) infection and at 6 months of age with a viral upper respiratory tract infection. She also has a history of rash around her mouth after eating scrambled eggs but is able to eat baked products containing eggs without difficulty. Question 1-1 Which of the following statements correctly describes the influenza vaccine( s) that can be offered to this child, and why? A) None, due to her history of egg allergy. B) Inactivated influenza virus (IIV) vaccine only, due to her history of wheezing. C) Live attenuated influenza virus (LAIV) vaccine only, due to her history of egg allergy. D) Both vaccines can be offered as there is no contraindication. E) None of the above. Discussion 1-1 The correct answer is "E:' Influenza vaccination is recommended yearly for all children older than 6 months of age. Children aged 6 months to 8 years require two doses (� 4 weeks apart) dur­ ing the first season they receive the vaccine. The IIV vaccine is given as an inj ection. The LAIV vaccine is given intranasally as a mist. The LAIV vaccine is contraindicated in ( 1 ) children younger than 2 years of age, (2) those who have had a prior allergic reaction to the LAIV, (3) those who are currently tak­ ing aspirin or aspirin-containing products, (4) those with egg allergy, (5) pregnant women, (6) immunosuppressed people, (7) those who have taken influenza antiviral medications in the last 48 hours, or (8) children aged 2 to 4 years with asthma or who had wheezed in the past 12 months. For older children with

asthma, the LAIV vaccine should be avoided as it is associated with increased risk of wheezing. Most formulations of influenza vaccines contain a small amount of egg protein as they are pro­ duced from chick embryos. All children with an egg allergy may receive the IIV vaccine but should be monitored in the office afterward. Children with egg anaphylaxis should receive their vaccine in an allergy clinic. New research suggests that the LAIV vaccine may be used in children with egg allergy, but the Advisory Committee on Immunization Practices (ACIP) and Centers for Disease Control and Prevention (CDC) currently recommend the IIV. This child should receive the IIV vaccine due to her history of egg allergy. Her wheezing was too long ago to matter. For any immunization questions, consult the CDC's website. (See Figure 28- 1 .)



Helpful Tip

:5.� The

occu rrence of G u i l l a i n-Ba rre syndrome with in

r1 1r 6 weeks fol lowing a previous dose of i nfl uenza vaccine is

considered a preca ution for use of all influenza vaccines.

What if the 2-year-old's favorite food was eggs and she had never wheezed, but she had a history of chronic kidney disease. In fact, she is eating a hardboiled egg in your office right now. Question 1-2 What vaccine would you recommend for her? A) IIV vaccine. B) LAIV vaccine. C) Either the IIV or LAIV vaccine. D) Neither; children with chronic health diseases are too fragile to receive vaccinations. Discussion 1-2 The correct answer is "A:' Children with chronic health condi­ tions should be vaccinated with the IIV. They are at high risk for complications from influenza infection. Those around them 625

626

MCG RAW- H I LL E D U C AT I O N S P E C I A LTY BOA R D REVI EW: P E D I AT R I C S

Figure 1 . Recommended immunization schedule for persons aged 0 through 1 8 years -United States, 201 5. { F O R THOSE W H O FALL B E H I N D OR START LATE, SEE THE CATCH-UP SCHEDULE { F I G U R E 2]). These recommendations must be read with the footnotes that follow. For those who fa ll behind or start late, provide catch-up vaccination at the earliest opportun ity as indicated by the green bars i n Figure 1 . To determine minimum interva ls between doses, see the catch-up sched ule (Fig ure 2). School entry and adolescent vaccine age groups are shaded.

Vaccine

Birth

1 mo

Hepatitis B1 (HepB)

1 '1 dose

2 mos

4 mos

1 '1 dose

series); RVS (3-dose series)

9 mos

1 2 mos

l S mos

1 9-23

1 8 mos

�-------------------------3 '' dose ·------------------------�

1'-t------ 2nd dose _____.,.

Rotavirus1 (RV) RVl {2-dose

6 mos

2nd dose

II

See footnote 2

I r------

Diphtheria, tetanus, & acellular pertussisJ (DTaP: 0.3 mg/d l

< 0.5 m L/kg/h x 6 h

Stage 2

2.0x i n c rease of base l i n e SCr

< 0.5 m l/kg/h x 1 2 h

Stage 3

3 .0x i n c rease of base l i n e SCr or req u i re re n a l re placement thera py

< 0.3 m l/kg/h x 24 h or a n u ric x 1 2 h

SCr, serum creati n i ne.

CHAPTER 31



R E N A L A N D U RO LOG I C D I S O R D E RS

TABLE 3 1 -5 CO M M O N ET I O LOG I E S OF ACUTE

TABLE 3 1 -6 I NV E S T I G ATI O N S TO D I ST I N G U I S H

K I D N EY I N J U RY

P R E R E N A L AZOT E M I A F RO M ATN

Decreased effective c i rcu lati ng vo l u m e with decreased re n a l perfu s i o n : dehyd ration, poor ca rd i a c output, h e patore n a l syn d ro m e, capi l l a ry leak, active neph rotic syn d ro m e N e p h rotoxi ns: NSAI Ds ( i b u p rofen, n a p roxe n, ketoro lac, i nd o m ethacin), va ncomycin, genta m ic i n , acyclovi r, cid ofovi r, or d i u retic use Hypoxic ischemic encep h a l opathy Acute t u b u l a r necrosis Tu mor lysis syn d rome Acute t u b u l o i nterstitial neph ritis Acute g l omeruloneph ritis Obstruction Va sc u l a r t h ro m bosis ( u m b i l ical vei n a n d a rte ry catheterization)



















1 . Fractional excretion of sod i u m ( F E N a ) :

FENa =











Helpful Tip

� The u r i n a lysis ca n

:5.

h e l p identify the ca use of AKI. On

r1 1r microscopic exa m, look for abnormal u ri n e sed i me nts.

G ra n u l a r casts: acute t u b u l a r necrosis, vascu l itis, g lomeru lonephritis RBC casts: vascul itis, glomeruloneph ritis



WBC casts: pyeloneph ritis, i nterstitia l neph ritis, vas­ c u l itis, g lomeru lonephritis Eosi n o p h i l s : i nterstiti a l neph ritis (most l a bs req u i re a s pecific req uest for th is) Hya l i n e casts: prote i n u ria

You suspect the etiology of her AKI is prerenal but you want to be sure you are not missing intrinsic kidney disease, such as acute tubular necrosis (ATN) . Question 23-3 Which finding will help confirm a prerenal etiology? A) Fractional excretion of sodium greater than 1 % . B) Urinalysis specific gravity 1 .005. C) Urine osmolarity less than 350 mOsm/kg. D) Fractional excretion of sodium less than 1 %.

PNa x UCr

x 1 00%

FENa: Child

F E N a : I nfa nt

I nterpretation

< 1%

< 3%

Pre ren a l azote m i a

1 -2%

2-3%

I ndeterm i n ate

> 2%

> 3%

MN

F E N a is used when the creati n i n e is a b n o r m a l . F E N a is d i ffi c u l t t o i nte rpret after d i u retic use a n d a g g ressive fl u i d resu scitation

E l evated > 500 mOsm/kg sug gests prese rved u r i n e concentrat i n g a b i l ity, typical o f pre ren a l azote m i a . Decreased < 350 mOs m/kg sug gests i m p roper u ri n e concentrating a b i l ity, typical of ATN .

3 . U r i n e specific g ravity: •

E l evated > 1 .020 s u g g ests prere n a l azote m i a .



Decreased < 1 .0 1 0 sugg ests ATN .

4 . B U N -to-creat i n i n e ratio (less rel i a ble): •

Discussion 23-2 The correct answer is "C:' Her AKI has resulted from hypo­ perfusion due to decompensated toxic shock syndrome. Initial diagnostic tests for AKI should include electrolytes, BUN, cre­ atinine, and urinalysis. (See Table 3 1 -5.)

U Na x PCr

2 . U r i n e osmola rity:

NSAI Ds, nonsteroidal a nti-i n fl a m matory d rugs.

Question 23-2 What is the etiology of her AKI? A) Nephrotoxic medication. B) Renal vein thrombosis. C) Hypoperfusion. D) Acute glomerulonephritis.

685

E l evated B U N/creati n i ne ratio > 20

=

prere n a l

azote m i a . •

N o r m a l B U N/creati n i ne ratio 1 0- 1 5

=

ATN .

ATN, acute t u b u l a r necrosis; B U N , bl ood u rea n itrogen; PCr, plasma creat i n i ne; PNa, plasma sod i u m ; UCr, u r i n e creat i n i n e; U Na, u ri n e sod i u m .

Discussion 23-3

The correct answer is "D:' AKI can be prerenal (poor perfu­ sion), intrinsic (ATN) , or postrenal (urinary tract obstruction) in etiology. Prerenal injury indicates inadequate perfusion of the kidney. Etiologies include decreased intravascular volume (dehydration, diuretics, hemorrhage), peripheral vasodilation (sepsis), and inadequate perfusion (heart failure) . ATN is due to intrinsic kidney injury. Etiologies include prolonged prerenal state, ischemia, nephrotoxic medications, rhabdomyolysis, and contrast dye. Urine concentrating ability is the key distinguish­ ing feature. In ATN, concentrating ability is lost. Table 3 1 -6 summarizes key tests and findings used in differentiating pre­ renal azotemia from ATN.



Helpful Tip

� When d i sti n g u i s h i n g prere n a l AKI from ATN (intrinsic

:5.

r1 1r kid n ey i nj u ry), t h i n k a bout concentrating a b i l ity. I f the

i nj u ry is prere n a l (hypoperfu s ion), the kid n ey wa nts to hold onto fl u i d (FENa is low, u r i n e is concentrated) . In

ATN, the kid n ey is da maged a n d ca n not concentrate the u ri n e (FENa is h i g h).

686

MCG RAW- H I LL E D U CAT I O N S P E C I A LTY BOA R D REVI EW: P E D I AT R I C S

A 6-year-old girl presents to your inpatient service for further evaluation. She has had fatigue, fever, and poor oral intake for the past week. She received two doses of ibuprofen each of the last 3 days for fever. Vital signs on admission are tem­ perature 38SC ( 1 0 1 .3°F), heart rate 136 bpm, respirations 25 breaths per minute, and blood pressure 85/5 1 mm Hg. Lab results include: •

CBC, glucose, C-reactive protein (CRP) normal



Sodium (Na) 1 48 mEq/L



Potassium (K) 3.6 mEq/L



Chloride (Cl) 1 08 mEq/L



C0 2 29 mEq/L



BUN 25 mg/dL



Creatinine 0.7 mg/dL



Phosphorus 3.8 mg/dL



Calcium (Ca) 8.2 mg/dL

Following administration of a 10 mL/kg normal saline (NS) bolus, and 6 hours of D 5 V2 NS maintenance IV fluids, the medical student reports to you that the patient's urine output has been 0.5 mL/kg/h since admission. Repeat vital signs show heart rate 120 bpm, respirations 25 breaths per minute, and blood pressure 90/56 mm Hg. Question 24- 1 What is NOT an appropriate next step in your management? A) Obtain a urinalysis. B) Measure fractional excretion of sodium. C) Give IV furosemide. D) Order a renal ultrasound. E) Give a second 10 mL/kg NS bolus. Discussion 24- 1 The correct answer is "C:' This patient has a multifactorial AKI presenting with oliguria, as diagnosed with urine output less than 0.5 mL/kg/h for 6 hours, along with elevated creatinine at 0.7 mg/dL. There is a strong history of prerenal dehydration prior to hospital admission, given her persistent fever and poor oral intake for the past week. She also received ibuprofen, which in the setting of dehydration could contribute to nephrotoxic AKI. Obtaining a urinalysis is helpful in many ways. Urine specific gravity greater than 1 .020 would suggest intravascular depletion. Hematuria and proteinuria may identify an active glomerular disease. Fractional excretion of sodium (FENa) less than 1 % can also confirm intravascular depletion. Giving IV furosemide will likely increase her urine output. However, when she is persistently tachycardic with borderline hypoten­ sion, this medication may worsen her AKI. Ordering a renal ultrasound can identify bilateral echogenic kidneys in the set­ ting of AKI and potentially rule out any ureteric obstruction (postrenal injury) .

You are staffing the day shift in the emergency department when a 5-year-old boy presents with worsening radiating abdominal pain rated as 8/1 0, associated with an episode of gross hematuria yesterday evening at around suppertime. His mother states that he has not voided since. He now has persistent emesis. IV morphine was given, providing some relief. Vital signs are temperature 36.4°C (97SF), heart rate 125 bpm, respirations 30 breaths per minute, and blood pressure l l0/70 mm Hg. He lies still in bed and identifies right-sided flank pain. Lab results are as follows: hemoglobin 10.2 g/dL, WBCs 3.0 x 10 3 /f!L, platelets 2 1 0 x 10 3/f!L, INR 1 . 1 , P T l l sec, PTT 2 9 sec, Na 140 mEq/L, K 4.0 mEq/L, Cl 1 09 mEq/L, C02 24 mEq/L, BUN 20 mg/dL, creatinine 0.8 mg/dL, and CRP less than 0.5 mg/dL. Question 25-1 Investigations and management should NOT include which of the following? A) IV NS bolus x 1 0 mL/kg. B) IV ceftriaxone. C) Abdominal X-ray. D) Renal ultrasound. E) Obtain urinalysis. Discussion 25-1 The correct answer is "B:' This patient most likely has an obstructive kidney stone presenting with oliguric AKI, as evi­ denced by oliguria and elevated creatinine of 0.8. There are no clinical signs suggestive of an infective process; therefore, IV antibiotic use is not warranted at this time. He may be clini­ cally dehydrated as indicated by his history, and a fluid bolus challenge given his tachycardia is appropriate. Abdominal X-ray (flat plate) and renal ultrasound are both useful studies that can identify radiopaque kidney stones and evidence of obstruction by examining for hydronephrosis and or hydroureter. Urinaly­ sis is helpful to look for persistent hematuria, or to collect for a kidney stone analysis.

A 3-year-old girl who was previously healthy presents with a history of fever and cough for the past week. Over the past 24 hours, she has had worsening respiratory distress and fatigue. She has only voided once in the past day. On exami­ nation, vital signs show temperature 36.5°C (97.7°F), heart rate 130 bpm, respirations 35 breaths per minute, Sa0 2 88% on room air, and blood pressure 1 18/71 mm Hg. Lab results are Na 135 mEq/L, K 4.9 mEq/L, Cl 108 mEq/L, C0 2 20 mEq/L, BUN 25 mg/dL, and creatinine 0.9 mg/dL. She has mild facial edema, is tachypneic with shallow breaths, has soft diffuse crackles on auscultation and a systolic murmur grade II/VI over left sternal border, but is otherwise warm

CHAPTER 31

and well perfused. She has mild abdominal distension that is nontender. There is no pitting edema to extremities. Question 26-1 What is an appropriate management option to consider? A) IV NS bolus x 10 mL!kg. B) IV furosemide. C) IV maintenance fluids D 5 1h NS + 20 mEq/L KCl. D) IV maintenance fluids D 5 NS. E) IV hydralazine. Discussion 26- 1 The correct answer is "B:' This patient is presenting with oli­ guric AKI and a preceding history of an upper respiratory ill­ ness within the past 7 to 10 days, suggestive of postinfectious glomerulonephritis. She is fluid overloaded as evidenced by tachycardia, cardiac murmur, elevated blood pressure, pulmo­ nary edema (tachypnea, hypoxia, crackles on auscultation), and facial edema. This is in the setting of abnormal creatinine and BUN. IV furosemide is the drug of choice to stimulate natri­ uresis and diuresis, which can alleviate her current fluid over­ load status. Although she is oliguric, providing a normal saline bolus at this time may worsen her fluid overload status, with the potential complication of precipitating or worsening acute congestive heart failure and pulmonary edema. IV maintenance fluids at this time, with or without potassium, would also be inappropriate fluid management given her oliguric state. Potas­ sium should never be included in IV fluids in a patient with an evolving AKI. IV hydralazine would be effective as an acute antihypertensive medication, acting as a vasodilator. However, it would not be a preferred agent over IV furosemide. •



1 1 1r

Helpful Tip Lea r n the ve rbiage. Acute ki d n ey i nj u ry (AKI) is the en vog u e term replaci ng acute re n a l fa i l u re (ARF).

You are seeing a 1 3-year-old adolescent boy who was admit­ ted to the local community hospital for chronic fatigue and significant iron deficiency anemia with hemoglobin of 5.8 g/dL. The history reveals that he has had no recent illnesses, fever, or travel history. He has decreased appe­ tite and nausea without abdominal pain. He is fatigued after sleeping 10 hours at night, and has trouble staying awake at school. He wakes to void several times at night. He complains of generalized weakness, having withdrawn from his soccer team this past month. Growth parameters are significant for weight at the 1 Oth percentile and height at the 50th percentile. Vital signs are temperature 36.7°C, heart rate 100 bpm, respirations 24 breaths per minute, and blood pressure 1 3 7/82 mm Hg. Upon admission, lab results are Na 1 3 5 mEq/L, K 5.2 mEq/L, Cl 1 0 1 mEq/L,



R E N A L A N D U RO LOG I C D I S O R D E RS

687

C0 2 16 mEq/L, glucose 102 mg/dL, albumin 3.2 g/dL, cal­ cium 6.8 mg/dL, phosphate 8.2 mg/dL, BUN 85 mg/dL, and creatinine 5.2 mg/dL. Urinalysis is negative for blood, but positive for 1 + protein. You recognize that this patient has renal failure and consider the next steps in his treatment. Question 27-1 Which of the following should NOT be part of your manage­ ment plans for him? A) Renal ultrasound. B) IV maintenance fluids D 5 1h NS. C) IV calcium chloride to correct for hypocalcemia. D) Low-phosphate and low-potassium diet. E) Pediatric nephrology consult at a tertiary care hospital for transfer of care to determine modality of renal replacement therapy. Discussion 27-1 The correct answer is "C' Hypocalcemia in the setting of end­ stage renal disease (ESRD) is most commonly associated with hyperphosphatemia and vitamin D deficiency. Control of hyperphosphatemia will generally normalize hypocalcemia by use of phosphate binders, low-phosphate diet, renal replace­ ment therapy (dialysis), or a combination of these. Unless there is symptomatic hypocalcemia, acute calcium repletion is not warranted. Renal ultrasound will be instrumental in diagnos­ ing the etiology of his renal failure. In this setting with a well­ grown adolescent, chronic kidney disease ( CKD) with a history of polyuria and bland urine may be consistent with a diagnosis of juvenile nephronophthisis, with typical features of normal but diffusely echogenic kidneys. The choice of maintenance IV fluids without potassium is optimal given mild to moder­ ate hyperkalemia, and he is not oliguric with signs of hyper­ volemia. Similarly, a low-phosphate diet and potassium dietary restrictions would be optimal until renal replacement therapy is provided. As the patient is clinically stable with signs of ESRD, consulting pediatric nephrology and transferring him to begin renal replacement therapy is appropriate. Question 27-2 Regarding CKD, which of the following is false? A) CKD has five stages. B) Most CKD in infants is acquired. C) Congenital anomalies are a common cause of CKD. D) Staging of CKD is based on GFR. E) GFR is reduced starting at stage 2. Discussion 27-2 The correct answer is "B:' CKD is staged 1 through 5 based on GFR. CKD stage 5 is the same as ESRD. Children with CKD stage 5 typically require transplant or dialysis. (See Table 3 1 -7.) Congenital anomalies of the kidney and urinary tract ( CAKUT) make up about 40% of all pediatric ESRD. CAKUT is also the most common cause of CKD in children and infants younger than age 10 years. After age 10, most cases of CKD are acquired ( eg, glomerulonephritis) .

MCG RAW- H I LL E D U CAT I O N S P E C I A LTY BOA R D REVI EW: P E D I AT R I C S

688

TABLE 3 1 -7 STAG ES O F C H RO N I C K I D N EY D I S E A S E

Glomeru l a r Fi ltration Rate (GF R) (ml/m in/1.73 m 2 )

Stage 1

> 90 (normal)

2

60-89

3

30-59

4

1 5-29

5

< 1 5 or d i a lysis

Question 27-3 Which is NOT a laboratory finding of oliguric AKI or CKD, or both? A) Hyperkalemia. B) Hypocalcemia. C) Hypophosphatemia. D) Azotemia. E) Metabolic acidosis. Discussion 27-3 The correct answer is "C:' AKI and CKD presenting with oli­ guria are generally associated with hyperphosphatemia. Addi­ tional findings include elevated creatinine, anemia, low vitamin D level and elevated parathyroid hormone. Hypophosphatemia may occur in cases of CKD with high urine output failure. This is common in CKD due to CAKUT. •

Helpful Tip

:S.� CKD

r11r

in c h i l d ren ca n be oliguric or nonoliguric ( h i g h

u ri n e output). Those with an obstructive nephropathy (ie, posterior u rethral va lves or refl ux n e p h ropathy)

Discussion 27-4 The correct answer is "E:' Due to the lapse of prekindergarten vaccination schedules, he would have missed the second dose of MMR vaccine. This is extremely important for pretransplanta­ tion candidates, as he will remain on chronic immunosuppres­ sion and will not be able to receive any live vaccines thereafter. He may be incomplete in the DTaP series; however, this vaccine may be received at any point. He should have received the full hepati­ tis B series by 18 months. HPV vaccine is not a live viral vaccine and does not have the absolute contraindication as does MMR.

� QUICKQUIZ What is NOT a goal of chronic renal replacement therapy (dialysis) ? A) Correct hyperparathyroidism. B) Prevent kidney transplantation. C) Maintain fluid status with fluid restriction. D) Normalize electrolytes. E) Treat hypertension. Discussion The correct answer is "B:' Chronic renal replacement therapy or dialysis is utilized as a bridge towards kidney transplantation in pediatrics. The overall CKD status is generally optimized dur­ ing the dialysis period. This generally includes normalization of electrolytes and acid-base derangements, control of anemia, control of hypertension, correction or stabilization of secondary hyperparathyroidism in metabolic bone disease, establishment of good nutrition with appropriate growth and development of the child, and determination of psychosocial readiness to receive a kidney transplant, which will require lifelong immu­ nosuppressive treatment.

a re typica l l y polyu ric a n d conseq uently may a l so have hypoka l e m i a and hypophosphatemia even though they have severe ren a l fa i l u re. I n contrast, o l i g u ric ren a l fa i l u re is typica l of the g lomeru lonephritides (ie, hemolytic u remic synd rome, posti nfectious g lomeru l o n e p h riti s).

The same patient has been started on peritoneal dialysis, and you are now his primary pediatrician. You review his medical records and learn that his immunizations are incomplete. His most recent vaccines were received at 18 months of age. Question 27-4 Recognizing that he is now on dialysis and would most likely move forward for kidney transplantation at some point, which is the vaccine that you would like him to catch up on as soon as possible? A) Diphtheria, tetanus, and acellular pertussis (DTaP) . B ) Hepatitis B . C ) Meningococcal. D) Human papillomavirus (HPV) . E) Measles, mumps, and rubella (MMR) .

A 5-year-old girl is on chronic dialysis from Escherichia coli shiga toxin-positive hemolytic uremic syndrome (HUS)­ induced ESRD. She is doing well on dialysis and is awaiting transplant. You discuss transplant options with the family; specifically, living versus deceased donor transplants. Question 28-1 Which of the following regarding living donor transplanta­ tion is false? A) Increased long-term survival. B) Lower risk of delayed graft function. C) Health of donor cannot be guaranteed. D) More control over planning time of transplant. E) May avoid need for a dialysis "bridge:' Discussion 28-1 The correct answer is "C:' Transplant is the best choice for pedi­ atric patients with ESRD (CKD stage 5). Overall, living donor

CHAPTER 31

TABLE 3 1 -8 DECEASED V E R S U S LIVI N G D O N O R

K I D N EY TRA N S PLANT

Living

Deceased

Ischemia time

Shorter

Longer

I m mediate fu nction

Better

May res u lt i n delayed g raft fu nction

Delayed g raft fu nction

Lower risk

H ig her risk

Donor health

Thoro u g h assessments of donors e n s u re excel lent health a n d ren a l fu nction

Donors may have su boptim a l health

H LA m i s m atches

Better a b i l ity to find a donor with few or zero H LA m i smatches if donor i s related

I ncreased risk of H LA m i s m atches

Long-term s u rviva l

Better

Va riables l i sted a bove can decrease overa l l g raft s u rviva l

Planning

Ca n identify a n appropriate time that i s most com pati ble with school a n d work sched u les

U nce rta i nty; can be on wa iti ng l i st for m a ny yea rs

Dia lysis needs

Preference

There i s a movement towa rd more preem ptive tra n spla ntation if ren a l prog ression is slow; therefore, c h i l d may not need bri d g i n g period with d i a lysis Fi rst choice for ped iatrics

transplants are superior to deceased donor transplants. (See Table 3 1 -8.)

Question 28-2 What do you tell them about measures that increase long­ term transplant survival? A) Be sure child always takes the prescribed immunosuppres­ sion medications. B) Prevent urinary tract infections.

689

Discussion 28-2 The correct answer is "E:' Survival, growth, and quality of life are better with transplant than dialysis so it is important to take care of the graft. Preventing acute and chronic rejection of the donor kidney is very important. Some factors that affect trans­ plant survival are within the patient's control, but many are not. (See Table 3 1 -9.)

Your 5-year-old patient received a living donor transplant and has been thriving in the years since. Now a 1 5-year-old adolescent, she presents to your office with fever and acute abdominal pain. She complains of dysuria, has had decreased intake for the past 2 days, and last voided 6 hours ago. She has no diarrhea or vomiting. She was on a band trip this past week and may have missed a few doses of her immunosup­ pressive medications. On focused abdominal physical exam, there is no guarding or rebound tenderness. However, there is focal tenderness on deep palpation over her transplant graft. Lab results show the following: Na 1 38 mEq/L, K 5.9 mEq/L, Cl 109 mEq/L, C02 15 mEq/L, BUN 53 mg/dL, and creati­ nine 2 . 1 mg/dL (her baseline creatinine is between 1 .3 and 1 .5 mg/dL) . Trough tacrolimus level is pending.

TABLE 3 1 -9 FACTO RS THAT A F F ECT LO N G-TERM

S U RVIVA L I N T RA N S P LA N T PAT I E NTS











Acute rejection (ce l l u l a r and a nti body med i ated) C h ronic rejection (ce l l u l a r and a nti body med i ated) Poor adherence to i m m u nosu ppression Recu rrent u ri n a ry tract infections Ongoing obstructive u ropathy especia l ly i n congen ita l n e u rogenic bladder (eg, posterior u reth ra l va lves) Recu rrence of pri m a ry d i sease •







The parents ask what they can do to make sure the transplant is successful. They want to be proactive.

R E N A L A N D U RO LOG I C D I S O R D E RS

C) Attend follow-up visits. D) Seek immediate medical care for fever. E) All of the above.



C h i l d wi l l most l i kely be on d i a l ysi s w h i l e wa iting on deceased donor l i st











I m m u ne mediated (mem bra noprol iferative g lomeruloneph ritis [M PG N ], focal seg menta l g lomeru losclerosis [FSGS], lgA neph ropathy, mem bra nous neph ropathy) Vasc u l itis (systemic l u pu s erythematosus, Henoch-Schon l e i n p u r p u ra, g ra n u lomatosis with polya n g i itis, microscopic polya n g i itis) Atypica l hemolytic u re m i c syn d rome (com plement dysreg u lation) Sickle cel l nephropathy Anti-g lomeru l a r basement mem b ra n e (Anti-G BM) d i sease Metabolic d i sorder with persistent abnormality of a s u bsta nce (eg, pri m a ry hyperoxa l u ria) Recurrent episodes of acute kidney i nj u ry Other health comorbid ities (hypertension, d i a betes, hyperl i pidem ia, obesity)

690

MCG RAW- H I LL E D U CAT I O N S P E C I A LTY BOA R D REVI EW: P E D I AT R I C S

Question 28-3 Your differential diagnosis of this acute kidney injury includes all of the following EXCEPT: A) Dehydration, which will require provision of iV fluid bolus. B) Pyelonephritis, which will require urine culture and start of IV antibiotics. C) Acute transplant rejection, which will require a transplant graft biopsy. D) Urinary obstruction, which will require ultrasound of trans­ plant graft. E) Recurrence of primary disease, which will require a trans­ plant graft biopsy.

in children aged 3 to 18 years is 3% to 5%. Risk factors such as age, male gender, obesity, physical activity, and dietary salt intake are major contributors to the risk of hypertension.

Discussion 28-3 The correct answer is "E:' There always needs to be a broad dif­ ferential diagnosis for acute kidney injury of the transplanted kidney. In this case there is sufficient history of decreased oral intake to suspect that intravascular repletion is war­ ranted. The patient has fever and complaints of dysuria that warrant workup and IV antibiotic therapy until proven other­ wise. Given her report of missed doses of immunosuppressive therapy and exam findings of transplant graft tenderness with fever, acute transplant rej ection should remain high on the dif­ ferential. The gold standard diagnosis for transplant rej ection is a renal biopsy. Urinary obstruction also warrants investiga­ tion as she is developing oliguria. A renal ultrasound should be ordered to identify any hydronephrosis or hydroureter to suggest an acute obstructive process. Disease recurrence should be considered in most patients who have undergone kidney transplant; therefore, understanding the underlying primary disease is important. However, E. coli shiga toxin­ positive HUS does not recur whereas atypical (non-diarrhea­ associated) HUS can recur.

The student is pleased to see that you, the attending, are still awake. He nervously smiles at you. You give him the thumbs up and say to forge ahead. His next PowerPoint slide defines a normal blood pressure for "kids" as systolic blood pressure of 1 20 mm Hg or lower and diastolic blood pressure 80 mm Hg or lower.

The medical student who started this chapter has decided that the kidney is quite interesting though complex. He is given a learning assignment about hypertension in children and adolescents. His presentation is going well, and he starts to discuss risk factors.

Your blood pressure is rising thinking about the audacity of defining hypertension as a single value. You will need to make an educational handout. Your mind starts to wander as you sip your third cup of coffee. You cannot remember if you have peed today.

Question 29-1 Which statement describes a risk factor for hypertension? A) Having a Y chromosome. B) Fast food addiction. C) Video game fanatic. D) "He's just 'huski" E) All of the above.



Helpful Tip

� C h i l d hood

=-

i1 1r

obes ity is associated with h i g h e r rates of

hyperte n s i o n. The l i ke l i hood of bei ng d i a g nosed with prehyperte n s i o n i s two times g reater and hyperte nsion i s fou r t i m es g reater i n overweig ht a d o l escents t h a n in those of n o r m a l weight.

Question 29-2 On his evaluation, you will be sure to note that: A) Blood pressure 75% or greater is the magical number to define hypertension. B) Blood pressure in pediatric patients must account for age. C) Blood pressure in pediatric patients must account for sex. D) Blood pressure in pediatric patients must account for height. E) Options B, C, and D. Discussion 29-2 The correct answer is "E:' Diagnosis of hypertension in a child or adolescent requires three separate visits for blood pressure measurements, with an average blood pressure at or above the 95th percentile for age, sex, and height. Hypertension can be diagnosed in a single visit if the child has symptomatic hyperten­ sion (headache, nausea, blurry vision, shortness ofbreath) or has stage 2 hypertension (blood pressure> 99%). (See Figure 3 1 -3.)

Prehypertension Mean

SBP

or

DBP

between 90th and 95th percentiles or > 120/80 if this was lowe r than the 90th percentile

between 95th and 99th percentile s + 5 mm

Hg

Mean

or

DBP

>99th perce nti l e +

5 mm

Discussion 29- 1 The correct answer is "E:' Hypertension is a major long-term health condition and a leading cause of premature death among adults. Owing to its hereditary nature, primary hypertension is detectable in children and adolescents. The estimated prevalence

SBP

>99th percenti l e or

Hg

in >2

sittings

F I G U R E 3 1 -3. Classification of hypertension in c h i l d ren a n d adolescents.

DBP, diastol i c blood pressu re; SBP, systol i c blood pressu re.

CHAPTER 31

Question 29-3 Which is true regarding your hormones currently? A) Renin is elevated. B) Angiotensin I is low. C) Aldosterone is low. D) Angiotensin II is low. E) Both A and C.



R E N A L A N D U RO LOG I C D I S O R D E RS

TABLE 3 1 - 1 0 L I M I TATI O N S OF B LO O D P R E S S U R E

M EA S U R E M ENTS

All methods









Discussion 29-3 The correct answer is "A:' You are dehydrated from excess caf­ feine ingestion. The renin-angiotensin system is the major hormonal system that affects blood pressure regulation. Renin is produced in the juxtaglomerular cells of the afferent renal arteriole and is stimulated by decreased glomerular perfusion, reduced sodium intake, or activity of the sympathetic nervous system. Renin cleaves angiotensinogen to form angiotensin I. Angiotensin-converting enzyme in the lungs converts angioten­ sin I to angiotensin II (maj or active component). Angiotensin II is a potent vasoconstrictor and increases blood pressure. It also stimulates aldosterone release from the adrenal gland, resulting in aldosterone-mediated salt and water retention with further increase in blood pressure. •



lllr

Au scu ltation









Osci l l ometric





Helpful Tip



A n g iotensi n-convert i n g enzyme i n h i bitors a n d a n g io­ te n s i n II receptor a nta g o n i sts a re key medications in hyperte nsion treatment.

Home blood pressu re mon itor Pa l pation









An 1 1 -year-old boy is currently hospitalized after surgery to repair a broken arm. He crashed his bike while wearing a helmet. There were no complications with surgery. He is otherwise healthy. You are paged for a pediatric consult in the middle of the night. The boy's blood pressure has been "high:' His latest measurement is 1 25/81 mm Hg using one of those machines the hospital has. The nurse reports it was measured on his left leg while he was calm in bed. He is 1 50 em (4 feet, 11 inches) tall. You give him a quick glance and notice he has a cast on his arm and is overweight. He otherwise looks fine. He is peeing well and the rest of his vital signs are normal. He has no complaints. Question 30-1 What should you do next? A) Obtain a manual blood pressure. B) Ask for blood pressure measurements to be obtained in the arm. C) Ensure the cuff size is correct. D) Ensure the cuff is placed correctly. E) All of the above. Discussion 30-1 The correct answer is "E." Doesn't it sometimes seem that it is impossible to get an accurate blood pressure measurement?

69 1

A m b u latory blood press u re mon itor (AB PM)







Machine ca l i bration error Wrong cuff size Cuff placement i ncorrect (arm a lways preferred) Not true rested blood pressu re (movi ng, arm elevated) Soft Korotkoff sounds Poor hea r i n g Deflati ng cuff too q u ickly Ter m i n a l d i g it prefe rence (round­ i n g n u m bers to 0, 5) Machine meas u res mean oscil­ lations of a rterial wa l l and ca lcu­ l ates systol i c blood pressu re (SBP) a nd d iasto l i c blood pressu re (DBP) with for m u l a u n iq u e to the mach i n e I n itial c u ff pressu re reaches 1 60- 1 80 mm Hg; can be u ncom­ forta ble for you n g c h i l d ren Need for sepa rate neonata l and child pressu re cuff cables Not ca l i brated Lack of cuff s izes a ppropriate for c h i l d ren Only mea s u res S B P Low accu racy Req u i res ABPMs that a re ca l i brated for u se i n c h i l d ren Not ava i lable for younger c h i l d ren H i g h cost

The blood pressure recorded for this child is 125/8 1 mm Hg (95%) . Before ordering tests, it is important to make sure the reading is accurate. Lots of things can go awry when taking a child's blood pressure. (See Table 3 1 -10.) Think about the infant happily kicking his leg as the cuff inflates. What about the toddler being held down to get a measurement, or the adolescent whose pressure is measured using a child-sized cuff? Another glance at the bed confirms your hunch: the cuff being used is appropri­ ate for a small child, not an overweight 1 1 -year-old. Remember that a blood pressure cuff is the correct size if the bladder length encircles 80% to 1 00% of the arm and the bladder width encircles 50% of the arm. You find a small adult cuff and retake the pres­ sure. The manual reading is 105/60 mm Hg ( 45%). Good night.

Let's suppose that same 1 1 -year-old has a blood pressure of 1 36/95 mm Hg (> 99%). You confirm the reading with a manual measurement on both arms using proper technique and an appropriately sized cuff. He is still asymptomatic and peeing clear urine at a rate of 1 mL!kg/h.

692

MCG RAW- H I LL E D U CAT I O N S P E C I A LTY BOA R D REVI EW: P E D I AT R I C S

Question 30-2 Do you go back to bed? A) Yes, he has prehypertension. B) Yes, he needs two more measurements to meet the defini­ tion of hypertension. C) Yes, he needs a BP at or above 95% and symptoms to meet the definition of hypertension. D) No, he has stage 2 hypertension which requires a diagnostic evaluation and treatment. E) No, he has stage 2 hypertension and you need to do some reading. Discussion 30-2 The correct answer is "D" but also "E:' He meets the definition of hypertension. If you didn't pick up on this, go back and reread the previous case discussions. Question 30-3 What is the next step in diagnosis? A) Renal ultrasound. B) BUN and creatinine. C) Urinalysis with microscopy. D) Review his medications. E) All of the above. Discussion 30-3 The correct answer is "E:' Always start with a thorough his­ tory, including lifestyle habits (physical activity and diet), sub­ stance exposure, risk factors, family history, and sleep history. This should help focus the evaluation on necessary areas of investigation. Initial testing should include four-point blood pressures; renal ultrasound to identify anomalies, dysplastic, or atrophic kidneys; echocardiogram to investigate for left ventricular hypertrophy, a sign of end-organ damage; labo­ ratory studies, including electrolytes, BUN, creatinine, and urinalysis to estimate baseline kidney function and electro­ lyte derangements; urine culture to rule out chronic pyelone­ phritis; and CBC to screen for anemia consistent with chronic kidney disease. Other tests that may be indicated include fasting lipids and glucose, drug screen to identify abuse of substances that may cause hypertension, and a sleep study to identify obstructive sleep apnea as a potential association with hypertension.

Discussion 30-4

The correct answer is "E:' Young children with stage 1 hyperten­ sion or adolescents with stage 2 hypertension should also have the following studies performed: •





Renin, aldosterone levels, to identify mineralocorticoid­ related disease Plasma metanephrines, to screen for pheochromocytoma Renovascular imaging; this includes DMSA, Doppler ultra­ sound of the kidneys, magnetic resonance angiography (MRA), and arteriography

You gather more details of the history and find the patient loves energy drinks. In fact, several empty cans of "Sleep No More" juice are at his bedside. A look at the label shows that one can has the same amount of caffeine as 3 cups of coffee, with only 10 calories. You need to try one of these. A red flag indicating new tests results flashes across the patient's elec­ tronic medical record. His urine drug screen is positive for amphetamines. Did you remember to ask about drug use? Could that be why he is hypertensive? Question 30-5 All of the drugs listed can cause hypertension EXCEPT: A) Prednisone. B) Caffeine. C) Lorazepam. D) Ibuprofen. E) Methylphenidate. Discussion 30-5 The correct answer is "C:' Lorazepam and other benzodiazepines cause hypotension and central nervous system depression. He may need a dose to fall asleep tonight. Methylphenidate, which is commonly prescribed for attention deficit hyperactivity dis­ order (ADHD) , can cause hypertension and a positive urine drug screen for amphetamines. (See Table 3 1 - 1 1 . )

You order all of these tests and arrange for a nephrology consult. You will call the specialist as soon as you get 2 more hours of sleep.

Th e boy tells you that h e has been taking "medicine" for ADHD for the past year. Although caffeine and methylpheni­ date can cause hypertension, you decide to take a full history and perform a physical exam to be sure no other key details have been overlooked. Previously, you did a "drive-by" peak. On exam, his skin is moist, he is tachycardic, and something is palpable at the midline of his neck. He seems to be staring at you with wide-open bug eyes.

Question 30-4 Will the specialist be happy with your initial workup? A) Yes. B) No. C) Maybe. D) It depends what time of day you call for a consult. E) Both C and D.

Question 30-6 What is the next step in his diagnostic workup? A) Thyroid studies. B) Hemoglobin A 1c . C) Four-point blood pressures. D) Serum caffeine level. E) Cortisol level.

CHAPTER 3 1



R E N A L A N D U RO LOG I C D I S O R D E RS

693

TABLE 3 1 - 1 1 COM M O N CAU S E S OF D R U G-I N D U C E D HYP E RT E N S I O N

Drug

Potentia l Mech a n isms

Sti m u l a nts caffei ne, coca i n e

Vasoconstriction with i ncreased sym pathetic nervous system activity, a n d decreased ba roreceptor sensitivity

Methyl phenidate

U nclear but l i kely secondary to i ncreased dopa m i n e agonist activity

Ora l contraceptives (estrogen, progesterone)

Alterations i n sod i u m retention secon d a ry to ren i n-a n g i otensi n-a ldosterone system sti m u lation

G l u cocorticoids

Sti m u lation of ren i n-a n g i ote n s i n system, e n h a n ced pressor sensitivity to endogenous vaso­ con strictors, i n h i bition of phospholi pase A 2, red uced activity of ka l l i krei n-ki n i n system B l ood pres s u re rise i s dose dependent

M i ne ra l ocorticoid (licorice, ketoconazole)

I n h i bits catabolism of a l dosterone; resu lts in susta i ned effect of a l dosterone effect, i n c reas­ ing sod i u m retention and potassi u m loss

Tobacco

I ncreased sym pathetic nervou s system activity, decreased endothel i u m -dependent vasod i­ lation, i ncreased a rterial wa l l stiffness

Ephedrine (herbal prod ucts)

Sti m u lation of catechol a m i ne release, i ncreased a l pha-, beta ,, beta 2 receptors I n h i bition of prostag l a n d i n , decrease in glomeru l a r fi ltration rate; resu lts in salt a n d water retention

N SA I Ds

Discussion 30-6 The correct answer is "A:' Aside from primary hypertension, which is a diagnosis of exclusion, it is important to consider secondary causes of hypertension. Etiologies include cardiac, endocrine, and renovascular disease. In this case, the child has

symptoms that suggest an endocrine cause of hypertension; specifically, hyperthyroidism due to Graves disease. Hyper­ thyroidism causes isolated systolic blood pressure elevation. A recheck of his blood pressure shows it is 1 3 6/70 mm Hg. (See Table 3 1 - 1 2 . )

TABLE 3 1 - 1 2 E N DOCRI N E CAU S E S O F SECON DA RY HYP E RTE N S I O N

Hormone Dysregu lation

Mecha nisms

Pri m a ry hyperaldostero n i s m

M i nera l ocorticoid excess resu lts i n excess a l dosterone without sti m u lation from ren i n

C u s h i n g syn d rome

G l ucocorticoid excess prod uces a ldosterone effect a n d i ncreases sensitivity t o endogenous vaso­ con strictors (epi nephrine and a n g iotensi n I I )

Pheoch romocytoma

Catechola m i ne-secreting t u m o r cel ls that m a y a rise from n e u ra l crest, a d re n a l g l a nds, or i n extra­ a d re n a l sites have a d i rect effect on blood pressu re, t h ro u g h c i rcu lati ng levels of catechol a m i nes (nora d re n a l i ne, a d ren a l i ne, dopa m i ne); associated with n e u rofi bromatosi s type 1 , m u ltiple endo­ crine neoplasia, a nd von H i ppei-Li ndau d i sease

Acromegaly (g rowth hormone excess)

G rowth hormone ca u ses sod i u m retention with vol u m e expa nsion

Pri m a ry hyperparathyroidism

Parathyroid hormone causes elevated i ntrace l l u l a r ca l c i u m , hypomag nese m i a, and ra ised plasma ren i n activity

Hypothyroidism

Red uces ren a l blood flow a n d glomeru l a r fi ltration rate a n d decreases card iac output; compensa­ tion i s by i ncreased peri phera l resi sta nce, which i n c reases d i a stol i c blood pressu re

Hyperthyro i d i s m

Thyroid hormone sti m u l ates ren i n-a n g i otensi n-a l d osterone axis a n d i ncreases sod i u m rea bsorp­ tion, resu lting i n isolated systol i c hypertension

Ren i n o m a

J uxtag lomeru lar cel l tumors of the kid ney or ectopic tumors secrete ren i n, resulting i n su bseq uent i ncrease of aldosterone; hypertension is often severe and presents acutely

Dia betes m e l l itus

Type 1 d i a betes m e l l itus: hypertension is secondary to prog ression of d i a betic nephropathy Type 2 d i a betes m e l l itus: hypertension is m u ltifactorial, rel ated to hyperi n s u l i n ism, sod i u m reten­ tion, vol u m e expa nsion, and i n c reased a rterial stiffness

MCG RAW- H I LL E D U CAT I O N S P E C I A LTY BOA R D REVI EW: P E D I AT R I C S

694

i QUICKQUIZ What diagnosis is associated with upper extremity blood pressure greater than lower extremity blood pressure on four-limb measurement? A) Aortic valve stenosis. B) Coarctation of the aorta. C) Renal vein thrombosis. D) Acute tubular necrosis. E) None; this is normal. Discussion The correct answer is "B:' Lower extremity blood pressures should be higher than the upper extremities. In coarctation, the reverse is true. Four-point (limb) blood pressure measurements are a cheap screening test in the workup of hypertension. You can get away with three points-both arms (in case an aberrant right subclavian artery is present) and one leg. Diminished or delayed femoral pulses (brachial-femoral delay) may be present with coarctation.



� I

llr

Discussion 3 1 - 1

The correct answer is "E:' For acute hypertensive emergency with suspected end-organ damage, early management in the intensive care unit with continuous infusion of antihyperten­ sive medication is essential. First-line management may include a bolus dose of IV hydralazine or labetalol. Labetalol should be used judiciously in a child with reactive airway disease as this could precipitate bronchospasm. Therefore, nicardipine (calcium channel blocker) would be the treatment of choice. This child has symptoms of severe hypertension, which could include congestive cardiac failure, palpitations, murmur, headache, vomiting, lethargy, acute hypertensive encephalopa­ thy, seizure, or cerebrovascular accident (hemorrhagic stroke). Causes of severe hypertension in pediatric patients include renal disease (glomerulonephritis, reflux nephropathy, obstructive uropathy, acute kidney injury, polycystic kidney disease, end­ stage renal disease at presentation), malignancy (pheochromo­ cytoma, Wilms tumor, neuroblastoma) , vascular abnormality (aortic coarctation, renal artery stenosis, hemolytic uremic syn­ drome), or medications (illicit substances, eg, MDMA; rapid withdrawal of clonidine or beta-blockers). Given the presenta­ tion of flushing and diaphoresis, this patient should receive a full workup to rule out pheochromocytoma or neuroblastoma.

Helpful Tip Coa rctation of the a o rta contributes to hyperten s i o n in th ree c l i n i ca l sett i n g s :

1.

2.

Presu rg i c a l : Prere n a l hypoperfu s i o n res u lts i n acti­ vation of ren i n - a n g iote n s i n system. I m mediate posto perative period (after repa i r) : Para­ d oxica l hyperten s i o n occurs fro m i n creased activity



� 1 1 1r

Helpful Tip

A hyperte n s ive urgency is a n e l evation of blood pres s u re with sym pto m s but no end - org a n d a mage. A hyperten s ive emergency is a n e l evation of b l ood pressu re with end-org a n d a m a g e.

of the ren i n - a n g i oten s i n a n d sympathetic n e rvou s

3.

syste m. Yea rs after repa i r: C h ro n i c hyperte n s i o n i s recog­ n ized in postoperative patie nts. Del ayed d iag nosis and repa i r is a ssociated with m o re severe c h ro n i c hypertension.

A 4-year-old girl with a history of reactive airway disease presents with worsening headache over the past 2 days that is unresponsive to acetaminophen. She has intermittent facial flushing, diaphoresis, and palpitations. On physical exam, her heart rate is 1 1 0 bpm, blood pressure is 1 50/ 1 00 mm Hg, and funduscopic exam reveals blurred optic discs. She is admitted for acute management of hypertension. Question 3 1 - 1 The choice of drug for management for the next 24 hours would be: A) Isradipine PO q6h. B) Captopril PO q6h. C) Hydralazine IV q4h prn. D) Labetalol continuous IV infusion. E) Nicardipine continuous IV infusion.

A 1 5-year-old overweight, adolescent boy who is at the 75th percentile for height presents from school physical examina­ tion with blood pressure of 1 35/85 mm Hg. He is asymptom­ atic, without headache, chest pain, or palpitations. He has mild shortness of breath on exertion. Question 32-1 Initial investigations should include all of the following EXCEPT: A) Plasma renin. B) Echocardiogram. C) Renal ultrasound. D) BUN and creatinine. E) Repeat blood pressure measurements. Discussion 32-1 The correct answer is ''A:' This is an adolescent patient with asymptomatic elevated blood pressure readings that fall within the stage 1 hypertensive range (we plotted it for you) . He is overweight and is otherwise well. The first-line investigations for adolescent hypertension include four-point blood pressure; renal ultrasound to investigate for dysplasia, atrophic kidneys,

CHAPTER 3 1

or other anomalies; renal function to determine whether there is underlying chronic kidney disease; and echocardiogram to eval­ uate for chronicity of hypertension and to detect left ventricular hypertrophy as a sign of end-organ damage. Repeat blood pres­ sure measurements at multiple clinic visits, and school or home blood pressure readings over a 3 -month period, are required to confirm the presence of true hypertension. Plasma renin is opti­ mally used when investigating a child or adolescent with stage 2 hypertension, and potentially with systemic conditions associ­ ated with hypertension.

Parents bring in their 3-year-old otherwise healthy son to your office with concerns of developmental delay. He has yet to speak in 2-word phrases. The mother has noticed that he prefers to play by himself and is very active all the time. He follows 1 -step commands well, but struggles with multistep commands. On physical exam, he is at the lOth percentile for height and weight, and the 40th percentile for head circum­ ference. Vital signs are normal except for manual blood pres­ sure of 1 1 5/65 to 1 20/69 mm Hg on his arms, and 1 20/70 to 1 25/72 mm Hg on his legs. He has palpable soft nodules on his upper back, brown patches on his trunk, and freckling in the axillary region. Question 33-1 You order a renal ultrasound, with specific attention to look for: A) Angiomyolipoma. B) Renal dysplasia. C) Renal artery stenosis. D) Middle aortic stenosis. E) Adrenal mass. Discussion 33-1 The correct answer is "C:' This child has developmental delay, macrocephaly, neurofibromas, cafe-au -lait spots, and axillary freckling, which strongly suggest neurofibromatosis type 1 (NF l ) . NF l is associated with hypertension, with renal artery stenosis being the predominant etiology. A Doppler renal ultrasound would be most appropriate. Angiomyolipoma is associated with tuberous sclerosis. Although tuberous scle­ rosis is associated with developmental delay and produces skin findings such as facial angiofibromas or hypomelanotic macules (ash-leaf spots), affected children do not have axil­ lary freckling or cafe-au-lait spots. Renal dysplasia can result in hypertension, but the primary genetic syndromes of neu­ rocutaneous findings and hypertension should be ruled out first. There is also no significant past medical history in this case. Supravalvular aortic stenosis is typically a concern in Williams-Beuren syndrome, and this is best investigated with CT angiogram or angiography. An adrenal mass should also be investigated with CT but is unlikely to be associated with developmental delay.



R E N A L A N D U RO LOG I C D I S O R D E RS

695

Helpful Tip

� Doppler

=

r1 1r



u ltraso u n d

has

low specifi city for

re n a l

a rtery ste nosis. If there is stro n g suspicion o f re n a l a rtery ste nosis, a n g i o g ra p h i c i m a g i n g m o d a l ities (CT a n g iogram, a n g i o g ra phy) a re defi n itive d i a g nostic tests.

Question 3 3-2 Which of the following disorders is/are associated with abnormal blood vessels that result in findings of renovascu­ lar hypertension secondary to either renal artery stenosis or middle aortic syndrome? A) Fibromuscular dysplasia. B) Takayasu arteritis. C) Tuberous sclerosis. D) Neurofibromatosis. E) Retroperitoneal fibrosis following abdominal irradiation. F) All of the above. Question 33 - 2

The correct answer is "F:' Middle aortic syndrome is charac­ terized by segmental narrowing along the abdominal or distal descending aorta. Narrowing may involve the ostia of the renal artery. Hypertension is noted above the stenosis, with a blood pressure gradient distal to the stenosis. These disorders typically are associated with severe and difficult-to-control hypertension, resulting in use of numerous antihypertensive agents. Surgical intervention, including angioplasty, may be required but has variable success. •

Helpful Tip

� Do not prescribe an ACE i n h i bitor if there is b i l atera l

=

r11r

re n a l a rtery stenosis. This c o u l d res u l t in serious re n a l fa i l u re!

� QUICKQUIZ What is NOT a sequela of childhood hypertension? A) Atherosclerosis. B) Left ventricular hypertrophy. C) Liver disease. D) Retinal arteriopathy. E) Stroke. Discussion The correct answer is "C:' Elevated blood pressure is associ­ ated with the development of end organ damage affecting the kidney, heart and blood vessels. End organ damage is typically seen after longer time periods of elevated blood pressures but if pressures are very high (hypertensive emergency) complica­ tions may develop over a short time period.

696

MCG RAW- H I LL E D U CAT I O N S P E C I A LTY BOA R D REVI EW: P E D I AT R I C S

BIBLIOGRAPHY Hypertension

Falkner B. Hypertension in children and adolescents: Natural history and epidemiology. Pediatr Nephrol. 20 1 0;25: 1 2 1 9 - 1 224. Flynn JT, Tullus K. Severe hypertension in children and ado­ lescents: Pathophysiology and treatment. Pediatr Nephrol. 2009;24(6) : 1 1 0 1 - 1 1 1 2 . Hansen ML, Gunn P W, Kaelber D C . Underdiagnosis of hypertension in children and adolescents. JAMA. 2007;298:8 74-879. National High Blood Pressure Education Program Working Group on High Blood Pressure in Children and Adoles­ cents. The fourth report on the diagnosis, evaluation, and treatment of high blood pressure in children and adoles­ cents. Pediatrics. 2004; 1 14(suppl 2):555-576. Tullus K, Brennan E, Hamilton G, et al. Renovascular hyper­ tension in children. Lancet. 2008;37 1 (9622) : 1453 - 1 463. Viera AJ, Neutze DM. Diagnosis of secondary hyperten­ sion: An age-based approach. Am Pam Physician. 20 1 0;82 ( 1 2) : 147 1 - 1 478. Chron ic Kidney Disease

Atkinson MA, Furth SL. Anemia in children with chronic kidney disease. Nat Rev Nephrol. 20 1 1 ;7:635-64 1 . Dharnidharka VR, Fiorina P, Harmon WE. Kidney transplan­ tation in children. N Engl J Med. 2014;37 1 (6):549-558. Mi.iller D, Goldstein SL. Hemodialysis in children with end­ stage renal disease. Nat Rev Nephrol. 20 1 1 ;7:650-658. Rees L, Mak RH. Nutrition and growth in children with chronic kidney disease. Nat Rev Nephrol. 20 1 1 ;7:6 1 5-623. Schaefer F, Warady BA. Peritoneal dialysis in children with end-stage renal disease. Nat Rev Nephrol. 20 1 1 ;7:659-668.

Schmitt CP, Mehls 0. Mineral and bone disorders in chil­ dren with chronic kidney disease. Nat Rev Nephrol. 20 1 1 ;7:624-634. Shroff R, Weaver DJ, Mitsnefes MM. Cardiovascular complica­ tions in children with chronic kidney disease. Nat Rev Nephrol. 20 1 1 ;7:642-649. Urologic Disorders

Ingraham SE, McHugh KM. Current perspectives on con­ genital obstructive nephropathy. Pediatr Nephrol. 20 1 1 ;26(9) : 1453- 1 46 1 . Strand WR. Initial management of complex pediatric disor­ ders: Prunebelly syndrome, posterior urethral valves. Ural Clin North Am. 2004;3 1 (3):399-4 1 5, vii. Glomerular Diseases

Cramer MT, Guay-Woodford LM. Cystic kidney disease: A primer. Adv Chronic Kidney Dis. 2 0 1 5;22(4):297-305. Gipson DS, Massengill SF, Yao L, et al. Management of childhood onset nephrotic syndrome. Pediatrics. 2009 ; 1 24(2):747-757. Harris PC, Torres VE. Polycystic kidney disease, autosomal dominant. In: Pagon RA, Adam MP, Ardinger HH, et al, eds. GeneReviews [Internet]. Seattle, WA: University of Washington, Seattle; 1 993-20 1 5. Acute Kidney I njury

Chan JCM, Williams DM, Roth KS. Kidney failure in infants and children. Pediatr Rev. 2002;23 (2) :47-60. Moffett BS, Goldstein SL. Acute kidney injury and increas­ ing nephrotoxic-medication exposure in noncritically ill children. Clin J Am Soc Nephrol. 20 1 1 ;6(4):856-863. Van De Voorde RG 3rd. Acute post-streptococcal glomerulo­ nephritis: The most common acute glomerulonephritis. Pediatr Rev. 20 1 5;36( 1 ) :3 - 1 2 .

32

Respi ratory D isorders Anthony Fischer

A 2-year-old fully immunized boy is brought to the emer­ gency department for sudden onset of barking cough associ­ ated with inspiratory stridor. He has had a runny nose and hoarse voice for the last day. Temperature is 37.6°C (99.7°F), heart rate is 150 beats per minute (bpm), respirations 40 breaths per minute, and oxygen saturation (Spo) by pulse oximeter is 98%. He has moderate suprasternal retractions and stridor. Lung fields are clear to auscultation. Question 1 - 1 Your treatment plan includes: A) Albuterol and prednisolone. B) Aerosolized 3% hypertonic saline. C) Ceftriaxone. D) Aerosolized epinephrine and dexamethasone. E) Amoxicillin. F) Further diagnostic testing. Discussion 1 - 1 The correct answer is "D:' The history and physical exam are consistent with viral laryngotracheobronchitis (croup), which is caused by an infection with mucosal edema of the larynx, trachea, and bronchi. Nasal congestion and cough appear first. Over 12 to 48 hours, the cough worsens, becomes barky, and stridor develops. Typically, children 6 months to 3 years of age are affected. If a child is older than age 6 years, it is not croup. Fall is the season of choice, and symptoms are worse at night. Parainfluenza viruses are usually to blame. Swelling makes the airway narrower, increasing airflow resistance and resulting in the classic barky cough with stridor. If a child barks like a seal, chances are he or she has croup. Diagnostic studies are not needed. Croup is a clinical diagnosis that even medical students can make. The edema improves with dexamethasone (typi­ cal dose 0.6 mg/kg) . For patients with respiratory distress and stridor at rest, aerosolized epinephrine should also be given. Symptoms can return 2 to 3 hours later when the medication wears off. Children should be monitored for 3 to 4 hours after

treatment with aerosolized epinephrine to ensure that stridor at rest does not return. The key is stridor at rest. Stridor will still be present if the child is crying. Albuterol or prednisolone would be appropriate treatments for an asthma exacerbation, which is characterized by inflammation and bronchoconstric­ tion in the intrathoracic airways (inside the chest). Symptoms include wheezing (not stridor) . Three percent hypertonic saline has been proposed as a treatment for bronchiolitis but has not been tested in croup. Ceftriaxone and amoxicillin could be used to treat bacterial pneumonia, but the normal oximetry and lack of fever or localized crackles makes this possibility unlikely. Immunization status is always a relevant question in pediatrics. Epiglottis caused by Haemophilus influenzae infection presents with stridor, respiratory distress, and drooling. Lying down makes everything worse so the child sits in a tripod position (upright, leaning forward, hands on knees) . This is a life-threatening emergency that could be mostly prevented with Hib vaccination. The airway may become obstructed by the big swollen epiglottis. Do everything in your power to not tick off a child with epiglottis. •

Helpful Tip

:5.� Forget the cool

i1 1 r

m i st a n d stea m therapies fo r c ro u p.

These therapies have not been s h own to work (sorry!) but a re not ha rmfu l , so when someone i n si sts they h e l p j u st s m i l e.



:5.� i1 1r

Helpful Tip Racem i c e p i n e p h r i n e i s a

1 :1

m ixtu re of D - a n d

L- isomers. It was fe lt t o have fewer side effects a n d preferred ove r L- e p i n e p h rine. lt i s now known that both a re effective and rates of tac hyca rd ia and hyperte nsion a re the s a m e.

An anteroposterior (AP) X-ray was ordered just in case the child swallowed a quarter when his dad was not watching. 697

698

MCG RAW- H I LL E D U CAT I O N S P E C I A LTY BOA R D REVI EW: P E D I AT R I C S

Question 1 -2 What is the classic radiographic finding with croup? (See Figure 32- 1 .) A) Thumb sign. B) Widened retropharyngeal space. C) Radiopaque circle. D) Steeple sign. E) Subcutaneous emphysema. Discussion 1 -2 The correct answer is "D:' In a child with croup an AP film of the neck shows narrowing of the subglottic air column, which

resembles a church steeple. Epiglottitis may show a thumbprint sign on lateral neck film. Remember what you just read-do not aggravate a child with epiglottitis. A widened retropharyngeal space on lateral film may be seen with retropharyngeal abscess. A radiopaque circle would suggest a foreign body. Subcutane­ ous emphysema is seen when air from a pneumomediastinum extends to the soft tissue.

A mother brings to the clinic a 6-week-old girl who has a high-pitched inspiratory noise that worsens with excitement. The girl has no respiratory distress or hoarseness. She has never turned blue or stopped breathing. She breastfeeds well and is growing normally. Question 2-1 What is the most likely cause of the inspiratory noise? A) Foreign body. B) Vocal cord paresis. C) Laryngomalacia. D) Tracheomalacia. E) Aberrant right subclavian artery.

A

B F I G U R E 3 2 - l A, Steeple sign in a 1 -yea r-ol d with crou p. B, Steeple sign i n a 1 2-yea r-old. (Reproduced with perm ission from Stone CK, H u m phries RL, eds. Current Diagnosis and Treatment: Emergency Medicine. 7th ed. New York, NY: McGraw-H i l l Education; 201 1 , Fig. 32-1 0.)

Discussion 2-1 The correct answer is "C:' The most common cause of stridor in newborns is laryngomalacia. Laryngomalacia causes inter­ mittent collapse of the supraglottic structures with inspiration. Infants are squeaky and musical sounding. It is noticed soon after birth, progressively worsens until 4 months of age, and then typically resolves by 1 year. The inspiratory stridor is ( 1 ) worse with crying, feeding, or excitement; (2) worse with viral upper respiratory tract infections; and (3) positional in nature (worse when supine) . Flip the infant prone either on the bed or over your forearm; the stridor should lessen. (People will think you are a diagnostic genius.) The differential diagnosis for stridor is long but all candidate conditions cause airway obstruction typically outside of the chest (extrathoracic). (See Table 32- 1 . ) Unilateral vocal cord paresis may be recognized by a weak cry, hoarseness, or aspiration with feeds. Bilateral disease causes stridor. Laryngomalacia does not cause hoarseness. Tra­ cheomalacia is less common (approximately 1 :2000) . Trachea­ malacia can cause stridor depending on the location, but more frequently is associated with cough and wheezing. Vascular rings are congenital anomalies of the aortic arch that compress the trachea or esophagus, or both, causing stridor. Complete rings fully encircle the trachea and esophagus (ie, double aortic arch) . Incomplete rings (ie, pulmonary sling) do not. An aber­ rant right subclavian artery (incomplete ring) courses posterior to the esophagus and can cause difficulty with swallowing ( dys­ phagia lusoria) . Let's hope this child's older sibling did not feed her a peanut that she aspirated.

CHAPTER 32

TABLE 32-1 CAU S E S OF STR I D O R BAS E D O N

T I M I N G D U R I N G R E S P I RATO RY CYC L E A N D LOCAT I O N O F OBSTRUCTI O N

I nspi ratory stridor •

S u p ra g l otti s

Laryngomalacia C ro u p Epiglottitis Peritonsi l l a r a bscess Retropharyngeal a bscess Foreign body L i n g u a l thyroid Retro/m icrog nathia Ton s i l hypertrophy Laryngospasm



R E S P I RATO RY D I SO R D E R S

699

D) Racemic epinephrine by nebulizer. E) Close outpatient monitoring of growth and respiratory symptoms. Discussion 2-2 The correct answer is "E:' Laryngomalacia usually has a benign course. For mild symptoms, as in this infant, no intervention is necessary. Severe symptoms warranting intervention and referral to otolaryngology include problems with feeding, poor growth, apneic spells, or persistent respiratory symptoms. Pharyngolaryn­ geal reflux is frequently associated with laryngomalacia; therefore, antireflux medications (proton pump inhibitor or H2 histamine antagonist) are recommended by some experts. If needed, a supra­ glottoplasty may be performed surgically to remove excess tissue. Tracheostomy is not necessary in this patient. Racemic epineph­ rine is a treatment for croup, which is a cause of acute stridor.

Ana phylaxis •





B i phasic stridor G l otti s U s u a l ly a fixed obstruction

Vocal cord paralysis S u bg l ottic hemangioma Foreign body Laryngeal web Laryngeal cyst Recu rrent respi ratory papi l l omatosi s Trachea l stenosis Complete or near-complete trachea l rings or tracheal stenosis

Expi ratory stridor S u bg lottis



S u bg l ottic stenosis Tracheoma lacia Bacterial tracheitis Vasc u l a r ring Foreign body



Helpful Tip

� Stri d o r fro m a s u b g l ottic h e m a n g io m a is often i1 1 r m i s d i a g nosed as crou p, a n d both i m p rove with steroids. =

Li ke hema n g io m a s on the skin, these s u b g l ottic lesions

g row ra p i d l y d u ri n g the early months of l ife, worse n i n g the a i rway obstruction. Diag nosis req u i res a h i g h i n d ex o f suspicion a n d e n d osco py. Look fo r recu rrent episod es, b i p h a s i c stridor, fac i a l h e m a n g iomas i n a beard d i stribution, a p nea, a n d l a c k of vira l prod rome.

Question 2-2 How should this infant girl's problem be managed? A) Supraglottoplasty. B) Tracheostomy. C) Therapeutic trial of lansoprazole.

A 5-year-old immunized girl develops biphasic stridor and high fever. She has a brassy-sounding cough and has severe respiratory distress. She is toxic appearing but not drooling and able to lie flat. Question 3-1 What diagnosis do you suspect? A) Croup. B) Peritonsillar abscess. C) Pneumonia. D) Tracheitis. E) Diphtheria. Discussion 3-1 The correct answer is "D:' The child has symptoms of bacterial tracheitis. This is a life-threatening infection that may require intubation to keep the airway open. Usually it occurs after a viral illness. Affected children want to lie flat, which is the oppo­ site of epiglottitis. Pneumonia does not cause wheezing. Croup produces a barky cough. A peritonsillar abscess gives a muffled "hot potato" voice. Diphtheria can cause pseudomembranous pharyngitis, but this child is immunized. The child is admitted to the PI CU. Question 3-2 Antibiotic coverage should include all of the following organ­ isms EXCEPT: A) Haemophilus influenzae. B) Staphylococcus aureus. C) Pseudomonas aeruginosa. D) Streptococcus pyogenes. E) Mycoplasma pneumoniae. Discussion 3-2 The correct answer is "E:' Bacterial tracheitis can be polymicro­ bial and has been reported with all of these organisms except

Mycoplasma.

700

MCG RAW- H I LL E D U CAT I O N S P E C I A LTY BOA R D REVI EW: P E D I AT R I C S

An infant has biphasic stridor, poor feeding, and respiratory distress. You suspect vascular compression of the trachea. Question 4- 1 Which of the following tests would be best to address this question? A) Chest X-ray. B) Noncontrast chest CT. C) Barium esophagram. D) Chest CT angiography. E) Echocardiography. F) Bronchoscopy. Discussion 4- 1 The correct answer is "D:' CT angiography would give the greatest sensitivity and specificity for identifying vascular compression.

A 7-year-old previously healthy girl was hospitalized for pneumonia requiring emergency intubation in the emer­ gency department of a community hospital. One month after discharge, she developed progressive dyspnea and biphasic stridor. She performs spirometry in your office. Question 5-1 Which flow-volume loop is most likely? (See Figure 32-2.) A) Normal. B) Expiratory small airway obstruction, bronchodilator responsive. C) Normal expiratory loop, inspiratory obstruction. D) Inspiratory and expiratory large airway obstruction. E) Expiratory obstruction, bronchodilator nonresponsive. Discussion 5-1 The correct answer is "D:' This child has developed subglot­ tic stenosis as a complication of traumatic intubation resulting

'

3:

Edema fro m i nfection o r l a ry n g o p h a ryngeal refl ux ma kes it worse.

Question 5-2 What is the next step in evaluation and management? A) Referral to otolaryngology for direct laryngoscopy and bronchoscopy. B) Tracheostomy. C) Therapeutic trial of lansoprazole.

C' .8 � 'iS..

iS..

(/)

(/)

E

3:

3:

0

Volume

u:::

C' 0



0

V olume

u:::

C'

tt

TLC V olume

0



X w

X w

·c..

·c..

A

B

c

·c..

may s i g n a l u nd e rlyi ng

s u b g l ottic ste nosis. The a i rway is a l ready na rrowed.

E

0



i1 1r

0

(/)

0

Helpful Tip



E

C'



:5.� Recu rrent o r persistent c ro u p

C'

C' .8 � ' iS..

u:::

in near complete central airway obstruction, which can be recognized by flow limitation on expiration and inspiration. Subglottic stenosis can be congenital or acquired. Acquired cases are usually associated with endotracheal intubation­ traumatic, prolonged (think preemies), or use of an over­ sized endotracheal tube (most common) . Tracheostomies may also cause stenosis. Listen for biphasic stridor. Flow-vol­ ume loops or curves plot inspiratory and expiratory airflow {y-axis) against lung volume {x-axis ) . {See Figure 3 2 - 3 . ) Looking a t the flow-volume loops i s helpful when assessing central airway obstruction. (See Figure 32-4.) A fixed central airway obstruction decreases inspiratory and expiratory air­ flow. When obstruction affects expiration or inspiration but not both, it is dynamic and determined by pressure changes in and out of the airways as well as location in or out of the chest. For example, an airway obstruction that is variable and extrathoracic reduces inspiratory flow with resultant stridor. The obstruction resolves with expiration. Think of drinking a milkshake through a flexible straw. When you apply suction, negative inspiratory pressure is generated, the straw collapses and nothing comes up the straw to your mouth. When you release or blow air down the straw (creating positive pressure), it opens again. Subglottic stenosis is a fixed obstruction so this child's loop would look like panel C in Figure 32-3.

X w

F I G U R E 32-2. Flow-vo l u m e loops. The y-axis measures a i rflow in l iters per second. The x-axis measures l u n g vol u m e in l iters. The expi ratory cu rve is a bove the

x-axis and i n s p i ratory cu rve below. (A) Normal: the expi ratory flow rate ra pidly rises to its pea k then fa l l s i n l i near fashion; the i n s p i rato ry cu rve is symmetric and saddle sha ped. (B) Ai rflow obstruction: the slope of the expi ratory cu rve is scooped out a n d the pea k flow is red uced. (C) Fixed central ai rway obstruction: flow is l i m ited i n both inspiration a n d expiration with flatte n i n g of both cu rves. RV, resi d u a l vol u me; TLC, tota l l u n g capacity. (Reproduced with perm ission from Kasper DL, Fa uci AS, Ha user SL et al: Harrison's Principles of Internal Medicine, 1 9th ed. McGraw- H i l l Education, I nc., 201 5 . Fig 306-4.)

CHAPTER 32

R E S P I RATO RY D I SO R D E R S



Trachea

A

B

701

Proximal esophagus

c

Posterior Exp i ration

t



Esophageal atresia

;;: 0 u::

I nspiration



- I ncreasing volume

D

E

RV-residual volume, TLC-total l u n g capacity

A N o rmal B Lowe r ai rways obstruction

C

Fixed central ai rway obstruction

D Valuable central extrathoracic obstruction E Valuable central intrathoracic obstruction

Distal esophagus

F I G U R E 32-3. The y-axis measures a i rfl ow in l iters per seconds. The x-axis

measures l u ng vol u m e i n l iters. The expi rato ry cu rve is a bove the x-axis a n d i n s p i ratory cu rve below. (A) N o r m a l : the expi ratory fl o w rate ra pidly rises to its pea k then fa l l s in l i near fashion; the i n s p i ratory cu rve is sym metric and saddle sha ped. (B) Ai rflow obstruction: the slope of the expi ratory cu rve i s scooped out and the pea k flow is red uced. (C) Fixed centra l a i rway obstruction: fl ow is l i m ited in both i n s p i ration a n d expiration with fl atte n i n g of both cu rves. (D) Va riable or dyn a m i c extrathoracic obstructio n : i n s p i ratory fl o w is l i m ited w i t h flatte n i n g of the i n s p i ratory cu rve. ( E ) Va riable i ntrathoracic obstruction: expi ratory flow is l i m ited with flatte n i n g of the expi ratory cu rve.

D) Racemic epinephrine by nebulizer. E) Close outpatient monitoring of growth and respiratory symptoms. Discussion 5-2 The correct answer is "A:' The patient should be referred to otolaryngology to assess surgical options. Treatment depends on severity and location of the stenosis. This patient already manifests severe respiratory symptoms and outpatient obser­ vation without referral is not appropriate. Surgical options include open reconstruction and endoscopic dilation/laser. Tracheostomy is avoided if possible. Acid-suppressive medica­ tions will not reverse the degree of airway obstruction. They may have a role in preventing subsequent insult, but this is not proven.

A newborn child was diagnosed with C-type esophageal atre­ sia and tracheoesophageal fistula (EA- TEF) (diagram below) and undergoes surgical repair. Question 6- 1 Which other airway abnormality is most likely to coexist with this defect?

A) B) C) D) E)

Left bronchial stenosis. Complete tracheal rings. Tracheomalacia. Posterior laryngeal cleft. Tracheal compression by anomalous left pulmonary artery.

Discussion 6-1 The correct answer is "C:' A tracheoesophageal fistula (TEF) generally results in a wider posterior tracheal membrane at the site of the defect as well as tracheal rings that are less complete. This causes the trachea to collapse with increased air flow. Left bronchial stenosis can be acquired as a complication of some thoracic surgeries, especially cardiac surgeries. However, this complication would be rare. Posterior laryngeal cleft is a disor­ der that results in communication between the airway and the esophagus at the level of the larynx, causing recurrent aspira­ tion and difficulty handling secretions. It is a rare occurrence compared with tracheomalacia. An anomalous left pulmonary artery (pulmonary sling) results in division of the trachea and esophagus and would be most unlikely in a patient with TEE

� QUICKQUIZ What is the most common type of TEH A) Type A. B) Type B. C) Type C. D) Type D. E) Type E. Discussion The correct answer is "C:' You would think the most common variant would be called type A, but not in this defect. In type C TEF, the upper esophagus ends in a blind pouch and the TEF is connected to the distal esophagus; this variant is present in

702

MCG RAW- H I LL E D U CAT I O N S P E C I A LTY BOA R D REVI EW: P E D I AT R I C S

t

Expi ration

TLC

I

RV

I I nspiration A

Expi ration

I nspiration

Fixed (intra or extrathoracic)

t

Expi ration

'i: 0 u:::

Paw < Patm

Paw < Patm

I nspiration

Expi ration

B

RV

TLC

I I nspiration

Variable extrathoracic

t

Expi ration

TLC

A Paw > ppl

Paw < Ppl

I nspiration

Expi ration

c

/.....,\' I

RV

'i: 0 u:::

I nspiration

Variable i ntrathoracic

F I G U R E 32-4. Flow-vo l u m e loops-centra l a i rway obstructions. (A) Fixed obstruction: flow is l i m ited i n both i n s p i ration and expiration with fl atte n i n g of both cu rves. (B) Va riable or dyn a m i c extrathoracic obstruction: i n s p i ratory flow is l i m ited with fl atte n i n g of the i n s p i ratory cu rve. (C) Va ria ble i ntrathoracic obstruction: expi ratory flow is l i m ited with flatte n i n g of the expi ratory cu rve. (Reproduced with permission fro m Bu rrows B, Knudson RJ, Quan SF, et al: Respiratory Disorders: A Pathophysiologic Approach. 2d ed. Chicago: Year Book Medical Publis hers; 1 983.)

87% of cases. In proximal TEF, the upper esophagus connects with the trachea allowing aspiration of oral secretions and feeds. In distal TEF, the lower esophagus connects with the trachea allowing aspiration of gastric contents. (See Figure 32-5.) Wanting to learn more about EA-TEF, you ask the father what symptoms the newborn had that led to the diagnosis. Question 6-2 Which is NOT a symptom of EA-TEF? A) Drooling. B) Coughing immediately after feeds. C) Respiratory distress. D) Polyhydramnios in utero. E) All of the above.

Discussion 6-2 The correct answer is "E:' Esophageal atresia (EA) is the most common congenital anomaly of the esophagus. Over 90% of cases have a concurrent tracheoesophageal fistula (TEF). The clinical presentation of TEF depends on the presence or absence of EA. Neonates with EA develop symptoms immediately after birth from excess secretions. Think of a bubbling fountains; affected infants have frothy saliva coming from their mouth and nose in a similar fashion. They will drool, cough, choke, pos­ sibly turn blue, and have respiratory distress. Feeding will make everything worse as oral content is aspirated through the proxi­ mal fistula. Gastric contents can reflux through a distal fistula, causing chemical pneumonitis. Crying introduces air into the stomach through a distal fistula, just as in a balloon that is being inflated.

CHAPTER 32

A

B

D



R E S P I RATO RY D I SO R D E R S

703

c

E

F I G U R E 32-5. Types of esophagea l atresia a n d tracheoesophagea l fistu la. (A) Type C: esophagea l atresia with d ista l tracheoesophagea l fistula (87% of cases). (B) Type A: esophagea l atres ia without tracheoesophagea l fistu l a {8% o f cases). (C) Type E: H-type fistula, tracheoesophagea l fistu la without esophagea l atresia (4% of cases). (D) Type D: esophageal atresia with proximal and d i stal tracheoesophagea l fistu la (< 1 % of cases). (E) Type B: esophagea l atresia with prox i m a l tracheoesophagea l fistu la ( 1 o/o of cases) .

F I G U R E 3 2-6. This i s a newborn with eso p h a g e a l atres i a a n d tra c h e o es o p h a g e a l fistu la. T h e feed i n g tube is coi l ed i n t h e u pper pouch of the esophagus. A tracheoesophageal fistu la is present as there is a i r below the d i a phra g m . (Reproduced with perm ission from Brun icard i FC, Andersen DK, B i l l i a r TR, et al, eds. Schwartz's Principles of Surgery. 1 Oth ed. New York, NY: McGraw- H i l l Education; 201 5, Fig. 39-1 0.)

(Reproduced with permission from Lalwa n i A K, ed. Current Diagnosis and Treatment in Otolaryngology: Head and Neck Surgery. 3 rd ed. New York, NY:

McGraw- H i l l Education; 201 1 , Fig. 3 5 - 1 A-E.)

� •

I I

Helpful Tip I nfa nts with an isolated TEF without EA (H - type fi st u l a ) may present later in i nfa n cy with recu rrent wheez i n g or a s p i ration p n e u m o n ias, or bot h .

Question 6-3 Which is NOT a true statement about EA-TEH A) It is associated with CHARGE syndrome. B) It may cause prenatal oligohydramnios. C) A nasogastric or orogastric tube will not pass into the stomach. D) An esophagogram is helpful to diagnosis H-type fistulas (type E). E) Vertebral defects and anal atresia are associated conditions. Discussion 6-3 The correct answer is "B:' Remember, the fetus swallows amni­ otic fluid then pees it out. An intestinal obstruction such as EA will cause buildup of unswallowed fluid (polyhydramnios). Concurrent congenital anomalies are present 50% of the time. CHARGE is a mnemonic for Coloboma, Heart defects, choanal Atresia, Retarded growth and development, Genital defects, and Ear defects. Do not be confused; there is no "T" but TEF is associated nonetheless. VACTERL stands for Vertebral defects, Anal atresia, Cardiac defects, TracheoEsophageal fistula, Renal

anomalies, and Limb abnormalities. If you find a TEF, look closely for deformities, call a geneticist, and order some imag­ ing. A quick test for EA is to try to pass a nasogastric or oro­ gastric tube. It won't work. On chest X-ray it may be coiled in the esophageal pouch. (See Figure 32-6.) Notice this is for EA that likely has a TEF. Type E TEF (H-type) does not have an EA. (See Figure 32-5.) An esophagogram isn't a perfect test. Endoscopy with bronchoscopy or CT imaging may be needed to clarify findings. For EA and TEF, call a surgeon right away. These defects are treated surgically.

A 2-year-old child acutely developed severe respiratory dis­ tress, and stridor. He is unable to swallow secretions. Vital signs are temperature 40°C ( 1 04°F), heart rate 190 bpm, res­ pirations 50 breaths per minute, and blood pressure (BP) 80/40 mm Hg. On exam he is toxic appearing, leaning for­ ward, drooling, and in distress. Question 7- 1 Which of the following should be performed first? A) Endotracheal intubation by a skilled provider. B) IV placement for fluid administration. C) Neck CT scan. D) Intramuscular ceftriaxone. E) Throat examination and pharyngeal culture.

704

MCG RAW- H I LL E D U CAT I O N S P E C I A LTY BOA R D REVI EW: P E D I AT R I C S

Discussion 7-1 The correct answer is "A:' The patient in this vignette has airway compromise from epiglottitis. The most important first step is to secure the airway (remember those ABCs)-preferably in an operating room setting. IV placement, fluid resuscitation, antibiotic treatment, and diagnostic studies should be delayed until after an artificial airway is established. Epiglottitis is a life-threatening emergency that rapidly progresses to upper air­ way obstruction from edema of the epiglottis and aryepiglottic folds. A child presents with the abrupt onset of fever, dysphagia, drooling, dysphonia, and distress (the 4 Ds) . The illness pro­ gresses rapidly over 1 2 to 24 hours. Children look toxic and sit in a tripod position (mouth open, neck hyperextended, sitting up, leaning forward) to make the airway as wide as possible. As a result of widespread vaccination against Hib, cases of epiglot­ titis now commonly involve bacteria other than H. influenzae and affect older children rather than toddlers. Immunocompro­ mised individuals and those with incomplete vaccination are at risk. Have you caught on that epiglottitis, although rare now with immunizations, is a big deal?

A 3-year-old boy has fever, stridor, respiratory distress, and torticollis. He has decreased range of motion of his neck, especially lateral rotation and extension. You suspect that the child has a retropharyngeal abscess. Question 8- 1 Which of the following statements about retropharyngeal abscess is true? A) Lateral neck X-ray has greater sensitivity for retropharyn­ geal abscess than neck CT with contrast. B) Anaerobic organism coverage is not necessary for patients with retropharyngeal abscesses. C) Lateral neck X-ray should be performed with the neck slightly flexed. D) Direct spreading of the infection can result in mediastinitis or meningitis. E) Culture of a drained abscess usually reveals a single organism. Discussion 8- 1 The correct answer is "D:' Deep neck space infections include retropharyngeal and parapharyngeal abscess. The retropharyn­ geal nodes that become infected involute by age 5 years, mak­ ing infection in older children uncommon. Direct spreading of retropharyngeal abscess can result in a variety of complica­ tions such as mediastinitis, meningitis, and venous thrombo­ sis. Computed tomography ( CT) scan with contrast has greater sensitivity than lateral neck X-ray for diagnosing retropharyn­ geal abscess. Iflateral neck X-ray is obtained, the neck should be slightly extended. Flexion of the neck makes the retropharyn­ geal space appear widened. Because retropharyngeal abscesses are often polymicrobial and can harbor anaerobic organ­ isms, treatment with ampicillin-sulbactam or clindamycin is recommended.

An 1 8-month-old boy presents with acute cough, stridor, and drooling. He was playing with his older brother when these symptoms developed. He was previously well. Question 9- 1 Which of the following statements about foreign body inges­ tion or aspiration is true? A) A chest X-ray may rule out the presence of an airway foreign body. B) Esophageal foreign bodies do not produce respiratory symptoms. C) Flexible bronchoscopy is the safest procedure for removing airway foreign bodies. D) Tracheoesophageal fistula is a potential complication ofbut­ ton battery ingestion. E) On chest X-rays, coins in the trachea usually appear more round on anteroposterior (AP) view compared with lateral view. Discussion 9- 1 The correct answer is "D:' This child aspirated a foreign body and it is obstructing his upper airway. Eighteen-month-olds like to put things in their mouths. Older siblings have fun toys with little parts; think Legos and Barbie shoes. The child is usually fine, then suddenly becomes blue and cannot breathe. A history of choking is not always present as the event was likely unwit­ nessed. Foreign bodies can land in any part of the airway but most commonly end up in the bronchi, especially on the right side. Symptoms vary and may be subtle as well as subacute ( eg, persistent wheezing unresponsive to treatment). Button bat­ tery ingestion is an emergency because it can cause esophageal necrosis resulting in mediastinitis or tracheoesophageal fistula. Do not mistake a button battery for a coin on a radiograph. But­ ton batteries have a double ring or rounded edge appearance. Coins have a sharp edge. Chest X-ray can locate a radiodense foreign body but will miss many life-threatening foreign bod­ ies such as plastic toys or peanuts. Flexible bronchoscopy is not recommended to remove foreign bodies because there is not a sufficient working channel to grab foreign material. Instead, rigid bronchoscopy should be performed to safely remove an airway foreign body. Coins in the esophagus generally appear more round on AP projection, but coins in the trachea are more round on a lateral view. This is because the posterior membrane of the trachea is more elastic than the cartilaginous anterior and lateral portions of the trachea.



Helpful Tip

=-� Most

i1 1 r

fo reig n bodies (pea n uts, plastic o bjects) a re

ra d i o l u cent a n d wi l l not be seen on ra d io g ra p h . Do not be fa lsely rea s s u red. If you r suspicion rema i n s h i g h, the c h i l d s h o u l d u n dergo rigid bronch oscopy not o n ly to m a ke the d i a g nosis but to rem ove the o bj ect.

CHAPTER 32

� QUICKQUIZ A friend's teenage son is eating peanuts when he starts laugh­ ing. He subsequently starts to cough and choke. He appears anxious and makes a hand gesture toward his throat. At this point, he is no longer coughing. What should be done next? A) Administer intramuscular epinephrine. B) Perform the Heimlich maneuver. C) Perform back blows and chest compressions. D) Do not intervene unless he loses consciousness. E) Sign up for a cardiopulmonary resuscitation (CPR) course. Discussion The correct answer is "B:' He is choking to death. This is a clas­ sic example of complete airway obstruction (cannot cough or speak) from aspiration of a little peanut. Remember to follow the American Heart Association guidelines. The Heimlich maneuver should be performed until the peanut is dislodged or the victim loses consciousness, at which time standard CPR should be per­ formed and 9- 1 - 1 called. Back blows and chest compressions are used to dislodge foreign bodies in infants. Blind finger sweeping is not recommended. Peanuts are a frequent cause of anaphylaxis, for which epinephrine should be administered. However, this sce­ nario does not have features of anaphylaxis as there is no involve­ ment of a second organ system ( eg, hives, angioedema, vomiting).



Helpful Tip

� In

i1 1 r

cases o f voc a l cord paralysis- i n d uced strid o r, d o

not sett l e after d i scove ring why the c h i l d has strid o r; look fo r the u n d erlyi ng etiology of the pa ra lysis. For exa m ple, res u l t i n g dys p h a g i a and fa i l u re to th rive a re sym pto m s of a C h i a ri m a lformation (bra i n stem c o m p ression) i n i nfa nts.

� QUICKQUIZ Which is NOT a typical cause of biphasic stridor? A) Subglottic stenosis. B) Vocal cord paralysis. C) Subglottic stenosis. D) Croup. E) Foreign body. Discussion The correct answer is "D:' Croup typically causes inspiratory stridor; however, if the swelling and inflammation became severe, the stridor could become biphasic.

A 1 6-year-old patient has a 6-month history of progressive hoarseness.



R E S P I RATO RY D I SO R D E R S

705

Question 1 0-1 What is the next step in evaluation? A) Reassurance. B) Trial of omeprazole. C) Laryngoscopy. D) Burst of prednisone. E) Trial of albuterol. Discussion 1 0-1 The correct answer is "C:' Chronic hoarseness may be related to damage to the vocal cords or abnormal vocal cord move­ ment. It is unlikely that hoarseness lasting 6 months will spontaneously resolve. Hoarseness could be a consequence of reflux or chronic cough, but before making such assump­ tions, the structure and function of the vocal cords must be examined. Question 1 0-2 Which of the following is the most likely risk factor for this disorder? A) Smoking. B) Human papillomavirus infection. C) History of intubation for a surgical procedure. D) Singing or prolonged vocalization. E) Gastroesophageal reflux. Discussion 1 0-2 The correct answer is "D:' Although each of the listed risk fac­ tors can contribute to hoarseness, the most common cause of chronic hoarseness in children and adolescents is singing or prolonged vocalization. Think about your voice after an intense sporting event where your team won in overtime or concert. Voice trauma can happen to anyone. Question 1 0-3 Vocal cord paresis or paralysis can be a complication of all the following EXCEPT: A) C2 spinal cord transection. B) Tracheal reconstruction. C) Patent ductus arteriosus (PDA) ligation. D) Chiari II malformation. E) Thyroid surgery. Discussion 1 0-3 The correct answer is ''A:' Vocal cord movement is controlled by the vagus nerve, which originates in the brainstem; spe­ cifically, the superior and recurrent laryngeal branches of the vagus nerve. Central nervous system lesions that increase pressure on the brainstem (eg, Chiari II malformation) may result in cranial neuropathies, including vocal cord paresis. An isolated transection of the spinal cord would not affect vocal cord movement. The left recurrent laryngeal nerve courses below the arch of the aorta. This increases its risk of damage during PDA ligation and other cardiac surgeries. Recurrent laryngeal nerves also pass along the trachea and thyroid tissues.

MCG RAW- H I LL E D U CAT I O N S P E C I A LTY BOA R D REVI EW: P E D I AT R I C S

706

A previously healthy I S-year-old college freshman was found unresponsive by his roommate. He recently broke up with his girlfriend. On examination, he was apneic, bradycardic, and pupils were pinpoint. SP02 was 90%. Question 1 1 - 1 Which o f the following venous blood gas values (pH, Pco2, HCO;, base excess) is most likely? A) 7.54, 22, 22, 0. B) 7. 1 , 80, 24, 0. C) 7. 1 , 1 8, 12, - 1 2. D) �35, 76, 35, + 1 Q 0

Discussion 1 1 - 1 Th e correct answer i s "B:' This patient has a n opioid toxidrome with acute respiratory depression. This presentation could also be consistent with alcohol poisoning from binge drinking. Both cause central nervous system depression. The pinpoint pupils (miosis} are suggestive of opioid ingestion. Either way the first step would be stabilization of his cardiorespiratory status. Acute hypoventilation results in a respiratory acidosis with hypercar­ bia (elevated partial pressure of carbon dioxide [Pco 2 ] ) . Option ''I\' is respiratory alkalosis (seen with hyperventilation), option "C" is an anion gap metabolic acidosis, and option "D" is a compensated chronic respiratory acidosis (HCO; and Pco2 are elevated) . (See Table 32-2. ) When interpreting blood gases take a two-step approach. First, determine the primary problem. For example, respiratory acidosis causes a low pH and high Pco 2• (See Table 32-2.) Second, look for compensation. The body tries to keep a neutral status. In respiratory disorders, the kidney will change the bicarbonate (HCO;) level to follow carbon dioxide (Pco) -both will increase or decrease. In metabolic disorders, the lungs change the Pco 2 opposite to the HCO;. Mixed disor­ ders are tough.





1 1 1r

Helpful Tip Oxyg enation i s n ot the s a m e a s venti lation. Venti lation contro l s

carbon

m e a s u re m e nt

of

d i oxide a

(CO) .

normal

A

pulse

hemog l o b i n

oxi metry oxyg e n

satu ration d oes n o t m e a n the C 0 2 level is norma l . Check t h e b l ood gases.

TABLE 32-2 P R I M A RY AC I D-BASE D I SO R D E R S

Respi ratory acidosis Meta bolic acidosis Respi ratory a l kalosis Meta bolic a l kalosis

pH

Pco2

j, j, i i

i

HCO�

2 6 . 8 40

80

Oxygen partial pres s u re

(Po2,

1 20

mm Hg)

F I G U R E 32-7. Oxygen hemog lobin d i ssociation cu rve. (Reprod uced with perm ission from McKean SC, Ross JJ, Dressier DD, Brotman DJ, G i n s berg JS, eds. Principles and Practice of Hospital Medicine. New York, NY: McGraw- H i l l Education; 201 2, F i g . 83-2.)

� QUICKQUIZ Which of the following is false regarding pulse oximetry? A) Continuous and noninvasive measurement of arterial oxy­ gen saturation. B) Normal and abnormal values are well defined. C) Oxygenated hemoglobin and deoxygenated hemoglobin absorb light at different wavelengths. D) Differences between upper and lower extremity oxygen saturation in a newborn may suggest congenital heart disease. E) Does not detect changes in Pco 2• Discussion The correct answer is "B:' Pulse oximetry is an easy way to detect hypoxemia. This "fifth vital sign" estimates arterial hemoglobin saturation (oxygen saturation [Spo 2 ] ) using a probe placed on the finger. In infants, probes can be placed on palms, feet, penis, and other sites. The ratio of deoxyhemoglobin and oxyhemoglo­ bin, each measured by the amount of light absorbed at different wavelengths, is used to estimate saturation. Nothing is perfect. The exact oxygen saturation at which tissue hypoxia occurs is unclear, so absolute normal and abnormal values do not exist. Pulse oximetry does not detect hypoventilation or hyperoxia. The hemoglobin dissociation curve is not linear; therefore, large changes in arterial partial pressure of oxygen (Paoz> may not change in the Spo 2 if it is already near 1 00%. For example, the Pao 2 would have to decrease from 145 to 65 mm Hg before the Spo2 would decrease significantly. (See Figure 32-7. )

j, j, i

A 1 0-year-boy with spinal muscular atrophy type 1 is brought by his parents to the emergency department. He has had a runny nose, low-grade fever, and cough for 3 days. His parents

CHAPTER 32

have performed suctioning and cough assistance at home. In the emergency department, oxygen saturation (Spo2) is 80% on room air with coarse crackles bilaterally on lung exam. A capillary blood gas analysis shows Pco2 of 50 mm Hg.

Spo2

Question 1 2- 1 Which i s the most important factor explaining the patient's hypoxemia? A) Hypoventilation. B) Shunting through an atrial or ventricular septal defect. C) Ventilation-perfusion mismatch. D) Increased physiologic dead space. E) Hemoglobin dissociation curve shifted to the left due to acidosis.



Discussion 1 2- 1 Th e correct answer i s "C:' Th e question implies that there is some degree of lung disease (pneumonia) causing ventilation­ perfusion (V/Q) mismatch as well as hypoventilation (elevated Pco2) from muscle weakness. Distinguishing the relative contri­ butions of each process requires knowledge of both the hemo­ globin dissociation curve and the alveolar gas equation. At Spo2 of 80%, the Pao2 (arterial oxygen level) is estimated to be 50 mm Hg based on the hemoglobin dissociation curve. In hypoxemia, the dissolved oxygen in the blood decreases. To compensate, oxygen is released from hemoglobin, resulting in decreased hemoglobin oxygen saturations. (See Figure 32-7, earlier.) At standard atmospheric pressure with no supplemental oxygen, the alveolar gas equation can be simplified to: PAo2 = 1 50 - Pco/0.8 = 1 50 - 50/0.8 = 1 50 - 62.5 = 87.5 mm Hg per Spo2

50 mm Hg

Normal Pao2

(no hypoventilation, no V/Q mismatch)

1 00 mm Hg

Predicted Pao2

(hypoventilation only)

87.5 mm Hg

The alveolar gas equation estimates the oxygen level at the alve­ oli. In the above example, hypoventilation alone cannot explain the hypoxemia because the actual is lower than that predicted value, meaning something else is contributing. The PAo2 (alveo­ lar partial pressure of oxygen) is used to calculate the alveolar­ arterial gradient (A-a gradient = PAo2 - Pao2). This gradient helps figure out the cause of hypoxemia. When oxygen is not transferred effectively from the alveoli to the blood the A-a gra­ dient is elevated, such as in cases of V/Q mismatch.



1 1 1r

reference

707

Pao2

50% = 27 mm Hg 80% = 50 mm Hg •

I

1 1r

r1J •

Helpful Tip The a lveo l a r g a s equation s i m p l ifies to PAo2 -

=

1 50

Pco/0.8.

Helpful Tip N o r m a l PAo2 is a p proxi mately 1 00 m m H g .

Question 1 2-2 Assuming that there was no V/Q mismatch, what is the highest that the Pco2 could be in this patient (PAo2 = Pao2 = 50 mm Hg)? A) 50 mm Hg. B) 65 mm Hg. C) 80 mm Hg. D) 1 00 mm Hg. E) 120 mm Hg. Discussion 1 2-2 The correct answer is "C:' This can be calculated as follows: PAo2 = 1 50 - Pco/0.8 Pco2 = 0.8 x ( 1 50 - PAo2) = 80 mm Hg Online calculators and smartphone apps are make it so much easier to check these values.

Question 1 2-3 Supportive cares for the patient should include: A) Cough assist. B) Suctioning. C) Bilevel positive airway pressure (BiPAP) . D) Enteral nutrition to prevent muscle catabolism. E) All of the above. Discussion 1 2-3 The correct answer is "E:' Cough assist and suctioning are essential aids for airway clearance to prevent atelectasis. This patient's mus­ cle weakness will make his cough weak and less effective. BiPAP will help maintain normal ventilation. Enteral nutrition is needed to prevent worsening weakness during prolonged periods of fast­ ing and to meet the increased calorie needs during acute illness. •

Helpful Tip

:5.� Conti n u o u s

r1 1r

b i l evel

positive

a i rway

pressu re

(CPAP)

and

positive a i rway pres s u re ( B i PA P) a re both

forms of n o n i nvasive vent i l at i o n . B i P A P g ives positive i n s p i ratory a n d expi ratory pressu re a n d has a backu p

Helpful Tip Quick

R E S P I RATO RY D I SO R D E R S

The patient is admitted to the intensive care unit and pre­ scribed intravenous antibiotics.

Estimated Pao2





est i m a tes

d i ssociation c u rve (see F i g u re

from

the

hemog l o b i n

32-7, earl ier)

rate. It h e l p s with venti lation a n d centra l a p nea. CPAP g ives positive expi ratory pressu re, h e l p i n g keep the a lveo l i open for oxyg enation a n d obstructive a p nea.

MCG RAW- H I LL E D U CAT I O N S P E C I A LTY BOA R D REVI EW: P E D I AT R I C S

708

e

Helpful Tip

� Ve ntilation

=.

i1 1 r

controls the carbon d ioxide l evel i n the

blood. It is eq u a l to the res p i ratory rate m u lt i p l ied by the tid a l vol u m e (RR

x

Vr) of each breath. C h i l d re n

w i t h wea kness or restrictive l u n g d i sease a re rate dependent beca use of low Vr.

An 1 8-year-old girl with cystic fibrosis presents to an emer­ gency department in summer with vomiting, cough, and dehydration. Her best FEV , (forced expiratory volume in 1 second} value in the last year was 60% of predicted. She was competing in a softball tournament when symptoms wors­ ened. Vital signs are as follows: heart rate 1 28 bpm, respira­ tions 28 breaths per minute, BP 1 1 5/70 mm Hg, and Sp02 90% on room air. Examination reveals bilateral inspiratory crack­ les, mild respiratory distress, and delayed capillary refill. The metabolic panel shows Na+ 1 30 mEq/L, K+ 3.3 mEq/L, CI- 88, total HCO; 32, BUN 29, Cr 0.9, and glucose 200. Question 1 3- 1 What i s her primary acid-base disturbance? A) Non-anion gap metabolic acidosis. B) Diabetic ketoacidosis. C) Metabolic alkalosis. D) Chronic respiratory acidosis. E) More diagnostic information is required. Discussion 1 3- 1 Th e correct answer i s "E:' Her cystic fibrosis i s poorly controlled. She has severe airway obstruction as evident by her low FEV , . The primary acid-base disturbance cannot be defined because respiratory acidosis with compensation and metabolic alkalo­ sis are both possible with increased total HCO;. Children and adolescents with cystic fibrosis are at risk of developing hypona­ tremic, hypochloremic, metabolic alkalosis as a result of chlo­ ride loss, especially after significant perspiration (option "C") . Additionally, this patient has evidence o f progressing pulmo­ nary disease {low FEV) and is at risk of developing respiratory acidosis (option "D"). The laboratory data obtained are not con­ sistent with metabolic acidosis (options "1\' and "B") because the total HCO; is increased. Remember: pH 7.45

Respiratory acidosis

Metabolic alkalosis

Metabolic acidosis

Respiratory alkalosis

An 1 8-year-old boy with Duchenne muscular dystrophy comes to your clinic for regular follow-up. He is a college freshman majoring in pre-law. He has no distress on exami­ nation and lung fields are clear bilaterally. He has a forced

vital capacity {FVC} of 45% of predicted and Spo2 of 97% on room air. Capillary blood gas analysis shows pH 7.35, Pco2 50, Hco-3 28, and base excess of 4. Chest X-ray shows osteopenia and restrictive changes, but no significant atelectasis. Question 1 4- 1 What i s the next step i n management? A) Oxygen at night. B) CPAP at night. C) BiPAP at night. D) Acetazolamide. E) Tracheostomy and mechanical ventilation. Discussion 1 4- 1 Th e correct answer i s "C:' Duchene muscular dystrophy i s a progressive disease that results in restrictive lung disease (low PVC) because of decreased ability to open up the lungs resulting from a combination of weak respiratory muscles and acquired scoliosis. The patient described has chronic respiratory failure due to neuromuscular weakness. His Pco2 is elevated with metabolic compensation (elevated HCO;). Nighttime BiPAP will improve tidal volume and ventilation related to neuromus­ cular weakness. CPAP can help overcome upper airway obstruc­ tion but it will not improve hypoventilation related to muscle weakness. Acetazolamide is a carbonic anhydrase inhibitor that can reduce bicarbonate levels but does not help the underlying problem of hypoventilation. Tracheostomy is not necessary and has the potential to interfere with the ability to speak. His oxy­ genation status is fine, with a normal Spo2 •

An 18-year-old boy has acute shortness of breath and left­ sided chest pain with inspiration after being involved in a motor vehicle collision. He is tachypneic, lightheaded, and unable to take deep breaths. Vital signs are temperature 36.5°C {97.7°F), heart rate 140 bpm, respirations 45 breaths per minute, BP 90/50 mm Hg, and Spo2 90%. Chest auscul­ tation reveals absent breath sound on the left side and tym­ panic sounds on percussion. Question 1 5- 1 What i s the next step i n management? A) Needle thoracostomy. B) Chest X-ray. C) Subcutaneous enoxaparin. D) Intravenous heparin. E) Albuterol by continuous nebulizer. Discussion 1 5- 1 Th e correct answer i s "A:' B y clinical examination, the patient has a tension pneumothorax, which is causing respiratory fail­ ure and obstructive shock. The patient should be immediately stabilized by needle thoracostomy, without awaiting further diagnostic studies. Options "C" and "D" allude to the possibility of pulmonary embolism, but there is clear evidence of pneumo­ thorax on examination. (See Figure 32-8.)

CHAPTER 32



R E S P I RATO RY D I SO R D E R S

709

A 4-year-old girl has had a chronic dry cough lasting 3 months that occurs both in the day and at night. She has had two prior acute care visits for the cough and was diagnosed with an upper respiratory infection. She also has had two prior hospitalizations for wheezing illnesses and has atopic dermatitis. Question 1 6- 1 What d o you recommend for the cough? A) Montelukast. B) Bronchoscopy. C) Albuterol as needed. D) Inhaled beclomethasone, albuterol as needed. E) Amoxicillin-clavulanic acid.

F I G U R E 32-8. Tension pneumothorax. On the right side, there is free a i r outside t h e l u ng with flatte n i n g o f t h e d i a p h ra g m . T h e mediasti n u m is sh ifted to the left. (Reproduced with perm ission from McKea n SC, Ross JJ,

Dressier DD, Brotman DJ, G i nsberg JS, eds. Principles and Practice of Hospital Medicine. New York, NY: McGraw- H i l l Education; 20 1 2, Fig. 1 07-7.)

� QUICKQUIZ Which of the following is true regarding tachypnea? A) Equals a respiratory rate greater than 20 for children 5 to 10 years of age. B) Definition varies by age. C) Equals a respiratory rate greater than 50 for newborns. D) Respiratory rate should be counted for a full 30 seconds. E) Just look at the child (who needs to count?) . Discussion The correct answer is "A:' Tachypnea varies by age. (See Table 32-3.) Respiratory rate should be counted over a full minute. Determining the actual respiratory rate is important.

TABLE 32-3 N O RMAL R E S P I RATO RY RATES BY AG E

Age

Normal Respiratory Rate (breaths per m i n ute)

I nfa nt

30-60

Todd le r

25-40

Child

20-30

Adolescent

1 2- 1 6

Discussion 1 6- 1 Th e correct answer i s "D:' Asthma i s a chronic inflammatory disease of the lungs with reversible bronchospasm. Inflamma­ tion means steroids will help. The patient described has persis­ tent or chronic asthma, likely with an allergic basis. Instead of wheezing, cough is her predominant symptom, often referred to as "cough variant" asthma. Albuterol alone would not alter the inflammatory component of the child's asthma, so addition of an inhaled corticosteroid is necessary. Leukotriene recep­ tor antagonists such as montelukast are generally less effective than inhaled corticosteroids. Bronchoscopy may be necessary if there is not an adequate response to a trial of asthma therapy. Amoxicillin-clavulanic helps resolve cough due to protracted bacterial bronchitis, which is usually a wet or productive cough. Question 1 6-2 Which is NOT consistent with persistent asthma? A) Wheezing 3 days or more during the week. B) Waking up at night weekly. C) Never needing albuterol. D) Cannot play tag with friends. E) Decreased FEV 1 on lung function tests. Discussion 1 6-2 The correct answer is "C:' Asthma management has two parts: ( 1 ) making the diagnosis and (2) long-term control. Symptoms must be recurrent-wheezing, cough, shortness of breath, or a combination of these. Symptoms occur at night or worsen with exposure to triggers (viral infections, exercise, irritant expo­ sure, allergen exposure). Once diagnosed, the next step is to determine the severity-intermittent or persistent. If asthma is persistent a daily preventive medication is needed. Remember the rule of 2s to decide if asthma is intermittent or persistent. Asthma is persistent if any of the following are present: ( 1 ) daytime symptoms occur 2 or more days per week, (2) nighttime symptoms occur 2 or more times per month, (3) albuterol is needed 2 or more days per week, and (4) nor­ mal activity is impaired in some way. Lung function tests (FEV 1 ) may also be used. At follow-up visits, asthma control becomes the focus. Finally, do not forget the asthma action plan; ensure

MCG RAW- H I LL E D U CAT I O N S P E C I A LTY BOA R D REVI EW: P E D I AT R I C S

710

that the girl and her family understand the details of the plan, including medication use and when to seek medical care for worsening symptoms.

The girl returns to your office in January. She has had a runny nose, cough, and fever. Over the past 24 hours, breathing has become increasingly difficult. Albuterol helps but only for a short time period and does not completely relieve the symptoms. On exam, her vital signs are heart rate 1 60 bpm, respirations 45 breaths per minute, and Spo2 of 88%. On exam she has retractions, nasal flaring, abdominal breathing, decreased breath sounds, and faint expiratory wheezes. Question 1 7- 1 What should you d o next? A) Administer albuterol. B) Administer oxygen if Spo 2 remains low after albuterol. C) Give systemic steroids. D) Consider hospital admission. E) All of the above. Discussion 1 7- 1 Th e correct answer i s "E:' She i s having a n acute asthma exacerba­ tion. Common triggers include viral respiratory tract infections, exercise, allergen exposure, and smoke exposure. Symptoms that do not respond to bronchodilators are called status asthmaticus. In this child, wheezing is minimal as her air movement is poor. This is a bad sign. With improved air flow, her wheezing may get louder. Albuterol is a beta2 -receptor agonist that causes bron­ chodilation by relaxing the bronchial smooth muscle. It is used for acute symptom relief and improving air flow. The true game changer is systemic steroids. Remember, this is also an inflam­ matory process. Steroids take time to work, so give them early. •



1 1 1r

Helpful Tip A n o r m a l Pco2 on b l ood gas a n a lys i s in a patient with an acute asthma exacerbati on is a s i g n of i m pe n d i n g

hyperinflation. Radiographs show flattened diaphragm, retro­ cardiac air, and a narrow mediastinum as the lungs are overin­ flated like balloons. Option "E" is also correct as a chest X-ray is not typically needed. Radiographs may be indicated in patients with suspected asthma complications like pneumothorax.

A 2-year-old boy has a chronic wet-sounding cough lasting 3 months that occurs both in the day and at night. Systemic steroids and inhaled albuterol have not improved the cough. Last year, he had a longstanding cough that fully resolved when he was being treated for a concurrent ear infection. Question 1 8- 1 What d o you recommend for his cough? A) Montelukast. B) Bronchoscopy. C) Albuterol as needed. D) Inhaled beclomethasone, albuterol as needed. E) Amoxicillin-clavulanic acid. Discussion 1 8- 1 Th e correct answer i s "E:' Th e boy has a chronic cough a s i t has been present for over 1 month. This vignette describes features of protracted bacterial bronchitis, which results from chronic lower airway inflammation and infection by Streptococcus pneumonia, Haemophilus influenzae, or Moraxella catarrhalis. Protracted bacterial bronchitis often occurs in association with tracheomalacia or bronchomalacia. Typically children younger than 5 years of age are affected and have a chronic wet cough but are otherwise well. Symptoms resolved with antibiotic treatment. It is important to rule out other causes of chronic cough, such as cystic fibrosis. (See Table 32-4.) The patient has not responded to albuterol or steroids in the past, so asthma is unlikely. A bronchoscopy may be necessary if response to treat­ ment with amoxicillin-clavulanic acid is not adequate. Consider immunodeficiency evaluation as recurrent sinopulmonary infections is a sign of an underlying immunodeficiency.

fa i l u re. T h e C 0 2 s h o u l d be l ow fro m hyperventilation. N o rm a l iz i n g may be a s i g n of i m pe n d i n g res pi ratory fa i l u re or "wea ring out."

Question 1 7-2 What would be expected on chest X-ray during an asthma exacerbation? A) Widened mediastinum. B) Pleural effusion. C) Underinflated lungs. D) Peribronchial thickening. E) Getting a radiograph is silly. Discussion 1 7-2 The correct answer is "D:' Asthma is an obstructive pulmo­ nary process. The air gets trapped in the lungs leading to

An 1 8-year-old boy who plays soccer feels short of breath dur­ ing both practices and competitions. He has no chest pain, palpitations, wheezing, cough, or stridor. Spirometry and baseline electrocardiogram (ECG) are within normal lim­ its. On treadmill testing, he reaches a maximum heart rate of 200 bpm and his oxygen consumption (VO) at maximal effort is 1 30% of predicted. Oxygen saturations are normal, and dead space ventilation is decreased with exercise. There is no airflow obstruction during or after exercise. An anaero­ bic threshold is detected at 65% of his maximum V02• His shortness of breath is reproduced during the later portions of the study.

CHAPTER 32

TABLE 32-4 CAU S E S OF C H RO N I C CO U G H

Asthma









Exercise i ntolera nce Noctu rnal sym ptoms







Protracted bacterial bronchitis









Psychogenic "ha bit"

Pertussis (whoo p i n g cou g h )

Wet cou g h C h i l d looks wel l You n g c h i l d ren Resolves with a ntibiotics B ra ssy sou n d i n g



Possi ble associated t h roat clearing



An noyi ng to others



Key: not p resent once asleep



Pa roxysm s of d ry cou g h





Known as the 1 00-day coug h Classic i n s p i ratory whoop (not a l ways)



Posttussive emesis



Deficient i m m u n izations





I m p roves over time Anti biotics do not m a ke coug h go away

Mycoplasma pneumoniae Chlamydia pneumoniae I nfl uenza



Poor g rowth



Prod uctive cou g h



Diagnosed b y sweat chloride test



Tracheoma­ lacia

Atelectasis or hyperi nflation in one a rea on chest X-ray

M a y fol l ow a vira l U RI



I m m u nodefi­ ciency

Asymmetric breath sounds on exa m (not a lways)





Cystic fi brosis

Resolves with a l b uterol a n d steroids H i story of choking episode (not always)





Prolonged infection

Personal or fa m i ly h i story of atopy Recurrent epi sodes triggered by vira l U Ri s





Foreign body

Most com mon

Can be detected on newborn metabol ic screen i n g



Recurrent s i n o p u l monary infections



Atypica l or severe infections



Poor g rowth



Sym ptoms may be worse s u pine





Hypoxe m i a and obstructive a pnea may occ u r Monophonic wheeze on exa m

U RI, u pper respi ratory tract infection.



R E S P I RATO RY D I SO R D E R S

71 1

Question 1 9- 1 Which o f the following explains his shortness o f breath? A) Metabolic acidosis. B) Pulmonary vascular disease. C) Interstitial lung disease. D) Vocal cord dysfunction. E) Exercise-induced asthma. F) Mitral insufficiency. Discussion 1 9- 1 Th e correct answer i s "A:' Th e athlete described i n this vignette has above-average performance on exercise testing and reached his physiologic limitation. Exercise above the anaerobic thresh­ old causes a lactic acidosis, which intensifies the feeling of shortness of breath. Pulmonary vascular disease and interstitial lung diseases should cause hypoxemia. Dead space normally decreases during exercise due to larger tidal volumes (deep breaths) . Exercise-induced asthma and vocal cord dysfunction are ruled out by the lack of airway obstruction on inspiration and exhalation. Physiologically significant mitral insufficiency would decrease vo2 relative to the heart rate and may reduce oxygen saturation. Basically, he is in great shape but must accept he may not be professional athlete material.

A 16-year-old female runner feels short of breath during competitions. A loud inspiratory noise has caused her to miss several track meets. She has no chest pain, palpitations, or wheezing. Spirometry and baseline electrocardiogram (ECG) are within normal limits. On treadmill testing, she reaches a maximum heart rate of 1 87 bpm and her V02 at maximal effort is 1 00% of predicted. Inspiratory flow limita­ tion and stridor occur near maximal effort. Question 20-1 What is the most likely diagnosis? A) Pulmonary hypertension. B) Interstitial lung disease. C) Vocal cord dysfunction. D) Exercise-induced asthma. E) Cardiomyopathy. Discussion 20- 1 Th e correct answer i s "C:' This athlete most likely suffers from vocal cord dysfunction triggered by exercise (a well-known trig­ ger). The vocal cords adduct with inspiration when they should not, causing chest or throat tightness, cough, dyspnea, and stri­ dor. The condition is often misdiagnosed as asthma. Girls are predominately affected, and academic, social, or athletic stress­ ors are typically present. The presentation may be dramatic, resulting in unnecessary invasive procedures such as emergency tracheostomy. Laryngoscopy can confirm the diagnosis. Speech therapy and patient education are the mainstays of treatment. (Did you know this condition was first called "hysteric croup;' reflecting the thought that a woman in a fit of hysteria could cause the dysfunction?)

71 2

MCG RAW- H I LL E D U CAT I O N S P E C I A LTY BOA R D REVI EW: P E D I AT R I C S

A 18-year-old boy is undergoing treatment for Hodgkin lymphoma. He is an avid tennis player but has diminished exercise tolerance. He informs you that he has a higher than normal resting heart rate and that he reaches a maximum heart rate of 200 bpm during vigorous exercise. Question 2 1 - 1 Which of the following is NOT consistent with his heart response during exercise? A) Large airway obstruction related to mediastinal lymphadenopathy. B) Valvular heart disease. C) Anemia. D) Deconditioning. E) Peripheral myopathy related to chemotherapy. Discussion 2 1 - 1 Th e correct answer i s ''A:' Patients with a pulmonary limitation to exercise become short of breath before they can achieve a high heart rate. The maximum heart rate expected is approxi­ mately 220 bpm minus age in years. The other causes listed would not be expected to impair heart rate response to exercise. Anemia, valvular heart disease, and deconditioning cause less oxygen to be delivered per heartbeat. Peripheral myopathy may decrease the efficiency of work, but should not affect the heart rate response.

� QUICKQUlZ Which of the following disorders is more strongly associated with obstructive sleep apnea compared with central sleep apnea? A) Prematurity. B) Opioid mediations. C) PHOX2B mutations. D) Heart failure. E) Congenital hypothyroidism. F) Pierre Robin sequence.

f lflfl flflJU li n JlJU

Question 22-1 Which of the following is present? A) Obstructive apnea. B) Central apnea. C) Sighing respiration. D) Desaturation. E) None of the above; the waveforms are normal.

Discussion 22-1 The correct answer is ''A:' No C0 2 is exhaled on the second and third breaths, despite evidence of respiratory effort. This implies obstructive apnea. Central apnea would be characterized by concurrent lack of respiratory effort. Ask if the chest is mov­ ing when the child or infant is not breathing. In patients with obstruction, the brain is telling the body to breath, so the chest is moving but the air does not flow.

A full-term neonate has recurrent apneic spells without noisy breathing or other symptoms of obstruction. Question 23-1 Which of the following tests should you obtain first? A) EEG. B) Bronchoscopy. C) Capillary blood gas analysis. D) Polysomnogram. E) Esophageal pH study.

Discussion The correct answer is "F:' Pierre Robin sequence (small jaw, pos­ teriorly displaced tongue, cleft palate) results in airway obstruc­ tion. When the brain does not tell the body to breath central apnea occurs. Prematurity, central nervous system depressants, heart failure, PHOX2B mutations, and congenital hypothyroid­ ism are all associated with central sleep apnea. PHOX2B muta­ tions cause congenital central hypoventilation syndrome (the so-called Ondine curse) .

Discussion 23-1 The correct answer is "C:' A basic laboratory evaluation should begin prior to ordering expensive tests. Possible reasons for apnea could include infection, metabolic disorders, or impaired control of breathing. A capillary blood gas analysis can be obtained more quickly than the other studies and may indicate whether there is an acid-base disorder or impaired regulation of ventilation. Bronchoscopy would be useful in the evaluation of suspected airway obstruction, but this infant's history sug­ gests central apnea. Polysomnography would allow for defini­ tive diagnosis of either central or obstructive sleep apnea.

The waveforms below represent a 20-second interval obtained during monitoring of a hospitalized patient.

A 6-year-old boy has obstructive sleep apnea (OSA) with an apnea-hypopnea index (AHI) of 10. His parents are

CHAPTER 32

concerned that his classroom performance may be affected by his sleep apnea. Question 24- 1 You advise: A) Adenotonsillectomy is likely to lower his AHI. B) Adenotonsillectomy will improve his neurocognitive performance. C) His sleep apnea will not improve spontaneously with watchful waiting. D) CPAP will improve neurocognitive performance. E) None of the above. Discussion 24- 1 The correct answer is ''A:' In OSA, the upper airway becomes obstructed during sleep. Sleep is disrupted, which may lead to behavioral problems. Signs and symptoms of OSA are listed in Table 32-5. A randomized controlled trial of adenotonsillectomy versus watchful waiting showed that adenotonsillectomy does decrease AHI on average, but this was not associated with an improvement in performance on a neurocognitive testing. In many children, there was spontaneous improvement in sleep apnea. •

� 1 1 1r

Helpful Tip The a p nea - hypo pnea i n d ex (A HI) i s dete r m i ned d u ri n g a s l eep study. It is calcu lated as the tota l n u m ber of episodes of a pnea a n d hypopnea d ivided by the h o u rs of s l eep. An A H I of g reater t h a n 1 episode per h o u r of sleep is consistent with OSA.

� QUICKQUIZ How is OSA diagnosed? A) History alone. B) History and tonsillar hypertrophy on exam. C) Polysomnography. D) Body mass index. E) Family members need ear plugs to sleep in the child's vicinity. TABLE 32-5 S I G N S A N D SYM PTO M S O F

O B STRUCTIVE S L E E P A P N EA

Sym ptoms

Signs

Freq uent snoring (� 3 n i g hts/week)

E n l a rged ton s i l s

Gasping breaths

Facial deform ities such a s m icrog nathia

Sleep e n u resis

Fa i l u re to th rive

Daytime sleepi ness

U n derweight

Headaches

Overwe i g ht or obese

Lea rning d ifficu lties

P u l m o n a ry hypertension

Hyperactivity/atten­ tion problems



R E S P I RATO RY D I SO R D E R S

71 3

Discussion The correct answer is "C:' History and physical exam are good screening tools. Snoring especially with viral respiratory infec­ tions is normal and common. The key is frequent (� 3 nights/ week) snoring and other signs and symptoms. The exam may be normal. When it comes to the tonsils, size does not matter! If OSA is suspected, the gold standard test is a sleep study (poly­ somnography), which will decide if OSA is present and if so how severe it is. Question 24-2 Which of the following ventilator settings is most appropri­ ate for a child who has central sleep apnea? (CPAP is con­ tinuous positive airway pressure, IPAP is inspiratory positive airway pressure, EPAP is expiratory positive airway pressure, rate refers to respiratory rate, and VT is tidal volume. CPAP, IPAP, and EPAP are measured as em H20.) A) CPAP 6. B) IPAP 1 4, EPAP 5. C) IPAP 1 4, EPAP 5, with 2 L/min of oxygen (O J D) IPAP 1 4, EPAP 5, rate 20. E) Volume control, VT 1 0 mL!kg of body weight, high pressure limit 20 em H2 0. Discussion 24-2 The correct answer is "D:' This is the only selection that has a mandatory rate. In central apnea, a rate is needed to guaran­ tee the child takes the needed number of breaths. The machine has to take over for the brain's dysfunctional respiratory drive. CPAP will not supply a rate, and BiPAP alone (options "B" and "C") relies on the patient's respiratory effort (ie, self-generated respiratory rate) to trigger the ventilator. Option "E" uses vol­ ume control, which would be more affected by leaks or discon­ nection. Moreover, there is also no rate given for choice E.

An 1 8-year-old girl with cystic fibrosis (FEV , 50% of pre­ dicted value) has bronchiectasis involving the right and left upper lobes. Question 25-1 Which of the following organisms is the most likely to be chronically colonizing her airways? A) Staphylococcus aureus. B) Pseudomonas aeruginosa. C) Burkholderia cepacia. D) Achromobacter xylosoxidans. E) Stenotrophomonas maltophilia. Discussion 25-1 The correct answer is "B:' Cystic fibrosis ( CF) is an autosomal recessive genetic condition resulting in dysfunction of the cystic fibrosis transmembrane regulator ( CFTR) protein. The CFTR protein controls the chloride channels in the lungs, pancreas, intestine, and skin. Secretions become thick and are difficult to

714

MCG RAW- H I LL E D U CAT I O N S P E C I A LTY BOA R D REVI EW: P E D I AT R I C S

clear. In the lungs, this leads to chronic infection, inflammation, and progressive airway obstruction (low FEV J In CF, the lungs are colonized with weird bacteria (all the listed options) that are specific to CF. By age 1 8 years, the most common CF airway pathogen is P. aeruginosa. In early childhood, S. aureus is more common.

Question 25-2 Which of the following pathogens is NOT commonly identi­ fied in sputum from patients with bronchiectasis due to cys­ tic fibrosis? A) Staphylococcus aureus. B) Pseudomonas aeruginosa. C) Burkholderia cenocepacia. D) Neisseria meningitidis. E) Stenotrophomonas maltophilia. F) Aspergillus fumigatus. G) Mycobacterium abscessus. Discussion 25-2 The correct answer is "D:' It probably is unfair to list seven instead of five choices. Consider it a test to see if you are pay­ ing attention. N. meningitidis would not routinely be isolated from CF sputum. The other species listed are regularly seen in CF sputum samples. Redflag alert: Finding these bacteria in the lungs is not normal. It means that some underlying disease pro­ cess is present. Think CF, ventilator-associated pneumonia, or immunodeficiency. Utilize those health care dollars and order diagnostic tests ! Question 25-3 CF lung disease is characterized by all of the following radio­ logic abnormalities EXCEPT: A) Bronchial wall thickening. B) Air trapping. C) Atelectasis. D) Bronchiectasis. E) Pleural effusion. Discussion 25-3 The correct answer is "E:' CF is a progressive disease. Eventu­ ally the airways and lungs are damaged and respiratory failure occurs. Early CF disease primarily affects conducting airways. Mucus plugging may result in focal areas of atelectasis and air trapping (hyperinflation). Inflammation can lead to bronchial wall thickening and bronchiectasis. Pneumothorax and pneu­ momediastinum can occur.

A 1 7-year-old boy with CF (FEV1 30% of predicted value) is hospitalized for pulmonary exacerbation. He produces over 200 mL of bright red blood after coughing.

Question 26- 1 What is the most likely source of the blood? A) Alveolar hemorrhage. B) Branch pulmonary artery. C) Bronchial artery. D) Submucosal vessels. E) Esophageal varices. Discussion 26- 1 The correct answer is "C:' CF patients with bronchiectasis are at risk of hemoptysis from hemorrhage of dilated bronchial arter­ ies. This can be life threatening because ( 1 ) the bleeding is rapid, owing to the bronchial arteries being under systemic pressure, and (2) there is the possibility for airway compromise. Blood pouring into your airways is not good. Alveolar hemorrhage would be more characteristic of idiopathic pulmonary hemosid­ erosis. Esophageal variceal bleeding presenting as hematemesis can also occur in patients with CF as a result of portal hyperten­ sion secondary to CF-associated liver disease. Question 26-2 Management of this patient's hemoptysis should include: A) Correction of vitamin K deficit. B) Discontinuation of ibuprofen. C) Consultation of interventional radiology. D) Temporarily ceasing chest physiotherapy. E) All of the above. Discussion 26-2 The correct answer is "E:' Correcting any underlying coagulopa­ thy is essential. Bronchial artery embolization by interventional radiology may be required if there is life-threatening hemopty­ sis. Obviously giving a medication (NSAID) that interferes with platelet function is not a good idea when a patient is hemor­ rhaging, nor is shaking and pounding on the chest.

A 14-year-old girl with CF (FEV1 70% of predicted value) is hospitalized for pulmonary exacerbation with methicillin­ resistant Staphylococcus au reus (MRSA) and mucoid Pseudo­ monas aeruginosa. She is receiving vancomycin, tobramycin, trimethoprim-sulfamethoxazole, and ceftazidime. During treatment, she develops severe abdominal pain, distension, and fever. Examination reveals tachycardia and a diffusely firm and tender abdomen with rebound tenderness. Question 27-1 Which of the following complications is most likely? A) Clostridium diffi cile colitis. B) Distal intestinal obstruction syndrome. C) Fibrosing colonopathy. D) Appendicitis. E) Nephrolithiasis.

CHAPTER 32

Discussion 27- 1 The correct answer is "A:' Patients with CF commonly receive broad-spectrum antibiotics and run a higher than ordinary risk of C. diificile colitis. In CF, pulmonary exacerbations character­ ized by worsening symptoms and decrease in pulmonary func­ tion on testing occur. Patients may be hospitalized depending on the severity. Treatment includes aggressive therapies such as chest physiotherapy to increase mucociliary clearance and sys­ temic antibiotics.

� QUICKQUIZ Which of the following is a manifestation of CF? A) Rectal prolapse. B) Nasal polyps. C) Diabetes. D) Intestinal obstruction. E) All of the above.



R E S P I RATO RY D I SO R D E R S

71 5

TABLE 32-6 C L I N I C A L FEAT U R E S OF CYSTI C

F I B RO S I S

Respi ratory •

C h ronic cou g h (prod uctive)



C h ronic respi ratory i nfections



Nasal polyps



C h ronic pansi n u sitis



C l u bbing



Pneumothorax



Pneumomediasti n u m



P u l monary hemorrhage

I ntesti n a l

Discussion The correct answer is "E:' The hallmarks of CF are chronic obstructive lung disease, pancreatic insufficiency, and salty sweat, but there are a multitude of other manifestations. Options ''I\' and "B" should make you wonder about CF. In CF, the pan­ creatic duct is plugged, resulting in fat malabsorption, diarrhea, and vitamin deficiencies (ADEK) . Endocrine function can be impaired, resulting in CF-related diabetes mellitus. Intestinal obstruction may occur in newborns as meconium ileus or in older children as distal intestinal obstructive syndrome. Sodium and chloride are lost through sweat, which may cause hypona­ tremic, hypochloremic metabolic alkalosis. Clinical manifes­ tations, especially if pancreatic function is normal, may not present until adulthood. (See Table 32-6.)

A patient of yours recently became pregnant. Her older brother has CF. The father of the child is Caucasian. Question 28-1 What is the probability that her child will carry at least one CFTR mutation? A) 1 in 3 . B) 1 i n 4 . C) 1 i n 2 5 . D) 1 i n 75. E) 1 in 2500. Discussion 28- 1 The correct answer is "A:' Because your patient does not have CF, the only remaining possibilities are wild-type (normal CFTR protein present) or carrier. Therefore, there is a 2 in 3 chance she is a carrier (see the Punnett square below) . Of the children without CF, two will be carriers and one will not. There is a 50% chance this mutation will be passed to her child. Thus, 2 the chance that her child will be a carrier is /3 x 'l2 1 /3 ' =



Mecon i u m i leus



Distal i ntesti n a l obstruction syn d rome (DIOS)



Recta l prola pse



B i l i a ry ci rrhosis



C i rrhosis with porta l hypertension



M a l a bsorption



G reasy, fou l-sme l l i ng stool s



Fat-sol u ble vita m i n deficiency



Pa ncreatitis

Other •

Dia betes or hyperg lyce m i a



Fa i l u re t o th rive



Hyponatre m i c, hypoch lore m i c meta bol i c a l kalosis



M a l e i nferti l ity (absent vas deferens, normal sperm prod uction)

Father ... Ql ..c ...

::E

0

CFTR+

CFTR-

C FTR+

Wi ld-type (normal)

Carrier

C FTR-

Carrier

CF

Question 28-2 What is the probability that the child will have CF? A) 1 in 3. B) 1 in 4. C) 1 in 75. D) 1 in 1 50. E) 1 in 2500. Discussion 28-2 The correct answer is "D:' The allele frequency of CFTR muta­ tions in the Caucasian population is approximately 1 in 25. There is a 1 in 3 chance the child will inherit a mutation from the mother (see Question 28- 1 ) . There is a 1 in 25 chance the father is a carrier. There is a 1 in 2 chance a paternal mutation

716

MCG RAW- H I LL E D U CAT I O N S P E C I A LTY BOA R D REVI EW: P E D I AT R I C S

would be transmitted to the child. Therefore, the probability the child will have CF is P 1 /3 x 1 /25 x ¥2 1 / 1 50 • =

=

A newborn child is identified by newborn screen as having an elevated immunoreactive trypsinogen level of 70 nanogram (ng)/mL and a single copy of CFTR-L1F508 mutation. Question 29-1 What is the next step? A) Prescribe pancreatic enzymes. B) Order full sequencing of CFTR. C) Start fat-soluble vitamin supplementation. D) Perform diagnostic sweat chloride testing. E) Reassure the family that a false-positive newborn screen is common. Discussion 29- 1 The correct answer is "D:' Infants with CF will typically have elevated blood levels of immunoreactive trypsinogen (IRT). If elevated, the next step is to test for CF gene mutations. If IRT is elevated and mutations are present, sweat chloride testing is done. If sweat chloride is 60 mEq/L or greater on two samples, the diagnosis is confirmed. Pancreatic enzymes and fat -soluble vitamins may not be necessary even in a patient with CF if there is enough residual CFTR function to permit pancreatic suffi­ ciency. Although there are false-positive results of newborn screening, it is not appropriate to reassure the parents until more information is known.

An asymptomatic newborn child is identified as having two del­ eterious CFTR mutations, L1F508 and Q890X. However, sweat chloride values are not diagnostic for CF (28 and 32 mEq/L). Question 30-1 Which of the following tests may explain this apparent discrepancy? A) Repeat sweat testing. B) Repeat genotype of the child. C) Genotype parents of the child for these mutations. D) Perform nasal potential difference testing. E) None of the above. Discussion 30-1 The correct answer is "C:' Many different mutations of the CFTR genes exist. The two CFTR mutations are either cis (on the same chromosome) or trans (on opposite chromosomes). If the mutations are in trans, then the patient would likely have CF. To distinguish between these possibilities, genotyping of the parents should be done. If one parent possesses both mutations and does not have CF, then the two mutations exist on the same

chromosome. To have CF, the mutations must be on different chromosomes (trans). Ifboth mutations are on the same chromo­ some (cis), the patient is essentially a CF carrier. Clear as mud?

Question 30-2 Which of the following gastrointestinal complications of CF is least likely to occur in infancy? A) Direct hyperbilirubinemia. B) Fibrosing colonopathy. C) Malabsorption. D) Growth failure. E) Intestinal obstruction. Discussion 30-2 The correct answer is "B:' Fibrosing colonopathy can occur if pancreatic enzyme therapy is dosed at inappropriately high lev­ els. This would not be expected in the infantile period. All of the other disorders are complications that can be attributed to loss of CFTR protein in hepatobiliary, pancreatic duct, or intestinal epithelial cells. Question 30-3 Which of the following adulthood complications of CF does NOT occur as a consequence of antibiotic therapy? A) Male infertility. B) Hearing loss. C) Chronic renal failure. D) Drug-resistant microorganisms. E) All of the above. Discussion 30-3 The correct answer is ''A:' Male infertility is secondary to bilat­ eral obstruction and obliteration of the vas deferens during fetal development. Spermatogenesis is normal. Hearing loss, chronic renal failure, and drug-resistant microorganisms all occur as consequences of antibiotic therapy. Question 30-4 Which of the following is characteristic of primary ciliary dyskinesia (immotile-cilia syndrome) but NOT CF? A) Bronchiectasis. B) Impaired mucociliary transport. C) Diminished fertility. D) Decreased nasal nitric oxide. E) Recurrent sinopulmonary infections. Discussion 30-4 The correct answer is "D:' Nasal nitric oxide is a useful non­ invasive screening test for primary ciliary dyskinesia (PCD), because patients with PCD have abnormally low exhaled nitric oxide. This is not true of patients with CF. Both CF and PCD predispose patients to bronchiectasis, abnormal mucociliary transport, decreased fertility, and recurrent sinopulmonary infections. The upper and lower respiratory tract epithelium is lined with small hairlike organelles called cilia. Cilia beat to help clear the airways (mucociliary clearance) . In PCD, the person

CHAPTER 32

is born with defective cilia. Impaired sperm motility and ovum transit down the fallopian tubes can occur causing infertility. Both CF patients and those with PCD share the impaired ability to clear respiratory secretions; therefore, clinical manifestations overlap. But unlike patients with CF, those with PCD commonly present in the newborn period. •

Helpful Tip

::.::"11.. Ka rta g e n e r

i1 1r

synd rome, a s u b g ro u p of PCD, res u lts

in the triad of situs i nversus, c h ro n i c s i n u sitis, a n d bronch iectasis.

Question 30-5 Which of the following is characteristic of CF but NOT pri­ mary ciliary dyskinesia? A) Mutations are linked to a single gene. B) Situs inversus. C) Abnormal microtubule structure. D) Susceptibility to hydrocephalus. E) Hypochloremic metabolic acidosis. Discussion 30-5 The correct answer is "A:' CF is caused by mutations in a single gene ( CFTR). PCD may be linked to hundreds of different genes that encode ciliary structures. PCD is associated with hydro­ cephalus and lateralization defects such as situs inversus (heart and abdominal organs are reversed, transposed, or duplicated) . Patients with CF have chloride loss and are at increased risk of hypochloremic metabolic alkalosis.

� QUICKQUIZ Which of the following causes of pulmonary hypertension is most likely to be alleviated by nighttime ventilation? A) Sickle cell disease. B) Restrictive lung disease from thoracic deformity. C) ANCA-positive vasculitis. D) Eisenmenger syndrome. E) Chronic thromboembolic disease. Discussion The correct answer is "B:' Restrictive lung diseases caused by thoracic deformity can result in nighttime hypoventilation, which can then lead to pulmonary hypertension. The other dis­ eases listed all can cause pulmonary hypertension, but do not have hypoventilation as a pathophysiologic mechanism.

� QUICKQUIZ Which of the following is most consistent with our current understanding of congenital airway malformations? A) Potter sequence-arrest in the development of bronchial tree.



R E S P I RATO RY D I SO R D E R S

71 7

B) Extralobar pulmonary sequestration-accessory bud from the foregut gives rise to lung tissue not connected to the tra­ cheobronchial tree and receives its blood supply from the systemic circulation. C) Congenital pulmonary airway malformation-renal agen­ esis leads to oligohydramnios and pulmonary hypoplasia. D) Congenital lobar emphysema-developing bronchus sepa­ rates to form a noncommunicating cyst. E) Bronchogenic cyst-partial bronchial obstruction, creating ball valve effect.

Discussion The correct answer is "B:' Pulmonary sequestrations do arise from accessory buds from the foregut. The Potter sequence arises from renal agenesis and oligohydramnios, causing pul­ monary hypoplasia and flattened facies. Congenital pulmonary airway malformations ( CCAM) are thought to occur due to arrested development of the bronchial tree. Congenital lobar emphysema develops as a result of partial obstruction, causing lobar hyperinflation. Bronchogenic cysts occur when a develop­ ing bronchus separates to form a cyst.

A child who recently underwent surgery for congenital heart disease develops a left-sided pleural effusion. The color is white, the cell count is predominantly lymphocytes, and there is a high concentration of cholesterol. Question 3 1 - 1 Why is there an effusion? A) Transudative fluid movement due to elevated pulmonary venous pressure. B) Transudative fluid movement due to protein-losing enteropathy. C) Transudative fluid movement due to malnutrition. D) Exudative process due to infection and capillary leak. E) Chylous effusion due to injury of the thoracic duct. Discussion 3 1 - 1 The correct answer is "E:' The patient has a chylothorax, which is a common complication of surgery to repair congenital heart defects. The thoracic duct is injured, spilling lymph fluid (chyle) from the intestines into the chest. Chyle contains lymphocytes and is milky white from the high concentration of triglycerides. Test the fluid for triglycerides! Not allowing the patient to eat should decrease the effusion. If the patient is eating, the diet should be low fat and long-chain triglycerides (LCTs) should be excluded. Medium-chain triglycerides (MCTs), unlike LCTs, are directly absorbed in the bloodstream rather than transported by the lymphatics. Infant formulas high in MCTs and low in LCTs are commercially available; an alternative is to supplement with MCT oil.

MCG RAW- H I LL E D U CAT I O N S P E C I A LTY BOA R D REVI EW: P E D I AT R I C S

718

� QUICKQUIZ All of the following are risk factors for sudden infant death syndrome (SIDS) EXCEPT: A) Prematurity. B) Smoke exposure. C) Sleeping on a couch or in the parents' bed. D) Sleeping in the supine position. E) Cushioned pads lining the infant's crib. Discussion The correct answer is "D." SIDS is defined as the death of an infant that cannot be explained after examination of the death scene and autopsy. Since institution of "Back to Sleep" recommendations, the incidence of SIDS has decreased in the United States. Prematurity, smoke exposure, and unsafe sleep conditions have all been implicated as risk factors for infant death. Sleeping in baby slings, swings, and or car seats is not safe. Breastfeeding and pacifier use is protective. (Go, binkies ! ) •



1 1 1r

Helpful Tip S l e e p i n g s u p i n e to p reve nt S I DS d o e s not i n crease the risk of choking a n d a s p i ration i n i nfa nts, even those with ga stroesophageal refl ux. E l evation of the head

Question 32-1 A patient takes a full deep breath and exhales as quickly as possible until she can no longer exhale. The amount of gas exhaled is the: A) Forced vital capacity. B) Expiratory reserve volume. C) Forced expiratory volume. D) Functional residual capacity. E) Total lung capacity. Discussion 32-1 The correct answer is "A:' Forced vital capacity (FVC) is the total volume of exhaled air. Spirometry measures flow and volume of air during a rapid, forceful, and maximum exhaled breath from total lung capacity (TLC) to residual volume (RV) . These measurements are plotted to give a flow-volume loop. (See Figure 32-9.) Question 32-2 After breathing comfortably, a patient inhales a full deep breath until he can no longer inhale. The amount of gas inhaled is the: A) Inspiratory capacity. B) Inspiratory reserve volume. C) Peak inspiratory flow rate. D) Tidal volume. E) Total lung capacity.

of bed does not decrease refl ux i n i nfa nts and i s not

Discussion 32-2 The correct answer is "A:' Inspiratory capacity is the amount of air that can be inhaled after normal expiration.

reco m m e n d e d .

You are demonstrating pulmonary function tests to a group of medical students.

Question 32-3 The amount of gas contained by the lungs and airways after exhalation during normal tidal breathing is the: 8 7

1

6

2:

- - - -

T

VT - - - - - 1- -

E Rv _ _ _ _ _ _

j

_ _ _ _ _ _ _ _ _ ,

TLC

FVC

5

Q)

E 4 ::::>

0 >

3 2

0

0

2

3

4

Time (sec) F I G U R E 32-9. P u l m o n a ry fu nction tests. Pul monary fu nction tests. ERV, expi ratory reserve vol u m e; FEF 2 5-75%, forced expi ratory flow measured d u ri n g

exh a l ation of 25% t o 75% o f the FVC; F EV 1 , forced expi ratory vol u m e i n 1 second; FRC, fu nctional reserve capacity; FVC, forced vita l capacity; I C , i n s p i ratory capacity; RV, residual vol u m e; TLC, tota l l u ng capacity; VC, vita l capacity; v,, tida l vo l u me. (Reproduced with perm ission from Stern SOC, Cifu AS, Altkorn D, eds. Symptom to Diagnosis: An Evidence-Based Guide. 3 rd ed. New York, NY: McGraw- H i l l Education; 201 5, Fig. 33-2.)

CHAPTER 32

A) B) C) D) E)

Residual volume. Total lung capacity. Functional residual capacity. Expiratory reserve volume. Peak expiratory flow rate.



R E S P I RATO RY D I SO R D E R S

719

TABLE 32-7 D I F F E R E N T I AT I N G A I R T RA P P I N G

F ROM RESTRICTIVE C HA N G E S

Discussion 32-3 The correct answer is "C:' Functional residual capacity (FRC) is the amount of air left in the lungs after exhaling a normal breath (tidal volume) .

Pa ra meter

Restrictive Disease

Air Trapping (Obstructive)

TLC

Decreased

Normal or i n c reased

FRC

Decreased

I ncreased

FVC

Decreased

Decreased

F RC, fu nctional res i d u a l capacity; FVC, forced vita l capacity; TLC, tota l l u ng capacity.

A 1 5-year-old patient with CF has had progressive worsening of pulmonary function since age 8. Currently, FEV 1 is 60% of predicted and FVC 70% of predicted values. Age

FEV 1 % Predicted

piece, the FRC is calculated. When air trapping occurs, gas dif­ fusion underestimates the FRC.

FVC % Predicted

8 years

1 00

1 00

1 5 years

60

70

Question 33-1 Why is the FVC diminished? A) Weakness related to acute illness. B) Chest wall deformity. C) Air trapping from mucus plugs. D) Fibrotic changes to lung parenchyma. E) All of the above are possible; the mechanism cannot be determined with the information given. Discussion 33-1 The correct answer is "E:' Options "/\_' through "D" are all pos­ sible explanations for a decline in vital capacity. This patient is not exhaling a large ( J- FVC) or fast ( J- FEV) breath. To suck in a big breath then blow it out, you need strong respiratory muscles as well as a chest that expands normally. The air must be able to flow freely in and out. Finally, the volume is dependent on what the lungs can hold. Less lung tissue such as in fibrosis means less air volume. Question 3 3-2 Which test would best distinguish between these possibilities? A) Chest X-ray. B) Whole body plethysmography. C) Repeat spirometry following bronchodilator. D) Arterial blood gas analysis. E) Gas diffusion testing. Discussion 3 3-2 The correct answer is "B:' Whole body plethysmography would be most informative because it can distinguish between air trap­ ping and restrictive changes. (See Table 32-7.) The entire child is placed in a plethysmograph or box that looks like a phone booth creating a closed system. Breathing through a mouth

For Questions 34- 1 through 34-5, refer to the figure below.

..J

ai E E ::>

g B

Volume, L

D

Time, s

Question 34- 1 Which of the following relationships is correct? A) FEV1 - C. B) PEFR - E. C) FVC - A. D) FVC - F. E) FEV1 - D. Discussion 34- 1 The correct answer is "D:' The volume that can be forcefully exhaled in a complete breath is the forced vital capacity ( FVC). Option "/\_' is incorrect because point C is a flow, not a volume. Option "B" is incorrect because point E is a volume, not a flow. Option "C" is incorrect because point A is a flow, not a volume. Option "E" is incorrect because point D labels a time, not a volume. •

Helpful Tip

-� If you a re confused by p u l m o n a ry fu nction tests, look

r1 1 r

at the u n its on fi g u res to g et hel pfu l c l ues. P u l m o na ry fu nction para m eters that end in "ca pacity" or "vo l u me" wi l l have u n its of vol u m e (L). P u l m o n a ry fu nction tests that end in "fl ow" or "fl ow rate" (eg, FEF 50, FEF 75, FEF 25_75, P E F R) wi l l have u n its of vo l u m e d ivided by ti me, such as L/s.

720

MCG RAW- H I LL E D U CAT I O N S P E C I A LTY BOA R D REVI EW: P E D I AT R I C S

Question 34-2 Which of the following points represents FVC? A) Point A. B) Point B. C) Point C. D) Point D. E) Point E.

a)

§

0 >

Discussion 34-2 The correct answer is "B:' The maximum volume exhaled in one breath is the FVC. Question 34-3 Which of the following points represents FEV 1 ? A) Point A. B) Point B. C) Point C. D) Point D. E) Point E. Discussion 34-3 The correct answer is "E:' FEV 1 is the volume that can be force­ fully exhaled in 1 second. Forced expiratory flow (FEF2 5 _75 ) mea­ sures airflow over the middle half of the FVC from 25% to 75% of the exhaled volume of air. Both FEV 1 and FEF 2 5 _75 are reduced in obstructive lung disorders, but FEF2 5 _75 is a sensitive indicator of small airway obstruction. Question 34-4 Which point is the PEFR? A) Point A. B) Point B. C) Point C. D) Point D. E) Point E. Discussion 34-4 The correct answer is "A:' Point A is the maximum flow or peak expiratory flow rate (PEFR) . Question 34-5 Point F measures the same parameter as point: A) Point A. B) Point B. C) Point C. D) Point D. E) Point E. Discussion 34-5 The correct answer is "B:' Points B and F are both representa­ tions of the FVC.

A medical student draws the following graphic in an attempt to illustrate the relationships between several measures of lung function.

c -t--�--"----�""-------\------,f--

Time

Question 35-1 Which of the lung volume relationships shown in the figure is INCORRECT?

Parameter

Volume

A)

Functional residual capacity (FRC)

c

B)

Total lung capacity (TLC)

A

C)

Inspiratory capacity (I C)

A-C

D)

Forced vital capacity (FVC)

A-D

E)

Expiratory reserve volume (ERV)

B-D

F)

Residual volume (RV)

D

G)

Tidal volume (VT)

B-C

Discussion 35-1 The correct answer is "E:' The expiratory reserve volume is the difference between FRC (point C, volume at the end of tidal exhalation) and RV (point D, the end of full exhalation) (see Figure 32-9, earlier.)

� QUICKQUIZ Which factor has the greatest effect on minute respiratory rate in normal individuals? A) Hypoxemia. B) Acidemia. C) Fever. D) Anemia. E) Dietary carbon source. Discussion The correct answer is "B:' Acidemia is the most powerful stimu­ lator of ventilation in normal individuals.

A 10-year-old has tachypnea (40 breaths per minute) with no cough, fever, or chest pain. He has no prior respiratory

CHAPTER 32

history. Lung fields are clear lung to auscultation and oxygen saturation is 99% on room air. Question 36-1 Which test is best for evaluating his tachypnea? A) Spirometry. B) Chest radiography. C) Lung volumes. D) Capillary blood gas analysis. E) Brain MRI. Discussion 36-1 The correct answer is "D:' The tachypnea is not accompanied by other respiratory symptoms. This suggests either that the tachy­ pnea is either primary hyperventilation or is a compensation for underlying metabolic acidosis.

A term newborn has cyanosis and tachypnea. You perform a hyperoxia test. After 10 minutes in a 1 00% oxygen tent, an arterial blood gas analysis is obtained, which shows pH 7.30, Paco2 35, Pao 2 98, and HC0 - 20. 3 Question 37-1 What is the likely etiology of the cyanosis? A) Pneumonia. B) Surfactant deficiency. C) Right-to-left shunt. D) Left-to-right shunt. E) Hemoglobinopathy.



R E S P I RATO RY D I SO R D E R S

721

Question 38- 1 Why does the patient appear cyanotic? A) Methemoglobinemia. B) Anemia. C) Hemolysis. D) Pulmonary fibrosis due to doxorubicin. E) Right-to-left shunting lesion. Discussion 38-1 The correct answer is ''A:' This child has cyanosis that is unre­ sponsive to oxygen and a normal arterial oxygen level. This strongly suggests methemoglobinemia acquired from dap­ sone. In methemoglobinemia, the iron molecule contained 2 within hemoglobin is oxidized from a ferrous (Fe +) to a ferric 3 (Fe +) state. Ferric iron cannot bind oxygen, so affected red blood cells are desaturated. The hemoglobin dissociation curve shifts to the left as the oxygenated red blood cells increase their affinity for oxygen. These processes result in decreased oxy­ gen delivery to the tissues. Look for chocolate brown-colored blood. Pulse oximeters are not reliable when methemoglo­ bin levels are elevated. Co-oximetry should be performed to confirm the presence of methemoglobin. Cyanosis becomes harder to detect in patients with anemia. Hemolysis is more likely to result in j aundice than cyanosis. Because the blood gas analysis shows a normal Pao 2 on room air, both pulmo­ nary and shunting lesions are unlikely to explain the cyanosis. Doxorubicin causes cardiotoxicity. Bleomycin causes pulmo­ nary toxicity.

A patient is referred to you to evaluate digital clubbing. Discussion 37-1 The correct answer is "C:' A hyperoxia test can help decide if cyanosis is a result of congenital cyanotic heart disease with right-to-left shunting. If not, the lungs or other pathology are to blame. The patient described has an abnormal hyperoxia test; the Pao 2 is low ( < 1 50 mm Hg) despite 1 00% oxygen. Supple­ mental oxygen can correct hypoxemia related to diffusion, V /Q mismatch, and hypoventilation. It cannot correct hypoxemia caused by right-to-left shunt from cyanotic heart disease. Left­ to-right shunt would not cause cyanosis. Abnormal hemoglobin could change the binding of oxygen by hemoglobin, but would not affect the Pao2.

A child is being treated for acute myeloid leukemia. After prescribing dapsone for chemoprophylaxis against Pneumocystis jiroveci, you note that he develops cyanosis. Arterial blood gas analysis shows that Pao 2 is 1 00 mm Hg on room air. Administering supplemental oxygen does not change his color.

Question 39-1 Which of the following diseases is associated with clubbing? A) Liver disease. B) Inflammatory bowel disease. C) Cyanotic congenital heart disease. D) Cystic fibrosis. E) All of the above. Discussion 39-1 The correct answer is "E:' Clubbing is a nonspecific physical finding. Although it is common among patients with structural lung diseases, clubbing is observed in a wide variety of diseases. (See Figure 32- 10.)

A 4-month-old infant has respiratory distress with increased nasal secretions, bilateral wheezing, and crackles. She is feed­ ing poorly and requires supplemental oxygen to maintain an Spo2 greater than 92%.

722

MCG RAW- H I LL E D U CAT I O N S P E C I A LTY BOA R D REVI EW: P E D I AT R I C S

Discussion The correct answer is "E:' Tracheostomy can increase aspiration by all of the listed mechanisms.

A 5-year-old boy has fever of 39°C ( 1 02.2"F), tachypnea (40 times per minute) , respiratory distress, and hypox­ emia. On examination, he has decreased right lower lung field sounds. He is diagnosed with pneumonia and hospital admission is arranged. Question 41 - 1 Which o f the following diagnostic tests has the strongest recommendation? A) Urine antigen testing for pneumococcus. B) Chest X-ray. C) Mycoplasma serology. D) Sputum culture. E) C-reactive protein to assess whether the infection is bacterial. F I G U R E 3 2 - 1 0. C l u b b i n g of the fi ngers a n d toes. (Reproduced with perm ission from Fuster V, Wa lsh RA, Ha rri ngton RA, eds. Hurst's The Heart. 1 3th ed. New York, NY: McGraw- H i l l Education; 201 1 , Fig. 1 4- 1 .)

Question 40- 1 In addition to oxygen therapy, cost-effective evaluation and management should include: A) RSV antigen testing. B) Chest X-ray. C) IV ampicillin. D) Azithromycin. E) Dexamethasone. F) Suctioning as needed. Discussion 40- 1 The correct answer is "F' The infant described has bronchiolitis, a viral infection of the lungs affecting children younger than 2 years of age. Management is supportive, and may include suctioning, oxygen, and IV fluids. Respiratory syncytial virus (RSV) antigen testing is not cost-effective. Chest X-ray often leads to unneces­ sary antibiotic treatment due to confusion between atelectasis and pneumonia. Antibiotics and steroids are not necessary.

� QUICKQUIZ Which statement regarding the relationship between trache­ ostomy and aspiration is true? A) Tracheostomy can impair deglutitive (swallowing) apnea. B) Tracheostomy impairs superior movement of the larynx during swallowing. C) Aspiration is increased in patients with tracheostomy. D) Swallowing is delayed in patients with tracheostomy. E) All of the above statements are true.

Discussion 41 - 1 Th e correct answer i s "B:' This child has pneumonia based on history. The finding of decreased breath sounds rather than crackles is concerning. A chest X-ray is the only test of those listed that can determine if a complication of pneumonia such as pleural effusion or empyema is present. Therefore, it receives the strongest recommendation from the Infectious Disease Society of America. Urine antigen testing has poor specificity and is not recommended. Mycoplasma testing is weakly recom­ mended in children with compatible clinical history. Sputum culture is weakly recommended in children who are able to produce sputum, but a 5-year-old is unlikely to produce spu­ tum. C-reactive protein should not be relied upon to distinguish between viral and bacterial pneumonia. Since the child is being admitted, blood cultures should be obtained.



Helpful Tip

:5.� C h i l d re n

i1 1 r



pneumonia

who

do

not

req u i re

test i n g or i m a g i ng, but with close fol l ow- u p.

Helpful Tip

:5.� A c h i l d

i1 1r

with

hospita l ization can be sta rted on a ntibiotics without

who i s bei ng treated for p n e u m o n i a s h o u l d

i m prove after 48 h o u rs of a nt i b iotic thera py. If not, i m a g i n g s h o u l d be o bta i n ed to look fo r c o m p l ications such a s effu sion or e m pyem a . C h est CT or u ltra s o u n d may be needed if the c h est X-ray i s i nconcl usive.

A chest X-ray is obtained and reveals a parapneumonic effusion.

CHAPTER 32

Question 4 1 -2 Which of the following are indications for chest tube drain­ age or video-assisted thoracoscopic surgery (VATS)? I. Effusion < 10 mm clinically stable. II. Effusion > 10 mm with empyema III. Effusion > 10 mm with respiratory distress IV. Effusion opacities over half of hemithorax and patient has respiratory distress A) IV only. B) II, III, and IV. C) II and IV. D) II and iii. E) All of the above. Discussion 41 -2 The correct answer is "B:' Chest tube drainage should be com­ pleted if there is respiratory distress or empyema. Pneumo­ nia complicated by small effusions ( < 1 0 mm or one-fourth of thorax opacified) usually can be treated with antibiot­ ics alone. Complicated pneumonia refers to the develop ­ ment of a loculated effusion. The pleural space becomes inflamed, forming a sterile effusion. Bacteria move into the fluid, forming pus and loculations-an empyema. Drainage options include VATS or a chest tube with fibrinolytic agents instilled into the chest. If chest tube drainage is unsuccessful, proceed to VATS . Infection spreads from the lung into the pleural space.

A diagnostic thoracentesis is performed on the patient, but only 1 mL of fluid is successfully aspirated. Question 4 1 -3 Which of the following tests on the pleural fluid is most important? A) Lactate dehydrogenase. B) Protein. C) Cell count and differential. D) Gram stain and culture. E) pH. Discussion 41 -3 The correct answer is "D:' Culture of pleural fluid gives a unique opportunity to determine the organism responsible for infec­ tion and tailor the antimicrobial therapy. Lactate dehydrogenase (LDH) , protein, and pH of pleural fluid are unlikely to change management. The cell count and differential would potentially help if malignancy or atypical infection was in the differential diagnosis.

A 3-year-old girl has had recurrent hospitalizations for pneu­ monia. Each illness was typified by fever, lethargy, hypox­ emia, and focal crackles, and each illness affected different lung segments. She also has chronic otitis media, recurrent skin abscesses, and severe eczema.



R E S P I RATO RY D I SO R D E R S

723

Question 42- 1 Which o f the following is/are NOT included in your initial evaluation? A) CBC with differential. B) Quantitative immune globulins. C) Pre- and postvaccination pneumococcal titers. D) Bronchoscopy. E) All of the above are important parts of the initial evaluation. Discussion 42- 1 The correct answer is "D:' The case describes features of recur­ rent pneumonia related to immune deficiency, with severe, recurrent infections of both the skin and respiratory tract. Ini­ tial evaluation should focus on identifying the immune defect. It is not likely to arise from an airway abnormality, since differ­ ent lung segments are affected.

A 10-year-old boy is admitted to the general pediatric ser­ vice with a diagnosis of community-acquired pneumonia. His condition worsens despite treatment with ceftriaxone and azithromycin. He is transferred to the intensive care unit with sepsis and respiratory failure. His initial X-ray shows interval development of abscess, and purulent secretions are aspirated with intubation. Question 43-1 Which bacterial pathogen is most likely to be responsible? A) Streptococcus pyogenes. B) Staphylococcus aureus. C) Streptococcus pneumonia. D) Haemophilus influenzae. E) Mycoplasma pneumoniae. Discussion 43- 1 The correct answer is "B:' The patient has a necrotizing pneu­ monia, likely due to methicillin-resistant S. aureus (MRSA). All of the other options are covered well by ceftriaxone and azithromycin.

A 4-year-old boy has had five hospitalizations for right mid­ dle lobe pneumonia since age 2. There were no respiratory symptoms prior to the initial pneumonia, and he has no his­ tory of recurrent otitis or other infections of the skin or soft tissues. On examination, his chest expands symmetrically, but there are crackles in the right middle lung fields. Question 44- 1 What is the most important test for evaluating his recurrent pneumonias?

724

A) B) C) D) E)

MCG RAW- H I LL E D U CAT I O N S P E C I A LTY BOA R D REVI EW: P E D I AT R I C S

Neutrophil oxidative burst assay. Quantitative immune globulins. Aeroallergen skin testing. Bronchoscopy. Diaphragm fluoroscopy.

Discussion 44- 1 The correct answer is "D:' Recurrent pneumonia in the same lobe suggests a localized anatomic obstruction, such as a retained foreign body. A neutrophil-killing defect or an anti­ body deficiency could result in pneumonia, but it would be unlikely to always infect the same lobe. One would also expect other infections such as otitis media to occur. Asthma can cause atelectasis and be confused with recurrent pneumonia, but the atelectasis tends to be migratory. Diaphragm fluoroscopy may be useful if there is asymmetry in chest expansion, suspected phrenic nerve injury, or both. Question 44-2 Which of the following disorders is least likely to result in bronchiectasis? A) Cystic fibrosis. B) Primary ciliary dyskinesia. C) Selective IgA deficiency. D) Chronically retained foreign body. E) Chronic granulomatous disease. Question 44-2 The correct answer is "C:' Selective IgA deficiency is often discovered incidentally and is rarely associated with bronchi­ ectasis. All of the other disorders listed are associated with bronchiectasis.

A child has a chest X-ray to evaluate cough and wheezing. (See the images below.)

Discussion 45-1 The correct answer is "C:' The patient has an anterior congeni­ tal diaphragmatic hernia, which was an incidental finding. The diaphragm fails to develop, leading to herniation of abdominal contents into the chest. Ninety percent of hernias are left sided. Symptomatic newborns have respiratory distress, unilateral decreased breath sounds, and a scaphoid abdomen. The lateral X-ray shows bowel contents in the anterior chest. The hyperlu­ cency is bowel gas and does not fit the shape of a hyperlucent lobe. •

� 1 1 1r

Helpful Tip Boc h d a l e k hernias a re on the l eft side. Th i n k "Back a n d to the Left." M o rg a g n i h e r n i a s a re on the right side.

Question 45-2 Which of the following statements about congenital dia­ phragmatic hernia is true? A) Symptoms vary depending on the size of the defect. B) Pulmonary hyperplasia of varying degrees occurs. C) Respiratory compliance is increased on the side of the diaphragm. D) Less than 1 % of patients with congenital diaphragmatic her­ nia are treated for pulmonary hypertension. E) None of the above. Discussion 45-2 The correct answer is ''A:' As in this case, diaphragmatic her­ nias may be asymptomatic. Congenital diaphragmatic hernia results in pulmonary hypoplasia. The more severe the defect, the worse oxygenation and ventilation become. The affected lung has decreased compliance. A recent large study shows approximately 30% of patients with congenital diaphragmatic hernias will receive some form of treatment for pulmonary hypertension.

A 5-year-old boy is brought to your office because his asthma is poorly controlled. He has had worsening symptoms of shortness of breath with exertion and his albuterol inhaler does not seem to help. On physical examination you note dis­ tended neck veins, peripheral edema, an enlarged liver and a loud second heart sound (S2). You suspect that the child has been misdiagnosed and is actually suffering from pulmonary hypertension. Question 45-1 What abnormality is identified by his X-ray? A) Congenital hyperlucent lobe (congenital lobar emphysema) . B) Congenital bronchogenic cyst. C) Anterior diaphragmatic hernia (Morgagni hernia) . D) Cardiomegaly. E) Hiatal hernia. F) Pneumomediastinum.

Question 46- 1 Which of the following statements about pulmonary hyper­ tension is false? A) Mean pulmonary arterial pressure in most children and adolescents is less than 25 mm Hg. B) Pulmonary arterial pressure may not be elevated in advanced pulmonary hypertension if right heart failure develops.

CHAPTER 32

C) Pulmonary hypertension can develop in children with pro­ longed exposure to hypoxemia, including sleep apnea. D) Exercise impairment is a late finding in pulmonary hypertension. E) Patients with pulmonary hypertension cannot increase car­ diac output sufficiently during exercise and illness, increas­ ing their risk of syncope or sudden death. Discussion 46- 1 The correct answer is "D:' Pulmonary hypertension often devel­ ops with few patient symptoms, but exercise impairment is usually the earliest sign. Patients are unable to increase their cardiac output in response to exercise because of high pulmo­ nary vascular resistance. Normal children and adolescents have mean pulmonary arterial pressures between 1 0 and 1 6 mm Hg. As the right heart fails, pulmonary arterial pressures may not be elevated because the right heart is unable to overcome high pulmonary vascular resistance. This could result in peripheral edema and hepatomegaly. Question 46-2 Which of the following factors is least likely to contribute to pulmonary hypertension in a premature infant? A) High pulmonary vascular resistance in the fetal circulation. B) Hypoxic vasoconstriction of pulmonary vasculature. C) High pulmonary vasoreactivity. D) Lower pulmonary arterial density due to incomplete lung development. E) Patent foramen ovale. Discussion 46-2 The correct answer is "E:' It is unlikely that left-to-right atrial­ level shunting could contribute to pulmonary hypertension in the newborn period unless the shunt is very large. Right-to-left atrial shunting through a patent foramen ovale could be protec­ tive by providing a "pop-off'' valve to increase left heart filling during a pulmonary hypertensive crisis (blood not entering the lungs from the right side of the heart so less coming back to the left side) . Options "/\_' through "D" are all factors that contribute to pulmonary hypertension in children born prematurely. Question 46-3 Which of the following is true about chronic lung disease of prematurity ( CLD)? A) Despite changes in ventilation strategy, extensive fibropro­ liferative changes remain the principal histopathologic find­ ing in CLD. B) Airway hyperreactivity is uncommon in CLD. C) Risk factors for CLD are highly interactive. D) Intrauterine betamethasone exposure significantly reduces the incidence of CLD but has no effect on infant survival. E) Higher Paco 2 in the first week of life and female sex are independent risk factors for CLD. Discussion 46-3 The correct answer is "C:' CLD, also known as bronchopulmo­ nary dysplasia (BPD), is defined as a persistent oxygen need



R E S P I RATO RY D I SO R D E R S

725

after 28 days of age. Risk factors for CLD include prematurity, surfactant deficiency, chorioamnionitis, sepsis, mechanical ventilation, and patent ductus arteriosus. Many of these risk factors coexist in individual patients. CLD has changed sig­ nificantly over time, with fibroproliferative changes becom­ ing less common. BPD formerly was associated with fibrotic changes from mechanical ventilation-induced barotrauma. This was before the days of pulmonary surfactant preparations. In the "new" BPD, fewer alveoli develop leading to decreased area for gas exchange. Airway hyperreactivity with wheezing is common in CLD. Low Paco 2 (related to increased mechani­ cal ventilation) and male sex are independent risk factors for CLD. Intrauterine steroids confer survival benefit and decrease respiratory distress syndrome but do not reduce the incidence of CLD.

You are caring for a former 24-week premature infant who is now 4 months old. She has CLD and requires '12 liter per minute of oxygen to maintain an Spo 2 greater than 92%. Question 47-1 You recommend all of the following EXCEPT: A) Regular immunizations. B) Passive RSV immunoprophylaxis. C) Avoidance of smoke exposure. D) Initiation of lansoprazole. E) Close monitoring of nutrition. Discussion 47- 1 The correct answer is "D:' In the absence of symptoms of gastroesophageal reflux disease (GERD), acid suppression is unnecessary and may increase the risk of pulmonary infec­ tions in infants and young children. Treatment of children with CLD aims at promoting lung growth through nutri­ tion, preventing lung injury related to infections or smoke exposure, and preventing complications such as pulmonary hypertension.

An infant is evaluated in the emergency department for an apparent life-threatening event (ALTE) . Question 48- 1 Which of the following findings on history and physical exam warrants admission for further evaluation? A) Inconsistent histories. B) Age younger than 2 weeks. C) Repeated episodes of ALTE. D) Stridor. E) CPR administration by parent. F) All of the above.

726

MCG RAW- H I LL E D U CAT I O N S P E C I A LTY BOA R D REVI EW: P E D I AT R I C S

Discussion 48- 1 The correct answer is "F:' ALTE is a frightening event witnessed by a caregiver that involves some change in color, breathing, or tone. Basically, it presents as "my baby may have stopped breathing and it was scarY:' Most often, there is no identified cause of ALTE. The history and physical exam will guide what tests if any are ordered. Each of the findings listed in options "X' through "E" could suggest an underlying abnormality and warrants further evaluation. The first 2 weeks of infancy may be when congenital heart diseases, seizure disorders, infections, or metabolic disorders present. Stridor suggests an abnormal air­ way that would contribute to apnea. Inconsistent history raises the possibility of abuse. CPR administration and repeated epi­ sodes both increase the possibility for identifying an underlying disorder. The most important thing to remember is that ALTE and SIDS are not related.

A 7 -year-old boy has cough, shortness of breath, and hemop­ tysis. Lung examination reveals bilateral crackles. Spirom­ etry shows decreased FVC and normal FEVJFVC. Diffusing capacity of the lung (DLCO) is increased. Lab studies show microcytic anemia with a hemoglobin of 9 mg/dL. Question 49- 1 Which of the following tests is consistent with a diagnosis of idiopathic pulmonary hemosiderosis? A) Anti-glomerular basement membrane antibody. B) Hemosiderin-laden macrophages on bronchoalveolar lavage. C) c-ANCA (antinuclear cytoplasmic) antibody. D) Endoglin mutations. E) Chest CT revealing dilated bronchial arteries. Discussion 49- 1 The correct answer is "B:' Idiopathic pulmonary hemosidero­ sis is a rare disorder (incidence of 1 : 1 ,000,000) that results in

recurrent alveolar hemorrhage and pulmonary fibrosis. Some investigators have proposed an autoimmune or allergic basis to this disease, although this is not proven. Its diagnosis requires evidence of alveolar hemorrhage and the exclusion of other causes of pulmonary hemorrhage. Option "X' is consistent with Goodpasture disease, option "C" with granulomatosis-poly­ angiitis (formerly Wegener granulomatosis), option "D" with hereditary hemorrhagic telangiectasia syndrome, and option "E" with bronchial artery (not alveolar) hemorrhage. BIBLIOGRAPHY

Bradley JS, Byington CL, Shah SS, et al; Pediatric Infectious Diseases Society, the Infectious Diseases Society of America. The management of community-acquired pneumonia in infants and children older than 3 months of age: Clinical practice guidelines by the Pediatric Infectious Diseases Society and the Infectious Diseases Society of America. Clin Infect Dis. 20 1 1 ;53 {7):e25-76. Canani RB, Cirillo P, Roggero P, et al. Therapy with gastric acidity inhibitors increases the risk of acute gastroenteritis and community-acquired pneumonia in children. Pediatrics. 2006; 1 1 7(5):e8 1 7-820. Kleigman RM, Stanton BF, St. Geme JW, Schor NF, Behrman RE, eds. Nelson Textbook ofPediatrics. 19th ed. Philadelphia, PA: Saunders; 20 1 1 . Manna A, Montella S , Maniscalco M , Maglione M , Santamaria F. Clinical application of nasal nitric oxide measure­ ment in pediatric airway diseases. Pediatr Pulmonol. 20 1 5;50{ 1 ) :85-99. Marcus CL, Brooks LJ, Draper KA, et al. Diagnosis and man­ agement of childhood obstructive sleep apnea syndrome. Pediatrics. 20 1 2 ; 1 30(3):e7 1 4-755. Martins RHG, Hidalgo Ribeiro CB, Fernandes de Mello BMZ, Branco A, Tavares ELM. Dysphonia in children. J Voice. 20 12 ; 26(5) :674.e6 1 7-e620. Wienberger M. Evaluation and initial management of chronic cough in children. First Consult. May 29, 20 1 3 . Zaoutis LB, Chiang V W. Comprehensive Pediatric Hospital Medicine. Philadelphia, PA: Mosby; 2007.

33

Ski n D isorders Wi l l Au g h e n ba u g h a n d Lisa M u c h a rd



An 8-year-old boy presents with recurrent erythema, scale, and pruritus of the antecubital and popliteal fossae. His mother notes that his skin problems tend to flare in the win­ ter months. Social history is notable for a pet cat. You suspect atopic dermatitis. Question 1 - 1 What additional cutaneous finding supports your diagnosis? A) Alopecia. B) Erythema and scale of alar creases. C) Keratosis pilaris. D) Nail pits. E) Scaling of elbows and knees. Discussion 1 - 1 The correct answer is "C:' The boy in this case has atopic der­ matitis, which is associated with keratosis pilaris (KP) . KP is an example of retention hyperkeratosis, characterized by adherent keratin at the apex of the hair follicle. KP has a rough, sandpa­ per texture and may be red in color. It is typically located on the extensor arms, thighs, cheeks, and buttocks. (See Figure 33- 1 . ) Erythema and scale o f the alar creases (creases near the tip o f the nose) is more typical of seborrheic dermatitis. The scale is yel­ low, greasy, and concentrates on the scalp, postauricular sulci, brows, glabella, alar creases, nasolabial folds, mental crease, central chest, axillae, and groin. Nail pits (small depressions) and oil spots (yellow discoloration of the nail bed) as well as scaling of elbows and knees are classic for psoriasis. The terms atopic dermatitis and eczema may be used interchangeably. In addition to KP, eczema may present with additional cutaneous findings, including ichthyosis vulgaris (dry skin with fish-scale appearance) , pityriasis alba (ill-defined hypopigmented patches of the face, trunk, and extremities), hyperlinear palms, Dennie­ Morgan lines (extra skinfold under the eyes) , and allergic salute (transverse nasal crease due to repeated rubbing of the nose) . (See Figure 33-2.)

Helpful Tip

:5.� Pityriasis

i1 1r

a l ba

and

viti l i g o

may

be

d iffi c u l t

to

d isti n g u i s h . Pityriasis i s off- wh ite i n color a n d res u lts fro m decreased p i gm e n t (hypo p i g m e ntatio n ) . (See F i g u re

33-3.)

Vit i l igo i s m i l k wh ite a n d resu lts fro m

a bsent p i g m e n t (dep i g m e ntatio n ) . S h i n i n g a Wood l a m p o n the skin accentuates d e p i g m e nted skin a n d has no effect on hypopi g m e nted s k i n , h e l p i n g to d iffe rentiate the two co n d itions. (See F i g u re

33-4.)

You counsel the patient and his mother on dry skin care, including bathing daily to every other day, and applying bland emollients immediately after bathing. Question 1 -2 Which topical steroid is most appropriate for both acute and chronic management of the rash? A) Clobetasol 0.05% cream. B) Desonide 0.05% cream. C) Hydrocortisone 1% cream. D) Pimecrolimus 0. 1 % cream. E) Triamcinolone 0. 1 % cream. Discussion 1 -2 The correct answer is "E:' Triamcinolone 0 . 1 % is a mid-potency steroid. It is a good starting point for rashes that occur on the trunk and extremities. Hydrocortisone and desonide are low­ potency steroids. They may not be sufficiently potent to treat eczema of the trunk and extremities and are more appropri­ ate for the face, groin, and skinfolds. Clobetasol is an ultra­ potent steroid that must be used cautiously due to the risk of steroid-induced skin atrophy. Pimecrolimus (Elidel) cream is a topical calcineurin inhibitor. It carries a black box warning requiring it to be used second line and on a rotational basis. (See Table 33- 1 . ) 727

728

MCG RAW- H I LL E D U CAT I O N S P E C I A LTY BOA R D REVI EW: P E D I AT R I C S

F I G U R E 3 3 - 1 . Keratosis pilaris is cha racterized b y small papu les with a sandpaper texture located on the extensor su rfaces of proximal arms and thighs. Surrounding erythema may be present. It is associated with atopic dermatitis or eczema. (Reproduced with perm ission from Wolff K, Johnson

RA, and Saavedra AP, eds. Fitzpatrick's Color A tlas and Synopsis of Clinical Dermatology, 7th ed. McGraw- H i l l Education, I nc., 201 3. Fig. 4-3.)

The boy and his mother return the next winter, as the rash has recurred. He resumed triamcinolone 0. 1 % cream twice daily but the symptoms have not improved. Itching is mild. You note fissures and serous drainage on exam.

F I G U R E 33-3. Pityriasis a l b a is cha racterized by i l l-defi ned off-wh ite­ colored hypo pigmented patches on the face, tru n k, and extrem ities. S h i n i n g a Wood l a m p on the skin does n o t c h a n g e t h e lesions. (Used with perm ission from Wi l l i a m Augehenbaugh, M D.)

Question 1 -3 What is your next step? A) Bacitracin. B) Biopsy. C) Bleach baths.

F I G U R E 33-2. Atopic Dermatitis. The characteristics of atopic dermatitis or

eczema va ry by age. In c h i l d hood, l ichenified erythematous plaques form on the flexor su rfaces of the el bows and knees. Secondary excoriations may be present from pruritus. In i nfa nts with atopic dermatitis, the tru n k and face may be i nvolved but the d i a per a rea is spared. (U sed with permission from Wi l l i a m Augehenbaugh, M D.)

F I G U R E 33-4. Vitiligo

is an auto i m m u n e process that destroys the melanocytes, res u lting i n depigmented m i l ky wh ite-colored patches.

S h i n i n g a Wood l a m p on the skin accentuates the lesions. (Used with perm ission from Wi l l i a m Augehenbaugh, M D.)

CHAPTER 33

TABLE 3 3 - 1 TO P I C A L STERO I D S U S E D I N THE

M A N AG E M E NT O F ATO P I C D E R MATI T I S (ECZEMA)

Potency U ltra potent

Topica l Corticosteroid

Location to Apply

Beta methasone d i propionate 0.05%

Sca l p

Clobetasol propionate 0.05% Mid potency

Triamci nolone 0 . 1 %

Low potency

Beta methasone va ler­ ate 0. 1 %

Pa l m s and soles Tru n k Extrem ities Face G ro i n

Desonide 0.05% Hyd rocortisone 1 %

D) Clobetasol 0.05% cream. E) Oral prednisolone.

Discussion 1 -3 Th e correct answer i s "C:' Atopic dermatitis (eczema) i s char­ acterized by a compromised skin barrier. It is not uncom­ mon for the eczema plaques to become secondarily infected with Staphylococcus aureus (secondary impetiginization). (See Figure 33-5.) Adding Vt cup of household bleach to the bath­ tub and bathing once or twice weekly may help treat eczema that fails to respond to standard therapy and prevent flares of eczema. Oral antibiotics are reserved for widespread infection or failure to respond to conservative therapy. Bacitracin has limited antibacterial properties and is associated with allergic contact dermatitis. Oral steroids may be added for severe flares once secondary infection has been addressed.



S KI N D I SO R D E R S

729

i QUICKQUIZ Patients with atopic dermatitis are at risk for developing which disease? A) Allergic contact dermatitis. B) Allergic rhinitis. C) Arthralgias. D) Food allergy. E) Psoriasis. Discussion The correct answer is "B:' The atopic triad is comprised of asthma, allergic rhinitis, and atopic dermatitis (eczema) . Patients may have one, two, or three of these elements of the triad. The association of food allergy and atopic dermatitis is a possible consideration in children younger than 2 years of age but does not typically occur in older children and adults.



Helpful Tip

� Do not confuse bacitra c i n with m u p i rocin (Bactro b a n ) .

-

r1 1r

M u p i rocin is a topica l a ntibiotic that treats sta phylo­ cocca l i nfecti o n s, i n c l u d i n g i m petigo. If seco n d a r i l y i nfected eczema i s foca l, topica l m u p i rocin may be a good choice for treatment. Bacitra c i n is a topica l a nti­ biotic that may cause a l l e rg ic contact dermatitis.



-� r1 1r

Helpful Tip I m petigo is a pri m a ry process whereby sta phyl ococca l bacteria cause i nfl a m mation a n d honey- colored cru st­ i n g of n o r m a l ski n . Seco n d a ry i m petig i n ization a rises when sta phyl ococca l bacteria sett l e i nto a b raded skin and d i s ru pt normal h ea l i n g .

A 14-year-old girl develops severe itching and linear ery­ thema and blisters of her arms and legs after hiking in the woods. (See Figure 33-6.) She has a history of allergic rhinitis. Question 2-1 What is the most likely cause of her rash? A) Allergic contact dermatitis. B) Atopic dermatitis. C) Irritant contact dermatitis. D) Factitious dermatitis. E) Seborrheic dermatitis.

F I G U R E 33-5. I m petig i n ized Ato pic Dermatitis. Lesions of atopic dermatitis

(eczema) may become acutely worse secondary to su bseq uent i nfection by Staphylococcus au reus, referred to as secondary i m petig i n ization. The lesions

become weepy, with serous d ra i nage a n d crusting. (Used with perm ission from Wi l l i a m Augehenbaugh, M D.)

Discussion 2-1 The correct answer is "A:' Rashes that are linear or geographic (square, triangular, etc) are commonly caused by agents that come in contact with the skin. Blisters arise from agents that elicit a severe inflammatory response. Blisters are common in

MCG RAW- H I LL E D U CAT I O N S P E C I A LTY BOA R D REVI EW: P E D I AT R I C S

730

You suspect she may have developed rhus dermatitis from exposure to poison ivy. Question 2-2 How do you explain the etiology of allergic contact dermatitis? A) Delayed-type hypersensitivity reaction, therefore the rash occurs with the first exposure. B) Delayed-type hypersensitivity reaction, therefore the rash requires a sensitizing event. C) Direct cellular cytotoxicity from a caustic agent. D) Immediate-type hypersensitivity reaction, therefore the rash occurs with the first exposure. E) Immediate-type hypersensitivity reaction, therefore the rash requires a sensitizing event. Discussion 2-2 The correct answer is "B:' (See Table 33-2.)

F I G U R E 33-6. Al lergic contact dermatitis is a delayed (type I V ) hypersensitivity

reaction to a pa rticular a l l ergen such as urushiol (found in poison ivy), as i n t h i s image. It is cha racterized b y erythema and blisters i n a l i near pattern. I n contrast, i rritant contact dermatitis is a chemical o r mechanical i rritation o f the skin that is not mediated by the i m m u n e system. (Used with perm ission from Wi l l i a m Augehenbaugh, MD.)

allergic contact and irritant contact dermatitis, but would not be associated with atopic dermatitis, factitious (self-induced) dermatitis or seborrheic dermatitis.



� I

1 1r

Helpful Tip A l l e rg i c contact derm atitis i s u n i q u e to the i n d iv i d u a l who develops it. The refore d ifferent peo p l e may or may not deve l o p a reaction, and the deg ree of i nfl a m mation

Question 2-3 Which treatment is NOT recommended in patients with rhus dermatitis? A) Apply a high-potency topical steroid. B) Avoid exposure to the offending plant. C) Bathe pets that may have been exposed to plant material. D) Prednisone 20 milligrams (mg) orally for 5 days. E) Wash all clothing and gardening tools that were used. Discussion 2-3 The correct answer is "D:' Allergic contact dermatitis due to rhus allergens typically incites a severe inflammatory response. Patients are advised to avoid further exposure and wash the plant resin from clothing, pets, and tools with soap and water. High-potency topical steroids are typically required to provide relief from itching. Oral steroids (prednisone) should be dosed over a 3-week period, as shorter durations often result in a rebound phenomenon.

a n d itc h i n g may va ry. I rrita nt contact dermatitis a ri ses in all i n d ividua ls, depen d i n g o n the d e g ree of exposu re. For exa m ple, i rrita nt hand d e rmatitis fro m repeated expo s u re to soa p and water i s most co m mo n in peo p l e who freq uently w a s h t h e i r h a n d s-ba rten d e rs, h a i r d ressers, h e a l t h ca re provid e rs, etc.

A 1 5-year-old girl presents with redness and scaling of her eyebrows and alar creases. No itching is reported. She applies moisturizers without benefit.

TABLE 33-2 ETI O LO G I E S OF D E RMATITIS A N D U RT I C A R I A

Pathogenesis

Ti m i n g

Deg ree o f Exposu re Req u i red for Reaction

A l l erg ic contact dermatitis

Delayed-type hypersensitivity (type IV)

Req u i res sensitizi ng event

Fi rst reaction occ u rs after at least 2 weeks of susta i ned exposu re; su bseq uent reactions may occ u r with low-level exposu res

I rrita nt contact dermatitis

Di rect cel l u l a r toxicity (not i m m u ne mediated)

No sensitizi ng event necessa ry, so may occ u r on fi rst exposu re

Req u i res sufficient exposu re to i rritate the skin; dependent on the ca u stic nat u re of the i rritant

U rticaria

I m mediate-type hypersensitivity (type I)

No sensitizi ng event necessary

Low-level exposu res

CHAPTER 33



S KI N D I SO R D E R S

731

Question 3-1 What is the most likely diagnosis? A) Allergic contact dermatitis. B) Atopic dermatitis. C) Irritant contact dermatitis. D) Psoriasis. E) Seborrheic dermatitis. Discussion 3-1 The correct answer is "E:' Seborrheic dermatitis may be present in infancy as "cradle cap:' It is acquired at birth with exposure to yeast during vaginal delivery. The yeast remains part of the normal flora throughout life. Outbreaks of seborrheic derma­ titis may reoccur at puberty and beyond. While most patients believe they have dry skin, the scale is typically yellow and moist in character. Commonly affected sites include scalp, postauricu­ lar sulci, eyebrows, glabella, alar creases, chin, chest, axillae, and groin. (See Figure 33-7.)

F I G U R E 33-8. G uttate psoriasis is cha racterized b y t h e abrupt onset of m u ltiple s m a l l sca l i n g plaq ues on the tru n k a n d extrem ities. It is freq uently trigg ered by Streptococcus pyogenes (gro u p A streptococcal) pharyng itis. (Used with perm ission fro m Wi l l i a m Augehenbaugh, M D.)

Question 3-2 What is your first-line treatment recommendation? A) Clobetasol 0.05% cream. B) Tacrolimus 0. 1 % ointment. C) Terbinafine 1 % cream. D) Triamcinolone 0. 1 % cream. E) Triamcinolone 0. 1 % ointment.

topical steroids, such as hydrocortisone 1 % or 2.5% cream. The two may be combined if a single agent is ineffective. Treatment of the scalp includes antiseborrheic shampoos containing zinc, selenium sulfide, ketoconazole, or tar. The treatments should be initiated when redness and scale are present and stopped once skin changes resolve.

Discussion 3-2 The correct answer is "C:' Seborrheic dermatitis typically responds well to topical treatment. Patients should be warned that seborrheic dermatitis is a recurrent condition. First-line treatments include topical antifungal agents or low-potency

F I G U R E 33-7. Seborrheic dermatitis is cha racterized by erythematous sca l i n g w i t h yel low greasy scales. (Reproduced with perm ission from Goldsm ith LA, Katz Sl, G i l c h rest BA, Pa l l e r AS, Leffel I DJ, Wolff K, eds. Fitzpatrick's Dermatology in General Medicine. 8th ed. New York, NY: McGraw-H i l l Education; 201 2, Fig. 22-4.)

An 18-year-old woman develops the acute onset of a mildly pruritic rash of the trunk and extremities. (See Figures 33-8 and 33-9.) She denies a history of atopy.

F I G U R E 33-9. Psoriatic N a i l Di sease. The n a i l s a re thickened, d i scolored (ye l l ow), and may be pa rti a l ly detached (onycholysis). Pitting may be seen. (Reproduced with perm ission from Goldsmith LA, Katz Sl, G i l c h rest BA, Pa l l e r A S , Leffel I D J , Wolff K , e d s . Fitzpatrick's Dermatology i n General Medicine. 8th

ed. New York, NY: McGraw- H i l l Education; 201 2, Fig. 89- 1 8.)

732

MCG RAW- H I LL E D U CAT I O N S P E C I A LTY BOA R D REVI EW: P E D I AT R I C S

Discussion 4-2 The correct answer is "E:' Guttate psoriasis is frequently trig­ gered by streptococcal pharyngitis. Psoriatic arthritis affects 5% to 30% of patients and is most common with the plaque subtype.

Question 4- 1 What is the most likely diagnosis? A) Atopic dermatitis. B) Contact dermatitis. C) Guttate psoriasis. D) Plaque psoriasis. E) Seborrheic dermatitis.

� QUICKQUIZ

Discussion 4- 1 The correct answer is "C:' Psoriasis is characterized by sharply defined circular erythematous plaques with silver colored scales. Pinpoint bleeding may be seen with removal of the scale; this is known as the Auspitz sign. The fingernails or toenails may be involved. (See Figure 33-9.) Guttate psoriasis (gutta is the Greek word for droplet) typically affects patients younger than 30 years of age and is characterized by the abrupt onset of small pink, scaling plaques of the trunk and extremities. (See Figure 33-8.) Plaque psoriasis typically affects extensor surfaces (elbows and knees), scalp, and trunk. Atopic dermatitis (eczema) is character­ ized by ill-defined plaques that are extremely itchy, concentrating on flexural surfaces. Neonates may develop facial eczema, which is less common as the child grows older. (See Table 33-3.) Question 4-2 What is the most likely associated symptom? A) Abdominal pain. B) Arthritis. C) Cough. D) Diarrhea. E) Sore throat.

Psoriasis may involve which of the following body areas? A) Scalp. B) Nails. C) Perineum. D) Elbows. E) All of the above. Discussion The correct answer is "E:' Psoriasis commonly involves the extensor surfaces of the elbows and knees, the back, and the scalp. Inverse psoriasis involves the intertriginous areas, including the inguinal, perineal, genital, intergluteal, and axil­ lary areas. Think about psoriasis in an infant with recalcitrant diaper rash.

A I S-year-old girl presents with mild acne vulgaris, predomi­ nantly characterized by comedones and rare inflammatory papules. (See Figure 33-10.)

TABLE 33-3 D I F F E R E N T I A L D I AG N O S I S O F SCALI N G D E R MATI T I S

Atopic dermatitis

Age of Onset

Distri bution

C l i n ical Features

C h i l d hood

Face (i nfa nts), flexu res

Erythema Sca le I l l-defined borders

Contact dermatitis

Any age

Dependent on a l l e rgen

Erythema Sca le Sharp borders

Seborrheic dermatitis

G uttate psoriasis

C h i l d hood

< 30 yea rs

Sca l p, posta u ri c u l a r su lci, brows, g l a bella, alar creases, nasola b i a l folds, sternal chest, g ro i n

Yel low, g reasy sca le

Tru n k a n d extrem ities

Erythema

Pi n k color I l l-defined borders Si lver sca le Small plaq ues Sharp borders

Plaque psoriasis

Any age Pea ks at 20-30 yea rs, and again at 50-60 yea rs

Sca l p, extensor extrem ities, tru n k, penis, g l utea l cleft

Erythema Si lver sca le Large plaques Sharp borders

CHAPTER 33

F I G U R E 33- 1 0. Acne Vu lgaris. The g i rl in this photo has comedonal acne of



S KI N D I SO R D E R S

733

Discussion 5-2 The correct answer is "D:' The increase in androgens at puberty contributes to the development of acne. Estrogens, conversely, have an inhibitory effect on acne. The hallmark of a hormonal influence on acne in young women includes irregular menses, perimenstrual flares, and chin and jawline (beard) distribution of hair. Spironolactone is typically used in combination with oral contraceptive pills (OCPs) . It competitively inhibits bind­ ing of androgens to androgen receptors, thereby reducing acne. Side effects include urinary frequency, headache, irregular men­ ses or spotting, and feminization of a male fetus. Isotretinoin does not address the underlying hormonal influence and will not have a permanent effect on acne. Most OCPs that improve acne combine estrogen with low-androgenic progestins, such as drospirenone.

her nose cha racterized by noni nfl a m m atory open a n d closed comedones, commonly referred to as blackheads and wh iteheads. (Used with perm ission from Wi l l i a m Augehenbaugh, M D.)

Question 5-1 Which agent is the most appropriate first-line therapy for this patient? A) Adapalene gel twice daily to acne lesions. B) Benzoyl peroxide gel once daily to the entire face. C) Clindamycin gel once daily to acne. D) Minocycline 1 00 mg once daily. E) Tretinoin gel to the entire face at bedtime. Discussion 5-1 The correct answer is "E:' Topical retinoids are the primary first-line treatment for comedonal acne. Retinoids prevent for­ mation of the microcomedone, a small blockage at the apex of the hair follicle, which is the first step in acne formation. Treti­ noin (Retin A) , adapalene (Differin) , and tazarotene (Tazorac) are examples of topical retinoids. The primary side effect is irritation, so prescribing instructions include application of a pea-sized amount to the entire face at bedtime. Because they are used for acne prevention, they should be applied to the entire face. Topical antibiotics treat existing acne and provide anti­ inflammatory effects. Therefore they are applied to individual inflamed acne lesions. Oral antibiotics are recommended for up to 3 months to reduce the burden of inflammatory acne.

A 1 7-year-old girl experiences a severe flare of acne. (See Figure 33- 1 1 . ) She has not responded to topical benzoyl per­ oxide wash and oral minocycline. You are concerned about scarring potential and decide to start her on isotretinoin. You counsel her on the side effects of isotretinoin. Question 6- 1 Which of the following is NOT a side effect of isotretinoin? A) Depression. B) Headache and blurred vision (pseudotumor cerebri) . C ) Hyperpigmentation. D) Photosensitivity. E) Teratogenicity.

Two years later, she notes a flare of acne, predominantly of the chin and jawline. She reports irregular periods but no increased facial hair. You start her on an estrogen-progestin oral contraceptive without improvement. Question 5-2 What is your next step? A) Oral doxycycline. B) Oral isotretinoin. C) Insertion of an intrauterine device to replace use of oral contraceptive pills. D) Oral spironolactone. E) Topical benzoyl peroxide.

F I G U R E 33-1 1 . I nflammatory Acne. Acne may progress, causing inflammatory lesions that i nclude pa pules, pustules, and nodules. (U sed with permission from Wi l l i a m Augehenbaugh, M D.)

734

MCG RAW- H I LL E D U CAT I O N S P E C I A LTY BOA R D REVI EW: P E D I AT R I C S

Discussion 6- 1 The correct answer is "C:' Patients on isotretinoin should be coun­ seled regarding the side effects of teratogenicity (female patients only), mood changes (controversial association with dysthymia, depression, suicidal ideation), dry skin, hyperlipidemia, arthral­ gias and myalgias, blurred vision, tinnitus, photosensitivity, and pseudotumor cerebri (leading to headache and blurred vision) . (You decide which is worse-the side effect profile of isotretinoin or having acne?)

A 1 5-year-old boy develops a single asymptomatic patch of alopecia. He was not concerned until multiple additional patches were noted. His mother has not witnessed him pulling his hair. Question 7-1 What is the most likely diagnosis? A) Alopecia areata. B) Syphilis. C) Telogen effluvium. D) Tinea capitis. E) Trichotillomania. Discussion 7-1 The correct answer is "A:' Alopecia areata is a common cause of nonscarring hair loss. It is characterized by the abrupt onset of patches of hair loss (focal alopecia areata; see Figure 3 3 - 1 2 ) , complete loss o f all scalp hair (alopecia totalis) , o r loss o f all body hair (alopecia universalis) . Alopecia tends to run a chronic

and recurrent course, with the more severe variants being more resistant to treatment. Secondary syphilis is associated with a "moth-eaten" appearance of hair loss, coupled with scaling patches of the palms, trunk, or extremities. Telogen effluvium typically occurs following a physical or emotional stressor and involves the entire scalp. Patients note diffuse hair thinning, as opposed to discrete patches of hair loss. Tinea capitis is pruritic; examination demonstrates scale and broken hairs. A boggy, elevated lesion known as a kerion may develop. Trichotilloma­ nia is characterized by focal or widespread twisted and broken hairs, as a result of physical pulling and manipulation. (See Table 33-4.)



Helpful Tip

=-� I n

i1 1 r

tric hoti l l oma n ia, look for broken h a i rs of va ryi ng

length in the patches of h a i r loss. I n a l opecia a reata, look fo r "exc l a mation point h a i rs"-s hort b ro ken-off h a i rs near the sca l p.

Question 7-2 What additional physical exam finding supports your diag­ nosis of alopecia areata? A) Hepatosplenomegaly. B) Lymphadenopathy. C) Nail thickening. D) Rubbery nodule on the skin. E) White patches of skin. Discussion 7-2 The correct answer is "E:' Alopecia areata is an autoimmune dis­ ease in which the immune system targets the hair bulb, resulting in miniaturization of the hair. It may be associated with vitiligo, characterized by bright white, nonscaling patches of depig­ mented skin with or without white hairs (poliosis). Hypothy­ roidism and pernicious anemia are additional associations. Nail pits, small depressions in the nail bed, may be seen as well. (See Table 33-4, earlier.) Question 7-3 You recommend a high-potency topical steroid, applied twice daily to the affected area. The patient returns after 3 months, noting no response to therapy. What is your next step? A) Intralesional steroid injection. B) Oral methylprednisolone. C) Topical minoxidil 5% solution. D) Oral terbinafine. E) Topical terbinafine.

F I G U R E 3 3 - 1 2. Alopecia a reata typica l l y affects the sca l p, causing patches of com p l ete h a i r loss. Notice the affected a reas a re smooth, circu l a r, a n d wel l defined without sca l i ng. (Used with permission fro m Wi l l i a m Augehenba u g h , M D.)

Discussion 7-3 The correct answer is ''A:' Steroid injections are effective for the motivated patient as it delivers the medication to the appropriate depth. Serial injections spaced 4 to 8 weeks apart may be necessary to achieve effect. Methylprednisolone or other systemic immunosuppressants are rarely indicated and are most commonly used for rapid shedding seen in alopecia

CHAPTER 33



S KI N D I SO R D E R S

735

TABLE 33-4 CO N D I T I O N S ASSOCIAT E D WITH HAI R LOS S

Alopecia Areata

Ti nea Capitis

Telogen Effl uvi u m

Trichoti l lomania

Etiology

Autoi m m u ne process with i nfla m mation a ro u nd h a i r fol l icle

Fungal i nfection of h a i r shaft or sca l p

Reaction t o physical o r emotional stress

Tra u m atic p u l l i ng of h a i rs

C l i nical fi n d i n g s

Rou nd patches

Ad herent wh ite sca le

Absence o f sca le

Black dots-ie, broken h a i rs at sca l p

Th i n n i n g d i stri buted across fu l l sca l p

B roken a n d twi sted h a i rs

H a i r p u l l test positive

Anxious demeanor

None

Psychiatric d i seases

Exclamation point (ta pered) h a i rs Pol iosis (white h a i r)

Pustu les and boggy plaque (kerion)

Nail pits Associated d i seases

Viti l i g o

None

Hypothyroidism Pernicious anemia

Comments

Often recu rrent

Treatment

Topical steroids

Terbinafine

E l i m i nate the stressor

Behaviora l modification

l ntralesional steroids

Gri seofulvin

M i noxi d i l

Psychothera py

Onset 2-3 months after physical o r emoti o n a l stressor

Psychotropic med ications

M i noxi d i l (2nd l i ne)

totalis or universalis. Minoxidil solution is used for androge­ netic alopecia, commonly referred to as male pattern baldness. It is occasionally used as an adjunctive therapy in alopecia areata to help minimize shedding. Since it does not have anti­ inflammatory effects, it is minimally effective in alopecia areata. Topical and oral antifungal agents (terbinafine) are employed for tinea capitis.

� QUICKQUIZ What body site is most commonly affected in trichotilloma­ nia (nervous hair pulling) ? A) Beard. B) Eyebrows. C) Eyelashes. D) Pubic hair. E) Scalp.



� 1 1 1r

p u l l i n g on the h a i r, such as the tight h a i r bra i d i n g seen i n certa i n c u l t u ra l practices.

A 6-year-old girl develops severe itching, erythema, and scale on the scalp. Physical exam does not reveal evidence of scarring. Her elbows, knees, and nails are clear. You initially diagnose seborrheic dermatitis, but she fails to respond to topical selenium sulfide shampoo.

TABLE 33-5 OSM- V C R I T E R I A F O R

T R I C HOTI L LOMA N I A •

Discussion The correct answer is "E:' Trichotillomania is most common among girls, with onset typically between the ages of 5 and 12 years. The scalp is most commonly affected. The Diagnostic and Statistical Manual of Mental Disorders, Fifth Edition (DSM- V) summarizes diagnostic criteria for this condition. (See Table 33-5.) There is no universally effective treatment for trichotillomania. Referral to a psychologist or psychiatrist may be helpful. Treatments include behavior modification, psycho­ therapy, and psychotropic medications.

Helpful Tip Traction a l opecia i s hair loss res u l t i n g fro m c h ro n i c









Recu rrent p u l l i ng o u t o f one's hair, resulting i n noticea ble hair loss An i ncreasing sense of tension i m mediately before p u l l ­ i ng o u t h a i r or w h e n atte m pting t o resist the behavior Plea s u re, g ratification, or rel i ef when p u l l i ng out h a i r T h e d i stu rba nce is n o t better accou nted for b y a n other menta l d i sorder a n d is not d u e to a genera l medical condition The d i stu rba nce provokes c l i n ically marked d i stress a nd/o r i m pa i rment in occu pational, soc i a l, o r other a reas of fu ncti o n i n g

MCG RAW- H I LL E D U CAT I O N S P E C I A LTY BOA R D REVI EW: P E D I AT R I C S

736

Question 8- 1 What is your next step? A) Bacterial culture. B) Empiric treatment with a topical antifungal cream. C) Scalp biopsy. D) Scrape scalp and send scale for fungal culture. E) Topical steroid solution to reduce inflammation.

C) Lymphadenopathy. D) Pustules. E) Twisted hairs.

Discussion 8-1 The correct answer is "D:' Given the high incidence of tinea capi­ tis among children, culture should be performed in those who do not respond to conservative treatment. (See Figure 33- 13.) Tinea capitis presents with single or multiple areas of scaly, patchy hair loss that fluoresce when looked at with a Wood lamp. Seborrheic dermatitis occurs more commonly around the time of puberty and has additional involvement of the brows, glabella, and alar creases. Itching tends to be mild. Bacterial culture may be considered if honey-colored crusting, erosions, or pustules are present. Topical antifungal creams are typically not sufficient to penetrate past the hair and offer inadequate absorption. Although topical steroids may reduce inflammation, they do not address the underlying source of the problem. It is premature to obtain a scalp biopsy. Pso­ riasis may affect the scalp, but this child has no involvement of the skin or nails. That was a key historical clue snuck into the question.



Helpful Tip

=� Dermatop hyte

i1 1r

i n fections

can

be

d i a g n osed

by

potassi u m hyd roxi d e (KOH) exa m of scra pi n g s fro m the lesions or fu n g a l cu ltu re. With sca l p a n d foot i nfections, look fo r fl uorescence u n d e r a Wood l a m p .

Question 8-2 Which of the following is NOT a potential presentation of tinea capitis? A) Black dots. B) Boggy plaques.

Discussion 8-2 The correct answer is "E:' Fungal scalp infections may be ectothrix or endothrix. Ectothrix (outside the hair shaft) infections occur with fungi that fail to penetrate the hair follicle and lead to an adherent white plaque on the scalp and hair follicles. Endothrix (inside the hair shaft) infections invade the hair follicle and lead to weakness of the hair shaft. Broken hairs are visible as black dots on the skin surface. Localized lymphadenopathy may be present. Boggy erythematous plaques and pustules, termed a kerion, arise from a severe inflammatory response to a more pathogenic fun­ gus. Kerions may be complicated by permanent scarring. Twisted hairs, often in association with thinning of the eyebrows, are indicative of self-induced plucking of hairs (trichotillomania). Question 8-3 What is the treatment of choice for tinea capitis? A) Griseofulvin. B) Itraconazole. C) Ketoconazole. D) Methylprednisolone. E) Terbinafine. Discussion 8-3 The correct answer is "E:' Tinea capitis is treated with oral anti­ fungal medication. Terbinafine has emerged as the treatment of choice for tinea capitis, as it demonstrates superior efficacy. Historically, griseofulvin was used. However, a longer treatment course is required and emerging resistance has made this a second-line therapy. Azole antifungals may be effective but have associated safety concerns, particularly with oral solutions.

� QUICKQUIZ Dermatophytes require what for growth? A) Keratin. B) Fibrin. C) Elastin. D) Collagen. E) None of the above. Discussion The correct answer is ''A:' Dermatophytes are fungi that metab­ olize keratin for growth. The skin, nails, and hair are affected as keratin is the major protein of these structures. •

Helpful Tip

� Tinea i n fections may present with a general ized i1 1 r pru ritic rash referred to as autoeczematization reactions =

F I G U R E 33- 1 3. Tinea capitis is a fu ngal i nfection o f t h e sca l p that presents as single or mu lti ple a reas of scaling hair loss. Erythema, lymphadenopathy, and pruritus may occur. (Reprod uced with permission from Wolff K, Johnson RA, and

Saaved ra AP, eds. Fitzpatrick's Color Atlas and Synopsis of Clinical Dermatology, 7th ed. McGraw- H i l l Education, I n c., 201 3. Fig. 26-43.)

or id reactions. Dermatophytid reactions ca n occur with

t i n ea pedis, corporis, capitis, or cruris. M a n a g e m e n t i nvolves treatment o f the pri m a ry fu n g a l i nfection.

CHAPTER 33



S KI N D I SO R D E R S

737

(eg, lip, ear, or nose hemangiomas) , obstruction of vision, ulceration, and psychosocial distress. A 3-month-old girl presents with a rapidly growing bright red papule on the forehead. (See Figure 33- 14.} It was first noted at 1 month of age. Question 9- 1 How do you counsel the family regarding the expected clini­ cal course? A) It will continue to grow until age 1 , with spontaneous reso­ lution by age 10. B) It will continue to grow until age of 3, with spontaneous resolution by age 1 5 . C ) Th e vascular lesion i s replaced b y prominent scarring in most cases. D) Ulceration and bleeding are typical for the majority of hemangiomas. E) It will not change. Discussion 9- 1 The correct answer is "A:' Infantile (strawberry or capillary} hemangiomas occur in up to 1 0 % of Caucasian infants, and are more common among female and premature infants. They may be superficial bright red papules or deep bluish compressible nodules or plaques. The maj ority are not visible at birth. They grow in the first 3 to 12 months of life, followed by slow resolution, with the maj ority resolv­ ing by age 9 or 10 years. Residual telangiectasias, scarring, or fibrofatty tissue changes may be left behind. Indications for treatment include potentially life-threatening loca­ tions ( including airway hemangiomas ) , disfigurement



A 2-month-old girl presents with a vascular plaque on the chin that has been present since birth. Parents report it has just started growing. Question 1 0-1 What is your next step in management of this infantile hemangioma? A) Otolaryngology consult. B) Follow-up in 6 months. C) Interferon. D) Magnetic resonance imaging (MRI) to define the extent of the lesion. E) Oral propranolol. Discussion 1 0- 1 The correct answer is "A:' Hemangiomas in a 'beard' distribu­ tion (lower lip, chin, mandible or neck) may be a marker for potentially life threatening airway hemangiomas. Referral to otolaryngology (ENT} is essential. Symptoms of stridor, hoarse­ ness, and cough typically occur in the first few months of life concurrent with growth of the lesion. Be aware, not all children with an airway hemangioma have cutaneous lesions. Lumbo­ sacral hemangiomas may be a marker for spinal dysraphism and should be evaluated with MRI of the spine. Large segmen­ tal facial hemangiomas are associated with PHACES syndrome (Posterior fossa malformations, Hemangioma, Arterial anoma­ lies, Cardiac anomalies and aortic coarctation, Eye abnormali­ ties, and Sternal clefting or supraumbilical abdominal raphe) . Oral propranolol has replaced systemic corticosteroids and interferon for management of complicated hemangiomas. Ulcerated hemangiomas may be treated with pulsed dye laser and wound care. Topical beta-blockers are reserved for superfi­ cial infantile hemangiomas .

.2% QUICKQUIZ Which is NOT a potential side effect of propranolol therapy for hemangiomas? A) Hypoglycemia. B) Hypotension. C) Bradycardia. D) Bronchodilation. E) All of the above.

F I G U R E 33- 1 4. I nfa nti le hemangiomas present after birth as s u perficial bright red ra ised lesions. Deep lesions have a b l u i s h hue. (Used with perm ission from Wi l l i a m Augehenbaugh, M D.)

Discussion The correct answer is "D:' Propranolol may cause bronchospasm. Oral propranolol peaks 1 to 3 hours after it is taken. Patients should be monitored during this time period when starting or increasing therapy.

MCG RAW- H I LL E D U CAT I O N S P E C I A LTY BOA R D REVI EW: P E D I AT R I C S

738



Helpful Tip

� S u b g l ottic

=

r1 1r

hemangiomas

a re

often

m i sta ke n l y

d i a g nosed as cro u p (a vira l i nfection) a n d treated with steroids. Beca use stero i d s treat the h e m a n g ioma, the patient i m proves tem porari ly.

A 2-month-old girl presents with a dusky purple patch involv­ ing the right forehead, upper and lower eyelids, and cheek in a trigeminal nerve ophthalmic branch (VI } distribution. Question 1 1 - 1 What i s the appropriate workup for this infant? A) Echocardiogram. B) ENT evaluation. C) Magnetic resonance angiogram (MRA) of the brain. D) MRI of the brain and eye exam. E) No workup is necessary as the birthmark will fade in the first few years of life. Discussion 1 1 - 1 Th e correct answer i s "D:' Port-wine stains present at birth with pink to dark red coloration. When located in a V 1 distribution, they may be associated with Sturge-Weber syndrome, charac­ terized by glaucoma and leptomeningeal angiomatosis leading to risk of seizures. Ophthalmologic manifestations (glaucoma) occur in approximately 60% of patients with Sturge-Weber syn­ drome and 20% of those with an isolated port -wine stain; there­ fore, an eye exam is an important aspect of care for a patient with a port-wine stain in a V1 distribution. MRI is the preferred imaging modality to evaluate for leptomeningeal angiomatosis, and the findings are usually ipsilateral to the lesion. Treatment of uncomplicated lesions is not mandatory. Laser treatments may help reduce the purpura and progressive thickening that are expected over time.





1 1 1r

Helpful Tip Sturge - Weber syn d ro m e is associated with a port- w i n e sta i n on the face in the d i stribution of the trig e m i n a l nerve. (See F i g u re

33-1 S.)

A 2-day-old healthy full-term Caucasian boy presents with papules and pustules and surrounding erythema on the face, trunk, and extremities. The parents report that the bumps were not present at birth. Question 1 2- 1 What is the most likely diagnosis? A) Erythema toxicum neonatorum. B) Neonatal acne.

F I G U R E 33- 1 5. Stu rge-Weber Syndrome. Port-wine sta i n s in a distribution of cra n i a l nerve V (trigeminal) warrant eva l uation for Stu rg e-Weber synd rome or g l a u coma, or both. This boy has Stu rge-Weber syndrome with a

port-wine sta i n i nvolving Vl (ophth a l m i c bra nch) a n d V2 (maxi l l a ry branch). (Reproduced with perm ission from Brun icard i FC, Andersen DK, Billiar TR, et al, eds. Schwartz's Principles of Surgery, l Oth ed. McGraw- H i l l Education, I n c., 201 5 . Fig. 45-246.)

C) Neonatal herpes. D) Neonatal impetigo. E) Transient neonatal pustular melanosis. Discussion 1 2- 1 Th e correct answer i s "A:' Erythema toxicum neonatorum i s a common disorder, occurring in up to 70% of healthy full-term Caucasian newborns characterized by pustules on an erythem­ atous base. Rarely present at birth, it occurs most commonly in the first 3 to 4 days of life, and can last up to approximately 2 weeks. The lesions may involve the trunk, extremities, or face but spare the palms and soles. (See Figure 3 3 - 1 6.} Tran­ sient neonatal pustular melanosis (TNPM) occurs in less than 1 % of newborns, most commonly in black infants. It presents at birth with pustules that heal with a collarette of scale sur­ rounding hyperpigmented macules. The palms and soles may be involved. This hyperpigmented macule resolves over weeks to months. (See Figure 3 3 - 1 7.} Neonatal acne, now referred to as neonatal cephalic pustulosis, is not due to androgen stimulation of the oil glands, as occurs in infantile acne ( usu­ ally developing at 3 months of life) . Papules and pustules are confined to the face and develop at age 3 weeks or older. Neo­ natal herpes is one not to miss. In utero infection with lesions at birth is rare. Typical onset is at 2 weeks of life with the development of weak vesicles on an erythematous base that

CHAPTER 33



S KI N D I SO R D E R S

739

i QUICKQUIZ A smear from a pustule of erythema toxicum stained with Giemsa would reveal: A) Bacteria. B) Eosinophils. C) Lymphocytes. D) Neutrophils. E) Only cellular debris.

FIGURE

33- 1 6. Erythema Tox i c u m Neonatoru m . Erythema tox i c u m is a common ben i g n newborn ras h c h a racterized b y pustu les o n a n erythematous base. C l ues i n c l ude s p a r i n g o f t h e pa l m s a n d sol es, deve l o p m e nt after b i rth, and presence of eos i n o p h i l s i n lesions. (Reprod uced with perm ission from G o l d s m ith LA, Katz 51, G i l c h rest BA, Pa l l e r AS, Leffe l l DJ, Wolff K, eds. Fitzpatrick's Dermatology in General Medicine. 8th ed. New York, NY: McGraw- H i l l Education; 2 0 1 2, Fig. 1 07-3.)

cluster and coalesce. Neonatal impetigo is a superficial bacte­ rial infection of the skin that can occur in the first few days to 2 weeks of life. It presents clinically as vesicles, pustules, or bullae on an erythematous base. These areas rupture easily and leave superficial erosions and crusting. Common locations of involvement are the skinfolds, such as the neck fold, axilla, and diaper regions.

A

Discussion The correct answer is "B:' A smear from the pustular fluid, stained with Wright or Giemsa stain, will reveal eosinophils. In contrast, a smear from a pustule in TNPM shows neutrophils, cellular debris, and occasionally a few eosinophils. Why do you need to know this? For three reasons: ( 1 ) it may be on the board exam, (2) to confirm the diagnosis, and (3) to stump residents and medical students on rounds.

� QUICKQUIZ What is the most common cause of a gray patch of the lumbo­ sacral region in an African-American neonate? A) Congenital nevus. B) Dermal melanocytosis. C) Nevus of Ito. D) Traumatic contusion. E) Vascular hemangioma.

8

F I G U R E 3 3 - 1 7. Tra nsient neonatal pustu l a r melanosis is a newborn ras h cha racterized by pustules that ruptu re, leaving a hyperpigmented macule. Clues

include presence at birth, i nvolvement of the pa l m s a n d soles, black race, a n d presence of neutro p h i l s i n lesions. (Reproduced with permission from Goldsmith LA, Katz 51, G i l c h rest BA, Pa l ler AS, Leffe l l DJ, Wolff K, eds. Fitzpatrick's Dermatology in General Medicine. 8th ed. New York, NY: McGraw- H i l l Education; 201 2, Fig 1 07-4A, B.)

740

MCG RAW- H I LL E D U CAT I O N S P E C I A LTY BOA R D REVI EW: P E D I AT R I C S

F I G U R E 33- 1 9. Cafe-a u-Lait M a c u l es. Two brown ovoid lesions cons istent with cafe-a u-la it m a c u l es a re present o n this c h i l d . M u l t i p l e lesions s h o u l d ra ise the s u s p i c i o n o f n e u rofi brom atos i s type 1 . (U sed w i t h perm ission fro m Wi l l i a m Augehenbaugh, M D.)

FIGURE

33- 1 8. D e r m a l Mela nocytosis. M u ltiple b l u i s h plaq ues a re present on the back a n d buttocks of this Asian c h i ld, consistent with dermal melanocytosis or mongolian spots. (Reproduced with perm ission from Goldsmith LA, Katz 51, G i l c h rest BA, Pa l l e r AS, Leffe l l DJ, Wolff K, eds. Fitzpatrick's Dermatology in General Medicine. 8th ed. New York, NY: McGraw- H i l l Education; 201 2, F i g . 9- 1 1 .)

Discussion The correct answer is "B:' Dermal melanocytosis (mongolian spot) is a benign melanocytic lesion that occurs in more than 90% of African-American infants, more than 80% of Asian infants, 70% of Hispanic infants, and close to 1 0 % of Cauca­ sian infants. The lesions appear as gray, blue-black, or brown patches, most commonly over the lumbosacral back and buttocks. (See Figure 3 3 - 1 8 . ) Most are present at birth and fade in the first 2 to 3 years of life, although occasionally they persist into adulthood.

A 5-year-old girl presents with five cafe-au-lait macules (CALMs) on the trunk, ranging from 5 to 9 millimeters (mm) in diameter. She is referred for possible neurofibromatosis. Question 1 3- 1 What additional physical finding i s most likely i n a child this age to support the diagnosis of neurofibromatosis type 1 (NF I ) ? A ) Axillary freckling. B) Neurofibroma. C) Lisch nodule. D) Optic glioma. E) None, as there are an insufficient number of CALMs for a diagnosis of NF l .

Discussion 1 3- 1 Th e correct answer i s "A:' Although NF 1 , also known a s von Recklinghausen disease, is an autosomal dominant disorder, approximately 50% of cases are due to new mutations. In order to diagnose the disorder, the patient must have two or more of the following: •



Six CALMs 5 mm or larger in prepubescent patients and 1 . 5 centimeters (em) or larger after puberty (see Figure 33- 19) Axillary or inguinal freckling



Plexiform neurofibroma



Two or more dermal neurofibromas



Two or more Lisch nodules



Optic glioma



Skeletal dysplasia (tibial or sphenoid wing dysplasia)



First degree relative with NF I

Although this patient had only five CALMs, more can develop over time. Axillary and inguinal freckling usually occurs by age 3 to 5 years old, affecting 20% to 50% of individuals with NF l . Lisch nodules (iris hamartomas) affect more than 90% of patients with NF I and have a postpubertal onset. Optic gliomas occur in 1 5 % of children with NF 1 and approximately one-third develop signs or symptoms, including decreased visual acuity, visual field defects, proptosis, or strabismus. Annual eye exam until age 1 0 is advised. Neurofibromas occur in later childhood and adolescence.

A 1 0-year-old boy presents for treatment of shiny, pink papules on the face. They were noted at age 6 years. Neither pustules nor comedones are seen on physical exam. (See Figure 33-20.)

CHAPTER 33



S KI N D I SO R D E R S

741

F I G U R E 33-20. A n g i ofi brom a s looking l i ke s h i ny, p i n k p a p u l e s a re present on the nose a n d m a l a r reg ion of t h i s c h i l d with tu berous scl eros is. A c l u e i s the lack of comedones. (Used with perm ission fro m Wi l l i a m Augehen b a u g h , M D.)

Question 1 4- 1 What i s your next step? A) Topical benzoyl peroxide. B) Oral isotretinoin. C) Oral antibiotic. D) Shave biopsy. E) Topical metronidazole. Discussion 1 4- 1 Th e correct answer i s "D:' Angiofibromas may be mistaken for inflammatory acne papules. The shiny surface, onset between 2 and 6 years, and absence of comedones helps in distinguishing angiofibromas from acne.

F I G U R E 33-2 1 . Ash-Leaf Spots. A hypopigmented ash-l eaf macules a re

present on the skin of this c h i l d with tu berous sclerosis. (Reproduced with perm ission from Goldsmith LA, Katz Sl, G i l c h rest BA, Pa l l e r AS, Leffe l l DJ, Wolff K, eds. Fitzpatrick's Dermatology in General Medicine. 8th ed. New York, NY: McGraw- H i l l Education; 201 2, Fig. 1 6-2A.)

see. Collagenomas ( Shagreen patches) typically present on the trunk during late childhood. They are flesh- colored subtle plaques with an "orange p eel" surface. Periungual fibromas occur in or after puberty around the fingernails and toenails. Dental pits typically occur in adulthood. (See Figure 3 3 - 2 2 . )

You perform a biopsy due to your suspicion that the facial papules represent angiofibromas of tuberous sclerosis. Question 1 4-2 What additional finding ( s ) on physical exam is/are expected with tuberous sclerosis in a child this age? A) Dark macules of the iris. B) Dental pits. C) Hypopigmented macule of the trunk. D) Skin-colored papule of the nail fold. E) None of the above. Discussion 1 4-2 The correct answer is "C:' Hypopigmented macules, referred to as "ash-leaf spots;' are usually present at birth. They may be lance- ovate resembling an ash leaf, small confetti-like, or oval in shape. (See Figure 3 3 - 2 1 . ) Use of a Wood lamp accentuates the lesions, which may be otherwise hard to



� 1 1 1r

Helpful Tip Syste m i c m a n ifestations o f tu bero u s scl erosis i n c l u d e seizu res, cortical tu bers, s u bependym a l nod u l es or g i a nt c e l l astrocytomas, ca rd iac rha bdomyo m a s, re n a l a n g i o myo l i po m a s, m u l t i p l e ren a l cysts, m u lt i p l e ret i n a l h a m a rtomas, a n d lym p h a n g i o l eio myo matosis, a mo n g others.

A 9-month-old boy presents with fevers of unknown origin. His parents report that he does not sweat with fevers or after exposure to hot weather. Physical examination reveals peri­ orbital wrinkling, frontal bossing, a thick everted lip, and small low-set ears.

MCG RAW- H I LL E D U CAT I O N S P E C I A LTY BOA R D REVI EW: P E D I AT R I C S

742

A

B

F I G U R E 33-22. Cuta neous fi n d i n g s of tu berous scl erosis. (A) Peri u n g u a l fi broma. (B) Shagreen patches a re connective tissue nevi made of collagen that a ppea r

as ski n-colored lesions with an orange peel textu re. (Reprod uced with permission from Goldsmith LA, Katz 51, G i l c h rest BA, Pa l l e r AS, Leffel I DJ, Wolff K, eds.

Fitzpatrick's Dermatology in General Medicine. 8th ed. New York, NY: McGraw- H i l l Education; 201 2, Fig. 1 6-SA, B.)

Question 1 5- 1 What additional feature supports your diagnosis o f hypohi­ drotic ectodermal dysplasia? A) Calcinosis cutis. B) Nail dystrophy. C) Numerous nevi. D) Short sparse hair and abnormal dentition. E) Thick, protuberant tongue. Discussion 1 5- 1 Th e correct answer i s "D:' This child has hypohidrotic (reduced sweating) ectodermal dysplasia, which is an X-linked reces­ sive disorder affecting males. Patients have absent or reduced sweating, abnormal dentition, characteristic facies, and alope­ cia. The reduced or absent ability to sweat may cause fevers. Gastroesophageal reflux is common. A concurrent immuno­ deficiency may occur in some cases. Treatment is aimed at avoiding overheating and cooling the skin. Multidisciplinary care, including dentistry, is important. The most common form of hidrotic ectodermal dysplasia is characterized by the triad of sparse, brittle hair, nail dystrophy, and palmoplan­ tar hyperkeratosis. It is inherited in an autosomal dominant fashion. This condition is associated with neither craniofacial abnormalities nor reduced sweating. Teeth are normal but are more likely to develop caries.



Helpful Tip

:5.� Ectod e r m a l

r1 1r

dys p l a s i a

is

a

g ro u p of d i sorders

A 14-year-old girl presents with a verrucous hairless plaque on the scalp. It was present at birth as a subtle yellow plaque that has gotten thicker over the past 1 to 2 years. Question 1 6- 1 What i s the most likely diagnosis? A) Aplasia cutis congenita. B) Congenital nevus. C) Nevus sebaceous. D) Seborrheic keratosis. E) Pilomatricoma. Discussion 1 6- 1 Th e correct answer i s "C:' Nevus sebaceous, a congenital ham­ artoma of the skin, usually presents at birth on the scalp or face and can be yellow or tan in color. It grows in proportion with the child and becomes thicker and more warty around puberty. Benign hyperplasia can occur within nevus sebaceous, but the risk of malignant growths is rare. (See Figure 33-23.) Aplasia cutis congenita affects the scalp. At birth, it is a circular open lesion that heals as a smooth, atrophic scar. Congenital nevi are typically pigmented with a cobbled surface. Pilomatricomas are hard, skin-colored to white nodules of the face and upper extremities. A nevus sebaceous is a warty, light-colored growth that appears stuck on the skin and occurs in adults.

in

w h i c h pa rts of the ectoderm, skin, teeth, h a i r, n a i ls, sweat g l a n d s, o r a c o m b i nation of th ese, d o not fo rm

co rrectly. The

d i sorders

a re

categ orized

as

hypo h i d rotic/a n h i d rotic (no or red uced sweating) and h i d rotic ( n o r m a l sweati n g ) .

A 2-year-old boy presents with a 6-month history of multiple brown macules of the trunk. His mother notes that a wheal forms when the lesions are scratched.

CHAPTER 33



S KI N D I SO R D E R S

743

F I G U R E 33-23. Nevus Sebaceous. On the sca l p is a yel low, thickened, wart l i ke lesion consistent with a nevus sebaceous. (Used with permission from Wi l l i a m Augehenba ugh, MD.)

Question 1 7- 1 What additional symptom supports your diagnosis of mastocytosis? A) Cough. B) Diarrhea. C) Headache. D) Hematuria. E) Hyperhidrosis. Discussion 1 7- 1 The correct answer is "B:' Cutaneous mastocytosis refers to collec­ tions of mast cells in the skin. Solitary mastocytoma presents as a single tan plaque, whereas urticaria pigmentosa is characterized by multiple tan to brown macules or papules. (See Figure 33-24.) Bullae may occur with either condition. A positive Darier sign is expected with all subtypes of mastocytomas, whereby stroking the skin leads to erythema and urticaria of the lesion. Mast cells may also collect in the gut. When mast cells are stimulated to degranulate, histamine release leads to urticarial wheals or diar­ rhea, depending on where the cell are located. There is no cure for mastocytosis. Treatment is directed at symptomatic relie£ Anti­ histamines are the mainstay of treatment. Topical or oral cortico­ steroids and phototherapy are rarely necessary.

� QUICKQUIZ

F I G U R E 33-24. Cuta neous Mastocytosis. Mast ce l l s col l ect in the skin to form brown to ta n macules i n mastocytosis. This patient has u rticaria pigmentosa, with m u ltiple mastocytomas present. (Used with perm ission from Wi l l i a m Augehenbaugh, M D.)

aspirin, nonsteroidal anti-inflammatory drugs (NSAIDs), nar­ cotics, egg whites, chocolate, strawberries, tomatoes, and citrus as these may stimulate mast cell degranulation.

A newborn develops blisters and erosions in areas of trauma. No scarring is noted on physical exam. You suspect epider­ molysis bullosa simplex.

Which is NOT a trigger of mast cell activation and degranulation? A) Spicy foods. B) Hot shower. C) Acetaminophen. D) Opioid medications.

Question 1 8- 1 What is appropriate counseling for the family? A) All types are associated with high mortality rate. B) Basal cell carcinomas are a common complication. C) Electron microscopy is required to subtype this disorder. D) Epidermolysis bullosa simplex is likely to resolve spontaneously. E) Scarring is common to all phenotypes of the disorder.

Discussion The correct answer is "C:' In mastocytosis, disease triggers are non-IgE mediated, such as stress and temperature (hot showers) . Patients severely affected by mastocytosis should avoid alcohol,

Discussion 1 8- 1 Th e correct answer i s "C:' Epidermolysis bullosa i s divided into three subtypes: simplex, junctional, and dystrophic. Each sub­ type has mild and severe phenotypes, depending on the defective

MCG RAW- H I LL E D U CAT I O N S P E C I A LTY BOA R D REVI EW: P E D I AT R I C S

744

or absent structure required to maintain cell-to-cell adhesion. Inheritance varies according to the different phenotypes. Most types show skin fragility leading to bullae and erosions in the newborn period. Additional findings may include growth retar­ dation, nail dystrophy, milia, scarring, and oral erosions. In the more severe subtypes, scarring may lead to mitten hand defor­ mities, joint contractures, and esophageal strictures. Electron microscopy and immunomapping are important for accurate diagnosis. In the recessive dystrophic subtype, squamous cell carcinoma occurs in 40% of patients by the age of 30 and can progress rapidly, resulting in death. Recent advances in gene therapy are revolutionizing treatment of epidermolysis bullosa.

A full-term girl is born with vesicles in a linear distribution on the arm. She is otherwise well. Workup for neonatal her­ pes and bullous impetigo are negative. You make the diagno­ sis of incontinentia pigmenti. Question 1 9- 1 How do you counsel the family about how the lesions will appear into adulthood? A) Atrophic plaques. B) Hyperpigmented patches. C) Hypopigmented patches. D) Scars. E) Verrucous papules. Discussion 1 9- 1 The correct answer i s "C:' Incontinentia pigmenti is a n X-linked dominant disorder that is lethal in males, although rare cases in patients with Klinefelter syndrome and somatic mosaicism have been reported. Neonates present with erythematous mac­ ules and vesicles in a linear and whorled distribution, following developmental lines of Blaschko. After several days to weeks, the vesicles resolve and are replaced by verrucous papules. These papules resolve leaving behind hyperpigmented macules in a swirled pattern. Over time, the hyperpigmentation is replaced by hypopigmented macules. Ocular, skeletal, and neurologic abnormalities may occur.



� I

1 1r

Helpful Tip The l i nes of Blasch ko a re u s u a l ly not visible. I n certa i n d iseases of the skin a n d m ucosa, they a p pea r as whorls

Question 20-1 What is your next step? A) Incision and drainage. B) MRI. C) Oral antibiotics. D) Punch biopsy. E) Surgical excision. Discussion 20- 1 The correct answer is "B:' Dermoid cysts are congenital lesions that arise along embryonic fusion planes. They may not become noticeable until later in childhood when they enlarge and become inflamed or infected. The lateral eyebrow is a common location; however, they may also be located on the midline nose, scalp, neck, back, and sternum. Lesions on the midline nose and midline scalp have higher risk of intracranial extension; there­ fore, imaging is required before any intervention is pursued.

A 1 5-year-old boy presents with erythematous targetoid lesions of the arms. He has no active mucosal lesions. You elicit a history of herpes labialis 10 days prior to onset. Question 2 1 - 1 What i s the most likely diagnosis? A) Erythema multiforme major. B) Erythema multiforme minor. C) Leukocytoclastic vasculitis. D) Stevens-Johnson syndrome. E) Toxic epidermal necrolysis. Discussion 2 1 - 1 Th e correct answer i s "B:' Erythema multiforme i s a self-lim­ ited eruption characterized by lesions that resemble a target. (See Figure 33-25.) The rash most commonly follows or occurs concurrently with a herpes simplex virus (HSV) infection. An alternative term is erythema multiforme minor (EM minor) . The term erythema multiforme major (EM major) has been replaced by Stevens-Johnson syndrome. This syndrome is a severe inflammatory reaction to medication, infection, or underlying inflammatory disease. Stevens-Johnson syndrome and toxic epi­ dermal necrolysis are variants of the same disease. Leukocytoclas­ tic vasculitis may also arise from infection or medication and is a nonblanching, purpuric papule. This condition may lead to renal dysfunction, so close monitoring is imperative. (See Table 33-6.)

or wave l i ke sha pes on the s ki n .

A healthy 2-year-old boy presents with a 2 e m nodule o n the midline nasal root. It was noted in infancy but has recently enlarged. You suspect a dermoid cyst. His parents request removal.

A 1 2-year-old boy with epilepsy develops acute-onset mal­ aise, fever, and painful skin 2 weeks after starting an anticon­ vulsant. The eruption starts on his trunk and then spreads to the entire body, resulting in widespread skin sloughing. You admit him to the hospital.

CHAPTER 33



S KI N D I SO R D E R S

745

Question 22-1 What finding helps support your diagnosis of toxic epidermal necrolysis (TEN) ? A) Angioedema of the lips. B) Facial edema. C) Hemorrhagic crust on the lips. D) Normal ocular conjunctivae. E) Urticarial plaques.

.:.. ,...

I



.

.. ,



F I G U R E 33-25. Erythema M u ltiforme M i nor. M u ltiple, confl uent, ta rget­

l i ke papules on the face of a 1 2-year-old boy. The target morphology of the lesions is best seen on the l i ps. (Reproduced with permission from Wolff K, Johnson RA, Saavedra AP, eds. Fitzpatrick's Color Atlas and Synopsis of Clinical Dermatology. 7th ed. McGraw- H i l l Education; 201 3, Fig. 1 4- 1 5.)

Discussion 22- 1 The correct answer is "C:' Stevens-Johnson syndrome (SJS) and TEN result in a febrile illness with painful exanthems and mucosal erosions that may involve the mouth, eyes, gastrointestinal tract, respiratory tract, or urethra. (See Figures 33-26 and 33-27.} The epidermis becomes necrotic, with sloughing of the skin. Nikol­ sky sign (stroking of the skin causes blistering) may be positive. Both manifestations are considered part of the continuum of the same disorder. SJS is associated with less than 10% body surface area (BSA) involvement, SJS-TEN with 10% to 30% BSA involve­ ment, and TEN with greater than 30% BSA involvement. {See Table 33-6, earlier.) The most common findings are the result of a severe reaction to medications (sulfa antibiotics, anticonvulsants, and allopurinol) . Mycoplasma pneumoniae is a known trigger. SJS and TEN may be life threatening and require hospitalization.

TABLE 33-6 COM PA R I S O N S OF E RYTHEMA M U LT I F O R M E, STEV E N S-J O H N S O N SYN DRO

�, A N D TOX I C

E P I D E R M A L N E C ROLYS I S

Synonym/Acronym

Erythema M u ltiforme

Stevens-Johnson Synd rome

Toxic Epiderma l Necrolysis

EM m i nor

EM major (outdated term)

TEN

SJ S Etiology

Herpes s i m plex virus (SO%)

Histoplasma

Med i cations: NSAI Ds, a nticonvu lsa nts, s u lfo n a m i d e a n d penici l l i n a ntibiotics I nfection: Mycoplasma pneumoniae, fu n g a l i nfections, cytomega lovi rus I nfla m m atory bowel d i sease

C l i nical fi n d i n g s

Treatment

No syste m i c prod rome

Prod rome

Prod rome

Ta rgetoid lesions

Foca l d u s ky red patches

Large d u s ky red patches

Ora l e rosions, typica l ly m i ld

Hemorrhagic crust on l i ps

Hemorrhag ic crust on l i ps

Ora l erosions, stomatitis

Ora l erosions, stomatitis

Conj u nctivitis

Conj u nctivitis

Anogenital erosions

Anogenital erosions

I nvolved BSA < 1 0%

I nvolved BSA > 30%

None req u i red

Disconti n u e offe n d i n g d rugs

S u p p ressive ora l a ntivi ra l for recurrent epi sodes

Treat of i nfection if present

Ad mit to hos pita l (burn u n it for TEN) S u p portive ca re Ophth a l mology cons u lt

Morta l ity

U p to 30%

BSA, body su rfa ce a rea; NSAI Ds, n o n stero i d a l a nti-infl a m m atory medications.

25-50%

MCG RAW- H I LL E D U CAT I O N S P E C I A LTY BOA R D REVI EW: P E D I AT R I C S

746

A 1 5-year-old healthy boy presents with crusting on the arm after falling off his skateboard 2 weeks earlier. {See Figure 33-28.) The erosion has not yet healed. Question 23-1 Which test is most appropriate to confirm the diagnosis? A) Bacterial culture. B) Biopsy. C) Fungal culture. D) Tzanck preparation. E) None of the above. F I G U R E 33 -26. Stevens-Johnson Syndrome. T h e skin b l i sters a n d sloughs i n Stevens-Johnson syndrome as a result of epidermal necrosis. (Reproduced with permission from Grippi MA, Elias JA, Fishman JA, Kotloff RM, Pack AI, Senior RM, eds. Fishman's Pulmonary Disease and Disorders. 5th ed. New York,

NY: McGraw- H i l l Education; 201 5, Fig. 29-36.)



Helpful Tip

:5.� SJS

r1 1 r

and

TEN

a re febri le

exa nthems

(rashes)

and

enanthems (mucosal eruptions). Look for erythematous lesions with a d u s ky center that become b u l lae, coa lesce, and ru ptu re.

Discussion 23-1 The correct answer is ''A:' Impetigo is an infection of the epider­ mis primarily caused by Staphylococcal aureus. Primary impe­ tigo results from bacterial invasion of normal skin and affects mainly toddlers. The face is a popular target, especially the chin and around the nose. Secondary impetigo commonly occurs following trauma, when the normal skin barrier is disrupted. Staphylococcal or streptococcal bacteria, or both, populate the wound base and impede healing. The classic appearance is a honey-colored crust. Lesions progress from papules to vesicles to pustules that rupture and become crusted. Although a bacte­ rial culture may be performed, empiric treatment is common if the clinical appearance supports the diagnosis of impetigo. A KOH examination is used to diagnose a superficial fungal infec­ tion. Fungal culture is rarely necessary. A Tzanck preparation is used to diagnose herpes infections.

You confirm the diagnosis of impetigo based on physical exam.

F I G U R E 33-27. M ucositis a n d Stevens-Johnson Syndrome. In this patient with Stevens-Johnson synd rome, the ora l mucosa and l i ps have pai nfu l hemorrhagic erosions. A g ray-wh ite mem bra ne may cover the lesions. (Reproduced with permission from Grippi MA, Elias JA, Fishman JA, Kotloff RM, Pack AI, Senior RM, eds. Fishman's Pulmonary Disease and Disorders. 5th ed. New York, NY: McGraw- H i l l Education; 20 1 5, Fig. 29-37.)

F I G U R E 33-28. I m petigo is a s u perficial bacteria l i nfection of the skin. I n

t h e n o n b u l l o u s form, pa p u l es become vesicles that ru ptu re, forming a honey-colored crust. Notice the thick honey-colored crust with s u rro u n d i n g erythematous pustules i n this patient. (U sed w i t h permission from Wi l l i a m Augehen baugh, M D.)

CHAPTER 33



S KI N D I SO R D E R S

747

Question 23-2 What is your recommended treatment? A) Bacitracin ointment. B) Cephalexin. C) Dicloxacillin. D) Erythromycin. E) Mupirocin ointment. Discussion 23-2 The correct answer is "E:' Topical agents are most appropriate for nonbullous impetigo, which is a superficial infection. Mupi­ rocin (Bactroban) is the treatment of choice. Oral antibiotics are typically reserved for bullous impetigo. Bacitracin provides little antibacterial effect and may be complicated by allergic contact dermatitis.





1 1 1r

Helpful Tip I m petigo lesions a re very conta g i o u s . Re m i n d fa m i l y m e m bers to wash t h e i r h a n d s a n d avoid tou c h i n g the

F I G U R E 33-29. B u l lous i m petigo i s less common that primary i m petigo a n d affects mainly c h i l d ren. Fl accid bullae form a n d fill with cloudy fl uid. The b u l l a e ru ptu re, leaving a n erythematous base with pee l i n g rim a n d brown crust. (Reproduced with perm ission from Goldsmith LA, Katz 51, G i l c h rest BA, Pa l l e r AS, Leffel I DJ, Wolff K, eds. Fitzpatrick's Dermatology in General Medicine. 8th ed. New York, NY: McGraw- H i l l Education; 201 2, Fig. 1 77-2.)

lesions.

� QUICKQUIZ Which of the following statements regarding impetigo is true? A) Bullous impetigo is an infection of the dermis. B) Bullous impetigo forms bullae on the skin. C) Bullous impetigo is more common than nonbullous impetigo. D) Bullous and nonbullous impetigo are characterized by a honey-colored crusting. E) Bullous impetigo and nonbullous impetigo are more com­ mon in adults. Discussion The correct answer is "B:' It seems intuitive that bullous impe­ tigo forms bullae. Impetigo may be bullous or nonbullous, but both are epidermal infections that primarily affect young chil­ dren. Nonbullous impetigo is the more common papulovesicu­ lar rash that forms a honey-colored crust. It can be caused by staphylococcal or streptococcal bacteria. In bullous impetigo, flaccid bullae form and rupture, leaving an erythematous base with a peeling rim and brown crust. (See Figure 33-29.) It is caused by toxin-producing Staphylococcus aureus.

A 5-year-old previously healthy boy is hospitalized with a 3-day history of malaise, fever, and generalized erythema. He complains of skin pain. He was given acetaminophen (Tylenol) for the past 3 days without relief. Examination is notable for flaccid bullae and desquamation of the chest and

axillae. Oral, ocular, and genital mucosae are clear. Touching the skin causes it to slough. Question 24- 1 What is the most likely diagnosis? A) Bullous impetigo. B) Morbilliform drug eruption. C) Pemphigus vulgaris. D) Staphylococcal scalded skin syndrome. E) Toxic epidermal necrolysis. Discussion 24- 1 The correct answer is "D:' Staphylococcal scalded skin syndrome (SSSS) is caused by an exotoxin secreted by Staphylococcus aureus. This toxin targets the proteins that bind keratinocytes (epidermal skin cells) together. Disruption of these bonds leads to superficial sloughing of the skin. (See Figure 33-30.) Bullous impetigo arises as a single or multiple cutaneous erosions, as the blisters rupture easily. It has the same etiology as SSSS (toxin mediated) but is more localized and does not cause diffuse ery­ thema. Morbilliform drug eruptions present as erythematous macules and papules that start on the trunk and spread to the extremities. Pemphigus vulgaris and toxic epidermal necrolysis (TEN) lead to blisters at the level of the dermoepidermal junction, resulting in higher morbidity and mortality. Both pemphigus vulgaris and TEN are associated with severe mucosal involvement.



Helpful Tip

:5.� Pa i nfu l

r1 1r

s k i n s h o u l d a l ways be ta ken seriously. Differ­

ential d i a g nosis of pa i nfu l s k i n i n c l u d es SSSS, Steve ns­ J o h nson!TEN, pem p h i g u s vu l g a ris, and therma l b u r n s .

748

MCG RAW- H I LL E D U CAT I O N S P E C I A LTY BOA R D REVI EW: P E D I AT R I C S

8

A F I G U R E 33-30. Sta phylococcal Sca lded Skin Syndrome. This i nfa nt devel oped sta phylococcal scalded skin syndrome from periora l i m petigo. The erythematous skin forms b u l l a e and sloug hs, leaving l a rge a reas of denuded skin. (Reproduced with perm ission from Goldsmith LA, Katz 51, G i l c h rest BA, Pa l l e r AS, Leffe l l DJ, Wolff K, eds. Fitzpatrick's Dermatology in General Medicine. 8th ed. New York, NY: McGraw-H i l l Education; 201 2, Fig 25-40A, B.)

Question 24-2 Which test do you perform to confirm your suspicion of SSSS? A) Biopsy. B) Blood culture. C) Culture of axillary vault. D) Culture of a bullous lesion. E) No tests are necessary as the diagnosis is made clinically. Discussion 24-2 The correct answer is "E:' Acute onset of painful skin, bul­ lae, and skin sloughing is classic for SSSS. Imagine a really painful sunburn. SSSS typically affects neonates and children younger than 5 years of age. It starts with fever and diffuse tender erythema of the skin, especially around the mouth, eyes, axilla, and groin. Fissuring occurs around the eyes, nose, and mouth. The skin sloughs with gentle pressure (posi­ tive Nikolsky sign ) , forms bullae, and later desquamates. (See Figure 33-30, earlier. ) Blood cultures are negative. Culture is of low yield, unless taken from the nares, nasopharynx, con­ junctivae, or purulent material on the skin. Conversely, skin culture is typically high yield in bullous impetigo. Treatment of SSSS includes intravenous antibiotics, wound care, and supportive care.



Helpful Tip

:5.� Loo k fo r

i1 1r

m u cosa l i nvolve m e n t to h e l p d iffe rentiate

sta phylococca l sca lded skin syn d ro m e fro m Steven s­ J o h nson syn d ro m e/toxic e p i d e r m a l necrolysis. M u co­ sitis i s present in SJS a n d TEN but not SSSS.

� QUICKQUIZ The Nikolsky sign is positive in all of the following EXCEPT: A) TEN. B) Scarlet fever. C) SSSS. D) Pemphigus vulgaris. E) Bullous impetigo. Discussion The correct answer is "B:' The Nikolsky sign is not a perfect clin­ ical test but it can be helpful. The test is positive when stroking the skin causes it to tear and wrinkle in a fashion similar to tis­ sue paper. This indicates problems with the bonds between skin cells in the epidermis.

CHAPTER 33





1 7-year-old boy presents with discrete, pruritic papules of the arms and legs that have been present for 2 weeks. (See Figure 33-3 1 .} No other family members are affected. You suggest a diagnosis of papular urticaria. The patient disagrees, noting "this has been going on too long to be bug bites?' A

Question 25-1 How do you counsel him? A) Tell him all family members will eventually develop itchy bumps. B) Tell him antihistamines are the treatment of choice. C) Tell him insect bites may occur year round. D) Tell him he is at risk for anaphylaxis. E) Tell him permethrin is the treatment of choice. Discussion 25-1 The correct answer is "C:' The cutaneous reaction to arthropod (bug) bites is termed papular urticaria. Although most common in the summer and fall, insect bites may occur at any time of the year. Potential sources include outdoor exposure, pets, bird nests, and house plants, among others. Immediate reactions present as edematous wheals (eg, as in a mosquito bite) . Delayed reac­ tions are discrete, erythematous papules that may be clustered or widespread. Nodules and vesicles may occur. Recurrent or chronic papules may occur, especially in younger children. Because lesions may not appear immediately and insect bites may not have been noticed, convincing patients or families of this diagnosis can be a hard sell. Lesions arise from an immune reac­ tion to allergens transmitted by the insect. The reaction is depen­ dent on individual immunity, so a single family member may be the only one who reacts. Anaphylaxis is uncommon and is most frequently associated with bee, wasp, hornet, and ant bites. Treatment includes topical antipruritics (eg, menthol), mid- to high-potency topical steroids, intralesional steroid injections, and insect repellant. Permethrin is used for scabies treatment.

F I G U R E 33-3 1 . Pa p u l a r u rticaria is a delayed response to i n sect bites that

presents as m u ltiple erythematous, pruritic pa p u l es. (Used with perm ission from Wi l l i a m Augehenbaugh, M D.)

1 1 1r



S KI N D I SO R D E R S

749

Helpful Tip DEET is the most effective i n sect repe l l a nt. It may be a p p l ied d i rectly to s ki n or c l oth i n g . The A m e rican Aca demy

of

Pediatrics

concentration to 1 Oo/o to

2

s u g g ests

30%

l i m it i n g

D E ET

in c h i l d re n o l d e r t h a n

months o f age. DEET s h o u l d n o t be u s e d on i nfa nts

you n g e r than

2 months of age.

1 2-year-old girl returns home from summer camp noting severe itching, which is most pronounced at night. You note that papules and linear tracks are concentrated on her wrists and the web spaces of her hands.

A

Question 26- 1 What is the most appropriate next step? A) Biopsy to confirm diagnosis. B) Empiric treatment with a topical steroid. C) KOH examination. D) Mineral oil preparation. E) Tzanck preparation. Discussion 26- 1 The correct answer is "D:' Pruritic rashes that concentrate in web spaces and display linear burrows are classic for scabies. (See Figure 33-32.} A mineral oil preparation (scabies prep) is used to confirm the diagnosis. (See Table 33-7.} Scabies mites infest the stratum corneum, so they are readily detected by skin scraping, application of a drop of mineral oil, and observation of mites, eggs, or scybala (feces) under the microscope. Biopsy is usually unnecessary to diagnose scabies. Not everyone has immediate access to a microscope, so it is reasonable to treat empirically for scabies.

F I G U R E 33-32. In sca b i es, look fo r s m a l l eryt h e m ato u s p a p u l es a n d b u rrows (t h i n red l i nes) i n t h e s k i n , espec i a l l y o n the ve ntra l s u rface of the wri st, fi n g e r webs, axi l l a, peri u m b i l i c a l a rea, b uttoc ks, and feet. Exco riations may m a s k fi n d i n g s . ( U sed with perm i s s i o n fro m Wi l l i a m A u g e h e n b a u g h , M D.)

MCG RAW- H I LL E D U CAT I O N S P E C I A LTY BOA R D REVI EW: P E D I AT R I C S

750

TABLE 33-7 M I C ROSCO P I C D I A G N OSTIC SKIN TESTS

Diagnostic Skin Test

Associated Disease(s)

Location of Test

M icroscopic Findings

Potassi u m hyd roxide (KOH) exa m i nation

Ti nea corporis, ped i s, etc

Lea d i n g e d g e o f sca le

Septate hyphae

Pityriasis versicolor

Any i nvolved site

Hyphae and spores ("spaghetti and meatba l l s")

Tza nck preparation

Herpes s i m plex, zoster

Base of vesicle

M u ltinucleated g i a nt cel l s

M i nera l o i l preparation

Sca bies

Lea d i n g edge o f bu rrow

M ites, eggs, scyba la (feces)



� I 1 1r

Helpful Tip Itc h i ng, especi a l l y at nig ht, is the h a l l mark of sca bies. Remem ber, sca bies ta rgets everyone, is very contagious, a n d is d ifficult to era d i cate.

Question 26-2 What is your recommended treatment? A) Ivermectin for the patient, only. B) Ivermectin for the entire family. C) Permethrin from the neck down overnight x 1 dose. Treat the entire family. D) Permethrin from the neck down overnight x 1 dose. Treat the patient, only. E) Permethrin from neck down overnight. Repeat in 1 week. Treat the entire family.

Discussion The correct answer is "A:' Nodular scabies presents as dome­ shaped discrete nodules without visible burrows. This presen­ tation is more common among infants. (See Figure 33-33.) Crusted (Norwegian) scabies presents as erythematous patches with thick, adherent scale. Patients then develop crusting and fissuring. This variant is often asymptomatic despite infestation with hundreds of thousands of mites. It is more common among immunosuppressed patients.

You discuss the importance of environmental controls to prevent reinfestation.

Discussion 26-2 The correct answer is "E:' As scabies is transmitted through direct contact, all close contacts, including family members, should be treated. Permethrin is applied from the neck down, including finger and toe webs, left on overnight, and rinsed off in the morning. Scabies may infest the scalp and face of infants, so these areas should be treated. Scabies treatments kill live mites by paralyzing their breathing apparatus. Ivermectin is an oral medication that is reserved for widespread outbreaks or the emerging cases of permethrin resistance. The female fer­ tilized mites lay eggs inside the skin-kind of like a gift that keeps on giving. Therefore, all scabies medications should be repeated in 1 week to kill any mites that hatched following the last treatment.

� QUICKQUIZ Which is NOT a clinical variant of scabies? A) Urticarial scabies. B) Nodular scabies. C) Norwegian scabies. D) Crusted scabies.

FIGURE

33-33. Nod u l a r scabies presents as dome-sha ped i nd ivid ual nod ules without bu rrows. It is a n atypical presentation of scabies. (Used with perm ission from Wi l l i a m Augehenbaugh, M D.)

CHAPTER 33



S KI N D I SO R D E R S

751

Question 26-3 What do you advise the family to do? A) Call pest control. B) Place clothing and bedding in a sealed bag for 24 hours. C) Treat pets to prevent transmission. D) Wash and dry all clothing and bedding. E) Burn all stuffed animals. Discussion 26-3 The correct answer is "D:' Environmental controls are essential to preventing reinfestation. All clothing and bedding should be washed in hot water, placed in the dryer, and heated on high or sealed in bags for 72 hours, as the mites cannot live apart from a host for longer than 24 to 36 hours. Pets cannot harbor the human mites, so veterinary treatments are not necessary. Do not pull a Velveteen rabbit and burn beloved toys.

� QUICKQUIZ What causes the pruritus associated with scabies? A) T-cell response. B) Venom released from the mite. C) Eggs deposited by the mite. D) Disruption of the skin barrier. E) Secondary bacterial infection of burrows. Discussion The correct answer is "A:' The itching, which is worse at night, results from a type IV delayed hypersensitivity reaction to the mite, its feces, or eggs. The itching may persist even after treat­ ment of the infestation. Antihistamines and topical steroids (once mites are gone) may be used.

F I G U R E 33 -34. Ti nea Corporis. T h e lesions o f tinea corporis a re a n n u l a r with a ra ised scaly rim a n d central clearing. T h e presence o f sca l i n g d ifferentiates it from g ra n u l o m a a n n u l a re. (U sed w i t h permission from

Wi l l i a m Augehenbaugh, M D.)

quality, and absence of scale. It is often confused with tinea corporis. The etiology is unknown. Treatment includes topical or intralesional steroids and phototherapy. A KOH exam confirms the presences of hyphae. (See Figure 33-35.) Question 27-2 What is the first-line treatment for tinea corporis? A) Bactroban ointment. B) Ketoconazole tablets. C) Nystatin cream. D) Terbinafine cream. E) Terbinafine tablets.

An 8-year-old girl develops round, scaling, pruritic patches on her arms. Question 27-1 Which is the least likely cause of these skin patches? A) Atopic dermatitis. B) Contact dermatitis. C) Granuloma annulare. D) Plaque psoriasis. E) Tinea corporis. Discussion 27- 1 The correct answer is "C:' Eczematous dermatitis is a classi­ fication that encompasses a variety of conditions that mimic atopic dermatitis. They are characterized by red, scaly papules and plaques. Atopic dermatitis, contact dermatitis, plaque psoriasis, and tinea corporis are examples of such conditions. (See Figure 33-34.) Granuloma annulare, conversely, has an annular configuration (raised rim with central clearing), a pink

F I G U R E 33-35. Microscopic exa m i nation of a KOH preparation of skin scra p i n g s from a tinea lesion wi l l show septate hyphae. (Used with perm ission from Wi l l i a m Augehenbaugh, M D.)

752

MCG RAW- H I LL E D U CAT I O N S P E C I A LTY BOA R D REVI EW: P E D I AT R I C S

Discussion 27-2 The correct answer is "D:' Tinea infections are generally super­ ficial and can be treated easily with topical agents. Derma­ tophyte infections respond to terbinafine, azole antifungals (eg, clotrimazole, ketoconazole, econazole) , and ciclopirox. Mupirocin (Bactroban) is a topical antibiotic used for bacterial infections. Nystatin targets Candida yeast and is useful when treating diaper dermatitis.

� QUICKQUIZ Which is NOT a presentation of tinea pedis? A) Moccasin tinea pedis. B) Two foot-two hand tinea pedis. C) Interdigital tinea pedis. D) Bullous tinea pedis. E) Pustular tinea pedis. Discussion The correct answer is "B:' It is two foot -one hand, not two hand. Tinea pedis has variable presentations. Interdigital tinea pedis produces erythema, scale, and maceration of the web spaces. A moccasin distribution describes erythema and scale of the entire plantar foot and extension to the lateral aspects. Two foot-one hand tinea describes erythema and scale of the plantar feet with scaling of one palm (palmar involvement is an immune "id" reaction, not true infection) . Bullous tinea pedis produces blisters. Pustular variants produce small pustules on an inflammatory background. Most types of tinea pedis may be treated with topical antifungals, as previously noted. Patients with severe inflammation and itch may require oral antifungals.

A 1 7-year-old boy develops small, round, scaling, pink mac­ ules that are coalescent into a large patch on his back. The rash has grown steadily over the past 2 years. He lifts weights at the school weight room. He notes that the spots turn white in the summer. Question 28-1 What is the most likely diagnosis? A) Atopic dermatitis. B) Pityriasis rosea. C) Pityriasis versicolor. D) Syphilis. E) Tinea corporis. Discussion 28-1 The correct answer is "C:' Pityriasis (tinea) versicolor is caused by the yeast Malassezia furfur. The infection causes scaly patches that may be small and discrete or coalescent into large patches. The color is pink in individuals with fair skin. (See Figure 33-36.) The patches are hypopigmented when affecting those with dark skin or when the surrounding skin is tan. The eruption is often

F I G U R E 33-36. Pityriasis versicolor is a yeast i nfection of the skin that

presents with sca l i n g pink patches on the tru n k. (Used with perm ission from Wi l l i a m Augehenbaugh, M D.)

chronic and recurrent. The yeast is spread by direct contact with an affected person or object, such as gym equipment. The KOH preparation has a "spaghetti and meatballs" appearance. Wood lamp examination may show fluorescence. Question 28-2 Which is NOT a treatment option for pityriasis versicolor? A) Topical ketoconazole. B) Topical terbinafine. C) Selenium sulfide shampoo. D) Oral itraconazole. E) All of the above. Discussion 28-2 The correct answer is "E:' Pityriasis versicolor typically responds well to antifungal agents. Topical antifungals (including terbin­ afine and azole antifungals) are preferred as they carry minimal risk. Oral antifungals may be added when the eruption is wide­ spread and located on sites that are difficult to reach. Patients with recurrent pityriasis versicolor should be counseled to wash the skin once or twice a month with selenium sulfide shampoo ( eg, Head and Shoulders or Selsun Blue). This decreases colonization with Malasseziafurfur and reduces the risk of recurrent infections.

A 1 -year-old girl cries during diaper changes. Noting a rash in the inguinal folds and gluteal cleft, you diagnose diaper dermatitis. Question 29-1 What clinical clue helps confirm cutaneous infection with

Candida?

A) B) C) D) E)

Bullae. Erythema. Satellite papules and pustules. Scale. White, adherent plaques.

CHAPTER 33

Discussion 29- 1 The correct answer is "C:' Candida diaper dermatitis is described as confluent beefy red papules and pustules involving the skinfolds, with satellite lesions. (See Figure 33-37.) In con­ trast, irritant diaper dermatitis produces erythema but without satellite lesions and spares the skinfolds. Irritant diaper derma­ titis is caused by wetness and contact with acidic stool, and may be complicated by secondary Candida infection. Consider an alternative cause or immunodeficiency if the diaper rash does not resolve, despite treatment.



Helpful Tip

� Treatment t i ps for d i a per d ermatitis i n c l u de:

=-

i1 1r

·

Wa s h with soa p l ess cleanser d u ri n g d i a per c h a n g es. Avoi d ba by wi pes, which may conta i n i rrita nts. Dry the skin thoro u g h ly. Avoid ba by powd e r a s they may a bra de the s k i n . Apply a topica l a ntica n d i d a l crea m (eg, nystatin or azo l e a ntifu ng a l ) . A p p l y a barrier pa ste, s u c h a s z i n c oxi d e pa ste.



Helpful Tip

j1 -.:l� 1 1r



S KI N D I SO R D E R S

753

Question 30-1 What additional clinical finding best supports your diagnosis? A) Blunting of papillae on the tongue. B) Erythematous plaque of the umbilicus. C) Nail pits and oil spots. D) Scalp erythema and scale. E) Single large, scaling plaque that preceded the current lesions. Discussion 30-1 The correct answer is "E:' Pityriasis rosea starts with one large pink, scaling patch termed the herald patch. It is followed by the eruption of smaller patches in a "Christmas tree" distribu­ tion on the trunk. Psoriasis may present with geographic tongue (blunting of papillae on the tongue), nail pits and oil spots, and erythema and scale of the scalp, postauricular sulci, elbows, knees, umbilicus, and groin. (See Figure 33-38.) Question 30-2 Which of the following is NOT recommended for patients with pityriasis rosea? A) Observation. B) Oral antihistamines. C) Oral erythromycin. D) Oral terbinafine. E) Triamcinolone ointment.

Cuta neous ca n d i d ia s i s i s a l so termed intertrigo (affects s k i nfo l d s ) . l t com m o n l y affects i nfra m a m m a ry, a bd o m ­ i n a l pa n n us, a n d i n g u i n a l fo l d s as the moist envi ron­ ment a l l ows Candida yeast to g row.

A 1 5-year-old boy presents for evaluation of multiple pink, scaling, thin plaques of the trunk. You suspect pityriasis rosea.

F I G U R E 33-37. Candida Diaper Dermatitis. Candida i nfections a re bright

beefy red, i nvolve the ski nfolds, a n d have sate l l ite lesions. (Used with perm ission from Wi l l i a m Augehenbaugh, M D.)

F I G U R E 33-38. Pityriasis rosea presents as erythematous scaly ova l s on the tru n k i n a "Ch ristmas tree" pattern. T h e r a s h m a y be preceded b y a s i n g l e l a rge lesion cal led a hera l d patch. (Used with perm ission from Wi l l i a m Augehenbaugh, M D.)

754

MCG RAW- H I LL E D U CAT I O N S P E C I A LTY BOA R D REVI EW: P E D I AT R I C S

Discussion 30-2 The correct answer is "D:' Pityriasis rosea arises from infection with human herpesvirus-? (HHV-7). The eruption typically runs a self­ limited course and resolves spontaneously after several months. When associated with pruritus, oral antihistamines or mid­ potency topical steroids may be added. Oral antibiotics, including erythromycin, have shown efficacy in shortening the duration of the rash. Oral and topical antifungal agents are not effective.

A 5-year-old girl develops five skin-colored and pink papules on the trunk and extremities. She denies itching. Question 3 1 - 1 What feature suggests a diagnosis of molluscum contagiosum? A) Black dots. B) Central dell. C) Excoriations. D) Pustule. E) Yellow color. Discussion 3 1 - 1 The correct answer is "B:' Molluscum contagiosum is caused by a pox virus that produces chronic infection of a localized area of skin. The lesions are contagious, hence contagiosum, and can be spread to distal sites by autoinoculation. Papules are umbili­ cated, characterized by a central depression. (See Figure 33-39.) This may be more apparent when the lesion is treated with cryo­ therapy. Black dots (often called "roots" or "seeds" by patients) are seen in warts, resulting from thrombosis of blood vessels. (See Figure 33-40. ) Question 3 1 -2 What is your first recommendation to the family? A) Acyclovir. B) Cantharidin.

F I G U RE 33-40. Wart (Verrucae). Warts have throm bosed ca p i l l a ries that look l i ke black dots. Warts also disru pt the skin l i nes. (Used with perm ission from Wi l l i a m Augehenbaugh, M D.)

C) Cryotherapy. D) Curettage. E) Reassurance. Discussion 3 1 -2 The correct answer is "E:' Most isolated papules of molluscum resolve spontaneously within 2 years. Indications for treatment include symptomatic, spreading, or cosmetically disfiguring lesions. Chemovesicants such as cantharidin (blister beetle extract) are painless with application, but must be rinsed off and carry the risks of dyspigmentation. Curettage (manual removal) and cryotherapy are painful and a good choice for older kids. Topical immunotherapy with tretinoin, imiquimod, or other irritants may be helpful. Oral antivirals are not effective in the treatment of molluscum.





1 1 1r

Helpful Tip On exa m, wa rts (verrucae) i nterru pt the skin l i nes a n d have t h ro m bosed ca p i l l a ries that look l i ke b l a c k dots.

� QUICKQUIZ Which human papillomavirus (HPV) subtype is most com­ monly associated with condyloma acuminata? A) HPV- 1 . B ) HPV-2. C) HPV-6. D) HPV- 16. E) HPV-3 1 . F I G U R E 33-39. Moll uscum Contagiosum. A single flesh-colored u m b i l icated pa pule is seen, consistent with molluscum contagiosum. (Used with perm ission from Wi l l i a m Augehenbaugh, M D.)

Discussion The correct answer is "C:' (See next quiz for discussion.)

CHAPTER 33

i QUICKQUIZ Which HPV subtype is considered high risk for malignancy? A) HPV- 1 . B ) HPV-2. C) HPV-6. D) HPV- 1 1 . E) HPV- 1 8 . Discussion The correct answer is "E:' More than 1 00 serotypes of HPV have been identified. HPV- 1 and HPV-2 are most frequently associ­ ated with common warts and palmoplantar warts, respectively. Condyloma acuminata (anogenital warts) most commonly occur with HPV-6 and HPV- 1 1 , which have low oncogenic potential. HPV- 16, - 1 8, - 3 1 , and -33 are responsible for many cases of cervical, vulvar, penile, and anal cancers. The Centers for Disease Control and Prevention (CDC) recommends HPV vaccination for boys and girls at ages 1 1 or 12 years. The most commonly utilized option is a quadrivalent vaccine covering HPV-6, - 1 1 , - 1 6, and - 1 8.



S KI N D I SO R D E R S

755

You choose permethrin 1 % cream and advise application to damp hair after shampooing, followed by rinsing after 10 minutes. Question 32-2 How often should the medication be applied? A) One application. B) Once daily for 1 week. C) Twice daily for 1 week. D) Twice daily for 2 weeks. E) Two applications 1 week apart. Discussion 32-2 The correct answer is "E:' Pediculosis capitis (head louse) is an obligate human parasite that cannot survive for more than 24 hours without a blood meal. The female louse lives for up to 30 days and lays 5 to 10 eggs per day. Eggs hatch 8 to 10 days later. After the initial application, treatment should be repeated in 7 to 10 days.





1 1 1r

Helpful Tip Ped i c u l o s i s p u b i s (cra b l i ce) may i n fest a n y reg ion of body h a i r, i n c l u d i n g eye l a s hes, eye b rows, m u stache, beard, axi l l ae, and pubic a reas. P h t h i r u s pubis (t h e crab l o u se) may be m o re d i ffi c u l t to d etect t h a n h e a d

A mother and her 5-year-old daughter rush to the clinic after they are notified of a lice outbreak at school. The mother saw her daughter scratching her head and found nits on examination.

l i ce a s t h ey a re skin c o l o r e d . S l ate g ray m a c u l e s of the tru n k a n d t h i g h s may be a c l u e to the d i a g nosis. Treatment of all h a i r-bea r i n g a reas i s essent i a l to e n s u re c o m p l ete e ra d icatio n . All cl ose co ntacts s h o u l d a l s o be treate d . Treatment i s t h e sa m e a s descri bed fo r

Question 32-1 What is recommended as first-line treatment? A) Ivermectin tablets. B) Lindane 1 % shampoo. C) Malathion 0.5% lotion. D) Permethrin 1% cream. E) Petroleum jelly. Discussion 32-1 The correct answer is "D:' All topical pediculicides are neuro­ toxic to lice. Permethrin 1 % cream is available over the counter. Increasing rates of resistance are reported throughout the world. Lindane is not recommended owing to risk of central nervous system toxicity. Malathion may have the best efficacy of a topical agent, but a longer contact time is required, making it a second­ line option. Oral ivermectin is effective but is off-label for head lice. Occlusive products, such as petroleum jelly and oil, are not pesticidal (do not kill lice) and therefore not recommended for the treatment of head lice.



Helpful Tip

� To diag nosis

=

i1 1 r

pedicu losis capitis, a l ive louse m ust be

seen. N its (eggs) may persist for months after successfu l treatment.

ped i c u l o s i s capitis.

A 2-month-old boy develops malaise, fussiness, and fever with edema and ill-defined erythema of the arm following an injury. Question 33-1 Of the organisms listed below, which is the most likely to cause cellulitis in this patient? A) Escherichia coli. B) Haemophilus influenzae. C) Pseudomonas aeruginosa. D) Streptococcus pyogenes. E) Sporothrix schenckii. F) Bartonella henselae. Discussion 33-1 The correct answer is "D:' Cellulitis is most commonly caused by group A beta-hemolytic streptococci (Streptococcus pyogenes) and Staphylococcus aureus. Gram-positive cocci and gram-negative aerobes and anaerobes are more common among patients who are immunosuppressed. Cellulitis occurs

MCG RAW- H I LL E D U CAT I O N S P E C I A LTY BOA R D REVI EW: P E D I AT R I C S

756

following disruption of the skin barrier and is characterized by spreading erythema, indistinct borders, edema, and warmth. •

Helpful Tip

:5.� Group A streptococca l i nfections may be c o m p l icated

i1 1 r

by s i n u sitis, mastoiditis, p n e u m o n ia, rheu m atic fever, or acute g l o m e ru l on e p h ritis.

20% to 40%. Polymicrobial infections are more common among diabetic and immunosuppressed patients. Necrotizing fasciitis is an infection of the deeper tissues, muscle fascia, and fat that progresses rapidly and may be fatal. The clinical presentation is a painful plaque resembling cellulitis. It evolves rapidly, becoming dusky to gray-blue within 36 hours of onset. Cutaneous necrosis develops with surrounding fibrosis. Patients must be managed aggressively with debridement of the tissue (including fascia), intravenous antibiotics, and supportive care.



An 8-year-old boy develops cellulitis and is treated with empiric oral penicillin. He does not respond to therapy. He is febrile but hemodynamically stable. His leg is red, warm, and edematous. Question 34- 1 What is your next step to help direct therapy? A) Blood culture. B) MRI. C) Tissue culture. D) Ultrasound. E) All of the above. Discussion 34- 1 The correct answer is "C:' When a patient is not responding to treatment, stop and ask yourself the following questions: ( 1 ) Is the patient compliant? (2) Is the diagnosis correct? (3) Is the organism resistant to the current antibiotic? (4) Is the organism covered by the current antibiotic? (5) Has a complication developed (abscess) ? (6) And, i s this a "normal host" (ie, not immunosuppressed, or affected by chronic kidney disease)? Treatment is often empiric unless the organism is cultured. Blood cultures are positive in fewer than 5% of cases of cellulitis. Tissue culture has a higher yield of 20% to 40%. Culture of pus or fluid from an intact bullae may be helpful, if present on exam. Additionally, the patients antibiot­ ics should be changed to provide coverage for methicillin-resistant Staphylococcus aureus (MRSA), and he may require admission for intravenous antibiotics.

� 1 1 1r

Discussion The correct answer is "C:' Necrotizing fasciitis is increasing among young, previously healthy patients following penetrating or blunt injury. It is a rapidly progressive infection with a mortality rate of

Ra p i d l y

p rog ress i n g

proportion to exa m

"cel l u l itis" w i t h

pa i n

out

of

is necrotizi ng fasciitis. Ca l l a

s u rgeon i m med iately fo r wou n d debridement.

A 1 7-year-old girl returns to your office 5 days after exci­ sion of a mole. She reports swelling, redness, and purulent discharge. Question 35-1 What is the most important step in management of this wound infection? A) Apply topical antibiotics. B) Culture purulent discharge. C) Give ibuprofen to reduce swelling and treat pain. D) Open the wound and drain pus. E) Start oral antibiotics. Discussion 35-1 The correct answer is "D:' Most postoperative infections of the skin are caused by Staphylococcus aureus. The most important step is draining the abscess by removing some or all of the sutures and rinsing the base of the wound. The wound is then allowed to heal by secondary intention. Oral antibiotics may not be necessary if the infection is uncomplicated, without sur­ rounding cellulitis. Routine culture is not required unless the wound fails to improve.

� QUIZ QUIZ What is the most common cause of necrotizing fasciitis in young, immunocompetent patients? A) Escherichia coli. B) Clostridium spp. C) Streptococcus pyogenes. D) Pseudomonas aeruginosa. E) Staphylococcus aureus.

Helpful Tip



Helpful Tip

� A bscesses

=

i1 1 r

a re treated with i ncision and d ra i nage.

Syste m i c a ntibiotics a re not a l ways needed.

BIBLIOGRAPHY

Bolognia JL, Jorizzo JL, Rapini RP, eds. Dermatology. 3rd ed. Philadelphia, PA: Elsevier/Saunders; 2012. Gupta D, Thappa DM. Mongolian spots. Indian J Dermatol Venereal Leprol. 2013;79(4) :469-478. Hirbe AC, Gutmann DH. Neurofibromatosis type 1: A multidisciplinary approach to care. Lancet Neural. 20 1 4; 1 3 (8) :834-843.

CHAPTER 33

Horn TM, Tidman MJ. The clinical spectrum of dystrophic epi­ dermolysis bullosa. Br J Dermatol. 2002; 146(2) :267-274. Metry DW, Hawrot A, Altman C, Frieden IJ. Association of solitary, segmental hemangiomas of the skin with visceral hemangiomatosis. Arch Dermatol. 2004; 140(5) : 5 9 1 -566.



S KI N D I SO R D E R S

O'Connor NR, McLaughlin MR, Ham P. Newborn skin: Part I. Common rashes. Am Pam Physician. 2008;77 ( 1 ) :47 -52. Thomas-Sohl KA, Vaslow DF, Maria BL. Weber syndrome: A review. Pediatr Neural. 2004;30(5) :303-3 1 0.

757

This page intentionally left blank

Sports Med i ci n e a n d Physica l Fitn ess

34

And rew R. Peterson

A teenage boy with one eye comes to your office for a prepar­ ticipation evaluation. He has a history of retinoblastoma that resulted in single enucleation. His remaining eye is healthy, with 20/20 vision. He has no other medical problems and no other concerns. He wants to play high school basketball. Question 1 - 1 You advise him: A) The risk of losing his remaining eye is too high for him to be allowed to play contact sports. B) He can play basketball without protective eyewear if he is cleared by an ophthalmologist. C) He can play basketball ifhe wears protective eyewear approved by the American National Standards Institute (ANSI). D) He can play basketball if he wears a protective hard contact lens. E) No restrictions. Discussion 1 - 1 The correct answer is "C:' This question is an example of a time when an athlete with a disability or medical condition can par­ ticipate in sports with minor modifications. In this case, the loss of a paired organ does not preclude participation in basketball as long as the remaining eye is effectively protected. While a comprehensive review of every medical condition requiring accommodations is beyond this scope of this review, there are several common situations that are ripe for board questions. The next seven cases cover these situations. A rule of thumb is that if a player can compete safely, he or she should be allowed to.

A high school basketball player was diagnosed with influ­ enza yesterday. You see him in the athletic training room the next day. He is febrile to 38.9°C ( I OZOF) . He wants to play in tonight's playoff game.

Question 2-1 You tell him: A) "No problem. Go get 'em:' B) "We can treat you with Tamiflu (oseltamivir) so that it's safe to play tonight:' C) "We can treat you with ibuprofen so that it's safe to play tonight:' D) "You shouldn't play tonight because of the risk of myocarditis:' E) "You shouldn't play tonight because of the risk of rhabdomyolysis:' Discussion 2-1 The correct answer is "D:' The reasons are fuzzy, but playing with a fever increases risk of myocarditis and arrhythmia. There is a theoretical risk of rhabdomyolysis if an athlete plays when dehydrated and febrile, but acute illness has not been demon­ strated to be an independent risk factor for muscle injury.

� QUICKQUIZ Which of the following is NOT a part of the standardized American Heart Association (AHA ) preparticipation screen­ ing evaluation? A) Electrocardiogram (ECG) . B) Blood pressure measurement. C) Family history of unexplained death. D) History of chest pain with exertion. E) Cardiac auscultation. Discussion The correct answer is "A:' This is very controversial, but the current American Academy of Pediatrics (AAP) recommenda­ tions for the preparticipation physical evaluation (PPE) do not include an ECG. The AAP endorses the AHA cardiac screening criteria, which involve obtaining a detailed history and basic cardiac examination. The PPE monograph changes frequently. You should make yourself familiar with the AAP endorsed PPE 759

MCG RAW- H I LL E D U CAT I O N S P E C I A LTY BOA R D REVI EW: P E D I AT R I C S

760

history and physical form (which is not reprinted here) . But rec­ ognize that it is likely to change every few years as this contro­ versial topic evolves.

You are evaluating a high school football player who suf­ fered a concussion 2 weeks ago. He says that he is completely back to normal. You use the Sideline Concussion Assessment Tool v.3 (SCAT3) in the office to evaluate for any lingering concussion symptoms. His symptom score is 0, but he does very poorly on the Balance Error Scoring System test and has trouble with tasks of attention and memory, including reverse order digits and months of the year in reverse. Question 3-1 Right now, you wish you: A) Had chosen a different career. B) Knew how to score the SCAT3. C) Had a computed tomography ( CT) scanner in your office. D) Knew the athlete's baseline neurocognitive abilities. E) Could talk the school into banning football. Discussion 3-1 The correct answer is "D:' OK, you may have wished for any of the others as well. But it is always useful to know the baseline neurocognitive status of a contact or collision sport athlete. The first step in returning to sport after a concussion is to return to feeling and functioning normally. Some offices and schools use computer-based neurocognitive tests to help tell when an athlete has returned to his or her neurocognitive baseline. If such testing is not available, knowing how an athlete would be expected to perform on tasks of reaction time, memory, and attention is important if you are going to use any tests (includ­ ing the SCAT3) to make a determination about return to play.





1 1 1r

Helpful Tip The SCAT3 is a sta n d a rd i zed tool to eva l uate a n ath lete 1 3 yea rs of a g e or o l d e r fo r a concussion. Havi ng a base l i n e score is h e l pfu l to i nterpret the c h i ld's score after a n i nj u ry or for retu r n i n g to play.

An 8-year-old boy with trisomy 21 (Down syndrome) comes to the clinic with his parents because he wants to participate in Special Olympics wrestling. Question 4- 1 How do you respond? A) Calmly explain that there is no Special Olympics wrestling. B) Order flexion-extension cervical spine X-rays. C) Discuss the risk of atlantoaxial instability in children with trisomy 2 1 .

D ) Encourage participation in sports with high dynamic or high static demands. E) All of the above. Discussion 4- 1 The correct answer is "E:' This is a poorly worded question. But it illustrates an important point. Athletes with trisomy 2 1 are at risk for atlantoaxial instability. For this reason, Special Olympics does not include any contact or collision sports or other activi­ ties that would put the athlete at risk for cervical spine injury. Obtaining flexion and extension cervical spine X-rays is an important part of screening in physically active children with trisomy 2 1 . But they are also at risk for early cardiovascular dis­ ease, so participation in strenuous activities is still a good idea.

A 15-year-old female swimmer has a long history of well­ controlled epilepsy. She recently switched clubs and her new coach is uncomfortable with her continuing to compete as a swimmer. The patient is asking for a letter to her coach explain­ ing that she should be allowed to swim whenever she likes. Question 5-1 You respond by: A) Telling her that unsupervised swimming is not recom­ mended for an athlete with well-controlled epilepsy. B) Advising that swimming increases risks of seizure and is not recommended for anyone with epilepsy. C) Suggesting she consider a safer sport such as archery. D) Handing her a pamphlet for your sky-diving school. E) Telling her that she can compete as long as she wears an approved personal flotation. device, at all times, including in class and when she goes to the prom. Discussion 5-1 The correct answer is "A:' Swimming is permitted for athletes with epilepsy as long as it is supervised. It is a common misconception that swimming increases seizure risk. It does not. In fact, exercise of any type decreases seizure risk. There are sports that are not safe for patients with epilepsy, but they are all pretty obvious: archery, riflery, power lifting, sports involving heights, and so on.

An 8-year-old with well-controlled type 1 diabetes mellitus is interested in playing tackle football. He currently uses an insulin pump. Question 6- 1 Which of the following is NOT true? A) He may be able to continue to use his insulin pump while playing tackle football if he embeds it in a special pad. B) He may be able to use a small dose of injectable insulin before practice and disconnect his pump while playing football.

CHAPTER 34



S P O RTS M E D I C I N E A N D P H Y S I C A L F I T N E S S

761

F I G U R E 34- 1 . I n s u l i n s i g n a l tra n sd uction i n skeleta l m u scle. Skeletal muscle g l ucose receptors (GLUT4) can transport g l u cose i nto t h e cel l without t h e need for i n s u l i n . CAP, Cbl-associated protein; I RS, i n s u l i n receptor su bstrate; Pl-3-kinase, phosphatidyl i n ositol 3-ki nase. (Reprod uced with permission from Bru nton LL, Chabner BA, Knol l man BC: Good m a n & G i l m a n's: The Pharmacological Basis of Therapeutics, 1 2th ed. McGraw- H i l l Education, I nc; 201 1 . Fig 43-4.)

C) His insulin needs will likely increase when he starts playing sports. D) He may have more trouble with overnight lows as a result of playing sports. E) His coaches or other team personnel should keep sugar­ containing beverages or glucagon, or both, on the sideline. Discussion 6-1 The correct answer is "C:' Skeletal muscle glucose receptors can take up glucose without the aid of insulin, so insulin needs often go down when a patient with type 1 diabetes becomes more physically active. (See Figure 34- 1 . ) There are many strategies for maintaining good glycemic control in the setting of contact sports. Athletes with diabetes should not be discouraged from playing any sport, but adjustments may need to be made in how and when they take their insulin, when they check their blood glucose, and when they eat.

A 1 6-year-old comes for a preparticipation evaluation. She is tall, wears thick glasses, and has long tapered fingers. She has a murmur on exam. Question 7-1 Which of the following is NOT a physical examination find­ ing in her genetic syndrome? A) Ectopia lentis. B) Hyperopia. C) Aortic dilation. D) Arm span exceeding height. E) Protrusio acetabuli.

Discussion 7-1 The correct answer is "B:' People with Marfan syndrome are nearsighted (myopic). There are several diagnostic criteria that variably rely on genetic testing and family history. The most common findings are listed in Table 34- 1 .

� QUICKQUIZ True or False: Adolescents are at higher risk of injury from sports participation than any other age group. A) True. B) False. Discussion The correct answer is "A:' Adolescents are indeed at increased risk of injury from sports participation, but it may be because of the types of sports they tend to play. For example, high school football is much more violent than youth football.

� QUICKQUIZ Which of the following measures has good evidence for pre­ venting sports-related injuries? A) Static stretching. B) Ballistic stretching. C) Neck strengthening to prevent concussions. D) General conditioning to prevent all injuries. E) Eccentric strengthening to prevent Achilles injuries.

762

MCG RAW- H I LL E D U CAT I O N S P E C I A LTY BOA R D REVI EW: P E D I AT R I C S

TABLE 34- 1 CHARACT E R I ST I C F I N D I N G S I N

MARFA N SYN D R O M E

Tru n k and l i m bs

Pectus carinatum, pectus excavatu m, red uced u p per­ to-lower seg ment ratio, arm span-to-height rati o > 1 .05, wrist and t h u m b sign, sco l i osis, s pondylo l i sthesis, red uced extension of el bow, pes pla­ n u s, joint hypermo b i l ity

Head and face

Arched pal ate, crowd i n g of teeth, dol ichocepha ly, m a l a r hypoplasia, enophtha l mos, retrog nathia, down s l a nting pa l pebra l fissures

Eyes

Ectopia lentis, a bnorma l ly flat corneas, i n c reased axia l length of g lobe, hypoplastic i ri s

Heart a nd vascu lat u re

D i l ation o f asce n d i n g aorta, d i ssection of asce n d i n g aorta, m itra l va lve prola pse, d i lation of main p u l monary a rtery

Lungs

S ponta neous pneumothorax, p u l monary a pical blebs

Abdomen a n d torso

Stretch marks, hernia, l u m bo­ sacra l d u ra l ectasia

Genetics

Fa m i ly h i story of Ma rfa n syn d rome, presence of FBN 7 m utation (fi bri l l i n 1 gene)

Discussion The correct answer is "D:' Very few measures have actually been proven to prevent injuries, but general conditioning is one. The other options listed all represent common misconceptions. How­ ever, landing training does prevent anterior cruciate ligament (ACL) injuries, eccentric strengthening prevents hamstring inju­ ries, and ankle proprioceptive drills prevent ankle sprains. And, in general, being in better shape prevents all kinds of injuries.

Today is injury follow-up day in the training room. You dread having to explain to college athletes why they cannot return to play immediately. (Rehabilitation is not a word in their vocab­ ulary.) You start thumbing through the charts to see how many physical therapy referrals you will be making today. Question 8- 1 Patients with which of the following injuries do NOT need to be rehabilitated before being allowed full return to sport? A) Ankle sprain. B) ACL tear after surgery.

C) Achilles tendon injury. D) Clavicle fracture. E) Quadriceps contusion. Discussion 8-1 The correct answer is "D:' Range of motion and proprioceptive training clearly prevents recurrent ankle sprains. ACL recon­ struction requires a prolonged recovery course that focuses on muscular control of the reconstructed knee. Achilles tendon injuries respond well to eccentric training. Quadriceps contu­ sions are less likely to develop heterotopic ossification (myosi­ tis ossificans) if they have early range of motion and activation therapy. However, patients who have clavicle fractures without any other associated injuries can return to sport when bony heal­ ing is adequate without any additional rehabilitation. In general, most injuries do better with a course of physical therapy.





Helpful Tip A n kl e proprioception d r i l l s decrease a n kl e s p ra i n s. To

1 1 1r red u ce a l l types of i nj u ries, a t h l etes m u st get in sha pe. Good physica l cond itio n i n g is key.

During a football game, a player is tackled. He was hit in the thigh by the other player's helmet, causing immediate pain and swelling. He is diagnosed with a quadriceps contusion and not allowed to return to play. Question 9- 1 Which of the following is NOT an appropriate treatment for a quadriceps contusion? A) Early mobilization. B) Immobilization in a fully flexed position. C) Immobilization in a fully extended position. D) Nonsteroidal anti-inflammatory drugs (NSAIDs) . E) Ice. Discussion 9- 1 The correct answer is "C:' Quadriceps contusions are at high risk for developing heterotypic ossification (myositis ossificans) . (See Figure 34-2.) Early mobilization, immobilization in a fully flexed position, NSAIDs, and ice have all been shown to decrease the risk of heterotypic ossification. Immobilization in a fully extended position clearly increases the risk of heterotopic ossification. Tra­ ditional knee immobilizers should be avoided at all costs.

An obese 16-year-old high school football player becomes dizzy and nauseated at the end of a full-pads practice in the August heat. He is having painful cramps and attempts to opt out of end-of-practice conditioning. His coach, angry that the

CHAPTER 34



S P O RTS M E D I C I N E A N D P H Y S I C A L F I T N E S S

763

TABLE 34-2 CHARACT E R I ST I C S OF HEAT STROKE

VERSUS HEAT EXHAU STI O N

Heat Stro ke

Heat Exha u stion

Hot, d ry s k i n

Hot, moist skin

Core body tem pe rature > 40°C ( 1 04°F)

Core body tem pe ratu re < 40°C ( 1 04°F)

Altered menta l status­ confusion, letha rgy, or coma

Overwhel m i n g fatigue

R h a bdomyolysis

M u scle cra m ps

M u ltiorg a n system fa i l u re

F I G U R E 34-2. Myositis ossifica ns occurs most com monly in the lower extremity, espec i a l ly the quad riceps. The muscle susta i n s a tra u matic hematoma that hea l s l i ke a fractu re, forming bone. I n the X-ray image, an

a rea of ossification is seen i n the h a m string (white a rrow). (Reproduced with permission from M i l l er AE, Davis BA, Beckley OA, et al: B i l ateral and recu rrent myos itis ossifica ns i n an athlete: a case report and review of treatment options, Arch Phys Med Rehabil. 2006 Feb;87(2):286-290.))

player is not training hard enough, punishes him by making him run additional cross-field sprints. The player collapses near mid-field. On initial evaluation, he is breathing, has a rapid pulse, has hot dry skin, and is unresponsive. Question 1 0-1 Which of the following is/are associated with heat stroke? A) Hot, dry skin. B) Hot, moist skin. C) Overwhelming fatigue. D) Core temperature greater than 40°C ( 1 04°F). E) Muscle cramps. F) Confusion, lethargy, or coma. G) Rhabdomyolysis. H) Multiorgan failure. I) Options A and D, only. J) Options A, D, and F, only. K) Options A, D, F, G, and H. Discussion 1 0-1 The correct answer is "K:' Athletes with heat stroke are at risk for death. They typically have hot, dry skin; a core temperature greater than 40°C, and profound mental status changes, and are at risk for muscle breakdown and organ failure. Question 1 0-2 Which of the following is/are associated with heat exhaustion? A) Hot, dry skin. B) Hot, moist skin. C) Overwhelming fatigue. D) Core temperature greater than 40°C ( 1 04°F). E) Muscle cramps. F) Confusion, lethargy, or coma.

G) H) I) J)

Rhabdomyolysis. Multiorgan failure. Options B and C, only. Options B, C, and E.

Discussion 1 0-2 The correct answer is 'T' Heat exhaustion, is much less severe than heat stroke. Athletes with heat exhaustion typically con­ tinue to sweat and maintain only a modestly elevated core temperature. Muscle cramps and overwhelming fatigue are common. (See Table 34-2.) Question 1 0-3 Which of the following is the best way to cool the athlete? A) Cold water immersion. B) Cool water immersion. C) Ice packs and a fan. D) Helicopter downdraft (seriously, this has been studied) . E) Cool mist spray. Discussion 1 0-3 The correct answer is "B:' Believe it or not, all of these meth­ ods have been studied and nothing scrubs heat faster than cold water immersion. The more rapidly the athlete is cooled, the more likely he or she is to survive without significant long-term effects of the injury. Question 1 0-4 Which of the following is/are acceptable as a method of mea­ suring the temperature of the athlete suspected of having heat stroke? A) Rectal thermistor with remote reader B) Rectal thermometer C) Axillary thermometer D) Oral thermometer E) Capsule thermistor F) Options A and B, only. G) Options A, B, and D. H) Options A, B, and E.

MCG RAW- H I LL E D U CAT I O N S P E C I A LTY BOA R D REVI EW: P E D I AT R I C S

764

Discussion 1 0-4 The correct answer is "H:' Axillary, oral, temporal, and otic temperatures are remarkably inaccurate in the setting of heat stroke because peripheral perfusion is typically markedly compromised.

� QUICKQUIZ Which of the following is NOT a risk factor for heat illness? A) Markedly thin body habitus. B) Markedly obese body habits. C) Young age (infants, toddlers). D) Old age. E) Stimulant use. Discussion The correct answer is "A:' This seems pretty obvious, but people with thin body habitus scrub heat more effectively than those with a thicker body habitus. Infants scrub heat poorly because they generally have less surface area for their body volume/mass and do not sweat very effectively. The elderly are less efficient at all kinds of physiologic processes and although they can sweat and may be thin, they are also at greater risk for heat illness. Whether there is a link between stimulant use and heat illness is controversial. High doses of strong stimulants clearly raise core temperature and can impair sweating. Whether low-dose stimulants, such as methylphenidate for attention deficit hyper­ activity disorder (ADHD ), might increase risk of heat illness is somewhat controversial. You should not be asked about that on a board exam. But know that someone who is abusing potent stimulants is clearly at risk.





1 1 1r

Helpful Tip Heat stroke ca u ses organ d a m age, rhabdo myolysis, a n d a l te red mental status. The skin is d ry. Vict i m s s h o u l d be i m me rsed i n cold water i m m ed iately t o l ower the b o d y tem perature.

You are the team physician for the Iowa State Cyclones. Fol­ lowing a game with the Iowa Hawkeyes, nearly all of your players are injured. The coaches have questions about when some of their key players might be able to return to play. Questions 1 1 - 1 through 1 1 -5 Pull out a piece of paper and outline your return-to-play rec­ ommendations for each of the following players:

1 1 - 1 : Starting quarterback suffered a concussion in the first half. It is now the next morning and he feels completely normal.

1 1 -2: Starting linebacker somehow managed to get poked in the eye by the referee. He complains of blurred vision, pain, and photosensitivity. 1 1 -3: Starting tailback was hit on the outside of his knee and now has pain but no laxity with valgus load, at both neutral and 30 degrees of flexion. 1 1 -4: Starting offensive lineman fell hard on the point of his shoulder and has a prominent bump and tenderness at his acromioclavicular (AC) j oint. 1 1 -5: Fourth-string backup punter suffered a neck sprain when the coach grabbed him by the facemask and told him to "get out there and put them in the ground:' Discussion 1 1 - 1 through 1 1 -5 Your recommendations should considerations:

include

the

following

1 1 - 1 : There are several key points to return-to-play decision making in concussion management. First, a concussed ath­ lete should not return to play on the same day as the injury. Some of the old guidelines were silent on this issue, but all modern concussion guidelines agree that this is a bad idea. There is a small risk of so-called second impact syndrome; this occurs when a concussed athlete suffers a second hit to the head and dies due to impaired vascular autoregulation, cerebral edema, and eventually brainstem herniation. This is thankfully a rare event. But it is very common for athletes to sustain a worse injury than they would have otherwise if they return to play too soon and suffer a second impact-type exposure. Second, a brief period of rest in the days follow­ ing the injury is clearly beneficial. Athletes should rest and recover as much as possible for the first week after the injury, as long as they are symptomatic. What a symptomatic ath­ lete should do after the first week of rest is controversial, so you probably won't be asked about that. Third, the athlete needs to return to normal before going back to play. This is largely for the reasons listed above. There are a lot of meth­ ods for ensuring this. A clinical history and examination are sufficient for most providers who are well-trained in con­ cussion management. Sometimes clinicians use tools such as the SCAT3 or computer-based neurocognitive testing to help with this decision making. Fourth, the athlete should complete a graduated return to play program before going back to full contact sports. This usually involves a progres­ sion from easy aerobic exercise to more difficult aerobic exercise to strength and skill work to contact practice and finally to full contact play. This process typically takes 5 to 7 days. People tend to focus on the steps of graduated return to play, rather than the point. Remember, the reason for the program is to look for the reemergence of symptoms. If the athlete becomes symptomatic again, he or she is not ready for return to play and should stop progressing through the graduated return to play program. Finally, be ready for the next concussion. Be sure to have a good baseline neurologic and, ideally, neuropsychological examination. This will make

CHAPTER 34



S P O RTS M E D I C I N E A N D P H Y S I C A L F I T N E S S

765

future decisions about return to play much easier. Consider changes that could be made to the athlete's style of play or position that would decrease concussion risk. It is unlikely that any piece of equipment is beneficial for decreasing con­ cussion risk.



Helpful Tip

:S.� Concu ssed c h i l d re n s h o u l d not return to play the sa m e

i1 1 r

day to avoid susta i n i ng a m o re severe i nj u ry if h it a g a i n (somet i m es ca l l e d second i m pact syn d rome). Before retu r n i n g to pl ay, the c h i l d m u st be back to n o r m a l a n d have su ccessfu l ly co m p l eted a g ra d u ated return to play prog ra m .

1 1 -2: Many injuries can occur from a finger t o the eye, but the two emergencies that are commonly encountered on the sideline are a hyphema and detached retina. A hyphe­ ma should be evident on physical examination as a pool of blood in the anterior chamber of the eye. (See Figure 34-3.) Although most hyphemas result in abnormal vision, not all do. However, in the absence of a hyphema, other severe eye injuries will cause abnormalities of vision. So, this athlete can return to play if he has no blood in the anterior chamber of his eye and normal vision. However, after the game, he should be further evaluated for other eye trauma (corneal abrasion, etc) . 1 1 -3: This athlete has a grade 1 medial collateral ligament (MCL) sprain. Grade 2 and 3 injuries are more severe and have laxity on examination. Athletes with these more severe injuries should be removed from play until they are healed

F I G U R E 34-4. Acrom ioclavicu l a r (AC) joint separation. The right AC joint is sepa rated with displacement of the clavicle from the acromion. (Reproduced with perm ission from Maitin I B, Cruz E, eds. Current Diagnosis and Treatment: Physical Medicine and Rehabilitation. New York, NY: McGraw- H i l l Education;

2 0 1 5, Fig. 29-4.)

and rehabilitated. However, an athlete with grade 1 MCL in­ juries can return to play if he or she is able to run and cut at full speed on the injured knee. Remember, MCL injuries are often associated with ACL and medial meniscus injuries (the "terrible triad") . Do not forget to examine these other structures. 1 1 -4: Acromioclavicular (AC) j oint injuries are not danger­ ous, but they are painful. They rarely need to be repaired, and athletes can return to play when they can tolerate the pain of the injury. Football players can often pad the AC joint effec­ tively, making return to play more tolerable. For the athlete described in this question, it would be a good idea to get a set of plain film X-rays to confirm the absence of associated fracture. (See Figure 34-4.) 1 1 -5: Neck injuries are very common in football, but in general, injuries that don't have a violent mechanism of injury and don't result in any neurologic symptoms are benign. This athlete can return to play when his symptoms allow.

F I G U R E 34-3. Posttra u matic hyphema and su bconj u n ctiva l hemorrhage

Layering blood is seen i n the a nterior c h a m ber of eye consistent with a hyphema. Med i a l ly, a su bconj u nctiva l hemorrhage is present. (Reproduced with perm ission from Riordan-Eva P, C u n n i n g h a m ET, eds. Vaughan & A sbury's General Ophthalmology. 1 8th ed. New York, NY: McGraw- H i l l Education; 201 1 , Fig. 1 9-9.)

A 14-year-old cross country runner comes to the clinic for evaluation of shin pain. She has tenderness at the medial bor­ der of the tibia but no pain with the hop test. X-rays are nor­ mal. She reports that she used to have regular periods, but has not had one in over a year. Her body mass index (BMI) is 20 kg/m2 •

766

MCG RAW- H I LL E D U CAT I O N S P E C I A LTY BOA R D REVI EW: P E D I AT R I C S

Question 1 2- 1 Which of the following statements i s true? A) Starting a birth control pill is an effective way to improve this patient's bone health. B) The amenorrhea of the female athlete triad is due to ovarian failure. C) Shin splints and tibial stress fractures are different grada­ tions of the same injury. D) Most athletes with the female athlete triad get adequate cal­ cium and vitamin D from their diet. E) This patient cannot have the female athlete triad because her BMI is in the normal range. Discussion 1 2- 1 Th e correct answer i s "C:' Shin splints and tibial stress fractures are different grades of the same injury. Shin splints is a mild bone stress injury that typically causes diffuse tenderness and pain at the medial border of the tibia (this is why it is called "medial tibial stress syndrome") . Stress fractures are more severe bone injuries that typically have more focal tenderness and more severe pain. If necessary, the two injuries can be differentiated on magnetic res­ onance imaging (MRI). Shin splints show only periosteal edema on T2-weighted images whereas stress fractures demonstrate edema in the cortex and marrow cavity of the bone. The female athlete triad comprises ( 1 ) disordered eating, (2) menstrual irreg­ ularities, and (3) impaired bone health. Note that low BMI is not necessary for the diagnosis. In fact it is common for particularly muscular female athletes with the female athlete triad to have normal or elevated BMI. The underlying problem is insufficient calories to sustain the level of activity. This causes impaired bone turnover and hypothalamic dysfunction that leads to menstrual irregularities (most commonly amenorrhea, but occasionally other patterns). The mainstay of treatment is increasing calories and decreasing exercise. Supplemental calcium and vitamin D is often necessary to meet dietary needs or make up for recent defi­ ciencies. Supplemental estrogen in the form of oral contraceptive pills does not improve bone health. One note on terminology. Recently, there has been a movement to rename this condition "relative energy deficiency in sport" or "RED-S" to make it clear that males can also suffer from hypothalamic dysfunction and impaired bone health when they do not have adequate nutrition to fuel their exercise. However, it is unlikely that you will see this term on a board exam in the near future.

E) Some sports require early entry in order to reach the highest levels of success. F) Options A and B, only. G) Options A, B, and D. H) Options A, C, and E. I) Options A, C, D, and E. Discussion The correct answer is "H:' Prior to skeletal maturity, tendon inju­ ries are fairly rare because the tendon tissue is fairly robust and the surrounding bone is not. Tibial tubercle avulsion is typically a traumatic injury and does not occur as a complication of tibial tubercle apophysitis (Osgood-Schlatter disease). Early sport spe­ cialization does increase risk of overuse injuries. Children who play multiple sports or spend their time in free play have fewer injuries than those who spend most of their active time in a single sport. A good rule of thumb is that a child should not spend more hours per week playing a sport than years they are old (obviously, there are exceptions, but this is a good place to start and illustrates the point that diverse athletic exposures are beneficial). However, there are some sports that seem to require early entry in order to succeed at the highest levels. Tennis and gymnastics have the best data to support the need for early entry. It does not seem to make much difference in most team sports.

A 7-year-old boy presents with heel pain. He no longer can play soccer. He does not remember falling or hurting his feet. On exam, he is tender on the underside of his heels. Question 1 3- 1 You suspect a n apophysitis, but o f which anatomic site? A) Tibial tubercle. B) Inferior pole of the patella. C) Base of the fifth metatarsal. D) Calcaneus. Discussion 1 3- 1 The correct answer i s "D:' H e has Sever disease, a n apophysitis of the calcaneus seen frequently in young athletic boys with heel pain. Other examples of apophysitis are listed below, with the site affected: •

� QUICKQUIZ Which of the following statements is/are true about overuse injuries in children? A) Bone injuries are more common than tendon injuries. B) Tibial tubercle avulsion is a common complication of Osgood-Schlatter disease. C) Sport specialization increases risk of overuse injuries. D) Decreasing free play is important for preventing overuse injuries when children are participating in several hours of organized sports per week.

Osgood-Schlatter disease: tibial tubercle (See Figure 34-5)



Sinding- Larsen-Johansson disease: inferior pole of the patella



Iselin disease: base of the fifth metatarsal



Sever disease: calcaneus

Question 1 3-2 Which is NOT an effective treatment for Sever apophysitis? A) Ice. B) Gel heel cups. C) Foot orthotics. D) Rest. E) Rehabilitation.

CHAPTER 34

o

S P O RTS M E D I C I N E A N D P H Y S I C A L F I T N E S S

767

TABLE 34-3 OTTAWA A N K L E R U L E S : I M AG I N G

C R I T E R I A I N ACUTE A N KL E S P RA I N S

An kle X-rays Are I ndicated: M a l leolar pa i n and

( 1 ) Posterior t i p o f the latera l or med i a l m a l leol u s tenderness

or (2) U na b l e to bea r weight M idfoot pa i n and

( 1 ) Base of the fifth metata rsal or navicu l a r pa i n

or (2) U na b l e to bea r weight

F I G U R E 34-5. Osgood-Sch latter d i sease is an apophysitis of the proxi m a l tibial tu bercle at the i n sertion o f the pate l l a tendon. Latera l X-ray s howi ng irreg u l a rity a n d fragmentation of the tu bercle (white a r row) . (Reprod uced with permission from Davis KW: I ma g i n g pediatric s ports inju ries: l ower extremity, Radio/ Clin North Am. 20 1 0 Nov;48(6) : 1 2 1 3-1 235.)

Discussion 1 3-2 The correct answer is "E:' Unfortunately rehabilitation does not make much difference for the treatment of most overuse apophysitis, including Sever. Ice makes the heel feel better, but does not fix the problem. Gel heel cups and foot orthotics decrease symptoms with activity. Rest does tend to improve the problem, but it commonly recurs with resumption of activity. Remember, apophysitis is not dangerous and does not cause long-term harm. There are not many conditions where it is okay to tell a 7 -year-old to play through the pain, but Sever apophysitis is one of them.

A 1 6-year-old girl was playing basketball when she stepped on another player's foot, rolling her ankle (inversion) . She is diagnosed with a lateral ankle sprain.

Discussion 1 4- 1 The correct answer is "C:' Inability to bear weight and ten­ derness at the posterior lateral malleolus are concerning for distal fibular fracture (this is the crux of the Ottawa Ankle Rules for ruling out ankle fractures by history and physical exam in the emergency department) . (See Table 34-3.) A posi­ tive ankle drawer test is seen with a grade 2 or 3 sprain of the anterior talofibular ligament, which is expected in more severe lateral ankle sprains. A positive syndesmosis squeeze test and widened medial clear space on ankle X-rays is concerning for a syndesmotic or "high" ankle sprain. This is a more severe injury that can lead to long-term ankle instability or arthritis, or both, and should be managed by someone with knowledge of the condition.

You explain to the girl that she sustained a grade 2 lateral ankle sprain involving the anterior talofibular ligament. She asks, "Is it completely torn" ? Question 1 4-2 Which of the following is true regarding a grade 2 ligament sprain? A) The ligament is not torn. B) The ligament has microscopic tears. C) The ligament is completely torn. D) The ligament is stretched. E) The ligament is incompletely torn. Discussion 1 4-2 The correct answer is "E:' The three grades of ligament sprain are described as follows: o

Question 1 4- 1 Which o f the following i s a n expected finding i n a lateral ankle sprain? A) Inability to bear weight. B) Tenderness at the posterior lateral malleolus. C) Positive ankle drawer test. D) Positive syndesmosis squeeze test. E) Widened medial clear space on ankle X-rays.

o

o

Grade 1 : Pain but no laxity. The ligament is only partially torn; there is internal disruption, but it is fully intact. The patient is able to bear weight and ambulate with minimal pain. Grade 2: Pain and mild laxity. The ligament is partially torn through, and there is more than just internal disruption. Weight bearing and ambulation are painful. Grade 3: Pain and laxity. The ligament is torn all the way through, causing significant instability and loss of motion. The patient is unable to bear weight or ambulate.

768

MCG RAW- H I LL E D U CAT I O N S P E C I A LTY BOA R D REVI EW: P E D I AT R I C S

� QUICKQUIZ The acronym PRICE is commonly used in describing the treatment of common sprains. What does PRICE stand for? A) Protection, Rest, Ice, Compression, Elevation. B) Pad, Rest, Ice, Compression, Elevation. C) Protection, Rotate, Ice, Compression, Elevation. D) Protection, Rest, Ice, Condition, Elevation. E) Protection, Rest, Ice, Compression, Eversion. Discussion The correct answer is ''A:' Common sprains are treated with these five measures:

P - Protection

Question 1 6- 1 Which o f the following i s true about athletes who have lost an eye? A) Due to their lack of depth perception, they should be pro­ hibited from playing any ball sport. B) They should wear ANSI-approved protective lenses for any sport that puts them at risk of being hit in the eye. C) They should wear a full polycarbonate face shield when playing football. D) They should have the vision in the remaining eye checked every 3 months to ensure that it has not suffered any damage. E) It's funny to tell them to "keep your eye on the ball:' Discussion 1 6- 1 Th e correct answer i s "B:' Th e buzzwords i n this question are ''ANSI approved:' Every policy statement and article on this topic uses the same phrase. So don't forget it.

R - Rest I - Ice C - Compression E - Elevation

You overhear a medical student say that helmets prevent con­ cussions in football players. Question 1 5- 1 You pull him aside and tell him what? A) He is wrong. B) He is really wrong. C) He is dead wrong. D) He needs to go home for the day. E) All of the above. Discussion 1 5- 1 The correct answer i s "E:' I t is well established that helmets do not prevent concussions in football players. Despite what you might hear, no helmet, helmet add-on, or other device pre­ vents concussion in football. This includes mouthguards. The only thing that seems to make a difference is limiting hitting or changing hitting technique. Helmets do prevent many other dangerous injuries, including skull fracture, intracranial hem­ orrhage, and death. A mouthguard does an excellent j ob of preventing dental trauma. Athletes in collision sports should absolutely wear helmets and mouthguards.

A teenage boy with one eye comes to your office for a pre­ participation evaluation. He has a history of retinoblastoma that resulted in single enucleation and needs a form filled out for school. The school wants a list of "dos and don'ts" for one-eyed athletes.

A high school wrestler suffered a blow to his ear and now has a large auricular hematoma. Question 1 7- 1 Which o f the following i s NOT true about his risk o f develop­ ing cauliflower ear? A) Draining the hematoma will decrease the risk. B) Using a pressure dressing over the drained ear will decrease the risk. C) Protecting the ear with a headgear is sufficient to prevent most of these injuries. D) Draining an auricular hematoma should only be done by an experienced provider because of the risk of infection. E) Hearing is never affected by cauliflower ear. Discussion 1 7- 1 Th e correct answer i s "D:' Cauliflower ear i s a permanent scar­ ring of the auricle following an auricular hematoma. Draining the wound and applying pressure (with a dental caulking gun, sutured-in bolster, or just a pressure dressing) decreases the risk of scarring. Severe cauliflower ear can cause conductive hearing loss (obviously) . Wrestling headgear is very effective in prevent­ ing auricular hematomas. Luckily, these injuries rarely become infected. In fact, many wrestlers drain each other's auricular hematomas in decidedly unsterile fashion with only very rare complications. The general pediatrician should feel comfortable draining an ear and applying a pressure dressing without fear of significant complications.

You are back on the sideline covering the Iowa State Cyclones. This week they are playing the Merciful Blind Sisters of the Poor (some people call them "Illini"). Your starting tailback is hit hard 5 yards deep in the backfield and does not get up.

CHAPTER 34

You rush onto the field and notice an open fracture of his lower leg. The player is not moving and does not respond to your ver­ bal commands. He is breathing and does have a pulse. Question 1 8- 1 Which of the following i s the first thing you should d o while stabilizing this patient? A) Remove the helmet and shoulder pads. B) Remove the facemask of the helmet. C) Splint the broken leg. D) Confirm neurovascular status distal to the fracture site. E) Stabilize the cervical spine. Discussion 1 8- 1 The correct answer is "E:' I t is easy to get distracted by the other injury in this setting, but do not forget than an uncon­ scious athlete should be assumed to have a cervical spine injury until proven otherwise. It is a bad idea to remove the helmet and shoulder pads because the movement can exacerbate an unstable cervical spine injury. Removing the facemask might be important for the athlete because the movement can exacerbate The broken leg, while dramatic, is low priority in this setting.

It is now 2 weeks later, and the Cyclones are playing the Junior Varsity team from the Barron County Branch Campus of Stout State University (BCBCSSU) . It is a close game. Your starting middle linebacker attempts to make a tackle and suf­ fers an anterior dislocation of his shoulder.



S P O RTS M E D I C I N E A N D P H Y S I C A L F I T N E S S

769

Question 1 8-2 Which of the following is NOT true about first-time shoulder dislocations? A) It is easier to reduce them on the field than to wait for the more controlled environment of the training room or emer­ gency department. B) Having the player reduce his own shoulder by clasping the hands around the knee and using the leg to provide a trac­ tional load with the shoulder in 30 degrees of forward flex­ ion is an effective way to safely reduce the shoulder. C) Early surgical repair of any bony injury to the glenoid dra­ matically decreases risk of repeat dislocation. D) It is important to check neurovascular status of the affected arm before attempting a reduction of an obvious shoulder dislocation. E) The most common mechanism for shoulder dislocation is a fall on an outstretched hand. Discussion 1 8-2 The correct answer is "E:' Most shoulder dislocations are hyper­ abduction with external rotation injuries. (See Figure 34-6.) Rapid reduction (before there is much muscle spasm) is much easier than waiting until the player is off the field. There are many effective methods of shoulder reduction, some of which require two providers and some of which can be done by the patient. Surgical repair following a first-time shoulder dislo­ cation is a very effective way of decreasing risk of future dis­ location. The reason to check neurovascular status prior to

Subcoracoid

Subglenoid

Subclavicular

I ntrathoracic

F I G U R E 34-6. Types of a nterior shoulder dislocation. (Reproduced with perm ission from Tinti n a l l i J E, Sta pczynski JS, M a OJ, et al: Tin tinalli's Emergency Medicine: A Comprehensive Study Guide, 8th ed. McGraw- H i l l Education, I nc., 201 6.)

770

MCG RAW- H I LL E D U CAT I O N S P E C I A LTY BOA R D REVI EW: P E D I AT R I C S

attempting dislocation is twofold: First, you want to be sure that your intervention did not cause nerve or vessel injury. Second, if the neurovascular function of the limb is not intact, it is more likely that there is more to the injury than just a simple dislo­ cation and urgent transport is warranted prior to attempting reduction on the field.

� QUICKQUIZ Children with trisomy 2 1 sometimes have atlantoaxial insta­ bility of the cervical spine. Because of this, they should not be allowed to participate in which of the following sports: A) Boxing. B) Weight lifting. C) Wrestling. D) Football. E) Swimming. F) Options A and B, only. G) Options A, C, and D. H) All of the above. Discussion The correct answer is "G:' Contact and collision sports are not permitted for children with trisomy 2 1 . While some patients do not have atlantoaxial instability, the risk is high enough that it is prudent to restrict all children with trisomy 2 1 from participat­ ing in sports that put them at risk for cervical spine instability.

A 1 7 -year-old pankration athlete comes to your clinic because he suffered a dislocation of his elbow during a bout. Question 1 9- 1 Which o f the following is true about this injury? A) Elbow instability is very rare after a first dislocation. B) Ulnar collateral ligament injury (UCL) is commonly the limiting factor for return to sport. C) Reduction of the elbow involves placing the forearm in pro­ nation and providing an axial load. D) Fall on an outstretched hand is the most common mecha­ nism of injury. E) Prolonged immobilization in full flexion accelerates recov­ ery and return to sport. Discussion 1 9- 1 The correct answer i s "B:' Elbow dislocations are dramatic events, but also fairly common in mixed martial arts fighting (pankration is a junior version of fighting; striking above the clavicle is typically prohibited in this style) . Reduction involves placing a tractional load on the forearm in a supinated position. Following reduction, early mobilization speeds recovery and return to sport. Splinting in flexion can cause a severe flexion contracture that is very difficult to overcome with rehabilitation. UCL injuries are present in nearly all elbow dislocations and can be very slow to heal. Rarely, surgical reconstruction of the UCL is necessary for return to full activity. (See Figure 34-7.)

F I G U R E 34-7. A posterior el bow d i slocation is s h o w n i n these X-rays. (Re produced w i t h perm ission from Tinti n a l l i J E, Sta pczynski JS, Ma O J , Yea l y OM, C l i n e O M , Meckler G O, e d s . Tin tinalli's Emergency Medicine: A Comprehensive Study Guide. 8 t h ed. N e w York, NY: McGraw- H i l l Education; 201 6, F i g . 270-1 1 .)

CHAPTER 34



S P O RTS M E D I C I N E A N D P H Y S I C A L F I T N E S S

771

I

A 1 2-year-old pitcher comes to the clinic to discuss pain and a "dead-arm" feeling he is getting with throwing. He plays on four teams in four different leagues, has an 80-mile-per hour (mph} fastball, and recently learned to throw a slider. Mallet finger

Question 20-1 Which of the following is NOT a risk factor for little league elbow? A) Throwing in multiple leagues. B) High-pitch velocity. C) Less advanced skeletal maturity. D) Throwing breaking pitches. E) Throwing from an elevated pitcher's mound. Discussion 20- 1 The correct answer is "E:' Little league elbow is a stress injury to the medial epicondylar physis in the throwing arm. Throw­ ing more and throwing harder are by far the biggest risk factors for this injury. It tends to occur in younger throwers, and other medial elbow injuries begin to replace little league elbow as the thrower reaches skeletal maturity. Children who throw breaking pitches (curveballs, sliders, etc) are at higher risk of developing the injury, but this seems to be a confounder-only children who are really interested in pitching, throwing a lot, throwing harder, and throwing in more than one league tend to start throwing breaking pitches. So, you could think of both little league elbow and throwing breaking pitches as the result of throwing a lot. The risk of shoulder and elbow injuries is higher in pitchers who throw from an elevated pitching mound, but this association has not been made with little league elbow in younger throwers. Hence the choice of option "E" as the correct answer might be a bit controversial.

� QUICKQUIZ Which of the following statements regarding jersey and mal­ let fingers is false? A) Distal finger flexion is lost with a jersey finger. B) A jersey finger requires urgent surgical evaluation. C) Distal finger extension is lost with a mallet finger. D) A mallet finger requires urgent surgical evaluation. E) None of the above. Discussion The correct answer is "D:' Jersey finger is an injury to the flexor tendon as it inserts into the distal phalanx of a finger. It occurs when a player grabs a j ersey as the opponent is pulling away, causing a hyperextension of the distal interphalangeal (DIP) joint while the proximal interphalangeal (PIP) is flexed and the flexor tendon is under extreme load. This causes a rup­ ture of the flexor tendon and loss of finger flexion at the DIP joint. This is a surgically urgent injury because the tendon can

F I G U R E 34-8. M a l let Finger. Extension of the d i stal i nterphalangeal joint is lost due to ru ptu re of the extensor tendon. (Reproduced with permission

from Tinti n a l l i J E, Sta pczynski JS, Ma OJ, Yea l y DM, Cline DM, Meckler G D, eds. Tintinalli's Emergency Medicine: A Comprehensive Study Guide. 8th ed. New Yo rk, NY: McGraw- H i l l Education; 201 6, Fig. 43-5 Part A.)

retract, making eventual repair and healing very difficult. Mal­ let finger is an injury to the extensor tendon as it inserts into the distal phalanx of a finger. (See Figure 34-8.} It occurs when a player is struck at the tip of the finger by a ball, forcing the DIP j oint into hyperflexion. This causes rupture of the extensor tendon and loss of finger extension at the DIP joint. This is a fairly benign injury that responds to a brief period of splinting in full extension.

A 19-year-old girl jammed her hand into her handlebars when she had to brake abruptly while riding her bike. On exam, she has pain in the anatomic snuff box. Question 2 1 - 1 Which o f the following i s NOT true about this type of fracture? A) Most scaphoid waist fractures heal without complications. B) The scaphoid is the most common fractured carpal bone. C) Fall on an outstretched hand is a common mechanism of injury. D) Prolonged immobilization is required for proper healing. E) None of the above. Discussion 2 1 - 1 The correct answer is "E:' This is another poorly worded ques­ tion, but none of those answers are not true. Most scaphoid waist fractures (fractures of the distal third of the scaphoid) heal with­ out complications. But enough do not heal well that vigilance is warranted. Check for pain to palpation in the anatomic snuff­ box. The scaphoid waist is a vascular watershed area and non­ union fracture healing with eventual degenerative changes and collapse is not uncommon. (See Figure 34-9.} These injuries should be protected in a thumb spica cast for at least 6 weeks and monitored closely for healing. Any question of poor healing should prompt a referral to a hand surgeon to consider surgical fixation. The scaphoid is the most commonly fractured carpal bone, and a fall on an outstretched hand (FOOSH) injury is the most common mechanism.

MCG RAW- H I LL E D U CAT I O N S P E C I A LTY BOA R D REVI EW: P E D I AT R I C S

772

8

A

F I G U R E 34-9. Sca phoid Fractu re. A fracture of the sca phoid bone (out l i ned) is seen in this a nteroposterior X-ray. (Reproduced with perm ission from Doherty

GM, ed. Current Diagnosis and Treatment: Surgery. 1 4th ed. New York, NY: McGraw- H i l l Education; 20 1 5, Fig. 40- 1 2A.)



Helpful Tip

=� Pa i n

r1 1 r

to

pa l pation

in

the

a nato m i c

s n uffbox

is

c h a racteristic of a sca phoid fra ctu re, w h i c h is a n i nj u ry at h i g h risk fo r poor hea l i n g .

A 1 3-year-old female athlete presents for evaluation of ante­ rior knee pain of several months' duration. She is a state champion in cross country running. Her pain is worse with running, which makes practice difficult, but she is able to "tough it out:' Recently she felt her left knee buckle a few times. This scared her, prompting today's visit to your office. Question 22-1 Which of the following knee tests is NOT useful for assessing a patient with patellofemoral stress syndrome (PFSS) ? A) Patellar compression test. B) One-legged step-down test. C) Patellar apprehension test. D) Thessaly test. E) Lachman test. F) None of the above.

Discussion 22-1 The correct answer is ''F:' This is another poorly worded question, and somewhat of a trick question. All of the listed tests are useful, so let's review what each test reveals. The patellar compression test is typically positive in PFSS. The subpatellar space is tender, and pushing the patella hard against the trochlea should recreate the patient's pain. The one-legged step -down test commonly shows poor hip abduc­ tion control in PFSS. The typical finding is a valgus thrust of the knee as the patient steps down off a step. Patients with PFSS and poor hip abduction control typically respond well to physical therapy for treatment of their pain. The patellar apprehension test is usually positive in patellar dislocation or subluxation. The most common direction of instability is laterally. A positive patellar apprehension test occurs when the patient has a sense of apprehension (not pain) when the patella is pushed laterally. The Thessaly test is positive in a meniscus tear. The patient stands on one leg, bends the knee to 30 degrees of flexion, and twists back and forth. A painful click is highly suggestive of a meniscus tear, although it can also be positive in other injuries to the cartilage (eg, osteo­ chondral defect or osteoarthritis) . The Lachman test reveals laxity in the anterior-posterior direction when translating ( sliding) the tibia on the femur. In a positive test there is increased translation, but also a soft end point when the tibia is translated anteriorly on the femur.

CHAPTER 34





1 1 1r



S P O RTS M E D I C I N E A N D P H Y S I C A L F I T N E S S

773

Helpful Tip Kee p i n g valgus versu s varus stra i g h t can be d ifficu lt. These terms refer to the orientation of the extrem ity d i sta l to the j o i n t of i nterest. Va rus: towa rd the m i d l i n e (eg, g e n u va rus = bow l egs) Va l g u s : away from the m i d l i n e (eg, genu va l g u s = knock kn ees)

Question 22-2 Which of the following is NOT a risk factor for developing PFSS? A) Hypermobility. B) Female gender. C) Tall stature. D) Increased BMI. E) Jumping sport. Discussion 22-2 The correct answer is "C:' All of the others are fairly strong risk factors for anterior knee pain. FIGURE

You will be serving as the team physician for a high school girls soccer team this year. You have been reading about the "epidemic" of knee injuries in girls and women's soccer and want to be sure you are ready for what you might see on the pitch. Questions 23-1 through 23-4 Describe the characteristic physical findings of each of the following knee injuries. Be sure to comment on appearance, sites of maximal tenderness, range of motion, strength, lax­ ity, and special tests that might be positive.

23- 1 : Anterior cruciate ligament (ACL) tear. 23-2: Posterior cruciate ligament (PCL) tear. 23-3: Medial meniscus tear. 23-4: Patellar dislocation. Discussion 23-1 through 23-4 Your descriptions should include the following information:

23- 1 : The defining physical exam finding an ACL tear is anterior laxity. Mechanisms of injury include being hit on the lateral side of the knee when the foot is planted or sudden deceleration followed by a quick change in direction (cutting) or pivoting. Immediately following injury a large effusion is noted. (See Figure 34- 1 0.) There may be tenderness at the femoral condyles or tibial plateau from bone bruising that occurs during the injury. (See Figure 34- 1 1 . ) Strength is often limited by quadriceps inhibition, but there is no true mus­ cular injury. There should be no laxity with varus or valgus

34- 1 0. Anterior

Cruciate Ligament (ACL) Tear. T h i s sagitta l

T1 -weig hted M R I i mage demonstrates a rupture of the ACL (arrow) a n d a very l a rge joint effusion. ACL tears bleed. A lot. A very l a rge j o i nt effusion j u st hours after the tea r is cha racteristic of the i nj u ry. (Reproduced with perm ission from Doherty GM, ed. Current Diagnosis and Treatment: Surgery. 1 4th ed. New York, NY: McGraw- H i l l Education; 201 5, Fig. 40-30.)

stress unless there is an associated collateral ligament injury. It is somewhat common to have a medial meniscus or medial collateral ligament tear, or both, at the same time as an ACL injury. This combination is called a terrible triad. The Lach­ man and anterior drawer tests should be markedly positive. A positive Lachman and anterior drawer consist of both increased travel as the tibia is translated anteriorly on the femur but also a sensation of a soft anterior end point. (See Figure 34- 1 2 . ) 23-2: Th e PCL injury i s very similar t o a n ACL injury but much less common. The typical mechanism of injury is a hard blow to the anterior proximal tibia, causing a posterior translation on the tibia on the femur. The knee is commonly very swollen, with significant tenderness at the anterior prox­ imal tibia. As in an ACL injury, there is laxity. However, this time it is in the posterior direction: the tibia sags posteriorly against the femur. (See Figure 34- 1 3 . ) Associated collateral and meniscus injuries are much less common. 23-3: Patients with isolated medial meniscus tears (see Figure 34- 1 4) commonly present with a small joint effusion and tenderness along the medial j oint line. Certain meniscus tears can cause knee locking or other mechanical symptoms. A locked knee will have very little range of motion. Expect normal strength and normal tests of ligamentous laxity. McMurray test, which passively loads and shears the medial and lateral compartments of the knee, may demonstrate a

774

MCG RAW- H I LL E D U CAT I O N S P E C I A LTY BOA R D REVI EW: P E D I AT R I C S

F I G U R E 34- 1 3. Posterior Cruciate Ligament (PCL) Sag. With a torn PCL, the tibia sags posteriorly below the fem u r when the leg is el evated and flexed at the hip and knee. (Reproduced with perm ission from Stone CK, H u m ph ries RL, eds. Current Diagnosis and Treatment: Emergency Medicine. 7th ed. New York, NY: McGraw-H i l l Education; 201 1 , Fig, 28-1 8.)

painful click felt along the medial j oint line. In the Thessaly test, the patient stands on the affected leg with the knee bent, then actively rotates the knee internally and externally, pro­ ducing a similar finding. F I G U R E 34- 1 1 . B o n e B r u i s e after Anterior Cruciate Ligament (ACL) Tear.

Sag itta l M R I image of the knee in a patient with an ACL tea r showing the classic associated bruising of the lateral femoral condyle a n d latera l tibial pl ateau (arrows). (Reproduced with perm ission from Chen MYM, Pope TL, Ott DJ, eds. Basic Radiology. 2nd ed. McGraw- H i l l Education, I nc., 201 1 . Fig 7-2 1 .)

F I G U R E 34- 1 2. Lac h m a n Test. The Lac h m a n test is u sefu l for d i a g nosing a tea r of the a nterior cruciate l i g a ment. The knee is held i n 30 deg rees of

flexion w h i l e the exa m i ner pulls forward tryi ng to a nteriorly tra n s l ate the tibia on the fem u r. I ncreased travel of the tibia forward i n d icates a positive test. (Reprod uced with permission fro m I m boden J B, H e l l m a n n DB, Stone J H , e d s . Current Diagnosis a n d Treatment: Rheumatology. 3 rd ed. N e w York, NY: McGraw- H i l l Education; 201 3, Fig. 1 2-2.)

23-4: Most patellar dislocations reduce spontaneously when the patient first tries to extend the knee after the injury. A patella that is still dislocated is very obvious; the knee should be locked in flexion and the patella is easily palpated lateral to the expected position. However, most of the time you will see a

F I G U R E 34- 1 4. Med i a l Meniscus Tear. A tea r in the posterior part of the med i a l meniscus (arrow) is shown i n this sag itta l M R I i mage of the knee. A normal meniscus should look l i ke a "bow tie" on M R I . (Reproduced with perm ission from M a itin I B, Cruz E, eds. Current Diagnosis and Treatment: Physical Medicine and Rehabilitation. New York, NY: McGraw- H i l l Education;

201 5, Fig. 29-2.)

CHAPTER 34

patellar dislocation after it is already reduced. The patient may have a small joint effusion, but it should not be as large as that seen in cruciate ligament injuries. There is typically exquisite tenderness at the medial pole of the patella and at the lateral femoral condyle. The patient demonstrates severe patellar ap­ prehension when a lateralizing force is applied to the patella.



Helpful Tip

=.:::'11.. Tea ring the ACL, med i a l m e n iscus, a n d med i a l col lat­

r1 1r

e ra l l i g a ment at the sa m e t i m e is known as the terri b l e tria d .

� QUICKQUIZ An osteochondral defect is most likely to be noticed at which common site in the knee? A) Lateral aspect of the medial femoral condyle. B) Medial aspect of the lateral femoral condyle. C) Trochlea. D) Medial patellar facet. E) Tibial tubercle. Discussion The correct answer is "A:' An osteochondral defect is most com­ monly seen at the lateral aspect of the medial femoral condyle. (See Figure 34- 1 5 . ) For some reason, this is a commonly tested

A



S P O RTS M E D I C I N E A N D P H Y S I C A L F I T N E S S

B

775

c

F I G U R E 34- 1 6. Forms of Osteochondra l Defects (Osteochond ritis Disseca ns). Different forms of osteochondra l defects seen i n c h i l d ren include (A) a n ossification center defect without ca rti lage defect, (B) a defect with a h i nged fla p, a n d (C) com p l ete sepa ration of bone and carti lage, which may cause a loose fra g ment i n the knee. (Reproduced with perm ission from

Skinner H B, McMahon PJ, eds. Current Diagnosis and Treatment in Orthopedics. 5th ed. New York, NY: McGraw- H i l l Education; 201 4, Fig. 1 0-27A-C.)

fact. The term osteochondritis dissecans is also sometimes used to describe the lesion. (See Figure 34- 16 .) Although occasion­ ally caused by trauma, these lesions most often arise due to avascular necrosis of the subchondral bone, leaving the articu­ lar cartilage unsupported and prone to damage. Patients who have not yet reached skeletal maturity can be treated with a brief period of nonweightbearing to see if the lesion recovers. If it does not, or in older patients, surgical techniques such as osteo­ chondral transfer procedures and microfracture procedures are useful. For the purpose of the pediatric boards, it is important to recognize the lesion on plain film X-rays and refer the patient to an orthopedic specialist.

A 1 5-year-old wrestler presents with anterior knee pain and swelling. He denies feeling his knee lock or give way. No liga­ mentous instability is detected on exam. A large fluid collec­ tion is felt over the patella. It is mildly tender. Question 24- 1 Which of the following is NOT an effective treatment for pre­ patellar bursitis in a high school wrestler? A) Drain the bursa and allow immediate return to wrestling. B) Drain the bursa and compress and ice for 1 week before allowing return to wrestling. C) Drain the bursa and inject a small dose of corticosteroids. D) Drain the bursa and inject a small dose of doxycycline to act as a sclerosis agent. E) Surgical bursectomy.

F I G U R E 34- 1 5. Osteochondra l Defect (Osteochond ritis Disseca ns). An osteochondral defect is seen at the l atera l as pect of the med i a l femoral condyle. It represents a sepa ration of s u bchondral bone a n d a rticu l a r ca rti lage from the u nderlying b o n e . (Reproduced w i t h permission from Davis KW: I maging pediatric sports i nj u ries: lower extremity, Radio/ Clin North Am. 201 0 Nov;48(6) : 1 2 1 3- 1 235.)

Discussion 24- 1 The correct answer is ''A:' Prepatellar bursitis is very common in wrestlers. It results from trauma that causes irritation of the bursa sandwiched between the patella and skin. Draining the bursa works very well to decrease the pain and swelling; how­ ever, the swelling returns very quickly if nothing is done to prevent recurrence. Treating with ice and firm compression is usually effective. For resistant lesions, injecting a small dose of corticosteroids into the bursa after following draining can lower

MCG RAW- H I LL E D U CAT I O N S P E C I A LTY BOA R D REVI EW: P E D I AT R I C S

776

the risk of recurrence. Historically, difficult-to-control lesions have been treated with surgical bursectomy; but recently, use of sclerotic agents such as doxycycline has gained popularity. As a general pediatrician, you should feel comfortable draining and compressing these injuries, but injecting anything into the bursa should be performed by an orthopedic specialist.

I f the pain resolves over 1 to 2 weeks, it i s unlikely to have been a fracture. While relatively benign at the distal fibula, the main concern with Salter-Harris fractures is the development of phy­ seal bars and growth arrest. •

Helpful Tip

::"'l.. The



i1 1r

i1 1 r

Helpful Tip

::"'l.. Ra n g e

=-

=-

of

motion

is

usua l ly

not

i m p a i red

with

Salter- H a rris

cla ssification

system

is

used

to

describe physea l fractu res that ca n ca use prematu re c l o s u re

of

the

physis.

The

h i g her

the

n u m ber

prepate l l a r b u rsitis as the swel l i n g is outside the j o i nt.

c l a ssification correlates with higher risk fo r g rowth

In contra st, a n acute knee effu sion may cause l i m ited

a rrest. (See F i g u re

34- 1 7.)

flexi o n .

You are seeing a 9-year-old tennis player who suffered an inversion ankle injury 1 hour before presentation. He is unable to bear weight and has exquisite tenderness on his dis­ tal fibula, approximately 1 em from the distal tip. You obtain ankle X-rays, which show no abnormalities. Question 25-1 Which of the following is an appropriate next step? A) Ankle rehabilitation. B) Weightbearing as tolerated. C) Walking boot, but no crutches. D) Cast and crutches. E) Lace-up ankle brace. Discussion 25-1 The correct answer is "D:' Of the available choices, cast immo­ bilization and nonweightbearing is the most appropriate. This 9-year-old is almost certainly skeletally immature. His inability to bear weight and the exquisite tenderness at the site of his dis­ tal fibular physis should make you very concerned for a Salter­ Harris fracture. (See Figure 34- 1 7.) His injury should be treated just like any other bony fracture. It is not uncommon to see no changes on plain film radiographs with either Salter-Harris I or Salter- Harris V injuries. If you are unsure if the patient has a Salter-Harris fracture, a brief period of immobilization and nonweightbearing with serial exams should clarify matters.

You have been asked to speak at a convention for youth football coaches in Arizona. Your topic of education is fluid replacement during sports. Arizona seems like a fitting place to talk about fluids, heat, and dehydration. Question 26- 1 Which of the following is NOT true regarding fluid replace­ ment during sports? A) Athletes should never be restricted from consuming fluids during practice and competition. It does not make them tougher and does not train them to need less water. B) Plain water is insufficient for maintaining hydration during short bouts of exercise and for rehydrating after exercise. C) Sports drinks with carbohydrates and sodium are absorbed more rapidly than plain water. They may be used for exercise lasting longer than 1 hour in duration. There is no need to use a sports drink for fluid replacement after exercise. D) For most athletes, using thirst as a guide for fluid intake it is adequate. E) Dehydration resulting in loss of as little as 2% ofbody weight can significantly impair performance. F) Dehydration can increase risk of heat illness when exercis­ ing in hot conditions. Discussion 26- 1 The correct answer is "B:' Plain water is sufficient for maintain­ ing hydration during short bouts of exercise and rehydrating

l t II

Ill

IV

v

F I G U R E 34- 1 7. Sa lter-Harris classification of g rowth pl ate (physea l) i nj u ries. Salter 1: Fractu re along the g rowth plate. Salter II: Fractu re along the g rowth plate with extension i nto the meta physis. Salter Ill: Fractu re along the g rowth pl ate with extension i nto the epiphys is. Salter IV: Fractu re across the g rowth plate with extension i nto the meta physis and epiphysis. Salter V: Crush i nj u ry to the growth pl ate without obvious fractu re. (Reproduced with perm ission from Doherty GM, ed. Current Diagnosis and Treatment: Surgery. 1 4th ed. New York, NY: McGraw- H i l l Education; 20 1 5, Fig. 40-22.)

CHAPTER 34

after exercise. Had you read option "C" closely, you would have been able to spot the inaccuracy in option "B:' Your talk at the coach's convention was a success. Now you are asked to sit on a panel of experts for a question-and-answer session with parents of pee wee wrestlers at the national Take Down and Pin Palooza. A burly man asks if dehydration is an effective way for a wrestling athlete to make weight. Question 26-2 You respond: A) Yes, that is true. B) No, that is false. C) It depends on the age of the wrestler. D) It depends on whether he or she will be wrestling in a single bout or tournament. E) It is effective for heavier but not lighter weight wrestlers. Discussion 26-2 The correct answer is ''A:' However, that does not mean that it is a good idea. Methods of rapid weight reduction typically impair performance. Some of these methods, such as exercis­ ing in very hot environments or severely restricting fluid intake, can be dangerous. Wrestling athletes should be reminded that dehydration can profoundly impair their ability to wrestle well. Most competitions require weigh-in only 1 hour prior to com­ petition and that is not nearly enough time to rehydrate ade­ quately. Similarly, using diuretics or laxatives to make weight should be discouraged. By far the safest and most effective method for making weight is slow gradual weight reduction focused on decreasing body fat as well as wrestling at a realistic body weight. A 200-pound child is never going to wrestle in the lightweight division.

A proud mother clad in a t-shirt bearing a picture of her sons dressed in singlets wants to know if her youngest son, with only 10% body fat, will be able to participate in high school wrestling when he gets older. Question 26-3 Which of the following is true about weight control practices for high school wrestlers? A) They are not allowed to compete at less than 12% body fat. B) It takes a 3500-calorie deficit to lose 1 pound of body fat. C) Impermeable rubber suits (sauna suits) are an allowed method of weight loss. D) Laxatives are an allowed method of weight loss. E) Less body fat does not provide a competitive advantage to the wrestler. Discussion 26-3 The correct answer is "B:' This is a favorite test question. It takes a fairly large calorie deficit to lose body fat. For this reason weight reduction should be done very gradually. High school wrestling has rules to prevent male wrestlers from competing at less than 7% body fat and female wrestlers from competing at less than 1 2 % body fat. NCAA (National Collegiate Athletic



S P O RTS M E D I C I N E A N D P H Y S I C A L F I T N E S S

777

Association) college wrestling permits body fat as low as 5%. Sauna suits and laxatives are both prohibited methods of rapid weight reduction. And clearly being less fat is a competitive advantage for weight -classified athletes. Your next question relates to using anabolic steroids. A mother asks if her teenage son's recent flare of his acne and incredible moodiness is a sign he is taking steroids. Question 26-4 Which of the following is true about the use of anabolic ste­ roids in athletes? A) Anabolic steroids do not provide a competitive advantage to male athletes with adequate testosterone levels. B) Anabolic steroids do not provide a competitive advantage to female athletes because they do not have enough testoster­ one receptors. C) The body efficiently aromatizes excess androgens into estro­ gens, preventing any unwanted virilization in women who take anabolic steroids. D) Injectable anabolic steroids are generally safer than oral ana­ bolic steroids. E) Anabolic steroids do not provide any strength benefit to ath­ letes who are not strength training. Discussion 26-4 The correct answer is "D:' This question addresses several of the misconceptions about anabolic steroids. Anabolic steroids are clearly effective for all athletes. Supraphysiologic doses of androgens improve muscle size and strength even in young man with normal testosterone production. Female athletes also see a large advantage from anabolic steroids and do have adequate testosterone receptors. One of the principal side effects seen in women who use anabolic steroids is virilization. However, some athletes who use anabolic steroids use aromatase inhibitors to prevent the estrogenic side effects and increase the effectiveness of the drugs. Contrary to popular belief, injectable anabolic ste­ roids are generally safer than oral anabolic steroids because they bypass first-pass metabolism in the liver, resulting in less hepa­ tocellular damage. But do not forget, these drugs are banned by most sport governing bodies and are mostly illegal. To answer the mother's question, steroids may cause acne and changes in personality but it is more likely her son is a normal moody teen­ ager trying to get through puberty.

� QUICKQUIZ True or false: Most nutritional supplements used by athletes are effective for improving performance. A) True. B) False. Discussion The correct answer is ''A:' But it is a trick question. By far the most common nutritional supplements taken by athletes are calorie replacement beverages and food, which are obviously

778

MCG RAW- H I LL E D U CAT I O N S P E C I A LTY BOA R D REVI EW: P E D I AT R I C S

effective for fueling exercise. The next most common supple­ ment is creatine, which is also safe and effective for increas­ ing muscle strength and size in power athletes who weight train. However, there is a whole industry devoted to nutri­ tional supplements that are ineffective and may be dangerous, depending on their ingredients. Athletes may come to you with questions about which supplements are safe, effective, and permitted. It is nearly impossible to know the entire sup­ plement market, so each of these cases should be addressed individually. It may require a significant time investment to research and evaluate these products for the athlete. Also, many supplements are made on equipment that is used to produce other supplements or drugs, which may impair the purity of the product.

You are caring for a high school football player who discloses to you that he is taking anabolic steroids. He asks you to do some laboratory tests to make sure that he is not harming himself with the drugs. Question 27-1 Which of the following is NOT an appropriate response? A) Since the drugs you are taking are illegal, it is illegal for me to provide you with medical care that makes the use of those drugs safer. B) We will order a complete blood count because you might have an elevated level of red blood cells or plate­ lets, or both. C) We will check your cholesterol because elevated low-density lipoprotein (LDL) cholesterol and suppressed high-density lipoprotein (HDL) cholesterol is a common side effect of anabolic steroids. D) We will check your follicle-stimulating hormone (FSH) and luteinizing hormone (LH) levels to confirm that you are really taking the drugs you think you are. E) We will check your blood pressure because it can become dangerously high due to abuse of anabolic steroids.

Discussion 27-1 The correct answer is "A:' This statement highlights a common misconception. It is illegal to provide the athlete with anabolic steroids, but it is legal and ethical to provide him with appro­ priate medical care. Options "B" through "E" address the most common side effects of anabolic steroid use. In steroid-abusing athletes, it is reasonable to monitor for erythrocytosis, throm­ bocytosis, dyslipidemia, gonadal axis suppression (which is expected, not really a side effect) and hypertension.

� QUICKQUIZ True or False: It is better to be fit than unfit. A) True. B) False. Discussion The correct answer is "A:' This is an actual content area for the general pediatrics board exam. I'm not sure what type of ques­ tion you could be asked about this, but there is almost no situ­ ation where it is better to be unfit. Maybe if you were getting ready for a movie role where you really needed to look doughy. Be aware that fitness and fatness are related, but not the same thing. Fit, but fat, adults have lower cardiovascular risk than the fat but unfit and there is some evidence that this tracks from childhood. That is, a fat but fit child may do better over the long term than if he or she was not fit. BIBLIOGRAPHY

Bernhardt DT, Roberts WO, American Academy of Family Physicians, American Academy of Pediatrics. PPE: Preparticipation physical evaluation. 4th ed. Elk Grove Village, IL: American Academy of Pediatrics; 2010. Peterson AR, Bernhardt DT. The preparticipation sports evaluation. Pediatr Rev. 20 1 1 ;32(5) :e53-65. doi: 1 0 . 1 542/ pir.32-5-e53. Rice SG, American Academy of Pediatrics Council on Sports Medicine and Fitness. Medical conditions affecting sports participation. Pediatrics, 2008; 1 2 1 ( 4):841 -848. doi: 10. 1 542/peds.2008-0080.

35

S u bsta n ce Ab use LaTi sha L. Bader a n d Ross Mathiasen

Question 1 When scientists began to study addictive behaviors in , they believed that issues of morality or willpower were the reasons for dependence. Now, addiction is consid­ ered a chronic relapsing disease, with the brain being the affected organ. A) 1 920s. B) 1 940s. C) 1 9 1 0s. D) 1 930s. E) 1 950s. ___

Discussion 1

The correct answer is "D:' Scientist began studying addiction in the 1 930s, and over the intervening decades researchers have disproved a number of myths and misconceptions. Addiction is now considered a disease of both the brain and behavior. Drugs change the brain, both structurally and functionally. Biologic and environmental factors have been identified that cause the development and progression of the disease. The Diagnostic and

Statistical Manual of Mental Disorders, Fifth Edition (DSM-5) characterizes addiction as compulsive drug seeking and use, despite harmful consequences. Question 2 There are several ways to take drugs; however, the most addic­ tive routes of administration are: A) Ingested and smoked. B) Smoked and orally. C) Ingested and snorted. D) Suppositories and intravenously. E) Intravenously and smoked.

Drugs interact with the pleasure pathway of the brain by flood­ ing it with dopamine, producing an intense reaction. A drug's addiction potential increases when it produces this rush fol­ lowed by a quick return to "normal;' causing people to chase the "high:' Question 3 Addiction is a disease that has many variables, which include: A) Biologic. B) Environmental. C) Neurologic. D) Genetic. E) All of the above. Discussion 3

The correct answer is "E:' Scientists estimate that 40% to 60% of a person's vulnerability to addiction is related to genetics, in addition to mental health, stage of development, and other medical conditions. How an individual responds to substance use and the thought patterns and beliefs that reinforce its use also help determine vulnerability. No one factor can predict an individual's risk for developing the disease of addiction. Rather, addiction is a collective influence of these factors that predis­ pose and reinforce aspects of addiction.

Discussion 2

Question 4 Individuals are times more likely to suffer from depen­ dence if one parent struggled with addiction himself or herself. A) 2. B) 3. C) 4. D) 5. E) 6.

The correct answer is "E:' Drugs can be taken into the body in various ways, but when a drug is smoked or injected it begins to interact with the brain in seconds. When nicotine is smoked it begins to affect the brain in just 7 seconds-as fast as heroin.

The best answer is "C:' The average rate of addiction in the pop­ ulation ranges from 8% to 1 5%. An individual with one parent

__

Discussion 4

779

780

MCG RAW- H I LL E D U CAT I O N S P E C I A LTY BOA R D REVI EW: P E D I AT R I C S

TABLE 3 5 - 1

RISK AND P ROTECTIVE FACTO RS FOR

DRUG A B U S E AND A D D I CT I O N

Risk Factors

Protective Factors

Agg ressive behavior i n c h i l d hood

Good self-control

Lack of parental su pervision

Pa renta l mon itoring and su pport

Poor social s ki l l s

Positive relati o n s h i ps

Drug experimentation

Acade m i c com petence

Ava i l a b i l ity of d rugs at school

School a ntidrug policies

Com m u n ity poverty

Neigh borhood pride

Discussion 6

The correct answer is "C:' The age of first use is important because of the implications for ongoing brain development, particularly of the prefrontal cortex. The longer an individual delays the age of first use, the more prefrontal cortex advance­ ment can occur. This part of the brain assesses situations, makes decisions, and modulates our emotions. Introducing drugs into our brains before they are fully developed can cause profound and long-lasting consequences. Question 7 Once a patient has been identified with a substance use disor­ der and has been referred for and participated in treatment, it is important to reassess the disease on a frequent basis because addiction has a rate of relapse. A) 10% to 20%. B) 30% to 50%. C) 40% to 60%. D) 50% to 70%. E) 60% to 80%. ___

Reproduced w i t h p e r m i s s i o n from National I n stitute on D r u g A b u s e . How science has revol ution ized the u n d e rsta n d i n g o f d rug a d d icti o n . In: Drugs, Brains a n d Behavior: The Science ofAddiction. N I H P u b N o. 1 4-5605. Bethesda, M D : National 20 1 4 (orig i n a l p u b l ication 2007).

I n stitutes of Health;

who has struggled with addiction is four times more likely to become addicted. An individual with two biologic parents who have suffered from addiction is six times more likely to become addicted. (Some studies cite a two- to ninefold increased risk for developing alcoholism.) Genetic factors also influence receptors for drugs (eg, A1 allele, dopamine D 2 receptor gene), levels of tolerance, and withdrawal as well as our predisposition for reac­ tions to substances. (See Table 35- 1 .) Question 5 Consequences of adolescent drug use are vast and include medical, social, economic, and criminal justice costs such as: A) Underachievement (declines in academics, dropout rates). B) Delinquency, violence. C) Unplanned pregnancies and infectious diseases. D) Depression. E) All of the above.

Discussion 7

The correct answer is "C:' As with any other chronic, progres­ sive disease (eg, hypertension, asthma, or diabetes), addiction has aspects of relapse within the disease model. It is important to remember that if an individual relapses following treatment, it does not mean that treatment failed. It may simply mean that the individual needs a higher level of care or return to effective means of coping with the disease. It is important to assess a person's recovery capital (supportive aspects they have around them: recovery community, financial support, access to services, etc) and how improvements can be made to provide adaptive coping. This is a lifelong disease and should be viewed as such. You are evaluating a 14-year-old boy at a well-child visit. You have some suspicion that he has been using substances. His brother, who is 2 years older, has experimented with mari­ juana and tobacco, and an older cousin has been in trouble with the law.

Discussion 5

The correct answer is "E:' Alcohol and drug use increase the risk of all these factors in adolescents. Addiction develops from the interaction of social, cognitive, cultural, attitudinal, personality, and developmental factors. Temperament, hyperactivity, high novelty seeking, low harm avoidance, and early use of violent behavior are cited risk factors for adolescent drug use. Question 6 The highest percentages of first-time substance users are between the ages of: A) 2 1 and 25 years. B) 18 and 20 years. C) 16 and 17 years. D) 14 and 15 years. E) 12 and 13 years.

Question 8 Which risk factors for use does this example illustrate? A) Biologic. B) Neurologic. C) Environmental. D) Genetic. E) All of the above. Discussion 8

The answer is "C:' Home and family represent significant risks factors during childhood and increase the early use of sub­ stances. Individuals in the child's environment, such as parents or older family members, who abuse substances or engage in criminal behavior increase the risk for children to develop a substance problem. Additionally, influences such as peers and

CHAPTER 35

the school milieu represent other environmental factors. Peer influence-as well as academic failure or poor social skills-has a strong effect on the likelihood that a child will use. Question 9 If you suspect that your adolescent patient may be struggling with substance use you may screen him or her using which of the following brief questionnaires? A) CAGE. B) SASSI-3. C) CASA. D) SADQ. E) CRAFFT.



S U B STA N C E A B U S E

781

TABLE 35-2 S U M M A RY O F DSM-5 D I A G N OSTIC

CRITERIA F O R S U B STA N C E USE D I S O R D E R

1 . Esca lation o f u se-ta king more o f the su bsta nce or u s i n g it for longer periods of time. 2. I na b i l ity to q u it or decrease use of the su bsta nce. 3 . A lot of time i s spent usi ng, recovering from or seeki n g the s u bsta nce. 4. Repeated attem pts to q u it or l i mit use 5 . Neglected d uties in order to u se. 6. Contin ued use despite negative effects. 7.

Giving up other activities to use.

8. Conti n u i ng to use despite self-identified hazards of u se.

Discussion 9

The correct answer is "E:' The CRAFFT is a behavioral health­ screening tool for use with children younger than age 2 1 and is recommended by the American Academy of Pediatrics' Com­ mittee on Substance Abuse. It is printed on a pocket card and can be requested for clinical use. It has been translated into sev­ eral languages. The screening questions are as follows: C - Have you ever ridden in a CAR driven by someone (in­ cluding yourself) who was "high" or had been using alcohol or drugs? R - Do you ever use alcohol or drugs to RELAX, feel better about yourself, or fit in?

9. Physical o r psychological problems related to use or made worse by u se. 1 0. Tolera n ce to the s u bsta nce. 1 1 . Withd raw! sym ptoms occ u r when not u s i n g . N otes: Two or th ree sympto m s = m i l d su bsta nce use d isorder. Fou r or five sym pto m s = moderate su bsta nce use d i sord e r. Six or m o re sym pto m s = severe su bsta nce use d i sord e r. Data from American Psyc h iatric Associati o n . Diagnostic and Statistical Manual of Mental Disorders. 5th ed. A r l i n gton, VA:

American Psych iatric Association;

20 1 3.

A - Do you ever use alcohol or drugs while you are by your­ self, ALONE? F - Do you ever FORGET things you did while using alcohol or drugs? F - Do your family or FRIENDS ever tell you that you should cut down on your drinking or drug use? T - Have you gotten in TROUBLE while you were using alcohol or drugs? The DSM lists 1 1 criteria for substance use disorder. (See Table 35-2.) Question 1 0 A child who smoked tobacco and drank is 65 times more likely to use than a child who never used. A child who used marijuana is 104 times more likely to use than peers who never used. A) Marijuana; cocaine. B) Hallucinogens; alcohol. C) Nicotine; opiates. D) Cocaine; stimulants. E) Opiates; steroids. _ _ _ _ _

Discussion 1 0

The correct answer is "A:' Early use of substances places children on a deadly path. Research suggests that the earlier a substance is used, the greater the effect it can have on its user. In combina­ tion with early use, brain development, unstable relationships,

genetics, and exposure to violence can reinforce maladaptive substance use. Substance abuse is a complex issue with inter­ twined factors that pose daunting treatment challenges. Early prevention and focus on modifiable factors are good places to start. Question 1 1 Physicians can best provide support to their patients, fami­ lies, local schools, and community through: A) Education regarding the disease of addiction. B) Mandating treatment. C) Frequent communication. D) Both A and B. E) Both A and C. Discussion 1 1

The correct answer is "E:' Although physicians are capable of mandating treatment according to law, they are often the most helpful when they share information with others about the facts of addiction and implications of substance use, and use open, honest communication to provide treatment options, referrals, medication, and encouragement. Educating others that this is a disease and not just a choice is important. It may help those close to the abuser gain understanding into his or her destruc­ tive patterns.

782

MCG RAW- H I LL E D U CAT I O N S P E C I A LTY BOA R D REVI EW: P E D I AT R I C S

Question 1 2 You recognize symptoms of early adolescent drinking in one of your patients. You know this patient well enough to recog­ nize that he responds better to education rather than fear tac­ tics. As you begin to explain the physiological consequences of alcohol use, you emphasize that it results in damage to which of the following? A) Brain. B) Endocrine system. C) Liver. D) Growth. E) All of the above. Discussion 1 2

The correct answer is "E:' Drinking can harm the liver, bones, heart, endocrine system, and brain, and interfere with growth. The brain is changing during adolescence, and when it is exposed to alcohol the effects can be lasting. Research suggests that adolescents may be less sensitive to some of the unpleasant effects of intoxication and more sensitive to alcohol's harmful effects on the brain. Elevated liver enzymes have been found in adolescents who drink, along with higher gamma glutamyl transpeptidase (GGT) and alanine aminotransferase (ALT). Moreover, young drinkers who also are overweight or obese exhibit elevated levels of serum ALT with even modest amounts of alcohol intake. Youth drinking has also been associated with early puberty, especially among females. Female teens are more sensitive to the consumption and long-term effects of alcohol than males. They also become more cognitively impaired, com­ pared with males. Alcohol is one of the five most significant risk factors for diseases, with more than 60% of alcohol-related diseases being chronic conditions, including cancer, cirrhosis of the liver, diabetes, and cardiovascular disease. Question 1 3 Parents report that the following behaviors are occurring in their child. You recognize they are all behavioral conse­ quences of alcohol use EXCEPT: A) Arguing with friends. B) Academic problems. C) Normal eating patterns. D) Risky sexual behavior. E) Sleep disruption. Discussion 1 3

The correct answer is "C:' When an adolescent begins drinking, the following behaviors can suggest difficulties: hangovers, aca­ demic problems, difficulties at work, arguing with friends and family, unwanted sexual activity, changes in eating patterns or weight, injuries, and damage to property. Question 1 4 Primary care physicians play an important role in supporting recovery of their substance-using patients. They may be the first to assess a patient's substance use or to follow up with the patient during or after treatment by providing medically assisted treatment. If the patient has struggled with tobacco

use, the provider may prescribe which of the following to help with cravings or relapse? A) Buprenorphine. B) Bupropion. C) Acamprosate. D) Naltrexone. E) Disulfiram. Discussion 1 4

The correct answer is "B:' Medications used for tobacco addiction include nicotine replacement therapies (patch, inhaler, gum), bupropion, or varenicline. Medications for opioid addiction include methadone, buprenorphine, or naltrexone. Medications for alcohol and drug addiction include naltrexone, disulfiram, or acamprosate. These medications can interfere with triggers for stress and cues linked to substance use (such as people, places, things and moods) and maintain recovery. Be careful to know the best practices for prescribing these medications to patients younger than 18 years of age because some have not been sys­ tematically studied or are regulated differently by law. The father of your 1 5-year-old male patient expresses con­ cern that his son is using drugs and brought him in today, seeking to have him tested for drug use. You recognize that before ordering any tests, several issues must be addressed. Question 1 5 Prior to testing for drugs of abuse, which of the following are important considerations that need to be discussed with the patient and family? A) False-positive and false-negative test results. B) Plan of action if the test results come back positive or negative. C) Specific concerns that prompted the request for drug testing. D) Consent of the patient. E) Specimen collection method. F) All of the above. Discussion 1 5

The correct answer is "F:' There are many issues related to test­ ing for drugs of abuse that both physician and family should be aware ofbefore the test is performed. This is one reason meeting with a health care professional is recommended over reliance on at-home or school-based testing. The plan of action after obtaining test results should be discussed before testing, includ­ ing whether referral to a specialist will be made, and how the concerns that prompted the request (including physical, mental health, and behavioral changes) will be further addressed. The adolescent's consent for testing should be obtained. Consent may be waived in special circumstances, such as an emergency (eg, patient with unexplained altered mental status) . Referral to a specialist should be performed if the patient declines con­ sent and the provider has suspicion for drug use. False-positive and false-negative testing is very important to discuss prior to testing. The timing of drug use, cross-reactivity between other drugs (see Table 35-3), proper specimen collection, tampering or masking, and the laboratory panels, including the drug or

CHAPTER 35

TABLE 35-3 EXA M P L E S OF CROSS-REACTIVE

S U B STA N C E S IN DRUGS O F ABUSE LABO RATO RY

TESTI N G

Drug of Abuse

M a y Cross- React With•

Phencyc l i d i n e (PCP)

Dextromethorphan

Coca i n e

Amoxici l l i n

Opiates

Fluoroq u i nolones

Benzod iazepi nes

Sertra l i ne

Ca n n a b i noids

I b u profen, n a p roxe n

A m pheta m i n e

Fluoxeti ne, bu propion

'Su bsta nces m a y c a u s e fa lse - positive d rug o f a buse test.

metabolite tested for and cutoff values for a positive or negative test, all may affect the validity of the test. It is imperative that the clinician realize that drug testing is an adjunct to management of substance abuse disorders and does not preclude manage­ ment of substance abuse. Question 1 6 In the primary care setting, which of the following is the most common specimen type collected when testing for drugs of abuse? A) Blood. B) Breath. C) Urine. D) Saliva. E) Hair. F) Sweat. Discussion 1 6

The correct answer is "C:' Each type of specimen has benefits and limitations to its utility. The timing of the use as related to the specimen collection must be considered. For example, hair can­ not be used to detect drug or alcohol use within the previous 7 to 10 days, whereas blood may be used to detect a substance used within the previous 12 hours. Obtaining a blood sample is inva­ sive. Obtaining a urine sample is less invasive than blood sam­ pling but is highly susceptible to tampering. Breath, saliva, and sweat testing are susceptible to tampering, and each has its own limitations in regard to timing of drug intake related to collection. A 1 2-year-old girl presents with symptoms that include shortness of breath during exercise and a persistent cough. She has taken some over-the-counter {OTC) medicine for respiratory concerns. Because of the sudden onset of symp­ toms, your initial assessment includes the possibility that she might be using substances. Question 1 7 What is considered a gateway drug for youth? A) Alcohol. B) Marijuana.



S U B STA N C E A B U S E

783

C) Stimulants. D) Tobacco. E) None of the above. Discussion 1 7

The correct answer is "D:' Tobacco use, specifically smoking, is the leading cause of preventable disease and mortality in the United States. The onset of tobacco use is usually during adoles­ cence and thus should be a focus of identification and prevention. Longitudinal research suggests that there have been declines in lifetime tobacco use in youth, due to strong cohort effects and a decrease in supportive attitudes toward smoking. Information from self-reports suggests that smoking is a less attractive quality in a romantic partner than in previous survey years. It is sur­ mised that external variables such as health campaigns, adverse publicity, and increases in federal tobacco taxes also make it less attractive for youth to purchase and use the substance. In some instances, as here, peer pressure can have positive effects, too. •



Helpful Tip E p i d e m i o l o g i c stu d ies have s h own that n icoti ne use is

1 1 1 r a gateway to the use of m a rij u a n a a n d coca i ne.

Question 1 8 True or False: Campaigns for e-cigarettes target advertise­ ments to youth smokers, in addition to current smokers as a means for smoking cessation. A) True. B) False. Discussion 1 8

The correct answer is "A:' Although many e-cigarette campaigns suggest that this form of smoking can help an individual with cessation effort, or provide a more polite way to smoke in pub­ lic, the tobacco industry continues to spend a large portion of its marketing budget targeting young smokers. Many early tobacco campaigns included cartoon animals (eg, Camel) to entice young individuals to try their product, or used flavors to decrease the harsh smell and taste of tobacco. £-cigarettes are also following this trend with flavored cartridges, such as bubble gum, and starter packs that are being sold in kiosks at shopping malls. The health risks of long-term use of e-cigarettes are not yet known, and they can be used to smoke illicit drugs. Question 1 9 Which of the following remains the leading cause of prevent­ able disease and death in the United States? A) Drunk driving. B) Alcohol poisoning. C) Cigarette smoking. D) Smokeless tobacco. E) Cocaine use. Discussion 1 9

The correct answer is "C:' This is a test of whether you are pay­ ing attention. Of course the answer is cigarettes-you learned

MCG RAW- H I LL E D U CAT I O N S P E C I A LTY BOA R D REVI EW: P E D I AT R I C S

784

that earlier. There are many reasons for this continued cause of mortality. Nicotine is highly addictive and works to change the mind and mood in an average of 7 seconds (which is similar to heroin). Although studies suggest that lifetime prevalence of cigarette use is on the decline, adolescents continue to make up a large portion of new users. Taxes are one way the government has attempted to decrease the use of tobacco, especially in teens; and health campaigns support education to increase disap­ proval rates. Because of its fast -acting nature, ability to change the mind and mood, easy access, and affordability, the use of nicotine is often seen in individuals who experience trauma or have mental health conditions. Question 20 Which of the following is also indicated as a risk factor for early initiation of substance use, especially tobacco? A) Socioeconomic status. B) Race or ethnicity. C) Early puberty. D) Gender. E) Living in an urban setting. Discussion 20

The correct answer is "C' Studies recognize several risk factors that influence an individual's chances of using substances. Early maturation seems to exacerbate a number of factors, such as peer influence, appearance of being older (access to substance), and even within the youth population tobacco has reportedly been used to offset weight gain, especially in young girls. Remember that early substance use has been associated with several nega­ tive outcomes, such as early pregnancy and criminal convictions. Each year that an individual can delay the onset of use decreases the likelihood of developing a lifetime substance use disorder. •

Helpful Tip

�A

=-

r1 1 r

l o n g i tu d i n a l study in c h i l d re n who were fo l l owed

over a period

1 5 yea rs

(ages

11

to

26 years)

s u g g ested

that i n d ividua l s who stru g g l e d with bei ng overwe i g h t or o b e s e h a d the h i g h est l i ke l i hood o f bei ng reg u l a r s m o kers.

An 1 1 -year-old boy presents to your office for a sports physi­ cal. During the review of symptoms and last year's history, his parents report a concern about changes in his behavior and a slight drop in his grades. He has started hanging out with different friends on the weekends and is being slightly secretive about belongings, such as his backpack. Question 2 1 What substance is the most widely used by youth? A) Tobacco. B) Alcohol. C) Marijuana. D) Stimulants. E) Synthetic marijuana.

Discussion 2 1

Th e correct answer i s "B:' Alcohol remains the most widely used substance by youth. Although declines in use have been reported over the years, consumption remains high, with approximately 3 in 1 0 children having consumed alcohol by the end of eight grade, and 7 in 1 0 students (68%) having consumed alcohol by the end of high school. Half of these individuals reporting being drunk at least once.





1 1 1r

Helpful Tip A l o n g itu d i n a l study s u g g ested that heavy episodic b i n g e d r i n ki n g pred icted the potential fo r driving while i ntoxicated (DWI) a n d rid i n g with i m pa i red d rivers. The study concl u d ed that pa renta l m o n itoring, especia l ly by fathers over mothers, was a protective fa cto r a g a i nst DWI.

Question 22 Marijuana can be taken in what forms? A) Orally. B) Mixed with food. C) Smoked. D) As hash oil in e-cigarettes. E) All of the above.

Discussion 22

The correct answer is "E:' Recreational use in the United States continues to be defined by smoking cannabis in rolled cigarettes ("joints"), in pipes or water pipes, or hollowed out cigarettes ("blunts") . With the recent advent and commer­ cialization of e-cigarettes, hashish oil can also be smoked or "vaped" using these devices. Individuals can also engage in "dabbing;' which is the consumption of a more concentrated version of cannabis that is placed on a heated surface and then inhaled as vapors. Increases in illicit drug use usually fluctu­ ates due to the changing beliefs and use of cannabis. The recent legalization of recreational marijuana use in the states of Col­ orado and Washington (20 14), serves to illustrate the chang­ ing beliefs of the overall population regarding this mind- and mood-altering substance. The longitudinal studies reveal that the perceived risk associated with marijuana use, as well as its disapproval rating, continues to decline in youth surveys, sug­ gesting changing beliefs regarding its negative consequences. A recent study surveyed youth's attitudes about the legalization of cannabis; it reported that 1 0% of non-cannabis-using students reported intent to initiate use if legal. Additionally, 1 8 % of life­ time users reported intent to use more often if marijuana was legal. Legalization appears to place an already high-risk group (males, white, cigarette smokers) at increased risk for contin­ ued use, and to position low-risk groups (noncigarette smok­ ers, religious individuals, with friends who disapprove) as a new group of users. Interestingly, the odds of using are reduced when friends disapprove.

CHAPTER 35

Question 2 3 How many 1 2th graders say they can get marijuana fairly eas­ ily or very easily if they want? A) 8 1 % to 90%. B) 75% to 82%. C) 50% to 60%. D) 65% to 75%. E) 85% to 90%.



S U B STA N C E A B U S E

785

Question 25 You are trying to identify some common behavioral conse­ quences of cannabis use and come across a list in a scientific journal. All of the following sequelae are listed EXCEPT: A) Increased tardiness. B) Delayed development of social skills. C) Decreased academic performance. D) Improved ability to shift attention. E) Increased accidents.

Discussion 23

The correct answer is "A:' Marijuana is believed to be a read­ ily accessible substance for youth. Research suggests that as far back as 1 975, 8 1 % to 90% of 1 2th graders each year said that they could get marijuana fairly or very easily if they wanted it. There are also synthetic versions of the drug, originally sold over the counter as Spice and K2. Synthetic cannabis was adver­ tised as a herbal material that has been sprayed with one or more synthesized chemicals that fall in the cannabinoid family. In March 20 1 1 , the Drug Enforcement Agency (DEA) made this substance illegal. In 2 0 1 3 , youth statistics suggested that syn­ thetic marijuana was the second most widely used illicit drug after marijuana among 1 2th graders. For 8th graders, it was the third, behind marijuana and inhalants. Question 24 Numerous physiologic consequences are associated with can­ nabis use. They include short-term and long-term effects. Which is not a short-term effect of cannabis use? A) Altered senses. B) Decreased heart rate. C) Impaired body movement. D) Difficulty with thinking and problem solving. E) Changes in mood. Discussion 24

The correct answer is "B:' The highest density of cannabinoid receptors is found in parts of the brain that influence pleasure, memory, thinking, concentration, sensory and time perception, and coordinated movement. Marijuana overactivates the endo­ cannabinoid system, causing the "high" and other effects that users experience. These effects include altered perceptions and mood, altered sense of time, impaired coordination, difficulty with thinking and problem solving, and disrupted learning and memory. Reported physical effects include breathing problems (daily coughing, phlegm, and higher risks of lung infections) , increased heart rate (raising heart rate for up to 3 hours after smoking, increased change of heart attack), and problems with child development during and after pregnancy (brain and behavioral problems in infants; and problems with atten­ tion, memory, and problem solving in children) . Additionally, research suggests that a loss of mental stability is one significant side effect from recreational use. Temporary hallucinations and paranoia have been reported, as well as strong links between cannabis use and later development of and vulnerability to psy­ chosis. Long-term use is associated with physiological conse­ quences that include impaired memory and loss of an average of eight IQ points.

Discussion 25

The correct answer is "D:' Cannabis compromises the ability to learn and remember information, thus affecting the ability to per­ form at work, home, and in social settings. Individuals fall behind in acquiring social skills, job training, and intelligence. Students are likely to attain lower grades and are less likely to graduate. They struggled with attention, memory, and learning. Canna­ bis users are more likely to have problems on the job, increased absences, tardiness, accidents, and to spend time on personal matters or daydreaming rather than on academics or work. Question 26 When an illicit drug first appears, its alleged benefits and consequences are not fully known. This period of time is called: A) Generational forgetting. B) Grace period. C) Trending. D) Legalization. E) Popularization. Discussion 26

The correct answer is "B:' The benefits of using a drug spread much faster than its adverse consequences, giving it a "grace period" before individuals receive an informed message about its benefits and dangers. Broadcasting of alleged benefits requires only a couple testimonials or rumors, usually spread by word of mouth, social media, or the Internet, whereas the adverse con­ sequences (overdose, deaths, disease, and addiction) take much longer to disseminate. The positives of drug use, or euphoric recall, persist among that generation of users until they are matched with enough negative outcomes or "war stories:' When the negatives begin to outweigh the benefits, use begins to decline (as seen recently with bath salts) . What is unfortunate is that out-of-favor drugs often make a comeback once enough people in a particular age group have not been directly affected by the negative consequences of use, or during periods when public campaigns are targeting other substances. This leads to "generational forgetting;' in which an older drug is rediscovered by a younger generation. For example, LSD and methamphet­ amine made a comeback in the 1 990s, after decreased use in the 1 960s. Currently, heroin use is spiking, after a period of decline. Your 14-year-old patient suffered a compound fracture in his right arm approximately 8 weeks ago when he fell while hik­ ing. It was treated appropriately with reduction and immo­ bilization. He continues to report intense pain, and you have

786

MCG RAW- H I LL E D U CAT I O N S P E C I A LTY BOA R D REVI EW: P E D I AT R I C S

ruled out pathology related to the injury. He requests addi­ tional opiates for his symptoms. You have some concern that he might be misusing the prescription. You begin to investi­ gate signs of possible misuse of the drug. Question 27 Which of the following is NOT typical of the opiate toxidrome? A) Lacrimation. B) Respiratory depression. C) Central nervous system depression. D) Pinpoint pupils. E) Bradycardia. Discussion 27

The correct answer is "A:' When deciphering whether or not someone could be suffering from opioid intoxication, it is help­ ful to think of opioids as "something that slows things down:' In addition to those listed above, symptoms may include hypoten­ sion, muscle flaccidity, and hypothermia. Lacrimation would be viewed as an increase in tear production, not a decrease. Question 28 What is the antidote to opioids? A) Physostigmine. B) 2-PAM. C) Naloxone. D) Atropine. E) N-acetylcysteine. Discussion 28

The correct answer is "C:' Atropine and pralidoxime (2-PAM) are used in organophosphate poisoning. N-acetylcysteine is used in acetaminophen toxicity. Physostigmine is sometimes used with benzodiazepines as the treatment for anticholinergic toxicity. Question 29 If ingested, which of the following is associated with the opiate toxidrome? A) Methadone. B) Heroin. C) Dextromethorphan. D) All of the above.

Question 30 Which of the following hallucinogens do you suspect the patient ingested? A) Lysergic acid diethylamide (LSD ) . B) Phencyclidine (PCP) . C) Mescaline. D) Dextromethorphan. Discussion 30

The correct answer is "B:' All options are hallucinogens­ substances that alter how a person feels, thinks, or senses. In addition to the effects of the other hallucinogens listed (which include tachycardia, hypertension, hyperthermia, acute psy­ chosis, and hallucinations), PCP is known to cause vertical or rotary nystagmus. Repetitive hallucinogen abuse is related to the euphoric effects provided, as most hallucinogens do not cause physiologic addiction. Question 3 1 Which of the following is contraindicated in management of patients under the influence of hallucinogens? A) Administration of benzodiazepines. B) Placing the patient in a low stimulation environment. C) Placing the patient in physical restraints. D) Administration of intravenous normal saline. E) Administration of antihypertensive medications. Discussion 3 1

The correct answer is "C:' Physical restraints may increase agi­ tation. The hypertension, hyperthermia, and tachycardia are secondary to the agitation. Calming the patient with a low stim­ ulation environment and pharmaceutically with benzodiaz­ epines are often enough to control these symptoms. In extreme cases of hyperthermia and hypertension, cooling measures such as use of intravenous fluids and antihypertensive medications may be necessary. Question 32 Typical users of hallucinogens fit the following profile: A) Young; male; employed. B) Young; female; legal problems. C) Young; male; unemployed. D) Young; male; legal problems; unemployed E) Young; female; unemployed; legal problems.

Discussion 29

The correct answer is "D:' Methadone is a prescription opiate medication. Heroin, an illicit drug, is a synthetic opiate. Dextro­ methorphan is structurally related to a synthetic opioid agonist and may cause symptoms of opiate ingestion, depending on the amount ingested. During your shift in the emergency department, you are evaluating a 17 -year-old boy with acute psychosis that devel­ oped while attending a party. In addition to hypertension and tachycardia, you note rotary nystagmus on examination of his eyes.

Discussion 32

The correct answer is "D:' The twenty-first century synthetic drugs (eg, bath salts) are finding company with previous gen­ erations of hallucinogens, thanks in part to "generational for­ getting:' When a generation becomes numb to the effects of a drug, lacking exposure to education and prevention cam­ paigns, as well as decreased peer pressure, it is common for a class of drugs to make a resurgence. The typical users of these substances are young and male, with similar education levels, unemployment, and legal problems, while differing in ethnicity and race identity.

CHAPTER 35

You are evaluating another 1 7-year-old boy who presents to the emergency department with agitation and hallucina­ tions, which developed while he was attending a party. His friend states that your patient also had a nosebleed earlier in the night. You astutely suspect cocaine intoxication, a sympa­ thomimetic toxidrome, as the cause of his symptoms. Question 3 3 Which o f the following i s NOT consistent with a sympatho­ mimetic toxidrome? A) Tachycardia. B) Dry skin. C) Muscular rigidity. D) Mydriasis. Discussion 33

The correct answer is "B:' Hallmarks of the sympathomimetic toxidrome include agitation, hyperthermia, tachycardia, hyper­ tension, mydriasis (dilated pupils), and diaphoresis. Distin­ guishing sympathomimetic from anticholinergic toxidromes (eg, diphenhydramine ingestion) is often difficult due to over­ lapping features, including tachycardia and pupillary dilation. •

� r1 1r

:5.

Helpful Tip I ntoxication with either sym pat h o m i m etics or a nticho­ l i n e rgics cau ses tac hyca rdia, hyperthermia, myd riasis (d i l ated p u p i l s), and h a l l u ci n ations, but: Sym pat h o m i m etic

=

d i a p h o resis

(wet

skin)

and

hyperactive bowe l sou n d s . Antic h o l i nergic

=

d ry skin ("d ry a s a bone") a n d

hypoactive bowe l s o u n d s .

You now know that your patient has ingested cocaine. Snort­ ing cocaine is associated with nosebleeds and, with chronic use, nasal septum perforation due to its vasoconstrictive effects. He is now complaining of chest pain, and you order an electrocardiogram (ECG) to evaluate the cardiac rhythm and look for ischemic changes, as dysrhythmias and ischemia are associated with cocaine-related chest pain. Question 34 Which of the following medications is contraindicated in patients who have ingested cocaine? A) Beta-blockers. B) Aspirin. C) Benzodiazepines. D) Nitroglycerin. E) Sodium bicarbonate. Discussion 34

The correct answer is "A:' Beta-blockers should be avoided in patients who have ingested cocaine due to concern of unop­ posed alpha-receptor stimulation. Aspirin and nitroglycerin



S U B STA N C E A B U S E

787

are safe for use. Along with supportive cares, benzodiazepines may be utilized for sedation, hyperthermia, tachycardia, sei­ zures, and muscular rigidity. Sodium bicarbonate is the treat­ ment for wide-complex dysrhythmias associated with cocaine intoxication. •

Helpful Tip

�A

:5.

r1 1r

b i n g e pattern of coca i n e use is m o re co m mo n ly

associated with s m o k i n g a n d i nj ecti ng d u e to fa ster a bsorption and s h o rter d u ration of action with these routes than with i ntra n a s a l use.

A 13-year-old girl presents to the emergency department seizing. Her parents found an empty bottle of methylpheni­ date next to her. Question 3 5 Which medication is the best choice for treatment of her seizure? A) Lorazepam. B) Phenytoin. C) Haloperidol. D) Phenobarbital. E) Options A and B are equally efficacious. Discussion 35

The correct answer is ''A:' Benzodiazepines are the treatment of choice for amphetamine-induced seizures. Other anticon­ vulsant medications, including phenytoin and phenobarbital, are generally not indicated for toxicologic-induced seizures. Amphetamines may be used medically, for example, in treat­ ment of narcolepsy and attention deficit hyperactivity disorder (ADHD), as in this case. These prescription medications are sometimes used for recreational purposes, as amphetamines increase norepinephrine and dopamine release, causing a feel­ ing of pleasure, self-confidence, and well-being. Question 36 MDMA (also known as ecstasy or Molly) is an amphetamine that is used recreationally. If a patient who is known to have used MDMA presents seizing, what electrolyte abnormality is most likely to be present? A) Hypokalemia. B) Hyperkalemia. C) Hypernatremia. D) Hyponatremia. Discussion 36

The correct answer is "D:' Similar to cocaine, amphetamines cause tachycardia, hypertension, hyperthermia, and diaphore­ sis. In addition, MDMA is associated with hyponatremia due to increased release of vasopressin (antidiuretic hormone) . MDMA i s known for being used recreationally at dance parties or "raves:' Isotonic fluid loss due to sweating while dancing and fluid replacement with oral free water compound the risk for hyponatremia.

788

MCG RAW- H I LL E D U CAT I O N S P E C I A LTY BOA R D REVI EW: P E D I AT R I C S

Methamphetamine (meth, crank, ice) can be used orally, intra­ venously, or inhaled. The symptoms and medical treatment of amphetamine ingestion are similar to those for cocaine. Sup­ portive care includes cooling if needed and intravenous hydra­ tion, and benzodiazepines are the mainstay of treatment. You are the treating physician on call when a group of intoxi­ cated adolescents are brought into the emergency depart­ ment. The teens have all been at a party together and give you conflicting reports about the substances available there. A friend who arrives with them reports it was a "pharm'' party. You quickly review the classes of over-the-counter medica­ tion and prescriptions and notice the symptoms observed best fit with stimulant intoxication. Question 3 7 Symptoms of intoxication following stimulant use include: A) Inappropriate aggressive behavior, mood lability, slurred speech, nystagmus, and stupor. B) Drowsiness, slurred speech, impaired attention, psychomo­ tor agitation, and pupillary constriction. C) Dizziness, incoordination, unsteady gait, depressed reflexes, general muscle weakness, and belligerence. D) Affective blunting, changes in sociability, hypervigilance, interpersonal sensitivity, chills, and pupillary dilation. Discussion 37

The correct answer is "D:' The symptoms listed in option "/\.' suggest sedative, hypnotic, and anxiolytic intoxication such as occurs with benzodiazepines. Option "B" suggests opioid intoxication. Option "C" suggests inhalant intoxication. After marijuana, prescription medications are the most common drugs abused by adolescents. In contrast to several previously described drugs, stimulants are often used to help teens study. Adolescents misuse stimulants prescribed to themselves or to others, such as Adderall (amphetamine and dextroamphet­ amine) , as performance enhancers. Casually referred to as the "midterm drug;' stimulants are used in both academic and sports settings to increase a person's competitive edge. Individu­ als exposed to stimulants or amphetamine-type stimulants can develop disordered use as rapidly as 1 week later. Studies sug­ gest that approximately 1 in every 6 high school seniors has had exposure to prescription stimulants, for medical or nonmedi­ cal reasons. Researchers went on to clarify that 59.3% of high school seniors had used prescription stimulants for medical reasons only, 22.9% reported medical use followed by nonmedi­ cal use, and 1 7.8% reported nonmedical use before medical use. A 1 2-year-old boy is brought to the emergency department by emergency medical services (EMS) after his parents found him in his room with altered mental status. His parents report he seemed normal at dinner 20 minutes before they found him in his room. EMS reports he was lethargic with a pulse of 40 beats per minute. In the emergency department, you are evaluating him 30 minutes after EMS arrived on scene. He is now acting normally and has normal vital signs.

Question 38 What is the most likely cause of this patient's presentation? A) LSD ingestion. B) Hydrocarbon inhalation. C) Concussion. D) Calcium channel blocker overdose. E) Beta-blocker overdose. Discussion 38

The correct answer is "B:' Hydrocarbons are a type of inhaled substance used for their euphoric effects. Inhalants are gases or fumes that can be inhaled for the purpose of getting high. Most are found as inexpensive household items and can be purchased and possessed legally by youth (glue, nail polish remover, gaso­ line, solvents, etc) . Sometimes referred to as "kids drugs;' this class of drugs differs from other drugs of abuse because it is most commonly used by younger adolescents, and then use declines as they age. The risk associated with use was well com­ municated in the mid- 1 990s by Partnership for a Drug-Free America, which educated youth and improved the accurate per­ ception of risk. Currently, over 80% of adolescents would disap­ prove of using inhalants. Inhalants have a rapid onset and their effects usually last no more than 30 minutes. The euphoric effect may be followed by depressive symptoms, including bradycar­ dia and lethargy, followed by return to baseline. Management is supportive, including exposure to fresh air. Prolonged symp­ toms suggest an alternate etiology. Long-term use of inhalants is associated with fatal dysrhythmias, sometimes termed "sud­ den sniffing death;' due to increased sensitivity of the heart to catecholamines. •

� I

1 1r

Helpful Tip G l u e s n iffe r's ra s h i s a periora l d e rmatitis ca u sed b y t h e d rying effects of i n ha l a nts.

Question 39 Which of the following is ingested for its euphoric effect and is found in many over-the-counter cough and cold medications? A) Salicylic acid. B) Red dye #40. C) Dextromethorphan. D) Guaifenesin. Discussion 39

The correct answer is "C:' Dextromethorphan is ingested rec­ reationally for its euphoric effect, and is dose dependent. At lower doses, a stimulant effect may be noted, while at higher doses its effects are similar to phencyclidine (PCP), including hallucination and dissociation. Salicylic acid ingestion can lead to salicylic intoxication, in which clinically the patient will have metabolic acidosis and compensatory respiratory alkalosis. Red dye #40 is a coloring agent. Guaifenesin is an expectorant found in many dextromethorphan-containing medications, its side effects include nausea and vomiting.

CHAPTER 35

BIBLIOGRAPHY

American Psychiatric Association. Diagnostic and Statistical Manual of Mental Disorders. 5th ed. Arlington, VA: American Psychiatric Association; 20 1 3 . Blok BK, Cheung D S , Platts-Mills T F. First Aid for the Emergency Medicine Boards. New York, NY: McGraw-Hill; 2009. Johnston LD, O'Malley PM, Miech RA, Bachman JG, Schulenberg JE. Monitoring the Future National Survey

Results on Drug Use: 1 975-201 3: Overview Key Findings on Adolescent Drug Use. Ann Arbor, MI: Institute for Social Research, University of Michigan; 2014. Kandel ER, Kandel DB. A molecular basis for nicotine as a gateway drug. New Engl J Med. 2014;371 :932-943. Lanza HI, Grella CE, Chung PJ. Does adolescent weight status predict problematic substance use patterns? Am J Health Behav. 2014;38(5):708-7 16. Lee JS, McCarty CA, Ahrens K, King KM, Stoep AV, McCauley EA. Pubertal timing and adolescent substance initiation. f Social Work Pract Addict. 20 1 4; 1 4(3) :286-307. Levy S, Siqueira LM, Committee on Substance Abuse. Testing for drugs of abuse in children and adolescents. Pediatrics. 2014; 1 33 ( 6) : 1 798- 1 807. Li K, Simo ?s-�orton BG, B �ooks-Russell A, Ehsani J, Hingson R. Dnnking and parentmg practices as predictors of impaired driving behaviors among US adolescents. J Stud Alcohol Drugs. 20 1 4;75 ( 1 ) : 5- 1 5.



S U B STA N C E A B U S E

789

Maxwell JC. Psychoactive substances-some new, some old: A scan of the situation in the US. Drug Alcohol Depend. 2014; 1 34:71 -77. McCabe SE, West BT. Medical and nonmedical use of prescription stimulants: Results from a national multi­ cohort study. J Am Acad Child Adolesc Psychiatry. 20 1 3;52( 12 ) : 1 272- 1 280. Meier MH, Caspi A, Ambler A, et al. Persistent cannabis users show neuropsychological decline from childhood to midlife. Proc Natl Acad Sci. 20 12; 1 09 (40) :E2657-E2664. National Institute on Drug Abuse. How science has revolu­ tionized the unde�standing of drug addiction. In: Drugs, . and Behavwr: The Science of Addiction. NIH Pub Brazns No. 1 4-5605. Bethesda, MD: National Institutes of Health; 20 1 4 (original publication 2007) . National Institute on Drug Abuse. Is there a link between marijuana use and mental illness? Research Report Series. NIH Pub No. 1 5-3859. Bethesda, MD: National Institutes of Health; 20 1 2 . Palamar JJ, Ompad D C , Petkova E. Correlates o f inten­ tions to use cannabis among US high school seniors in the case of cannabis legalization: Int J Drug Policy. 20 14;25(3) :424-435.

This page intentionally left blank

Fi na l Exa m

Answer key is on page 807. Discussion of the concept being tested can be found on the page number listed in bold text after each question. 1 . A teenage boy asks how tall he can expect to be as a full­ grown adult. His mother is 1 60 em tall and his father is 1 80 em tall. He is healthy and has had normal growth and development. His full-grown brother is 1 82 em tall. You tell him that his approximate expected adult height is: ( 1 ) A) B) C) D) E)

180 em 1 60 em 1 82 em 1 76.5 em 163.5 em

2. Which of the following is the most effective method of preventing unwanted pregnancy? ( 1 5) A) B) C) D) E)

Rhythm method Male condom Female condom Implantable etonogestrel rod Oral contraceptive pills

3. Which of the following is a characteristic of middle adolescence? (20) A) B) C) D) E)

Appearance of acne and body odor Mainly interested in same-sex relationships Desires privacy from parents Physically mature Becomes less influenced by peers

4. Which of the following is true of a typical adolescent girl's first few menstrual cycles? {2 1 ) A ) Anovulation i s rare B) Age of first menses is usually around 1 0 years in healthy well-nourished Americans

C) Typically precedes thelarche by 1 to 2 years D) Occurs at regular 30-day intervals E) Typical menstrual product use is 3 to 6 pads or tam­ pons per day 5. Which of the following is more characteristic of T-cell immunodeficiencies than of other types of immunodefi­ ciencies? (25-27) A) Vaccines tend not to produce an immune response B) Live virus vaccines may lead to active infection in the patient C) Skin abscesses are common D) Overwhelming infections occur with common bacterial pathogens E) Oral corticosteroids can provide effective treatment 6. Which of the following is a characteristic of mild persis­ tent asthma? {39) A) Has symptoms that wake from sleep 3 to 4 times per month B) Requires a short -acting beta -agonist daily for symptom control C) FEV 1 is less than 80% of predicted D) Never requires systemic corticosteroids for symptom control E) Has daytime symptoms less than 2 days per week. 7. Which of the following is appropriate treatment for an adolescent with intermittent asthma? {41 ) A) B) C) D) E)

Short-acting beta-agonists a s needed Low-dose inhaled corticosteroids Medium-dose inhaled corticosteroids Inhaled corticosteroids plus long-acting beta-agonist Inhaled corticosteroids plus leukotriene receptor antagonist

791

MCG RAW- H I LL E D U CAT I O N S P E C I A LTY BOA R D REVI EW: P E D I AT R I C S

792

80 Which of the following is the most effective long-term controller medication for young children with asthma? (42) A) B) C) D) E)

Short-acting beta-agonists Long-acting beta-agonists Inhaled corticosteroids Leukotriene receptor antagonists Mast cell stabilizers

90 Which of the following is the most common cause of food allergy in childhood? (45) A) B) C) D) E)

Soy Wheat Shellfish Peanut Milk

100 Which of the following is NOT a marker of toilet training readiness? (52-53) A) B) C) D) E)

Has sibling who was toilet trained at the same age Can walk to the toilet Can follow simple commands Wants to please caregivers Seems distressed or bothered by dirty diapers

1 1 . Which is NOT an appropriate intervention for managing a tantrum? (54) A) B) C) D)

Ignore the behavior until the child stops Time-out Place the child in a safe area to prevent injury Immediately give in to the child's demand so the tantrum does not escalate E) Offer to help the child after he or she calms down

120 Which of the following is a characteristic of good sleep hygiene? (58} A) B) C) D)

Go to bed when tired No naps during the day No electronic devices or gadgets in the bedroom Start planning for the child's bedtime when the parents become tired E) Set out pajamas in the morning so they will be ready for the child at bedtime

1 3 0 Which of the following is true about excessive infant crying (colic)? (59) A) Typically starts around 2 weeks of age B) Typically resolves by 12 to 1 6 weeks of age C) Is defined by crying more than 3 hours per day, more than 3 days per week, and more than 3 weeks in duration D) Is equally common in breastfed and formula-fed babies E) All of the above

140 Which of the following is NOT an anxiety disorder? ( 64) A) B) C) D) E)

Social phobia Obsessive compulsive disorder Specific phobia Panic disorder Separation anxiety disorder

1 5 0 Which of the following laboratory values is consistent with iron deficiency anemia? (73) A) B) C) D) E)

Elevated hemoglobin Low mean corpuscular volume Elevated ferritin Elevated haptoglobin Absent reticulocytes

16 0 Which of the following is associated with painless hematuria? (75-76} A) B) C) D) E)

Wilms tumor Pyelonephritis Nephrocalcinosis Rhabdomyolysis Urethral stricture

1 7 0 Which of the following is an appropriate first-line treatment for immune thrombocytopenia (ITP)? (77} A) B) C) D) E)

Observation Intravenous immunoglobulin (IVIG) Anti-D immune globulin Systemic corticosteroids All of the above

1 8 0 Which of the following laboratory findings is common in acute tumor lysis syndrome? (9 1 } A) B) C) D) E)

Hyperkalemia Hypophosphatemia Hypercalcemia Hypouricemia Hypernatremia

19 0 Which of the following is true regarding the treatment of sickle cell disease? ( 1 02) A) Penicillin prophylaxis should be started at birth B) Once the spleen involutes, patients with sickle cell disease are at increased risk for infections with viruses C) Treatment with hydroxyurea decreases risk of ischemic stroke by increasing the production of fetal hemoglobin (HbF) D) Siblings of the affected child have at least a 75% chance of being affected as well E) The standard 1 3-valent pneumococcal vaccine (PCV- 13) should be given before age 1

F I N A L EXAM

20. Which of the following cardiac conditions will cause a holosystolic murmur? ( 1 12, 1 14) A) B) C) D) E)

Atrial septal defect Ventricular septal defect Hypertrophic cardiomyopathy Patent ductus arteriosus All of the above

2 1 . Which of the following arrhythmias will present with the fastest ventricular rate? ( 1 22, 1 25) A) B) C) D) E)

Atrial fibrillation Supraventricular tachycardia Sinus tachycardia Third-degree heart block Pulseless electrical activity

22. Which of the following heart conditions is associated with Turner syndrome? ( 1 27) A) B) C) D) E)

Coarctation of the aorta Dilated aorta Bicuspid aortic valve Partial anomalous pulmonary venous return All of the above

23. Which of the following is true about tetralogy of Fallot? (127- 1 28) A) The murmur is due to the large ventricular septal defect (VSD) B) The murmur is due to pulmonary stenosis C) Oxygen therapy can reverse the left-to-right shut across the VSD during a "Tet" spell D) Ketamine may improve oxygenation by decreasing systemic vascular resistance during a "Tet" spell E) The condition is autosomal dominant 24. When evaluating a newborn with persistent cyanosis, which of the following conditions is UNLIKELY to be the cause? ( 1 29) A) B) C) D) E)

Truncus arteriosus Transposition of the great arteries (DTGA) Tricuspid atresia Ebstein anomaly Peripheral pulmonary stenosis

25. Which of the following is an early symptom of autism spectrum disorders? ( 1 33) A) B) C) D) E)

Attachment disorder Failure to roll over by 6 months of age Limited babbling Excessive imaginary play Higher activity level

793

26. Initial screening for autism spectrum disorders should occur at what age? ( 1 34) A) B) C) D) E)

< 6 months 6- 1 2 months 12- 1 8 months 1 8-24 months 24-30 months

27. Which receptive language skill is matched with the correct age of normal acquisition? ( 1 37) A) B) C) D) E)

Alerts or quiets to sound-birth Responds to own name- 1 2 months Stops when told "no" - 1 8 months Follows simple commands-24 months Answers simple questions-4 years

28. Which expressive language skill is NOT matched with the correct age of normal acquisition? ( 1 38) A) B) C) D) E)

Cooing-2 months Babbling-6 months Pointing-6 months Two-word phrases- 1 8 months Able to tell or retell a familiar story-36 months

29. Which of the following is NOT required by law? (144- 146) A) An individual education plan (IEP) for a student with dyslexia B) Continuing academic modifications for a student with a 504 plan who will attend college C) Braille education for a blind child D) Wheelchair access at a public elementary school E) An IEP for a student with a concussion who is unable to complete classwork now but is expected to make a full recovery 30. Which of the following is NOT a common laboratory finding in systemic lupus erythematosus (SLE)? ( 1 52) A) B) C) D) E)

Leukocytosis Lymphopenia Anemia Decreased C3 and C4 complement levels Hematuria

3 1 . Which of the following is NOT a major manifestation of acute rheumatic fever? ( 1 53) A) B) C) D) E)

Polyarthritis Erythema migrans Chorea Carditis Subcutaneous nodules

MCG RAW- H I LL E D U CAT I O N S P E C I A LTY BOA R D REVI EW: P E D I AT R I C S

794

32. Which of the following is a late manifestation of Henoch­ Schonlein purpura? ( 1 54) A) End-stage kidney disease B) Chronic abdominal pain C) Scarring of the skin on the buttocks and lower extremities D) Chronic headaches E) Chronic arthritis 33. Which of the following accurately describes systemic onset juvenile idiopathic arthritis (JIA) ? ( 1 57) A) B) C) D) E)

Gradual onset Girls affected more than boys Diffuse lymphadenopathy More than four joints involved HLA-B27 positive

34. Which of the following is NOT seen in juvenile dermato­ myositis (JDM)? ( 1 59) A) B) C) D) E)

Gottron papules Periungual telangiectases Distal muscle weakness Heliotrope rash Photosensitivity

35. What is the appropriate size of cuffed endotracheal tube for a 4-year-old? ( 1 68) A) B) C) D) E)

2.0 3.0 4.0 5.0 6.0

36. Which of the following substances can cause hepatic fail­ ure in overdose? ( 1 68) A) B) C) D) E)

Acetaminophen Ibuprofen Phenylephrine Diphenhydramine All of the above

37. What is the most appropriate place to check for a pulse in an infant who has become unresponsive? ( 1 69) A) B) C) D) E)

Carotid artery Chest wall (point of maximal impulse) Brachial artery Radial artery Femoral artery

38. What is the appropriate first treatment for a patient with ven­ tricular tachycardia with a pulse and poor perfusion? ( 170) A) Adenosine 0. 1 mg/kg IV B) Synchronized cardioversion at 0.5 J/kg

C) Nonsynchronized defibrillation at 2 J/kg D) Epinephrine 0 .0 1 mg/kg IV E) Application of ice to the face 39. What is the most common bacterial cause of acute otitis media? ( 1 80) A) B) C) D) E)

Streptococcus pneumoniae Nontypeable Haemophilus influenzae

Moraxella catarrhalis Staphylococcus aureus Pseudomonas aeruginosa

40. Which is NOT a complication of acute bacterial sinusitis? ( 1 88) A) B) C) D) E)

Subdural abscess Cavernous sinus thrombosis Meningitis Orbital cellulitis Tonsillitis

4 1 . What is the most common complication of a tonsillec­ tomy and adenoidectomy? ( 1 9 1 ) A) B) C) D) E)

Aspiration pneumonia Bleeding Dental trauma Dehydration Velopharyngeal insufficiency

42. What is the most common midline neck mass in children? ( 1 96) A) B) C) D) E)

Cystic hygroma Brachial cleft cyst Thyroglossal duct cyst Lymphoma Thyroid nodule

43. Which of the following detects sensorineural hearing loss? ( 197- 198) A) B) C) D) E)

Evoked otoacoustic emissions Tympanometry Pneumatic otoscopy Automated brainstem response Magnetic resonance imaging

44. Which is NOT a part of the evaluation of a febrile neonate? (204) A) B) C) D) E)

Blood culture Urine culture Sputum culture Cerebrospinal fluid culture Chest X-ray (if respiratory symptoms present)

F I N A L EXAM

45. Tetanus immunization is indicated in all of the following EXCEPT: (206) A) Patient with simple, clean wound and unknown vaccination history B) Patient with contaminated wound and unknown vaccination history C) Patient with a simple, clean wound who has received < 3 doses of the tetanus vaccine D) Patient with a simple, clean wound who has received � 3 doses of the tetanus, vaccine with the last dose received 5 years earlier E) Patient with a contaminated wound who has received � 3 doses of the tetanus vaccine, with the last dose received 6 years earlier 46. Which is NOT a sign of a pediatric skull fracture? (21 1 ) A) B) C) D) E)

Scalp hematoma Posttraumatic clear rhinorrhea Bruising behind the ear Palpable step-off or crepitus Forehead contusion

47. Which is a late sign of compartment syndrome complicat­ ing a tibia fracture? (218) A) B) C) D) E)

Pulselessness Pain Pallor Paresthesia Poikilothermia

48. In classic congenital adrenal hyperplasia due to 2 1 -hydroxlyase deficiency, which of the following adrenal steroids is NOT elevated? (219-22 1 ) A) B) C) D) E)

Androstenedione Aldosterone Testosterone 1 7-Hydroxyprogesterone Dehydroepiandrosterone

49. Which is NOT a cause of tall stature in childhood? (226) A) B) C) D) E)

Hypothyroidism Overnutrition Growth hormone excess Precious puberty Familial tall stature

50. Which is an example of precious puberty? (226-228) A) B) C) D) E)

8-year-old girl with breast buds 1 1 -year-old boy with pubic hair 1 2-year-old girl with menarche 7-year-old girl with breast buds 9-year-old boy with testicle enlargement

795

5 1 . Which laboratory value is NOT characteristic of Graves disease? (23 1 ) A) B) C) D) E)

Low thyroid-stimulating hormone (TSH) Elevated free T4 Elevated T 3 Elevated thyroid peroxidase antibodies Elevated thyroid-simulating immunoglobulins

52. Which laboratory value is NOT characteristic of acquired primary adrenal insufficiency (Addison disease) ? (232) A) B) C) D) E)

Hyponatremia Elevated adrenocorticotropic hormone (ACTH) Hypokalemia Low cortisol Hypoglycemia

53. Which hormone is NOT made by the anterior pituitary gland? (234) A) B) C) D) E)

Adrenocorticotropic hormone (ACTH) Antidiuretic hormone (ADH) Thyroid-stimulating hormone (TSH) Growth hormone (GH) Follicle-stimulating hormone (FSH)

54. Which of the following positively correlates with improved treatment adherence by patients? (242) A) B) C) D) E)

Physician's specialty Physician's years of experience Physician's knowledge Effective communication of the health care team Ethics consultation

55. Which of the following is NOT true? (247-25 1 ) A ) I n the setting o f an emergency and parental refusal of treatment, treatment may be provided to a child with­ out a court order overriding the parents' refusal B) A pediatrician may treat his or her own children or the children of family members for minor condi­ tions, emergencies, and in underserved areas where no other physician capable of providing pediatric care is available C) Expensive gifts from patients should not be accepted, as they could seem to influence the pediatrician's pro­ fessional judgment D) Genetic testing for carrier status of conditions in child­ hood is not recommended if medical management during childhood does not change E) Parental consent is always needed to treat adolescents for contraception, sexually transmitted diseases, and mental illness

MCG RAW- H I LL E D U CAT I O N S P E C I A LTY BOA R D REVI EW: P E D I AT R I C S

796

56. A 3 -day-old newborn has acute onset of bilateral eyelid swelling, conjunctival chemosis and hyperemia, puru­ lent drainage, and fever. What is the most likely cause of infection? (257) A) B) C) D) E)

Respiratory syncytial virus

Neisseria gonorrhea Chlamydia trachomatis Herpes simplex virus

Streptococcus agalactiae

57. Which of the following signs or symptoms distinguishes orbital cellulitis from preseptal cellulitis? (260) A) B) C) D) E)

Painful extraocular movements Vision changes Proptosis Ophthalmoplegia All of the above

58. Chronic inflammation of an obstructed meibomian gland in the eyelid is known as: (262) A) B) C) D) E)

Chalazion Hordeolum Molluscum contagiosum Stye Blepharitis

59. The triad of epiphora, blepharospasm, and photophobia with corneal clouding on exam is consistent with what pediatric ophthalmologic emergency? (265-267) A) B) C) D) E)

Congenital nasolacrimal duct obstruction Orbital cellulitis Primary congenital glaucoma Endemic keratoconjunctivitis Congenital Horner syndrome

60. What is the most common cause of amblyopia? (270) A) B) C) D) E)

Uncorrected refractive error Cataract Eyelid hemangioma Strabismus Ptosis

6 1 . Which is NOT a sign of neonatal hypoglycemia? (283) A. B. C. D. E.

Hypothermia Respiratory distress Strong suck Lethargy Exaggerated Moro reflex

62. Which is NOT a risk factor for developing severe neonatal hyperbilirubinemia? (286) A) Exclusive breastfeeding with weight loss B) East Asian race

C) ABO incompatibility D) Discharge after 72 hours of age E) Cephalohematoma 63. An ultrasound of the head in a premature infant shows hem­ orrhage involving the ventricles without extension of the bleeding to the cerebral cortex or ventricle dilation. What grade of intraventricular hemorrhage (IVH) is present? (290) A) B) C) D) E)

Grade I Grade II Grade III Grade IV Grade V

64. A 29-week gestational age infant has respiratory distress and hypoxia after delivery. A chest X-ray shows decreased lung expansion and diffuse ground glass opacities. This is consistent with what condition? (29 1 -292) A) B) C) D) E)

Transient tachypnea of the newborn Neonatal pneumonia Persistent pulmonary hypertension Meconium aspiration syndrome Respiratory distress syndrome

65. Premature infants are at risk for which of the following? (292-293) A) B) C) D) E)

Respiratory distress syndrome Necrotizing enterocolitis Sepsis Apnea All of the above

66. A newborn is unable to move his right arm. The arm is adducted, internally rotated, and extended. The forearm is pronated and the wrist flexed. This is most consistent with an injury to which of the following nerves? (294) A) B) C) D) E)

C5, C6, and C7 C5 and C6 C7, CS, and T 1 C S and T l C 5 , C6, C7, CS, and T l

67. Sunken eyes, parched skin, anuria, and altered mental sta­ tus are associated with what percentage of dehydration in a 3-year-old child? (30 1 ) A) B) C) D) E)

3% 6% 9% 10% 15%

F I N A L EXAM

68. Which is NOT a cause of hypocalcemia? (304) A) B) C) D) E)

Vitamin D deficiency Hypoparathyroidism Hypermagnesemia Pancreatitis Hyperphosphatemia

69. What metabolic derangement is seen in dehydrated infants with pyloric stenosis? (305) A) B) C) D) E)

Hypochloremic metabolic alkalosis Hyperchloremic metabolic acidosis Hyperkalemia Hyponatremia Hypophosphatemia

70. Which is NOT a treatment of hyperkalemia? {308-309) A) B) C) D) E)

Albuterol Insulin and intravenous (IV) dextrose Calcium gluconate Sodium polystyrene sulfonate Normal saline IV fluid bolus

7 1 . Which is NOT a cause of an anion-gap metabolic acidosis? {3 14) A) B) C) D) E)

Isoniazid Diabetic ketoacidosis Salicylate ingestion Diarrhea Uremia

72. A blood gas measurement is obtained, revealing the fol­ lowing: pH 7.39, PC0 2 28, HC0 3 18. This is consistent with what metabolic derangement? {3 15) A) B) C) D) E)

Compensated metabolic acidosis Uncompensated respiratory acidosis Uncompensated respiratory alkalosis Compensated respiratory alkalosis Partially compensated respiratory acidosis

73. Which of the following causes of acute abdominal pain is common in infants and adolescents, but not school-aged children? (324) A) B) C) D) E)

Appendicitis Intussusception Ovarian torsion Gastroenteritis Pyelonephritis

74. Which is NOT a common symptom of acute appendicitis? (324) A) Pain B) Anorexia C) Nausea

797

D) Fever E) Flatulence 75. Which of the following is the most common cause of acute pancreatitis? (326) A) B) C) D) E)

Viral infections Hyperlipidemia Cholelithiasis Medications Scorpion bites

76. At what age does malrotation typically become symptomatic? {327) A) B) C) D) E)

First month of life 1 month to 3 months 3 months to 3 years 3 years to puberty After puberty

77. At what ages does intestinal intussusception most com­ monly occur? {328) A) B) C) D) E)

First month of life 1 month to 3 months 3 months to 3 years 3 years to puberty After puberty

78. Which of the following pathogens is most likely to cause "pseudoappendicitis;' in which severe abdominal pain occurs before the onset of diarrhea? {337) A) B) C) D) E)

Campylobacter Escherichia coli Clostridium diffi cile Epstein-Barr virus

Salmonella

79. Which of the following conditions is more common in children with trisomy 2 1 ? (363) A) B) C) D) E)

Leukemia Duodenal atresia Atlantoaxial instability Celiac disease All of the above

80. Which of the following is NOT a feature of 22q1 1 .2 deletion syndrome? {365) A) B) C) D) E)

Abnormal facies Cardiac defects Thymus hypoplasia Cleft palate Hyperparathyroidism

MCG RAW- H I LL E D U CAT I O N S P E C I A LTY BOA R D REVI EW: P E D I AT R I C S

798

8 1 . A pregnant woman is a known carrier of Duchenne mus­ cular dystrophy. What is the chance that her child will be affected? {373} A) B) C) D) E)

1 00% 50% 25% 3% 0%

82. Which pathogen is the most common cause of urinary tract infection in otherwise healthy children? {378} A) B) C) D) E)

Escherichia coli Pseudomonas aeruginosa Enterococcus faecalis Streptococcus pneumonia Streptococcus viridans

83. Which of the following is true when using DDAVP in the treatment of nocturnal enuresis? {385} A) It may cause hypernatremia if given at too high a dose B) The intranasal solution is considered safer than oral tablets C) An ECG should be obtained before starting treatment D) The dose may need to be titrated over time to achieve all-night dryness E) It may make the child more difficult to awaken in the morning 84. The "blue dot sign" is indicative of what condition? (389) A) B) C) D) E)

Torsion of the testicular appendage Testicular torsion Henoch-Schonlein purpura Congenital adrenal hyperplasia Solitary testicle

85. What is the most common type of kidney stone? {395) A) B) C) D) E)

Calcium oxalate dihydrate Calcium phosphate Uric acid Ammonium acid urate Struvite

86. Which of the following testicular masses is associated with precocious puberty? (397) A) B) C) D) E)

Testicular hamartoma Epidermoid cyst Teratoma Leydig cell tumor Yolk-sac tumor

87. Which of the following is NOT a complication of cranio­ synostosis? {408) A) Facial asymmetry B) Intracranial hypotension

C) Hydrocephalus D) Strabismus E) Dental malocclusion 88. What is expected average daily weight gain for a 1 -month­ old infant? {410) A) B) C) D) E)

� � � � �

1 00 g/day 60 g/day 20 g/day 1 0 g/day 7 g/day

89. "Symmetric" failure to thrive refers to: {41 1 } A) B) C) D)

Low weight but otherwise normal growth Low weight, followed by decreased linear growth Low weight, followed by decreased head growth Low weight, with decreased linear growth and head growth E) Shortened lower and upper extremities

90. A 12-year-old boy weighs 75 kg and is 1 50 em tall. What are his body mass index (BMI) and body weight classification? {412) A) B) C) D) E)

22.2-healthy 22.2-overweight 22.2-obese 33.3-healthy 33.3-obese

9 1 . Which of the following infant reflexes will not be preset at day of life 7? {414) A) B) C) D) E)

Palmar grasp Parachute Moro Tonic neck Rooting

92. Which of the following would be considered normal speech development for an 1 8 -month-old? {416) A) B) C) D) E)

Two-word sentences Speech 50% understandable by a stranger Vocabulary of 10 to 50 words Use of plurals Able to name seven body parts

93. Which infectious disease is a contraindication to breast­ feeding in the United States? {424) A) B) C) D) E)

Latent tuberculosis Group B streptococcus Cytomegalovirus Hepatitis B (in the setting of cracked nipples) Herpes simplex virus with active labial and genital lesions

F I N A L EXAM

94. Which mother received adequate intrapartum antibiotic prophylaxis for prevention of group B streptococcus (GBS) transmission? (424-425) A) Term infant, mother GBS positive, mother received IV penicillin 4 hours prior to delivery B) Premature infant, mother GBS unknown, mother received no antibiotics prior to delivery C) Premature infant, mother GBS unknown, mother received IV clindamycin 2 hours prior to delivery D) Term infant, mother GBS negative but history of prior infant with GBS pneumonia, mother received no anti­ biotics prior to delivery E) All of the above 95. Erythema migrans is the hallmark rash of which infec­ tious agent? (430) A) B) C) D) E)

Ixodes scapularis Borrelia burgdorferi Epstein-Barr virus

Streptococcus pyogenes Human immunodeficiency virus

96. Which of the following viruses is characterized by airborne (as opposed to contact or droplet) transmission? (432) A) B) C) D) E)

Mumps Measles Influenza Adenovirus Human metapneumovirus

97. Which of the following is NOT associated with measles infection? (449-450) A) B) C) D) E)

Fever Cough Coryza Conjunctivitis Morbilliform rash that spreads from the extremities to the trunk

98. Which of the following types of inborn errors of metabo­ lism can be effectively managed by avoiding fasting? (460) A) B) C) D) E)

Glycogen storage diseases Aminoacidurias Organic acidemias Urea cycle defects Mitochondrial myopathies

99. Which of the following conditions is NOT associated with elevated ammonia? (460) A) B) C) D) E)

Phenylketonuria Maple syrup urine disease Propionic acidemia Ornithine transcarbamylase deficiency Tay-Sachs disease

799

1 00. Which of the following is associated with ketotic hypogly­ cemia? (464) A) B) C) D) E)

Infant of a diabetic mother Exogenous insulin Fatty acid oxidation disorder Fasting/starvation Nesidioblastosis

1 0 1 . Which of the following causes of ketotic hypoglycemia will NOT respond to a glucagon test? (464) A) B) C) D) E)

Panhypopituitarism Adrenal insufficiency Organic acidemia Hereditary fructose intolerance Mitochondrial defects

1 02. Which of the following is NOT a potential complication of phenylketonuria (PKU) ? (475) A) B) C) D) E)

Seizures Cardiomyopathy Hypopigmentation of the hair and skin Microcephaly Eczema

103. Which is NOT a finding in congenital clubfoot (talipes equinovarus) ? (484) A) B) C) D) E)

Metatarsus adductus Hindfoot valgus Rigid equinus Cavus foot Full range of motion in the hip and knee

1 04. Which of the following is true of supracondylar humerus fractures in children? (495) A) B) C) D)

Traction injuries are the most common mechanism They are often associated with child abuse The median nerve is the most commonly injured nerve Vascular insufficiency to the forearm almost always occurs immediately, and delayed vascular complica­ tions are rare E) The presence of a posterior elbow fat pad can be nor­ mal, and comparison views should be obtained if one is seen on X-ray

105. Legg-Calve-Perthes disease is defined as: (50 1 ) A) B) C) D)

Idiopathic osteonecrosis o f the femoral head Avascular necrosis of the femoral head Physeal fracture involving the proximal femoral physis Avascular necrosis of the medial aspect of the lateral femoral condyle E) Insufficiency fracture of the mid-shaft of the femur

MCG RAW- H I LL E D U CAT I O N S P E C I A LTY BOA R D REVI EW: P E D I AT R I C S

800

106. Spondylolysis may be seen on which of the following types of images? (504-506) A) B) C) D) E)

X-ray (especially oblique views of the lumbar spine) CT scan MRI Bone scan All of the above

107. Which of the following is an indication for brain MRI when evaluating a child with headaches? (5 1 1 -5 1 2) A) B) C) D)

Focal neurologic symptoms (weakness, numbness) Headache worse later in the day Presence of aura prior to onset of headache Presence of rebound headaches when over-the-counter analgesics wear off E) Age greater than 6 years

108. Which is associated with central, rather than peripheral, hypotonia in children? (514) A) B) C) D) E)

Normal mental status Absent reflexes Presence of seizures Muscle fasciculations Weak cough

1 09. Which is the most common cause of fatal meningitis infection among college students? (5 1 5) A) B) C) D) E)

Streptococcus pneumoniae Escherichia coli Listeria monocytogenes Neisseria meningitidis Enterovirus

l l O. Which is a characteristic of spinal fluid in acute bacterial meningitis? (516) A) Elevated glucose B) Low protein C) Elevated white blood cell (WBC) count with eosino­ philic predominance D) Elevated WBC count with lymphocytic predominance E) Elevated WBC count with neutrophilic predominance 1 1 1 . Which of the following is associated with complex partial seizures? (522) A) B) C) D) E)

Impaired level of consciousness Induced by hyperventilation Clusters upon awakening Absence of a postictal period Presence of hypsarrhythmia on EEG

1 1 2. Which of the following is NOT a benefit of exclusive breast milk feeding? (542) A) Decreased respiratory tract infections B) Increased diarrheal infections

C) Decreased likelihood of atopic disease D) Decreased risk of obesity E) Decreased risk of sudden infant death syndrome (SIDS) 1 1 3. Which is NOT an indication for soy formula? (543) A) B) C) D) E)

Galactosemia Congenital lactase deficiency Family wanting a vegetarian diet Prematurity All of the above

l l4. Which disease is NOT paired with its correct vitamin deficiency? (545-548) A) B) C) D) E)

Scurvy and vitamin C (ascorbic acid) deficiency Beriberi and vitamin B 1 (thiamine) deficiency Pellagra and vitamin B 3 (niacin) deficiency Severe measles infection and vitamin A deficiency Night blindness and vitamin E deficiency

1 1 5. Which is NOT a clinical feature of rickets? (549) A) B) C) D) E)

Tall stature Genu valgus Craniotabes Kyphoscoliosis Physeal widening

l l 6. Which of the following has been shown to be the most common contributor to medical errors and sentinel events? (563) A) B) C) D) E)

Poor communication Inadequate staffing Computerized physician order entry Incomplete documentation Lack of proper education for staff

l l 7. Harm that reaches the patient in the absence of medical error is classified as which of the following? (562) A) B) C) D) E)

Lapse Sentinel event Nonpreventable adverse event Preventable adverse event Near-miss event

1 1 8. Successful quality improvement projects utilize repeated PDSA cycles until change is achieved. Which is NOT part of the PDSA acronym? (564) A) B) C) D) E)

Perform Study Do Act Plan

F I N A L EXAM

1 1 9. Which of the following describes the best approach for identifying the top few factors causing the majority of events? (566) A) B) C) D) E)

Key driver diagram Pareto chart Run chart Statistical process control chart Shewhart chart

120. A healthy person starts taking a prescribed medication twice daily that has a half-life of 10 hours. How long will it take for the medication to reach a steady-state concentra­ tion? (574) A) B) C) D) E)

50 hours 24 hours 72 hours 36 hours 96 hours

1 2 1 . The fraction of an administered dose of unchanged drug that reaches the systemic circulation is known as what? (575) A) B) C) D) E)

Half-life Bioavailability Steady-state concentration Peak concentration Clearance

122. What type of adverse drug effect results from altered drug metabolism based on racial background? (577) A) B) C) D) E)

Pharmacokinetic Pharmacodynamic Pharmacogenomic Idiosyncratic Allergic

123. Which is NOT an adverse effect of systemic glucocorti­ coid use? (579) A) B) C) D) E)

Immunosuppression Adrenal suppression Short stature Osteoporosis Hypoglycemia

1 24. Which is NOT an adverse effect of stimulant medications? (583) A) B) C) D) E)

Hallucinations Hypertension Blurry vision Hyperphagia Insomnia

801

125. For a nonemergent procedure requiring sedation, what is the minimum recommended time between ingestion of infant formula and administration of a sedative? (589-590) A) B) C) D) E)

2 hours 3 hours 4 hours 6 hours 8 hours

126. What monitoring is necessary when performing moderate or deep sedation? (591 -592) A) B) C) D)

Continuous pulse oximetry Continuous cardiorespiratory monitoring Frequent blood pressure monitoring Licensed provider whose sole responsibility is to mon­ itor the patient E) All of the above

127. Which is NOT a treatment for salicylate poisoning? (597-598) A) B) C) D) E)

Intravenous sodium bicarbonate Hemodialysis Activated charcoal N-acetylcysteine Urine alkalization

128. In acute acetaminophen ingestions with known time of ingestion, when should the acetaminophen level first be drawn? (60 1 ) A) B) C) D) E)

2 hours postingestion 4 hours postingestion 6 hours postingestion 8 hours postingestion 12 hours postingestion

129. Right upper quadrant pain, oliguria, and rising values for liver function tests are characteristic of which phase of acetaminophen toxicity? ( 600) A) B) C) D) E)

Phase 1 Phase 2 Phase 3 Phase 4 Phase 5

1 30. Which is NOT a sign or symptom of opioid ingestion? (603) A) B) C) D) E)

Altered mental status Hypotension Tachypnea Miosis Seizures

MCG RAW- H I LL E D U CAT I O N S P E C I A LTY BOA R D REVI EW: P E D I AT R I C S

802

1 3 1 . Which is NOT a symptom of anticholinergic toxicity? {603) A) B) C) D) E)

Hypothermia Urinary retention Mydriasis Hallucinations Flushed skin

132. Which is NOT an ECG finding seen with tricyclic antide­ pressant toxicity? ( 606) A) B) C) D) E)

Prolonged QRS Sinus tachycardia Ventricular tachycardia Right bundle branch block Shortened PR interval

1 3 3 . A 3 -year-old presents after ingesting a liquid. Laboratory testing shows an anion-gap metabolic acidosis, elevated serum osmolar gap, elevated creatinine, and hypocalcae­ mia. This is consistent with ingestion of which of the fol­ lowing? {608-609) A) B) C) D) E)

Ethanol Methanol Isopropyl alcohol Ethylene glycol Salicylate

1 34. At what lead level should chelation therapy be initiated? (616) A) B) C) D) E)

1 0 mcg/dL 15 mcg/dL 25 mcg/dL 30 mcg/dL 45 mcg/dL

1 3 5 . Which is NOT an indication for endoscopic removal of a foreign body? (61 8) A) B) C) D) E)

Disc battery in the esophagus Coin in the proximal esophagus with dysphagia Coin in the stomach for 24 hours Coin in the esophagus for 36 hours Multiple magnets in the stomach

136. Which is NOT a contraindication to the live attenuated influenza virus vaccine? {625) A) B) C) D) E)

Age less than 2 years old Immunosuppression Pregnancy Egg allergy History of wheezing as an infant

137. Which is NOT a contraindication to the single antigen varicella vaccine? {628-629) A) Pregnant woman B) Child with severe combined immunodeficiency

C) Child with human immunodeficiency virus (HIV) D) Child with history of anaphylaxis to neomycin E) Child with leukemia 1 38. Which of the following regarding water safety is FALSE? (635-637) A) Vest- or horseshoe-style personal floatation devices are appropriate for children who are weak swimmers B) Bath rings should not be used in place of close supervi­ sion during bath time C) Swimming pools should be surrounded by a four­ sided fence with a latching gate D) Children should begin swimming lessons at 1 year of age E) Five-gallon buckets full of water should not be left unattended around young children 1 39. Which of the following regarding sun safety is FALSE? {635) A) Sunscreens should provide sun protective factor (SPF) 45 or higher B) Peak sun hours should be avoided C) Sunscreen should be reapplied every 2 hours or after swimming or sweating D) Young infants should be kept in the shade E) Children should wear sunglasses, a hat, and tightly woven clothing when in the sun 140. Which of the following is an adverse risk of secondhand smoke exposure? (640) A) B) C) D) E)

Increased risk of sudden infant death syndrome (SIDS) Increased number of viral respiratory tract infections Increased number of ear infections Increased number of asthma exacerbations All of the above

1 4 1 . Which of the following is not one of Elisabeth Kiibler­ Ross's stages of loss and grief? ( 644) A) B) C) D) E)

Denial and isolation Anger Bargaining Anxiety Acceptance

142. Which of the following statistics about foster care in the United States is true? (647) A) There are approximately 750,000 children in foster care B) There are approximately 75,000 children in foster care C) Each year, approximately 5,000 children "age out" of the foster care system D) Each year, approximately 1 00,000 children "age out" of the foster care system E) Only approximately 10% of foster children are placed with foster families due to child abuse or denial of crit­ ical care

F I N A L EXAM

143. Which is NOT a risk factor for vulnerable child syndrome? (648) A) B) C) D) E)

Child's life was at risk during pregnancy Prematurity Excessive reassurance from physician Maternal history of multiple spontaneous abortions Mother had postpartum depression

144. Which of the following is NOT an injury suggestive of child abuse? (653) A) B) C) D) E)

Rib fractures Femur fracture in nonambulatory child Tibia fracture in ambulatory child Sternum fracture Bilateral long-bone fracture

145. Which of the following sites of bruising is most concern­ ing for child abuse? (655) A) B) C) D) E)

Knees Abdomen Shins Forehead Forearms

146. Which of the following characteristics is most concerning for fabricated childhood illness (Munchausen syndrome by proxy) ? (658) A) B) C) D) E)

Father brings the child to medical appointments Parent is overly attentive to the child Family has minimal or no knowledge of medicine Family is reassured when a diagnosis is ruled out Seeks care at only one facility or with one provider

147. Of the following, which is considered the highest level of evidence? {66 1 ) A) B) C) D) E)

Cross-sectional study Randomized controlled trial Prospective cohort Case-control study Animal research

148. Which of the following is a characteristic of a case-control study? (662-663) A) B) C) D) E)

Controls for all variables Prospective Retrospective Does not involve a control group All of the above

149. What is the difference between statistical and clinical significance? (667) A) Clinical but not statistical significance can be deter­ mined by the P value

803

B) A result can be clinically significant but not statistically significant C) A result can be statistically significant but not clinically significant D) A statistically significant result is always clinically significant E) None of the above 1 50. The "P" in PICO question stands for: {667) A) B) C) D) E)

Pilot Preliminary Pathologic Premature Patient

1 5 1 . Which of the following is the most common cause of recurrent, painless gross hematuria? (672) A) B) C) D) E)

Kidney stones Urinary tract infection IgA nephropathy Hemolytic uremic syndrome Minimal change disease

1 52. How is the urine in myoglobinuria different from that in hematuria? (673) A) Dipstick urinalysis will not show blood B) Urine color will be dark yellow instead of pink, red, or brown C) Red blood cells (RBCs) will be more fragile and lyse soon after collection D) There will be no RBCs on microscopy E) It only becomes evident after IVF rehydration 1 53. Which of the following medications is NOT generally considered to be nephrotoxic? (676) A) B) C) D) E)

Acetaminophen Acyclovir Penicillin Ibuprofen Tacrolimus

1 54. Which of the following is the equation to calculate the free water deficit for a patient with hypernatremia? {680) A) Free water deficit (liters) Weight (kg) x (Actual Na - Goal Na)/Goal Na B) Free water deficit (liters) = 0.6 x Weight (kg) x (Goal Na - Actual Na)/Actual Na C) Free water deficit (liters) = 0.6 x Weight (kg) x (Actual Na - Goal Na)/Goal Na D) Free water deficit (liters) = 0.6 x Weight (kg) x (Goal Na - Actual Na)/Goal Na E) Free water deficit (liters) = Weight (kg) x (Goal Na ­ Actual Na)/ Actual Na

MCG RAW- H I LL E D U CAT I O N S P E C I A LTY BOA R D REVI EW: P E D I AT R I C S

804

1 55.

A) B) C) D) E) 1 56 .

163.

Laryngomalacia Croup Epiglottitis Vascular ring Vocal cord paralysis

1 64.

20-40

165.

12-30 12- 1 6

Lung disease Cyanotic heart disease Inflammatory bowel disease Liver disease All of the above

1 66 .

A) It is autoimmune B) It is made worse by trauma C) Shining a Wood lamp on the skin does not accentuate the lesions

Echocardiogram Electrocardiogram Baseline concussion testing Detailed physical examination of each major joint None of the above

Which of the following is true regarding glucose metabo­ lism in skeletal muscle? (76 1 ) A ) Th e insulin receptor i s only necessary for restoring glycogen stores B) Insulin can act directly on GLUT4 C) GLUT4 can transport glucose into the cell without the need for insulin when a muscle is exercising D) The insulin receptor can transport glucose into the cell without the need for insulin when the muscle is exercising E) Skeletal muscles do not have insulin receptors

Streptococcus pyogenes Staphylococcus aureus Streptococcus pneumoniae Haemophilus influenza Mycoplasma pneumoniae

Which of the following is true about vitiligo? (727-728)

Which of the following is a routine part of the sports pre­ participation physical evaluation? (759) A) B) C) D) E)

1 67. 161.

Topical antibiotics Oral antibiotics Oral corticosteroids IV antibiotics Incision and drainage

60-80

Which of the following is the most common organism in necrotizing pneumonia of a child? {723) A) B) C) D) E)

Herpes simples virus Nonsteroidal anti-inflammatory drugs (NSAIDs) Fungal infections Cytomegalovirus Inflammatory bowel disease

Which of the following is the most effective method of treating a skin abscess? (756) A) B) C) D) E)

30-60

Scalp Nails Elbow Perineum Face

Which of the following is the most common cause of erythema multiforme? (745) A) B) C) D) E)

Clubbing o f the digits can be associated with which o f the following types of illnesses? (72 1 ) A) B) C) D) E)

1 60 .

Thumb sign Widened retropharyngeal space Radiopaque circle Steeple sign Subcutaneous emphysema

Which of the following body regions is least likely to be affected by psoriasis? (732) A) B) C) D) E)

The normal respiratory rate for an infant (in breaths per minute) is: (709) A) B) C) D) E)

1 59 .

1 62 .

Which of the following causes expiratory stridor (instead of inspiratory or biphasic stridor) ? ( 699) A) B) C) D) E)

1 58 .

Lower urinary tract pathology Upper urinary tract pathology Muscle breakdown Toxin ingestion Urethral stricture

What is the classic radiographic finding in croup? (698) A) B) C) D) E)

1 5 7.

D) It can result in hyperpigmentation of the involved skin E) All of the above

Bright red blood in the urine is most commonly seen with: (682)

Which of the following sports should an athlete with trisomy 2 1 be allowed to participate in? (770) A) B) C) D) E)

Boxing Wrestling Football Weight lifting None of the above

F I N A L EXAM

1 68. Which of the following physical examination maneuvers is used to test for an anterior cruciate ligament injury? (774) A) B) C) D) E)

Lachman Posterior sag McMurray Valgus stress Varus stress

1 69. Which of the following is the most common specimen type to use when testing for drugs of abuse? (783) A) B) C) D) E)

Blood Breath Urine Saliva Hair

805

1 70. Which of the following is the leading cause of preventable death and disease in the United States? (783) A) B) C) D) E)

Drunk driving Alcohol poisoning Cocaine Heroin Cigarettes

1 7 1 . Which of the following medications is contraindicated in patients who have ingested cocaine? (787) A) B) C) D) E)

Beta-blockers Aspirin Benzodiazepines Nitroglycerin Sodium bicarbonate

This page intentionally left blank

Answer Key

l. D

33. c

65. E

2. D

34. c

66. A

3. A

35. D

67. c

4. E

36. A

68. c

5. B

37. c

69. A

6. A

38. B

70. E

7. A

39. A

71. D

8. c

40. E

72 . D

9. E

41. B

73. c

1 0. A

42. c

74. E

1 1. D

43. D

75. A

12. c

44. c

76. A

13. E

45. D

77. c

14. B

46. E

78. A

1 5. B

47. A

79. E

16. A

48. B

80. E

1 7. E

49. A

81. c

18. A

50. D

82. A

1 9. E

51. D

83. D

20. B

52. c

84. A

21. B

53. B

85. A

22. E

54. D

86. D

23. B

55. E

87. B

24. E

56. B

88. c

25. c

57. E

89. D

26. D

58. A

90. E

27. A

59. c

91. B

28. c

60. D

92. c

29. B

61. c

93. D

30. A

62. D

94. A

31. B

63. B

95. B

32. A

64. E

96. B 807

808

MCG RAW- H I LL E D U CAT I O N S P E C I A LTY BOA R D REVI EW: P E D I AT R I C S

97. E

1 22 . c

1 47. B

98. A

123. E

1 48 . c

99. E

1 24. D

1 49. c

1 00. D

1 25 . D

1 50. E

101. D

1 26. E

151. c

1 02 . B

1 27. D

1 52 . D

103. B

1 28 . B

1 53. A

1 04. c

1 29. B

1 54. c

105. A

1 30. c

1 55. A

1 06. E

131. A

1 56. D

1 07. A

1 32 . E

1 5 7. D

108. c

133. D

1 58 . A

1 09. D

1 34. E

1 59. E

1 1 0. E

1 35. c

1 60. B

1 1 1. A

1 36. E

161. A

1 1 2. B

1 3 7. c

1 62 . E

1 1 3. D

1 38. D

1 63 . A

1 1 4. E

1 3 9. A

1 64. E

1 1 5. A

1 40. E

1 65 . E

1 1 6. A

141. D

1 66. c

1 1 7. c

1 42 . A

1 67. D

1 1 8. A

143. c

1 68 . A

1 1 9. B

1 44. c

1 69. c

1 20. A

1 45 . B

1 70. E

121. B

1 46. B

171. A

I n dex Note: Page numbers followed by f refer to figures; those followed by t refer to tables. A

ABC(s), 169, 1 7 1 ABCDE survey, 2 1 0 Abdominal masses, 329 Abdominal migraines, 332 Abdominal pain, 323-332, 324t chronic, 3 3 1 , 3 3 l t functional, 329-330, 3 3 1 ovarian torsion and, 3 9 1 visceral, 3 3 2 Abdominal trauma, blunt, 209 ABO incompatibility, 1 0 7 Abscesses. See specific sites and types Absolute risk reduction, 666 Academic achievement assessment of, l44t factors affecting, 1 3 9 - 1 40, 1 40f Acanthosis nigricans insulin resistance and, 236, 236f, 4 1 2 -4 1 3 i n polycystic ovary syndrome, l 3 f Accommodations, 146, l46f Acetaminophen, 590, 592-593 Acetaminophen intoxication, l 6S, 599-602, 600t, 6 0 l f Acetylcholine receptor antibody test, in myasthenia gravis, 534 N-Acetylcysteine (NAC) , for acetaminophen intoxication, 600-60 l Achievement testing, 1 42 Achilles tendon injuries, 762 Achondroplasia, 367, 4S l -4S2 Acid-base disorders, primary, 706, 706t Acid-blocking medications, caution regarding, 276 Acidemia, as stimulator of ventilation, 720 Acne inflammatory, 733, 733f neonatal, 73S Acne vulgaris, 732-733, 733f Acquired immunodeficiency syndrome (AIDS). See HIV/AIDS Acrocyanosis, 277 Acrodermatitis enteropathica, 477, 552, 5 52f Acromegaly, 693t Acromioclavicular (AC) joint injuries, 765, 765f

Acromioclavicular (AC) separation, 2 1 7-2 1 S Activated charcoal, indications and contraindications to administering, 597, 59S, 59St Acute chest syndrome (ACS), 94 in sickle cell disease, S4 Acute demyelinating encephalomyelitis (ADEM) , 525 Acute hemolytic events, 93 Acute kidney injury (AKI) criteria for, 6S4-6S5, 6S4t differential diagnosis of, 690 drug-induced, 675, 676t ethylene glycol intoxication and, 609 etiologies of, 6S5, 6S5t multifactorial, 6S6 in nephrotic syndrome, 6S4 nephrotoxic, 320- 3 2 1 oliguric, 6S6-6S7 physiologic derangements in, 3 2 1 Acute lymphoblastic leukemia (ALL), 7 S , S6, SS, 90-92, 9 1 ( 96, 105- 1 06 Acute myeloid leukemia (AML), 96, 97, 72 1 Acute otitis media (AOM), l S O - l S 5 , l S l f complications of, l S l - l S2, 1 S4- l S 5 diagnostic criteria for, l S I , l S2f organisms causing, ISO with perforated tympanic membrane, 1 82, 1 84 treatment of, 1S0- 1 S l , l S2, l S2t, l S 3 , 434-435 Acute renal failure. See Acute kidney injury (AKI) Acute respiratory distress syndrome (ARDS), 1 76 - 1 77, l 76f Acute rheumatic fever (ARF), 1 53 , l 53f, l 53t Acute tubular necrosis (ATN) , distinguishing prerenal azotemia from, 6S5, 6S5t Acyclovir, 676t adverse effects of, 5S4 for herpes encephalitis, 5 1 7 Adams forward-bending test, 499 Adaptive behavioral assessment, l44t Addiction, 779-7SO, 7SOt Addison disease, 232-233, 232f

Adenoidectomy, 1 9 1 , l 9 l t Adenotonsillectomy, 7 1 3 Adhesive strips, 206 Adolescents, 1 -22 age of puberty, l, 2f-4f, 3t, 4-6 amenorrhea in, 1 0 - 1 2 anaphylaxis in, 30-32 death of, causes of, 7 -S, Sf, 67 delinquency in, 63 depression in, 64-67 eating disorders in, S - 1 0, 9t, l Ot male, gynecomastia in, 3-4 manic episodes in, 67 -6S menses and, 1 0 - 1 5 , 2 1 -22 peptic ulcer disease in, 349 pregnancy in, 1 6 , 1 7 psychosis in, 6S, 7 1 -72 psychosocial screening for, 6-7 recreational drug use by, S risk of injury from sports in, 7 6 1 risk-taking behavior i n , 6 - 7 safety recommendations for, 636t school refusal in, 6 1 , 6 l t scoliosis in, 499 sexually transmitted infection in, 7, I S , 1 9-20, 435-436 stages of adolescence and, 7, 20-2 1 , 20t suicide in, 7 -S, Sf, 67 tuba-ovarian abscess in, 19 urethritis in, 1 9-20 Adoption developmental understanding of, 646t of different-race child, 645-646 international, 443, 443t Adrenal insufficiency, primary, 232-233, 232f Adrenal tumors, 1 3 - 1 4 Adrenarche, 6, 22S premature, 22S Advanced maternal age (AMA), trisomy 2 1 and, 364 Adverse events drug, risk of, 562 nonpreventable, 562 preventable, 5 6 1 -562 AEIOU-TIPS mnemonic, 5 14

81 0

I n dex

Age. See also specific age groups bone, 403 maternal, trisomy 21 and, 364 Ages and Stages Questionnaire (ASQ) , 4 1 3 Aggression, 70, 7 1 Agoraphobia, 6 5 Airborned precautions, 43 1 Air enema, 329 Air leak syndrome, in neonates, 293 Airway infections, 445 Alagille syndrome, 365 Alarms, sensitivity and specificity of, 570 Alcohol abuse of, 782, 784 epistaxis due to, 1 8 6 fetal alcohol syndrome and, 2 9 6 , 3 6 6 , 366f for methanol ingestion, 609 physiological consequences of using, 782 Alcohol intoxication, 607-608, 609, 6 1 0, 706 Alert, definition of, 5 1 3 Aliskiren, 585 Allergen immunotherapy, 34 Allergic reactions, 36 Allergic rhinitis, 33-34, 1 99-200, 729 Allergy testing, 34-35 Alloimmune thrombocytopenia, neonatal, 92 All-terrain vehicles (ATVs) , safety with, 635 Alopecia areata, 734-735, 734f, 735t Alpha- 1 -antitrypsin (AT) deficiency, 346 Alpha-thalassemia minor, 1 04 Alport syndrome, 6 7 1 -672 Alprazolam, 588t Altered mental status, 5 1 3 , 5 14, 5 1 5 Alternate cover test, 274 Alternative care, parents' right to seek, 249-250 Alternative/complementary treatments, 595 Alveolar gas equation, 707 Amblyopia, 264, 270, 270t, 273 Amenorrhea, 1 0 - 1 2 Aminoglycosides, 676t for otitis externa, 179 Amitriptyline intoxication, 605-607 Amlodipine, 585 Ammonium acid urate stones, 396 Amniotic band sequence, 297 Amoxicillin rash and, 423 for urinary tract infections, 3 78 Amphetamines, fetal exposure to, 296 Amphotericin B, 676t nephrotoxicity of, 5 8 1 AMPLE history, 2 1 0 Amyoplasia, 482 Anabolic steroids, athletes' use of, 777, 778 Anaphylactic shock, 3 1 Anaphylaxis, 30-32, 32t, 33 allergy testing and, 35 food-related, 46 hymenoptera stings and, 208 immunotherapy and, 34

nonimmunologic, 32 Anaplasmosis, 44 1 , 453 Androgen, excess, 1 3 - 1 4 Androgen insensitivity syndrome, 1 1 , 2 2 1 Androgen replacement therapy, for delayed puberty, 1 Anemia, 73-75 aplastic, 88-90, 89t Diamond-Blackfan, 80, 8 l t, 89t, 3 7 1 Fanconi, 89t, 9 2 , 3 7 1 hemolytic, 74, 8 8 iron deficiency. See Iron deficiency anemia (IDA) macrocytic megaloblastic, 5 5 1 pernicious, 550-5 5 1 o f prematurity, 280-28 1 sickle cell. See Sickle cell anemia; Sickle cell disease Angelman syndrome (AS), 366 Anger, 70 Angiofibromas, 740-74 1 , 74 1 f Angiokeratomas, i n Fabry disease, 469, 470f Angiotensin-converting enzyme (ACE) inhibitors, 585, 6 9 1 nephrotoxic, 676t pacemaker and, 1 2 7 Angiotensin II receptor antagonists, 6 9 1 Angiotensin receptor blockers (ARBs), nephrotoxic, 676t Angular stomatitis, 547f Anion gap, 3 1 3- 3 1 4 methanol ingestion and, 608 Aniridia, 372, 372f Ankle, inversion injury of, 776, 776f Ankle proprioception drills, 762 Ankle sprains, 762 imaging criteria in, 767, 767t lateral, 767-768, 767t Ankyloglossia, 1 92 Ankylosing spondylitis, 1 60- 1 6 1 Anomalous coronary arteries, 1 12 Anomalous left coronary artery from the pulmonary artery (ALCAPA), 1 1 1 Anorexia, cancer-associated, 556 Anorexia nervosa, 8-10, 9t, l Ot, 358 Anosmia, 6 Anovulatory abnormal uterine bleeding, 12- 1 5 Anoxia, cerebral, 1 73 Anterior chamber, 272 Anterior cruciate ligament (ACL) tears, 762, 773, 773f Anterior drawer test, 773 Anterior talofibular ligament sprain, 767 Anthrax, cutaneous, 622f Antibiotics. See also specific antibiotics for acute chest syndrome, 84 for acute otitis media, 1 80- 1 8 1 , 434 for acute rheumatic carditis, 1 1 8 bacteriostatic, 579 beta-lactam, 579 for brain abscesses, 5 1 9

for Clostridium difficile colitis, 33 7 Clostridium difficile colitis due to, 337, 435, 578-579 for dog bites, 206-207 for GAS pharyngitis, 190 for Neisseria meningitidis meningitis, 439-440 nephrotoxic, 676t for ophthalmia neonatorum, 433 for peritonsillar abscesses, 1 89 for respiratory syncytial virus infections, 447-448 for retropharyngeal abscesses, 1 90- 1 9 1 for septic shock, 1 72 for sexually transmitted infections, 1 8 for tracheitis, 699 Anticholinergic toxicity, 603-604, 787 Antidepressants, for bipolar disorder, 68 Anti-D immune globulin, for immune thrombocytopenia, 77t Antiepileptics, 574-576 oral contraceptive pills and, 1 6 serum drug concentration of, 574 trough and peak levels of, 576 Antifungal drugs, toxicities of, 5 8 1 , 582t Antihistamines for allergic rhinitis, 199 for anaphylaxis, 3 1 for urticaria, 33 Antimalarial drugs, for systemic lupus erythematosus, 1 52 Antimotility agents, for giardiasis, 336 Antineutrophil antibodies, 1 04 Antipsychotics, for bipolar disorder, 68 Antivenom for black widow spider bites, 208 for scorpion stings, 208 for snake bites, 207 Ant stings, 208 Anus, imperforate, 284 Anxiety disorders, 63 -64, 64t Aortic insufficiency, 1 1 3 Aortic stenosis, 1 10- 1 1 1 Aortic valve, bicuspid, 1 1 6 Apert syndrome, 3 7 1 Apgar score, 277, 278t, 289 Aphthous ulcers, 1 94t Aplastic anemia, 88-90, 89t Aplastic crisis, 94 Apnea-hypopnea index (AHI), 7 1 3 Apology, for medical errors, 246 Apparent life-threatening events (ALTEs), 725-726 Appendicitis, 323 Appendix, perforated, 323 Appendix testis, torsion of, 390 Applied behavioral analysis (ABA), 136

Arcanobacterium haemolyticum infections, 423 Arizona bark scorpion stings, 207-208 Arnold-Chiari malformation, 257, 528, 528f Arrhythmias, fetal, 275

I nd ex

Arrhythmic right ventricular cardiomyopathy (ARVC) , 1 1 2 Arteriovenous malformations (AVMs), 530, 5 3 l f Arthritis bacterial, 425-426 idiopathic, juvenile, 1 57- 1 58, 1 57t inflammatory j oint symptoms of, 1 52 Lyme, 1 5 7 postinfectious, 1 6 1 pyogenic, 492-493, 493f reactive, 1 6 1 septic, 425-426, 433-434 Arthrogryposis, 482 Arthroopthalmopathy, hereditary, 1 9 5 Arthropod bites, 2 0 8 , 749 Ascariasis, 449 Asherman syndrome, 1 1 Ash -leaf spots, in tuberous sclerosis, 369-370, 370£ 74 1 , 74 1 f Aspiration, tracheostomy and, 722 Aspirin for acute rheumatic carditis, 1 1 8 for Kawasaki disease, 1 1 8, 1 56 metabolic acidosis caused by, 3 1 6-3 1 7 Reye syndrome and, 580 toxicity of, 596-598 Assent, from patient, 24 1 Asthma, 35-45, 3 7f-39f, 40t, 4 l f, 43f,

protective eyewear for, 759, 768 return to play after injury and, 762, 764-765 sudden cardiac death in, 1 1 2 with trisomy 2 1 , 760, 770 with type 1 diabetes mellitus, 760-76 1 , 76 l f weight control practices of, 777 weight reduction methods for, 777 Atlantoaxial instability, of cervical spine, in trisomy 2 1 , 770 Atopic dermatitis, 46, 48-49, 727-729, 728f, 732t, 75 1 allergy testing and, 3 5 Atrial septal defect (ASD ) , 1 1 3 - 1 14, 1 29 Atrioventricular (AV) heart block, 1 2 5 Atrioventricular nodal reentrant tachycardia (AVNRT), 1 2 1 , 1 22, 1 22f Atrioventricular reentrant tachycardia (AVRT), 1 2 1 - 122, 1 22f Atropine for beta-blocker intoxication, 604-605 for organophosphate exposure, 6 1 3 Attachment, 5 8 Attention deficit hyperactivity disorder (ADHD) , 6, 64, 69-70 interventions for, 1 47- 148 Auricular hematoma, 768 Autism spectrum disorder (ASD ) , 1 3 3 - 1 35, 1 3 3t, 520

709-710, 7 1 1 t as contraindication to immunotherapy, 34 control of, monitoring, 42, 43f exercise-induced, 37 hallmark characteristics of, 35-36 obesity and, 4 1 3 pathogenesis of, 3 8 , 38f prevalence of, 36, 3 7 risk factors for death from, 44 severity of, classification of, 38-40, 39f, 40t, 42 treatment of, 40-45, 4 l f, 5 8 1 Ataxia, cerebellar, acute, 524-525 Ataxia-telangiectasia (AT), 30, 525, 525f Atenolol, 585 Athletes, 759-778. See also specific injuries

and conditions anabolic steroid use by, 777, 778 cervical spine stabilization in, 769 cooling, 763 dietary supplements for, 558 energy and sports drinks and, 558 with epilepsy, 760 fever in, 759 fluid replacement during sports and, 776-777 headgear for, 768 injury prevention for, 76 1 - 762 with Marfan syndrome, 76 1 , 762t nutritional supplement use by, 777-778 overuse injuries in, 766 preparticipation physical examination and, 759-760

conditions associated with, 1 3 5 , 136 genetic testing in, 373-375, 3 74t interventions for, 1 3 6 specific language impairment distinguished from, 134 vaccination and, 1 36, 627 Autoimmune hemolytic anemia (AIHA) , 74, 88 Autoimmune thyroiditis, 229-230 Automated brainstem response (ABR) testing, 198 Automated external defibrillators (AEDs), 1 70 Automatisms, 5 2 1 Autosomal dominant disorders, 3 6 1 Autosomal dominant polycystic kidney disease (ADPKD), 675, 677t Autosomal recessive disorders, 3 6 1 -362 Autosomal recessive polycystic kidney disease, 675-676, 677t Avascular necrosis, steroids and, 1 52 Average for gestational age (AGA) , 282 Axillary temperature, 203 Azotemia, prerenal, distinguishing acute tubular necrosis from, 685, 685t B

Babesiosis, 429, 453 Back to Sleep campaign, 634 Bacterial infections. See also specific

infections asthma and, 38

81 1

in chronic granulomatous disease, 28, 29 conjunctivitis due to, 258-260, 259t infective endocarditis and, 1 1 9, 1 20t parotitis due to, 192 pericarditis and, 1 1 1 transmitted by blood transfusion, 93 in X-linked agammaglobulinemia, 28 Bag-and-mask skills, 588 Bagging, 6 1 1 Bag-valve-mask, 1 69 Ballard score, 279, 280f Barbiturates, 588 Bark scorpion stings, 207-208

Bartonella henselae infections, 195, 430-43 1 Battery ingestions, 620-62 1 , 62 l f B C G vaccination, 438 Becker's muscular dystrophy (BMD), 373, 536 Beckwith-Wiedemann syndrome (BWS), 96, 3 5 1 -352, 369, 394 Bedtime, 58 Bedwetting, 384-385, 385f, 673 Bee stings, 208 Behavioral staring, 5 2 1 Behavior problems, 5 3 - 5 6 , 54t Beighton scoring system, for benign j oint hypermobility syndrome, 1 6 1 , 1 6 l t "Bell clapper" deformity, 390 Bell palsy, 533 Benign j oint hypermobility syndrome (BJHS), 1 6 1 , 1 6 l t Benign paroxysmal torticollis, 483 Benign positional vertigo (BPV) , 1 8 5 Benzodiazepines, 5 8 8 , 588t, 590 for amphetamine-induced seizures, 787 for anxiety disorders, 63-64 hypertension due to, 692 Bernard-Soulier syndrome (BSS), 1 0 1 , 1 0 l f Beta, -agonists long-acting, for asthma, 4 1 , 4 l f long- vs. short-acting, 5 8 1 short-acting, for asthma, 44-45 Beta-blockers contraindication in patients who have ingested cocaine, 787 intoxication by, 604-605 1 1 -Beta-hydroxylase deficiency, 380- 3 8 1 Betamethasone, fetal lung maturity and, 289 Beta-oxidation defects, 46 l t Beta-thalassemia intermedia, 103- 1 04 Beta-thalassemia major, 1 0 7 Beta-thalassemia minor, 1 04 Bezold abscesses, 182 Bhutani nomogram, 287 Bias, 665 Bicarbonate (HCO), 3 1 3, 3 14, 3 1 5- 3 1 6 Bicuspid aortic valve (BAV), 1 1 6 Bicycle safety, 64 1 Bilevel positive airway pressure (BiPAP), 707 for central sleep apnea, 7 1 3 Biliary atresia extrahepatic, 346-347, 355 stool in, 347

81 2

I n dex

Biliary tract disease, 3 5 5 Bilirubin-induced neurologic dysfunction (BIND) , 286-287 Biologic response modifiers, for juvenile idiopathic arthritis, 1 58 Biophysical profile, 275 Biotinidase deficiency, 459 Bipolar disorder, 62-63, 65-66, 68 Birth injuries, 293-294 Birthweight, 2 8 1 -283 changes from, during first 1 2 months, 404 low, 289, 290 BISCUIT, 1 3 5 Bites arthropod, 208, 749 cat, 449 dog, 206-207, 449 "fight;' 207 human, 53, 207 snake, 207 spider, 208 tick, 637-638 . See also Lyme disease Biting, 53 Black box warnings, 575, 583 Black widow spider bites, 208 Bladder control, 53 Bleeding. See also specific sites vitamin K deficiency, 284 Bleeding disorders. See also Hemophilia A; Hemophilia B inherited, 1 5 Blepharoconjunctivitis, 258 Blepharospasm, 267 Blood group incompatibility, 107 Blood pressure. See also Hypertension; Hypotension measurement of, 640, 69 1 , 6 9 l t Blood pressure cuffs, size of, 640 Blood transfusion for acute chest syndrome, 84 fever with, 92-93 Blood urea nitrogen (BUN), in dehydration, 302 Blood volume, circulating, 302 Blount disease, 4 1 3 Blowout fractures, 2 6 1 , 272 Blueberry muffin, 453, 453f Blue dot sign, 389f Blunt errors, 562 Body mass index (BMI) , 4 1 2 Body temperature, measurement of, 203, 764 Body weight, percentage of accounted for by water, 3 1 9 Bone age, 403 Bone age test, 1 Bone cysts aneurysmal, 498 unicameral, 497-498, 497f Bone dysplasias, 48 1 -482 Bone marrow failure syndromes, 88-90, 89t Borderline personality disorder, 66

Borrelia burgdorferi infections. See Lyme disease Botulism, 442-443 infantile, 534 Bowel control, 53 Brachial plexus injuries, in neonates, 294 Brachycephaly, 408, 408f Bradycardia fetal, 275 symptomatic, 1 70 in tricyclic antidepressant overdose, 607 Brain, hypoxia to, in newborns, 288 Brain abscesses, 188, 5 1 8-5 1 9, 5 1 St Brain death, criteria for, 1 73 , 1 7 4t, 245, 245t Brain injury. See Traumatic brain injury Branchial cleft cysts, 196 "Breakbone fever;' 429, 453 Breast(s) development of, 5, 6 fibroadenomas and, 5 gynecomastia and, 3 -4 in pure gonadal dysgenesis, 1 1 Tanner stages of development of, 226-227, 227f Breastfeeding contraindications to, 42 1 , 424, 54 1 - 542 duration of, 54 1 , 545, 557 immunologic benefits of, 542, 542t Breast milk jaundice, 287, 346 Breath-holding spells, 55-56, 56t, 5 2 1 Bristol stool chart, 383, 383f Bronchiectasis in cystic fibrosis, 7 1 3 selective IgA deficiency and, 724 Bronchiolitis, 722 viral, 167 Bronchitis, bacterial, 7 1 0 , 7 l l t Bronchodilators for anaphylaxis, 3 1 for asthma, 37, 44 toxicity of, 45 Bronchopulmonary dysplasia (BPD ) , 2 9 6 , 725 Bronchoscopy, in pneumonia, 724 Brown recluse spider bites, 208 Brucella infections, 43 1 , 456 Brugada syndrome, 1 1 2 Bruising child abuse and, 654-655, 655f, 655t, 656f suspicious versus innocent, 98 unintentional, 654 Bruton's X-linked agammaglobulinemia (XLA) , 25, 28, 30 "Buffalo hump;' 233 Bulimia nervosa, 8, 9t, lOt Bullying, 71 Burn injuries, 2 14-2 1 6 child abuse and, 656f estimating total body surface area involved in, 2 14, 2 1 4t, 2 1 St fluid resuscitation for, 2 1 4, 2 1 6, 2 1 6t full-thickness (third-degree) , 2 1 6

infections i n children with, 455 nutritional support for, 555-556 partial-thickness (second-degree) , 2 1 6 Burst fractures, spinal, 2 1 4 Burton lines, 6 1 6 c

Cafe-au-lait macules (CALMs), 369-370, 370f, 394, 740, 740f Calcaneal apophysitis, 50 1 - 502, 766-767 Calcium channel blocker intoxication, 605 Calcium chloride for beta-blocker intoxication, 605 for calcium channel blocker intoxication, 605 Calcium gluconate for beta-blocker intoxication, 605 for calcium channel blocker intoxication, 605 for hyperkalemia, 309 Calcium homeostasis, 237, 237f hypoalbuminemia and, 304-305 Calcium infusion, 304 Calcium oxalate stones, 396 Calicivirus infections, 457 Campylobacter gastroenteritis, 337 Campylobacter jejuni infections, 447 CaNa2 EDTA, for lead poisoning, 6 1 7 Canavan disease, 478 Cancer predisposition syndromes, 95-96

Candida infections, 456 cutaneous, 753 diaper dermatitis due to, 752-753 "Canker sores;' 1 94t Capillary blood gas analysis, 72 1 Capillary hemangiomas, 273, 273f, 737 Capillary refill, 165 Captopril, 585 Carbamazepine for myoclonic seizures, 523 serum drug concentration of, 574 Carbohydrates, 539-540 malabsorption of, 352, 354 Carbon monoxide poisoning, 6 14-6 1 5 Carboxyhemoglobin level (COHb), 6 1 4 Cardiac syncope, 1 1 1 - 1 1 2 Cardiogenic shock, 1 6 7 Cardiomyopathy, dilated, 1 1 4 Cardiopulmonary resuscitation (CPR), rate of, 1 70 Carditis, Lyme, 1 2 5 Caregivers, vaccinations for, 2 8 5 Carglumic acid, 475 Carnitine palmitoyltransferase (CPT- 1 ) deficiency, 473 Car seats, 634 Case-control studies, 662-663, 663f, 665 Case reports, 663-664 Case series, 665 Cat(s), toxoplasma and, 453 -454 Cataplexy, 536 Cataracts, congenital, 267-268, 268f

I nd ex

Cat bites, 449 CATCH-22 mnemonic, 365 Catecholaminergic polymorphic ventricular tachycardia (CPVT) , 1 1 2 Cat -scratch disease, 195, 258, 430-43 1 Cauliflower ear, 768 Caustic ingestions, 620 Cavernous sinus thrombosis, 188, 262 Ceftriaxone, adverse effects of, 578 Celiac disease, 1 59, 352, 3 52f, 353-354 Cellulitis, 755-756 orbital, 188, 260, 260f, 2 6 1 -262, 262t preseptal, 260-26 1 , 260f, 262t Central sleep apnea, 7 1 2 , 7 1 3 Cephalohematomas, 285, 286f Cephalosporins, 676t Cerebellar ataxia, acute, 524-525 Cerebral anoxia, 173 Cerebral blood flow (CBF) studies, brain death and, 17 4 Cerebral edema, 1 73 Cerebral palsy, 529-530, 529f, 530f nutritional needs in, 557 Cervical spine atlantoaxial instability of, in trisomy 2 1 , 770 stabilization of, in athletes, 7 69 Cervical spine injuries, 2 1 3- 2 1 4 Cesarean delivery, 29 1 -292 Chalazia, 262-263, 263f CHAMP mnemonic, 6 1 1 Chancre, in syphilis, 1 8 Charcot-Marie-Tooth (CMT) disease, 533 CHARGE syndrome, 136, 1 86, 365, 365f, 703 otitis media with effusion and, 1 8 3 Chediak-Higashi syndrome, 2 9 , 30 Chelation therapy, 6 1 7 Chemical hazards, 622 Chemotherapy anorexia due to, 556 empiric therapy for infections in patients receiving, 452 risks of, 105- 1 06 in trisomy 2 1 , 96 vaccinations prior to, 633 "Cherry red" spots, 473, 473f Chest pain, causes of, 1 1 0- 1 1 1 Chest trauma, blunt, 2 1 0-2 1 1 Chest tube drainage, 723 Chest wall rigidity, fentanyl and, 5 9 1 Chest X-rays in asthma, 7 1 0 i n pneumonia, 722 Chiari type II malformations, 528, 528f Chickenpox, 43 1 , 449 prevention of, 45 1 Chikungunya fever, 429, 453 Child abuse, 248, 6 5 1 -658 bruises and, 654-655, 655f, 655t, 656f burns and, 656f fabricated childhood illness and, 658, 658t fractures and, 652-653, 653f

physical, 97-98 red flags for, 652 risk factors for, 652 sexual, 657-658, 657t "shaken baby syndrome" and, 654 suspected, diagnostic workup for, 652 types of, 656 Child care, infection prevention and, 422 Childproofing the home, 635 Children, safety recommendations for, 636t Child welfare system, 64 7 Chlamydia! infections, 1 8

Chlamydia trachomatis infections epididymitis due to, 390-391 ophthalmia neonatorum due to, 257 screening for, 17 Chloral hydrate, 587t Chloramphenicol, adverse effects of, 578 Choana! atresia, 1 8 6 Choking, 705 Cholangitis, 325, 325t Cholecystectomy, 325 Cholecystitis, 325, 325t acalculous, 325 Choledochal cysts, 34 7 Choledocholithiasis, 324-325, 325t Cholelithiasis, 325t Cholesteatomas, 1 84, 1 84f Chronic granulomatous disease (CGD ) , 28, 29, 30 Chronic illness, coping in families of children with, 650 Chronic kidney disease (CKD), 687-688, 688t Chronic lung disease of prematurity (CLD ) , 296, 725 Chronic nonspecific diarrhea o f childhood, 337-338, 338t Chronic pain syndrome, 648-649 Chronic renal replacement therapy. See Dialysis Chronic suppurative otitis media (CSOM), 1 8 3 - 1 84 Chylothorax, 7 1 7 Cidofovir, 676t Cigarette smoking, 783-784 hypertension due to, 693t maternal, fetus and, 296 secondhand smoke and, 640 Ciliary dyskinesia, primary, 7 1 6 - 7 1 7 Cimetidine, adverse effects of, 580 Circumcision, 3 8 1 -382 Citalopram, toxicity of, 607 Citrullinemia, 460t Clavicular fractures, 2 1 7- 2 1 8, 7 62 Cleft lip and palate, 1 93 - 1 95 , 1 95f Clinical significance, statistical significance vs., 667 Clobetasol, for atopic dermatitis, 727, 729t Clonidine intoxication, 605

Clostridium diffi cile colitis, 336, 337, 435, 578-579, 7 1 5

81 3

Clostridium tetani infections, 43 1 . See also Tetanus Clubfoot, congenital, 484-485, 484f Coagulase-negative staphylococcus (CONS) , 452 Coarctation of the aorta, 1 1 6, 694 Cocaine abuse of, 78 1 , 787 epistaxis due to, 186 Coccidioidomycosis, 445 Cochlear implants, 253 Coffee ground emesis, 349 Cognitive behavioral treatment, for oppositional defiant disorder, 1 49 Cognitive development, 4 1 7 Cognitive disability, i n fragile X syndrome, 369 Cohort studies, 662, 665 Coin(s), aspirated, 6 1 7- 6 1 8 , 6 1 8f Coining, 657f Cold panniculitis, 1 92 "Cold" shock, 1 70 Cold sores, 427 Cold stress, in newborns, 276-277 Colic, 59 Colitis bacterial, 338-339 Clostridium diffi cile, 336, 337, 435, 578-579, 7 1 5 ulcerative, 356 Collagenomas, 74 1 , 742f Colorado tick fever, 429 Column of Bertin, hypertrophied, 392 Coma, 245 definition of, 5 1 3 substances inducing, 5 1 3 Common variable immunodeficiency (CVID ) , 28-29 Communication medical error prevention and, 563, 570 therapeutic relationship and, 242-243 Communication disorders, 1 34 Compartment syndrome, acute, 494 Complement deficiency, 28 Complement protein levels, 682 Compression fractures, spinal, 2 1 4 Computed tomography ( CT) in craniosynostosis, 407-408 for orbital infections, 2 6 1 Concussion, i n athletes, 760, 765 Conduct disorder, 62-63 Conductive hearing loss, 198 Condyloma acuminata, 754-755 Confidence interval, 666 Confidentiality exceptions to, 67 minors' consent to treatment and, 242 therapeutic relationship and, 243 Confounding variables, 665 Congenital adrenal hyperplasia (CAH), 1 3 - 14, 2 1 9-22 1 , 2 2 l f, 380-38 1 , 380f, 3 8 l f

814

I n dex

Congenital anomalies of the kidney and urinary tract (CAKUT), 679, 6SO, 6S7 Congenital heart disease brain abscesses secondary to, 5 1 9 chylothorax following surgery to repair, 7 1 7 nutrition in, 556 Congenital motor nystagmus (CMN), 256 Congenital pulmonary airway malformations (CCAMs), 7 1 7 Conjunctivitis, 25S, 270 allergic, 25S atopic, 25S, 259t infectious, 25S-260, 259t in Kawasaki disease, 1 55, 1 56f neonatal, prevention of, 279 oculocutaneous, 25S Conscious sedation, SS6 Consent full information and, 249 by mature minors, 249-250 Constipation, 53, 3 4 1 - 342 formula feeding and, 542 functional, 34 1 , 3S3 in Hirschsprung disease, 340 Constitutional delay of growth and puberty (CDGP), 224-225 Contact dermatitis, 732t, 75 1 allergic, 729-730, 730f, 730t with atopic dermatitis, 49 irritant, 730t Contact sports, von Willebrand disease and, 1 0 1 - 1 02, 1 02t Continuous positive airway pressure (CPAP), 707 for central sleep apnea, 7 1 3 Contraception, failure rates for, 1 5 , 1St Contraction stress tests ( CSTs), 2 7 5 Controlled clinical trials, 661 -662 Convergence-retraction nystagmus, 257 Conversion disorder, 6S, 524 Copper, 477 Coral snakes, 207 Corkscrew hairs, 552f Corneal abrasion, 270-27 1 , 2 7 l f Corneal disease, herpetic, 2 5 9 Corneal foreign bodies, 2 7 1 -272 Cornelia DeLange syndrome (CDLS), 136, 365 Corticosteroids. See also Glucocorticoids; Mineralocorticoids for allergic rhinitis, 199 for anaphylaxis, 3 1 for Henoch-Schonlein purpura, 1 54 for hydrocarbon inhalation, 6 1 1 for immune thrombocytopenia, 77t inhaled, adverse effects of, 579 inhaled, for asthma, 40-4 1 , 4 l f, 42, 44 oral, for asthma, 43 for superior vena cava syndrome, 1 0 5 topical, for atopic dermatitis, 4 S Cortisol excess, 233, 234f

Cor triatriatum sinister, 12S Cost(s), associated with preventable adverse events, 5 6 1 Costochondritis, 1 1 0 Cough, chronic, 7 1 0 , 7 l lt Cover-uncover test, 274 Cow's milk allergy, 33S, 543-544, 543t, SSS-559 Coxsackie viruses, 450 Crab lice, 755 CRAFFT screening tool, 2 1 , 7S l Cranial sutures, 406, 406f premature fusion of, 407 -40S, 40Sf Craniofacial anomalies, otitis media with effusion and, 1 S 3 Craniopharyngioma, S 6 Craniosynostosis, 4 0 7-40S, 40Sf lamboid, unilateral, 409, 409f, 409t Creatine kinase (CK), screening for Duchenne's muscular dystrophy and, 536 Creatinine values, 669, 670t in glomerulonephritis, 6S3 Cremaster reflex, 3S9 Cri-du-chat syndrome, 405 Crigler massages, 265 Critical illness, evaluation and treatment of, 463-465 Crohn disease, 356-357, 3 56t Cross-sectional studies, 663, 663f Croup, 697-69S, 69S£ 705 recurrent, 700 Crouzon syndrome, 3 7 1 Crypt abscesses, 3 5 6 Cryptitis, 3 5 6 Cryptorchidism, 3S7-3SS Cryptosporidium infections, 446 "Cultural competence;' 647-64S Culture, foods and feeding practices and, SSS Cushing syndrome, 233, 234f, 693t Cutting, 66 Cyanosis etiology of, 72 1 methemoglobinemia and, 72 1 in newborns, 129 peripheral, 277 Cyberbullying, 71 Cyclophosphamide, for systemic lupus erythematosus, 1 52 Cyst( s) . See specific sites Cysticercosis, 449 Cystic fibrosis (CF), 1 99, 355, 70S, 7 l l t, 7 1 3 -7 1 7 bronchiectasis in, 7 1 3 - 7 1 4 clinical features of, 7 1 5 , 7 1 5t complications of, 7 1 4 - 7 1 5 , 7 1 6 dehydrated patients with, 307 genetics of, 7 1 5- 7 1 6 hemoptysis in, 7 1 4 vitamin deficiencies in, 545, 5 5 5 Cystic hygromas, 1 96 Cystitis, viral, 6 S l

Cytomegalovirus ( CMV) infections breastfeeding and, 424 congenital, 453, 453f, 454f ubiquitousness of, 454 D

Dacryocystocele, congenital, 265, 266f Dantrolene, for malignant hyperthermia, 166 Day care, infection prevention and, 422 Daydreaming, 5 2 1 Death. See also Sudden infant death syndrome (SIDS); Suicide in adolescence, causes of, 7 -S, Sf, 67 from asthma, 44 cardiac, sudden, 1 1 2 child's response to, 644 due to hydrocarbon inhalation, 6 1 1 due to preventable medical errors, 5 6 1 due t o suicide, 7 - S , Sf, 6 6 , 6 7 , 6St smoking and, 7S3-7S4 stages of, 644 Decerebrate posturing, 1 74, 1 75f Decision making, shared, model of, 242 Decorticate posturing, 1 74, 1 75f DEET (N,N-diethyl-meta-toluamide) , 429 Deferoxamine, for iron overdose, 6 1 9, 620 Dehydration clinical signs to estimate, 3 0 1 , 3 0 l t degree of, estimating, 302 mild, 306-307 Dehydroepiandrosterone (DHEA), puberty and, 6 Delinquency, 63 Delusions, 72 Dengue fever, 429, 453 Dennie-Morgan lines, 727 Dental caries, 634 Denys-Drash syndrome, 394 Depression, 64-67 Dermatitis atopic, 46, 4S-49, 727-729, 72Sf, 732t, 7 5 1 contact. See Contact dermatitis diaper, 752-753 eczematous, 75 1 kwashiorkor, 54 l f scaling, differential diagnosis of, 732t seborrheic, 730-73 1 , 73 l f, 732t Dermatographism, allergy testing and, 35 Dermatomyositis, 1 59, 1 59f, 347 Dermatophyte infections, 736 Dermoid cysts, 744 Desmopressin acetate (DDAVP) for enuresis, 3SS for von Willebrand disease, 1 02 Desonide, for atopic dermatitis, 727, 729t Developmental assessment, formal, 4 1 6 Developmental dysplasia o f the hip (DDH), 505-507, 507f Developmental lines of Blaschko, 744 Developmental milestones, 4 14-4 1 6 , 4 1 5-4 1 7, 4 1 St-4 1 9t loss of, evaluation of, 520

I nd ex

Dexamethasone, maternal treatment with, to reduce virilization, 220-22 1 Dexmedetomidine, 587t Dextroamphetamine, adverse effects of, 583 Dextromethorphan, abuse of, 788 Diabetes insipidus (DI), 674 hypernatremia due to, 3 1 8- 3 1 9 Diabetes mellitus hypertension in, 693t maternal, infants and, 295-296 type 1 , 235-236, 760-76 1 , 76 1 f type 2, 236-237, 236f, 4 1 3 Diabetic ketoacidosis (DKA), 2 3 5 Dialysis, 688 in acute renal failure, 3 2 1 for methanol ingestion, 609 Diamond-Blackfan anemia (DBA), 80, 8 lt, 89t, 3 7 1 Diaper dermatitis, 752-753 Diaper rash, treatment of, 579 Diaphragmatic hernia anterior (Morgagni) , 724 Bochdalek, 724 congenital, 724 Diarrhea in Campylobacter gastroenteritis, 337 in Clostridium difficile colitis, 337 in congenital lactase deficiency, 339 diagnostic tests in, 352-353 differential diagnosis of, 340, 340t in enterocolitis, 340-341 in enterocyte heparan sulfate deficiency, 339-340 functional, 337-338, 338t in giardiasis, 336 infectious, 352, 422, 446 metabolic acidosis and, 3 1 4 most common cause of, 339 Diastrophic dysplasia, 482 Diazepam, 588t, 590 for status epilepticus, 524t Diet gluten-free, for celiac disease, 353 for infantile rotavirus infection, 554 to prevent renal stones, 396 in renal disease, 555 DiGeorge syndrome, 25, 237, 365 Digibind, 6 1 2 Digital clubbing, 72 1 , 722f Digitalis purpurea intoxication, 604, 6 1 2 Dilated cardiomyopathy, 1 14 Diltiazem intoxication, 605 Dimercaprol (BAL), for lead poisoning, 6 1 7 Diphtheria, 456 Diphtheria, tetanus, and acellular pertussis (DTap) vaccine, 632 Discipline, 53-54 Discitis, 504, 504f Disease-modifying antirheumatic drugs (DMARDs), for juvenile idiopathic arthritis, 1 5 8 Dishwasher p o d ingestion o r aspiration, 620

Disruptive mood dysregulation disorder (DMDD ) , 66 Disseminated intravascular coagulation (DIC), 96-97, 5 5 1 Distracted driving, 634 Diuretics, adverse effects of, 585-586 Divorce, 643, 644t DNR orders, 243, 244 Dog bites, 206-207, 449 Domestic violence, 650-65 1 , 6 5 1 t Done nomogram, 5 9 7 , 598 Dopamine, for septic shock, 172 Dose-range checking, 564 "Double bubble" sign, 334, 334f, 363f Downbeat nystagmus, 257 Down syndrome. See Trisomy 2 1 Doxycycline, for Rocky Mountain spotted fever, 44 1 Drawing, development of, 4 1 7 Droplet precautions, 43 1 , 444 Drowning, 1 72- 1 74, 637 Drug abuse. See Substance abuse Drug allergy, 48 Drug testing, 8, 2 1 , 782-783, 783t D -transposition of the great arteries (DTGA), 128- 1 29 Duchenne muscular dystrophy (DMD ) , 372-373, 535-536, 708 DUMBELS mnemonic, 604, 6 1 2 Duodenal atresia, 333-334, 363f Dye disappearance test, 264 Dysentery, 446 Dysfunctional uterine bleeding (DUB), 1 2- 1 5 Dyskeratosis congenita, 89t Dyslexia, 1 3 9 Dyslipidemia, risk factors for, 640, 640t, 64 1 t Dysmenorrhea, 2 1 -22 Dysostosis multiplex, 468 Dystonia, acute, drugs reversing, 525-526 Dysuria, 680-68 1 E

Eagle-Barrett syndrome, 678 Ear(s) cauliflower, 768 foreign bodies in, 1 8 0 hematoma of, 180 Ear pain, causes of, 1 85t Eastern equine encephalitis, 453 Eating problems, in toddlers, 5 1 -52 Echocardiography, sedation for, 588 £-cigarettes, 783 Ecstasy, 787 Ectodermal dysplasia, 742 Ectopic pregnancy, 1 6 Ectothrix infections, 736 Eczema. See Atopic dermatitis Eczema herpeticum, 428f Edema, in kwashiorkor, 540 Edrophonium test, in myasthenia gravis, 534 Educational abuse, 656

81 5

"Eff on a string;' on chest X-ray, 129 Egg allergy, influenza vaccine and, 625, 626 Ehlers-Danlos syndrome (EDS), 1 62, 1 62f, 1 62t, 163( 368-369, 369f Ehrlichiosis, 429, 44 1 Elbow( s), little league, 7 7 1 Elbow dislocation, 7 7 0 , 770f Electrocardiography (ECG), in hyperkalemia, 309, 3 1 0 Electroencephalography (EEG), brain death and, 1 74 Electrolyte abnormalities. See also specific

abnormalities in dehydrated patients with cystic fibrosis, 307 in refeeding syndrome, 358-359, 560 Electrophysiologic studies, in myasthenia gravis, 534 Emancipation, 242 Emergencies, parental refusal of treatment for, 249 Emergent upper endoscopy (EGD ) , 348 Emesis. See Vomiting Emotional abuse, 656 Encephalitis, 5 1 7, 5 1 7t differential diagnosis of, 5 1 7, 5 1 7t Encephalocele, 265, 266f, 529 Encephalopathy, 5 1 3 Encopresis, 341 Endocarditis, 5 1 9 bacterial, subacute, 120- 1 2 1 Endometrial cancer, i n polycystic ovary syndrome, 1 3 Endothrix infections, 736 Endotracheal tube determining internal diameter of, 168 verifying position of, 1 68- 1 69 End stage renal disease (ESRD), hypocalcemia in, 687 End-tidal C02 monitoring, of moderately or deeply sedated patients, 592 Energy drinks, 558 Enhancement therapies, 252 Entamoeba histolytica infections, 43 1 Enteral feedings, for failure to thrive, 559 Enterobiasis, 448-449 Enterococcus infections, 447 Enterocolitis, acute, 340-341 Enterocyte heparan sulfate deficiency, 339-340 Enteroviruses, 450-45 1 Enuresis daytime, 384, 673 nocturnal, 384-385, 385f, 673 Environmental contaminants, in food and water supplies, 62 1 -622 Enzyme replacement therapy (ERT), 469, 470, 470f Eosinophilia, in parasitic disease, 443 Ephedrine, hypertension due to, 693t Epiblepharon, 264 Epidermolysis bullosa simplex, 743-744

81 6

I n dex

Epididymitis, bacterial, 390-39 1 Epidural hematoma (EDH), 2 1 2 -2 1 3 , 2 1 3f, 529 Epiglottitis, 703-704 Epilepsy. See also Seizures absence, 5 2 1 definition of, 52 1 Epinephrine for anaphylaxis, 30-3 1 in hydrocarbon inhalation, 6 1 1 racemic, 697 Epiphora, 267 Epispadias, 382f Epistaxis, 1 0 1 - 1 02, 1 86- 1 87, l 87t Epstein-Barr virus (EBV) , infectious mononucleosis due to, 422-424, 424t Erb palsy, 533-534 in neonates, 294 Erythema infectiosum, 1 6 1 , 422, 450 Erythema marginatum, 1 1 7- 1 1 8 in acute rheumatic fever, l 53f Erythema migrans, in Lyme disease, 430f Erythema multiforme, 744, 745f, 745t major, 20, 445, 523, 744, 745t, 746, 746f minor, 744, 745f Erythema toxicum neonatorum, 738, 739, 739f Erythromycin, drug interactions of, 576-577 Escherichia coli infections, 338-339 day care attendance and, 422 diarrhea due to, 446 meningitis due to, 5 1 5, 5 1 St neonatal, 292 urinary tract, 378 Esophageal atresia (EA) , 702-703, 703f Esophagitis eosinophilic, 336 pill, 349 Esotropia, 256, 273 Estimated energy requirement, 539 Estrogen, for labial adhesions, 384 Ethanol. See Alcohol; Alcohol intoxication Ethosuximide, for myoclonic seizures, 523 Ethylene glycol intoxication, 3 1 1 , 3 12-314, 609, 6 1 0 Eucalyptus, 429 Euthanasia, 246 Evans syndrome, 74 Event reporting, 563 Evoked otoacoustic emissions (OAEs) , 1 98 Ewing sarcoma, 99 Exanthem subitum. See Roseola Exercise asthma and, 37 heart response during, 7 1 2 a s trigger for anaphylaxis, 3 2 vocal cord dysfunction induced by, 7 1 1 Exotropia, 256, 274 Expert witness testimony, 247-248 Expiratory positive airway pressure (EPAP) , central, 7 1 3 Expiratory reserve volume, 720

Extrahepatic biliary atresia (EHBA), 355 Extremely low birthweight (ELBW) preterm infants, 289 Extremity changes, in Kawasaki disease, 1 55 Eye exams, 639 Eyewear, protective, for athletes, 759 F

Fabricated childhood illness, 658, 658t Fabry disease, 469, 470, 470f Facial nerve injury, 532-533, 533f Facial nerve palsy, 1 82, 533 Factitious disorder, 68 Factor V deficiency, 83 Factor V Leiden (FVL) mutation, 95 Factor VII deficiency, 83 Factor VIII deficiency, 82-83, 1 0 1 Factor I X deficiency, 82, 1 0 1 Factor XII deficiency, 8 3 Failure mode and effects analysis, 566, 566f Failure to thrive, 4 1 0-412, 4 l lt, 559 Familial adenomatous polyposis (FAP) , 3 5 1 , 352 Familial hypocalciuric hypercalcemia, 238 Familial lipoprotein lipase deficiency, 468 Family-centered medical homes, 650 Family environment, as risk factor for substance abuse, 780-78 1 Family therapy, divorce and, 643 Fanconi anemia (FA), 89t, 92, 3 7 1 Fat as main energy source for infants, 539 malabsorption of, 352, 355 Fatty acid oxidation disorders, 472-473 Fecal occult blood testing false-positive results on, 350 in intussusception, 328 Feeding problems systolic dysfunction and, 1 1 5- 1 1 6 in toddlers, 5 1 -52 Female athlete triad, 13 Femoral anteversion, 487 Fentanyl, 5 9 1 , 5 9 l t, 593 Fetal alcohol syndrome (FAS) , 296, 366, 366f Fetal Dilantin syndrome, 3 7 1 Fetal gestational age, predicting, 278-279 Fetal hydantoin syndrome, 3 7 1 Fetal lung maturity, improving, 289 Fetal well-being, assessment of, 275 Fetus, intrauterine growth restriction and, 282-283 Fever, 203-205 in athletes, 759 with blood transfusion, 92-93 evaluation of, 203, 204f in infants, 93-94 in Kawasaki disease, 1 1 8 in malaria, 445 management of, 204-205 in sickle cell disease, 84 Fibroadenomas, 5, 3 5 1

Fibroblast growth factor receptor 3 (FGFR3 ) , 48 1 -482 Fibromas, oral, 1 94t Fibrosing colonopathy, in cystic fibrosis, 7 1 6 Fibula, distal, Salter-Harris type I fracture of, 496, 497f Fidelity, 242-243 Fifth disease, 1 6 1 , 422, 450 "Fight bites;' 207 Firearm safety, 638 Fissured tongue, l 94t Fitz-Hugh-Curtis syndrome, 1 8 504 Plans, 145, 1 45t Flail chest, 17 5 Flecainide, for tricyclic antidepressant overdose, 607 Flexion-distraction injuries, 2 1 4 Floppy infant, 5 14-5 1 5 Flotation devices, 637 Flow-volume loops, 700, 700f, 70 l f Fluid maintenance, 307-308, 308t Fluid replacement, during sports, 776-777 Fluid resuscitation, 303 for burn injuries, 2 14, 2 1 6, 2 1 6t in shock, 1 7 1 - 1 72 time required for, 3 1 9 Fluorescent in situ hybridization (FISH), in autism spectrum disorder, 373-374, 3 74t, 375 Fluoride supplementation, 633-634 Folate deficiency, 1 04, 1 07, 5 5 1 during pregnancy, 5 5 1 Folic acid, during pregnancy, 527 Follicle-stimulating hormone (FSH), in hypogonadism, 12 Fomepizole, for methanol ingestion, 609 Fontanelles, closure of, 406, 406f Food(s) to avoid in glucose-6-phosphate dehydrogenase deficiency, 87 environmental contaminants in, 6 2 1 -622 preoperative fasting recommendations and, 589-590, 590t solid, introduction of, 544-545 as triggers for anaphylaxis, 32, 35 Food allergies, 33, 45-47, 557 to eggs, influenza vaccine and, 625, 626 Foot deformities, 484-485 Forced expiratory flow (FEF 25 75% ) , 7 1 8f, 720 Forced expiratory volume in 1 second (FEY,) , 7 1 8f, 720 Forced vital capacity (FVC), 7 1 8 , 7 1 8f, 7 1 9-720 Foreign bodies aspirated, 704, 7 1 lt corneal, 2 7 1 -272 in ear canal, 1 8 0 ingested, 704-705 in nose, 1 8 7 swallowed, requiring endoscopic removal, 6 1 7- 6 1 8 , 6 1 8t Foreskin, abnormalities of, 382, 382f, 398

I nd ex

Formal developmental assessment, 4 1 6 Formula feeding, 542, 543, 544 duration of, 545 47,XXX individuals, 3 7 1 47,XYY individuals, 3 7 1 Fosphenytoin, for status epilepticus, 524t Foster care, 646-647 Foxglove intoxication, 604, 6 1 2 Fractures. See also specific sites child abuse and, 652-653, 653f greenstick, 2 1 6, 2 1 7f patterns of, 2 1 7f vascular or nerve injury with, 2 1 8 Fragile X syndrome, 1 3 6 , 369 Fragile X syndrome molecular testing, 3 74, 375 Francisella tularensis infections, 1 9 5 Frenulum, torn, 6 5 1 -652, 652f Frontal bone, osteomyelitis of, 1 8 8 Frontal-occipital head circumference, 405, 406 Fruit juices, amount in diet, 545 Functional joint pain, 163 Functional residual capacity (FRC), 7 1 8 Fungal infections. See also specific infections in chronic granulomatous disease, 28, 29 of scalp, 736 Futility, physiologic, 245 G

Galactosemia, 460t, 466-467 as contraindication to breastfeeding, 541 Galactose- 1 -phosphate uridyltransferase (GALT), 466-467 Gallbladder, hydrops of, 1 56, 325 Gallstones in common bile duct, 324-325 risk factors for, 325-326 Gardnerella vagina/is vaginosis, 1 7 Gastric ulcers, 348 Gastroenteritis bacterial, 338-339 Campylobacter, 337 Gastroesophageal reflux (GER) , 200, 333, 4 1 1 Gastroesophageal reflux disease (GERD ) , 335-336 Gastrointestinal bleeding lower, 348t upper, 347, 348, 348t Gastroschisis, 294-295 Gaucher disease, 470, 473 type I, 469 Gel phenomenon, 1 5 7 Generalizability, 665 Generalized anxiety disorder (GAD ) , 63-64, 64t Genetic counseling, 463 Genetic disorders. See also specific disorders autosomal dominant, 3 6 1 autosomal recessive, 3 6 1 -362 mitochondrial, 362

multifactorial, 362 recessive, X-linked, 362 Genetic screening informing child of results of, 250-2 5 1 potential risks of, 2 5 1 -252 Genetic testing, in autism spectrum disorder, 373-375, 3 74t Genitalia ambiguous, 2 1 9-22 1 , 220f, 2 2 l f, 380, 380f formation of, 390, 390f Genu valgum, 485-487 Genu varum, 485-486 in rickets, 549, 549f Geographic tongue, 1 94t Germ cell tumors, primary, 86 Giardiasis, 336, 446 chronic, 352 Gift(s), to physicians from patients and families, 247 Gifted children, siblings of, 649-650 Gilbert syndrome, 343 Gingivostomatitis, 427-428 herpetic, acute, 1 94t Glanzmann thrombasthenia, 1 0 1 Glasgow Coma Scale (GCS), 1 74, 1 75t, 2 1 0, 2 1 0t, 2 1 1 Glaucoma, congenital, primary, 264, 265f, 267 Glomerular filtration rate (GFR) , 669, 670t Glomerulonephritis, 682 membranoproliferative, 682-683 postinfectious, 6 8 1 -682 poststreptococcal, 682 Glossopexy, 195 Glucagon for anaphylaxis, 3 1 for beta-blocker intoxication, 605 Glucocorticoids for acquired adrenal insufficiency, 232, 233 for asthma, 41 for congenital adrenal hyperplasia, 2 1 9, 220 hypertension due to, 693t Glucose-6-phosphate dehydrogenase (G6PD) deficiency, 74, 75, 87, 1 0 7 "Glue-sniffers rash;' 6 1 1 Glutaric acidemia, 460t Glutaric aciduria, type I, 472, 478 Glycine encephalopathy, 474 Glycogen storage diseases, 460t, 465-466 Glycosylation, congenital disorders of, 464 Gonadal dysgenesis, pure, 1 1 Gonadarche, 6 Gonadotropin-releasing hormone (GnRH) deficiency of, 6 puberty and, 6 Gonococcal urethritis, 435 Gonorrhea, 1 8 Gower sign, 1 59 Grace period, substance abuse and, 785 Granulocyte colony-stimulating factor (GCSF), for neutropenia, 104

81 7

Granuloma(s), in chronic granulomatous disease, 29 Granuloma annulare, 75 1 Gratification behavior, 5 2 1 Graves disease, 2 3 1 "Gray baby syndrome:' 578 Great arteries, D-transposition of, 1 28- 1 29 Greenstick fractures, 2 1 6, 2 1 7f Grisel syndrome, 483 Group A streptococcus (GAS), 455 carriers of, 190 cellulitis due to, 755 cervical lymphadenitis due to, 1 9 5 neonatal screening for, 424 pharyngitis due to, 1 89- 1 90, 423 Sydenham chorea due to, 526 Group B streptococcus (GBS) cervical lymphadenitis due to, 1 9 5 meningitis due to, 5 1 5, 5 1 St neonatal infections due to, 292, 293f Growth intrauterine, restriction of(IUGR) , 282-283 poor, evaluation of, 4 1 0 tracking toward genetic potential, 404-405 Growth charts, neonatal, 282f Growth curves, for genetic syndromes, 4 1 0 Growth deceleration (failure), 225-226, 233 Growth rate, linear, 403 Growth spurt, pubertal, 403 Guanfacine, 585 Guardians, consent from, lack of need for, 242 Guillain-Barre syndrome (GBS), 5 1 9, 525, 534 Campylobacter jejuni infection and, 337 differential diagnosis of, 532, 532t influenza vaccine and, 625 Gynecomastia, 3-4 H

Haemophilus influenzae infections acute otitis media due to, 1 80, 1 8 1 preseptal cellulitis due to, 261 Haemophilus influenzae type b (Hib) infections, 456 Haemophilus influenzae type b (Hib) vaccine, 627 Hair(s) loss of, 734-736, 735t twisted, 736 Hallucinations, 72, 536 Hallucinogens, abuse of, 786 Hand-foot-and-mouth disease, 1 94t, 450 Hashimoto thyroiditis, 229-230 H2 blockers, 580 Headache, S l l - 5 1 3 intracranial hypertension and, 5 1 1 management of, 5 1 2 - 5 1 3 migraine, 3 3 2 , 5 1 2 Head circumference, 405, 406-407

81 8

I n dex

school placement and, 145 sensorineural, 198 Hearing tests, 1 97, 198 Heart block, third-degree, 125 Heart disease. See Congenital heart disease Heart failure, 1 1 5 volume overload, 1 1 2 Heart murmurs, 109 Heat exhaustion, 763, 763t Heat illness, risk factors for, 7 64 Heat stroke, 763-764, 763t Height, midparental, 405 Heimlich maneuver, 705 Helicobacter pylori infections, peptic ulcer disease and, 3 3 1 Helmets. See Headgear Hemangiomas, 200

Henoch-Schiinlein purpura (HSP), 1 5 3 - 1 54, 1 54f, 5 5 1 , 683 Heparin-induced thrombocytopenia (HIT) , 94-95, 95t Hepatic failure, fulminant, 1 6 7- 168 Hepatitis autoimmune, 343-344 chronic, 345 viral, 344-345 Hepatitis A, transmission of, 437 Hepatitis A vaccine, 627 Hepatitis B breastfeeding and, 424 transmission of, 436-437 Hepatitis B core antibody (HBcAb) , 437 Hepatitis B immune globulin (HBIG), 436-437 Hepatitis B surface antibody (HBsAb), 437 Hepatitis B surface antigen (HBsAg), 437 Hepatitis B vaccine, 437, 627 Hepatitis C breastfeeding and, 424 transmission of, 43 7 Hepatoblastoma, 96, 97, 3 5 1 -352 Hepatomegaly, 3 5 1 Hepatoportoenterostomy, 347 Hereditary arthroopthalmopathy, 1 9 5 Hereditary spherocytosis ( H S ) , 7 4 , 75, 88 Hernias

capillary, 273, 273f, 737 infantile, 273, 273f, 737 infantile periocular, 273, 273f liver, 3 5 1 oral, 1 94t strawberry, 737 subglottic, 699, 737 Hematochezia, 3 5 1 Hematomas of ear, 180 epidural, 2 1 2- 2 1 3 , 2 1 3f nasal, 1 8 7 subdural, 2 1 3, 2 1 3f Hematopoietic stem cell transplant (HSCT), for congenital microthrombocytopenia, 92 Hematuria, 672-673 gross, 672 microscopic, 1 54, 6 7 1 Hemodialysis, for salicylate intoxication, 597-598 Hemoglobin dissociation curve, 706, 706f Hemoglobin SS disease, 83-84, 94 Hemolytic disease of the fetus and newborn (HDFN) , 1 0 7 Hemolytic uremic syndrome (HUS), 1 0 6 , 446, 683 Hemophilia A, 82-83, 1 0 1 Hemophilia B , 82, 1 0 1 Hemoptysis, i n cystic fibrosis, 7 1 4 Hemosiderosis, pulmonary, idiopathic, 726 Hemothorax, 1 76

incarcerated, 397 inguinal, 397 Herpangina, 1 94t, 427, 428f Herpes, neonatal, 738-739 Herpes labialis, 427 Herpes simplex virus (HSV) breastfeeding and, 424 as contraindication to breastfeeding, 542 encephalitis caused by, 5 1 7 neonatal infections with, 426-428, 427f, 428f ophthalmia neonatorum due to, 257, 433 Herpetic gingivostomatitis, acute, 1 94t Herpetic whitlow, 428, 428f Hers disease, 466 Heterophile antibody test, 423 Heterotopic ossification, 762 Heterozygous familial hypercholesterolemia (HeFH), 468-469 Hip, bacterial or septic arthritis of, 426 Hip dysplasia, 505-507, 507f Hirschberg test, 274 Hirschsprung disease, 284, 295, 340, 341 Histiocytosis, 504, 505f Histoplasma, 445 HIVI AIDS, 1 00 blood transfusion and, 93 breastfeeding and, 424 as contraindication to breastfeeding, 542 contraindication to MMRV vaccine in, 628 failure to thrive and, 4 1 1

Headgear for all-terrain vehicle use, 635 for athletes, 768 Head lice, 422, 755 HEADSSS screening tool, 6-7 Hearing loss, 140- 1 4 1 , 1 97- 198 in autism, 135 cochlear implants for, 253 conductive, 198 mild, moderate, and severe, 198 with otitis media with effusion, 1 8 3 risk factors for, 1 97, 1 97t

diaphragmatic. See Diaphragmatic hernia

informing child of status, 248-249 screening for, 1 7 testing for, 422 transmission of, 7, 42 1 -422 Hives. See Urticaria Hoarseness, 200, 705 Hodgkin lymphoma, 77-78, 78f, 7 1 2 Holt-Oram syndrome, 370-37 1 , 3 7 l f Home environment, a s risk factor for substance abuse, 780-78 1 Homocystinuria, classical, 368 Homozygous familial hypercholesterolemia (HoFH), 468-469 Honey, botulism and, 443 Hordeolum, 263, 263f, 264 Hormones. See also specific hormones puberty and, 6 Horner syndrome, 265, 533, 534f, 535f Hot potato voice, 189 Howell-Jolly bodies, 103, 1 03f H2 receptors, 580 Huffing, 6 1 0-6 1 1 Human bites, 207 biting and, 53 Human herpesvirus 6 (HHV-6), 451 Human immunodeficiency virus (HIV). See HIVIAIDS Human papillomavirus (HPV) , 200, 436 condyloma acuminata due to, 754-755 transmission of, 7 Human papillomavirus (HPV) vaccine, 632 Human T-lymphotropic virus (HTLV) infections, as contraindication to breastfeeding, 424, 542 Hunter syndrome, 467, 468, 470 Hurler syndrome, 467, 467f, 468, 470 Hydralazine, for hypertensive emergency, 694 Hydration preoperative, 306 withholding, 246 Hydration status, weight change as determinant of, 3 0 1 Hydrocarbons abuse of, 788 household products containing, 6 1 0t inhalation of, 6 1 0- 6 1 2 , 6 1 lf Hydrocephalus, 407 obstructive, 527 Hydrochlorothiazide intoxication, 605 Hydrocortisone for atopic dermatitis, 727, 729t for congenital adrenal hyperplasia, 2 1 9, 220 Hydromorphone, 5 9 l t Hydronephrosis, 385-386, 386f, 392 Hydrops, of gallbladder, 1 56, 325 Hydroureteronephrosis, bilateral, 679 Hygromas, cystic, 1 96 Hymenoptera stings, 208 Hyperaldosteronism, primary, 693t Hyperammonemia, 470

I nd ex

Hyperbaric oxygen, 6 1 5 Hyperbilirubinemia conjugated, 2 9 1 direct, 342-343, 344t, 346 indirect, 343, 343t in neonates, 286-287, 286t, 290-29 1 Hypercalcemia, 237-238, 238t causes of, 304t hypocalciuric, familial, 238 Hypercalciuria, 671 Hypercalprotectinemia, 477 Hypercoagulable state, in nephrotic syndrome, 684 Hyperglycemia, nonketotic, 474 Hyperimmunoglobulin E syndrome (HIES), 29, 30 Hyperinsulinism, 468 hypoglycemia and, 464, 464f Hyperkalemia, 308-3 1 0 , 309t Hyperleukocytosis, 1 0 5 Hypernatremia, 3 1 7- 3 1 9 Hyperopia, high, 256 Hyperosmolar hyperglycemic state (HHS), 3 1 7 Hyperparathyroidism primary, 693t secondary, 555 Hyperprolactinemia, amenorrhea and, 1 1 Hypersensitivity reactions classification of, 47, 47t to contrast media, 32 Hypertension, 1 09- 1 1 0, 233, 690-695 acute hypertensive emergency and, 694 classification of, 690, 690f coarctation of the aorta and, 694 diagnosis of, 692 drug-induced, 692, 693t evaluation of, 694-695 intracranial, 408, 5 1 1 pulmonary, 1 1 3 - 1 14 renal artery stenosis and, 695 risk factors for, 690 secondary, endocrine causes of, 692-693 , 693t sequelae of, 695 Hypertensive emergency, 694 Hypertensive urgency, 694 Hyperthermia, 205 Hyperthyroidism, 23 1 , 693t Hypertrophic cardiomyopathy (HCM), 1 1 2 Hypertyrosinemia, transient, 462-463 Hyperviscosity, 1 0 5 Hypervitaminosis A, 555 Hyperzincuria, 477 Hyphemas, 272, 272f, 765, 765f Hypoalbuminemia, 304-305, 683 Hypocalcemia, 237, 477 acute-onset, clinical manifestations of, 304t causes of, 304t in end stage renal disease, 687 Hypochondriasis, 68

Hypogammaglobulinemia, 544 Hypoglycemia in anorexia nervosa, l O hypoketotic, 464, 464f ketotic, 464 neonatal, 283 Hypogonadism hypogonadotropic, 229, 229f primary and secondary, 1 2 Hypohidrotic ectodermal dysplasia, 74 1 - 742 Hypokalemia, 309t in anorexia nervosa, lO beta2-agonists and, 5 8 1 Hypomagnesemia, 477 Hypomelanotic macules, in tuberous sclerosis complex, 369-370, 370f Hyponatremia, 3 1 0, 3 1 lf in anorexia nervosa, 1 0 correcting, 303 MDMA and, 787 Hypophosphatemia, 358 in refeeding syndrome, 1 0 , 560 Hypopituitarism, amenorrhea and, 1 1 Hypoplastic left heart syndrome (HLHS), 1 1 5, 1 1 5f Hypospadias, 382, 382f Hypotension by age and systolic blood pressure, 300t in shock, 299, 300t Hypothermia, 1 72- 1 73 in newborns, 276-277 Hypothyroidism, 693t amenorrhea and, 1 1 congenital, 230-2 3 1 Hypotonia, central vs. peripheral, 5 14, 5 14t Hypoxemia, 707 detection of, 706, 706f Hypoxia, to brain, in newborns, 288

Ibuprofen, 676t Icthyosis vulgaris, 727 Idiopathic thrombocytopenia (ITP), maternal, infant and, 296-297 Idiopathic toe walking, 487-488 Idiosyncratic drug reactions, 577 Ilntrauterine growth restriction (IUGR) , 282-283 "Image Gently" campaign, 394 Imipramine, for enuresis, 385 Immotile-cilia syndrome, 7 1 6 - 7 1 7 Immune thrombocytopenia (ITP), 76-77, 77t Immunocompromised patients necrotizing fasciitis in, 756 pneumonia in, 448 vaccinations in, 45 1 Immunodeficiency, 25-30. See also HIVI AIDS; Severe combined immunodeficiency (SCID) common variable, 28-29

81 9

cough and, 7 1 l t red flags for, 27-28, 27t Immunoglobulin, following measles exposure, 628 Immunoglobulin A (IgA) deficiency, selective, bronchiectasis and, 724 Immunoglobulin A (IgA) nephropathy, 672 Immunomodulators, for urticaria, 33 Immunosuppressives, nephrotoxic, 676t Imperforate anus, 284 Impetigo, 729, 746-747, 746f, 747f neonatal, 739 Inactivated polio vaccine (IPV) , 633 Inborn errors of metabolism, 459, 460t-46 l t screening for, 459, 46 1 -462 Incidence, 666 Incontinence fecal, overflow, 341 urinary, 382-383 . See also Enuresis Incontinentia pigmenti, 744 Individual family service plan (IFSP), 145 Individualized education plan (IEP), 145, 146 Individuals with Disabilities Act (IDEA), 144, 145 - 1 46, 145t Individual therapy, for children experiencing divorce, 643 Indomethacin, 676t Induced childhood illness, 658, 658t Infant(s). See also Neonates attachment and, 59 average for gestational age, 282 colic in, 59 fever in, 93-94 large for gestational age, 2 8 1 , 282, 283 main energy source for, 539 nonaccidental trauma in, 97-98 nutrition of, 539-540, 541 -545 postterm, 279-280, 2 8 l f preterm. See Preterm infants safety recommendations for, 636t small for gestational age, 2 8 1 , 282-283 term, definition of, 279 tetralogy of Fallot in, 1 2 7- 1 28, 1 2 7f Infantile hypertrophic pyloric stenosis (IHPS), 332-333 Infantile periocular hemangiomas, 273, 273f Infantile spasms, 522t, 523 Infants of diabetic mothers (IDMs) , 295-296 Infection(s). See also Bacterial infections; Fungal infections; Viral infections;

specific infections and pathogens with burn injuries, 455 definition of, 1 7 l t malnutrition and, 457 preventable, 456 Infection prevention, 422, 43 1 , 432t Infectious mononucleosis, 78, 258, 422-424, 424t Infective endocarditis, 1 1 9- 1 2 1 , 1 20t Inflammatory bowel disease (IBD), 356-357, 3 57t

820

I n dex

Infliximab, for Kawasaki disease, 1 56 Influenza, prevention of, 45 1 Influenza vaccine, 625-627 for children with chronic health conditions, 625-626 effectiveness of, 626-627 egg allergy and, 625, 626 Guillain-Barre syndrome and, 625 inactivated (IIV) , 625-626 Inguinal canal, swelling in, 397 Inheritance patterns, 3 6 1 -362 Inspiratory capacity, 7 1 8 Institute for Healthcare Improvement (IHI) model for improvement, 568 Insulin for beta-blocker intoxication, 605 need for, physical activity and, 760-76 1 , 76 l f Insulin resistance, acanthosis nigricans and, 236, 236f, 4 1 2 -4 1 3 Intellectual disability (ID), 1 3 5 - 1 36 in autism, 1 3 5 diagnosis of, 1 3 1 - 1 3 3 , 1 32t Intelligence testing, 142- 1 44, l44t interpretation of, 143- 144 testing situation and, 1 43 Intention to treat analysis, 665 Interferon gamma release assays (IGRAs), 438-439 Intertrigo, 753 Intestinal obstruction acute, 329 signs of, 334 Intimate partner violence, 650-65 1 , 6 5 l t Intracranial aneurysm, 1 1 6 Intracranial hemorrhage (ICH), 88, 98 Intracranial hypertension, 408, 5 1 1 Intracranial pressure (ICP), elevated, 2 12, 2 1 2t, 269 impending coma from, 1 6 5- 1 66 Intraocular pressure (IOP), elevated, 272 Intraosseous (I 0) access, 1 7 1 Intravenous fluids, isotonic, 3 1 9 Intravenous immunoglobulin (IVIG) hemolytic anemia caused by, 1 5 7 for immune thrombocytopenia, 77t for Kawasaki disease, 1 56 for severe combined immunodeficiency, 27 for varicella prevention, 4 5 1 for X-linked agammaglobulinemia, 2 8 Intravenous medication delivery, 565-566 Intraventricular hemorrhage (IVH), in low birthweight and premature infants, 290 Intussusception, 328-329, 350 Ipecac, 596 Iron deficiency, 553, 554t vegan diets and, 556 Iron deficiency anemia (IDA) , 73, 74f, 90 in celiac disease, 354 early introduction of cow's milk and, 553, 554t occult gastrointestinal bleeding and, 348

Iron overdose, 6 1 8-620 Iron replenishment therapy, 90 Iron tablets, on X-rays, 6 1 8- 6 1 9 Irritable bowel syndrome (IBS), 330-3 3 1 , 330t Iselin disease, 766 Isolation precautions, 43 1 , 432t Isopropyl alcohol ingestion, 3 1 1 Ivermectin, for lice, 755

J Janeway lesions, 1 1 9, 1 20 Jaundice in alpha - ! -antitrypsin deficiency, 346 breastmilk, 346 neonatal, 342 in neonates, 287, 290-29 1 Jejunal atresia, in preterm infants, 295 Jellyfish stings, 209 Jersey finger, 7 7 1 Jimson weed toxicity, Poinsettia toxicity, 6 1 2 Job syndrome, 2 9 Jones Criteria for acute rheumatic fever, 1 1 7, 1 1 7t, 1 53 , l 53f, l 53t Juvenile ankylosing spondylitis (JAS), 1 60- 1 6 1 Juvenile idiopathic arthritis ( JIA) , 1 57- 1 58, l 5 7t Juvenile polyposis syndrome, 350 K

Kallman syndrome, 6, 186 Kartagener syndrome, 7 1 7 Kasabach-Merritt syndrome, 273 Kasai procedure, 347 Kawasaki disease, 20, 1 1 1 , 1 1 8, 1 1 9t, 1 5 5 - 1 57, 1 55f, 1 56f, 450 Kearns-Sayre syndrome, 46 l t Keratoconjunctivitis atopic, 258 vernal, 258 Keratosis pilaris (KP), 727, 728f Kerions, 736 Kernicterus, 529 in neonates, 2 9 1 Ketamine, 587t Ketogenic diet, for seizures, 557 Ketorolac, 676t Key driver diagrams, 568 Kidneys. See also Acute kidney injury (AKI); Renal entries multicystic, dysplastic, 675, 676, 677t role in homeostasis, 669 Kidney stones, obstructive, with oliguric acute kidney injury, 686 Kidney transplantation deceased vs. living donor, 688-690, 689t vaccines prior to, 688 Klinefelter syndrome, 136, 368, 368f, 3 7 1 Klippel-Feil syndrome, 3 70, 483, 483f Kocher criteria, 493, 494

Koplik spots, 449, 449f Kostmann syndrome, 28 Krabbe disease, 473, 478, 520 Kwashiorkor, 540-54 1 , 540f, 54 l f Kyphosis, 500, 500f L

Labetalol, for hypertensive emergency, 694 Labial adhesions, 384-385, 385f Lacerations, 205-206 pulmonary, 2 1 1 Lachman test, 773, 774f LaCrosse virus, 429 Lactase deficiency, congenital, 339, 543 Lactated Ringer solution, components of, 306 Lactic acidosis, 470 Lactose intolerance, 554 Langerhans cell histiocytosis (LCH), 8 1 -82, 82f, 83t Langley model for improvement, 568 Language expressive, development of, 1 3 7 , 1 38t hearing impairment and, 1 4 1 receptive, development of, 1 37, 1 3 7t speech distinguished from, 1 3 7 Language disorders, 1 3 1 , 1 34, 1 3 8 expressive, 1 3 1 Large for gestational age (LGA), 2 8 1 , 282, 283 Larsen syndrome, 482 Laryngeal clefts, 200 Laryngeal webs, 200 Laryngitis, viral, 200 Laryngomalacia, 698-699, 699f Laryngotracheobronchitis, 705 recurrent, 700 viral, 697-698, 698f Latent errors, 562 Latrodectus bites, 208 Laundry pod ingestion or aspiration, 620 "Lazy eye;' 270, 270t Lead exposure environmental sources of, 639 screening for, 638-639 Lead poisoning, 5 1 3, 6 1 5-6 1 7, 6 1 6t risks of, 639 Lean methods, 568 Learning culture, mature, 569 Learning disabilities, 1 3 8 - 1 3 9 psychological problems associated with, 149 Leber hereditary optic neuropathy (LHON), 476 Legg-Calve-Perthes disease, 493-494, 50 1 , 50lf Leg-length discrepancy (LLD ) , 490-49 1 , 490t, 499 Lesch-Nyhan syndrome, 476-477 Lethargic, definition of, 5 1 3 Leukocoria, 97, 256, 268

I nd ex

Leukocyte adhesion deficiency, 29 Leukodystrophies, 477-478, 478f Leukostasis, 1 0 5 Leukotriene antagonists, for allergic rhinitis, 1 99 Leukotriene receptor antagonists (LTRAs) , for asthma, 4 1 , 4 l f, 4 2 , 44 Levetiracetam for myoclonic seizures, 522-523 for status epilepticus, 524t Levofloxacin, adverse effects of, 578 Levothyroxine, for autoimmune (Hashimoto) thyroiditis, 230 Lice, 422, 755 Lichen simplex chronicus, atopic dermatitis and, 49 Lidocaine, for tricyclic antidepressant overdose, 607 Li-Fraumeni syndrome (LFS) , 96, 97 Likelihood ratio, 666 Limit setting, 53-54 Limp, differential diagnosis of, 493-494 Lines of Blaschko, 744 Lips, cleft, 193- 1 95, 1 95f Lisinopril, 585 Lisinopril intoxication, 605 Lissencephaly, 407 Listeria monocytogenes infections, meningitis due to, 5 1 5, 5 1 St

Listeria monocytogenes sepsis, 457 Little league elbow, 771 Liver disease, cholestatic, 557-558 Liver enzymes, in anorexia nervosa, 10 Liver failure, 5 5 1 Liver function tests, 345t Lomotil intoxication, 602 Long QT syndrome, 1 1 2, 126, 1 26f Lorazeparn, 588t for status epilepticus, 524t Losartan, 585 Low birthweight (LBW) preterm infants, 289, 290 Lower extremities, angular deformities of, 485-487, 487t Loxosceles bites, 208 Lumbar spine injuries, 2 1 4 Lung(s) air trapping vs. restrictive changes in, 7 1 8 , 7 1 8t fetal, maturity of, improving, 289 Lung disease, of prematurity, chronic, 296 Lupus nephritis, 682-683 Luteinizing hormone (LH) in hypogonadism, 12 puberty and, 6 Lying, 59 Lyme carditis, 1 2 5 Lyme disease, 429-430, 430f, 4 5 3 , 494 arthritis associated with, 1 57, 426, 433-434 Bell palsy and, 533

Lymphadenitis, 483 cervical, bilateral, 196 cervical, unilateral, 1 95 - 1 96 mycobacterial, nontuberculous, 454-455 Lymphadenopathy, 1 0 5 cervical, i n Kawasaki disease, 1 5 5 Lymphangiomas, 1 9 6 oral, 1 94t Lymphocyte count, low, 25 Lymphoid hyperplasia, 349 M

Macrocephaly, 407 Macrophage activation syndrome (MAS), 1 5 8 Macrosomia, 294 Macrothrombocytopenia, 1 0 1 Magnesium, 477 Magnetic resonance imaging (MRI), in increased intracranial pressure, 5 1 1 Magnet ingestion, 6 1 5 Major depressive disorder (MDD), 66 Malabsorption, 352 Malaria, 445 Malathion, for lice, 755 Malignant hyperthermia (MH), 1 66

Malingering, 68 Mallampati classification, 590 Mallet finger, 77 1 , 7 7 l f Malnutrition, 540-541 chemotherapy and, 555, 556 infections and, 457 Malrotation, with volvulus, 327-328, 327f, 328f, 333-334 Mandibular distraction, 195 Mandibular hypoplasia, 366 Mandibulofacial dysostosis, 1 86, 1 95, 372, 372f Manic episodes, 63, 68 Maple syrup urine disease, 460t, 474 Marasmus, 540, 54 l f Marcus-Gunn pupil, 268 Marfan syndrome, 1 6 2 - 1 6 3 , 1 63f, 368, 368f, 499 Marijuana, abuse of, 78 1 , 784-785 Maroteaux-Lamy disease, 470 Marshal syndrome, 3 72 Mastocytosis, cutaneous, 743, 743f allergy testing and, 35 Mastoiditis, acute, 1 8 5 Maternal age, trisomy 2 1 and, 364 McArdle disease, 465, 466 MDMA, abuse of, 787 Mean arterial pressure (MAP), of newborn, 279 Measles, 43 1 , 449, 449f, 450f postexposure prophylaxis for, 628 Mechanical ventilation, in persistent vegetative state, 245 Meckel diverticulum, 350

82 1

Meconium delayed passage of, 284 suction of, 287-288 Meconium aspiration syndrome, 288, 293 Meconium ileus, 284 Meconium plug syndrome, 284 Medial collateral ligament (MCL) sprain, 765 Mediastinal masses, 1 0 5 Medical abuse, 6 5 6 Medical errors, 5 6 1 - 570 alarm sensitivity and specificity and, 570 communication to prevent, 563, 570 compensation for consequences of, 567 disclosing, 246 disease, 567 failure mode and effects analysis and, 566, 566f learning culture and, 569 medication, reducing, 564-566, 569 model for improvement and, 563, 564f preventable, 5 6 1 private rooms vs. p o d arrangement and, 569 rates of, in PICUs, 562 reduction in variation to reduce, 566-568, 567f "second victim" syndrome and, 567 sentinel events and, 563 "Swiss Cheese Model" of, 562, 569, 569f Medication(s), 573-593. See also specific

medications and classes of medications adverse effects of, 577-586 for alcohol and drug addition, 782 to avoid in glucose-6-phosphate dehydrogenase deficiency, 87 to avoid in long QT syndrome, 126 black box warnings and, 575, 583 for constipation, 34 1 , 342, 342t drug interactions of, 576-577 epistaxis due to, 1 86, 1 8 7t half-life of, 574 high-alert, 565 for juvenile idiopathic arthritis, 1 5 8 linear pharmacokinetics of, 575 maternal, Apgar score and, 289 nephrotoxic, 675, 676t ototoxic, 198 pharmacodynamics of, 575 pharmacokinetics of, 575 to reverse acute dystonia, 525-526 serum concentration of, 5 74 syndrome of inappropriate antidiuretic hormone secretion caused by, 3 1 1 taking with food, 573, 575 tall-man lettering for labels on, 565 taste of, 573-574 transmission in breast milk, 542 tremors caused by, 525 trough and peak levels of, 576 verbal orders for, error associated with, 564

822

I n dex

Medication reconciliation, S6S ME DIE A mnemonic, 608 Medium-chain acyl-CoA dehydrogenase deficiency (MCADD), 46 lt, 462, 473 Medullary thyroid carcinoma (MTC), 1 9 7 Medulloblastoma, 8S-86 MedWatch, S77 Megalencephaly, 407 Megameatus, with intact prepuce, 382 Melanocytosis, dermal, 739-740, 740f Membrane transport defects, 46 l t Menarche, S, 2 2 Meningitis, S 1 S- S 1 6 aseptic, S 1 6 bacterial, S 1 6, S 1 6t cerebrospinal fluid composition in, S 1 6, S 1 6t complications of, S 1 6 enteroviral, S 1 S- S 1 6 Neisseria meningitidis, 439-440, 439f Meningococcal vaccine, 627, 6 3 1 Meningococcemia, 9 6 Meningoencephalitis, S 1 7 Meniscus tears, medial, 773-774, 774f Menkes disease, 4 77 Menometrorrhagia, 12 Menorrhagia, 12, 1 4- 1 S Menses absence of, 1 0 - 1 2 dysfunctional uterine bleeding and, 12- 1 S dysmenorrhea and, 2 1 -22 onset of, S, 22 Mental status ahered, S 1 3 , S 14, S 1 S documenting, S 1 3 Mercury thermometers, 203 Meta-analyses, 663, 664f, 66S Metabolic acidosis, 3 1 3-3 14, 3 1 S, 3 1 St, 706, 706f, 707 deferoxamine therapy and, 620 hypochloremic, in anorexia nervosa, 1 0 with normal anion gap, 674-67S shortness of breath and, 7 1 1 Metabolic alkalosis, 3 1 St, 706, 706f, 707 hypochloremic, 30S Metabolic screening, newborn, 276 "Metabolic syndrome;' 237 Metachromatic leukodystrophy (MLD), 473 -474, 478 Metaphyseal changes, in rickets, S49,SSOf Metatarsus adductus, 48S Methadone, S 9 l t Methamphetamine, abuse of, 788 Methanol ingestion, 3 1 1 , 3 1 2- 3 1 4, 608-609, 6 1 0 Methemoglobinemia, cyanosis and, 72 1 Methicillin-resistant Staphylococcus aureus (MRSA) infections, 4SS antibiotics for treatment of, 189 pneumonia due to, 723

Methicillin-sensitive Staphylococcus aureus (MSSA) infections, 4SS antibiotics for treatment of, 189 Methylmalonic acidemia, 460t, 472 Methylphenidate, S 8 1 , S83 abuse of, 787 hypertension due to, 692, 693t Methylxanthines, adverse drug reactions to, S78 Metoprolol intoxication, 60S Metrorrhagia, 12 Microarray comparative genomic hybridization (CGH), in autism spectrum disorder, 373, 3 74, 3 74t, 37S Microcephaly, 40S Microthrombocytopenia, congenital, 92 Midazolam, S86, S88, S88t, S93 Middle aortic syndrome, 69S Midnight salivary cortisol test, 233 Midparental height, 40S Miglustat, 47S Migraines abdominal, 332 with aura, S 1 2 Milk cow, allergy to, 338, S43-S44, S43t, SS8-SS9 human. See Breastfeeding unpasteurized, 447 "Milkmaid's grip;' S26 Milrinone, for cardiogenic shock, 1 6 7 Mineralocorticoids for acquired adrenal insufficiency, 232 for congenital adrenal hyperplasia, 220 hypertension due to, 693t Mineral oil preparation, 7SOt Minimally conscious state, 24S Mitochondrial disorders, 362 Mitochondrial encephalopathy, lactic acidosis, and stroke syndrome (MELAS) , 46 lt, 476 Mitochondrial myopathies, 373 MMR vaccine, 444, 627-629 contraindications to, 627, 629 postexposure prophylaxis with, 628 MMRV vaccine, 627 contraindications to, 628 Mobitz type I heart block, 1 2 S Mobitz type II heart block, 1 2 S Modified Checklist for Autism i n Toddlers (M-CHAT) , 1 3 3 , 1 34, 136, 4 1 3-414 Modified Checklist for Autism in Toddlers- ­ Revised (M-CHAT-R) , 1 33 , 1 34, 1 3 6 Molluscum contagiosum, 2S7-2S8, 2S8f, 7S4, 7S4f Molly, 787 Monitoring, of moderately or deeply sedated patients, S 9 1 -S92 Monro-Kellie doctrine, 1 66, 1 66f Mood stabilizers, for bipolar disorder, 68 "Moonshine;' 608-609 Morgagni hernia, 724

Moro reflex, 4 1 4 Morphine, S90, S 9 1 t Morquio disease, 4 6 7 , 470 Mosquitos, diseases transmitted by, 429, 4S2-4S3 Motor development, 4 1 7 Moyamoya disease, S 3 1 Mucoceles, 1 94t Mucopolysaccharidoses, 46 l t MUDPILES mnemonic, 3 1 4 Muhicystic dysplastic kidney (MCKD ) , 67S, 676, 677t Muhidisciplinary rounds, medical error prevention and, S63 Muhiple endocrine neoplasia (MEN) , types 2A and 2B, 1 9 7 Muhiple sclerosis, S 1 9 Mumps, 3 9 1 , 443-444, 444f Munchausen syndrome by proxy, 6S8, 6S8t Mupirocin (Bactroban), 729 Murmurs, in tetralogy of Fallot, 1 28 Muscular dystrophies, S34-S36 Muscular torticollis, congenital, 482-483 Mushroom intoxication, 604 Myasthenia gravis, S34 Myasthenic syndromes, congenital, S34 Mycobacterium avium-intracellulare (MAl) infections, 4SS Mycoplasma infections, 423, 44S Myelomeningocele, S27-S28 Myocarditis, 1 14- 1 1 S Myoclonic epilepsy and ragged red fibers (MERRF), 46 1 t Myopia, i n Marfan syndrome, 1 62 Myositis ossificans, 762 Myotonic dystrophy, congenital, 373 N

Naloxone, S 9 1 , 602 duration of effectiveness of, 602 in neonates, 602 Naproxen, 676t intoxication by, S98 Narcolepsy, S36 Narcotics. See also Opioids definition of, S 9 1 Nasal polyps, 1 99 Nasogastric tubes, complications of, SS9 Nasolacrimal duct (NLD) congenital obstruction of, 264-26S, 264f massages of, 26S National Association of the Deaf (NAD) , 2S3 Near-miss events, S 6 1 Neck injuries, 76S Neck masses, differential diagnosis of, 1 96, 1 96t Necrotizing enterocolitis (NEC), in preterm infants, 29S Necrotizing fasciitis, 7S6 Negative predictive value, 666

I nd ex

Neisseria gonorrhoeae infections ophthalmia neonatorum due to, 257, 433 pyogenic arthritis due to, 492 screening for, 1 7 Neisseria meningitidis infections meningitis due to, 439-440, 439f, 5 1 5 nasopharyngeal carriage of, 440 septic shock caused by, 1 72 vaccine for, 440 Neonatal abstinence syndrome, 296 Neonatal cephalic pustulosis, 738 Neonatal conjunctivitis, prevention of, 279 Neonatal growth chart, 282F Neonatal herpes, 738-739 Neonatal impetigo, 739 Neonatal jaundice, 342 Neonates. See also Preterm infants ABO incompatibility and, 1 0 7 amniotic band sequence in, 2 9 7 Apgar score and, 2 7 7 , 278t, 2 8 9 bilirubin-induced neurologic dysfunction in, 286-287 criteria for early discharge of, 278 cyanosis in, 129 delayed passage of meconium in, 284 D -transposition of the great arteries in, 128- 1 29 failure to urinate within first 24 hours, 276 Graves disease in, 2 3 1 head-to-toe examination of, 284-285 hearing testing of, 1 9 7 hepatitis B vaccination of, 2 8 5 hyperbilirubinemia i n , 2 8 6 , 286t hypoglycemia in, 283 j aundice in, 287, 290-2 9 1 meconium suctioning of, 287-288 metabolic screening of, 276 naloxone in, 602 requiring resuscitation after delivery, 277-278 resuscitation of, 288 seizures in, 297, 521 spitting up and emesis in, 275-276 subgaleal hemorrhage in, 285, 286f transient tachypnea of, 291 vitamin K deficiency bleeding in, 284 warmers for, 276-277 Nephritis, lupus, 682-683 Nephrogenesis, 669 Nephrolithiasis, 237-238, 394-396 Nephroma, mesoblastic, 392 Nephropathy IgA, 672 reflux, acute pyelonephritis with, 6 8 1 Nephrotic syndrome, 683-684 Nerve injury, with fractures, 2 1 8 Neural tube defects, 527-529, 528t folic acid and, 527, 5 5 1 Neuroblastoma, 7 6 , 78-79, 8 6 , 8 7 , 9 7 , 105, 257, 392, 525

Neurodermatitis, atopic dermatitis and, 49 Neurofibromatosis, 136 type 1 (NF 1 ) , 96, 97, 369-370, 370f type 2 (NF2) , 370 Neurologic impairment, nutritional needs and, 557 Neuropsychiatric manifestations, in vitamin B 12 deficiency, 5 5 1 Neutropenia, 74, 84-85, SSt, 94 chronic, 1 04- 1 0 5 congenital, 2 8 Neutrophils, hypersegmented, 7 9 , SOf

823

0

Obesity, 4 1 2 -4 1 3 exogenous, growth and, 226 hypertension and, 690 smoking and, 784 Obstructive nephropathy, 679-680 Obstructive sleep apnea (OSA), 712-713, 713t Obtunded, definition of, 5 1 3 Ocular infections, congenital, 2 5 5 Ocular torticollis, 483 Odds ratio, 666

Nevus sebaceous, 742, 743f New Ballard score, 279, 280f Niemann-Pick disease, 473 Nightmares, 57t Night terrors, 57, 57t Nighttime awakening, 58, 1 00 Nikolsky sign, 748 Nitrous oxide (N,), 587-588 Nonaccidental trauma (NAT), 97-98 Nonallergic rhinitis with eosinophilia syndrome (NARES), 34 Non-Hodgkin lymphoma (NHL), 78 Nonsteroidal anti-inflammatory drugs (NSAIDS), 592, 593 for acute chest syndrome, 84 adverse effects of, 580 for benign j oint hypermobility syndrome, 161

Oil of lemon, 429 Oligoarthritis, 269 Oligomenorrhea, 12 Oliguria, 686-687 Omalizumab for asthma, 4 1 , 4 l f, 44 for urticaria, 33 Omenn syndrome, 27 Omphalocele, 294-295 Ophthalmia neonatorum, 257, 2 8 l f, 432-433 Ophthalmology exams, 639 Opiates, definition of, 5 9 1 Opioids, 590-591 abuse of, 785-786 antidote to, 786 dependency on, 593

for chest pain, 1 1 0 for dysmenorrhea, 22 for erythema infectiosum, 1 6 1 for Henoch-Schonlein purpura, 1 54 hypertension due to, 693t intoxication by, 599, 676t for juvenile idiopathic arthritis, 1 5 8 nephrotoxic, 676t for osteoid osteoma, 99 for systemic lupus erythematosus, 1 52 upper gastrointestinal bleeding due to, 348 Nonstress tests (NSTs), 275 Nontreponemal tests, 442 Noonan syndrome, 1 1 6- 1 1 7, 367, 367f Norepinephrine, for septic shock, 1 72 Normal saline, for opioid ingestions, 602 Nose bleeding from, 1 8 6 - 1 87, 1 8 7t foreign bodies in, 1 8 7 hematomas of, 1 87 Nucleic acid amplification testing (NAAT), 433 Null hypothesis, 665 Number needed to treat, 666 "Nursemaid's elbow;' 2 1 7 Nutrition, 539-560 withholding, 246 Nutritional supplements, athletes' use of, 777-778 Nystagmus, 267-268 acquired, 256-257

ingestions of, 602-603, 706 for pain management, 245-246 toxidrome associated with, 786 withdrawal from, 593 Opportunistic infections, 26 Oppositional defiant disorder (ODD) , 62, 63, 66, 1 48- 149 Opsoclonus myoclonus, 257 Opsoclonus-myoclonus syndrome, 79, 525 Oral allergy syndrome, 45 Oral contraceptive pills (OCPs) , 1 4- 1 6 for abnormal uterine bleeding, 1 4 contraindications to, 1 5 drug interactions of, 1 6 for dysmenorrhea, 22 hypertension due to, 693t Oral hygiene, 634 Oral lesions, differential diagnosis of, 193, 1 94t Oral polio virus vaccine (OPV) , 633 Oral rehydration therapy (ORT), components of, 307 Oral temperature, 203 Orbital abscesses, 188 Orbital cellulitis, 1 88, 260, 260f, 2 6 1 -262, 262t Orchitis, mumps and, 3 9 1 , 443-444 Organic acidemias, 471 -472 Organophosphate exposure, 6 1 2 - 6 1 3 , 6 1 3t Ornithine transcarbamylase deficiency, 460t

fetal exposure to, 296

824

I n dex

Oropharyngeal changes, in Kawasaki disease, 1 5 5 Osgood-Schlatter disease (OSD), 502, 502f, 766, 767f Osmolar gap, methanol ingestion and, 608 Osmotic demyelination syndrome (ODS) , 303 Osteochondral defect, 775, 775f Osteochondritis dissecans, 775, 775f Osteogenesis imperfecta, 48 1 , 482, 653 Osteoid osteoma, 98-99 Osteomyelitis, 491 -492, 492f of frontal bone, 1 8 8 Osteopetrosis, 407 Osteosarcoma (OS), 86, 98- 1 00, 99f, l OOf Otitis externa, 1 79 Otitis media acute, 1 80- 1 82, 1 8 1 f, 1 84- 1 8 5 brain abscesses secondary to, 5 1 8 chronic suppurative, 1 8 3 - 1 84 with effusion, 1 8 3 , 1 83t recurrent, 34 Otitis media with effusion (OME), 1 8 3 , 1 83t Otoacoustic emission test, 1 97, 198 Otorrhea, 1 83 - 1 84, 1 84t Ovarian cysts, in polycystic ovary syndrome, 1 3 , 1 3 f, 14 Ovarian torsion, 3 9 1 Overnutrition, growth and, 226 Overuse injuries, in athletes, 766 Oxcarbazepine, for myoclonic seizures, 523 Oxycodone, 5 9 1 1 Oxygen hemoglobin dissociation curve, 706, 706f Oxygen supplementation, for carbon monoxide poisoning, 6 1 4 Oxygen therapy, for acute chest syndrome, 84 p

Pacemakers, 126- 1 2 7 Pacifiers, sudden infant death syndrome and, 634 Page kidney, 387 Pain. See also specific sites management of, for terminally ill patients, 245-246 opioids for relief of, 590-591 Palate, cleft, 1 93 - 1 95, 1 95f Palivizumab, for respiratory syncytial virus infections, 447 Palliation, 245-246 Palm(s), hyperlinear, 727 Palmar grasp reflex, 4 1 4 Pancolitis, 3 5 6 Pancreatic exocrine insufficiency, 3 5 5 Pancreatitis, 468 acute, 326, 326t chronic, 326, 327t Panic disorder, 64t

Papanicolaou (Pap) testing, 1 7 Papilledema, 268-269, 268f Papillomas, 200 Paracetamol. See Acetaminophen Parachute reflexes, 4 1 4 Parapharyngeal abscesses, 1 9 1 Paraphimosis, 382f Parapneumonic effusion, 722-723 Parechovirus, 450-45 1 Parent(s) consent from, lack of need for, 242 divorce of, 643, 644t domestic violence and, 650-65 1 , 6 5 1 t right t o seek alternative care, 249-250 treatments requested by, 252-253 Parent-infant attachment, 58 Parinaud oculoglandular syndrome, 258 Parkland formula, 2 1 4, 2 1 6, 2 1 6t Parotid gland, 193 Parotitis, 1 92- 1 93 Paroxysmal nocturnal hemoglobinuria, 89t Pasteurella, animal bites and, 449 Patau syndrome, 364-365 Patellar dislocations, 774-775 Patellofemoral stress syndrome, 772-773 Patent ductus arteriosus (PDA), 1 1 3 , 1 1 5 Patent foramen ovale (PFO), 1 1 7 Patient -controlled analgesia (PCA) , 592 Patient -parent -pediatrician relationship, 242-244 reo,, 3 1 5- 3 1 6

Peak expiratory flow rate (PEFR) , 720 Pediatric advanced life support (PALS) training, 587 Pediculosis capitis, 755 Pediculosis pubis, 755 Pelizaeus-Merzbacher disease, 478 Pelvic examination, in adolescents, indications for, 1 7 - 1 8 Pelvic inflammatory disease (PID), 1 9 Pelvic pain, causes of, 1 9t Penicillins, 676t allergy to, 48 Penis, circumcision of, 3 8 1 -382 Pentobarbital, 588 Peptic ulcer disease, 3 3 1 i n adolescents, 349 Pepto Bismol, gastric ulcers due to, 348 Perianal fistula, 358 Pericarditis, 1 1 0, 1 1 7 acute, 1 1 1 Peripheral neuropathy, in vitamin B12 deficiency, 5 5 1 Peritonitis, i n nephrotic syndrome, 684 Peritonsillar abscesses, 1 88 - 1 89 Periungual fibromas, 74 1 , 742f Periventricular calcifications, 530, 530f Periventricular leukomalacia, 529, 529f Permethrin, 429 for lice, 755 for scabies, 750

Pernicious anemia, 550- 5 5 1 Persistent depressive disorder, 6 6 Persistent vegetative state (PVS), 245 Personality disorders, 66 Pertussis, 43 1 , 456, 632 Petechiae in meningitis, 5 1 5 in septic shock, 1 70- 1 7 1 Pfeiffer syndrome, 3 7 1 PHACE(S) syndrome, 273 Pharyngeal abscesses, lateral, 1 9 1 Pharyngitis, 423, 427-428 differential diagnosis of, 190 group A streptococcus, 1 89- 1 90 Pharyngolaryngeal reflux, 699 Phencyclidine (PCP), 786 Phenobarbital, for status epilepticus, 524t Phenylalanine control, in phenylketonuria, 475 Phenylketonuria (PKU) , 460t, 474-476 Phenytoin, dosage of, 574-575 Pheochromocytoma, 693t Phimosis, 382f, 398 Phobias, 64t, 65 Phorias, 274 Photophobia, 267 Physical abuse, 656 Physical examination, preparticipation, for athletes, 759-760 Physical restraints, for hallucinogen abuse, 786 Physicians disclosure of medical errors by, 246 as expert witnesses, 247-248 gifts to, 24 7 Physiologic futility, 245 Physostigmine for anticholinergic intoxication, 604 for opioid ingestions, 602 for tricyclic antidepressant overdose, 607 Phytoestrogens, 4 Picaridin, 429 Picky eaters, 5 1 -52 "PICO" questions, 667 Pierre Robin sequence, 193- 1 95, 365-366, 365f, 7 1 2 Pill esophagitis, 349 Pimecrolimus, for atopic dermatitis, 727 Pimozide, drug interactions of, 576-577 Pinworms, 448-449 Pituitary, lesions near, 233-235, 235f Pityriasis alba, 727, 728f Pityriasis rosea, 753-754, 753f Pityriasis versicolor, 752, 752f Plagiocephaly, 408, 408f positional, 408-4 1 0 , 409f, 409t Plan, Do, Study, Act model, 563, 564f Platelet transfusion, for purpura fulminans, 96-97 Play cooperative, 4 1 7 parallel, 4 1 6 -4 1 7

I nd ex

Plethysmography, whole body, 7 1 8 Pleural effusion, 7 1 7 Pleural fluid culture, 723 Pleuritic chest pain, 1 10 Pneumococcal conjugate (PCV- 1 3 ) vaccine, 627, 630 Pneumococcal polysaccharide unconjugated vaccine (PPSV-23) vaccine, 630 Pneumocystis jiroveci pneumonia, 448 Pneumonia, 444-445, 722-724 bacterial, 1 6 7 community-acquired, 723 pneumocystis, 448 recurrent, evaluation of, 723-724 Pneumothorax in neonates, 293 tension, 2 1 1 , 2 1 1 f Poison Control Center, 595-596 Poisoning. See also specific substances unknown ingestions and, 595 Polio vaccines, 633 Pollen-food allergy, 45 Polycystic ovary syndrome (PCOS), 1 3 , 1 3 f, 14 Polycythemia, maternal diabetes mellitus and, 295 Polydactyly, 489 Polyhydramnios, 295, 703 Polymenorrhea, 12 Pompe disease, 465, 466, 470 "Popsicle panniculitis;' 1 92 Port-wine stains, 737 Positional plagiocephaly, 408-4 1 0 , 409f, 409t "Positive agreement;' 24 1 Positive future expectations, 647 Positive predictive value, 666 Posterior cruciate ligament (PCL) tears, 773, 774f Posterior urethral valves (PUVs), 679 Postexposure prophylaxis, for Neisseria meningitidis, 440 Postexposure prophylaxis (PEP), for rabies, 207 Posttest probability, 666 Posttraumatic stress disorder (PTSD), 64 Potassium hydroxide (KOH) examination, 750t Potassium levels, 308 Potter sequence, 7 1 7 Pott puffy tumor, 1 8 8 Poverty-related stressors, 645 Prader-Willi syndrome (PWS), 366, 4 1 2 Pralidoxime, for organophosphate exposure, 6 1 3 Preauricular sinuses, pits, and skin tags, 1 79 Precocious puberty, 226 Pregnancy in adolescence, 1 6 , 1 7 contraindication to MMR vaccine in, 627 folate deficiency during, 5 5 1 folic acid during, 527

Prehn sign, 3 9 1

825

Psychogenic "habit;' 7 1 1 t Psychosis. See also Bipolar disorder causes of, 7 1 -72, 7 l t

Prehypertension, 1 1 0 Premenstrual dysphoric disorder, 66 Preparticipation physical examination, for athletes, 7 59-7 60 Prepatellar bursitis, 775-776 Prepuce, abnormalities of, 382, 382f, 398 Preschool children eating problems in, 5 1 -52 tantrums in, 54-55, 56 thumb-sucking in, 5 1 toilet training and, 52-53

Psychosomatic disorders, 68-69 Psychostimulant medications abuse of, 788 for attention deficit hyperactivity disorder, 69-70, 1 47- 1 48 for bipolar disorder, 68 for oppositional defiant disorder, 1 49 Psychotherapy for anxiety disorders, 63

Preseptal cellulitis, 260-26 1 , 260f, 262t Preterm infants, 280-28 1 , 290 chronic lung disease of prematurity in, 296 extremely low birthweight, 289 low birthweight, 289 nutritional needs of, 540 pulmonary hypertension in, 725 respiratory distress syndrome in, 292 vaccination of, 630, 63 1 -632, 63 l t very low birthweight, 289 Pretest probability, 666 Prevalence, 666 PRICE acronym, 768 Primary congenital glaucoma (PCG), 264, 265f, 267 Primary survey, 2 1 0

for children experiencing divorce, 643 for depression, 66 Ptosis congenital, 265-266, 266f severe, 265 Pubarche, 5 female, 228 Puberty age of, 1, 2f-4f, 3t, 4-6 delayed, 1 early, substance use and, 784 female, 226-227, 227f female, delayed, 228-229 precocious, 226 Pubic hair development of, 5 Tanner stages of development of,

Procainamide, for tricyclic antidepressant overdose, 607 Proctocolitis, allergic, 338, 543 Progesterone challenge, 1 2 Prolonged rupture o f membranes (PROM), 292, 293f Propionic acidemia, 460t, 471 -472 Propofol, 587t, 588 Propranolol, adverse effects of, 584-585, 737 Prostaglandin, for systolic dysfunction, 1 1 5- 1 1 6 Protein-losing enteropathy (PLE), 352, 544, 544t Proteinuria, 669-670 Proton pump inhibitors (PPis), 580 for gastroesophageal reflux disease, 335 Prune belly syndrome (PBS), 678 Pseudogynecomastia, 4 Pseudohyperkalemia, 3 1 0 Pseudohyponatremia, 3 1 0 Pseudomembranous colitis, 336, 337, 435, 578-579, 7 1 5 Pseudomonas aeruginosa infections bronchiectasis due to, 7 1 3 , 7 1 5 otitis externa due to, 1 79 puncture wounds of foot and, 206 Pseudoseizures, 524 Pseudostrabismus, 255 Psoas sign, 504 Psoriasis, 72 7 arthritis associated with, 1 58f guttate, 730-73 1 , 73 1 -732, 7 3 l f, 732t plaque, 732t, 75 1

227, 227f Pulmonary function tests, 7 1 8-720, 7 1 8f Pulmonary hemosiderosis, idiopathic, 726 Pulmonary hypertension, 1 1 3 - 1 14 exercise impairment and, 724-725 persistent, in neonates, 293 in preterm infants, 725 relieved by nighttime ventilation, 7 1 7 Pulmonary laceration, 2 1 1 Pulmonary stenosis, 1 1 7 Pulselessness, 1 70 Puncture wounds, 206 Pure red cell aplasia, 80 Purified protein derivative (PPD), 438, 439 Purine metabolism defects, 46 l t Purpura fulminans, 96-97 in meningitis, 5 1 5 meningitis with, 439-440, 439f Pyelonephritis, 76, 678-679 acute, with reflux nephropathy, 6 8 1 Pyloric stenosis, 305-306 infantile, hypertrophic, 332-333 Pyrimidine metabolism defects, 46 l t Pyuria, sterile, i n Kawasaki disease, 1 56

Q Quadriceps contusion, 762, 763f Quality improvement, 568-569 Quantiferon gold test, 438-439 R

Rabies vaccine, 206-207 "Raccoon eyes;' 79

826

I n dex

Rectal bleeding, 349-3SO Rectal prolapse, 3SO Rectal temperature, 203 Red reflex, abnormal, 2S6 Refeeding syndrome, 9, 1 0 , 3S8-3S9, S60 Reflexes cremaster, 389 Moro (startle) , 4 1 4

case series and, 66S cohort studies and, 662, 66S controlled clinical trials and, 661 -662 cross-sectional studies and, 663, 663f meta-analyses and, 663, 664f, 66S randomized controlled trials and, 66 1 , 662f, 664 sample size and, 664 study design and, 66 1 terminology related to, 66S Resilience, 6 1 -62, 62t Respiratory acidosis, 3 1 S, 3 1 St, 706, 706f, 707 Respiratory alkalosis, 3 1 St, 3 1 6-3 1 7, 706, 706f, 707 Respiratory distress, management of, 72 1 -722 Respiratory distress syndrome, in preterm infants, 292 Respiratory failure, in neonates, 293 Respiratory rate, normal, by age, 709, 709t Respiratory syncytial virus (RSV) , 447 Response to Intervention (RTI) , 144 Resuscitation, for hypothermia, 1 72- 1 73 Reticulocytosis, 90 Retinal disorders, inherited, 2SS Retinoblastoma (RB), 97 Retinoids, for acne vulgaris, 733-734 Retinopathy of prematurity (ROP), 2SS, 269

attendance and, 422 Salter-Harris fractures, 496, 497f, 776, 776f Sample size, 664 Sandifer syndrome, 336, 483, S 2 1 Sapropterin hydrochloride (Kuvan) , for phenylketonuria, 47S Sarcoidosis, 1 60 SBAR (situation, background, assessment, recommendation) , S70 Scabies, 749-7S 1 , 749f, 7SOf crusted (Norwegian), 7SO nodular, 7SO, 7SOf Scaphocephaly, 408, 408f Scaphoid waist fractures, 77 1 , 772f

palmar grasp, 4 1 4 parachute, 4 1 4 rooting, 4 1 4 tonic neck, 4 1 4 Refusal o f treatment in emergencies, 249 by mature minor, 249-2SO Regurgitation, in rumination syndrome, 334-33S Reiter syndrome, 20 Relapse rate, for substance abuse, 780 Relative afferent pupillary defect (RAPD ) , 268 Relative risk, 666 Relative risk reduction, 666 Reliability, 66S Religious beliefs, care decisions and, 2SO Remediation, 146, 146f Renal artery stenosis, 69S Renal cysts, 67S-676, 676t, 677t Renal disease, 1 06- 1 0 7 diet in, SSS Renal injury, 386-387 Renal stones, 394-396 Renal tubular acidosis, 674-67S Renin - angiotensin -aldosterone system (RAAS) , S8S Renin-angiotensin system, 691 Reninoma, 693t Research case-control studies and, 662-663, 663f, 66S case reports and, 663-664

Retropharyngeal abscesses, 1 90- 1 92, 704 Retropharyngitis, 483 Retroviral syndrome, acute, 1 8 Rett syndrome, 366, S 1 9-S20 Return-to-play decision, for athletes, 762, 764-76S Rewarming, 1 72- 1 73 Reye syndrome, S80, S97 Rhabdomyosarcoma (RMS), 76 Rheumatic fever, 1 1 7- 1 1 8, 1 1 7t Rh incompatibility, 1 0 7 Rhinitis medicarnentosa, 1 99 Rhinoviruses, 1 99-200 Rhus dermatitis, 730 Rickets, 238-239, 238f, S47, S49, S49f, S49t calcipenic, S49 phosphopenic, S49 Rifampin, oral contraceptive pills and, 1 6 Right-to-left shunting, 72 1 atrial, pulmonary hypertension and, 72S Right ventricular outflow tract obstruction, severe, 128 Risk-taking behavior, 6-7 Risser sign, 499, SOOf Rocky Mountain spotted fever (RMSF) , 429, 440-44 1 Rolling, from front to back, 4 1 S Rooting reflex, 4 1 4 Rosacea, 2S8 Roseola, 4SO Rotavirus infections, 4S6 Routine health maintenance exams, screening and surveillance in, 4 1 4

Scheuermann kyphosis, SOO, SOOf School early intervention and, 147 individualized education plan and, 14S, 146 Individuals with Disabilities Act and, 144, 14S- 1 46, 14St most inclusive education in, 1 46 options for hearing-impaired children, 14S special education services evaluation and, 144t special education services placement and, 148 specialized, pull-out instruction in, 147 School refusal, 61, 6 1 t Scimitar syndrome, 1 28- 1 29 Scleroderma, linear, 1 S9- 1 60, 1 60f Scoliosis, 498-SOO, 499f congenital, 483-484, 484f, 499 idiopathic, adolescent, 499 Scopolamine, adverse effects of, S77 Scorpion stings, 207-208, 326 Screening Tool for Autism in Toddlers and Young Children (STAT), 1 3 6 Screen time behavioral effects of, 644-64S limiting, 641 Scrotal pain, 390- 3 91 Scurvy, S S 1 , S S l f Secondhand smoke, 640 "Second victim" syndrome, S67

Radiation therapy risks of, 106 vaccinations prior to, 633 Radiocontrast agents, nephrotoxic, 676t Randomized controlled trials, 66 1 , 662f, 664 Ranula, 193 Rapid plasma reagin (RPR) , 442 Rash. See also specific disorders amoxicillin and, 423 blueberry muffin, 4S3, 4S3f diaper, S79, 7S2-7S3 in erythema nodosum, 1 6 1 evaluation of, 44 1 with fever, 422 in Kawasaki disease, 1 S S i n systemic lupus erythematosus, 1 S 1 , 1 S2f viral, 449-4S 1 , 449f-4S l f

Recommendations for Preventive Pediatric Health Care, 4 1 4 Recreational drug use. See Substance abuse

Rubella, 449 congenital, 4S4 Rumack-Matthew nomogram, 6 0 1 f, 602 Rumination, 648 Rumination syndrome, 334-33S Russell-Silver syndrome (RSS), 367-368 s

Sacroiliitis, 1 6 1 Safety culture, S70 Salicylate intoxication, S96-S98 Salmonella, pyogenic arthritis due to, 492

Salmonella enteritidis infections, 446 Salmonella typhi infections, day care

I nd ex

Sexually transmitted infection (STI), 7, 1 8 , 1 9-20. See also specific infections screening for, 17, 1 8 Sexual maturity rating (SMR) , 1 , 3f, 3t, 4-5, 4f Sexual preference, in adolescence, 7 Shagreen patches, 74 1 , 742f

Sedation, 586-592 conscious, 586 "crash cart" for, 587 deep, 586, 587 discharge following, 588-589 for echocardiography, 588 equipment for, 592 inadequate, 589 minimal, 586-587 moderate, 586, 587 monitoring and, 5 9 1 -592 opioids and, 590-59 1 , 5 9 l t

"Shaken baby syndrome;' 653-654, 654f Shewhart charts, 566-567, 567f, 568 Shigella infections, 446 Shin splints, 766 Shock, 1 65 - 1 66, 1 6 7

preoperative fasting recommendations for, 589-590, 590t Seesaw nystagmus, 257 Seizures, 520-524, 522t absence, 52 1 , 522t amphetamine-induced, 787 complex partial, 52 1 , 522t evaluation of, 520-52 1 febrile, complex, 523-524 generalized, 522t ketogenic diet for, 557 myoclonic, 522-523 neonatal, 297, 521 simple partial, 522t treatment of, 303-304 Selective serotonin reuptake inhibitors (SSRis)

anaphylactic, 3 1 cardiogenic, 1 67, 300t compensated, 1 6 5 distributive, 300t hemodynamics in, 165, 1 66f hypovolemic, 299-300, 300t septic, 1 67, 1 70- 1 7 1 , 1 7 l t treatment of, 299-301 "warm" and "cold;' 1 70 Shortness of breath, 7 1 0 -7 1 1 Short stature, 2 2 1 -223, 403, 405 Shoulder dislocations, 769-770, 769f Shoulder dystocia, 293-294 Shprintzen syndrome, 1 9 5 Shwachman-Diamond syndrome, 89t, 3 5 5 Siblings

for anxiety disorders, 63 for bipolar disorder, 68 for depression, 66 discontinuation syndrome with, 584 intoxication with, 607 Self-stimulation, 5 2 1 Sensitivity, 666 Sensorineural hearing loss (SNHL), 198 Sentinel events, 563 Separation anxiety, 59-60 Separation anxiety disorder (SAD), 60-6 1 , 64t Sepsis definition of, 1 7 l t i n galactosemia, 466 neonatal, 457 Septic shock, 1 67, 1 70- 1 7 1 definition of, 1 7 l t treatment of, 1 72 Serotonin syndrome, 583-584 Serum-specific IgE testing, 34-35 Severe combined immunodeficiency (SCID) , 25-27, 28, 30, 1 00- 1 0 1 phenotypes of, 26, 27t X-linked, 26 Sever's disease, 5 0 1 -502, 766-767 Sexual abuse, 657-658, 657t Sexual assaults, reporting of, 1 6 Sexual behavior, normal for age, 6 1 , 62t Sexual differentiation, 2 1 9 , 220f Sexually transmitted diseases (STDs) , 435-436. See also specific diseases

arrival of, helping a child adjust to, 56 conflicts among, 56-57 of gifted children, 649-650 Sickle cell anemia, 83-84, 94 vaccinations in, 429 Sickle cell disease (SCD), 88 fetal hemoglobin and, 1 0 2 - 1 0 3 , 1 03f ischemic stroke due to, 530-53 1 , 5 3 l t vaccinations in, 630 Sideline Concussion Assessment Tool v.3 (SCAT3 ), 760 Sinding-Larsen-Johansson disease, 766 Sinus cavities, present at birth, 1 8 8 Sinusitis, 262 bacterial, acute, 1 87 - 1 8 8 brain abscesses secondary to, 5 1 8 chronic, risk factors for, 1 8 8 recurrent, 3 4 Sinus tachycardia, i n anticholinergic intoxication, 603-604 Six sigma, 568 Skin hyperelastic, in Ehlers-Danlos syndrome, 1 62, 1 62f hyperpigmentation of, in acquired adrenal insufficiency, 232, 232f Skin adhesives, 206 Skin, eye, and mouth disease (SEM), 426 Skin necrosis, warfarin-induced, 96 Skin tests for allergies, 34-35 for pruritic rashes, 7 SOt Skull fracture, 2 1 1 -2 1 2, 2 1 lf

827

Sleep apnea central, 712, 7 1 3 obstructive, 7 1 2 - 7 1 3 , 7 1 3t Sleep hygiene, 57-58, 169- 1 70 Sleep patterns, 58, 58t Slipped capital femoral epiphysis (SCFE), 503-504, 503f obesity and, 4 1 3 SLUD mnemonic, 604 Small for gestational age (SGA) , 2 8 1 , 282-283 Small intestinal bacterial overgrowth (SIBO), 3 54 Small left colon syndrome, 284 Smith-Lemli-Opitz syndrome, 405 Smoke, secondhand, 640 Smoking. See Cigarette smoking Snake bites, 207 Sniffing, 6 1 1 "Snowman sign;' on chest X-ray, 1 1 2 SOAP ME mnemonic, 592 Social Communication Questionnaire (SCQ), 1 3 6 Social phobia, 64t Social Responsiveness Scale (SRS), 1 3 6 Socioeconomic stressors, 645 Sodium bicarbonate for beta-blocker intoxication, 605 for opioid ingestions, 602 for salicylate intoxication, 597 for tricyclic antidepressant overdose, 606 Sodium chloride, administration of, 3 1 1 Sodium polystyrene sulfonate, 309 Solid foods, introduction of, 544-545 Scorbutic rosary, 5 5 1 Sotos syndrome, 1 3 6 Soy formula, 544 Spanking, 59 Spasmus nutans, 257 Spastic diplegia, 529 Special education services evaluation for, 144t placement in, 1 48 Special Olympics, 760 Specificity, 666 Specific language impairment (SLI), autism spectrum disorder distinguished from, 134 Specific learning disabilities (SLDs), 1 3 8 - 1 39, 1 44 Specific phobia, 64t, 65 Speech development of, 4 1 6 hearing impairment and, 1 4 1 language distinguished from, 1 3 7 Speech disorders, 1 34, 1 3 8 Sphingolipidoses, 46 l t Spider bites, 208 Spina bifida, 527-528 Spinal cord compression, 86-87

828

I n dex

Spinal cord injuries, 2 1 3-214 without radiographic abnormality (SCIWORA), 2 1 4 Spinal cord tumors, 5 1 9 Spinal dysraphism, 528 Spinal muscle atrophy (SMA) , 5 14-5 1 5 Spitting up, in newborns, 275-276 Spleen, rupture of, 423 Splenectomy for Hodgkin lymphoma, 78 for immune thrombocytopenia, 77 Splenic injury, 209 Splenic sequestration, 94 Splenomegaly, 1 0 5 Spondylitis, 1 6 1 Spondylolysis, 504-505, 506f isthmic, 505 Sports. See Athletes Sports drinks, 558 Spousal abuse, 650-65 1 , 6 5 l t Sprengel deformity, 3 7 0 , 483 Standard deviation, 666 Staphylococcal scalded skin syndrome (SSSS), 747-748, 748f Staphylococcus aureus infections, 43 1 , 455 arthritis and, 425-426 cellulitis due to, 755 otitis externa due to, 1 79 postoperative, 756 pyogenic arthritis due to, 492 Startle reflex, 4 1 4 Statistical control charts, 566-567, 567f, 568 Statistical significance, clinical significance vs. , 667 Stature short, 2 2 1 -223, 403, 405 tall, 226, 403 Stature-for-age percentiles, 222, 223f, 224, 225 Status epilepticus, 523, 524t Stealing, 59 Steeple sign, 698, 698f Steroids. See also Corticosteroids; Glucocorticoids; Mineralocorticoids for allergic rhinitis, 33 for alopecia areata, 734-735 anabolic, athletes' use of, 777, 778 for atopic dermatitis, 727, 729t avascular necrosis and, 1 5 2 for juvenile idiopathic arthritis, 1 5 8 for Kawasaki disease, l l 8, 1 56 for urticaria, 33 Stevens-Johnson syndrome, 20, 445, 523, 744, 745t, 746, 746f Stickler syndrome, 195, 372 Still disease, 15 7- 1 58, 15 7t Stimulants. See Psychostimulant medications Stings ant, 208 hymenoptera, 208

jellyfish, 209 scorpion, 207-208, 326 Stool in biliary atresia, 34 7 bloody, 337, 338-339, 340-34 1 , 349, 3 5 1 , 355-3 56, 446 Bristol stool chart and, 383, 383f normal output of, 3 1 4 Strabismus, 256, 267, 268, 270 evaluation of, 273-274 sensory, 256 Stranger anxiety, 4 1 5 Strawberry hemangiomas, 737 "Strep throat;' 1 89- 1 90 Streptococcus pneumoniae infections, acute otitis media due to, 1 80, 1 8 1

Streptococcus pyogenes infections, 455 Sydenham chorea due to, 526 Stretch marks, in nephrotic syndrome, 684 Stridor, 200, 698-700 biphasic, 705 flow-volume loops in, 700, 700f, 70 l f i n laryngomalacia, 698-699, 699f from subglottic hemangioma, 699 in tracheitis, 699 vocal cord paralysis-induced, 705 Stroke, ischemic, 530-53 1 , 5 3 l t Struvite stones, 396 Study design, 6 6 1 Stupor, definition of, 5 1 3 Sturge-Weber syndrome, 738, 738f Styes, 263, 263f, 264 Subacute bacterial endocarditis (SBE), 120- 1 2 1 Subcutaneous nodules, i n rheumatic fever, l l 7, l l 8 Subdural hematoma (SDH), 2 1 3 , 2 1 3f Subgaleal hemorrhage, 285, 286f Subglottic hemangiomas, 737 Subglottic stenosis, 200 Subperiosteal abscesses, 1 88 Substance abuse, 8, 779-788 addiction and, 779-780, 780t administration routes and, 779 of alcohol, 782, 784 of cocaine, 78 1 , 787 consequences of, 780, 782 of dextromethorphan, 788 diagnostic criteria for, 78 l t drug testing and, 8, 2 1 , 782-783, 783t first-time, age of, 780 grace period and, 785 of hallucinogens, 786 of hydrocarbons, 788 of marijuana, 78 1 , 784-785 of MDMA, 787 of methamphetamine, 788 of methylphenidate, 787 of opioids, 785-786 physicians' role in managing, 7 8 1 relapse rate for, 780 risk factors for, 780-78 1

of stimulants, 788 of sympathomimetics, 787 of tobacco, 783-784 treatment of, 782 Sudden cardiac death, in athletes, l l 2 Sudden infant death syndrome (SIDS), 296 prevention of, 634 risk factors for, 7 1 8 Sudden sniffing death, 6 1 1 Suicide in adolescence, 7-8, Sf, 67 depression and, 66 protective and risk factors for, 67, 68t "Sunburst" pattern, on X-rays, 99, l OOf Sun safety, 635 Sunscreen, 635 Superior vena cava (SVC) syndrome, 105, 126- 1 2 7 Supracondylar fractures, 494-495 , 495f, 496 Supraventricular tachycardia (SVT), 1 2 1 - 1 22, 125, 1 70 Sutures, 206 Sweat chloride testing, 7 1 6 Swimmer's ear, 1 79 Swimming pool safety, 635, 637 "Swiss Cheese Model" of organizational accidents, 562, 569, 569f Sydenham chorea, 526 Sympathomimetics, abuse of, 787 Syncope, 5 2 1 cardiac, l l 1 - l l 2 vasovagal, 1 1 1 Syndrome of inappropriate antidiuretic hormone secretion, 3 1 0-3 1 2 Synovitis, 1 52 Syphilis, 1 8 congenital, 44 1 -442, 44 l f testing for, 442 Syrup of ipecac, 596 Systemic inflammatory response syndrome (SIRS), 167, 1 7 l t Systemic lupus erythematosus (SLE), 1 5 1 - 1 53, 1 52t Systolic dysfunction, l l 5- l l 6 T

T4, for autoimmune (Hashimoto) thyroiditis, 230 Tachycardia causes of, 299, 300t in tricyclic antidepressant overdose, 607 Tachydysrhythmias, with tricyclic antidepressant intoxication, 607 Tachypnea, 709, 720-72 1 transient, of newborn, 2 9 1 Tall-man lettering, for medication labels, 565 Tall stature, 226, 403 Tamsulosin, for renal stones, 396 Tanner staging, 1, 3f, 3t, 4f, 226-227, 227f, 228f

I nd ex

829

Tantrums, 54-55 , 56, 59-60 Tarui disease, 466 Tay-Sachs disease, 473-474, 473f, 520 T-cell receptor excision circle (TREC), 25 Teeth fluoride supplementation and, 633-634 permanent, avulsed, 1 9 5 permanent, eruption of, 1 9 5 primary, eruption of, 1 9 5 Teleoroentgenograms, 486-487 Telogen effluvium, 735t Tendon injuries, in athletes, 766

Tibial tubercle apophysitis, 502, 502f, 766, 767f Tibial tubercle avulsion, 766 Tic(s), 52 1 Ticks infections borne by, 429, 440-44 1 , 453 . See also Lyme disease prophylactic treatment for bites of, 637-638 removing, 638 Timed toilet sitting, for functional constipation, 3 4 1 , 342

Transfusion-related acute lung injury (TRALI) , 92-93 Transient erythroblastopenia of childhood (TEC), 80, 8 l t Transient hypogammaglobulinemia (THI), 25 Transient neonatal pustular melanosis (TNPM) , 738 Transmalleolar angle (TMA), 489 Transportation defects, 46 1 t Transpyloric tubes, complications of, 559 Transverse myelitis (TM), 5 1 9

Tensilon test, in myasthenia gravis, 534 Tension pneumothorax, 2 1 1 , 2 1 1 f, 708-709, 709f

Time-outs, 54-55 Tinea capitis, 735-736, 735t, 736f Tinea corporis, 75 1 -752, 75 1 f Tinea pedis, 752 Tinea versicolor, 752, 752f Tobacco. See Cigarette smoking Toddlers biting in, 53 breath-holding spells in, 55-56 immunodeficiency in, 26-28 night terrors in, 57 renal disease in, 1 06- 1 0 7 safety recommendations for, 635, 636t separation anxiety in, 59-60 sexual behavior in, 6 1 , 62t tantrums in, 59-60 toilet training and, 52

"Trapdoor" fractures, 272 Traumatic brain injury (TBI), 17 4- 1 75, 1 75f, 1 75t, 209-2 1 0 , 2 1 0t, 2 1 1 -2 1 2, 2 1 1 f Treacher-Collins syndrome, 1 86, 195, 372, 372f Tremors, drug-induced, 525 Treponema! tests, 442 Triamcinolone, for atopic dermatitis, 727, 729t Trichomonas, 436 Trichomoniasis, 17 transmission of, 7 Trichotillomania, 735, 735t Tricyclic antidepressant overdose, 605-607 Trigger tool methodology, 565 Trigonocephaly, 408, 408f Trisomy 13, 364-365

Teratoma, 97 Terbinafine, for tinea capitis, 736 Terminal illness, pain management for, 245-246 Terrible triad, 773, 775 Testicles enlargement of, 6 undescended, 387-388 Testicular cancer, 4, 396-397, 396f Testicular torsion, 388-390, 389f Testicular volume, 4-5 Testosterone, resistance to, 1 1 Test results, informing child of, 250-2 5 1 Tetanus, 443 Tetanus vaccine, 43 1 -432 indications for, 206t Tethered cord, 499 Tetracyclines caution regarding, 263 dental staining and, 578 Tetralogy of Fallot, 1 27- 128, 1 2 7f Tet spells, 128 Thalassemia, 90 Thalassemia major, 88 Thelarche, 5, 6 Therapeutic relationship, 242-244 Thermometers, 203 Thessaly test, 772 Thigh-foot angle (TFA), 489, 489f Third nerve palsy, 265 Thoracic spine injuries, 2 1 4 Thrombocytopenia, 74 neonatal, 297 Thrombocytopenia and absent radii (TAR) syndrome, 92, 3 7 1 Thumb sucking, 5 1 Thyroglossal duct cysts, 1 96 Thyroid cancer, 1 9 7 Thyroiditis, autoimmune (Hashimoto) , 229-230 Thyroid nodules, 230 Thyroid storm, 23 1 Tibial fractures, distal, occult, 495-496 Tibial stress fractures, 766 Tibial torsion, 487 internal, 488-489

Toddler's diarrhea, 337-338, 338t Toe walking, idiopathic, 487-488 Toilet training, readiness for, 52-53, 53t Tongue fissured, 1 94t geographic, 1 94t Tongue fasciculations, 5 1 4 Tongue-lip plication, 195 Tongue-tie, 1 92 Tonic neck reflex, 4 1 4 Tonsillectomy, 1 9 1 , 1 9 l t Topiramate, for headaches, 5 1 3 Total anomalous pulmonary venous return (TAPVR), 1 1 2- 1 1 3, 1 1 3f Total body surface area (TBSA) , 2 14, 2 1 4t, 2 1 St Tourette syndrome, 64, 526 Toxic epidermal necrolysis (TEN), 745, 745t, 746 Toxic exposure, ataxia due to, 525 Toxic ingestions. See also specific substances unknown, 595 Toxic shock syndrome, 455 Toxocariasis, 449 Toxoplasma gondii infections, 453-454 Tracheitis, 699 Tracheoesophageal fistula (TEF), 70 1 -703, 703f in preterm infants, 295 Tracheomalacia, 70 1 , 7 1 lt Tracheostomy, aspiration and, 722 Tramadol, for dysmenorrhea, 22

Trisomy 18, 364-365, 364f Trisomy 2 1 , 96, 1 36, 352, 362-364, 363f, 405, 482 athletes with, 760, 770 conditions associated with, 363-364, 363f pulmonary hypertension in, 1 14 risk of, 364 Tryptase, anaphylaxis and, 32 T-SPOT test, 438-439 Tuberculin skin tests (TSTs), 437-439, 438, 438f, 439 Tuberculosis as contraindication to breastfeeding, 424, 541 failure to thrive and, 4 1 1 Tuberous sclerosis, 1 36, 74 1 , 742f Tuberous sclerosis complex (TSC), 369-370, 370f Tubo-ovarian abscess, 1 9 Tularemia, 1 9 5 Tumor lysis syndrome (TLS), 9 1 -92 Turner syndrome, 12, 1 27, 367, 367f 33q1 1 deletion syndrome. See DiGeorge syndrome Twisted hairs, 736 Tympanic membrane, perforation of, 1 82, 1 84 Tympanic temperature, 203 Tympanometry, 198 Tympanosclerosis, 182 Tympanostomy tube placement, 1 82, 1 83 Types I and II error, 665 Tzanck preparation, 750t

830

I n dex

u

in sickle cell anemia, 429

Ulcer(s) gastric, 348 oral, traumatic, 1 94t Ulcerative colitis, 3 56 Ulnar collateral ligament (UCL) injury, 770, 770f Umbilical artery, single, 6 8 1 Umbilical cord care of, 278 delayed clamping of, in premature infants, 2 8 1 Upbeat nystagmus, 2 5 7 Upper respiratory infection, viral, rhinitis due to, 1 99-200 Urea cycle, 470, 47 l f Urea cycle disorders, 470 Ureter(s), ectopic, 383-384 Ureteral stones, 394, 395, 395f Urethral valves, posterior, 392, 393f Urethritis, 1 9-20 gonococcal, 435 in Kawasaki disease, 1 56 Uric acid stones, 396 Urinalysis, 378 in acute kidney injury, 685 Urinary tract infection (UTI), 377-379, 673 recurrent, 680 signs and symptoms of, 680, 680f Urination, lack of, in neonate's first 24 hours, 276 Urine samples, collection of, 377 Urolithiasis, 396 Urticaria, 730t allergy testing and, 35 cholinergic, 33 cold-induced, 33 papular, 749, 749f Uveitis anterior, 269-270 in juvenile idiopathic arthritis, 1 5 8 v

Vaccinations, 625-633. See also specific types

of vaccine adverse effects of, 632 autism spectrum disorder and, 1 3 6 BCG, 438 for caregivers, 285 hepatitis B, for neonates, 285 in immunocompromised patients, 45 1 international adoption and, 443 in Kawasaki disease, 1 56 live vaccines for, 633 MMR, 444 for Neisseria meningitidis, 440 prior to chemotherapy and radiation, 633 prior to kidney transplantation, 688 rabies, 206-207 recommended for newborns through 18 years of age, 626f

in sickle cell disease, 103 situations to avoid vaccines in, 27, 30, 100 tetanus, 206t, 43 1 -432 varicella zoster, 449 VACTERL association, 34 1 , 3 7 1 , 703 VACTERL/VATER association, 1 86 Vaginosis, bacterial, 1 7 Valgus angulation, 486 Validity, 665 Valproate, for status epilepticus, 524t Valsartan, 585 Vancomycin, 676t trough and peak levels of, 576 Vanderbilt scales, 6 Van der Woude syndrome, 195, 372 Vanishing white matter disease, 478 Varicella, 449 congenital, 454 Varicella immune globulin (VariZIG) , 45 1 Varicella vaccine, 627 contraindications to, 629 single antigen, contraindications to, 628 Varicella zoster vaccine, 449 Varicoceles, 397-398 Vascular access, in shock, 1 7 1 Vascular injury, with fractures, 2 1 8 Vasculitis, 5 5 1 Vaso-occlusion, i n sickle cell disease, 1 0 3 Vasovagal syncope, 1 1 1 Vegan diets, 550, 556-557 Velocardiofacial syndrome, 195 Velopharyngeal insufficiency, 1 92 Venereal disease research laboratory (VDRL) test, 442 Venous hum, 1 09 Ventricular fibrillation, 1 2 5 Ventricular septal defect (VSD) , 1 1 2, 1 1 3, 127, 1 27f, 128 Ventricular tachycardia, 1 70 Vergence system, development of, 255 Vertebra plana, 504 Very long-chain acyl-CoA dehydrogenase deficiency (VLCADD), 472-473 Very low birthweight (VLBW) preterm infants, 289 Vesicoureteral reflux (VUR) , 378, 379f, 679 Video-assisted thoracoscopic surgery (VATS), 723 Violaceous striae, 4 1 2 Viper envenomation, 207 Viral infections. See also specific infections asthma and, 38 conjunctivitis due to, 259, 259t, 260 in hemoglobinopathies, 75 myocarditis due to, 1 14- 1 1 5 neutropenia and, 104 parotitis due to, 1 92- 1 93 pericarditis and, 1 1 1

rhinitis due to, 1 99-200 transmitted by blood transfusion, 93 urticaria and, 33 Visual impairment, 1 4 1 - 1 42 braille instruction services and, 1 44 Visual system, development of, 255 Vitamin(s) fat-soluble, 545, 548t water-soluble, 545, 546t-547t Vitamin A, 548t excess of, 555 Vitamin B 1 , 546t Vitamin B2, 546t Vitamin B3, 546t Vitamin B5, 546t Vitamin B6, 546t Vitamin B7, 546t Vitamin B9, 546t Vitamin B12, 546t, 550 Vitamin B12 deficiency, 79, 80f, 1 04, 550-55 1 , 580 Vitamin C, 547t Vitamin C deficiency, 5 5 1 -552, 5 5 l f, 552f Vitamin D, 548t Vitamin D deficiency, 545, 547, 548t, 549, 549f, 549t Vitamin D toxicity, 550 Vitamin E, 548t Vitamin K, 548t Vitamin K deficiency bleeding (VKDB), 88, 284 Vitiligo, 727, 728f Vocal cord dysfunction, exercise-induced, 711 Vocal cord paralysis, 200, 705 Vocal cord paresis, 705 Vocal fold granulomas, 200 Vocal fold nodules, 200 Voiding cystourethrogram (VCUG), 378-379, 3 79f, 676, 677t, 679 Volume overload heart failure, 1 1 2 Volvulus, malrotation with, 327-328, 327f, 328f, 333-334 Vomiting coffee ground emesis and, 349 cyclic, 332 differential diagnosis of, 334 in gastroesophageal reflux, 333 in infantile hypertrophic pyloric stenosis, 332-333 in malrotation with volvulus, 333-334 in newborns, 275-276 Von Gierke disease, 465, 466 von Willebrand disease (vWD), 1 5 , 82-83, 1 0 1 - 1 02, 1 02t, 1 04, 107 Vulnerable child syndrome (VCS), 648, 648t Vulvovaginitis, 3 9 1 -392 w

WAGR syndrome, 372, 372f, 394 Wallace Rule of 9s, 2 1 4, 2 1 4t

I nd ex

Warfarin-induced skin necrosis, 96 Warmers, for neonates, 276-277 "Warm" shock, 1 70 Warts, anogenital, 754-755 Wasp stings, 208 Water environmental contaminants in, 62 1 -622 percentage of body weight accounted for by, 3 1 9 Water safety, 635, 637 epilepsy and, 760 Weight. See also Birthweight; Obesity control of, by athletes, 777 Weight-for- age percentiles, 222, 223f, 224, 225 Weight gain from birth to 3 years of age, 4 1 0 , 4 1 0t in childhood, 404 "Weight gainers;' 558 Wenckebach phenomenon, 125 Werdnig-Hofmann disease, 5 14-5 1 5

West Nile virus, 429, 453 West syndrome, 522t, 523 Wet bandages or wraps, for atopic dermatitis, 49 Wheezing. See also Asthma atopic and nonatopic, 36 risk factors for, 3 5 WHIM syndrome, 3 0 White pupillary reflex, 2 5 6 Whole body plethysmography, 7 1 8 Whole bowel irrigation, indications for, 602, 603 "Whooping cough;' 632 Wide-complex tachycardia, 1 24f, 1 2 5 Williams syndrome, 3 6 6 , 374 Wilms tumor (WT) , 75-76, 76f, 95-96, 392-394, 393f Wilson disease, 345 Wiskott-Aldrich syndrome (WAS), 28, 30, 92 Withdrawal, from opioids, 593 Worms, 448-449

831

X

X-linked adrenoleukodystrophy; 477-478, 478f X-linked agammaglobulinemia (XLA) , 25, 28, 30 X-linked recessive disorders, 362 X-rays of chest. See Chest X-rays in osteomyelitis, 492 situations to avoid X-rays in, 30 Xylene, huffing, 6 1 0-6 1 1 y

Yeast infections, vaginal, 1 7 Yellow fever, 453 Yersinia infections, 337 Young adults, superior vena cava syndrome in, 1 26- 1 2 7 z

Zellweger syndrome, 46 1 1 Zidovudine, 42 1 Zinc, 477 Zinc deficiency, 552, 552f